മുഹമ്മദ് നബി

/മുഹമ്മദ് നബി
“കൊള്ളയുദ്ധങ്ങളിൽ കീഴടക്കിയ ഗോത്രങ്ങളിലെ സ്ത്രീകളെ അവരുടെ ഭർത്താക്കൻമാരുടേയും കുട്ടികളുടേയും പിതാക്കൻമാരുടേയും മുന്നിലിട്ട് ബലാൽസംഗം ചെയ്ത ലിസ്റ്റ്…” മറുപടി പെരും നുണകൾ യാതൊരു ലജ്ജയുമില്ലാതെ പടച്ചുവിട്ടിരിക്കുകയാണിവിടെ എന്ന് പ്രത്യേകം പറയേണ്ടതില്ലല്ലൊ. “സ്ത്രീകളോട് പളുങ്കുപാത്രങ്ങൾ പോലെ സൗമ്യതയോടെ ഇടപെടണം” (സ്വഹീഹുൽ ബുഖാരി: 6209) എന്ന് പഠിപ്പിച്ച മുഹമ്മദ് നബി (സ)… “സ്ത്രീകളെ അവർ വെറുക്കുന്നത് ചെയ്യാൻ നിങ്ങൾ നിർബന്ധിക്കരുത്.” എന്ന് പഠിപ്പിച്ച മുഹമ്മദ് നബി (സ)… (മുസ്വന്നഫ് അബ്ദുർ റസാഖ്: 10320) “അല്ലാഹുവാണെ സാക്ഷി, അനാഥ, സ്ത്രീ എന്നീ രണ്ട് ദുർബല വിഭാഗങ്ങളുടെ അവകാശങ്ങളെ (നിറവേറ്റുന്നതിൽ പരാജയപ്പെടുന്നതിലുള്ള പാപത്തെ) സംബന്ധിച്ച് ഞാൻ നിങ്ങൾക്ക് ശക്തമായ താക്കീത് നൽകുന്നു.” എന്ന് സമുദായത്തെ താക്കീത് ചെയ്ത മുഹമ്മദ് നബി (സ)… (മുസ്നദു അഹ്‌മദ്‌: 2/439, റിയാളുസ്സ്വാലിഹീൻ: 146) തന്റെ ഭാര്യയുടെ അടിമസ്ത്രീയുമായി ലൈംഗിക ബന്ധത്തിൽ ഏർപ്പെട്ട ഒരു വ്യക്തിയുടെ കാര്യത്തിൽ വിധി പറയവെ അടിമസ്ത്രീ ബന്ധത്തിന് നിർബന്ധിക്കപ്പെടുകയാണ് ഉണ്ടായതെങ്കിൽ അവൾ സ്വതന്ത്രയാണെന്ന് പ്രഖ്യാപിച്ച മുഹമ്മദ് നബി (സ)… (മുസ്വന്നഫ് അബ്ദുർ റസാഖ്: 13417) “അടിമ സ്ത്രീകളെ പോലും വേശ്യാവൃത്തിക്ക് നിർബന്ധിക്കരുത്…” (ക്വുർആൻ: 24: 33) എന്ന് ലോകത്തെ പഠിപ്പിച്ച മുഹമ്മദ് നബി (സ)… ഈ മുഹമ്മദ് നബി (സ), യുദ്ധത്തിൽ ബന്ധികളാക്കപ്പെട്ട സ്ത്രീകളെ ബലാൽസംഗം ചെയ്യാൻ, അതും അവരുടെ ഭർത്താക്കന്മാരുടേയും കുട്ടികളുടേയും പിതാക്കൻമാരുടേയും മുന്നിൽ വെച്ച് ദ്രോഹിക്കാൻ അനുയായികളെ അനുവദിച്ചു അല്ലെങ്കിൽ അദ്ദേഹം തന്നെ ദ്രോഹിച്ചു എന്നത് ഈ നൂറ്റാണ്ടിലെ ഏറ്റവും വലിയ നുണയാണ്. മനുഷ്യ മനസ്സിൽ ഇന്നേ വരെ വർഗീയ വാദികൾ പാകിയ വിഷ വിത്തുകളിൽ ഏറ്റവും വലിയ വിധ്വേഷ വിത്താണ്. ഈ വിഷയകമായുള്ള കൂടുതൽ വിവരങ്ങൾക്ക് ആറ് ഭാഗങ്ങളായി സ്നേഹ സംവാദം വെബ്സിനിൽ ഈയുള്ളവൻ തന്നെ എഴുതിയ “ഇസ്‌ലാം അടിമസ്ത്രീകളെ ദ്രോഹിച്ചുവോ ?” എന്ന ലേഖനം വായിക്കുക: (https://www.snehasamvadam.org/ഇസ്‌ലാം-അടിമസ്ത്രീകളെ-ദ്/) അടുത്ത ആരോപണം മുഹമ്മദ് നബിക്ക് (സ) മക്കളുണ്ടായിരുന്നില്ല എന്ന കല്ലുവെച്ച നുണയാണ്. “മക്കളുണ്ടായിരുന്നില്ല” എന്ന ഭാഷയല്ല വിമർശകർ “മുഹമ്മദിന്റെ പെണ്ണുങ്ങൾ” എന്ന ലേഖനത്തിൽ പ്രയോഗിച്ചിരിക്കുന്നത് എന്ന് വായനക്കാർ മനസ്സിലാക്കണം. മറിച്ച് വിമർശകരുടെ വിമർശന ഭാഷയിലെ അശ്ലീലത കാരണം അതിന്റെ ശബ്‌ദാന്തര രചനയാണ് ലേഖകൻ ഇവിടെ ചേർത്തിരിക്കുന്നത്. ഏതായാലും, സ്വഹീഹായ നിവേദനങ്ങൾ പ്രകാരം മുഹമ്മദ് നബിക്ക് (സ) ഉണ്ടായിരുന്ന ഏഴു മക്കളുടെ പേരുകൾ അക്കമിട്ട് ഇവിടെ ചേർക്കാം: ആൺമക്കൾ: 1. അൽകാസിം 2. അബ്ദുല്ല 3. ഇബ്രാഹീം പെൺമക്കൾ: 1. സൈനബ് 2. റുക്വിയ്യ 3. ഉമ്മുകുൽസൂം 4. ഫാത്വിമ (സാദുൽ മആദ്: 1/103) മുഹമ്മദ് നബിയോടുള്ള(സ) വെറുപ്പ് സിരയിലും ശിരസ്സിലുമേന്തി നടക്കുന്നവരോട് അവസാനമായി ഒന്നേ പറയാനുള്ളു. പുരുഷോത്തമനായ ഈ തിരുദൂതൻ ലോകത്തിന് മുമ്പിൽ കൊളുത്തി വെച്ച ആദർശ – ധാർമ്മിക പാഠങ്ങളോടുള്ള വെറുപ്പിന്റെ ജ്വരമാണ്, ചരിത്ര നീതിയൊ വൈജ്ഞാനിക ധർമ്മമൊ തൊട്ടു തീണ്ടിയിട്ടില്ലാത്ത, ഈ അധപതിച്ച നിരൂപണ സംസ്കാരത്തിലേക്ക് നിങ്ങളെ എത്തിച്ചിരിക്കുന്നത്. നിഷ്പക്ഷതയോടെ ലോകത്തിന്റെ വിവിധ കോണുകളിലുള്ളവർ മുഹമ്മദ് നബിയെ(സ) കൂടുതൽ പഠിക്കാനും അദ്ദേഹത്തിൽ ആകൃഷ്ടനാകാനും, വിമർശകരുടെ ആദർശ രാഹിത്യം തിരിച്ചറിയാനും മാത്രമെ നിങ്ങളുടെ വിഷം പുരട്ടിയ വിമർശനങ്ങളും അപഹാസങ്ങളും ഉപകരിക്കൂ എന്ന് തിരിച്ചറിയുക… ആ മഹത് വ്യക്തിത്വം പരത്തുന്ന നന്മയുടെ പ്രഭാകിരണങ്ങളുടെ ഊഷ്മളതയിൽ പൊട്ടി പതിക്കുന്ന നീർകുമിളകളുടെ സ്ഥാനമേ മാനവ ചരിത്രത്തിൽ നിങ്ങളെയൊക്കെ കാത്തിരിക്കുന്നുള്ളൂ. { إِنَّ شَانِئَكَ هُوَ ٱلۡأَبۡتَرُ } “(നബിയെ,) തീര്‍ച്ചയായും നിന്നോട് വിദ്വേഷം വെച്ചു പുലര്‍ത്തുന്നവന്‍ തന്നെയാകുന്നു വാലറ്റവന്‍ (ഭാവിയില്ലാത്തവന്‍).” (സൂറത്തുൽ കൗസർ: 3) എല്ലാ നീർക്കുമിളകൾക്കും ‘ഊഷ്മളമായ’ “ഭാവി” നേരുന്നു…

“സ്വശരീരം മുഹമ്മദിന്റെ ലൈംഗികതയിലേക്ക് ദാനം ചെയ്ത സ്ത്രീകൾ ആണ് ബാക്കിയുള്ളവർ….” എന്ന വാചകം ‘ഹിബത്ത് വിവാഹങ്ങൾ’ എന്താണെന്നറിയാത്തതിൽ നിന്നൊ അറിവില്ലായ്മ നടിക്കുന്നതിൽ നിന്നൊ ഉണ്ടായതാണ്. അറബി ഭാഷയൊ ഇസ്‌ലാമിലെ സാങ്കേതിക പദങ്ങളൊ വിമർശകർക്ക് യാതൊരു അറിവുമില്ല. നാവും അസഭ്യങ്ങൾ നിറഞ്ഞ മനസ്സും മാത്രമാണ് ആധാരം. എതായാലും ‘ഹിബത്ത് വിവാഹങ്ങളെ’ സംബന്ധിച്ച് മറ്റൊരു ലേഖനത്തിൽ വ്യക്തമാക്കിയ വിവരം ഇവിടെയും ആവർത്തിക്കാം:

“ശരീരദാനം ചെയ്യുന്നുവെന്ന ആലങ്കാരിക പദപ്രയോഗത്തിലൂടെയുള്ള ‘ഹിബത്ത് വിവാഹങ്ങൾ’ കൊണ്ട് എന്താണ് ഉദ്ദേശിക്കുന്നത്? വിമര്‍ശകന്മാര്‍ ജല്‍പ്പിക്കും പ്രകാരം സ്വന്തം ശരീരത്തെ ഒരു സ്ത്രീ ലൈംഗികാസ്വാദനത്തിനായി നബി(സ്വ)ക്ക് സമര്‍പ്പിക്കുന്ന ഏര്‍പ്പാടാണോ അത്. അല്ലേ അല്ല.!! നാസ്തിക/മിഷനറി/ഷോവനിസ്റ്റ് ഞരമ്പുരോഗികളുടെ ശിഫക്കായി (ശമനം) നമുക്കതു മന്ത്രിച്ചു കൊടുക്കാം.

വിശുദ്ധ ക്വുര്‍ആന്‍ പറഞ്ഞു: “സത്യവിശ്വാസിയായ സ്ത്രീ സ്വന്തത്തെ പ്രവാചകന് ദാനം ചെയ്യുകയും അവളെ വിവാഹം കഴിക്കാനുദ്ദേശിക്കുകയുമാണെങ്കില്‍ അതിനും വിരോധമില്ല.” (ക്വുര്‍ആന്‍: 33: 50)

‘വിവാഹം കഴിക്കാനുദ്ദേശിക്കുകയുമാണെങ്കില്‍’ എന്നാണ് ക്വുര്‍ആന്‍ ഇവിടെ പരാമര്‍ശിച്ചിരിക്കുന്നത്. അഥവാ വിമര്‍ശകന്മാര്‍ ആരോപിക്കും വിധം ലൈംഗികാസ്വാദനത്തിനായി നബി(സ്വ)ക്ക് ഒരു സ്ത്രീ സ്വശരീരത്തെ സമര്‍പ്പിക്കുന്ന ഏര്‍പ്പാടല്ല ഇത്. മറിച്ച് വിവാഹത്തിനായി സ്വന്തത്തെ സമര്‍പ്പിക്കലാണ്. അഥവാ സാധാരണ വിവാഹത്തില്‍ നിന്നും വ്യത്യാസമായി ഇവിടെ സ്ത്രീ മഹ്ര്‍ ഒഴിവാക്കും. ‘ഹിബത്ത്’ വിവാഹം എന്നാണ് ഇതറിയപ്പെടുന്നത്. ഇത് പ്രവാചകനു മാത്രം ബാധകമായ നിയമമാണ്. എല്ലാ പണ്ഡിതന്മാരും അതു വ്യക്തമാക്കിയിട്ടുണ്ട്.

”സത്യവിശ്വാസിനിയായ ഒരു സ്ത്രീ സ്വദേഹം നബിക്ക് ദാനം ചെയ്യുന്ന പക്ഷം നബി അവളെ വിവാഹം കഴിക്കാന്‍ ഉദ്ദേശിക്കുന്നെങ്കില്‍ അതും (അനുവദിച്ചിരിക്കുന്നു.)” അഥവാ മഹ്ര്‍ ഇല്ലാതെ വിവാഹം ചെയ്യാന്‍ ആവശ്യപ്പെട്ടാല്‍. ഹിബത്ത് (സ്വദേഹം ദാനം ചെയ്യുന്നു) എന്ന പദം ഉപയോഗിച്ച് മഹ്ര്‍ ഇല്ലാത്ത നിക്കാഹാണ് ഇവിടെ ഉദ്ദേശം.” (തഫ്‌സീറുല്‍ ജലാലൈനി: 33: 50 ന്റെ വ്യാഖ്യാനം)

“ഇബ്‌നു അബ്ബാസ് (റ) പറഞ്ഞു: അല്ലാഹുവിന്റെ ദൂതന്റെ പത്‌നിമാരില്‍ സാധാരണ ചടങ്ങിലൂടെയൊ വലങ്കൈ ഉടമപ്പെടുത്തിയതോ അല്ലാതെ ആരും ഉണ്ടായിരുന്നില്ല. ഹിബത്ത് (സ്വദേഹം ദാനം ചെയ്യുന്നു) എന്ന പദം ഉപയോഗിച്ച് മഹ്ര്‍ ഇല്ലാത്ത നിക്കാഹിന് തയ്യാറായ ഒരു സ്ത്രീയേയും പ്രവാചകന്‍ (സ) സ്വീകരിച്ചിട്ടില്ല. അവരില്‍ പെട്ട ആരും പ്രവാചകന് (സ) ഉണ്ടായിരുന്നില്ല.” (തഫ്‌സീറുല്‍ കുര്‍തുബി: 33: 50 ന്റെ വ്യാഖ്യാനം)

“മഹ്ര്‍ കൂടാതെ, എന്നെ വിവാഹം ചെയ്യണോ എന്ന വിധി താങ്കള്‍ക്ക് ഞാന്‍ ഇഷ്ടദാനം ചെയ്യുന്നു.” (അല്‍ മുഫ്ഹിം: ഇമാം കുര്‍ത്തുബി: പേജ്: 4/128)

“ഹിബത്ത് (സ്വദേഹം ദാനം ചെയ്യുന്നു) എന്ന പദം ഉപയോഗിച്ച് മഹ്ര്‍ ഇല്ലാത്ത നിക്കാഹിന് തയ്യാറായ സ്ത്രീകളെ വിവാഹം ചെയ്യല്‍ പ്രവാചകന് അനുവദിക്കപ്പെട്ടിട്ടും പ്രവാചകന്‍ (സ) അതു ചെയ്തില്ല എന്നാണ് ഇബ്‌നു അബ്ബാസ് (റ) പറയുന്നത്.” (ഫത്ഹുല്‍ ബാരി: 8: 526)

“ഞാന്‍ എന്നെ താങ്കള്‍ക്കു മുമ്പില്‍ വിവാഹത്തിനായി ഇഷ്ടദാനം ചെയ്യുന്നു’ എന്ന വാചകത്തില്‍ ഒരു മുള്വാഫ് (Possession) (ഭാഷാ പരമായ ഭംഗിക്കായി) വിട്ടുകളഞ്ഞതാണ്. യഥാർത്ഥത്തില്‍ വാചകത്തിന്റെ വിവക്ഷ ഇപ്രകാരമാണ്: ‘ഞാന്‍ എന്നെ അഥവാ എന്റെ വിവാഹ കാര്യത്തെ താങ്കള്‍ക്കു മുമ്പില്‍ ഇഷ്ടദാനം ചെയ്യുന്നു’. കാരണം ഒരു സ്വതന്ത്ര സ്ത്രീ ഉടമപ്പെടുത്തപ്പെടുകയോ ദാനം ചെയ്യപ്പെടുകയോ ഇല്ലല്ലോ. മഹ്ര്‍ ഇല്ലാതെ തന്നെ താങ്കളെ വിവാഹം ചെയ്യാന്‍ ഞാന്‍ തയ്യാറാണ് എന്നാണ് ആ സ്ത്രീ പറഞ്ഞതിന്റെ വിവക്ഷ.” (ഫത്ഹുല്‍ ബാരി: 9/112, ഫത്ഹുല്‍ മുന്‍ഇം: 5/540)

നബി(സ്വ)ക്ക് ലൈംഗികാസ്വാദനത്തിനായി സ്ത്രീകള്‍ അവരുടെ ശരീരം ദാനം ചെയ്യുന്ന ഏര്‍പാടല്ല ‘ഹിബത്ത്’. മറിച്ച് മഹ്ര്‍ (വിവാഹ മൂല്യം) ഇല്ലാതെ തന്നെ അവരെ വിവാഹം ചെയ്യാനുള്ള അവകാശം പ്രവാചകനു സമര്‍പിച്ചുകൊണ്ട് നടത്തപ്പെടുന്ന വിവാഹ രീതിയാണത്. നബി(സ്വ)യുടെ പത്‌നിപദം ആഗ്രഹിച്ചുകൊണ്ടാണവര്‍ അപ്രകാരമുള്ള വിവാഹത്തിനു സന്നദ്ധത അറിയിക്കുന്നത്. എന്നാല്‍ അത്തരത്തിലുള്ള ‘ഹിബത്ത്’ വിവാഹം നബി (സ്വ) ഒരിക്കലും ചെയ്തിട്ടില്ലെന്നാണ് പ്രവാചക ശിഷ്യനും സന്തതസഹചാരിയുമായ ഇബ്‌നു അബ്ബാസ് (റ) പറയുന്നത്.” (നബിപാഠങ്ങളിൽ പെൺ വിരുദ്ധതയില്ല !!! 8)

മറുപടി:

ആറ് വരികളിൽ ആറും ദാക്ഷിണ്യമില്ലാത്ത നുണകളാണ്, മിഷണറിമാരുടെ നുണകളും നവനാസ്തിക തെറികളും കൂട്ടി കുഴച്ചു ചുട്ടെടുത്ത ഈ ലേഖനത്തിൽ നിന്ന് നാം വായിക്കുന്നത്.

ക്വുർആൻ കൊണ്ടോ സ്വഹീഹായ ഹദീസുകൾ കൊണ്ടോ അവലംബനീയമായ സീറ: (നബി ചരിത്രം) കൊണ്ടോ മുഹമ്മദ് നബിക്ക്(സ) 61 പോയിട്ട് ഒരു അവിഹിത ബന്ധമെങ്കിലും ഉണ്ടായിരുന്നെന്ന് തെളിയിക്കാൻ കഴിയാതെ നിരാശരായ മിഷണറിമാർ നുണകളും അർധ സത്യങ്ങളും ദുർവ്യാഖ്യാനങ്ങളും നടത്തി പണികഴിച്ച ചീട്ടു കൊട്ടാരങ്ങളാണ് ഈ ആരോപണങ്ങൾ. പാപസുരക്ഷിതരും വിശുദ്ധരുമായ ദൈവദൂതന്മാരെ പോലും ബലാൽസംഗക്കാരും, സ്ത്രീ പീഢകരും, അഗമ്യഗമനക്കാരുമൊക്കെയായി സ്വന്തം വേദഗ്രങ്ങളിൽ വക്രീകരിച്ച് അവതരിപ്പിക്കാൻ യാതൊരു സങ്കോചവും ഇല്ലായിരുന്ന ഒരു വിഭാഗത്തിൽ നിന്ന് ഇതിൽ കുറവൊന്നും പ്രതീക്ഷിക്കേണ്ടതില്ലല്ലൊ! (ബൈബിൾ: 2 സാമുവൽ 11-12, ഉല്പത്തി 19:30-35, 1 രാജാക്കന്മാർ: 11:3)

ബാലാനുരാഗവും അഗമ്യഗമനവും കുടുംബരതിയും മൃഗരതിയും ശവരതിയും എന്തിനേറെ ബലാത്സംഗം പോലും വസ്തുനിഷ്ടമായ തെറ്റല്ല എന്ന് തുറന്നടിക്കുകയും മോണോഗമി പ്രകൃതി വിരുദ്ധമാണെന്ന് വാദിക്കുകയുമെല്ലാം ചെയ്ത നവനാസ്തികർ (https://youtu.be/FFWkS5nwXjo

https://youtu.be/sDLUYpgzEwo

https://www.washingtonpost.com/national/on-faith/richard-dawkins-under-fire-for-mild-pedophilia-remarks/2013/09 )

ഏത് ധാർമ്മിക ഭൂമികയിൽ ഊന്നി നിന്നു കൊണ്ടാണ് മുഹമ്മദ് നബിയുടെ(സ) (ആരോപിതമായ) ഈ ‘വിവാഹങ്ങളെ’ വിമർശിക്കുന്നത് എന്ന് മനസ്സിലാവുന്നില്ല !!

ബഹുഭാര്യത്വത്തെയും അടിമ സ്ത്രീകളുമായുള്ള ഉപവിവാഹത്തെയും ബൈബിൾ വിശ്വാസികൾ അവമതിക്കുന്നത് ഏറെ ആശ്ചര്യജനകമായ ഒരു നവീന പ്രതിഭാസം തന്നെയാണ്.

അബ്രഹാം പ്രവാചകന് മൂന്ന് ഭാര്യമാരും അടിമ സ്ത്രീകളുമുണ്ടായിരുന്നെന്ന് ബൈബിൾ പഴയ നിയമം വാദിക്കുന്നു. (ഉല്പത്തി: 12:20, 16:3, 1:25, 25:6)

യാകോബ് പ്രവാചകന് നാല് ഭാര്യമാരും അടിമ സ്ത്രീകളുമടക്കം നാല് സ്ത്രീകളുണ്ടായിരുന്നു. (32 : 22).

ദാവൂദിന് ഒമ്പത് സ്ത്രീകളുണ്ടായിരുന്നു. (2 സാമുവേൽ: 3:1-6, 6:23, 11:26, 1 രാജാക്കന്മാർ: 1:1-5)

സോളമൻ രാജാവിന് എഴുന്നൂറ് ഭാര്യമാരും മുന്നൂറ് ഉപഭാര്യമാരും ഉണ്ടായിരുന്നു. (1 രാജാക്കന്മാർ: 3:11)

ഇതൊന്നും പോരാതെ, യുദ്ധം ചെയ്ത് കീഴ്‌പ്പെടുത്തിയ സമൂഹങ്ങളിലെ ചെറിയ പെൺകുട്ടികളെ കണക്കില്ലാതെ കൈവശം വെക്കാം എന്ന വിചിത്രമായ ദൈവകൽപ്പനയും ബൈബിളിൽ കാണാം.

ജൂബിലി ബൈബിൾ, ലിവിംഗ് ബൈബിൾ, വെബ്സ്റ്റേഴ്സ് എന്നീ ബൈബിളിന്റെ ഇംഗ്ലീഷ് പരിഭാഷകൾ ഒന്നിച്ച് വെച്ച് ബൈബിളിലെ സംഖ്യാപുസ്തകത്തിലെ 31 ആം അധ്യായത്തിലെ 17, 18 വാക്യങ്ങളിൽ മോശെയുടെ കൽപ്പനയായി ഇപ്രകാരം നാം വായിക്കുന്നു:

1. But all the female children that have not known a man by lying with him keep alive for yourselves. (Jubilee Bible 2000: Numbers 31:18)

(https://www.biblegateway.com/passage/?search=Numbers%2031%3A18&version=JUB)

2. Only the little girls may live; you may keep them for yourselves. (Living Bible. (TLB) : Numbers 31:18)

(https://www.biblegateway.com/passage/?search=Numbers%2031%3A18&version=JUB)

3. “But all the female children that have not known a man by lying with him, keep alive for yourselves.” (Webster’s Bible: Numbers 31:18)

(https://www.biblestudytools.com/search/?s=bibles&q=Female+children&t=wbt&c=all)

“ഇനി എല്ലാ ആൺകുട്ടികളേയും നിങ്ങൾ കൊല്ലുക. പുരുഷന്മാരുമായി ശയിച്ചിട്ടുള്ള സ്ത്രീകളേയും കൊന്നുകളയുക. പക്ഷെ പുരുഷന്മാരുമായി ഒരിക്കലും ശയിച്ചിട്ടില്ലാത്ത ചെറിയ പെൺകുരുന്നുകളെ നിങ്ങൾക്ക് വേണ്ടി എടുത്തു കൊൾക.” (Numbers 31: 17-18)

മോശെയുടെ ഈ കൽപ്പന പ്രകാരം കന്യകളും ചെറുപ്രായക്കാരുമായ 32,000 പെൺകുഞ്ഞുങ്ങൾ ശത്രു സമൂഹങ്ങളിൽ നിന്ന് അടിമകളായി എടുക്കപ്പെട്ടു. (Numbers 31: 35-40)

പീറ്റർ പെറ്റിന്റെ ബൈബിൾ വിവരണം സംഖ്യ പുസ്തകത്തിലെ വാക്യത്തെ (31: 17-18) ഇപ്രകാരം വിശദീകരിച്ചു:

“അത്തരം പെൺകിടാങ്ങളെ വിവാഹം ചെയ്യാനോ ദാസ്യവേല ചെയ്യാനോ വേണ്ടി സ്വീകരിക്കാമല്ലോ. അങ്ങനെ വരുമ്പോൾ അവർ ഭർത്താവിന്റെ അല്ലെങ്കിൽ യജമാനന്റെ മതം സ്വീകരിക്കുന്നത് പ്രതീക്ഷിക്കാം. അവർ കാലക്രമേണ ഇസ്രാഈൽ സമൂഹത്തിന്റെ ഭാഗമായി തീരുകയും ചെയ്യും.”

ദി ന്യൂ അമേരിക്കൻ കമന്ററി:

“ചെറിയ പെൺകുട്ടികളെ മാത്രമേ ജീവിക്കാൻ അനുവദിക്കാവൂ. അപ്പോഴാണല്ലൊ തോറയിലെ നിയമാവർത്തനത്തിലെ (20: 13-14; 21: 10-14) യുദ്ധ വിധികൾ പ്രകാരം അവരെ ഭാര്യമാരായോ ഇസ്രാഈലി പുരുഷന്മരാരുടെ അടിമകളായോ സ്വീകരിക്കാൻ സാധിക്കൂ. ഇതിലൂടെ അവരെ വിശ്വാസി സമൂഹത്തിന്റെ കുടകീഴിൽ കൊണ്ടുവരാൻ സാധിക്കുമല്ലൊ.”

ജോൺ ഡമ്മലോവിന്റെ ബൈബിൾ വിവരണം:

“ആൺകുഞ്ഞുങ്ങളെ വധിക്കാൻ കൽപ്പിച്ചത് വിഗ്രഹാരാധകരുടെ വർഗത്തെ തന്നെ വേരോടെ പിഴുതുകളയുക എന്ന ലക്ഷ്യത്തോടെയാണ്. മുതിർന്ന സ്ത്രീകളാകട്ടെ വിശ്വാസത്യാഗത്തിന്റെ പ്രധാന കാരണക്കാരായതിനാലും ഭാവിയിൽ ആളുകളെ വഴിതെറ്റിക്കാൻ സാധ്യതയുള്ളതിനാലും കൊല്ലപ്പെടുന്നു. എന്നാൽ ഇളം പ്രായക്കാരായ പെൺകുട്ടികളെ വധശിക്ഷയിൽ നിന്ന് ഒഴിവാക്കിയത്, അവരെ അടിമകളോ ഭാര്യമാരോ ആയി എടുക്കാനും, ഒരുപക്ഷേ എബ്രായ ജനതയിലേക്ക്‌ മതപരിവർത്തനം നടത്താനും സാധിക്കുമെന്നതിനാലുമാണ്.”

ഇനി ഹിന്ദു മതത്തിലേക്ക് വരാം. ആമുഖങ്ങളൊന്നും ഇല്ലാതെ തന്നെ സുലൈമാൻ റസ്‌വിയുടെ ‘ബഹുഭാര്യത്വം ഹിന്ദു ധർമ്മത്തിൽ’ (Polygamy in Hindu Dharma) എന്ന ഒരു ലേഖത്തിലെ പ്രധാന ഭാഗങ്ങൾ ചേർക്കുന്നതിന് ഇവിടെ പ്രസക്തിയുണ്ട്:

“ഗവൺമെന്റിന്റെ 1961 ലെ സെൻസസ് അനുസരിച്ച്, മുസ്‌ലിങ്ങളുടെ 5.7% മായി താരതമ്യം ചെയ്യുമ്പോൾ ഹിന്ദുക്കളിൽ 5.8% ബഹുഭാര്യത്വത്തിൽ ഏർപ്പെടുന്നവരായിരുന്നു. ബുദ്ധമതത്തിലെ പുരുഷന്മാർക്കിടയിൽ ബഹുഭാര്യത്വം 7.9%, ജൈനർ 6.7%, ബഹുഭാര്യത്വം ഏറ്റവും കൂടുതലുള്ളത് ആദിവാസികളിലാണ്, 15.25% പേർ ബഹുഭാര്യത്വം ആചരിക്കുന്നു.

ഇന്ത്യയിലെ അമുസ്‌ലിംകൾക്ക് ഒന്നിലധികം ഭാര്യമാരുള്ളത് നിയമവിരുദ്ധമാണ്. ഇതൊക്കെയാണെങ്കിലും, പല ഹിന്ദുക്കൾക്കും ഒന്നിലധികം ഭാര്യമാരുണ്ട്. 1974-ൽ പുറത്തുവന്ന ഔദ്യോഗിക റിപ്പോർട്ടുകൾ, ഹിന്ദു ദ്വിഭാര്യത്വം നിരോധിച്ച് ഏകദേശം രണ്ട് പതിറ്റാണ്ടുകൾക്ക് ശേഷം, ഹിന്ദുക്കൾക്കിടയിൽ ബഹുഭാര്യത്വം മുസ്‌ലിങ്ങളെക്കാൾ കൂടുതലാണെന്ന ഞെട്ടിക്കുന്ന വസ്തുത എടുത്തുകാട്ടുന്നു (ആദിവാസികൾ: 15 ശതമാനം, ഹിന്ദുക്കൾ: 5.8 ശതമാനം, ജൈനർ: 6.7 ശതമാനം, ബുദ്ധമതക്കാർ: 7.9 ശതമാനം, മുസ്‌ലിങ്ങൾ: 5.6 ശതമാനം). തുടർന്നുള്ള ദശാബ്ദങ്ങളിലെ കണക്കുകൾ ലഭ്യമല്ല. വ്യത്യാസം നിസ്സാരമെന്ന് തോന്നുമെങ്കിലും യഥാർത്ഥത്തിൽ അത് വളരെ വലുതാണ് – ഒരു കോടിയോളം വരുന്ന ഹിന്ദു പുരുഷന്മാർക്ക് ഒന്നിലധികം ഭാര്യമാരുണ്ടായിരുന്നു, വെറും 12 ലക്ഷം മുസ്‌ലിങ്ങൾക്കും.

2006-ൽ നടത്തിയ ദേശീയ കുടുംബാരോഗ്യ സർവേ കാണിക്കുന്നത് 2.5% മുസ്‌ലിം പുരുഷന്മാരും 1.7% ഹിന്ദു പുരുഷന്മാരും 2.1% ക്രിസ്ത്യൻ പുരുഷന്മാരും ബഹുഭാര്യത്വം ആചരിക്കുന്നവരാണെന്നാണ്. ഹിന്ദുക്കളിലും മുസ്‌ലിങ്ങളിലും ബഹുഭാര്യത്വ വിവാഹങ്ങളുടെ എണ്ണം ഗണ്യമായി കുറഞ്ഞു. എന്നാൽ ഇത്തവണ ഹിന്ദുക്കളും മുസ്‌ലിങ്ങളും തമ്മിൽ 0.8% അന്തരമുള്ള മുസ്‌ലിങ്ങൾക്കിടയിൽ ബഹുഭാര്യത്വത്തിന്റെ ശതമാനം കൂടുതലാണ്. എന്നാൽ ഈ 0.8% വിടവ് ഒരു വലിയ സംഖ്യയല്ല. ഇന്ത്യൻ നിയമം മുസ്‌ലിങ്ങൾക്ക് ബഹുഭാര്യത്വം അനുവദിക്കുമ്പോൾ, ഹിന്ദു വിവാഹ നിയമം 1955 ഹിന്ദു പുരുഷന്മാർക്ക് ഒന്നിലധികം ഭാര്യമാരെ ഒരേസമയം വിലക്കുന്നുണ്ട്, ഈ നിരോധനം ഉണ്ടായിരുന്നിട്ടും 1.7% ഹിന്ദുക്കൾ ഇപ്പോഴും ബഹുഭാര്യത്വം ആചരിക്കുന്നു എന്നത് ശ്രദ്ധിക്കേണ്ടതാണ്. ഒന്നിലധികം ഭാര്യമാരുണ്ടാകാൻ ഇന്ത്യൻ നിയമം അനുവദിക്കുന്നെങ്കിൽ ഹിന്ദുക്കളിൽ ബഹുഭാര്യത്വത്തിന്റെ തോത് സങ്കൽപ്പിക്കുക. ഇന്ത്യൻ നിയമകൂടം ശരീഅ: നിയമത്തെ അടിസ്ഥാനമാക്കി മുസ്‌ലിങ്ങൾക്ക് ബഹുഭാര്യത്വം അനുവദിക്കുന്നു, എന്നിട്ടും 2.5% പുരുഷന്മാർ മാത്രമേ ബഹുഭാര്യത്വം അനുഷ്ഠിക്കുന്നുള്ളൂ. 1961-ലെയും 1974-ലെയും സെൻസസ് കൂടുതൽ പ്രാധാന്യമർഹിക്കുന്നു, കാരണം വിവാഹങ്ങളെ മതപരമായ രീതിയിൽ നോക്കിക്കാണുന്ന രണ്ട് ഔദ്യോഗിക സെൻസസ് മാത്രമാണവ.

സ്രോതസ്സുകൾ:-

http://scroll.in/article/669083/muslim-women-and-the-surprising-facts-about-polygamy-in-india

http://timesofindia.indiatimes.com/articleshow/5004493.cms

http://www.cpsindia.org/index.php/pub/87-religious-demography-of-india/reviews/84-indian-currents-lies-half-truths-and-statistics-by-john-dayal

http://indianexpress.com/article/opinion/columns/three-is-a-crowd/

ഹൈന്ദവ ഗ്രന്ഥങ്ങളിൽ ബഹുഭാര്യത്വം:

പദ്‌മ പുരാണ V.57.27-40 പ്രകാരം രാമന്റെ പിതാവ് ദശരഥ രാജാവിന് 3 ഭാര്യമാർ ഉണ്ടായിരുന്നത്രെ. കൗശല്യ, സുമിത്ര, കൈയേയി… പദ്‌മ പുരാണ V.116.42-45 പ്രകാരം അദ്ദേഹത്തിന് നാല് ഭാര്യമാരാണ്.

ശ്രീകൃഷ്ണന് എട്ട് മുഖ്യ ഭാര്യമാരെ കൂടാതെ 16,100 ഉപഭാര്യമാർ ഉണ്ടായിരുന്നു.

വിഷ്ണുപുരാണം പുസ്തകം 5.28.1-5 “ചാരുദേശൻ, സുദേശൻ, ചാരുദേഹം, സുഷേണൻ, ചാരുഗുപ്തൻ, ഭദ്രാചാരു, ചാരുവിന്ദൻ, സുചരു, അതിശക്തനായ ചാരു എന്നീ പുത്രന്മാരെ രുക്മിണി കൃഷ്ണനിൽ പ്രസവിച്ചു; ചാരുമതി എന്ന ഒരു മകളും. കൃഷ്ണന് മറ്റ് ഏഴ് സുന്ദരിമാരായ ഭാര്യമാരുണ്ടായിരുന്നു, കാളിന്ദി, മിത്രവൃന്ദ, സദാചാരിയായ നാഗജിതി, രാജ്ഞി ജാംബവതി; രോഹിണി, മദ്രയിലെ രാജാവായ മാദ്രിയുടെ സ്നേഹനിധിയും മികച്ചതുമായ പുത്രി; സത്യഭാമ, ശത്രുജിത്തിന്റെ മകൾ; മനോഹരമായ പുഞ്ചിരിക്കുടമ ലക്ഷ്മണയും. ഇവരെക്കൂടാതെ അദ്ദേഹത്തിന് പതിനാറായിരം ഭാര്യമാരും ഉണ്ടായിരുന്നു.” (വിവർത്തന: എച്ച്.എച്ച്.വിൽസൺ)

ബ്രഹ്മപുരാണം 95.12-18 “…നരകന്റെ ശേഖരത്തിൽ നിന്ന് സേവകർ കൊണ്ടുവന്ന ആനകളും കുതിരകളും മറ്റ് സമ്പത്തും കൃഷ്ണൻ കൈവശപ്പെടുത്തി. കൃഷ്ണൻ, ഒരു ശുഭദിനത്തിൽ നരകന്റെ വസതിയിൽ നിന്ന് കൊണ്ടുവന്ന യുവതികളെ വിവാഹം കഴിച്ചു. ഹേ ഉത്തമ ബ്രാഹ്മണരേ, ഇവരിൽ ഓരോരുത്തർക്കും പ്രത്യേകമായ ശരീരത്തോടെ, കൃഷ്ണൻ ഭക്തിയോടെ അവരെ വിവാഹം കഴിച്ചു. പതിനാറായിരത്തി നൂറോ അതിലധികമോ സ്ത്രീകൾ ഉണ്ടായിരുന്നു അവർ. ഭഗവാൻ കൃഷ്ണൻ പല രൂപങ്ങൾ സ്വീകരിച്ചു. എന്നാൽ ആ കന്യകമാർ അവനെ തങ്ങളുടെ ഏക ഭർത്താവായി നിനച്ചു; ‘കൃഷ്ണൻ എന്നെ മാത്രം വിവാഹം കഴിച്ചു എന്ന്.’ രാത്രികളിൽ, ബ്രാഹ്മണരേ, പ്രപഞ്ചത്തിന്റെ സ്രഷ്ടാവായ കൃഷ്ണൻ, അവരുടെ എല്ലാവരുടെയും വാസസ്ഥലങ്ങളിൽ താമസിച്ചു.” (വിവർത്തനം: ബോർഡ് ഓഫ് സ്കോളേഴ്സ്, എഡിറ്റ് ചെയ്തത് ജെ.എൽ. ശാസ്ത്രി)

കൃഷ്ണന്റെ പിതാവായ വാസുദേവന് 14 ഭാര്യമാരുണ്ടായിരുന്നു.

ബ്രഹ്മപുരാണം 112.35 “വാസുദേവന് തന്റെ ഭാര്യമാരായി പതിനാല് വിശിഷ്ട സ്ത്രീകളുണ്ടായിരുന്നു. ആദ്യത്തെ അഞ്ച് പേരായിരുന്നു: രോഹിണി, മദിര, വൈശാഖി, ഭദ്ര, സുനാംനി എന്നിങ്ങനെ പേരുള്ള പുരിയുടെ സന്തതികൾ. സഹദേവൻ, ശാന്തിദേവൻ, ശ്രീദേവി, ദേവരക്ഷിത, വൃകാദേവി, ഉപാദേവി, ദേവകി എന്നിവരടങ്ങിയ ഏഴ് സ്ത്രീകളുടെ രണ്ടാമത്തെ ശൃംഖലയും. പതിമൂന്നാമത്തേതും പതിന്നാലാമത്തേതും സുതനുവും യാദവിയും ആയിരുന്നു. ഇവർ രണ്ടുപേരും ആദ്യം വേലക്കാരികളായിരുന്നു. പ്രസിദ്ധനായ സൗരി (കൃഷ്ണൻ) ദേവകിയുടെയും വസുദേവയുടെയും മകനായി ജനിച്ചു…” (വിവർത്തനം: ബോർഡ് ഓഫ് സ്കോളേഴ്സ്, എഡിറ്റ് ചെയ്തത് ജെ.എൽ. ശാസ്ത്രി)

വാസുദേവന് 14 ഭാര്യമാരുണ്ടെന്ന് ഹരിവംശ ​​പുരാണത്തിൽ 1.35.3-ലും പരാമർശിച്ചിട്ടുണ്ട്.

ചന്ദ്രദേവനായ സോമയ്ക്ക് 27 ഭാര്യമാരുണ്ടായിരുന്നു.

ബ്രഹ്മപുരാണം 1.173: “സോമന്റെ ഇരുപത്തിയേഴ് ഭാര്യമാരെന്ന് പരാമർശിക്കപ്പെട്ട പുണ്യ ചടങ്ങുകളിൽ വിവാഹിതരായ സ്ത്രീകളിൽ നിന്ന് അളവറ്റ തേജസ്സുള്ള മിടുക്കരായ കുട്ടികൾ ജനിച്ചു.”

27 ഭാര്യമാരുള്ള സോമയെ സംബന്ധിച്ച് സ്കന്ദപുരാണം V.ii.26.1-6-ലും വരാഹപുരാണം 35.1-2 ലും പരാമർശിക്കപ്പെട്ടിട്ടുണ്ട്.

ബ്രഹ്മ വൈവർത്ത പുരാണം, ബ്രഹ്മ ഖണ്ഡം 9.42: “…ഇപ്പോൾ ഞാൻ ചന്ദ്രദേവന്റെ ഭാര്യമാരുടെ പേരുകളും പുരാണങ്ങളുടെ സാരാംശം ഉൾക്കൊള്ളുന്ന അവരുടെ സ്വഭാവ സവിശേഷതകളും നിങ്ങൾക്ക് വിവരിക്കാൻ പോകുന്നു. അവർ 27 എണ്ണമായിരുന്നു, അവരുടെ പേരുകൾ ഇപ്രകാരമാണ്: അശ്വിനി, ഭ്രാണി, കൃതിക, രോഹിണി, മൃഗ-സിരി, ആർദ്ര, പുനർവസു, പുഷ്യ, അസ്ലേസ, മാഘ, പൂർവ-ഫൽഗുനി, ഉത്തര-ഫൽഗുനി, ഹസ്ത, ചിത്ര, സ്വാതി, വിശാഖം, അനുരാധ, ജ്യേഷ്ഠ, മാഫ, പൂർവാശാദ, പൂർവാസാദ, ഉത്തരാസാദം, ധനിഷ്‌ഠ, ശ്രവണ, ശതാഭിഷ, ഉത്തരാഭിദ്രാപാദി, രേവതി…” (വിവ: രാജേന്ദ്ര നാഥ് സെൻ)

ശ്രീമദ് ഭാഗവതം 6.6.2: — അവൻ പത്തു പെൺമക്കളെ ധർമ്മരാജാവിന് [യമരാജ], പതിമൂന്ന് കശ്യപന് [ആദ്യം പന്ത്രണ്ടും പിന്നെ ഒന്ന് കൂടി], ഇരുപത്തിയേഴ് പേരെ ചന്ദ്രദേവനും, രണ്ട് വീതം അങ്കിരനും കൃഷ്ണാശ്വനും ബുധനും നൽകി. മറ്റ് നാല് പെൺമക്കളെ കശ്യപന് നൽകി. [അങ്ങനെ കശ്യപന് ആകെ പതിനേഴു പുത്രിമാരെ ലഭിച്ചു.]

ഹനുമാന്റെ പിതാവ് കേസരിക്ക് അഞ്ജന, അദ്രിക എന്നീ രണ്ട് ഭാര്യമാരുണ്ടായിരുന്നു. അഞ്ജനയിൽ നിന്നും വായുദേവനിൽ നിന്നുമാണ് ഹനുമാൻ ജനിച്ചത്.

ബ്രഹ്മപുരാണം: ഗൗതമി മാഹാത്മ്യം 14.1-4 “…ഹേ നാരദാ, അഞ്ജന പർവ്വതമുണ്ട്. ആ പർവതത്തിൽ, ഹേ ശ്രേഷ്ഠമായ മഹർഷി, അഞ്ജന എന്ന മഹത്തായ ഒരു ദേവതയുണ്ടായിരുന്നു. ഒരു ശാപത്താൽ അവൾക്ക് ഒരു വീഴ്ച സംഭവിച്ചു. അവളുടെ മുഖം ഒരു കുരങ്ങനോട് സാമ്യമുള്ളതായി മാറി. കേസരി എന്നായിരുന്നു അവളുടെ ഭർത്താവിന്റെ പേര്. കേസരിയുടെ മറ്റൊരു ഭാര്യയായിരുന്നു അദ്രിക. അവളും ഒരു ശാപത്താൽ തകർച്ച നേരിട്ട ഒരു സ്വർഗ്ഗീയ പെൺകുട്ടിയായിരുന്നു. അവളുടെ മുഖവും തലയും പൂച്ചയുടേതിനോട് സാമ്യമുള്ളതായിരുന്നു. അവളും അഞ്ജന പർവതത്തിൽ താമസിച്ചു.” (വിവ: ബോർഡ് ഓഫ് സ്കോളേഴ്സ്, എഡിറ്റർ: ജെ.എൽ. ശാസ്ത്രി)

രുദ്രന് പതിനൊന്ന് ഭാര്യമാരുണ്ടായിരുന്നു

ബ്രഹ്മ വൈവർത്ത പുരാണം, ബ്രഹ്മ ഖണ്ഡം 9.13-22 “അല്ലയോ മഹാനായ സന്യാസി, ഇപ്പോൾ ഞാൻ രുദ്രന്റെ ഭാര്യമാരുടെ പേരുകൾ വായിക്കാൻ പോകുന്നു. (1) കാല (2) കലാവതി (3) കാഷ്ട (4) കാളിക (5) കലഹപ്രിയ (6) കന്ദലി (7) ഭീഷണ (8) ബസ്ന (9) പ്രമോച (10) ഭൂഷണ (11) സുകി എന്നീ പേരുകളിൽ അവ ആഘോഷിക്കപ്പെടുന്നു. അവർ നിരവധി കുട്ടികളെ ജനിപ്പിച്ചു, എല്ലാവരും ശിവയുടെ അനുയായികളായിരുന്നു…” (വിവ: രാജേന്ദ്ര നാഥ് സെൻ)

പതിനൊന്ന് ഭാര്യമാരുള്ള രുദ്രനെ സംബന്ധിച്ച് ശ്രീമദ് ഭാഗവതം 3.12.13-ലും പരാമർശമുണ്ട്.

അഗ്നിക്ക് രണ്ട് ഭാര്യമാരുണ്ടായിരുന്നു.

ദേവീഭാഗവതം 12.10.81-100 പ്രകാരം അഗ്നിക്ക് സ്വാഹ, സ്വധ എന്നിങ്ങനെ രണ്ട് ഭാര്യമാരുണ്ടായിരുന്നു.

ഗണേശന് സിദ്ധി, ബുദ്ധി എന്നീ രണ്ട് ഭാര്യമാരുണ്ടായിരുന്നു, അവർ ഇരുവരും പ്രജാപതി വിശ്വരൂപയുടെ പുത്രിമാരായിരുന്നു.

ശിവപുരാണം, രുദ്ര സംഹിത 2.20.1-10, ഗണേശന് പ്രജാപതി വിശ്വരൂപയുടെ പുത്രിമാരായ സിദ്ധി, ബുദ്ധി എന്നിങ്ങനെ രണ്ട് ഭാര്യമാരുണ്ടായിരുന്നു, അവർ രണ്ട് ആൺമക്കളെ ജനിപ്പിച്ചു എന്ന് വ്യക്തമാക്കുന്നു.

ശിവപുരാണം, രുദ്ര സംഹിത 2.20.13: “ഗണേശന് സന്തോഷത്തോടെ രണ്ട് ഭാര്യമാരെ ലഭിച്ചു. അവർ പ്രജാപതി വിശ്വരൂപന്റെ ഉത്തമപുത്രിമാരാണ്. അവൻ തന്റെ ഐശ്വര്യമുള്ള ശരീരത്തിന് ഉടമകളായ രണ്ട് ഭാര്യമാരിൽ നിന്ന് രണ്ട് പുത്രന്മാരെ ജനിപ്പിച്ചു, സിദ്ധിയുടെ ക്ഷേമ, ബുദ്ധിയുടെ ലാഭ. അവർ എല്ലാവർക്കും സന്തോഷം നൽകുന്നു. ” (വിവ: ജെ.എൽ.ശാസ്ത്രി)

വിഷ്ണുവിന് മൂന്ന് ഭാര്യമാരുണ്ടായിരുന്നു

ബ്രഹ്മ വൈവർത്ത പുരാണം, പ്രകൃതി ഖണ്ഡ 6.13-21: “…ലക്ഷ്മി, സരസ്വതി, ഗംഗ എന്നിവരാണ് ഹരിയുടെ മൂന്ന് ഭാര്യമാർ…” (വിവ: രാജേന്ദ്ര നാഥ് സെൻ)

മറ്റൊരു പതിപ്പ് അനുസരിച്ച്, വിഷ്ണുവിന് നാല് ഭാര്യമാരുണ്ടായിരുന്നു, തുളസിയെ ബലാത്സംഗം ചെയ്തതിന് ശേഷം ഭാര്യമാരിൽ ഉൾപ്പെടുത്തപ്പെടുത്തി കൊണ്ടാണ് ഈ കണക്ക്. ബ്രഹ്മ വൈവർത്ത പുരാണം പ്രകാരം അവൾ തന്റെ ശരീരം ഉപേക്ഷിച്ച് വിഷ്ണുവിന്റെ ഭാര്യയായി എന്നാണുള്ളത്.

ബ്രഹ്മാവിന് രണ്ട് ഭാര്യമാരുണ്ടായിരുന്നു.

സ്കന്ദപുരാണം III.i.41.98-99 പ്രകാരം ബ്രഹ്മാവിന് ഗായത്രിയും സരസ്വതിയും എന്നിങ്ങനെ രണ്ട് ഭാര്യമാരുണ്ടായിരുന്നു.

യമരാജന് പത്തു ഭാര്യമാരുണ്ടായിരുന്നു.

ശ്രീമദ് ഭാഗവതം 6.6.4 പ്രകാരം യമരാജന് നൽകിയ പത്ത് പെൺമക്കളുടെ പേരുകൾ ഭാനു, ലംബ, കകുദ്, യാമി, വിശ്വം, സാധ്യ, മരുത്വതി, വാസു, മുഹൂർത്ത, സംകൽപ എന്നിങ്ങനെയാണ്. ഇപ്പോൾ അവരുടെ ആൺമക്കളുടെ പേരുകൾ കേൾക്കുക.

ഗരുഡന് ധാരാളം ഭാര്യമാരുണ്ടായിരുന്നു.

ബ്രഹ്മാണ്ഡപുരാണം 2.3.7.448-454; വായുപുരാണം 8.319 “…ഗരുഡന്റെ ഭാര്യമാർ മറ്റ് അഞ്ച് പേരായിരുന്നു.-ഭാസി, ക്രൗഞ്ചി, സുകി, ധൃതരാഷ്ട്രി, സയേനി…” (വിവ: ജി.വി. ടാഗരെ)

ബ്രഹ്മാവിന്റെ പുത്രനായ സ്വയംഭുവ മനുവിന് രണ്ട് പുത്രന്മാരുണ്ടായിരുന്നു; പ്രിയവ്രതനും ഉത്താനപാദനും. ഉത്താനപാദന് രണ്ട് ഭാര്യമാരുണ്ടായിരുന്നു; സുരുസിയും സുനിതിയും.

ലിംഗപുരാണം 62.1-5: ഉത്താനപാദന് സുനിതി, സുരുചി എന്നിങ്ങനെ രണ്ട് ഭാര്യമാരുണ്ടായിരുന്നു, ധ്രുവൻ തന്റെ മൂത്ത ഭാര്യയായ സുനിതിയിൽ ജനിച്ചു.

മനുവിന് പത്തു ഭാര്യമാരുണ്ടായിരുന്നു എന്ന് ശ്രീമദ് ഭാഗവതം 4.8.8-ലും പരാമർശിച്ചിട്ടുണ്ട്. മൈത്രായണി സംഹിത 1.5.8 പ്രകാരവും മനുവിന് പത്ത് ഭാര്യമാരുണ്ടായിരുന്നു.

യാജ്ഞവൽക്യ മുനിക്ക് രണ്ട് ഭാര്യമാരുണ്ടായിരുന്നു.

ബൃഹദാരണ്യക ഉപനിഷത്ത് 4.5.1 പ്രകാരം യാജ്ഞവൽക്യന് മൈത്രേയി, കാത്യായൻ എന്നീ രണ്ട് ഭാര്യമാരുണ്ടായിരുന്നു.

മന്ദാർകിണി മഹർഷിക്ക് അഞ്ച് ഭാര്യമാരുണ്ടായിരുന്നു.

വാല്മീകി രാമായണം അനുസരിച്ച്, (ആരണ്യകാണ്ഡം 3, അദ്ധ്യായം 11 ) മന്ദകർണി മഹർഷിക്ക് അഞ്ച് അപ്സരസുകൾ (Nymphs) ഭാര്യമാരായിരുന്നു.

പ്രജാപതി അംഗിരയ്ക്ക് രണ്ട് ഭാര്യമാരുണ്ടായിരുന്നു.

ശ്രീമദ് ഭാഗവതം 6.6.19: പ്രജാപതി അംഗിരയ്ക്ക് സ്വധ എന്നും സതി എന്നും പേരുള്ള രണ്ട് ഭാര്യമാരുണ്ടായിരുന്നു. സ്വധ എന്ന ഭാര്യ എല്ലാ പിതാക്കന്മാരെയും പുത്രന്മാരായി സ്വീകരിച്ചു, സതി അഥർവാംഗിരസ വേദത്തെ പുത്രനായി സ്വീകരിച്ചു.

സൗഭരി മുനിക്ക് അമ്പത് ഭാര്യമാരുണ്ടായിരുന്നു.

സൗഭരി മുനിക്ക് അമ്പത് ഭാര്യമാരുണ്ടെന്ന് ശ്രീമദ് ഭാഗവതം 9.6.52 പറയുന്നു.

വിശ്രവാസന് നാല് ഭാര്യമാരുണ്ടായിരുന്നു.

വായുപുരാണം 9.32-34: “വിശ്രവാസൻ എന്ന മുനി ഇടവിടയിൽ ആണ് ജനിച്ചത്. പുലസ്ത്യ കുടുംബത്തെ അഭിവൃദ്ധിപ്പെടുത്താൻ അദ്ദേഹത്തിന് നാല് ഭാര്യമാരുണ്ടായിരുന്നു. ദേവാധ്യാപകനായ ബൃഹസ്പതിക്ക് ദേവിണിനി എന്ന പ്രശസ്തയായ ഒരു മകളുണ്ടായിരുന്നു. അവൻ (വിശ്രവാസൻ) ആ പെൺകുട്ടിയെ വിവാഹം കഴിച്ചു. അവൻ (വിശ്രവാസൻ) മാല്യവന്റെ പുത്രിമാരായ പുഷ്പ്ത്കതയെയും വാകയെയും മാലിൻ്റെ മകളായ കൈകസിയെയും വിവാഹം കഴിച്ചു. അവരുടെ സന്തതികളെ കേൾക്കുക.” (വിവ: ജി.വി. ടാഗാരെ, എഡിറ്റർ: ജി.പി. ഭട്ട്)

അത്രിക്ക് പത്ത് ഭാര്യമാരുണ്ടായിരുന്നു.

വായുപുരാണം 9.64: “മൂന്നാം പ്രജാപതിയായ അത്രിയുടെ വംശപരമ്പരയെ ഞാൻ ഇപ്പോൾ വിവരിക്കാം. അദ്ദേഹത്തിന് ശുദ്ധരും സുന്ദരികളുമായ പത്ത് ഭാര്യമാരുണ്ടായിരുന്നു. (വിവ: ജി.വി. ടാഗാരെ, എഡിറ്റർ: ജി.പി. ഭട്ട്)

ഭൃഗുവിന് രണ്ട് ഭാര്യമാരുണ്ടായിരുന്നു.

ബ്രഹ്മാണ്ഡ പുരാണം 2.3.1.74-76: “ഭ്രഗുവിന്റെ രണ്ട് ഭാര്യമാരും ജന്മനാ ഉത്തമ കുലീനരായിരുന്നു. അവർ സമാനതകളില്ലാത്തവരും ഗംഭീരവുമായിരുന്നു. (അവരിൽ ഒരാൾ) ദിവ്യ എന്ന പേരിൽ അറിയപ്പെടുന്ന ഹിരണ്യകശിപുവിന്റെ മകളായിരുന്നു. രണ്ടാമത്തേത് പൗലോമി ആയിരുന്നു, പുലോമന്റെ മകൾ. (ജി.വി. ടാഗരെ)

മാരീസി മുനിക്ക് നാല് ഭാര്യമാരുണ്ടായിരുന്നു, അവർ 60,000 കുട്ടികളെ ജനിപ്പിച്ചു.

ബ്രഹ്മപുരാണം 1.195-8: “ഉപാദാനവ, ഹയശിരസിന്റെ മകളും ശർമ്മിഷ്ഠ, വ്രസപർവ്വന്റെയും പുലോമനും കലകയും, വൈശ്വാനരന്റെ രണ്ടു പുത്രിമാരും ആയിരുന്നു. അവർ മാരീസിയുടെ ഭാര്യമാരായിരുന്നു. അവർക്ക് വലിയ ശക്തി ഉണ്ടായിരുന്നു, അവർ ധാരാളം കുട്ടികളെ പ്രസവിച്ചു. ദാനവനെ സന്തോഷിപ്പിച്ച അറുപതിനായിരം പുത്രന്മാർ അവർക്കുണ്ടായിരുന്നു…” (ബോർഡ് ഓഫ് സ്കോളേഴ്സ്, എഡിറ്റർ: ജെ.എൽ. ശാസ്ത്രി)

ബ്രഹ്മാണ്ഡപുരാണം 2.3.6.26-ലും പരാമർശിച്ചിട്ടുണ്ട്.

ബ്രഹ്മാവിന്റെ മകൻ മാരീചി, മാരീചിയുടെ മകൻ കശ്യപൻ. കശ്യപ ഋഷിക്ക് പതിമൂന്ന് ഭാര്യമാരുണ്ടായിരുന്നു, അവരെല്ലാം സഹോദരിമാരായിരുന്നു, കശ്യപിന് ദിതി, അദിതി എന്നിങ്ങനെ രണ്ട് പ്രധാന ഭാര്യമാരുണ്ടായിരുന്നു.

ശ്രീമദ് ഭാഗവതം 6.6.24-26: “…ഹേ പരീക്ഷിത് രാജാവേ, ഇപ്പോൾ കശ്യപന്റെ ഭാര്യമാരുടെ പേരുകൾ എന്നിൽ നിന്ന് കേൾക്കൂ, അവരുടെ ഗർഭപാത്രങ്ങളിൽ നിന്ന് പ്രപഞ്ചത്തിലെ മുഴുവൻ ജനസംഖ്യയും ഉണ്ടായി. അവർ പ്രപഞ്ചത്തിലെ മിക്കവാറും എല്ലാ ജനസംഖ്യയുടെയും അമ്മമാരാണ്, അവരുടെ പേരുകൾ കേൾക്കുന്നത് വളരെ ശുഭകരമാണ്. അവ അദിതി, ദിതി, ദനു, കാഷ്ട, അരിഷ്ട, സുരസ, ഇല, മുനി, ക്രോധവശ, താമ്ര, സുരഭി, സാരമ, തിമി… ആകുന്നു. ”

ദേവീഭാഗവതം 4.3.21-22: വ്യാസൻ പറഞ്ഞു:– ദക്ഷ പ്രജാപതിക്ക് ദിതി, അദിതി എന്നീ രണ്ട് പെൺമക്കളുണ്ടായിരുന്നു. ഉയർന്ന പദവിയിലുള്ള ഈ രണ്ടുപേരും കശ്യപനെ വിവാഹം കഴിച്ചു; അവർ അദ്ദേഹത്തിന് പ്രിയപ്പെട്ടവരായിരുന്നു. അദിതി ദേവരാജാവായ അതിശക്തനായ ഇന്ദ്രനെ പ്രസവിച്ചു. ദിതിയും ഇന്ദ്രന്റെ അതേ ശക്തിയും പ്രതാപവുമുള്ള ഒരു പുത്രനെ ആവശ്യപ്പെട്ടു.”

ബ്രഹ്മപുരാണം 1.164-5: “ഹേ പ്രമുഖ ബ്രാഹ്മണരേ, ഇപ്പോൾ പ്രജാപതി കശ്യപന്റെ ഭാര്യമാരുടെ പേരുകൾ ശ്രദ്ധിക്കുക. അദിതി, ദിതി, ദനു, അരിസ്ത, സുരസ, ഖസ, സുരഭി, വിനത, താമ്ര, ക്രോധവാസ, ഇര, കദ്രു, മുനി എന്നിവയാണ് അവ. ഹേ ബ്രാഹ്മണരേ, അവരിൽ നിന്ന് ജനിച്ച കുട്ടികളെ അറിയുക. (വിവ: ബോർഡ് ഓഫ് സ്കോളേഴ്സ്, ജെ.എൽ. ശാസ്ത്രി എഡിറ്റുചെയ്തത്.)

ബ്രഹ്മാണ്ഡ പുരാണത്തിൽ 2.3.3.56-ലും ഇത് പരാമർശിച്ചിട്ടുണ്ട്.

മത്സ്യപുരാണം 4.53-54: “താൻ സൃഷ്ടിച്ച പെൺകുട്ടികളിൽ നിന്ന് പത്ത് പേർ ധർമ്മത്തിനും പതിമൂന്ന് കശ്യപർക്കും…” (വിവ: താലുഖ്ദാർ ഓഫ് ഔദ്, എഡിറ്റർ: ബി.ഡി. ബസു)

അദിതിയുടെ മകൻ ആദിത്യന് നാല് ഭാര്യമാരുണ്ടായിരുന്നു.

കൂർമ്മപുരാണം I.20.1-2: “അദിതി തന്റെ മകനായ ഭഗവാൻ ആദിത്യനെ (സൂര്യദേവൻ) കശ്യപനിൽ നിന്ന് പ്രസവിച്ചു. ഈ ആദിത്യന് നാല് ഭാര്യമാരുണ്ടായിരുന്നു. അവർ സംജ്ഞ, രജനി, പ്രഭ, ഛായ എന്നിവരായിരുന്നു…” (വിവ: ജി.വി. ടാഗരെ)

ആദിത്യയ്ക്ക് മൂന്ന് ഭാര്യമാരുണ്ടായിരുന്നു, നാലാമത്തെ ഭാര്യ ഛായ, സംജ്ഞയുടെ നിഴലായിരുന്നു.

അദിതിയുടെ ഏഴാമത്തെ പുത്രനും നാല് ഭാര്യമാരുണ്ടായിരുന്നു.

ശ്രീമദ് ഭാഗവതം 6.18.3-4: അദിതിയുടെ ഏഴാമത്തെ പുത്രനായ ധാതയ്ക്ക് കുഹു, സിനിവാലി, രാക, അനുമതി എന്നിങ്ങനെ നാല് ഭാര്യമാരുണ്ടായിരുന്നു. ഈ ഭാര്യമാർ യഥാക്രമം സായം, ദർശ, പ്രാതഃ, പൂർണ്ണമാസ എന്നീ നാല് പുത്രന്മാരെ ജനിപ്പിച്ചു. അദിതിയുടെ എട്ടാമത്തെ പുത്രനായ വിധാതയുടെ ഭാര്യയുടെ പേര് ക്രിയ എന്നാണ്. അവളുടെ വിധാതയിൽ പുരിഷ്യൻമാർ എന്ന് പേരുള്ള അഞ്ച് അഗ്നിദേവന്മാരെ ജനിപ്പിച്ചു. അദിതിയുടെ ഒമ്പതാമത്തെ പുത്രനായ വരുണന്റെ ഭാര്യയുടെ പേര് കർഷാണി എന്നാണ്. ബ്രഹ്മാവിന്റെ പുത്രനായ ഭൃഗു അവളുടെ ഉദരത്തിൽ വീണ്ടും ജനിച്ചു.

ക്ഷത്രിയർക്കിടയിൽ ബഹുഭാര്യത്വം കൂടുതൽ പ്രബലമായിരുന്നു, അത് ഹിന്ദു വാക്താക്കൾ തന്നെ അംഗീകരിക്കുന്നു. ഋഗ്വേദവും അതുതന്നെയാണ് കാണിക്കുന്നത്.

ഋഗ്വേദം 6.18.2: “തന്റെ ഭാര്യമാരിൽ ഒരു രാജാവിനെപ്പോലെ നീ വസിക്കുന്നു: മഹത്വങ്ങളോടെ, ഒരു മഹർഷിയെപ്പോലെ, ഞങ്ങളെ വളയുകയും സഹായിക്കുകയും ചെയ്യുക…”

ദേവീഭാഗവതം പുസ്തകം 2, അദ്ധ്യായം 7 അനുസരിച്ച്, സാധാരണ ഭാര്യയായ ദ്രൗപതിയെ കൂടാതെ, അർജുനന് കൃഷ്ണന്റെ സഹോദരിയായ സുഭദ്ര എന്ന പേരിൽ ഒരു ഭാര്യ കൂടി ഉണ്ടായിരുന്നു. കൃഷ്ണന്റെ സമ്മതപ്രകാരം അർജുനൻ അവളെ ബലപ്രയോഗത്തിലൂടെ തട്ടിക്കൊണ്ടുപോയി.

വേദപുത്രനായ ഹരിശ്ചന്ദ്ര രാജാവിന് നൂറ് ഭാര്യമാരുണ്ടായിരുന്നു.

ഐതരേയ ബ്രാഹ്മണൻ, അദ്ധ്യായം 3, ഖണ്ഡിക 13, ഇക്ഷവാകു വംശത്തിൽപ്പെട്ട വേധസിന്റെ മകൻ ഹരിശ്ചന്ദ്ര, പുത്രനില്ലാത്ത രാജാവായിരുന്നു. അദ്ദേഹത്തിന് നൂറ് ഭാര്യമാരുണ്ടായിട്ടും അവർക്ക് ഒരു പുത്രൻ ജനിച്ചില്ല.

പാണ്ഡുവിന് കുന്തിയും മാദ്രിയും എന്നീ രണ്ട് ഭാര്യമാരുണ്ടായിരുന്നു.

പത്മപുരാണം II.79.1-2 പ്രകാരം യയാതി രാജാവിന് മൂന്ന് ഭാര്യമാരുണ്ടെന്ന് പറയപ്പെടുന്നു.

രാജാക്കന്മാർക്ക് നിരവധി ഭാര്യമാരുണ്ടെന്നതിന് ഡസൻ കണക്കിന് ഉദാഹരണങ്ങളുണ്ട്, പക്ഷേ ഞാൻ അത് കുറച്ച് മാത്രമായി പരിമിതപ്പെടുത്തട്ടെ.

ബഹുഭാര്യത്വം അനുവദിക്കുന്ന ഹിന്ദു ഗ്രന്ഥങ്ങൾ

വിഖ്യാത ഹിന്ദു പണ്ഡിതനായ വിജ്ഞാനേശ്വരൻ എഴുതുന്നു:

“വർഗങ്ങളുടെ ക്രമമനുസരിച്ച്, ബ്രാഹ്മണർക്ക് മൂന്ന്, ക്ഷത്രിയർക്ക് രണ്ട്, വൈശ്യർക്ക് ഒരു ഭാര്യ എന്നാണ് വിധി. ഒരു ശൂദ്രന് ഒരേ ജാതിയിൽ ജനിച്ച ഒരു ഭാര്യ മാത്രമേ ഉണ്ടാകാവൂ. (യാജ്ഞവൽക്യ സ്മൃതിയുടെ മിതാക്ഷര വ്യാഖ്യാനത്തിൽ വിജ്ഞാനേശ്വരൻ 3: 57)

ബഹുഭാര്യത്വത്തിൽ ഒരു പ്രശ്നവുമില്ലെന്ന് സ്വാമി വിവേകാനന്ദൻ അഭിപ്രായപ്പെട്ടു:

“സ്ത്രീകളെ സംബന്ധിച്ചിടത്തോളം, അവർ പവിത്രതയെ ഏറ്റവും പ്രധാനപ്പെട്ട പുണ്യമായി കണക്കാക്കുന്നു, സംശയമില്ല. ഒരേ സമയം ഒന്നിലധികം ഭർത്താക്കന്മാരുള്ള ഒരു സ്ത്രീ എന്നതോളം ഒരു പുരുഷൻ ഒന്നിലധികം ഭാര്യമാരെ വിവാഹം ചെയ്യുന്നത് സമൂഹത്തിന് അത്ര ദോഷകരമല്ല, കാരണം ഒന്നാമത്തേത് വംശത്തിന്റെ ക്രമാനുഗതമായ അപചയത്തിലേക്ക് നയിക്കുന്നു.”

The Complete Works of Swami Vivekananda/Volume 5/Writings: Prose and Poems/The East and The West/France-Paris https://en.wikisource.org/wiki/The_Complete_Works_of_Swami_Vivekananda/Volume_5/Writings:_Prose_and_Poems/The_East_and_The_West/France-Paris

ബഹുഭാര്യത്വത്തെക്കുറിച്ചുള്ള ISKCON സ്ഥാപകൻ സ്വാമി പ്രഭുപാദ

“…നമ്മുടെ വേദപ്രക്രിയ അനുസരിച്ച് ബഹുഭാര്യത്വം അനുവദനീയമാണ്. ഉദാഹരണത്തിന്, കൃഷ്ണൻ 16,000 ഭാര്യമാരെ വിവാഹം കഴിച്ചു, അർജുനൻ 3 അല്ലെങ്കിൽ 4 ഭാര്യമാരെ വിവാഹം കഴിച്ചു, കൃഷ്ണന്റെ പിതാവ് വാസുദേവൻ, 16 അല്ലെങ്കിൽ 18 ഭാര്യമാരെ വിവാഹം കഴിച്ചു. അതിനാൽ വൈദിക സമ്പ്രദായമനുസരിച്ച് ബഹുഭാര്യത്വം നിഷിദ്ധമല്ല…” (Swami Prabhupada, Letter to Karandhara written from Bombay http://vanisource.org/wiki/730109_-_Letter_to_Karandhara_written_from_Bombay)

“ഒരു വശത്ത് ബഹുഭാര്യത്വത്തെ എതിർക്കുകയും മറുവശത്ത് അവർ സ്ത്രീകളെ പല തരത്തിൽ വേട്ടയാടുകയും ചെയ്യുന്ന തരത്തിൽ ഈ കാലഘട്ടത്തിൽ ആളുകൾ വളരെ അധഃപതിച്ചിരിക്കുന്നു. ഈ ക്ലബ്ബിലോ ആ കടയിലോ ടോപ്‌ലെസ്സ് പെൺകുട്ടികൾ ലഭ്യമാണെന്ന് പല ബിസിനസ്സ് സംഘങ്ങളും പരസ്യമായി പരസ്യം ചെയ്യുന്നു. അങ്ങനെ സ്ത്രീകൾ ആധുനിക സമൂഹത്തിൽ ഇന്ദ്രിയ ആസ്വാദനത്തിന്റെ ഉപകരണങ്ങളായി മാറിയിരിക്കുന്നു. ബ്രാഹ്മണർ, ക്ഷത്രിയർ, വൈശ്യർ, എന്തിനേറെ ശൂദ്രർ പോലും ഒരു ഉയർന്ന സാമൂഹിക ക്രമത്തിലുള്ള പുരുഷന്മാർ എന്ന നിലയിൽ, ഒന്നിലധികം ഭാര്യമാരെ ആസ്വദിക്കാനുള്ള പ്രവണത പുരുഷനുണ്ടാകാൻ ഇടയുള്ളത് പോലെ ഉണ്ടായാൽ, കൂടുതൽ വിവാഹം കഴിക്കാൻ വേദങ്ങൾ അനുശാസിക്കുന്നു. ഒന്നിലധികം ഭാര്യമാരെ അവന് വിവാഹം ചെയ്യാൻ അനുവദിച്ചു. വിവാഹം എന്നാൽ ഒരു സ്ത്രീയുടെ പൂർണ്ണമായ ഉത്തരവാദിത്തം ഏറ്റെടുക്കുകയും വ്യഭിചാരത്തെ സമീപിക്കാതെ സമാധാനത്തോടെ ജീവിക്കുകയും ചെയ്യുക എന്നതാണ്. ഇന്ന്, വ്യഭിചാരം അനിയന്ത്രിതമായി നടക്കുന്നു. അതേസമയം, ഒന്നിലധികം ഭാര്യമാരെ വിവാഹം ചെയ്യാൻ പാടില്ലെന്ന നിയമം സമൂഹം ഉണ്ടാക്കുന്നു. ഇത് ഒരു പൈശാചിക സമൂഹത്തിന്റെ സവിശേഷതയാണ്. ”

(Swami Prabhupada on Srimad Bhagavatam 4.26.6 http://vanisource.org/wiki/SB_4.26.6)

അദ്ദേഹം എഴുതുന്നു,

“ഭാര്യ ഗർഭിണിയായിരിക്കുമ്പോൾ ലൈംഗികത ആസ്വദിക്കാൻ കഴിയാത്തതിനാൽ ഒരു പുരുഷന് ഒന്നിലധികം ഭാര്യമാരെ സൂക്ഷിക്കാൻ അനുവാദമുണ്ട്. അത്തരമൊരു സമയത്ത് ലൈംഗികത ആസ്വദിക്കണമെങ്കിൽ അയാൾ ഗർഭിണിയല്ലാത്ത മറ്റൊരു ഭാര്യയുടെ അടുത്തേക്ക് പോയേക്കാം. മനു-സംഹിതയിലും മറ്റു ഗ്രന്ഥങ്ങളിലും പറഞ്ഞിരിക്കുന്ന നിയമങ്ങളാണിവ.”

(Swami Prabhupada on Srimad Bhagavatam 4.27.5 http://vanisource.org/wiki/SB_4.27.5)

ഹിന്ദുമതത്തിലെ ഏറ്റവും പൗരാണിക വേദഗ്രന്ഥമായ വേദങ്ങൾ ബഹുഭാര്യത്വത്തെ അനുവദിക്കുകയോ നിരോധിക്കുകയോ ചെയ്യുന്നില്ല. എന്നാൽ ബഹുഭാര്യത്വത്തിന്റെ അടയാളങ്ങൾ വേദങ്ങളിൽ കാണാം, അത് ബഹുഭാര്യത്വം വേദ കാലഘട്ടത്തിൽ പ്രയോഗിക തലത്തിൽ ഉണ്ടായിരുന്നുവെന്ന് സൂചിപ്പിക്കുന്നു, ഞാൻ ശ്രീരാം ശർമ്മയുടെ ഹിന്ദി വിവർത്തനമാണ് ഉപയോഗിക്കുന്നത്.

ഋഗ്വേദം 5.42.12 ശ്ലോകത്തിൽ ഇന്ദ്രന് മൂന്ന് ഭാര്യമാരുണ്ടെന്ന് പറയുന്നു.

മൈത്രായണി സംഹിത 3.11.1 [140, 10-11] ഇന്ദ്രന്റെ ഭാര്യമാരായ സരസ്വതിയെയും, ഇഡയെയും, ഭാരതിയെയും സംബന്ധിച്ച് പരാമർശിക്കപ്പെടുന്നു. (വിവ. കാതറിൻ ലുഡ്വിക്)

വേദങ്ങൾ, ബഹുഭർതൃത്വത്തെയും അംഗീകരിക്കുന്നതായി തോന്നുന്നു, ഋഗ്വേദം 6.49.7 സരസ്വതിയെ ഇന്ദ്രന്റെ ഭാര്യയായി കണക്കാക്കുന്നു, ഇത് വൈരുദ്ധ്യമല്ലെങ്കിൽ മുകളിലുള്ള വാക്യങ്ങൾ (ബ്രഹ്‌മാവിന്റെ ഭാര്യയാണ് സരസ്വതി എന്ന് വാദിക്കുന്ന വാക്യങ്ങൾ) വൈരുദ്ധ്യമാണെന്ന് പറയേണ്ടിവരും. യജുർവേദം 19.94-ൽ സരസ്വതിയെ അശ്വിനി സഹോദരന്മാരുടെ ഭാര്യ എന്നും വിളിക്കുന്നു, ഇത് വ്യക്തമായും ബഹുഭർതൃത്വത്തിന്റെ കേസാണ്.

ഋഗ്വേദം 10.43.1. “സ്വർഗ്ഗത്തിന്റെ വെളിച്ചം കണ്ടെത്തുന്ന, എന്റെ എല്ലാ സ്തുതികളും തികഞ്ഞ ഐക്യത്തോടെ ഇന്ദ്രനെ സ്തുതിച്ചു. ഭാര്യമാർ തങ്ങളുടെ യജമാനനായ സുന്ദരനായ വരനെ ആലിംഗനം ചെയ്യുന്നതുപോലെ, അവർ മഘവാനെ സഹായത്തിനു വേണ്ടി ആലിംഗനം ചെയ്യുന്നു.”

https://vedkabhed.files.wordpress.com/2016/05/rig-veda-10-43-1.png?w=517&h=93

ബഹുഭാര്യത്വത്തെ സ്തുതിക്കുന്ന മറ്റൊരു വാക്യത്തിന്റെ വ്യക്തമായ പരിഭാഷ:

ഋഗ്വേദം 4.58.8 “ഗിയുടെ പ്രവാഹങ്ങൾ, പുഞ്ചിരിക്കുന്ന, അർപ്പണബോധമുള്ള ഭാര്യമാരായി അഗ്നിയിലേക്ക് ചായുന്നു: അവർ ഇന്ധനം പോലെ (ജ്വാലയെ) പോഷിപ്പിക്കുന്നു, കൂടാതെ ജാതവേദങ്ങൾ അവരെ പ്രീതിപ്പെടുത്തി സ്വീകരിക്കുന്നു.” (വിവ: എച്ച്.എച്ച്. വിൽസൺ)

കൃഷ്ണ യജുർവേദം 6.5.1.4 … “അതിനാൽ ഒരാൾ പോകുമ്പോൾ പലരും പിന്തുടരുന്നു; അതിനാൽ ഒരുവൻ പലരിലും ശ്രേഷ്ഠനാകുന്നു; അതിനാൽ ഒരാൾ ധാരാളം ഭാര്യമാരെ നേടുന്നു …”

ഒരു പഴയ ഋഷിയായ ച്യവന താൻ പുനരുജ്ജീവിപ്പിച്ചതിന് ശേഷം നിരവധി പെൺകുട്ടികളെ വിവാഹം കഴിച്ചിരുന്നു എന്നാണ് ഋഗ്വേദത്തിലെ ഒരു വാക്യം വ്യക്തമായി കാണിക്കുന്നത്.

ഋഗ്വേദം 1.116.10 “ഹേ നാസത്യരേ, പഴയ ച്യവനത്തിൽ നിന്ന്, ശരീരത്തിലെ തൊലി ഉരിഞ്ഞുകളഞ്ഞു, എല്ലാവരും അവനെ നിസ്സഹായനാക്കിയപ്പോൾ അവന്റെ ആയുസ്സ് നീട്ടപ്പെട്ടു, ദാസരേ! അവനെ യൗവനക്കാരിയായ കന്യകമാരുടെ അധിപതിയാക്കി.”

https://vedkabhed.files.wordpress.com/2016/05/rig-veda-1-116-10.png?w=554&h=120

മുകളിൽ ശ്രീരാം ശർമ്മയുടെ ഹിന്ദി വേദവിവർത്തനവും തുടർന്നുള്ള വിവർത്തനം പണ്ഡിറ്റ് രാം ഗോവിന്ദ് ത്രിവേദിയുടേതുമാണ്.

https://vedkabhed.files.wordpress.com/2016/05/rig-veda-1-116-10-govind-trivedi.png?w=501&h=122

നിരുക്തം 4.19, മഹാഭാരതം ആദിപർവ്വം 1.177, പഞ്ചവിംസ ബ്രാഹ്മണം 14.6.10 എന്നിവയിലും ച്യവന പുനരുജ്ജീവിപ്പിക്കപ്പെട്ടതായി പരാമർശിക്കപ്പെടുന്നു, എന്നാൽ അത് അവന്റെ വിവാഹത്തെക്കുറിച്ച് ഒന്നും പറയുന്നില്ല. ഇവിടെ കന്യകകൾ എന്നത് നിരവധി കന്യകമാരെയോ രണ്ട് കന്യകമാരെയോ സൂചിപ്പിക്കാം. ഹിന്ദു ഗ്രന്ഥങ്ങളിൽ അദ്ദേഹത്തിന്റെ രണ്ട് ഭാര്യമാരെ മാത്രമേ പരാമർശിക്കുന്നുള്ളൂ. ച്യവനന്റെ ഭാര്യമാരെക്കുറിച്ച് തിരുവെഴുത്തുകൾ കൂടുതൽ വിശദാംശങ്ങൾ നൽകുന്നില്ല, കൂടാതെ സരയാതി രാജാവിന്റെ മകളായ സുകന്യയെ കൂടുതൽ ഊന്നിപ്പറയുന്നു. ശ്രീമദ് ഭാഗവതം 9.3-ൽ പരാമർശിച്ചിരിക്കുന്നതുപോലെ സുകന്യ ച്യവനയുടെ ഭാര്യയായി, മറ്റ് ഗ്രന്ഥങ്ങൾ മനുവിന്റെ മകളായ ആരുഷി എന്ന മറ്റൊരു ഭാര്യയെക്കുറിച്ച് സംസാരിക്കുന്നു, അദ്ദേഹത്തിന് ഔർവ എന്നൊരു മകനെ പ്രസവിച്ചു, ഇത് മഹാഭാരതം ആദിപർവ്വം 1.66.47 ൽ പരാമർശിച്ചിരിക്കുന്നുണ്ട്.

മറ്റ് ഹിന്ദു ഗ്രന്ഥങ്ങൾ ബഹുഭാര്യത്വത്തെക്കുറിച്ച് വ്യക്തമായ നിലപാടാണ് സ്വീകരിക്കുന്നത്. അവ ഒരു സംശയവുമില്ലാതെ ബഹുഭാര്യത്വം അനുവദിക്കുന്നു.

മനു സ്മൃതി 3.12-13: പുരുഷന്മാരോട് തുല്യ ജാതിയിൽ നിന്നുള്ള ആദ്യ വിവാഹത്തിന് ശുപാർശ ചെയ്യുന്നു; എന്നാൽ (വീണ്ടും വിവാഹം കഴിക്കാൻ) ആഗ്രഹിക്കുന്നവർ താഴെപ്പറയുന്ന (ജാതികളിൽ പെട്ട) സ്ത്രീകളെ ക്രമപ്രകാരം വിവാഹം കഴിക്കുന്നത് അംഗീകരിക്കുന്നു. ഒരു ശൂദ്ര സ്ത്രീക്ക് ഒറ്റയ്ക്ക് ശൂദ്രന്റെ ഭാര്യയാകാം, അവളെയും വൈശ്യന്റെ സ്വന്തം ജാതിയിൽപ്പെട്ട ഒരാളെയും വൈശ്യന് വിവാഹം ചെയ്യാം, ആ രണ്ടുപേരും ഒപ്പം തന്റെ സ്വന്തം ജാതിയിൽപ്പെട്ട ഒരാളെയും ക്ഷത്രിയന് വിവാഹം ചെയ്യാം. ഈ മൂന്ന് പേരേയും, സ്വന്തം ജാതിയിൽ പെട്ട ഒരാളെയും ബ്രാഹ്മണന് വിവാഹം ചെയ്യാം.

അഗ്നിപുരാണം 285:63-67 “…നിരവധി ഭാര്യമാരുള്ള ഒരു ഭർത്താവ്, എല്ലാ ദിവസവും, ത്രിഫല, പിപ്പലി, തേൻ, വെണ്ണ, പൊടിച്ച അമലാക്കി, അതേ പഴത്തിന്റെ നീര് എന്നിവ ചേർത്ത ഒരു സിറപ്പ് നാവ് കൊണ്ട് നക്കി കഴിക്കണം. ശേഷം വെള്ളം കുടിക്കൂ…” (വിവ: എം.എൻ. ദത്ത്)

മനു സ്മൃതി 9.85: “ദ്വിജാതിയനായി ജനിച്ച പുരുഷന്മാർ അവരുടേതും മറ്റേ (താഴ്ന്ന ജാതിയിലുള്ള) സ്ത്രീകളെയും വിവാഹം കഴിച്ചാൽ, അവരുടെ (ഭാര്യമാരുടെ) സീനിയോറിറ്റി, ബഹുമാനം, വാസസ്ഥലം എന്നിവ ജാതികളുടെ (വർണ്ണ) ക്രമം അനുസരിച്ചായിരിക്കണം നിശ്ചയിക്കേണ്ടത്.”

മഹാഭാരതം 1.160.36: “ഇതിൽ പാപമില്ല. പുരുഷനെ സംബന്ധിച്ചിടത്തോളം ബഹുഭാര്യത്വം ഒരു പുണ്യ പ്രവൃത്തിയാണ്, എന്നാൽ ഒരു സ്ത്രീയെ സംബന്ധിച്ചിടത്തോളം ആദ്യ ഭർത്താവിന് ശേഷം രണ്ടാമത്തെ ഭർത്താവിനെ സ്വീകരിക്കുന്നത് വളരെ പാപമാണ്. (വിവ. കെ.എം. ഗാംഗുലി)

മഹാഭാരതം 14.80.12-18: “ഓ, വിജയാ, ഗുഡകേശൻ എന്ന് വിളിക്കപ്പെടുന്നവൻ, ചുവന്ന കണ്ണുകളുള്ള ഈ വീരൻ മടങ്ങിവരട്ടെ, ജീവിതമേ. ഹേ ഭാഗ്യവതി, ബഹുഭാര്യത്വം പുരുഷന്മാരുടെ കുറ്റമല്ല. ഒന്നിലധികം ഭർത്താക്കന്മാരെ സ്വീകരിക്കുന്നതിലൂടെ മാത്രമേ സ്ത്രീകൾക്ക് തെറ്റ് സംഭവിക്കൂ.” (വിവ: കെ.എം. ഗാംഗുലി)

മഹാഭാരതം 1.197.27-28: “ദ്രുപദൻ മറുപടി പറഞ്ഞു, ‘ഓ കുരുവിന്റെ വംശത്തിലെ സന്തതി, ഒരു പുരുഷന് അനേകം ഭാര്യമാരുണ്ടാകാമെന്ന് നിർദ്ദേശിക്കപ്പെട്ടിരിക്കുന്നു. എന്നാൽ ഒരു സ്‌ത്രീക്ക്‌ അനേകം ഭർത്താക്കന്മാരുണ്ടാകുമെന്ന്‌ ഇതുവരെ കേട്ടിട്ടില്ല!” (വിവ: കെ.എം. ഗാംഗുലി)

ഭർത്താവിന് നിരവധി ഭാര്യമാരുണ്ടെങ്കിൽ ഏത് ഭാര്യയാണ് അവനോടൊപ്പം മതപരമായ കർത്തവ്യങ്ങൾ ചെയ്യേണ്ടതെന്ന് ഹിന്ദു ഗ്രന്ഥങ്ങളും നിർദ്ദേശങ്ങൾ നൽകുന്നുണ്ട്.

വിഷ്ണു സ്മൃതി 26.1-4: “ഒരു പുരുഷന് സ്വന്തം ജാതിയിൽ പെട്ട നിരവധി ഭാര്യമാരുണ്ടെങ്കിൽ, അവൻ മൂത്ത (അല്ലെങ്കിൽ ആദ്യ വിവാഹിത) ഭാര്യയോടൊപ്പം തന്റെ മതപരമായ കർത്തവ്യങ്ങൾ നിർവഹിക്കണം. (അവന് നിരവധി) വിവിധ ജാതികളിൽപ്പെട്ട ഭാര്യമാർ ഉണ്ടെങ്കിൽ സ്വജാതിയിൽ പെട്ടവളുമായി അവൻ അത് അനുഷ്ഠിക്കണം; അവൾ ഇളയഭാര്യ ആണെങ്കിൽ പോലും. സ്വന്തം ജാതിയിലുള്ള ഒരു ഭാര്യ അനുഷ്ഠാനത്തിൽ പരാജയപ്പെട്ടാൽ (അവൻ അവ നിർവഹിക്കേണ്ടത്) തന്റെ സ്വന്തം ജാതിയുടെ തൊട്ട് താഴെയുള്ള ജാതിയിൽപ്പെട്ടവളുമായിട്ടാണ്…

കാത്യായന സംഹിത 8.6: “ഒരേ ജാതിയിൽപ്പെട്ടവരും നിലവിലുള്ള മറ്റ് ജാതികളിൽപ്പെട്ടവരുമായ അനേകം ഭാര്യമാർ ഉണ്ടാകുന്ന സന്ദർഭത്തിൽ, അഗ്നിയെ ഉൽപ്പാദിപ്പിക്കുന്നതിനുള്ള ചടങ്ങ്, ജന്മത്തിന്റെ ശ്രേഷ്ഠത കണക്കിലെടുത്ത്, ഒരേ ജാതിയിലുള്ള ശുദ്ധിയുള്ള ഭാര്യമാരാൽ ചെയ്യണം.” (വിവ: മന്മഥ നാഥ് ദത്ത്)

നാല് ഭാര്യമാർ:

ഹിന്ദു ഗ്രന്ഥങ്ങളിലെ ബഹുഭാര്യത്വത്തെ സംബന്ധിച്ച ഏക ആശയക്കുഴപ്പം ചില ഹിന്ദു ഗ്രന്ഥങ്ങൾ ഒരു ബ്രാഹ്മണന് നാല് സ്ത്രീകളെ വിവാഹം കഴിക്കാൻ അനുവാദം നൽകുന്നു, ചിലത് മൂന്ന് സ്ത്രീകളെ മാത്രമേ അനുവദിക്കൂ എന്നതാണ്. ഒരു ബ്രാഹ്മണന് നാല് ഭാര്യമാരും, ഒരു ക്ഷത്രിയന് മൂന്ന് ഭാര്യമാരും, ഒരു വൈശ്യന് രണ്ട്, ശൂദ്രന് ഒന്ന് മാത്രം അനുവദനീയമാണ് എന്നിങ്ങനെ നിർദ്ദേശിക്കുന്ന ശ്ലോകങ്ങൾ നിരവധിയാണ്.”

മഹാഭാരതം 13.47.4:

“അല്ലയോ മുത്തശ്ശി, ഒരു ബ്രാഹ്മണന് നാല് ഭാര്യമാരെ എടുക്കാം. അതായത്, സ്വന്തം ക്രമത്തിൽ പെട്ടവളെ, ഒരു ക്ഷത്രിയയെ, ഒരു വൈശ്യയെ, ഒരു ശൂദ്ര സ്ത്രീയെ എന്നിങ്ങനെയുള്ള നാല് ഭാര്യമാരെ സ്വീകരിക്കാം എന്ന് പറഞ്ഞിരിക്കുന്നു; ബ്രാഹ്മണന് ലൈംഗികബന്ധത്തിൽ ഏർപ്പെടാൻ ആഗ്രഹമുണ്ടെങ്കിൽ.” (വിവ: കെ.എം. ഗാംഗുലി)

മഹാഭാരതം 13.48.4 : “ബ്രാഹ്മണന് നാല് ജാതികളിൽ നിന്ന് ഓരോരുത്തർ വീതം നാല് ഭാര്യമാരെ സ്വീകരിക്കാം. അവയിൽ രണ്ടിൽ (അതായത്, ഭാര്യ തന്റെ സ്വന്തം ജാതിയിൽ നിന്ന് എടുത്തതും താഴെയുള്ളതിൽ നിന്ന് എടുത്തതും), അവൻ തന്നെ ജനിക്കുന്നു (അവരിൽ ജനിക്കുന്ന കുട്ടികൾ തന്റേതിന് സമാനമായ പദവിയിൽ നിക്ഷേപിക്കപ്പെട്ടവരായി കണക്കാക്കപ്പെടുന്നു)… ഒരു ക്ഷത്രിയൻ മൂന്ന് ഭാര്യമാരെ സ്വീകരിക്കാം… വൈശ്യന് രണ്ട് ഇണകളെ സ്വീകരിക്കാം…ശൂദ്രന് ഒരു ഭാര്യയെ മാത്രമേ സ്വീകരിക്കാൻ കഴിയൂ, അതായത്, സ്വന്തം ജാതിയിൽ നിന്ന് സ്വീകരിക്കപ്പെട്ടവൾ. അവളിൽ അവനാൽ ജനിച്ച മകൻ ശൂദ്രനാകുന്നു…” (വിവ: കെ.എം. ഗാംഗുലി)

അഗ്നിപുരാണം 153.1: “പുഷ്കരൻ പറഞ്ഞു:- ഒരു ബ്രാഹ്മണന് നാല് ഭാര്യമാരെയും, ഒരു ക്ഷത്രിയൻ മൂന്നും, ഒരു വൈശ്യന് രണ്ട് ഭാര്യമാരെയും, ശൂദ്ര ജാതിയിൽപ്പെട്ട ഒരാൾക്ക് ഒരു ഭാര്യയിൽ കൂടുതലും പാടില്ല.” (വിവ: എം.എൻ. ദത്ത്)

മനു സ്മൃതി 9.149: “ജാതികളുടെ നേരിട്ടുള്ള ക്രമത്തിൽ ഒരു ബ്രാഹ്മണന്റെ നാല് ഭാര്യമാരുണ്ടെങ്കിൽ, അവരിൽ ജനിച്ച പുത്രന്മാർക്കിടയിൽ (ഭൂമി) വിഭജിക്കാനുള്ള നിയമം ഇപ്രകാരമാണ്:… ”

വിഷ്ണു സ്മൃതി 24.1-5: “ഇപ്പോൾ ഒരു ബ്രാഹ്മണന് (നാല്) ജാതികളുടെ നേരിട്ടുള്ള ക്രമത്തിൽ നാല് ഭാര്യമാരെ എടുക്കാം; ഒരു ക്ഷത്രിയൻ, മൂന്ന്; ഒരു വൈശ്യന് രണ്ട്; ഒരു ശൂദ്രൻ, ഒരാൾ മാത്രം.”

ബൗധ്യാന ധർമ്മ ശാസ്ത്രം, പ്രശ്നം I, അധ്യായ 8, കണ്ടിക 16, വാക്യങ്ങൾ 1-5: ബ്രാഹ്മണർ, ക്ഷത്രിയർ, വൈശ്യർ, ശൂദ്രർ എന്നിങ്ങനെ നാല് ജാതികളുണ്ട്. (പുരുഷന്മാർ) ജാതികളുടെ ക്രമമനുസരിച്ച് ഭാര്യമാരെ സ്വീകരിക്കാം, (അതായത്) ഒരു ബ്രാഹ്മണൻ നാല്, ഒരു ക്ഷത്രിയൻ മൂന്ന്, ഒരു വൈശ്യൻ രണ്ട്, ഒരു ശൂദ്രൻ ഒന്ന്.

മൂന്ന് ഭാര്യമാർ:

പരസ്കര ഗൃഹ്യസൂത്രം I കാണ്ഡ, 4 കാണ്ഡിക, 8-11:

ബ്രാഹ്മണന് മൂന്ന് (ഭാര്യമാർ അനുവദനീയമാണ്), ജാതികളുടെ ക്രമം അനുസരിച്ച്, രാഗണ്യന് രണ്ട്, വൈശ്യന് ഒരാൾ…

യാജ്ഞവൽക്യ സ്മൃതി 3.57:

“ജാതിയുടെ ക്രമമനുസരിച്ച്, മൂന്ന്, രണ്ട്, ഒന്ന് യഥാക്രമം ബ്രാഹ്മണനും ക്ഷത്രിയനും വൈശ്യനും (ഭാര്യമാരായിരിക്കാം). ശൂദ്രനായി ജനിച്ച ഒരാൾക്ക് സ്വന്തം ജാതിയിൽപ്പെട്ട പെൺകുട്ടിയാണ് ഭാര്യ.” ശ്രീസ ചന്ദ്ര വാസു

ഇതേ കാര്യം തന്നെ ശംഖ സംഹിത 4.7,വസിഷ്ഠ സംഹിത 1.24, മഹാഭാരതം 13.44.11-12 എന്നിവയിലും ആവർത്തിക്കുന്നു…”

(https://vedkabhed.com/index.php/2018/05/05/polygamy-in-hindu-dharma/)

(ഹിന്ദു ധർമ്മശാസ്ത്രത്തിലെ ബഹുഭാര്യത്വത്തെ സംബന്ധിച്ച് ഇത്രയും വിവരിച്ചത് സുലൈമാൻ റസ്‌വിയുടെ polygamy in Hindu Dharma എന്ന ലേഖനത്തിൽ നിന്നാണ്. കൂടുതൽ വിശദാംശങ്ങളും പരിശോധനകളും പ്രസ്തുത ലേഖനത്തിൽ നിന്ന് നേരിട്ട് കാണുക.)

ഇതൊക്കെ ന്യായീകരിച്ചും വെളുപ്പിച്ചും ക്ഷീണിതരായി, രണ്ട് ദിവസം നന്നായി ഉറങ്ങി, എഴുന്നേറ്റിട്ട് പോരെ മുഹമ്മദ് നബിയുടെ(സ) പരിശുദ്ധവും മാതൃകാപരവുമായി ദാമ്പത്യങ്ങളെ കാമ പ്രേരിതമായ രതിവേട്ടകളായി വക്രീകരിക്കാൻ ?!

മുഹമ്മദ് നബിയെ (സ) സംബന്ധിച്ചിടത്തോളം ഒരു സ്ത്രീയേയും അദ്ദേഹം ബലം പ്രയോഗിച്ച് വിവാഹം ചെയ്തിട്ടില്ല. വിവാഹം ചെയ്തവരോട് തന്നെ വേണമെങ്കിൽ മാത്രം തന്റെ കൂടെ ജീവിച്ചാൽ മതിയെന്ന് പരസ്യമായി പ്രഖ്യാപിച്ചിട്ടുമുണ്ട്:

“നബിയേ, നിന്‍റെ ഭാര്യമാരോട് നീ പറയുക: ഐഹികജീവിതവും അതിന്‍റെ അലങ്കാരവുമാണ് നിങ്ങള്‍ ഉദ്ദേശിക്കുന്നതെങ്കില്‍ നിങ്ങള്‍ വരൂ! നിങ്ങള്‍ക്ക് ഞാന്‍ ജീവിതവിഭവം നല്‍കുകയും, ഭംഗിയായ നിലയില്‍ ഞാന്‍ നിങ്ങളെ മോചിപ്പിച്ച് അയച്ചുതരികയും ചെയ്യാം.” (ക്വുർആൻ:33: 28)

വലിച്ചിഴച്ച് പുറത്തിടുമെന്നല്ല പ്രഖ്യാപനം, “ഭംഗിയായ നിലയില്‍ ഞാന്‍ നിങ്ങളെ മോചിപ്പിച്ച് അയച്ചുതരികയും ചെയ്യാം” എന്നാണ്. തന്നിൽ നിന്ന് ശരണം തേടിയ ഉമൈമ എന്ന ഭാര്യയെ സമ്മാനങ്ങൾ നൽകി ആദരിച്ച് വീട്ടിലേക്ക് തിരിച്ചു പൊയ്കൊള്ളാൻ പറഞ്ഞ മുഹമ്മദ് നബിയുടെ(സ) മാതൃക ഈ ക്വുർആൻ വാക്യത്തിന്റെ ജീവിത വ്യാഖ്യാനമായിരുന്നു. (സ്വഹീഹുൽ ബുഖാരി: 5254, 5255)

أيها الرجل إن كان إنما بك الحاجة جمعنا لك حتى تكون أغنى قريش رجلا، وإن كان إنما بك الباه فاختر أي نساء قريش شئت فلنزوجك عشرا. ” മുഹമ്മദെ, നിനക്ക് ധനമാണ് ആവശ്യമെങ്കിൽ ഞങ്ങൾ ധനം ഒരുമിച്ചു കൂട്ടി നിന്നെ ക്വുറൈശികളിലെ ഏറ്റവും പണക്കാരനാക്കി ഞങ്ങൾ മാറ്റാം. നിനക്ക് രതിയാണാവശ്യമെങ്കിൽ ക്വുറൈശികളിലെ നീ തിരഞ്ഞെടുക്കുന്ന പത്തു സ്ത്രീകളെ ഞങ്ങൾ നിനക്ക് വിവാഹം കഴിപ്പിച്ചു തരാം..” (അൽ ബിദായ വന്നിഹായ: ഇബ്നുകസീർ: 3: 80) എന്ന ക്വുറൈശികളുടെ വാഗ്ദാനം യാതൊരു സങ്കോചവുമില്ലാതെ നിമിഷാർദ്ധം കൊണ്ട് തിരസ്കരിച്ച വ്യക്തിയാണ് മുഹമ്മദ് നബി (സ).

സ്വഹീഹായ ഹദീസുകളുടെ വെളിച്ചത്തിൽ പരിശോധിച്ചാൽ പതിനൊന്ന് ഭാര്യമാരായിരുന്നു നബിക്ക്(സ) ഉണ്ടായിരുന്നത് എന്ന് വ്യക്തമാവുന്നതാണ്. 1. ഖദീജ, 2. സൗദ, 3. ആഇശ, 4. ഹഫ്‌സ, 5. സൈനബ് ബിൻത് ഖുസൈമ, 6. ഉമ്മു സലമ, 7. ഉമ്മു ഹബീബ, 8. ജുവൈരിയ, 9. മൈമൂന, 10. സ്വഫിയ്യ, 11. സൈനബ് ബിൻത് ജഹ്ശ്… എന്നിവരാണവർ. റെെഹാന ബിൻത് സൈദിന്റെ വിഷയത്തിൽ അഭിപ്രായ വ്യത്യാസമുണ്ട്. (സാദുൽ മആദ്: 1: 79)

عن ابن عباس، قال: «توفي رسول الله صلى الله عليه وسلم وعنده تسع نسوة (നബി (സ) ജീവിത കാലത്തു തന്നെ മരണമടഞ്ഞ ഖദീജ, സൈനബ് ബിൻത് ഖുസൈമ എന്നീ രണ്ട് ഭാര്യമാരെ കൂട്ടാതെ) നബി (സ) മരണപ്പെടുമ്പോൾ അദ്ദേഹത്തിന് ഒമ്പത് ഭാര്യമാരെ ഉണ്ടായിരുന്നുള്ളു എന്ന് പ്രവാചക ശിഷ്യൻ ഇബ്നു അബ്ബാസ് (റ) വ്യക്തമാക്കുന്ന സ്വഹീഹായ ഹദീസ് നിലവിലുണ്ട്. (സുനനുന്നസാഈ: 3197)

വിമർശകർ ഉദ്ധരിക്കുന്ന ഈ അറുപതോളം വരുന്ന വിവാഹങ്ങളുടെ കെട്ടുകഥകൾ ഇസ്‌ലാമിക പ്രമാണങ്ങളായ ക്വുർആനിൽ നിന്നൊ സ്വഹീഹായ ഹദീസുകളിൽ നിന്നൊ തെളിയിക്കാൻ സാധിക്കുമൊ? ഒരിക്കലുമില്ല. എല്ലാം എടുക്കപ്പെട്ടിരിക്കുന്നത് ചരിത്ര ഗ്രന്ഥങ്ങളിൽ നിന്നാണ്; അതിൽ ഭൂരിഭാഗത്തിലും തങ്ങളുടേതായ (ദുർ) വ്യാഖ്യാനങ്ങളും അഭിപ്രായങ്ങളും സാങ്കൽപ്പിക അശ്ലീലങ്ങളും വിമർശകരുടെ വകയായി തിരുകി കയറ്റുകയും ചെയ്തിട്ടുണ്ട്. വിമർശകർ ദുർബല നിവേദനങ്ങൾ ചികയാൻ നൽകുന്ന ഈ ചരിത്ര ഗ്രന്ഥങ്ങളൊ സീറയൊ ഇസ്‌ലാമിക പ്രമാണങ്ങളല്ല. നബിയുടെ(സ) സമകാലികരായ സ്വഹാബികൾ നിവേദനം ചെയ്ത ഹദീസുകൾ നിലനിൽകെ പ്രവാചക വിയോഗത്തിന് കാലങ്ങൾക്ക് ശേഷം പല സ്രോതസ്സുകളിൽ നിന്നും ആധികാരികത പരിശോധിക്കാതെ വാരിക്കൂട്ടിയ കഥകളുടെ സമാഹാരങ്ങളായ സീറകളിൽ വിമർശകരുടെ കണ്ണു പതിയാൻ കാരണം മറ്റൊന്നുമല്ല. തങ്ങളെ പോലെ വ്യാജ വാർത്തകളും അർദ്ധ സത്യങ്ങളും പ്രചരിപ്പിച്ചു നടക്കുന്ന ദുർബല നിവേദകരുടെ ഒരു ശൃംഖല തന്നെ സീറാ ഗ്രന്ഥങ്ങളിൽ സ്ഥാനം പിടിച്ചിട്ടുണ്ട് എന്ന് മുമ്പ് വായിച്ച ‘ദുർബല ഹദീസുകളും കള്ളകഥകളും’ ഭാഗങ്ങളിൽ വ്യക്തമാക്കപ്പെട്ടിട്ടുണ്ട്. മുഹമ്മദ് നബിയുമായി(സ) ബന്ധപ്പെടുത്തി ഈ വ്യാജ വിവാഹ കഥകൾ പ്രചരിപ്പിക്കുന്നതിൽ ഈ ദുർബലരും, വ്യാജഹദീസ് നിർമ്മാതാക്കളുമായ നിവേദകർക്ക് പല ലക്ഷ്യങ്ങളുമാണ് ഉണ്ടായിരുന്നത്. മുഹമ്മദ് നബിയുടെ (സ) വ്യക്തിത്വത്തെ വികൃതമായി ചിത്രീകരിക്കുക എന്നതായിരിക്കാം പ്രധാന ലക്ഷ്യം. മറ്റു ചില നിവേദകരാകട്ടെ തീർത്തും വിപരീതമായ ഉദ്ദേശത്തോടെ വ്യാജ വിവാഹങ്ങൾ ആരോപിച്ചവരാണ്. ഉദാഹരണതിന് വ്യാജ നിവേദനങ്ങൾ നിർമ്മിക്കുന്നവനാണെന്ന് ഹദീസ് നിദാനശാസ്ത്ര പണ്ഡിതർ ഒരു പോലെ അഭിപ്രായപ്പെട്ട കൽബ് ഗോത്രക്കാരനായ ഒരു നിവേദകൻ കൽബ് ഗോത്രത്തിലെ പല സ്ത്രീകളിലേക്കും ചേർത്ത് പ്രവാചകന് (സ) വിവാഹങ്ങൾ ആരോപിക്കുന്നത് വായനക്കാർ ശ്രദ്ധിച്ചിരിക്കുമല്ലൊ. തങ്ങളുടെ ഗോത്രത്തിൽ നിന്നും പ്രവാചകൻ (സ) വിവാഹം ചെയ്തുവെന്ന ആദരവും ശ്രേഷ്ടതയും പിടിച്ചു പറ്റാനുള്ള വിഫല ശ്രമമാണിത് എന്ന് ന്യായമായും കരുതാം.

ഇത്തരം വ്യാജന്മാരെ കൂട്ടു പിടിക്കാതെ സ്വഹീഹായ സനദുകൾ (നിവേദക പരമ്പര) ഉള്ള അവലംബനീയമായ സ്രോതസ്സുകളിൽ നിന്ന്, മുഹമ്മദ് നബി (സ) 61 സ്ത്രീകളുമായി രമിച്ചെന്നോ പത്തൊമ്പത് സ്ത്രീകളെ വിവാഹം ചെയ്ത് ഭോഗിച്ചുവെന്നോ തെളിയിക്കാൻ ലോകത്ത് ഇന്ന് ജീവിച്ചിരിക്കുന്ന സർവ്വ നാസ്തികരേയും വെല്ലുവിളിക്കുന്നു.

അപ്പോൾ പിന്നെ സീറ രചയിതാക്കളായ മുസ്‌ലിം പണ്ഡിതർ എന്തിന് ഈ വ്യാജ കഥകൾ തങ്ങളുടെ ഗ്രന്ഥങ്ങളിൽ രേഖപ്പെടുത്തി എന്ന ചോദ്യം ഉയർന്നേക്കാം.

അതിനുള്ള ഉത്തരം എത്രയോ ആവർത്തി നാം വ്യക്തമാക്കി കഴിഞ്ഞു. എങ്കിലും മറുപടിയുടെ രത്നചുരുക്കം മറ്റൊരു ലേഖനത്തിൽ നിന്നും ഇവിടെ പേർക്കാം:

“ഹദീസ് ഗ്രന്ഥങ്ങൾക്കു പുറമെ സീറ (നബി ചരിത്ര) ഗ്രന്ഥങ്ങളുടേയും ഇസ്‌ലാമിക ചരിത്ര ഗ്രന്ഥങ്ങളുടേയും കാര്യവും സമാനമാണ്. പൗരാണിക ഹദീസ് – സീറ ഗ്രന്ഥങ്ങളിലെല്ലാം ഓരോ നിവേദനങ്ങൾക്കുമൊപ്പം സനദും കണിശമായി ചേർക്കപ്പെട്ടിരിക്കും. അവയിൽ എഴുതപ്പെട്ട ഹദീസുകളും നിവേദനങ്ങളുമെല്ലാം സ്വീകാര്യമാണെന്ന് ആ ഗ്രന്ഥകാരന്മാരായ പണ്ഡിതർക്കു തന്നെ അഭിപ്രായമില്ലെന്നാണ് ഇത് തെളിയിക്കുന്നത്. മറിച്ച് ഏത് നിവേദനങ്ങളുടെ സനദുകളാണോ സ്വഹീഹ് അവ മാത്രമാണ് -പ്രസ്തുത ഗ്രന്ഥകർത്താക്കളുടെ അടുക്കൽ തന്നെ – സ്വീകാര്യം, ദഈഫ് ആയ നിവേദനങ്ങൾ അസ്വീകാര്യങ്ങളും. ഈ ഗ്രന്ഥങ്ങളിൽ സ്വഹീഹായ നിവേദനങ്ങൾ മാത്രമല്ല പ്രസ്തുത പണ്ഡിതന്മാർ ഉൾപ്പെടുത്തിയിരിക്കുന്നത്. ഒരു വിഷയത്തെയൊ കാലഘട്ടത്തെയൊ ചർച്ച ചെയ്യുമ്പോൾ അതുമായി ബന്ധപ്പെട്ട് അവർക്കു കിട്ടിയ സർവ്വ നിവേദനങ്ങളും അവർ അവരുടെ ഗ്രന്ഥങ്ങളിൽ ഉൾപ്പെടുത്തിയിട്ടുണ്ട്. ആ നിവേദനങ്ങളിൽ ഏതൊക്കെയാണ് സത്യസന്ധം ഏതൊക്കെയാണ് വ്യാജം എന്നത് വേർതിരിച്ച് മനസ്സിലാക്കൽ ആ ഗ്രന്ഥങ്ങൾ പഠനവിധേയമാക്കുന്നവരുടെ ബാധ്യതയായാണ് ആ ഗ്രന്ഥങ്ങളുടെ രചയിതാക്കൾ മനസ്സിലാക്കിയത്. ഉദാഹരണത്തിന് ഇസ്‌ലാമിക ലോകത്തെ ഏറ്റവും പൗരാണികവും പ്രശസ്തവുമായ ചരിത്ര ഗ്രന്ഥത്തിന് ഉടമയായ ഇമാം ഇബ്നു ജരീർ ത്വബ്‌രി തന്റെ ചരിത്ര ഗ്രന്ഥത്തിന്റെ ആമുഖത്തിൽ പറയുന്നത് കാണുക:

“നമ്മുടെ ഈ ഗ്രന്ഥം വായിക്കുന്നവർ അറിയേണ്ട ഒരു വസ്തുതയുണ്ട്. അതായത് നാമിവിടെ കൊണ്ട് വന്നതും പറഞ്ഞതുമായ കാര്യങ്ങളിൽ എന്റെ അവലംബം, ഞാൻ തന്നെ നേരിട്ട് അറിയുന്ന ചില കാര്യങ്ങളും, നിവേദകന്മാരിലേക്ക് ഞാൻ ചേർത്തി ഉദ്ധരിച്ച സംഭവങ്ങളുമാണ്.

എന്നാൽ, ഭൂതകാലത്തെ പറ്റി നാം സംസാരിക്കവെ, വായനക്കാരന് അപരിചിതമായതോ കേൾക്കുന്നവന് വെറുപ്പുണ്ടാക്കുന്നതോ ആയ എന്തെങ്കിലുമുണ്ടെങ്കിൽ – അഥവാ ഒരു തരത്തിലും സ്വീകാര്യമല്ലാത്തവയോ വസ്തുതകൾക്ക് നിരക്കാത്ത ആശയത്തിലുള്ളതോ ഒക്കെ – അത് നമ്മുടെ അടുക്കൽ നിന്നുള്ളതല്ലെന്നും നിവേദകന്മാരിൽ നിന്നും സംഭവിച്ചതാണെന്നും അറിയണം. (ഏതെങ്കിലും വിഷയാസ്പദമായി) നമുക്ക് ലഭിച്ച നിവേദനങ്ങൾ അപ്പടി ഉദ്ധരിക്കുക മാത്രമാണ് നാം (ഈ ഗ്രന്ഥത്തിൽ) ചെയ്തിരിക്കുന്നത്. (അവയിലെ നെല്ലും പതിരും വ്യവഛേദിക്കൽ ഗ്രന്ഥ പഠിതാക്കളുടെ ബാധ്യതയാണ്)” (മുഖദ്ദിമ: താരീഖുത്വബ്‌രി: 5) ”

ചരിത്രത്തിൽ എവിടെയും രേഖപ്പെടുത്താൻ കഴിയാത്ത പീഢനകഥകൾ ‘ഇനിയും ഉണ്ടായേക്കാം ‘അവ നമുക്ക്’ നഷ്ടമായതായിരിക്കാം ‘എന്നൊക്കെ മുതലക്കണ്ണീരൊഴുക്കുന്നതിനും’ ആഞ്ഞ് ‘നിഗമിക്കുന്നതിനും യാതൊരു സാധുതയുമില്ല എന്ന് ഇതിൽ നിന്നും വ്യക്തമാവുന്നു. കാരണം ഉള്ളതെല്ലാം – നല്ലതും ചീത്തയും നോക്കാതെ, വിശ്വസ്‌തരോ വ്യാജരോ എന്ന് വേർതിരിക്കാതെ – തങ്ങളുടെ ഗ്രന്ഥത്തിൽ കൂട്ടി കുഴക്കുന്നത് ശീലമാക്കിയവരായിരുന്നു സീറക്കാർ എന്ന് വരുമ്പോൾ അവർക്ക് കിട്ടാതെ പോയ ചരിത്ര ‘വിവരങ്ങളെ’ ഓർത്ത് വിലപിക്കുന്ന ചരിത്രപരമായ അജ്ഞത മച്ചി പെറ്റ കുട്ടിക്ക് ചരമഗീതം പാടുന്നതു പോലെ മൂഢത്വമാണ്. “അതിന്റെ ബാഹുല്യം കാരണമാവാം… എവിടേയും പറയുന്നില്ല… ഒരു പക്ഷേ അത് പിൻ കാലത്ത് പതിവു പോലെ ചരിത്രത്തിൽ നിന്ന് നീക്കം ചെയ്തതാവാം…” എന്ന വാചകം ഒരു നിരാശാജനകമായ നിശ്വാസം മാത്രമാണ്. കഴിവിന്റെ പരമാവധി ന്യൂനതകൾ മുഹമ്മദ് നബിക്കെതിരെ(സ) കുത്തിപ്പൊക്കാൻ ശ്രമിച്ചിട്ടും സാധിക്കാതിരിക്കുന്നതിന്റെ ഗാഢമായ ദുഃഖം മുറ്റിയ മോഹഭംഗം മാത്രമാണത്.

മുഹമ്മദ് നബിയുടെ (സ) വിവാഹങ്ങളൊന്നും കാമ പ്രേരിതമായിരുന്നില്ല. ഓരോന്നിന്നും മതപരവും സാമൂഹികവും രാഷ്ട്രീയവുമായ മഹ്ത്‌ലക്ഷ്യങ്ങളും കാരണങ്ങളുമുണ്ടായിരുന്നു.

സാമൂഹികമായ കാരണങ്ങൾ:

ആദ്യ വിവാഹം നാൽപതുകാരിയായ ഖദീജയെയായിരുന്നു. അദ്ദേഹത്തേക്കാൾ പതിനാല് വയസ്സ് കൂടുതൽ പ്രായമുള്ള അവരെ വിവാഹം കഴിക്കുകയും അവർ മരണമടയുന്നതു വരെ 25 വർഷം അദ്ദേഹം അവരെ മാത്രം ഭാര്യയായി സ്വീകരിച്ച് ജീവിച്ചു. ബുദ്ധിമതിയും പക്വമതിയുമായ അവരുടെ വ്യക്തിത്വത്തിലും സ്നേഹ സമ്പന്നതയിലും അദ്ദേഹം ആകൃഷ്ടനായിരുന്നു. വ്യക്തിത്വ സമ്പൂർണ്ണതയെത്തിയ ഒരു സ്ത്രീയായിരുന്നു അവരെന്ന് മുഹമ്മദ് നബി (സ) തന്നെ വ്യക്തമാക്കുന്നുണ്ട്. അദ്ദേഹത്തിന്റെ സൽസ്വഭാവത്തിലും സത്യസന്ധതയിലും അവരും ആകൃഷ്ടയായിരുന്നു.

ഖദീജയുടെ(റ) മരണത്തിന് ശേഷം അദ്ദേഹം വിവാഹം കഴിച്ചത് സൗദയെയാണ്. നാല് മക്കളുള്ള വിധവയായിരുന്നു സൗദാ ബീവി (റ). അവരുടെ സംരക്ഷണവും പരിപാലനവും എളുപ്പമാക്കുന്നതായിരുന്നു ഈ വിവാഹത്തിലൂടെ.

ഹഫ്‌സ ബിൻത് ഉമറിനെ (റ) അവർ വിധവയായപ്പോൾ നബി (സ) വിവാഹം ചെയ്തു. അവർക്ക് താങ്ങും തണലുമാകാനും അവരുടെ പിതാവിന് ആദരവായി കൊണ്ടുമായിരുന്നു ഈ വിവാഹം.

ഉഹ്ദ് യുദ്ധത്തിൽ ഭർത്താവ് രക്തസാക്ഷിയായി, വിധവയായ സൈനബ് ബിൻത് ഖുസൈമയെ (റ) വിവാഹം ചെയ്ത് നബി (സ) അവർക്ക് അഭയം നൽകി.

ഒരുപാട് മക്കളും പ്രാരാബ്ദവുമുണ്ടായിരുന്ന പ്രവാചകന്റെ പ്രിയ അനുചരനായ അബൂസലമയുടെ (റ) വിധവയായ ഉമ്മു സലമയെ നബി (സ) ഏറ്റെടുത്തു.

മുസ്‌ലിംകൾക്കായി സ്വന്തം ജീവൻ ത്യാഗം ചെയ്ത ഒരുപറ്റം മനുഷ്യരുടെ വിധവകളെയാണ് പ്രവാചകൻ (സ) വിവാഹം ചെയ്തതിൽ അധികവും. അവരെ ആശ്വസിപ്പിക്കാനും, അവരുടെ കണ്ണീരൊപ്പാനും അവരുടെ സന്താനങ്ങളുടെ പരിപാലനം അന്യത്വമില്ലാതെ ഏറ്റെടുക്കാനും, നിരാലംബരായ അവരുടെ പ്രാർത്ഥനകൾക്ക് മറുപടിയെന്നോണമെല്ലാമാണ് ഈ വിവാഹങ്ങൾ എല്ലാം നടന്നത്.

മത നിയമ സ്ഥാപനത്തിന്റെ ഭാഗമായ വിവാഹങ്ങൾ:

ചെറുപ്പക്കാരിയും ബുദ്ധിമതിയുമായ ആഇശയാണ് (റ) പ്രവാചകന്റെ ജീവിത സന്ദേശങ്ങൾ ഹദീസുകളായി ഏറ്റവും കൂടുതൽ ഉദ്ധരിച്ച പ്രവാചക പത്നി; അല്ല സ്ത്രീകളിൽ തന്നെ ഏറ്റവും കൂടുതൽ ഉദ്ധരിച്ചിട്ടുള്ളത്. പുരുഷന്മാരേയും കൂട്ടി ഏറ്റവുമധികം ഹദീസുകൾ ഉദ്ധരിച്ച അഞ്ചാമത്തെ പ്രവാചകാനുചര. ദാമ്പത്യം, കുടുംബം, സ്ത്രീ വിഷയങ്ങൾ എന്നിവയിലെല്ലാം അവരാണ് ഇസ്‌ലാമിക കർമ്മശാസ്ത്ര വിജ്ഞാനങ്ങൾക്ക് അവലംബം. പ്രവാചകൻ (സ) മരണപ്പെടുമ്പോൾ ചെറുപ്പക്കാരിയായിരുന്നതിനാൽ പിന്നീട് ജീവിച്ച ദീർഘമായ കാലഘട്ടം വ്യത്യസ്‌ത സന്ദർഭങ്ങളിൽ അനവധി വിജ്ഞാന കുതുകികൾക്കും സ്ത്രീ ജനങ്ങൾക്കും ഈ ഹദീസ് പാഠങ്ങൾ പകരാനും ആവർക്ക് സാധിച്ചു. ഇതെല്ലാം കൊണ്ട് തന്നെയാണ് വഹ്‌യ് അഥവാ ദിവ്യ ബോധനം വഴി ആഇശയെ(റ) വിവാഹം ചെയ്യാൻ ദൈവകൽപ്പന ഉണ്ടായത്.

സൈനബ് ബിൻത് ജഹ്ശ്നെ(റ), പ്രവാചകന്റെ വളർത്ത് പുത്രനും അടിമ വംശജനുമായിരുന്ന സൈദിന് (റ) പ്രവാചകൻ (സ) വിവാഹം ചെയ്ത് കൊടുത്തതാണ്. ഈ ദമ്പതികൾക്ക് പരസ്പരം യാതൊരു തൃപ്തിയുമുണ്ടായിരുന്നില്ലെങ്കിലും, ക്ഷമ ദീക്ഷിക്കാനുള്ള പ്രവാചകന്റെ (സ) കൽപ്പന മാനിച്ചു കൊണ്ട് മാത്രമായിരുന്നു അവർ പരസ്പരം സഹജീവിച്ചത്. വളർത്തു പുത്രന്മാരെ സ്വപുത്രന്മാരായി പരിഗണിക്കുന്ന അറബികളിലെ സമ്പ്രദായം അവസാനിപ്പിച്ചു കൊണ്ട് നിയമം അവതരിപ്പിക്കപ്പെട്ടപ്പോൾ (ക്വുർആൻ: 33:5) ആ നിയമം മുസ്‌ലിങ്ങളുടെ മനസ്സിൽ സുസ്ഥാപിതമാക്കാനായി, മുസ്‌ലിംകളുടെ മനസ്സിലെ ധാർമ്മിക മാതൃകയും വിശുദ്ധിയുടെ പ്രതീകവുമായ മുഹമ്മദ് നബി(സ)യോട് തന്നെ വളർത്തു “പുത്രനായ” സൈദിന്റെ വിവാഹ മോചിതയായ ഭാര്യയെ വിവാഹം ചെയ്യാൻ അല്ലാഹു കൽപ്പന നൽകി. ഇതിലൂടെ അവരുടെ മനസ്സിലെ തെറ്റായ പുത്ര സങ്കൽപ്പത്തെ അല്ലാഹു വേരോടെ പിഴുതെറിഞ്ഞു.

രാഷ്ട്രീയ കാരണങ്ങൾ:

ശത്രുത ഉന്മൂലനം ചെയ്യാനും ഹൃദയങ്ങളെ കൂട്ടിയിണക്കാനും യുദ്ധാനന്തര ബന്ധികളെ അടിമത്തത്തിൽ നിന്ന് മോചിപ്പിക്കാനുമെല്ലാം ഏറ്റവും സ്നേഹജനകമായ മാധ്യമമായി വിവാഹത്തെ പരിഗണിച്ചു കൊണ്ട് നടന്നതാണ് മറ്റു പല വിവാഹങ്ങളും. നബിയോട് (സ) യുദ്ധം ചെയ്ത് പരാജിതരായ ബനൂ മുസ്ത്വലക് ഗോത്രക്കാരിൽ നിന്നും ബന്ധിയാക്കപ്പെട്ട ജുവൈരിയയെ(റ) നബി (സ) വിവാഹം ചെയ്തത് ഈ ലക്ഷ്യത്തോടെയായിരുന്നു. ബനൂ മുസ്ത്വലക് ഗോത്രക്കാരുടെ നേതാവിന്റെ പുത്രിയായിരുന്നു അവർ. ഈ വിവാഹത്തിലൂടെ ശത്രുത ഇല്ലാതാവുകയും പ്രവാചകന്റെ(സ) സ്നേഹ ബന്ധുക്കളായി ബനൂ മുസ്ത്വലക് ഗോത്രക്കാർ പരിണമിക്കുകയും ചെയ്തു.

പ്രവാചകന്റേയും ഇസ്‌ലാമിന്റെയും കൊടിയ ശത്രുവായിരുന്ന കുറൈശി നേതാവ് അബൂസുഫ്‌യാന്റെ(റ) – പിന്നീടദ്ദേഹം ഇസ്‌ലാം സ്വീകരിച്ചു – മകൾ ഉമ്മു ഹബീബയും (റ) അവരുടെ ഭർത്താവും മുസ്‌ലിംകളായിരുന്നു. എന്നാൽ പിൻകാലത്ത് അവരുടെ ഭർത്താവ് അഭയാർത്ഥിയായി ചെന്ന അബിസീനിയലിൽ വെച്ച് മരണപ്പെട്ടു. ഈ സന്ദർഭത്തിൽ നിരാലംബയും അഭയാർത്ഥിയുമായ ഉമ്മു ഹബീബയെ(റ) നബി (സ) ഏറ്റെടുത്തു. ഇത് അബൂസുഫ്‌യാന്റെ(റ) മനസ്സിലെ ശത്രുതയെ കെടുത്താനും പ്രവാചകന്റെ(സ) വ്യക്തിത്വത്തോട് ബഹുമാനം ജനിക്കാനും കാരണമായി. കൂടാതെ ഇസ്‌ലാമിൽ ഉറച്ചു നിന്നതിന്റെ പേരിൽ കൂടുതൽ പ്രതിഫലമല്ലാതെ നഷ്ടമൊന്നും ഒരു വിശ്വാസിക്കുണ്ടാവില്ല എന്ന പൊതുവായ ഗുണപാഠവും ഈ വിവാഹം വിശ്വാസികൾക്ക് പകർന്നു നൽകി.

സമാനമായ ലക്ഷ്യങ്ങളോടെ തന്നെയാണ് സ്വഫിയ (റ), മൈമൂന (റ) എന്നിവരുമായുള്ള വിവാഹവും നടന്നത്.

മുമ്പ് സൂചിപ്പിച്ചതു പോലെ നബിയുടെ (സ) ബഹുഭാര്യത്വം കൃത്യവും വ്യക്തവുമായ യുക്തിയുടേയും ന്യായത്തിന്റെയും അടിസ്ഥാനത്തിലായിരുന്നു. അവയ്ക്ക് സാമൂഹികവും ധാർമ്മികവും രാഷ്ട്രീയവുമായ ധർമ്മങ്ങളുണ്ടായിരുന്നു. അവ കാമ പ്രേരിതങ്ങളായിരുന്നില്ല.

കുറൈശികളിലെ സുന്ദരികളും കുലീനകളും മികച്ചവരുമായ സ്ത്രീകളെ തരാമെന്ന ക്വുറൈശികളുടെ വാഗ്ദാനങ്ങൾ തിരസ്കരിച്ച നബി (സ) ഈ പ്രയാസങ്ങളും പ്രാരാബ്ധങ്ങളും നിറഞ്ഞ വിധവകളേയും വിവാഹ മോചിതകളേയും മാതാക്കളേയും തന്നേക്കാൾ പ്രായം കൂടിയവരേയുമെല്ലാം വിവാഹം ചെയ്തത് കാമ പ്രേരിതനായി കൊണ്ടാണെന്ന് വിവേകവും നീതി ബോധവുമുള്ള ആർക്കാണ് വാദിക്കാൻ കഴിയുക ?!

പ്രവാചകന്റെ(സ) കൂടെ നിന്ന സ്ത്രീകൾ ഓരോരുത്തരും അദ്ദേഹത്തിന്റെ സാന്നിധ്യത്തിനായി കൊതിച്ചു. അദ്ദേഹത്തിന്റെ സഹവാസത്തിൽ ആനന്ദിച്ചു. ഭൂമിയിൽ തനിക്ക് കിട്ടാവുന്നതിൽ ഏറ്റവും നല്ല ഭർത്താവായി അദ്ദേഹത്തെ മനസ്സിലാക്കി. മറിച്ചൊരു ചിത്രം ഏത് സീറ പൊടിതട്ടിയെടുത്ത് ദുർബല നിവേദനങ്ങൾ ചികഞ്ഞ് കൊണ്ടുവന്നാലും തെളിയിക്കാനാവില്ല. പള്ളിയിൽ അബ്സീനിയക്കാർ കുന്തപ്പയറ്റ് കളിക്കുന്ന പ്രദർശനം കാണാൻ നബി (സ) ആഇശ ബീവിയെ വിളിച്ചു. നബി (സ) നിൽക്കുകയായിരുന്നു, ആഇശ (റ) പിന്നിലൂടെ വന്ന് അദ്ദേഹത്തിന്റെ തോളിൽ താടി വെച്ച് കവിളിൽ കവിളു ചേർത്ത് വാതിൽക്കൽ നിന്ന് കളി കണ്ടു കൊണ്ടിരുന്നു. കുറച്ചു കഴിഞ്ഞപ്പോൾ പ്രവാചകൻ (സ) മതിയായില്ലെ എന്നു ആഇശയോട് (റ) തിരക്കി. അവർ പറഞ്ഞു: അല്ലാഹുവിന്റെ ദൂതരേ, നിങ്ങൾ തിരക്കല്ലെ… കുറച്ചു കഴിഞ്ഞപ്പോൾ വീണ്ടും പ്രവാചകൻ (സ) മതിയായില്ലെ എന്നു ആഇശയോട്(റ) തിരക്കി. അവർ പറഞ്ഞു: അല്ലാഹുവിന്റെ ദൂതരേ, നിങ്ങൾ തിരക്കല്ലെ… അപ്പോൾ അദ്ദേഹം ആഇശക്ക്(റ) വേണ്ടി അങ്ങനെ നിന്നു കൊടുത്തു. ആഇശ (റ) പറയുന്നു: “എനിക്ക് അവരുടെ കളി കാണാനുള്ള ആഗ്രഹം കൊണ്ടല്ല ഞാൻ പ്രവാചകനെ പിടിച്ചു നിർത്തിയത്. മറിച്ച്, അദ്ദേഹത്തിന്റെ മനസ്സിൽ എനിക്കുള്ള സ്ഥാനവും എന്റെ മനസ്സിൽ അദ്ദേഹത്തിനുള്ള സ്ഥാനവും സ്ത്രീകൾ അറിയട്ടെ എന്ന കൊതിയായിരുന്നു അത്…” (സുനനുൽ കുബ്റാ: നസാഈ: 8:181, മുശ്കിലുൽ ആസാർ: ത്വഹാവി: 292, അഹ്കാമുന്നദ്ർ: 360, സിൽസിലത്തു സ്വഹീഹ: 7/818) പ്രവാചക പത്നി ഉമ്മു സലമ (റ) പറയുന്നു: അബൂ സലമ (റ) മരണപ്പെട്ടപ്പോൾ ഞാൻ നബിയുടെ (സ) അടുക്കൽ ചെന്ന് വിവരം അറിയിച്ചു. അദ്ദേഹം എനിക്ക് ഒരു പ്രാർത്ഥന പഠിപ്പിച്ചു തന്നു. اللَّهمَّ اغفِر لي ولَهُ وأعقِبني منْهُ عقبى حسَنةً “അല്ലാഹുവേ, എനിക്കും അദ്ദേഹത്തിനും (മരണപ്പെട്ട വ്യക്തിക്കും) നീ പാപങ്ങൾ പൊറുത്തു തരേണമേ. അദ്ദേഹത്തിൽ നിന്ന് ഏറ്റവും നല്ല പര്യവസാനവും നൽകേണമേ ”

ഞാൻ ചിന്തിച്ചു: “അബൂ സലമയേക്കാൾ നല്ല പര്യവസാനം (അല്ലെങ്കിൽ അദ്ദേഹത്തേക്കാൾ ഉത്തമായ ഇണ) ആരാണ് ?!” എങ്കിലും ഞാൻ ആ പ്രാർത്ഥന പ്രാർത്ഥിച്ചു.

അങ്ങനെ അബൂ സലമയേക്കാൾ ഉത്തമനായ ഇണയെ അല്ലാഹു എനിക്ക് നൽകി. അത് അല്ലാഹുവിന്റെ ദൂതനായിരുന്നു. (സ്വഹീഹു മുസ്‌ലിം: 919, 918, തുർമുദി: 977)

പ്രവാചക പത്നി ഉമ്മു ഹബീബ (റ), പ്രവാചകനോടൊപ്പമുള്ള തന്റെ ഊഷ്മളമായ ദാമ്പത്യ ജീവിത്തിൽ ആനന്ദ പ്രേരിതയായി തന്റെ സഹോദരിയെ കൂടി അദ്ദേഹം വിവാഹം കഴിച്ചിരുന്നെങ്കിൽ എന്ന് കൊതിക്കുകയുണ്ടായി. أَحَبُّ مَن شَرِكَنِي في الخَيْرِ أُخْتِي “എന്നോടൊപ്പം ഈ സൗഖ്യത്തിൽ പങ്കാളിയാവാൻ ഞാൻ ഏറ്റവും ഇഷ്ടപ്പെടുന്നത് എന്റെ സഹോദരിയാണ്…” എന്ന് അവർ പ്രവാചകനോട് അപേക്ഷിച്ചു. രണ്ട് സഹോദരിമാരെ ഒരുമിച്ച് വിവാഹം കഴിക്കുന്നത് അനുവദനീയമല്ല എന്ന് പഠിപ്പിച്ചു കൊണ്ട് അദ്ദേഹം ആ വിവാഹാഭ്യർത്ഥന നിരസിച്ചു. എന്നു മാത്രമല്ല തങ്ങളുടെ ഭാര്യമാരോട്, (فلا تَعْرِضْنَ عَلَيَّ بَنَاتِكُنَّ، وَلَا أَخَوَاتِكُنَّ) “നിങ്ങളുടെ മക്കളുടേയൊ സഹോദരിമാരുടേയോ വിവാഹാഭ്യർത്ഥനയുമായി എന്റെ അടുത്ത് വരരുത്” എന്ന് വിലക്കുക കൂടി ചെയ്തു. (സ്വഹീഹു മുസ്‌ലിം: 1449)

ഇതും ഇതല്ലാത്തതുമായ മറ്റു പല സന്ദർഭങ്ങളിലും പ്രവാചകൻ (സ) വിവാഹാഭ്യർത്ഥനകൾ വിലക്കുന്നതും നിരസിക്കുകയും ചെയ്യുന്നതായി സ്വഹീഹായ ഹദീസുകളിൽ കാണാം. ഒരു പക്ഷെ സ്വീകരിച്ച വിവാഹാഭ്യർത്ഥനകളേക്കാൾ അദ്ദേഹം നിരസിച്ച വിവാഹാഭ്യർത്ഥനകളാണ് എണ്ണത്തിൽ കൂടുതലും. ഒരു സ്ത്രീലംബടനും കാമവെറിയനും ഇപ്രകാരം ചെയ്തതായി നമുക്കിതുവരെ അറിവില്ല.

പ്രവാചക പത്നി ഖദീജക്ക്(റ) പ്രവാചകനെ പറ്റിയുള്ള അഭിപ്രായം നോക്കൂ:

كَلَّا وَاللَّهِ مَا يُخْزِيكَ اللَّهُ أَبَدًا ، إِنَّكَ لَتَصِلُ الرَّحِمَ ، وَتَحْمِلُ الكَلَّ ، وَتَكْسِبُ المَعْدُومَ ، وَتَقْرِي الضَّيْفَ ، وَتُعِينُ عَلَى نَوَائِبِ الحَقِّ

“അല്ലാഹുവാണേ, അല്ലാഹു നിങ്ങളെ ഒരിക്കലും അപമാനിക്കുകയില്ല. തീർച്ചയായും നിങ്ങൾ കുടുംബ ബന്ധങ്ങൾ ചേർക്കുന്ന വ്യക്തിയാണ്, മറ്റുള്ളവരുടെ ദുഖഭാരം ഏറ്റെടുക്കുന്നവനും, പാവപ്പെട്ടവർക്ക് വേണ്ടി പണിയെടുക്കുന്നവനും, അതിഥികളെ തീറ്റുന്നവനും, സാമൂഹത്തിന് ദുരന്തം നേരിടുമ്പോൾ സഹായത്തിൽ മുഴുകുന്നവനുമാണ്…” (സ്വഹീഹുൽ ബുഖാരി: 3)

പ്രവാചകപത്നി സ്വഫിയ്യയുടെ(റ) അടിമത്ത മോചനവുമായി ബന്ധപ്പെട്ട അനുഭവം തന്നെ പരിശോധിക്കുക. ഖൈബർ യുദ്ധത്തിൽ ബന്ദികളാക്കപ്പെട്ടവരുടെ കൂട്ടത്തിൽ സ്വഫിയ്യയും ഉണ്ടായിരുന്നു. പ്രവാചകൻ (സ) അവരെ സ്വന്തത്തിനായി തിരഞ്ഞെടുക്കുകയും അവർക്ക് മുമ്പിൽ രണ്ട് വഴികളിലൊന്ന് തിരഞ്ഞെടുക്കാനായി അവസരം നൽകുകയും ചെയ്തു. ഒന്നുകിൽ മോചിതയാവുകയും തന്റെ ഭാര്യയാവുകയും ചെയ്യുക. അല്ലെങ്കിൽ മോചിതയാവുകയും കുടുംബത്തിലേക്ക് ചെന്നുചേരുകയും ചെയ്യാം. മോചിതയാവുകയും പ്രവാചകന്റെ പത്നിയാവുകയും ചെയ്യുക എന്നതാണ് സ്വഫിയ്യ സ്വമനസ്സാൽ തിരഞ്ഞെടുത്തത്. (സ്വഹീഹു ഇബ്നുഹിബ്ബാൻ: 4613)

വിമർശനം:

“പരിപൂർണ്ണതയിലെത്താത്ത മറ്റു ഡിവോഴ്സുകൾ”

ഉമ്രാ ബിൻത് റിഫാ… മേൽ പറഞ്ഞ റിഫയുടെ സഹോദരി…. (ഇബ്നു സാദ് 8: 107)

മറുപടി:

ഈ നാമത്തിലുള്ള ഒരു സ്ത്രീയെ സംബന്ധിച്ച വിവാഹ കഥ ഈയുള്ളവന്റെ പരിശോധനയിൽ എവിടെയും കണ്ടെത്താൻ കഴിഞ്ഞില്ല. കിട്ടിയ പേര് വെച്ച് ഒരു സാങ്കൽപിക ഭാര്യയെ ഭാവനാത്മകമായി രൂപകൽപ്പന ചെയ്തെടുത്തതാവാം. അങ്ങനെയാണെങ്കിൽ ഭൗതികവാദികളെ സംബന്ധിച്ച് അത് ഒരു അത്ഭുതമൊ പുതിയതൊ അല്ല.

വിമർശനം:

ബിൻത് ജുദൂബ് ഇബ്ന് ദാമ്രാ ഓഫ് ജൻദ്രാ…. കെട്ടി … ഒഴിവാക്കി..(ഇബ്നു സാദ് 8: 106)

മറുപടി:

ജന്ദഇയ്യ ഇബ്നതു ജുന്ദുബ് ഇബ്നു ദംറയെ നബി (സ) വിവാഹം അന്വേഷിച്ചു എന്നത് പോലും സ്ഥിരപ്പെട്ടിട്ടില്ല.

ഇബ്നു സഅ്ദ് തന്റെ ത്വബകാത്തിൽ കഥ ഉദ്ധരിച്ച നിവേദക പരമ്പരകൾ രണ്ടും കണ്ണി മുറിഞ്ഞതും ദുർബലരായ നിവേദകരാൽ നിർഭരവുമാണ്. എല്ലാം ഉദ്ധരിക്കുന്നത് മുഹമ്മദ് ഇബ്നു ഉമർ ആണ്:

أخبرنا محمد بن عمر قال حدثني محمد بن عبد الله عن الزهري مثل ذلك بنت جندب ابن ضمرة الجندعي أخبرنا محمد بن عمر حدثني عبد الله بن جعفر عن يزيد بن بكر أن رسول الله صلى الله عليه وسلم تزوج بنت جندب بن ضمرة الجندعي

മുമ്പത്തെ ലേഖനങ്ങളിൽ വിശദീകരിച്ചതു പോലെ മുഹമ്മദ് ഇബ്നു ഉമർ ദുർബലനാണ്. എന്ന് മാത്രമല്ല ഈ കഥ ഉദ്ധരിച്ച മുഹമ്മദ് ഇബ്നു ഉമർ തന്നെ ഈ കഥ കെട്ടുകഥയാണെന്നും നബി (സ) ഒരു കിനാന ഗോത്രക്കാരിയെയും ഒരിക്കലും വിവാഹം ചെയ്തിട്ടില്ലെന്നും പറയുന്ന നിവേദനവും ഇബ്നു സഅ്ദിന്റെ ത്വബക്കാത്തിൽ (8:149), കഥയുടെ തൊട്ട് താഴെയുണ്ടെങ്കിലും നാസ്തികർ അത് കണ്ട മട്ടില്ല.

قال محمد بن عمر وأصحابنا ينكرون ذلك ويقولون لم يتزوج رسول الله صلى الله عليه وسلم كنانية قط

വിമർശനം:

ജാമ്രാ ബിൻത് അൽ ഹരിത്…. മുഹമ്മദ് വിഹാഹ കരാറിൽ ഒപ്പിട്ട ശേഷം അറിഞ്ഞു അവൾക്ക് രോഗമുണ്ട് എന്ന്… ഒഴിവാക്കി (തബാരി v9, P 140- 141)

മറുപടി:

‘വിവാഹ കരാർ ചെയ്തു’, ‘വിവാഹ കരാറിൽ ഒപ്പിട്ടു’ എന്നൊക്കെ എല്ലാ കഥകളിലും നാസ്തികർ ആവർത്തിച്ചു കൊണ്ടിരിക്കുന്ന പദങ്ങൾ മിഷണറി പരിഭാഷകളിൽ നിന്ന് കടമെടുത്തവയാണ്. അറബിയിൽ ഖത്വബ (خطب) അഥവാ വിവാഹം അന്വേഷിച്ചു എന്നെയുള്ളു. അതിൽ ഒരു പഞ്ച് പോരാത്തതു കൊണ്ടും നബിയെ(സ) ഒരു കരാർ ലംഘകനാക്കാനും വേണ്ടിയാണ് ഈ പദങ്ങളിലെ ബോധപൂർവ്വമുള്ള തിരിമറികൾ. ഏതായാലും കഥ എല്ലായ്‌പ്പോഴും പോലെ വ്യാജമാണ്. സനദു പോലുമില്ലാത്ത രണ്ട് വരി!!. (താരീഖുത്വബ്‌രി: 3: 169)

വിമർശനം:

അൽ ഷാൻബാ ബിൻത് അമ്ര്… പ്രവാചകനല്ല എന്ന് ആദ്യ ദിവസം തന്നെ കളിയാക്കിയത് കൊണ്ട് ഒഴിവാക്കി (തബാരി 9, P 136)

മറുപടി:

ശൻബാഅ് ബിൻത് ഉമർ അൽ ഗിഫാരി. കഥ ഇപ്രകാരമാണ്:

ശൻബാഅ്നെ പ്രവാചകൻ (സ) വിവാഹം ചെയ്ത ആദ്യ രാത്രി അദ്ദേഹത്തിന്റെ പുത്രൻ ഇബ്രാഹിം മരണപ്പെട്ടു. അപ്പോൾ ശൻബാഅ് പറഞ്ഞു: അദ്ദേഹം പ്രവാചകനായിരുന്നെങ്കിൽ അദ്ദേഹത്തിന് ഏറ്റവും പ്രിയപ്പെട്ടവർ എന്തുകൊണ്ട് മരണപ്പെട്ടു? ഇസ്‌ലാമിക പാഠങ്ങൾ തീരെ അറിവില്ലാത്തതിനാലും ദുർബല വിശ്വാസി ആയതിനാലും പ്രതിസന്ധി ഘട്ടത്തിൽ തന്നെ ആശ്വസിപ്പിക്കാനുള്ള പക്വത ഇല്ലാത്തതിനാലുമെല്ലാമാവാം നബി (സ) അവരെ വിവാഹ മോചനം ചെയ്തത് എന്ന് അനുമാനിക്കാം. പക്ഷെ നാസ്തികൻ സ്വന്തം വക കാരണം കണ്ടെത്തി.

“പ്രവാചകനല്ല എന്ന് ആദ്യ ദിവസം തന്നെ ‘കളിയാക്കിയതിനാൽ’ ഒഴിവാക്കി”

ഒരു തമാശ പറഞ്ഞതിന് ഒഴിവാക്കി എന്ന് വരുത്തി തീർത്തു. മകൻ മരിച്ച കാര്യം സൂചിപ്പിച്ചുമില്ല. അതിന് മറ്റൊരു കാരണം കൂടി ഉണ്ട്. “മുഹമ്മദിന്റെ പെണ്ണുങ്ങൾ” എന്ന ലേഖനത്തിന്റെ അവസാനത്തിൽ “ഇത്രയൊക്കെയായിട്ടും മുഹമ്മദിന് കുട്ടികളില്ല” എന്ന ഒരു കല്ലുവെച്ച നുണ പറഞ്ഞാണ് ലേഖനം അവസാനിപ്പിക്കുന്നത്. അപ്പോൾ പിന്നെ നിവേദനം മുഴുവനായും ഉദ്ധരിക്കാൻ കഴിയില്ലല്ലൊ.

ഏതായാലും ഈ നിവേദനവും വ്യാജമാണ്. സനദ് (നിവേദക പരമ്പര) പോലും കഥയ്ക്കില്ല.

വിമർശനം:

“പരിപൂർണ്ണതയിലെത്താത്ത മറ്റു ഡിവോഴ്സുകൾ”

സനാ അൽ-നഷാത്ത് ബിൻത് റിഫാ… മുഹമ്മദിന്റെ ഒരു സാദാ പടയാളിയുടെ മകൾ… സ്ഥാനക്കയറ്റത്തിന് വേണ്ടി മുഹമ്മദിന് മകളെ വിവാഹം ചെയ്തു കൊടുത്തു… മുഹമ്മദ് കരാറിൽ ഒപ്പിട്ട ശേഷം മരിച്ചു (ആത്മഹത്യയാവാം) (അൽ തബാരി v9, P 135-136, അൽ തബാരി v 39 P 166)

(മുഹമ്മദിന്റെ പെണ്ണുങ്ങൾ: നാസ്‌തിക സോഷ്യൽ മീഡിയ തെറിമാല)

മറുപടി:

നാസ്‌തിക സോഷ്യൽ മീഡിയ തെറിമാലകളിൽ ഒന്നായ “മുഹമ്മദിന്റെ പെണ്ണുങ്ങൾ” എന്ന കുറിപ്പിൽ നിന്നുള്ള ചില വരികളെയാണ് നാം തുടർച്ചയായി നിരൂപണം ചെയ്‌തു കൊണ്ടിരിക്കുന്ന്. മുമ്പ് സൂചിപ്പിച്ചതു പോലെ, സ്ത്രീവിമോചകനായ നബിയെ (സ), സ്ത്രീ പീഢകനും ലമ്പടനുമായി പ്രചരിപ്പിക്കാൻ വേണ്ടി കല്ലുവച്ച നുണകളും, അർദ്ധ സത്യങ്ങളും, ദുർവ്യാഖ്യാനങ്ങളും, വൈരുദ്ധ്യങ്ങളും കൂട്ടി കുഴച്ചുണ്ടാക്കിയ വിധ്വേഷ കഷായമാണ് ലേഖനം.

ആരോപണ വിധേയമായ വിഷയത്തിലേക്ക് കടന്നു വരാം…

സനാഅ് ബിൻത്ത് സ്വൽത്, നഷാത്ത് ബിൻത് രിഫാഅ എന്നിങ്ങനെ രണ്ട് സ്ത്രീകളുടെ നാമങ്ങൾ കൂട്ടിയോജിപ്പിച്ച് നാസ്‌തികരുണ്ടാക്കിയ ഒരു പുതിയ ഭാര്യയാണ് “സനാ അൽ-നഷാത്ത് ബിൻത് റിഫാ… ” !!

ഇതിനൊരു കാരണമുണ്ട്. ഇത് രണ്ടും രണ്ട് സ്ത്രീകളാണൊ ? അതൊ ഒരു സ്ത്രീയെ തന്നെ ഉദ്ദേശിച്ച് രണ്ട് പേര് പറയപ്പെട്ടതാണൊ ? എന്നതിലൊന്നും ആർക്കും ഒരു നിശ്ചയവുമില്ല. ഇങ്ങനെയൊരു സ്ത്രീയുണ്ടൊ എന്നതിന് തന്നെ കൃത്യമായ തെളിവൊന്നും ഇല്ല എന്നത് മറ്റൊരു വസ്തുത.

ഇവരുടെ നാമത്തിലൊ അസ്തിത്വത്തിലൊ യാതൊരു തീർച്ചയുമില്ല എന്ന് നിവേദനം ഉദ്ധരിച്ച ത്വബ്‌രി തന്നെ വ്യക്തമാക്കിയിട്ടുമുണ്ട്. (താരീഖു ത്വബ്‌രി: 2: 416)

ഇനി, നിവേദനത്തിന്റെ പരമ്പരകളിലേക്ക് വരാം. ത്വബ്‌രി തന്റെ താരീഖിൽ ഒരു പരമ്പരയുമില്ലാതെ ചില അഭിപ്രായങ്ങൾ രേഖപ്പെടുത്തുകയാണ് ചെയ്തിരിക്കുന്നത്. എന്നാൽ ത്വബ്‌രിയുടെ മറ്റൊരു ഗ്രന്ഥത്തിൽ അദ്ദേഹത്തിന് ഈ വ്യാജ വാർത്ത ലഭിച്ച നിവേദക പരമ്പര അദ്ദേഹം വ്യക്തമാക്കുന്നുണ്ട്:

قال هشام بن محمد الكلبي حدثني رجل من رهط عبد الله بن خازم السلمي أن رسول الله صلى الله عليه وسلم تزوج سنا بنت الصلت بن حبيب السلمية فماتت قبل أن يصل إليها

“ഹിശാമിബ്നു മുഹമ്മദ് അൽ കൽബി പറഞ്ഞു: അബ്ദുല്ലാഹിബ്നു ഖാസിം അസ്സുലമിയുടെ സംഘത്തിൽ പെട്ട’ ഏതോ ഒരാൾ’ എന്നോട് പറഞ്ഞു: ദൈവദൂതൻ (സ) സനാഅ് ബിൻത്ത് സ്വൽത് ഇബ്നു ഹബീബ് അസ്സുലമിയയെ വിവാഹം ചെയ്തു. അവരുടെ അടുത്തെത്തുന്നതിന് മുമ്പ് തന്നെ അവർ മരണമടഞ്ഞു…” (അൽ മുൻതഖബ് മിൻ ദൈലിൽ മുദയ്യൽ: ത്വബ്‌രി: 90)

സനദ് (നിവേദക പരമ്പര):

പരമ്പരയിലെ ഹിശാമിബ്നു മുഹമ്മദ് അൽ കൽബിയെ സംബന്ധിച്ച വിശദ വിവരങ്ങൾ മുൻ കഴിഞ്ഞ ലേഖനങ്ങളിൽ വന്നിട്ടുണ്ടല്ലൊ. ഹിശാമിബ്നു മുഹമ്മദ് അൽ കൽബിയും അദ്ദേഹത്തിന്റെ പിതാവും നുണ പറയുന്നവരും ദുർബലരുമാണെന്നതിൽ ഹദീസ് നിദാന ശാസ്ത്ര പണ്ഡിതർക്കിടയിൽ ഇരു പക്ഷമില്ല.

ഹിശാമിന് ഈ കഥ കിട്ടിയതാവട്ടെ “അബ്ദുല്ലാഹിബ്നു ഖാസിം അസ്സുലമിയുടെ സംഘത്തിൽ പെട്ട ‘ഏതോ ഒരാൾ’ എന്നോട് പറഞ്ഞു” കൊണ്ടാണ്. ഏതോ ആളുകൾ പറഞ്ഞ കഥകളൊക്കെ തെളിവിന് നിരക്കുന്നതാണൊ ?!

ഇനി നിവേദനത്തിന്റെ മത്‌ന് അഥവാ ഉള്ളടക്കത്തിലേക്ക് വരാം: “ദൈവദൂതൻ (സ) സനാഅ് ബിൻത്ത് സ്വൽത് ഇബ്നു ഹബീബ് അസ്സുലമിയയെ വിവാഹം ചെയ്തു. അവരുടെ അടുത്തെത്തുന്നതിന് മുമ്പ് തന്നെ അവർ മരണമടഞ്ഞു…” ഇത്രയെ നിവേദനത്തിലുള്ളു. പക്ഷെ നബിയെ സ്ത്രീ പീഢകനും ലമ്പടനുമാക്കാൻ വേണ്ടിയുള്ള ലേഖനമായതിനാൽ ഈ കഥക്ക് ഒരു പഞ്ച് പോര !! അപ്പോൾ നാസ്‌തിക ബുദ്ധിയിലെ പാഷാണം ഉറവ പൊട്ടി ഒഴുകാൻ തുടങ്ങി…..

“മുഹമ്മദിന്റെ ഒരു സാദാ പടയാളിയുടെ മകൾ… സ്ഥാനക്കയറ്റത്തിന് വേണ്ടി മുഹമ്മദിന് മകളെ വിവാഹം ചെയ്തു കൊടുത്തു… മുഹമ്മദ് കരാറിൽ ഒപ്പിട്ട ശേഷം മരിച്ചു (ആത്മഹത്യയാവാം)….”

ഇപ്പോഴാണ് ഒരു പഞ്ച് വന്നത് !! നബിക്ക് ഒരു വില്ലൻ പരിവേഷം കിട്ടി ആശ്വാസമായി !! ചരിത്ര ധർമ്മവും വൈജ്ഞാനിക നീതിയും സത്യസന്ധതയുമൊക്കെ അവിടെ നിൽക്കട്ടെ എന്നങ്ങ് തീരുമാനിച്ചു. അല്ലെങ്കിലും ധാർമ്മികതക്ക് ഈ വിഡ്ഢി പരിഷകളുടെ അടുക്കൽ വല്ല അസ്തിത്വമൊ അടിത്തറയൊ ഉണ്ടൊ ?!

നഷാത്ത് ബിൻത് രിഫാഅ എന്ന നാമം -ചില നിവേദനങ്ങിൽ – ലോപിച്ച് ‘ഷാത്ത്’ ബിൻത് രിഫാഅ എന്ന് വന്നതായി കാണാം.

മുഫദ്ദൽ അൽ ഗസ്സാനി തന്റെ താരീഖിൽ സൂചിപ്പിക്കുകയും ഇബ്നുൽ കയ്യിം തന്റെ ‘സബീലുൽ ഹുദാ വർറശാദ് ഫീ സീറതി ഖൈറിൽ ഇബാദ് എന്ന ഗ്രന്ഥത്തിൽ ക്വോട്ട് ചെയ്യുകയും ചെയ്ത നിവേദനത്തിന്റെ പരമ്പര ഇപ്രകാരമാണ്:

روى المفضل بن غسان العلائي في تاريخه من طريق سيف بن عمر عن أبي عمر عثمان بن مقسم عن قتادة قال:

മുഫദ്ദൽ ബിൻ ഗസ്സാൻ അൽ അലാഈ തന്റെ താരീഖിൽ ഉദ്ധരിച്ചു: സൈഫ് ഇബ്നു ഉമർ വഴി: അബൂ ഉമർ ഉസ്മാനിബ്നു മിക്സം ൽ നിന്ന്: കത്താദയിൽ നിന്ന്, അദ്ദേഹം പറഞ്ഞു:…

നിവേദനം ഉദ്ധരിച്ച ഇബ്നുൽ കയ്യിം തന്നെ പറയുന്നു:

وعثمان بن مقسم متروك

നിവേദകനായ ഉസ്മാനിബ്നു മിക്സം മത്റൂക് (അതി ദുർബലൻ) ആകുന്നു. പ്രവാചക ശിഷ്യൻമാരെ തെറി വിളിക്കുന്ന, മീസാൻ പോലെയുള്ള ഇസ്‌ലാമിലെ വിശ്വാസ കാര്യങ്ങൾ നിഷേധിക്കുന്ന, കദരിയ്യായ വ്യക്തിയാണ് ഉസ്മാനിബ്നു മിക്സം എന്ന് ഇമാം ദഹബി വ്യക്തമാക്കിയിട്ടുണ്ട്. വ്യാജ ഹദീസുകൾ നിർമ്മിക്കാറുള്ള വ്യക്തിയാണിയാൾ എന്ന് യഹ്‌യബ്നു മഈൻ വ്യക്തമാക്കി. ധാരാളം കളവുകൾ പറയുന്ന വ്യക്തിയാണിയാൾ എന്ന് ജൂസജാനി പറഞ്ഞു. (മീസാനുൽ ഇഅ്തിദാൽ: 3: 56, അൽ ജർഹു വതഅ്ദീൽ: 6: 176, അദ്ദുഅഫാഉൽ കബീർ: 3: 220)

മറ്റൊരു നിവേദകനായ കത്താദ താബിഈ ആകുന്നു. ആരിൽ നിന്നാണ് സംഭവം കേട്ടതെന്ന് വ്യക്തമാക്കാത്തതിനാൽ പരമ്പര മുറിഞ്ഞതാണ്.

സൈഫിബ്നു ഉമർ അത്തീമി എന്ന നിവേദകനും ദുർബലനാണ്. أبو حاتم بن حبان البستي : يروي الموضوعات عن الأثبات، اتهم بالزندقة، قالوا إنه كان يضع الحديث

ഇബ്നു ഹിബ്ബാൻ പറഞ്ഞു: വിശ്വസ്‌തരിൽ നിന്ന് വ്യാജ നിവേദനങ്ങൾ ഉദ്ധരിക്കുമായിരുന്നു. ഉള്ളിൽ അവിശ്വാസം ഒളിപ്പിച്ചിരുന്നവരുടെ കൂട്ടത്തിൽ ആണെന്ന് സംശയിക്കപ്പെട്ടിരുന്ന വ്യക്തി. അദ്ദേഹം വ്യാജ ഹദീസുകൾ നിർമ്മിക്കുമായിരുന്നു.

വിമർശനം:

“പരിപൂർണ്ണതയിലെത്താത്ത മറ്റു ഡിവോഴ്സുകൾ”

ശറാഫ് ബിൻത് കലീഫ … മേൽ പറഞ്ഞ ഖവ്‌ല മരിച്ചതിന് ശേഷം അവരുടെ കുടുംബം മുഹമ്മദുമായി ബന്ധം പുനസ്ഥാപിച്ച് തടി രക്ഷിക്കാൻ വേണ്ടി അയച്ചു കൊടുത്ത സ്ത്രീ… ഇവരെ ഡിവോഴ്സ് ചെയ്ത് നബി ആ ഗോത്രവും ആയുള്ള കരാർ ഇല്ലാതാക്കി.. (തബാരി v9, P 138; ഇബ്നു സാദ് v 8, P 116 -117)

(മുഹമ്മദിന്റെ പെണ്ണുങ്ങൾ: നാസ്‌തിക സോഷ്യൽ മീഡിയ തെറിമാല)

മറുപടി:

നാസ്‌തിക സോഷ്യൽ മീഡിയ തെറിമാലകളിൽ ഒന്നായ “മുഹമ്മദിന്റെ പെണ്ണുങ്ങൾ” എന്ന കുറിപ്പിൽ നിന്നുള്ള ചില വരികളെയാണ് നാം തുടർച്ചയായി നിരൂപണം ചെയ്‌തു കൊണ്ടിരിക്കുന്ന്. മുമ്പ് സൂചിപ്പിച്ചതു പോലെ, സ്ത്രീവിമോചകനായ നബിയെ (സ), സ്ത്രീ പീഢകനും ലമ്പടനുമായി പ്രചരിപ്പിക്കാൻ വേണ്ടി കല്ലുവച്ച നുണകളും, അർദ്ധ സത്യങ്ങളും, ദുർവ്യാഖ്യാനങ്ങളും, വൈരുദ്ധ്യങ്ങളും കൂട്ടി കുഴച്ചുണ്ടാക്കിയ വിധ്വേഷ കഷായമാണ് ലേഖനം.

വിമർശനത്തിലേക്ക് വരാം…

ശറാഫ് ബിൻത് ഖലീഫയെ ദൈവദൂതൻ വിവാഹം ചെയ്തു എന്ന് സൂചിപ്പിക്കുന്ന നിവേദനം ഇപ്രകാരമാണ്:

أَخْبَرَنَا هِشَامُ بْنُ مُحَمَّدِ بْنِ السَّائِبِ ، قَالَ : حَدَّثَنَا الشَّرْقِيُّ بْنُ الْقَطَّامِيُّ قَالَ : لَمَّا هَلَكَتْ خَوْلَةُ بِنْتُ الْهُذَيْلِ تَزَوَّجَ رَسُولُ اللَّهِ صَلَّى اللَّهُ عَلَيْهِ وَسَلَّمَ شَرَافَ بِنْتَ خَلِيفَةَ أُخْتَ دِحْيَةَ وَلَمْ يَدْخُلْ بِهَا

ഹിശാമിബ്നു മുഹമ്മദ് ഇബ്നുസ്സാഇബ് അറിയിച്ചു: നമ്മോട് ശർക്വിയ്യിബ്നു കത്വാമി പറയുകയുണ്ടായി: ഖൗല ബിൻത് ഹുദൈൽ മരണപ്പെട്ടപ്പോൾ ദൈവദൂതൻ (സ) ശറാഫ് ബിൻത് ഖലീഫയെ – ദിഹ്‌യയുടെ സഹോദരി – വിവാഹം ചെയ്തു. അവരോടൊപ്പം താമസിച്ചില്ല (അതിനു മുമ്പ് വിവാഹ മോചനം നടന്നു)

1. സനദിലെ ഹിശാമിബ്നു മുഹമ്മദ് അൽ കൽബി നുണയനും ദുർബലനുമാണെന്ന, ഹദീസ് നിദാന ശാസ്ത്ര പണ്ഡിതന്മാരുടെ അഭിപ്രായങ്ങൾ മുമ്പത്തെ ലേഖനത്തിൽ ചേർത്തിട്ടുണ്ട്.

2. മറ്റൊരു നിവേദകനായ ‘ശർകിയ്യിബ്നു ക്വത്വാമി’ യും ദുർബലനാണ്. ഇതും മുമ്പത്തെ ലേഖനത്തിൽ വിശദമായി പ്രതിപാദിച്ചിട്ടുണ്ട്.

3. ശർകിയ് പ്രവാചകാലഘട്ടക്കാരനല്ല; താൻ ആരിൽ നിന്നാണ് ഈ കഥ കേട്ടതെന്ന് വ്യക്തമാക്കിയിട്ടുമില്ല. അഥവാ പരമ്പര കണ്ണി മുറിഞ്ഞതാണ്.

ഈ മൂന്ന് കാരണങ്ങളാൽ തന്നെ ഇത്തരമൊരു വിവാഹം പ്രവാചക ജീവിതത്തിൽ നടന്നിട്ടില്ലെന്നും, ചില തൽപര കക്ഷികൾ കെട്ടിച്ചമച്ചുണ്ടാക്കിയ വ്യാജകഥ മാത്രമാണിതെന്നും സുതരാം വ്യക്തമാണ്.

ബനൂ കൽബ് ഗോത്രക്കാരനായ ഹിശാമിബ്നു മുഹമ്മദ് അൽ കൽബി എന്ന റാവി തന്റെ ഗോത്രത്തിലെ ഒരുപാട് സ്ത്രീകളുമായി നബി(സ)ക്ക് വിവാഹ ബന്ധമുണ്ടായിരുന്നു എന്ന് ഗോത്ര മഹിമ പ്രചരിപ്പിക്കാൻ ചമച്ചുണ്ടാക്കിയതാവാം ഈ കഥകൾ മുഴുവൻ. എന്നിട്ട് ആരും ഈ വിവാഹങ്ങൾ അറിയാതിരുന്നതെന്തെ? എന്ന് നബി ചരിത്രത്തെയും നബി കുടുംബത്തെയും സംബന്ധിച്ച് വിവരമുള്ളവർ ചോദിക്കാനുള്ള സാധ്യത ഇല്ലാതാക്കാൻ, പെട്ടെന്ന് തന്നെ വിവാഹ മോചനങ്ങളും നടന്നു എന്നും കഥയിൽ റ്റ്വിസ്റ്റുണ്ടാക്കി പ്രചരിപ്പിച്ചു എന്ന് കരുതാനെ വഴിയുള്ളു.

സ്വമേധയാ ഇസ്‌ലാം സ്വീകരിച്ച ഗോത്രമാണ് കൽബ് ഗോത്രം. ആദ്യ കാലഘട്ടത്തിൽ തന്നെ ആവേശത്തോടെ ഇസ്‌ലാം ആശ്ലേഷിച്ച വ്യക്തിയായിരുന്നു – ശറാഫിന്റെ സഹോദരനായ – ദിഹ്‌യ ഇബ്നു ഖലീഫ. (സിയറു അഅ്ലാമിന്നുബലാഅ്: 2: 551)

പിന്നെ എവിടെ നിന്നാണ് “മുഹമ്മദുമായി ബന്ധം പുനസ്ഥാപിച്ച് തടി രക്ഷിക്കാൻ വേണ്ടി അയച്ചു കൊടുത്ത സ്ത്രീ… ” “ഇവരെ ഡിവോഴ്സ് ചെയ്ത് നബി ആ ഗോത്രവും ആയുള്ള കരാർ ഇല്ലാതാക്കി..” എന്ന് തുടങ്ങിയ നാസ്‌തികർ വകയായുള്ള വാചകങ്ങൾ നിവേദനത്തിൽ തിരുകി കയറ്റപ്പെട്ടത് ? വാചകം കേട്ടാൽ തോന്നുക മുഹമ്മദ് നബി (സ) ആ ഗോത്രത്തെ സായുധശക്തിയാൽ കീഴ്‌പ്പെടുത്തി സ്ത്രീകളെ അയച്ചു കൊടുക്കാൻ വല്ല കരാറും ഉണ്ടാക്കിയിട്ടുണ്ട് എന്ന്. ഇങ്ങനെ ചരിത്ര വസ്തുതകൾക്ക് നിരക്കാത്ത കുപ്രചരണങ്ങൾ നിവേദനങ്ങൾക്കിയിൽ തിരുകി കയറ്റി വളരെ സാധാരണമായ ഒരു വിവാഹത്തെ, ഭയത്തിന്റെ നിഴൽപ്പാടിൽ നടന്ന ‘പെണ്ണയച്ച് കൊടുക്കൽ’ ആക്കി മാറ്റാനുള്ള കുൽസിത ശ്രമമാണിത്. യുദ്ധമൊ കലാപമൊ ഒന്നും നിലനിൽക്കാത്ത, നബിയുടെ(സ) ഏറ്റവും പഴയ ആദർശ സ്നേഹികളും സുഹൃത്തുക്കളും നബിക്ക് കൊണ്ടുവന്ന വിവാഹാലോചന, എന്തിൽ നിന്ന് തടി രക്ഷിക്കാനാണ് എന്ന് തെളിവ് സഹിതം വ്യക്തമാക്കണം. വ്യംഗ്യമായ വാചകങ്ങളിലൂടെ ഇല്ലാക്കഥ നിർമ്മിക്കുന്ന സ്ഥിരം പരിപാടി തന്നെയാണ്, നടക്കാത്ത കല്യാണത്തിൽ നാസ്‌തികർ വക ഒരു ഇല്ലാത്ത കരാറും ‘പെണ്ണയച്ചു കൊടുക്കലും’.

അവസാനമായി സൂചിപ്പിക്കട്ടെ, ശറാഫ് ബിൻത് ഖലീഫയെ നബി (സ) വിവാഹം ചെയ്തിട്ടില്ല എന്ന നിവേദനവും ‘ത്വബകാത്തി’ ൽ ഉണ്ട് (ത്വബകാത്തു ഇബ്നു സഅ്ദ്: 8: 161) എങ്കിലും നാസ്‌തികർ അത് കാണില്ല. കാരണം, ‘ത്വബകാത്ത്’ കണ്ടിട്ടുള്ള, വായിച്ചിട്ടുള്ള ഒരു നാസ്‌തികനുമില്ലെന്ന് ഏവർക്കുമറിയാമല്ലൊ. എല്ലാത്തിന്റെയും സ്രോതസ്സുകൾ ഇസ്‌ലാമോഫോബിക്ക് വെബ്സൈറ്റുകൾ മാത്രമാണല്ലൊ.

വിമർശനം:

“പരിപൂർണ്ണതയിലെത്താത്ത മറ്റു ഡിവോഴ്സുകൾ”

ലൈല ബിൻത് ഖുത്യാം… മക്കയിലെ മുഹമ്മദിന്റെ ആദ്യകാലത്ത് ഇസ്ലാം സ്വീകരിച്ച സ്ത്രീ… പിന്നീട് മുഹമ്മദിനെ വിവാഹം കഴിക്കാൻ ആഗ്രഹം പ്രകടിപ്പിച്ചു… മുഹമ്മദ് പതിവു പോലെ അത് സ്വീകരിച്ചു… പക്ഷേ പിന്നീട് മുഹമ്മദിന്റെ ഭാര്യക്കൂട്ടത്തിൽ നിനക്ക് ഒത്തുപോവാൻ സാധിക്കില്ല എന്ന സ്വന്തം കുടുംബത്തിന്റെ വാക്കു കേട്ട് പിൻ വാങ്ങി (തബാരി v9, P 139, ഇബ്നു സാദ് 8:7, P 108 – 109, 231)

(മുഹമ്മദിന്റെ പെണ്ണുങ്ങൾ: നാസ്‌തിക സോഷ്യൽ മീഡിയ തെറിമാല)

മറുപടി:

നാസ്‌തിക സോഷ്യൽ മീഡിയ തെറിമാലകളിൽ ഒന്നായ “മുഹമ്മദിന്റെ പെണ്ണുങ്ങൾ” എന്ന കുറിപ്പിൽ നിന്നുള്ള ചില വരികളെയാണ് നാം തുടർച്ചയായി നിരൂപണം ചെയ്‌ത് കൊണ്ടിരിക്കുന്നത്. മുമ്പ് സൂചിപ്പിച്ചതു പോലെ, സ്ത്രീവിമോചകനായ നബിയെ(സ), സ്ത്രീ പീഢകനും ലമ്പടനുമായി പ്രചരിപ്പിക്കാൻ വേണ്ടി കല്ലുവച്ച നുണകളും, അർദ്ധ സത്യങ്ങളും, ദുർവ്യാഖ്യാനങ്ങളും, വൈരുദ്ധ്യങ്ങളും കൂട്ടി കുഴച്ചുണ്ടാക്കിയ വിധ്വേഷ കഷായമാണ് ലേഖനം.

ലൈല ബിൻത് അൽ ഖുതൈമിനെ വിവാഹം ചെയ്‌തതുമായി ബന്ധപ്പെട്ട് വന്ന കഥ ഇപ്രകാരമാണ്:

ലൈല ബിൻത് അൽ ഖുതൈം വിവാഹാഭ്യർത്ഥനയുമായി നബിയുടെ(സ) അടുക്കൽ വരുകയുണ്ടായി: ഞാൻ എന്റെ സ്വന്തത്തെ വിവാഹത്തിനായി അഭ്യർത്ഥിച്ചു കൊണ്ട് വന്നതാണ്. അതിനാൽ എന്നെ താങ്കൾ വിവാഹം ചെയ്‌താലും. നബി (സ) ചെയ്യാമെന്ന് അംഗീകരിച്ചു. ഉടനെ ലൈല സന്തോഷത്തോടെ തന്നെ നാട്ടുകാരുടെ അടുത്തു ചെന്ന് വിളമ്പരം ചെയ്‌തു: എന്നെ ദൈവദൂതൻ വിവാഹം ചെയ്‌തിരിക്കുന്നു. ഇതു കേട്ടപ്പോൾ നാട്ടുകാർ പറഞ്ഞു: നീ ചെയ്‌തത് എത്ര മോശം. നീ വളരെ അഭിമാന രോഗമുള്ള ഒരു സ്ത്രീയാണ്. നബിക്കാകട്ടെ വേറെയും ഭാര്യമാരുണ്ട്. അവരോട് നീ വഴക്കിലാവുക തന്നെ ചെയ്യും. അപ്പോൾ അദ്ദേഹം നിനക്കെതിരെ പ്രാർത്ഥിച്ചാലോ? അതിനാൽ നീ അദ്ദേഹത്തിൽ നിന്ന് മോചനം വാങ്ങുക. അങ്ങനെ ലൈല നബി(സ)യുടെ അടുത്ത് വന്ന് പറഞ്ഞു: ദൈവദൂതരേ, എന്നെ വിവാഹ മോചനം ചെയ്‌താലും. നബി (സ) പറഞ്ഞു: ശരി, ഞാനിതാ നിന്നെ മോചിപ്പിച്ചിരിക്കുന്നു. അങ്ങനെ ലൈല പിന്നീട് മസ്ഊദിബ്നു ഔസിനെ വിവാഹം ചെയ്യുകയുണ്ടായി…”

(ത്വബകാത്തു ഇബ്നു സഅ്ദ്: 8/150)

നിവേദനം സ്വഹീഹ് ആയ (സ്വീകാര്യയോഗമായ) പരമ്പരയിലൂടെ വന്നിട്ടില്ല. രണ്ട് സ്രോതസ്സുകളാണ് കഥക്കുള്ളത്.

ഒന്ന്:

أخبرنا هشام بن محمد بن السائب عن أبيه عن أبي صالح عن بن عباس قال أقبلت ليلى بنت الخطيم…..

രണ്ട്:

أخبرنا محمد بن عمر حدثنا عبد الله بن جعفر عن بن أبي عون أن ليلى بنت الخطيم

ഹിശാമിബ്നു മുഹമ്മദ് ഇബ്നു സാഇബ് തന്റെ പിതാവിൽ നിന്ന്… ഉദ്ധരിക്കുന്നതാണ് ഒന്നാമത്തെ നിവേദനം. ഹിശാമിബ്നു മുഹമ്മദ് അൽ കൽബിയും അദ്ദേഹത്തിന്റെ പിതാവും നുണ പറയുന്നവരും ദുർബലരുമാണെന്നതിൽ ഹദീസ് നിദാന ശാസ്ത്ര പണ്ഡിതർക്കിടയിൽ ഇരു പക്ഷമില്ല.

ഇമാം ദഹബി എഴുതി: …കൂഫക്കാരനായ ഇദ്ദേഹം ശിഈയും കളവു പറയുന്ന വ്യക്തിയുമായിരുന്നു; അദ്ദേഹത്തിന്റെ പിതാവും തഥൈവ. അഹ്മദിബ്നു ഹമ്പൽ പറഞ്ഞു: …അദ്ദേഹത്തിൽ നിന്ന് ആരും ഹദീസ് ഉദ്ധരിക്കുമെന്ന് ഞാൻ കരുതുന്നില്ല. ഇമാം ദാറകുത്നിയും മറ്റു പണ്ഡിതരും പറഞ്ഞു: ഇദ്ദേഹം കളവു പറയുന്നതായി ആരോപിക്കപ്പെട്ടിരിക്കുന്നതിനാൽ ഇദ്ദേഹത്തിന്റെ ഹദീസുകൾ തള്ളപ്പെട്ടവയാണ്. ഇബ്നു അസാകിർ പറഞ്ഞു: അദ്ദേഹം റാഫിദിയാണ്, വിശ്വസ്തനല്ല. ക്വുർആൻ മൂന്ന് ദിവസം കൊണ്ട് താൻ മനപാഠമാക്കി എന്നെല്ലാം അസത്യ വീര വാദങ്ങൾ മുഴക്കുമായിരുന്നു.

(സിയറു അഅ്ലാമിന്നുബലാഅ്: 10: 101, 102, മീസാനുൽ ഇഅ്തിദാൽ)

മുഹമ്മദ് ഇബ്നു ഉമറാണ് രണ്ടാമത്തെ നിവേദനത്തിന്റെ മൂല നിവേദകൻ. അദ്ദേഹം ദുർബലനാണ്. അഹ്മദിബ്നു ഹമ്പൽ പറഞ്ഞു: മുഹമ്മദിബ്നു ഉമർ വാക്കിദുൽ അസ്‌ലമി നുണയനാണ്; അയാൾ ഹദീസുകളിൽ കോട്ടിമാട്ടുമായിരുന്നു.

യഹ്‌യ പറഞ്ഞു: അയാൾ വിശ്വസ്തനല്ല. അയാളുടെ ഹദീസുകൾ എഴുതിവെക്കാൻ കൊള്ളാത്തത്രയും അവിശ്വസനീയമാണ്.

ഇമാം ബുഖാരി, റാസി, നസാഈ എന്നിവർ പറഞ്ഞു: അയാൾ കളവു കൊണ്ട് ആരോപിതനാണ്. റാസി, നസാഈ എന്നിവർ പറഞ്ഞു: അയാൾ വ്യാജ ഹദീസുകൾ ഉണ്ടാക്കുന്ന വ്യക്തിയായിരുന്നു. ഇമാം ദാറക്കുത്നി പറഞ്ഞു: അയാളിൽ ദൗർബല്യമുണ്ട്. ഇസ്ഹാകിബ്നു റാഹൂയ പറഞ്ഞു: അയാൾ നുണയനാണ്.

(അദ്ദുഅഫാഉ വൽ മത്റൂകീൻ: ഇബ്നുൽ ജൗസി: 3/ 87, അദ്ദുഅഫാഉ സ്സ്വഗീർ: ബുഖാരി: 334, അൽ ജർഹുവതഅദീൽ: അബൂഹാതിം: 8/ 21, അൽ കാമിൽ ഇബ്നു അദിയ്യ്: 7/ 481)

എന്ന് മാത്രമല്ല, ലൈലയുമായുള്ള വിവാഹ കഥ നിവേദനം ചെയ്‌ത മുഹമ്മദ് ഇബ്നു ഉമർ അൽവാക്വിദി എന്ന നിവേദകൻ തന്നെ അത്തരമൊരു വിവാഹം നടന്നിട്ടില്ല എന്ന് നിവേദനത്തിന് തൊട്ടു തുടർച്ചയായി വിശദീകരിച്ചിട്ടുണ്ട്.

أخبرنا محمد بن عمر حدثنا عبد الله بن جعفر عن بن أبي عون أن ليلى بنت الخطيم وهبت نفسها للنبي صلى الله عليه وسلم ووهبن نساء أنفسهن فلم يسمع أن النبي صلى الله عليه وسلم قبل منهن أحدا

(ത്വബകാത്തു ഇബ്നു സഅ്ദ്: 8: 151)

അഥവാ കഥ ഉദ്ധരിച്ച നിവേദകൻ തന്നെ കഥയുടെ സത്യത നിഷേധിച്ചിട്ടുണ്ട് എന്നർത്ഥം. പക്ഷെ ഇബ്നു സഅ്ദിന്റെ ത്വബകാത്തിൽ നിന്ന് വിവാഹ കഥ മാത്രം കാണുകയും തൊട്ടുടനെ അതിന്റെ സത്യതയെ നിരാകരിച്ചു കൊണ്ടുള്ള പ്രസ്‌താവന നാസ്‌തികർ കാണാതെ പോയത് എങ്ങനെയാണ് ? ഉത്തരം ലളിതം, മിഷണറിമാരുടെ പഴയ വിധ്വേഷ വീഞ്ഞ് നാസ്‌തികൻ പുതിയ കുപ്പിയിൽ പുതിയ സ്റ്റിക്കറൊട്ടിച്ച് അവതരിപ്പിച്ചതാണ്. സ്വന്തമായി ഒരു പഠനമോ പരിശോധനയോ നടത്താനുള്ള മാന്യതയൊ കഴിവോ ഉള്ള ഒരാളും ഈ വ്യാജ ഗവേഷകർക്കിടയിൽ ഇല്ല.

ഈ നിവേദനം സ്വഹീഹ് ആണെന്ന് കരുതുക. സ്ത്രീ പീഢകനും ലൈംഗിക കൊതിയനുമായ ഒരാളുടെ ചിത്രമല്ല ഈ കഥയിൽ ആർക്കും ദർശിക്കാൻ കഴിയുന്നത് എന്ന് സാന്ദർഭികമായി നാം ശ്രദ്ധയിലേക്ക് കൊണ്ടു വരട്ടെ. മുഹമ്മദ് നബിയല്ല(സ) ലൈലയാണ് വിവാഹം ആഗ്രഹിച്ചു കൊണ്ട് അഭ്യർത്ഥനയുമായി വന്നത്. തന്റെ ന്യൂനതകളും കോപ പ്രകൃതിയും പരിഗണിച്ച് വിവാഹ മോചനത്തെ സംബന്ധിച്ച് ആലോചിച്ചതും ലൈല തന്നെ. തനിക്ക് വിവാഹ മോചനം വേണമെന്ന് ആവശ്യപ്പെട്ട ഉടനെ വിവാഹ മോചനം നൽകിയത് ആ അറബ് ഉപഭൂഖണ്ഡത്തിന്റെ ആത്മീയവും ഭൗതികവുമായ നേതാവായ പ്രവാചകൻ (സ) !! ഒഴികഴിവുകൾ പറഞ്ഞില്ല, ബലാൽകാരങ്ങളില്ല, പ്രകോപനങ്ങളൊ ഭീഷണികളൊ ഇല്ല, കേസും കോടതി വിചാരണകളും ഇല്ല. തന്റെ ഇഷ്ടാനുസാരം സ്വാതന്ത്ര്യത്തിന്റെ ചക്രവാള വിശാലതയിൽ പാറി പറക്കുന്ന സ്ത്രീകളുടെ ചിത്രം മാത്രം… വിവാഹവും വിവാഹ മോചനവും ഇണയും തുണയുമെല്ലാം അവളുടെ തൃപ്തിക്ക് വിട്ടു കൊടുത്ത ഒരു സ്ത്രീവിമോചനത്തിന്റെ സാമ്രാട്ട്; അതായിരുന്നു മുഹമ്മദ് നബി (സ).

വിമർശനം:

“പരിപൂർണ്ണതയിലെത്താത്ത മറ്റു ഡിവോഴ്സുകൾ”

ഖൗല ബിൻത് ഹുധ്യാൽ… ഒരു ക്രിസ്ത്യൻ ഗോത്രത്തിലെ രാജകുമാരി… മുഹമദുമായി വിവാഹം നടത്തിയതിന് ശേഷം മദീനയിലേക്കുള്ള യാത്രാമധ്യേ അവർ മരിച്ചു (ആത്മഹത്യയാവാം) (അൽ തബാരി – വോളിയം 9, പേജ് 139, 166; ഇബ്നു സാദ് 8:116)

(മുഹമ്മദിന്റെ പെണ്ണുങ്ങൾ: നാസ്‌തിക സോഷ്യൽ മീഡിയ തെറിമാല)

മറുപടി:

നാസ്‌തിക സോഷ്യൽ മീഡിയ തെറിമാലകളിൽ ഒന്നായ “മുഹമ്മദിന്റെ പെണ്ണുങ്ങൾ” എന്ന കുറിപ്പിൽ നിന്നുള്ള ചില വരികളെയാണ് നാം തുടർച്ചയായി നിരൂപണം ചെയ്ത് കൊണ്ടിരിക്കുന്നത്. മുമ്പ് സൂചിപ്പിച്ചതു പോലെ, സ്ത്രീവിമോചകനായ നബിയെ(സ), സ്ത്രീ പീഢകനും ലമ്പടനുമായി പ്രചരിപ്പിക്കാൻ വേണ്ടി കല്ലുവച്ച നുണകളും, അർദ്ധ സത്യങ്ങളും, ദുർവ്യാഖ്യാനങ്ങളും, വൈരുദ്ധ്യങ്ങളും കൂട്ടി കുഴച്ചുണ്ടാക്കിയ വിധ്വേഷ കഷായമാണ് ലേഖനം.

ഖൗല ബിൻത് അൽ ഹുദൈലുമായുള്ള നബിയുടെ(സ) വിവാഹത്തെ സംബന്ധിച്ച് ഇബ്നു സഅ്ദ് തന്റെ ത്വബകാത്തിൽ ഉദ്ധരിച്ച സനദ് (നിവേദക പരമ്പര) ഇപ്രകാരമാണ്:

أخبرنا هشام بن محمد حدثني الشرقي بن القطامي

1. സനദിലെ ഹിശാമിബ്നു മുഹമ്മദ് അൽ കൽബി നുണയനും ദുർബലനുമാണെന്നതിൽ ഹദീസ് നിദാന ശാസ്ത്ര പണ്ഡിതർക്കിടയിൽ ഇരു പക്ഷമില്ല.

ഇമാം ദഹബി എഴുതി: …കൂഫക്കാരനായ ഇദ്ദേഹം ശിഈയും കളവു പറയുന്ന വ്യക്തിയുമായിരുന്നു; അദ്ദേഹത്തിന്റെ പിതാവും തഥൈവ. അഹ്മദിബ്നു ഹമ്പൽ പറഞ്ഞു: …അദ്ദേഹത്തിൽ നിന്ന് ആരും ഹദീസ് ഉദ്ധരിക്കുമെന്ന് ഞാൻ കരുതുന്നില്ല. ഇമാം ദാറകുത്നിയും മറ്റു പണ്ഡിതരും പറഞ്ഞു: ഇദ്ദേഹം കളവു പറയുന്നതായി ആരോപിക്കപ്പെട്ടിരിക്കുന്നതിനാൽ ഇദ്ദേഹത്തിന്റെ ഹദീസുകൾ തള്ളപ്പെട്ടവയാണ്. ഇബ്നു അസാകിർ പറഞ്ഞു: അദ്ദേഹം റാഫിദിയാണ്, വിശ്വസ്തനല്ല. ക്വുർആൻ മൂന്ന് ദിവസം കൊണ്ട് താൻ മനപാഠമാക്കി എന്നെല്ലാം അസത്യ വീര വാദങ്ങൾ മുഴക്കുമായിരുന്നു. (സിയറു അഅ്ലാമിന്നുബലാഅ്: 10:101,102, മീസാനുൽ ഇഅ്തിദാൽ)

2. മറ്റൊരു നിവേദകനായ ‘ശർകിയ്യിബ്നു ക്വത്വാമി’ യും ദുർബലനാണ്.

ഇമാം ദഹബി എഴുതി: സ്വഹീഹായ ഹദീസുകൾക്ക് വിരുദ്ധമായി പത്തോളം അതി ദുർബലമായ (മുൻകറായ) ഹദീസുകൾ ശർകിയ് ഉദ്ധരിക്കുമായിരുന്നു. അദ്ദേഹം ദുർബലനാണെന്ന് സകരിയ്യ അസ്സാജിയും അഭിപ്രായപ്പെട്ടിട്ടുണ്ട്. (മീസാനുൽ ഇഅ്തിദാൽ: 2: 268)

وقال النديم في الفهرست: اسمه الوليد بن الحصين قرأت بخط اليوسفي كان كذابا ويكنى أبا المثنى.))

ഇബ്നു നദീം തന്റെ ഫഹ്റസത്തിൽ പറഞ്ഞിരിക്കുന്നത് ശർകിയ് കളവു പറയുന്ന വ്യക്തിയാണ് എന്നാണ്. (ലിസാനുൽ മീസാൻ: ഇബ്നു ഹജർ:4: 241)

3. മാത്രമല്ല, ശർകിയ് പ്രവാചകാലഘട്ടകാരനല്ല; താൻ ആരിൽ നിന്നാണ് ഈ കഥ കേട്ടതെന്ന് വ്യക്തമാക്കിയിട്ടുമില്ല. അഥവാ പരമ്പര കണ്ണി മുറിഞ്ഞതാണ്.

ഈ മൂന്ന് കാരണങ്ങളാൽ തന്നെ ഇത്തരമൊരു വിവാഹം പ്രവാചക ജീവിതത്തിൽ നടന്നിട്ടില്ലെന്നും, ചില തൽപര കക്ഷികൾ കെട്ടിച്ചമച്ചുണ്ടാക്കിയ വ്യാജകഥ മാത്രമാണിതെന്നും സുതരാം വ്യക്തമാണ്.

സാങ്കൽപ്പിക വിവാഹത്തിലെ, ഭാവനാത്മകമായ “പ്രവാചക പത്നി”, വഴിയിൽ വെച്ച് മരണപ്പെട്ടു (فهلكت في الطريق) എന്ന ഒരു വ്യാജ നിവേദനം പൊക്കിപ്പിടിച്ച്, “ആത്മഹത്യയാവാം” എന്ന മനസ്സിലെ വിഷവിത്ത് തിരുകി കയറ്റി, മുതല കണ്ണീരൊഴുക്കി… നബി വിധ്വേഷം ഞെക്കി തുറിപ്പിച്ചുണ്ടാക്കാനുള്ള കഷ്ടപ്പാടെത്രയാണ്!!

വിമർശനം:

“…വിവാഹം ചെയ്ത് ഭോഗിച്ചു… 4 എണ്ണത്തിനെ ബാലിശമായ കാരണങ്ങൾ പറഞ്ഞ് ഡിവോഴ്സ് ചെയ്തു…. അധികം പേർക്കും അറിയാത്ത (മുഹമ്മദ് നബി) മൊഴിചൊല്ലിയ സ്ത്രീകളുടെ പേരുകൾ ഇവയാണ്…

3) ഫാത്തിമ അൽ അലിയാ ബിൻത് സാബിയാൻ അൽ ദഹാക്ക്…. മറ്റൊരു പുരുഷനെ ഒളിഞ്ഞു നോക്കി എന്നും പറഞ്ഞ് മൊഴി ചൊല്ലി (തബാരി v9, P 138; തബാരി v39 P 186 – 188)

4) അമ്ര ബിൻത് യാസിദ്…. (ഇബ്നു ഇഷാഖ്, സീറത്തുൽ റസൂലള്ളാ P 155)”

(മുഹമ്മദിന്റെ പെണ്ണുങ്ങൾ: നാസ്തിക സോഷ്യൽ മീഡിയ തെറിമാല)

മറുപടി:

നാസ്തിക സോഷ്യൽ മീഡിയ തെറിമാലകളിൽ ഒന്നായ “മുഹമ്മദിന്റെ പെണ്ണുങ്ങൾ” എന്ന കുറിപ്പിൽ നിന്നുള്ള ചില വരികളെയാണ് നാം തുടർച്ചയായി നിരൂപണം ചെയ്ത് കൊണ്ടിരിക്കുന്നത്. മുമ്പ് സൂചിപ്പിച്ചതു പോലെ, കല്ലുവച്ച നുണകളും, അർദ്ധ സത്യങ്ങളും, ദുർവ്യാഖ്യാനങ്ങളും, വൈരുദ്ധ്യങ്ങളും നിറഞ്ഞതാണ് ലേഖനം.

“കിലാബ് ഗോത്രക്കാരിയായ ഒരു സ്ത്രീയെ നബി (സ) വിവാഹം ചെയ്തു എന്ന് പറയപ്പെടുന്നു… എന്താണ് ആ സ്ത്രീയുടെ നാമം എന്ന കാര്യത്തിൽ പണ്ഡിതന്മാർക്കിടയിൽ ധാരാളം അഭിപ്രായ വ്യത്യാസങ്ങളുണ്ട്. ചിലർ പറഞ്ഞു: അവരുടെ നാമം ഫാത്വിമ ബിൻത് ദഹ്ഹാക് ഇബ്നു സുഫ്യാൻ അൽ കിലാബിയ്യ എന്നാണ്. വേറെ ചിലർ പറഞ്ഞു: അവരുടെ നാമം അംറ ബിൻത് യസീദ് ബിൻ ഉബൈദ് ബിൻ റുവാസ് ബിൻ കിലാബ് ബിൻ റബീഅ ബിൻ ആമിർ എന്നാണ്. മറ്റൊരാൾ പറയുന്നത് അവരുടെ നാമം ആലിയ ബിൻത് ളബ്യാൻ ബിൻ അംറ് ബിൻ ഔഫ് ബിൻ കഅ്ബ് ബിൻ അബ്ദ് ബിൻ അബൂബകർ ബിൻ കിലാബ് എന്നാണ്. ഒരാൾ പാഞ്ഞു: അവർ സബാ ബിൻത് സുഫ്യാൻ ബിൻ ഔഫ് ബിൻ കഅ്ബ് ബിൻ അബ്ദ് ബിൻ അബൂബകർ ബിൻ കിലാബ് എന്നാണ്. ഇക്കാര്യത്തിൽ നാം കേട്ട നിവേദനങ്ങളെല്ലാം നാം ഇവിടെ എഴുതി എന്നു മാത്രം. ചിലർ പറയുന്നു: ഒരൊറ്റ കിലാബ് കാരിയെ മാത്രമെ നബി (സ) വിവാഹം കഴിച്ചിട്ടുള്ളു. അവരുടെ പേരിന്റെ കാര്യത്തിൽ സംശയങ്ങളുണ്ടായതാണ്…” (ത്വബകാതു ഇബ്നു സഅ്ദ്: 8/112)

ഇതാണ് കിലാബ് കാരിയായ സ്ത്രീയുമായി ബന്ധപ്പെട്ട കഥയുടെ അവസ്ഥ !! അത്തരമൊരു വിവാഹം നടന്നിട്ടുണ്ടൊ ഇല്ലേ ? നടന്നെങ്കിൽ അവരുടെ പേരെന്താണ്? ആ പേരുകൾ എല്ലാം ഒരേ ആളാണൊ അതൊ വ്യത്യസ്തരായ സ്ത്രീകളാണൊ എന്നൊന്നും ഉള്ളതിന് കൃത്യമായ ഒരു തെളിവുമില്ല. വിമർശകർ തന്നെ – മുഹമ്മദിന്റെ പെണ്ണുങ്ങൾ എന്ന ലേഖനത്തിൽ – ഉദ്ധരിച്ച പേര് “ഫാത്തിമ അൽ അലിയാ ബിൻത് സാബിയാൻ അൽ ദഹാക്ക്” എന്നാണ് !! ഇത് രണ്ട് പേരുകൾ കൂട്ടിയോജിപ്പിച്ച ഒരു പേരാണ്. 1. ഫാത്വിമ ബിൻത് ദഹ്ഹാക് 2. ആലിയ ബിൻത് ളബ്യാൻ എന്ന രണ്ടു പേരുകൾ ചേർത്ത് “ഫാത്തിമ അൽ അലിയാ ബിൻത് സാബിയാൻ അൽ ദഹാക്ക്” എന്നായി.

കഥയുടെ ഉള്ളടക്കം ഉമൈമയുടെ കഥക്ക് സമാനം തന്നെ ! പ്രവാചകൻ (സ) ‘ഫാത്വിമ ബിൻത് ദഹ്ഹാക്’ അല്ലെങ്കിൽ ‘ആലിയ ബിൻത് ളബ്യാനെ’ അല്ലെങ്കിൽ ‘അംറ ബിൻത് യസീദിനെ’ അതോ ‘സബാ ബിൻത് സുഫ്യാനെ’ (!) വിവാഹം ചെയ്യുകയും അവരുടെ അടുത്ത് പ്രവേശിക്കുകയും ചെയ്തപ്പോൾ അവർ അദ്ദേഹത്തിൽ നിന്ന് അല്ലാഹുവിൽ ശരണം തേടി. അപ്പോൾ നബി (സ) അവരെ വീട്ടിലേക്ക് പൊയ്കൊള്ളാൻ നിർദ്ദേശിച്ച് സ്വതന്ത്രരാക്കി.

മുമ്പ് പല ലേഖനങ്ങളിൽ സൂചിപ്പിച്ചതു പോലെ ഉമൈമ എന്ന ഭാര്യയുടെ കഥ പല നാമങ്ങളിലായി പുനർ നിർമ്മിച്ചെടുത്ത വ്യാജ കഥകളും വ്യാജ ഭാര്യമാരുമാണ് ഇവ/ ഇവർ എല്ലാം.

وأشار ابن سعد إلى أنها واحدة اختلف في اسمها ، والصحيح أن التي استعاذت منه هي الجونية . وروى ابن سعد من طريق سعيد بن عبد الرحمن بن أبزى قال : لم تستعذ منه امرأة غيرها . ഇബ്നു ഹജർ (റ) പറഞ്ഞു: ഒരൊറ്റ ഭാര്യയുടെ കാര്യത്തിൽ നടന്നതാണ് ഈ ശരണ തേട്ടവും വിവാഹ മോചനവും; അവരുടെ പേരെന്താണ് എന്ന കാര്യത്തിൽ അഭിപ്രായ വ്യത്യാസം വന്നു എന്ന് മാത്രം. നബിയിൽ(സ) നിന്ന് ശരണം തേടിയത് (ഉമൈമ) ജുവനിയ ആണെന്നതാണ് സ്വഹീഹ് (വിശ്വസ്‌തമായ ഹദീസിലൂടെ സ്ഥാപിതമായ വസ്തുത). സഈദിബ്നു അബ്ദുർറഹ്‌മാൻ ബിൻ അബ്സാ പറഞ്ഞതായി ഇബ്നു സഅ്ദ് ഉദ്ധരിച്ചിരിക്കുന്നു: ഉമൈമയല്ലാതെ മറ്റൊരു സ്ത്രീയും നബിയിൽ നിന്ന് അല്ലാഹുവിൽ ശരണം തേടിയിട്ടില്ല. (ഫത്ഹുൽ ബാരി: 9:269)

ത്വബ്‌രി, ഇബ്നു സഅ്ദ്, ദഹബി പോലുള്ള എല്ലാ ചരിത്രകാരന്മാരും ഈ കഥകൾ ഉദ്ധരിക്കുന്നത് മുഹമ്മദ് ഇബ്നു ഉമർ ഇബ്നുൽ വാക്വിദ് എന്ന ചരിത്രകാരനിൽ നിന്നാണ്. കഥകളുടെ വ്യത്യസ്ത നിവേദനങ്ങളുടെ സനദുകൾ കാണുക:

1. أَخْبَرَنَا مُحَمَّدُ بْنُ عمر. حَدَّثَنَا مُحَمَّدُ بْنُ عَبْدِ اللَّهِ عَنِ الزُّهْرِيِّ قَالَ: … 2. أَخْبَرَنَا مُحَمَّدُ بْنُ عُمَرَ. حَدَّثَنِي مُحَمَّدُ بْنُ عَبْدِ اللَّهِ عَنِ الزُّهْرِيِّ عَنْ عُرْوَةَ عَنْ عَائِشَةَ قَالَتْ: … 3. أَخْبَرَنَا مُحَمَّدُ بْنُ عُمَرَ. حَدَّثَنَا عَبْدُ اللَّهِ بْنُ جَعْفَرٍ عَنْ عَبْدِ الْوَاحِدِ بن أبي عون عن ابْنِ مَنَّاحٍ قَالَ: … 4. أَخْبَرَنَا مُحَمَّدُ بْنُ عُمَرَ. حَدَّثَنَا عَبْدُ اللَّهِ بْنُ سُلَيْمَانَ عَنْ عَمْرِو بْنِ شُعَيْبٍ عَنْ أَبِيهِ عَنْ جَدِّهِ قَالَ: … 5. أَخْبَرَنَا مُحَمَّدُ بْنُ عُمَرَ. أَخْبَرَنَا عَبْدُ اللَّهِ بْنُ جَعْفَرٍ عَنْ مُوسَى بْنِ سَعِيدٍ وَابْنِ أَبِي عَوْنٍ قَالا: …

എല്ലാ നിവേദക പരമ്പരകളും ആരംഭിക്കുന്നത് തന്നെ മുഹമ്മദ് ഇബ്നു ഉമർ ഇബ്നുൽ വാക്വിദ് എന്ന ചരിത്രകാരനിൽ നിന്നാണ്. അദ്ദേഹത്തെ സംബന്ധിച്ച ഹദീസ് – ചരിത്ര നിദാന ശാസ്ത്ര പണ്ഡിതന്മാരുട അഭിപ്രായം ഇവിടെ ആവർത്തിക്കട്ടെ:

അഹ്മദിബ്നു ഹമ്പൽ പറഞ്ഞു: മുഹമ്മദിബ്നു ഉമർ വാക്കിദുൽ അസ്‌ലമി നുണയനാണ്; അയാൾ ഹദീസുകളിൽ കോട്ടിമാട്ടുമായിരുന്നു.

യഹ്‌യ പറഞ്ഞു: അയാൾ വിശ്വസ്തനല്ല. അയാളുടെ ഹദീസുകൾ എഴുതിവെക്കാൻ കൊള്ളാത്തത്രയും അവിശ്വസനീയമാണ്.

ഇമാം ബുഖാരി, റാസി, നസാഈ എന്നിവർ പറഞ്ഞു: അയാൾ കളവു കൊണ്ട് ആരോപിതനാണ്.

റാസി, നസാഈ എന്നിവർ പറഞ്ഞു: അയാൾ വ്യാജ ഹദീസുകൾ ഉണ്ടാക്കുന്ന വ്യക്തിയായിരുന്നു.

ഇമാം ദാറക്കുത്നി പറഞ്ഞു: അയാളിൽ ദൗർബല്യമുണ്ട്.

ഇസ്ഹാകിബ്നു റാഹൂയ പറഞ്ഞു: അയാൾ നുണയനാണ്.

(അദ്ദുഅഫാഉ വൽ മത്റൂകീൻ: ഇബ്നുൽ ജൗസി: 3 / 87, അദ്ദുഅഫാഉ സ്സ്വഗീർ: ബുഖാരി: 334, അൽ ജർഹുവതഅദീൽ: അബൂഹാതിം: 8/21, അൽ കാമിൽ ഇബ്നു അദിയ്യ്: 7/ 481)

സാന്ദർഭികമായി ചില ചോദ്യങ്ങൾ ആവർത്തിക്കട്ടെ ?

കഥ യാഥാർഥ്യമാണെങ്കിൽ തന്നെ നബി (സ) പെണ്ണുങ്ങളെ കെട്ടി ഭോഗിച്ചു, ബാലിശമായ കാരണങ്ങൾ പറഞ്ഞ് മൊഴി ചൊല്ലി എന്നതെല്ലാം നാസ്തിക നുണകൾ മാത്രമല്ലെ ?! ലൈംഗിക ബന്ധം നടന്നിട്ടില്ല എന്നതും നബിയല്ല, നവവധുവാണ് ഇഷ്ടമില്ലായ്മ പ്രകടിപ്പിച്ചത് എന്നതും എന്തുകൊണ്ട് മറച്ചു പിടിക്കുന്നു ?! താൽപര്യമില്ലാത്ത വിവാഹ ബന്ധത്തിൽ നിന്ന് സ്വഭാര്യയെ മോചിപ്പിച്ച നബി, സ്ത്രീകളോട് പുലർത്തിയ മാന്യതയും ദയാപരതയും ലൈംഗിക വിശുദ്ധിയും എന്തുകൊണ്ട് കട്ടുമുക്കുന്നു ?!!

ചില (വ്യാജ) നിവേദനങ്ങളിൽ, ഫാതിമയെ വിവാഹ മോചനം ചെയ്യാനുണ്ടായ സാഹചര്യം അവർ സ്ഥിരമായി പുരുഷന്മാരെ ഒളിഞ്ഞു നോക്കാറുണ്ടായിരുന്നു എന്നും, ഇത് നബി (സ) നേരിട്ട് കാണുകയുണ്ടായി എന്നും വന്നിട്ടുണ്ട്. ഇതിനെ നിസ്സാര കാര്യമെന്നോണം നാസ്തികൻ ഇപ്രകാരം കുറിച്ചിട്ടിരിക്കുന്നത് വായിക്കാം: “മറ്റൊരു പുരുഷനെ ഒളിഞ്ഞു നോക്കി എന്നും പറഞ്ഞ് മൊഴി ചൊല്ലി”… ! പുരുഷന്മാരെ ഒളിഞ്ഞു നോക്കുന്ന ഒരു സ്ത്രീയെ ഭാര്യയായി വെക്കാൻ നാസ്തികർ തയ്യാറാകുമോ എന്നു കൂടി നാസ്തിക കോപ്പിയടിയന്മാർ വ്യക്തമാക്കേണ്ടതുണ്ട്.

വിമർശനം:

” …വിവാഹം ചെയ്ത് ഭോഗിച്ചു… 4 എണ്ണത്തിനെ ബാലിശമായ കാരണങ്ങൾ പറഞ്ഞ് ഡിവോഴ്സ് ചെയ്തു…. അധികം പേർക്കും അറിയാത്ത (മുഹമ്മദ് നബി) മൊഴിചൊല്ലിയ സ്ത്രീകളുടെ പേരുകൾ ഇവയാണ്…

2) മുലൈഖ… തന്റെ പിതാവിനേയും മറ്റും മുഹമ്മദ് നേരിട്ടാണ് കൊന്നത് എന്ന് തിരിച്ചറിഞ്ഞതിനു ശേഷം ഡിവോഴ്സ് ആവശ്യപ്പെട്ടു.. മുഹമ്മദ് കൊടുത്തു,… (തബാരി v39, P 165)”

(മുഹമ്മദിന്റെ പെണ്ണുങ്ങൾ: നാസ്‌തിക സോഷ്യൽ മീഡിയ തെറിമാല)

മറുപടി:

നാസ്‌തിക സോഷ്യൽ മീഡിയ തെറിമാലകളിൽ ഒന്നായ “മുഹമ്മദിന്റെ പെണ്ണുങ്ങൾ” എന്ന കുറിപ്പിൽ നിന്നുള്ള ചില വരികളാണ് ഈ വായിച്ചത്. മുമ്പ് സൂചിപ്പിച്ചതു പോലെ, കല്ലുവച്ച നുണകളും, അർദ്ധ സത്യങ്ങളും, ദുർവ്യാഖ്യാനങ്ങളും, വൈരുദ്ധ്യങ്ങളും നിറഞ്ഞതാണ് ലേഖനം.

1. മുലൈഖ ബിൻത് കഅ്ബിനെ വിവാഹ മോചനം ചെയ്തതുമായി ബന്ധപ്പെട്ട് വിമർശകർ അവലംബിക്കുന്ന നിവേദനം വളരെയേറെ ദുർബലമാണ്. നിവേദനത്തിന്റെ ഉള്ളടക്കം ഇപ്രകാരമാണ്:

تزوج النبي صلى الله عليه وسلم مليكة بنت كعب، وكانت تُذكر بجمال بارع، فدخلَت عليها عائشة، فقالت لها: أما تستحين أن تنكحي قاتِلَ أبيك، فاستعاذت من رسول الله صلى الله عليه وسلم، فطلَّقها، فجاء قومها إلى النبي صلى الله عليه وسلم، فقالوا: يا رسول الله، إنها صغيرة، وإنها لا رأيَ لها وإنها خدعت فارتجعها، فأبى رسول الله صلى الله عليه وسلم، وكان أبوها قتل في يوم فتح مكة، قتَله خالد بن الوليد بالخندمة

മക്കാ വിജയ വേളയിൽ നബിയുടെ(സ) ശിഷ്യൻ ഖാലിദിബ്നു വലീദ് കഅ്ബിനെ വധിക്കുകയുണ്ടായി. അദ്ദേഹത്തിന്റെ മകൾ മുലൈഖ ബിൻത് കഅ്ബിനെ നബി (സ) വിവാഹം ചെയ്തു. അവരോട് അസൂയ തോന്നിയ ആഇശ (റ) അവരോട് ഇപ്രകാരം പറഞ്ഞു: നിങ്ങളുടെ പിതാവിന്റെ ഘാതകനെ വിവാഹം ചെയ്യാൻ നിങ്ങൾക്ക് മടിയില്ലെ ? അങ്ങനെ അവരുടെ അടുത്തേക്ക് നബി (സ) വന്നപ്പോൾ മുലൈഖ, നബിയിൽ (സ) നിന്ന് ശരണം തേടി. അപ്പോൾ നബി (സ) അവരെ വിവാഹ മോചനം നൽകി സ്വാതന്ത്രയാക്കി. (ത്വബകാതു ഇബ്നു സഅ്ദ്: 8 /148, താരീഖു ദ്ദഹബി: 1 /335)

ഈ നിവേദനത്തിൽ എവിടെയാണ് – നാസ്‌തികർ വാദിക്കുന്നതു പോലെ – നബി മുലൈഖയെ ഭോഗിച്ചു എന്നുള്ളത് ?! ഭോഗിക്കുക പോയിട്ട് സ്വന്തം ഭാര്യയായിരുന്നിട്ടും, തന്നെ ഇഷ്ടമില്ലാത്തതു കൊണ്ട് നബി (സ) മുലൈഖയെ സ്പർശിക്കുക പോലുമുണ്ടായില്ല. മുലൈഖയെ പിടിച്ചു വെക്കുകയൊ ബലം പ്രയോഗിക്കുകയൊ ചെയ്തില്ല. സൗമ്യതയോടെ വീട്ടിലേക്ക് പൊയ്കൊള്ളാൻ പറഞ്ഞു! ഇതാണൊ ഒരു സ്ത്രീപീഢകന്റെ ചിത്രം ?!

തനിക്ക് ഭർത്താവിനെ ഇഷ്ടമില്ലെന്ന് പറയുന്ന സ്ത്രീക്ക് യാതൊരു സങ്കോചമോ വൈമനസ്യമോ കൂടാതെ വീട്ടിലേക്ക് പോകാനും വിവാഹ മോചനം ചെയ്യാനുമുള്ള അവകാശം നൽകുകയാണ് നബി (സ) ഇവിടെ ചെയ്യുന്നതായി നിവേദനത്തിൽ പ്രസ്‌താവിക്കുന്നത്. ഇത് സ്ത്രീ പീഢകരുടെ സ്വഭാവമല്ല, സ്ത്രീവിമോചകന്റേതാണ്.

“ബാലിശമായ കാരണങ്ങൾ പറഞ്ഞ് ഡിവോഴ്സ് ചെയ്തു” എന്ന ആരോപണവും നുണ തന്നെയെന്ന് ഈ നിവേദനവും വ്യക്തമായി തെളിയിക്കുന്നു. മുലൈഖയാണ് നബിയിൽ (സ) താൽപര്യമില്ലെന്ന അഭിപ്രായം അവതരിപ്പിച്ചത്, നബിയല്ല (സ). തന്നെ ഇഷ്ടമല്ലാത്ത ഒരാളെ മോചിപ്പിക്കുക മാത്രമാണ് നബി (സ) ചെയ്തത്.

2. ത്വബ്‌രിയും ഇബ്നു സഅ്ദും എല്ലാം ഈ കഥ ഉദ്ധരിച്ചിരിക്കുന്നത് ചരിത്രകാരനായ വാക്വിദിയിൽ നിന്നാണ്. നിവേദനം ഉദ്ധരിച്ച തൊട്ടുടനെ തന്നെ കഥയുടെ അസത്യത വാക്വിദിയും, അദ്ദേഹത്തിൽ നിന്ന് ഇമാം ത്വബ്‌രിയും ഇബ്നു സഅ്ദും എല്ലാവരും വ്യക്തമാക്കിയിട്ടുമുണ്ട്. അതു പക്ഷെ നാസ്‌തിക പകർത്തെഴുത്തുകാർ കണ്ടിട്ടില്ല. കാരണം, ഈച്ച കോപ്പിയടിക്കാൻ ഉപയോഗിച്ച മിഷണറി ദുർവ്യാഖ്യാന ലേഖനങ്ങളിൽ അതു കാണില്ല; അതവർ ഉദ്ധരിക്കില്ല എന്നതിൽ അത്ഭുതപ്പെടാനൊന്നുമില്ല. നിവേദനത്തിന്റെ തൊട്ടുടനെ നിവേദകൻ തന്നെ പ്രസ്‌താവിക്കുന്നത് ഇപ്രകാരമാണ്.

“വാക്വിദി പറഞ്ഞു: നമ്മുടെ സഹചരിത്രകാരന്മാർ ഇതിനെ നിശിതമായി നിഷേധിക്കുന്നു. നബി (സ) ഒരിക്കലും ഒരു കിനാനക്കാരിയെയും വിവാഹം ചെയ്തിട്ടില്ല എന്നതാണ് വസ്തുത.” (മുൻതഖബു മിൻ ദൈലിൽ മിദ്‌യൽ 89, താരീഖുത്വബ്‌രി: 11:596, അൽ ഇസ്വാബ: ഇബ്നു ഹജർ: 8: 320)

3. مما يضعف هذا الحديث، ذِكر عائشة أنها قالت: ألا تستحين، وعائشةُ لم تكن مع النبي صلى الله عليه وسلم عام الفتح.

നിവേദനം ദുർബലമാണെന്നതിനുള്ള മറ്റൊരു തെളിവാണ്, അവരോട് അസൂയ തോന്നിയ ആഇശ (റ) അവരോട് ഇപ്രകാരം ചോദിച്ചു എന്ന ഭാഗം: “നിങ്ങളുടെ പിതാവിന്റെ ഘാതകനെ വിവാഹം ചെയ്യാൻ നിങ്ങൾക്ക് മടിയില്ലെ ?”. മക്കാ വിജയ ദിവസം ആഇശ (റ) നബിയോടൊപ്പം (സ) ഉണ്ടായിരുന്നില്ല. പിന്നെ എങ്ങനെയാണ് മുലൈഖ ബിൻത് കഅ്ബിനോട് ആഇശ (റ) ഇപ്രകാരം ചോദിക്കുക ?! 500 കിലോമീറ്ററുകൾക്കപ്പുറം മദീനയിലുള്ള ആഇശ (റ), നവ വധുവായ മുലൈഖയുമായി എങ്ങനെ സംഭാഷണം നടത്തും ?! ഇതിന്റെ അടിസ്ഥാനത്തിൽ തന്നെ നിവേദനം ദുർബലമാണെന്ന് പണ്ഡിതന്മാർ വ്യക്തമാക്കിയിട്ടുണ്ട്.

4. നബിയുടെ (സ) പത്നിമാരിൽ, നബിയിൽ നിന്നും ശരണം തേടിയത് ഒരൊറ്റ പത്നി മാത്രമാണ്. അത് ഉമൈമ ബിൻത് നുഅ്മാനാണ് (ബുഖാരി: 5254).

ഉമൈമയുടെ ചരിത്രം വിശദമായി മറ്റൊരു ലേഖനത്തിൽ പ്രതിപാദിച്ചിട്ടുള്ളതിനാൽ ഇവിടെ ആവർത്തിക്കുന്നില്ല:

ഉമൈമയുടെ ഈ സംഭവം മുലൈഖ ബിൻത് കഅ്ബിലേക്കും മറ്റു പല സ്ത്രീകളിലേക്കും ചേർത്തി കൊണ്ട് ചില നിവേദനങ്ങൾ രേഖപ്പെടുത്തപ്പെട്ടിട്ടുണ്ട്. ഇത് നിവേദകന്മാരിൽ നിന്നും സംഭവിച്ച ഓർമ്മ പിശകു മാത്രമാണ്. നാമങ്ങൾ പരസ്പരം കൂടി കലർന്നു കൊണ്ടുള്ള ചില ആശയക്കുഴപ്പങ്ങൾ ചില ചരിത്രകാരന്മാരിൽ നിന്ന് സംഭവിച്ചു എന്നതിനുള്ള വ്യക്തമായ തെളിവാണ് മുലൈഖ ബിൻത് കഅ്ബിന്റെയും, ഉമൈമ ബിൻത് നുഅ്മാന്റെയും കഥകളുടെ ഉള്ളടക്കത്തിലെ സാമ്യത.

5. നിവേദനത്തിന്റെ പരമ്പരകൾ ദുർബലമാണ് എന്നതും ഈ കഥയുടെ അസത്യതയെ തെളിയിക്കുന്നതാണ്.

ഒന്നാമത്തെ പരമ്പര: ذكر ابن عمر أن عبد العزى بن الجندعي حدثه عن أبيه عن عطاء بن يزيد الجندعي قال

രണ്ടാമത്തെ പരമ്പര: قال ابن عمر وحدثني محمد بن عبد الله عن الزهري مثل ذلك

മൂന്നാമത്തെ പരമ്പര: أخبرنا محمد بن عمر حدثني أبو معشر قال

മൂന്നു പരമ്പരകളിലേയും റാവിയായ മുഹമ്മദിബ്നു ഉമർ വാക്കിദുൽ അസ്‌ലമി (സത്യസന്ധതയിൽ) ദുർബലനാണ്. അഹ്മദിബ്നു ഹമ്പൽ പറഞ്ഞു: മുഹമ്മദിബ്നു ഉമർ വാക്കിദുൽ അസ്‌ലമി നുണയനാണ്; അയാൾ ഹദീസുകളിൽ കോട്ടിമാട്ടുമായിരുന്നു.

യഹ്‌യ പറഞ്ഞു: അയാൾ വിശ്വസ്ഥനല്ല. അയാളുടെ ഹദീസുകൾ എഴുതിവെക്കാൻ കൊള്ളാത്തത്രയും അവിശ്വസനീയമാണ്. ഇമാം ബുഖാരി, റാസി, നസാഈ എന്നിവർ പറഞ്ഞു: അയാൾ കളവു കൊണ്ട് ആരോപിതനാണ്. റാസി, നസാഈ എന്നിവർ പറഞ്ഞു: അയാൾ വ്യാജ ഹദീസുകൾ ഉണ്ടാക്കുന്ന വ്യക്തിയായിരുന്നു. ഇമാം ദാറക്കുത്നി പറഞ്ഞു: അയാളിൽ ദൗർബല്യമുണ്ട്. ഇസ്ഹാകിബ്നു റാഹൂയ പറഞ്ഞു: അയാൾ നുണയനാണ്. (അദ്ദുഅഫാഉ വൽ മത്റൂകീൻ : ഇബ്നുൽ ജൗസി: 3 / 87, അദ്ദുഅഫാഉ സ്സ്വഗീർ: ബുഖാരി: 334, അൽ ജർഹുവതഅദീൽ: അബൂഹാതിം: 8/21, അൽ കാമിൽ ഇബ്നു അദിയ്യ്: 7/ 481)

പുറമെ മൂന്നു സനദുകൾക്കും നബിയിലേക്കെത്തുന്ന നിവേദകരുടെ കണ്ണികളില്ല, പരമ്പര മുറിഞ്ഞതാണ്. രണ്ടു സനദുകൾക്കും മറ്റു പല ന്യൂനതകളുമുണ്ട്. بو معشر: نجيح ابن عبد الرحمن السِّندي،. ضعَّفه أيوب بن عبد الرحمن صعصعة،: لم يوثقه غير ابن حبان മൂന്നാമത്തെ സനദിലെ അബൂ മഅ്ഷർ: നജീഹിബ്നു അബ്ദുർറഹ്‌മാൻ അസ്സിന്ദി ദുർബലനാണ്; അയ്യൂബിബ്നു അബ്ദുർറഹ്മാൻ സ്വഅ്സ്വഅ അദ്ദേഹത്തെ ദുർബലനാണെന്ന് വ്യക്തമാക്കിയിട്ടുണ്ട്, ഇബ്നു ഹിബ്ബാൻ അല്ലാത്ത ആരും അബൂ മഅ്ഷർ വിശ്വസ്‌തനാണെന്ന് അഭിപ്രായപ്പെട്ടിട്ടില്ല.

ചുരുക്കത്തിൽ മൂന്ന് പരമ്പരകളും ദുർബലമാണ്.

വിമർശനം:

” …വിവാഹം ചെയ്ത് ഭോഗിച്ചു… അതിൽ 4 എണ്ണത്തിനെ ബാലിശമായ കാരണങ്ങൾ പറഞ്ഞ് ഡിവോഴ്സ് ചെയ്തു…. അധികം പേർക്കും അറിയാത്ത (മുഹമ്മദ് നബി) മൊഴിചൊല്ലിയ സ്ത്രീകളുടെ പേരുകൾ ഇവയാണ്…

1) അസ്മ ബിൻത് അൽ നുമാൻ… ആയിഷ കുതത്രം പ്രയോഗിച്ച് ഡിവോഴ്സ് ചെയ്യിച്ചു.. (ഇബ്നു സാദ് 8: 101 -105, 153)”

(മുഹമ്മദിന്റെ പെണ്ണുങ്ങൾ: നാസ്‌തിക സോഷ്യൽ മീഡിയ തെറിമാല)

മറുപടി:

നാസ്‌തിക സോഷ്യൽ മീഡിയ തെറിമാലകളിൽ ഒന്നായ “മുഹമ്മദിന്റെ പെണ്ണുങ്ങൾ” എന്ന കുറിപ്പിൽ നിന്നുള്ള ചില വരികളാണ് ഈ വായിച്ചത്. കല്ലുവച്ച നുണകളും, അർദ്ധ സത്യങ്ങളും, ദുർവ്യാഖ്യാനങ്ങളും, വൈരുദ്ധ്യങ്ങളും നിറഞ്ഞ ഒരു മിഷനറി ലേഖനം ഫോട്ടോസ്റ്റാറ്റ് എടുത്ത് അരികുകൾ ചെത്തി കളഞ്ഞുണ്ടാക്കിയതാണ് ഈ നാസ്‌തിക അക്ഷര ഗരളം.

മറുപടിയിലേക്ക് വരാം…

1. അസ്‌മാഅ് ബിൻത് അന്നുഅ്മാൻ എന്ന സ്ത്രീയെ മുഹമ്മദ് നബി (സ) വിവാഹം ചെയ്യുകയും, “ഭോഗി”ക്കുകയും ബാലിശമായ കാരണങ്ങൾ പറഞ്ഞ് ഡിവോഴ്സ് ചെയ്യുകയുമുണ്ടായി എന്നാണ് ആരോപണം. ആരോപണത്തിനായി വിമർശകർ അവലംബിച്ച നിവേദനത്തിന്റെ ഉള്ളടക്കം ഇതാണ്:

അസ്‌മാഅ് ബിൻത് അന്നുഅ്മാനെ നബി (സ) വിവാഹം ചെയ്തു. വിവാഹ ശേഷം നബി (സ) അവരുടെ അടുത്തു ചെന്നു. അപ്പോൾ അവർ പറഞ്ഞു: “നിങ്ങളിൽ നിന്നും ഞാൻ അല്ലാഹുവിൽ ശരണം തേടുന്നു”. അപ്പോൾ നബി (സ) തന്റെ വസ്ത്രം കൊണ്ട് മുഖം മറച്ചു കൊണ്ട് പറഞ്ഞു: നീ ശരണം തേടിയിരിക്കുന്നത് ഏറ്റവും നല്ല രക്ഷകനിലാണ്… നീ നിന്റെ വീട്ടിലേക്ക് പൊയ്‌ക്കൊള്ളു…”

സംഭവം വായിച്ചാൽ പ്രവാചകനോട് വെറുപ്പല്ല ആദരവാണ് ഉണ്ടാവുക എന്ന് മിഷണറിമാർക്കറിയാം. അതുകൊണ്ട് സംഭവം മുഴുവൻ കൊടുക്കാതെ “വിവാഹം ചെയ്ത് ഭോഗിച്ചു… അതിൽ 4 എണ്ണത്തിനെ ബാലിശമായ കാരണങ്ങൾ പറഞ്ഞ് ഡിവോഴ്സ് ചെയ്തു…. അസ്മ ബിൻത് അൽ നുമാൻ… ആയിഷ കുതത്രം പ്രയോഗിച്ച് ഡിവോഴ്സ് ചെയ്യിച്ചു..” എന്നങ്ങു പറഞ്ഞു നിർത്തി !!

എവിടെ നിന്ന് ഭോഗിച്ച വിവരം നാസ്‌തികർക്ക് കിട്ടി ?! ഭോഗിയ്ക്കുക പോയിട്ട് സ്വന്തം ഭാര്യയായിരുന്നിട്ടും ഇഷ്ടമില്ലാത്തതു കൊണ്ട് സ്പർശിക്കുക പോലുമുണ്ടായില്ല. തന്നെ ഇഷ്ടമില്ലാത്തതിനാൽ തന്നെ മുഖം മറച്ച് ദൃഷ്ടി പോലും അവരിൽ നിന്നും മറച്ചുവെച്ചു. പിടിച്ചു വെക്കുകയൊ ബലം പ്രയോഗിക്കുകയൊ ചെയ്തില്ല. സൗമ്യതയോടെ വീട്ടിലേക്ക് പൊയ്കൊള്ളാൻ പറഞ്ഞു! ഇതാണൊ ഒരു സ്ത്രീപീഢകന്റെ ചിത്രം ?!

തനിക്ക് ഭർത്താവിനെ ഇഷ്ടമില്ലെന്ന് പറയുന്ന സ്ത്രീക്ക് യാതൊരു സങ്കോചമോ വൈമനസ്യമോ കൂടാതെ വീട്ടിലേക്ക് പോകാനും വിവാഹ മോചനം ചെയ്യാനുമുള്ള അവകാശം നൽകിയ ഒരാളെ “ബലാൽസംഗ വീരനും” ” സ്ത്രീ പീഢകനും” ആയി ചിത്രീകരിക്കാൻ വർഗീയ വിഷ വാഹകർക്കെ സാധിക്കു. അല്ലാത്തവർക്ക് അദ്ദേഹത്തെ സ്ത്രീവിമോചകനായാണ് മനസ്സിലാവുക. തന്നെ സ്നേഹത്തോടെ തിരഞ്ഞെടുത്തവരെ മാത്രമെ നബി (സ) ഭാര്യയാക്കിയിട്ടുള്ളു, സ്നേഹിച്ചിട്ടുള്ളു, സ്പർശിച്ചിട്ടുള്ളു എന്നത് അദ്ദേഹത്തിന്റെ മാന്യ വ്യക്തിത്വത്തിനും ലൈംഗിക വിശുദ്ധിക്കും മികച്ച തെളിവാണ്.

“ബാലിശമായ കാരണങ്ങൾ പറഞ്ഞ് ഡിവോഴ്സ് ചെയ്തു” എന്ന ആരോപണവും നുണ തന്നെ. അസ്‌മാഅ് ആണ് നബിയിൽ (സ) താൽപര്യമില്ലെന്ന അഭിപ്രായം അവതരിപ്പിച്ചത്, നബിയല്ല (സ). അവരുടെ അഭിപ്രായത്തെ മാനിച്ചു എന്നത് ഒരു തെറ്റായി മനസ്സിലാക്കാൻ മാത്രം ബുദ്ധിക്ക് വിധ്വേഷജ്വരം ബാധിച്ചോ നാസ്‌തികരെ നിങ്ങൾക്ക് ?!

2. അസ്‌മാഅ് ബിൻത് അന്നുഅ്മാൻ എന്ന സ്ത്രീയെ മുഹമ്മദ് നബി (സ) വിവാഹ മോചനം ചെയ്തു എന്ന് പോയിട്ട് വിവാഹം ചെയ്തു എന്നു പോലും സ്ഥിരപ്പെട്ടിട്ടില്ല എന്നതാണ് യാഥാർഥ്യം. ഈ പ്രസ്‌താവനക്ക് ഉപോൽബലകമായി രണ്ട് കാരണങ്ങൾ ഇവിടെ ഉദ്ധരിക്കാം.

ഒന്നാമത്തെ കാരണം:

നബിയുടെ (സ) പത്നിമാരിൽ, നബിയിൽ നിന്നും ശരണം തേടിയത് ഒരൊറ്റ പത്നി മാത്രമാണ്. അത് ഉമൈമ ബിൻത് നുഅ്മാനാണ് (ബുഖാരി: 5254). നബിയെ (സ) സംബന്ധിച്ചും അദ്ദേഹത്തിന്റെ മഹനീയമായ വ്യക്തിത്വത്തെ സംബന്ധിച്ചും പൂർവ്വ അറിവില്ലാത്ത ഉമൈമ പ്രവാചകനിൽ അനിഷ്ടം പ്രകടിപ്പിക്കുകയുണ്ടായി. താന്‍ വിവാഹം ചെയ്ത സ്ത്രീക്ക് തന്നോടൊപ്പം ജീവിക്കുവാന്‍ താല്‍പര്യമില്ലെന്നറിഞ്ഞപ്പോള്‍, നിര്‍ബന്ധിച്ച് കൂടെ താമസിപ്പിക്കാതെ മാന്യമായി അവരെ സ്വഗൃഹത്തിലേക്ക് യാത്രയാക്കുകയും വേര്‍പിരിയും മുമ്പ് അവര്‍ക്ക് സമ്മാനങ്ങള്‍ നല്‍കുകയും ചെയ്ത മാതൃകാപരമായ ഒരു നപടിയാണ് നബിയിൽ നിന്നുണ്ടായത്.

ﻻَ ﺗَﺤْﻤِﻠُﻮا اﻟﻨِّﺴَﺎءَ ﻋَﻠَﻰ ﻣَﺎ ﻳَﻜْﺮَﻫْﻦَ

“സ്ത്രീകളെ അവർക്ക് വെറുക്കുന്നത് ചെയ്യാൻ നിങ്ങൾ നിർബന്ധിക്കരുത്.” (മുസ്വന്നഫ് അബ്ദുർ റസാഖ്: 10320) എന്ന് അനുചരന്മാരെ പഠിപ്പിക്കുക മാത്രമല്ല കാരുണ്യ മൂർത്തിയായ പ്രവാചകൻ (സ) ചെയ്തത്, പ്രത്യുത ഉമൈമയോട് അനുവർത്തിച്ച നിലപാടിലൂടെ തന്റെ ആദർശനിഷ്ഠത സ്വജീവിതത്തിൽ പ്രാവർത്തികമായി തെളിയിക്കുക കൂടി അദ്ദേഹം ചെയ്തു.

പ്രവാചകന്‍ (സ) ഉദ്ദേശിച്ചിരുന്നെങ്കില്‍ അവരെ നിര്‍ബന്ധപൂര്‍വ്വം കൂടെ താമസിപ്പിക്കുവാന്‍ യാതൊരു തടസ്സവുമുണ്ടായിരുന്നില്ല. കാരണം അവിടുന്ന് ഇസ്‌ലാമിക സാമ്രാജ്യത്തിന്റെ അധിപനായിരുന്നു. രാജാക്കന്മാരും ചക്രവര്‍ത്തിമാരും ഒരു പെണ്ണിനെ ആഗ്രഹിച്ചു കഴിഞ്ഞാല്‍ അവളുടെ താല്‍പര്യം അന്വേഷിക്കുന്ന പതിവില്ലെന്ന് എല്ലാവര്‍ക്കുമറിയാവുന്ന വസ്തുതയാണ്. അതിനെതിരെ ഒരു ശബ്ദവുമവിടെ ഉയരുകയില്ല. ഇവിടെ പ്രവാചകന്‍ (സ) മാതൃകയാവുകയാണ്. താന്‍ വിവാഹം ചെയ്ത ഒരു സ്ത്രീക്ക് തന്നോടൊപ്പം ജീവിക്കുവാന്‍ താല്‍പര്യമില്ലെന്നറിഞ്ഞ നിമിഷം അവളെ ആശ്വസിപ്പിക്കുകയും സമാധാനിപ്പിക്കുകയും നിര്‍ഭയത്വത്തോടെ സ്വഗൃഹത്തിലേക്ക് മടങ്ങാന്‍ അവസരമൊരുക്കുകയും ചെയ്യുന്ന പ്രവാചകന്‍, ഒരു രാഷ്ട്രത്തിന്റെ ചോദ്യം ചെയ്യപ്പെടാത്ത ഭരണാധിപനാണെന്ന വസ്തുതയും ചേര്‍ത്തു മനസ്സിലാക്കുമ്പോള്‍ എത്രമാത്രം ആദരവും താല്‍പര്യവുമാണ് ആ വ്യക്തിത്വത്തിനോട് തോന്നേണ്ടത്. ഇസ്‌ലാം വിമര്‍ശകര്‍ക്ക് പക്ഷെ അത്തരം ഊഷ്മളമായ ചിന്തയും വികാരവുമൊന്നും ഉണ്ടാവുകയില്ല. കാരണം അവരുടെ ഹൃദയം കടുത്തു പോയിരിക്കുന്നു. ഊഷരമായ ചിന്തയും വികാരവുമാണ് അവരെ നയിക്കുന്നത്. വെറുപ്പും വിദ്വേഷവും മാത്രമാണ് അവരെ ഭരിക്കുന്നത്.

ഉമൈമയുടെ ചരിത്രം വിശദമായി മറ്റൊരു ലേഖനത്തിൽ പ്രതിപാദിച്ചിട്ടുള്ളതിനാൽ ഇവിടെ ആവർത്തിക്കുന്നില്ല:

ഉമൈമയുടെ ഈ സംഭവം അസ്‌മാഅ് ബിൻത് നുഅ്മാനടക്കം മറ്റു പല സ്ത്രീകളിലേക്കും ചേർത്തി കൊണ്ട് ചില നിവേദനങ്ങൾ രേഖപ്പെടുത്തപ്പെട്ടിട്ടുണ്ട്. ഇത് നിവേദകന്മാരിൽ നിന്നും സംഭവിച്ച ഓർമ്മ പിശകു മാത്രമാണ്. ഉമൈമ ബിൻത് നുഅ്മാൻ, അസ്‌മാഅ് ബിൻത് നുഅ്മാൻ എന്നീ പേരുകളിലെ സാമ്യതയാണ് ഇവിടെ അതിനു കാരണം. നാമങ്ങൾ പരസ്പരം കൂടി കലർന്നു കൊണ്ടുള്ള ചില ആശയക്കുഴപ്പങ്ങൾ ചില ചരിത്രകാരന്മാരിൽ നിന്ന് സംഭവിച്ചു എന്നതിനുള്ള വ്യക്തമായ തെളിവാണ് ഉമൈമ ബിൻത് നുഅ്മാന്റെയും, അസ്‌മാഅ് ബിൻത് നുഅ്മാന്റെയും കഥകളുടെ ഉള്ളടക്കത്തിലെ സാമ്യത.

രണ്ടാമത്തെ കാരണം:

അസ്‌മാഅ് ബിൻത് നുഅ്മാനെ നബി (സ) വിവാഹം ചെയ്തു, നബി (സ) അവരെ സന്ദർശിച്ചപ്പോൾ അവർ വെറുപ്പു പ്രകടിപ്പിച്ചു, (“നിങ്ങളിൽ നിന്നും ഞാൻ അല്ലാഹുവിൽ ശരണം തേടുന്നു” എന്ന് നബിയോട് പറഞ്ഞു കൊള്ളാൻ അസ്‌മായോട് ഉപദേശിച്ചു കൊണ്ട്) ആഇശ ബീവി തന്ത്രം പ്രയോഗിച്ചു എന്നൊക്കെ സൂചിപ്പിക്കുന്ന നിവേദനങ്ങളുടെ പരമ്പരകൾ ആസകലം ദുർബലമാണ് എന്നതാണ് മറ്റൊരു കാരണം.

പ്രസ്തുത നിവേദനങ്ങളുടെ പരമ്പരകളും അവയെ സംബന്ധിച്ച നിരൂപണങ്ങളും താഴെ ചേർക്കാം:

ഒന്നാമത്തെ പരമ്പര:

ഇബ്നു സഅ്ദ് (ത്വബകാത് 8/143-148) നിവേദനം ഉദ്ധരിച്ചിരിക്കുന്നത് മുഹമ്മദ് ഇബ്നു ഉമർ വാകിദിയിൽ നിന്നാണ് അദ്ദേഹം ഹദീസിന്റെ മേഖലയിൽ (ദഈഫ്) ദുർബലമാണ്.

രണ്ടാമത്തെ പരമ്പര:

أخبرنا محمد بن عمر حدثني عبد الله بن جعفر عن عمرو بن صالح عن سعيد بن عبد الرحمن بن أبزى قال

ഇബ്നു സഅ്ദ് (ത്വബകാത്: 8/144), സഈദിബ്നു അബ്ദുർറഹ്മാൻ ബിൻ അബ്സയിൽ നിന്ന് ഉദ്ധരിച്ച പരമ്പര പൂർണമല്ല, മുർസലാണ്. പരമ്പരയിലെ മുഹമ്മദിബ്നു ഉമർ വാകിദ് ഹദീസിന്റെ വിഷയത്തിൽ ദുർബലനാണ്. നിവേദനത്തിലെ അംറിബ്നു സ്വാലിഹ് എന്ന റാവി മജ്ഹൂൽ (അജ്ഞാതൻ) ആണ്. ഇബ്നു അദിയ്യ് തന്റെ ‘അൽ കാമിൽ ഫീ ദുഅഫാഇർ രിജാലിൽ’, അംറിബ്നു സ്വാലിഹ് എന്ന റാവിയെ ദുർബലനായി പരിഗണിച്ചിരിക്കുന്നു.

മൂന്നാമത്തെ പരമ്പര:

أخبرنا هشام بن محمد بن السائب ، عن أبيه ، عن أبي صالح ، عن بن عباس قال…

ഇബ്നു സഅ്ദ് (ത്വബകാത്: 8/145) ഉദ്ധരിച്ച ഈ പരമ്പരയും അങ്ങേയറ്റം ദുർബലമാണ്. നിവേദനത്തിലെ ഹിശാമിബ്നു മുഹമ്മദിബ്നുസ്സാഇബിനെ സംബന്ധിച്ച ഹദീസ് നിദാനശാസ്ത്ര പണ്ഡിതരുടെ അഭിപ്രായം ഇപ്രകാരമാണ്. ഇബ്നു ഇറാക്: കളവു പറയുന്നവനായി ആരോപിതൻ ഇബ്നു അസാകിർ പറഞ്ഞു: അദ്ദേഹം റാഫിളിയാണ്, വിശ്വസ്തനല്ല. ദാറകുത്നി പറഞ്ഞു: മത്റൂക് (നുണയനെന്ന് ആരോപിക്കപ്പെട്ടതിനാൽ ഹദീസ് സ്വീകരിക്കപ്പെടാത്ത വ്യക്തി) ദഹബി പറഞ്ഞു: അദ്ദേഹം വിശ്വസ്‌തനല്ല. ( http://hadith.islam-db.com/narrators/33196/ )

നിവേദനത്തിലെ മുഹമ്മദിബ്നുസ്സാഇബിനെ സംബന്ധിച്ചു ഹദീസ് നിദാനശാസ്ത്ര പണ്ഡിതരുടെ അഭിപ്രായം:

കള്ള ഹദീസുകൾ നിർമ്മിക്കുന്ന വ്യക്തിയാണയാൾ എന്നാണ് ബുർഹാനുദ്ദീൻ അൽ ഹലബിയുടെ അഭിപ്രായം (അൽ കശ്ഫുൽ ഹസീസ് അമ്മൻ റുമിയ ബി വദ്ഇൽ ഹദീസ്)

ഇബ്നുൽ ജവ്സി തന്റെ ‘മൗദൂആത്ത്’ ൽ, പ്രസ്തുത റാവി കള്ള ഹദീസ് നിർമ്മാതാക്കളുടെ നേതാവാണെന്ന് പ്രസ്ഥാവിക്കുന്നു. ‘അലി മരണപ്പെട്ടിട്ടില്ലെന്നും, മേഘങ്ങൾക്കു മുകളിൽ വസിക്കുകയാണ്…’ എന്നെല്ലാം വിശ്വസിക്കുന്ന തീവ്ര ശീഈ വിഭാഗത്തിൽ പെട്ട വ്യക്‌തിയാണെന്ന് ഇബ്നു ഹിബ്ബാൻ, റാവിയെ പറ്റി അഭിപ്രായപ്പെടുന്നു.

ദാറകുത്നി തന്റെ ‘അദ്ദുഅഫാഉ വൽ മത്റൂ കീൻ’ ലും, അബു നുഐം അൽ അസ്ബഹാനി തന്റെ ‘ദുഅഫാഅ്’ ലും, റാവി കള്ള ഹദീസുകൾ പടച്ചുണ്ടാക്കുന്ന വ്യക്തിയായി തന്നെ വിശദീകരിക്കുന്നുണ്ട്.

മറ്റൊരു സനദ് ഇപ്രകാരമാണ്: وروى أيضا قال : أخبرنا هشام بن محمد ، حدثني ابن الغسيل ، عن حمزة بن أبي أسيد الساعدي ، عن أبيه – وكان بدريا – قال :

പരമ്പരയിലെ ഹിശാമിബ്നു മുഹമ്മദ് കളവു പറയുന്ന വ്യക്തിയാണെന്ന വിശദീകരണം മുമ്പ് പ്രസ്താവിക്കപ്പെട്ടിട്ടുണ്ട്. നിവേദനത്തിലെ ഇബ്നുൽ ഗസീൽ വിശ്വസ്തനാണെങ്കിലും ഓർമ്മശക്തി കുറഞ്ഞ വ്യക്തിയാണെന്ന് ഇബ്നു ഹജറും വ്യക്തമാക്കിയിട്ടുണ്ട്.

വിമർശനം:

വിവസ്ത്രരായ കുറേ ആളുകൾ മുഹമ്മദ് നബിയുമായി ലൈംഗിക ബന്ധത്തിൽ ഏർപ്പെട്ടു, തന്നിമിത്തം അദ്ദേഹം തളർന്നുവെന്നുമെല്ലാം ഹദീസിൽ ഉണ്ട്. ഇത് സ്വവർഗ്ഗ സംഘരതിയിൽ നബി ഏർപ്പെട്ടു എന്നതിന് തെളിവല്ലെ ?

മറുപടി:

ആരോപിക്കുന്നതെന്ത് ?! ആരോപണത്തിനു പിൻബലമായി ഉദ്ധരിക്കുന്ന ഹദീസിന്റെ വിഷയമെന്ത്?!! പുലബന്ധം പോലുമില്ലാത്ത എന്തോ സംഭവത്തിലെ, ഏതോ വാചകം ലജ്ജയുടെ ലാഞ്ചന പോലും ഇല്ലാത്ത വിധം ദുർവ്യാഖ്യാനിക്കുക… എന്നിട്ട് മുഹമ്മദ് നബിക്കെതിരെ(സ) ഒരു അശ്ലീല കഥ പടച്ചുവിടുക. ഈ മിഷണറി ജീർണ്ണതകൾ പുതിയ ചായം പൂശി അവതരിപ്പിക്കുന്ന നാസ്തികരുടെ ഗതികേട് നോക്കണേ.

ലൈംഗികതയുമായി ബന്ധപ്പെട്ട ഒരു പരാമർശം പോലുമില്ലാത്ത ഒരു ഹദീസിനെ ഓർജിയാക്കി മാറ്റണമെങ്കിൽ എത്രമാത്രം രോഗാതുരമായിരിക്കണം ഇവരുടെയൊക്കെ മനസ്സും ജീവിതവും !? ധൈഷണികമായി എത്രമാത്രം ശൂന്യവും ഹീനവുമായിരിക്കണം ഇവരുടെയൊക്കെ ചിന്താമണ്ഡലം !!?

വിമർശകർ ഉദ്ധരിക്കുന്ന ഹദീസ് ഇപ്രകാരം നമുക്ക് വായിക്കാം:

اسْتَتبْعَنِي رَسُولُ اللهِ صَلَّى اللهُ عَلَيْهِ وَسَلَّمَ، قَالَ: فَانْطَلَقْنَا، حَتَّى أَتَيْتُ مَكَانَ كَذَا وَكَذَا فَخَطَّ لِي خِطَّةً، فَقَالَ لِي: ” كُنْ بَيْنَ ظَهْرَيْ هَذِهِ لَا تَخْرُجْ مِنْهَا، فَإِنَّكَ إِنْ خَرَجْتَ هَلَكْتَ ” . قَالَ: فَكُنْتُ فِيهَا، قَالَ: فَمَضَى رَسُولُ اللهِ صَلَّى اللهُ عَلَيْهِ وَسَلَّمَ، حَذَفَةً ، أَوْ أَبْعَدَ شَيْئًا، أَوْ كَمَا قَالَ: ثُمَّ إِنَّهُ ذَكَرَ هَنِينًا كَأَنَّهُمْ الزُّطُّ (قَالَ عَفَّانُ: أَوْ كَمَا قَالَ عَفَّانُ: إِنْ شَاءَ اللهُ) : لَيْسَ عَلَيْهِمْ ثِيَابٌ، وَلَا أَرَى سَوْآتِهِمْ، طِوَالًا، قَلِيلٌ لَحْمُهُمْ ، قَالَ: فَأَتَوْا، فَجَعَلُوا يَرْكَبُونَ رَسُولَ اللهِ صَلَّى اللهُ عَلَيْهِ وَسَلَّمَ . قَالَ: وَجَعَلَ نَبِيُّ اللهِ صَلَّى اللهُ عَلَيْهِ وَسَلَّمَ يَقْرَأُ عَلَيْهِمْ . قَالَ: وَجَعَلُوا يَأْتُونِي فَيُخَيِّلُونَ، أَوْ يَمِيلُونَ حَوْلِي، وَيَعْتَرِضُونَ لِي . قَالَ عَبْدُ اللهِ: فَأُرْعِبْتُ مِنْهُمْ رُعْبًا شَدِيدًا . قَالَ: فَجَلَسْتُ، أَوْ كَمَا قَالَ . قَالَ: فَلَمَّا انْشَقَّ عَمُودُ الصُّبْحِ جَعَلُوا يَذْهَبُونَ، أَوْ كَمَا قَالَ . قَالَ: ثُمَّ إِنَّ رَسُولَ اللهِ صَلَّى اللهُ عَلَيْهِ وَسَلَّمَ جَاءَ ثَقِيلًا وَجِعًا، أَوْ يَكَادُ أَنْ يَكُونَ وَجِعًا، مِمَّا رَكِبُوهُ .

അബ്ദുല്ലാഹിബ്നു മസ്ഊദ് (റ) പറഞ്ഞു: അല്ലാഹുവിന്റെ ദൂതൻ എന്നെ ഒരു യാത്രക്കായി കൂട്ട് വിളിച്ചു. അങ്ങനെ ഞങ്ങൾ പുറപ്പെട്ടു. ഇന്ന സ്ഥലമെത്തിയപ്പോൾ അദ്ദേഹം എനിക്ക് നിലത്ത് ഒരു വരവരച്ചു തന്നു. എന്നിട്ട് എന്നോട് പറഞ്ഞു: നീ എന്റെ പിറകിൽ ഈ വരക്കപ്പുറം നിൽക്കണം, അതിനപ്പുറം വരരുത്. ഈ വരവിട്ട് നീ മുന്നോട്ടു വന്നാൽ നീ മരിച്ചേക്കും. അങ്ങനെ ഞാൻ അവിടെ തന്നെ നിന്നു.

എന്നിട്ട് അല്ലാഹുവിന്റെ ദൂതൻ (സ) കുറച്ച് മുന്നോട്ട് നിന്നു. കുറച്ച് കഴിഞ്ഞപ്പോൾ ഒരു സംഘം പ്രാകൃതരായ ആളുകൾ വന്നു. അവരുടെ മേൽ വസ്ത്രമുണ്ടായിരുന്നില്ല, എന്നാൽ അവരുടെ നഗ്നത എനിക്ക് കാണാൻ കഴിഞ്ഞില്ല. അവർ വളരെ നീളമുള്ളവരും മാംസം കുറഞ്ഞവരുമായിരുന്നു. അങ്ങനെ അവർ അദ്ദേഹത്തിന്റെ അടുത്തു വന്നു. അവർ അല്ലാഹുവിന്റെ ദൂതരെ തിക്കിതിരക്കാൻ തുടങ്ങി.

നബിയാകട്ടെ അവർക്ക് ക്വുർആൻ പാരായണം ചെയ്തു കൊണ്ടിരിക്കാനും തുടങ്ങി. പിന്നീടവർ എന്റെയടുത്തു വരാൻ തുടങ്ങി, എന്നെ വലയം വെക്കാനും എനിക്ക് തടസ്സമുണ്ടാക്കാനും തുടങ്ങി. ഞാനാകെ പേടിച്ചരണ്ടു. അങ്ങനെ ഞാനവിടെ ഇരുന്നു. നേരം പുലരുന്നതിനനുസരിച്ച് അവർ പിരിഞ്ഞു പോവാൻ തുടങ്ങി. ശേഷം അല്ലാഹുവിന്റെ ദൂതൻ ക്ഷീണിതനും അവശനുമായി വന്നു; തിക്കും തിരക്കും കാരണം…” (സുനനു തുർമുദി: 2861)

ആത്മീയ ജീവികളായ ജിന്നുകളുമായി നബി (സ) നടത്തിയ സംവാദവും പ്രബോധനവുമാണ് ഹദീസിന്റെ ഇതിവൃത്തം. സംഭവത്തെ സംബന്ധിച്ച് ക്വുർആനും പ്രസ്താവിക്കുന്നുണ്ട്. “ജിന്നുകളില്‍ ഒരു സംഘത്തെ നാം നിന്‍റെ അടുത്തേക്ക് ഖുര്‍ആന്‍ ശ്രദ്ധിച്ചുകേള്‍ക്കുവാനായി തിരിച്ചുവിട്ട സന്ദര്‍ഭം (ശ്രദ്ധേയമാണ്‌.) അങ്ങനെ അവര്‍ അതിന് സന്നിഹിതരായപ്പോള്‍ അവര്‍ അന്യോന്യം പറഞ്ഞു: നിങ്ങള്‍ നിശ്ശബ്ദരായിരിക്കൂ. അങ്ങനെ അത് കഴിഞ്ഞപ്പോള്‍ അവര്‍ തങ്ങളുടെ സമുദായത്തിലേക്ക് താക്കീതുകാരായിക്കൊണ്ട് തിരിച്ചുപോയി.” (ക്വുർആൻ: 46:29)

“ഒരു സംഘം പ്രാകൃതരായ ആളുകൾ വന്നു…” എന്ന് ഹദീസിൽ പ്രസ്താവിച്ചിരിക്കുന്നത് ആത്മീയ ജീവികളായ ജിന്നുകളെ ഉദ്ദേശിച്ചാണ്. ചില നിവേദനങ്ങളിൽ ഇത് വ്യക്തമായി പ്രസ്താവിക്കപ്പെട്ടിട്ടുമുണ്ട്. “مَن هؤلاءِ يا رسولَ اللهِ؟ قال: هؤلاء جِنُّ نَصِيبينَ، جاؤوا يَختصِمون إليَّ في أمورٍ كانتْ بينهم”. “ഇബ്നു മസ്ഊദ് ചോദിച്ചു: അല്ലാഹുവിന്റെ ദൂതരേ, ആരാണവർ ? അപ്പോൾ അദ്ദേഹം (സ) പറഞ്ഞു: അവർ ജിന്നുകളാണ്. അവർക്കിടയിലുള്ള ചില വിഷയങ്ങളെ സംബന്ധിച്ച് എന്നോട് സംവദിക്കാൻ വന്നതാണ് അവർ.”

“നീ എന്റെ പിറകിൽ ഈ വരക്കപ്പുറം നിൽക്കണം, അതിനപ്പുറം വരരുത്. ഈ വരവിട്ട് നീ മുന്നോട്ടു വന്നാൽ നീ മരിച്ചേക്കും” എന്ന് നബിയും(സ), “പിന്നീടവർ എന്റെയടുത്തു വരാൻ തുടങ്ങി, എന്നെ വലയം വെക്കാനും എനിക്ക് തടസ്സമുണ്ടാക്കാനും തുടങ്ങി. ഞാനാകെ പേടിച്ചരണ്ടു.” എന്ന് അബ്ദുല്ലാഹിബ്നു മസ്ഊദും (റ) പറയാനുള്ള കാരണം ആ സംഘം അദൃശ്യ ലോകത്തു നിന്നുള്ള അഭൗതിക ജീവവർഗമായതു കൊണ്ടും അവരുമായി ഭൗതികമായി സന്ധിക്കുവാൻ അല്ലാഹു ഒരുക്കിയ അസുലഭവും അത്ഭുതകരവുമായ ഒരു സംഗമമായിരുന്നു അത് എന്നതു കൊണ്ടുമാണ്.

ആത്മീയ ജീവികളായ ജിന്നുകൾ മുഹമ്മദ് നബിയുമായി സ്വവർഗരതിയിൽ ഏർപ്പെട്ടു എന്ന് നാസ്തികർക്ക് എങ്ങനെയാണ് വാദിക്കാനാവുക എന്നത് മറ്റൊരു കാര്യം.

ഭീമാകാരമായ ശരീരമുള്ള, മാംസമില്ലാത്ത ഈ ആത്മീയ ജീവവർഗ്ഗം (ഒരു തരം ജിന്ന് വിഭാഗം) വസ്ത്രം ധരിച്ചതായി കാണപ്പെട്ടില്ലെങ്കിലും അവരുടെ നഗ്നതയും കാണപ്പെട്ടിരുന്നില്ലെന്ന് നിവേദനത്തിൽ തന്നെ പ്രസ്‌താവിക്കുന്നുണ്ട്. ഒരു നിവേദനത്തിൽ لا أرَى عَورةً ولا أرَى قِشرًا “അവരുടെ നഗ്നതയൊ തൊലിയൊ ഞാൻ കണ്ടില്ല” എന്ന് വന്നിരിക്കുന്നു. അഥവാ ഗുഹ്യാവയവങ്ങളൊ ലൈംഗികാവയവങ്ങളൊ പോയിട്ട് ശരീരത്തിൽ തൊലി പോലും കാണപ്പെടാത്ത വിധമാണ് അവരുടെ ശരീര സൃഷ്ടി ! ലൈംഗികാവയവങ്ങളില്ലാതെ പിന്നെ എങ്ങനെയാണ് സ്വവർഗരതിയൊക്കെ നടന്നത് എന്ന് നാസ്തികർ തന്നെ വ്യക്തമാക്കണം.

ഇസ്‌ലാം പഠിക്കാൻ വേണ്ടി വന്ന ജിന്നുകളായിരുന്നു അവർ. പ്രവാചകനെ(സ) ആദ്യമായി കണ്ട ആവേശത്തിൽ അവർ അദ്ദേഹത്തിന് ചുറ്റും തിങ്ങി കൂടി… “അങ്ങനെ അവർ അദ്ദേഹത്തിന്റെ അടുത്തു വന്നു. അവർ അല്ലാഹുവിന്റെ ദൂതരെ തിക്കിതിരക്കാൻ തുടങ്ങി. നബിയാകട്ടെ അവർക്ക് ക്വുർആൻ പാരായണം ചെയ്തു കൊണ്ടിരിക്കാനും തുടങ്ങി..” എന്ന ഹദീസിലെ വരികൾ ശ്രദ്ധിക്കുക. അവർ തിക്കിതിരക്കി വന്നത് ‘ഓർജി’ക്കായിരുന്നില്ല. ഇസ്‌ലാം പഠിക്കാനായിരുന്നു. അവർ വന്നപ്പോൾ പ്രവാചകൻ (സ) ചെയ്തത് ക്വുർആൻ പാരായണമായിരുന്നു; ഏതെങ്കിലും തരത്തിലുള്ള ലൈംഗിക ചേഷ്ടകളായിരുന്നുമില്ല.

ഇവിടെ ലൈംഗികതയുമായി എങ്ങിനെയാണ് ഈ സംഭവം ബന്ധപ്പെട്ടു കിടക്കുന്നത് എന്ന് ഒരു സാധാരണക്കാരൻ ആശ്ചര്യപ്പെട്ടേക്കാം. പക്ഷെ, ഒരു മാനസിക രോഗിയുടെ ചിന്തകളുടെ സഞ്ചാരപഥം എത്ര സങ്കീർണ്ണമാണ് എന്ന് മിഷണറിമാരുടെയും നാസ്തികരുടെയും ദുർവ്യഖ്യാനങ്ങൾ നമുക്ക് ബോധ്യപ്പെടുത്തി തരും. ദുർവ്യാഖ്യാനം ഇപ്രകാരമാണ്:

1. ജിന്നുകളുടെ ശരീര പ്രകൃതിയെ വർണ്ണിച്ചു കൊണ്ട് “അവരുടെ മേൽ വസ്ത്രമുണ്ടായിരുന്നില്ല, എന്നാൽ അവരുടെ നഗ്നത എനിക്ക് കാണാൻ കഴിഞ്ഞില്ല.” എന്ന് ഇബ്നു മസ്ഊദ് പറഞ്ഞ വാചകം ആദ്യം ദുർവ്യാഖ്യാനിച്ചു വഷളാക്കി. വസ്ത്രം അഴിച്ചു വന്ന ഒരു പറ്റം മനുഷ്യരായി അവരെ വിമർശകർ വ്യാഖ്യാനിച്ചു. യഥാർത്ഥത്തിൽ പ്രവാചകന്റെ(സ) അടുത്തു വന്ന ജിന്നു വിഭാഗത്തിന്റെ ശരീരഘടനയാണ് ഇവിടെ പ്രസ്‌താവിക്കുന്നത്. മനുഷ്യർക്ക് സമാനമായ ശരീരമല്ലായിരുന്നു അവരുടെത്. മനുഷ്യേതര ജീവജാലങ്ങളെ പോലെ വസ്ത്രം ധരിക്കാത്ത ജീവികൾ; എന്നാൽ അവരുടെ നഗ്നത കാണാനും കഴിയില്ല. മൃഗങ്ങളിലെ രോമകുപ്പായം (fur) പോലെ വേണമെങ്കിൽ ആ ശരീര പ്രകൃതിയെ – മനുഷ്യർക്ക് മനസ്സിലാക്കാക്കാനായി – വ്യാഖ്യാനിക്കാം. ഇങ്ങനെ അത്ഭുതകരമായ ശരീര പ്രകൃതിയെ സൂചിപ്പിച്ചു കൊണ്ടുള്ള വരികളെ ഓർജിക്കു തയ്യാറായി വരുന്ന ഒരു സംഘം സ്വവർഗരതിക്കാരായി വ്യാഖ്യാനിക്കാൻ നിസ്സാര തൊലിക്കട്ടി ഒന്നും പോര.

2. “അങ്ങനെ അവർ അദ്ദേഹത്തിന്റെ അടുത്തു വന്നു. അവർ അല്ലാഹുവിന്റെ ദൂതരെ തിക്കിതിരക്കാൻ തുടങ്ങി. നബിയാകട്ടെ അവർക്ക് ക്വുർആൻ പാരായണം ചെയ്തു കൊണ്ടിരിക്കാനും തുടങ്ങി..” എന്ന ഹദീസിലെ വരികളാണ് അടുത്തതായി ലൈംഗികവൽകരിക്കപ്പെട്ടിരിക്കുന്നത്. ഇവിടെ “അവർ അല്ലാഹുവിന്റെ ദൂതരെ തിക്കിതിരക്കാൻ തുടങ്ങി…” എന്ന വാചകത്തിലെ ‘യർകബൂന’ (അവർ തിക്കിതിരക്കുന്നു يركبون) എന്ന അറബി പദത്തിനെ ആദ്യം ഇംഗ്ലീഷിലേക്ക് വിവർത്തനം ചെയ്തു. മുപ്പതിലേറെ അർത്ഥങ്ങളുള്ള ഈ ഒരു അറബി പദത്തിന് ദുർവ്യാഖ്യാനത്തിനുതകുന്ന ഒരു ഇംഗ്ലിഷ് അർത്ഥം അവർ കണ്ടെത്തുന്നു; Drive/Ride. Ride എന്ന ഇംഗ്ലിഷ് പദത്തിനെ അനൗപചാരികമായി (colloquial) ലൈംഗികമായി ബന്ധപ്പെടുക എന്ന ഒരു അർത്ഥത്തിൽ ഉപയോഗിക്കാറുണ്ടല്ലൊ. അപ്പോൾ ജിന്നുകൾ നബിയെ Ride ചെയ്തു; ലൈംഗികമായി ബന്ധപ്പെട്ടു എന്നർത്ഥം വരുന്നു !! എങ്ങനെയുണ്ട് കണ്ടെത്തൽ?!

പണ്ട് സ്കൂളിൽ പഠിക്കുന്ന കാലത്ത് കലോത്സവത്തിനിടയിൽ “ഇന്നയിന്ന വിദ്യാർത്ഥികൾ ഇന്ന സാറുമായി എത്രയും പെട്ടെന്ന് ‘ബന്ധപ്പെടുക ‘…. ജഡ്ജസുമായി ‘ബന്ധപ്പെടുക ‘… സ്റ്റേജുമായി ‘ബന്ധപ്പെടുക ‘…” അനൗൺസ്മെന്റ് കേൾക്കുമ്പോൾ ഒരു പറ്റം അലവലാതി ചെക്കന്മാർ പൊട്ടിച്ചിരിക്കുമായിരുന്നത് ഓർമ്മ വന്നു പോവുകയാണ്. അവരുടെ മനസ്സിലെ ‘ബന്ധപ്പെടുക’ എന്ന് പറഞ്ഞാൽ അത് ലൈംഗിക അർത്ഥത്തിലായിരുന്നു. അവരൊക്കെ “പഠിച്ച്” “മിടുക്കന്മാരായി” യുക്തിവാദി സംഘത്തിന്റെ നേതൃത്വത്തിൽ എത്തി എന്ന് ഇപ്പോൾ മനസ്സിലാവുന്നു. സന്തോഷം.

ഏതായിരുന്നാലും ഹദീസിലെ فَأَتَوْا، فَجَعَلُوا يَرْكَبُونَ رَسُولَ اللهِ صَلَّى اللهُ عَلَيْهِ وَسَلَّمَ എന്ന വാചകത്തിനർത്ഥം “അവർ അല്ലാഹുവിന്റെ ദൂതരുടെ അടുത്ത് തിക്കിതിരക്കാൻ തുടങ്ങി” എന്നാണ് എന്നതിൽ യാതൊരു സംശയവും ഇല്ല. സംഭവത്തിന്റെ മറ്റൊരു നിവേദനത്തിൽ ‘യർകബൂന’ (അവർ തിക്കിതിരക്കുന്നു يركبون) എന്ന പദത്തിന്റെ വിവക്ഷയെന്താണെന്ന് വ്യക്തമായും വന്നിട്ടുണ്ട്:

انْطَلَقْتُ مَعَ النَّبِيِّ صَلَّى اللهُ عليه وآله وَسَلَّمَ لَيْلَةَ الْجِنِّ حَتَّى إِذَا أَتَى الْحَجُونَ، فَخَطَّ عَلَيَّ خَطًّا ثُمَّ تَقَدَّمَ إليهم فَازْدَحَمُوا عَلَيْهِ …

” …ശേഷം അദ്ദേഹം (നബി (സ)) ജിന്നുകളുകളുടെ അടുത്തേക്ക് ചെന്നു. അവർ അദ്ദേഹത്തിന് ചുറ്റും തിങ്ങി തിരക്കുകൂട്ടി.” (ദലാഇലുന്നുബുവ്വ: 2:232)

ഈ നിവേദനത്തിൽ ‘യർകബൂന’ (يركبون) എന്ന പദത്തിന് പകരം ‘ഇസ്ദഹമൂ’ (ازْدَحَمُوا crowded) എന്ന് തന്നെ അർത്ഥശങ്കക്കിടയില്ലാത്ത വിധം വന്നിരിക്കുന്നു. (https://translate.google.com/?sl=ar&tl=en&text=%D8%A7%D8%B2%D8%AF%D8%AD%D9%85%D9%88%D8%A7%0A&op=translate )

يركبون، أي: يزحمونه ويقربون منه ഭാഷാ പണ്ഡിതർ പറഞ്ഞിരിക്കുന്നു: “യർകബൂൻ (يركبون) എന്നാൽ ചുറ്റും തിങ്ങി തിരക്കുകൂട്ടുകയും അടുക്കുകയും ചെയ്യുക എന്നാണ്…” (തഹ്കീക്: മുസ്നദു അഹ്മദ്: 6:337 )

ക്വുർആനും ഈ സംഭവത്തെ സംബന്ധിച്ച് സംസാരിക്കുന്നതും ഇതേ അർത്ഥത്തിൽ തന്നെയാണ്: “അല്ലാഹുവിന്‍റെ ദാസന്‍ (നബി) അവനോട് പ്രാര്‍ത്ഥിക്കുവാനായി എഴുന്നേറ്റ് നിന്നപ്പോള്‍ അവര്‍ (ജിന്നുകൾ) അദ്ദേഹത്തിന് ചുറ്റും ‘തിങ്ങിക്കൂടുവാനൊരുങ്ങി’ എന്നും…” (ക്വുർആൻ: 72: 19)

3. ഈ തിക്കും തിരക്കും കഴിഞ്ഞ് പ്രവാചകൻ (സ) ക്ഷീണിച്ച് വന്ന് ഉറങ്ങി എന്ന് ഹദീസിൽ സൂചിപ്പിക്കപ്പെട്ടതാണ് അടുത്ത ദുർവ്യാഖ്യാനം. രതിക്കു ശേഷമുള്ള ക്ഷീണമായിരുന്നത്രെ അത്.?!

ഭീകരവും ഭീമാകാര രൂപികളുമായ ഒരു വലിയ സംഘം ജിന്നുകൾക്ക് ഒരു ദിവസം മുഴുവൻ ക്വുർആൻ ഓതി കൊടുത്ത്, സമ്മേളനം കഴിഞ്ഞ് വന്നു കിടന്നുറങ്ങിയ നിഷ്കളങ്കമായ ആ ഉറക്കത്തെ ലൈംഗികവൽകരിക്കാൻ ലജ്ജയുടെ ഒരു കണിക പോലും ബാക്കിയില്ലെ ഇവരുടെയൊന്നും മനസ്സിൽ. തലയിൽ മുഴുവൽ ലൈംഗിക ചിന്തകൾ മാത്രമാണൊ അവശേഷിക്കുന്നത് ?!

ഇനി നിവേദനത്തിന്റെ പരമ്പരയും സ്വീകാര്യതയും നിരൂപണ വിധേയമാക്കാം:

നിവേദനത്തിന്റെ സനദും (പരമ്പര) മത്നും (ഉള്ളടക്കവും) ദുർബലമാണെന്ന് ഒരുപാടു പണ്ഡിതർ വീക്ഷിച്ചിട്ടുണ്ട്.

നിവേദനത്തിന്റെ സനദ് ഇപ്രകാരമാണ്:

حَدَّثَنَا عَارِمٌ، وَعَفَّانُ، قَالَا: حَدَّثَنَا مُعْتَمِرٌ، قَالَ: قَالَ أَبِي: حَدَّثَنِي أَبُو تَمِيمَةَ، عَنْ عَمْرٍو – لَعَلَّهُ أَنْ يَكُونَ قَدْ قَالَ: الْبِكَالِيَّ يُحَدِّثُهُ عَمْرٌو – عَنْ عَبْدِ اللهِ بْنِ مَسْعُودٍ – قَالَ عَمْرٌو إِنَّ عَبْدَ اللهِ -، قَالَ:

ഇബ്നു മസ്ഊദിൽ നിന്ന് അംറ് അൽ ബക്കാലിയാണ് സംഭവം ഉദ്ധരിക്കുന്നത്.

അംറ് അൽ ബക്കാലിയാകട്ടെ ഇബ്നു മസ്ഊദിൽ നിന്ന് ഹദീസൊന്നും കേട്ടിട്ടില്ല. ഇമാം ബുഖാരി പറഞ്ഞു: ولا يُعرف لعمرو سماعٌ من ابن مسعود അംറ് അൽ ബക്കാലി, ഇബ്നു മസ്ഊദിൽ നിന്ന് ഹദീസൊന്നും കേട്ടതായി അറിയപ്പെട്ടിട്ടില്ല. (അത്താരീഖുൽ കബീർ: 2: 200)

അംറ് അൽ ബക്കാലിക്ക് ‘സുഹ്ബത്ത്’ (പ്രവാചകാനുചരനാകാനുള്ള അവസരം) ഉണ്ടായിട്ടുണ്ട് എന്ന ഒരു അഭിപ്രായം ചില പണ്ഡിതർക്കുണ്ട്. ഇത് ശരിയാണെങ്കിൽ തന്നെ ഈ സംഭവം അംറ് അൽ ബക്കാലി, ഇബ്നു മസ്ഊദിൽ നിന്ന് ഒരു മധ്യവർത്തി മുഖേനയാണ് ഉദ്ധരിക്കുന്നത് എന്ന് വരുന്നു. ആ മധ്യവർത്തി മജ്ഹൂൽ (അറിയപ്പെടാത്ത നിവേദകൻ) ആണ്. അപ്പോൾ നിവേദനം ദുർബലമാണെന്ന് വരുന്നു.

കൂടാതെ നിവേദനം “വളരെ ഒറ്റപ്പെട്ടതാണെന്ന്” ഇബ്നു കസീറും, “പരമ്പര ദുർബലമാണെന്ന്” ശുഐബ് അൽ അർനാവൂത്വും, ഡോ. തൈസീറിബ്നു സഅ്ദ് അബൂ ഹൈമദും പ്രസ്‌താവിക്കുന്നു. (തഫ്സീറു ഇബ്നു കസീർ: 7:293, തഹ്കീക്: മുസ്നദു അഹ്മദ്: 6: 334, തഹ്കീക്: താരീഖുൽ ഔസത്: 2:1072)

(ശൈഖ് അബൂ ഉമർ അൽ ബാഹിസിന്റെ ലേഖനത്തിൽ നിന്ന് https://www.antishubohat.net/2012/09/08/3orah/)

ജിന്നുകളുമായുള്ള നബിയുടെ (സ) ഈ സംവാദം അബ്ദുല്ലാഹിബ്നു മസ്ഊദ് (റ) സാക്ഷിയായതായിട്ടാണല്ലൊ വിമർശന വിധേയമായ നിവേദനം ഉദ്ധരിക്കപ്പെട്ടിരിക്കുന്നത്. വളരെ പ്രബലമായ നിവേദനത്തിൽ അബ്ദുല്ലാഹിബ്നു മസ്ഊദ് (റ) ഈ സംഭവത്തിന് സാക്ഷിയായിട്ടില്ല എന്ന് അദ്ദേഹം തന്നെ പ്രസ്താവിക്കുന്നുണ്ട്.

عَنْ عَبْدِ اللهِ قَالَ: لَمْ أَكُنْ لَيْلَةَ الْجِنِّ مَعَ رَسُولِ اللهِ صَلَّى اللهُ عَلَيْهِ وَسَلَّمَ، وَوَدِدْتُ أَنِّي كُنْتُ مَعَهُ

അബ്ദുല്ലാഹിബ്നു മസ്ഊദ് (റ) പറഞ്ഞു: “ജിന്നുകൾ സന്ദർശിച്ച രാത്രി ഞാൻ അല്ലാഹുവിന്റെ ദൂതനോടൊപ്പം ഉണ്ടായിരുന്നില്ല, ഞാൻ അദ്ദേഹത്തോടൊപ്പം ആയിരുന്നെങ്കിൽ എന്ന് ഞാൻ ആഗ്രഹിച്ചു പോയി.” (സ്വഹീഹു മുസ്‌ലിം: 152)

പിന്നെ എങ്ങനെയാണ് ചർച്ചാ വിഷയകമായ നിവേദനം സ്വഹീഹ് (സ്വീകാര്യമാവുക) എന്ന് ചില ഹദീസ് പണ്ഡിതർ ചോദ്യം ചെയ്യുന്നുണ്ട്.

വിമർശനം: ഇസ്‌ലാമിലെ പ്രധാനപ്പെട്ട പുണ്യകർമങ്ങളായ ബാങ്ക്, ഇമാമത്ത് എന്നിവ സ്ത്രീകൾക്ക് തീർത്തും നിഷിദ്ധമാക്കുകയും പുരുഷന്മാർക്ക് യഥേഷ്‌ടം അനുവദിച്ചു കൊടുക്കുകയും ചെയ്തതിലൂടെ പുരുഷാധിപത്യത്തിനു കൊടിപിടിക്കുകയാണ് നബി ചെയ്തത്. സ്വന്തം മതത്തിലെ വലിയ പുണ്യകർമങ്ങൾ പോലും സ്ത്രീക്കു നിരുപാധികം നിഷിദ്ധമാക്കിയ ഒരാളെ ദൈവദൂതനായി കണക്കാക്കുന്നതു തന്നെ സ്ത്രീവിരുദ്ധമാണ്.

ഉത്തരം: ഇസ്‌ലാമിലെ പ്രധാനപ്പെട്ട പുണ്യകർമങ്ങളായ ബാങ്കും ഇമാമത്തും (നമസ്കാരത്തിനു നേതൃത്വം നൽകൽ) നിർവഹിക്കുന്നത് സ്ത്രീകൾക്കു നിഷിദ്ധമാക്കപ്പെട്ട കാര്യങ്ങളാണെന്ന അറിവില്ലായ്മയിൽ നിന്നാണ് ഈ വിമർശനം ജന്മമെടുത്തിരിക്കുന്നത്. ഇസ്‌ലാംവിമർശനങ്ങളിൽ പലതും ഉടലെടുക്കുന്നത് തികഞ്ഞ അറിവില്ലായ്മയിൽ നിന്നാണെന്ന് ഇത്തരം വിമർശനങ്ങളെ അവലോകനം ചെയ്താൽ ഏതൊരാൾക്കും നിഷ്പ്രയാസം മനസ്സിലാക്കാവുന്ന വസ്തുതയാണ്. വാസ്തവത്തിൽ ബാങ്കും ഇമാമത്തും നബി(സ്വ) സ്ത്രീകൾക്ക് വിലക്കുകയോ വിരോധിക്കുകയോ ചെയ്തിട്ടുണ്ടോ?. ഇല്ല എന്നതാണ് യാഥാർഥ്യം. വിശുദ്ധ ഖുർആനോ പ്രവാചക വചനങ്ങളോ ബാങ്കോ, ഇമാമത്തോ നിർവഹിക്കുന്നതിൽ നിന്നും സ്ത്രീകളെ വിലക്കുന്നില്ല. പിന്നെ എവിടെ നിന്നാണാവോ വിമർശകർ ഇത്തരം വിവരക്കേടുകൾ ഗവേഷണം ചെയ്തെടുക്കുന്നതെന്ന് മസ്സിലാകുന്നില്ല. മാത്രമല്ല സ്ത്രീകൾക്ക് ബാങ്കില്ല എന്ന നിലപാടിനെ കുറ്റകരമായ കാര്യമായാണ് സ്വഹാബികൾ മനസ്സിലാക്കിയിരുന്നത്. വസ്തുതകൾ നമുക്കൊന്നു വിശകലനം ചെയ്യാം.

سئل ابن عمر هل على النساء أذان؟ فغضب، وقال: أنا أنهى عن ذكر الله؟

സ്ത്രീകൾക്ക് ബാങ്ക് ഉണ്ടോ ? എന്ന് ഇബ്നു ഉമർ (റ) ചോദിക്കപ്പെട്ടു. അപ്പോൾ അദ്ദേഹം കോപിഷ്ടനായി പറഞ്ഞു: (സ്ത്രീകൾക്ക് ബാങ്കില്ല എന്ന് പറഞ്ഞ്) അല്ലാഹുവെ സ്മരിക്കുന്നതിൽ നിന്ന് ഞാൻ അവരെ തടയണൊ ?!. (മുസ്വന്നഫ് ഇബ്നു അബീ ശൈബ: 1/223). സ്ത്രീകൾക്ക് ബാങ്കില്ല എന്ന നിലപാടിനെ, അല്ലാഹുവെ സ്മരിക്കുന്നതിൽ നിന്നും അവരെ തടയുന്ന നിലപാടായാണ് ഇബ്നു ഉമർ (റ) കണ്ടതെന്നു ഈ നിവേദനം വ്യക്തമാക്കുന്നുണ്ട്. മാത്രമല്ല പ്രവാചക പത്നി ആഇശ (റ) തന്നെ ബാങ്കും ഇക്കാമത്തും ഇമാമത്തും നിർവഹിച്ചതായും ഹദീസ് ഗ്രന്ഥങ്ങളിൽ നിവേദനം ചെയ്യപ്പെട്ടിട്ടുണ്ട്.

وعن عائشة أنها كانت تؤذن وتقيم، وتؤم النساء، وتقف وسطهن

ആഇശയിൽ (റ) നിന്നും: അവർ ബാങ്കുവിളിക്കുകയും ഇകാമത്ത് കൊടുക്കുകയും സ്ത്രീകൾക്ക് ഇമാമായി (നേതാവായി) – അവരുടെ നടുവിൽ- നിൽക്കുകയും ചെയ്യുമായിരുന്നു. (സുനനുൽ കുബ്റാ: 1/408, 3:131, മുസ്തദ്‌റക്: ഹാകിം: 1/203-204, മുസ്വന്നഫ് ഇബ്നു അബീ ശൈബ: 1/223, 2/89).

മറ്റു ചില നിവേദനങ്ങൾ കൂടി നമുക്കു പരശോധിക്കാം.

حديث رائطة الحنفية أن عائشة أمت نسوة في المكتوبة فأمتهن بينهن وسطا.

നിർബന്ധ നമസ്കാരങ്ങളിൽ ആഇശ (റ) സ്ത്രീകൾക്ക് ഇമാമായി (നേതൃത്വം നൽകുന്നവരായി) നിന്നു. അവരുടെ നടുവിൽ നിന്നു കൊണ്ടാണ് ആഇശ (റ) നേതൃത്വം നൽകിയിരുന്നത്. (മുസ്വന്നഫ് അബ്ദുർറസാക് : 3/141, ദാറകുത്നി: 1/404, ബൈഹകി: 3/131) സ്ത്രീകൾക്കു വേണ്ടി സ്ത്രീ ഇമാം നിക്കുമ്പോൾ നടുവിലാണ് നിൽക്കേണ്ടതെന്നും, ബാങ്കും ഇക്കാമത്തും നിർവഹിക്കാമെന്നും ഈ നിവേദനങ്ങൾ വ്യക്തമാക്കുന്നുണ്ട്. ആഇശ(റ) മാത്രമല്ല മറ്റു നബിപത്നിമാരും സ്ത്രീകൾക്കു വേണ്ടിയുള്ള നമസ്ക്കാരങ്ങൾക്ക് നേതൃത്വം നൽകിയതായും ഇസ്‌ലാമിക പ്രമാണങ്ങളിൽ കാണാവുന്നതാണ്. അത്തരമൊരു റിപ്പോർട്ട് കാണുക.

رواية حجيرة بنت حصين قالت: ” أمتنا أم سلمة في صلاة العصر قامت بيننا “. رواه عبد الرزاق أيضا، وابن أبي شيبة (2 / 88)،

ഹജീറ ബിൻത് ഹുസ്വൈൻ (റ) പറഞ്ഞു: അസർ നമസ്കാരത്തിന് (പ്രവാചക പത്നി) ഉമ്മു സലമ (റ) ഞങ്ങൾക്ക് ഇമാമായി (നേതൃത്വം നൽകുന്നവരായി) നിന്നു. അവർ ഞങ്ങളുടെ നടുവിൽ നിന്നു കൊണ്ടാണ് നേതൃത്വം നൽകിയത്. (മുസ്വന്നഫ് അബ്ദുർറസാക്, മുസ്വന്നഫ് ഇബ്നു അബീ ശൈബ: 2:88)

باب إِمَامَةِ النِّسَاءِ അഥവാ സ്ത്രീകളുടെ ഇമാമത്ത് (നേതൃത്വം) എന്ന അധ്യായങ്ങൾ ഇസ്‌ലാമിക ഗ്രന്ഥങ്ങളിൽ സ്ഥാനം പിടിച്ചിരിക്കെ, ഇസ്‌ലാം സ്ത്രീകളെ ബാങ്കും ഇമാമത്തും നിർവഹിക്കുന്നതിൽ നിന്നും തടഞ്ഞു എന്ന കല്ലുവെച്ച കള്ളം ഒരു പെൺപക്ഷ ഗവേഷണ പഠനമായി അവതരിപ്പിക്കാൻ അസാമാന്യ തൊലിക്കട്ടി തന്നെ വേണം!. വൈജ്ഞാനികസത്യസന്ധതക്കു കുഴിവെട്ടിയവരല്ലാതെ ഈ കാലഘട്ടത്തിൽ ഇസ്‌ലാം വിമർശനം നടത്തുന്നില്ലെന്നു പറയുന്നത് വെറുതെയല്ലെന്ന് ഇത്തരം വിമർശനങ്ങൾ ഓർമപെടുത്തുന്നുണ്ട്.

നബി പത്നിമാരല്ലാത്ത സ്വഹാബി വനിതകളും സ്ത്രീകൾക്കു വേണ്ടി ഇമാമത്ത് നിർവഹിച്ചിരുന്നു എന്നതും ഹദീസ് ഗ്രന്ഥങ്ങളിൽ ധാരാളം കാണാവുന്നതാണ്. അത്തരം ഒരു റിപ്പോർട്ട് കൂടി നമുക്കു പഠനവിധേയമാക്കാം.

عَنْ أُمِّ وَرَقَةَ بِنْتِ عَبْدِ اللَّهِ بْنِ نَوْفَلٍ الْأَنْصَارِيَّةِ أَنَّ النَّبِيَّ صَلَّى اللَّهُ عَلَيْهِ وَسَلَّمَ … وَكَانَتْ قَدْ قَرَأَتْ الْقُرْآنَ فَاسْتَأْذَنَتْ النَّبِيَّ صَلَّى اللَّهُ عَلَيْهِ وَسَلَّمَ أَنْ تَتَّخِذَ فِي دَارِهَا مُؤَذِّنًا فَأَذِنَ لَهَا قَالَ وَكَانَتْ قَدْ دَبَّرَتْ غُلَامًا لَهَا وَجَارِيَةً فَقَامَا إِلَيْهَا بِاللَّيْلِ فَغَمَّاهَا بِقَطِيفَةٍ لَهَا حَتَّى مَاتَتْ وَذَهَبَا فَأَصْبَحَ عُمَرُ فَقَامَ فِي النَّاسِ فَقَالَ مَنْ كَانَ عِنْدَهُ مِنْ هَذَيْنِ عِلْمٌ أَوْ مَنْ رَآهُمَا فَلْيَجِئْ بِهِمَا فَأَمَرَ بِهِمَا فَصُلِبَا فَكَانَا أَوَّلَ مَصْلُوبٍ بِالْمَدِينَةِ

ഉമ്മു വറക്ക ബിൻത് അബ്ദുല്ലാഹിബ്നു നൗഫൽ അൽ അൻസ്വാരിയ്യ (റ) യിൽ നിന്ന്: ….അവർ ക്വുർആൻ മനപാഠമാക്കിയ ഒരു സ്ത്രീയായിരുന്നു. തന്റെ ഭവനത്തിൽ ബാങ്കുകൊടുക്കുന്ന ഒരാളെ ഏർപ്പാടു ചെയ്യാൻ അവർ നബിയോട് അനുവാദം ചോദിച്ചു. അദ്ദേഹം അവർക്ക് അനുവാദം നൽകി. അവർക്ക് ഭൃത്യരായ ഒരു പയ്യനും പെൺകുട്ടിയും ഉണ്ടായിരുന്നു. അവരെ രണ്ടു പേരെയും കൂട്ടി അവർ രാത്രി നമസ്ക്കരിക്കുമായിരുന്നു…(സുനനു അബൂദാവൂദ്: 591).

വീട്ടിലുള്ള പുരുഷന്മാർക്കുവരെ നമസ്കാരത്തിനു സ്ത്രീകൾ നേതൃത്വം നൽകിയ സംഭവങ്ങൾ ഹദീസ് ഗ്രന്ഥങ്ങൾ പരമ്പരകളോടെ നമുക്കു പറഞ്ഞുതരുന്നുണ്ട്. അത്തരം ഒരു നിവേദനം നമുക്കു കാണാം.

عَنْ أُمِّ وَرَقَةَ بِنْتِ عَبْدِ اللَّهِ بْنِ الْحَارِثِ بِهَذَا الْحَدِيثِ وَالْأَوَّلُ أَتَمُّ قَالَ *وَكَانَ رَسُولُ اللَّهِ صَلَّى اللَّهُ عَلَيْهِ وَسَلَّمَ يَزُورُهَا فِي بَيْتِهَا وَجَعَلَ لَهَا مُؤَذِّنًا يُؤَذِّنُ لَهَا وَأَمَرَهَا أَنْ تَؤُمَّ أَهْلَ دَارِهَا قَالَ عَبْدُ الرَّحْمَنِ فَأَنَا رَأَيْتُ مُؤَذِّنَهَا شَيْخًا كَبِيرًا

പ്രവാചകൻ (സ) ഉമ്മു വറകയുടെ വീട് സന്ദർശിക്കാറുണ്ടായിരുന്നു. അദ്ദേഹം അവർക്ക്, ബാങ്ക് വിളിക്കാൻ ആളെ ഏർപ്പാടു ചെയ്തു കൊടുത്തിരുന്നു. തന്റെ വീട്ടിലുള്ളവർക്ക് നേതൃത്വം നൽകി നമസ്ക്കരിക്കാൻ അവരോട് പ്രവാചകൻ (സ) കൽപ്പിക്കുകയും ചെയ്തു. അബ്ദുർറഹ്മാൻ പറഞ്ഞു: അവരുടെ ബാങ്കുവിളിക്കാരനായ പടു വൃദ്ധനെ ഞാൻ കാണുകയുണ്ടായി. (സുനനു അബൂദാവൂദ്: 592).

“അവരുടെ ബാങ്കുവിളിക്കാരനായ പടു വൃദ്ധനെ ഞാൻ കാണുകയുണ്ടായി.” എന്ന നിവേദനത്തിലെ വാചകങ്ങൾ, വീട്ടിലെ പുരുഷന്മാർക്കു വരെ ഇമാം നിൽക്കുന്നതിന് സ്ത്രീകളെ ഇസ്‌ലാം അനുവദിക്കുന്നുണ്ടെന്നു വ്യക്തമാക്കുന്നതാണ്.

ഇമാം സ്വൻആനി പറഞ്ഞു:

والحديث دليل علـى صحة إمامة المرأة أهل دارها، وإن كان فيهم الرجال، فإنه كان لها مـؤذن وكان شيخًا كما في الرواية والظاهر أنها كانت تؤمه وغلامها وجاريتهـا

വീട്ടിലുള്ളവർക്ക് സ്ത്രീ ഇമാമായി (നേതൃത്വം നൽകി) നമസകരിക്കൽ സ്വീകാര്യമാണെന്നതിന് ഹദീസ് തെളിവാണ്; വീട്ടുകാരിൽ പുരുഷൻ ഉൾപ്പെടുമെങ്കിലും. അവർക്ക് (ഉമ്മു വറക്കക്ക്) ഒരു ബാങ്കുവിളിക്കാരനുണ്ടായിരുന്നു. അയാൾ വൃദ്ധനായ ഒരു പുരുഷനായിരുന്നു എന്നാണ് നിവേദനത്തിൽ വന്നിരിക്കുന്നത്. അവർ ഭൃത്യരായ ആൺകുട്ടിക്കും പെൺകുട്ടിക്കും ഇമാമായി നമസ്ക്കരിച്ചു എന്ന് നിവേദനത്തിന്റെ പ്രത്യക്ഷാർത്ഥം സൂചിപ്പിക്കുന്നു. (സുബുലുസ്സലാം: 2:48)

ശംസുൽ ഹക്ക് അസീമാബാദി പറഞ്ഞു:

ثبت من هذا الحديث أن إمامة النساء وجماعتهن صحيحة ثابتة من أمر رسول الله صلى الله عليه وسلم ، وقد أمت النساء عائشة رضي الله عنها وأم سلمة رضي الله عنها في الفرض والتراويح قال الحافظ في تلخيص الحبير : حديث عائشة أنها أمت نساء فقامت وسطهن رواه عبد الرزاق… وظهر من هذه الأحاديث أن المرأة إذا تؤم النساء تقوم وسطهن معهن ولا تقدمهن . قال في السبل : والحديث دليل على صحة إمامة المرأة أهل دارها وإن كان فيهم الرجل فإنه كان لها مؤذنا وكان شيخا كما في الرواية ، والظاهر أنها كانت تؤمه وغلامها وجاريتها ، وذهب إلى صحة ذلك أبو ثور والمزني والطبري…

സ്ത്രീകളുടെ ഇമാമത്തും ജമാഅത്തും സ്വീകാര്യമാണെന്നും പ്രവാചകന്റെ (സ) കൽപ്പനയാൽ അത് സ്ഥാപിതമാണെന്നുമാണ് ഹദീസിൽ നിന്നും സ്ഥിരപ്പെടുന്നത്. ആഇശയും(റ) ഉമ്മു സലമയും(റ) സ്ത്രീകൾക്ക് നിർബന്ധ നമസ്ക്കാരത്തിലും തറാവീഹിലും നേതൃത്വം നൽകിയിട്ടുണ്ട്.

സ്ത്രീകൾ ഇമാം നിൽക്കുകയാണെങ്കിൽ നടുവിലാണ് നിൽക്കേണ്ടത് എന്നും മുന്നിൽ നീങ്ങി നിൽക്കുകയല്ല വേണ്ടത് എന്നും ഹദീസുകളിൽ നിന്നും വ്യക്തമാവുന്നു. പുരുഷന്മാർ ഉൾപ്പെടെ വീട്ടിലുള്ളവർക്ക് സ്ത്രീ ഇമാമായി (നേതൃത്വം നൽകി) നമസകരിക്കൽ സ്വീകാര്യമാണെന്നതിന് ഹദീസ് തെളിവാണ് എന്ന് സുബുലുസ്സലാമിൽ പറയപ്പെട്ടിരിക്കുന്നു… അബു സൗർ, മുസ്നി, ത്വബ്‌രി തുടങ്ങിയവർ ഈ അഭിപ്രായത്തെ ശരിവെക്കുന്നു… (ഔനുൽ മഅ്ബൂദ്: 2:225)

കാര്യങ്ങൾ വളരെ ലളിതമാണ്, വ്യക്തമാണ്, സ്ത്രീകൾക്ക് ഇസ്‌ലാം ബാങ്കും ഇമാമത്തും വിരോധിചിട്ടില്ല, വിലക്കിയിട്ടില്ല. സ്ത്രീകൾക്ക് സ്ത്രീ ഇമാം നിൽക്കുന്നത് മാത്രമല്ല, വീട്ടിലെ പുരുഷന്മാർക്ക് പോലും സ്ത്രീക്ക് ഇമാം നിൽക്കാൻ ഇസ്‌ലാം അനുവാദം നൽകിയിട്ടുണ്ട്. ഇസ്‌ലാമിക പ്രമാണങ്ങൾ ഇത്രമേൽ ചർച്ച ചെയ്ത ഒരു വിഷയത്തിൽ പോലും, അതെല്ലാം മറച്ചുവെച്ചും ദുർവ്യാഖ്യാനിച്ചും ഇത്തരം കളവുകൾ പ്രചരിപ്പിക്കുവാൻ ഇസ്‌ലാംവിമർശകർക്ക് ഒരു തരിമ്പ് പോലും ലജ്ജയില്ലാതായത് അവരുടെ ചർമസൗഭാഗ്യത്തിന്റെ അപാരതയല്ലാതെ മറ്റെന്താണ് കുറിക്കുന്നത്.

സാമൂഹ്യബാധ്യതയുടെ ഭാരം ഇസ്‌ലാം സ്ത്രീയുടെ ചുമലിൽ കെട്ടിവെച്ചില്ല.

എന്നാൽ സാമൂഹ്യബാധ്യത എന്ന നിലയിൽ പുരുഷന്മാരടക്കമുള്ള മൊത്തം സമൂഹത്തിനുവേണ്ടി ബാങ്കും ഇമാമത്തും നിർവഹിക്കേണ്ട ബാധ്യതയിൽ നിന്നും ഇസ്‌ലാം സ്ത്രീയെ മാറ്റി നിർത്തിയിട്ടുണ്ട്. ആ രംഗത്ത് ഇസ്‌ലാം ബാധ്യതയുടെ ഭാരം പുരുഷനെയാണ് ഏൽപ്പിച്ചിരിക്കുന്നത്. അതു പക്ഷെ സ്ത്രീകൾക്ക് ഏതെങ്കിലും തരത്തിലുള്ള ആത്മീയ അർഹതകുറവുണ്ടെന്നു കണകാക്കികൊണ്ടുള്ള ഒരു നിലപാടല്ല. മറിച്ച് പുരുഷ പ്രകൃതിയാണ് ആ ബാധ്യതയുടെ ഭാരം ഏറ്റെടുക്കാൻ ഏറ്റവും അനുയോജ്യമെന്നതു കൊണ്ടാണ്. ഇസ്‌ലാമിന്റെ ഈ വിഷയകമായുള്ള സമീപനത്തെ യുക്തിപരമായി സമീപിക്കുന്ന ഏതൊരാൾക്കും അതിലെ ന്യായം ഉൾകൊള്ളാൻ സാധിക്കുന്നതാണ്. നൈതികതയുടെ ഒരു പ്രശ്നവും ആ രംഗത്ത് കണ്ടെത്താൻ നിഷ്പക്ഷമായ ഒരു വിശകലനത്തിനും സാധ്യമല്ല തന്നെ. ഓരോ കാര്യങ്ങളും നമുക്ക് യുക്തിപരമായി വിശകലം ചെയ്യാം.

1, സ്വഫ്ഫിന്റെ ഘടനയും പെണ്ണിമാമത്തും:

ജമാഅത്ത് നമസ്കാരത്തിലെ (സംഘ നമസ്കാരം) സ്വഫ്‌ഫിന്റെ (അണി) ഘടനക്ക് ഇസ്‌ലാം പുണ്യം നിശ്ചയിച്ച ഒരു രീതിയുണ്ട്. സ്വഫ്ഫിലെ ആദ്യ ഭാഗമാണ് പുരുഷന്മാർക്ക് ഏറ്റവും പുണ്യകരം. അതിനു ശേഷം ഉള്ള സ്വഫ്ഫുകൾ പിറകിലേക്ക് പോകുന്തോറും പുണ്യം കുറഞ്ഞു വരുന്നു. സ്ത്രീകൾക്കാവട്ടെ അണിയുടെ അവസാനത്തിൽ നിന്നാണ് പുണ്യം തുടങ്ങുന്നത്. അത് മുന്നിലേക്ക് പോകുന്തോറും പുണ്യം കുറഞ്ഞു വരുന്നു. ഇതാണ് സ്വഫ്ഫിൽ ഇസ്‌ലാം നിശ്ചയിച്ച പുണ്യത്തിന്റെ ഘടന. അതു പ്രകാരം സ്ത്രീ, പുരുഷന്മാർ അടക്കമുള്ള സംഘനമസ്കാരത്തിന് നേതൃത്വം നൽകുമ്പോൾ സ്വാഭാവികമായും അവളുടെ തൊട്ട് പിറകിൽ നിൽക്കേണ്ടത് പുരുഷന്മാരാണ്. നമസ്കാരമെന്നത് വിവിധ ചലനങ്ങൾ ഉൾകൊള്ളുന്ന ഒരു പ്രാർത്ഥനയാണ്. സ്ത്രീ നിന്നും വളഞ്ഞും ഇരുന്നും ഇളകിയും കുമ്പിട്ടുകിടന്നും നമസ്കാരത്തിന് നേതൃത്വം നൽകുമ്പോൾ അവൾക്കു പിറകിൽ നിൽക്കുന്ന പുരുഷന്മാർക്ക് അതു ജൈവികമായ ചില പ്രശ്നങ്ങൾ സൃഷിച്ചേക്കാം. കാരണം അത്തരം സന്ദർഭങ്ങളിൽ സ്ത്രീയുടെ നിമ്‌നോന്നതികള്‍ വെളിവാക്കപ്പെടാനുള്ള സാധ്യത കൂടുതലാണ്. പിറകിൽ നിൽക്കുന്ന പുരുഷന്മാരിൽ ചിലർക്കെങ്കിലും അതുമൂലം ആത്മസാന്നിധ്യം നഷ്ട്ടപെടാൻ ഇടയുണ്ട്. മാത്രമല്ല സ്ത്രീകൾക്കും അത്തരം ഒരു സന്ദർഭത്തിൽ മനസാന്നിധ്യത്തോടെ നമസ്കാരത്തിനു നേതൃത്വം നൽകാൻ ബുദ്ധിമുട്ടുണ്ടാകുമെന്നത് ആർക്കും നനസ്സിലാക്കാവുന്ന വസ്തുതയാണ്. സ്ത്രീ-പുരുഷ മനഃശാസ്ത്രവും ലൈംഗിക ശാസ്ത്രവും പഠിച്ച ഏതൊരാൾക്കും ഇവിടെ യുക്തിപരമായി സമർപ്പിക്കാവുന്ന നിർദേശം, സാമൂഹ്യ ബാധ്യതയുള്ള സംഘനമസ്കാരത്തിനു സ്ത്രീയേക്കാൾ നല്ലത് പുരുഷനെ തിരഞ്ഞെടുക്കുന്നതാണെന്നായിരിക്കും. കേവലം യുക്തിപരമായി ചിന്തിച്ചാൽ പോലും സാമൂഹ്യ ബാധ്യതയായ സംഘനമസ്കാരത്തിന് നേതൃത്വം കൊടുക്കാൻ ഏറ്റവും അനുയോജ്യമായ പ്രകൃതിഘടന സ്ത്രീയേക്കാൾ പുരുഷനിലാണ് കാണാനാകുന്നത്. അപ്പോൾ മതം ആ നിലപാട് സ്വീകരിക്കുമ്പോൾ അതിലൊരു പെൺവിരുദ്ധതയും കാണേണ്ടതില്ലെന്നർത്ഥം.

2, സ്ത്രീയുടെ സ്വകാര്യ സമയങ്ങളും ഇമാമത്തും:

സ്ത്രീകളെ സംബന്ധിച്ച് അവരുടെ സ്വകാര്യ സമയങ്ങളിൽ പെട്ടതാണ് ആർത്തവ സമയം. അതു മറ്റുള്ളവർ അറിയുന്നത് പൊതുവേ സ്ത്രീകൾ ഇഷ്ട്ടപെടാറില്ല. അതെല്ലാം ജനമധ്യത്തിൽ കൊട്ടിഘോഷിക്കുന്നതിൽ അഭിരമിക്കുന്ന പിടിവിട്ട പരിഷ്കരണവാദികളായ സ്ത്രീകളെ ഒഴിച്ചു നിർത്തിയാൽ, ആത്മാഭിമാനമുള്ള പല സ്ത്രീകളും അത്തരം സമയങ്ങൾ സ്വകാര്യമാക്കി വെക്കാനാണ് ഇഷ്ട്ടപ്പെടുക. ഒരു സ്ത്രീക്ക് സാമൂഹ്യബാധ്യത എന്ന നിലയിൽ പുരുഷന്മാരടക്കമുള്ള മൊത്തം സമൂഹത്തിനുവേണ്ടി ബാങ്കും ഇമാമത്തും നിർവഹിക്കേണ്ട ഉത്തരവാദിത്തം ഏൽപ്പിക്കപ്പെട്ടാൽ ഫലത്തിൽ സംഭവിക്കുന്നത് അവളുടെ അത്തരം സ്വകാര്യ സമയങ്ങളെ പരസ്യമാക്കാൻ നിർബന്ധിക്കലായിരിക്കും. കാരണം ആർത്തവ സമയങ്ങളിൽ സ്ത്രീക്ക് നമസ്കാരമടക്കമുള്ള പല ആരാധനകളും നിർവഹിക്കുവാൻ പാടില്ലാത്തതാണ്. അപ്പോൾ മാസത്തിൽ ചുരുങ്ങിയത് ഏഴ് ദിവസങ്ങളെങ്കിലും അവൾക്ക് സമൂഹത്തിന് ഇമാം നിൽക്കാൻ സാധ്യമല്ലാതെ വരും. നിരന്തരമായി മാസത്തിലെ പ്രത്യേക ദിവസങ്ങളിൽ ഇമാം ഇല്ലാതെ വരുമ്പോൾ സമൂഹം തിരിച്ചറിയും ഈ ദിനങ്ങളാണ് നമ്മുടെ ഇമാമിന്റെ ആർത്തവ ദിനങ്ങളെന്ന്. മാത്രമല്ല ആർത്തവചക്രം കൃത്യമല്ലെങ്കിൽ ഇമാമിന്റെ ആർത്തവ പ്രശ്നവും സമൂഹമറിയും. ഇനി ദീർഘിച്ചതോ ചുരുങ്ങിയതോ ആയ ആർത്തവചക്രമുള്ള സ്ത്രീയാണ് ഇമാം എങ്കിൽ അതും പൊതുസമൂഹത്തിന്റെ ശ്രദ്ധയിൽ പെടും. ഇതെല്ലാം തന്നെ അഭിമാനബോധമുള്ള സ്ത്രീകൾക്ക് വലിയ പ്രയാസം സൃഷ്ടിക്കുന്ന കാര്യങ്ങളാണെന്ന് ആർക്കും പറഞ്ഞുകൊടുക്കേണ്ടതില്ലല്ലോ. ഇത്തരം സ്വകാര്യ സമയങ്ങൾ സമൂഹത്തിൽ കൊട്ടിഘോഷിക്കുന്നതിൽ ഹരം കൊള്ളുന്ന പരിഷ്കാരിണികൾ ഇതിനെല്ലാം അപവാദമാണെങ്കിലും, മതത്തിനു പക്ഷെ മാന്യമഹിളകളെ കണ്ടല്ലേ ഒരു നിയമം ആവിഷ്കരിക്കാനാവുക. പരിഷ്കാരിണികൾ തൽക്കാലം ക്ഷമിക്കുകയല്ലാതെ മറ്റു പോംവഴികളില്ല. ഇതെല്ലാം മതം സ്വീകരിച്ച നിലപാടുകളിൽ ന്യായങ്ങൾ കണ്ടെത്താനുള്ള കേവല യുക്തിചിന്താപരിശ്രമം എന്നതിനപ്പുറം, ഇതാണ് മതത്തിനു പറയാനുള്ള ന്യായവാദങ്ങൾ എന്നു വെറുതെ തെറ്റിദ്ധരിച്ചിടരുത് എന്ന് ഇസ്‌ലാംവിമർശകരെ പ്രത്യേകം ഓർമപ്പെടുത്തുന്നു.

3, സ്ത്രീയുടെ അബലതകളെ പരിഗണിക്കാത്ത നിലപാടാണ് പെണ്ണിമാമത്ത്:

പുരുഷനെ അപേക്ഷിച്ച് സ്ത്രീകൾക്ക് പ്രകൃതിപരമായി തന്നെ ധാരാളം അബലതകൾ ഉണ്ടെന്നത് ഒരു വസ്തുതയാണ്. ഗർഭധാരണം, പ്രസവം, പ്രസവാനന്തരമുള്ള ദീർഘകാലയളവിലെ പ്രശ്നങ്ങൾ, മുലയൂട്ടൽ, ആർത്തവം, ആർത്തവ വിരാമം തുടങ്ങിയ പ്രശ്നങ്ങളും കൂടാതെ ഈ കാലയളവിൽ സ്ത്രീ അനുഭവിക്കുന്ന ശാരീരികവും മാനസീകവും ഹോർമോൺ സംബന്ധവുമായ മറ്റൊരുപാട് പ്രശ്നങ്ങളും വേറെയുമുണ്ട്. അത്തരം അവസ്ഥകളിൽ, സാമൂഹ്യബാധ്യത എന്ന നിലയിൽ പുരുഷന്മാരടക്കമുള്ള മൊത്തം സമൂഹത്തിനുവേണ്ടി ബാങ്കും ഇമാമത്തും നിർവഹിക്കേണ്ട ഉത്തരവാദിത്തം സ്ത്രീയെ ഏൽപ്പിക്കുന്നത് അവളോട് ചെയ്യുന്ന വലിയ ക്രൂരതയായിരിക്കും. കാരണം ഒരു ദിവസം പുലരി മുതൽ രാത്രി വരെ സമയ ബന്ധിതമായി നിർവഹിക്കാൻ ബാധ്യതപെട്ട ഒരു ഉത്തരവാദിത്തം ഈ പ്രകൃതിപരമായ പ്രശ്നങ്ങൾ ഒന്നുമില്ലാത്ത പുരുഷൻ ഉണ്ടായിരിക്കെ സ്ത്രീയെ പിടിച്ച് അതേൽപ്പിക്കുന്നത് കടുത്ത അപരാധമായിരിക്കുമെന്നു യുക്തിക്കു സ്വീകാര്യമായ ഒരു നിലപാടാണല്ലോ. ഒരുപാട് പ്രകൃതിപരമായ അബലതകൾ ഉള്ള ഒരു വിഭാഗവും അത്തരം പ്രശ്നങ്ങൾ ഒന്നുമില്ലാത്ത മറ്റൊരു വിഭാഗവും മുന്നിലിരിക്കെ ഇത്തരം സാമൂഹിക ഉത്തരവാദിത്തം ഏതു വിഭാഗത്തെയാണ് ഏൽപ്പിക്കേണ്ടത്?. യുക്തിപരമായി ചിന്തിക്കുന്ന ഏതൊരാൾക്കും പറയാനുള്ള മറുപടിയെന്താണോ അതു മാത്രമല്ലേ ഇസ്‌ലാം ഇവിടെ സ്വീകരിച്ചിരിക്കുന്ന നിലപാട്. അപ്പോൾ സ്ത്രീക്ക് ബാങ്കും ഇമാമത്തും ഇസ്‌ലാം വിലക്കി എന്ന വിമർശനം പ്രമാണവിരുദ്ധവും, സാമൂഹ്യബാധ്യതയായ ബാങ്കും ഇമാമത്തും ഇസ്‌ലാം സ്ത്രീകളെ ഏൽപ്പിച്ചില്ല എന്ന വിമർശനം യുക്തിവിരുദ്ധവുമാകുന്നു. എങ്ങനെ നോക്കിയാലും ഇസ്‌ലാംവിമർശനമെന്ന പരിപ്പ് ബുദ്ധിയുള്ളവർക്കിടയിൽ വേവില്ലെന്ന് മനസ്സിലാക്കി ആ കലം അടുപ്പിൽ നിന്നും ഇറക്കിവെക്കുന്നതാണ് ഇസ്‌ലാംവിമർശകർക്കെല്ലാം നല്ലത്. കലത്തിന് തുള വീഴുമെന്നല്ലാതെ പരിപ്പ് വേവില്ല കൂട്ടരേ. കാരണം ഇതു ദൈവിക മതമാണ്.

മൃഗങ്ങളെ നാണിപ്പിക്കുന്ന മൃഗരതി കളവുകൾ !! വിമർശനം:

Sahih al-Bukhari 2, 357:

Ibn Sharib narrated, Ibn Abdul Talib said:

“Always when his wives had their period, I saw the Prophet (pbuh) near by his camel herd. There he had lovingly intercourse with the female animals, sometimes he also turned towards the young animals of both sexes.”

സ്വഹീഹുൽ ബുഖാരി 2, 357:

ഇബ്നു ശരീബ് വിവരിച്ചു, ഇബ്നു അബ്ദുൽ താലിബ് പറഞ്ഞു:

“എല്ലായ്‌പ്പോഴും അദ്ദേഹത്തിന്റെ ഭാര്യമാർക്ക് ആർത്തവം വരുമ്പോൾ നബി(സ)യെ അദ്ദേഹത്തിന്റെ ഒട്ടകക്കൂട്ടത്തിനരികിൽ ഞാൻ കാണാറുണ്ട്. അവിടെ അവൻ പെൺ മൃഗങ്ങളുമായി സ്നേഹപൂർവ്വം ലൈംഗിക ബന്ധത്തിൽ ഏർപ്പെട്ടു, ചിലപ്പോൾ അവൻ രണ്ട് ലിംഗത്തിലുള്ള മൃഗങ്ങളിലേക്കും തിരിഞ്ഞു.

മറുപടി:

സോഷ്യൽ മീഡിയയിലൂടെ പ്രചരിക്കപ്പെടുന്ന ഈ കള്ള കഥയുടെ ഇംഗ്ലീഷ് രൂപവും അത് കേരളത്തിലെ ഭൗതികവാദികൾ ‘ഗൂഗിൾ ട്രാൻസ്ലേറ്റർ’ ഉപയോഗിച്ച് (ഇല്ലാ കഥയാണെന്ന് അറിയാവുന്നതു കൊണ്ട് മിനക്കെട്ട് പരിഭാഷ ചെയ്യെണ്ടന്ന് തീരുമാനിച്ചതായിരിക്കാം) ഉണ്ടാക്കിയെടുത്ത് പ്രചരിപ്പിക്കുന്ന മലയാള തിരകഥയുമാണ് മുകളിൽ കൊടുത്തിരിക്കുന്നത്.

സ്വഹീഹുൽ ബുഖാരി 2, 357 എന്ന നമ്പറിട്ട് കൊടുത്തിട്ടുള്ള ഈ മൃഗരതി ഉള്ളടക്കമുള്ള ഒരു ഹദീസും സ്വഹീഹുൽ ബുഖാരിയിൽ എവിടെയുമില്ല! ഗൂഗിളിൽ ഈ നമ്പറും ഹദീസിന്റെ ഇംഗ്ലീഷ് ടെക്സ്റ്റും സെർച്ച് ചെയ്താൽ ഒരിക്കലും തുറക്കാൻ കഴിയാത്ത ചില വ്യാജ വെബ്സൈറ്റുകളുടെ പേരുകൾ പ്രത്യക്ഷപ്പെട്ടേക്കാം…. പല ഇസ്‌ലാമിക സൈറ്റുകളിലും വീഡിയൊകളുടെ കമന്റ് ബോക്സുകളിലെ നാസ്തിക വിധ്വേഷ പ്രചാരകരുടെ കമന്റുകളിലും മാത്രം ഈ കള്ള കഥയും കള്ള നമ്പറുകളും കാണാം. അതല്ലാതെ സ്വഹീഹുൽ ബുഖാരിയിൽ നേരിട്ടൊ, ബുഖാരിക്ക് മുസ്‌ലിം പണ്ഡിതന്മാർ എഴുതിയ പരിഭാഷകളിലൊ ഇങ്ങനെയൊരു കള്ള കഥ കാണാൻ സാധ്യമെ അല്ല. അഥവാ യാതൊരു അടിസ്ഥാനവുമില്ലാത്ത, മൃഗങ്ങളെ പോലും ലജ്ജിപ്പിക്കാനുതകുന്ന ഒരു വൃത്തികെട്ട പോൺ സ്റ്റോറി എഴുതി, തോന്നിയ നമ്പറിട്ട് അത് നബിയുടെ (സ) പേരിൽ പ്രചരിപ്പിക്കുകയാണിവിടെ.

നവനാസ്തികരുടെ ലൈംഗിക സ്വാതന്ത്ര്യകാംക്ഷയുടെ സ്‌മൃതിലഹരിയിൽ ഉന്മത്തനായി, സ്വഹീഹുൽ ബുഖാരിയുടെ ഇംഗ്ലീഷ് വിവർത്തനം വായിച്ചപ്പോൾ സംഭവിച്ച ഭാവനാവിലാസമാണ് ഈ കള്ള ഹദീസ്. അല്ലാതെ സ്വഹീഹുൽ ബുഖാരിയടക്കം ഒരു ഹദീസ് ഗ്രന്ഥത്തിലും ഇത്തരം, നാസ്തിക രതി വൈകൃത ചിത്രങ്ങൾ കാണാൻ ആർക്കും കഴിയില്ല. ഒരു ഹദീസ് ഗ്രന്ഥത്തിന്റെ പേരെഴുതി, എന്തെങ്കിലും നമ്പർ കുറിച്ചിട്ട്, സ്വന്തം കാമാഭിലാഷങ്ങളും സ്വന്തം ഗൃഹാന്തര കാഴ്ച്ചകളും നബിയുടെയും സ്വഹാബികളുടേയും പേരിൽ എഴുതി പിടിപ്പിക്കുക എന്ന നാസ്തിക-മിഷണറി ശൈലിയാണ് ഇവിടെയും ആവർത്തിക്കപ്പെട്ടിരിക്കുന്നത്. ഏതോ ഒരു നാസ്തിക മൃഗാനുരാഗിയുടെ രോഗാതുരമായ മനസ്സിലെ -മൃഗങ്ങളെ പോലും നാണിപ്പിക്കുന്ന- ഭോഗലാലസതയുടെ വിരൂപസൃഷ്ടി !

കളവിന്റെ ആഴവും അശ്ലീലതയുടെ തീക്ഷ്ണതയും ഈ വർഗീയവാദികളുടെ അന്തരാളങ്ങളിൽ നുരഞ്ഞു പൊന്തുന്ന വെറുപ്പിന്റെ ആധിക്യമാണ് തെളിയിക്കുന്നത്. എന്തും എങ്ങനെയും നേടിയെടുക്കുക എന്ന നാസ്തിക ധർമ്മരാഹിത്യത്തിന്റെ പ്രകടമായ ഉദാഹരണങ്ങളാണ് ഇത്തരം കള്ള കഥാ നിർമ്മാണങ്ങൾ. ഉള്ളിലെ ജീർണത തുളുമ്പി തെറിപ്പിച്ച് മറ്റുള്ളവരെയും തങ്ങളെ പോലെ മലിനമാക്കുക എന്ന ഉപജാപത്തിന്റെ ഭാഗമാണിതും. ഇത്തരക്കാരോട് ഒന്നേ പറയാനുള്ളു:

“…നിങ്ങളുടെ അരിശം കൊണ്ട് നിങ്ങള്‍ മരിച്ചുകൊള്ളൂ. തീര്‍ച്ചയായും അല്ലാഹു മനസ്സുകളിലുള്ളത് അറിയുന്നവനാകുന്നു.” (ക്വുർആൻ 3: 119)

വിമർശനം:

In Islam, sex with animals and sex with dead bodies is halal.

Book: Sunan Abu Dawood (Ifa), Chapter: 33 / Provision of Punishment, Hadith Number: 4406

4406. Narrated from Ahmad Ibn Yunus (R): Ibn Abbas (R). He said: There is no punishment for having intercourse with an animal.

There are two opinions if a woman inserts the penis of an animal (into her vagina), and if she inserts a separate penis; The most accurate is that the woman’s genitals should be washed.

MuslimSahih Muslim – Book of Menstruation – hadith # 525 – Commentary

Comment: The sex or separate male genitalia of any animal of Momina can be inserted into its female genitalia.

According to some Sunni Islamic scholars, sexual intercourse with an animal does not invalidate fasting or Hajj.

(വിമർശകരുടെ തന്നെ (ഗൂഗിൾ) പരിഭാഷ:)

ഇസ്‌ലാമിൽ മൃഗങ്ങളുമായുള്ള ലൈംഗികതയും മൃതദേഹവുമായുള്ള ലൈംഗികതയും ഹലാലാണ്.

പുസ്തകം: സുനൻ അബു ദാവൂദ് അധ്യായം: 33 / ശിക്ഷയുടെ വ്യവസ്ഥ, ഹദീസ് നമ്പർ: 4406 4406.

അഹ്മദ് ഇബ്നു യൂനുസ് (റ): ഇബ്നു അബ്ബാസ് (റ) ൽ നിന്ന് നിവേദനം. അദ്ദേഹം പറഞ്ഞു: മൃഗവുമായി ഇണചേരുന്നതിന് ശിക്ഷയില്ല. ഒരു സ്ത്രീ ഒരു മൃഗത്തിന്റെ ലിംഗം (അവളുടെ യോനിയിൽ) പ്രവേശിപ്പിക്കുകയാണെങ്കിൽ, അവൾ ഒരു ച്ഛേദിക്കപ്പെട്ട ലിംഗം പ്രവേശിപ്പിക്കുകയൊ ആണെങ്കിൽ അക്കാര്യത്തിൽ രണ്ട് അഭിപ്രായങ്ങളുണ്ട്; സ്ത്രീയുടെ ജനനേന്ദ്രിയങ്ങൾ കഴുകണം എന്നതാണ് ഏറ്റവും കൃത്യമായത്.

സഹിഹ് മുസ്ലിം – ഹദീസ് # 525 – വ്യാഖ്യാനം അഭിപ്രായം: ഏതെങ്കിലും മൃഗത്തിന്റെ ലിംഗഭേദം അല്ലെങ്കിൽ പ്രത്യേക പുരുഷ ജനനേന്ദ്രിയം അതിന്റെ സ്ത്രീ ജനനേന്ദ്രിയത്തിൽ ചേർക്കാവുന്നതാണ്.

ചില സുന്നി ഇസ്ലാമിക പണ്ഡിതരുടെ അഭിപ്രായത്തിൽ, മൃഗവുമായുള്ള ലൈംഗികബന്ധം നോമ്പിനെയോ ഹജ്ജിനെയോ അസാധുവാക്കില്ല.

മറുപടി:

1. ഇസ്‌ലാമിലെ പ്രമാണങ്ങൾ ക്വുർആനും സ്വഹീഹായ ഹദീസുകളുമാണ്. അല്ലാതെ പണ്ഡിതാഭിപ്രായങ്ങളല്ല. പണ്ഡിത വ്യാഖ്യാനങ്ങൾ പ്രമാണങ്ങളെ മനസ്സിലാക്കി തരുന്ന ഗൈഡൻസുകളാണ്. സുനനു അബൂദാവൂദ് അടക്കം ഹദീസ് ഗ്രന്ഥങ്ങളിൽ ഹദീസുകൾക്ക് പുറമെ ഫിക്ഹ് (കർമ്മശാസ്ത്രമുണ്ട്), ക്വുർആൻ വ്യാഖ്യാനമുണ്ട്, പണ്ഡിതാഭിപ്രായങ്ങളുണ്ട്, ഭാഷാ ചർച്ചകൾ… എന്നിങ്ങനെ പലതുമുണ്ട്. അങ്ങനെ പല വിവരങ്ങളും ഉൾകൊള്ളുന്ന ഒരു ഗ്രന്ഥമാണത്. അതിലെ ഹദീസുകളാണ് മുസ്‌ലിംകൾ പ്രമാണമായി കാണുന്നത്.

( https://www.snehasamvadam.org/ഇസ്‌ലാമിലെ-രണ്ട്-അടിസ്ഥാ/ )

പ്രവാചകന്റെ(സ) വാക്കുകളുടേയും പ്രവർത്തനങ്ങളുടേയും ക്രോഡീകരണമാണ് ഹദീസ്. (നുസ്ഹത്തുന്നദ്ർ: 1:36, മുഖദ്ദിമ ഫീ ഉസൂലുൽ ഹദീസ്: 1:33, അൽഫദ്‌ലുൽ മുബീൻ അലാ അക്ദി ജൗഹരി സ്സമീൻ: 61)

ഇവിടെ, ഇസ്‌ലാം മൃഗരതിക്ക് അനുവാദം നൽകുന്നു എന്നതിന് തെളിവായി വിമർശകർ എടുത്തുദ്ധരിച്ചിരിക്കുന്നത് ക്വുർആനൊ, സ്വഹീഹായ ഹദീസുകളോ അല്ല. പണ്ഡിതാഭിപ്രായങ്ങളാണ്. ഇത്തരം പണ്ഡിതാഭിപ്രായങ്ങൾ ഇസ്‌ലാമിൽ പ്രമാണങ്ങളല്ല. അവ ഫിക്ഹ് (കർമ്മശാസ്ത്രം) ആകുന്നു. ഫിക്ഹ് എന്നത് ഒരു പഠനശാഖയാണ് എന്ന് ഏവർക്കുമറിയാം. അതിൽ പ്രമാണങ്ങളോട് യോജിക്കുന്നവ മുസ്‌ലിംകൾ സ്വീകരിക്കുകയും പ്രമാണങ്ങളോട് യോജിക്കാത്തവ തിരസ്കരിക്കുകയും ചെയ്യും. ഹദീസുകൾക്ക് പുറമെ, ഇത്തരത്തിലുള്ള ഫിക്‌ഹും ഹദീസ് ഗ്രന്ഥങ്ങളിലും അവക്ക് പണ്ഡിതന്മാർ രചിച്ച വിശദീകരണങ്ങളിലുമുണ്ട്. ആ ഫിക്ഹുകൾ മുസ്‌ലിംകളുടെ അടുക്കലൊ ഇസ്‌ലാമിലോ അലംഘനീയമായ പ്രമാണങ്ങൾ അല്ല. (ഉലൂമുൽ ഹദീസ്: ഇബ്നു സ്വലാഹ്: 22, 23)

2. ഇസ്‌ലാമിക പ്രമാണങ്ങൾ മൃഗരതിയെ എങ്ങനെ കാണുന്നു എന്ന് തിരിച്ചറിയാൻ ക്വുർആനിലൂടെയും സ്വഹീഹായ ഹദീസുകളിലൂടെയും ഒന്ന് കണ്ണോടിച്ചാൽ പോരെ ?! ഫിക്ഹിന്റെ അന്തരാളങ്ങളിൽ ഊളിയിടണമെന്നുണ്ടോ ?!!

ക്വുർആൻ പറയുന്നു: “തങ്ങളുടെ ലൈംഗികവിശുദ്ധി കാത്തുസൂക്ഷിക്കുന്നവരുമാണ്. തങ്ങളുടെ ഭാര്യമാരുമായോ, തങ്ങളുടെ അധീനത്തിലുള്ള അടിമസ്ത്രീകളുമായോ ഉള്ള ബന്ധം ഒഴികെ. അപ്പോള്‍ അവര്‍ ആക്ഷേപാര്‍ഹരല്ല. എന്നാല്‍ അതിന്നപ്പുറം ആരെങ്കിലും ആഗ്രഹിക്കുന്ന പക്ഷം അവര്‍ തന്നെയാണ് അതിക്രമകാരികള്‍.” (ക്വുർആൻ: 23: 5-7)

ക്വുർആനിൽ ഇത്ര വ്യക്തമായി പറഞ്ഞിരിക്കെ പിന്നെ മൃഗരതിയെ ഇസ്‌ലാമിന്റെ പേരിൽ അവതരിപ്പിക്കുന്നതിൽ എന്ത് അർത്ഥമാണുള്ളത് ?!

നബി (സ) പറഞ്ഞു: لعن اللهُ مَن وقع على بهيمةٍ لعن اللهُ من عمِل عملَ قومِ لوطٍ “മൃഗങ്ങളുമായി രതിയിൽ ഏർപ്പെട്ടവനെ ദൈവം ശപിച്ചിരിക്കുന്നു. ലൂത്തിന്റെ ജനതയുടെ പ്രവർത്തനത്തിൽ (സ്വവർഗരതി) ഏർപ്പെട്ടവനെ അല്ലാഹു ശപിച്ചിരിക്കുന്നു.” (മുസ്നദ് അഹ്മദ്: 2816, ഇബ്നുഹിബ്ബാൻ: 4417, ഹാകിം: 8052 )

ملعونٌ مَنْ وقعَ على بهيمَةٍ ، ملعونٌ مَنْ عمِلَ بعمَلِ قومِ لوطٍ

“മൃഗങ്ങളുമായി രതിയിൽ ഏർപ്പെട്ടവൻ ശപിക്കപ്പെട്ടിരിക്കുന്നു. ലൂത്തിന്റെ ജനതയുടെ പ്രവർത്തനത്തിൽ (സ്വവർഗരതി) ഏർപ്പെട്ടവൻ ശപിക്കപ്പെട്ടിരിക്കുന്നു.” (മുസ്നദു അഹ്മദ്: 1875, ഇബ്നുഹിബ്ബാൻ: 4417, ത്വബ്റാനി: 11546 )

3. പ്രവാചകാനുചരന്മാരുടെ കാലഘട്ടത്തിലെ അവസാന സന്ധിയിൽ ഇസ്‌ലാമിന്റെ വളർച്ച ദ്രുതഗതിയിലാവുകയും ലക്ഷോപലക്ഷങ്ങൾ ഇസ്‌ലാമിലേക്ക് കടന്നുവരികയും ചെയ്യാൻ തുടങ്ങിയപ്പോൾ ദുർബല വിശ്വാസികളും കപട വിശ്വാസികളും ഇസ്‌ലാമിക സമൂഹത്തിൽ സ്ഥാനം പിടിച്ചു. വളർച്ച പ്രാപിച്ച മറ്റേത് സമുദായങ്ങളിലേതുമെന്ന പോലെ മുസ്‌ലിംകൾക്കിടയിലും ജീർണതകളും അധാർമികതകളും വർധിച്ചു. മദ്യം, വ്യഭിചാരം, കളവ്, മോഷണം, കൊള്ള, കൊല തുടങ്ങിയ സാമൂഹിക- വൈയക്തിക തിന്മകൾ എല്ലാ സമൂഹങ്ങളിലും ഇടതടവില്ലാതെ നടക്കുന്നുണ്ട്. മുസ്‌ലിം സമൂഹത്തിൽ ദുർബല വിശ്വാസികളും കപട വിശ്വാസികളും സ്ഥാനം പിടിച്ചതോടെ അവരിലും ഇത്തരം ദൂഷ്യങ്ങൾ പെരുകുക സ്വഭാവികം മാത്രം. അതുകൊണ്ട് തന്നെ ഈ സാമൂഹിക ഭൂമികയെ പരിഗണിച്ച് കർമ്മശാസ്ത്ര ചർച്ചകളെ വാർത്തെടുക്കാൻ കർമശാസ്ത്ര പണ്ഡിതർ നിർബന്ധിതരായി. അല്ലാതെ സാമൂഹിക യഥാർത്ഥ്യങ്ങളെ അവഗണിച്ച് എങ്ങനെ ഫിക്ഹ് (കർമ്മശാസ്ത്ര) ചർച്ച ചെയ്യും? അധാർമിക പ്രവണതകളെയും ആ പ്രവണതകളുടെ അനന്തരഫലമായുണ്ടാകുന്ന സാമൂഹിക- കുടുംബ- വൈയക്തിക പ്രശ്നങ്ങളെയുമെല്ലാം കർമ്മശാസ്ത്ര പണ്ഡിതർ ചർച്ച ചെയ്തിട്ടുണ്ട്. അതിൽ ഒരു വിഷയവും അറപ്പു കൊണ്ട് മാറ്റി വെച്ചിട്ടില്ല. ഒരു കൈപ്പുറ്റ യാഥാർത്ഥ്യത്തെയും അവഗണിച്ച് ഫിക്ഹിനെ (കർമ്മശാസ്ത്ര) വാർത്തെടുക്കാൻ ശ്രമിച്ചിട്ടുമില്ല, ശ്രമിക്കാനും പാടില്ല. സ്വാഭാവികമായും നിഷിദ്ധമായ രതി വൈകൃതങ്ങൾ സമൂഹത്തിൽ നടക്കാനുള്ള സാധ്യത മുന്നിൽ കണ്ട് അത്തരം പ്രശ്നങ്ങളിലെ കർമ്മശാസ്ത്ര വിധികളെ പറ്റി ചർച്ച ചെയ്തിട്ടുണ്ട്. അതിനർത്ഥം അത്തരം അവിഹിതങ്ങളെയൊ രതി വൈകൃതങ്ങളെയൊ ഇസ്‌ലാമിക കർമ്മശാസ്ത്രമോ, കർമ്മശാസ്ത്ര ചർച്ചകളുടെ അവലംബമായ ഇസ്‌ലാമിക പ്രമാണങ്ങളൊ അംഗീകരിക്കുന്നു എന്നല്ല. ഒരു രാജ്യത്ത് കൊലപാതകവും കലാപവും ബലാത്സംഗവുമെല്ലാം നടന്നാലുള്ള അനന്തര നടപടികളെ സംബന്ധിച്ച നിയമങ്ങൾ ഉണ്ടെന്നതും ഭരണഘടന അത് ചർച്ച ചെയ്യുന്നുണ്ട് എന്നതും ആ രാജ്യവും ഭരണഘടനയും ഈ ദ്രോഹങ്ങൾക്ക് അംഗീകാരം നൽകുന്നു എന്നതിന് തെളിവാണോ ? അല്ലല്ലൊ. ഈ വസ്തുത മനസ്സിലാക്കിയതിന് ശേഷമാവണം ഹദീസ് ഗ്രന്ഥങ്ങളിലെ കർമ്മശാസ്ത്ര ചർച്ചയെ നാം സമീപിക്കേണ്ടത്.

“In Islam, sex with animals and sex with dead bodies is halal… ” “ഇസ്‌ലാമിൽ മൃഗങ്ങളുമായുള്ള ലൈംഗികതയും മൃതദേഹവുമായുള്ള ലൈംഗികതയും ഹലാലാണ്…” എന്ന പെരും നുണ പറഞ്ഞു കൊണ്ടാണ് വിമർശകർ പ്രചരിപ്പിക്കുന്ന ഈ വിമർശനം ആരംഭിക്കുന്നത്. തുടർന്ന് അതിന് തെളിവായി ഉദ്ധരിക്കുന്നത് ഇതാണ്:

Book: Sunan Abu Dawood (Ifa), Chapter: 33 / Provision of Punishment, Hadith Number: 4406

4406. Narrated from Ahmad Ibn Yunus (R): Ibn Abbas (R). He said: There is no punishment for having intercourse with an animal.

“പുസ്തകം: സുനൻ അബു ദാവൂദ് അധ്യായം: 33 / ശിക്ഷയുടെ വ്യവസ്ഥ, ഹദീസ് നമ്പർ: 4406 4406.

അഹ്‌മദ്‌ ഇബ്നു യൂനുസ് (റ): ഇബ്നു അബ്ബാസ് (റ) ൽ നിന്ന് നിവേദനം. അദ്ദേഹം പറഞ്ഞു: മൃഗവുമായി ഇണചേരുന്നതിന് ശിക്ഷയില്ല.”

സുനനു അബൂദാവൂദിൽ നിന്ന് ദുർവ്യാഖ്യാനിക്കാൻ സാധ്യത കാണുന്ന ഒരു വാചകമങ്ങെടുത്ത് ചേർത്തു! അതിന്റെ മുന്നും പിന്നുമൊന്നുമില്ല.!! അധ്യായം മുഴുവനായും ഇവിടെ ഉദ്ധരിച്ചിട്ടാവാം നമ്മുടെ ചർച്ച:

30 – باب فيمن أتى بهيمةً

4464 – حدَّثنا عبدُ الله بنُ محمَّد النُّفيليُّ، حدَّثنا عبدُ العزيز بنُ محمدٍ، حدَّثني عَمرُو بنُ أبي عَمرو، عن عِكرمة

عن ابنِ عباس، قال: قال رسولُ الله – صلَّى الله عليه وسلم -: “مَن أتى بهيمةً، فاقتلوهُ واقتلوها مَعَهم” قال: قُلتُ له: ما شأنُ البهيمةِ؟ قال: ما أُراه قال ذلك إلا أنه كَرِهَ أن يُؤكَلَ لحمُها، وقد عُمِلَ بها ذلك العملُ…

4465 – حدَّثنا أحمدُ بنُ يونسَ، أن شريكاً وأبا الأحوصِ وأبا بكر بنَ عياشٍ حدَّثوهم، عن عاصِمٍ، عن أبي رزينٍ

عن ابنِ عباس، قال: ليس على الذي يأتي البهيمة حدٌّ.

قال أبو داود: وكذا قال عطاءٌ، وقال الحكم: أرى أن يُجلَدَ ولا يُبْلَغَ به الحدّ، وقال الحسنُ: هو بمنزلةِ الزَّاني

“അധ്യായം: മൃഗവുമായി രതിയിൽ ഏർപ്പെട്ടവന് എന്ത് ശിക്ഷാ നടപടിയാണ് സ്വീകരിക്കേണ്ടത്.” എന്ന അധ്യായത്തിന്റെ നാമം തന്നെ വായിച്ചാൽ കാര്യം സ്പഷ്ടമാണ്. മൃഗരതി അനുവദനീയമാണൊ അല്ലെ എന്നല്ല അധ്യായം. മൃഗരതി എന്ന വൻപാപം ഒരാൾ ചെയ്താൽ അയാളുടെ മേൽ ഒരു ഇസ്‌ലാമിക ഭരണകൂടം ഭൗതികമായ ശിക്ഷാ നടപടികൾ എന്തെങ്കിലും നടപ്പാക്കണൊ വേണ്ടെ എന്നത് മാത്രമാണ് ചർച്ച. മൃഗരതി എന്ന വൻപാപത്താൽ പരലോകത്ത് നൽകപ്പെടുന്ന ദൈവികമായ ശിക്ഷയൊ, ഭൗതികമായ പ്രായശ്ചിത്തമൊ ‘തഅ്സീറൊ’ മാത്രമാണ് അയാളുടെ മേൽ ഉള്ളു, വധശിക്ഷയൊ സമാനമായ ഭൗതിക ശിക്ഷാ നടപടികൾ ഇല്ല എന്ന് ചില പണ്ഡിതന്മാർ അഭിപ്രായപ്പെട്ടിട്ടുണ്ട്. നബിയുടെ (സ) കാലഘട്ടത്തിൽ മൃഗരതിയിൽ ആരും ഏർപ്പെടാത്തതിനാൽ ശിക്ഷാ നടപടികൾ ഒന്നും നടപ്പാക്കിയതായി ഇസ്‌ലാമിക പ്രമാണങ്ങളിൽ രേഖപ്പെടുത്തപ്പെട്ടിട്ടില്ല എന്നതാണ് ഈ ഒറ്റപ്പെട്ട അഭിപ്രായത്തിന് കാരണം. ഹദ്ദ് അഥവാ ശിക്ഷക്ക് പകരം കഫ്ഫാറത്ത് അഥവാ പ്രായശ്ചിത്തം ആണ് വിധിയെന്ന് അവർ വാദിക്കുന്നു.

മൃഗരതിക്ക് ഭൗതിക ശിക്ഷാ നടപടികൾ ഒന്നുമില്ല എന്ന ഇബ്നു അബ്ബാസിന്റെ അഭിപ്രായം സുനനു അബൂദാവൂദിൽ നിന്നും കോട്ടിമാട്ടിയ വിമർശകർ ആ വാചകത്തിന്റെ തൊട്ടു മുകളിലും താഴെയുമുള്ള വാചകങ്ങൾ കട്ടു മറച്ചു.

مَن أتى بهيمةً، فاقتلوهُ واقتلوها مَعَهم

“മൃഗരതിയിൽ ഏർപ്പെടുന്നവന് വധശിക്ഷയുണ്ട്” എന്ന ഇബ്നു അബ്ബാസിന്റെ(റ) തന്നെ അഭിപ്രായം വിമർശകർ മുക്കി.

രണ്ടാമതായി വിമർശകർ മുക്കിയ വാചകമിതാണ്: وقال الحكم: أرى أن يُجلَدَ ولا يُبْلَغَ به الحدّ، وقال الحسنُ: هو بمنزلةِ الزَّاني

“ഹകം പറഞ്ഞു: മൃഗരതിയിൽ ഏർപ്പെട്ടവന് മർദ്ദന ശിക്ഷ നടപ്പാക്കണം. അങ്ങേയറ്റം ശിഷക്ക് വിധേയനാക്കണം. ഹസൻ പറഞ്ഞു: അവൻ വ്യഭിചാരിക്ക് സമാനമായതിനാൽ വ്യഭിചാരത്തിനുള്ള ശിക്ഷ അവനിൽ നടപ്പാക്കണം.”

ഇതൊന്നും കാണാത്ത മട്ടിൽ ഭൗതിക ശിക്ഷാവിധി ഉണ്ടെന്നും ഇല്ലെന്നും ഒരു പണ്ഡിതൻ തന്നെ അഭിപ്രായപ്പെട്ട രണ്ട് അഭിപ്രായത്തിൽ ശിക്ഷാവിധി ഇല്ലെന്ന വാചകം മാത്രമെടുത്തു വെച്ച്, ഫ്രെയ്‌മിട്ടു. ചർച്ച ശിക്ഷാവിധി ഇല്ല എന്ന ആശയത്തിൽ നിന്നും അനുവദനീയം എന്ന ആശയത്തിലേക്ക് തള്ളി മാറ്റി… എങ്ങനെയെങ്കിലും തങ്ങളുടെ മനസ്സുകളിലെ ലൈംഗിക വൈകൃതങ്ങൾ മറ്റുള്ളവരിൽ പ്രതിഫലിപ്പിച്ച് കാണിക്കുന്ന ഈ മ്ലേച്ഛത എത്ര ഭീകരം !!

ഇസ്‌ലാമിൽ മൃഗരതി അനുവദനീയമാണ് എന്ന് കെട്ടിച്ചമക്കാൻ വിമർശകർ ഉദ്ധരിക്കുന്ന മറ്റൊരു വാചകമിതാണ്:

There are two opinions if a woman inserts the penis of an animal (into her vagina), and if she inserts a separate penis; The most accurate is that the woman’s genitals should be washed.

MuslimSahih Muslim – Book of Menstruation – hadith # 525 – Commentary

According to some Sunni Islamic scholars, sexual intercourse with an animal does not invalidate fasting or Hajj.

“ഒരു സ്ത്രീ ഒരു മൃഗത്തിന്റെ ലിംഗം (അവളുടെ യോനിയിൽ) പ്രവേശിപ്പിക്കുകയാണെങ്കിൽ, അവൾ ഒരു ച്ഛേദിക്കപ്പെട്ട ലിംഗം പ്രവേശിപ്പിക്കുകയൊ ആണെങ്കിൽ അക്കാര്യത്തിൽ രണ്ട് അഭിപ്രായങ്ങളുണ്ട്; സ്ത്രീയുടെ ജനനേന്ദ്രിയങ്ങൾ കഴുകണം എന്നതാണ് ഏറ്റവും കൃത്യമായത്.

സഹിഹ് മുസ്ലിം – ഹദീസ് # 525 – വ്യാഖ്യാനം അഭിപ്രായം: ഏതെങ്കിലും മൃഗത്തിന്റെ ലിംഗഭേദം അല്ലെങ്കിൽ പ്രത്യേക പുരുഷ ജനനേന്ദ്രിയം അതിന്റെ സ്ത്രീ ജനനേന്ദ്രിയത്തിൽ ചേർക്കാവുന്നതാണ്.

ചില സുന്നി ഇസ്ലാമിക പണ്ഡിതരുടെ അഭിപ്രായത്തിൽ, മൃഗവുമായുള്ള ലൈംഗികബന്ധം നോമ്പിനെയോ ഹജ്ജിനെയോ അസാധുവാക്കില്ല.”

മുകളിൽ ഉദ്ധരിക്കപ്പെട്ടത് ക്വുർആനൊ നബിയുടെ ഹദീസൊ അല്ല. അറബിയിൽ എഴുതപ്പെടുന്നതെല്ലാം ഇസ്‌ലാമാണെന്ന തെറ്റിദ്ധാരണ നാം മാറ്റി വെക്കുക.

ക്രിസ്താബ്ദം 1277 ൽ അഥവാ ഹിജ്റാബ്ദം 676 ൽ നിര്യാതനായ ഇമാം നവവി എന്ന ശാഫിഈ കർമ്മശാസ്ത്ര പണ്ഡിതന്റെ വാചകമാണ് മുകളിലെ ആദ്യത്തെ പാരഗ്രാഫ്. പ്രവാചക വിയോഗത്തിന് ശേഷം ആറ് നൂറ്റാണ്ട് കഴിഞ്ഞ് വന്ന ഒരു പണ്ഡിതൻ, ഹദീസിന്റെ വ്യാഖ്യാനത്തിൽ കുറിച്ചിട്ട തന്റെ കർമ്മശാസ്ത്ര സംബന്ധമായ അഭിപ്രായങ്ങങ്ങളെ ഇസ്‌ലാമിന്റെ പ്രമാണമൊ വിശുദ്ധ ഗ്രന്ഥമൊ ആയി അവതരിപ്പിക്കുന്നതു തന്നെ അബദ്ധമാണ്.

മാത്രമല്ല, ഇമാം നവവി കുറിച്ചിട്ട ചർച്ചയും ശ്രദ്ധ അർഹിക്കുന്നുണ്ട്. മൃഗരതി എന്ന വൻപാപം അനുവദനീയമൊ നിഷിദ്ധമൊ എന്നതല്ല അദ്ദേഹവും ചർച്ച ചെയ്യുന്നത്!! അത്തരമൊരു മ്ലേച്ഛത ഒരാൾ ചെയ്താൽ അതിനെ തുടർന്നു വരുന്ന മറ്റു കർമ്മശാസ്ത്ര വിധികളെ പറ്റി മാത്രമാണ് ഇവിടെയും ചർച്ച.

മൃഗരതിയെ സംബന്ധിച്ച ധാർമ്മിക വിധി എന്താണെന്ന് വളരെ വ്യക്തമായി ഇമാം നവവി തന്നെ രേഖപ്പെടുത്തുന്നത് കാണുക:

ﻭﻳﺤﺮﻡ اﺗﻴﺎﻥ اﻟﺒﻬﻴﻤﺔ ﻟﻘﻮﻟﻪ ﻋﺰ ﻭﺟﻞ (ﻭاﻟﺬﻳﻦ ﻫﻢ ﻟﻔﺮﻭﺟﻬﻢ ﺣﺎﻓﻈﻮﻥ اﻻ ﻋﻠﻰ ﺃﺯﻭاﺟﻬﻢ ﺃﻭ ﻣﺎ ﻣﻠﻜﺖ ﺃﻳﻤﺎﻧﻬﻢ ﻓﺈﻧﻬﻢ ﻏﻴﺮ ﻣﻠﻮﻣﻴﻦ) ﻓﺈﻥ ﺃﺗﻰ اﻟﺒﻬﻴﻤﺔ ﻭﻫﻮ ﻣﻤﻦ ﻳﺠﺐ ﻋﻠﻴﻪ ﺣﺪ اﻟﺰﻧﺎ ﻓﻔﻴﻪ ﺛﻼﺛﺔ ﺃﻗﻮاﻝ…

“മൃഗങ്ങളുമായുള്ള രതി നിഷിദ്ധമാണ്. കാരണം, അല്ലാഹു ഇപ്രകാരം പറഞ്ഞിരിക്കുന്നു: ‘തങ്ങളുടെ ഗുഹ്യാവയവങ്ങളെ കാത്തുസൂക്ഷിക്കുന്നവരുമത്രെ അവര്‍. തങ്ങളുടെ ഭാര്യമാരുമായോ, തങ്ങളുടെ അധീനത്തിലുള്ള അടിമസ്ത്രീകളുമായോ ഉള്ള ബന്ധം ഒഴികെ. അപ്പോള്‍ അവര്‍ ആക്ഷേപാര്‍ഹരല്ല. എന്നാല്‍ അതിന്നപ്പുറം ആരെങ്കിലും ആഗ്രഹിക്കുന്ന പക്ഷം അവര്‍ തന്നെയാണ് അതിക്രമകാരികള്‍.’ (ക്വുർആൻ: 23:5-7). ഇനി ആരെങ്കിലും മൃഗങ്ങളെ സമീപിച്ചാൽ അയാളിൽ നടപ്പാക്കേണ്ട ശിക്ഷാവിധിയിൽ മൂന്ന് അഭിപ്രായങ്ങൾ പണ്ഡിതന്മാർക്കിടയിലുണ്ട്…” (അൽ മജ്മൂഉ ശർഹുൽ മുഹദ്ദബ്: 20:30)

തുടർന്ന് ഇമാം നവവി മൂന്ന് അഭിപ്രായങ്ങൾ വിശദീകരിച്ചു.

(ﺃﺣﺪﻫﺎ) ﺃﻧﻪ ﻳﺠﺐ ﻋﻠﻴﻪ اﻟﻘﺘﻞ

അതിൽ പ്രഥമവും പ്രധാനവുമായ അഭിപ്രായം അയാൾക്ക് വധശിക്ഷ നൽകപ്പെടണം എന്നതാണ്.

രണ്ടാമത്തെ അഭിപ്രായം മൃഗരതിക്കുള്ള ശിക്ഷ വ്യഭിചാരത്തിനുള്ള ശിക്ഷ പോലെയാണ് എന്നാണ്. വിവാഹിതൻ വ്യഭിചരിച്ചാൽ വധശിക്ഷയും അവിവാഹിതനാണെങ്കിൽ മർദ്ദനവുമാണ് ശിക്ഷ. മൂന്നാമത്തെ അഭിപ്രായം അയാൾക്ക് ഭൗതികമായ ശിക്ഷാവിധികളൊന്നും ഭരണാധികാരി നടപ്പാക്കേണ്ടതില്ല എന്നതാണ്. (അൽ മജ്‌മൂഉ ശർഹുൽ മുഹദ്ദബ്: 20:30)

അത്തരമൊരു രതി വൈകൃതത്തിൽ ഏർപ്പെട്ടാൽ പിന്നീടു മറ്റു കർമ്മങ്ങൾ നിർവ്വഹിക്കാൻ അംഗശുദ്ധിയും കുളിയും നിർബന്ധമാണൊ എന്നതാണ് ഒരു ചർച്ച.

‏وَلَوْ اسْتَدْخَلَت الْمَرْأَة ذَكَرَ بَهِيمَة وَجَبَ عَلَيْهَا الْغُسْل , وَلَوْ اسْتَدْخَلَت ذَكَرًا مَقْطُوعًا فَوَجْهَانِ أَصَحّهمَا يَجِب عَلَيْهَا الْغُسْل

“ഒരു സ്ത്രീ, മൃഗത്തിന്റെ ലിംഗം തന്റെ ഗുഹ്യാവയവത്തിൽ പ്രവേശിപ്പിച്ചാൽ കുളിച്ച് ശുദ്ധി വരുത്തൽ അവൾക്ക് നിർബന്ധമാണ്. ഒരു സ്ത്രീ, ച്ഛേദിക്കപ്പെട്ട ഒരു ലിംഗം തന്റെ ഗുഹ്യാവയവത്തിൽ പ്രവേശിപ്പിച്ചാലുള്ള വിധിയിൽ രണ്ടഭിപ്രായമുണ്ട്. കുളിച്ച് ശുദ്ധി വരുത്തൽ നിർബന്ധമാണ് എന്നതാണ് ശരിയായ അഭിപ്രായം.” ഇതാണ് ഇമാം നവവിയുടെ വാചകം. ഭൗതികവാദി അത് ഇംഗ്ലീഷിലേക്ക് മാറ്റിയപ്പോൾ പരിഭാഷയിൽ വേണ്ടുവോളം വെള്ളം ചേർത്തു എന്ന് സാന്ദർഭികമായി സൂചിപ്പിക്കട്ടെ. ഭൗതികവാദിയുടെ പരിഭാഷ ശ്രദ്ധിക്കുക: There are two opinions if a woman inserts the penis of an animal (into her vagina), and if she inserts a separate penis; The most accurate is that the woman’s genitals should be washed.

അത്തരമൊരു രതി വൈകൃതത്തിൽ ഏർപ്പെട്ടാൽ മഹാപാപിയായി പരിണമിക്കുമെങ്കിലും ആ വൻപാപം മൂലം വ്രതമോ ഹജ്ജ് കർമ്മമോ അസാധുവായി പോവുമോ എന്ന ചർച്ചയും ചില കർമ്മശാസ്ത്ര പണ്ഡിതർ നടത്തിയിട്ടുണ്ടാവാം. അതാണ് ഈ വാചകത്തിന്റെ പൊരുൾ: “According to some Sunni Islamic scholars, sexual intercourse with an animal does not invalidate fasting or Hajj.” മൃഗരതിയുടെ ധാർമ്മിക വിധിയല്ല ഇവിടെയും അന്വേഷിക്കപ്പെടുന്നത്.

പൈശാചിക സ്വാധീനത്താൽ ഒരു ദുർബല നിമിഷത്തിൽ മൃഗ രതിയിൽ ഒരാൾ ഏർപ്പെട്ടു എന്ന് കരുതുക. ചിലപ്പോൾ, മൃഗരതി ലൈംഗിക അവകാശത്തിന്റെ ഭാഗമായി കാണുന്ന ഒരു എക്സ് മുസ്‌ലിമിന്റെയൊ ഫ്രീതിങ്കേഴ്സിന്റെയൊ അവകാശ വാദങ്ങളിൽ സ്വാധീനിക്കപ്പെട്ട് കൊണ്ട് അങ്ങനെയൊന്ന് ഒരാളിൽ നിന്ന് സംഭവിച്ചു എന്ന് കരുതുക. ശേഷം, അയാൾ, “മൃഗങ്ങളുമായി രതിയിൽ ഏർപ്പെട്ടവനെ ദൈവം ശപിച്ചിരിക്കുന്നു…”(മുസ്നദ് അഹ്മദ്: 2816) എന്ന നബിയുടെ (സ) ഹദീസിൽ നിന്ന് ആ പാപത്തിന്റെ ഭയാനകത മനസ്സിലാക്കുകയും ചെയ്തു. ഇത്തരമൊരു സാഹചര്യത്തിൽ തുടർന്ന് അയാൾ പാലിക്കേണ്ട ശുദ്ധി, ആരാധനാ കർമ്മങ്ങൾ എന്നിങ്ങനെയുള്ള പശ്ചാത്താപ-ആത്മീയ അനുഷ്ഠാനങ്ങളെ സംബന്ധിച്ച വിധിവിലക്കുകൾ കർമ്മശാസ്ത്രം ചർച്ച ചെയ്യേണ്ടതില്ലെ ?! അത്തരം തുടർന്നടപടികൾ ചർച്ച ചെയ്താൽ അതിനർത്ഥം ആ വൻപാപത്തെ ആ കർമ്മശാസ്ത്ര പണ്ഡിതർ അനുകൂലിക്കുന്നു എന്നാണെന്ന് വർഗീയ തിമിരം ബാധിച്ചവരല്ലാതെ വാദിക്കുകയില്ല.

ചുരുക്കത്തിൽ, മൃഗരതി നന്മയാണോ തിന്മയാണോ എന്നതല്ല ഇവിടെയൊന്നും ചർച്ച. ആ തിന്മ ഒരാൾ ചെയ്താൽ മറ്റു കർമ്മങ്ങളെ അത് എപ്രകാരം ബാധിക്കും എന്നാണ് ചർച്ച. അങ്ങനെ സംഭവിച്ചാൽ തുടർന്നുള്ള വിധികളെ സംബന്ധിച്ച ചില പണ്ഡിതരുടെ ചർച്ചകൾ “ഇസ്‌ലാമിന്റെ വിശുദ്ധ പ്രമാണങ്ങൾ” എന്ന വ്യാജേന അവതരിപ്പിക്കൽ ഭൗതികവാദികളുടെ സ്ഥിരം ഗവേഷക സംസ്കാരമാണ്.

Comment: The sex or separate male genitalia of any animal of Momina can be inserted into its female genitalia.

മൃഗരതിയുടെ ധാർമ്മിക വിധി ചർച്ച ചെയ്തു കൊണ്ട് ഇമാം നവവിയൊ മറ്റൊരു മുസ്‌ലിം പണ്ഡിതനൊ ഇത്തരമൊരു അഭിപ്രായം കുറിച്ചിട്ടിട്ടില്ല. ഇത്തരമൊരു വാചകം ഭൗതീകവാദികൾക്ക് കിട്ടിയത് “ഇമാം” ഡോകിൻസിൽ നിന്നൊ “ഇമാം” സാം ഹാരിസിൽ നിന്നൊ ഒക്കെയാവാനാണ് സാധ്യത.!!

വിമർശനം:

അടിമ സ്ത്രീകളെ വിവസ്ത്രരാക്കാനും ഇഷ്ടാനുസാരം ശരീരാവയവങ്ങൾ സ്പർശിക്കുവാനും ഇസ്‌ലാം അനുവദിച്ചുവെന്ന് ഹദീസുകളിൽ ഇല്ലേ ?

മറുപടി:

അടിമ സ്ത്രീകളെ വേശ്യാവൃത്തിക്കും അശ്ലീലതകൾക്കും വിധേയരാക്കിയിരുന്ന ഒരു കാലഘട്ടത്തിൽ ലൈംഗിക ശുദ്ധി കാത്തുസൂക്ഷിക്കാനുള്ള അവരുടെ അവകാശത്തിനായി ഘോരമായി ശബ്ദിക്കുകയും നിയമങ്ങൾ ആവിഷ്കരിക്കുകയും ചെയ്ത മതമാണ് പരിശുദ്ധ ഇസ്‌ലാം.

“നിങ്ങളുടെ അടിമസ്ത്രീകള്‍ ചാരിത്രശുദ്ധിയോടെ ജീവിക്കാന്‍ അഗ്രഹിക്കുന്നുണ്ടെങ്കില്‍ ഐഹികജീവിതത്തിന്‍റെ വിഭവം ആഗ്രഹിച്ചു കൊണ്ട് നിങ്ങള്‍ അവരെ വേശ്യാവൃത്തിക്ക് നിര്‍ബന്ധിക്കരുത്‌…” (ഖുർആൻ: 24:33)

അന്യ സ്ത്രീകളെ ഇഷ്ടാനുസാരം വിവസ്ത്രരാക്കാനും ശരീരാവയവങ്ങൾ സ്പർശിക്കുവാനും പോയിട്ട് ഒന്ന് തൊടുന്നത് പോലും ഇസ്‌ലാം വിലക്കിയിട്ടുണ്ട്.

لَأنْ يُطعَنَ في رأسِ رجلٍ بِمِخْيَطٍ من حديدٍ خيرٌ من أن يمَسَّ امرأةً لا تَحِلُّ له

“നിന്റെ ശിരസ്സിൽ ഒരു ഇരുമ്പിന്റെ ആണികൊണ്ട് കുത്തി തറക്കുന്നതാണ് നിനക്ക് അനുവദനീയമല്ലാത്ത ഒരു സ്ത്രീയെ സ്പർശിക്കുന്നതിനേക്കാൾ നിനക്ക് നല്ലത്.” (ത്വബ്റാനി: 487, മുസ്നദു റുയാനി: 1283) എന്നാണ് മുഹമ്മദ് നബി (സ) മുസ്‌ലിംകളെ പഠിപ്പിച്ചത്.

ﻗَﺎﻝَ ﺭَﺳُﻮﻝُ اﻟﻠَّﻪِ ﺻَﻠَّﻰ اﻟﻠﻪُ ﻋَﻠَﻴْﻪِ ﻭَﺳَﻠَّﻢَ: ﻻَ ﺗَﺤْﻤِﻠُﻮا اﻟﻨِّﺴَﺎءَ ﻋَﻠَﻰ ﻣَﺎ ﻳَﻜْﺮَﻫْﻦَ

“സ്ത്രീകളെ അവർ വെറുക്കുന്നത് ചെയ്യാൻ നിങ്ങൾ നിർബന്ധിക്കരുത്.” (മുസ്വന്നഫ് അബ്ദുർ റസാഖ്: 10320) എന്നും മുഹമ്മദ് നബി (സ) പഠിപ്പിച്ചിട്ടുണ്ട്.

ഇക്കാര്യങ്ങളിലെല്ലാം സ്വതന്ത്ര സ്ത്രീകളും അടിമ സ്ത്രീകളും സമമാണ്.

“സ്വതന്ത്ര സ്ത്രീകളുടെ കാര്യത്തിൽ നിഷിദ്ധമായവ അടിമ സ്ത്രീകളുടെ വിഷയത്തിലും നിഷിദ്ധമാണ്” എന്നാണ് പ്രവാചകാനുചരന്മാർ (റ) മനസ്സിലാക്കിയിട്ടുള്ളതും. (അൽ ഉമ്മ്: ഇമാം ശാഫിഈ: 5:3)

കാര്യങ്ങൾ ഇങ്ങനെയൊക്കെ ആണെങ്കിലും, ഇസ്‌ലാമിലെ പ്രമാണങ്ങളായ ക്വുർആനും സ്വഹീഹായ ഹദീസുകളും മറച്ചു പിടിച്ച് പകരം കുറേ കള്ള കഥകളും ദുർബല നിവേദനങ്ങളും സോഷ്യൽ മീഡിയകളിലൂടെ പ്രചരിപ്പിക്കുന്ന ഉദ്യമത്തിൽ വിരാജിക്കുകയാണ് ഇസ്‌ലാമോഫോബിയക്കാർ.

അടിമ സ്ത്രീകളെ വിവസ്ത്രരാക്കാനും ഇഷ്ടാനുസാരം ശരീരാവയവങ്ങൾ സ്പർശിക്കുവാനും ഇസ്‌ലാം അനുവദിച്ചുവെന്ന് വരുത്തി തീർക്കാൻ ഉദ്ധരിക്കുന്ന നിവേദനങ്ങളാകട്ടെ സാങ്കേതികമായി ഹദീസുകൾ പോലുമല്ല !! ‘അറബി കിതാബു’കളിൽ ഉള്ളതെല്ലാം ഇസ്‌ലാമാണെന്നും മുസ്‌ലിംകൾക്കിടയിൽ അറബിയിലെ വരികൾക്കെല്ലാം ആത്മീയ പ്രാധാന്യമുണ്ട് എന്നും വിശ്വസിക്കുന്ന നിലയിലേക്ക് കൂപ്പ് കുത്തി പോയോ ഇസ്‌ലാം – നാസ്തികത സംവാദങ്ങൾ?!! ഗവേഷണാത്മകതയുടെയും വൈജ്ഞാനിക ധർമ്മത്തിന്റെയും ഒരു കണിക പോലും നിങ്ങളുടെ ഇസ്‌ലാം അവലോകനത്തിൽ ബാക്കി ഇല്ലാതായി പോയോ?

പ്രതിപക്ഷ മര്യാദ അന്യംനിന്നു പോയിട്ടില്ലാത്തവർക്ക് വേണ്ടി ഇസ്‌ലാമിന്റെ രണ്ട് അടിസ്ഥാന തത്ത്വങ്ങൾ അടങ്ങുന്ന ഒരു ലേഖനം ഇവിടെ ചേർത്തു വെക്കട്ടെ:

വിമർശകർ പ്രചരിപ്പിച്ചു കൊണ്ടിരിക്കുന്ന നിവേദനങ്ങളുടെ റെഫറൻസും നിവേദക പരമ്പരകളും ആദ്യം ചർച്ചാവിധേയമാക്കാം:

പരമ്പര: 1 മുസ്വന്നഫ് അബ്ദുർറസാക്: 13208

13208 – عبد الرزاق عن بن جريج قال أخبرنى من أصدق عمن سمع عليا يسأل عن الأمة تباع أينظر إلى ساقها وعجزها وإلى بطنها قال لا بأس بذلك لا حرمة لها إنما وقفت لنساومها

അടിമ സ്ത്രീയെ വാങ്ങുമ്പോൾ അവളുടെ തണ്ടങ്കാലും അരയും നോക്കാം എന്ന് അലി (റ) അനുവാദം നൽകിയതായ നിവേദനം…

പരമ്പരയിലെ ഇബ്നു ജുറൈജ് ഉദ്ധരിക്കുന്നത് ഒരു ‘മജ്ഹൂൽ’ (അജ്ഞാതൻ) ൽ നിന്നാണ് ആ ‘മജ്ഹൂൽ’ ഉദ്ധരിക്കുന്നത് മറ്റൊരു ‘മജ്ഹൂൽ’ (അജ്ഞാതൻ) ൽ നിന്നും.!!! പരമ്പര വളരെ ദുർബലം.

പരമ്പര: 2 മുസ്വന്നഫ് അബ്ദുർറസാക്: 13206

13206 – عبد الرزاق عن بن جريج عن رجل عن بن المسيب أنه قال يحل له أن ينظر إلى كل شيء فيها ما عدا فرجها

താബിഈ പണ്ഡിതനായ (ഒരു പണ്ഡിതന്റെ അഭിപ്രായം !) സഈദിബ്നുൽ മുസ്വയ്യിബിലേക്ക് ചേർക്കപ്പെടുന്ന മറ്റൊരു നിവേദനം…

പരമ്പരയിൽ ഇബ്നു ജുറൈജ് ഉദ്ധരിക്കുന്നത് ഒരു ‘മജ്ഹൂൽ’ (അജ്ഞാതൻ) ൽ നിന്നാണ്. പരമ്പര അങ്ങേയറ്റം ദുർബലം.

പരമ്പര: 3 മുസ്വന്നഫ് അബ്ദുർറസാക്: 13207

13207 – عبد الرزاق عن الثوري عن جابر عن الشعبي قال إذا كان الرجل يبتاع الأمة فإنه ينظر إلى كلها إلا الفرج

താബിഈ പണ്ഡിതനായ (ഒരു പണ്ഡിതന്റെ അഭിപ്രായം !) ശുഅ്ബിയിലേക്ക് ചേർക്കപ്പെട്ട അഭിപ്രായം…

പരമ്പരയിലെ ജാബിർ അൽ ജഅ്ദി ശീഈയും നുണയനായി അരോപിക്കപ്പെട്ട വ്യക്തിയുമാണെന്ന് ഒട്ടനവധി ഹദീസ് വിശാരദർ വ്യക്തമാക്കിയിട്ടുണ്ട്. (തഹ്ദീബുൽ കമാൽ: മിസ്സി: 4:468)

പരമ്പര: 4 മുസ്വന്നഫ് ഇബ്നു അബീ ശൈബ: 2662 (29)

نا علي بن مسهر عن عبيد الله عن نافع عن ابن عمرأنه كان إذا أراد أن يشتري الجارية وضع يده على أليتيها أو بين فخذها وربما كشف عن ساقيها

ഇബ്നു ഉമറിലേക്ക് ചേർത്തി ഉദ്ധരിക്കപ്പെട്ട സമാനമായ ഒരു നിവേദനം…

പരമ്പരയിൽ ‘അലിയ്യുബ്നു മുസ്ഹിർ’ എന്ന റാവി അന്ധത ബാധിച്ചതിനു ശേഷം ധാരാളം ഒറ്റപ്പെട്ട, അസ്വീകാര്യമായ നിവേദനങ്ങൾ ഉദ്ധരിക്കുമായിരുന്നു എന്ന് വിമർശിക്കപ്പെട്ടിട്ടുണ്ട്. (തക്’രീബു തഹ്ദീബ്: 1:703)

പരമ്പര: 5 മുസ്വന്നഫ് അബ്ദുർറസാക്: 13200

13200 – عبد الرزاق عن عبد الله بن عمر عن نافع عن ابن عمر، ومعمر عن أيوب عن نافع عن ابن عمر، كان إذا أراد أن يشتري جارية، فراضاهم على ثمن، وضع يده على عجزها، وينظر إلى ساقيها، وقبلها، يعني بطنها

മൂന്ന് പരമ്പരകൾ ഈ നിവേദനത്തിൽ സമന്വയിപ്പിക്കപ്പെട്ടിട്ടുണ്ട്.

ഒന്നാമത്തെ പരമ്പര: عَنْ عَبْدِ اللَّهِ بْنِ عُمَرَ ، عَنْ نَافِعٍ ، عَنِ ابْنِ عُمَرَ

ഈ പരമ്പരയിൽ സ്മരിക്കപ്പെടുന്ന ഇബ്നു ഉമർ, അബ്ദുല്ലാഹിബ്നു ഉമർ എന്ന പ്രവാചക അനുചരനല്ല. അബ്ദുല്ലാഹിബ്നു ഉമർ ബിൻ ഹഫ്സ് എന്ന പിൻ തലമുറക്കാരനായ ഒരു വ്യക്തിയാണ്. (പ്രവാചകാനുചരനായ ഇബ്നു ഉമറിലേക്ക് ചേർത്തു കൊണ്ട് ഇത്തരമൊരു ആരോപണം ഉന്നയിക്കപ്പെട്ടിട്ടുണ്ട്. ആ പരമ്പരയെ സംബന്ധിച്ച ചർച്ച തുടർന്ന് വരുന്നുണ്ട് എന്ന് സാന്ദർഭികമായി സൂചിപ്പിക്കട്ടെ). ഇദ്ദേഹം ദുർബലനാണെന്നും ഇദ്ദേഹത്തിൽ നിന്ന് ഹദീസ് ഉദ്ധരിച്ചു കൂട എന്നും ഇബ്നുൽ മദീനി, യഹ്‌യൽ കത്വാൻ തുടങ്ങി പണ്ഡിതർ സൂചിപ്പിച്ചിട്ടുണ്ട്. (സിയറു അഅ്ലാമിന്നുബലാഅ്: 7: 340)

രണ്ടാമത്തെ പരമ്പര: وَمَعْمَرٍ ، عَنْ أَيُّوبَ ، عَنْ نَافِعٍ ، عَنِ ابْنِ عُمَرَ

പരമ്പരയിലെ മഅ്മർ ഇറാഖുകാരിൽ നിന്ന് ഉദ്ധരിക്കുന്നത് ‘ദഈഫ്’ ദുർബലമാണെന്ന് ഹദീസ് നിരൂപണ ശാസ്ത്ര പണ്ഡിതർ വ്യക്തമാക്കിയിട്ടുണ്ട്. ( ശർഹു ഇലലു തുർമുദി: ഇബ്നു റജബ്: 2: 774)

മഅ്മർ ‘ഇറാഖു’കാരനായ ‘അയ്യൂബ് അസ്സഖ്തിയാനി’യിൽ നിന്ന് (സിയറു അഅ്ലാമിന്നുബലാഅ്: 6:16) ഉദ്ധരിക്കുന്നതായാണ് പരമ്പര. അതിനാൽ തന്നെ ദുർബലവും.

മൂന്നാമത്തെ നിവേദനം: عَنْ مَعْمَرٍ ، عَنِ الزُّهْرِيِّ ، عَنْ سَالِمٍ ، عَنِ ابْنِ عُمَرَ مِثْلَهُ

ഈ പരമ്പരയിൽ രണ്ട് ന്യൂനതകൾ ഉണ്ട്. ഒന്നാമതായി പരമ്പരയുടെ ഉള്ളടക്കം എന്താണെന്ന് കൃത്യമായി ഉദ്ധരിക്കപ്പെട്ടിട്ടില്ല. രണ്ടാമതായി, പരമ്പരയിലെ റാവിയായ ഇമാം സുഹ്‌രി, നേരിട്ട് കേട്ടു എന്ന് വ്യക്തമായി സൂചിപ്പിക്കുന്ന പദം ഉപയോഗിക്കാത്തതിനാൽ സനദ് ‘മുദല്ലസ്’ ആവാൻ സാധ്യത നിലനിൽക്കുന്നു. (ത്വബക്കാത്തുൽ മുദല്ലിസീൻ: ഇബ്നു ഹജർ: 45). അതിനാൽ തന്നെ പരമ്പര ദുർബലമായി (ദഈഫ്) തീരുന്നു.

പരമ്പര: 6 മുസ്വന്നഫ് അബ്ദുർറസാക് : 13205 13205 – عبد الرزاق عن بن جريج عن نافع أن بن عمر كان يكشف عن ظهرها وبطنها وساقها ويضع يده على عجزها

പരമ്പരയിലെ ‘ഇബ്നു ജുറൈജ്’ നേരിട്ട് കേട്ടു എന്ന് വ്യക്തമായി സൂചിപ്പിക്കുന്ന പദം ഉപയോഗിക്കാത്തതിനാൽ പരമ്പര ദഈഫ് (ദുർബലം) ആകുന്നു.

‘ഇബ്നു ജുറൈജ്’ നേരിട്ട് കേട്ടു എന്ന് വ്യക്തമായി സൂചിപ്പിക്കാത്ത നിവേദനങ്ങൾ അങ്ങേയറ്റം ദുർബലവും വ്യർത്ഥവുമാണെന്ന് ഹദീസ് നിദാന ശാസ്ത്രം സൂചിപ്പിക്കുന്നു. (സിയറു അഅ്ലാമിന്നുബലാഅ്: 6: 328, തഹ്ദീബു തഹ്ദീബ്: 6/405 )

പരമ്പര: 7 മുസ്വന്നഫ് അബ്ദുർ റസാക് : 13198 13198 -عبد الرزاق عن بن جريج عن عطاء قال قلت له الرجل يشتري الأمة أينظر إلى ساقيها وقد حاضت أو إلى بطنها قال نعم قال عطاء كان بن عمر يضع يده بين ثدييها وينظر إلى بطنها وينظر إلى ساقيها أو يأمر به

അടിമ സ്ത്രീയെ വാങ്ങുമ്പോൾ അവളുടെ തണ്ടങ്കാലും അരയും നോക്കാം എന്ന്, ഇബ്നു ജുറൈജ് അത്വാഅ് എന്ന താബീഈ പണ്ഡിതനിലേക്ക് ചേർത്തു കൊണ്ട് ഉദ്ധരിക്കപ്പെടുന്ന നിവേദനം…

ഇബ്നു ജുറൈജ് എന്ന റാവി അത്വാഅ് ൽ നിന്ന് ഉദ്ധരിക്കുന്ന നിവേദനങ്ങൾ അങ്ങേയറ്റം ദുർബലമാണ് എന്ന് കാര്യകാരണ സഹിതം ഹദീസ് വിശാരദർ വ്യക്തമാക്കിയിട്ടുണ്ട്. (തഹ്ദീബു തഹ്ദീബ്: 6/406)

പരമ്പര: 8 മുസ്വന്നഫ് അബ്ദുർ റസാക് : 13199 13199 – أخبرنا عبد الرزاق قال أخبرنا بن جريج قال أخبرني عمرو أو أبو الزبير عن بن عمر أنه وجد تجارا مجتمعين على أمة فكشف عن بعض ساقها ووضع يده على بطنها

പരമ്പരയിലെ ഇബ്നു ജുറൈജ്, തന്നോട് നിവേദനം ഉദ്ധരിച്ചു കേൾപ്പിച്ചത് “അംറ് (അംറിബ്നു ദീനാർ) അല്ലെങ്കിൽ അബു സ്സുബൈർ ആകുന്നു” എന്നാണ്. അഥവാ ആരിൽ നിന്നാണ് ഈ നിവേദനം ഉദ്ധരിക്കുന്നതെന്ന് വ്യക്തമായി പ്രസ്ഥാവിക്കുന്നില്ല എന്നർത്ഥം. മാത്രമല്ല അബു സ്സുബൈർ എന്ന റാവിയെ സംബന്ധിച്ച് ഒരുപാട് ഭിന്നാഭിപ്രായങ്ങൾ നിലനിൽക്കുന്നു. പല പണ്ഡിതരും അദ്ദേഹം ദുർബലനാണ് എന്ന് സൂചിപ്പിച്ചിട്ടുണ്ട്. (തഹ്ദീബുത്തഹ്ദീബ്: 9: 391)

അബു സ്സുബൈർ നേരിട്ട് കേട്ടു എന്ന് വ്യക്തമായി സൂചിപ്പിക്കാത്ത നിവേദനങ്ങൾ അങ്ങേയറ്റം ദുർബലമാണെന്നും ഹദീസ് നിദാന ശാസ്ത്രം സൂചിപ്പിക്കുന്നുണ്ട്. (ത്വബകാതുൽ മുദല്ലിസീൻ: 13)

പരമ്പര: 9 സുനനുൽ കുബ്റാ: ബൈഹക്വി: 10513

أنَّ ابنَ عمرَ كان يضعُ يدَهُ بيْنَ ثَديَيها ( يعنى الجاريةَ ) وعلى عُجُزِها من فوقِ الثيابِ ويكَشفُ عن ساقِها

ഇബ്നു ഉമർ (റ) തന്റെ കൈ അടിമ പെൺകുട്ടിയുടെ മാറിടത്തിനിടയിലും അരയിലും വസ്ത്രത്തിന് മുകളിലൂടെ കൈ വെച്ച് നോക്കുമായിരുന്നു, അവളുടെ തണ്ടം കാലും വെളിവാക്കി നോക്കുകയും ചെയ്യുമായിരുന്നു. (സുനനുൽ കുബ്റാ: ബൈഹക്വി: 10513)

أخبرنا أبو الحسين بن بشران العدل ببغداد ، أنا إسماعيل بن محمد الصفار، ثنا الحسن بن علي بن عفان، ثنا ابن نمير عن عبيد الله بن عمر عن نافع عن ابن عمر

പരമ്പരയിൽ ‘ഉബൈദുല്ലാഹിബ്നു ഉമർ’ എന്ന ‘റാവി’യിൽ (നിവേദകൻ) നിന്ന് ഉദ്ധരിക്കുന്നത് ‘ഇബ്നു നുമൈർ’ എന്ന ‘റാവി’യാണ് (നിവേദകൻ). ‘ഇബ്നു നുമൈർ’ എന്ന പേര് മാത്രം ഉദ്ധരിക്കപ്പെടുമ്പോൾ സാധാരണ ഗതിയിൽ ഹദീസ് പണ്ഡിതർ ഉദ്ദേശിക്കാറുള്ളത് ‘മുഹമ്മദ് ഇബ്നു അബ്ദുല്ലാഹിബ്നു നുമൈർ’ ആണ്.

ഉദാഹരണത്തിന് ഇമാം ബൈഹക്വി തന്നെ ‘അയ്യാമുത്തശ്‌രീകി’നെ സംബന്ധിച്ച ഹദീസിനെ സംബന്ധിച്ച് വിവരിക്കവെ ഇപ്രകാരം എഴുതുകയുണ്ടായി: رواه مسلم في الصحيح عن ابن نمير. “ഈ ഹദീസ് ഇമാം മുസ്‌ലിം തന്റെ സ്വഹീഹിൽ ‘ഇബ്നു നുമൈറി’ൽ നിന്ന് ഉദ്ധരിക്കുന്നുണ്ട്… ” (മഅ്’രിഫതു സ്സുനനു വൽ ആസാർ: ബൈഹക്വി: 3: 439. നമ്പർ: 2599)

സ്വഹീഹു മുസ്‌ലിമിൽ ഈ ഹദീസ് ഉദ്ധരിച്ച ‘ഇബ്നു നുമൈർ’, ‘മുഹമ്മദിബ്നു അബ്ദുല്ലാഹിബ്നു നുമൈർ’ ആണ്.

1141 وحدثنا سريج بن يونس حدثنا هشيم أخبرنا خالد عن أبي المليح عن نبيشة الهذلي قال قال رسول الله صلى الله عليه وسلم أيام التشريق أيام أكل وشرب حدثنا محمد بن عبد الله بن نمير حدثنا إسمعيل يعني ابن علية عن خالد الحذاء حدثني أبو قلابة عن أبي المليح عن نبيشة قال خالد فلقيت أبا المليح فسألته فحدثني به فذكر عن النبي صلى الله عليه وسلم بمثل حديث هشيم وزاد فيه وذكر لله

(സ്വഹീഹു മുസ്‌ലിം: 1141)

ولد سنة نيف وستين ومائة قال البخاري : مات في شعبان أو رمضان سنة أربع وثلاثين ومائتين

മുഹമ്മദിബ്നു അബ്ദുല്ലാഹിബ്നു നുമൈറിന്റെ ജനനം ഹിജ്റാബ്ദം 161 ൽ ആണ്. അദ്ദേഹം മരണമടയുന്നത് ഹിജ്റാബ്ദം 234 ൽ ആണ്. (സിയറു അഅ്ലാമിന്നുബലാഅ്: 11: 456)

قال الهيثم بن عدي : مات سنة سبع وأربعين ومائة ‘ഉബൈദുല്ലാഹിബ്നു ഉമർ’ മരണമടയുന്നത് 167 ലാണ്. (സിയറു അഅ്ലാമിന്നുബലാഅ്: 6: 305 )

ഈ രണ്ട് ‘റാവി’മാർ തമ്മിൽ പരസ്പരം കണ്ടുമുട്ടിയിട്ടില്ല എന്നതിനാൽ തന്നെ പരമ്പര ‘മുറിഞ്ഞ’താണ് (മുൻകത്വിഅ്).

അതേസമയം, പരമ്പരയിൽ ‘ഇബ്നു നുമൈർ’ എന്നതുകൊണ്ട് ഉദ്ദേശിക്കപ്പെട്ടിരിക്കുന്നത് ‘അബ്ദുല്ലാഹിബ്നു നുമൈർ’ എന്ന റാവിയാണ് എന്ന് അഭിപ്രായപ്പെട്ട പണ്ഡിതർ പരമ്പര ‘മുൻകത്വിഅ്’ അല്ല, പരമ്പര സ്വഹീഹാണ് എന്ന് നിരീക്ഷിക്കുകയുണ്ടായി. (ഇർവാഉൽ ഗലീൽ: അൽബാനി: 6: 201)

ഈ പരമ്പരയെ സംബന്ധിച്ച വീക്ഷണ വ്യത്യാസത്തിൽ നിന്നും ചുരുങ്ങിയ പക്ഷം ഈ പരമ്പരയും വിമർശന വിധേയമാണെന്ന് മനസ്സിലാക്കാം.

ഈ പരമ്പരകളിലൂടെ എല്ലാം ഒരു ഓട്ട പ്രദക്ഷിണം നടത്തിയാൽ താഴെ പറയുന്ന കാര്യങ്ങൾ മനസ്സിലാക്കാം:

1. ഇവയൊന്നും തന്നെ ഹദീസുകൾ അല്ല. പ്രവാചകന്റെ (സ) വാക്കോ പ്രവർത്തനമൊ ഉൾക്കൊള്ളുന്നതല്ല ഇവയൊന്നും തന്നെ. ഇസ്‌ലാമിലെ പ്രമാണങ്ങൾ ക്വുർആനും സ്വഹീഹായ ഹദീസുകളുമാണ്.

2. ചില പണ്ഡിതരുടെ അഭിപ്രായങ്ങളാണ് പല നിവേദനങ്ങളും. പ്രവാചകാനുചരന്മാരുടെ തന്നെ അഭിപ്രായങ്ങൾ ഇസ്‌ലാം മതത്തിൽ സ്വമേധയാ പ്രമാണങ്ങളൊ തെളിവുകളൊ അല്ല.

3. എല്ലാ നിവേദനങ്ങളുടെ പരമ്പരകളും ദഈഫ് (ദുർബലം) ആകുന്നു. സ്വഹീഹാണെന്ന് ചില പണ്ഡിതന്മാർ അഭിപ്രായപ്പെട്ടിട്ടുള്ളത് പരമ്പര: 9 മാത്രമാണ്. ആ പരമ്പരയുടെ സ്വീകാര്യതയിൽ തന്നെ അഭിപ്രായാന്തരം നിലനിൽക്കുന്നു.

പരമ്പര: 9 തന്നെ കൂലങ്കഷമായ ഒരു വിശകലനത്തിനെടുത്താൽ വിമർശകർ വർണ്ണിക്കുന്നതു പോലെ അശ്ലീലതയൊന്നും ഉള്ളടക്കത്തിൽ ഇല്ലെന്ന് മനസ്സിലാക്കാം.

ഒന്നാമതായി, അടിമ സ്ത്രീയെ വിവസ്ത്രയാക്കുന്നതായൊ, നഗ്നത വെളിവാക്കി നിരീക്ഷിക്കുന്നതായൊ, ലൈംഗിക അവയവങ്ങൾ സ്പർശിക്കുന്നതായൊ ഒന്നും പരമ്പര 9ൽ ഇല്ല. “വസ്ത്രത്തിന് മുകളിലൂടെ”യാണ് (من فوقِ الثيابِ) ഇബ്നു ഉമർ സമീപിക്കുന്നത്, “മാറിടത്തിനിടയിൽ” (بيْنَ ثَديَيها) ആണ് കൈ വെക്കുന്നത്, മാറിടത്തിൽ അല്ല, “അരക്ക് മുകളിൽ” (على عُجُزِها) വസ്ത്രം പിടിച്ചു കൊണ്ടുമാണ് പരിശോധന. ആ അടിമ സ്ത്രീ പ്രായപൂർത്തിയും വളർച്ചയും എത്തിയ സ്ത്രീ തന്നെയാണൊ എന്ന് പരിശോധിക്കുവാൻ മാത്രമായിരുന്നു ആ പ്രവർത്തനം. അതാകട്ടെ അദ്ദേഹത്തിന്റെ വ്യക്തിപരമായ ഒരു നിലപാട് മാത്രമാണ്. ഇസ്‌ലാമിൽ അതിന് ഒരു പ്രാമാണികതയും ഇല്ല. എല്ലാത്തിനുമുപരി അവിതർക്കിതവും സ്വഹീഹുമായ നിവേദക പരമ്പരയിലൂടെ അങ്ങനെ ഒരു പ്രവർത്തനം അദ്ദേഹത്തിൽ നിന്ന് സംഭവിച്ചിട്ടുണ്ട് എന്ന് ഖണ്ഡിതമായി ഉദ്ധരിക്കപ്പെട്ടിട്ടില്ല. ‘സ്വഹീഹാ’യ പരമ്പരകൾ മാത്രമാണ് മുസ്‌ലിംകൾ സ്വീകരിക്കുക. സ്വഹീഹായ ‘ഹദീസുകൾ’ (പ്രവാചക പാഠങ്ങൾ) ആകുന്നു ഇസ്‌ലാമിലെ പ്രമാണം.

ആയിശ(റ)യുടെ വിവാഹപ്രായം: നബിനിന്ദകരോട് പറയാനുള്ളത്

ഇണകളായി ജീവിക്കുന്ന രണ്ടുപേരെ നോക്കി ഒരാൾ ഇവർ മാതൃകാ ദമ്പതികളാണെന്ന് പരിചയപെടുത്തിയാൽ അതിൽ നിന്ന് കേൾവിക്കാരൻ മനസ്സിലാക്കുന്നത് എന്തായിരിക്കും? അവർക്ക് ഒരേ പ്രായമായിരിക്കുമെന്നാണോ? പുരുഷൻ പ്രായക്കൂടുതലും സ്ത്രീ പ്രായക്കുറവുമുള്ളവരാണെന്നാണോ? സ്ത്രീ പ്രായക്കൂടുതലും പുരുഷൻ പ്രായക്കുറവുമുള്ളവരാണെന്നാണോ? ഇതൊന്നുമല്ലെന്ന് എല്ലാവർക്കുമറിയാം. മാതൃകാദമ്പതികളാണെന്ന് പരിചയപ്പെടുത്തുക സംതൃപ്തവും മാതൃകാപരവുമായ ജീവിതം നയിക്കുന്ന ഇണകളെയായിരിക്കും. സ്നേഹവും കാരുണ്യവും പരസ്പരം നൽകിക്കൊണ്ടും സ്ത്രീ പുരുഷനിലും തിരിച്ചും ശാന്തി കണ്ടെത്തുകയും ചെയ്തുകൊണ്ടുള്ള ജീവിതത്തെക്കുറിച്ചാണ് മാതൃകാ ദാമ്പത്യമെന്ന് പറയുന്നത്. മുഹമ്മദ് നബി(സ)യും ആയിഷയും നയിച്ച ദാമ്പത്യജീവിതം അവസാനനാളുവരെയുള്ള മനുഷ്യർക്കെല്ലാം മാതൃകയായ ദാമ്പത്യജീവിതമാണെന്ന് പറയുന്നത് ഈ മാനകങ്ങളുടെ അടിസ്ഥാനത്തിലാണ്. ഇണയുമൊത്തുള്ള തന്റെ ജീവിതത്തെക്കുറിച്ച് മധുരമൂറുന്ന പദങ്ങളുപയോഗിച്ച് ഇണ മരണപ്പെട്ട് പതിറ്റാണ്ടുകൾ കഴിഞ്ഞ ശേഷവും ഒരാൾ അനുസ്മരിക്കുന്നുണ്ടെങ്കിൽ അതിനർത്ഥം അയാൾ അനുഭവിച്ച ദാമ്പത്യജീവിതം അത്രത്തോളം സംതൃപ്തവും സ്നേഹ-കാരുണ്യങ്ങൾ നിറഞ്ഞതുമായിരുന്നുവെന്ന് തന്നെയാണ്. ആയിഷ (റ) മരണപ്പെടുന്നത് 67 ആമത്തെ വയസ്സിലാണ്. പ്രവാചകൻ (സ) മരണപ്പെടുമ്പോൾ അവർക്ക് വയസ്സ് 18. അര നൂറ്റാണ്ടോളം കാലം അവർ ചെയ്ത സേവനമെന്തായിരുന്നുവെന്നതിനുള്ള ഉത്തരം ഹദീഥ് ഗ്രന്ഥങ്ങൾ പരിശോധിച്ചാൽ ഏതൊരാൾക്കും ലഭിക്കും. തന്റെ മടിയിൽ കിടന്ന് ഇഹലോകവാസം വെടിഞ്ഞ അന്തിമദൂതന്റെ ജീവിതത്തിൽ താൻ കണ്ടതും കേട്ടതും അനുഭവിച്ചതുമായ കാര്യങ്ങൾ അടുത്ത തലമുറയ്ക്ക് പകർന്നുകൊടുക്കുകയെന്ന മഹാദൗത്യമാണ് ഇക്കാലമത്രയും അവർ നിർവ്വഹിച്ചത്. മുഹമ്മദ് നബിയുമൊത്തുള്ള തന്റെ ദാമ്പത്യജീവിതത്തിന്റെ സൂക്ഷ്മാംശങ്ങൾ പോലും അവസാനനാളുവരെയുള്ള മനുഷ്യർക്ക് മാതൃകയായിത്തീരുന്ന വിധത്തിൽ അവർ തന്റെയടുത്തെത്തുന്നവർക്ക് പറഞ്ഞുകൊടുത്തു. അവരിൽ നിന്ന് ഈ ഹദീഥുകളിൽ പലതും നിവേദനം ചെയ്തവരുമായുള്ള സമ്പർക്കത്തെയും അവരുപയോഗിച്ച ഭാഷയെയും സൂക്ഷ്മമായി അപഗ്രഥിച്ചാൽ ഈ സംപ്രേക്ഷണം കൂടുതലായി നടന്നത് അവരുടെ ജീവിതത്തിന്റെ അവസാനത്തെ രണ്ട് പതിറ്റാണ്ടുകളിലായിരുന്നുവെന്ന് മനസ്സിലാവും. നബിയോടോപ്പമുള്ള ആയിഷയുടെ ദാമ്പത്യം എത്രമാത്രം സംതൃപ്തവും സ്നേഹസുരഭിലവുമായിരുന്നുവെന്ന് മനസ്സിലാക്കുവാൻ ജീവിതസായാഹ്നത്തിൽ പോലും നബിജീവിതത്തെക്കുറിച്ച് പറയുമ്പോൾ അവരുടെ വാക്കുകളിലൂറുന്ന മധുരം മാത്രം പഠനവിധേയമാക്കിയാൽ മതി. ഈ സംതൃപ്തി തന്നെയാണ് മറ്റ് ഇണകളുമായുള്ള ദാമ്പത്യജീവിതത്തിന്റെ കാര്യത്തിലെന്ന പോലെത്തന്നെ നബി-ആയിഷ ദാമ്പത്യത്തിന്റെയും മാതൃക; പൂർണ്ണാർത്ഥത്തിലുള്ള സ്നേഹ-കാരുണ്യങ്ങളുടെ പാരസ്പര്യം വഴി സമാധാനപൂർണ്ണവും സംതൃപ്തവുമായ ദാമ്പത്യം എങ്ങനെ സാധിക്കാമെന്നതിന് ലോകാവസാനം വരെയുള്ള മുഴുവൻ മനുഷ്യർക്കുമുള്ള മാതൃക.

തന്നോട് ഇഷ്ടമുള്ള സന്ദർഭമാണോ ദേഷ്യമുള്ള അവസരമാണോ എന്ന് ഇണയുടെ വാക്കുകൾ മാത്രം പരിശോധിച്ച് മനസ്സിലാക്കുന്ന പ്രവാചകന്റെ(സ) പാടവത്തെക്കുറിച്ച് അനുസ്മരിക്കുന്ന ആയിഷയുടെ(റ) ഹദീഥ് (സ്വഹീഹുൽ ബുഖാരി) എത്രമേൽ മധുരമുള്ള ദാമ്പത്യത്തെയാണ് അടയാളപ്പെടുത്തുന്നതെന്ന് അതിലെ പദങ്ങൾ പരിശോധിച്ചാൽ ബോധ്യമാകും. താൻ രണ്ട് തവണ പ്രവാചകനോടൊപ്പം ഓട്ടപ്പന്തയം നടത്തിയിട്ടുണ്ടെന്നും ആദ്യ തവണ താൻ ജയിച്ചുവെന്നും രണ്ടാം തവണ താൻ പരാജയപ്പെട്ടുവെന്നും തന്നെ പരാജയപ്പെടുത്തിയപ്പോൾ ‘ഇത് മുമ്പത്തേതിനുള്ള മറുപടിയാണ്’ എന്ന് പ്രവാചകൻ (സ) പറഞ്ഞുവെന്നുമുള്ള ആയിഷയുടെ(റ) ഓർമ്മകളിൽ (അബൂദാവൂദ് സ്വഹീഹായ സനദോടെ നിവേദനം ചെയ്തത്) വെളിപ്പെടുന്നത് കളിയും തമാശയുമെല്ലാം വഴി ദാമ്പത്യത്തെ സുരഭിലമാക്കിയ നബിയെ അവർ വല്ലാതെ ഇഷ്ടപ്പെട്ടിരുന്നുവെന്ന സത്യമാണ്. ‘ഞാൻ കുടിച്ചു വെച്ച പാനപാത്രത്തിൽ നിന്ന് അതിന്റെ ബാക്കി പ്രവാചകൻ (സ) കുടിച്ചപ്പോൾ ഞാൻ എവിടെയാണോ ചുണ്ട് വെച്ചത് അവിടെത്തന്നെ ചുണ്ട് വെച്ചാണ് തിരുമേനി കുടിച്ചത്’ എന്നും ‘ഒരു എല്ലിൻ കഷ്ണത്തിലുണ്ടായിരുന്ന ഇറച്ചിക്കഷ്ണങ്ങൾ ഞാൻ തിന്ന ശേഷം അതിന്റെ ബാക്കി നബി (സ) തിന്നപ്പോൾ ഞാൻ തിന്നിടത്ത് നിന്ന് തന്നെ തിരുമേനി തീറ്റയാരംഭിച്ചു’വെന്നും ആയിഷ (റ) പറയുമ്പോൾ (നസാഈ സ്വഹീഹായ സനദോടെ നിവേദനം ചെയ്തത്) തീറ്റയിലും കുടിയിലുമെല്ലാം സ്വീകരിച്ച ചെറിയ ചെറിയ ശ്രദ്ധകൾ വഴി എത്ര സമർത്ഥമായാണ് പ്രവാചകൻ (സ) ദാമ്പത്യത്തെ ആസ്വദിച്ചതും ആസ്വദിപ്പിച്ചതുമെന്ന മഹാമാതൃക അനുവദനീയമായ ഇണജീവിതത്തിലൂടെയാകണം ലൈംഗികാസ്വാദനമാകേണ്ടതെന്ന് കരുതുന്നവർക്കെല്ലാം ലഭിക്കുന്നുണ്ട്. വീടുകളിലെത്തിയാൽ ഇണകളെ ഗാർഹികജോലികളിൽ സഹായിക്കാനാണ് പ്രവാചകൻ (സ) സമയം ചെലവഴിച്ചിരുന്നതെന്ന ആയിഷ(റ)യുടെ സാക്ഷ്യം (സ്വഹീഹുൽ ബുഖാരി) ആണ്കോയ്മയുടെ ലാഞ്ചന പോലുമേശാതെയാണ് അന്തിമപ്രവാചകൻ (സ) ജീവിച്ചതെന്നതിനുള്ള ഇണയുടെ സാക്ഷ്യത്തോടൊപ്പം തന്നെ അത്തരമൊരു ജീവിതത്തിൽ നിന്ന് ഇണകൾ എത്രത്തോളം ആശ്വാസവും സംതൃപ്‌തിയുമനുഭവിച്ചിരുന്നുവെന്നും വ്യക്തമാക്കുന്നുണ്ട്. ലൈംഗികവൃത്തികൾ അനുവദിക്കപ്പെട്ടിട്ടില്ലാത്ത വൃതാനുഷ്ഠാനത്തിന്റെ പകലുകളിൽ പോലും സ്നേഹപ്രകടനമെന്നവണ്ണം ഇണകളെ ചുംബിക്കുന്ന പ്രവാചകന്റെ ചിത്രം അനുയായികൾക്ക് നൽകുന്ന ആയിഷ (റ) (സ്വഹീഹുൽ ബുഖാരി, സ്വഹീഹ് മുസ്‌ലിം) പ്രവാചകന്റെ സ്നേഹപ്രകടനങ്ങൾ വഴി ഇണകൾക്ക് സ്നേഹവും സംതൃപ്തിയും സമാധാനവും എത്രത്തോളം ലഭിച്ചിരുന്നുവെന്ന് മനസ്സിലാക്കിക്കൊടുക്കുകയാണ് ചെയ്യുന്നത്.

ഇണകൾക്ക് ആവർത്തവമുള്ള സന്ദർഭത്തിൽ പോലും യോനീസുരതമൊഴിച്ചുള്ള ലൈംഗികചേഷ്ടകളെല്ലാം അവരുമായി പ്രവാചകൻ (സ) നടത്തുമായിരുന്നുവെന്ന് സാക്ഷീകരിക്കുന്ന ആയിഷ (റ)(സ്വഹീഹുൽ ബുഖാരി, സ്വഹീഹ് മുസ്‌ലിം) രജസ്വലകളായിരിക്കുമ്പോൾ പോലും തങ്ങൾക്ക് നബിസ്നേഹവും സഹവാസവും നിഷേധിക്കപ്പെട്ടിരുന്നില്ലെന്ന് വ്യക്തമാക്കുക മാത്രമല്ല, സ്വാഭാവികമായ ആ സ്ത്രീപ്രക്രിയ അവളുടെ വിസർജ്ജ്യസ്ഥലമൊഴികെ മറ്റ് ശരീരഭാഗങ്ങളെയൊന്നും അശുദ്ധമാക്കുന്നില്ലെന്ന് അന്ന് ജീവിച്ചിരുന്നവരും ഇന്ന് ജീവിക്കുന്നവരും നാളെ ജീവിക്കാനിരിക്കുന്നവരുമായ മുഴുവൻ സ്ത്രീ-പുരുഷന്മാരെയും ബോധ്യപ്പെടുത്തുക കൂടിയാണ് ചെയ്യുന്നത്. ആർത്തവകാലത്തുപോലും ഇണകളോടൊപ്പം തിന്നുകയും കുടിക്കുകയും മാത്രമല്ല, അവരുപയോഗിച്ച ഭക്ഷണപാനീയങ്ങളുടെ ബാക്കി അതേ പാത്രങ്ങളിൽ നിന്ന് തന്നെ ഉപയോഗിക്കുകയും സഹശയനം നടത്തുകയുമെല്ലാം ചെയ്തിരുന്ന നബിയെക്കുറിച്ച് വാചാലമാകുന്ന ആയിഷ (റ) (നസാഈ സ്വഹീഹായ സനദോടെ നിവേദനം ചെയ്തത്) വൈകാരികപ്രശ്നങ്ങളാൽ പ്രയാസപ്പെടുന്ന പെൺനാളുകളിൽ പോലും നബിസ്നേഹത്തിന്റെ ശീതളിമയും ആഴങ്ങളുമനുഭവിച്ചപ്പോഴുള്ള സംതൃപ്തിയെ വെളിപ്പെടുത്തുന്നതോടൊപ്പം തന്നെ മറ്റ് പല സംസ്കാരങ്ങളിലെയും പോലെ രജസ്വലയെ മൊത്തം അശുദ്ധിയായും അസ്പൃശ്യയായും കാണുകയും വൈയക്തികവ്യവഹാരങ്ങളിൽ നിന്ന് പോലും മാറ്റിനിർത്തുകയും ചെയ്യുന്നത് വഴിയുള്ള മനഃസംഘർഷമോ അപകർഷതാബോധമോ അനുഭവിക്കേണ്ട ദൗർഭാഗ്യമൊന്നും പ്രവാചകാനുചരികളായ വനിതകൾക്കില്ലെന്ന് വിളിച്ച് പറയുകയും ചെയ്യുന്നുണ്ട്.

അമ്പത് കഴിഞ്ഞ പ്രവാചകൻ (സ) ഒൻപതുകാരിയായ ആയിഷയോടൊപ്പം ദാമ്പത്യജീവിതമാരംഭിച്ചതിൽ എന്ത് മാതൃകയാണുള്ളതെന്ന് ചോദിക്കുന്നവരോട് ആ പ്രായവ്യത്യാസത്തിലൂടെ തന്നെയാണ് നബിജീവിതം വലിയൊരു സന്ദേശം മാനവരാശിക്ക് നൽകുന്നത് എന്ന് തന്നെയാണ് ഉത്തരം. പ്രായമല്ല പൊരുത്തമാണ് ദാമ്പത്യവിജയത്തിന്റെ അടിത്തറയെന്ന സന്ദേശം നൽകുന്നതാണ് ആ ദാമ്പത്യത്തിന്റെ പത്ത് വർഷങ്ങൾ. ഇരുപത്തിയഞ്ചുകാരനായിരിക്കുമ്പോൾ നാല്പതുകാരിയോടൊപ്പം ദാമ്പത്യമാരംഭിക്കുകയും പരസ്പരം മധുരം നൽകിക്കൊണ്ട് ജീവിക്കുകയും കാൽ നൂറ്റാണ്ടുകാലം ആവോളം സ്നേഹം നൽകുകയും വാങ്ങുകയും ചെയ്ത് യഥാർത്ഥ ഇണകളും തുണകളുമായിത്തീരുകയും ചെയ്ത മുഹമ്മദ്-ഖദീജ ദമ്പതികൾ സംതൃപ്തദാമ്പത്യത്തിന് പെൺപ്രായം കൂടുതലാണെന്നത് തടസ്സമേയല്ലെന്ന് മാനവതയെ പഠിപ്പിച്ചത് പോലെയുള്ള മഹാമാതൃക. ഇതിനർത്ഥം എക്കാലഘട്ടങ്ങളിലെയും എല്ലാ മനുഷ്യർക്കും ഈ പ്രായവ്യത്യാസം തുടരാൻ കഴിയുമെന്നോ കഴിയണമെന്നോ അല്ല, പ്രത്യുത പ്രായവ്യത്യാസമല്ല സംതൃപ്തദാമ്പത്യത്തിന്റെ മാനകമെന്ന സത്യം മനുഷ്യരെ പഠിപ്പിക്കുകയാണ് നബി തന്റെ വിവാഹങ്ങളിലൂടെയെല്ലാം ചെയ്തത് എന്ന് മാത്രമാണ്. ഇണകൾ ഏത് പ്രായത്തിലുള്ളവരാണെങ്കിലും പരസ്പരം സ്നേഹം നൽകാനും ഉൾക്കൊള്ളുവാനും മനസ്സിലാക്കുവാനും കഴിയുമെങ്കിൽ സംതൃപ്തിയുടെ കൊടുമുടിയിലെത്താൻ സാധിക്കുമെന്ന വലിയ പാഠമാണ് നബിവിവാഹങ്ങളെല്ലാം മാനവരാശിക്ക് നൽകുന്നത്.

ഒമ്പത്കാരിയുമായി ദാമ്പത്യജീവിതമാരംഭിച്ച മുഹമ്മദ് നബിയെ ശിശുകാമിയെന്നും പീഡോഫൈലെന്നും വിളിച്ച് ആക്ഷേപിക്കുന്നവരോട് വിനീതമായി പറയാനുള്ളത് അങ്ങനെയെങ്കിൽ നിങ്ങളും ഞാനുമെല്ലാം പീഡോഫൈലുകളുടെ മക്കളും പേരമക്കളുമാണെന്നാണ്. ഇന്ത്യയിലെ സ്ത്രീകളുമായി ലൈംഗികബന്ധത്തിലേർപ്പെടണമെങ്കിൽ പന്ത്രണ്ട് വയസ്സെങ്കിലുമാകണമെന്ന നിയമം (The Indian Criminal Law Amendment Act, 1891) ബ്രിട്ടീഷ് സർക്കാർ കൊണ്ടു വന്നപ്പോൾ അതിന്നെതിരെ നമ്മുടെ മുത്തച്ഛന്മാർ സമരം ചെയ്തിട്ട് ഒന്നേകാൽ നൂറ്റാണ്ട് മാത്രമേ കഴിഞ്ഞിട്ടുള്ളൂ. പത്ത് വയസ്സുകാരികളെ വിവാഹം ചെയ്തുകൊടുക്കാനും അവരുമായി രതിയിലേർപ്പെടുവാനുമുള്ള തങ്ങളുടെ മതപരമായ അവകാശം അനുവദിച്ചുകിട്ടണമെന്ന് ആവശ്യപ്പെട്ടുകൊണ്ട് സമരം ചെയ്തവർക്ക് മുന്നിലുണ്ടായിരുന്നത് ഇന്ത്യൻ ദേശീയപ്രസ്ഥാനത്തിന്റെ നേതൃത്വത്തിലുണ്ടായിരുന്ന ബാല ഗംഗാധരതിലകനായിരുന്നുവെന്ന സത്യം നമ്മുടെയെല്ലാം മുത്തച്ഛന്മാർ പീഡോഫൈലുകളാണെന്നാണോ വ്യക്തമാക്കുന്നതെന്ന് പറയാൻ ആയിഷാവിവാഹത്തിന്റെ പേരിൽ നബിയെ തെറി പറയാൻ ധൃഷ്ടരാകുന്നവർക്ക് ബാധ്യതയുണ്ട്. മതപരവും സാമൂഹികവുമായ തങ്ങളുടെ രീതികളെ ചോദ്യം ചെയ്യുന്ന യാതൊരുവിധ ഇടപെടലുകളും നടത്താൻ സർക്കാരിനെ അനുവദിക്കില്ലെന്ന് പ്രഖ്യാപിച്ചു കൊണ്ടാണ് ബാലവിവാഹത്തെയും രതിയെയും നിരോധിച്ചുകൊണ്ടുള്ള ബില്ലിനെതിരെ പോരാടാൻ ജനങ്ങളെ തിലകൻ പ്രചോദിപ്പിച്ചതെന്ന് മീര കൊസാംബി Economic and Political Weekly (1991 August 3-10)യിൽ എഴുതിയ “Girl-Brides and Socio-Legal Change: Age of Consent Bill (1891) Controversy” എന്ന പ്രബന്ധത്തിൽ നിരീക്ഷിക്കുന്നുണ്ട്. 1891 മാർച്ച് 19 ന് മുമ്പ് നമ്മുടെ മുത്തച്ഛന്മാരിൽ പലരും പീഡോഫൈലുകളായിരുന്നുവെന്നാണോ ഇതെല്ലാം അർത്ഥമാക്കുന്നതെന്ന് വ്യക്തമാക്കേണ്ടത് തിലകൻ മുന്നോട്ട് വെച്ച ദേശീയസങ്കല്പത്തിന്റെ പേരിൽ അഭിമാനിക്കുകയും ഒപ്പം നബിയെ തെറി പറയാൻ കിട്ടുന്ന അവസരങ്ങളെല്ലാം ഉപയോഗിക്കുകയും ചെയ്യുന്നവരാണ്.

ഭാരതീയതയിൽ അഭിമാനിക്കുന്നവരാണ് തങ്ങളെന്ന് ആണയിട്ടുകൊണ്ട് നബി(സ)യെ തെറിപറയുന്നവർ വാൽമീകിരാമായണമെങ്കിലും വായിക്കാൻ സന്നദ്ധമായാൽ പ്രവാചകപ്രഭുവിനെതിരെയുള്ള അവരുടെ കാർക്കിച്ച് തുപ്പലുകൾ അവരുടെ തന്നെ മുഖത്ത് വന്നു വീഴുന്നതാണ് നമുക്ക് കാണാനാവുക. സമ്പൂർണ്ണ പുരുഷനും മാതൃകാഭർത്താവുമായി രാമായണം അവതരിപിപ്പിക്കുന്ന ശ്രീരാമൻ സീതയെ വിവാഹം ചെയ്യുന്നത് അദ്ദേഹത്തിന് പതിമൂന്ന് വയസ്സും സീതക്ക് ആറ് വയസ്സുമുള്ളപ്പോഴാണെന്നാണ് വാല്മീകി രാമായണം, ആരണ്യകാണ്ഡത്തിന്റെ നാല്പത്തേഴാം സർഗ്ഗത്തിലെ വ്യത്യസ്ത ശ്ലോകങ്ങൾ വായിച്ചാൽ ആർക്കും മനസ്സിലാവുക. ഇതിന്റെ അടിസ്ഥാനത്തിൽ ആരെങ്കിലും ശ്രീരാമനെ പീഡോഫൈൽ എന്ന് വിളിച്ചാൽ നമുക്ക് അവരോട് പറയേണ്ടി വരിക രാമായണത്തിലെ ആദ്യത്തെ വചനത്തിന്റെ തുടക്കത്തിലെ നിർദേശം മാത്രമാണ്. ‘മാ നിഷാദ’ (അരുത് കാട്ടാളാ..). ഇന്നത്തെ സാമൂഹ്യമാനദണ്ഡങ്ങൾ വെച്ചുകൊണ്ട് പുരാതനകാലത്തെ മഹാവ്യക്തിത്വങ്ങളെ അപഗ്രഥിക്കുകയും അവരെ തെറി പറയുവാൻ കാരണങ്ങൾ കണ്ടെത്തുകയും ചെയ്യുന്നവരെ കാട്ടാളന്മാർ എന്ന് വിളിച്ചാൽ കാട്ടുമൂപ്പന്മാർ നമ്മെ വിചാരണ ചെയ്ത് ശിക്ഷിക്കുമോയെന്ന് ഭയപ്പെടണം.

എന്ന് മുതൽക്കാണ് നാം ഇന്ത്യക്കാർക്ക് ചെറിയ പ്രായത്തിലുള്ളവരുമായുള്ള വിവാഹം പീഡോഫീലിയയായി അനുഭവപ്പെടാൻ തുടങ്ങിയത്? ഭാരതത്തിന്റെ ആത്മീയ പാരമ്പര്യത്തെക്കുറിച്ച് എന്തെങ്കിലുമൊക്കെ വിവരമുള്ള ആരെങ്കിലും ഇത്തരം വൃത്തികേടുകൾ പറയുമോ? ആധുനിക ഇന്ത്യയുടെ ആത്മീയ പാരമ്പര്യത്തെക്കുറിച്ച് പറയുമ്പോൾ ആരുടെയും മനസ്സിലുദിക്കുന്ന ചിത്രം വിവേകാനന്ദ സ്വാമികളുടേതാണ്. കുസൃതിക്കുറുമ്പനായ നരേന്ദ്രനിൽ നിന്ന് ലോകമതസമ്മേളനത്തിലെ ക്ഷണിതാവായ വിവേകാനന്ദനിലേക്കുള്ള പരിവർത്തനത്തിന്റെ ചാലകമായി വർത്തിച്ചത് അദ്ദേഹത്തിന്റെ ആത്മീയഗുരുവായ ശ്രീരാമകൃഷ്ണ പരമഹംസനായിരുന്നു. ആധുനിക ഭാരതത്തിലെ ആത്മീയാചാര്യന്മാരിൽ പ്രമുഖനായ ശ്രീരാമകൃഷ്ണ പരമഹംസർ തന്റെ ഇരുപത്തിമൂന്നാമത്തെ വയസ്സിലാണ് അഞ്ച് വയസ്സുകാരിയായ ശാരദാ ദേവിയെ വിവാഹം ചെയ്തത്. ശ്രീരാമകൃഷ്‌ണ മിഷനിലുള്ളവർ അമ്മയെന്ന് വിളിക്കുന്ന ശാരദാ ദേവിയെ പരമഹംസർ വേൾക്കുന്നത് 1859 ലാണെന്ന് നാം മനസ്സിലാക്കണം. അന്നത്തെ ഇന്ത്യയിലുണ്ടായിരുന്ന വിവാഹരീതിയെയല്ലാതെ മറ്റൊന്നും ഇത് അടയാളപ്പെടുത്തുന്നില്ലെന്ന് ആർക്കാണറിയാത്തത്? പരമഹംസനോടുള്ള ആദരവിനെയോ ശാരദാദേവിയോടെയുള്ള ഭക്തിയേയോ ബാധിക്കുന്ന കാര്യമായി അവരുടെ വിവാഹപ്രായത്തെ വേദാന്തികളൊന്നും മനസ്സിലാക്കുന്നില്ലെങ്കിൽ പിന്നെ മുഹമ്മദ് നബിയെ ഭൽസിക്കുവാൻ ഇണയുടെ പ്രായമെങ്ങനെയാണ് നിമിത്തമായിത്തീരുന്നത്?!!

ഇരുപതാം നൂറ്റാണ്ടിന്റെ തുടക്കത്തിലെ ഹൈന്ദവ നവോത്ഥാനത്തിന്റെ മുഖമായിരുന്നു മഹാദേവ് ഗോവിന്ദ് റാനഡെ. സമൂഹത്തിൽ പൊതുവെ നിരോധിക്കപ്പെട്ടിരുന്ന വിധവാ വിവാഹം അനുവദിക്കുന്നതിനുവേണ്ടിയുള്ള സമരത്തിൽ മുന്നിൽ നിന്നായാൾ; ഇന്ത്യൻ നാഷണൽ കോൺഗ്രസ്സിന്റെ രൂപീകരണയോഗത്തിൽ പങ്കെടുത്ത ഇന്ത്യക്കാരിലൊരാൾ. അദ്ദേഹത്തിന്റെ ആദ്യഭാര്യ മരണപ്പെട്ടപ്പോൾ തന്റെ മുപ്പത്തിയൊന്നാമത്തെ വയസ്സിൽ രണ്ടാമതായി വിവാഹം ചെയ്തത് പതിന്നുകാരിയായ വയസ്സുള്ള രമാഭായിയെന്ന കന്യകയെയാണ്. നമ്മുടെ രാഷ്ട്രപിതാവായ ഗാന്ധിജി തന്റെ പതിമൂന്നാത്തെ വയസ്സിലാണ് പതിനാല് വയസ്സുള്ള കസ്തൂർബായെ വിവാഹം ചെയ്തത്. ഇന്ത്യൻ ഭരണഘടനയുടെ ശില്പിയായ അംബേദ്‌കർ 1906 ൽ തന്റെ പതിനഞ്ചാം വയസ്സിൽ രമാ ഭായിയെ വിവാഹം ചെയ്യുമ്പോൾ അവർക്ക് ഒൻപത് വയസ്സായിരുന്നു. ലോകപ്രശസ്തനായ ഇന്ത്യൻ ഗണിത ശാസ്ത്രജ്ഞൻ ശ്രീനിവാസ രാമാനുജ അയ്യങ്കാർ 1909 ൽ തന്റെ ഇരുപത്തിയൊന്നാമത്തെ വയസ്സിൽ ജാനകി അമ്മാളിനെ വിവാഹം ചെയ്യുമ്പോൾ അവർക്ക് പത്ത് വയസ്സാണുള്ളത്. ഭാരതീയ ഭൗതികശാസ്ത്രജ്ഞരിൽ അഗ്രഗണ്യനായ സത്യേന്ദ്രനാഥ് ബോസ് തന്റെ ഇരുപതാമത്തെ വയസ്സിൽ ഉഷാപതി ഘോഷ് എന്ന പതിനൊന്ന്കാരിയെ വിവാഹം ചെയ്യുന്നത് 1914 ലാണ്. പ്രവാചകനെ തെറി പറയുവാൻ ധൃഷ്ടരാവുന്നവരുടെ മാനദണ്ഡങ്ങൾ പ്രകാരം പരിശോധിച്ചാൽ ഇരുപതാം നൂറ്റാണ്ടിന്റെ തുടക്കം വരെ ഇന്ത്യ നിറയെ പീഡോഫൈലുകളായിരുന്നു ജീവിച്ചിരുന്നതെന്ന് പറയേണ്ടി വരും. നമ്മുടെ നാടിന് സ്വാതന്ത്ര്യം നേടിത്തരികയും കാര്യമായ സംഭാവനകൾ അർപ്പിക്കുകയും ചെയ്തവരെല്ലാം അവരുടെ കണ്ണിൽ പീഡോഫൈലുകളാണ്. ‘ഹാ പീഡോഫീലിയ; എത്ര നല്ല കാര്യം’ എന്ന് പറയേണ്ടി വരുമോ !!!

ഇസ്‌ലാമുമായുള്ള ആദർശസമരത്തിൽ പരാജയപ്പെട്ടുകൊണ്ടേയിരിക്കുന്നതിന്റെ കെർവ്വ് തീർക്കാൻ നബിയെ തെറി പറഞ്ഞ് ആസ്വദിക്കുകയും ആശ്വസിക്കുകയും ചെയ്യുന്ന ചിലരും ഇതിന്നിടയിൽ വർദ്ധിതമായ ആവേശത്തോടെ പീഡോഫീലിയ ആരോപണവുമായി രംഗത്തുണ്ട്. അവരോടും വിനീതമായി പറയാനുള്ളത് തങ്ങളുടെ കയ്യിലുള്ള വേദഗ്രന്ഥം ഇടക്കെങ്കിലുമൊന്ന് വായിക്കുന്നതും ചരിത്രമെല്ലാം ഓർക്കുന്നതും നല്ലതാണെന്നാണ്. ബൈബിളിലെ ഉല്പത്തി പുസ്തകത്തിൽ വിവിധ ഇടങ്ങളിലായി പരാമർശിച്ചിരിക്കുന്ന പലരുടെയും പ്രായങ്ങൾ താരതമ്യം ചെയ്‌താൽ അബ്രഹാമിന്റെ രണ്ടാമത്തെ മകനായ ഇസഹാക്ക് റബേക്കയെ വിവാഹം ചെയ്യുമ്പോൾ അവർക്ക് മൂന്ന് വയസ്സായിരുന്നുവെന്നാണ് മനസ്സിലാവുക. എന്നാൽ ഇരുപത്തിനാലാം അധ്യായത്തിന്റെ തുടക്കത്തിലെ റബേക്കയെക്കുറിച്ച പരാമർശങ്ങൾ പത്തിൽ കുറയാത്ത പ്രായമുണ്ടെന്നാണ് വ്യക്തമാക്കുന്നത്. ഇത്തരം വൈരുധ്യങ്ങൾ ബൈബിളിൽ സാധാരണമാണല്ലോ. ഈ വിഷയത്തിൽ കൃത്യമായ വിവരം നൽകുന്നത് അപ്പോക്രിഫാഗ്രന്ഥമായ യാഷെറിന്റെ പുസ്തകമാണ്. വിവാഹ സമയത്ത് റബേക്കക്ക് പത്ത് വയസായിരുന്നു പ്രായമെന്നാണ് യാഷെറിന്റെ പുസ്തകം പറയുന്നത് (24: 40). പുസ്തകം വായിക്കണമെന്നുള്ളവർ https://www.sacred-texts.com/chr/apo/jasher/index.htm എന്ന ലിങ്കിൽ പരതിയാൽ മതി. ആയിഷാവിവാഹത്തിന്റെ പേരിൽ മുഹമ്മദ് നിബി(സ)യെ തെറി പറയാൻ തക്കം പാർത്തിരിക്കുന്നവർ ആദ്യമായി പീഡോഫൈൽ എന്ന് വിളിക്കേണ്ടി വരിക യേശുവിന്റെ മുതുമുത്തച്ഛനെയായിരിക്കുമെന്നർത്ഥം.

യോസേഫ് എന്ന തച്ചനുമായി വിവാഹ നിശ്ചയം കഴിഞ്ഞ ശേഷമാണ് ക്രിസ്തുവിന്റെ ജനനത്തെക്കുറിച്ച സന്തോഷവാർത്തയുമായി മാലാഖമാർ യേശുമാതാവായ മറിയക്കടുത്തെത്തിയതെന്ന് സംഹിതസുവിശേഷങ്ങൾ പറയുന്നുണ്ടെങ്കിലും അവരുടെ രണ്ട് പേരുടെയും വയസ്സിനെക്കുറിച്ച് അവയിലൊന്നുമില്ല. എന്നാൽ പുതിയ നിയമത്തിൽ ഉൾപ്പെടുത്തിയിട്ടില്ലാത്ത ചില അപ്പോക്രിഫാ ഗ്രന്ഥങ്ങളിൽ അവരുടെ പ്രായത്തെക്കുറിച്ച് പരാമർശിക്കുന്നുണ്ടെന്ന് കാത്തലിക്ക് എൻസൈക്ലോപീഡിയയിൽ പറയുന്നുണ്ട്. വിശുദ്ധ യോസേഫിന് നാല്പത് വയസ്സുള്ളപ്പോഴാണ് ആദ്ദേഹം സലോമിയെ വിവാഹം ചെയ്തത്; 49 വർഷം നീണ്ടു നിന്ന് അവരുടെ വൈവാഹികജീവിതം അവസാനിച്ചത് സലോമിയുടെ മരണത്തോടെയാണ്. അത് കഴിഞ്ഞ് ഒരു വർഷത്തിന് ശേഷമാണ് പന്ത്രണ്ട്കാരിയായ മറിയയുമായുള്ള (പന്ത്രണ്ടിനും പതിനാലിനുമിടയിലായിരുന്നു മറിയയുടെ പ്രായം) വിവാഹനിശ്ചയം നടന്നത്. ഇക്കാര്യത്തിൽ സംശയമുള്ളവർക്ക് കാത്തലിക് എൻസൈക്ലോപീഡിയയുടെ https://www.newadvent.org/cathen/08504a.htm എന്ന ലിങ്ക് സന്ദർശിക്കാവുന്നതാണ്. പന്ത്രണ്ടുകാരിയെ വിവാഹം കഴിക്കാനൊരുമ്പെട്ട തൊണ്ണൂറുകാരനായ പീഡോഫൈലായിരുന്നു കത്തോലിക്കാ സഭ വിശുദ്ധനായി വാഴ്ത്തുന്ന യോസേഫെന്നാണ് നബിനിന്ദക്ക് വേണ്ടി മാത്രം വായ തുറക്കുന്ന ചില മിഷനറിമാരുടെ അളവുകോലുപയോഗിച്ചാൽ പറയേണ്ടി വരിക.

അല്ലെങ്കിലും സഭയ്‌ക്കെന്നാണ് ചെറിയ പ്രായത്തിലുള്ളവരുടെ വിവാഹം വിലയ്ക്കപ്പെട്ടതായിരുന്നത്? നാലും അഞ്ചും വയസ്സുള്ളവരുടെ എത്രയെത്ര വിവാഹകൂദാശകളാണ് മാർപ്പാപ്പമാർ മുതൽ വികാരിയച്ചന്മാർ വരെ നടത്തിയിയിരിക്കുന്നത് !! സ്‌കോട്ട്ലാൻഡ് രാജാവായിത്തീർന്ന ഡേവിഡ് രണ്ടാമന് അദ്ദേഹത്തിന്റെ നാലാം വയസ്സിൽ ഏഴ് വയസ്സുകാരിയായ ജോനിനെ (Joan of the Tower) വിവാഹം ചെയ്തു കൊടുത്തത് ആരായിരുന്നു? പ്രസിദ്ധ സ്കോട്ടിഷ് അധിപതിയായിരുന്ന റോബർട്ട് ഒന്നാമന്റെയും എലിസബത്തിന്റെയും മകനായ ഡേവിഡ് രണ്ടാമനും ഇംഗ്ലണ്ട് രാജാവായിരുന്ന എഡ്വേർഡ് രണ്ടാമന്റെയും ഫ്രാൻസിലെ പ്രസിദ്ധയായ ഇസബെല്ലയുടെയും മകളായ ജോനും 1328 ജൂലൈ പതിനേഴിന് വിവാഹിതരായപ്പോൾ ഇല്ലാത്ത എന്തെങ്കിലും പുതിയ കാനോനുകൾ മുഹമ്മദ് നബിയെ ഭൽസിക്കാന് വേണ്ടി മാത്രമായി ചർച്ചിന് ലഭിച്ചിട്ടുണ്ടോയെന്നറിയില്ല. എപ്പോഴോ ഒരിക്കൽ മാത്രം സംഭവിച്ചതൊന്നുമല്ല ഇത്. ഇംഗ്ലണ്ടിലും ഫ്രാൻസിലുമെല്ലാം ഇത്തരം വിവാഹങ്ങൾ സർവ്വസാധാരണമായിരുന്നുവെന്ന് രാജവംശങ്ങളുടെ ചരിത്രം വായിച്ചാൽ മനസ്സിലാവും. ഫ്രാൻസിലെ രാജാവായിരുന്ന ചാൾസ് ആറാമന്റെ മകൾ ഇസബെല്ല ഇംഗ്ലണ്ട് രാജാവായിരുന്ന റിച്ചാർഡ് രണ്ടാമന്റെ രണ്ടാം ഭാര്യയാകുമ്പോൾ അവർക്ക് ഏഴ് വയസ്സായിരുന്നു. 1396 നവംബർ നാലിന് കാലായിസിലെ സെന്റ് നിക്കോളാസ് ചർച്ചിൽ വെച്ച് അവരുടെ വിവാഹ കൂദാശ നടത്തുമ്പോൾ ലഭിച്ചിട്ടില്ലാത്ത വെളിപാടുകളെന്തെങ്കിലും പുതുതായി ഉണ്ടായിട്ടുണ്ടോയെന്ന് പറയേണ്ടത് നബിനിന്ദക്കായി മാത്രം സോഷ്യൽ മീഡിയ തുറന്നുവെച്ചിരിക്കുന്ന വെറുപ്പുൽപ്പാദകരാണ്. സഭയുടെ ആശീർവാദത്തോടെ നടന്ന എത്രയോ ബാലവിവാഹങ്ങൾ !!! അത് ചൂണ്ടി പീഡോഫീലിയ അനുവദിക്കുകയാണ് സഭ ചെയ്തതെന്ന് പറയണമെങ്കിൽ ചരിത്രത്തെക്കുറിച്ച ചെറിയ അജ്ഞതയൊന്നും പോരാ. ആഢംബരഭ്രമത്താൽ ഇസ്‌ലാംവിരോധത്തിന്റെ പൊട്ടക്കിണറ്റിൽ പെട്ട് കരഞ്ഞുകൊണ്ടിരിക്കുന്നവരുടെ സ്ഥിതിയതാണ്.

പാശ്ചാത്യൻ സഭകൾക്ക് മാത്രമൊന്നുമല്ല ചെറിയ പ്രായത്തിലുള്ളവരുടെ വിവാഹങ്ങൾ തിന്മയാണെന്ന ഇപ്പോൾ ചില വെറുപ്പുതീനികൾക്ക് മാത്രം ലഭിച്ച വെളിപാട് ലഭിക്കാതിരുന്നത്; കേരളത്തിലെ സഭകൾക്കും അത് ലഭിച്ചിരുന്നില്ലെന്നതിന്ന് നമ്മുടെയെല്ലാം ചുറ്റും തന്നെ നിരവധി ഉദാഹരണങ്ങൾ കാണാനും വായിക്കാനും കഴിയും. മലയാള മനോരമയുടെ മാമ്മൻ മാപ്പിള ഏത് പ്രായത്തിലാണ് മാമ്മിയെ വിവാഹം ചെയ്തതെന്നറിയണമെങ്കിൽ മകൻ കെ. എം. മാത്യുവിന്റെ ‘എട്ടാമത്തെ മോതിരം’ എന്ന ആത്മകഥ വായിച്ചാൽ മതി. 1888 ൽ അവരുടെ വിവാഹം നടക്കുബോൾ അദ്ദേഹത്തിന് പതിനഞ്ച് വയസ്സും അവർക്ക് പത്ത് വയസ്സുമായിരുന്നു പ്രായം. അന്നൊന്നുമില്ലാത്ത പീഡോഫീലിയ ആരോപണം ഇന്ന് കൂർപ്പിച്ചെടുക്കുന്നത് നാടിന്റെ ബഹുസ്വരതയെ തകർക്കാൻ ശ്രമിക്കുന്നവർക്ക് ഊർജ്ജം നൽകാൻ വേണ്ടിയാണെങ്കിൽ ലൂക്കോസ് (23:34) ഉദ്ധരിച്ച യേശുവചനം മാത്രമേ അവർക്കായി നമുക്ക് പറയാനുള്ളൂ: “കർത്താവേ, ഇവർ ചെയ്യുന്നതെന്തെന്ന് ഇവർക്ക് തന്നെ അറിയില്ല; ഇവർക്ക് നീ പൊറുത്തു കൊടുക്കേണമേ”

നബിനിന്ദക്ക് അവസരങ്ങൾ കാത്തിരിക്കുന്നവരും ലഭിക്കുന്ന അവസരങ്ങൾ പാഴാക്കാത്തവരും വെറുപ്പുൽപ്പാദനം മാത്രം ജീവിത സപര്യയായി സ്വീകരിച്ചിരിക്കുന്നവരുമായ ചില നാസ്തികന്മാരാണ് കേരളത്തിൽ നബി(സ)യെ ഭൽസിക്കുവാൻ മുന്നിലുള്ളത്. ആസക്തിയാണ് സംതൃപ്തിയെന്ന് തെറ്റിദ്ധരിച്ച ചില ‘സാധു’ക്കളുടെ വർത്തമാനങ്ങൾ കേട്ടാൽ തങ്ങൾക്ക് ലഭിക്കാത്ത സംതൃപ്തദാമ്പത്യം അനുഭവിക്കുകയും അക്കാര്യത്തിൽ ലോകത്തിന് മാതൃക കാണിക്കുകയും ചെയ്ത പ്രവാചകനോടുള്ള അസൂയയും വിരോധവുമാണോ അവരുടെ എല്ലില്ലാത്ത നാവിനെ പ്രവർത്തിക്കുന്നത് എന്ന് തോന്നിപ്പോകും. മനുഷ്യരുടെ സ്വാഭാവികമായ ജൈവികപ്രക്രിയയായ ലൈംഗികതയുടെ ആസ്വാദനത്തിന് വിവാഹം വരെ പോലും കാത്തിരിക്കരുതെന്ന് യുവാക്കളെ പഠിപ്പിക്കുന്നവർക്ക് പക്ഷെ പ്രവാചകന്റെ ദാമ്പത്യജീവിതം മാത്രം ചതുർത്ഥിയാകുന്നതിന്റെ മനഃശാസ്ത്രം അസൂയയും വിരോധവുമല്ലാതെ മറ്റെന്താണ്? തങ്ങൾക്ക് വഴി കാണിച്ച നേതാക്കളിൽ പലരെയും പീഡോഫീലുകളാക്കിക്കൊണ്ടല്ലാതെ അവർക്കും പ്രവാചകനെ തെറി പറയാൻ കഴിയില്ല. ഇന്ത്യയിലെ ഏറ്റവും പ്രഗത്ഭനായ നിരീശ്വരവാദികളിലൊരാളായ പെരിയാർ രാമസ്വാമി നാഗമ്മയെ വിവാഹം ചെയ്തത് അവർക്ക് പതിമൂന്ന് വയസ്സുള്ളപ്പോഴായിരുന്നുവെന്ന സത്യം നിഷേധിക്കാൻ ആർക്കാണ് കഴിയുക? കേരളത്തിലെ കമ്യൂണിസ്റ്റ് പ്രസ്ഥാനത്തിന്റെ ആചാര്യന്മാരിലൊരാളായ എ.കെ. ഗോപാലൻ പിന്നീട് തന്റെ ജീവിതസഖിയാക്കിത്തീർത്ത സുശീലാഗോപാലനുമായി പ്രേമബന്ധമാരംഭിക്കുന്നത് അവർക്ക് പന്ത്രണ്ട് വയസ്സുള്ളപ്പോഴാണെന്ന് അദ്ദേഹത്തിന്റെ ജീവചരിത്രം വായിച്ചാൽ മനസ്സിലാകും. അവരൊന്നും പീഡോഫീലുകളായിരുന്നുവെന്ന് വെളിവുള്ള ആരെങ്കിലും പറയുമെന്ന് തോന്നുന്നില്ല; ഈ വെളിവില്ലായ്മയാണ് നബിനിന്ദകരുടെ കാര്യമായ കൈമുതൽ.

നബിജീവിതം വിമർശിക്കപ്പെട്ടതു പോലെ മറ്റൊരാളുടെയും ജീവിതം തലമുടി നാരിഴ കീറി വിമർശിക്കപ്പെട്ടിട്ടില്ല. നബി ജീവിച്ച കാലം മുതൽ ഇന്ന് വരെ ആ വിമർശനങ്ങൾ അഭംഗുരം തുടർന്നുകൊണ്ടേയിരിക്കുകയാണ്. എന്നാൽ ഇരുപതാം നൂറ്റാണ്ടിന്റെ തുടക്കം വരെ വിമർശകരാരും തന്നെ ആയിശയുമായുള്ള നബിദാമ്പത്യത്തിന്റെ പേരിൽ അദ്ദേഹത്തെ വിമർശിച്ചതായി കാണാൻ കഴിയില്ല. തന്റെ അമ്പത് വയസ്സിന് ശേഷം ഒമ്പതുകാരിയുമായി ദാമ്പത്യബന്ധത്തിലേർപ്പെട്ടത് അദ്ദേഹം പ്രവാചകനല്ലെന്നതിന് തെളിവായി ജൂതന്മാരോ ക്രൈസ്തവരോ ആയ വിമർശകരാരെങ്കിലും ഇരുപതാം നൂറ്റാണ്ടിന് മുമ്പ് പറഞ്ഞതായി രേഖകളൊന്നുമില്ല. നബി(സ)യുടെ അധാർമ്മികത സ്ഥാപിക്കാനായി ഇക്കാലയളവിൽ ജീവിച്ച നാസ്തികരായ വിമർശകരൊന്നും തന്നെ മുഹമ്മദ്- ആയിഷ ദാമ്പത്യത്തെ ചൂണ്ടിക്കാട്ടിയിട്ടില്ല. ഇതിന്നർത്ഥമെന്താണ്? ഇരുപതാം നൂറ്റാണ്ടിന്റെ തുടക്കം വരെ അത്തരം ദാമ്പത്യങ്ങൾ സർവ്വസാധാരണമായിരുന്നുവെന്ന് തന്നെ. അതിൽ ഒരു തെറ്റും ആരും കണ്ടിരുന്നില്ല; അത്തരം ദാമ്പത്യങ്ങളിലേർപ്പെട്ടവരുടെ പ്രായക്കുറവോ പ്രായവ്യത്യാസമോ അവരുടെ ദാമ്പത്യവിജയത്തെയോ സംതൃപ്തിയെയോ ബാധിച്ചിരുന്നുവെന്നതിന് തെളിവുകളൊന്നുമില്ല. പുതിയ കാലത്തെ സാഹചര്യങ്ങളിൽ അത്തരം വിവാഹങ്ങൾ പ്രോത്സാഹിപ്പിക്കപ്പെട്ടുകൂടാ എന്നുതന്നെയാണ് എല്ലാവരുടെയും അഭിപ്രായം. പതിനാല് നൂറ്റാണ്ടുകൾക്ക് മുമ്പ് നടന്ന ഒരു വിവാഹത്തിന്റെ പേരിൽ പ്രവാചകനിൽ ശിശുരതിക്കാരനെ തെരയുന്നവർ ആത്മാർത്ഥതയുള്ളവരാണെങ്കിൽ ചെയ്യേണ്ടത് ഇരുപതാം നൂറ്റാണ്ടിന് മുമ്പ് ഇസ്‌ലാം വിമർശനങ്ങളുടെ കാര്യത്തിൽ തങ്ങളുടെ മുൻഗാമികളായ ആരെങ്കിലും പ്രവാചകനെ ഇവ്വിഷയകമായി വിമർശിച്ചിട്ടുണ്ടോയെന്ന് പരിശോധിക്കുകയാണ്. ഇല്ലെന്ന സത്യം ബോധ്യപ്പെട്ടിട്ടും ഇതേ വിമർശനവുമായി പ്രവാചകനിന്ദ നടത്താനാണ് പുറപ്പാടെങ്കിൽ അവർ വിളിക്കപ്പെടേണ്ടത് വെറുപ്പുൽപ്പാദന കേന്ദ്രങ്ങളെന്നാണ്; അവർ പാദസേവ ചെയ്യുന്നത് പിശാചിനാണ്; സകല തിന്മകളെയും ഇളക്കിവിട്ട് മനുഷ്യരെ ദ്രോഹിക്കുന്ന പിശാചിനെ. ഇന്നല്ലെങ്കിൽ നാളെ അതിന്റെ ദുഷ്ടഫലം അവർ അനുഭവിക്കുക തന്നെ ചെയ്യും.

വിമർശനം:

അവിശ്വാസികളെ ചങ്ങലകളിൽ തളച്ച്, നിർബന്ധിച്ച് ഇസ്‌ലാം സ്വീകരിപ്പിക്കുന്നവരാണ് “ഉത്തമ ജനത” എന്ന് ഹദീസുകളിൽ വന്നിരിക്കുന്നു !! ഇസ്‌ലാം നിർബന്ധിത മതപരിവർത്തനത്തെ പ്രോത്സാഹിപ്പിക്കുന്നു എന്നല്ലെ ഇത് തെളിയിക്കുന്നത്?

മറുപടി:

1. നിർബന്ധിത ആദർശപരിവർത്തനത്തെ നിശിതമായി വിമർശിച്ച മതമാണ് ഇസ്‌ലാം. ആദർശ സ്വാതന്ത്ര്യത്തിന് അടിവരയിട്ടു കൊണ്ടുള്ള ക്വുർആൻ വചനങ്ങൾ അനവധിയാണ്:

“മതകാര്യത്തില്‍ ഒരുവിധ ബലപ്രയോഗവുമില്ല. ‎നന്മതിന്മകളുടെ വഴികള്‍ വ്യക്തമായും ‎വേര്‍തിരിഞ്ഞുകഴിഞ്ഞിരിക്കുന്നു…” (ക്വുർആൻ: 2:256)

“നിന്റെ നാഥന്‍ ഇച്ഛിച്ചിരുന്നെങ്കില്‍ ഭൂമിയിലുള്ളവരൊക്കെയും സത്യവിശ്വാസം സ്വീകരിക്കുമായിരുന്നു. എന്നിരിക്കെ ജനങ്ങള്‍ വിശ്വാസികളാകാന്‍ നീ അവരെ നിർബന്ധിക്കുകയോ? യാതൊരാള്‍ക്കും അല്ലാഹുവിന്‍റെ അനുമതിപ്രകാരമല്ലാതെ വിശ്വസിക്കാന്‍ കഴിയുന്നതല്ല. ചിന്തിച്ചു മനസ്സിലാക്കാത്തവര്‍ക്ക് അല്ലാഹു നികൃഷ്ടത വരുത്തിവെക്കുന്നതാണ്‌.”

“പറയുക: സത്യം നിങ്ങളുടെ രക്ഷിതാവിങ്കല്‍ നിന്നുള്ളതാകുന്നു. അതിനാല്‍ ഇഷ്ടമുള്ളവര്‍ വിശ്വസിക്കട്ടെ.” (ക്വുർആൻ: 18:29)

യുദ്ധങ്ങളിൽ ബന്ദികളാക്കപ്പെടുന്നവർക്ക് നിരുപാധിക മത സ്വാതന്ത്ര്യം ഇസ്‌ലാം എക്കാലത്തും വകവെച്ചു കൊടുത്തിട്ടുണ്ട്. ഖൈബറിൽ ബന്ദിയാക്കപ്പെട്ടിരുന്ന സ്വഫിയ്യയോട് പ്രവാചകൻ (സ്വ) ഇപ്രകാരം പറയുകയുണ്ടായി:

.ﺇﻥْ ﺃَﻗَﻤْﺖ ﻋﻠﻰ ﺩﻳﻨﻚ ﻟَﻢْ ﺃُﻛْﺮِﻫْﻚ، ﻭَﺇِﻥْ اﺧْﺘَﺮْﺕ اﻟﻠﻪَ ﻭَﺭَﺳُﻮﻟَﻪُ ﻓَﻬُﻮَ ﺧَﻴْﺮٌ ﻟَﻚ

“നിന്റെ പഴയ മതത്തില്‍ തന്നെ നില്‍ക്കാനാണ് തീരുമാനമെങ്കില്‍ അതുപേക്ഷിക്കാന്‍ ഞാന്‍ നിന്നെ നിർബന്ധിക്കുകയില്ല.” (അൽ മഗാസി: വാഖിദി: 2/675)

2. അവിശ്വാസികളെ ചങ്ങലകളിൽ തളച്ച്, നിർബന്ധിച്ച് ഇസ്‌ലാം സ്വീകരിപ്പിക്കുന്നവരാണ് “ഉത്തമ ജനത” എന്ന് ഒരു സ്വഹീഹായ ഹദീസിലും വന്നിട്ടില്ല. വിമർശകർ ഉന്നയിക്കുന്ന ആശയത്തോട് പുലബന്ധം പോലുമില്ലാത്ത ഒരു ഹദീസിനെ അപനിർമ്മിച്ചുകൊണ്ട് കെട്ടിയുണ്ടാക്കിയ ഒരു വികൃതമായ ആരോപണം മാത്രമാണ് അത്.

വിമർശന വിധേയമായ ഹദീസ് ഇപ്രകാരമാണ്:

.عن أبى هريرة رضى الله عنه عن النبى صلى الله عليه وسلم قال: (عَجِبَ اللَّهُ مِنْ قَوْمٍ يَدْخُلُونَ الجَنَّةَ فِي السَّلاَسِلِ)

അബൂഹുറൈറ (റ) യിൽ നിന്ന്, അദ്ദേഹം പ്രവാചകനിൽ (സ്വ) നിന്നും ഉദ്ദരിക്കുന്നു. അദ്ദേഹം (സ്വ) പറഞ്ഞു: ചങ്ങലകെട്ടുകളിലായി സ്വർഗത്തിൽ പ്രവേശിക്കുന്ന ഒരു പറ്റം ജനങ്ങളെ സംബന്ധിച്ച് അല്ലാഹു തൃപ്തി പ്രകടിപ്പിക്കും. (സ്വഹീഹുൽ ബുഖാരി: 3010)

സത്യനിഷേധികളുടെ അടുക്കൽ ബന്ദികളായി ജീവിക്കുകയും അതെ അവസ്ഥയിൽ മരണപ്പെടുകയൊ, കൊല്ലപ്പെടുകയൊ ചെയ്യുന്ന മുസ്‌ലിംകളെ സംബന്ധിച്ചാണ് ഹദീസ് പരാമർശിക്കുന്നത്. അത്തരം മുസ്‌ലിംകൾ അവർ മരണപ്പെട്ട ബന്ധനാവസ്ഥയിൽ തന്നെ പുനർജീവിപ്പിക്കപ്പെടാൻ കാരണം അവരുടെ ത്യാഗത്തിനും രക്തസാക്ഷിത്വത്തിനും ആദരവ് എന്ന നിലക്കാണ്. ഇത്രയും സുവ്യക്തമായ ഒരു ആശയത്തെ എത്ര വിദൂരമായ അർത്ഥതലത്തിലേക്കാണ് വിമർശകർ ദുർവ്യാഖ്യാനിച്ച് വലിച്ചു നീട്ടിയത് എന്ന് ശ്രദ്ധിക്കുക?!

വിമർശകർ വളച്ചൊടിച്ച് ദുർവ്യാഖ്യാനിക്കാറുള്ള, സമാനമായ മറ്റൊരു നിവേദനം ഇപ്രകാരമാണ്:

عن أبي هريرة في قوله تعالى: (كنتم خير أمة أخرجت للناس ) قال : ( خير الناس للناس ، يأتون بهم في السلاسل في أعناقهم حتى يدخلوا في الإسلام.

അല്ലാഹു പറഞ്ഞു: “മനുഷ്യവംശത്തിനു വേണ്ടി രംഗത്ത് കൊണ്ടുവരപ്പെട്ട ഉത്തമസമുദായമാകുന്നു നിങ്ങള്‍…” (ക്വുർആൻ: 3:110) എന്ന ക്വുർആൻ വചനത്തെ വ്യാഖ്യാനിച്ചു കൊണ്ട് അബൂഹുറൈറ (റ) പറഞ്ഞു: ജനങ്ങൾക്കായുള്ള ജനങ്ങളിൽ നിന്നുള്ള ഏറ്റവും ഉത്തമർ. കഴുത്തുകളിൽ ചങ്ങലകളണിയപ്പെട്ട നിലക്ക് അവരെ കൊണ്ട് വരും; അവർ ഇസ്‌ലാം സ്വീകരിക്കും വരെ.

ഈ രണ്ടാമത്തെ നിവേദനം അബൂ ഹുറൈറയുടെ (റ) വാചകമാണ്, മുഹമ്മദ് നബിയുടെ (സ) ഹദീസ് അല്ല എന്ന് സാന്ദർഭികമായി സൂചിപ്പിക്കട്ടെ. ഈ നിവേദനത്തിന്റെയും ഉള്ളടക്കം മുമ്പത്തെ ഹദീസിന്റെ ആശയം തന്നെയാണ് എന്നതിൽ സംശയമില്ല. അവിശ്വാസികളെ ചങ്ങലകളിൽ തളക്കാനൊ നിർബന്ധിതമായി മതപരിവർത്തനത്തിന് വിധേയമാക്കാനൊ രണ്ട് നിവേദനങ്ങളിലും ഒരു സൂചനയും ഇല്ല.

ഈ നിവേദനങ്ങൾക്ക് മുസ്‌ലിം പണ്ഡിതന്മാർ നൽകിയ രണ്ട് വ്യാഖ്യാനങ്ങൾ ശ്രദ്ധിച്ചാൽ തന്നെ ഈ വസ്തുത സുതരാം വ്യക്തമാവുന്നതാണ്.

വ്യാഖ്യാനം: ഒന്ന്

وقال غيره يحتمل أن يكون المراد المسلمين المأسورين عند أهل الكفر يموتون على ذلك أو يقتلون فيحشرون كذلك وعبر عن الحشر بدخول الجنة لثبوت دخولهم عقبة والله أعلم സത്യനിഷേധികളുടെ അടുക്കൽ ബന്ദികളായി ജീവിക്കുകയും അതെ അവസ്ഥയിൽ മരണപ്പെടുകയൊ കൊല്ലപ്പെടുകയൊ ചെയ്യുന്ന മുസ്‌ലിംകളാണ് ഹദീസിന്റെ താൽപര്യമെന്ന് പല പണ്ഡിതന്മാരും ഹദീസിനെ വ്യാഖ്യാനിക്കുന്നു. അത്തരം മുസ്‌ലിംകൾ അവർ മരണപ്പെട്ട ബന്ധനാവസ്ഥയിൽ തന്നെ പുനർജീവിപ്പിക്കപ്പെടുകയും ചെയ്യുമെന്നർത്ഥം. (ഫത്ഹുൽ ബാരി: 6:101)

വ്യാഖ്യാനം: രണ്ട്

قال ابن الجوزي معناه انهم أسروا وقيدوا فلما عرفوا صحة الاسلام دخلوا طوعا فدخلوا الجنة فكان الاكراه على الأسر والتقييد هو السبب الأول وكانه أطلق على الاكراه التسلسل ولما كان هو السبب في دخول الجنة أقام المسبب مقام السبب

ഇബ്നുൽ ജൗസി പറഞ്ഞു: ഹദീസിന്റെ ഉദ്ദ്യേശമിതാണ്: അവർ യുദ്ധങ്ങളിൽ പിടിക്കപ്പെടുകയും ബന്ദികളാക്കപ്പെടുകയും ചെയ്ത (അമുസ്‌ലിംകളായ) വരാണ്. എന്നാൽ പിന്നീട് ഇസ്‌ലാമിന്റെ സത്യത അവർ സ്വമേധയാ തിരിച്ചറിയുകയും സ്വേച്‌ഛയാൽ ഇസ്‌ലാം സ്വീകരിക്കുകയും ചെയ്തു. അവർ ഇസ്‌ലാം സ്വീകരിച്ചത് നിർബന്ധിതമായിട്ടല്ലെങ്കിലും ഇസ്‌ലാമിനെ (അവർ അറിയാനും ആശ്ലേഷിക്കാനും) വഴിവെച്ചത് ബന്ധനമാണ് എന്നതിനാൽ ബന്ധനത്തെ സ്വർഗപ്രവേശനത്തിന്റെ (യും ഇസ്‌ലാം ആശ്ലേഷണത്തിന്റെയും) കാരണമായി ആലങ്കാരികമായി പ്രയോഗിക്കപ്പെട്ടിരിക്കുകയാണ് ഇവിടെ. (ഫത്ഹുൽ ബാരി: 6:101)

പ്രവാചകാനുചരൻ അബൂ അസീസ് ഇബ്നു ഉമൈറിന്റെ (റ) ജീവിതാനുഭവം ഈ വ്യാഖ്യാനത്തിന് ഒരു ഉദാഹരണമാണ്. അദ്ദേഹം (റ) പറഞ്ഞു: ബദ്ർ യുദ്ധാനന്തരം (ശത്രു പക്ഷത്ത് നിന്ന്) ബന്ദികളാക്കപ്പെട്ടവരിൽ ഞാനും ഉണ്ടായിരുന്നു. (അദ്ദേഹം പിന്നീടാണ് ഇസ്‌ലാം സ്വീകരിക്കുന്നത്) അപ്പോൾ അല്ലാഹുവിന്റെ ദൂതർ മുസ്‌ലിംകളോട് കൽപ്പിച്ചു: “ബന്ദികളോട് നന്മ ചെയ്യാൻ ഞാൻ നിങ്ങളോട് അനുശാസനം നല്‍കുന്നു”. ഞാൻ അൻസ്വാരികളുടെ ഒരു കൂട്ടത്തിലാണ് (ബന്ധനസ്ഥനായ നിലക്ക്) ഉണ്ടായിരുന്നത്. രാവിലേയും വൈകുന്നേരവും അവരുടെ അടുക്കൽ ഭക്ഷണം കൊണ്ടു വരപ്പെടുമ്പോഴെല്ലാം -ബന്ദികളോട് നന്മ ചെയ്യാനുള്ള പ്രവാചകന്റെ അനുശാസനം പരിഗണിച്ച് – അവർ ഈത്തപഴം ഭക്ഷിക്കുകയും എനിക്ക് റൊട്ടി നൽകുകയും ചെയ്യുമായിരുന്നു. (ത്വബ്റാനി: മുഅ്ജമു സ്വഗീർ: 409, അൽ ഖബീർ: 977, മജ്മഉ സവാഇദ്: 10007)

(ബദർ യുദ്ധം നടത്തിയതും തുടക്കം കുറിച്ചതും സത്യനിഷേധികളാണ് എന്ന് സാന്ദർഭികമായി സൂചിപ്പിക്കട്ടെ. യുദ്ധത്തിൽ സത്യനിഷേധികൾ പരാചയപ്പെടുകയും ബന്ദികളാക്കപ്പെടുകയും ചെയ്തപ്പോൾ കൂട്ടത്തിൽ പ്രവാചകാനുചരൻ ഇബ്നു ഉമൈറും (റ) ഉണ്ടായിരുന്നു).

ഇസ്‌ലാമിന്റെ നന്മകൾ അടുത്തറിയാനും മുസ്‌ലിംകളിലെ മൂല്യങ്ങൾ അനുഭവിച്ചറിയാനും ഇസ്‌ലാം മതത്തിൽ ആകൃഷ്ടനാവാനും തുടർന്ന് സ്വേച്‌ഛ പ്രകാരം മതത്തെ സ്വീകരിക്കാനും അദ്ദേഹത്തിന് നിമിത്തമായത് ബദർ യുദ്ധാനന്തരമുണ്ടായ ബന്ധനമാണ്. ഇങ്ങനെ ഇസ്‌ലാമാശ്ലേഷിച്ച, അബൂ അസീസ് ഇബ്നു ഉമൈറിനെ (റ) പോലെയുള്ളവരെ സംബന്ധിച്ചാണ് ഹദീസിലെ ഉദ്ദേശ്യം എന്നതാണ് രണ്ടാമത്തെ വ്യാഖ്യാനം.

ഇസ്‌ലാം സ്ത്രീകളെ ആദരിച്ച മതമാണെന്നത് മുസ്‌ലീംകളുടെ കേവലം അവകാശവാദം മാത്രമാണ്. കാരണം പ്രവാചകന്‍ തന്നെ പല ഘട്ടങ്ങളിലും സ്ത്രീകളെ വളരെ മോശമായ രൂപത്തില്‍ ആക്ഷേപിച്ചു സംസാരിച്ചത് ഹദീസ് ഗ്രന്ഥങ്ങളില്‍ തന്നെ രേഖപ്പെടുത്തപ്പെട്ടിരിക്കുന്നു. സ്ത്രീയും നായയും കഴുതയും നമസ്‌ക്കാരത്തെ മുറിച്ചുകളയുമെന്ന നബി പാഠം തന്നെ തികഞ്ഞ സ്ത്രീവിരുദ്ധത നിറഞ്ഞു നില്‍ക്കുന്ന പരാമര്‍ശമാണ്. നായയോടും കഴുതയോടും പെണ്ണിനെ ഉപമിക്കുക വഴി തികഞ്ഞ സ്ത്രീവിരുദ്ധ പരാമര്‍ശമാണ് പ്രവാചകന്‍ നടത്തിയിരിക്കുന്നത്. പിന്നെ എങ്ങനെയാണ് ഇസ്‌ലാം സ്ത്രീകളെ ആദരിച്ച മതമാണെന്ന് മുസ്‌ലീംകള്‍ക്ക് വാദിക്കാനാവുന്നത് ?

മറുപടി:

അധമവിചാരികള്‍ക്ക് സത്യം എപ്പോഴും അപ്രാപ്യമായിരിക്കുമെന്ന നിരീക്ഷണം എത്രയോ വസ്തുതാപരമാണെന്ന് ഈ വിമര്‍ശനം വിളിച്ചറിയിക്കുന്നുണ്ട്. ഒറ്റ വായനയില്‍ തന്നെ സാമാന്യബുദ്ധിക്കു ഗ്രാഹ്യമായ ഒരു ആശയത്തെ എത്രമാത്രം സാഹസപ്പെട്ടാണ് ഇസ്‌ലാംവിരോധികളിവിടെ വികൃതവല്‍ക്കരിച്ചിരിക്കുന്നത്. വിമര്‍ശകരുടെ “വക്രീകര ശാസ്ത്ര സാഹസം” എത്രമാത്രം ബാലിശമാണെന്ന് മനസ്സിലാക്കാനായി നമുക്കൊരു സര്‍ക്കാര്‍ മാര്‍ഗനിര്‍ദ്ദേശ പത്രിക ഉദാഹരണമായെടുക്കാം. കോവിഡ് 19 വ്യാപനം തടയുന്നതിന്റെ ഭാഗമായി കേന്ദ്ര-സംസ്ഥാന സര്‍ക്കാറുകള്‍ പുറത്തിറക്കിയ പ്രസ്തുത മാര്‍ഗനിര്‍ദ്ദേശ പത്രികയില്‍ പരാമര്‍ശിച്ച പ്രത്യേക സംരക്ഷണം നല്‍കേണ്ടവരുടെ പട്ടികയില്‍ ഇപ്രകാരം കാണാം: “കുട്ടികള്‍, അംഗപരിമിതര്‍, മാനസിക രോഗികള്‍, സ്ത്രീകള്‍…” ഈ പത്രിക ചൂണ്ടികാട്ടി ഇസ്‌ലാംവിരോധികളാരെങ്കിലും പറഞ്ഞേക്കുമോ, കേന്ദ്ര-സംസ്ഥാന സര്‍ക്കാറുകള്‍ സ്ത്രീകളെയും മാനസിക രോഗികളെയും ഒരേ തരത്തിലാണ് കാണുന്നത്. അതുകൊണ്ടാണ് അവരെ ഒരേ പട്ടികയില്‍ തന്നെ ചേര്‍ത്തതെന്ന്!. വാസ്തവത്തില്‍ ഇസ്‌ലാംവിരോധികളുടെ ഹദീസ് വിമര്‍ശനവും ഈ വിഢിത്തത്തിനപ്പുറം ഒന്നുമല്ലെന്ന് വിമര്‍ശന വിധേയമായ ഹദീസിന്റെ യാഥാര്‍ഥ്യം കൃത്യമായി പരിശോധിച്ചാല്‍ തന്നെ ബോദ്ധ്യമാകുന്നതാണ്. അതിനാല്‍ നമുക്ക് ഇസ്‌ലാംവൈരികള്‍ വിമര്‍ശനവിധേയമാക്കിയ ഹദീസും അതിന്റെ താല്‍പര്യവും മനസ്സിലാക്കാം.

“അബൂഹുറയ്‌റ (റ) പറയുന്നു: റസൂല്‍ (സ്വ) പറഞ്ഞു: സ്ത്രീയും കഴുതയും നായയും നമസ്‌ക്കാരം മുറിക്കും. ഒട്ടകക്കട്ടിലിന്റെ പിന്നിലെ വടിപോലുള്ള ഒന്ന് (ഉണ്ടായാല്‍) അത് തടുക്കാവുന്നതാണ്.” (മുസ്‌ലിം 790, അഹ്‌മദ്, ഇബ്‌നു അബീശൈബ, ഇബ്‌നു ഹിബ്ബാന്‍) ഈ ഹദീസ് വായിച്ചപ്പോള്‍ സ്ത്രീയെ മൃഗങ്ങളോട് ഉപമിച്ചതായി ഇസ്‌ലാംവിരോധികള്‍ക്കു തോന്നി എന്നതല്ലാതെ മറ്റൊരു പ്രശ്നവുമിവിടെയില്ല. ഇസ്‌ലാംവിരോധികള്‍ക്കു തോന്നുന്ന അധമവിചാരങ്ങളും വികാരങ്ങളും കൊണ്ടുതള്ളാനുള്ള പുറമ്പോക്കല്ല ഇസ്‌ലാമെന്ന് അവര്‍ തിരിച്ചറിഞ്ഞാല്‍ തീരാവുന്ന പ്രശ്‌നമേ ഇവിടെയുള്ളൂ. ഈ ഹദീസിനെ പറ്റി പരാമര്‍ശിക്കവെ ഇമാം ക്വുര്‍ത്തുബിയുടെ(റ) വാചകങ്ങള്‍ “അല്‍ മുഫ്ഹിം ലിമാ അശ്കല മിന്‍ തല്‍ഖീസ്വി സ്വഹീഹ് മുസ്‌ലിമി”ല്‍ ഇപ്രകാരം രേഖപ്പെടുത്തിയിരിക്കുന്നു: “കാരണം തീര്‍ച്ചയായും സ്ത്രീ (സാന്നിദ്ധ്യം പുരുഷമനസ്സുകളില്‍) പ്രലോഭനമുണ്ടാക്കും, കഴുതയാകട്ടെ ഒച്ചയുണ്ടാക്കുകയും നായ ഭയം സൃഷ്ടിക്കുകയും ചെയ്യും. അപ്പോള്‍ (നമസ്‌ക്കരിക്കുന്ന) ചിന്താശേഷിയുള്ള വ്യക്തിയില്‍ (അവയുടെ സാന്നിദ്ധ്യം) ആശയക്കുഴപ്പം സൃഷ്ടിക്കുകയും അവന്റെ നമസ്‌ക്കാരം മുറിഞ്ഞു പോകുമാറ് കുഴപ്പമുണ്ടാക്കുകയും ചെയ്യും. അപ്പോള്‍ നമസ്‌ക്കാരത്തെ (ഏകാഗ്രത) മുറിച്ചു കളയുന്നതിലേക്ക് കാര്യങ്ങള്‍ എത്തുന്നതുകൊണ്ടാണ് (അവയെ) നമസ്‌ക്കാരം മുറിച്ചുകളയുന്ന കാര്യങ്ങളിലുള്‍പ്പെടുത്തിയത്.” (അല്‍ മുഫ്ഹിം ലിമാ അശ്കല മിന്‍ തല്‍ഖീസ്വി സ്വഹീഹ് മുസ്‌ലിം: 2/ 109)

“നമസ്ക്കാരത്തെ മുറിക്കും” എന്നാൽ നമസ്ക്കാരത്തിലെ ഏകാഗ്രത, പൂർണ്ണത മുറിക്കുകയും കുറക്കുകയും ചെയ്യുമെന്നാണ് വിവക്ഷയെന്ന് ഒട്ടുമിക്ക പണ്ഡിതരും വ്യക്തമാക്കിയിട്ടുണ്ട്.

وقال مالك وأبو حنيفة والشافعي رضي الله عنهم وجمهور من السلف والخلف: لا تبطل الصلاة بمرور شيء من هؤلاء ولا من غيرهم، وتأول هؤلاء هذا الحديث على أن المراد بالقطع نقص الصلاة لشغل القلب بهذه الأشياء وليس المراد إبطالها

“ഇമാം മാലിക്, അബൂ ഹനീഫ, ശാഫിഈ തുടങ്ങി പൂർവ്വീകരും ശേഷക്കാരുമായ ഭൂരിഭാഗം പണ്ഡിതന്മാരും പറയുന്നത്: ഈ വിഭാഗക്കാരുൾപ്പെടെ ഒന്നും തന്നെ നമസ്ക്കാരത്തെ നിഷ്ഫലമാക്കും എന്ന അർത്ഥത്തിൽ ‘മുറിക്കും’ എന്നല്ല. നമസ്ക്കാരത്തെ മുറിക്കും എന്നാൽ, ഇത്തരം കാര്യങ്ങളിൽ മനസ്സ് വ്യാപൃതമാവുക വഴി ഏകാഗ്രത മുറിയുകയും നമസ്ക്കാരത്തിൽ ന്യൂനത സംഭവിക്കും എന്നുമാണ്.” (തുഹ്ഫതുൽ അഹ്‌വദി: 1:371)

നമസ്കാരത്തെ മുറിക്കും എന്നാൽ നമസ്ക്കാരത്തിന്റെ ഭയ ഭക്തിക്ക് ഭംഗം വരുത്തും എന്നാണ്. അല്ലാതെ നമസ്ക്കാരത്തിൽ നിന്ന് പുറത്താവുമെന്നല്ല. (ഫത്ഹുൽ ബാരി: 2:273)

ഇമാം അഹ്‌മദ് പറഞ്ഞു: നമസ്ക്കരിക്കുന്നതിനിടയിൽ തന്റെ തൊട്ട് മുമ്പിൽ ഒരു നായയൊ ഒരു സ്ത്രീയൊ നിൽക്കുമ്പോൾ മനസ്സിൽ നമസ്കാരമല്ലാത്ത മറ്റു ചിന്തകൾ കടന്നുവന്നേക്കാം…. (ഫത്ഹുൽ ബാരി: 2:273)

സുവ്യക്തമാണ് കാര്യങ്ങള്‍, വിമര്‍ശന വിധേയമായ ഹദീസ് ഒരിക്കലും സ്ത്രീയെ മൃഗങ്ങളോട് ഉപമിക്കുകയല്ല ചെയ്തിരിക്കുന്നത്, മറിച്ച് ഒരു പുരുഷന്റെ നമസ്‌ക്കാരത്തില്‍ ഭംഗമുണ്ടാക്കുന്ന കാര്യങ്ങളെ പറ്റി പറയുമ്പോള്‍ ആ കൂട്ടത്തില്‍ സ്ത്രീയേയും പരാമര്‍ശിച്ചെന്ന് മാത്രം. നമസ്‌ക്കരിക്കുന്ന മനുഷ്യനില്‍, നായ ഭയം ജനിപ്പിച്ചും കഴുത ഒച്ചവെച്ചും നമസ്‌ക്കാരത്തിലെ ശ്രദ്ധമുറിച്ചു കളയാന്‍ സാധ്യതയുള്ളതു പോലെ സ്ത്രീ സാന്നിദ്ധ്യം പുരുഷ ഹൃദയങ്ങളില്‍ പ്രലോഭനങ്ങള്‍ സൃഷ്ടിക്കാനും തന്നിമിത്തം നമസ്‌ക്കാരത്തിലെ ശ്രദ്ധ മുറിയുവാനും ഇടയാക്കുമെന്ന് പഠിപ്പിക്കുക മാത്രമേ ഹദീസ് ചെയ്യുന്നുള്ളൂ. നായ, കഴുത, സ്ത്രീ എന്നിവയുടെ സാന്നിദ്ധ്യം നിരുപാധികം നമസ്‌കാരം മുറിച്ചു കളയുമെന്ന് ഹദീസ് പഠിപ്പിക്കുന്നില്ല.

അപ്പോള്‍ അസ്തിത്വത്തിലെ നീചത്വം കൊണ്ടാണ് ഇവയെല്ലാം നമസ്‌കാരം മുറിച്ചുകളയുന്നതെന്ന് ഹദീസ് ഒരിക്കലും പഠിപ്പിക്കുന്നില്ല മറിച്ച് അവരുടെ സാന്നിദ്ധ്യം സൃഷ്ടിച്ചേക്കാവുന്ന ശ്രദ്ധതെറ്റിക്കലാണ് ഹദീസിന്റെ പ്രതിപാദനം. അങ്ങനെ വരുമ്പോള്‍ വാസ്തവത്തില്‍ ഇവിടെ സ്ത്രീ നിന്ദിക്കപ്പെടുകയാണോ ചെയ്യുന്നത്?! പരോക്ഷമായ സ്തുതിയാണ് ഹദീസിൽ നിന്ന് സ്ത്രീത്വത്തിന് ലഭിക്കുന്നത്. നമസ്‌കാര സമയത്തു പോലും പുരുഷ ഹൃദയങ്ങളെ ആകര്‍ഷിക്കുവാനും സ്വാധീനിക്കുവാനും പ്രലോഭിപ്പിക്കുവാനും സ്ത്രീ സൗന്ദര്യത്തിനു സാധിക്കുമെന്ന് പറയുന്നത് സ്ത്രീകളെ അവമതിക്കലാണെന്ന് തെറ്റുദ്ധരിക്കാന്‍ മാത്രം സാധുക്കളല്ല ഇസ്‌ലാംവിരോധികളെന്ന് ആര്‍ക്കും പ്രത്യേകം പറഞ്ഞു തരേണ്ടതില്ലല്ലോ?. ഇത്രയേ ഉള്ളൂ, ഇസ്‌ലാംവിരോധികള്‍ ഹദീസിനുമേല്‍ അടയിരുന്നു വിരിയിച്ച അധമവിചാരങ്ങളുടെ അവസ്ഥ. ഹദീസുകളല്ല; ഇസ്‌ലാംവിരോധികളുടെ അന്തഃരംഗമാണ് ഇരുട്ട് പരത്തുന്നതെന്നര്‍ഥം. ഹദീസുകളെടുത്തുവെച്ച് തങ്ങളുടെ വികൃതമായ താല്‍പര്യങ്ങള്‍ക്കനുസൃതമായി അതിനെ വ്യാഖ്യാനിക്കുകയും എന്നിട്ട് അത് പ്രവാചകനും ഇസ്‌ലാമിനും മേല്‍ ആരോപിച്ച് പുളകം കൊള്ളുകയും ചെയ്യുക എന്നത് ഇസ്‌ലാംവിരോധികളുടെ സ്ഥിരം പതിവാണ്. വൈജ്ഞാനിക സത്യസന്ധതയില്ലാത്തവര്‍ക്ക് എന്തുമാകാമല്ലോ. പക്ഷെ ഇത്തരം അൽപജ്ഞാനികളുടെ “ശര്‍ഹ്” കേട്ടപ്പോഴേക്കും ചിന്താശേഷിയില്‍ എട്ടുകാലിവല കെട്ടിയ “മഹാ പിള്ള”മാരുടെ കാര്യമാണ് കഷ്ടം.! നബി(സ)യില്‍ നിന്നും നിവേദനം ചെയ്യപ്പെട്ട ഹദീസുകള്‍ ഇസ്‌ലാമിക പണ്ഡിത ലോകം എപ്രകാരമാണ് ഗ്രഹിച്ചതും ഉള്‍കൊണ്ടതുമെന്ന അന്വേഷണത്തില്‍ നിന്നു വേണം സത്യസന്ധമായ ഹദീസ് വിമര്‍ശനം തുടങ്ങേണ്ടതെന്ന വൈജ്ഞാനിക മര്യാദ പോലും ഇസ്‌ലാംവിരോധികള്‍ക്കില്ലെന്ന വസ്തുതക്ക് ഒന്നു കൂടി അടിവരയിടുകയാണിവിടെ. എന്തിനാണ് തങ്ങള്‍ ഹദീസുകളെ വിമര്‍ശിക്കുന്നതെന്ന ബോധം പോലും ഇസ്‌ലാംവിമര്‍ശകര്‍ക്കു നഷ്ടപ്പെട്ടിരിക്കുന്നു. ഒരു ഹദീസ് വായിച്ചപ്പോള്‍ ഇസ്‌ലാംവിരോധികള്‍ക്കു തോന്നിയ അധമവിചാരത്തെ അടിസ്ഥാനപ്പെടുത്തിയല്ല ഹദീസുകള്‍ മനസ്സിലാക്കപ്പെടേണ്ടത്; മറിച്ച് ഇസ്‌ലാമിക പണ്ഡിത ലോകം അതിനെ എപ്രകാരമാണ് ഗ്രഹിച്ചതും വ്യാഖ്യാനിച്ചതും ഉള്‍കൊണ്ടതുമെന്നതിന്റെ അടിസ്ഥാനത്തിലായിരിക്കണം. കാരണം അതാണല്ലോ നബിപാഠങ്ങള്‍ ഏതു തരം സ്വാധീനമാണ് സമൂഹത്തിലുണ്ടാക്കിയത് എന്ന് തിരിച്ചറിയാനുള്ള പ്രത്യക്ഷ മാര്‍ഗം. തെറ്റായ യാതൊരു സ്വാധീനവും സമൂഹത്തില്‍ സൃഷ്ടിച്ചതായി അവകാശപ്പെടാന്‍ സാധ്യമല്ലാത്ത ഒരു ഹദീസ്, ഇസ്‌ലാംവിരോധവായനകള്‍ക്കു തോന്നിയ അധമവിചാരങ്ങളെ അടിസ്ഥാനപ്പെടുത്തി വിമര്‍ശന വിധേയമാക്കുക എന്നത് വൈജ്ഞാനിക സത്യസന്ധതക്ക് ഒരിക്കലും ചേര്‍ന്ന സമീപനമല്ല.

ഹദീസുകള്‍ പെണ്ണിനെ നായയോടും കഴുതയോടും ഉപമിച്ചു എന്ന ഇസ്‌ലാംവിരോധികളുടെ വിമര്‍ശനം പാടേ നുള്ളി കളഞ്ഞത് പ്രവാചക പത്‌നിയായ ആഇശ (റ) തന്നെയാണ്. അതും പ്രവാചകനുമായുള്ള തന്റെ സ്വന്തം അനുഭവത്തെ മുന്‍ നിര്‍ത്തികൊണ്ട്. പ്രസ്തുത സംഭവം ഹദീസ് ഗ്രന്ഥങ്ങളില്‍ നിന്നു തന്നെ നമുക്കു വായിക്കാം.

“ആഇശ(റ)യുടെ അടുക്കല്‍വച്ച്, നമസ്‌ക്കാരത്തെ മുറിക്കുന്നവയാണ് നായ, കഴുത, സ്ത്രീ എന്നിവയെന്ന് പറയപ്പെടുകയുണ്ടായി. അന്നേരം അവര്‍ ചോദിച്ചു: കഴുതകളോടും നായ്ക്കളോടുമാണോ നിങ്ങള്‍ ഞങ്ങളെ സാദൃശ്യപ്പെടുത്തിയിരിക്കുന്നത്? അല്ലാഹുവാണ്, നബി(സ്വ)ക്കും ക്വിബ്‌ലക്കുമിടയിലെ കട്ടിലില്‍ ഞാന്‍ കിടക്കവെ, അവിടുന്ന് നമസ്‌കരിക്കുന്നത് ഞാന്‍ കണ്ടിട്ടുണ്ട്. അന്നേരം എനിക്ക് (വിസര്‍ജനത്തിന്) ആവശ്യം ഉണ്ടാകും. അപ്പോള്‍ എഴുന്നേറ്റിരുന്ന് നബിക്ക് പ്രയാസം വരുത്താന്‍ ഞാന്‍ ഇഷ്ടപ്പെടാത്തതിനാല്‍ കട്ടിലിന്റെ കാലുകളുടെ ഭാഗത്തുകൂടെ ഞാന്‍ ഊര്‍ന്നിറങ്ങും.” (ബുഖാരി 484, മുസ്‌ലിം)

നമസ്‌കരിക്കുന്നവനില്‍ പ്രലോഭനമുണര്‍ത്തി അതിനു ഭംഗം വരുത്താവുന്ന ഒരു കാര്യമായി സ്ത്രീയെ പരാമര്‍ശിച്ചു എന്നതല്ലാതെ, പ്രവാചകന്‍ ഒരിക്കലും സ്ത്രീയെ നായയോടും കഴുതയോടും ഉപമിച്ചിട്ടില്ല. അഥവാ, അസ്തിത്വത്തിലെ നീചത്വം കൊണ്ടാണ് സ്ത്രീ സാന്നിധ്യംമൂലം നമസ്‌കാരം മുറിഞ്ഞുപോകുന്നതെന്ന് പ്രവാചകൻ (സ) പഠിപ്പിച്ചിട്ടില്ലെന്നർത്ഥം. മറിച്ച് അവരുടെ സാന്നിദ്ധ്യം സൃഷ്ടിച്ചേക്കാവുന്ന ശ്രദ്ധതെറ്റിക്കലാണ് പ്രവാചകൻ (സ) ഉദ്ദേശിച്ചത്. അതു വ്യക്തമാക്കി കൊടുക്കാനാണ്, തന്റെ സാന്നിധ്യം തൊട്ടുമുമ്പിൽ ഉണ്ടായിരിക്കെയാണ് പ്രവാചകൻ (സ) പലപ്പോഴും നമസ്ക്കാരം നിർവ്വഹിച്ചിരുന്നതെന്നു ആഇശ (റ) വ്യക്തമാക്കിയതും.

സ്ത്രീ സാന്നിദ്ധ്യം നിരുപാധികം നമസ്‌കാരം മുറിക്കുന്ന കാര്യമായിരുന്നെങ്കില്‍, നബിയുടെ മുമ്പില്‍ സ്ത്രീയായ ആഇശ (റ) പല തവണ നിവര്‍ന്നു കിടന്നിട്ടും ഒരിക്കല്‍ പോലും അതിന്റെ പേരില്‍ തന്റെ നമസ്‌കാരം മുറിച്ചെന്നു പറഞ്ഞ് നബി(സ്വ) അവരെ ആക്ഷേപിച്ചിട്ടില്ല. മാത്രമല്ല നബിയുടെ മുമ്പില്‍ അവരെങ്ങനെ കിടന്നുവോ അതേ രൂപത്തില്‍ തന്നെ അവരെ തുടരാന്‍ അനുവദിച്ചുകൊണ്ടു തന്നെ തന്റെ നമസ്‌കാരം പൂര്‍ത്തീകരിക്കുകയാണ് നബി (സ്വ) ചെയ്തതെന്ന ആഇശയുടെ അനുഭവ സാക്ഷ്യം ഇസ്‌ലാംവിരോധികളുടെ വിമര്‍ശനത്തില്‍ യാതൊരു കഴമ്പുമില്ലെന്ന് വ്യക്തമാക്കുന്നതാണ്. എങ്കില്‍ പിന്നെ അനാവശ്യമായ ഇത്തരം ഉണക്ക വേവലാതികള്‍ മാറ്റിവെച്ച് കാര്യപ്പെട്ട വല്ല ഏര്‍പ്പാടിനും സമയം കണ്ടെത്തുന്നതല്ലേ ഇസ്‌ലാംവിരോധികള്‍ക്കു നല്ലത്. പക്ഷെ, അങ്ങനെയൊക്കെ നേരെ ചൊവ്വായ അര്‍ഥതലങ്ങളൊന്നും ആരും ഹദീസുകള്‍ക്കു മനസ്സിലാക്കികൂടാ എന്നത് ഇസ്‌ലാംവിരോധികളുടെ നിതാന്ത നികൃഷ്ഠ നിര്‍ബന്ധബുദ്ധിയാണ്. അതില്‍ സംഖി, കൃസംഖി, എമു, ഫെമിനി, യുക്തിവാദി… തുടങ്ങിയ ഒരു ജനുസ്സിലും പെട്ട ഇസ്‌ലാംവിരോധികളാരും വ്യത്യസ്തരല്ല എന്നതാണ് വസ്തുത.

വിമർശനം:

പ്രവാചക പത്നി ഹഫ്‌സയുടെ വീട്ടിൽ വെച്ച് മാരിയത്ത് എന്ന അടിമ സ്ത്രീയുമായി മുഹമ്മദ് നബി ലൈംഗിക ബദ്ധത്തിൽ ഏർപ്പെട്ടു. ഇത് ഹഫ്‌സ അറിഞ്ഞപ്പോൾ അവരോട് അക്കാര്യം ആരുമായും പങ്കു വെക്കരുത് എന്ന് നബി അപേക്ഷിക്കുകയും ചെയ്തു.

മറുപടി:

ഒരു വ്യാജ നിവേദനത്തിൻമേൽ കെട്ടിപടുത്ത കാൽപനിക സൗധമാണ് ഈ കഥ. പ്രവാചകൻ (സ), തന്റെ പത്നി ഹഫ്‌സയുടെ വീട്ടിൽ വെച്ച് “അവിഹിത”ത്തിൽ ഏർപ്പെടുകയും ഭാര്യ “പിടികൂടിയപ്പോൾ” ഈ “അവിഹിതം” രഹസ്യമാക്കി വെക്കാൻ ഭാര്യയോട് കേണപേക്ഷിച്ചു എന്നൊക്കെയാണ് വിമർശകരുടെ തിരക്കഥ!! അതിനായി അവർ അവലംബിക്കുന്ന വ്യാജവും അങ്ങേയറ്റം ദുർബലവുമായ നിവേദനങ്ങളിലാകട്ടെ വിമർശകരുടെ ഈ ഭാവനാചിത്രണങ്ങൾ ഒന്നും ഉള്ളടങ്ങിയിട്ടില്ല എന്നതാണ് യാഥാർത്ഥ്യം.

വിഷയകമായ ദുർബല ഹദീസിന്റെ ഹ്രസ്വരൂപം ആദ്യം നമുക്ക് പരിചയപ്പെടാം: പ്രവാചകന്റെ(സ) അടിമസ്ത്രീയായിരുന്നു മാരിയ. ഒരിക്കൽ ഹഫ്‌സയുടെ വീട്ടിൽ വെച്ച് പ്രവാചകൻ (സ) അവരുമായി ശാരീരിക ബന്ധത്തിലേർപ്പെട്ടു. ഇത് മനസ്സിലാക്കിയപ്പോൾ ഹഫ്‌സ (റ) പ്രവാചകനോട് ഇപ്രകാരം പരാധിപ്പെട്ടു:

“താങ്കൾ എന്റെ വീട്ടിൽ വെച്ച് ‘മാരിയ’യുമായി ബന്ധപ്പെട്ടു എന്നത് (എന്നോട് താങ്കൾക്കുള്ള അതൃപ്തിയായി) മറ്റുള്ള ഭാര്യമാർക്കിടയിൽ (തെറ്റിദ്ധാരണ ജനിപ്പിച്ചേക്കാം എന്നതിനാൽ) എനിക്ക് അപമാനത്തിന് കാരണമായേക്കും.”

അപ്പോൾ പ്രവാചകൻ (സ) ‘ഈ സംഭവം ആരോടും പങ്കു വെക്കേണ്ട’ എന്ന് അവരെ ഉപദേശിച്ചു. തുടർന്ന് ‘മാരിയ’യുമായി മേലിൽ ശാരീരിക ബന്ധം സ്ഥാപിക്കില്ലെന്നും പ്രവാചകൻ (സ) പ്രസ്ഥാവിച്ചു. അപ്പോൾ ഹഫ്‌സ ചോദിച്ചു: ( وكيف تحرم عليك وهي جاريتك) “അവരുമായുള്ള ശാരീരിക ബന്ധം താങ്കൾക്കെങ്ങനെ നിഷിദ്ധമാകും; അവർ താങ്കളുടെ അടിമ സ്ത്രീ ആയിരിക്കെ ?!”

‘മാരിയ’യെ സമീപിക്കില്ലെന്ന് ഹഫ്‌സയോട് ശപഥം ചെയ്ത പ്രവാചകന്റെ(സ) നിലപാട് തിരുത്തി കൊണ്ട് വിശുദ്ധ ക്വുർആനിൽ ചില വചനങ്ങൾ അവതരിപ്പിക്കപ്പെടുന്നു: “നബിയേ, നീയെന്തിനാണ് ഭാര്യമാരുടെ പ്രീതി കാംക്ഷിച്ച് അല്ലാഹു അനുവദനീയമാക്കിയത് നിഷിദ്ധമാക്കുന്നത്?”(സൂറത്തു തഹ്‌രീം: 1). (മുഅ്ജമുൽ അവ്സത്: ത്വബ്റാനി: 2316, സുനനു ദാറകുത്നി: 122)

മാരിയ പ്രവാചകന്റെ അടിമസ്ത്രീയാണ്. അടിമസ്ത്രീകളുമായി അവരുടെ ഹിതപ്രകാരം ബന്ധം സ്ഥാപിക്കാൻ ഇസ്‌ലാം പുരുഷന്മാർക്ക് അനുവാദം നൽകിയിട്ടുണ്ട്. (അടിമ സ്ത്രീകളുമായുള്ള ലൈംഗികബന്ധം ഇസ്‌ലാം അനുവദിച്ചതിലെ യുക്തിയും നീതിയും വിശാലമായ മറ്റൊരു ചർച്ചയാണ്. അതിന് ഇവിടെ മുതിരുന്നില്ല. ചുരുങ്ങിയ വായനക്കായി: (https://www.snehasamvadam.org/responds-to-criticism/അടിമസ്ത്രീകളുമായി-ബന്ധം/)

ഈ ബന്ധം ഒരു “അവിഹിത”മായി അക്കാലഘട്ടത്തിലെ ഒരു രാജ്യത്തിലേയും, ഒരു മതവും, ഒരു മത സമൂഹവും, മതരഹിതരുമടക്കം ആരും കരുതിയിട്ടില്ല എന്നിരിക്കെ ഇസ്‌ലാമിക സമൂഹത്തിൽ മാത്രം നിലനിന്നിരുന്ന “അവിഹിത”മായി ഈ ബന്ധത്തെ വീക്ഷിക്കുന്നതിലെ വർഗീയ വിവേചനം വ്യക്തമാണ്. പ്രവാചകന്റെ നാട്ടിലെ അമുസ്‌ലിംകൾ പോലും വിഹിതവും വിവാഹ തുല്യവുമായി കണ്ട ഒന്നാണ് അടിമസ്ത്രീ – യജമാന ബന്ധം എന്നതിന് ഏറ്റവും നല്ല തെളിവാണ് പ്രവാചക പത്നി ഹഫ്‌സയുടെ(റ) (ഹദീസിലെ) വാചകം: “‘മാരിയ’യുമായുള്ള ശാരിരിക ബന്ധം താങ്കൾക്കെങ്ങനെ നിഷിദ്ധമാകും; അവർ താങ്കളുടെ അടിമ സ്ത്രീ ആയിരിക്കെ ?!”.

പ്രവാചകൻ (സ) അവിഹിതത്തിൽ ഏർപ്പെട്ടുവെന്നോ, തന്റെ വീട്ടിൽ വെച്ച് അവിഹിതത്തിൽ മുഴുകിയെന്നോ ഒന്നും ഹഫ്‌സ (റ) പരാതിപ്പെട്ടിട്ടില്ല എന്നും (ദുർബല) ഹദീസിൽ നിന്ന് സുവ്യക്തം. തന്റെ വീട്ടിൽ വെച്ച് ‘മാരിയ’യുമായി ബന്ധപ്പെട്ടു എന്നത് തന്നോട് പ്രവാചകനുള്ള അതൃപ്തിയായി മറ്റുള്ള ഭാര്യമാർ തെറ്റിദ്ധരിക്കാൻ ഇടയാകുമെന്നും ഇത് തനിക്ക് അപമാനത്തിന് കാരണമായേക്കും എന്ന് മാത്രമാണ് ഹഫ്‌സയുടെ (റ) പരാതി. എങ്കിൽ ഇക്കാര്യം ആരുമായും പങ്കു വെക്കേണ്ടതില്ല എന്ന് പ്രവാചകൻ (സ) മറുപടി പറഞ്ഞു. അല്ലാതെ – വിമർശകർ ദുർവ്യാഖ്യാനിക്കുന്നതു പോലെ- തന്റെ “അവിഹിതം” ആരോടും വെളിപ്പെടുത്തരുതേ എന്ന ഭാര്യയോടുള്ള അപേക്ഷയൊന്നുമായിരുന്നില്ല അത്.

എല്ലാത്തിനുമുപരി വിമർശകർ ഓർക്കേണ്ട ഏറ്റവും സുപ്രധാനമായ കാര്യം മാരിയയുടെ ഈ കഥ സ്ഥിരപ്പെട്ട ഒരു സംഭവമൊ ഇസ്‌ലാമിന്റെയൊ പ്രവാചക ചരിത്രത്തിന്റെ ഭാഗമൊ അല്ല എന്നതാണ്.

മാരിയയുമായി ബന്ധപ്പെട്ട കഥയുടെ സനദ് അഥവാ നിവേദക പരമ്പര:

ഹഫ്‌സയുടെ (റ) വീട്ടിൽ വെച്ച് മാരിയയുമായി (റ) പ്രവാചകൻ (സ) ബന്ധത്തിലേർപ്പെട്ടു എന്ന വ്യാജ കഥയുടെ ഇരുപതോളം സനദുകൾ (നിവേദക പരമ്പര) ഓരോ മുടിനാരിഴ കീറി മുറിച്ച് പഠനം ത്വാഹിർ ഇബ്നു സഈദ് ഇബ്നു അബ്ദുർ റഊഫ് അസ്സയിൽഹീതി നടത്തിയിട്ടുണ്ട്. മാരിയയുടെ കഥയുമായി ബന്ധപ്പെട്ട എല്ലാ സനദുകളും വ്യാജങ്ങളൊ (الباطل), കള്ള ഹദീസുകളൊ (الموضوع) അങ്ങേയറ്റം ദുർബലമൊ (ضعيف جدا) ഒക്കെയാണ് എന്ന് തെളിവു സഹിതം സമർത്തിക്കുന്നുണ്ട് അദ്ദേഹം. സനദു സമ്പന്ധമായ ആ ദീർഘമായ ചർച്ച പഠനവിധേയമാക്കാൻ ആഗ്രഹിക്കുന്നവർ ആ ലേഖനം വായിക്കുക. (http://darhadith.com/pubs/asbab-nuzl-sura-tahrim-maria/)

ആയത്തിന്റെ ശരിയായ അവതരണ പശ്ചാത്തലം:

മാരിയയുമായി ബന്ധപ്പെട്ട് ചർച്ചാ വിഷയകമായ നിവേദനങ്ങൾ വ്യാജവും അങ്ങേയറ്റം ദുർബലവുമാണ് എന്നതിന് പുറമെ ഇസ്‌ലാമിക പ്രമാണങ്ങൾക്കും സ്വഹീഹായ ഹദീസുകൾക്കും എതിരുമാണ്. മാരിയയുമായി ബന്ധപ്പെട്ട (ദുർബല) നിവേദനത്തിൽ പ്രസ്ഥാവിക്കുന്നത്, സൂറത്തു തഹ്‌രീമിലെ ആയത്തുകൾ (ദിവ്യ വചനങ്ങൾ) അവതരിപ്പിക്കപ്പെടാനുള്ള പശ്ചാത്തലം മാരിയയുമായുള്ള ശാരീരിക ബന്ധം പ്രവാചകൻ (സ) സ്വന്തത്തിനു മേൽ നിഷിദ്ധമായി പ്രഖ്യാപിക്കുകയും, ശപഥം ചെയ്യുകയും ചെയ്തു എന്നതുമാണ്. ഇത് സ്വഹീഹായ ഒട്ടനവധി നിവേദനങ്ങൾക്കും ഹദീസുകൾക്കും വിരുദ്ധമാണ്.

സൂറത്തു തഹ്‌രീമിലെ ആയത്തുകളുടെ യഥാർത്ഥ അവതരണ പശ്ചാത്തലം ഇപ്രകാരമാണ്: മുഹമ്മദ് നബി (സ) തന്റെ പത്നിയായ സൈനബ് ബിൻത് ജഹ്ശിന്റെ (റ) അടുക്കൽ താമസിക്കുമ്പോൾ തേൻ കുടിക്കുക പതിവായിരുന്നു. ഇത് ഇഷ്ടപ്പെടാതിരുന്ന ആഇശ (റ), ഹഫ്‌സ (റ) എന്നീ പത്നിമാർ, തേൻ കുടിക്കുന്നത് മൂലം ‘മഗാഫിർ’ കുടിച്ചാലുണ്ടാവുന്ന ദുർഗന്ധം പോലെ അനുഭവപ്പെടുന്നു എന്ന് അഭിപ്രായപ്പെട്ടു. എങ്കിൽ താൻ ഇനി തേൻ കുടിക്കില്ലെന്ന് പ്രവാചകൻ (സ) ശപഥം ചെയ്തു. പ്രവാചകന്റെ(സ) ഈ നിലപാടിനെ തിരുത്തി കൊണ്ട് ക്വുർആൻ വചനം അവതരിപ്പിക്കപ്പെട്ടു: “നബിയേ, നീയെന്തിനാണ് ഭാര്യമാരുടെ പ്രീതി കാംക്ഷിച്ച് അല്ലാഹു അനുവദനീയമാക്കിയത് നിഷിദ്ധമാക്കുന്നത്?”(സൂറത്തു തഹ്‌രീം: 1). പ്രവാചക പത്നി ആഇശയിൽ (റ) നിന്ന് ഒട്ടനവധി ഹദീസ് ഗ്രന്ഥങ്ങളിൽ, സ്വഹീഹായ സനദോടു(നിവേദക പരമ്പര)കൂടെ ഉദ്ധരിക്കപ്പെട്ട ഹദീസ് ആകുന്നു ഇത്. (സ്വഹീഹുൽ ബുഖാരി: 4966, സ്വഹീഹു മുസ്‌ലിം: 2787, സുനനു അബൂദാവൂദ്: 3281, തുർമുദി: 1834, ഇബ്നുമാജ: 3342, മുസ്നദു അഹ്മദ്: 23769, ഇബ്നു ഹിബ്ബാൻ: 5344, നസാഈ: 3645, ഹാകിം: 7174, ദാരിമി: 2062)

മാരിയയുമായി പ്രവചകൻ (സ) ഹഫ്‌സയുടെ വീട്ടിൽ വെച്ച് ബന്ധപ്പെട്ടു എന്ന കഥ വ്യാജമൊ അങ്ങേയറ്റം ദുർബലമൊ ആകുന്നു എന്നാണ് ഹദീസ്-ചരിത്ര പണ്ഡിതന്മാർ അഭിപ്രായപ്പെട്ടത്:

ഉകൈലി (മരണം: ഹി: 322) പറഞ്ഞു: “(‘മാരിയ’യുമായി ബന്ധപ്പെട്ട) ഈ കഥ പ്രമാണ വിരുദ്ധവും വ്യാജവുമാണ്.” (മീസാനുൽ ഇഅ്തിദാൽ: 4:201)

ഇമാം ദഹബി (ജനനം: ഹി: 673) ഈ കഥ പ്രമാണ വിരുദ്ധവും നുണ കഥയുമാണെന്നും വിധി പറഞ്ഞിരിക്കുന്നു. (ലിസാനുൽ മീസാൻ: 8:191)

നൂറുദ്ദീൻ അൽ ഹൈസമി (ജനനം: ഹി: 735) ഈ കഥ ദുർബലമാണെന്ന് തന്റെ ‘മജ്‌മഉസ്സവാഇദ്’ (5:10) എന്ന ഗ്രന്ഥത്തിൽ മൂന്നിടങ്ങിൽ വ്യക്തമാക്കിയിരിക്കുന്നു.

ഇബ്നുൽ അറബി (ജനനം: ഹി: 558) പറഞ്ഞതായി ഇമാം ക്വുർതുബി രേഖപ്പെടുത്തുന്നു: “(‘മാരിയ’യുമായി ബന്ധപ്പെട്ട കഥയുടെ) ദുർബലത, അതിന്റെ നിവേദകന്മാർ വിശ്വസ്തരും നീതിമാന്മാരുമല്ല എന്നതാണ്.” (തഫ്സീറുൽ ക്വുർതുബി: 18: 179)

കാദി ഇയാദ് (ജനനം: ഹി: 476) പറഞ്ഞു: “നബിയേ, നീയെന്തിനാണ് ഭാര്യമാരുടെ പ്രീതി കാംക്ഷിച്ച് അല്ലാഹു അനുവദനീയമാക്കിയത് നിഷിദ്ധമാക്കുന്നത്?” എന്ന ക്വുർആൻ വചനം (സൂറത്തു തഹ്‌രീം: 1) അവതരിപ്പിക്കപ്പെടാനുള്ള പശ്ചാത്തലത്തെ സംബന്ധിച്ച് വ്യത്യസ്ത നിവേദനങ്ങൾ ഉദ്ധരിക്കപ്പെട്ടിട്ടുണ്ട്. പ്രവാചകൻ തേൻ കുടിച്ചതുമായി ബന്ധപ്പെട്ട സംഭവത്തിലാണ് വചനം ഇറക്കപ്പെട്ടത് എന്നാണ് പ്രവാചക പത്നി ആഇശ (റ) വ്യക്തമാക്കുന്നത്… സ്വഹീഹായ (സ്വീകാര്യയോഗ്യം) ഒരു പരമ്പരയിലൂടെയും സ്ഥാപിതമായിട്ടില്ലാത്ത ‘മാരിയ’യുമായി ബന്ധപ്പെട്ട കഥയല്ല അവതരണ പശ്ചാത്തലം.” (അൽ ബദറുൽ മുനീർ: ഇബ്നു മുലക്കിൻ: 8:79)

ഇമാം സ്വൻആനി ഈ അഭിപ്രായം തന്നെ ആവർത്തിക്കുന്നു. (അത്തഹ്‌രീറുൽ ഈദാഹ് മആനി തയ്സീർ: 2: 418)

പരമ്പര മുറിഞ്ഞ കാരണത്താൽ ഇമാം മിസ്സിയും (ജനനം: ഹി 654) ഈ കഥ ദുർബലമാണെന്ന് വ്യക്തമാക്കുന്നു. (തുഹ്ഫതുൽ അശ്റാഫ് ബി മഅ്’രിഫതിൽ അത്റാഫ്: 12: 170)

ഇബ്നു കുദാമ (മുഗ്നി: 9:509), ഇബ്നു ആശൂർ (അത്തഹ്‌രീർ വത്തൻവീർ: 28: 345), ഇബ്നു ഉസൈമീൻ (ഫത്ഹു ദിൽ ജലാലി വൽ ഇക്റാം: 5:71) എന്നീ പണ്ഡിതരും മാരിയയുമായ ബന്ധപ്പെട്ട ഈ കഥ ദുർബലമാണെന്ന് അഭിപ്രായപ്പെടുന്നുണ്ട്.

ശൈഖ് സ്വുഹൈബ് ഹസൻ ഇബ്നു അബ്ദുൽ ഗഫ്ഫാറിനോട് ഈ കഥയെ സംബന്ധിച്ച് ചോദിക്കപ്പെട്ടപ്പോൾ അദ്ദേഹം പൊട്ടിച്ചിരിച്ചു കൊണ്ട് പറഞ്ഞു: “ഒരു പരിഗണനയും അർഹിക്കാത്തതാണ് ഈ കഥ.”

മുബാറക്ഫൂരി രേഖപ്പെടുത്തി: “ഖാസിൻ (ജനനം: ഹി 678) തന്റെ തഫ്സീറിൽ പറഞ്ഞു: പണ്ഡിതന്മാർ ഇപ്രകാരം പ്രസ്ഥാവിച്ചിരിക്കുന്നു: ക്വുർആൻ വചനം (സൂറത്തു തഹ്‌രീം: 1) അവതരിപ്പിക്കപ്പെടാനുള്ള പശ്ചാത്തലത്തെ സംബന്ധിച്ച സ്വഹീഹായ (സ്വീകാര്യയോഗ്യം) നിവേദനം, പ്രവാചകൻ തേൻ കുടിച്ചതുമായി ബന്ധപ്പെട്ട സംഭവത്തിലാണ്, അല്ലാതെ മാരിയയുമായി ബന്ധപ്പെട്ട കഥയല്ല അവതരണ പശ്ചാത്തലം. മാരിയയുമായി ബന്ധപ്പെട്ട കഥ ഒരു സ്വഹീഹായ പരമ്പരയിലൂടെയും വന്നിട്ടില്ല. ഇമാം നസാഈ പറഞ്ഞു: തേൻ കുടിച്ചതുമായി ബന്ധപ്പെട്ട, പ്രവാചക പത്നി ആഇശ ഉദ്ധരിച്ച സംഭവം അങ്ങേയറ്റം സ്വഹീഹ് ആണ്.” (തുഹ്‌ഫതുൽ അഹ്‌വദി: 9 : 162)

ഇമാം നവവി പറഞ്ഞു: “ക്വുർആൻ വചനം (സൂറത്തു തഹ്‌രീം: 1) അവതരിപ്പിക്കപ്പെടാനുള്ള പശ്ചാത്തലത്തെ സംബന്ധിച്ച നിവേദനങ്ങളിൽ സ്വഹീഹായ (സ്വീകാര്യയോഗ്യം) നിവേദനം, പ്രവാചകൻ തേൻ കുടിച്ചതുമായി ബന്ധപ്പെട്ട സംഭവത്തിലാണ് വചനം ഇറക്കപ്പെട്ടത് എന്നാണ്. അല്ലാതെ സ്വഹീഹുൽ ബുഖാരിയിലൊ മുസ്‌ലിമിലൊ വന്നിട്ടില്ലാത്ത, ‘മാരിയ’യുമായി ബന്ധപ്പെട്ട കഥയല്ല അവതരണ പശ്ചാത്തലം. മാരിയയുമായി ബന്ധപ്പെട്ട കഥ ഒരു സ്വഹീഹായ പരമ്പരയിലൂടെയും വന്നിട്ടില്ല. ഇമാം നസാഈ പറഞ്ഞു: തേൻ കുടിച്ചതുമായി ബന്ധപ്പെട്ട പ്രവാചക പത്നി ആഇശ ഉദ്ധരിച്ച സംഭവം അങ്ങേയറ്റം സ്വഹീഹ് ആണ്. ഇതാണ് ഇമാം കാദി ഇയാദിന്റെയും അഭിപ്രായം.” (ശർഹു മുസ്‌ലിം: 1:77)

ശൈഖ് അബൂ ഹംസ പറഞ്ഞു: “മാരിയയുമായി ബന്ധപ്പെട്ട കഥ സ്വഹീഹായ ഒരു പരമ്പരയിലൂടെയും ഉദ്ധരിക്കപ്പെട്ടിട്ടില്ല എന്ന് കാദി ഇയാദ് പറഞ്ഞിരിക്കുന്നു.”

ഇമാം ഇബ്നുൽ അറബി പറഞ്ഞു: “മാരിയുമായുള്ള ലൈംഗികത പ്രവാചകൻ (സ) സ്വയം നിരോധിച്ചുവെന്ന കഥ സ്വഹീഹായ ഒരു സനദിലൂടെയും ക്രോഡീകരിക്കപ്പെട്ടിട്ടില്ല, നീതിമാന്മാരും വിശ്വസ്തരുമായ നിവേദകന്മാരല്ല അത് ഉദ്ധരിച്ചിരിക്കുന്നത്. പരമ്പര മുറിഞ്ഞ നിലയിലും ഈ കഥ ഉദ്ധരിക്കപ്പെട്ടിട്ടുണ്ട്… സ്വഹീഹായ നിവേദനം ക്വുർആൻ വചനം (സൂറത്തു തഹ്‌രീം: 1) അവതരിപ്പിക്കപ്പെടാനുള്ള പശ്ചാത്തലം പ്രവാചകന്റെ(സ) തേൻ കുടിയുമായി ബന്ധപ്പെട്ടതാണ്. സൈനബിന്റെ അടുക്കൽ വെച്ചാണ് അദ്ദേഹമത് കുടിച്ചത്.” (അഹ്കാമുൽ ക്വുർആൻ: 4:252)

ഖത്വാബി (ജനനം: ഹി 319) പറഞ്ഞു: “പ്രവാചകൻ (സ) തന്റെ അടിമ സ്ത്രീയായ മാരിയയുമായുളള ശാരീരിക ബന്ധം സ്വയം വർജിച്ചതുമായി ബന്ധപ്പെട്ട കഥയിലാണ് ‘പ്രവാചക ശപഥം’ നടന്നത് എന്ന ചിലരുടെ വാദം ശരിയല്ലെന്നും, തേൻ കുടിക്കുന്നത് പ്രവാചകൻ (സ) സ്വയം നിഷിദ്ധമാക്കിയതുമായി ബന്ധപ്പെട്ടാണ് ‘പ്രവാചക ശപഥം’ നടന്നത് എന്നും ഈ ഹദീസ് തെളിയിക്കുന്നു”. (മആലിമുസുനന് : 4:272)

വിമർശനം1: അന്ത്യ വേളയെ സംബന്ധിച്ച് ചില ഗ്രാമീണർ മുഹമ്മദ് നബിയോട് നിരന്തരം ചോദിച്ചു കൊണ്ടിരുന്നപ്പോൾ അവരിലെ ഏറ്റവും പ്രായം കുറഞ്ഞ കുട്ടിയെ ചൂണ്ടി അദ്ദേഹം ഇപ്രകാരം പറയുകയുണ്ടായി: “ഈ പയ്യൻ ജീവിക്കുകയാണെങ്കിൽ ഇവന് വാർധക്യം പ്രാപിക്കുമ്പോഴേക്കും അന്ത്യം സംഭവിചേക്കാം” (സ്വഹീഹു മുസ്‌ലിം: 7276) ആ ‘പയ്യൻ’ മരണപ്പെട്ട് നൂറ്റാണ്ടുകൾ കഴിഞ്ഞിട്ടും ലോകാവസാനം സംഭവിക്കാതിരുന്നത് മുഹമ്മദ് നബിയുടെ പ്രവചനം പരാജയപ്പെട്ടുവെന്നതിന് തെളിവല്ലെ ? മറുപടി: a) മനുഷ്യർ ആസകലം ഈ ലോകത്തു നിന്ന് ഉച്ചാടണം ചെയ്യപ്പെടുന്ന, ജൈവലോകം സമ്പൂർണമായി ഉന്മൂലനം ചെയ്യപ്പെടുന്ന, ഭൗതീക പ്രപഞ്ചത്തിന്റെ അന്തർദ്ധാനത്തെ കുറിക്കുന്ന ‘അസ്സാഅ: അൽഉള്മാ’ (മഹാ അന്ത്യം) എന്നറിയപ്പെടുന്ന ‘ലോകാവസാന’ത്തെ സംബന്ധിച്ച അറിവ്, ദൈവം മാത്രമാണ് ഉടമപ്പെടുത്തുന്നത്. മുഹമ്മദ് നബിയടക്കം (സ) സൃഷ്ടികളിൽ ഒരാളുമായി ഈ അഭൗതീക ജ്ഞാനം ദൈവം പങ്കു വെച്ചിട്ടില്ല എന്ന് ക്വുർആനും മുഹമ്മദ് നബിയും (സ) അടിക്കടി ആവർത്തിച്ചു ഉറപ്പിക്കുന്നുണ്ട്: “തീര്‍ച്ചയായും അല്ലാഹുവിന്‍റെ പക്കലാണ് അന്ത്യസമയത്തെപ്പറ്റിയുള്ള അറിവ്‌..” (ക്വുർആൻ: 31:34) “അന്ത്യസമയത്തെപ്പറ്റി അവര്‍ നിന്നോട് ചോദിക്കുന്നു; അതെപ്പോഴാണ് വന്നെത്തുന്നതെന്ന്‌. പറയുക (നബിയെ): അതിനെപ്പറ്റിയുള്ള അറിവ് എന്‍റെ രക്ഷിതാവിങ്കല്‍ മാത്രമാണ്‌. അതിന്‍റെ സമയത്ത് അത് വെളിപ്പെടുത്തുന്നത് അവന്‍ മാത്രമാകുന്നു. ആകാശങ്ങളിലും ഭൂമിയിലും അത് ഭാരിച്ചതായിരിക്കുന്നു. പെട്ടെന്നല്ലാതെ അത് നിങ്ങള്‍ക്കു വരുകയില്ല. നീ അതിനെപ്പറ്റി ചുഴിഞ്ഞന്വേഷിച്ചു മനസ്സിലാക്കിയവനാണെന്ന മട്ടില്‍ നിന്നോടവര്‍ ചോദിക്കുന്നു. പറയുക: അതിനെപ്പറ്റിയുള്ള അറിവ് അല്ലാഹുവിങ്കല്‍ മാത്രമാണ്‌. പക്ഷെ അധികമാളുകളും (കാര്യം) മനസ്സിലാക്കുന്നില്ല.” (ക്വുർആൻ: 7:187) “ജനങ്ങള്‍ അന്ത്യസമയത്തെപ്പറ്റി നിന്നോട് ചോദിക്കുന്നു. പറയുക (നബിയെ): അതിനെപ്പറ്റിയുള്ള അറിവ് അല്ലാഹുവിങ്കല്‍ മാത്രമാകുന്നു. നിനക്ക് (അതിനെപ്പറ്റി) അറിവുനല്‍കുന്ന എന്തൊന്നാണുള്ളത്‌? അന്ത്യസമയം ഒരു വേള സമീപസ്ഥമായിരിക്കാം.” (ക്വുർആൻ: 33:63) متى الساعة؟ فقال: ما المسئول عنها بأعلم من السائل ലോകാന്ത്യം എപ്പോഴാണ് എന്ന് ചോദിച്ച മലക്ക് ജിബ്‌രീലിനോട് പ്രവാചകൻ (സ) പറഞ്ഞ മറുപടി: “ചോദിക്കപ്പെട്ടവൻ ചോദ്യകർത്താവിനേക്കാൾ അതിനെ സംബന്ധിച്ച് അറിവുള്ളവനല്ല.” (സ്വഹീഹുൽ ബുഖാരി: 50, സ്വഹീഹു മുസ്‌ലിം: 99) ലോകാവസാനത്തെ സംബന്ധിച്ച അറിവ് തനിക്കില്ലെന്ന് ആവർത്തിച്ചാവർത്തിച്ച് പ്രഖ്യാപിക്കുന്ന മുഹമ്മദ് നബി (സ) തന്നെ അതിന്റെ സമയവും കാലവും ക്ലിപ്തമായി പ്രസ്ഥാവിച്ചു എന്ന് വാദിക്കുന്നതിലെ യുക്തിരാഹിത്യം സുവ്യക്തമാണ്. b) തനിക്ക് ശേഷം നൂറ്റാണ്ടുകൾ തന്റെ സമുദായം ജീവിക്കുമെന്ന് പ്രവാചകൻ (സ) തന്നെ വ്യക്തമാക്കിയതായി ഒട്ടനവധി ഹദീസുകൾ സ്വഹീഹുൽ ബുഖാരിയിലും സ്വഹീഹു മുസ്‌ലിമിലുമൊക്കെ ഉണ്ടെങ്കിലും അതിന്നും കാണാത്ത മട്ടിലാണ് വിമർശകർ. ഒരു ഉദാഹരണം കാണുക: خَيْرُ النَّاسِ قَرْنِي، ثُمَّ الَّذِينَ يَلُونَهُمْ، ثُمَّ الَّذِينَ يَلُونَهُمْ، ثُمَّ يَجِيءُ أَقْوَامٌ تَسْبِقُ شَهَادَةُ أَحَدِهِمْ يَمِينَهُ، وَيَمِينُهُ شَهَادَتَهُ “മനുഷ്യരിൽ ഏറ്റവും ഉത്തമർ എന്റെ തലമുറയാണ്, ശേഷം അവരുടെ പിന്നിൽ വരുന്ന തലമുറ, ശേഷം അവരുടെ പിന്നിൽ വരുന്ന തലമുറ. പിന്നീട് ചില ജനതകൾ വരും, അവരിൽ ഒരുവന്റെ സാക്ഷ്യത്തെ അവന്റെ ശപഥം മുൻകടക്കും…” (സ്വഹീഹുൽ ബുഖാരി: 2652, സ്വഹീഹു മുസ്‌ലിം: 2533) ഈ ഹദീസിനെ ‘ക്വർന്’ (قَرْن) എന്ന പദത്തിന് തലമുറ എന്നും നൂറ്റാണ്ട് എന്നും അർത്ഥമുണ്ട്. അപ്പോൾ മുഹമ്മദ് നബിക്ക്(സ) ശേഷം ചുരുങ്ങിയത് മൂന്നും അതിലധികവും തലമുറകൾ അല്ലെങ്കിൽ നൂറ്റാണ്ടുകൾ മനുഷ്യ ജീവിതം ദീർഘിപ്പിക്കപ്പെടുമെന്ന് അദ്ദേഹം തന്നെ വ്യക്തമാക്കി കഴിഞ്ഞു. പിന്നെ എങ്ങനെയാണ് ഒരു തലമുറയോടെ ‘ലോകാവസാനം’ സംഭവിക്കുമെന്ന് അദ്ദേഹം പറഞ്ഞതായി (ദുർ)വ്യാഖ്യാനിക്കപ്പെടുക ?! c) വിമർശന വിധേയമായ ഹദീസിലേക്ക് തന്നെ വരാം: അന്ത്യ വേളയെ സംബന്ധിച്ച് നിരന്തരമായി ചോദിച്ചു കൊണ്ടിരുന്നവരോട് അവരിലെ ഏറ്റവും പ്രായം കുറഞ്ഞ കുട്ടിയെ ചൂണ്ടി “ഈ പയ്യൻ ജീവിക്കുകയാണെങ്കിൽ ഇവന് വാർധക്യം പ്രാപിക്കുമ്പോഴേക്കും അന്ത്യം സംഭവിചേക്കാം” എന്ന് പ്രവാചകൻ പറഞ്ഞു എന്ന് പ്രവാചകാനുചരൻ അനസ് ബിൻ മാലിക് (റ) പറഞ്ഞ സംഭവം (സ്വഹീഹു മുസ്‌ലിം: 7276) വിശദവും വ്യക്തവുമായ രൂപത്തിൽ പ്രവാചകപത്നി ആഇശ (റ) ഉദ്ധരിക്കുന്നുണ്ട്. ആ നിവേദനവും സ്വഹീഹുൽ ബുഖാരിയിലും സ്വഹീഹു മുസ്‌ലിമിലും എഴുതപ്പെട്ടിട്ടുണ്ട്. പക്ഷെ അവ കൂടി ചേർത്ത് വെച്ച് അവതരിപ്പിച്ചാൽ ഹദീസിനെ ദുർവ്യാഖ്യാനിക്കാൻ കഴിയില്ല എന്നതുകൊണ്ട് സൗകര്യപൂർവ്വം ഒഴിവാക്കുകയാണ് ഇസ്‌ലാം വിമർശകരുടെ പതിവ്. ﻋﻦ ﻋﺎﺋﺸﺔ، ﻗﺎﻟﺖ: ﻛﺎﻥ ﺭﺟﺎﻝ ﻣﻦ اﻷﻋﺮاﺏ ﺟﻔﺎﺓ، ﻳﺄﺗﻮﻥ اﻟﻨﺒﻲ ﺻﻠﻰ اﻟﻠﻪ ﻋﻠﻴﻪ ﻭﺳﻠﻢ ﻓﻴﺴﺄﻟﻮﻧﻪ: ﻣﺘﻰ اﻟﺴﺎﻋﺔ؟ ﻓﻜﺎﻥ ﻳﻨﻈﺮ ﺇﻟﻰ ﺃﺻﻐﺮﻫﻢ ﻓﻴﻘﻮﻝ: «ﺇﻥ ﻳﻌﺶ ﻫﺬا ﻻ ﻳﺪﺭﻛﻪ اﻟﻬﺮﻡ ﺣﺘﻰ ﺗﻘﻮﻡ ﻋﻠﻴﻜﻢ ﺳﺎﻋﺘﻜﻢ»، ﻗﺎﻝ ﻫﺸﺎﻡ: ﻳﻌﻨﻲ ﻣﻮﺗﻬﻢ ആഇശ (റ) പറഞ്ഞു: അപരിഷ്കൃതരായ ഗ്രാമീണ അറബികളിൽ ചിലർ പ്രവാചകന്റെ(സ) അടുത്തു വന്ന് “എന്നാണ് അന്ത്യ സമയം ?” എന്ന് ചോദിക്കുമായിരുന്നു. അപ്പോൾ അവരിലെ ഏറ്റവും പ്രായം കുറഞ്ഞവനെ നോക്കി കൊണ്ട് അദ്ദേഹം ഇപ്രകാരം പറയും: “ഈ പയ്യൻ ജീവിക്കുകയാണെങ്കിൽ ഇവന് വാർധക്യം പ്രാപിക്കുമ്പോഴേക്കും നിങ്ങളുടെ അന്ത്യ സമയം സംഭവിക്കും” ഹിശാം പറഞ്ഞു: അഥവാ അവരുടെ മരണം. (സ്വഹീഹുൽ ബുഖാരി: 6511) ﺇﻥ ﻳﻌﺶ ﻫﺬا، ﻟﻢ ﻳﺪﺭﻛﻪ اﻟﻬﺮﻡ، ﻗﺎﻣﺖ ﻋﻠﻴﻜﻢ ﺳﺎﻋﺘﻜﻢ “ഇവൻ ജീവിക്കുകയാണെങ്കിൽ നിങ്ങളുടെ അന്ത്യ സമയം നിങ്ങളുടെ മേൽ സംഭവിച്ചിട്ടല്ലാതെ ഇവന് വാർധക്യം പ്രാപിക്കുകയില്ല.” (സ്വഹീഹുൽ മുസ്‌ലിം: 136) ദൈവത്തിനല്ലാതെ സൃഷ്ടികളിൽ മറ്റാർക്കും അറിവ് നൽകപ്പെട്ടിട്ടില്ലാത്ത മനുഷ്യരാശിയുടെ ‘അന്ത്യനാളിനെ’ സംബന്ധിച്ച് ആവശ്യത്തിനുപരി ചിന്തിച്ചും ചോദിച്ചും സമയവും ഊർജവും പാഴാക്കുന്ന ഗ്രാമീണരോട്, ‘അവരുടെ അന്ത്യ സമയത്തെ’ (അഥവാ മരണത്തെയും ലോകത്തു നിന്നുമുള്ള വിയോഗത്തെയും) സംബന്ധിച്ച് ചിന്തിക്കാൻ വഴി കാട്ടുകയാണ് പ്രവാചകൻ (സ) ചെയ്തത്. അല്ലാതെ ലോകാവസാനത്തിന്റെ തിയ്യതി കുറിച്ചു കൊടുക്കുകയല്ല. (ﻗﺎﻣﺖ ﻋﻠﻴﻜﻢ ﺳﺎﻋﺘﻜﻢ) “നിങ്ങളുടെ അന്ത്യ സമയം നിങ്ങളുടെ മേൽ സംഭവിക്കും” എന്ന് പ്രവാചകൻ (സ) പറഞ്ഞതായി ഹദീസിന്റെ വിശദരൂപത്തിൽ തന്നെ വ്യക്തമാക്കപ്പെട്ടല്ലൊ. കൂടാതെ ഹദീസ് ഉദ്ധരിക്കുന്ന റാവി ഹിശാം ഇബ്നു ഉർവ്വ (ജനനം: 61 ഹിജ്റാബ്ദം) തന്നെ “നിങ്ങളുടെ അന്ത്യ സമയം” എന്നതുകൊണ്ട് പ്രവാചകൻ (സ) ഉദ്ദേശിച്ചത് അദ്ദേഹത്തിന്റെ (സ) സംബോധിതരായ ആളുടെ മരണമാണെന്നും ഹദീസിൽ തന്നെ നാം കണ്ടു. ഹദീസിൽ തന്നെ പ്രവാചക വാചകത്തിൽ ഉദ്ദേശിക്കപ്പെട്ട ‘അസ്സാഅ:’യുടെ (അന്ത്യം) അർത്ഥം വ്യക്തമാണ് എന്നിരിക്കെ ഹദീസിന്റെ ഏറ്റവും സംഗ്രഹരൂപത്തിലുളള നിവേദനം മാത്രം എടുത്തു വെച്ച് ‘അസ്സാഅ:’ക്ക് ‘ലോകാവസാനം’ എന്ന് അർത്ഥം നൽകുന്നത് ദുരുദ്ദേശ്യപരമാണ്. ഇത് ഹദീസിന്റെ പൂർണ രൂപത്തിനും അറബി ഭാഷക്കും എല്ലാം എതിരാണ്. പൗരാണി അറബികൾ പ്രസ്ഥുത പദത്തെ ഉപയോഗിച്ചിരുന്ന അർത്ഥവ്യാപ്തിയെ സംബന്ധിച്ച് യാതൊരു വിവരവുമില്ലാത്തവർക്കു മാത്രമെ ഈ തെറ്റിദ്ധാരണ സംഭവിക്കു. ‘അസ്സാഅ:’ (الساعة) എന്ന പദത്തിന്റെ ഭാഷാർത്ഥം സമയം, മണിക്കൂർ, മണി (O’clock), വാച്ച്, ക്ലോക്ക് എന്നൊക്കെയാണ് ആധുനിക ഡിക്ഷ്ണറികളിൽ കാണുക. ക്വുർആനിലും ഹദീസിലും സാങ്കേതികമായി ‘അന്ത്യ സമയത്തെ’ കുറിക്കാൻ ഈ പദം ധാരാളമായി പ്രയോഗിച്ചിട്ടുണ്ട്. അന്ത്യ സമയം, അന്ത്യ ഘട്ടം, അന്ത്യ മണിക്കൂർ, അന്ത്യദിനം എന്നിങ്ങനെയൊക്കെ സാങ്കേതിക പദമായി ഉപയോഗിക്കപ്പെടുമ്പോൾ ‘അസ്സാഅ:’ക്ക് അർത്ഥം നൽകപ്പെടാറുണ്ട്. ലോകാവസാനത്തിന് പുറമെ ഒരു വ്യക്തിയുടെയൊ സംഘത്തിന്റെയൊ മരണത്തെയും, ഒരു തലമുറയുടെയൊ ജനതയുടെയൊ തിരോധാനത്തെയും, ഒരു നൂറ്റാണ്ടിന്റെയൊ സമുദായത്തിന്റെയൊ അവസാനത്തെയും എല്ലാം ‘അസ്സാഅ:’ അഥവാ ‘അന്ത്യ ഘട്ടം’ എന്ന് അറബി ഭാഷയിൽ പ്രയോഗിക്കപ്പെടും. ലോകാവസാനം, ജീവിതാവസാനം, തലമുറയുടെ അവസാനം എന്നൊക്കെയുളള ഉദ്ദേശ്യാർത്ഥം ഓരോ സാഹചര്യത്തിനനുസരിച്ച് ‘അസ്സാഅ:’ (അന്ത്യ സമയം) എന്ന പദം പ്രയോഗിക്കപ്പെടും. ഇബ്നു തീമിയ പറഞ്ഞു: “‘അസ്സാഅ:’ (അന്ത്യ വേള) എന്നതുകൊണ്ട് പ്രവാചകൻ (സ) ഉദ്ദേശിച്ചത് ഒരു തലമുറയുടെ അന്ത്യമാണ് എന്ന് സ്വഹീഹുൽ ബുഖാരിയിലൂടെ സ്ഥാപിതമായ കാര്യമാണ്… മനുഷ്യരെല്ലാം തങ്ങളുടെ ശ്മശാനങ്ങളിൽ നിന്ന് ദൈവ സന്നിധിയിലേക്ക് ഉയിർത്തെഴുന്നേൽക്കപ്പെടുന്ന ലോകാവസാനമല്ല ഇവിടെ ഉദ്ദേശിക്കപ്പെട്ടിരിക്കുന്നത്. ഈ പയ്യൻ വയസ്സാവുമ്പോഴേക്കും സംഭവിക്കുമെന്ന് പറഞ്ഞ ‘അസ്സാഅ:’ ശ്രോദ്ധാക്കളായ മനുഷ്യരുടെ മരണവും പ്രസ്ഥുത തലമുറയുടെ തിരോധാനവുമാണ്. പ്രവാചക ശിഷ്യൻ മുഗീറത്തിബ്നു ശുഅ്ബ പറഞ്ഞത് ഇതിന് തെളിവാണ്: ജനങ്ങളെ, നിങ്ങൾ ‘ക്വിയാമ:’ (ഉയിർത്തെഴുന്നേൽപ്പ്) എന്ന് വിളിക്കുന്നത് (സർവ്വ മനുഷ്യരും ദൈവസന്നിധിയിൽ ഹാജരാക്കപ്പെടുന്ന മഹാ) ഉയിർത്തെഴുന്നേൽപ്പിനെയാണ്. എന്നാൽ ആർ മരണപ്പെട്ടുവോ അവന്റെ ക്വിയാമത്ത് (ഉയിർത്തെഴുന്നേൽപ്പ്) സംഭവിച്ചു കഴിഞ്ഞു. (അൽ ഇസ്തികാമ: 1: 67) ഇമാം നവവി എഴുതി: “കാദി ഇയാദ് (ജനനം ഹിജ്റാബ്ദം:476) പറഞ്ഞു: ‘നിങ്ങളുടെ സാഅ:’ (അന്ത്യ വേള) എന്ന് പ്രവാചകൻ (സ) പറഞ്ഞതിന്റെ അർത്ഥം ശ്രോദ്ധാക്കളായ തലമുറയുടെ മരണവും വിയോഗവുമാണ്. പ്രസ്ഥുത അഭിസംബോധകർ അല്ലെങ്കിൽ ആ തലമുറയിൽ പെട്ടവരുടെ അന്ത്യമാണ് ഉദ്ദേശ്യം.” (ശർഹു മുസ്‌ലിം : 18:90) ‘അസ്സാഅ: അൽഉള്മാ’ ( الساعة العظمى മഹാ അന്ത്യം) എന്നറിയപ്പെടുന്ന ‘ലോകാവസാന’ത്തെ സംബന്ധിച്ച, ദൈവത്തിന് മാത്രം അറിവുള്ള ഒരു കാര്യത്തെ സംബന്ധിച്ച് ആവർത്തിച്ച് ചോദിച്ച് സമയവും ജീവിതാവസരങ്ങളും നഷ്ടപ്പെടുത്തരുത്. കാരണം നിങ്ങളുടെ ഓരോരുത്തരുടെയും ‘അസ്സാഅ:’ (അന്ത്യ വേള) ഏകദേശം നൂറു വർഷത്തിനിടയിൽ സംഭവിക്കും. അതിനാൽ സ്വന്തം അവസാന വേളയെ സംബന്ധിച്ച് ചിന്തിച്ച് അതിനായി ഒരുങ്ങുകയും സൽകർമ്മങ്ങളിൽ നിരതരാവുകയും ചെയ്യുക എന്നാണ് ഹദീസിലൂടെ പ്രവാചകൻ (സ) നൽകുന്ന പാഠം. ‘അസ്സാഅ: അൽഉള്മാ’ (മഹാ അന്ത്യം) എന്നറിയപ്പെടുന്ന ലോകാവസാനത്തെ സംബന്ധിച്ച ചോദ്യങ്ങളുടെ ധാരാളിത്വം പ്രവാചകന് (സ) പ്രിയമുള്ളതായിരുന്നില്ല എന്ന് ഹദീസുകളിൽ നിന്ന് തന്നെ വ്യക്തമാണ്. ഒരിക്കൽ അന്ത്യ വേളയെ സംബന്ധിച്ച് ആരാഞ്ഞ ഒരാളോട് പ്രവാചകൻ (സ), “കഷ്ടം ! ആ അവസരത്തിനായി നീ എന്താണ് (സൽകർമ്മങ്ങൾ) ഒരുക്കി വെച്ചിട്ടുള്ളത്?!” എന്ന് പ്രവാചകൻ (സ) തിരിച്ചു ചോദിക്കുകയാണുണ്ടായത്. (സ്വഹീഹുൽ ബുഖാരി:6167, സുനനു തുർമുദി: 2385) ഇബ്നു ഹജർ പറഞ്ഞു: “ഇസ്മാഈലി പറഞ്ഞു: ‘അസ്സാഅ:’ (അന്ത്യ വേള) എന്നത് കൊണ്ട് ഉദ്ദേശ്യം പ്രവാചകന്റെ അടുക്കൽ സന്നിഹിതരായവരുടെ അന്ത്യമാണ്. അഥവാ അവരുടെ മരണം. അവരുടെ മരണത്തെ ‘അസ്സാഅ:’ (അന്ത്യ വേള) എന്ന് പ്രയോഗിക്കപ്പെടുന്നതിന്റെ കാരണം മരണം പരലോക കാര്യങ്ങളിലേക്ക് അവരെ വഹിച്ചുകൊണ്ടു പോവുന്നു എന്നതാണ്. ‘അസ്സാഅ: അൽഉള്മാ’ (മഹാ അന്ത്യം) എന്നറിയപ്പെടുന്ന ലോകാവസാനത്തെ സംബന്ധിച്ച അറിവ് ദൈവത്തിന് മാത്രം അറിയുന്നതാണ് എന്ന് ഈ ഹദീസ് ഉറപ്പിക്കുന്നു. ക്വുർആനും ധാരാളം ഹദീസുകളും വ്യക്തമാക്കുന്നത് ലോകാവസാനത്തെ സംബന്ധിച്ച അറിവ് ദൈവത്തിന് മാത്രം അറിയുന്നതാണ് എന്ന വസ്തുതയാണ്… ‘അസ്സാഅ:'(അന്ത്യ വേള) എന്നതിനെ സംബന്ധിച്ച ഈ അർത്ഥാന്തരങ്ങൾ അറബികളുടെ അടുക്കൽ വളരെ പ്രചാരത്തിലുള്ളതാണ്…” (ഫത്ഹുൽ ബാരി:10:556) ഹദീസിന്റെ പൂർണരൂപമടങ്ങുന്ന ആഇശയുടെ നിവേദനത്തിന്റെ വെളിച്ചത്തിൽ ഇതേ വ്യാഖ്യാനം തന്നെ ‘അൽമസ്വാബീഹ്’, ‘അൽ മശാരിക്’ എന്നീ ഗ്രന്ഥങ്ങൾക്ക് വ്യാഖ്യാനം രചിച്ച പണ്ഡിതന്മാരും ഇമാം ദാവൂദിയും, കാദി ഇയാദും, ക്വുർതുബിയും എല്ലാം വിശദീകരിച്ചിട്ടുണ്ട്. (ഫത്ഹുൽ ബാരി: 10:572) ഇബ്നുകസീർ പറഞ്ഞു: “അനസിന്റെ(റ) നിവേദനത്തിലുള്ള പോലെ, ‘അസ്സാഅ:’ (അന്ത്യ വേള) എന്ന പദം നിരുപാധികമായി വന്ന നിവേദനങ്ങൾ, ആഇശയുടെ(റ) നിവേദനത്തിൽ സോപാധികമായി വന്ന “നിങ്ങളുടെ അന്ത്യ വേള” എന്ന പദപ്രയോഗത്തിലൂടെയാണ് വ്യാഖ്യാനിക്കപ്പെടേണ്ടത്.” (മആരിജുൽ കുബൂൽ: 2: 592) ‘അൽക്വിയാമ:’ (اﻟﻘﻴﺎﻣﺔ) അഥവാ ‘ഉയിർത്തെഴുന്നേൽപ്പ് ‘എന്ന സാങ്കേതിക പദത്തിന്റെ കാര്യം ഇപ്രകാരം തന്നെയാണ്. സർവ്വ മനുഷ്യരും ദൈവസന്നിധിയിൽ ഹാജരാക്കപ്പെടുന്ന ‘മഹാ ഉയിർത്തെഴുന്നേൽപ്പ്’ എന്നതിന് പുറമെ ഓരോ മനുഷ്യരുടെയും മരണത്തെ അവരവരുടെ ‘ഉയിർത്തെഴുന്നേൽപ്പ് ‘ എന്ന് ഇസ്‌ലാമിക പ്രമാണങ്ങൾ ധാരാളമായി വിളിച്ചിട്ടുണ്ട്. “ഓരോരുത്തരുടേയും ക്വിയാമത്ത് അഥവാ ‘ഉയിർത്തെഴുന്നേൽപ്പ് ‘ അവരവരുടെ മരണമാണ്… ഒരാളുടെ മൃതശരീരം മറമാടിയതിന് ശേഷം അൽക്വമ പറഞ്ഞു: എന്നാൽ ഇദ്ദേഹത്തിന്റെ ക്വിയാമത്ത് (ഉയിർത്തെഴുന്നേൽപ്പ്) സംഭവിച്ചു കഴിഞ്ഞു. (തഫ്സീറു ത്വബ്‌രി: 24:43) “ആർ മരണപ്പെട്ടുവോ അവന്റെ ക്വിയാമത്ത് (ഉയിർത്തെഴുന്നേൽപ്പ്) ആയി.” (ശർഹുസ്സുന്ന: ബഗ്‌വി: 10:97, തഖ്‌രീജുൽ അഹാദീസു വൽ ആസാർ: സൈല’ഇ: 1:436) d) ‘അന്ത്യ വേളയെ സംബന്ധിച്ചുള്ള അറിവ് അല്ലാഹുവിന്റെ അടുക്കൽ മാത്രമാണ് ഉള്ളത്. അല്ലാഹുവെ സാക്ഷിയാക്കി കൊണ്ട് ഞാൻ സത്യം ചെയ്യുന്നു. ഇപ്പോൾ ഭൂമിക്കു മുകളിൽ ശ്വസിക്കുന്ന ഒരാളുടെയും മേൽ നൂറ് വർഷത്തിനപ്പുറം കടന്നുപോകില്ല’ (സ്വഹീഹു മുസ്‌ലിം: 6363) എന്ന് മുഹമ്മദ് നബി (സ) പറഞ്ഞതായി വന്ന ഹദീസിലെയും ‘അസ്സാഅ:’ (അന്ത്യ വേള) എന്നത് ഈ അർത്ഥവ്യാപ്തിയെ ഉൾകൊണ്ടാണ് പ്രയോഗിച്ചിരിക്കുന്നത്. ‘അസ്സാഅ:’ (അന്ത്യ വേള) എന്നത് കൊണ്ട് ലോകത്തിന്റെ/ പ്രപഞ്ചത്തിന്റെ അവസാനമാണ് ഉദ്ദേശിക്കുന്നതെങ്കിൽ അത് എപ്പോഴാണ് സംഭവിക്കുക എന്നത് ദൈവത്തിന് മാത്രമെ അറിയു. തനിക്ക് ആ അറിവില്ല. ഇനി, ‘അസ്സാഅ:’ (അന്ത്യ വേള) എന്നത് കൊണ്ട് ഈ തലമുറയുടെ അവസാനമാണ് ഉദ്ദേശിക്കപ്പെടുന്നതെങ്കിൽ ഒരു നൂറ്റാണ്ടു കൊണ്ട് നിങ്ങൾ നിങ്ങളുടെ ‘അസ്സാഅ:’ (അന്ത്യ വേള)ക്ക് പാത്രീയരാവും എന്നാണ് മുകളിൽ ഉദ്ധരിക്കപ്പെട്ട ഹദീസിന്റെ അർത്ഥം. പ്രവാചകാനുചരൻ ഇബ്നു ഉമർ (റ) തന്നെ ഇത് വ്യക്തമാക്കുന്നുണ്ട്: ‘ഇന്ന് ഭൂമിക്കുമുകളിലുള്ള ആരും തന്നെ അവശേഷിക്കില്ല’ എന്ന് മാത്രമാണ് അല്ലാഹുവിന്റെ ദൂതൻ (സ) പറഞ്ഞത്. ( يريد بذلك أن ينخرم ذلك القرن) ഒരു തലമുറയുടെ അന്ത്യമാണ് അദ്ദേഹം ഈ പറഞ്ഞതു കൊണ്ട് ഉദ്ദേശിച്ചത്. (സ്വഹീഹു മുസ്‌ലിം : 6361) വിമർശനം2: രണ്ടു വിരലുകൾ ചേർത്തുപിടിച്ചു കൊണ്ട് താനും ലോകാന്ത്യവും ഇത്രയും അടുത്താണെന്ന് മുഹമ്മദ് നബി പറഞ്ഞിട്ട് പതിനാല് നൂറ്റാണ്ട് പിന്നിട്ടില്ലെ?! ഇത് പ്രവചനത്തിന്റെ പരാജയത്തിലേക്കല്ലെ വിരൽ ചൂണ്ടുന്നത്? മറുപടി: പ്രവാചകൻ (സ) തന്റെ നടുവിരലും ചൂണ്ടുവിരലും കാണിച്ചു കൊണ്ട് “ഞാൻ നിയോഗിക്കപ്പെട്ടതും അന്ത്യനാളും ഇപ്രകാരമായാണ്” എന്ന് പറഞ്ഞതായി ഹദീസിൽ വന്നിട്ടുണ്ട്. (സ്വഹീഹു മുസ്‌ലിം: 2951) ചൂണ്ടുവിരലിനും നടുവിരലിനുമിടയിൽ ഒരു വിരലില്ല എന്നത് പോലെ മുഹമ്മദ് നബിക്കും (സ) അന്ത്യനാളിനുമിടയിൽ പുതുതായി ഒരു പ്രവാചക നിയോഗമനം ഉണ്ടാവില്ല എന്നതാണ് ഹദീസിന്റെ അർത്ഥം. ഈ പ്രവചനം ഒരു നിലയിലും അസാധുവാക്കപ്പെട്ടിട്ടില്ല; ഭാവിയിൽ അസാധുവാക്കപ്പെടുകയുമില്ല. ഹദീസിനെ വ്യാഖ്യാനിച്ചു കൊണ്ട് ഇബ്നു ഹജർ ഇപ്രകാരം രേഖപ്പെടുത്തി: “കാദി ഇയാദും മറ്റും പറഞ്ഞു: മുഹമ്മദ് നബിയുടെ(സ) പ്രവാചകത്വവും ലോകാവസാനവും തമ്മിലുള്ള അടുപ്പമാണ് ഹദീസ് സൂചിപ്പിക്കുന്നത്. ഈ രണ്ട് കാര്യങ്ങൾക്കുമിടയിലെ ചുരുങ്ങിയ കാലയളവ് അല്ലെങ്കിൽ ഒന്നിന് ശേഷം വരുന്നതായ അടുത്ത കാര്യം എന്ന അർത്ഥത്തിൽ ഒക്കെയാണ് ഹദീസ്… ‘അന്ത്യനാളിനെ സംബന്ധിച്ച് ചോദിച്ചക്കപ്പെട്ടവന്, ചോദ്യകർത്താവിനേക്കാൾ അറിവില്ല’ എന്ന ഹദീസിന് ഈ ഹദീസ് എതിരല്ല. ഈ ഹദീസിന്റെ അർത്ഥം, ലോകാന്ത്യത്തിനും മുഹമ്മദ് നബിക്കും(സ) ഇടയിൽ പുതുതായി ഒരു നബിയും വരാനില്ല എന്നാണ്; ചൂണ്ടുവിരലിനും നടുവിരലിനും ഇടയിൽ മറ്റൊരു വിരലില്ല എന്നതു പോലെ… ദഹ്ഹാക് പറഞ്ഞു: ലോകാവസാനത്തിന്റെ അടയാളങ്ങളിൽ പെട്ടതാണ് മുഹമ്മദ് നബിയുടെ(സ) പ്രവാചക നിയോഗം. മതിമറന്നവരെ ഉൽബുദ്ധരാക്കുക അന്ത്യനാളിനായി തയ്യാറെടുപ്പിക്കുകയും പശ്ചാത്താപത്തിനായി പ്രോത്സാഹിപ്പിക്കുകയും ചെയ്യുക എന്നതാണ് ലോകാവസാനത്തിന് അടയാളങ്ങൾ നിയോഗിച്ചതിലെ യുക്തി.” (ഫത്ഹുൽ ബാരി: 11:357) ഹദീസിന് നൽകപ്പെട്ടിട്ടുള്ള മറ്റൊരു വ്യാഖ്യാനം, രണ്ടു വിരലുകൾക്കിടയിലുള്ള അടുപ്പം പോലെയാണ് മുഹമ്മദ് നബിയുടെ (സ) ആഗമനവും ലോകാന്ത്യവും തമ്മിലുള്ള കാല ചുരുക്കം എന്നതാണ്. ഈ വ്യാഖ്യാനവും പ്രവചനത്തെ അസാധുവാക്കുകയൊ ദുർബലപ്പെടുത്തുകയൊ ചെയ്യുന്നില്ല. കാരണം ലോകത്തിന്റെയും പ്രപഞ്ചത്തിന്റെയും മൊത്തം പ്രായവും പഴക്കവുമായി താരതമ്യം ചെയ്തു കൊണ്ടാണല്ലൊ മുഹമ്മദ് നബിയുടെയും (സ) ലോകാവസാനത്തിന്റെയും ഇടയിലുള്ള കാലയളവ് ചുരുക്കമൊ ദീർഘമൊ എന്ന് നിർണയിക്കാൻ. പ്രപഞ്ചത്തിന് ഏകദേശം 13.8 ബില്യൺ വർഷം പഴക്കമുണ്ടെന്നാണ് നിഗമിക്കപ്പെടുന്നത്. (https://www.google.com/amp/s/www.space.com/amp/universe-age-14-billion-years-old ഭൂമിയുടെ പ്രായം ഏകദേശം 4.54 ബില്യൺ വർഷങ്ങളായി കണക്കാക്കപ്പെടുന്നു, ഈ കണക്കിൽ നിന്ന് ഏകദേശം 50 ദശലക്ഷം വർഷങ്ങൾ കൂടിയൊ കുറഞ്ഞൊ വരാം. (https://www.nationalgeographic.org ) ഭൂമിയിൽ ജീവൻ ആരംഭിച്ചത് കുറഞ്ഞത് 3.7 ബില്യൺ വർഷങ്ങൾക്ക് മുമ്പാണെന്ന് നിഗമിക്കപ്പെടുന്നു. ആദ്യത്തെ മനുഷ്യ പൂർവ്വികർ പ്രത്യക്ഷപ്പെട്ടത് ഏഴ് ദശലക്ഷം മുതൽ നാലൊ രണ്ടൊ ദശലക്ഷം വർഷങ്ങൾക്ക് മുമ്പാണെന്നാണ് വാദിക്കപ്പെടുന്നത്. അപ്പോൾ ഇനിയും നൂറോ ആയിരമൊ വർഷങ്ങൾക്ക് ശേഷമാണ് പ്രപഞ്ചവും ലോകവും പ്രതിഭാസവ്യതിയാനമൊ ഉന്മൂലനാശമൊ പ്രാപിക്കുന്നതെങ്കിൽ പോലും പ്രപഞ്ചത്തിന്റെയൊ ഭൂമിയുടെയൊ ജൈവലോകത്തിന്റെയൊ മൊത്തം പ്രായത്തിന്റെയും കാലപഴക്കത്തിന്റെയും മുമ്പിൽ മുഹമ്മദ് നബിയുടെ (സ) ആഗമനവും ലോകാവസാനവും തമ്മിൽ വളരെ ചെറിയ കാല അകൽച്ച മാത്രമെ മനസ്സിലാക്കാൻ സാധിക്കു.
വിമർശനം: ഒറ്റ ദിവസം തന്നെ എല്ലാ ഭാര്യമാരോടൊപ്പവും ലൈംഗിക ബന്ധം സ്ഥാപിച്ചുവെന്നത് മുഹമ്മദ് നബി ഒരു സ്ത്രീലമ്പടനായിരുന്നു എന്നതിന് തെളിവല്ലെ ? മറുപടി: മുഹമ്മദ് നബിക്കെതിരെ(സ) ഈ വ്യാജ ആരോപണം അഴിച്ചു വിടാൻ വിമർശകർ അവലംബിക്കുന്ന ഹദീസ് ആദ്യം നമുക്കൊന്ന് വായിക്കാം: അനസ് (റ) പറഞ്ഞു: പ്രവാചകൻ (സ), പകലിൽ നിന്നും രാത്രിയിൽ നിന്നും ഒരൊറ്റ ചുറ്റലിൽ തന്റെ ഭാര്യമാരെയെല്ലാം സന്ദർശിക്കാറുയുണ്ടായിരുന്നു. അവർ പതിനൊന്നു പേരുണ്ടായിരുന്നു. ഞാൻ (കത്താദ) അനസിനോട് ചോദിച്ചു: അദ്ദേഹത്തിന് (പ്രവാചകന്) അത് സാധിക്കുമായിരുന്നോ ? അപ്പോൾ അനസ് (റ) പറഞ്ഞു: അദ്ദേഹത്തിന് മുപ്പതു പേരുടെ കരുത്ത് നൽകപ്പെട്ടിട്ടുണ്ടെന്ന് ഞങ്ങൾ പരസ്പരം സംസാരിക്കാറുണ്ടായിരുന്നു. (സ്വഹീഹുൽ ബുഖാരി: 268) പ്രവാചകൻ (സ) ഒരു സ്ത്രീലമ്പടനുമായിരുന്നു എന്ന് ആക്ഷേപിക്കാനായി വിമർശകർ വികലമായ അർത്ഥം നൽകി വിവാദവൽക്കരിക്കാറുള്ള ഒരു ഹദീസാണ് ഇത്. ഒരു ദിവസം തന്നെ പതിനൊന്ന് ഭാര്യമാരുമായി ലൈംഗിക ബന്ധത്തിൽ ഏർപ്പെടുക എന്നത് അദ്ദേഹത്തിന്റെ അമിതാസക്തിയും സ്‌ത്രീലോലുപത്വവുമാണ് തെളിയിക്കുന്നത് എന്ന് അസഭ്യഭാഷയിൽ അവതരിപ്പിക്കുകയാണ് നബിവിമർശകർ ചെയ്യാറുള്ളത്. വാസ്ഥവത്തിൽ, ഹദീസിൽ പ്രസ്ഥാവിക്കപ്പെട്ട സംഭവത്തിന്റെ യാഥാർത്ഥ്യം മനസ്സിലാക്കിയാൽ നബി വിമർശകർ സൃഷ്ടിച്ചെടുക്കുന്ന അശ്ലീല വ്യാഖ്യാനത്തിനൊന്നും യാതൊരു അടിത്തറയുമില്ലെന്ന് ആർക്കും വ്യക്തമാകുന്നതാണ്. 1. പ്രവാചകന്റെ(സ) പത്നിമാർക്കെല്ലാം അദ്ദേഹം പ്രിയങ്കരനായിരുന്നു. കേവല ശരീരങ്ങൾ തമ്മിലുള്ള ബന്ധം മാത്രമായിരുന്നില്ല അവർ തമ്മിലുള്ള ദാമ്പത്യം. അദ്ദേഹത്തോടൊപ്പമുള്ള സഹവാസവും സമ്പർക്കവും ആത്മീയമായ നേട്ടത്തിന് പുറമെ അവർക്ക് മാനസികവും ഭൗതീകവുമായ ആസ്വാദനവും സമാധാനവും അവലംബവുമായിരുന്നു. അതുകൊണ്ട് തന്നെ അദ്ദേഹത്തിന്റെ സാന്നിധ്യത്തിനായി അവർ ഓരോരുത്തരും അതിയായി ആഗ്രഹിച്ചിരുന്നു. അവരുടെ ആഗ്രഹം കണക്കിലെടുത്തു കൊണ്ടും ഭാര്യമാർക്കിടയിൽ നീതി പാലിക്കുക എന്ന ആദർശനിഷ്ട പ്രയോഗവൽകരിച്ചു കൊണ്ടും ഓരോ ഭാര്യക്കും ഓരോ ദിവസം പ്രവാചകൻ (സ) വീതിച്ചിരുന്നു എന്നും ചില യാത്രകളിൽ കൂടെ കൂട്ടേണ്ടതാരെയാണെന്ന് തിരഞ്ഞെടുക്കാനായി അവർ നറുക്കിട്ടിരുന്നു എന്നും പ്രവാചക പത്നിമാർ തന്നെ പ്രസ്ഥാവിച്ചിട്ടുണ്ട്. (സ്വഹീഹുൽ ബുഖാരി: 2593) ഓരോരുത്തർക്കായും വീതം വെച്ച ദിവങ്ങൾക്കു പുറമെ ചില ദിവസങ്ങളും ഉണ്ടാകാറുണ്ട്. ദീർഘയാത്രകൾ, ആരാധനാ- തീർത്ഥാടന കർമ്മങ്ങൾ തുടങ്ങി പലതിനുമായും ഒഴിച്ചു വെക്കപ്പെട്ട ഇത്തരം ദിവസങ്ങൾ പ്രത്യേകിച്ച് ഒരു ഭാര്യക്കും അവകാശപ്പെട്ടതല്ലാത്തവയാണ്. ഇത്തരം അവസരങ്ങളിൽ ഭാര്യമാരുമായി പിരിയുകയോ അവരിലേക്ക് തിരിച്ചു വരുകയോ ഒക്കെ ചെയ്യുന്ന സന്ദർഭത്തിൽ എല്ലാവരുമായി പ്രവാചകൻ (സ) അൽപ്പനേരം സമ്പർക്കം പുലർത്തുകയും സഹവസിക്കുകയും ചെയ്യുമായിരുന്നു. ഇതാണ്, “പ്രവാചകൻ (സ) ഒറ്റ ദിവസം തന്നെ എല്ലാ ഭാര്യമാരേയും ചുറ്റിസഞ്ചരിക്കുമായിരുന്നു” എന്ന ഹദീസുകളുടെ ഉള്ളടക്കത്തിന്റെ വിവക്ഷ. ഒരു സ്ത്രീലോലുപതയും അമിതാസക്തിയുമില്ലാത്ത സ്വാഭാവിക സന്ദർശനം!! പ്രവാചകന്റെ(സ) ‘ഒറ്റ ദിവസത്തെ കൂട്ട സന്ദർശനത്തിന്റെ’ ലക്ഷ്യമെന്തായിരുന്നെന്ന് അദ്ദേഹത്തിന്റെ പത്നിമാർ തന്നെ വ്യക്തമായി വിശദീകരിച്ചിട്ടുണ്ട്. പ്രവാചകന്റെ (സ) ഭാര്യമാരുടെ അടുത്തേക്കുള്ള അദ്ദേഹത്തിന്റെ ഈ സന്ദർശനത്തിന്റെ അർത്ഥവും ലക്ഷ്യവും എന്താണെന്ന് ഏറ്റവും നന്നായി അറിയുക പ്രവാചകന്റെ ഭാര്യമാർക്ക് തന്നെയാണല്ലൊ; അല്ലാതെ അനസിനെ(റ) പോലെയുള്ള പ്രവാചകന്റെ ഏതെങ്കിലും ശിഷ്യർക്കല്ല. عائشة رضي الله عنها، قالت: ( كان رسولُ اللهِ صلَّى الله عليه وسلم لا يُفضِّلُ بعضنا على بعضٍ في القَسمِ ، من مُكثه عِندنا ، وكان قلَّ يومٌ إلا وهو يَطُوفُ علينا جميعاً ، فيدنو مِنْ كُلِّ امرأة ، مِن غير مَسِيسٍ ، حتى يَبْلُغَ إلى التي هو يَوْمُها فيبيتُ عندها ) . പ്രവാചക പത്നി ആഇശ (റ) പറയുകയുണ്ടായി: “ഞങ്ങളുടെ അടുക്കൽ താമസിക്കുന്നതിനായി വിഭജിച്ച ദിവസങ്ങളിൽ ഒരു ഭാര്യക്ക് മറ്റൊരാളേക്കാൾ പ്രമാഖ്യം അല്ലാഹുവിന്റെ ദൂതൻ (സ) കാണിച്ചിരുന്നില്ല. ഞങ്ങളുടെ എല്ലാവരുടേയും അടുക്കൽ അദ്ദേഹം ചുറ്റിസഞ്ചരിക്കാത്ത ദിവസങ്ങൾ കുറവായിരുന്നു.’ലൈംഗിക ബന്ധത്തിൽ ഏർപ്പെടാതെ’ അദ്ദേഹം എല്ലാ ഭാര്യമാരുടെ അടുത്തേക്കും ചെല്ലുമായിരുന്നു. അവസാനം,(അടുത്ത് താമസിക്കുന്നതിനായി ഭാര്യമാർക്കിടയിൽ വിഭജിച്ച ദിവസങ്ങളിൽ) ഏതു ഭാര്യയുടെ ദിവസമാണോ ആ ഭാര്യയുടെ അടുക്കൽ രാപ്പാർക്കും.” (മുസ്നദ് അഹ്മദ്: 24765, സുനനു അബൂദാവൂദ്: 2135, സ്വഹീഹു അബൂദാവൂദ്: അൽബാനി: 1852) فيدنو مِنْ كُلِّ امرأة ، مِن غير مَسِيسٍ “ലൈംഗിക ബന്ധത്തിൽ ഏർപ്പെടാതെ അദ്ദേഹം എല്ലാ ഭാര്യമാരുടെ അടുത്തേക്കും ചെല്ലുമായിരുന്നു…” എന്ന് ഹദീസിൽ ആഇശ (റ) വളരെ വ്യക്തമായി, പ്രത്യേകം എടുത്തു പറയുന്നുണ്ട് എന്ന് കണ്ടുവല്ലൊ. പ്രവാചകൻ (സ) ഒറ്റ ദിവസം തന്നെ എല്ലാ ഭാര്യമാരേയും ‘സന്ദർശിക്കാറുണ്ടായിരുന്നു’, എല്ലാ ഭാര്യമാരുടേയും അടുത്ത് ‘ചുറ്റിസഞ്ചരിക്കാറുണ്ടായിരുന്നു’ (يَدُورُ – يَطُوفُ) എന്നിങ്ങനെയുള്ള പദപ്രയോഗങ്ങൾ കൊണ്ടുള്ള ഉദ്ദേശം സമ്പർക്കത്തിലൂടെയും സഹവാസത്തിലൂടെയും “അവരുമായി ബന്ധം പുതുക്കുകയാണ്” (ﻳُﺮَاﺩَ ﺑِﻪِ ﺗَﺠْﺪِﻳﺪُ اﻟْﻌَﻬْﺪِ ﺑِﻬِﻦَّ). അഥവാ, ദീർഘയാത്രകൾക്ക് മുമ്പോ ശേഷമോ “എല്ലാവരുമായി സമയം ചെലവഴിക്കുക” എന്ന സ്വാഭാവികമായ സന്ദർശനമാണ് ഉദ്ദേശ്യമെന്ന് പൗരാണികരും ആധുനികരുമായ പല പണ്ഡിതന്മാരും ആഇശയുടെ (റ) ഹദീസുമായി ബന്ധിപ്പിച്ചു കൊണ്ട് വ്യാഖ്യാനിച്ചിട്ടുണ്ട്. ذَكَرْتُهُ لِعَائِشَةَ فَقَالَتْ : يَرْحَمُ اللَّهُ أَبَا عَبْدِ الرَّحْمَنِ كُنْتُ أُطَيِّبُ رَسُولَ اللَّهِ صَلَّى اللَّهُ عَلَيْهِ وَسَلَّمَ فَيَطُوفُ عَلَى نِسَائِهِ ، ثُمَّ يُصْبِحُ مُحْرِمًا يَنْضَخُ طِيبًا ആഇശ (റ) പറഞ്ഞു: …ഞാൻ അല്ലാഹുവിന്റെ ദൂതന് (അദ്ദേഹത്തിന്റെ ശരീരത്തിൽ) സുഗന്ധം തേച്ച് കൊടുക്കുമായിരുന്നു. എന്നിട്ട് അദ്ദേഹം തന്റെ എല്ലാ ഭാര്യമാരേയും അടുത്ത് ‘ചുറ്റി സന്ദർശിക്കും’ (فَيَطُوفُ عَلَى نِسَائِهِ). പിന്നീട് രാവിലെ – (ഞാൻ തേച്ചു കൊടുത്ത) സുഗന്ധം, വമിക്കുന്ന നിലയിൽ അദ്ദേഹം ഇഹ്റാമിൽ പ്രവേശിക്കും. (സ്വഹീഹുൽ ബുഖാരി:267, സ്വഹീഹു മുസ്‌ലിം: 1192) ആഇശ (റ) ഈ പ്രസ്ഥാവിച്ച, ഒരു ദിവസം കൊണ്ട് എല്ലാ ഭാര്യമാരെയുമുള്ള സന്ദർശനത്തിൽ അവരുമായുള്ള ലൈംഗിക ബന്ധം ഉൾപ്പെട്ടിരുന്നെങ്കിൽ ദിവസത്തിന്റെ അന്ത്യത്തിൽ അദ്ദേഹത്തിന് കുളി നിർബന്ധമാകുമായിരുന്നു. കുളിച്ചിരുന്നെങ്കിൽ പിറ്റേന്ന് ഇഹ്റാമിൽ പ്രവേശിച്ചപ്പോൾ ശരീരത്തിൽ തേച്ച സുഗന്ധം അവശേഷിക്കില്ലായിരുന്നു. അത് പിറ്റേന്നും വമിക്കുന്നുണ്ടായിരുന്നു എന്ന ആഇശയുടെ പ്രസ്ഥാവനയിൽ നിന്നും പ്രസ്ഥുത സന്ദർശനം ലൈംഗിക ബന്ധത്തിന് വേണ്ടിയായിരുന്നില്ല, അവരുമായുള്ള സമ്പർക്കത്തിനും സഹവാസത്തിനും വേണ്ടി മാത്രമായിരുന്നു എന്ന് ഇസ്മാഈലി (ജനനം ഹിജ്റ 277) തന്റെ ‘സ്വഹീഹി’ൽ രേഖപ്പെടുത്തി. ഇസ്മാഈലിയിൽ നിന്നും ഈ വ്യാഖ്യാനം ഇബ്നു ഹജറും (ഫത്ഹുൽ ബാരി: 1:377), ബദറുദ്ദീൻ അൽഐനിയും (ഉംദത്തുൽ ക്വാരി: 3:213, 214) ഇബ്നു റജബും (ഫത്ഹുൽ ബാരി: 1:297) കസ്ത്വല്ലാനിയും (ഇർശാദുസ്സാരി: 1:325), ശൻക്വീതിയും (കൗസറുദ്ദുറാറി അൽ മആനി:5:418) എല്ലാം രേഖപ്പെടുത്തിയിട്ടുണ്ട്. 2. പക്ഷെ അനസിന്റെ ഹദീസിൽ “അദ്ദേഹത്തിന് മുപ്പത് ആളുകളുടെ കരുത്ത് നൽകപ്പെട്ടിട്ടുണ്ട്…” എന്നടങ്ങുന്ന പരാമർശത്തിൽ ഒറ്റ ദിവസം കൊണ്ട് എല്ലാ ഭാര്യമാരുമായുള്ള ‘ചുറ്റി സന്ദർശനത്തിൽ’ ലൈംഗിക ബന്ധവും ഉള്ളടങ്ങുന്നതായ സൂചനയുണ്ട് എന്നതാണ് വിവാദങ്ങൾക്ക് നിദാനം. സ്വഭാവം, ബുദ്ധി, ഭക്തി, ശരീരം തുടങ്ങി വ്യക്തിത്വത്തിന്റെ സർവ്വഭാവങ്ങളിലും സമ്പൂർണ്ണത നൽകപ്പെട്ടവരാണ് ദൈവദൂതന്മാർ. പുറമെ മറ്റു മനുഷ്യരിൽ നിന്ന് വ്യത്യസ്ഥമായി അവരിലൂടെ അല്ലാഹു പ്രകടമാക്കുന്ന മുഅ്ജിസത്തിൽ (അമാനുഷിക ദൃഷ്ടാന്തങ്ങൾ) ഉൾകൊള്ളുന്നതാണ് അവരുടെ അസാധാരണമായ കായിക ശക്തിയും ശാരീരിക ശേഷിയും. സ്വാഭാവികമായും മുഹമ്മദ് നബിക്കും(സ) അസാധാരണ ആരോഗ്യവും, അപാരമായ കായികശേഷിയും നൽകപ്പെട്ടിട്ടുണ്ടായിരുന്നു. സൂറത്തുൽ ബക്വറ, സൂറത്തു ആലു ഇംറാൻ, സൂറത്തുന്നിസാഅ് തുടങ്ങിയ ക്വുർആനിലെ നീളൻ അധ്യായങ്ങൾ പാരായണം ചെയ്ത് രാത്രി ഭൂരിഭാഗവും നിന്ന് നമസ്കരിക്കുമായിരുന്നു പ്രവാചകൻ (സ). (സ്വഹീഹു മുസ്‌ലിം: 772) ഇടമുറിയാതെ രണ്ടു ദിവസം ചേർത്ത് വ്രതം അനുഷ്ടിക്കുന്ന രീതിയായ ‘വിസ്വാൽ’ പ്രവാചകൻ (സ) അനുഷ്ടിച്ചിരുന്നു. നിങ്ങൾക്ക് അതിന് ശാരീരികമായി സാധിക്കില്ലെന്നും ഇത് തനിക്ക് മാത്രം നിശ്ചയിക്കപ്പെട്ട വ്രതത്തിന്റെ രീതിയാണെന്നും അദ്ദേഹം തന്റെ അനുചരന്മാരോട് ഉപദേശിക്കുകയും ചെയ്തിരുന്നു.(സ്വഹീഹുൽ ബുഖാരി:7299) ഖന്ദക്ക് യുദ്ധ സന്ദർഭത്തിൽ പ്രവാചകാനുചരന്മാർ തങ്ങളുടെ മഴു കൊണ്ട്, സംഘം ചേർന്ന് തകർക്കാൻ ശ്രമിച്ചിട്ടും തകരാതിരുന്ന ഒരു ഭീമൻ പാറ പ്രവാചകൻ (സ) തന്റെ മഴു കൊണ്ട് വെട്ടി തകർക്കുകയുണ്ടായി. (സുനനുൽ കുബ്റാ: നസാഈ: 8858, മുസ്നദു റൂയാനി: 410) പ്രവാചകന്(സ) അസാധാരണ കായിക ശക്തി ഉണ്ടായിരുന്നു എന്നത് ശരിയാണെന്ന് ചുരുക്കം. അദ്ദേഹം ഇടക്ക്, ഒറ്റ ദിവസം അദ്ദേഹത്തിന്റെ എല്ലാ ഭാര്യമാരേയും സന്ദർശിക്കാറുണ്ടായിരുന്നു എന്നതും വാസ്തവം. പക്ഷെ ഈ രണ്ടു കാര്യങ്ങൾക്കു തമ്മിലുള്ള ബന്ധിപ്പിക്കുന്ന ഘടകം ലൈംഗിക വേഴ്ച്ചയാകണമെന്ന് യാതൊരു നിർബന്ധവുമില്ല. കാരണം, ഒമ്പതു ഭാര്യമാരേയും ഒരു ദിവസം തന്നെ സന്ദർശിക്കുകയാണെങ്കിൽ അവരുമായി ഹൃദ്യമായി സല്ലപിക്കുകയും, അവരുടെ സുഖ ദുഖങ്ങളിൽ പങ്കാളിയാവുകയും, എന്നിട്ട് വഴക്കോ- അസ്വാരസ്യങ്ങളോ ഉണ്ടാകാത്ത നിലയിൽ- അവരുടെയെല്ലാം ഭർതൃ സങ്കൽപങ്ങളെ തൃപ്തിപ്പെടുത്തി വിടപറയുകയും എല്ലാം വേണമല്ലൊ. ലൈംഗിക ബന്ധമില്ലാതെ തന്നെ ഇത് അങ്ങേയറ്റം ശ്രമകരമായ യത്നം തന്നെയാണ്. ഒരു സാധാരണ മനുഷ്യന് കഴിയാൻ സാധ്യതയില്ലാത്ത അനൽപമായ ക്ഷമയും, സഹാനുഭൂതിയും, നീതി ബോധവും, ത്യാഗവും, സ്നേഹവുമെല്ലാം അനിവാര്യമായ ഒരു മഹാ ഉദ്യമമാണത്. മതം, രാഷ്ട്രീയം, സാമൂഹിക സേവനം, യുദ്ധം, ആരാധന, പ്രബോധനം തുടങ്ങി കൃത്യാന്തര ബാഹുല്യത്തിനിടയിലും ഒമ്പതു ഭാര്യമാരും മക്കളും മരുമക്കളം, പേരക്കുട്ടികളുമെല്ലാമടങ്ങുന്ന കുടുംബ-ദാമ്പത്യ ജീവിതത്തിലും പ്രവാചകനെ പോലെ മാതൃകയും തുല്യനീതിയും ആദർശനിഷ്ടയും പുലർത്താൻ അശക്തരായ അനുചരന്മാർ പ്രവാചകന്റെ ഈ ബഹുമുഖ മാതൃകയെ ആശ്ചര്യത്തോടെ ആവിഷ്‌കരിച്ചതാണ് “അദ്ദേഹത്തിന് മുപ്പത് ആളുകളുടെ കരുത്ത് നൽകപ്പെട്ടിട്ടുണ്ട്…” എന്ന വാചകം. ഒരാൾക്ക്, ഒരു ആയുഷ്ക്കാലം കൊണ്ട് ചെയ്തു തീർക്കാൻ കഴിയുന്നതിനുമപ്പുറമായിരുന്നു അദ്ദേഹത്തിലർപ്പിതമായതും അദ്ദേഹം പൂർണതയോടെ നിറവേറ്റിയതുമായ -കുടുംബ ജീവിതമുൾപ്പെടെയുള്ള- ദൗത്യങ്ങൾ എന്നേ അനസിന്റെ ആ വാചകം കൊണ്ട് മനസ്സിലാക്കേണ്ടതുള്ളു. അല്ലാതെ മുപ്പത് പുരുഷന്മാരുടെ ലൈംഗിക ശക്തി അദ്ദേഹത്തിനുണ്ടായിരുന്ന എന്ന് ആ വാചകത്തെ ദുർവ്യാഖ്യാനിക്കുന്നത് ന്യായമില്ല. പ്രവാചകന്റെ(സ) ഈ ‘കൂട്ട സന്ദർശനത്തിന്റെ’ അർത്ഥവും ലക്ഷ്യവും സ്വാഭാവികമായ സഹവാസവും സമ്പർക്കവും മാത്രമായിരുന്നെന്ന് അദ്ദേഹത്തിന്റെ സന്ദർശനത്താൽ അനുഗ്രഹിക്കപ്പെട്ട ഭാര്യമാർ വ്യക്തമാക്കിയിരിക്കെ അനസിന്റെ(റ) വാചകം ദുർവ്യാഖ്യാനിച്ചുള്ള അശ്ലീല ആരോപണങ്ങളുടെ പ്രസക്തിയെന്താണ്?! “അദ്ദേഹത്തിന് മുപ്പത് ആളുകളുടെ കരുത്ത് നൽകപ്പെട്ടിട്ടുണ്ട്…” എന്ന വാചകത്തിന്റെ ഉദ്ദേശ്യം ലൈംഗിക ബന്ധത്തിനുള്ള കരുത്താണ് എന്ന വ്യാഖ്യാനം വാദത്തിന് അംഗീകരിച്ചാൽ തന്നെ അത് ആ സന്ദർശനത്തെ സംബന്ധിച്ച പ്രവാചക ശിഷ്യൻ അനസിന്റെ(റ) ഒരു തെറ്റിദ്ധാരണയാണെന്നേ സ്ഥാപിക്കപ്പെടുന്നുള്ളു. കാരണം, പ്രവാചകന്റെ(സ) ഈ ‘കൂട്ട സന്ദർശനത്തിന്റെ’ അർത്ഥവും ലക്ഷ്യവും, സ്വാഭാവികമായ സഹവാസവും സമ്പർക്കവും മാത്രമായിരുന്നെന്ന് അദ്ദേഹത്തിന്റെ സന്ദർശനത്താൽ അനുഗ്രഹിക്കപ്പെട്ട ഭാര്യമാർ വ്യക്തമാക്കിയിരിക്കെ അനസിന്റെ(റ) ധാരണ യഥാർത്ഥ്യമാകാൻ വല്ല നിർവ്വാഹവുമുണ്ടോ?! അനസാണോ(റ) ആ സന്ദർശനത്തെ സംബന്ധിച്ച് സൂക്ഷ്മമായി അറിയുക ? അതോ പ്രവാചകപത്നിമാരോ ?!കൂടാതെ, അനസിന്റെ(റ) ആ വാചകത്തിൽ നിന്നും എല്ലാ ഭാര്യമാരുമായി അദ്ദേഹം ലൈംഗികവേഴ്ച്ചയിൽ ഏർപ്പെട്ടു എന്നും സ്ഥാപിതമാകുന്നില്ല. കൂടി വന്നാൽ ഒന്നിലേറെ പേരുമായി ബന്ധപ്പെട്ടു എന്നേ ഈ വ്യാഖ്യാനം ശരിവച്ചാലും സ്ഥാപിതമാകുന്നുള്ളു. “ആർക്കെങ്കിലും രണ്ട് ഭാര്യമാർ ഉണ്ടായിട്ട് അവരിൽ ഒരാളിലേക്ക് (പക്ഷപാതപരമായി) അയാൾ താൽപര്യം കാണിച്ചാൽ അന്ത്യനാളിൽ തന്റെ ശരീരത്തിന്റെ ഒരു ഭാഗം ചെരിഞ്ഞതായി/ സ്വാധീനമറ്റതായിട്ടാണ് അയാൾ വരിക” (മുസ്നദു അഹ്മദ്: 2:295, സുനനു അബൂദാവൂദ്: 2:601) എന്ന് ഉണർത്തി, ഭാര്യമാർക്കിടയിൽ നീതിയോടെ വർത്തിക്കണമെന്ന് സ്വന്തം അനുചരൻമാരെ അനുശാസിക്കുക മാത്രമല്ല പ്രവാചകൻ (സ) ചെയ്തത്. ഉപദേശത്തിന് പുറമെ, മരണം വരെയുള്ള സ്വന്തം ദാമ്പത്യജീവിതത്തിലൂടെ അതിന് മാതൃക ലോകത്തിന് സമ്മാനിക്കുക കൂടി ചെയ്തു അദ്ദേഹം. വിവാദ വിധേയമായ ഹദീസ് പ്രവാചകന്റെ നീതിയേയും സമത്വത്തെയുമാണ് ജ്വലിപ്പിക്കുന്നത്.

ന്ത്യൻ മുസ്‌ലിംകളുടെ ദേശക്കൂറും വിശ്വസ്തതയും ചോദ്യം ചെയ്തു കൊണ്ട് വർഗീയ ദ്രുവീകരണം സൃഷ്ടിച്ച് രാഷ്ട്രീയ നേട്ടങ്ങൾ കൊയ്യാൻ സംഘ് പരിവാറും കൂട്ടാളികളും അശ്രാന്ത പരിശ്രമത്തിലാണ്. ഇസ്‌ലാമിനെതിരെ നുണകളും അർദ്ധ സത്യങ്ങളും പ്രചരിപ്പിക്കുന്നതിനു പുറമെ ഇസ്‌ലാമിക പ്രമാണങ്ങളെ ദുർവ്യാഖ്യാനിച്ചും തെറ്റിദ്ധരിപ്പിച്ചും ജനമനസ്സുകളിൽ ഇസ്‌ലാം ഭീതി പടർത്താൻ നിരന്തരം പ്രയത്നിച്ചു കൊണ്ടിരിക്കുന്ന വെറുപ്പിന്റെ അപ്പോസ്തലന്മാർ ഏറ്റവുമൊടുവിൽ പുതിയ കുപ്പിയിലാക്കി ഇറക്കുമതി ചെയ്ത പഴയ വീഞ്ഞാണ് ‘ഗസ്‌വത്തുൽ ഹിന്ദ്’ (ഹിന്ദ് യുദ്ധം). ഇന്ത്യയോട് യുദ്ധം ചെയ്യാൻ പ്രവാചക കൽപ്പനയുണ്ടെന്നും ഏതു നിമിഷവും പൊട്ടാവുന്ന ബോംബാണ് മുസ്‌ലിംകളെന്നുമാണ് ദേശസ്നേഹികളും സമാധാന ചിത്തരുമായ ഇന്ത്യൻ മുസ്‌ലിംകളുടെ തലയിൽ വെച്ചുകെട്ടുന്ന പുതിയ ആരോപണം. ഈ അവസരത്തിൽ വാദപ്രതിവാദങ്ങളുടെ വൈകാരിക തലം ഒട്ടും സ്പർശിക്കാതെ, വിവാദ വിഷയകമായ ഹദീസിനെ സംബന്ധിച്ച ഒരു വൈചാരികമായ ചർച്ചയാണ് ഈ കുറിപ്പ് ലക്ഷ്യം വെക്കുന്നത്. ﻋﺼﺎﺑﺘﺎﻥ ﻣﻦ ﺃﻣﺘﻲ ﺃﺣﺮﺯﻫﻤﺎ اﻟﻠﻪ ﻣﻦ اﻟﻨﺎﺭ: ﻋﺼﺎﺑﺔ ﺗﻐﺰﻭ اﻟﻬﻨﺪ ﻭﻋﺼﺎﺑﺔ ﺗﻜﻮﻥ ﻣﻊ ﻋﻴﺴﻰ ﺑﻦ ﻣﺮﻳﻢ ﻋﻠﻴﻪ اﻟﺴﻼﻡ. “എന്റെ സമുദായത്തിലെ രണ്ട് സംഘത്തെ അല്ലാഹു നരകത്തിൽ നിന്നും സംരക്ഷിക്കും. ‘ഹിന്ദി’നോട് (ഇന്ത്യ) യുദ്ധം ചെയ്യുന്ന സംഘവും ഈസബ്നു മർയത്തോടൊപ്പം(അ) ഉണ്ടാകുന്ന സംഘവുമാണത്.” (നസാഈ: 2/64)

‘ഹിന്ദു’മായുള്ള യുദ്ധത്തെ സംബന്ധിച്ച് ചില ഹദീസ് ഗ്രന്ഥങ്ങളിൽ ഉദ്ധരിക്കപ്പെട്ടിട്ടുള്ള ഒരു നിവേദനമാണ് നാം മുകളിൽ ഉദ്ധരിച്ചത്. ഹദീസ് ‘സ്വഹീഹ്’ (സ്വീകാര്യതയുടെ ഹദീസ് നിദാന ശാസ്ത്ര മാനദണ്ഡങ്ങൾ പൂർത്തീകരിക്കപ്പെട്ടത്) ആണ് എന്ന് വന്നാൽ തന്നെ ഹദീസിന്റെ ഉള്ളടക്കത്തിൽ, ഇസ്‌ലാമോഫോബിയ പ്രചാരകർ ഊതി വീർപ്പിച്ച് ഉരുട്ടി കാണിക്കുന്നതു പോലെ ഇന്ത്യക്ക് ഭീഷണിയായ ഒന്നും തന്നെ അടങ്ങിയിട്ടില്ല എന്നതാണ് മറ്റൊരു സത്യം. ഹദീസിലെ ‘ഹിന്ദ്’ (اﻟﻬﻨﺪ) എന്ന പദത്തിനാണ് ‘ഇന്ത്യ’ എന്ന് പരിഭാഷ നൽകപ്പെടാറുള്ളത്.

In ancient times, India was much more extended to the North West and west (consisting of parts of modern Pakistan and Afghanistan). (https://www.culturalindia.net)

പൗരാണിക ഇന്ത്യയിൽ അധുനിക പാകിസ്ഥാനും അഫ്ഗാനിസ്ഥാന്റെ പല ഭാഗങ്ങളും ഉൾകൊണ്ടിരുന്നു എന്ന് നമുക്കേവർക്കും അറിയാമല്ലൊ. (ആധുനിക പാക്കിസ്ഥാനിൽ ഉൾപ്പെടുന്ന) സിന്ദു നദി തീരപ്രദേശങ്ങളെയാണ് പൗരാണിക കാലത്ത് ‘ഹിന്ദ്’ കൊണ്ട് ഉദ്ദേശിക്കപ്പെട്ടിരുന്നത്, നമ്മുടെ ഇന്നത്തെ ഇന്ത്യയെയല്ല എന്നതിനാൽ തന്നെ ‘ഹിന്ദി’നോടുള്ള യുദ്ധം എന്നതുകൊണ്ട് (ആധുനിക) ഇന്ത്യയോടുള്ള യുദ്ധമല്ല എന്ന് തിരിച്ചറിയാൻ വലിയ പ്രയാസമൊന്നുമില്ല.

കൂടാതെ, പ്രവാചക കാലഘട്ടത്തിലാവട്ടെ -പ്രവാചകനും പ്രവാചകാനുചരന്മാരും ഉൾപ്പെടെ- അറബികൾ ‘ഹിന്ദ്’ (ഇന്ത്യ) എന്ന് വിളിച്ചിരുന്നത് ആധുനിക ഇന്ത്യയെയല്ല. ഇന്ത്യയുടെ അന്നത്തെ ഭൂമിശാസ്ത്ര ഘടന പ്രകാരമായാലും ശരി പൗരാണിക അറേബ്യൻ മുസ്‌ലിംകളുടെ സാങ്കേതിക ഭാഷ പ്രകാരമായാലും ശരി, ഹദീസിൽ പറയപ്പെട്ടിട്ടുള്ള ‘ഹിന്ദ്’ അഥവാ ‘ഇന്ത്യ’ നമ്മുടെ രാജ്യമായ ഇന്ത്യ (ഭാരതം) അല്ലേയല്ല.

പൂർവ്വസൂരികളായ മുസ്‌ലിംകൾ ‘ഹിന്ദ്’ (ഇന്ത്യ) എന്ന് വിളിച്ചിരുന്നത് ‘ബസ്വറ’യെയാണ്. ഇന്നത്തെ ഇറാക്കിലെ പ്രസിദ്ധമായ ഒരു പട്ടണമാണ് ‘ബസ്വറ’. ‘ബസ്വറ’ ഉൾപ്പെടെയുള്ള പൗരാണിക ‘ഇറാക്’ -പ്രവാചക കാലഘട്ടത്തിൽ- പേർഷ്യൻ സാമ്രാജ്യത്തിന്റെ ഭാഗമായിരുന്നു. (https://mawdoo3.com)

അറേബ്യയിൽ നിന്നും ഇന്ത്യയിലേക്കുള്ള കരമാർഗ്ഗത്തിലാണ് ബസ്വറ നിലകൊള്ളുന്നത് എന്നത് കൊണ്ട്, ഇന്ത്യയുടെ ദിക്കിലുള്ള നാട് എന്ന നിലയിൽ ‘ബസ്വറ’യെ പ്രവാചക കാലഘട്ടം ‘ഹിന്ദ്’ (ഇന്ത്യ) എന്ന് വിളിച്ചു.

ﻭﻛﺎﻧﻮا ﻳﺴﻤﻮﻥ اﻟﺒﺼﺮﺓ ﻫِﻨﺪًا، ﻷﻧﻬﺎ ﻣﻦ ﺟﻬﺔ اﻟﻬﻨﺪ، ﻭﻣﻨﻬﺎ ﻳُﺴﻠﻚ ﺇِﻟَﻰ اﻟﻬﻨﺪ، ﻭﻟﻬﺬا ﻗﺎﻝ ﺧﺎﻟﺪ ﻟﻤﺎ ﻋﺰﻟﻪ ﻋﻤﺮ ﻋﻦ اﻟﺸﺎﻡ: ﺇﻥ ﻋﻤﺮ ﺃﻣﺮﻧﻲ ﺃﻥ [ ﺁﺗﻲ] اﻟﻬﻨﺪ. ﻗﺎﻝ اﻟﺮﻭاﻱ: ﻭﻛﺎﻧﺖ اﻟﻬﻨﺪ ﻋﻨﺪﻧﺎ اﻟﺒﺼﺮﺓ. ഇബ്നു റജബ് എഴുതി: പൂർവ്വകാല മുസ്‌ലിംകൾ ‘ബസ്വറ’ക്ക് ‘ഹിന്ദ്’ എന്നായിരുന്നു പേര് വെച്ചിരുന്നത്. ‘ബസ്വറ’, ഇന്ത്യയുടെ ദിക്കിലായതു കൊണ്ടും ബസ്വറയിലൂടെയാണ് ഇന്ത്യയിലേക്ക് എത്താനുള്ള കരമാർഗം എന്നതുകൊണ്ടുമായിരുന്നു അത്. ഉമർ, ഖാലിദിനെ ശാമിൽ നിന്നും നീക്കിയപ്പോൾ ഖാലിദ് ഇപ്രകാരം പറഞ്ഞത് അതുകൊണ്ടാണ്: എന്നോട് ഉമർ ‘ഹിന്ദി’ലേക്ക് (ഇന്ത്യ) ചെല്ലാൻ കൽപ്പിച്ചു. നിവേദകൻ പറയുന്നു: ‘ഹിന്ദ്’ (ഇന്ത്യ) എന്നാൽ ഞങ്ങളുടെ അടുക്കൽ ‘ബസ്വറ’യായിരുന്നു. (മജ്മൂഉ റസാഇലു ഇബ്നു റജബ്: 3:205)

‘ബസ്വറ’ക്കടുത്ത ‘ഉബുല്ല’ എന്ന സ്ഥലത്തെ ‘ഇന്ത്യൻ പുൽത്തകിടി’ (مرج الهند) എന്നാണ് വിളിക്കപ്പെട്ടിരിക്കുന്നത് എന്ന് ഇബ്നു ഖൽദൂൻ തന്റെ ‘താരീഖ്’ (2:507) ൽ പ്രസ്ഥാവിക്കുന്നുണ്ട്.

ഒട്ടനവധി ഹദീസ്-ചരിത്ര ഗ്രന്ഥങ്ങളിൽ ഉദ്ധരിക്കപ്പെട്ട സുപ്രധാനമായ ഒരു നിവേദനം ഇപ്രകാരമാണ്: “പ്രവാചക ശിഷ്യൻ ഖാലിദിബ്നു വലീദ്(റ) പറഞ്ഞു: ശാം അതിന്റെ സമൃതി ഇട്ടു തന്നതിന് ശേഷം വിശ്വാസികളുടെ നേതാവ്, ഉമർ ബിൻ ഖത്താബ് എനിക്ക് കത്തെഴുതി, ഞാൻ ഹിന്ദിലേക്ക് (ഇന്ത്യ) സഞ്ചരിക്കാൻ കൽപ്പന നൽകി – ഹിന്ദ് (ഇന്ത്യ) എന്നാൽ ഞങ്ങളുടെ മനസ്സിൽ ബസ്വറയാണ് – എനിക്കാകട്ടെ ഹിന്ദിലേക്ക് പോകാൻ വൈമനസ്യമുണ്ടായിരുന്നു…” (മുസ്നദു അഹ്മദ്: 4:90, ദലാഇലുന്നുബുവ്വ: 6:387, ജാമിഉൽ മസാനിദ്: 2: 389, അൽ ജിഹാദ്: ഇബ്നു അബീ ആസിം: 2: 666, മുഅ്ജമുൽ കബീർ: ത്വബ്റാനി: 4:137, മുഅ്ജമുൽ അവ്സത്വ് : 8:277, അൽ മഅ്’രിഫതു വത്താരീഖ് : ഫസ്വി: 3:115-116, അൽമുത്തഫകു വൽമുഫ്തറകു:3:1743-1744, താരീഖു ദിമശ്ക്: ഇബ്നു അസാകിർ: 40:310. ഒരു ഹദീസ് ആയിട്ടല്ലെങ്കിൽ, ചരിത്രപരമായ ഭൂമിശാസ്ത്ര (Historical geography) സാക്ഷ്യമായെങ്കിലും ഈ നിവേദനം പരിഗണിക്കപ്പെടേണ്ടതുണ്ട്.)

ﺃﻥ اﻟﺴﻠﻒ اﻟﺼﺎﻟﺢ «ﻛﺎﻧﻮا ﻳﺴﻤﻮﻥ (اﻟﺒﺼﺮﺓ) (ﻫﻨﺪاً)… ﺇﻥ اﻟﻬﻨﺪ ﻛﺎﻧﺖ ﻓﻲ ﻧﻔﻮﺳﻬﻢ اﻟﺒﺼﺮﺓ، ﻭﺑﻪ ﺗﻔﻬﻢ ﺳﺎﺋﺮ اﻷﺣﺎﺩﻳﺚ اﻟﻮاﺭﺩِ ﻓﻴﻬﺎ ﺫﻛﺮُ (اﻟﻬﻨﺪ) .

അബൂ ഉബൈദ മശ്ഹൂറിബ്നു ഹസൻ ഇബ്നു മഹ്മൂദ് ആലു സൽമാൻ വ്യക്തമാക്കുന്നു:

“പൂർവ്വസൂരികളായ സച്ഛരിതർ ‘ബസ്വറ’ക്ക് പേര് നൽകിയിരുന്നത് ‘ഇന്ത്യ’ (ഹിന്ദ്) എന്നായിരുന്നു… തീർച്ചയായും ഇന്ത്യയെന്നാൽ (ഹിന്ദ്) അവരുടെ മനസ്സിൽ ബസ്വറയാണ്. ഇന്ത്യയെ (ഹിന്ദ് ) സംബന്ധിച്ച് സ്മരിക്കുന്ന എല്ലാ ഹദീസുകളും ഇപ്രകാരം തന്നെയാണ് മനസ്സിലാക്കേണ്ടത്.” (അൽ ഇറാക്ക് ഫിൽ അഹാദീസി വ ആസാറുൽ ഫിതൻ: 1:360-364)

മസ്ഊദി (മരണം:346 ഹിജ്‌റ) പറഞ്ഞു: ബസ്വറയിൽ അതബതുബ്നു ഗസ്‌വാൻ ഹിജ്‌റ 16 അല്ലെങ്കിൽ 17 ന് ഖലീഫയുടെ കല്പന പ്രകാരം കടന്നു വന്നു. അന്ന് ബസ്വറയെ വിളിക്കപ്പെട്ടിരുന്നത് ‘ഇന്ത്യൻ ഭൂമി’ (അർദുൽ ഹിന്ദ് أرض الهند) എന്നായിരുന്നു. അത് വെളുത്ത പാറകളും ചരൽക്കല്ലുകളും നിറഞ്ഞ ഒരു ഭൂമിയായിരുന്നു. അതബതുബ്നു ഗസ്‌വാൻ ആണ് അതിനെ ഒരു പട്ടണമായി വാർത്തെടുക്കുന്നത്. ബസ്വറയും ചുറ്റുപാടുമുള്ള മറ്റു പട്ടണങ്ങളിൽ നിന്നും പേർഷ്യൻ സാമ്രാജ്യത്തേക്ക് ചെല്ലുന്ന യുദ്ധ സന്നാഹങ്ങളും സഹായങ്ങളും തടയുക കൂടി ബസ്വറക്ക് മേൽ ഉള്ള വിജയത്തിന് പിന്നിലെ ലക്ഷ്യത്തിൽ പെട്ടതായിരുന്നു.. (താരീഖുത്വബ്‌രി: 3:596, അത്തംബീഹ് വൽഇഷ്റാഫ് :1:310,)

At the time of the Muhammadan conquest, the county about Basra was called Arz-ul-Hind, the Land of India…

“മുഹമ്മദിയ അധിനിവേശ കാലഘട്ടത്തിൽ, ബസ്രയെ ‘അർസ്-ഉൾ-ഹിന്ദ്’, ഇന്ത്യൻ നാട് എന്നാണ് വിളിക്കപ്പെട്ടിരുന്നത്… ” എന്ന് പല ഇന്ത്യൻ ചരിത്രകാരന്മാരും വ്യക്തമാക്കിയിട്ടുണ്ട്. (The Indian Encyclopedia: Edited by Subodh Kapoor: Cosmo Publication: New Delhi: 2002, Vol: page: 4718)

ബ്രിട്ടീഷ് ആർമി ഓഫീസറും ഒറിയന്റലിസ്റ്റും പേർഷ്യൻ ഭാഷാ പണ്ഡിതനുമായ സർ എച്ച്. റൗളിൻസണും ഫ്രഞ്ച് ഒറിയന്റലിസ്റ്റും ചരിത്രകാരനുമായ ഹെൻറി കോർഡിയറും ഈ വസ്തുതയിലേക്ക് വിരൽ ചൂണ്ടുന്നുണ്ട്. (Cathay and the Way Thinker. Being a Collection of Medieval Notices of China)

പേർഷ്യൻ സാസാനിയ്യ ഭരണകൂടത്തിന്റെ കാലഘട്ടത്തിൽ (226 AD-651 AD) ഇറാഖിന്റെ ദക്ഷിണ ഭാഗത്തും ടൈഗ്രിസ് നദിയുടെ ഇടയിലുമായി ‘മീഷാൻ’ എന്ന പേരിൽ ഒരു ‘അമീർ ഭരണം’ (Emirate) നിലനിന്നിരുന്നു. മീഷാൻ രാജ്യത്തിന്റെ കേന്ദ്രസ്ഥാനം ബസ്വറയുടെ പേർഷ്യൻ ഗൾഫ് തീരത്തായിരുന്നു. ബസ്വറയുൾപ്പെടുന്ന ഇത്തരം എമിറേറ്റുകൾക്ക് ആ കാലഘട്ടത്തിൽ പേർഷ്യൻ സാമ്രാജ്യത്തിൽ തന്ത്രപ്രധാനവും അതി പ്രസക്തവുമായ സ്ഥാനമുണ്ടായിരുന്നു. ഇറാഖിൽ പേർഷ്യൻ സാമ്രാജ്യത്വ ശക്തികളോട് മുസ്‌ലിംകൾ ഏറ്റുമുട്ടുന്ന ആദ്യത്തെ യുദ്ധങ്ങളാണ് ഹഫീർ, ദാത്തു സലാസിൽ യുദ്ധങ്ങൾ. യുദ്ധത്തിനിടയിൽ തങ്ങളുടെ സൈന്യത്തിൽ നിന്നും ആരും ഓടി രക്ഷപ്പെടാതിരിക്കാൻ വലിയ ചങ്ങല വലയങ്ങളുമായാണ് പേർഷ്യക്കാർ യുദ്ധത്തിന് വന്നത് എന്നതിനാൽ ‘ദാത്തു സലാസിൽ’ അഥവാ ‘ചങ്ങല കെട്ടുകളുടെ യുദ്ധം’ എന്നാണ് ആ പോരാട്ടത്തെ മുസ്‌ലിംകൾ വിളിച്ചത്. (മുഖ്തസറു താരീഖിൽ ബസ്വറ :അലീ ളരീഫ് അൽ അഅ്‌സമി: 7-12)

ബസ്വറയെ ഇന്ത്യ എന്നായിരുന്നു പ്രവാചക കാലഘട്ടത്തിൽ വിളിക്കപ്പെട്ടിരുന്നത് എന്നത് ഒട്ടനവധി ചരിത്രജ്ഞർ തങ്ങളുടെ വിശ്വപ്രസിദ്ധ ചരിത്ര ഗ്രന്ഥങ്ങളിൽ വ്യക്തമാക്കിയിട്ടുണ്ട്:

* അബൂ യുസുഫ് (മരണം: 182: ഹിജ്റ): ‘അൽഖറാജ്’ (1:73).

* ഇബ്നു സഅ്ദ് (മരണം: 230: ഹിജ്റ): ‘ത്വബകാത്ത്’ (7:3).

* ഖലീഫ ബിൻ ഖയ്യാത്ത് (മരണം: 240: ഹിജ്റ): ‘താരീഖ് ‘ (1:117).

* ത്വബ്‌രി (മരണം: 310: ഹിജ്റ): ‘താരീഖ് ‘ (3:591).

* അദ്ദാരിമി അൽ ബുസ്തി (മരണം :354 ഹിജ്‌റ): ‘അസ്സീറത്തുന്നബവിയ്യ വ അഖ്ബാരിൽ ഖുലഫാ’ (2:476).

* മുത്വഹ്‌ഹിർ ഇബ്നു ത്യാഹിർ അൽമക്ദസി (മരണം: 355 ഹി): ‘അൽബദ്ഉ വത്താരീഖ്’ (5:175).

* ഇബ്നുൽ അസീർ (മരണം: 630 ഹി): ‘അൽ കാമിൽ ഫിത്താരിഖ് ‘ (2:316).

* ദഹബി (മരണം: 748 ഹി): ‘സിയറു അഅ്ലാമിന്നുബലാഅ്’ (2:393).

* ഇബ്നു കസീർ (മരണം: 774 ഹി): ‘അൽ ബിദായ വന്നിഹായ’ (7:57).

ഖലീഫ ഉമറിന്റെ കാലഘട്ടത്തിലാണ് ബസ്വറ മുസ്‌ലിംകൾ വിജയിച്ചടക്കുന്നതും പട്ടണമാക്കുന്നതും എന്നും ഇതിനെ പറ്റി പ്രവാചകൻ (സ) സുവിശേഷമറിയിച്ചിട്ടുണ്ട് എന്നും ഇബ്നുൽ വർദ്ദി (മരണം: 749) തന്റെ ‘താരീഖിൽ’ (1:137) രേഖപെടുത്തുന്നു. ഇന്ത്യയെ സംബന്ധിച്ച, നാം ചർച്ച ചെയ്യുന്ന ഹദീസിനെ സംബന്ധിച്ചാണ് ഇബ്നുൽ വർദ്ദി സൂചിപ്പിക്കുന്നത്.

വെള്ള കലർന്ന മിനുസമുള്ള പാറയെയും കല്ലുകളേയുമാണ് ‘ബസ്റ’ (اﻟﺒَﺼْﺮَﺓُ) എന്ന് അറബിയിൽ വിളിക്കപ്പെടുന്നത്. വളരെ കട്ടിയുള്ള ഭൂമിയെ ‘അൽ ബസ്റു’ എന്ന് പറയുമെന്ന് കസ്സാസ് തന്റെ ‘ജാമിഅ്’ ൽ ഭാഷാ പണ്ഡിതന്മാരിൽ നിന്നും ഉദ്ധരിക്കുന്നു. ഈ ഭൂമി മുസ്‌ലിംകൾ വിജയിച്ചടക്കിയപ്പോൾ നിറയെ കല്ലുകൾ ഉള്ളതിനാൽ അവർ ആ നാടിന് ‘ബസ്വറ’ എന്ന് പേര് മാറ്റി വിളിക്കാൻ തുടങ്ങി എന്ന് ഒട്ടനവധി ഭാഷാ പണ്ഡിതരും ചരിത്രകാരൻമാരും രേഖപ്പെടുത്തുന്നുണ്ട്. (താജുൽ ഉറൂസ്: 10:203, ഉംദത്തുൽ ക്വാരി: 6:57)

ശർഖി ഇബ്നുൽ ക്വുത്വാമി പറഞ്ഞു: മുസ്‌ലിംകൾ ബസ്വറയിലേക്ക് വന്നപ്പോൾ ദൂരെ നിന്ന് നിരീക്ഷിച്ച സന്ദർഭത്തിൽ കുറെ കല്ലുകളാണ് അവർ കണ്ടത്. അപ്പോൾ അവർ പറഞ്ഞു: ഇത് കല്ലുകൾ നിറഞ്ഞ ഭൂമി അഥവാ ബസ്വറ ആണ്. അങ്ങനെയാണ് മുസ്‌ലിംകൾക്കിടയിൽ ആ നാടിന് അപ്രകാരം പേര് വന്നത്. (താരീഖു മദീനത്തുൽ ബസ്വറ: അബ്ദുല്ലാഹിബ്നു ഈസബ്‌നു ഇസ്മാഈൽ അന്നജ്ദി :19)

ഹിന്ദ് അഥവാ ബസ്വറയോടുള്ള യുദ്ധം എന്തുകൊണ്ട് പ്രോത്സാഹിപ്പിക്കപ്പെട്ടു? ഹിന്ദ് അഥവാ ബസ്വറയോട് യുദ്ധം എന്തുകൊണ്ട് പ്രോത്സാഹിപ്പിക്കപ്പെട്ടു? എന്നതാണ് അവശേഷിക്കുന്ന ചോദ്യം.

മതേതരത്വവും ജനാധിപത്യവും രാഷ്ട്രങ്ങളുടെ ചട്ടക്കൂടും രാഷ്ട്രമീമാംസയുടെ അടിസ്ഥാന ശിലയുമായ, ആധുനിക ലോക വ്യവസ്ഥയിലല്ല ഹിന്ദ് അഥവാ ബസ്വറയോട് യുദ്ധം ചെയ്യുന്നത് പ്രവാചകൻ (സ) പ്രോത്സാഹിപ്പിച്ചത്. ഇസ്‌ലാമിന്റെ ആവിർഭാവകാലഘട്ടത്തിൽ പ്രവാചക ശിഷ്യന്മാർ നേതൃത്വം വഹിച്ച പടയോട്ടങ്ങളുടെ ലക്ഷ്യവും പശ്ചാത്തലവും ആദ്യമായി നാം മനസ്സിലാക്കണം. ആദർശ സഹവർത്തിത്വമോ ജനാധിപത്യ മൂല്യങ്ങളോ തൊട്ടു തീണ്ടിയിട്ടില്ലാത്ത തീവ്ര മത വികാരത്തിലും അന്ധമായ മൗലികവാദത്തിലും വേരുറച്ച മതാധിഷ്‌ഠിത ഏകാധിപത്യ ഭരണങ്ങളായിരുന്നു (Theocratic Autocracy) അന്നത്തെ രാഷ്ട്രങ്ങളിൽ നിലനിന്നിരുന്നത്. ചരിത്രത്തിന്റെ അപൂർവം ചില ദശകളിൽ ചില നാടുകളിൽ അപൂർവ്വം ഭരണാധികാരികൾ മാത്രമെ അപര വിശ്വാങ്ങൾക്കും ആദർശങ്ങൾക്കും പ്രത്യയ ശാസ്ത്രങ്ങൾക്കും സഹവർത്തിത്വത്തിന് അവസരം നൽകിയിരുന്നുള്ളു. മതസ്വാതന്ത്ര്യവും സഹസ്ഥിതിയും വിവിധ പ്രത്യയശാസ്‌ത്രങ്ങളോട് സമാധാനപരമായ സഹവർത്തിത്വവുമൊക്കെ തീർത്തും അസംഭവ്യങ്ങളായിരുന്ന പൗരാണിക ലോക വ്യവസ്ഥയുടെ പശ്ചാത്തലത്തിൽ നിന്നു കൊണ്ട് വേണം ഹദീസിലെ യുദ്ധാഹ്വാനത്തെ മനസ്സിലാക്കാൻ.

ഇസ്‌ലാം സ്വീകരിക്കുകയോ ആചരിക്കുകയോ അതിന്റെ ചിഹ്നങ്ങൾ പ്രകടിപ്പിക്കുകയോ, ഇസ്‌ലാം പ്രചരിപ്പിക്കുകയോ ചെയ്യുന്നതിനുള്ള സ്വാതന്ത്ര്യം നിഷേധിക്കപ്പെടുകയും അവയുടെ പേരിൽ മർദ്ദനങ്ങളും പീഢനങ്ങളും അനുഭവിക്കുകയും ചെയ്യേണ്ടിവരുന്ന അവസ്ഥ ഇസ്‌ലാമികേതര രാജ്യങ്ങളിൽ ഇല്ലാതാവുകയും ഇസ്‌ലാം മതം പൂർണമായും ഉൾക്കൊണ്ട് ജീവിക്കാൻ കഴിയുന്ന അവസ്ഥ സംജാതമാവുകയും ചെയ്യുന്നതിന് മാത്രമായിരുന്നു ഈ യുദ്ധങ്ങൾ.

“ഫിത്‌ന ഇല്ലാതാകുന്നതുവരെ യുദ്ധം നടത്തി കൊള്ളുക” ( وَقَاتِلُوهُمْ حَتَّىٰ لَا تَكُونَ فِتْنَةٌ ) എന്ന ഖുർആൻ വചനത്തിന് പ്രവാചക ശിഷ്യൻ ഇബ്നു ഉമർ (റ) നൽകിയ വ്യാഖ്യാനം കാണുക:

ﻓﻌﻠﻨﺎ ﻋﻠﻰ ﻋﻬﺪ ﺭﺳﻮﻝ اﻟﻠﻪ ﺻﻠﻰ اﻟﻠﻪ ﻋﻠﻴﻪ ﻭﺳﻠﻢ ﻭﻛﺎﻥ اﻹﺳﻼﻡ قليلا، ﻓﻜﺎﻥ اﻟﺮﺟﻞ ﻳﻔﺘﻦ ﻓﻲ ﺩﻳﻨﻪ: ﺇﻣﺎ ﻗﺘﻠﻮﻩ، ﻭﺇﻣﺎ ﻳﻌﺬﺑﻮﻧﻪ، ﺣﺘﻰ ﻛﺜﺮ اﻹﺳﻼﻡ ﻓﻠﻢ ﺗﻜﻦ ﻓﺘﻨﺔ

“അല്ലാഹുവിന്റെ തിരുദൂതരുടെ(സ) കാലത്ത് ഞങ്ങള്‍ അങ്ങനെ (ഫിത്‌ന ഇല്ലാതാകുന്നതുവരെ യുദ്ധം നടത്തുക എന്ന പ്രവർത്തനം) ചെയ്തിട്ടുണ്ട്. അന്ന് ഇസ്‌ലാം (മുസ്‌ലിംകള്‍) അല്‍പമായിരുന്നു. അതിനാല്‍, ഒരു മുസ്‌ലിം തന്‍റെ മത കാര്യത്തില്‍ ഫിത്‌നക്ക് (പരീക്ഷണത്തിന്/ കുഴപ്പത്തിന്) വിധേയനാകുമായിരുന്നു. ഒന്നുകില്‍ അവിശ്വാസികൾ അവനെ വധിക്കുമായിരുന്നു, അല്ലെങ്കില്‍ അവിശ്വാസികൾ അവനെ മര്‍ദ്ദനങ്ങൾക്കും പീഢനങ്ങൾക്ക് വിധേയമാക്കുകയും ചെയ്യുമായിരുന്നു. അങ്ങനെ, ഇസ്‌ലാം (മുസ്‌ലിംകള്‍) വര്‍ദ്ധിച്ചു, അപ്പോള്‍ ഈ ഫിത്‌ന ഇല്ലാതായി….’ (സ്വഹീഹുൽ ബുഖാരി: 4514)

ഇസ്‌ലാം സ്വീകരിക്കുവാനും ജീവിതത്തിൽ പ്രാവർത്തികമാക്കാനും, വസ്ത്രധാരണ രീതി, ബാങ്ക്, ആരാധനാലയങ്ങൾ, മതപഠനം, ആഘോഷങ്ങൾ തുടങ്ങിയ ഇസ്‌ലാമിക ചിഹ്നങ്ങൾ ആചരിക്കുവാനും, ഇസ്‌ലാമിക പ്രബോധന-പ്രചാരണങ്ങളിൽ ഏർപ്പെടാനും ഒരു വിശ്വാസിക്ക് ദൈവത്തിന്റെ ഭൂമിയിൽ എവിടെയും സാധ്യമാകുന്ന സ്ഥിതി വിശേഷം നിലവിൽ വരണം. അഥവാ മുസ്‌ലിം സമൂഹം ഇസ്‌ലാം വിരുദ്ധ രാഷ്ട്രങ്ങളിൽ നിന്ദ്യരും മർദ്ദിതരുമായി കഴിയുന്ന അവസ്ഥ ഇല്ലാതാവുകയും അല്ലാഹുവിന്റെ മതം ഭൂമിയിലെ ഏത് സാമൂഹിക വ്യവസ്ഥിതിയിലും പ്രൗഢിയോടെ പുലർത്താൻ സാധിക്കണം. (അതിനർത്ഥം മറ്റു മതങ്ങളോ ആദർശങ്ങളോ ഇസ്‌ലാമിന് കീഴ്‌പ്പെടുന്ന അവസ്ഥ സൃഷ്ടിക്കണമെന്നോ ഭൂമിയിൽ മുഴുവൻ ഇസ്‌ലാമിക ഭരണം സ്ഥാപിക്കപ്പെടണം എന്നോ അല്ല. ഇസ്‌ലാമും മുസ്‌ലിംകളും മറ്റു മതങ്ങളാലും ആദർശങ്ങളാലും അടിച്ചമർത്തപ്പെടുകയും നിന്ദ്യരാക്കപ്പെടുകയും ചെയ്യുന്ന ദുരവസ്ഥ ഇല്ലാതാവുകയും ഇസ്‌ലാമിക പ്രബോധനം സാധ്യമാവുകയും ചെയ്യണം എന്നാണ്.) അതിനു തടസ്സമായി നിൽക്കുന്ന ഓട്ടോക്രസികളോടാണ് -ഇസ്‌ലാമിക രാഷ്ട്രത്തിന്റെ ഭരണാധികാരി എന്ന നിലയിൽ- ചില ‘സ്വതന്ത്ര്യ സമരങ്ങൾ/ യുദ്ധങ്ങൾ’ നടത്താൻ മുഹമ്മദ് നബി (സ) ആഹ്വാനം ചെയ്തത്. ഇതാണ് ഈ സ്വാതന്ത്ര്യ യുദ്ധങ്ങളുടെ ലക്ഷ്യം.

“മര്‍ദ്ദനം ഇല്ലാതാവുകയും, മതം അല്ലാഹുവിന് വേണ്ടിയാവുകയും ചെയ്യുന്നത് വരെ നിങ്ങളവരോട് യുദ്ധം നടത്തിക്കൊള്ളുക. എന്നാല്‍ അവര്‍ (യുദ്ധത്തില്‍ നിന്ന്‌) വിരമിക്കുകയാണെങ്കില്‍ (അവരിലെ) അക്രമികള്‍ക്കെതിരിലല്ലാതെ പിന്നീട് യാതൊരു കയ്യേറ്റവും പാടുള്ളതല്ല.” (ഖുർആൻ: 2:193)

“എന്നാല്‍ അവര്‍ (യുദ്ധത്തില്‍ നിന്ന്‌) വിരമിക്കുകയാണെങ്കില്‍ (അവരിലെ) അക്രമികള്‍ക്കെതിരിലല്ലാതെ പിന്നീട് യാതൊരു കയ്യേറ്റവും പാടുള്ളതല്ല.” എന്ന ഭാഗം ഈ യുദ്ധങ്ങളുടെ പശ്ചാത്തലത്തിലേക്ക് കൂടുതൽ വെളിച്ചം വീശുന്നുണ്ട്. ആധുനിക മതേതര ജനാധിപത്യ രാഷ്ട്രങ്ങളിൽ ഇഷ്ടമുള്ള മതം സ്വീകരിക്കാനും അനുഷ്ഠിക്കാനും പ്രചരിപ്പിക്കാനും സ്വാതന്ത്ര്യമനുവദിക്കുന്ന അവസ്ഥ നിലനിൽക്കുമ്പോൾ ഇസ്‌ലാമിലെ സ്വാതന്ത്ര്യ യുദ്ധങ്ങൾക്ക് പ്രസക്തിയില്ല.

മുമ്പ് സൂചിപ്പിച്ചത് പോലെ, സഹവർത്തിത്വവും ജനാധിപത്യവും ഒരു നിലക്കും ഉൾകൊള്ളാത്ത തീവ്ര മത വികാരത്തിലധിഷ്ഠിതമായ തിയോക്രസികളായിരുന്നു ഇസ്‌ലാം ഉദയം കൊണ്ട കാലഘട്ടത്തിൽ ലോകത്തുടനീളം നിലനിന്നിരുന്നത്; പ്രത്യേകിച്ച് അറബ് ഉപഭൂഖണ്ഡത്തിൽ അധികാരം വാണിരുന്നത്. അറബ് ഉപഭൂഖണ്ഡത്തിലെ ഗോത്ര വ്യവസ്ഥ മതസ്വാതന്ത്രത്തെ മുഴുവനായും നിഷേധിച്ചിരുന്നു. പ്രസ്തുത സമൂഹങ്ങളിലെയും ഗോത്രങ്ങളിലേയും രാജാക്കന്മാർക്ക് പോലും സമൂഹ മനസ്സിനെതിരായ ആദർശങ്ങളോ മതമോ സ്വീകരിക്കാനും വിശ്വസിക്കാനുമുള്ള അവകാശവും സ്വാതന്ത്ര്യവും ഇല്ലാതിരിക്കാൻ മാത്രം തീവ്രമായിരുന്നു അന്ന് നിലനിന്നിരുന്ന വർഗീയതയും അസഹിഷ്ണുതയും.

റോമൻ ചക്രവർത്തിക്ക് പ്രവാചകൻ (സ) ഒരു കത്തയക്കുകയുണ്ടായി. ഹാരിസിബ്നു ഉമൈർ അൽ അസ്ദിയായിരുന്നു കത്തുമായി പുറപ്പെട്ട ദൂതൻ. റോമൻ ചക്രവർത്തിയുടെ, ശാമിലെ ഗവർണറായ ശർഹബീലിബ്നു അംറ് അൽ ഗസ്സാനി, പ്രവാചകന്റെ ദൂതനെ ബന്ദിയാക്കുകയും കെട്ടിയിട്ട് കഴുത്തറുക്കുകയും ചെയ്തു. തങ്ങളുടെ ക്രിസ്ത്യൻ സാമ്രാജ്യത്തിന്റെ അതിർത്തിയിൽ കാലു കുത്താൻ പോലും ഒരു മുസ്‌ലിമിനും അനുവാദമില്ലെന്ന്, ദൂതന്മാരെ വധിക്കുക എന്ന, അക്കാലഘട്ടത്തിലെ അസഹിഷ്ണുതയുടെ പാരമ്യരൂപത്തിലൂടെ റോമൻ സാമ്രാജ്യം പ്രഖ്യാപിച്ചു. ഇതാണ് ഹിജ്റ എട്ടാം വർഷം ശാമിലെ ക്രിസ്ത്യാനികളും മുസ്‌ലിംകളും തമ്മിൽ നടന്ന യുദ്ധമായ മുഅ്ത യുദ്ധത്തിലേക്ക് നയിച്ചത് എന്ന് ചില ചരിത്ര ഗ്രന്ഥങ്ങൾ സൂചിപ്പിക്കുന്നു. (മഗാസി: വാക്വിദി: 2:755, ത്വബകാത്തു ഇബ്നു സഅ്ദ്: 4:256)

പ്രേർഷ്യൻ സാമ്രാജ്യത്വത്തിന്റെ ആദർശ അസഹിഷ്ണുത ചരിത്ര പ്രസിദ്ധമാണ്. സ്വാതന്ത്ര്യ നിഷേധത്തിൽ പരസ്പരം മത്സരിച്ചിരുന്ന ക്രിസ്ത്യൻ റോമും പേർഷ്യൻ ജൂതന്മാരും തമ്മിലുള്ള അടിച്ചമർത്തലുകളും വർഗീയ യുദ്ധങ്ങളും കലാപങ്ങളും അക്കാലഘട്ടത്തിന്റെ പ്രധാന ഇതിവൃത്തം ആയിരുന്നു.

ഹദീസിൽ ഉദ്ദേശിക്കപ്പെട്ട ഹിന്ദ് അഥവാ ബസ്വറ ആധുനിക ഇറാഖിന്റെ ഭാഗമാണെന്നും ക്രിസ്താബ്ദം 638 ൽ ഖലീഫ ഉമറിന്റെ കാലഘട്ടത്തിൽ മുസ്‌ലിംകൾ വിജയിച്ചടക്കുന്നതു വരെ ഇറാഖ് പേർഷ്യൻ സാമ്രാജ്യത്വത്തിന്റെ കീഴിലായിരുന്നു എന്നും സൂചിപ്പിച്ചല്ലൊ.

എൻസൈക്ലോപീഡിയ ഇറാനിക (Encyclopedia iranica) പറയുന്നു:

സാസാനിയൻ രാജാക്കൻമാരാണ് പേർഷ്യൻ സാമ്രാജ്യത്തെ നയിച്ച അവസാന രാജവംശം. പേർഷ്യൻ സാമ്രാജ്യത്തിനു മേൽ ഇസ്‌ലാമിന്റെ അധിജയത്തിന് തൊട്ട് മുമ്പ് (224 CE–650 CE) വരെ ഭരണത്തിന് നേതൃത്വം വഹിച്ച വംശപരമ്പരയാണ് സാസാനിയൻ രാജവംശം. (https://iranicaonline.org/articles/sasanian-dynasty )

അബ്ദുശ്ശാഫി മുഹമ്മദ് അബ്ദുല്ലത്തീഫ് എഴുതി:

“പേർഷ്യൻ രാജാവായ കിസ്റക്ക് -അദ്ദേഹത്തെ ഇസ്‌ലാമിലേക്ക് ക്ഷണിച്ചു കൊണ്ട്- മുഹമ്മദ് നബി (സ) അയച്ച കത്ത് അഹങ്കാരത്തോടെയും ധാർഷ്ട്യത്തോടെയും പിച്ചിച്ചീന്തുകയാണ് ആ ഏകാധിപതി ചെയ്തത്. മാത്രമല്ല യമനിൽ ഭരണം നടത്തിയിരുന്ന തന്റെ പ്രതിനിധിയായ ബാദാനോട് മുഹമ്മദ് നബിയെ പിടികൂടാനും കൊല്ലാനും ആവശ്യപ്പെടുകയും ചെയ്തു. രാജാധിരാജനായ തന്നെ മറ്റൊരു മതത്തിലേക്ക് ക്ഷണിക്കാനുള്ള തന്റേടത്തിനു ശിക്ഷയായി കൊണ്ടാണ് മുഹമ്മദ് നബിയെ വധിക്കാൻ അദ്ദേഹം ഉത്തരവിട്ടത് (താരീഖു ത്വബ്‌രി 2:654). തുടക്കം മുതലേ അങ്ങേയറ്റം ശത്രുതാ മനോഭാവത്തോടെയാണ് പേർഷ്യൻ സാമ്രാജ്യത്വം ഇസ്‌ലാമിനെ അഭിമുഖീകരിച്ചത്. കാലാന്തരത്തിൽ ഈ അനീതി നിറഞ്ഞ ശത്രുത ഇസ്‌ലാമിക രാഷ്ട്രത്തിന്റെ അതിർത്തികളിലുള്ള അറബ് ഗോത്രങ്ങളെ ഭരണകൂടത്തിനെതിരെ ഇളക്കി വിടുകയും സായുധ വിപ്ലവങ്ങൾക്ക് പ്രോത്സാഹിപ്പിച്ചു കൊണ്ടിരിക്കുകയും ചെയ്തു. ഇത് യുദ്ധത്തിലാണ് കലാശിച്ചത്. ആഭ്യന്തര ലഹളക്കാരുമായും സഹായികളായ പേർഷ്യക്കാരുമായും മുസ്ന ഇബ്നു ഹാരിസ അശൈബാനിയുടെ നേതൃത്വത്തിലുള്ള മുസ്‌ലിം സൈന്യം അറബ് ഗൾഫ് തീരത്ത് നിരന്തരം ഏറ്റുമുട്ടി. ഇറാൻ മുഴുവൻ വിജയിച്ചടക്കുകയും ഇസ്‌ലാമിക നേതൃത്വത്തിനു കീഴിൽ സമ്പൂർണ്ണമായി പ്രവേശിക്കുകയും ചെയ്യുന്നത് വരെ പേർഷ്യക്കാരിൽ നിന്നുമുള്ള വൈദേശിക കലാപങ്ങൾ കെട്ടടങ്ങിയില്ല. മുസ്‌ലിംകൾ അല്ല ശത്രുതയ്ക്ക് തുടക്കം കുറിച്ചത് എന്നതിനാൽ തന്നെ നീതിയുള്ള നിഷ്പക്ഷരായ ആർക്കും പേർഷ്യക്കാരോടുള്ള യുദ്ധത്തിൽ മുസ്‌ലിംകളെ പഴിക്കാൻ കഴിയില്ല… പേർഷ്യൻ രാജ്യങ്ങളിലുള്ള വിജയത്തിന്റെ സുവിശേഷങ്ങളും യുദ്ധാർജിത സ്വത്തുക്കളും മദീനയിൽ ഖലീഫാ ഉമറിലേക്ക് മുസ്‌ലിം സൈന്യത്തിന്റെ നേതാക്കളിൽ ഒരാളായ സഅദ്ബ്നു അബീ വഖാസ് അയക്കുകയും പേർഷ്യൻ രാജ്യങ്ങളിൽ യുദ്ധം തുടരാനുള്ള അനുവാദം ആവശ്യപ്പെടുകയും ചെയ്തപ്പോൾ ഉമർ (റ) അത് നിരസിച്ചു കൊണ്ട് പറഞ്ഞു: “അവർക്കും നമുക്കുമിടയിൽ ഒരു വലിയ പർവതമുണ്ടാവുകയും നാം അവരിൽ നിന്നും, അവർ നമ്മിൽ നിന്നും സുരക്ഷിതരായിരിക്കുകയും ചെയ്തിരുന്നെങ്കിൽ എന്നാണ് ഞാൻ ആശിച്ചു പോവുന്നത്… യുദ്ധാർജിത സ്വത്തിനേക്കാൾ മുസ്‌ലിംകളുടെ സുരക്ഷയെയാണ് ഞാൻ മുന്തിക്കുന്നത്” (താരീഖു ത്വബ്‌രി 4:28). ഖലീഫയുടെ ഈ നിലപാട് മുസ്‌ലിംകൾ യുദ്ധക്കൊതിയന്മാർ അല്ലായിരുന്നു എന്നതിന് ഏറ്റവും വലിയ തെളിവാണ്. ശക്തിയും വാളും ഉപയോഗിച്ച് ഇസ്‍ലാം പ്രചരിപ്പിക്കൽ ആയിരുന്നില്ല മുസ്‌ലിംകളുടെ ഉദ്ദേശ്യം. (അസ്സീറത്തുന്നബവിയ്യ വത്താരീഖുൽ ഇസ്‌ലാം: 1:259)

എൻസൈക്ലോപീഡിയ ഇറാനിക (Encyclopedia iranica) പറയുന്നു:

സാസാനിയൻ രാജവംശത്തിലെ ഏറ്റവും കൂടുതൽ കാലം (r. 309-79 CE) ഭരിച്ച ചക്രവർത്തി ‘ശാഹ്പോർ രണ്ടാമൻ’ പേർഷ്യൻ സ്വേച്ഛാധിപത്യത്തിന്റെയും അസഹിഷ്ണുതയുടേയും പ്രതീകമായിരുന്നു.

ഇസ്‌ലാമിന് മുമ്പത്തെ അറബ് ഗോത്രങ്ങളെ അടിച്ചമർത്താനും സാമ്രാജ്യത്തിന്റെ അതിർത്തികൾ സുരക്ഷിതമാക്കാനും അയാൾ ഒരു സൈനിക പ്രചാരണം ആരംഭിച്ചു. (താരീഖുത്വബ്‌രി, തജാറുബുൽ ഉമം: മസ്കവൈഹി).

ﻓﺄﻓﺸﻲ ﻓﻴﻬﻢ اﻟﻘﺘﻞ، ﻭﺳﻔﻚ ﻓﻴﻬﻢ ﻣﻦ اﻟﺪﻣﺎء ﺳﻔﻜﺎ ﺳﺎﻟﺖ ﻛﺴﻴﻞ اﻟﻤﻄﺮ، ﺣﺘﻰ ﻛﺎﻥ اﻟﻬﺎﺭﺏ ﻣﻨﻬﻢ ﻳﺮﻯ ﺃﻧﻪ ﻟﻦ ﻳﻨﺠﻴﻪ ﻣﻨﻪ ﻏﺎﺭ ﻓﻲ ﺟﺒﻞ، ﻭﻻ ﺟﺰﻳﺮﺓ ﻓﻲ ﺑﺤﺮ….ﻭﺇﻥ ﺳﺎﺑﻮﺭ ﺿﺮﻱ ﺑﻘﺘﻞ اﻟﻌﺮﺏ، ﻭﻧﺰﻉ ﺃﻛﺘﺎﻑ ﺭﺅﺳﺎﺋﻬﻢ ﺇﻟﻰ ﺃﻥ ﻫﻠﻚ. ﻭﻛﺎﻥ ﺫﻟﻚ ﺳﺒﺐ ﺗﺴﻤﻴﺘﻬﻢ ﺇﻳﺎﻩ ﺫا اﻷﻛﺘﺎﻑ

ത്വബ്‌രി എഴുതുന്നു: “അറബികളെ അയാൾ പരക്കെ കൊന്നു. മഴ കുത്തിച്ചൊരിയുന്നതു പോലെ അറേബ്യയിൽ രക്തം ചിന്നി ചിതറി. മരണത്തിൽ നിന്ന് ഓടി രക്ഷപ്പെടാൻ ശ്രമിച്ചവർ തന്നെ മനസ്സിൽ മന്ത്രിച്ചു; ഒരു പർവ്വതത്തിനു മൂലയിലുള്ള ഗുഹയും ഒരു സമുദ്ര പ്രാന്തത്തിലുള്ള ദ്വീപും തന്നെ ഒരിക്കലും രക്ഷിക്കില്ല എന്ന്. ‘ശാഹ്പോർ’ ചക്രവർത്തി അറബികളെ കൂട്ടക്കുരുതി നടത്തി. അറബികളുടെ നേതാക്കളുടെ തോളെല്ലുകൾ വലിച്ചൂരി അവരെ ക്രൂരമായ രീതിയിൽ കൊലപ്പെടുത്തി. അക്കാരണത്താൽ അയാളെ ‘ദുൽ അക്താഫ്’ (ﺫﻱ اﻻﻛﺘﺎﻑ) ‘തോളെല്ലുകളുടെ ഉടമ’ എന്ന് അറബികൾ വിളിച്ചു.” (താരീഖു ത്വബ്‌രി: 2:57)

‘ശാഹ് പോർ’ രണ്ടാമൻ ഇറാഖിലെ അയാദുകളെ ആദ്യം ആക്രമിച്ചു. പിന്നീട് പേർഷ്യൻ ഗൾഫ് മുറിച്ച് കടന്ന് ബഹ്റൈൻ, കത്വർ എന്നീ പ്രദേശങ്ങളും, തമീം, ബകർ, വാഇൽ, അബ്ദുൽ കൈസ് തുടങ്ങിയ ഗോത്രക്കാരുടെ വാസസ്ഥലമായ ഹജറും ആക്രമിച്ചു. (താരീഖു ത്വബ്‌രി) ഒട്ടുമിക്ക ഗോത്രവർഗ്ഗക്കാരേയും കശാപ്പു ചെയ്യുന്നതിന് പുറമെ അറബികൾക്ക് വെള്ളം കിട്ടാതിരിക്കാൻ ജലസ്രോതസ്സുകൾ മണ്ണിട്ടു മൂടി.

തുടർന്ന് കിഴക്കൻ അറേബ്യയിലും സിറിയയിലും ആക്രമണം അഴിച്ചു വിടുകയും യമാമ, ബക്കർ, തഗ്‌ലിബ് നഗരങ്ങൾക്കുമെതിരെ ആഞ്ഞടിച്ചു. (ഗററു അഖ്ബാരി മുലൂകുൽ ഫറസ്: സഗാലിബി, തജാറുബുൽ ഉമം: മസ്കവൈഹി)

ചില അറബ് ഗോത്രങ്ങളെ നിർബന്ധിതമായി മാറ്റിപ്പാർപ്പിക്കുകയും സാസാനിയൻ സാമ്രാജ്യത്തിലേക്ക് നാടുകടത്തുകയും ചെയ്യപ്പെട്ടു.

അറബികളിൽ നിന്ന് തുടർന്ന് ആക്രമണങ്ങൾ ഉണ്ടാകാതിരിക്കാൻ, ‘ശാഹ്പോർ’ II, “അറബി മതിൽ” എന്ന പേരിൽ ഒരു പ്രതിരോധ സംവിധാനം നിർമ്മിച്ചു. (T. Daryaee, “Memory and History: The Construction of the Past in Late Antique Persia,” Nāma-ye Irān-e Bāstan/The International Journal of Ancient Iranian Studies 1/2, 2001-02, pp. 1-14)

‘ശാഹ് പോർ’ II ന്റെ ഭരണകാലത്ത് സാസാനിയൻ സാമ്രാജ്യത്തിലെ ക്രിസ്ത്യാനികൾ നേരിടേണ്ടി വന്ന കൊടും പീഡനങ്ങൾ അർമേനിയൻ സ്രോതസ്സുകൾ പോലും രേഖപ്പെടുത്തുന്നുണ്ട്.

‘ശാഹ് പോർ’ രണ്ടാമന്റെ വേട്ടക്കിരയായ പ്രശസ്ത ക്രിസ്ത്യൻ രക്തസാക്ഷികളുടെ പേരുകളിൽ അയാളുടെ പ്രധാന കരകൗശലത്തൊഴിലാളിയായ പോസി (പുസൈ), അദ്ദേഹത്തിന്റെ മകൾ മാർത്ത, തെക്ല, ദാനക്, ബൗത എന്നിവരും ഉൾപെടുന്നു. സ്ത്രീകൾ ഉൾപ്പെടെ സാൻ, മാമ, മെസഖ്യ, അന്ന, അബ്യത്, സാത്തായ്, മെസഖ്യ എന്നിങ്ങനെ മറ്റ് ഒരുപാട് രക്തസാക്ഷികളും അവരുടെ കൂട്ടത്തിൽ ഉണ്ടായിരുന്നു. (Brock and S. A. Harvey, Holy Women of the Syrian Orient, Berkeley, 1998, pp. 68-77) (https://iranicaonline.org/articles/shapur-ii )

സൊസോമന്റെ Ecclesiastical History ൽ, ‘ശാഹ്പോർ’ രണ്ടാമന്റെ കീഴിൽ രക്തസാക്ഷികളായ പേർഷ്യൻ ക്രിസ്ത്യാനികളെക്കുറിച്ച് ഗണ്യമായ വിശദാംശങ്ങൾ അടങ്ങിയിരിക്കുന്നു: “ഈ കാലയളവിൽ രക്തസാക്ഷികളായ പുരുഷന്മാരുടെയും സ്ത്രീകളുടെയും പേരുകളിൽ -സ്ഥിരീകരിക്കപ്പെട്ടവരുടെ എണ്ണം- പതിനാറായിരത്തിലധികമായി കണക്കാക്കപ്പെടുന്നു. എന്നാൽ ഈ കാലയളവിൽ പേരറിയാത്ത, രക്തസാക്ഷികളായ സ്ത്രീകളുടെയും പുരുഷന്മാരുടെയും എണ്ണം പതിനാറായിരത്തിലും എത്രയോ അധികമായതിനാൽ പേർഷ്യക്കാരും, സിറിയക്കാരും, എഡെസ്സ നിവാസികളും, എത്ര ശ്രമിച്ചിട്ടും ആ കണക്ക് നിജപ്പെടുത്താൻ സാധിക്കാതെ പരാജയമടഞ്ഞു.” ( Ecclesiastical History, Book II, Chapter XIV )

ഇസ്‌ലാമിന്റെ ആവിർഭാവത്തിനു മുമ്പ് തന്നെ പേർഷ്യൻ റോമൻ സാമ്രാജ്യങ്ങൾ മത സ്വേച്ഛാധിപത്യത്തിൽ കേളികേട്ടവരായിരുന്നു. ഇരു സാമ്രാജ്യങ്ങളേയും ആദർശപരമായി നയിച്ചിരുന്ന സൊറോസ്ട്രിയർ, ക്രിസ്ത്യാനികൾ, ജൂതന്മാർ തുടങ്ങി വ്യത്യസ്ഥ ആദർശവിശ്വാസികൾ – അവർ ഓരോരുത്തർക്കും ലഭ്യമാകുന്ന രാഷ്ട്രീയ അനുകൂലാവസ്ഥക്ക് അനുസൃതമായി- നടപ്പാക്കിയിരുന്ന അന്യമത ധ്വംസനങ്ങളും ഭരണ വടം വലികളും അന്നത്തെ ലോക വ്യവസ്ഥയിലെ അസഹിഷ്ണുതയുടെ ആഴം വ്യക്തമാക്കിത്തരുന്നുണ്ട്.

ഇസ്‌ലാംഭരണത്തിനു മുമ്പുള്ള പേർഷ്യയിലെ അവസാനസാമ്രജ്യമായിരുന്ന സാസാനിയൻ സാമ്രാജ്യത്തിന്റെ കാലത്ത് സൊറോസ്ട്രിയൻ മതം സാമ്രാജ്യത്തിലെ ഔദ്യോഗികമതമായിരുന്നു. കൂടാതെ ഈ സമയത്ത് സൊറോസ്ട്രിയൻ മതം സംഘടനാരൂപം കൈവരിക്കുകയും ചെയ്തു.

സാസാനിയൻ കാലഘട്ടത്തിലെ സൊറാസ്ട്രിയനിസം (224-651) പേർഷ്യൻ ചക്രവർത്തിയായ ശഹൻ-ശായുടെ (രാജാധി രാജൻ) രക്ഷാകർതൃത്വം ആസ്വദിച്ചു പോന്നു.

ഹഖാമനീഷിയാൻ പേർഷ്യൻ രാജാക്കന്മാരുടെ മഹത്വത്തിന്റെ നേരിട്ടുള്ള അവകാശികളായി സ്വയം കണ്ട സാസാനിയൻ സാമ്രാജ്യത്വ ഭരണകൂടം മസ്ദ-ആരാധനയിൽ അധിഷ്ടിതമായ, സോറോസ്ട്രിയൻ വിശ്വാസത്തിന്റെ സംരക്ഷകരായി സ്വയം അവരോധിച്ചു. (Zoroastrians and Christians in Sasanian Iran: A.V Williams: Department of Religions and Theology, University of Manchester: Page: 2)

ഈ കാലഘട്ടത്തിൽ അഹ്രിമാനും ദേവുകളും വലിയ പ്രഹരവും ഉപദ്രവവും ഭരണകൂടത്തിൽ നിന്ന് നേരിട്ടു. അവരുടെ വിശ്വാസങ്ങൾ ഭൂമിയിൽ നിന്ന് പുറത്താക്കപ്പെടുകയും വിശ്വാസ്യത നഷ്ടപ്പെടുകയും ചെയ്തു. യഹൂദരും ബുദ്ധമതക്കാരും ബ്രാഹ്മണരും അരാമിക്, ഗ്രീക്ക് ഭാഷകൾ സംസാരിക്കുന്ന ക്രിസ്ത്യാനികളും സ്നാപനമേറ്റവരും മനിക്കേയന്മാരും പേർഷ്യൻ ഭൂമിയിൽ ആക്രമിക്കപ്പെട്ടു. ചിത്രങ്ങൾ തകർക്കപ്പെട്ടു, ആരാധനാലയങ്ങളും വാസസ്ഥലങ്ങളും നശിപ്പിക്കപ്പെട്ടു, അഗ്നിക്ഷേത്രങ്ങൾ ധാരാളമായി സ്ഥാപിക്കപ്പെട്ടു. (Mary Boyce, Textual sources for the study of Zoroastrianism (Manchester: Manchester University Press, 1984), 112, translating from M. Back, Die sassanidischen Staatsinschriften (Acta Iranica, 18, 1978), 384ff.)

ഏറ്റവും കൂടുതൽ കുപ്രസിദ്ധമായ പീഡന പരമ്പരയുടെ കണക്കുകൾ ശാഹ്പോർ രണ്ടാമന്റെ ഭരണകാലഘട്ടത്തിൽ (309-79) നിന്നുള്ള ഇരുപത്തിയൊമ്പത് രക്തസാക്ഷികളാണ്. മുഖ്യ ആത്മീയ നേതാവായ അദുർബാദിന്റെ നേതൃത്വത്തിൽ ക്രിസ്ത്യാനികളെ നാൽപത് വർഷത്തോളം സുസ്ഥിരവും നിഷ്കരുണവും ദംശിച്ചു കൊണ്ടിരുന്നു. കോൺസ്റ്റന്റൈൻ ക്രിസ്തുമതം സ്വീകരിച്ച് ക്രിസ്തുമതത്തെ റോമൻ സാമ്രാജ്യത്തിന്റെ മതമാക്കിയതിന് തൊട്ടുപിന്നാലെയാണ് പീഡനത്തിന്റെ തുടക്കം.

ക്രിസ്ത്യൻ പീഡനത്തിന്റെ രണ്ടാം കാലഘട്ടം യാസ്ദെജർദ് ഒന്നാമന്റെയും (399-421) പിന്നീടുള്ള വർഷങ്ങളിൽ അദ്ദേഹത്തിന്റെ പിൻഗാമിയായ ബഹ്റാം അഞ്ചാമന്റെയും ആദ്യ വർഷങ്ങളിലുമായിരുന്നു (c. 421-39).

മൂന്നാമതായി, യാസ്ദെജർദ് രണ്ടാമന്റെ (439-57) ഭരണകാലത്ത്, ക്രിസ്ത്യൻ വിരുദ്ധ മത വിദ്വേഷം പീഡനവും വർദ്ധിച്ചു. ക്രിസ്ത്യൻ അർമേനിയയെ സൊറോസ്ട്രിയനിസത്തിലേക്ക് നിർബന്ധ മതപരിവർത്തനം ചെയ്യാനുളള ശ്രമം ശക്തിയാർജിച്ചു.

പേർഷ്യൻ ക്രിസ്ത്യാനികളുടെ, സുറിയാനി ക്രിസ്ത്യൻ രക്തസാക്ഷികളുടെ പ്രവർത്തനങ്ങളെ (Acts of the Persian Martyrs) സംബന്ധിച്ച രേഖകളിൽ പീഡനവും താഢനവും രക്തസാക്ഷിത്വവും വികാരഭരിതമായ വാക്കുകളിൽ രേഖപ്പെടുത്തപ്പെട്ടിട്ടുണ്ട്. (Zoroastrians and christians in Sasanian Iran: A.V Williams: Department of Religions and Theology, University of Manchester: Page: 4)

സോറാസ്ട്രിയൻ പുരോഹിതന്മാർ സാധാരണക്കാരോടൊപ്പം ഭക്ഷണം കഴിക്കുന്നത് ഒഴിവാക്കണം എന്ന ശക്തമായ കൽപ്പന പുറപ്പെടുവിക്കപ്പെട്ടു. ഒരു അവിശ്വാസിക്ക് എന്തെങ്കിലും ദാനം നൽകുന്നത് നിരോധിച്ചുകൊണ്ടുള്ള സോറോസ്ട്രിയൻ മതശാസനകൾ ഇറക്കപ്പെട്ടു. മരിക്കുമെന്ന് നിങ്ങൾ കരുതുന്നില്ലെങ്കിൽ അല്ലാതെ, ഒരു അവിശ്വാസിയ്ക്ക് നിങ്ങൾ ഭക്ഷണമുൾപ്പെടെ ഒന്നും നൽകരുത് എന്നായിരുന്നു പുരോഹിത ശാസന. (Pahlavi Rivayat, ed. Williams, 1990, 14.7, voL 2, 27.)

ശക്തനായ ഒരു പ്രഭു, ഉദാഹരണത്തിന് ഗിർഡർ, രാജാവിന്റെ മേൽ സ്വാധീനം നേടുമ്പോളെല്ലാം, പല കാരണങ്ങളും ആരോപിച്ച് ക്രിസ്ത്യാനികളും മറ്റ് ന്യൂനപക്ഷങ്ങളും രാഷ്ട്രീയവും മതപരവുമായ ബലിയാടുകളാക്കപ്പെടുക എന്നത് ഒരു സ്ഥിരം പ്രതിഭാസമായിരുന്നു. (Zoroastrians and christians in Sasanian Iran: A.V Williams: Department of Religions and Theology, University of Manchester: Page: 8)

മത പരിത്യാഗം മഹാ കുറ്റകൃത്യമായി നിശ്ചയിക്കപ്പെട്ടു. ഭീഷണികളും പീഡനങ്ങളും ഉപയോഗിച്ച് ക്രിസ്തുമതത്തിൽ നിന്ന് സോറാസ്ട്രിയൻ മതത്തിലേക്ക് നിർബന്ധപൂർവ്വം വലിച്ചിഴക്കപ്പെടുന്ന വർണനകൾ ക്രിസ്ത്യൻ സാധുചരിത്രവർണ്ണന (hagiography) രേഖകൾ മുഴുവൻ നിറഞ്ഞു നിൽക്കുന്നു.

സൂര്യൻ, അഗ്നി, ജലം എന്നിവയെ ആരാധിക്കാൻ പല ക്രിസ്ത്യാനികളേയും സോറാസ്ട്രിയൻ പൗരോഹിത്യം വധഭീഷണിയിലൂടെയും മർദ്ദനങ്ങളിലൂടെയും നിർബന്ധിച്ചു കൊണ്ടിരുന്നു.

ഈസ്റ്റ് ചർച്ച് കത്തോലിക്കരുടെയും ബിഷപ്പുമാരുടെയും തലവനായ ‘മാർ സിമൺ ബാർ സബ്ബാ’യുടെ വധത്തിന് കാരണം, ക്രിസ്ത്യാനികൾക്ക് ഇരട്ട നികുതി ചുമത്താൻ വിസമ്മതിച്ചതാണ്. സിമണേയും അദ്ദേഹത്തിന്റെ എല്ലാ സഹോദരന്മാരെയും വാളാൽ ശിരസ്സു ചേദിച്ച് വധിക്കാൻ രാജ കൽപ്പന നൽകപ്പെട്ടു. ഈ കൂട്ടകൊലക്ക് തൊട്ടു മുമ്പ് സോറാസ്ട്രിയൻ മുഖ്യ പുരോഹിതൻ ഇരകളോട് ഇപ്രകാരം ആവശ്യപ്പെടുകയുണ്ടായി: “ദൈവമായ സൂര്യനെ ആരാധിക്കുക, രാജാക്കന്മാരുടെ രാജാവും എല്ലാ കർത്താക്കളുടെയും കർത്താവുമായ ‘ശാഹ്പോറി’ന്റെ ഹിതം അനുസരിച്ച് പ്രവർത്തിക്കുക, എങ്കിൽ നിങ്ങൾക്ക് ജീവിക്കാം.” അപ്പോൾ അവർ എല്ലാവരും ഉറക്കെ മറുപടി നൽകി: “തീർച്ചയായും ഞങ്ങൾ രാജാക്കന്മാരുടെ രാജാവും മുഴുവൻ ഭൂമിയുടെയും അധിപനും, നിത്യതയുടെ ഉടമയുമായ (സാക്ഷാൽ ദൈവത്തിന്റെ) ഹിതം പ്രവർത്തിക്കും.”

സിമണിന്റെ രക്തസാക്ഷിത്വത്തിന് സാക്ഷ്യം വഹിച്ച പുസൈ ക്രിസ്ത്യൻ യുവാവ്, മറ്റൊരു ക്രിസ്ത്യൻ വയോധികനെ വധിക്കുന്ന ചടങ്ങിൽ വെച്ച് പരസ്യമായി പ്രതികരിക്കുകയുണ്ടായി. ഇത് പേർഷ്യൻ രാജാധിരാജനോടുള്ള ധിക്കാരമായി പരിഗണിക്കപ്പെട്ടു. പുസൈയെ ചങ്ങലകളിൽ വലിച്ചിഴക്കുകയും ക്രിസ്തുമതം ഉപേക്ഷിക്കാൻ നിർബന്ധിക്കുകയും ചെയ്തു. വിസമ്മതിച്ചപ്പോൾ വധശിക്ഷക്ക് വിധേയമാക്കി. മാർ അബാ, മാർ ഗിവർഗീസ് എന്നിവരും ക്രിസ്തുമത രക്തസാക്ഷികളിൽ എണ്ണപ്പെടുന്നു. (Zoroastrians and christians in Sasanian Iran: A.V Williams: Department of Religions and Theology, University of Manchester: Page: 12)

പേർഷ്യൻ സാമ്രാജ്യത്വത്തിന്റെ ക്രൂര രാഷ്ട്രീയത്തിന്റെ മറ്റൊരു ഉദാഹരണമാണ് വലേറിയൻ ചക്രവർത്തിയുടെ വധം: റോമൻ ചക്രവർത്തിയായ (ക്രിസ്ത്യൻ വിരുദ്ധനായിരുന്നു) വലേറിയൻ പണം വാഗ്ദാനം ചെയ്ത് സാസാനിയൻമാരുമായുള്ള യുദ്ധം അവസാനിപ്പിക്കാൻ ആഗ്രഹിച്ചു. സാസാനിയൻ രാജാവായ ശാഹ്പോർ ഒന്നാമൻ -ചക്രവർത്തി നേരിട്ട് വന്ന് അദ്ദേഹത്തോട് അഭ്യർത്ഥിക്കാൻ ആഗ്രഹിക്കുന്നു എന്ന സന്ദേശവുമായി- അംബാസഡർമാരെ വലേറിയനിലേക്ക് തിരിച്ചയച്ചു. വലേറിയൻ ഒരു ചെറിയ സംഘത്തോടൊപ്പം ശാഹ്പോറിനടുത്തേക്ക് ചെന്നപ്പോൾ, അദ്ദേഹത്തെ സാസാനിയക്കാർ പിടികൂടി. ബൈസന്റൈൻ ചരിത്രകാരനായ സോസിമസിന്റെ അഭിപ്രായത്തിൽ, പേർഷ്യക്കാർക്കിടയിൽ ഒരു അടിമയെന്ന നിലയിൽ വലേറിയൻ തന്റെ ജീവിതത്തോട് വിട പറഞ്ഞു. പുരാതന ക്രിസ്ത്യൻ എഴുത്തുകാരനും കോൺസ്റ്റന്റൈൻ ഒന്നാമന്റെ ഉപദേശകനുമായ ലാക്റ്റൻഷ്യസ് വാദിക്കുന്നത്, ശാഹ്പോർ ഒന്നാമൻ കുതിരപ്പുറത്ത് കയറാൻ ആഗ്രഹിക്കുമ്പോഴെല്ലാം വലേറിയൻ ചക്രവർത്തിയെ മനുഷ്യ പാദപീഠമായി ഉപയോഗിച്ചിരുന്നു എന്നാണ്.” അദ്ദേഹത്തെ കൊന്നുകളയുകയും, അദ്ദേഹത്തിന്റെ തൊലി മാംസത്തിൽ നിന്ന് ഉരിച്ച് വേർപെടുത്തി, സിന്ദൂരം കൊണ്ട് ചായം പൂശി, ബാർബേറിയൻമാരുടെ ദൈവങ്ങളുടെ ക്ഷേത്രത്തിൽ പ്രതിഷ്ടിക്കുകയും ചെയ്തു” എന്നും ലാക്റ്റൻഷ്യസ് വാദിക്കുന്നു. (https://www.ancient-origins.net/history/what-really-happened-valerian-was-roman-emperor-humiliated-and-skinned-hands-enemy-008598)

വലേറിയന്റെ തൊണ്ടയിലേക്ക് ഉരുക്കിയ സ്വർണ്ണ ദ്രാവകം ഒഴിച്ചാണ് പേർഷ്യക്കാർ വധിച്ചതെന്നും പറയപ്പെടുന്നു.

പേർഷ്യൻ സാമ്രാജ്യം ജൂതവിരുദ്ധമായിരുന്ന കാലഘട്ടത്തിൽ ആദർശ അസഹിഷ്ണുതയുടെ പേരിൽ ജൂതന്മാർ മത ദ്വംസനങ്ങൾക്കും അക്രമങ്ങൾക്കും പാത്രമായി. പഴയനിയമത്തിന്റെ ഭാഗമായ ഗ്രന്ഥങ്ങളിൽ ഒന്നായ ‘എസ്തേറിന്റെ പുസ്തക’ത്തിൽ ‘പൂരിം’ എന്ന പേരിൽ അറിയപ്പെടുന്ന ജൂതാഘോഷത്തിനു പിന്നിലുള്ള കഥ പ്രതിപാദിക്കുന്നുണ്ട്. പൂരിം തിരുനാളിലെ സായാഹ്നത്തിലും അടുത്ത പ്രഭാതത്തിലും ഈ പുസ്തകത്തിന്റെ സമ്പൂർണ്ണപാഠം ജൂതർ ഉറക്കെ വായിക്കുന്നു. പേർഷ്യൻ രാജാവിന്റെ കൊട്ടാരത്തിൽ, ഹാമാന്റെ നേതൃത്വത്തിൽ ജൂതർക്കെതിരെ നടന്ന ഒരു ഉപജാപത്തിന്റേയും, തൽഫലമായി ഉണ്ടാകാനിരുന്ന വംശഹത്യയിൽ നിന്ന്, ജൂതന്മാർ രക്ഷപെടുന്നതിന്റേയും കഥയാണിത്. (The Religious Policy of Xerxes and the “Book of Esther” Robert J. Littman)

ഒരു ജൂത ഉദ്യോഗസ്ഥനായ മൊർദ്ദെകായ്, രാജാവിന്റെ ഉന്നത സഹായിയായ ഹാമാന്റെ മുമ്പിൽ തലകുനിച്ച് വണങ്ങാൻ വിസമ്മതിച്ചു എന്ന കാരണത്താലാണ് രാജ്യത്തെ എല്ലാ ജൂതന്മാരെയും ഉന്മൂലനം ചെയ്യാൻ രാജാവ് പദ്ധതിയിട്ടത്. യഹൂദ മതം ഒരു ജൂതനും, ദൈവമല്ലാതെ മറ്റേതെങ്കിലും വ്യക്തിയെയോ വിഗ്രഹത്തെയോ തലകുനിച്ച് വണങ്ങാൻ അനുവദിക്കുന്നില്ല.

ലബ്‌നാനിലെ പ്രസിദ്ധ (ക്രിസ്ത്യൻ) സാഹിത്യകാരനും ചരിത്രകാരനുമായ ജോർജി സൈദാൻ എഴുതി: പേർഷ്യയും റോമും തമ്മിലുളള ശത്രുത പൗരാണികമാണ്. ഒരു പക്ഷെ ബി.സി അഞ്ചാം നൂറ്റാണ്ടിനുമപ്പുറം അതിന്റെ വേരുകൾ എത്തി നിൽക്കുന്നുണ്ടാകാം. ലോകത്തെ അടക്കി ഭരിക്കാനുള്ള ഇരു സാമ്രാജ്യങ്ങളുടെയും അത്യാഗ്രഹമായിരുന്നു ഈ ശത്രുതയുടെ അടിത്തറ. നൂറ്റാണ്ടുകളോളം നീണ്ടു നിന്ന ഈ അധികാര വടം വലി ഇസ്‌ലാമിന്റെ ആവിർഭാവ കാലഘട്ടത്തിലും തുടർക്കഥയായിരുന്നു. പേർഷ്യൻ സാമ്രാജ്യത്തിന്റെ ആധിപത്യം ഇസ്ര അനൂഷർവാൻ ചക്രവർത്തിയിൽ എത്തിച്ചേർന്നപ്പോൾ റോമൻ സാമ്രാജ്യത്തെ അൽപാൽപ്പമായി പിടിച്ചടക്കാൻ അദ്ദേഹം സൈന്യ വ്യൂഹത്തെ വിന്യസിച്ചു. സിറിയ പിടിച്ചടക്കുകയും അന്താഖിയ ചുട്ടു നശിപ്പിക്കുകയും ഏഷ്യാ മൈനർ കൊള്ളയടിക്കുകയും ചെയ്തു. അന്നത്തെ റോമൻ ചക്രവർത്തിയായിരുന്ന ജസ്റ്റീനിയൻ ചക്രവർത്തിയും വിട്ടുവീഴ്ചയ്ക്ക് തയ്യാറല്ലായിരുന്നു. ക്രിസ്താബ്ദം 541 മുതൽ 561 വരെ ഇരുപതു വർഷം ഇരു രാഷ്ട്രങ്ങളും യുദ്ധത്തിൽ മുഴുകി.

പർവേസ് ചക്രവർത്തിയുടെ കാലഘട്ടത്തിൽ തന്റെ സുഹൃത്ത് മോറിസിന്റെ കൊലപാതകത്തിന് പ്രതികാരമെന്ന പേരിൽ റോമൻ സാമ്രാജ്യത്തെ പർവേസ് ചക്രവർത്തി ആക്രമിക്കുകയുണ്ടായി. ക്രിസ്താബ്ദം 614 ൽ സിറിയ പിടിച്ചടക്കി…

(പേർഷ്യയിൽ ജൂതന്മാർക്ക് സ്വാധീനമുണ്ടായിരുന്ന കാലത്ത്) ജൂതന്മാരുടെ അകമഴിഞ്ഞ സഹായത്താൽ ബൈസാന്റിയൻ പടയെ പർവേസ് ചക്രവർത്തി ഒന്നൊന്നായി കീഴടക്കി. ഈജിപ്ത്, അന്താഖിയ, ദമാസ്ക്കസ്, ബൈത്തുൽ മുഖദസ്‌ തുടങ്ങിയവ പിടിച്ചടക്കി. ജറുസലേമിലെ ബൈത്തുൽ മുഖദസ്‌ കൊള്ളയടിക്കാനും ക്രിസ്ത്യൻ പള്ളികളും പുണ്യപുരുഷന്മാരുടെ കല്ലറകൾ തീയിടാനും അവിടെയുള്ള വിലമതിക്കാനാകാത്ത സ്വത്തുക്കൾ പിടിച്ചു പറിക്കാനും തന്റെ സൈന്യത്തിന് പർവേസ് ചക്രവർത്തി അനുവാദം നൽകി. സിറിയ വരെ ഈ കൊലയും കൊള്ളയും തുടർന്നു. 90000 ക്രിസ്ത്യാനികളെ സൈന്യം കൊന്നൊടുക്കി… ഇതെല്ലാം കണ്ടിട്ടും കാണാത്ത മട്ടിൽ ഭീരുവായി (അന്നത്തെ) ഹെറാക്ലിയസ് ചക്രവർത്തി കൊട്ടാരത്തിൽ തന്നെ ഇരുന്നു; ക്രിസ്താബ്ദം 632 ൽ ഏഷ്യാ മൈനറിൽ വെച്ച് കൊല്ലപ്പെടുന്നത് വരെ… (താരീഖുത്തമദ്ദുനുൽ ഇസ്‌ലാമി:ജോർജി സൈദാൻ: 1: 43-48)

ഹെറാക്ലിയസിന്റെ കാലഘട്ടത്തിൽ ജൂതന്മാർ തങ്ങളുടെ സഹായികളായ പേർഷ്യക്കാരിൽ നിന്നും 80000 ക്രിസ്ത്യൻ ബന്ദികളെ വിലയ്ക്ക് വാങ്ങി അറുത്തു കൊന്നു… (താരീഖുത്തമദ്ദുനുൽ ഇസ്‌ലാമി: ജോർജി സൈദാൻ: 1: 43-48)

ഈ രാഷ്ട്രീയ വ്യവസ്ഥയിലെ, പേർഷ്യൻ അധിനിവേശത്തിനു കീഴിലായിരുന്ന ‘ബസ്വറ’യോടാണ് (ഹിന്ദ്) മുഹമ്മദ് നബി (സ) യുദ്ധത്തിന് ആഹ്വാനം ചെയ്തത്. ഈ യുദ്ധ ആഹ്വാനത്തിന് പശ്ചാത്തലവുമായി ഗാഢമായ ബന്ധമുണ്ട് എന്ന് വ്യക്തമാണല്ലൊ. ഏതെങ്കിലും ഒരു രാഷ്ട്ര നേതാവ് ഒരു രാജ്യത്തോട് ഒരു പ്രത്യേക പശ്ചാത്തലത്തിലും സാഹചര്യത്തിലും യുദ്ധത്തിന് പ്രോത്സാഹനം നൽകിയ ഒരു വാചകം എടുത്തു വെച്ച്, അവ എക്കാലത്തേക്കുമുള്ള കൽപ്പനയായി അവതരിപ്പിക്കുന്നതിലെ ദുരുദ്ദേശം തിരിച്ചറിയപ്പെടണം. ബ്രിട്ടീഷ് സാമ്രാജ്യത്വ അധിനിവേശത്തോട് സമരവും യുദ്ധവും പ്രഖ്യാപിച്ച് കൊണ്ടുള്ള ഇന്ത്യൻ സ്വാതന്ത്ര്യ സമര നേതാക്കളുടെ വാചകങ്ങൾ എടുത്തുദ്ദരിച്ച് ഇന്നത്തെ, അല്ലെങ്കിൽ എക്കാലത്തേയും ബ്രിട്ടനോട് ശത്രുത വെച്ചുപുലർത്താനാണ് അവയുടെ ഉൾസാരമെന്ന് ദുർവ്യാഖ്യാനിക്കുന്നതു പോലെ തികച്ചും ബാലിശമാണ് ഈ ഹദീസ് ദുർവ്യാഖ്യാനം.

ഹദീസിൽ ഉദ്ദേശിക്കപ്പെട്ട ഹിന്ദ് അഥവാ ബസ്വറ നൂറ്റാണ്ടുകളായി മുസ്‌ലിംകളുടേതായി കഴിഞ്ഞു. അപ്പോൾ ബസ്വറയോടുള്ള യുദ്ധ കൽപ്പന എക്കാലത്തേയും ബാധിക്കുന്ന, പൊതു കൽപനയാണെന്ന് ദുർവ്യഖ്യാനിച്ചാൽ ‘മുസ്‌ലിം ഇറാഖി’നോട് തന്നെ യുദ്ധം ചെയ്യാനാണ് മുഹമ്മദ് നബി (സ) കൽപ്പിച്ചത് എന്ന് ഹദീസിന് വിരുദ്ധാത്ഥം കൽപ്പിക്കേണ്ടി വരില്ലേ ?! ഇസ്‌ലാം മുഹമ്മദ് നബിയിലൂടെ കടന്നുവന്ന കാലഘട്ടത്തിൽ ലോകത്ത് നിലനിന്നിരുന്ന രാഷ്ട്രീയ വ്യവസ്ഥയെ സംബന്ധിച്ച് ചരിത്രത്തിന്റെ താളുകളിലൂടെ അൽപ്പമെങ്കിലും കണ്ണോടിച്ചാൽ ജനാധിപത്യ മൂല്യങ്ങളും മത സഹവർത്തിത്വത്തിലും ഊന്നി, ഇന്ന് ലോകത്ത് നിലനിൽക്കുന്ന രാഷ്ട്ര ഘടനയായിരുന്നില്ല അന്നത്തേത് എന്ന് മനസ്സിലാക്കാവുന്നതെയുള്ളു.

അന്യ മതങ്ങളേയോ ആദർശങ്ങളേയോ സംസ്കാരങ്ങളേയോ വെച്ചുപൊറുപ്പിക്കാത്ത സ്വേച്ചാധിപത്യ ഭരണകൂടങ്ങളായിരുന്നു ലോകത്ത് ഭൂരിഭാഗവും രാജ്യങ്ങളിലും നിലനിന്നിരുന്നത്. ജനാധിപത്യമോ മത സഹിഷ്ണുതയോ തൊട്ടു തീണ്ടിയിട്ടില്ലാത്ത കിരാത വാഴ്ച്ച. ഇതാണ് ഇസ്‌ലാമിന്റെ ആവിർഭാവ ഭൂമിക. ഈ സാമൂഹിക രാഷ്ട്രീയ പശ്ചാത്തലത്തിൽ ഊന്നി നിന്നു കൊണ്ടേ ഇസ്‌ലാമിലെ യുദ്ധങ്ങളെയും യുദ്ധ ആഹ്വാനങ്ങളേയും വായിച്ചറിയാവൂ. പ്രതിരോധത്തിനും മത സ്വാതന്ത്ര്യത്തിനും വേണ്ടിയായിരുന്നു ഇസ്‌ലാമിക യുദ്ധങ്ങൾ മുഴുവനും. അതല്ലാതെ ഇന്ന് ലോകത്ത് നിലനിൽക്കുന്ന മതേതര സ്വഭാവമോ ആദർശ സഹവർത്തിത്വമോ രാഷ്ട്രീയ പ്രകൃതിയായി സ്വീകരിച്ച സാമൂഹിക പശ്ചാത്തലത്തോടായിരുന്നില്ല പ്രസ്തുത സമരങ്ങൾ. ഈ സമൂഹത്തോട് അവകാശങ്ങൾക്കും സ്വാതന്ത്ര്യത്തിനും വേണ്ടി സായുധ സമരത്തിൽ ഏർപ്പെടുകയല്ലാതെ മറ്റെന്താണ് മുസ്‌ലിംകളുടെ – ആവിർഭാവ ഘട്ടത്തിൽ – മുമ്പിലുണ്ടായിരുന്ന വഴി !!

പ്രതിരോധ യുദ്ധങ്ങളല്ലാത്ത ഇസ്‌ലാമിക യുദ്ധങ്ങൾ തന്നെയും നിർബന്ധിത മതപരിവർത്തനത്തിനോ അന്യ മത ധ്വംസനത്തിനോ വേണ്ടിയല്ല നടന്നത് എന്നർത്ഥം. മറിച്ച്, മത സ്വാതന്ത്ര്യത്തിനും അവകാശങ്ങൾക്കും വേണ്ടിയാണ്. ഇസ്‌ലാമിക ഭരണത്തിനു കീഴിൽ കൊണ്ടുവന്ന് അന്യ മത സമൂഹങ്ങളെ അടക്കിഭരിക്കുകയല്ല ലക്ഷ്യം. മറിച്ച്, അന്നത്തെ തിയോക്രാറ്റിക്ക് ഓട്ടോക്രസികളിൽ (തീവ്രമതവികാരത്തിൽ അധിഷ്ഠിതമായ സ്വേച്ഛാധിപത്യ ഭരണകൂടങ്ങൾ) ഇസ്‌ലാമിൽ ആകൃഷ്ടരാകുന്നവർക്ക് ഇസ്‌ലാം സ്വീകരിക്കാനുള്ള വഴിയൊരുക്കാനും ഇസ്‌ലാം അനുസരിച്ചുള്ള ജീവിതം എളുപ്പമാവാനും ഇസ്‌ലാമിക പ്രബോധനം സാധ്യമാകാനും അവരുടെ മേൽ ഭരണം നേടിയെടുക്കുകയല്ലാതെ മറ്റൊരു വഴിയും അന്നത്തെ രാഷ്ട്രീയ ഘടനയിൽ നില നിന്നിരുന്നില്ല. ഇസ്‌ലാമിന്റെ കടന്നുവരവിനെ ആ കാലഘട്ടത്തിലെ ഗോത്ര സമൂഹങ്ങളും നാട്ടുരാജ്യങ്ങളും മത സമുദായങ്ങളും എത്രത്തോളം അസഹിഷ്ണുതയോടെയാണ് സ്വീകരിച്ചത് എന്ന ചരിത്രസാക്ഷ്യങ്ങൾ മുമ്പ് പല ലേഖനങ്ങളിൽ നാം വിശദീകരിച്ചിട്ടുണ്ട്. (വിശദാശംങ്ങൾക്കായി: www.snehasamvadam.org: ‘ഇസ്‌ലാമിന്റെ യുദ്ധഭൂമിക’ എന്ന ലേഖനം വായിക്കുക: https://bit.ly/3BHqSuX)

ഇത്തരമൊരു ലോക വ്യവസ്ഥയിൽ ഇസ്‌ലാമിന്റെ അതിജീവനവും പ്രചാരണവും മറ്റെന്തു മാർഗത്തിലൂടെയാണ് സാധ്യമാകുമായിരുന്നത്? ഇത്തരമൊരു ഭൂമികയിൽ സമ്പൂർണ അഹിംസ രാഷ്ട്രീയ നയമായി ഇസ്‌ലാം സ്വീകരിച്ചിരുന്നുവെങ്കിൽ ഇതര സമൂഹങ്ങൾ ഇസ്‌ലാമിനെ വിഴുങ്ങുമായിരുന്നു. ഒരു നൂറ്റാണ്ടിനപ്പുറം വെളിച്ചം കാണാതെ അജ്ഞതയുടെ അഗണ്യകോടിയിൽ അലിഞ്ഞില്ലാതെയാവുമായിരുന്നു (;ദൈവത്തിൽ നിന്നുള്ള മതമായിരുന്നില്ല ഇസ്‌ലാം എങ്കിൽ.)

പക്ഷെ ഇസ്‌ലാം ദൈവത്തിൽ നിന്നുള്ള ആദർശമായത് കൊണ്ടും പ്രാവർത്തിക മതമായത് കൊണ്ടും അനിവാര്യ ഘട്ടങ്ങളിലുള്ള സായുധ സമരങ്ങൾക്ക് അനുവാദം നൽകപ്പെട്ടു. ഇസ്‌ലാം ദൈവത്തിൽ നിന്നുള്ള സന്മാർഗ സന്ദേശമാണ്. അത് ലോകം മുഴുവനും – കുടിലും കൊട്ടാരവും വ്യത്യാസമില്ലാതെ – എത്തിക്കുക എന്നത് ദൈവത്തിന്റെ ബാധ്യതയാണല്ലോ. അവൻ സ്വയം ഏറ്റെടുത്ത ബാധ്യത. അതിന് സാമൂഹിക അസഹിഷ്ണുതയും രാഷ്ട്രീയ തടസ്സങ്ങളും വിഘ്നങ്ങളായി കൂടാ. സത്യത്തിന്റെ ശത്രുക്കൾ രാഷ്ട്രങ്ങളുടേയും നാട്ടുരാജ്യങ്ങളുടേയും അതിർത്തികളിൽ വാളേന്തി നിൽക്കുകയും വാതിലുകൾ കൊട്ടിയടക്കുകയും ചെയ്തിരുന്ന കാലഘട്ടത്തിൽ വാളുകൊണ്ടല്ലാതെ ആ തടസ്സത്തെ നീക്കാൻ കഴിയില്ലായിരുന്നു. ഇന്നത്തെ ലോക-രാഷ്ട്ര വ്യവസ്ഥ അന്നത്തേതല്ല. രാഷ്ട്രങ്ങളുടെ വാതിലുകൾ ഏത് ആദർശങ്ങൾക്കും മതങ്ങൾക്കും മുമ്പിൽ – താത്ത്വികമായെങ്കിലും – മലർക്കേ തുറന്നിടപ്പെട്ടിരിക്കുകയാണ്, ഇന്ന്. ഇവിടെ, വാതിലുകളുടെ കാവൽക്കാർ വാളേന്തി നിൽക്കുന്നില്ല. പിന്നെയെന്തിന് മുസ്‌ലിംകൾ വാളെടുക്കണം ?!. വാതിലുകൾ കൊട്ടിയടക്കപ്പെട്ടാലല്ലേ ചവിട്ടി തുറക്കേണ്ടതുള്ളു ?!. തുറന്നു കിടക്കുന്ന വാതിൽ ആരെങ്കിലും ചവിട്ടി തുറക്കാൻ ശ്രമിക്കുമോ?!!

ഇസ്‌ലാമിക ഭരണത്തിനു കീഴിൽ പൂർണ്ണ മത സ്വാതന്ത്ര്യങ്ങളോടെയും അവകാശങ്ങളോടെയും ജീവിക്കാൻ ഒരു അമുസ്‌ലിമിന് വഴിയൊരുക്കണമെന്നാണ് മുഹമ്മദ് നബി (സ) അദ്ദേഹത്തിന്റെ അനുചരന്മാരെ പഠിപ്പിച്ചത് എന്ന് സാന്ദർഭികമായി സൂചിപ്പിക്കട്ടെ. ഇസ്‌ലാം മുന്നോട്ടുവെക്കുന്ന ഭരണ വ്യവസ്ഥ തീവ്രമതവികാരത്തിൽ അധിഷ്ഠിതമായ സ്വേച്ഛാധിപത്യ ഭരണകൂടമല്ല; ജനാധിപത്യ മൂല്യങ്ങളിൽ അധിഷ്ഠിതമായ ആത്മീയഭരണമാണ്. അവിടെയും ഇസ്‌ലാമിക ഭരണത്തിനു കീഴിൽ പ്രസ്തുത സമൂഹങ്ങളെ അടക്കിഭരിക്കുകയല്ല ലക്ഷ്യം; മുമ്പ് സൂചിപ്പിച്ചതു പോലെ അന്നത്തെ തിയോക്രാറ്റിക്ക് ഓട്ടോക്രസികളിൽ ജീവിക്കുന്ന മനുഷ്യർക്കിടയിൽ ഇസ്‌ലാം പ്രബോധനം ചെയ്യാനും, ഇസ്‌ലാം സ്വീകരിക്കാനും ഉള്ള വഴി വെട്ടുക മാത്രമാണ്.

‘ഹിന്ദി’നോടുള്ള യുദ്ധത്തെ സംബന്ധിച്ച പ്രവാചക പ്രവചനം കഴിഞ്ഞ കാലഘട്ടത്തിൽ പുലർന്നിട്ടില്ല എന്നും അത് ഭാവിയിൽ നടക്കാനിരിക്കുന്നതാണ് എന്നും ചില മുസ്‌ലിം പണ്ഡിതന്മാർ വ്യാഖ്യാനിക്കാറുണ്ട്. ഈ പണ്ഡിതാഭിപ്രായം അംഗീകരിച്ചാലും, സ്വതന്ത്രാനന്തര അതിർത്തിരേഖ ഉൾകൊള്ളുന്ന ആധുനിക മതേതര ജനാധിപത്യ ഇന്ത്യയോടുള്ള യുദ്ധമല്ല ഇതു കൊണ്ടും ഉദ്ദേശിക്കപ്പെടുന്നത്. ഈ പണ്ഡിത വ്യഖ്യാനം സ്വീകരിച്ചാലും ഇന്ത്യക്കെതിരെയുള്ള ഭീകരാക്രമണങ്ങൾ ഒരു സാഹചര്യത്തിലും ഈ ഹദീഥിൽ പ്രദിപാദിക്കുന്നില്ല. അന്ത്യനാളിനോടനുബന്ധിച്ച് തീർത്തും വ്യത്യസ്ഥമായ ഒരു ലോക ഘടനയിൽ, ലോകം മുഴുവൻ ബലപ്രയോഗത്തിലൂടെ അധിനിവേശം നടത്തുന്ന, മസീഹുദ്ദജ്ജാൽ എന്ന ഒരു ഏകശസനാധിപന്റെ കാലത്താണ് സംഭവം നടക്കുക. ഈ ഏകശാസകനിൽ നിന്നും ഹിന്ദിനെ മോചിപ്പിച്ച് സ്വാതന്ത്ര്യത്തിലേക്കും, നീതിയിലേക്കും തിരിച്ച് കൊണ്ടുപോകാനാണ് ഹിന്ദ് യുദ്ധം.

യുദ്ധം നടക്കുന്ന ലോകം നമ്മൾ ഇന്ന് ജീവിക്കുന്ന ലോകമല്ല. യുദ്ധം ചെയ്യപ്പെടുന്ന ‘ഹിന്ദ്’ ഇന്നത്തെ മതനിരപേക്ഷ ജനാധിപത്യ ഇന്ത്യയുമല്ല. ദജ്ജാലെന്ന ഏകാധിപതിയുടെ കീഴിൽ ഞെരിഞ്ഞമരുന്ന ഒരു ഹിന്ദിനോട് അന്ത്യദിനത്തോട് അടുത്തായി നടക്കുന്ന ഒരു യുദ്ധത്തിനെ (ഒരു സ്വാതന്ത്ര്യ സമരത്തിനെ) സ്വതന്ത്രാനന്തര അതിർത്തിരേഖ ഉൾകൊള്ളുന്ന ആധുനിക മതനിരപേക്ഷ ഇന്ത്യയോടുള്ള യുദ്ധമായി ദുർവ്യാഖ്യാനിക്കുകയാണ് ഇസ്‌ലാമോഫോബിക്കുകൾ ചെയ്തു കൊണ്ടിരിക്കുന്നത്. എന്നാൽ സമാധാന ചിത്തരായി ജീവിക്കുന്ന സാധാരണ അമുസ്‌ലിംകളോട് എന്ത് നിലപാടാണ് സ്വീകരിക്കേണ്ടത് എന്നതുമായി ബന്ധപ്പെട്ട ഇസ്‌ലാമിക നിയമവും പ്രവാചക പാഠവും മറ്റു ഹദീസുകളിൽ വ്യക്തമായി വായിക്കാൻ സാധിക്കും, അതിനോട് എതിരായി അന്യായമായി ഒരു നാടിനേയും ആക്രമിക്കാൻ പ്രവാചക ശ്രേഷ്ഠൻ (സ) ഒരിക്കലും കൽപ്പിക്കുകയില്ല.

അല്ലാഹു പറഞ്ഞു: “മതകാര്യത്തില്‍ നിങ്ങളോട് യുദ്ധം ചെയ്യാതിരിക്കുകയും, നിങ്ങളുടെ വീടുകളില്‍ നിന്ന് നിങ്ങളെ പുറത്താക്കാതിരിക്കുകയും ചെയ്യുന്നവരെ സംബന്ധിച്ചിടത്തോളം നിങ്ങളവര്‍ക്ക് നന്മ ചെയ്യുന്നതും നിങ്ങളവരോട് നീതി കാണിക്കുന്നതും അല്ലാഹു നിങ്ങളോട് നിരോധിക്കുന്നില്ല. തീര്‍ച്ചയായും അല്ലാഹു നീതി പാലിക്കുന്നവരെ ഇഷ്ടപ്പെടുന്നു. മതകാര്യത്തില്‍ നിങ്ങളോട് യുദ്ധം ചെയ്യുകയും നിങ്ങളുടെ വീടുകളില്‍ നിന്ന് നിങ്ങളെ പുറത്താക്കുകയും നിങ്ങളെ പുറത്താക്കുന്നതില്‍ പരസ്പരം സഹകരിക്കുകയും ചെയ്തവരെ സംബന്ധിച്ചു മാത്രമാണ് -അവരോട് മൈത്രികാണിക്കുന്നത്- അല്ലാഹു നിരോധിക്കുന്നത്‌. വല്ലവരും അവരോട് മൈത്രീ ബന്ധം പുലര്‍ത്തുന്ന പക്ഷം അവര്‍ തന്നെയാകുന്നു അക്രമകാരികള്‍.” (കുർആൻ: 60:8,9)

പ്രവാചകൻ മുഹമ്മദ് (സ) പറഞ്ഞു: مَنْ قَتَلَ مُعَاهَدًا لَمْ يَرَحْ رَائِحَةَ الْجَنَّةِ، وَإِنَّ رِيحَهَا لَيُوجَد مِنْ مَسِيرَةِ أَرْبَعِينَ عَامًا. “സമാധാന സന്ധിയിലുള്ള ഒരു അമുസ്‌ലിമിനെ ആരെങ്കിലും കൊന്നാൽ അവന് സ്വർഗത്തിന്റെ സുഗന്ധം പോലും ലഭിക്കില്ല.” (സ്വഹീഹുൽ ബുഖാരി: ഹദീസ് നമ്പർ: 3166)

عَنْ صَفْوَانَ بْنَ سُلَيْمٍ عَنْ رَسُولِ اللَّهِ صلى الله عليه وسلم قَالَ‏ “‏ أَلاَ مَنْ ظَلَمَ مُعَاهِدًا أَوِ انْتَقَصَهُ أَوْ كَلَّفَهُ فَوْقَ طَاقَتِهِ أَوْ أَخَذَ مِنْهُ شَيْئًا بِغَيْرِ طِيبِ نَفْسٍ فَأَنَا حَجِيجُهُ يَوْمَ الْقِيَامَةِ

സ്വഫ്‌വാനു ബ്‌നു സുലൈമില്‍(റ) നിന്ന് നിവേദനം: പ്രവാചകൻ (സ) പറഞ്ഞു: ‘അറിയണം, ആരെങ്കിലും സമാധാന സന്ധിയിലുള്ള അമുസ്‌ലിമിനെ ഉപദ്രവിക്കുകയോ, അവന് കിട്ടേണ്ട അവകാശങ്ങളില്‍ കുറവ് വരുത്തുകയോ, സാധ്യമാകുന്നതിലുപരി വഹിക്കാന്‍ അവനെ നിര്‍ബന്ധിക്കുകയോ, മനപ്പൊരുത്തമില്ലാതെ അവനില്‍ നിന്നും വല്ലതും കവര്‍ന്നെടുക്കുകയോ ചെയ്താൽ ഉയിര്‍ത്തെഴുന്നേല്‍പ്പ് നാളില്‍ ഞാന്‍ അവനുമായി (ആ അമുസ്‌ലിമിന്റെ അവകാശങ്ങളുടെ കാര്യത്തിൽ) തര്‍ക്കത്തിലേര്‍പെടും. (അബൂദാവൂദ്: 3052).

ഇതാണ് സമാധാന ചിത്തരായി ജീവിക്കുന്ന സാധാരണ അമുസ്‌ലിംകളോട് സ്വീകരിക്കേണ്ട നിലപാടായി ഇസ്‌ലാം പഠിപ്പിച്ച നിയമവും ശാസനയും. എത്ര തവണ വ്യക്തമാക്കിയാലും ഈ ആയത്തുകളും ഹദീസുകളും വിമർശകർ കാണാത്ത മട്ടാണ് !!

ദജ്ജാലിനോടും ദജ്ജാലിന്റെ അധിനിവേശത്തോടുമുള്ള യുദ്ധം അന്ത്യദിനത്തോടനുബന്ധിച്ചാണ്, ഈസാ നബിയുടെ(അ) (യേശു) നേതൃത്വത്തിലാണ് നടക്കുക. (ഫത്ഹുൽ ബാരി: 6: 610, അത്തൗദീഹു ലി ശർഹിൽ ജാമിഅ്: 17:663)

അന്ത്യനാളിനോടടുത്ത് പ്രത്യക്ഷനാകുമെന്ന് പല മത ഗ്രന്ഥങ്ങളും പ്രവചിച്ച, അന്തിക്രിസ്‌തു (antichrist) എന്ന പേരിൽ അറിയപ്പെടുന്ന ഒരു ഭാവി വ്യക്തിത്വമാണ് ‘മസീഹു ദ്ദജ്ജാൽ’. ലോകത്ത് ജീവിച്ചിരുന്നതും ജീവിച്ചിരിക്കുന്നവരുമായ ഏകാധിപതികളേക്കാൾ ഏറ്റവും കിരാതനും ക്രൂരനുമായ ഏകാധിപതിയായിരിക്കും (dictator) മസീഹു ദ്ദജ്ജാൽ എന്നാണ് ഹദീസുകൾ പഠിപ്പിക്കുന്നത്. മനുഷ്യരെ ഭൗതീകവും ആത്മീയവുമായ പാരതന്ത്ര്യത്തിലേക്ക് വലിച്ചിഴക്കുന്ന ഈ ദജ്ജാലിന്റെ ഉപദ്രവങ്ങളിൽ നിന്ന് രക്ഷ തേടാൻ പ്രവാചകൻ (സ) തന്റെ അനുചരന്മാരോട് പ്രത്യേകം ഉപദേശിക്കുക കൂടി ചെയ്തതായി കാണാം. (സ്വഹീഹു മുസ്‌ലിം: 924)

ദജ്ജാലിനാൽ ഭാവിയിൽ, ലോകത്ത് വിതക്കപ്പെടാനിരിക്കുന്ന കുഴപ്പങ്ങളും ഛിദ്രതകളും സമാനതകൾ ഇല്ലാത്തതായിരിക്കും.

مَا بَيْنَ خَلْقِ آدَمَ إِلَى أَنْ تَقُومَ السَّاعَةُ فِتْنَةٌ أَكْبَرُ مِنْ فِتْنَةِ الدَّجَّالِ . പ്രവാചകൻ (സ) പറഞ്ഞു: “(ആദ്യ മനുഷ്യൻ) ആദമിനെ സൃഷ്ടിച്ചതു മുതൽ അന്ത്യദിനം സംഭവിക്കുന്നതു വരെ ദജ്ജാലിന്റെ കുഴപ്പത്തേക്കാൾ വലിയ ഒരു ആപത്തും ഇല്ലതന്നെ.” (മുസ്നദു അഹ്മദ്: 15831)

ലോക രാജ്യങ്ങൾ മുഴുവൻ വെട്ടിപ്പിടിച്ച് തന്റെ അധികാരത്തിനും ആജ്ഞാപനത്തിനും കീഴിലാക്കുന്ന ലോകം കണ്ട ഏറ്റവും വലിയ ഏകാധിപതിയായിരിക്കും ‘മസീഹു ദ്ദജ്ജാൽ’.

ليس من بلد إلا سيطؤه الدجال إلا مكة والمدينة ليس له من نقابها نقب إلا عليه الملائكة صافين يحرسونها

ദജ്ജാൽ കാലു കുത്താത്ത ഒരു നാടും അവശേഷിക്കില്ല; മക്കയും മദീനയും ഒഴികെ. ആ രണ്ട് നാടുകളെയും സംരക്ഷിച്ചു കൊണ്ട് മലക്കുകൾ വലയം ചെയ്യുന്നുണ്ടാകും. (സ്വഹീഹുൽ ബുഖാരി: 1881, സ്വഹീഹു മുസ്‌ലിം: 2943)

നാടുകളിൽ കാലുകുത്തുക എന്നതുകൊണ്ടുദ്ദേശം പടയോട്ടത്തിലൂടെ അധികാരത്തിന് കീഴിലാക്കുക എന്നാണ്. فلا يبقى له موضع إلا ويأخذه غير مكة والمدينة “മക്കയും മദീനയും തുടങ്ങിയ സ്ഥലങ്ങളല്ലാതെ ഒരു സ്ഥലവും അവൻ പിടിച്ചടക്കാത്തതായി അവശേഷിക്കില്ല” എന്ന് ചില നിവേദനങ്ങളിൽ കാണാം. (ഉംദത്തുൽ കാരി: 10:244)

“എല്ലാ ജല തടത്തിലും അവന്റെ അധികാരമെത്തും” (يبلغ سلطانه كل منهل) എന്ന് മറ്റു ചില നിവേദനങ്ങളിലും വന്നിരിക്കുന്നു. (മുസ്നദു അഹ്മദ്: 23139)

(സ്വാഭാവികമായും ദജ്ജാലിന്റെ ഈ അധിനിവേശം ഇന്ത്യയിലുമെത്തുമല്ലൊ. ആദർശ സ്വാതന്ത്ര്യവും ജനാധിപത്യ മൂല്യങ്ങളും ദജ്ജാലിനാൽ തിരോധാനം ചെയ്യപ്പെടുന്ന കാലത്ത് അവ തിരിച്ചു പിടിക്കാൻ രക്ഷകരായി കൊണ്ടാണ് ഈ പണ്ഡിതാഭിപ്രായപ്രകാരം മുസ്‌ലിംകൾ ഇന്ത്യയിലേക്ക് കടന്നുവരിക എന്നർത്ഥം)

ദജ്ജാലിന്റെ സാമ്രാജ്യത്വ അധിനിവേശത്തിനു കീഴിൽ മനുഷ്യ ജീവിതങ്ങൾ ഞെരിഞ്ഞമരും. അങ്ങനെ ലോകം മുഴുവൻ അക്രമവും അനീതിയും അടിച്ചമർത്തലുകളും നിറയുന്ന ഘട്ടത്തിൽ മഹ്ദി എന്ന സ്ഥാനപേരിലുള്ള ഒരു ഭരണാധികാരിയുടേയും പ്രവാചകനായ ഈസാ(അ)യുടേയും നേതൃത്വത്തിൽ അന്ത്യദിനത്തോടടുത്ത് നടക്കുന്ന സ്വാതന്ത്ര്യ സമരങ്ങൾ ഒരുപാട് ഹദീസുകളുടെ ഇതിവൃത്തമായിട്ടുണ്ട്. (ഇക്കൂട്ടത്തിൽ തന്നെയാണ് ‘ഗസ്‌വത്തുൽ ഹിന്ദും’ എന്നാണ് ഈ വിഭാഗം പണിതന്മാരുടെ അഭിപ്രായം)

أبشركم بالمهدي يبعث على اختلاف من الناس وزلازل فيملأ الأرض قسطاً وعدلاً كما ملئت جوراً وظلماً “ലോകം മുഴുവൻ അക്രമങ്ങളും സ്വേച്ഛാധിപത്യവും കൊണ്ട് നിറഞ്ഞ സന്ദർഭത്തിൽ ലോകത്തെ നീതിയും ന്യായവും കൊണ്ട് മഹ്ദി നിറക്കുമെന്ന്” ഹദീസിൽ പ്രസ്ഥാവിക്കുന്നുണ്ട്. (മുസ്നദു അഹ്മദ്: 11344, അൽ അഹ്കാമുശറഇയ്യ അൽ കുബ്റാ: അബ്ദുൽ ഹക്ക് അൽ ഇശ്ബീലി: 4/532, മജ്മഉസ്സവാഇദ്: 7/316)

അത്ഭുത സിദ്ധികൾ പലതും പ്രദർശിപ്പിച്ച് ജനങ്ങളെ വശീകരിക്കുന്നതിന് പുറമെ, ദജ്ജാലെന്ന ഏകശസനാധികാരി ആളുകളെ ചതിയിലൂടെ അവരെ അഗ്നിയിലേക്ക് നയിക്കുമെന്ന സൂചനകൾ ഹദീസുകളിൽ കാണാം.

‘അയാളുടെ കൂടെ രണ്ട് ജല തടങ്ങളുണ്ടാകും. ഒന്ന് പ്രത്യക്ഷ ദൃഷ്ട്യാ വെള്ളമായിരിക്കും. മറ്റൊന്ന് ദൃഷ്ട്യാൽ അഗ്നിയായിരിക്കും. എന്നാൽ വെള്ളമെന്ന് തോന്നിപ്പിക്കപ്പെടുന്ന തടങ്ങൾ യഥാർത്ഥത്തിൽ അഗ്നിയായിരിക്കും എന്നും’ ഹദീസുകൾ സൂചിപ്പിക്കുന്നുണ്ട്. (സ്വഹീഹു മുസ്‌ലിം: 5223)

അത്ഭുത പ്രവർത്തനങ്ങൾ പ്രദർശിപ്പിച്ച് ദിവ്യത്വം വാദിക്കുക കൂടി ചെയ്യും. അധികാരത്തിലൂടെ തന്റെ ദിവ്യത്വം ജനങ്ങളിൽ അടിച്ചേൽപ്പിക്കുക എന്നത് ദജ്ജാലിന്റെ പദ്ധതികളിൽ ഒന്നായിരിക്കും.

“ദജ്ജാൽ ജനങ്ങളോട്‌ പറയും: ഈ വ്യക്തിയെ (ഒരു മുസ്‌ലിം) ഞാൻ കൊല്ലുകയും പിന്നീട് ജീവിപ്പിക്കുകയും ചെയ്താൽ (ഞാൻ നിങ്ങളുടെ ദൈവമാണെന്നതിൽ) നിങ്ങൾ സംശയിക്കുമോ? ജനങ്ങൾ പറയും: ഇല്ല. അപ്പോൾ ദജ്ജാൽ അയാളെ കൊല്ലുകയും ജീവിപ്പിക്കുകയും ചെയ്യും. അപ്പോൾ ജീവൻ തിരിച്ചു കിട്ടിയ വ്യക്തി പറയും: “അല്ലാഹുവാണേ, മുമ്പൊരിക്കലും നിന്നെക്കുറിച്ച്‌ എനിക്കില്ലാതിരുന്ന വ്യക്തമായ തിരിച്ചറിവ് ഇന്നെനിക്ക് ലഭിച്ചു.” (അഥവാ നബി പറഞ്ഞ ദജ്ജാൽ നീ തന്നെയാണു എന്ന് എനിക്കിപ്പോൾ ഉറപ്പായി എന്നയാൾ ആണയിടും. കാരണം ഇപ്രകാരം ഒരു സംഭവമുണ്ടാകുമെന്ന പ്രവാചകന്റെ ഹദീസിന് ജീവിക്കുന്ന തെളിവായി അയാൾ മാറുമല്ലൊ) അപ്പോൾ ദജ്ജാൽ അയാളെ വീണ്ടും കൊല്ലാൻ ശ്രമിക്കും. പക്ഷെ അതിന് ദജ്ജാലിനെ അല്ലാഹു അനുവദിക്കില്ല.” (സ്വഹീഹുൽ ബുഖാരി: 6599)

തന്റെ ദിവ്യത്വത്തിൽ ജനങ്ങളെ നിർബന്ധിച്ച് വിശ്വസിപ്പിക്കാനായി അധികാരവും മായാജാലവുമൊക്കെ ദജ്ജാൽ ഉപയോഗിക്കും. എന്നിട്ടും വിശ്വസിക്കാത്തവർക്ക് വധശിക്ഷയാണ് ദജ്ജാലിന്റെ ഭരണകൂടം വിധിക്കുക. വിശ്വാസ സ്വാതന്ത്ര്യം നിശ്ശേഷം നിഷേധിക്കപ്പെടുന്ന ഈ സാമ്രാജ്യത്വ സ്വേച്ഛാധിപത്യത്തോടാണ് മുസ്‌ലിംകൾ പടക്കിറങ്ങുന്നത് എന്നർത്ഥം.

ഈ സ്വേച്ഛാധിപതിയുടെ രാജാധികാരത്തോടും അയാളുടെ കിങ്കരന്മാരോടുമാണ് അന്ത്യ നാളിനോടടുത്ത് സ്വാതന്ത്ര്യ സമരങ്ങൾ നടക്കുക. അല്ലാതെ സമാധാനത്തോടെ ജീവിക്കുന്ന അമുസ്‌ലീംകളോടോ അവരുടെ നാടുകളോടോ അല്ല. ചുരുക്കത്തിൽ, ഗസ്വത്തുൽ ഹിന്ദ് പരാമർശിക്കുന്ന ഹദീഥുകളെ തെറ്റിദ്ധരിപ്പിച്ചു കൊണ്ട് ഇന്ത്യൻ മുസ്‌ലിംകൾ ഇന്ത്യയോട് യുദ്ധം ചെയ്യാൻ തക്കം പാർത്തിരിക്കുന്നവരാണെന്ന പ്രചാരണം അബദ്ധജടിലവും അടിസ്ഥാനരഹിതവുമാണ്. അകാരണമായും അന്യായമായും ഒരു നാടിനോടും യുദ്ധം ചെയ്യാനും അക്രമിക്കുവാനും ഈ ഹദീസിൽ എന്നല്ല ഇസ്‌ലാമിക പ്രമാണങ്ങളിൽ എവിടെയും പഠിപ്പിക്കുന്നില്ല.

വിമർശനം:

“സ്ത്രീകൾക്ക് എഴുത്ത് പഠിപ്പിക്കരുത്” എന്ന് മുഹമ്മദ് നബി തന്റെ അനുചരന്മാരോട് കൽപ്പിച്ചതായി ഹദീസുകളിൽ വന്നിട്ടില്ലേ ?

മറുപടി:

പ്രവാചകന്റെ(സ) കൽപ്പനകളും അധ്യാപനങ്ങളും വിദ്യാഭ്യാസത്തേയും എഴുത്തും വായനയേയും പ്രോത്സാഹിപ്പിക്കുന്നവയാണ്. വിമർശകർ ഉദ്ധരിക്കുന്ന ഈ കള്ള ഹദീസ് പ്രവാചകന്റെ(സ) വാചകമല്ല, അദ്ദേഹത്തിന്റെ പേരിൽ കെട്ടിച്ചമക്കപ്പെട്ടതാണ്. ഈ വസ്തുത സമൂഹത്തെ ബോധ്യപ്പെടുത്താൻ ഉൽപതിഷ്ണുക്കളായ മുസ്‌ലിം പണ്ഡിതന്മാർ, പുരോഗമ വിരോധികളായ ഒരു പറ്റം പൗരോഹിത്യവൃന്ദത്തോട് ആദർശസമരത്തിൽ ഏർപ്പെട്ടിരുന്ന ഒരു കാലഘട്ടമുണ്ടായിരുന്നു.

പതിറ്റാണ്ടുകൾക്ക് മുമ്പ് കേരളത്തിലെ മുസ്‌ലിം സമൂഹം -പ്രത്യേകിച്ച് മുസ്‌ലിം സ്ത്രീകൾ- വിദ്യാഭ്യാസ രംഗത്ത് പിന്നോക്കാവസ്ഥയിലായിരുന്ന ആ കാലഘട്ടത്തിൽ നവോത്ഥാനത്തിന്റെ അമരക്കാരായ കേരളത്തിലെ മുസ്‌ലിം പണ്ഡിതർ നടത്തിയ ഇസ്‌ലാമിക പ്രബോധനങ്ങളുടെ സദ്‌ഫലമാണ് മുസ്‌ലിം കൈരളിയുടെ ശോഭനമായ വിദ്യാഭ്യാസ രംഗം. ഇന്ന് കേരളത്തിൽ, ഓരോ വർഷവും പത്താം ക്ലാസ് റിസൽട്ട് പ്രഖ്യാപിക്കപ്പെടുമ്പോഴും ഏറ്റവും കൂടുതൽ ഏ പ്ലസ്സുകാരുള്ള ജില്ല മലപ്പുറമായി മാറിയിരിക്കുന്നു; ആ ഏ പ്ലസ്സുകാരിൽ കൂടുതൽ പേരും പെൺകുട്ടികളും!! അല്ലാഹുവിന് സ്തുതി.

പണ്ട് സാമൂഹികോദ്ധാരണങ്ങളിൽ നിന്നും സമുദായത്തെ പിന്നോട്ട് വലിച്ചിരുന്ന പൗരോഹിത്യം തന്നെ ‘ഇസ്‌ലാമിന്റെ യഥാർത്ഥ അധ്യാപനങ്ങളെ ഉൾകൊണ്ട്’, തങ്ങൾ പേറി നടന്നിരുന്ന പല കള്ള വൃത്താന്തങ്ങളും അന്ധവിശ്വാസ അനാചാരങ്ങളും ചരിത്രത്തിന്റെ ചവറ്റുകൊട്ടയിലേക്ക് സ്വയം ചുരുട്ടി വലിച്ചെറിയുകയാണുണ്ടായത്. അവർ പോലും കൈയ്യൊഴിഞ്ഞ ഈ വ്യാജ വാറോലകൾ തപ്പി തിരഞ്ഞ് തിരിച്ച് കൊണ്ടുവരാനുള്ള ബദ്ധപ്പാടിലാണ് ആധുനിക ഭൗതീകവാദികൾ !! ഇസ്‌ലാമിനെ വിമർശിക്കാൻ എന്തെങ്കിലുമൊക്കെ വേണ്ടേ ?! ഇത്തരം ചപ്പുചവറുകളും ഭൗതികവാദികൾക്ക് നിലനിൽപ്പിന് അനിവാര്യമായി തീർന്നിരിക്കുന്നു എന്നതാണ് ദുരവസ്ഥ !!

ഈ വിദ്യാഭ്യാസ നവോദ്ധാനത്തിൽ വല്ല പങ്കും ഭൗതീക വാദികൾക്ക് അവകാശപ്പെടാനുണ്ടോ ?! ക്വുർആൻ വാഖ്യങ്ങളും ഹദീസുകളും സ്‌തുതിഘോഷിച്ച് ഈ സാക്ഷരതാ സമരത്തിന് നേതൃത്വം വഹിച്ചത് വക്കം മൗലവി, കെ.എം മൗലവി, ചാലിലകത്ത് കുഞ്ഞഹമ്മദ് ഹാജി, അബുസ്സബാഹ് മൗലവി, എം.സി.സി സഹോദരങ്ങൾ, ഹലീമാബീവി, പി.കെ സുബൈദ തുടങ്ങിയ മുസ്‌ലിം പണ്ഡിതന്മാരും പണ്ഡിതകളും തന്നെയാണ് എന്നിരിക്കെ ചില പിന്നാക്ക സംഘടനകളുടെ കാലഹരണം സംഭവിച്ച പ്രമേയങ്ങളും കള്ള ഹദീസുകളുമൊക്കെ കുത്തിപ്പൊക്കാൻ ഭൗതീകവാദികൾക്ക് എന്ത് ധാർമിക അവകാശമാണുള്ളത് ?!

‘സ്ത്രീകൾക്ക് എഴുത്ത് പഠിപ്പിക്കരുത്, അവർക്ക് തുന്നലും സൂറത്തുന്നൂറും പഠിപ്പിക്കുക…’ എന്നൊക്കെ പ്രവാചകൻ (സ) കൽപ്പിച്ചതായി പറയപ്പെടുന്ന വ്യാജ നിവേദനം പരിശോധന വിധേയമാക്കാം.

1. ആദ്യമായി ഈ നിവേദനങ്ങളുടെ നിവേദക പരമ്പരകൾ (സനദ്) പരിശോദിക്കുക:

* ത്വബ്റാനി തന്റെ മുഅ്ജമിൽ ഉദ്ധരിച്ച നിവേദക പരമ്പര ഇപ്രകാരമാണ്:

حدثنا محمد بن عبدالله الحضرمي قال ثنا محمد بن ابراهيم الشامي قال ناشعيب بن اسحاق عن هشام بن عروة عن ابيه عن عائشة قالت…

നിവേദക പരമ്പരയിൽ ‘മുഹമ്മദ് ഇബ്നു ഇബ്റാഹീം അശ്ശാമി’ എന്ന ‘റാവി’യുണ്ട്. ഇയാൾ കളവു പറയുകയും വ്യാജ നിവേദനങ്ങൾ നിർമ്മിക്കുകയും ചെയ്യുന്ന വ്യക്തിയാണ്.

هذا الحديث لا يصح… قال أبو حاتم بن حبان: كان محمد بن إبراهيم الشامي يضع الحديث على الشاميين لا يحل الرواية عنه إلا عند الاعتبار. روى أحاديث لا أصول لها من كلام رسول الله صلى الله عليه وسلم لا يحل الاحتجاج به.

ഇബ്നുൽ ജൗസി പറഞ്ഞു: “ഈ ഹദീസ് സ്വഹീഹ് അല്ല… ഇബ്നു ഹിബ്ബാൻ പറഞ്ഞു: ‘മുഹമ്മദ് ഇബ്നു ഇബ്റാഹീം അശ്ശാമി’ ശാമുകാരുടെ പേരിൽ ഹദീസുകൾ വ്യാജമായി നിർമ്മിക്കുമായിരുന്നു. അയാളിൽ നിന്ന് ഹദീസ് ഉദ്ധരിക്കൽ അനുവദനീയമല്ല. അല്ലാഹുവിന്റെ ദൂതന്റെ(സ) സംസാരങ്ങളിൽ പെടാത്ത, അവയുടെ യാതൊരു അടിത്തറയുമില്ലാത്ത ഹദീസുകൾ അയാൾ പടച്ചുണ്ടാക്കുമായിരുന്നു. അവ പ്രമാണമായി സ്വീകരിക്കൽ അനുവദനീയമല്ല.” (അൽ മൗദൂആത്ത്: 2/269)

ഇബ്നു അദിയ്യ് പറഞ്ഞു: വിശ്വസ്ഥരായ നിവേദകർക്കെതിരായി, അങ്ങേയറ്റം ദുർബലമായ നിവേദനങ്ങൾ ഉദ്ധരിക്കുന്ന വ്യക്തിയാണയാൾ. അയാളുടെ ഭൂരിഭാഗം ഹദീസുകളും പ്രാമാണികമായ ഹദീസുകൾക്ക് വിരുദ്ധമാണ്. ദാറകുത്നി പറഞ്ഞു: അയാൾ നുണയനാണ്. ഹാകിം, നികാശ് എന്നിവർ പറഞ്ഞു: അയാൾ ധാരാളം കള്ള ഹദീസുകൾ ഉദ്ധരിച്ചിട്ടുണ്ട്. (തഹ്ദീബു തഹ്ദീബ്: 9:13)

* ഹാകിം തന്റെ മുസ്തദ്റകിൽ ഉദ്ധരിച്ച, അതേ നിവേദനത്തിന്റെ മറ്റൊരു നിവേദക പരമ്പര ഇപ്രകാരമാണ്:

حدثنا أبوعلي الحافظ انبأ محمد بن سليمان حدثنا عبدالوهاب بن الضحاك حدثنا شعيب بن اسحاق عن هشام بن عروة عن أبيه عن عائشة رضي الله عنها قالت…

ഈ നിവേദകപരമ്പരയിലെ ‘അബ്ദുൽ വഹാബ് ഇബ്നു ദഹ്ഹാക് ഇബ്നു അബ്ബാൻ അൽ ഉർദി’ നുണയനാണെന്ന് അബൂഹാതിമും, ഇബ്നു ഹജർ അൽഅസ്കലാനിയും വ്യക്തമാക്കുന്നുണ്ട്. (തക്‌രീബു തഹ്‌ദീബ്: 1:626)

ശീഈ സൈറ്റുകളിൽ ഓടുന്ന ഈ നിവേദനത്തിന് വ്യക്തമായ നിവേദക പരമ്പരകൾ പോലുമില്ല എന്നത് സാന്ദർഭികമായി സൂചിപ്പിക്കട്ടെ. നുണയന്മാരിൽ നിന്നും വ്യാജഹദീസ് നിർമ്മാതാക്കളിൽ നിന്നും ഉദ്ധരിക്കുന്നതിന് പുറമെ നിവേദക പരമ്പരയിലെ റാവിമാരെ വ്യക്തമാക്കാതെ “ആരൊക്കെയോ ഉദ്ധരിച്ചു” എന്ന് പറഞ്ഞ് പ്രവാചകന്റെ മേൽ കെട്ടിയുണ്ടാക്കിയ ഇത്തരം കള്ള ഉദ്ധരണികളൊന്നും ഇസ്‌ലാമിൽ പ്രമാണമല്ല.

2. ഇത്തരം വ്യാജ ഉദ്ധരണികൾ ഇസ്‌ലാമിലെ പ്രമാണങ്ങളായ ക്വുർആനിനും സ്വഹീഹായ ഹദീസുകൾക്കും എതിരാണ് എന്നതാണ് മറ്റൊരു വസ്തുത. ഇസ്‌ലാമിന്റേയും മുഹമ്മദ് നബിയുടേയും(സ) സ്ഥിരപ്പെട്ട എല്ലാ അധ്യാപനങ്ങൾക്കും ഈ വ്യാജ വൃത്താന്തം എതിരാണ്.

അറിവിനും വിദ്യാ സമ്പാധനത്തിനും അളവില്ലാത്ത പ്രാധാന്യം നൽകിയ മതമാണ് ഇസ്‌ലാം. പ്രവാചകന് (സ) ആദ്യമായി അവതരിപ്പിക്കപ്പെട്ട ദിവ്യബോധനത്തിലെ രണ്ട് വരി ഇപ്രകാരമായിരുന്നു:

“നീ വായിക്കുക നിന്‍റെ രക്ഷിതാവ് ഏറ്റവും വലിയ ഔദാര്യവാനാകുന്നു. പേന കൊണ്ട് പഠിപ്പിച്ചവന്‍…” (ക്വുർആൻ: 96:3,4)

‘ക്വലം’ അഥവാ പേന എന്ന പേരിൽ ഒരു അധ്യായം തന്നെ ക്വുർആനിലുണ്ട്. ആ അധ്യായം ആരംഭിക്കുന്നത് തന്നെ ഇപ്രകാരമാണ്: “നൂന്‍. പേനയും അവര്‍ എഴുതിവെക്കുന്നതും സാക്ഷി…” (ക്വുർആൻ: 68:1)

മനുഷ്യർക്ക് ദൈവം നൽകിയ ഏറ്റവും വലിയ അനുഗ്രഹവും അവന്റെ ഏറ്റവും വലിയ ദൃഷ്ടാന്തവുമായാണ് പേനയെ ക്വുർആൻ കാണുന്നത്. അതുകൊണ്ടെല്ലാം തന്നെ എഴുത്ത് പഠിക്കുന്നതിനെ പ്രവാചകൻ (സ) അങ്ങേയറ്റം പ്രോത്സാഹിപ്പിച്ചു.

“അൻസ്വാരി സ്ത്രീകൾ എത്ര നല്ലവരാണ്. മതപാണ്ഡിത്യം നേടുന്ന കാര്യത്തിൽ ലജ്ജ അവരെ തടയുന്നില്ല” (സ്വഹീഹു മുസ്‌ലിം: 332) എന്ന് ശ്ലാഘിച്ചു കൊണ്ട്, അറിവിന്റേയും പഠനത്തിന്റേയും വിഷയത്തിൽ അന്തർമുഖരായ സ്ത്രീ ജനങ്ങളെ മുന്നോട്ടാനയിക്കുകയാണ് പ്രവാചകൻ (സ) ചെയ്തത്. സ്ത്രീ അറിവു നേടുന്നതിൽ തടസ്സമായി ഒന്നും തന്നെ നില നിൽക്കരുത് എന്ന വിപ്ലവ പാഠം ലോകത്തിന് മുമ്പിൽ വിളംബരം ചെയ്ത മുഹമ്മദ് നബി (സ) അറിവിന്റെ സുപ്രധാന സ്രോതസ്സായ വായനയും മാധ്യമമായ എഴുത്തും അവർക്കു തടഞ്ഞു എന്ന് വിഡ്ഢികളല്ലാതെ ആരാണ് വിശ്വസിക്കുക.?! അടിമ സ്ത്രീകളുടെ വിദ്യാഭ്യാസത്തെ പോലും പ്രവാചകൻ (സ) പ്രചോദിപ്പിക്കുകയാണുണ്ടായത്: “തന്റെ കീഴിലുള്ള അടിമ സ്‌ത്രീയെ സംസ്കാര സമ്പന്നയാക്കുകയും അവൾക്ക്‌ ഏറ്റവും നന്നായി വിദ്യാഭ്യാസം നൽകുകയും പിന്നീട്‌ അവളെ മോചിപ്പിച്ച്‌ സ്വയം വിവാഹം കഴിക്കുകയും ചെയ്തവനും ഇരട്ടി പ്രതിഫലമുണ്ട്‌” (ബുഖാരി: 5083).

ബദ്ർ യുദ്ധാനന്തരം ബന്ധികളാക്കപ്പെട്ട പടയാളികൾക്ക് മോചനദ്രവ്യമായി നിശ്ചയിക്കപ്പെട്ടിരുന്നത് നാൽപത് ഊക്കിയ വെള്ളി, അഥവാ നാലായിരം വെള്ളി ദിർഹമുകളായിരുന്നു മോചനദ്രവ്യം. അത് നൽകാൻ കഴിയാത്തവർക്ക് പത്ത് മുസ്‌ലിം കുട്ടികൾക്ക് എഴുത്ത് പഠിപ്പിക്കൽ മോചനദ്രവ്യമായി പ്രവാചകൻ (സ) നിശ്ചയിച്ചു.

ഇബ്നു അബ്ബാസ് (റ) പറഞ്ഞു: ബദ്ർ യുദ്ധാനന്തരം ബന്ധികളാക്കപ്പെട്ടവരിൽ ചിലർക്കും മോചനദ്രവ്യം നൽകാൻ ഒന്നുമുണ്ടായിരുന്നില്ല. അൻസ്വാരികളുടെ കുട്ടികൾക്ക് ‘എഴുത്ത് പഠിപ്പിക്കൽ’ മോചന മാർഗമായി അല്ലാഹുവിന്റെ ദൂതൻ അവർക്ക് നിശ്ചയിച്ച് കൊടുത്തു. (മുസ്നദ് അഹ്മദ്: 2217)

പുരുഷന്മാർക്ക് പുറമെ, അക്കാലഘട്ടത്തിലെ ധാരാളം സ്ത്രീകൾക്കും എഴുത്തും വായനയും അറിയാമായിരുന്നുവെന്ന് ചരിത്രകാരനും ഭൂമിശാസ്‌ത്രഗ്രന്ഥകാരനുമായ ബലാദുരി വ്യക്തമാക്കുന്നുണ്ട്. (ഫുതൂഹുൽ ബുൽദാൻ: 581)

ഹഫ്‌സ ബിൻത്ത് ഉമർ, ശിഫാഅ് ബിൻത്ത് അബ്ദുശ്ശംസ്, ഉമ്മു കുൽസൂം ബിൻത്ത് ഉക്ബ, ആഇശ ബിൻത്ത് സഅ്ദ്, കരീമ ബിൻത്ത് മിക്‌ദാദ് എന്നിവർ ഉദാഹരണം. (ഫുതൂഹുൽ ബുൽദാൻ: 581, അൽ ഇസ്തീആബ്: 4: 1811,1869,1953, ത്വബകാത്തു ഇബ്നു സഅ്ദ്: 8: 467, തക്‌രീബു തഹ്ദീബ്: 2: 857)

എഴുത്തും വായനയുമറിയാവുന്ന സ്ത്രീകളെ കൊണ്ട് തന്റെ പത്നിമാർക്കും എഴുത്തും വായനയും പഠിപ്പിക്കാനും പ്രവാചകൻ (സ) ശ്രമിച്ചിരുന്നു. ശിഫാഅ് ബിൻത്ത് അബ്ദുശ്ശംസ് പ്രവാചക പത്നി ഹഫ്‌സക്ക് എഴുത്ത് പഠിപ്പിക്കുന്ന സന്ദർഭത്തിൽ വീട്ടിൽ കയറി വന്ന പ്രവാചകൻ (സ), അധ്യാപികയോട് “താങ്കൾ അവൾക്ക് (ഹഫ്‌സ) എഴുത്തു പഠിപ്പിച്ചതു പോലെ രോഗ പ്രാർത്ഥനയും പഠിപ്പിച്ചു കൂടേ?” എന്ന് ചോദിച്ചതായി ഹദീസുകളിൽ കാണാം. (സ്വഹീഹു അബൂദാവൂദ്: 3887)

ഇസ്‌ലാമിക ചരിത്രത്തെ സംബന്ധിച്ച് അൽപ്പമെങ്കിലും പരിജ്ഞാനമുള്ള ആരും തന്നെ ഇസ്‌ലാം, സ്ത്രീകൾ കയ്യെഴുത്ത് പരിശീലിക്കുന്നതിനെ എതിർത്തുവെന്ന് പറയാൻ യാതൊരു സാധ്യതയുമില്ല. കാരണം ഇസ്‌ലാമിക ചരിത്രത്തിലുടനീളം പുരുഷന്മാരേക്കാൾ സ്ത്രീകൾ അതിവിദൂരം മുന്നിട്ടു നിന്ന പ്രത്യേകം ചില മേഖലകളിലൊന്നായിരുന്നു ‘കയ്യെഴുത്ത്’. ചില ഉദാഹരണങ്ങൾ കുറിക്കാം:

* അച്ചടി യന്ത്രമോ ആധുനിക സാങ്കേതിക ഉപകരണങ്ങളോ നിലവിലില്ലായിരുന്ന മധ്യ കാലഘട്ടത്തിൽ ഗ്രന്ഥങ്ങളുടെ പ്രതികൾ കൈപ്പടം കൊണ്ട് പകർത്തി എഴുതുന്ന തൊഴിൽ ചെയ്തിരുന്നവരെ ‘വർറാക്കു’കൾ (الوراقون) എന്നാണ് അറിയപ്പെട്ടിരുന്നത്. പൗരാണിക കാലഘട്ടത്തിലെ അച്ചടിശാലകളായിരുന്നു അവർ. സ്ത്രീകൾ ചേതോഹരമായ കൈയ്യക്ഷരത്തിന് ഉടമകളായത് കൊണ്ട് തന്നെ അവർ തന്നെയായിരുന്നു ഈ മേഖലയിൽ പ്രധാനികൾ.

* കോർഡോവയിൽ നൂറ്റി എഴുപത് സ്ത്രീകൾ കൂഫി ലിപിയിൽ മുസ്ഹഫുകൾ ഇറക്കിയിരുന്നുവെന്ന് സ്പെയിനിലെ ചരിത്രകാരനായ അബുൽഫിയാദ് രേഖപ്പെടുത്തുന്നുണ്ട്.

* കവയത്രി ത്വലൈത്വലിയ (ഹി. 540) ഇക്കൂട്ടത്തിൽ പെടുന്നു.

* ബാഗ്‌ദാദിലെ ഏറ്റവും വലിയ അച്ചടിശാലയിൽ പ്രൂഫ് റീഡിംഗിലും പകർപ്പിലും പങ്കെടുത്തിരുന്ന ജാരിയ്യ തൗഫീക് സൗദാഇനെ സംബന്ധിച്ച് സാഹിത്യകാരൻ അബുൽ അലാഅ് മഅരി സ്മരിക്കുന്നുണ്ട്.

* ആറാം നൂറ്റാണ്ടുകാരിയായ പണ്ഡിത മർയം ബിൻത് അബ്ദുൽ കാദിർ, അബൂ നസ്ർ അൽജൗഹരിയുടെ ‘കാമൂസു സ്വിഹാഹ്’ എന്ന ഗ്രന്ഥം പകർത്തിയെഴുതിയിട്ടുണ്ട്, അതിന്റെ പ്രതി ബാഗ്‌ദാദിലെ ഹൈദർഖാന ലൈബ്രറിയിൽ സൂക്ഷിക്കപ്പെട്ടിട്ടുമുണ്ട്. ഗ്രന്ഥത്തിന്റെ അവസാനത്തിൽ അവർ ഇപ്രകാരം രേഖപ്പെടുത്തിയതായി കാണാം : “ഈ പ്രതിയിൽ വല്ല സ്ഖലിതങ്ങളും ശ്രദ്ധയിൽ പെട്ടാൽ വായനക്കാർ എന്നോട് പൊറുക്കുമെന്ന് ഞാൻ ആശിക്കുന്നു. കാരണം, എന്റെ വലതു കൈ കൊണ്ട് ഞാൻ ഗ്രന്ഥം പകർത്തുമ്പോൾ ഇടതു കൈ കൊണ്ട് എന്റെ കുഞ്ഞിനെ ഞാൻ തൊട്ടിലിൽ ആട്ടുകയും ചെയ്യുമായിരുന്നു.”

* ഹിജ്റ 374ൽ മരണപ്പെട്ട ലുബ്ന അൽ ഖുർത്വബിയ, ഇസ്‌ലാമിക് സ്പെയിനിലെ കവിയത്രി, കൈയെഴുത്തുശാസ്‌ത്രത്തിലും (calligraphy) ലിപിന്യാസത്തിലും (penmanship) അഗ്രേസരയായിരുന്നു. സ്പെയിനിലെ ഉമവി ഖലീഫയായിരുന്ന അൽ ഹകം അൽ മുസ്തൻസിർബില്ലായുടെ കൊട്ടാര എഴുത്തുകാരിയായിരുന്നു. ബഹുമുഖപ്രതിഭയായിരുന്ന ഇവർ ശാസ്‌ത്രചിന്തകയും ഗണിതജ്ഞയും വ്യാകരണപണ്ഡിതയുമെല്ലാമായിരുന്നു. ഒരു അടിമ പെൺകുട്ടിയായിട്ടാണ് ജനിച്ചതെങ്കിലും ഖലീഫയുടെ സെക്രട്ടറിയായിത്തീർന്നു. ഖുർത്വുബയിലെ സഹ്റാഅ നഗരത്തിലെ പ്രശസ്തമായ ലൈബ്രറിയുടെ നിർമ്മാണത്തിലും നടത്തിപ്പിലും വിജ്ഞാനകുതുകിയായ ഇവർ നിസ്തുലമായ പങ്ക് വഹിച്ചു. എണ്ണമറ്റ കൈയ്യെഴുത്തു പ്രതികൾ ലൈബ്രറിക്കായി വിവർത്തനം ചെയ്തു.

* ഹിജറാബ്ദം 440 ൽ മരണപ്പെട്ട, പണ്ഡിതയും എഴുത്തുകാരിയുമായ ‘ബിൻത്തുൽ അക്റഅ് ‘ജനങ്ങളെ മനോഹരമായ കൈപ്പടയിൽ എഴുതാൻ പഠിപ്പിച്ചിരുന്നു. അന്നത്തെ ഖലീഫയുടെ എഴുത്തുകുത്തുകൾ കൈകാര്യം ചെയ്യാൻ ഔദ്യോഗികമായി നിയോഗിതയായ ഇവരുടെ എഴുത്ത് അക്കാലഘട്ടത്തിലെ മികച്ച കയ്യെഴുത്തിന് മാതൃകയായി അവലംബിക്കപ്പെട്ടു എന്ന് ചരിത്ര ഗ്രന്ഥമായ സിയറു അഅ്ലാമിന്നുബലാഅ് വർണിക്കുന്നു.

* ഉമ്മു മുഹമ്മദ് ശുഹ്ദ ബിൻത്ത് അഹ്മദ്. ഹിജ്റാബ്ദം 484 ഇറാഖിലെ ബാഗ്‌ദാദിൽ ജനനം. ഹദീസ് പണ്ഡിത; ഇമാം മാലികിൻ്റെ മുവത്വയിലെ ഹദീഥുകൾ മുഴുവനായും ചെറുപ്രായത്തിലെ മനപാഠമാക്കി. സാഹിത്യകാരി, പ്രാസംഗിക, ‘കൈയെഴുത്ത് കലാ പ്രതിഭ’ തുടങ്ങി സകല കലാ വല്ലഭയായ ശുഹ്ദ ‘സ്ത്രീകൾക്ക് അഭിമാനഹേതുവായി’രുന്നതിനാൽ ‘ഫഖ്റുന്നിസാഅ്’ എന്ന് വിളിക്കപ്പെട്ടു. ഫഖ്റുന്നിസയുടെ കീഴിൽ ഹദീഥ് പഠിക്കുക എന്നത് ഇസ്‌ലാമിക ലോകത്തെ നാനാ ദിക്കുകളിൽ നിന്നുമുള്ള ഹദീസ് പണ്ഡിതർ അനുഗ്രഹവും അഭിമാനവുമായി പരിഗണിച്ചു. ഇബ്നു അസാകിർ, സംആനി, ഇബ്നുൽ ജൗസി തുടങ്ങിയ എണ്ണമറ്റ വിശ്വപ്രസിദ്ധ പണ്ഡിതർ ഫഖ്റുന്നിസയിൽ നിന്നും ഹദീസ് പഠിച്ചവരാണ്.

* ഹിജ്റാബ്ദം 460 ൽ മരണപ്പെട്ട ഹദീസ് പണ്ഡിതയും പ്രഭാഷകയുമായ ആഇശ ബിൻത്ത് ഹസനിബ്നു ഇബ്റാഹീം അൽ ഇസ്ബഹാനിയ്യയുടെ പാണ്ഡിത്യത്തെ സംബന്ധിച്ച് ഇമാം ദഹബി, സിയറിൽ വാനോളം വാഴ്ത്തുന്നുണ്ട്. ഇബ്നു മന്ദയുടെ ആമാലി എന്ന ഗ്രന്ഥം അദ്ദേഹത്തിൽ നിന്ന് അവർ നേരിട്ട് കേട്ടെഴുതിയ കയ്യെഴുത്തു പ്രതിയായിരുന്നു ഗ്രന്ഥത്തിന്റെ പ്രചാരണത്തിന്റെ പ്രധാന അവലംബം.

ഇമാം ദഹബിയുടെ സിയറു അഅ്ലാമിന്നുബലാഅ്, ഇമാം ഇബ്നു ഹജറിന്റെ അൽ ഇസ്വാബ, ഇമാം മിസ്സിയുടെ തഹ്ദീബുൽ കമാൽ, ഇമാം ഇബ്നു കസീറിന്റെ അൽ ബിദായത്തു വന്നിഹായ, ഉമർ രിദായുടെ അഅ്ലാമുന്നിസാഅ് തുടങ്ങിയ ഗ്രന്ഥങ്ങളിൽ പ്രവാചക കാലഘട്ടം മുതൽ മധ്യകാലഘട്ടം വരെയുള്ള എഴുത്തുകാരികളായ ആയിരക്കണക്കിന് മുസ്‌ലിം പണ്ഡിതകളുടെ ജീവചരിത്രം നമുക്ക് വായിക്കാൻ സാധിക്കും.

ഹിജറാബ്ദം 384 ൽ ജനിച്ച ലോക പ്രസിദ്ധ കർമ്മശാസ്ത്ര പണ്ഡിതനായ ഇബ്നു ഹസം പൗരാണിക ഇസ്‌ലാമിക സമൂഹത്തിലെ പണ്ഡിതകളെ സംബന്ധിച്ച് വിശദീകരിക്കവെ അവരുടെ ‘കയ്യെഴുത്തി’നെ സംബന്ധിച്ച് പ്രത്യേകം പരാമർശിക്കുന്നുണ്ട്.

“സ്ത്രീകളുടെ മടിത്തട്ടിലാണ് ഞാൻ വളർന്നത്, അവരുടെ മുമ്പിലാണ് ഞാൻ പിച്ചവെച്ചതും. വലുതായതിന് ശേഷമാണ് പുരുഷന്മാരുമായി ഞാൻ സഹവസിക്കാൻ ആരംഭിച്ചത്… (അതിന് മുമ്പേ) സ്ത്രീകളാണ് എന്നെ കുർആൻ പഠിപ്പിച്ചത്. ധാരാളം കവിതകൾ പാടാനും ‘നല്ല കൈപ്പടയിൽ എഴുതാനും’ എന്നെ പരിശീലിപ്പിച്ചതും അവർ തന്നെയായിരുന്നു…”

വിമർശനം:

വഹ്‌യ് (ദിവ്യബോധനം) ദീർഘകാലം തടയപ്പെടുമ്പോഴെല്ലാം വിഷാദം ബാധിച്ച് മുഹമ്മദ് നബി മലമുകളിൽ നിന്ന് ചാടി ആത്മഹത്യ ചെയ്യാൻ ശ്രമിക്കുക പതിവായിരുന്നു. അല്ലാഹുവിന്റെ മലക്കായ ജിബ്‌രീൽ അതിൽ നിന്നും അദ്ദേഹത്തെ തടഞ്ഞു കൊണ്ടുമിരുന്നു. ഇത് ‘സ്വഹീഹുൽ ബുഖാരി’യിൽ രേഖപ്പെടുത്തപ്പെട്ടിരിക്കുന്നു.

മറുപടി:

1. പ്രവാചകനിലേക്ക്(സ) ചേർക്കപ്പെടുന്ന ഈ “ആത്മഹത്യകഥ”ക്ക് യാതൊരു വിധ പ്രാമാണിക സ്രോതസ്സും അടിത്തറയും ഇല്ല. ദുർബലമായ ചില ചരിത്ര നിവേദങ്ങളിൽ നിന്നും ഒരു നിവേദകന്റെ കേട്ടറിവ് മാത്രമാണ് ഈ അവാസ്തവികമായ “കഥ”യുടെ ആധാരം. ഇസ്‌ലാമിക പ്രമാണങ്ങൾ ക്വുർആനും സ്വഹീഹായ ഹദീസുകളുമാണ്. പ്രവാചക ചരിത്രത്തിന്റെ സ്രോതസ്സും ഇവ രണ്ടും തന്നെ. അല്ലാതെ ഏതെങ്കിലും ഹദീസ് നിവേദകന്റെ – തെളിവുകളുടെ പിൻബലമില്ലാത്ത- “കേട്ടറിവോ” ദുർബല നിവേദനങ്ങളൊ അല്ല.

2. ദിവ്യബോധനത്തിന്റെ ആരംഭം എങ്ങനെയായിരുന്നു എന്ന് വിവരിക്കുന്ന ഒരു ഹദീസുമായി ബന്ധപ്പെട്ടാണ് ഈ കഥ കടന്നുവരുന്നത്. ദിവ്യബോധനത്തിന്റെ ആരംഭത്തെ സംബന്ധിച്ച ആഇശയുടെ(റ) ഹദീസ് സ്വഹീഹുൽ ബുഖാരിയിൽ മൂന്ന് സ്ഥലങ്ങളിൽ (ഹദീസ് നമ്പറുകൾ: 3, 4953, 6581) ആവർത്തിക്കപ്പെടുന്നുണ്ട്. ദീർഘമായ ഈ ഹദീസിന്റെ ചുരുക്കം ഇപ്രകാരമാണ്:

പ്രവാചകന് വഹ്‌യുകളുടെ (ദിവ്യബോധനത്തിന്റെ) ആരംഭം സ്വപ്നങ്ങളിലൂടെയായിരുന്നു. അദ്ദേഹം കാണുന്ന സ്വപ്നങ്ങളൊക്കെ പ്രഭാതോദയം പോലെ പുലരുമായിരുന്നു. പിന്നീട് ഹിറാ ഗുഹയില്‍ ആരാധനയില്‍ മുഴുകി കഴിയുമ്പോൾ അദ്ദേഹത്തിന്റെ അടുത്ത് മലക്ക് – ജിബ്‌രീല്‍ വന്ന് അല്ലാഹുവിൽ നിന്നുള്ള വഹ്‌യ് ആദ്യമായി നേരിട്ട് നൽകി. (ക്വുർആനിലെ 96 ആം അധ്യായത്തിലെ ഒന്നു മുതൽ അഞ്ച് വരെയുള്ള വാക്യങ്ങൾ പ്രവാചകന് ഓതി കേൾപ്പിച്ചു.) ഉടനെ പ്രവാചകൻ (സ) പേടിച്ചുവിറയ്ക്കുന്ന ഹൃദയത്തോടെ ഈ സൂക്തങ്ങളുമായി പത്നി ഖദീജയുടെ അടുക്കല്‍ ചെന്നു. തനിക്കെന്തെങ്കിലും കുഴപ്പം ബാധിച്ചോ എന്ന് ഭയപ്പെട്ട പ്രവാചകനെ പത്നി ആശ്വസിപ്പിച്ചു. ഖദീജ (റ) പറഞ്ഞു: “ഇല്ല അല്ലാഹുവാണെ, അവനൊരിക്കലും അങ്ങയെ അപമാനിക്കുകയില്ല. അങ്ങ് കുടുംബബന്ധം പുലര്‍ത്തുന്നു. നിരാലംബരുടെ ഭാരം ചുമക്കുന്നു. അഗതികള്‍ക്ക് (അവകാശങ്ങള്‍) നേടിക്കൊടുക്കുന്നു. അതിഥികളെ സൽക്കരിക്കുന്നു. ദുരിതബാധിതരെ സഹായിക്കുന്നു.” പിന്നീടവര്‍ പ്രവാചകനേയും കൂട്ടി തന്റെ പിതൃവ്യപുത്രന്‍ വറകതിബ്‌നു നൗഫലിനെ സമീപിച്ചു. വറക്വ ജാഹിലിയ്യാ കാലത്ത് ക്രിസ്ത്യാനിയായിരുന്നു. അദ്ദേഹത്തിന് ഹീബ്രു എഴുത്ത് അറിയാമായിരുന്നു. അന്ധനായ പടുവൃദ്ധനായി കഴിഞ്ഞിരുന്ന അദ്ദേഹത്തോട് ഖദീജ (റ) പറഞ്ഞു. പിതൃവ്യപുത്രാ താങ്കളുടെ സഹോദരപുത്രന് പറയാനുള്ളത് കേള്‍ക്കുക, വറക്വ ചോദിച്ചു നീ കണ്ടെതെന്താണ്? കണ്ട വിശേഷങ്ങളെല്ലാം പ്രവാചകൻ (സ) പറഞ്ഞുകേൾപ്പിച്ചു. വറക്വ പ്രതിവചിച്ചു: “മൂസാ(അ)യുടെ അടുത്തേക്ക് അല്ലാഹു അയച്ച രഹസ്യ സൂക്ഷിപ്പുകാരന്‍ (ജിബ്‌രീല്‍) ആണത്. (താങ്കള്‍ പ്രവാചകനാവുമ്പോള്‍) ഞാനൊരു യുവാവായിരുന്നെങ്കില്‍? താങ്കളുടെ ജനത താങ്കളെ ബഹിഷ്‌കരിക്കുന്ന ഘട്ടത്തില്‍ ഞാന്‍ ജീവിച്ചിരുന്നെങ്കിലെന്ന് ആശിക്കുന്നു.” ഉടനെ പ്രവാചകൻ (സ) ചോദിച്ചു: “അവരെന്നെ പുറത്താക്കുമെന്നോ?” വറക്വ പറഞ്ഞു: “അതെ, താങ്കള്‍ കൊണ്ടുവന്നതു പോലെയുള്ളത് കൊണ്ടുവന്നവരൊക്കെ എതിര്‍ക്കപ്പെട്ടിട്ടുണ്ട്. ആ നാളുകളില്‍ ഞാനുണ്ടെങ്കില്‍ സുശക്തമായ പിന്‍ബലം നല്‍കി സഹായിക്കും.” ഏറെ കഴിയുംമുമ്പേ വറക്വ മരണമടഞ്ഞു. കുറച്ച് കാലത്തേക്ക് വഹ്‌യും നിലച്ചു. പ്രവാചകനെ (സ) ഈ രണ്ടു സംഭവങ്ങളും ദുഖിപ്പിച്ചു.

സ്വഹീഹുൽ ബുഖാരിയിൽ മൂന്ന് സ്ഥലങ്ങളിൽ (ഹദീസ് നമ്പറുകൾ: 3, 4953, 6581) ആവർത്തിക്കപ്പെടുന്ന സ്വഹീഹായ ഹദീസിന്റെ ഉള്ളടക്കം ഇതാണ്. ഇതിൽ പ്രവാചകൻ (സ) പിന്നീട് വിഷാദത്തിൽ ആണ്ടു പോയെന്നോ ആത്മഹത്യ ചെയ്യാൻ ശ്രമിച്ചു എന്നോ ഒന്നുമില്ല.!!

3. എന്നാൽ 6581 ആം നമ്പർ ഹദീസിൽ നാം വിവരിച്ച സംഭവത്തിന് തുടർച്ചയായി ഹദീസ് ഉദ്ധരിക്കുന്ന റാവിമാരിൽ ഒരാളായ ഇബ്നു ശിഹാബ് അസ്സുഹ്‌രി ഈ സംഭവവുമായി ബന്ധപ്പെട്ട് തനിക്കു കിട്ടിയ ഒരു ചരിത്ര നിവേദനം സാന്ദർഭികമായി കൂട്ടി ചേർത്ത് പറയുന്നുണ്ട്. അതിന്റെ ചുരുക്കം ഇപ്രകാരമാണ്:

വറക്വയുടെ മരണവും കുറച്ചു കാലത്തേക്ക് വഹ്‌യ് നിലച്ചതും പ്രവാചകന് തീവ്രമായ വിഷാദമുണ്ടാക്കി. അദ്ദേഹം മലയിൽ നിന്ന് ചാടി മരിക്കാൻ തുനിഞ്ഞു. അപ്പോൾ അല്ലാഹുവിന്റെ മലക്കായ ജിബ്‌രീൽ അദ്ദേഹത്തെ തടയുകയും “താങ്കൾ അല്ലാഹുവിന്റെ സത്യദൂതൻ തന്നെ” എന്ന് ഉറപ്പ് നൽകുകയും ചെയ്തു.

ഈ കഥ, ദിവ്യബോധനത്തിന്റെ ആരംഭത്തെ സംബന്ധിച്ച, ആഇശയുടെ (റ) സ്വഹീഹായ ഹദീസിന്റെ ഭാഗമല്ല. ആയിരുന്നെങ്കിൽ ആഇശയുടെ (റ) ഹദീസ് ആവർത്തിക്കപ്പെടുന്നിടത്തെല്ലാം (ഹദീസ് നമ്പറുകൾ: 3, 4953, 6581) ഈ “ആത്മഹത്യ കഥ”യും ആവർത്തിക്കപ്പെടുമായിരുന്നു. സ്വഹീഹുൽ ബുഖാരിയിലെ ഹദീസ് നമ്പർ: 3, ഹദീസ് നമ്പർ: 4953 എന്നിവിടങ്ങളിൽ ഈ “ആത്മഹത്യ കഥ”യുടെ വാൽകഷ്‌ണം നമുക്ക് കാണാൻ കഴിയില്ല. 6581 ആം നമ്പറായി ഉദ്ധരിക്കപ്പെട്ടിടത്ത് മാത്രമാണ്, ഹദീസിന് അനുബന്ധമെന്നോണം “ആത്മഹത്യ കഥ” ചേർക്കപ്പെട്ടിരിക്കുന്നത്.

“നമ്മുടെ അടുക്കൽ എത്തിയ ഒരു വിവരമനുസരിച്ച്” (فيما بلغنا) എന്ന് സൂചിപ്പിച്ച് കൊണ്ട് ഇബ്നു ശിഹാബ് അസ്സുഹ്‌രിയാണ് ആത്മഹത്യാ ശ്രമത്തെ സംബന്ധിച്ച ഈ കഥ ഒരു അനുബന്ധമായി കൂട്ടി ചേർത്തിട്ടുള്ളത്. സ്വഹീഹുൽ ബുഖാരിയിൽ 6581 ആം നമ്പർ ഹദീസിന്റെ അവസാന ഭാഗത്ത് ഇത് നമുക്ക് പ്രത്യേകം കാണാൻ സാധിക്കുന്നതാണ്.

അതല്ലാതെ സ്വഹീഹുൽ ബുഖാരിയിൽ മൂന്ന് സ്ഥലങ്ങളിൽ ആവർത്തിക്കപ്പെട്ടിട്ടുള്ള -ദിവ്യബോധനത്തിന്റെ ആരംഭത്തെ സംബന്ധിച്ച- ആഇശയുടെ (റ) ഹദീസിന്റെ ഭാഗമല്ല അത്. ഹദീസുകൾ ഉദ്ധരിക്കുമ്പോഴും കൈമാറുമ്പോഴും ഹദീസുകളുടെ അനുബന്ധമായി അൽപം ചില റാവിമാർ (നിവേദകന്മാർ) ചില അധിക വിവരങ്ങൾ അനുബന്ധമായി ചേർത്തെന്നു വരാം. ഇത്തരം അധികരിച്ച അനുബന്ധങ്ങളെ ഹദീസ് നിദാനശാസ്ത്രത്തിന്റെ സാങ്കേതിക ഭാഷയിൽ ‘മുദ്റജ്’ (المدرج) എന്നാണ് വിളിക്കപ്പെടുക. നിവേദകന്റെ ഈ അധികരിച്ച അനുബന്ധം അഥവാ ‘മുദ്റജ്’ ആ നിവേദനത്തിന്റെ ഭാഗമല്ല. അത് നിവേദകന്റെ വ്യക്തിപരമായ അഭിപ്രായം മാത്രമായാണ് പരിഗണിക്കപ്പെടുക. അത്തരം അഭിപ്രായങ്ങൾ ഹദീസുമല്ല, സ്വഹീഹുമല്ല, ഇസ്‌ലാമിൽ പ്രമാണവുമല്ല.

അതുകൊണ്ട് തന്നെ, “നമ്മുടെ അടുക്കൽ എത്തിയ ഒരു വിവരമനുസരിച്ച്” ( فيما بلغنا ) എന്ന് സൂചിപ്പിച്ചുകൊണ്ട് ഇബ്നു ശിഹാബ് അസ്സുഹ്‌രി അനുബന്ധമായി കൂട്ടി ചേർത്ത, ഈ “ആത്മഹത്യാ ശ്രമത്തിന്റെ കഥ”, ദിവ്യബോധനത്തിന്റെ ആരംഭത്തെ സംബന്ധിച്ച, ആഇശയുടെ (റ) ഹദീസിന്റെ ഭാഗമല്ല; സ്വഹീഹായ (സ്ഥിരപ്പെട്ടതും വിശ്വസനീയവുമായ) സംഭവവുമല്ല.

ശൈഖ് സ്വാലിഹ് അൽ മുനജ്ജിദ് പറഞ്ഞു: ആത്മഹത്യയുമായി ബന്ധപ്പെട്ട അധികഭാഗം ആഇശയുടെ (റ) സംസാരത്തിലോ ഹദീസിലോ പെട്ടതല്ല. മറിച്ച്, അത് (നിവേദകന്മാരിൽ ഒരാളായ) സുഹ്‌രിയുടെ വാചകമാണ്. അദ്ദേഹം ‘താബിഈ’കളിൽ(പ്രവാചകാനുചരന്മാരുടെ ശിഷ്യന്മാരായ പിൻതലമുറക്കാർ)പെട്ട വ്യക്തിയാണ്. അതുകൊണ്ട് തന്നെ ഈ സംഭവത്തിന് അദ്ദേഹം സാക്ഷിയല്ല. പ്രവാചക ശിഷ്യന്മാരിൽ ആരിൽ നിന്നെങ്കിലുമാണ് ഈ കഥ ഉദ്ധരിക്കുന്നത് എന്ന് സുഹ്‌രി പറഞ്ഞിട്ടുമില്ല. ഇങ്ങനെ, കൃത്യമായ അടിത്തറകളിൽ ഊന്നി കൊണ്ടല്ല ഈ കഥ ഉദ്ധരിക്കുന്നത് എന്നതിനാലാണ് “നമ്മുടെ അടുക്കൽ എത്തിയതനുസരിച്ച്” (فيما بلغنا) എന്ന് സൂചിപ്പിച്ചുകൊണ്ട് ഇബ്നു ശിഹാബ് അസ്സുഹ്‌രി, ആത്മഹത്യയുമായി ബന്ധപ്പെട്ട അധികഭാഗം ഉദ്ധരിച്ചത്.” (ഇസ്‌ലാം സുആൽ വൽജവാബ്: 152611)

ثم إن القائل ” فيما بَلَغَنا ” هو الزهري ، ومعنى الكلام : أن في جملة ما وصل إلينا من خبر رسول الله صلى الله عليه وسلم في هذه القصة . وهو من بلاغات الزهري وليس موصولاً ، وقال الكرماني : هذا هو الظاهر .

ഇബ്നു ഹജർ അൽ അസ്കലാനി പറഞ്ഞു: ” “നമ്മുടെ അടുക്കൽ എത്തിയ ഒരു വിവരമനുസരിച്ച്” (فيما بلغنا) എന്ന് പറഞ്ഞിരിക്കുന്നത് സുഹ്‌രിയാണ്. ആ പറഞ്ഞതിന്റെ അർത്ഥം: ഈ കഥയുമായി ബന്ധപ്പെട്ട് നമ്മുടെ അടുക്കൽ എത്തിപ്പെട്ട മൊത്തം വിവരങ്ങളിൽ പെട്ടതാണ് (പ്രവാചകൻ (സ) ആത്മഹത്യ ചെയ്യാൻ തുനിഞ്ഞു എന്ന കഥ.) ഈ കഥയാകട്ടെ സുഹ്‌രിയുടെ കേവല കേട്ടറിവുകളിൽ പെട്ടത് മാത്രമാണ്. പ്രവാചകനിലേക്കെത്തുന്ന കൃത്യമായ ഒരു നിവേദക പരമ്പരയും അദ്ദേഹത്തിനടുക്കൽ ഇല്ല. കർമാനി പറഞ്ഞു: ഇതാണ് പ്രത്യക്ഷ സത്യം.” (ഫത്ഹുൽ ബാരി: 12:359)

നിവേദകപരമ്പര മുറിഞ്ഞ അല്ലെങ്കിൽ നിവേദകപരമ്പര പറയാത്ത സുഹ്‌രിയുടെ ഇത്തരം അധികരിച്ച അനുബന്ധാഭിപ്രായം മുർസൽ (مرسل) എന്ന ദുർബല നിവേദനങ്ങളിലാണ് ഉൾപ്പെടുക എന്നും ഒരു കാറ്റു പോലെ അടിത്തറയില്ലാത്തവയാണ് സുഹ്‌രിയുടെ ഇത്തരം മുർസലുകൾ എന്നും ഇമാം യഹ്‌യബ്നു സഈദ് അൽകത്വാൻ വ്യക്തമാക്കുന്നുണ്ട്. (ജാമിഉത്തഹ്സ്വീൽ: സ്വലാഹുദ്ദീൻ അലാഈ: 78, തദ്കിറത്തുൽ ഹുഫ്ഫാദ്: 1:1-5)

هذا من كلام الزهري أو غيره ، غير عائشة ، والله أعلم ؛ لقوله : ” فيما بلغنا ” ، ولم تقل عائشة في شيء من هذا الحديث ذلك . അബൂ ശാമ അൽ മക്ദസി പറഞ്ഞു: ” “നമ്മുടെ അടുക്കൽ എത്തിയ ഒരു വിവരമനുസരിച്ച്” (فيما بلغنا) എന്ന വാചകം സുഹ്‌രിയുടേതോ മറ്റൊരു റാവിയുടെതോ ആകുന്നു; അത് ആഇശയുടെ (റ) ഹദീസിൽ പെട്ടതല്ല. ഹദീസിലെവിടെയും “നമ്മുടെ അടുക്കൽ എത്തിയ ഒരു വിവരമനുസരിച്ച്” എന്ന് ആഇശ (റ) പറയുന്നില്ല…” (ശർഹുൽ ഹദീസിൽ മുക്തഫാ : 177)

4. സ്വഹീഹുൽ ബുഖാരി എന്ന ഗ്രന്ഥത്തിന്റെ യഥാർത്ഥ നാമം “അല്ലാഹുവിന്റെ ദൂതന്റെ കാര്യവിവരങ്ങൾ- ചര്യകൾ- യുദ്ധങ്ങൾ എന്നിവയുടെ ഹ്രസ്വ വിവരണങ്ങളായ ‘സ്വഹീഹും’ ‘മുസ്നദു’മായ ഹദീസുകളുടെ സമാഹാരം” (الجامع المسند الصحيح المختصر من أُمور رسول الله صلى الله عليه وسلّم وسننه وأيامه) എന്നാണ്.

ഗ്രന്ഥത്തിന്റെ നാമം സൂചിപ്പിക്കുന്നതു പോലെ പ്രവാചക ഹദീസുകളിലെ ‘സ്വഹീഹും’ ‘മുസ്നദു’മായവ മാത്രമാണ് ഇസ്‌ലാമിലെ പ്രമാണം. മുസ്നദ് (المسند) എന്നാൽ പ്രവാചകനിലേക്ക് എത്തുന്ന സനദ് (നിവേദക പരമ്പര) ഉള്ളത് എന്നാണ് അർത്ഥം. സ്വഹീഹ് (الصحيح) എന്നാൽ ഈ നിവേദക പരമ്പര ഹദീസ് നിദാന ശാസ്ത്രത്തിന്റെ നിബന്ധനകൾ പൂർത്തീകരിക്കപ്പെട്ട സ്വീകാര്യ യോഗ്യമായ ഹദീസുകളുമാണ്.

സ്വഹീഹുൽ ബുഖാരിയിൽ ‘മുസ്നദായി’ ഉദ്ധരിക്കപ്പെട്ട ‘ഹദീസുകൾ’ എല്ലാം സ്വഹീഹാണെന്നും പ്രമാണമാണെന്നും മാത്രമാണ് മുസ്‌ലിംകൾ വിശ്വസിക്കുന്നത്. എന്നാൽ സ്വഹീഹുൽ ബുഖാരിയിൽ ഇത്തരം മുസ്നദായ ഹദീസുകൾക്ക് പുറമെ ധാരാളം ഭാഷാ ചർച്ചകളും, കർമ്മശാസ്ത്ര ചർച്ചകളും, മുഅല്ലക്കായ (പരമ്പര കളഞ്ഞ) നിവേദനങ്ങളും, പണ്ഡിതാഭിപ്രായങ്ങളും തുടങ്ങി അറബി കവിതകൾ വരെ ഉൾപ്പെട്ടിട്ടുണ്ട്… അവയെല്ലാം സ്വഹീഹ് ആണെന്നോ ഇസ്‌ലാമിൽ പ്രമാണമാണെന്നോ മുസ്‌ലിംകളാരും മനസ്സിലാക്കുന്നില്ല; ഇമാം ബുഖാരിയും അഭിപ്രായപ്പെട്ടിട്ടുമില്ല. ഇമാം ബുഖാരി തന്നെയും വാദിച്ചത് തന്റെ ഈ ഗ്രന്ഥത്തിലെ ‘മുസ്നദായ ഹദീസുകളെല്ലാം സ്വഹീഹാണെന്ന്’ മാത്രമാണ്. ആത്മഹത്യക്കായി പ്രവാചകൻ (സ) ചിന്തിച്ചു എന്ന കഥയാകട്ടെ മുസ്നദായ (സനദോടു കൂടിയ) ഒരു ഹദീസ് അല്ല. ആഇശയുടെ ഹദീസിന് അനുബന്ധമായി, സുഹ്‌രി ഉദ്ധരിച്ച സനദില്ലാത്ത ഒരു കേവലാഭിപ്രായമാണ്.

5. വറക്വയുടെ മരണവും ദീർഘ കാലത്തേക്ക് വഹ്‌യ് നിലച്ചതും പ്രവാചകനിലുണ്ടാക്കിയ (സ) തീവ്രമായ വിഷാദത്താൽ പ്രേരിതനായി അദ്ദേഹം (സ) മലമുകളിൽ നിന്ന് ചാടി മരിക്കുന്നതിനെ സംബന്ധിച്ച് ചിന്തിച്ചുവെന്ന് സൂചിപ്പിക്കുന്ന ഈ നിവേദനം സ്വഹീഹുൽ ബുഖാരിയിലല്ലാതെ മറ്റു ചില ചരിത്ര ഗ്രന്ഥങ്ങളിലും വന്നിട്ടുണ്ട് എങ്കിലും പ്രസ്തുത നിവേദനങ്ങളെല്ലാം സംശയലേശമന്യേ ദുർബലവും കെട്ടു കഥകളും തന്നെയാണ്.

a) ഇബ്‌നു മർദൂയയുടെ തഫ്‌സീറിൽ ഉദ്ധരിക്കപ്പെട്ട, കഥയുടെ നിവേദക പരമ്പരയിൽ മുഹമ്മദിബ്‌നു കസീർ എന്ന റാവിയുണ്ട്. ഇയാൾ ദുർബലനും ഓർമ്മക്കുറവുള്ള വ്യക്തിയുമാണ്. അതുകൊണ്ട് തന്നെ “നമ്മുടെ അടുക്കൽ എത്തിയ ഒരു വിവരമനുസരിച്ച്” (فيما بلغنا) എന്ന സുഹ്‌രിയുടെ വാചകം ഒഴിവാക്കി കൊണ്ടാണ് ഇയാൾ കഥ ഉദ്ധരിക്കുന്നതും. (സിൽസിലത്തു ദ്ദഈഫ: 10:453)

b) ഇബ്‌നു സഅ്ദിന്റെ ‘ത്വബക്കാത്തി’ൽ ഉദ്ധരിക്കപ്പെട്ട, കഥയുടെ നിവേദനം വ്യാജമാണ് (മൗദൂഅ്). നിവേദക പരമ്പര ഇപ്രകാരമാണ്.

أخبرنا محمد بن عمر قال حدثني إبراهيم بن محمد بن أبي موسى عن داود بن الحصين عن أبي غطفان بن طريف عن ابن عباس أن رسول الله صلى الله عليه وسلم…

നിവേദക പരമ്പരയിലെ മുഹമ്മദിബ്‌നു ഉമർ എന്ന റാവി, നിവേദനങ്ങൾ വ്യാജമായി നിർമ്മിച്ചുണ്ടാക്കുന്ന വ്യക്തിയാണെന്ന് ആരോപിക്കപ്പെടുന്നു. മറ്റൊരു റാവിയായ ഇബ്രാഹിം ഇബ്‌നു മുഹമ്മദിബ്‌നു അബീ മൂസാ, ‘മത്റൂക്’ (متروك) അഥവാ കളവു പറയുന്നതായി ആരോപിക്കപ്പെടുന്ന വ്യക്തിയാണ്. (സിൽസിലത്തു ദ്ദഈഫ: 10:451)

c) ത്വബ്‌രി ഉദ്ധരിച്ച നിവേദനത്തിന്റെ പരമ്പര:

حدثنا ابن حميد قال حدثنا سلمة عن محمد بن إسحاق قال حدثني وهب بن كيسان مولى آل الزبير قال سمعت عبد الله بن الزبير وهو يقول لعبيد بن عمير بن قتادة الليثي : حدِّثنا يا عبيد كيف كان بدء ما ابتدئ به رسول الله صلى الله عليه و سلم من النبوة حين جاء جبريل عليه السلام فقال عبيد…

ഈ നിവേദകപരമ്പരയിലെ പ്രധാനപ്പെട്ട മൂന്ന് ന്യൂനതകൾ ഇവയാണ്:

ഒന്ന്, നിവേദകപരമ്പരയിലെ റാവിയായ ഉബൈദ് ഇബ്‌നു ഉമർ പ്രവാചക ശിഷ്യനല്ലാത്തതിനാൽ സംഭവത്തിന് സാക്ഷിയല്ല. അപ്പോൾ പ്രവാചകനിലേക്കെത്തുന്ന പരമ്പരയില്ല. രണ്ട്, സലമ ഇബ്‌നുൽ ഫദ്ൽ അൽ അബ്റശ് എന്ന റാവി തീരെ ഓർമ്മ പിശകുള്ള വ്യക്തിയാണ്. മൂന്ന്, ഇബ്‌നു ഹുമൈദ് അങ്ങേയറ്റം ദുർബലനാണ്. അബൂ സർഅ അർറാസിയെ പോലെയുള്ള ഒരു സംഘം ഹദീസ് പണ്ഡിതർ അയാൾ കളവു പറയുന്ന വ്യക്തിയാണെന്ന് പോലും അഭിപ്രായപ്പെട്ടിരിക്കുന്നു. (സിൽസിലത്തു ദ്ദഈഫ: 10:455-457)

പ്രവാചകൻ (സ) മലയിൽ നിന്ന് ചാടി മരിക്കാൻ തുനിഞ്ഞപ്പോൾ അല്ലാഹുവിന്റെ മലക്കായ ജിബ്‌രീൽ അദ്ദേഹത്തെ തടയുകയും “താങ്കൾ അല്ലാഹുവിന്റെ സത്യദൂതൻ തന്നെ” എന്ന് ഉറപ്പ് നൽകുകയും ചെയ്തു എന്ന കഥയിലെ പ്രസ്ഥാവനയിൽ നിന്നു തന്നെ കഥ വാസ്തവ വിരുദ്ധമാണെന്ന് ഏതൊരാൾക്കും മനസ്സിലാക്കാവുന്നതേയുള്ളു. കാരണം, താൻ പ്രവാചകനാണെന്ന സത്യത്തിൽ ഒരു നിമിഷം പോലും മഹാനായ പ്രവാചകൻ മുഹമ്മദ് (സ) ശങ്കിച്ചതായി ക്വുർആനോ സ്വഹീഹായ ഹദീസുകളോ നമ്മുക്ക് അറിവു നൽകുന്നില്ല. താൻ ജീവിക്കുന്ന സമൂഹത്തിലെ അന്ധവിശ്വാസ അനാചാരങ്ങളോടുള്ള തന്റെ വിമുഖതയുടെ കാരണമറിയാതെ വിഷാദനായി ഹിറാ ഗുഹയിലേക്ക് ധ്യാനത്തിന് പോയ പ്രവാചകന് (സ) വഹ്‌യിലൂടെ ജീവിത ലക്ഷ്യവും പ്രവാചകത്വ ദൗത്യവും ഏൽപ്പിക്കപ്പെടുകയാണ് ഉണ്ടായത്. ഇത് ഒരാളുടെ വിഷാദത്തെ ആവേശത്തിലേക്കും നിരാശയെ ജീവിതോന്മുഖതയിലേക്കുമാണ് നയിക്കുക. “മനുഷ്യനായതു കൊണ്ട് തന്നെ വഹ്‌യ് ആദ്യം നൽകപ്പെട്ടപ്പോൾ ഉടലെടുത്ത ആകസ്‌മിക ഭയപ്പാടിൽ നിന്ന് പൂർണ്ണമായ വിമുക്തി കൈവരിക്കാൻ സാവകാശം നൽകി കൊണ്ടാണ് രണ്ടാം വഹ്‌യ് നൽകുന്നതിൽ കാലതാമസമുണ്ടായത്…” എന്ന് ഇബ്‌നു ത്വൂലൂൻ നിരീക്ഷിക്കുന്നുണ്ട്. (സുബുലുൽ ഹുദാ വർറശാദ്: 2:272)

അപ്പോൾ, വറക്വയുടെ മരണവും കുറച്ച് കാലത്തേക്ക് വഹ്‌യ് നിലച്ചതും പ്രവാചകനെ (സ) ദുഖിപ്പിച്ചുവെങ്കിലും, ആത്മഹത്യയിലേക്ക് നയിക്കാൻ മാത്രം ദുർബലമായിരുന്നില്ല ഹിറായിലെ ദിവ്യബോധനത്തിൽ നിന്ന് പുതുതായി അദ്ദേഹത്തിന് ലഭ്യമായ ലക്ഷ്യബോധവും വിശ്വാസ ദർശനവും. ഹൃദയാന്തരാളങ്ങളിലേക്ക് പെയ്തിറങ്ങിയ ആ പുതുമഴ, ഇതിനേക്കാളൊക്കെ എത്രയോ ഭയാനകരമായ പരീക്ഷണാനുഭവങ്ങളെ അതിജീവിക്കാനുള്ള കുളിർമ്മ അദ്ദേഹത്തിന്റെ ജീവിതത്തിന് സമ്മാനിച്ചിട്ടുണ്ട്. അസഭ്യങ്ങൾ, പരിഹാസങ്ങൾ, കല്ലേറുകൾ, കുറ്റപ്പെടുത്തലുകൾ, കൊലവിളികൾ, കൂക്കുവിളികൾ എന്നിവയിൽ തുടങ്ങി… ശത്രുക്കൾ അദ്ദേഹത്തേയും അനുചരന്മാരേയും ക്രൂരമായി പീഡിപ്പിച്ചു, പലരേയും കൊന്നു കളഞ്ഞു, പട്ടിണിക്കിട്ടു, ബഹിഷ്ക്കരിച്ചു, യുദ്ധങ്ങൾ അഴിച്ചു വിട്ടു… ഇങ്ങനെ ആയിരമായിരം പരീക്ഷണങ്ങളുടെ അലമാലകൾക്കുമുമ്പിൽ കണ്ണു ചിമ്മാത്ത, ആദർശ അചഞ്ചലതയുടെ ഗിരിവീര്യമായിരുന്നു മുഹമ്മദ് നബി (സ).

വിമർശനം:

അബ്ബാസിന്റെ മകളായ ഉമ്മു ഹബീബ് ഓടിക്കളിക്കുന്നതു കണ്ട് “അവൾ പ്രായപൂർത്തിയാകുന്ന കാലം വരെ ഞാൻ ജീവിച്ചിരുന്നാൽ ഞാൻ അവളെ വിവാഹം കഴിക്കും” എന്നു മുഹമ്മദ് നബി പറഞ്ഞു. ഇത് നബി കുട്ടികളോട് ആസക്തിയുള്ള വ്യക്തിയായിരുന്നു (pedophile) എന്ന് തെളിയിക്കുന്നു.

മറുപടി:

ഒരു വ്യാജ നിവേദനത്തെ അടിസ്ഥാനപ്പെടുത്തി പ്രചരിക്കപ്പെടുന്ന കള്ളകഥയാണ് ഇത്.

നിവേദനം വ്യാജമാണ് എന്ന വസ്തുതയുടെ തെളിവുകൾ ചർച്ച ചെയ്യുന്നതിന് മുമ്പ് ഈ കഥ, പ്രവാചകൻ ‘പീഡൊഫൈൽ’ (കുട്ടികളോട് ലൈംഗീകാസക്തിയുള്ള വ്യക്തി) ആയിരുന്നു എന്ന് തെളിയിക്കുന്നുണ്ടോ എന്നതാണ് ആദ്യം ചർച്ച ചെയ്യേണ്ട ചോദ്യം. ഇല്ല, എന്നു മാത്രമല്ല പ്രവാചകൻ (സ) പീഡൊഫൈൽ ആയിരുന്നില്ല എന്നാണ് ഈ കഥ തെളിയിക്കുന്നത്. കാരണം, ഒരു പീഡോഫൈലും ഒരു കുഞ്ഞ് വളർന്നു വലുതായിട്ട് അവളെ വിവാഹം ചെയ്യാനല്ലല്ലൊ ആഗ്രഹിക്കുക; അവളുടെ ചെറുപ്രായത്തിൽ തന്നെ അവളെ ലഭിക്കണമെന്നല്ലെ ആശിക്കുക ?! ഉമ്മു ഹബീബിനെ വിവാഹം കഴിക്കാൻ പ്രവാചകനെ പ്രേരിപ്പിച്ച ഘടകം അവളുടെ ചെറുപ്രായമായിരുന്നു എങ്കിൽ വലുതായിട്ട് വിവാഹം ചെയ്യാമെന്നായിരുന്നോ പ്രവാചകൻ (സ) ആഗ്രഹിക്കുക ?! വിമർശകർ സാധാരണയായി വാദിക്കുന്നതു പോലെ പ്രവാചകൻ (സ) ഒരു സ്വേച്ഛാധിപതിയും അവിഹിത ബന്ധക്കാരനും ഒന്നും ആയിരുന്നില്ല എന്ന് കൂടി ഈ കഥ തെളിയിക്കുന്നു. കാരണം ഒരു ഏകാധിപതി, അയാൾ പീഡൊഫൈൽ ആയിരുന്നെങ്കിൽ ആ കുട്ടി വലുതാവാൻ കാത്തിരിക്കുമായിരുന്നോ, ഉടനെ ആഗ്രഹം സഫലീകരിക്കുമായിരുന്നില്ലേ?! ആസക്തി ശമിപ്പിക്കാൻ എന്തിന് ‘വിവാഹം’ ചെയ്യാൻ ആഗ്രഹിക്കുന്നു ?! അവിവാഹിത ബന്ധത്തിൽ ഏർപ്പെടുകയല്ലെ ഒരു പീഡോഫൈൽ ചെയ്യുക?! യുക്തിക്കും ബുദ്ധിക്കും മനശ്ശാസ്ത്രത്തിനും എതിരായ ഇത്തരം പീഡോഫൈൽ ആരോപണം വിമർശകരുടെ ഉള്ളിലെ പീഡോഫീലിയയെ മാത്രമാണ് പ്രതിബിംബിപ്പിക്കുന്നത്.

വിവാഹങ്ങളിലൂടെ ബന്ധങ്ങൾ ഊട്ടി ഉറപ്പിക്കുക എന്നത് അക്കാലഘട്ടത്തിൽ വളരെ പ്രബലമായ ഒരു സാമൂഹിക ശൈലിയായിരുന്നു. തന്റെ പ്രിയ സുഹൃത്തുക്കളും സന്തതസഹചാരികളുമായിരുന്ന അബൂബക്കറിന്റെയും(റ) ഉമറിന്റെയും(റ) മക്കളായ ആഇശയേയും ഹഫ്സയേയും പ്രവാചകൻ (സ) വിവാഹം ചെയ്തത് ഇതേ സാമൂഹിക കാരണങ്ങളിലും സൗഹൃദ പ്രചോദനങ്ങളിലും പ്രേരിതമായി കൊണ്ടായിരുന്നു. അല്ലാതെ അവരുടെ പ്രായമോ സൗന്ദര്യമോ പരിഗണിച്ചു കൊണ്ടായിരുന്നില്ല. ആ കാലഘട്ടത്തിലെ സാമൂഹിക ശീലങ്ങൾ മനസ്സിലാക്കിക്കൊണ്ട് നബി ചരിത്രം പഠിച്ച നിഷ്പക്ഷരായ അമുസ്‌ലിം ചരിത്രകാരന്മാർ പോലും ഈ വസ്തുത വ്യക്തമാക്കിയിട്ടുണ്ട്.

സ്കോട്ടിഷ് ഒറിയന്റലിസ്റ്റും, ചരിത്ര പണ്ഡിതനും, എഡിംഗ്ബർഗ് സർവ്വകലാശാലയിലെ പ്രൊഫസറും, ആംഗ്ലിക്കൻ (കത്തോലിക്കൻ) പുരോഹിതനുമായ വില്യം മോണ്ട്ഗോമറി വാട്ട് (W. Montogomory Watt) പറഞ്ഞു:

“ഇവിടെ നാം ഓർക്കേണ്ട ഒരു വസ്തുത, തീർച്ചയായും ഏഴാം നൂറ്റാണ്ടിലെ അറേബ്യയിലെ പെൺകുട്ടികൾ വളരെ നേരത്തെ പ്രായപൂർത്തി എത്തിയിരുന്നു എന്നതാണ്… ഒരു ബഹുഭാര്യത്വ കുടുംബത്തിൽ ഒരു പെൺകുട്ടിക്ക് ഉണ്ടാകാവുന്നതിൽ വെച്ച് ഏറ്റവും സന്തുഷ്ടയായിരുന്നു ആഇശ എന്ന് നാം കാണുന്നു. അവളുടെ വിവാഹം വ്യക്തമായും അബുബക്കറിനെയും മുഹമ്മദിനെയും തമ്മിൽ ബന്ധിപ്പിക്കുന്നതിനുള്ള രാഷ്ട്രീയ കാരണത്താലായിരുന്നു, അബുബക്കർ മുഹമ്മദിന്റെ ചീഫ് ലെഫ്റ്റനന്റ് ആയിരുന്നതുപോലെ, ആഇശ അദ്ദേഹത്തിന്റെ ഭാര്യമാരിൽ മുഖ്യയായിരുന്നു…” (W. Montgomary Watt, Muhammad: Prophet and Statesman, p.102, ഉദ്ധരണം: ibnehashim.blogspot.com)

മത താരതമ്യ ഗവേണപുസ്തകങ്ങൾക്ക് പേരുകേട്ട ബ്രിട്ടീഷ് എഴുത്തുകാരിയും ഐറിഷ് കത്തോലിക്കാ ബൈബിൾ വ്യാഖ്യാതാവുമായിരുന്ന കാരെൻ ആംസ്ട്രോംഗ് എഴുതി:

“ഭൗമിക ആനന്ദങ്ങളുടെ ഒരു പൂന്തോട്ടത്തിൽ അത്യന്തം സുഖലോലുപനായി മുഹമ്മദ്‌ വെയിൽ കായുകയായിരുന്നു എന്ന് സങ്കൽപ്പിക്കുന്നത് തികച്ചും അനർത്ഥമാണ്… നാം രണ്ട് കാര്യങ്ങൾ ശ്രദ്ധിക്കണം. ഒന്ന്, സൗദയെയും ആഇശയെയും അവരുടെ ലൈംഗിക ആകർഷണീയതയാലല്ല തിരഞ്ഞെടുക്കപ്പെട്ടത്. ആഇശ ഒരു കൊച്ചു പെൺകുട്ടി മാത്രമായിരുന്നു, മുപ്പതാം വയസ്സിൽ, സൗദ തന്റെ ആദ്യ യൗവനത്തെ മറികടന്ന് തടികൂടാൻ തുടങ്ങിയിരുന്നു. (രണ്ടു പേരിലും ലൈംഗീക ആകർഷണീയതയൊന്നുമുണ്ടായിരുന്നില്ല) രണ്ട്, രണ്ട് വിവാഹങ്ങൾക്കും ഒരു രാഷ്ട്രീയ മാനമുണ്ടായിരുന്നു: മുഹമ്മദ് പ്രധാനപ്പെട്ട പല ബന്ധങ്ങളും സൃഷ്ടിക്കുകയായിരുന്നു ആ വിവാഹങ്ങളിലൂടെ.” (Karen Armstrong, Muhammad: A Biography, p.145)

പ്രവാചകന്റെ പിതൃവ്യനായിരുന്നു അബ്ബാസ്. അദ്ദേഹത്തിന്റെ മകൾ ഉമ്മു ഹബീബ് ഏതോ ഒരു പെൺകുട്ടിയായിരുന്നില്ല… പ്രവാചകന്റെ ‘മുറപ്പെണ്ണി’ന്റെ സ്ഥാനമായിരുന്നു അവൾക്കുണ്ടായിരുന്നത്. “വഴിയിൽ ഓടി കളിക്കുന്ന ഏതോ പെൺകുട്ടിയെ പ്രവാചകൻ (സ) വിവാഹം ചെയ്യാൻ ആഗ്രഹിച്ചു” എന്ന രൂപേണ കഥയെ അവതരിപ്പിക്കുന്ന വിമർശകരുടെ ശൈലി ദുർവ്യാഖ്യാനമാണെന്നർത്ഥം.

“അബ്ബാസിന് നൽകുന്ന സ്ഥാനമോ ആദരവോ മറ്റൊരാൾക്കും പ്രവാചകൻ (സ) നൽകുന്നതായി ഞാൻ കണ്ടിട്ടില്ല..” എന്ന് ആഇശ (റ) പറയുകയുണ്ടായി. (അൽ ഫവാഇദ്: അബൂബക്കർ അൽബസ്സാസ്: 266 )

പ്രവാചകൻ (സ) അങ്ങേയറ്റം ആദരിക്കുകയും സ്നേഹിക്കുകയും ചെയ്തിരുന്ന പിതൃവ്യൻ അബ്ബാസിന്റെ മകളെ, അവൾ വലുതായാൽ -വിവാഹങ്ങളിലൂടെ ബന്ധങ്ങൾ ഊട്ടി ഉറപ്പിക്കുക എന്ന അക്കാലഘട്ടത്തിലെ സാമൂഹിക സമ്പ്രദായമനുസരിച്ച് – വിവാഹം ചെയ്യണമെന്ന് പ്രവാചകൻ (സ) ആഗ്രഹിച്ചു. ഇതിൽ ആസക്തിയും പീഡോഫീലിയയുമൊക്കെ കുത്തി കയറ്റപ്പെടുന്നത് വിമർശകരിലൂടെ മാത്രണ്. വിമർശകരുടെ രോഗാതുര മനസ്സുകളിലെ ലൈംഗീക ഭാവനകളിൽ നിന്നാണ് ഇത്തരം അശ്ലീല വ്യാഖ്യാനങ്ങൾ ഉടലെടുക്കുന്നത്. വസ്തുനിഷ്ഠമായ ഒരു തെളിവിന്റേയും അടിസ്ഥാനത്തിലല്ല.

ഇനി ആദ്യത്തെ പോയന്റിലേക്ക് മടങ്ങി വരാം. ഒരു വ്യാജ നിവേദനത്തെ ലൈംഗിക വ്യാഖ്യാനങ്ങളുടെ മസാല തേച്ച് അവതരിപ്പിക്കപ്പെട്ടതാണ് ഈ കഥ എന്ന് പറഞ്ഞുവല്ലൊ. നിവേദനം വ്യാജമാണെന്നതിന് ഒരുപാട് തെളിവുകളുണ്ട്. അവയിൽ പ്രധാനപ്പെട്ടവ ഇവിടെ ഹ്രസ്വമായി സൂചിപ്പിക്കാം:

1. ഇമാം അഹ്‌മദാകട്ടെ മറ്റാരുമാകട്ടെ ഈ കഥ ഉദ്ധരിക്കുന്നത് ‘ഇബ്നു ഇസ്ഹാക്കി’ൽ നിന്നാണ്. കഥയുടെ നിവേദനങ്ങളുടെയെല്ലാം മൂല സ്രോതസ്സ് ഇബ്നു ഇസ്ഹാക്കിന്റെ നബിചരിത്ര കൃതിയായ (സീറ) ‘സീറത്തുന്നബവിയ്യ’യിൽ ഉദ്ധരിച്ച ഒരു വ്യാജ നിവേദനമാണ്.

ഇബ്നു ഇസ്ഹാകിൽ നിന്ന് ഈ നിവേദനം ഉദ്ധരിച്ച ഇമാം അഹ്‌മദ് തന്നെ ഇബ്നു ഇസ്ഹാകിനെ സംബന്ധിച്ച് അഭിപ്രായപ്പെട്ടത് ഇപ്രകാരമാണ്: “ഹദീസിന്റെ വിഷയത്തിൽ അദ്ദേഹം പ്രമാണമല്ല. വിശ്വസ്ഥരല്ലാത്ത പലരിൽ നിന്നും ഇബ്നു ഇസ്ഹാക് നിവേദനങ്ങൾ ഉദ്ധരിച്ചിരുന്നു…” (സിയറു അഅ്ലാമിന്നുബലാഅ്: 7:46)

ഇബ്നു ഇസ്ഹാക് ഒരു ‘സീറ’ക്കാരനാണ്; ഹദീസ് പണ്ഡിതനല്ല. ആണെന്ന് അദ്ദേഹം സ്വയം വാദിച്ചിട്ടുമില്ല. അദ്ദേഹത്തിന്റേയും മറ്റു ചരിത്രകാരന്മാരുടേയും ചരിത്ര ഗ്രന്ഥങ്ങളിൽ ഏത് സംഭവം ഉദ്ധരിക്കുമ്പോഴും അതിനോടൊപ്പം സനദും (നിവേദക പരമ്പര) ഉദ്ധരിച്ചിരിക്കും. ഇബ്നു ഇസ്ഹാകിന്റെ സീറ ആരംഭിക്കുന്നതു തന്നെ ഇപ്രകാരം പറഞ്ഞു കൊണ്ടാണ്: ﻛﻞ ﺷﻲء ﻣﻦ ﺣﺪﻳﺚ اﺑﻦ ﺇﺳﺤﻖ ﻣﺴﻨﺪ “ഇബ്നു ഇസ്ഹാക്കിന്റെ എല്ലാ നിവേദനങ്ങളും സനദ് (നിവേദക പരമ്പര) ഉള്ളതാണ്.”

ഈ സനദുകൾ (നിവേദക പരമ്പരകൾ) എന്തിനാണ് ഇബ്നു ഇസ്ഹാക്കടക്കമുള്ള ചരിത്രകാരന്മാർ ഓരോ നിവേദനത്തിനൊപ്പവും എഴു വെച്ചിരിക്കുന്നത് എന്ന് വിമർശകർ ചിന്തിച്ചിട്ടുണ്ടോ ? മൈലാഞ്ചി എഴുത്തു പോലെയോ കാലിഗ്രഫി പോലെയോ ചന്തം കൂട്ടാനല്ല ഇത്. എഴുതപ്പെട്ട സംഭവം സത്യമാണോ നുണയാണോ, സ്ഥാപിതമാണോ അതോ കേവല ആരോപണമാണോ എന്ന് വായനക്കാർക്ക് പരിശോധിക്കാനാണ് ഈ സനദുകൾ. “ഈ സനദുകളിലെ നിവേദകർ വിശ്വസ്ഥരല്ലെങ്കിൽ അവർ പറഞ്ഞ കഥയും വിശ്വസ്ഥമല്ലെന്നും അവ സ്വീകരിക്കരുതെന്നും” ചരിത്രകാരന്മാർ തന്നെ വ്യക്തമാക്കിയിട്ടുണ്ട്. (മുഖദ്ദിമി: താരീഖുത്വബ്‌രി: 5)

വിശ്വാസ്യതയിൽ ദുർബലരോ, വ്യാജന്മാരോ ആയിട്ടുള്ള നിവേദകർ ഉദ്ധരിച്ച നിവേദനങ്ങൾ വല്ലതും ഇത്തരം ഗ്രന്ഥങ്ങളിൽ ചേർക്കപ്പെട്ടിട്ടുണ്ടെങ്കിൽ അവ സ്ഥാപിത വസ്തുതകളല്ലെന്നും കേവല കള്ള കഥകളാണെന്നും അവ തള്ളിക്കളയണമെന്നും ഈ ചരിത്ര ഗ്രന്ഥകാരന്മാർ തന്നെ വായനക്കാരോട് അഭ്യർത്ഥിക്കുന്നുണ്ട് എന്നർത്ഥം.

ഒരു വിഷയത്തെയൊ ഒരു കാലഘട്ടത്തെയൊ ചർച്ച ചെയ്യുമ്പോൾ അവയുമായി ബന്ധപ്പെട്ട്, തങ്ങൾക്കു ലഭിച്ച സർവ്വ നിവേദനങ്ങളും ചരിത്രകാരന്മാർ അവരുടെ ഗ്രന്ഥങ്ങളിൽ ഉൾപ്പെടുത്തിയിട്ടുണ്ട്. ആ നിവേദനങ്ങളിൽ ഏതൊക്കെയാണ് സത്യസന്ധം ഏതൊക്കെയാണ് വ്യാജം എന്നത് വേർത്തിരിച്ച് മനസ്സിലാക്കൽ ആ ഗ്രന്ഥങ്ങൾ പഠനവിധേയമാക്കുന്നവരുടെ ബാധ്യതയായാണ് ആ ഗ്രന്ഥങ്ങളുടെ രചയിതാക്കൾ മനസ്സിലാക്കിയത്.

2. ഇനി, ചർച്ച ചെയ്യപ്പെടുന്ന നിവേദനത്തിന്റെ ഉള്ളടക്കവും (മത്‌ന്) അതിന്റെ നിവേദക പരമ്പരയും (സനദ്) കാണുക:

സനദ്: ﻧﺎ ﺃﺣﻤﺪ: ﻧﺎ ﻳﻮﻧﺲ ﻋﻦ اﺑﻦ ﺇﺳﺤﻖ ﻗﺎﻝ: ﺣﺪﺛﻨﻲ اﻟﺤﺴﻴﻦ ﺑﻦ ﻋﺒﺪ اﻟﻠﻪ ﺑﻦ ﻋﺒﻴﺪ اﻟﻠﻪ ﺑﻦ ﻋﺒﺎﺱ ﻋﻦ ﻋﻜﺮﻣﺔ ﻋﻦ اﺑﻦ ﻋﺒﺎﺱ ﻗﺎﻝ:

ഇബ്നു ഇസ്ഹാക് പറഞ്ഞു: എന്നോട് ഹസനിബ്നു അബ്ദുല്ലാഹിബ്നു ഉബൈദുല്ലാഹിബ്നു അബ്ബാസ് പറഞ്ഞു: അദ്ദേഹം ഇക്‌രിമയിൽ നിന്ന് ഉദ്ധരിക്കുന്നു: അദ്ദേഹം അബ്ബാസിൽ നിന്ന് ഉദ്ധരിക്കുന്നു: അദ്ദേഹം പറഞ്ഞു:…

മത്‌ന്: ﻧﻈﺮ ﺭﺳﻮﻝ اﻟﻠﻪ ﺻﻠﻰ اﻟﻠﻪ ﻋﻠﻴﻪ ﻭﺳﻠﻢ ﺇﻟﻰ ﺃﻡ ﺣﺒﻴﺐ اﺑﻨﺔ ﻋﺒﺎﺱ ﻭﻫﻲ ﺑﺪﺭ ﺑﻴﻦ ﻳﺪﻳﻪ ﻓﻘﺎﻝ ﺭﺳﻮﻝ اﻟﻠﻪ ﺻﻠﻰ اﻟﻠﻪ ﻋﻠﻴﻪ ﻭﺳﻠﻢ: ﻟﺌﻦ ﺑﻠﻐﺖ ﻫﺬﻩ ﻭﺃﻧﺎ ﺣﻲ ﻷﺗﺰﻭﺟﻨﻬﺎ…

അബ്ബാസിന്റെ മകൾ ഉമ്മു ഹബീബ് ചെറുതായിരിക്കെ അവളെ നോക്കി അല്ലാഹുവിന്റെ ദൂതൻ (സ) പറഞ്ഞു: ഇവൾക്ക് പ്രായപൂർത്തിയാവുമ്പോൾ ഞാൻ ജീവിച്ചിരിക്കുന്നുവെങ്കിൽ ഞാൻ ഇവളെ വിവാഹം ചെയ്യും… (സീറത്തു ഇബ്നു ഇസ്ഹാക്: 1: 268)

ഇബ്നു ഇസ്ഹാകിനോട് ഈ കഥ ഉദ്ധരിക്കുന്നത്, ഹസനിബ്നു അബ്ദുല്ലാഹിബ്നു ഉബൈദുല്ലാഹിബ്നു അബ്ബാസ് എന്ന റാവിയാണ് (നിവേദകൻ) എന്ന് സനദിൽ നിന്നും വ്യക്തം. ഈ റാവിയുടെ സത്യസന്തതയെയും വിശ്വാസ്യതയേയും സംബന്ധിച്ച് ഹദീസ് നിദാന ശാസ്ത്ര പണ്ഡിതന്മാരും ചരിത്രകാരന്മാരും വിശദീകരിക്കുന്നത് ഇപ്രകാരമാണ്:

ഇമാം നസാഈ പറഞ്ഞു: ഹസനിബ്നു അബ്ദുല്ലാഹിബ്നു ഉബൈദുല്ലാഹിബ്നു അബ്ബാസ് നുണയനാണെന്ന് ആരോപിതനാണ്. (അൽ കാമിൽ ഫി ദുഅഫാഉർ രിജാൽ: ഇബ്നു അദിയ്യ: 3:214)

ഇമാം ബുഖാരി പറയുന്നു: ഹസനിബ്നു അബ്ദുല്ലാഹിബ്നു ഉബൈദുല്ലാഹിബ്നു അബ്ബാസ് വ്യാജനാണ്. (അദ്ദുഅഫാഉ വൽമത്റൂകൂൻ: 145)

ഹസനിബ്നു അബ്ദുല്ലാഹിബ്നു ഉബൈദുല്ലാഹിബ്നു അബ്ബാസ് വ്യാജനാണ് എന്ന് ഇബ്നു ഹജറും (തഹ്ദീബുൽ കമാൽ: 6:385, ഉകൈലിയും (അദ്ദുഅഫാ: 111) സ്വാലിഹ് ഇബ്നു അലി നൗഫലിയും വ്യക്തമാക്കുന്നുണ്ട്.

ഹാകിം പറഞ്ഞു: അയാൾ ശക്തനല്ല.

അബൂ ബിശ്ർ അദ്ദൗലാബി പറഞ്ഞു: അയാളുടെ ഹദീസുകൾ വളരെയധികം മുൻകർ (വാസ്തവ വിരുദ്ധം) ആകുന്നു.

* ബൈഹകി, അബൂഹാതിം അർറാസ്സി, ദഹബി, ഇബ്നു ഹജർ, ഇബ്നുൽ മദീനി എന്നിവർ പറഞ്ഞു: അയാൾ ദുർബലനാണ്.

* ഇബ്നുഹിബ്ബാൻ പറഞ്ഞു: അയാൾ സനദുകൾ കോട്ടിമാട്ടുമായിരുന്നു.

* അഹ്‌മദ് പറഞ്ഞു: വിശ്വസ്ഥർക്കെതിരായ നിവേദനങ്ങൾ ഉദ്ധരിക്കുന്ന വ്യക്തിയാണയാൾ.

* ജൊസ്ജാനി പറഞ്ഞു: അയാളുടെ നിവേദനങ്ങൾ വായിച്ച് സമയം കളയരുത്. (https://hadith.islam-db.com/narrators/1334/)

അപ്പോൾ, ഒരു ചരിത്രകാരൻ, ആയിരക്കണക്കിന് നിവേദനങ്ങളുള്ള തന്റെ ഒരു ഗ്രന്ഥത്തിൽ, ഏതോ ഒരു മൂലയിൽ ഒരു നാലു വരി നീളത്തിൽ എഴുതി ചേർത്ത, ‘നുണയനെന്ന് ആരോപിക്കപ്പെട്ട ഒരു റാവി’യിൽ നിന്ന് ഉദ്ധരിക്കപ്പെട്ട ഒരു ‘വ്യാജ നിവേദന’ത്തിൽ, പ്രവാചകൻ (സ) തന്റെ ‘മുറപ്പെണ്ണായ’ ഉമ്മു ഹബീബ് ചെറുതായിരിക്കുമ്പോൾ, ‘വലുതായാൽ’ ഞാൻ ഇവളെ വിവാഹം ചെയ്യുമെന്ന് പറഞ്ഞതിനെ മസാല തേച്ച് കുട്ടികളോടുള്ള ആസക്തിയാക്കി ദുർവ്യാഖ്യാനിക്കുകയാണ് വിവർശകർ ചെയ്തിരിക്കുന്നത് !! ആരോപണത്തിന് വസ്തുതകളുമായി അകന്ന ബന്ധം പോലുമില്ല.

വിമർശനം:

അനസിന്റെ വീട്ടിനടുത്ത് സ്ഥിതിചെയ്യുന്ന കിണറ്റിൽ മുഹമ്മദ് നബി മൂത്രമൊഴിച്ചു. അതിനു ശേഷം കിണറ്റിലെ വെള്ളത്തിൽ അനുഗ്രഹം നൽകപ്പെട്ടുവെന്നും മധുരകരമായ ആ വെള്ളം ശിഷ്യന്മാർ കുടിക്കാറുണ്ടായിരുന്നെന്നും അനസ് പറയുന്നു.

മറുപടി:

പ്രവാചകൻ (സ) കിണറ്റിൽ മൂത്രമൊഴിച്ചു എന്നത് യഥാർത്ഥ്യവുമായി പുലബന്ധം പോലുമില്ലാത്ത ഒരു വ്യാജകഥയാണ്. മുസ്‌ലിം സമുദായത്തിലെ ചില അന്ധവിശ്വാസികൾ ഇത്തരം കഥകളെ കൊണ്ടു നടക്കുന്നു എന്നത് കൊണ്ട് അവ ഇസ്‌ലാമിന്റേയോ, ഇസ്‌ലാമിക പ്രമാണങ്ങളുടേയോ, പ്രവാചക ചരിത്രത്തിന്റേയോ ഭാഗമാണെന്ന് സ്ഥാപിതമാകില്ല.

കാരണങ്ങൾ:

1. കിണർ ഉൾപ്പെടെ ഒഴുകാതെ കെട്ടി നിൽക്കുന്ന ജലത്തിൽ മല-മൂത്ര വിസർജനം നടത്തുന്നത് പ്രവാചകൻ (സ) ശക്തമായി നിരോധിച്ച കാര്യമാണ്.

“ഒഴുകാതെ കെട്ടി നിൽക്കുന്ന വെള്ളത്തിൽ മൂത്രമൊഴിക്കപ്പെടുന്നത് അല്ലാഹുവിന്റെ ദൂതൻ (സ) വിലക്കിയിരിക്കുന്നു” (സ്വഹീഹു മുസ്‌ലിം: 281) എന്ന് പ്രവാചകാനുചരൻ ജാബിർ (റ) പറഞ്ഞതായി സ്വഹീഹായ ഹദീസുകളിൽ വന്നിരിക്കുന്നു. ഇതാണ് ഇസ്‌ലാമിലെ, ഈ വിഷയത്തിലെ സ്ഥിരപ്പെട്ട പ്രവാചക അധ്യാപനം.

“നിശ്ചലമായ അല്ലെങ്കിൽ ഒഴുകാത്ത വെള്ളത്തിൽ മൂത്രമൊഴിക്കുകയും ശേഷം അതിൽ നിന്ന് അംഗശുദ്ധി വരുത്തുകയോ കുളിക്കുകയോ ചെയ്യുന്നത് പ്രവാചകൻ (സ) നിരോധിച്ചു.” (സ്വഹീഹു മുസ്‌ലിം: 282, സുനനു നസാഈ: 399, മുസ്നദു അഹ്‌മദ്‌: 7868, സുനനു ഇബ്നുമാജ: 344)

ഒഴുകാത്ത ജലത്തിൽ ‘ജനാബത്തു'(വലിയ അശുദ്ധി)കാരനായിരിക്കെ കുളിക്കുന്നത് പോലും പ്രവാചകൻ (സ) നിരോധിച്ചു. അതിൽ നിന്ന് വെള്ളം കോരിയെടുത്തേ കുളിക്കാവൂ എന്ന് നിർദ്ദേശിക്കുകയും ചെയ്തു. (സ്വഹീഹു മുസ്‌ലിം)

ജനാബത്തുകാരനായിരിക്കെ ഒഴുകാത്ത ജലസ്രോതസ്സിൽ മുങ്ങി കുളിച്ചാൽ ജലത്തെ അശുദ്ധമാക്കിയേക്കാം എന്നതു പോലും ശ്രദ്ധയോടെ മനസ്സിലാക്കി, അത് നിരോധിച്ച പ്രവാചകൻ (സ) എങ്ങനെയാണ് കിണറ്റിൽ മൂത്രിക്കുക ?!

2. അനസിന്റെ(റ) വീടിനടുത്ത കിണറ്റിൽ പ്രവാചകൻ (സ) മൂത്രമൊഴിച്ചു എന്ന കള്ള കഥയുടെ സ്രോതസ്സ് അബൂ നുഐമിന്റെ ‘ദലാഇലുന്നുബുവ്വ’ എന്ന ഗ്രന്ഥത്തിലെ ഒരു നിവേദനത്തെ അടിസ്ഥാനപ്പെടുത്തി നിർമ്മിക്കപ്പെട്ടതാണ്.

അബൂ നുഐമിന്റെ ‘ദലാഇലുന്നുബുവ്വ’യിലെ ഈ നിവേദനത്തിൽ പ്രവാചകൻ (സ), ആ കിണറ്റിൽ ‘തുപ്പി’ എന്നാണ് വന്നിട്ടുള്ളത്.

أن النبي صلى الله عليه وسلم بزق في بئر داره

നിവേദനത്തിലെ ‘ബസക’ (بزق) അഥവാ ‘തുപ്പി’ എന്ന പദത്തിന് പകരം ‘ബാല’ (بال) അഥവാ ‘മൂത്രിച്ചു’ എന്ന്, അബൂ നുഐമിന്റെ ‘ദലാഇലുന്നുബുവ്വ’ എന്ന ഗ്രന്ഥത്തിന്റെ ചില പ്രതികളിൽ തെറ്റി ചേർക്കപ്പെട്ടിട്ടുണ്ട്. ഇത് കൈയ്യെഴുത്തു പ്രതികൾ ഉണ്ടാക്കുന്ന ചിലരിൽ നിന്ന് ഉടലെടുത്ത് പിന്നീട് പ്രചരിച്ചതാവാം. എന്നാൽ ‘ദലാഇലുന്നുബുവ്വ’ എന്ന ഗ്രന്ഥത്തിന്റെ ശരിയായ പ്രതിയിൽ നിന്ന് നിവേദനം വായിച്ച, ഉദ്ധരിച്ച ഒരുപാടു പണ്ഡിതന്മാർ ഇത് വ്യക്തമാക്കിയിട്ടുണ്ട്. ചില ഉദാഹരണങ്ങൾ കാണുക:

കാദി ഇയാദ് പറഞ്ഞു: “അദ്ദേഹം അനസിന്റെ വീട്ടിലെ കിണറിൽ ‘തുപ്പി’…” (കിതാബു ശിഫാ: 1/246)

ഹാഫിദ് ഇബ്നു ഹജർ പറഞ്ഞു: “ബുജൈരിയുടെ ഹദീസിൽ പ്രവാചകൻ (സ) അനസിന്റെ വീട്ടിലെ കിണറിൽ ‘തുപ്പി’ എന്ന് പ്രസ്ഥാവിക്കുന്നു…” (ഇത്ഹാഫുൽ മഹ്റ: 1/575: 784)

ജലാലുദ്ദീൻ സുയൂത്വി പറഞ്ഞു: “അബൂ നുഐം അനസിൽ നിന്നും ഉദ്ധരിക്കുന്ന ഒരു നിവേദനത്തിൽ പ്രവാചകൻ (സ) അദ്ദേഹത്തിന്റെ വീട്ടിലെ കിണറ്റിൽ ‘തുപ്പി’യെന്ന് പറയുന്നു…” (ഖസാഇസുൽ കുബ്റാ: 1/105)

അബൂ നുഐം അനസിൽ നിന്നും ഉദ്ധരിക്കുന്ന നിവേദനത്തിൽ പ്രവാചകൻ (സ) അദ്ദേഹത്തിന്റെ വീട്ടിലെ കിണറ്റിൽ ‘തുപ്പി’യെന്നാണ് പറയുന്നത് എന്ന് അലിയ്യുബ്നു അബ്ദുല്ല അസ്സംഹൂദിയും (ഖുലാസത്തുൽ വഫാ: 268) മുഹമ്മദ് ഇബ്നു യൂസുഫ് അസ്സ്വാലിഹിയും വ്യക്തമാക്കുന്നുണ്ട്. (സുബുലുൽ ഹുദാ വർറശാദ്: 7/223)

അപ്പോൾ അബൂ നുഐമിന്റെ ‘ദലാഇലുന്നുബുവ്വ’ എന്ന ഗ്രന്ഥത്തിന്റെ ശരിയായ പ്രതികളിൽ കിണറ്റിൽ ‘തുപ്പി’ എന്നാണുള്ളത്. ‘തുപ്പി’ എന്ന പദത്തിന് പകരം ‘മൂത്രിച്ചു’ എന്ന് വന്നത്, ഗ്രന്ഥത്തിന്റെ ചില പ്രതികളിൽ ഉണ്ടായ അക്ഷര തെറ്റോ, പകർത്തി എഴുത്തുകാരുടെ (വർറാക്കുകൾ) ഓർമ്മ പിശകോ മാത്രമാണ്.

3. ഇനി, കിണറ്റിൽ തുപ്പി എന്ന തെറ്റില്ലാത്ത നിവേദനം തന്നെ സ്വഹീഹ് (ഹദീസ് നിദാനശാസ്ത്ര മാനദണ്ഡങ്ങൾ പ്രകാരം സ്വീകാര്യമായത്) ആണോ എന്ന് അടുത്തതായി നമുക്ക് പരിശോധിക്കാം.

കഥയുടെ നിവേദക പരമ്പര ഇപ്രകാരമാണ്: حدثنا علي بن هارون ثنا موسى بن هارون قال: ثنا عبيد الله بن النعمان المنقري قال: ثنا محمد بن عبد الله الأنصاري قال: حدثني أبي، عن ثمامة، عن أنس قال: كان رسول الله صلى الله عليه وسلم يصلي…

ഈ സനദ് അഥവാ നിവേദക പരമ്പര അങ്ങേയറ്റം ദഈഫ് (ദുർബലം ضعيف) ആകുന്നു. പരമ്പരയിലെ അലിയ്യിബ്നു ഹാറൂൻ എന്ന ‘റാവി’ (നിവേദകൻ) ഓർമ്മക്കുറവുള്ള വ്യക്തിയും നിവേദനങ്ങളുടെ ആശയങ്ങളിൽ കൂടി കലരൽ സംഭവിക്കുന്ന വ്യക്തിയുമാണ്. പരമ്പരയിലെ മറ്റൊരു റാവി (നിവേദകൻ) ഉബൈദുല്ലാഹിബ്നു നുഅ്മാൻ അൽ മിൻകരി ‘മജ്ഹൂൽ’ (مجهول) അഥവാ അറിയപ്പെടാത്ത റാവിയാകുന്നു. പരമ്പരയിലെ മൂന്നാമതൊരു റാവിയായ അബ്ദുല്ലാഹിബ്നുൽ മുസന്നാ അൽ അൻസ്വാരി: നിവേദനത്തിൽ ധാരാളം സ്ഖലിതങ്ങൾ സംഭവിക്കുന്ന റാവിയാണ്. (അൽ ജർഹു വതഅ്ദീൽ: 5/ 177)

ചുരുക്കത്തിൽ, കിണറ്റിൽ ‘തുപ്പി’ എന്ന നിവേദനം തന്നെ ‘ദഈഫ്’ (ഹദീസ് നിദാനശാസ്ത്ര പ്രകാരം ദുർബലമായ നിവേദനം) ആകുന്നു. ഇസ്‌ലാം മതത്തിന്റെയും പ്രവാചക ചരിത്രത്തിന്റേയും അടിത്തറ ക്വുർആനും ‘സ്വഹീഹായ ഹദീസുകളും’ മാത്രമാണ്. ദഈഫ് (ദുർബലം) ആയ ഹദീസുകൾ ഇസ്‌ലാമിക പ്രമാണമല്ല.

വിമര്‍ശനം:

നബിയുടെ മാതൃസഹോദരി ഖൗല ബിന്‍ത് ഹകീം എന്ന സ്ത്രീ സ്വന്തം ശരീരം നബിക്കു ദാനം നല്‍കുകയുണ്ടായി എന്നും അതിനെതിരെ സ്വപത്‌നി ആഇശ പോലും നബിയോട് ‘ഒരു സ്വതന്ത്ര സ്ത്രീ സ്വദേഹം ദാനം ചെയ്യുകയോ’ എന്ന് പ്രതികരിച്ചുവെന്നും ഹദീഥ് ഗ്രന്ഥങ്ങളില്‍ കാണാം. എന്നാല്‍ നബി ഒരിക്കലും അതിനെ മോശമായി കണ്ടിരുന്നില്ല എന്നും ഹദീഥ് ഗ്രന്ഥങ്ങള്‍ തന്നെ മനസ്സിലാക്കി തരുന്നുണ്ട്. സ്ത്രീകളെ കൊണ്ട് സ്വശരീരങ്ങള്‍ തനിക്കായി ദാനം ചെയ്യാന്‍ പ്രേരിപ്പിക്കുകയും സ്വന്തം മാതൃസഹോദരിയെ പോലും അതില്‍ നിന്നും ഒഴിവാക്കാതിരിക്കുകയും ചെയ്ത പ്രവാചക നടപടിയെ അല്‍പമെങ്കിലും ധാര്‍മികബോധമുള്ളവര്‍ക്ക് അംഗീകരിക്കാനാകുമോ?

മാത്രമല്ല, ഉമ്മു ഹറാം, ഉമ്മു സുലൈം എന്നീ സ്ത്രീകളെ നബി സ്വകാര്യ സന്ദര്‍ശനം നടത്താറുണ്ടായിരുന്നെന്നും അവരുടെ മടിയില്‍ പലപ്പോഴും തലവെച്ചു കിടക്കാറുണ്ടായിരുന്നു എന്നും ഹദീഥ് ഗ്രന്ഥങ്ങള്‍ തന്നെ വ്യക്തമാക്കുന്നുണ്ട്. അവിഹിത ബന്ധങ്ങള്‍ക്ക് പ്രവാചക ജീവിതത്തിലിടമുണ്ടായിരുന്നു എന്നു വ്യക്തമാക്കുന്ന സംഭവങ്ങളാണിതെല്ലാം. ഇത്തരത്തിലുള്ള ഒരു വ്യക്തിയെ എങ്ങനെയാണ് മാര്‍ഗദര്‍ശകനായി സ്വീകരിക്കാനാവുക.!?

മറുപടി:

അറബിയില്‍ ഒരു പഴമൊഴിയുണ്ട് ‘നാട്ടിയത് വളഞ്ഞാല്‍ നിഴല്‍ നേരെയാവുമോ?’. അക്ഷരാര്‍ഥത്തില്‍ ഈ അറേബ്യന്‍ പഴമൊഴിയെ ഓര്‍മപ്പെടുത്തുകയാണീ വിമര്‍ശനങ്ങള്‍. വളഞ്ഞ വസ്തു നാട്ടിയിട്ട് അതിന്റെ നിഴലിനെ നേരെയാക്കാന്‍ നോക്കിയിട്ട് കാര്യമുണ്ടോ!?. നിഴല്‍ നേരെയാകണമെങ്കില്‍ ആദ്യം വസ്തു നേരെയാക്കണം. തലമുടി മുതല്‍ കാല്‍നഖം വരെ ലൈംഗികവൈകൃതം പേറുന്നവരില്‍ ഇസ്‌ലാംവിരോധം ഇണചേര്‍ന്നാല്‍ ഇതല്ല ഇതിനപ്പുറവും നബിജീവിതത്തില്‍ നിന്നും ‘ഉല്‍ഘനനം’ ചെയ്‌തെടുത്തുകളയുമവര്‍. ചികിത്സിക്കേണ്ടത്, ലൈംഗിക വൈകൃതാനുരക്ത ഇസ്‌ലാമോഫോബിക്കുകളുടെ (Sexually Deviant Islamophobes) ഹൃദയങ്ങളില്‍ കട്ടപിടിച്ചു കിടക്കുന്ന ഇസ്‌ലാംവെറുപ്പും ലൈംഗിക വൈകൃതാസക്തിയുമാണ്. അപ്പോള്‍ മാത്രമേ അവരുടെ ഹൃദയങ്ങളില്‍ നിന്നും പ്രതിഫലിപ്പിക്കപ്പെടുന്ന വിമര്‍ശനങ്ങള്‍ക്ക് സത്യസന്ധതയും മാന്യതയുമുണ്ടാവുകയുള്ളൂ. ആരായിരുന്നു പ്രവാചകരെന്നും എന്തായിരുന്നു അവിടുത്തെ ജീവിതവും സന്ദേശവുമെന്നും നേര്‍ക്കുനേരെ ഇസ്‌ലാമിക പ്രമാണങ്ങളില്‍ നിന്നും നിഷ്പക്ഷമായി പഠിക്കാന്‍ തയ്യാറാകാതെ, വെറുപ്പു കച്ചവടത്തിന്റെ വിപണന സാധ്യതകള്‍ മാത്രം തേടി നബിചരിത്രം പരിശോധിക്കുന്നവരില്‍ നിന്നും ഒരിക്കലും സത്യസന്ധമായ ചരിത്ര വായന കേള്‍ക്കാന്‍ സാദ്ധ്യമല്ലെന്ന് ബധിരത ബാധിക്കാത്തവര്‍ക്കെല്ലാം അറിയാവുന്ന വസ്തുതയാണ്.

ഇസ്‌ലാംപൂര്‍വ കാലഘട്ടത്തില്‍ നിലനിന്നിരുന്ന സകല വിധ ലൈംഗിക അധാര്‍മികതകള്‍ക്കും നീചവൃത്തികള്‍ക്കുമെതിരെ സമൂഹത്തെ ബോധവല്‍ക്കരിക്കുകയും വിശുദ്ധമായ ലൈംഗിക ജീവിതത്തിലേക്ക് തന്റെ ജനതയെ വഴിനടത്തുകയും ചെയ്തത് പ്രവാചകരായിരുന്നു എന്ന് അദ്ധേഹത്തിന്റെ സമകാലികരില്‍ – ശത്രു മിത്ര വ്യത്യാസമില്ലാതെ- ഒരാള്‍ പോലും അംഗീകരിക്കാതിരുന്നിട്ടില്ല എന്ന ചരിത്ര വസ്തുതയെങ്കിലും മുന്നില്‍ വെച്ചുകൊണ്ടായിരുന്നു വിമര്‍ശകന്മാര്‍ നബിചരിത്രം വായിക്കേണ്ടിയിരുന്നത്. ബദ്ധശത്രുക്കള്‍ പോലും പ്രവാചകനെതിരെ വ്യഭിചാരാരോപണമോ ലൈംഗികവൈകൃതമോ ഉന്നയിക്കാതിരുന്നത് എന്തുകൊണ്ടാണെന്ന് നബിവിമര്‍ശകന്മാര്‍ ഏറെ ചിന്തിക്കേണ്ട വസ്തുതയാണ്. ഇന്ന് നബിവൈരികള്‍ ദുര്‍വ്യാഖ്യാനിച്ച് ദുഷിപ്പിച്ചവതരിപ്പിക്കുന്ന ചരിത്ര സന്ദര്‍ഭങ്ങളും ഹദീഥ് നിവേദനങ്ങളും അവര്‍ പ്രചരിപ്പിക്കുന്ന വിധമായിരുന്നുവെങ്കില്‍ എത്രമാത്രം ഒച്ചപ്പാടുകളും ബഹളകോലാഹലങ്ങളും പ്രവാചക കാലഘട്ടത്തില്‍ തന്നെ അതുമൂലം സൃഷ്ടിക്കപ്പെടുമായിരുന്നു. പക്ഷെ ഇസ്‌ലാമിക ചരിത്ര ഗ്രന്ഥങ്ങളിലൊരിടത്തും അത്തരത്തിലുള്ള ഒച്ചപ്പാടുകളെ പറ്റിയോ ബഹളകോലാഹലങ്ങളെ പറ്റിയോ ഉള്ള യാതൊരു സൂചനയും കാണുകയില്ല. നബിയെ പറ്റിയുള്ള വിവരങ്ങള്‍ അന്വേഷിച്ച ഹിര്‍ക്കിലിനെ പേലെയുള്ള ആ കാലഘട്ടത്തിലെ ചക്രവര്‍ത്തിമാരോടും രാജാക്കന്മാരോടും പരമാവധി പ്രവാചകനെ പ്രതികൂലിച്ചു വിവരങ്ങള്‍ പങ്കുവെക്കാന്‍ ശ്രമിച്ച ശത്രുക്കളിലൊരാള്‍ പോലും നബിക്കെതിരെ വ്യഭിചാരാരോപണമോ ലൈംഗികവൈകൃതമോ ഉന്നയിച്ചിട്ടില്ല എന്നതും വളരെ ശ്രദ്ധേയമായ വസ്തുതയാണ്.

മാത്രമല്ല, പ്രവാചകനെതിരെ ഈ ലൈംഗിക വൈകൃതാനുരക്ത ഇസ്‌ലാമോഫോബിക്കുകള്‍ (Sexually Deviant Islamophobes) ആരോപിക്കുന്ന വ്യഭിചാരാരോപണങ്ങളും ലൈംഗികവൈകൃതങ്ങളും ഇസ്‌ലാമില്‍ ഹദ്ദ് (ശിക്ഷ) നല്‍കപ്പെടുന്ന കുറ്റകൃത്യങ്ങളാണ്. ഇവര്‍ ആരോപിക്കും വിധം നബി(സ്വ) തന്നെ അത്തരം കുറ്റങ്ങള്‍ ചെയ്‌തെന്നിരിക്കെ എന്തു ന്യായം പറഞ്ഞാണ് ഇസ്‌ലാമിക രാഷ്ട്രങ്ങള്‍ പൗരന്മാര്‍ക്കുമേല്‍ പ്രസ്തുത തെറ്റുകള്‍ക്ക് ശിക്ഷ നടപ്പാക്കുക; കര്‍മശാസ്ത്ര ഗ്രന്ഥങ്ങള്‍ എങ്ങനെയാണ് പ്രസ്തുത തെറ്റുകള്‍ക്ക് വിധിക്കപ്പെടേണ്ട ശിക്ഷകളെ പറ്റി ചര്‍ച്ച ചെയ്യുക. എന്തിനധികം പറയണം നബി (സ്വ) തന്നെ അത്തരം കുറ്റകൃത്യങ്ങള്‍ക്ക് ഭരണാധികാരി എന്ന നിലയില്‍ ഹദ്ദ് (ശിക്ഷ) നല്‍കിയിട്ടുണ്ട്. വിമര്‍ശകര്‍ ആരോപിക്കും വിധമാണ് കാര്യങ്ങളെങ്കില്‍, നബി തന്നെ ചെയ്തിട്ടുള്ള അത്തരം കുറ്റങ്ങളുടെ പേരില്‍ മറ്റുള്ളവരെ ശിക്ഷിക്കാന്‍ എന്തു ന്യായമായിരിക്കും അദ്ധേഹത്തിനു പറയാനുണ്ടാവുക. വ്യഭിചാരത്തിന്റെ പേരില്‍ ശിക്ഷിക്കപ്പെട്ട കുറ്റവാളികളില്‍ ഒരാള്‍ പേലും പ്രവാചക ജീവിതത്തെ ചൂണ്ടി കാണിച്ച് തങ്ങളുടെ ചെയ്തികളെ ന്യായീകരിക്കാന്‍ ശ്രമിച്ചിട്ടില്ല. ചൂണ്ടിക്കാണിക്കുവാന്‍ അത്തരം ന്യായങ്ങളുണ്ടായിരുന്നെങ്കില്‍ തീര്‍ച്ചയായും അവരതു ചെയ്യുമായിരുന്നു. അത്തരം ചെയ്തികളുടെ പേരില്‍ ശിക്ഷക്കപ്പെട്ടവരില്‍ മുസ്‌ലിംകളും ജൂതന്മാരുമടക്കം നാനാവിധത്തിലുള്ളവരുണ്ടായിരുന്നിട്ടും ഒരാള്‍ പോലും അതു പറഞ്ഞില്ലെങ്കില്‍ അതിനര്‍ഥമെന്താണ്?. നബിവൈരികള്‍ ആരോപിക്കും വിധം ഒരു പുഴുക്കുത്തും നബിജീവിതത്തില്‍ നിന്നും കണ്ടെടുക്കാനാവില്ല എന്നുതന്നെ. ഒരു ചരിത്ര സംഭവത്തിലേക്കു ദൃഷ്ടി തിരിക്കാം നമുക്ക്:

”അബ്ദുല്ലാഹിബ്‌നു ഉമര്‍ (റ) നിവേദനം: വ്യഭിചാരത്തിലുള്‍പ്പെട്ട ഒരു യഹൂദനേയും യഹൂദ സ്ത്രീയേയും നബി(സ്വ)യുടെ മുന്നില്‍ ഹാജരാക്കി. നബി (സ്വ) യഹൂദന്മാരുടെ അടുക്കല്‍ ചെന്നു ചോദിച്ചു: ‘വ്യഭിചാരത്തിന്റെ ശിക്ഷ തൗറാത്തില്‍ എങ്ങനെയാണ് കാണുന്നത്?’ അവര്‍ പറഞ്ഞു: ‘അവര്‍ രണ്ടുപേരുടേയും മുഖം കറുപ്പിക്കും. കഴുതപ്പുറത്ത് കയറ്റി പുറം തിരിച്ചിരുത്തും. അവരെ പ്രദക്ഷിണം ചെയ്യിക്കും.’ നബി(സ്വ) പറഞ്ഞു: ‘നിങ്ങള്‍ തൗറാത്ത് കൊണ്ടു വരിക; നിങ്ങള്‍ സത്യം പറയുന്നവരാണെങ്കില്‍.’ അവര്‍ തൗറാത്തുമായി വന്നു വായിക്കാന്‍ തുടങ്ങി. അങ്ങനെ എറിഞ്ഞു കൊല്ലുന്ന ശിക്ഷാവിധി അടങ്ങിയ ഭാഗത്തെത്തിയപ്പോള്‍ അത് വായിച്ചിരുന്ന ചെറുപ്പക്കാരന്‍ ആ വിധികളടങ്ങിയ സൂക്തങ്ങളുടെ മേല്‍ കൈവച്ചു. അതിന്റെ അപ്പുറവും ഇപ്പുറവും വായിച്ചു. നബി(സ്വ)യുടെ കൂടെയുണ്ടായിരുന്ന അബ്ദുല്ലാഹി ബ്‌നു സലാം (റ) പറഞ്ഞു: ‘അയാളോട് കൈ മാറ്റാന്‍ ആവശ്യപ്പെടുക.’ അയാള്‍ കൈ ഉയര്‍ത്തി. അതില്‍ എറിഞ്ഞുകൊല്ലണമെന്ന വിധി അടങ്ങിയ സൂക്തമുണ്ടായിരുന്നു. നബി (സ്വ) അവര്‍ രണ്ടുപേരേയും എറിഞ്ഞുകൊല്ലാന്‍ കല്‍പ്പിക്കുകയും അങ്ങനെ എറിഞ്ഞു കൊല്ലുകയും ചെയ്തു,” (സ്വഹീഹ് മുസ്‌ലിം: 1699) ഈ ചരിത്ര സംഭവം എന്താണ് നമുക്ക് വിവരിച്ചുതരുന്നത്. വ്യഭിചാരത്തിലേര്‍പ്പെട്ട യഹൂദന്‍ ഉന്നത സ്ഥാനത്തിനുടമയായതുകൊണ്ട് അയാളെ രക്ഷപ്പെടുത്താന്‍ സ്വന്തം വേദഗ്രന്ഥത്തിലുള്ള വിധിപോലും മറച്ചു പിടിച്ച് പ്രവാചകനെ തെറ്റിദ്ധരിപ്പിക്കാന്‍ പരിശ്രമിക്കുകയാണ് യഹൂദന്മാര്‍. പക്ഷെ തൗറാത്ത് നന്നായി അറിയുന്ന, യഹൂദ പണ്ഡിതനായിരുന്ന അബ്ദുല്ലാഹി ബ്‌നു സലാം (റ) അത് കൈയ്യോടെ പിടികൂടുകയും അങ്ങനെ കുറ്റക്കാരനായ യഹൂദനെ രക്ഷിക്കാന്‍ മറ്റു വഴികളില്ലാതെ യഹൂദന്മാര്‍ക്ക് തൗറാത്തിലെ വിധി നടപ്പാക്കേണ്ടി വരികയും ചെയ്യുന്നു. നബിവൈരികള്‍ ആരോപിക്കും വിധം പ്രവാചക ജീവിതം തന്നെ അവിഹിത ബന്ധങ്ങള്‍ നിറഞ്ഞതോ ലൈംഗിക വൈകൃതങ്ങള്‍ പേറിയതോ ആയിരുന്നെങ്കില്‍ യഹൂദന്മാര്‍ നബിയുടെ നിയമനടപടിക്കെതിരെ ആദ്യമായി ഉന്നയിക്കുന്ന തുരുപ്പുചീട്ട് അതാകുമായിരുന്നു. താങ്കള്‍ തന്നെ ചെയ്തുകൊണ്ടിരിക്കുന്ന തെറ്റ് ഞങ്ങളുടെ കൂട്ടത്തില്‍ ഒരാള്‍ ചെയ്‌തെന്നിരിക്കെ അതിനെതിരെ ശിക്ഷാ നടപടി സ്വീകരിക്കാന്‍ താങ്കള്‍ക്കെന്തധികാരം എന്നവര്‍ ചോദിക്കുമായിരുന്നു. പക്ഷെ ഒരു യഹൂദനും അതിനു സാധിച്ചില്ല. നിശബ്ദരായി തൗറാത്തിലെ വിധി നടപ്പാക്കേണ്ടി വന്നു അവര്‍ക്ക്. എന്തുകൊണ്ട്!? ഉത്തരം വളരെ ലളിതമാണ്. നബിവിരോധികള്‍ കച്ചകെട്ടിയുണ്ടാക്കിയ ഈ കളവുകള്‍ അവര്‍ക്കറിയില്ലായിരുന്നു. സ്വന്തം വേദഗ്രന്ഥത്തില്‍ വരെ കളവു കാണിക്കുന്ന യഹൂദനെ പോലും തോല്‍പ്പിക്കുന്ന ഈ പുതിയ കാല മഹാ കള്ളന്മാരെ അന്ന് ആ യഹൂദന്മാര്‍ പരിചയപ്പെട്ടിട്ടില്ലായിരുന്നു.

മാത്രമല്ല, ലൈംഗിക വിശുദ്ധിയുമായി ബന്ധപ്പെട്ട നബിയുടെ ഉപദേശങ്ങള്‍ അതീവ കാര്‍ക്കശ്യ സ്വഭാവമുള്ളവയാണെന്ന് അതില്‍ നിന്നും അല്‍പമെങ്കിലും കേട്ടിട്ടുള്ളവര്‍ക്കെല്ലാം അറിയാവുന്ന വസ്തുതയാണ്. ഒരു സ്ത്രീയും പുരുഷനും ഒറ്റക്കായാല്‍ മൂന്നാമന്‍ പിശാചാണ്, അന്യ സ്ത്രീയുടെ നേര്‍ക്കുള്ള രണ്ടാമത്തെ നേട്ടം പുരുഷനു നിഷിദ്ധമാണ്, അന്യസ്ത്രീയെ സ്പര്‍ശിക്കുന്നതിനേക്കാള്‍ പുരുഷനു നല്ലത് തലയില്‍ ഇരുമ്പാണി തറക്കുന്നതാണ്, മനുഷ്യരെ ഏറ്റവുമധികം നരകാര്‍ഹരാക്കുന്നത് നാവിന്റെയും ലൈംഗികാവയവങ്ങളുടെയും വിശുദ്ധിയില്ലായ്മയാണ്, ആര്‍ ലൈംഗികാവയവങ്ങളെ കാത്തു സൂക്ഷിക്കുന്നുവേ അവനു ഞാന്‍ സ്വര്‍ഗം ജാമ്യം നില്‍ക്കുന്നു, ലൈംഗിക വിശുദ്ധി കാത്തു സൂക്ഷിക്കുവാനായി വിവാഹം ചെയ്യാന്‍ കഴിവില്ലാത്തവന്‍ വ്രതമനുഷ്ഠിക്കട്ടെ, വ്രതം ലൈംഗിക വിശുദ്ധിയുടെ പരിചയാണ്, അന്യസ്ത്രീയാല്‍ ആകര്‍ഷിക്കപ്പെട്ടവന്‍ സ്വന്തം ഭാര്യയുടെ അരികില്‍ പേകട്ടെ കാരണം അവള്‍ക്കുള്ളതു തന്നെയാണ് ഇവള്‍ക്കുള്ളതും, വൃദ്ധനായ വ്യഭിചാരിയോട് ദൈവം മുഖം തിരിക്കും, അത്തരക്കാര്‍ക്ക് ദൈവത്തിന്റെ കാരുണ്യ കടാക്ഷം ലഭിക്കില്ല, അയല്‍ വാസിയുടെ ഭാര്യയെ വ്യഭിചരിക്കല്‍ വന്‍ പാപമാകുന്നു, ദമ്പധിമാര്‍ കിടപ്പറ രഹസ്യങ്ങള്‍ പരസ്യപ്പെടുത്തുന്നത് പാപമാണ്, കുടുംബത്തെ മ്ലേഛ വൃത്തിക്കു കൂട്ടികൊടുക്കുന്നവന്‍ സ്വര്‍ഗത്തില്‍ പ്രവേശിക്കുകയില്ല, പരപുരുഷന്മാരെ വശീകരിക്കാനായി സുഖന്ധമുപയോഗിക്കുന്ന സ്ത്രീകള്‍ നരകത്തിലാണ്; അവളെ നോക്കുന്ന കണ്ണുകളും നരകത്തിലാണ്, സ്വവര്‍ഗ സംഭോഗിയെ അല്ലാഹു ശപിച്ചിരിക്കുന്നു, മൃഗരതിയിലേര്‍പ്പെടുന്നവനെയും മൃഗത്തെയും വധിക്കണം, വിവാഹിതനായ വ്യഭിചാരിയെ ചാട്ടവാറുകൊണ്ട് നൂറടിയടിക്കണം; എറിഞ്ഞുകൊല്ലണം, അവിവാഹിതനായ വ്യഭിചാരിയെ ചാട്ടവാറുകൊണ്ട് നൂറടിയടിക്കണം; നാടുകടത്തണം തുടങ്ങി പാരാവാരം കണക്കെ രേഖപ്പെടുത്തപ്പെട്ടിരിക്കുന്നു ലൈംഗിക വിശുദ്ധിയുടെ നബിപാഠങ്ങള്‍. വിമര്‍ശകര്‍ ആരോപിക്കും വിധം അവിഹിത ബന്ധങ്ങളും ലൈംഗികവൈകൃതങ്ങളും നിറഞ്ഞതായിരുന്നു നബിജീവിതമെങ്കില്‍ പിന്നെ എങ്ങനെയാണ് പ്രവാചകന്‍ തന്റെ ജനതക്ക് ഇത്രമാത്രം ലൈംഗിക വിശുദ്ധിയുടെ ജീവിത പാഠങ്ങള്‍ പകര്‍ന്നു നല്‍കിയത്. പ്രവാചകനില്‍ നിന്നും വിശുദ്ധിയുടെ ജീവിത പാഠങ്ങള്‍ കേട്ട് സ്വജീവിതത്തെ അതിനനുസൃതമായി ചിട്ടപ്പെടുത്താന്‍ അങ്ങേയറ്റത്തെ ത്യാഗ പരിശ്രമങ്ങള്‍ നടത്തിയ ഒരു ജനത, തങ്ങള്‍ക്കു നല്‍കിയ ഉപദേശങ്ങള്‍ക്കു കടക വിരുദ്ധമായി ജീവിക്കുന്ന ഉപദേശകനെ ജീവനേക്കാള്‍ അധികമായി സ്‌നേഹിച്ചുവെന്ന് കരുതാന്‍ ബുദ്ധിഭ്രമം പിടിപെടുക തന്നെ വേണം. സ്വന്തം ജീവനും ജീവിതവും സമ്പാദ്യവും സന്തോഷവുമെല്ലാം ആ ഉപദേശകന്റെ കാല്‍കീഴില്‍ കൊണ്ടു സമര്‍പ്പിക്കുവാന്‍ ആ ജനത തയ്യാറായത്, കേട്ട ഉപദേശങ്ങളെക്കാള്‍ മഹത്തരമായ ജീവിതത്തെ നേരില്‍ കണ്ടതുകൊണ്ടാണെന്ന് തിരിച്ചറിയാന്‍ സത്യസന്ധതക്ക് ജീവിതത്തില്‍ ചെറിയ ഒരിടം കൊടുത്താല്‍ മാത്രം മതി.

ഇമാം അഹ്‌മദ്‌ മുസ്‌നദിലും (5/256) ത്വബ്‌റാനി മുഅ്ജമുല്‍ കബീറിലും (8/162, 183) ബൈഹക്വി ശുഅ്ബുല്‍ ഈമാനിലും (4/362(5415)) ഉദ്ദരിച്ച ഒരു ഹദീഥ് കാണുക: ”അബൂ ഉമാമഃ(റ)യില്‍ നിന്നും നിവേദനം: ഒരു യുവാവ് നബി(സ്വ)യുടെ അടുക്കലേക്ക് വന്നു. എന്നിട്ടവന്‍ പറഞ്ഞു: അല്ലാഹുവിന്റെ ദൂതരെ എനിക്കു വ്യഭിചരിക്കാന്‍ അനുവാദം നല്‍കിയാലും. അപ്പോള്‍ ജനങ്ങള്‍ അയാളുടെ അടുത്തു ചെന്ന് അയാളെ തടഞ്ഞുകൊണ്ട് മിണ്ടാതിരിക്കാന്‍ ആവശ്യപ്പെട്ടു. അപ്പോള്‍ നബി(സ്വ) പറഞ്ഞു: അദ്ധേഹത്തെ എന്റടുത്തേക്ക് വിടൂ. അപ്പോള്‍ ആ യുവാവ് നബി(സ്വ)യുടെ അരികിലേക്ക് വന്നിരുന്നു. എന്നിട്ട് നബി(സ്വ) അയാളോട് ചോദിച്ചു:

‘നിന്റെ മാതാവിനു നീ അത് (വ്യഭിചാരം) ഇഷ്ടപ്പെടുന്നുണ്ടോ? അയാള്‍ പറഞ്ഞു: ഇല്ല. അല്ലാഹുവാണേ, അല്ലാഹു എന്നെ താങ്കള്‍ പ്രായശ്ചിത്തമാക്കട്ടെ. അപ്പോള്‍ നബി(സ്വ) പറഞ്ഞു: ജനങ്ങളും അവരുടെ മാതാക്കള്‍ക്ക് അത് (വ്യഭിചാരം) ഇഷ്ടപ്പെടുന്നില്ല.’ ‘നബി(സ്വ) ചോദിച്ചു: നിന്റെ മകള്‍ക്ക് നീ അത് (വ്യഭിചാരം) ഇഷ്ടപ്പെടുന്നുണ്ടോ? അയാള്‍ പറഞ്ഞു: ഇല്ല. അല്ലാഹുവാണേ, അല്ലാഹു എന്നെ താങ്കള്‍ പ്രായശ്ചിത്തമാക്കട്ടെ. അപ്പോള്‍ നബി(സ്വ) പറഞ്ഞു: ജനങ്ങളും അവരുടെ പുത്രിമാര്‍ക്ക് അത് (വ്യഭിചാരം) ഇഷ്ടപ്പെടുന്നില്ല.’ ‘എന്നിട്ട് വീണ്ടും നബി(സ്വ) അയാളോട് ചോദിച്ചു: നിന്റെ സഹോദരിക്ക് നീ അത് (വ്യഭിചാരം) ഇഷ്ടപ്പെടുന്നുണ്ടോ? അയാള്‍ പറഞ്ഞു: ഇല്ല. അല്ലാഹുവാണേ, അല്ലാഹു എന്നെ താങ്കള്‍ പ്രായശ്ചിത്തമാക്കട്ടെ. അപ്പോള്‍ നബി(സ്വ) പറഞ്ഞു: ജനങ്ങളും അവരുടെ സഹോദരിമാര്‍ക്ക് അത് (വ്യഭിചാരം) ഇഷ്ടപ്പെടുന്നില്ല.’

‘നബി(സ്വ) വീണ്ടും ചോദിച്ചു: നിന്റെ അമ്മായിക്ക് നീ അത് (വ്യഭിചാരം) ഇഷ്ടപ്പെടുന്നുണ്ടോ? അയാള്‍ പറഞ്ഞു: ഇല്ല. അല്ലാഹുവാണേ, അല്ലാഹു എന്നെ താങ്കള്‍ പ്രായശ്ചിത്തമാക്കട്ടെ. അപ്പോള്‍ നബി(സ്വ) പറഞ്ഞു: ജനങ്ങളും അവരുടെ അമ്മായിമാര്‍ക്കും അത് (വ്യഭിചാരം) ഇഷ്ടപ്പെടുന്നില്ല.’

‘വീണ്ടും അവിടുന്ന് ചോദിച്ചു: നിന്റെ മാതൃസഹോദരിക്ക് നീ അത് (വ്യഭിചാരം) ഇഷ്ടപ്പെടുന്നുണ്ടോ? അയാള്‍ പറഞ്ഞു: ഇല്ല. അല്ലാഹുവാണേ, അല്ലാഹു എന്നെ താങ്കള്‍ പ്രായശ്ചിത്തമാക്കട്ടെ. അപ്പോള്‍ നബി(സ്വ) പറഞ്ഞു: ജനങ്ങളും അവരുടെ മാതൃസഹോദരിമാര്‍ക്കും അത് (വ്യഭിചാരം) ഇഷ്ടപ്പെടുന്നില്ല.’

എന്നിട്ട് നബി (സ്വ) അദ്ധേഹത്തിന്റെ കൈകള്‍ അയാളുടെ ശരീരത്തില്‍ വെച്ചുകൊണ്ട് പ്രാര്‍ഥിച്ചു: ‘അല്ലാഹുവേ ഇദ്ധേഹത്തിന്റെ പാപം നീപൊറുത്തുകൊടുക്കുകയും ഹൃദയം ശുദ്ധീകരിക്കുകയും ലൈംഗികാവയവങ്ങളെ സംരക്ഷിക്കുകയും ചെയ്യേണമേ.’ അതിനു ശേഷം ആ യുവാവ് ഒന്നിലേക്കും (ലൈംഗികമായ ദുര്‍വൃത്തിയിലേക്ക്) തിരിഞ്ഞു നോക്കിയിരുന്നില്ല.”

വ്യഭിചാരത്തിന് അനുവാദം നല്‍കണമെന്നാവശ്യപ്പെട്ടുകൊണ്ട് പ്രവാചക സന്നിധിയിലേക്ക് കടന്നു വന്ന ഒരു യുവാവിനെ മനഃശാസ്ത്രപരമായി സമീപിക്കുകയാണിവിടെ പ്രവാചകന്‍ (സ്വ). അവനോടായി അദ്ധേഹം പറയുകയാണ്, നിന്റെ മാതാവും മകളും സഹോദരിയും അമ്മായിയും മാതൃസഹോദരിയുമൊക്കെ വ്യഭിചരിക്കുന്നത് നീ ഇഷ്ടപ്പെടുന്നില്ല. എന്നാല്‍ നീ വ്യഭിചരിക്കുന്ന സ്ത്രീയാകട്ടെ തീര്‍ച്ചയായും മറ്റൊരുവന്റെ മാതാവോ മകളോ സഹോദരിയോ അമ്മായിയോ മാതൃസഹോദരിയോ ആയിരിക്കും. അവനും നിന്നെപ്പോലെ സ്വന്തം മാതാവും മകളും സഹോദരിയും അമ്മായിയും മാതൃസഹോദരിയുമൊക്കെ വ്യഭിചരിക്കുന്നത് ഇഷ്ടപ്പെടുന്നവനല്ല. സ്വന്തക്കാരായ സ്ത്രീകള്‍ വ്യഭിചരിക്കുന്നത് വെറുക്കുന്ന നീ മറ്റുള്ളവരുടെ സ്വന്തക്കാരായ സ്ത്രീകളെ വ്യഭിചരിക്കാനനുവാദം ചോദിക്കുന്നത് എത്രമാത്രം അക്രമമാണ്. അയാള്‍ക്ക് ബോധോദയമുണ്ടായി ആ നികൃഷ്ഠമായ ആഗ്രഹം വെടിഞ്ഞ് വിശുദ്ധമായ ജീവിതം അയാള്‍ തിരഞ്ഞെടുത്തു. ഇനി നാം ചിന്തിക്കുക, നബിവൈരികള്‍ ആരോപിക്കും വിധം പ്രവാചക ജീവിതം തന്നെ വ്യഭിചാരബന്ധങ്ങള്‍ നിറഞ്ഞതോ ലൈംഗികവൈകൃതങ്ങള്‍ പോറിയതോ ആയിരുന്നെങ്കില്‍ ആ യുവാവിനെ നബി എങ്ങനെയാണ് ഉപദേശിക്കുക. അയാള്‍ അത് എങ്ങനെയാണ് ഉള്‍കൊള്ളുക. എന്നെ നന്മയും സദാചാരവും ഉപദേശിക്കാന്‍ താങ്കള്‍ക്കെന്തര്‍ഹത എന്നയാള്‍ തിരിച്ചു ചോദിക്കുമായിരുന്നില്ലേ.? എന്തുകൊണ്ടയാള്‍ അതു ചോദിച്ചില്ല.! കാരണം അയാള്‍ നബിയെ പഠിച്ചതും അറിഞ്ഞതും അനുഭവങ്ങളില്‍ നിന്നും നേര്‍ക്കുനേരെയായിരുന്നു; അല്ലാതെ എതെങ്കിലും നവനാസ്തിക ഞരമ്പുരോഗികളില്‍ നിന്നോ മിഷണറി നുണഫാക്ടറികളില്‍ നിന്നോ ഷോവനിസ്റ്റ് ചാണക പുരകളില്‍ നിന്നോ ആയിരുന്നില്ല.

ഇത്രയും നാം പറഞ്ഞത്, ലൈംഗികവൈകൃതാനുരക്ത ഇസ്‌ലാമോഫോബിക്കുകള്‍ (Sexually Deviant Islamophobes) നബിജീവിതത്തിനെതിരെ ഉന്നയിച്ചുകൊണ്ടിരിക്കുന്ന ഏതൊരു ആരോപണത്തെയും പഠനവിധേയമാക്കുന്നതിനു മുമ്പ് ചിന്തയിലേക്ക് കൊണ്ടുവരേണ്ട ആമുഖ ചിന്താധാരകളെ പറ്റിയാണ്. കല്ലുവെച്ച കള്ളങ്ങള്‍ മാത്രമാണ് ഏതൊരു നബിനിന്ദാ പ്രമേയങ്ങളുമെന്ന ബോധ്യം പകര്‍ന്നു നല്‍കാന്‍ പര്യാപ്തമാണ് ആ വിശുദ്ധ ജീവിതത്തെപ്പറ്റിയുള്ള പ്രാഥമിക ചരിത്രപാഠങ്ങള്‍ പോലും. ഇനി നമുക്കു പരിശോധിക്കുവാനുള്ളത് ഉന്നയിക്കപ്പെട്ട ആരോപണങ്ങളുടെ നിജസ്ഥിതികളാണ്. അക്കമിട്ടു നമുക്കതു പരിശോദിക്കാം.

1, നബിയുടെ മാതൃസഹോദരി ഖൗല ബിന്‍ത് ഹകീം എന്ന സ്ത്രീ സ്വന്തം ശരീരം നബിക്കു ദാനം നല്‍കുകയുണ്ടായി എന്നും അതിനെതിരെ സ്വപത്‌നി ആഇശ പോലും നബിയോട് ‘ഒരു സ്വതന്ത്ര സ്ത്രീ സ്വദേഹം ദാനം ചെയ്യുകയോ’ എന്ന് പ്രതികരിച്ചുവെന്നും ഹദീഥ് ഗ്രന്ഥങ്ങളില്‍ കാണാം. എന്നാല്‍ നബി ഒരിക്കലും അതിനെ മോശമായി കണ്ടിരുന്നില്ല എന്നും ഹദീഥ് ഗ്രന്ഥങ്ങള്‍ തന്നെ മനസ്സിലാക്കി തരുന്നുണ്ട്. സ്ത്രീകളെ കൊണ്ട് സ്വശരീരങ്ങള്‍ തനിക്കായി ദാനം ചെയ്യാന്‍ പ്രേരിപ്പിക്കുകയും സ്വന്തം മാതൃസഹോദരിയെ പോലും അതില്‍ നിന്നും ഒഴിവാക്കാതിരിക്കുകയും ചെയ്ത പ്രവാചക നടപടിയെ അല്‍പമെങ്കിലും ധാര്‍മികബോധമുള്ളവര്‍ക്ക് അംഗീകരിക്കാനാകുമോ?

നാം നിരൂപണവിധേയമാക്കേണ്ട വിമര്‍ശനങ്ങള്‍ ഇവയാണ്:

a) എന്താണ് ശരീരദാനം കൊണ്ട് ഉദ്ദേശിക്കുന്നത്? വിമര്‍ശകന്മാര്‍ ജല്‍പ്പിക്കും പ്രകാരം സ്വന്തം ശരീരത്തെ ഒരു സ്ത്രീ ലൈംഗികാസ്വാദനത്തിനായി നബി(സ്വ)ക്ക് സമര്‍പ്പിക്കുന്ന ഏര്‍പ്പാടാണോ അത്. അല്ലേ അല്ല.!! നാസ്തിക/മിഷനറി/ഷോവനിസ്റ്റ് ഞരമ്പുരോഗികളുടെ ശിഫക്കായി (ശമനം) നമുക്കതു മന്ത്രിച്ചു കൊടുക്കാം.

വിശുദ്ധ ക്വുര്‍ആന്‍ പറഞ്ഞു: ”സത്യവിശ്വാസിയായ സ്ത്രീ സ്വന്തത്തെ പ്രവാചകന് ദാനം ചെയ്യുകയും അവളെ വിവാഹം കഴിക്കാനുദ്ദേശിക്കുകയുമാണെങ്കില്‍ അതിനും വിരോധമില്ല.” (ക്വുര്‍ആന്‍: 33: 50)

‘വിവാഹം കഴിക്കാനുദ്ദേശിക്കുകയുമാണെങ്കില്‍’ എന്നാണ് ക്വുര്‍ആന്‍ ഇവിടെ പരാമര്‍ശിച്ചിരിക്കുന്നത്. അഥവാ വിമര്‍ശകന്മാര്‍ ആരോപിക്കും വിധം ലൈംഗികാസ്വാദനത്തിനായി നബി(സ്വ)ക്ക് ഒരു സ്ത്രീ സ്വശരീരത്തെ സമര്‍പ്പിക്കുന്ന ഏര്‍പ്പാടല്ല ഇത്. മറിച്ച് വിവാഹത്തിനായി സ്വന്തത്തെ സമര്‍പിക്കലാണ്. അഥവാ സാധാരണ വിവാഹത്തില്‍ നിന്നും വ്യത്യാസമായി ഇവിടെ സ്ത്രീ മഹ്ര്‍ ഒഴിവാക്കും. ‘ഹിബത്ത്’ വിവാഹം എന്നാണ് ഇതറിയപ്പെടുന്നത്. ഇത് പ്രവാചകനു മാത്രം ബാധകമായ നിയമമാണ്. എല്ലാ പണ്ഡിതന്മാരും അതു വ്യക്തമാക്കിയിട്ടുണ്ട്.

”സത്യവിശ്വാസിനിയായ ഒരു സ്ത്രീ സ്വദേഹം നബിക്ക് ദാനം ചെയ്യുന്ന പക്ഷം നബി അവളെ വിവാഹം കഴിക്കാന്‍ ഉദ്ദേശിക്കുന്നെങ്കില്‍ അതും (അനുവദിച്ചിരിക്കുന്നു.)” അഥവാ മഹ്ര്‍ ഇല്ലാതെ വിവാഹം ചെയ്യാന്‍ ആവശ്യപ്പെട്ടാല്‍. ഹിബത്ത് (സ്വദേഹം ദാനം ചെയ്യുന്നു) എന്ന പദം ഉപയോഗിച്ച് മഹ്ര്‍ ഇല്ലാത്ത നിക്കാഹാണ് ഇവിടെ ഉദ്ദേശം.” (തഫ്‌സീറുല്‍ ജലാലൈനി: 33:50 ന്റെ വ്യാഖ്യാനം)

”ഇബ്‌നു അബ്ബാസ് (റ) പറഞ്ഞു: അല്ലാഹുവിന്റെ ദൂതന്റെ പത്‌നിമാരില്‍ സാധാരണ ചടങ്ങിലൂടെയൊ വലങ്കൈ ഉടമപ്പെടുത്തിയതോ അല്ലാതെ ആരും ഉണ്ടായിരുന്നില്ല. ഹിബത്ത് (സ്വദേഹം ദാനം ചെയ്യുന്നു) എന്ന പദം ഉപയോഗിച്ച് മഹ്ര്‍ ഇല്ലാത്ത നിക്കാഹിന് തയ്യാറായ ഒരു സ്ത്രീയേയും പ്രവാചകന്‍ (സ) സ്വീകരിക്കരിച്ചിട്ടില്ല. അവരില്‍ പെട്ട ആരും പ്രവാചകന് (സ) ഉണ്ടായിരുന്നില്ല.” (തഫ്‌സീറുല്‍ കുര്‍തുബി: 33:50 ന്റെ വ്യാഖ്യാനം)

”മഹ്ര്‍ കൂടാതെ, എന്നെ വിവാഹം ചെയ്യണോ എന്ന വിധി താങ്കള്‍ക്ക് ഞാന്‍ ഇഷ്ടദാനം ചെയ്യുന്നു.” (അല്‍ മുഫ്ഹിം: ഇമാം കുര്‍ത്തുബി: പേജ്: 4/128)

”ഹിബത്ത് (സ്വദേഹം ദാനം ചെയ്യുന്നു) എന്ന പദം ഉപയോഗിച്ച് മഹ്ര്‍ ഇല്ലാത്ത നിക്കാഹിന് തയ്യാറായ സ്ത്രീകളെ വിവാഹം ചെയ്യല്‍ പ്രവാചകന് അനുവദിക്കപ്പെട്ടിട്ടും പ്രവാചകന്‍ (സ) അതു ചെയ്തില്ല എന്നാണ് ഇബ്‌നു അബ്ബാസ് (റ) പറയുന്നത്.” (ഫത്ഹുല്‍ ബാരി: 8:526)

”ഞാന്‍ എന്നെ താങ്കള്‍ക്കു മുമ്പില്‍ വിവാഹത്തിനായി ഇഷ്ടദാനം ചെയ്യുന്നു’ എന്ന വാചകത്തില്‍ ഒരു മുള്വാഫ് (Possession) (ഭാഷാ പരമായ ഭംഗിക്കായി) വിട്ടുകളഞ്ഞതാണ്. യഥാർത്ഥത്തില്‍ വാചകത്തിന്റെ വിവക്ഷ ഇപ്രകാരമാണ്: ‘ഞാന്‍ എന്നെ അഥവാ എന്റെ വിവാഹ കാര്യത്തെ താങ്കള്‍ക്കു മുമ്പില്‍ ഇഷ്ടദാനം ചെയ്യുന്നു’. കാരണം ഒരു സ്വതന്ത്ര്യ സ്ത്രീ ഉടമപ്പെടുത്തപ്പെടുകയോ ദാനം ചെയ്യപ്പെടുകയോ ഇല്ലല്ലോ. മഹ്ര്‍ ഇല്ലാതെ തന്നെ താങ്കളെ വിവാഹം ചെയ്യാന്‍ ഞാന്‍ തയ്യാറാണ് എന്നാണ് ആ സ്ത്രീ പറഞ്ഞതിന്റെ വിവക്ഷ.” (ഫത്ഹുല്‍ ബാരി: 9/112, ഫത്ഹുല്‍ മുന്‍ഇം: 5/540)

നബി(സ്വ)ക്ക് ലൈംഗികാസ്വാദനത്തിനായി സ്ത്രീകള്‍ അവരുടെ ശരീരം ദാനം ചെയ്യുന്ന ഏര്‍പാടല്ല ‘ഹിബത്ത്’. മറിച്ച് മഹ്ര്‍ (വിവാഹ മൂല്യം) ഇല്ലാതെ തന്നെ അവരെ വിവാഹം ചെയ്യാനുള്ള അവകാശം പ്രവാചകനു സമര്‍പിച്ചുകൊണ്ട് നടത്തപ്പെടുന്ന വിവാഹ രീതിയാണത്. നബി(സ്വ)യുടെ പത്‌നിപദം ആഗ്രഹിച്ചുകൊണ്ടാണവര്‍ അപ്രകാരമുള്ള വിവാഹത്തിനു സന്നദ്ധത അറിയിക്കുന്നത്. എന്നാല്‍ അത്തരത്തിലുള്ള ‘ഹിബത്ത്’ വിവാഹം നബി (സ്വ) ഒരിക്കലും ചെയ്തിട്ടില്ലെന്നാണ് പ്രവാചക ശിഷ്യനും സന്നദ്ധസഹചാരിയുമായ ഇബ്‌നു അബ്ബാസ് (റ) പറയുന്നത്. എങ്കില്‍ എന്തുകൊണ്ടാണ് പ്രവാചക പത്‌നി ആഇശ (റ) ‘ഒരു സ്വതന്ത്ര സ്ത്രീ സ്വദേഹം ദാനം ചെയ്യുകയോ’?! എന്ന് പ്രവാചകനോട് പ്രതികരിച്ചത്? ഒരു സ്വതന്ത്ര സ്ത്രീയുടെ കുലീനതക്കു യോജിച്ച കാര്യമായി അവര്‍ക്കതിനെ ഉള്‍കൊള്ളാന്‍ പ്രയാസം തോന്നിയതുകൊണ്ടു മാത്രമാണ് അവരപ്രകാരം നബി(സ്വ)യോടു പ്രതികരിച്ചത്. കാരണം മഹ്ര്‍ എന്നത് സ്ത്രീയുടെ കുലീനതയുടെ ഭാഗമായാണ് അന്നും ഇന്നും അറേബ്യന്‍ സ്ത്രീകള്‍ കണക്കാക്കി പോരുന്നത്. അതിനാല്‍ സ്വാഭാവികമായും ഉണ്ടായ ഒരാശ്ചര്യം പ്രകടിപ്പിക്കുക മാത്രമായിരുന്നു അവര്‍. എന്നാല്‍ നബി (സ്വ) അത് മോശമായി കാണാതിരുന്നത് ‘ഹിബത്ത്’ വിവാഹത്തിന് സന്നദ്ധരായ സ്ത്രീകള്‍ അതുകൊണ്ടാഗ്രഹിച്ചത് പ്രവാചകന്റെ പത്‌നിപദം മാത്രമാണെന്ന് അവിടുത്തേക്ക് അറിയാവുന്നതു കൊണ്ടായിരുന്നു. ആദര്‍ശ പ്രണയമാണ് ശരീര പ്രണയമല്ല അതിനു പിറകിലുള്ള ചേദോവികാരമെന്ന് പ്രവാചകന് അറിയാമായിരുന്നു.

b) ‘നബിയുടെ മാതൃസഹോദരി ഖൗല ബിന്‍ത് ഹകീം എന്ന സ്ത്രീ സ്വന്തം ശരീരം നബിക്കു ദാനം നല്‍കുകയുണ്ടായി’. വിമര്‍ശനമിതാണ്; സ്വന്തം മാതൃസഹോദരിയെ കൊണ്ടു പോലും ശരീരം ദാനം (ഹിബത്ത്) ചെയ്യപ്പിച്ചു പ്രവാചകന്‍.!! ആരായിരുന്നു വാസ്തവത്തില്‍ ഖൗല ബിന്‍ത് ഹകീം എന്ന സ്ത്രീ. നബിയുടെ മാതൃസഹോദരിയായിരുന്നോ?. അല്ല. മറിച്ച് അവര്‍ പ്രസിദ്ധ താബിഈയായ (പ്രവാചകാനുചരന്മാരുടെ ശിഷ്യന്‍) സഈദിബ്‌നുല്‍ മുസ്വയ്യിബി(റ)ന്റെ മാതൃസഹോദരിയായിരുന്നു. പിന്നെ എന്തുകൊണ്ടാണവര്‍ നബിയുടെ മാതൃസഹോദരിയാണെന്ന തെറ്റിദ്ധാരണയുണ്ടായി?. ആദ്യം നാം പരിശോധിക്കുന്നത് അവര്‍ സഈദിബ്‌നുല്‍ മുസ്വയ്യിബി(റ)ന്റെ മാതൃസഹോദരിയായിരുന്നു എന്ന വസ്തുതയാണ്. അതോടൊപ്പം, എന്തുകൊണ്ടാണവര്‍ നബിയുടെ മാതൃസഹോദരിയാണെന്ന തെറ്റിദ്ധാരണയുണ്ടായതെന്നും പരിശോധിക്കാം.

”അബുല്‍ വലീദ് അത്ത്വയാലിസി പറഞ്ഞു: ശുഅ്ബ നമ്മോട് പറഞ്ഞു: അതാഅ് അല്‍ ഖുറാസാനിയില്‍ നിന്ന്, അദ്ദേഹം പറഞ്ഞു: സഈദിബ്‌നുല്‍ മുസ്വയ്യിബ് പറയുന്നതായി ഞാന്‍ കേട്ടു: ‘എന്റെ മാതൃസഹോദരി’ ഖൗല ബിന്‍ത്ത് ഹകീം അസ്സുലമിയ്യ അല്ലാഹുവിന്റെ തിരുദൂതരോട് സ്ത്രീകളുടെ സ്‌കലനത്തെ സംബന്ധിച്ച് ചോദിച്ചു. അപ്പോള്‍ അത് സംഭവിച്ചാല്‍ കുളിക്കുവാന്‍ പ്രവാചകന്‍ കല്‍പ്പന നല്‍കി.” (സുനനു ദാരിമി: 762)

എന്നാല്‍ ഇതേ ഹദീഥിന്റെ ചില നിവേദനങ്ങളില്‍ പ്രവാചകന്റെ മാതൃസഹോദരിയാണ് ഖൗല ബിന്‍ത്ത് ഹകീം എന്ന് വന്നത് ഹദീസ് നിവേദകന് സംഭവിച്ച ഓര്‍മ പിശക് മാത്രമാണ്. നിവേദക പരമ്പരയിലെ ശുഅ്ബ എന്ന നിവേദകനാണ് ഈ ഓര്‍മ പിശക് സംഭവിച്ചത് എന്നും ചില പണ്ഡിതര്‍ അഭിപ്രായപ്പെടുന്നു. (https://al-maktaba.org/book/31621/19145#p35)

നിവേദക പരമ്പരയിലെ ‘അതാഅ് അല്‍ ഖുറാസാനി’ യില്‍ നിന്ന് പലപോഴും ഹദീഥിന്റെ പദപ്രയോഗങ്ങളില്‍ വഹ്‌മ് അഥവാ ധാരണ പിശക് സംഭവിക്കാറുണ്ട്. (സിയറു അഅ്‌ലാമിന്നുബലാഅ്: 6/140, തക്‌രീബു തഹ്ദീബ്: 1/392, തഹ്ദീബുല്‍ കമാല്‍: 20/107)

അത്തരത്തില്‍ സംഭവിച്ച ധാരണ പിശകാണ്, സഈദിബ്‌നുല്‍ മുസ്വയ്യിബ്, തന്റെ മാതൃസഹോദരിയാണ് ഖൗല ബിന്‍ത്ത് ഹകീം എന്ന് പറഞ്ഞത് പ്രവാചകന്റെ മാതൃസഹോദരിയാണ് ഖൗല ബിന്‍ത്ത് ഹകീം എന്ന് തെറ്റായി മനസ്സിലാക്കിയത്. ഇസ്ഹാകിബ്‌നു റാഹൂയ പറഞ്ഞു: ഖൗല ബിന്‍ത്ത് ഹകീം വഴി അല്ലാഹുവിന്റെ ദൂതനില്‍ നിന്ന് ഉദ്ധരിക്കപ്പെട്ട ഹദീഥ്. അബൂ യഅ്കൂബ് പറഞ്ഞു: ഖൗല ബിന്‍ത്ത് ഹകീം, സഈദിബ്‌നു മുസ്വയ്യിബിന്റെ മാതൃസഹോദരിമാരില്‍ ഒരാളാണ്. (മുസ്‌നദു ഇസ്ഹാകിബ്‌നു റാഹൂയ: 5:44: ഹദീസ് നമ്പര്‍: 2147)

ഇനി നാം പരിശോധിക്കുന്നത് നബി(സ്വ)യുടെ മാതാവ് ആമിനയുടേയും ഖൗല ബിന്‍ത്ത് ഹകീമിന്റേയും കുടുംബ പരമ്പരകളാണ്. അപ്പോള്‍ വ്യക്തമാകുന്ന കാര്യമാണ് ഖൗല ബിന്‍ത്ത് ഹകീം നബി(സ്വ)യുടെ മാതൃസഹോദരിയല്ലെന്ന വസ്തുത.

നബി(സ്വ)യുടെ മാതാവ് ആമിനയുടെ കുടുംബ പരമ്പര കാണുക:

സഹ്‌റയുടെ പുത്രന്‍ അബ്ദു മനാഫിന്റെ പുത്രന്‍ വഹബിന്റെ പുത്രി ആമിന. അബ്ദു യഗൂസിബ്‌നു വഹബ് എന്ന ഒരു മാതൃസഹോദരന്‍ മാത്രമേ പ്രവാചകന് ഉണ്ടായിരുന്നുള്ളു. (അസ്സികാത്ത്: ഇബ്‌നു ഹിബ്ബാന്‍: 1/26)

ഖൗല ബിന്‍ത്ത് ഹകീമിന്റെ കുടുംബ പരമ്പര കാണുക: സുലൈമിന്റെ പുത്രന്‍ ബഹ്തയുടെ പുത്രന്‍ ഇംറുല്‍ കൈസിന്റെ പുത്രന്‍ ദക്വാന്റെ പുത്രന്‍ സഅ്‌ലബയുടെ പുത്രന്‍ ഫാലിജിന്റെ പുത്രന്‍ ഹിലാലിന്റെ പുത്രന്‍ മുര്‍റയുടെ പുത്രന്‍ അവ്കസിന്റെ പുത്രന്‍ ഉമയ്യയുടെ പുത്രന്‍ ഹകീമിന്റെ പുത്രി ഖൗല. (ത്വബകാത്തു ഇബ്‌നു സഅ്ദ്: 8/158)

ഇനി നബിയുടെ മാതാവ് ആമിനയുടെയും സഈദിബ്‌നുല്‍ മുസ്വയ്യിബി(റ)ന്റെ മാതൃസഹോദരിയായ ഖൗല ബിന്‍ത്ത് ഹകീം അസ്സുലമിയ്യയുടെയും മാതാക്കളുടെ പരമ്പരയും കൂടി കാണുക:

“ആമിനയുടെ മാതാവ്, 'ബർറ' ബിൻത്ത് അബ്ദുൽ ഉസ്സായാണ്; കിലാബിന്റെ മകൻ കുസ്വയ്യിന്റെ മകൻ അബ്ദുദ്ദാറിന്റെ മകൻ ഉസ്മാന്റെ മകൻ അബ്ദുൽ ഉസ്സായുടെ മകൾ ബർറ. (ത്വബകാതുൽ കുബ്റാ: ഇബ്നു സഅ്ദ്: 1:59, സീറത്തു ഇബ്നു ഹിശാം: 1:110)

ഖൗലയുടെ മാതാവ് 'ദഈഫ'യാണ്. അബ്ദുശ്ശംസിന്റെ മകൻ ഉമയ്യയുടെ മകൻ ആസിന്റെ മകൾ ദഈഫ. (ത്വബകാതുൽ കുബ്റാ: ഇബ്നു സഅ്ദ്: 8:158, സിയറു അഅ്ലാമിന്നുബലാഅ്: 1:164)

ഇനി സാക്ഷാല്‍ ഖൗല ബിന്‍ത്ത് ഹകീമില്‍ നിന്ന് ഹദീഥ് ഉദ്ധരിച്ച സഈദിബ്‌നുല്‍ മുസ്വയ്യിബിന്റെ മാതാവ് ആരാണെന്ന് കാണുക:

അവ്കസിന്റെ പുത്രന്‍ ഉമയ്യയുടെ പുത്രന്‍ ഹകീമിന്റെ പുത്രി ഉമ്മു സഈദ്. (ത്വബകാത്തു ഇബ്‌നു സഅ്ദ്: 5/119)

സഈദിബ്‌നുല്‍ മുസ്വയ്യിബിന്റെ മാതാവും ഖൗല ബിന്‍ത്ത് ഹകീമും ഉപ്പയും ഉമ്മയുമൊത്ത സഹോദരിമാരാണ്. അല്ലാതെ നബി(സ്വ)യുടെ മാതൃസഹോദരിയല്ല ഖൗല ബിന്‍ത്ത് ഹകീം എന്നര്‍ഥം.

നബി(സ്വ)യോട് ‘ഹിബത്ത്’ വിവാഹത്തിന് താല്‍പര്യമറിയിച്ച ഖൗല ബിന്‍ത്ത് ഹകീമിന്റെ പിതാവിന്റെ പേര് ഹകീം എന്നാണെങ്കില്‍ നബി(സ്വ)യുടെ മാതാവ് ആമിനയുടെ പിതാവിന്റെ പേര് വഹബ് എന്നാണ്. അഥവാ ഖൗല ബിന്‍ത്ത് ഹകീം ആമിന ബിന്‍ത്ത് വഹബിന്റെ സഹോദരിയല്ല. അതുകൊണ്ടു തന്നെ നബിയുടെ മാതുലയുമല്ല. ഇസ്‌ലാമിക ചരിത്ര രേഖകളില്‍ വളരെ വ്യക്തമായി ഇതു രേഖപ്പെടുത്തപ്പെട്ടിട്ടും പ്രവാചകനു മേല്‍ ബന്ധുരതി (Incest) ആരോപിക്കുവാന്‍ പരിശ്രമിക്കുന്നവരില്‍ നിന്നും ഒരു നന്മയും നാം പ്രതീക്ഷിക്കേണ്ടതില്ല. ഹദീഥുകളെയും ഇസ്‌ലാമിക ചരിത്രത്തെയും പച്ചയായി ദുര്‍വ്യാഖ്യാനിച്ച് പ്രവാചകനെ തെറിയഭിഷേകം ചെയ്യുന്നവര്‍, ക്രൂരമായ മതനിന്ദയാണോ നിര്‍വഹിക്കുന്നത് അതല്ല വിമര്‍ശന സ്വാതന്ത്ര്യം ഉപയോഗപ്പെടുത്തുകയാണോ ചെയ്യുന്നതെന്ന് മാന്യതയുള്ളവര്‍ മൂന്നുവട്ടം ആലോചിക്കട്ടെ. ആവിഷ്‌കാര സ്വാതന്ത്ര്യമെന്നത് തെറിവിളിക്കാനുള്ള സ്വാതന്ത്ര്യമായി പരിണമിച്ചാല്‍ പരിക്കേല്‍ക്കുന്നത് നാം പടുത്തുയര്‍ത്തിയ നമ്മുടെ ബഹുസ്വര സംസ്‌കാരത്തിനു മേലായിരിക്കും. അതുതന്നെയാണ് ഈ മനോരോഗികളുടെ ലക്ഷ്യമെന്ന് വിവരമുള്ളവരെല്ലാം തിരിച്ചറിയേണ്ടിയിരിക്കുന്നു. സ്വന്തം മാതുലയെ തനിക്കു വേണ്ടി ശരീരം ദാനം ചെയ്യാന്‍ പ്രേരിപ്പിച്ച വ്യക്തിയായി പ്രവാചകനെ ചിത്രീകരിക്കാന്‍ പരിശ്രമിച്ചവര്‍ വായിക്കേണ്ട ഒരു ഹദീഥ് ഇവിടെ കുറിക്കട്ടെ.

” അബൂസുഫ്‌യാന്റെ പുത്രി ഉമ്മുഹബീബ പറയുന്നു: ഞാന്‍ പറഞ്ഞു: ‘അല്ലാഹുവിന്റെ ദൂതരേ, അബൂസുഫ്‌യാന്റെ പുത്രിയായ എന്റെ സഹോദരിയെക്കൂടി അങ്ങ് വിവാഹം കഴിച്ചാലും’. നബി ചോദിച്ചു: ‘നിനക്ക് അതിഷ്ടമാണോ?’ ഞാന്‍ പറഞ്ഞു: ‘അതേ. ഞാന്‍ അങ്ങയുടെ ഏക പത്‌നിയല്ല. നന്മകളില്‍ എന്നോടൊപ്പം പങ്കുകൊള്ളുന്നത് എന്റെ സഹോദരി ആയിരിക്കുന്നതാണ് എനിക്കേറ്റവും ഇഷ്ടം’. നബി പറഞ്ഞു: ‘അത് എനിക്ക് അനുവദനീയമല്ല’……’നിങ്ങളുടെ പെണ്‍കുട്ടികളെയും സഹോദരിമാരെയും വിവാഹം കഴിക്കാന്‍ നിങ്ങള്‍ എന്നോടാവശ്യപ്പെടരുത്.” (സ്വഹീഹുല്‍ ബുഖാരി: 1755) ഇനി അടുത്ത ആരോപണത്തിന്റെ നിജസ്ഥിതികൂടി നമുക്കു പരിശോദിക്കാം.

2, ‘ഉമ്മു ഹറാം, ഉമ്മു സുലൈം എന്നീ സ്ത്രീകളെ നബി സ്വകാര്യ സന്ദര്‍ശനം നടത്താറുണ്ടായിരുന്നെന്നും അവരുടെ മടിയില്‍ പലപ്പോഴും തലവെച്ചു കിടക്കാറുണ്ടായിരുന്നു എന്നും ഹദീഥ് ഗ്രന്ഥങ്ങള്‍ തന്നെ വ്യക്തമാക്കുന്നുണ്ട്. അവിഹിത ബന്ധങ്ങള്‍ക്ക് പ്രവാചക ജീവിതത്തിലിടമുണ്ടായിരുന്നു എന്നു വ്യക്തമാക്കുന്ന സംഭവങ്ങളാണിതെല്ലാം.’

ആരായിരുന്നു ഉമ്മു ഹറാമും, ഉമ്മു സുലൈമും.? സ്വകാര്യ സന്ദര്‍ശനം നടത്താനും മടിയില്‍ തലവെച്ചു കിടക്കാനും മാത്രം എന്തു ബന്ധമായിരുന്നു പ്രവാചകന് അവരോടുണ്ടായിരുന്നത്.? നമുക്കു പരിശോധിക്കാം.

ഉമ്മു സുലൈം, യഥാർത്ഥ നാമം സഹ്‌ല. ജുന്‍ദുബിന്റെ പുത്രന്‍ ഹറാമിന്റെ പുത്രന്‍ സൈദിന്റെ പുത്രന്‍ ഖാലിദിന്റെ പുത്രന്‍ മല്‍ഹാന്റെ പുത്രിയാണ് ഉമ്മു സുലൈം. അനസ് ഇബ്‌നു മാലികിന്റെ മാതാവ്. (അല്‍ ഇസ്വാബ: 8/227)

ഉമ്മു ഹറാം ബിന്‍ത്ത് മല്‍ഹാന്‍, ഉമ്മു സുലൈമിന്റെ സഹോദരി, അനസ് ഇബ്‌നു മാലികിന്റെ മാതൃസഹോദരി. (തഹ്ദീബുല്‍ കമാല്‍: 35/338: നമ്പര്‍: 7962)

ഇമാം നവവി പറഞ്ഞു: അവര്‍ പ്രവാചകന് വിവാഹം നിഷിദ്ധമായ കുടുംബക്കാരായിരുന്നു എന്നതില്‍ പണ്ഡിതന്മാര്‍ക്കിടയില്‍ ഏകാഭിപ്രായമുണ്ട്. എന്നാല്‍ ഈ കുടുംബ ബന്ധം ഏത് രൂപത്തിലാണ് എന്നതില്‍ മാത്രമാണ് അഭിപ്രായ വ്യത്യാസം. ഇബ്‌നു അബ്ദുല്‍ ബിര്‍റും മറ്റു പണ്ഡിതന്മാരും പറഞ്ഞത് പ്രവാചകന്റെ മുലകുടി ബന്ധത്തിലെ മാതൃസഹോദരിയാണ് അവര്‍ എന്നാണ്. മറ്റു പല പണ്ഡിതന്മാരും അഭിപ്രായപ്പെടുന്നത്, അവര്‍ പ്രവാചകന്റെ പിതാവിന്റെയൊ പിതാമഹന്റെയൊ മാതൃസഹോദരിയാണ് എന്നാണ്. കാരണം പ്രവാചകന്റെ പിതാമഹന്‍ അബ്ദുല്‍മുത്വലിബിന്റെ മാതാവ് ബനൂനജ്ജാര്‍ ഗോത്രത്തില്‍ നിന്നാണ്. (ഉമ്മു ഹറാം, ഉമ്മു സുലൈം എന്നിവരും ബനൂനജ്ജാറുകാരാണ്). (ശര്‍ഹുന്നവവി അലാ മുസ്ലിം: 13/50, ഔനുല്‍ മഅ്ബൂദ്: 7/122, സിറാജുല്‍ വഹ്ഹാജ്: 4/332, ഫത്ഹുല്‍ ബാരി: 11/78, തന്‍വീറുല്‍ ഹവാലിക്: 1/309, ശര്‍ഹു സുയൂതി: 4/505, ഇര്‍ശാദുസ്സാരി: 5/36, നള്‌റത്തു നഈം: 1/549, മിര്‍കാത്തുല്‍ മഫാത്തീഹ്: 9/3478)

താബിഉകളിലെ (പ്രവാചകാനുചരന്മാരുടെ ശിഷ്യര്‍) ഇളതലമുറക്കാരുമായി സഹവസിച്ചിട്ടുള്ള ഇബ്‌നു വഹബ് പറയുന്നു: ഉമ്മു ഹറാം പ്രവാചകന്റെ മുല കുടിബന്ധത്തിലെ മാതൃസഹോദരിയാണ്. (ഫത്ഹുല്‍ ബാരി: 11/78)

നബി(സ്വ)യുടെ മാതൃസഹോദരിമാരാണ് ഉമ്മു ഹറാമും, ഉമ്മു സുലൈമും. അവരെ മകനായ പ്രവാചകന്‍ സന്ദര്‍ശിച്ചതിനെയും ആ മാതാക്കളുടെ മടിയില്‍ തലവെച്ചു കിടന്നതിനെയുമാണ് നബിവൈരികള്‍ ലൈംഗിക ഛായം പൂശിയവതരിപ്പിച്ചത്. എത്രമാത്രം നികൃഷ്ഠമാണ് നബിവിമര്‍ശകന്മാരുടെ അന്തരംഗം. മാതൃ-പുതൃ ബന്ധങ്ങളില്‍ പോലും അശ്ലീലതകള്‍ തിരയുന്നവരെ ഏതു മുക്കാലിയില്‍ കെട്ടിയിട്ടാണാവോ അടിക്കേണ്ടത്. മൃഗരതിയും ശവരതിയും സ്വവര്‍ഗരതിയും ബന്ധുരതിയും ശിശുരതിയുമെല്ലാം അവകാശവും സ്വാതന്ത്ര്യവുമായി ഉദ്‌ഘോഷിക്കുന്ന, അതിനു വേണ്ടിയുള്ള പോരാട്ടങ്ങളെ ആക്ടിവിസമായി നെറ്റിയിലൊട്ടിച്ചു നടക്കുന്ന നവനാസ്തികരില്‍ നിന്നും ഇത്രയൊന്നും പ്രതീക്ഷിച്ചാല്‍ പോരാ. ഇതിലും വലിയ വൃത്തികേടുകള്‍ക്ക് അടയിരിക്കുന്നവരാണവര്‍. വിശുദ്ധരും സ്വാത്തികരുമായവര്‍ക്കെതിരെ ഇനിയുമെന്തെല്ലാം വൃത്തികേടുകള്‍ അവരില്‍ നിന്നും കേള്‍ക്കാനിരിക്കുന്നു. കാരണം അവര്‍ക്ക് വെറുപ്പ് വെളിച്ചത്തിലേക്ക് വിളിച്ചവരോടാണ്. വിശുദ്ധിയിലേക്ക് നയിച്ചവരോടാണ്. മാനവികതക്ക് ജീവിത മൂല്ല്യങ്ങളിലേക്ക് വഴിവെട്ടിയവരോടാണ്. പക്ഷെ മോക്ഷത്തെപ്പറ്റിയും വിശുദ്ധിയെപ്പറ്റിയും മൂല്ല്യങ്ങളെപ്പറ്റിയും നിരന്തരം ഗിരിപ്രഭാഷണങ്ങളും സുവിശേഷഭാഷണങ്ങളും നടത്തുന്ന ഹിന്ദുത്വ വാദികളും മിഷനറിമാരും, ഇസ്‌ലാമിനും പ്രവാചകനുമെതിരെയാണെങ്കില്‍ ഏതു നവനാസ്തുക ഞരമ്പുരോഗിയേയും പേറുമെന്നാണെങ്കില്‍ വിസ്മരിക്കുരുത്; നാറിയവനെ പേറിയാല്‍ പേറിയവനും നാറുമെന്ന യാഥാര്‍ഥ്യത്തെ.

വിമർശനം:

(വിമർശകർ ഹദീസിന് നൽകുന്ന പരിഭാഷ അതേപടി ഇവിടെ ചേർക്കുന്നു:)

മുഹമ്മദ് നബിയുടെ മദ്യപാനത്തെ കുറിച്ച് ഗാബെർബിൻ അബ്ദുള്ള റിപ്പോർട്ട് ചെയ്തത്: അല്ലാഹുവിൻറെ ദൂതന് സമാധാനം ഉണ്ടാവട്ടെ. കൂടെ ആയിരുന്നപ്പോൾ, നബി അല്പം മദ്യം ആവശ്യപ്പെട്ടു. ഇതുകേട്ട് അനുചരന്മാരിൽ ഒരാൾ ചോദിച്ചു: അല്ലയോ അല്ലാഹുവിന്റെ ദൂതരെ ഞങ്ങൾ അങ്ങേയ്ക്ക് മദ്യം നൽകാൻ പാടുണ്ടോ..? തിരുമേനി: പറഞ്ഞു ഉവ്വ്. അയാൾ മദ്യം അന്വേഷിച്ചു പുറത്തേക്ക് പോയി ഒരു കോപ്പ മദ്യവുമായി തിരികെ വന്നു. മുഹമ്മദ് നബി പറഞ്ഞു: നീ മരച്ചില്ല കുത്തനെ പിടിച്ചു കൊണ്ട് (മദ്യം) മറച്ചിരുന്നോ, അവൻ (ഗാബെർ) പറഞ്ഞു ഉവ്വ്. അപ്പോൾ നബി അതു പാനം ചെയ്തു. sahih muslim,Hadith-3753. മറുപടി:

ഹദീസ് സ്വഹീഹ് ആണെന്നതിൽ സംശയമില്ല. പക്ഷെ ഹദീസിന്റെ വിവർത്തനത്തിൽ ധാരാളം കൃത്രിമം നടത്തി കൊണ്ടാണ് പ്രവാചകൻ (സ) മദ്യപിച്ചിരുന്നു എന്ന കള്ള പ്രചാരണത്തിന് ദുർവ്യാഖ്യാനക്കാർ ‘തെളിവ്’ മെനഞ്ഞെടുത്തിരിക്കുന്നത്.

ഹദീസും ഹദീസിനോട് പരമാവധി നീതി പുലർത്തിക്കൊണ്ടുള്ള വിവർത്തനവും ശ്രദ്ധിക്കുക:

ﻋﻦ ﺟﺎﺑﺮ ﺑﻦ ﻋﺒﺪ اﻟﻠﻪ، ﻗﺎﻝ: ﻛﻨﺎ ﻣﻊ ﺭﺳﻮﻝ اﻟﻠﻪ ﺻﻠﻰ اﻟﻠﻪ ﻋﻠﻴﻪ ﻭﺳﻠﻢ ﻓﺎﺳﺘﺴﻘﻰ، ﻓﻘﺎﻝ ﺭﺟﻞ: ﻳﺎ ﺭﺳﻮﻝ اﻟﻠﻪ ﺻﻠﻰ اﻟﻠﻪ ﻋﻠﻴﻪ ﻭﺳﻠﻢ، ﺃﻻ ﻧﺴﻘﻴﻚ ﻧﺒﻴﺬا؟ ﻓﻘﺎﻝ: «ﺑﻠﻰ»، ﻗﺎﻝ: ﻓﺨﺮﺝ اﻟﺮﺟﻞ ﻳﺴﻌﻰ، ﻓﺠﺎء ﺑﻘﺪﺡ ﻓﻴﻪ ﻧﺒﻴﺬ، ﻓﻘﺎﻝ ﺭﺳﻮﻝ اﻟﻠﻪ ﺻﻠﻰ اﻟﻠﻪ ﻋﻠﻴﻪ ﻭﺳﻠﻢ: «ﺃﻻ ﺧﻤﺮﺗﻪ ﻭﻟﻮ ﺗﻌﺮﺽ ﻋﻠﻴﻪ ﻋﻮﺩا»، ﻗﺎﻝ: ﻓﺸﺮﺏ

ജാബിർ ഇബ്നു അബ്ദുല്ല (റ) പറഞ്ഞു: (ഒരിക്കൽ) ഞങ്ങൾ അല്ലാഹുവിന്റെ ദൂതന്റെ (സ) കൂടെയായിരുന്നപ്പോൾ അദ്ദേഹം വെള്ളം ആവശ്യപ്പെട്ടു. അപ്പോൾ ഒരാൾ ചോദിച്ചു: അല്ലാഹുവിന്റെ ദൂതരേ, താങ്കൾക്ക് ഞങ്ങൾ നബീദ് (ഈത്തപ്പഴച്ചാറ്) കുടിക്കാൻ തരട്ടെ ? അദ്ദേഹം അതെ എന്ന് പറഞ്ഞു. അങ്ങനെ അയാൾ പുറപ്പെടുകയും ഒരു പാത്രത്തിൽ ഈത്തപ്പഴച്ചാറ് കൊണ്ട് വരികയും ചെയ്തു. അപ്പോൾ അല്ലാഹുവിന്റെ ദൂതൻ (സ) ചോദിച്ചു: ഒരു മരക്കമ്പ് വെച്ചു കൊണ്ടെങ്കിലും നീ പാത്രം മൂടിയിരുന്നില്ലേ ? (അയാൾ അതെ എന്ന് അറിയിച്ചപ്പോൾ) പ്രവാചകൻ (സ) അത് കുടിച്ചു. (സ്വഹീഹു മുസ്‌ലിം: 2011)

വിമർശകരുടെ വിവർത്തനവും ഹദീസിന്റെ ശരിയായ വിവർത്തനവും തമ്മിലുള്ള അന്തരം നോക്കൂ !! എത്രമാത്രം കൃത്രിമങ്ങളും ദുർവ്യാഖ്യാനങ്ങളുമാണ് വിമർശകർ പടച്ചുണ്ടാക്കിയ ‘മദ്യപാനിയായ മുഹമ്മദ്’ എന്ന നാടകത്തിന്റെ ‘തിരകഥയിൽ’ തിരുകി കയറ്റിയിരിക്കുന്നത് !!! അവ ഓരോന്നും അക്കമിട്ട് വിശദീകരിക്കാം.

1. അല്ലാഹുവിന്റെ ദൂതന്റെ(സ) “വെള്ളം ആവശ്യപ്പെട്ടു” (ﻓﺎﺳﺘﺴﻘﻰ) എന്നതിന് പകരം “നബി അല്പം മദ്യം ആവശ്യപ്പെട്ടു.” എന്നതാണ് ആദ്യത്തെ ‘കള്ളകടത്ത്’. ഹദീസിലെ ‘ഇസ്തസ്ക്വാ’ (اﺳﺘﺴﻘﻰ) എന്ന പദം കുർആനിൽ പോലും വന്നിട്ടുള്ള ഒന്നാണ്:

وَإِذِ ٱسۡتَسۡقَىٰ مُوسَىٰ لِقَوۡمِهِۦ فَقُلۡنَا ٱضۡرِب بِّعَصَاكَ ٱلۡحَجَرَۖ فَٱنفَجَرَتۡ مِنۡهُ ٱثۡنَتَا عَشۡرَةَ عَیۡنࣰاۖ … “മൂസാ നബി തന്റെ ജനതയ്ക്കുവേണ്ടി വെള്ളത്തിനപേക്ഷിച്ച സന്ദര്‍ഭവും (ശ്രദ്ധിക്കുക.) അപ്പോള്‍ നാം പറഞ്ഞു: നിന്റെ വടികൊണ്ട് പാറമേല്‍ അടിക്കുക. അങ്ങനെ അതില്‍ നിന്ന് പന്ത്രണ്ട് ഉറവുകള്‍ പൊട്ടി ഒഴുകി… ” (കുർആൻ: 2:60)

‘ഇസ്തസ്ക്വാ’ (ٱسۡتَسۡقَىٰ) അഥവാ വെള്ളത്തിനപേക്ഷിച്ചു എന്ന പദത്തിന് അൽപ്പം മദ്യം ആവശ്യപ്പെട്ടു എന്ന ഇല്ലാത്ത അർത്ഥം വിമർശകർ സൃഷ്ടിച്ചത് ഹദീസിന്റെ ഉള്ളടക്കം മൊത്തം അപനിർമിക്കാൻ വേണ്ടിയാണ്.

2. വെള്ളം ചോദിച്ച നബിയോട് “താങ്കൾക്ക് ഞങ്ങൾ നബീദ് (ഈത്തപ്പഴച്ചാറ്) കുടിക്കാൻ തരട്ടെ ?” (ﺃﻻ ﻧﺴﻘﻴﻚ ﻧﺒﻴﺬا؟) എന്ന് അനുചരന്മാരിൽ ഒരാൾ പ്രതികരിച്ച സ്വഭാവിക പ്രതികരണത്തെയാണ് രണ്ടാമതായി ദുർവ്യാഖ്യാന വീരന്മാർ അർത്ഥം മാറ്റിയിരിക്കുന്നത്. “താങ്കൾക്ക് ഞങ്ങൾ നബീദ് (ഈത്തപ്പഴച്ചാറ്) കുടിക്കാൻ തരട്ടെ ?” എന്ന വാചകത്തെ വളച്ചൊടിച്ച് “അല്ലയോ അല്ലാഹുവിന്റെ ദൂതരെ ഞങ്ങൾ അങ്ങേയ്ക്ക് മദ്യം നൽകാൻ പാടുണ്ടോ..?” എന്ന ഇല്ലാത്ത അർത്ഥം നൽകി !! ഈ ‘പാടുണ്ടോ’ എന്ന വിവർത്തനമൊക്കെ ‘അലാ നുസ്കീക നബീദാ” എന്ന അറബി വാചകത്തിലെ ഏത് പദത്തിന്റെ പരിഭാഷയാണെന്ന് വിമർശകർക്ക് പറയാനൊക്കുമോ ?!! യാതൊരു പുലബന്ധവുമില്ലാത്ത തോന്യാസങ്ങൾ ഹദീസിന്റെ ഇംഗ്ലീഷ് പരിഭാഷകളിൽ കള്ളക്കടത്തു നടത്തുന്ന മിഷണറി അപനിർമാണ സൈറ്റുകളിലും, ഹദീസ് വിരുദ്ധ ശിഈ സൈറ്റുകളിലും ഓടുന്ന ഹദീസുകളുടെ നിഴലുകൾ മാത്രമാണ് ഇത്തരം വിവർത്തനങ്ങൾ. യഥാർത്ഥ ഹദീസുകളിൽ മാറ്റം വരുത്തി നിർമ്മിക്കപ്പെടുന്ന ഫേക്ക് ഹദീസുകൾ.

3. അടുത്ത കൃത്രിമം ‘നബീദ്’ (النَّبِيذُ) എന്ന പദത്തിൻമേലാണ്. പ്രവാചകനോട് കുടിക്കാൻ തരട്ടെയോ? എന്ന് അനുചരന്മാർ ചോദിച്ചതും പ്രവാചകൻ (സ) കുടിച്ചതുമായി ഹദീസിൽ പ്രസ്ഥാവിക്കുന്നത് നബീദ് എന്ന മധുര പാനീയമാണ്. ആ പദത്തിന് മദ്യമെന്ന് വിവർത്തനം നൽകിയുള്ള ഹദീസ് തിരിമറികൾ ഒരുപാട് നാളായി നടന്നു വരുന്നു. മദ്യത്തിന്റെ കാര്യത്തിൽ പത്തുപേരെ/ കാര്യങ്ങളെ അല്ലാഹുവിന്റെ ദൂതൻ (സ) ശപിച്ചിട്ടുണ്ട്. മദ്യത്തെയും, അത് കുടിക്കുന്നവനേയും കുടിപ്പിക്കുന്നവനേയും വിൽക്കുന്നവനേയും ആർക്കുവേണ്ടി വിൽക്കപ്പെടുന്നുവൊ അവനേയും അതു ഉണ്ടാക്കുന്നവനേയും ആർക്കുവേണ്ടി ഉണ്ടാക്കുന്നവോ അവനേയും അത് വഹിച്ചു കൊണ്ടു പോകുന്നവനേയും അത് അരുടെ അടുത്തേക്ക് വഹിച്ചു കൊണ്ട് പോകുന്നുവൊ അവനേയും അത് വിറ്റു കിട്ടുന്ന പ്രതിഫലം ഭക്ഷിക്കുന്നവനേയും. (മുസ്നദു അഹ്‌മദ്: 4787, ഹാകിം: 7311)

ഇതേ പ്രവാചകൻ മദ്യപാനിയാണെന്ന് വരുത്തി തീർക്കാനുള്ള ദുർവ്യാഖ്യാനം അതീവ സാഹസികത തന്നെയാണെന്നതിൽ സംശയമില്ല. അതിനായി നബീദ് എന്ന ‘പഴം ജ്യൂസ്’ മദ്യമായി മാറ്റുന്ന ‘വാറ്റിയെടുക്കൽ’ പ്രക്രിയയാണ് ഈ വിവർത്തനം.

പ്രവാചകൻ (സ) കുടിച്ച നബീദ് എന്ന മധുര പാനീയം എന്താണെന്ന് ഹദീസുകളിൽ നിന്ന് തന്നെ നമ്മുക്ക് മനസ്സിലാക്കാം.

ﻋَﻦْ ﻗَﺮْﺻَﺎﻓَﺔَ ﺑِﻨْﺖِ ﻋُﻤَﺮَ ﻗَﺎﻟَﺖْ: ﺩَﺧَﻠْﺖُ ﻋَﻠَﻰ ﻋَﺎﺋِﺸَﺔَ ﻓَﻄَﺮَﺣَﺖْ ﻟِﻲ، ﻭَﺳَﺎﺩَﺓً ﻓَﺴَﺄَﻟَﺘْﻬَﺎ اﻣْﺮَﺃَﺓٌ ﻋَﻦِ اﻝﻧﺒﻴﺬ ﻓَﻘَﺎﻟَﺖْ: «ﻧَﺠْﻌَﻞُ اﻟﺘَّﻤْﺮَﺓَ ﻓِﻲ اﻟْﻜُﻮﺯِ ﻓَﻨَﻄْﺒُﺨُﻪُ ﻓَﻨَﺼْﻨَﻌَﻪُ ﻧﺒﻴﺬا ﻓَﻨَﺸْﺮَﺑُﻪُ» ﻓَﻘَﺎﻟَﺖْ: «اﺷْﺮَﺑِﻲ ﻭَﻻَ ﺗَﺸْﺮَﺑِﻲ ﻣُﺴْﻜِﺮًا»

ഒരു സ്ത്രീ നബീദിനെ സംബന്ധി ആഇശയോട്(റ) പറഞ്ഞു: ഈത്തപ്പഴം മൊന്തയിൽ ഇട്ട് പാകം ചെയ്ത് നബീദ് ഞങ്ങൾ കുടിക്കാറുണ്ട്. ആഇശ (റ) പറഞ്ഞു: നീ അത് കുടിച്ചു കൊള്ളുക. എന്നാൽ ലഹരി വന്നത് കുടിക്കരുത്. (മുസ്വന്നഫ് അബ്ദുർറസാക്: 16952)

ആഇശ (റ) പറയുന്നു: ഞങ്ങൾ അല്ലാഹുവിന്റെ ദൂതന് തോൽപാത്രത്തിൽ നബീദ് നൽകാറുണ്ടായിരുന്നു. ഞങ്ങൾ ഒരു പിടി ഈത്തപ്പഴവും ഒരു പിടി ഉണക്കമുന്തിരിയുമെടുത്ത് തോൽ പാത്രത്തിൽ ഇടും. ശേഷം അതിൽ വെള്ളമൊഴിക്കും. എന്നിട്ട് രാവിലെ ഉണ്ടാക്കിയ ആ പഴച്ചാറ് അദ്ദേഹം വൈകുന്നേരം കുടിക്കും. വൈകുന്നേരമാണ് ഉണ്ടാക്കുന്നതെങ്കിൽ രാവിലെ കുടിക്കും. (ഇബ്നുമാജ: 3398)

എന്താണ് പ്രവാചകരും അനുചരന്മാരും കുടിച്ചിരുന്ന നബീദ് എന്ന് ഈ രണ്ടു ഹദീസുകളിൽ നിന്ന് സുതരാം വ്യക്തമാണ്. തോൽപാത്രത്തിൽ ഈത്തപ്പഴമോ മറ്റു പഴവർഗങ്ങളൊ ഇട്ട് വെള്ളം ഒഴിച്ച് വെച്ച് അതിൽ നിന്നും ലഭിക്കുന്ന മധുരമുള്ള ചാറ് അല്ലെങ്കിൽ ജ്യൂസ്… ഇതാണ് നബീദ്.

ഈ പഴച്ചാറ് അല്ലെങ്കിൽ ഫ്രൂട്ട് ജ്യൂസ് ഇസ്‌ലാമിൽ നിഷിദ്ധമല്ല. കാരണം അത് മദ്യമല്ല. മദ്യം എന്നാൽ “ബുദ്ധിയെ തിരിച്ചു കളയുന്ന” ലഹരിയുള്ള വസ്തുക്കളും പാനീയങ്ങളുമാണ് എന്ന് പ്രവാചകൻ (സ) പഠിപ്പിച്ചിട്ടുണ്ട്. (സ്വഹീഹുൽ ബുഖാരി:5266)

നബീദ് എന്ന പഴച്ചാറ് അല്ലെങ്കിൽ ഫ്രൂട്ട് ജ്യൂസ് ലഹരിയുള്ളതല്ലാത്തതു കൊണ്ടാണ് പ്രവാചകൻ (സ) അതു കുടിക്കുന്നത് തെറ്റായി കാണാതിരുന്നത്.

സ്വഹീഹു മുസ്‌ലിമിലെ വിവാദ വിധേയമായ ഹദീസിനെ സംബന്ധിച്ച് ഇമാം നവവി ഇപ്രകാരം പറഞ്ഞു: “ജാബിർ (റ) പ്രവാചകന് കൊണ്ടുവന്നു കൊടുത്ത നബീദ് മൂപ്പെത്തിയിട്ടില്ലാത്തതും ലഹരി ബാധിച്ചിട്ടില്ലാത്തതുമായ പാനീയമാണ്.” (ശർഹു മുസ്‌ലിം: 13: 173)

എന്നാൽ ഈ പഴച്ചാറ് തണുപ്പുള്ള പാത്രത്തിൽ സൂക്ഷിക്കാതിരിക്കുകയോ മൂന്നിൽ അധിക ദിവസം എടുത്തു വെക്കുകയോ ചെയ്താൽ അതിൽ ലഹരി വന്നുചേരും. ഇത്തരം ലഹരി വന്ന നബീദ് പ്രവാചകനോ അനുചരന്മാരോ കുടിച്ചിരുന്നില്ല. “എന്നാൽ ലഹരി വന്നത് കുടിക്കരുത് “എന്ന് ആഇശ (റ) പ്രത്യേകം ഉണർത്തിയതായും മുകളിൽ ഉദ്ധരിച്ച ഹദീസിൽ (മുസ്വന്നഫ് അബ്ദുർറസാക്: 16952) തന്നെ കാണാം. ലഹരി വന്നാൽ അവ ‘ഖംറ്’ (الخمر) അഥവാ മദ്യമായി പരിണമിക്കുന്നു.

പ്രവാചകൻ (സ) തന്നെയും നബീദ് (അഥവാ ഈത്തപ്പഴച്ചാറിൽ) ലഹരിയുടെ ലാഞ്ചനയെങ്കിലും വന്നെന്ന് മനസ്സിലാക്കിയാൽ അത് വലിച്ചെറിയുമായിരുന്നു എന്ന് ഒട്ടനവധി ഹദീസുകളിൽ കാണാം:

باب في النبيذ إذا غلى

‘നബീദ് ലഹരിയുള്ളതായി മാറിയാൽ’ എന്ന അധ്യായത്തിൽ ഇബ്നുമാജ ഉദ്ധരിച്ച ഹദീസ് കാണുക:

ﻋﻦ ﺃﺑﻲ ﻫﺮﻳﺮﺓ، ﻗﺎﻝ: ﻋﻠﻤﺖ ﺃﻥ ﺭﺳﻮﻝ اﻟﻠﻪ ﺻﻠﻰ اﻟﻠﻪ ﻋﻠﻴﻪ ﻭﺳﻠﻢ ﻛﺎﻥ ﻳﺼﻮﻡ، ﻓﺘﺤﻴﻨﺖ ﻓﻄﺮﻩ ﺑﻧﺒﻴﺬ ﺻﻨﻌﺘﻪ ﻓﻲ ﺩﺑﺎء ﺛﻢ ﺃﺗﻴﺘﻪ ﺑﻪ ﻓﺈﺫا ﻫﻮ ﻳﻨﺶ، ﻓﻘﺎﻝ: «اﺿﺮﺏ ﺑﻬﺬا اﻟﺤﺎﺋﻂ، ﻓﺈﻥ ﻫﺬا ﺷﺮاﺏ ﻣﻦ ﻻ ﻳﺆﻣﻦ ﺑﺎﻟﻠﻪ ﻭاﻟﻴﻮﻡ اﻵﺧﺮ

അബൂഹുറൈറ (റ) പറഞ്ഞു: അല്ലാഹുവിന്റെ ദൂതൻ (സ) നോമ്പുകാരനാണെന്ന് ഞാൻ അറിഞ്ഞു. അപ്പോൾ ഞാൻ നോമ്പുതുറക്കായുള്ള നബീദ്, ദുബ്ബാഅ് പാത്രത്തിൽ ഒരുക്കി. ശേഷം അദ്ദേഹത്തിന്റെ അടുത്തേക്ക് ഞാൻ അത് കൊണ്ടുപോയപ്പോൾ അത് ലഹരിയുള്ളതായി മാറിയിരുന്നു. ആ പാത്രം അടുപ്പിച്ച് പിടിക്കാൻ അദ്ദേഹം പറഞ്ഞു. ഞാൻ അത് അടുപ്പിച്ച് പിടിച്ചപ്പോൾ അദ്ദേഹമത് മണത്ത് നോക്കി. അപ്പോൾ (മണത്തിൽ നിന്ന്) അതിന് ലഹരി ബാധിച്ചിട്ടുണ്ടെന്ന് മനസ്സിലാക്കിയപ്പോൾ പ്രവാചകൻ (സ) പറഞ്ഞു: ഇത് വലിച്ചെറിയുക. തീർച്ചയായും ഈ പാനീയം അല്ലാഹുവിലും അന്ത്യദിനത്തിലും വിശ്വസിക്കാത്തവരുടേതാകുന്നു. (മുസ്‌ലിമിനുള്ളതല്ല). (സുനനു ഇബ്നുമാജ: 3716, സുനനു നസാഈ: 5100, സുനനു ദാറകുത്‌നി: 4642) ﻋﻦ ﺃﺑﻲ ﻣﻮﺳﻰ، ﺭﺿﻲ اﻟﻠﻪ ﻋﻨﻪ ” ﺃﻧﻪ ﺃﺗﻰ اﻟﻨﺒﻲ ﺻﻠﻰ اﻟﻠﻪ ﻋﻠﻴﻪ ﻭﺳﻠﻢ ﺑﻧﺒﻴﺬ ﺟﺮ ﻳﻨﺶ، ﻓﻘﺎﻝ «اﺿﺮﺏ ﺑﻬﺬا اﻟﺤﺎﺋﻂ، ﻓﺈﻧﻪ ﻻ ﻳﺸﺮﺑﻪ ﻣﻦ ﻳﺆﻣﻦ ﺑﺎﻟﻠﻪ ﻭاﻟﻴﻮﻡ اﻵﺧﺮ»

അബൂ മൂസൽ അശ്അരിയും ലഹരിയുള്ളതായി മാറിയ നബീദ് പ്രവാചകന്റെ അടുത്തേക്ക് കൊണ്ടുപോയപ്പോൾ അദ്ദേഹം (സ) പറഞ്ഞു: ഇത് വലിച്ചെറിയുക. തീർച്ചയായും ഈ പാനീയം അല്ലാഹുവിലും അന്ത്യദിനത്തിലും വിശ്വസിക്കാത്ത ജനതയുടേതാകുന്നു. (മുസ്നദുൽ ബസ്സാർ: 3192)

പ്രവാചകൻ (സ) പറഞ്ഞു: “നിങ്ങൾ നബീദ് കുടിച്ചു കൊള്ളുക. എന്നാൽ ലഹരിയുള്ളത് ഞാൻ അനുവദിക്കില്ല…” (മുസ്നദു അഹ്‌മദ്: 11329)

അപ്പോൾ പ്രവാചകൻ (സ) കുടിച്ചതായി വിവാദ വിഷയകമായ ഹദീസിൽ പറയുന്ന നബീദ് ലഹരി വന്നിട്ടില്ലാത്ത കേവലം ഈത്തപ്പഴച്ചാറാണ്. അഥവാ തനിക്കു നൽകപ്പെടുന്ന നബീദിൽ ലഹരിയുടെ വാസനയൊ ലാഞ്ചനയോ മനസ്സിലാക്കിയാൽ അവ പ്രവാചകൻ (സ) വലിച്ചെറിയുകയാണ് പതിവ് എന്നർത്ഥം.

ഈ പ്രവാചകനെയാണോ നിങ്ങൾ മദ്യപാനിയായി ചിത്രീകരിക്കുന്നത് ?!!

അപ്പോൾ നബീദ് സ്വമേധയാ അനുവദനീയമായ ഒരു മധുര പാനീയമാണ്. അവ ഒരുപാട് ദിവസം സൂക്ഷിച്ചു വെച്ച് പാകപ്പെടുത്തിയെടുത്ത് ലഹരിയുള്ളതാക്കി മാറ്റുമ്പോളാകട്ടെ ഇസ്‌ലാമിൽ അത് നിഷിദ്ധമാണ്. ഈ ലഹരിയുള്ള നബീദ് അഥവാ വീഞ്ഞായി പരിണമിച്ച നബീദ് പ്രവാചകൻ (സ) കുടിച്ചതായി ഒരു ഹദീസും നമുക്ക് പറഞ്ഞു തരുന്നില്ല. ഈ വസ്തുത അമുസ്‌ലിംകളായ ഗവേഷകർ പോലും രേഖപ്പെടുത്തിയിട്ടുണ്ട്:

ബ്രിട്ടൺ ഗവൺമെന്റിന്റെ ‘നാവൽ ഇന്റലിജൻസ് ഡിവിഷൻ’ (Naval Intelligence Division) തയ്യാറാക്കുകയും ഓക്സ്ഫോർഡ് യൂനിവേഴ്സിറ്റി പ്രസ്സ് പുറത്തിറക്കുകയും ചെയ്ത ‘western Arabia and the Red Sea’ എന്ന പുസ്തകത്തിൽ (പേജ്: 489) ഇപ്രകാരം രേഖപ്പെടുത്തപ്പെട്ടിരിക്കുന്നു: “പുതിയ ഈത്തപ്പഴങ്ങളുടെ മേൽ വെള്ളം ഒഴിച്ച് ഒറ്റരാത്രികൊണ്ട് ഉണ്ടാക്കപ്പെട്ട നബീദ് എന്നറിയപ്പെടുന്ന ഒരു ലഹരി ബാധിതമല്ലാത്ത പാനീയം മുഹമ്മദിന്റെ കാലം മുതൽ കുടിക്കപ്പെട്ടിരുന്നു.” (അവലംബം: https://dawatetowheed.wordpress.com)

നബീദിൽ ലഹരി കടന്നുവരാനുള്ള സാധ്യതയുള്ളതിനാൽ തന്നെ അത്തരം സാധ്യതകളെ പോലും പ്രവാചകൻ (സ) വിലക്കുകയാണുണ്ടായത് എന്നും ഹദീസുകൾ നമ്മെ പഠിപ്പിക്കുന്നു.

ദുബ്ബാഅ്, നകീർ, മുസഫ്ഫത്ത്, ഹൻത്തം എന്നീ പാത്രങ്ങളിൽ നബീദ് സൂക്ഷിക്കുന്നതും കുടിക്കുന്നതും പ്രവാചകൻ (സ) സ്വയം വർജിക്കുകയും അനുചരന്മാരെ വിലക്കുകയും ചെയ്തത് ഇക്കാരണത്താലാണ്. ഇത്തരം പാത്രങ്ങൾക്ക് താഴ്ന്ന താപനില നിലനിർത്തി പഴച്ചാറുകളുടെ ലഹരിവൽകരണത്തെ മന്ദഗതിയിലാക്കാൻ കഴിയില്ല. അതേസമയം തോൽ പാത്രം താഴ്ന്ന താപനില നിലനിർത്തുന്നതിന് സഹായിക്കുന്നു. അതിനാൽ ലഹരി വൽക്കരണം തടയപ്പെടുന്നു. രണ്ടോ മൂന്നോ ദിവസം ഈത്തപ്പഴ വെള്ളം ‘ഫ്രെഷായി’ അവശേഷിക്കുന്നു. മറ്റൊരർത്ഥത്തിൽ പറഞ്ഞാൽ അന്നത്തെ ഒരു ‘മിനി ഫ്രിഡ്ജാ’യിരുന്നു തോൽ പാത്രം. ഇക്കാര്യം വ്യക്തമാക്കുന്ന ഹദീസുകൾ കാണുക:

പ്രവാചകൻ (സ) പറഞ്ഞു: “ദുബ്ബാഅ്, ഹൻത്തം, നകീർ… തുടങ്ങിയ പാത്രങ്ങളിൽ നബീദ് കുടിക്കുന്നത് ഞാൻ നിങ്ങളെ വിലക്കുന്നു. എന്നാൽ തോൽ പാത്രത്തിലുള്ള നബീദ് കുടിച്ചു കൊള്ളുക.” (സ്വഹീഹു മുസ്‌ലിം: 33 – 1993)

“ദുബ്ബാഅ്, നകീർ, മുസഫ്ഫത്ത്, ഹൻത്തം എന്നീ പാത്രങ്ങളിൽ നബീദ് കുടിക്കുന്നത് പ്രവാചകൻ (സ) വിലക്കി.” (സ്വഹീഹു മുസ്‌ലിം: 1995)

പഴച്ചാറുകളുടെ ലഹരിവൽക്കരണത്തിൽ അഥവാ ഫെർമെന്റേഷനിൽ (fermentation) താപനിലക്കും വായുവിനും വളരെ വലിയ സ്വാധീനമുണ്ട്. താപനില കുറയുന്നത് പദാർത്ഥങ്ങളെ പുളിപ്പിച്ച് ലഹരിപാനീയങ്ങളാക്കി മാറ്റുന്ന പ്രക്രിയയെ മന്ദഗതിയിലാക്കുന്നു. ഉയർന്ന താപനിലയിൽ പഴച്ചാറുകളുടെ പുളിക്കൽ, അതിവേഗത്തിൽ ആകുന്നു. തണുത്ത താപനില നൽകാനും നിലനിർത്താനും തോൽപാത്രം സഹായിക്കുന്നു.

ഇബ്നു ഹജർ പറഞ്ഞു: “ദുബ്ബാഅ്, ഹൻത്തം, നകീർ തുടങ്ങിയ പാത്രങ്ങളിൽ നബീദ് കുടിക്കരുത് എന്ന് ഹദീസുകളിൽ ഉദ്ധരിക്കപ്പെട്ടിട്ടുണ്ട്. തോൽ പാത്രവും ഇത്തരം പാത്രങ്ങളും തമ്മിലുള്ള വ്യത്യാസം തോൽ പാത്രമല്ലാത്ത മറ്റു പാത്രങ്ങളിലെ സൂക്ഷ്മായ വിടവുകളിലൂടെ വായു ഉള്ളിലേക്ക് പ്രവേശിക്കും എന്നതാണ്. ഇത് മൂലം തോൽ പാത്രത്തിലുള്ള നബീദിൽ നിന്ന് വ്യത്യസ്ഥമായി മറ്റു പാത്രങ്ങളിലേത് കേടു വരാനും ലഹരി ബാധിക്കാനും ഇടയാകുന്നു. അതുകൊണ്ടാണ് അത്തരം പാത്രങ്ങളിൽ നബീദ് സൂക്ഷിക്കുന്നത് പ്രവാചകൻ (സ) വിരോധിച്ചത്. കൂടാതെ, തോൽപാത്രം ഭദ്രമായി കെട്ടിവെച്ചാൽ ലഹരി ബാധയിൽ നിന്നും സുരക്ഷിതമാവുന്നു. കാരണം നബീദ് പുളിപ്പ് വരികയും സ്ഥിതിഭേദം സംഭവിക്കുകയും ചെയ്താൽ അത് ലഹരിയുള്ളതായി മാറും…” (ഫത്ഹുൽ ബാരി: 10: 61)

ഇമാം നവവി പറഞ്ഞു: “പണ്ഡിതന്മാർ പറഞ്ഞിരിക്കുന്നു: തോൽപാത്രം ഭദ്രമായി കെട്ടിവെച്ചാൽ ലഹരി ബാധയിൽ നിന്നും സുരക്ഷിതമാവുന്നു. കാരണം നബീദ് പുളിപ്പ് വരികയും സ്ഥിതിഭേദം സംഭവിക്കുകയും ചെയ്താൽ അത് ലഹരിയുള്ളതായി മാറും… ദുബ്ബാഅ്, ഹൻത്തം, മുസഫ്ഫത്ത് തുടങ്ങിയ പാത്രങ്ങളിൽ നബീദ് സൂക്ഷിച്ചാൽ അവ ലഹരിയുള്ളതായി മാറും…” (ശർഹു മുസ്‌ലിം: 13: 160)

ദുബ്ബാഅ് (اﻟﺪُّﺑَّﺎء) എന്നാൽ മത്തങ്ങ തുരന്ന് ഉണ്ടാക്കിയ പാത്രമാണ്. നകീർ (اﻟﻨَّﻘﻴﺮ), ഹൻത്തം (اﻟﺤﻨَﺎﺗِﻢ) എന്നിവ ഈത്തപ്പനയുടെ മുരട്, നടുവിലെ കഴമ്പ് കളഞ്ഞ് ഉപയോഗിക്കുന്ന പാത്രങ്ങളാണ്. മുസഫ്ഫത്ത് (اﻟﻤُﺰَﻓَّﺖ) ടാറ് പൂശിയ മരപാത്രം. (മുസ്നദു ശാഫിഈ: 309, 310)

ഇത്തരം പാത്രങ്ങൾ നബീദിന്റെ പുതുമയും ശുദ്ധതയും കുളിർമയും നിലനിർത്താൻ ഉതകുന്നവയല്ലാത്തത് കൊണ്ട് അവയിൽ സൂക്ഷിക്കപ്പെടുന്ന നബീദ് ലഹരി ബാധിതമാകുമായിരുന്നു. അതുകൊണ്ട് തന്നെ ഇത്തരം പാത്രങ്ങളിൽ നബീദ് കുടിക്കുന്നത് പ്രവാചകൻ (സ) വിലക്കുകയും തോൽപാത്രത്തിൽ മാത്രം നബീദ് കുടിക്കുന്നത് അദ്ദേഹം (സ) അനുവദിക്കുകയും ചെയ്തു. എന്നാൽ ഈ യുക്തി തിരിച്ചറിയാതിരുന്ന ചിലർ ഈ പാത്രങ്ങൾ സ്വമേധയാ നിഷിദ്ധമാണെന്ന് തെറ്റിദ്ധരിച്ചു. ചിലർ ഇതല്ലാത്ത പാത്രങ്ങളുടെ ലഭ്യതയുടെ കുറവിനെ സംബന്ധിച്ച് പ്രവാചകനോട് പരാതിപ്പെട്ടു. തദവസരത്തിൽ ആ പാത്രങ്ങൾ പ്രവാചകൻ (സ) അവർക്ക് അനുവദിച്ച് ഇളവു നൽകി. എന്നാൽ അന്നേരവും അത്തരം പാത്രങ്ങളിൽ സൂക്ഷിച്ച നബീദുകളിൽ ലഹരി വന്നാൽ കുടിക്കരുത് എന്ന് പ്രത്യേകം ഉണർത്തുകയും ചെയ്തു.

പ്രവാചകൻ (സ) പറഞ്ഞു: “ചില പാത്രങ്ങളിലെ നബീദിൽ നിന്നും ഞാൻ നിങ്ങളെ വിലക്കിയിരുന്നു. എന്നാൽ നിങ്ങൾ അറിയുക ആ പാത്രങ്ങളിൽ ഉള്ളതിനാൽ മാത്രം അവ ഒന്നിനേയും നിഷിദ്ധമാക്കുന്നില്ല. എന്നാൽ എല്ലാ ലഹരിയുള്ളവയുമാണ് നിഷിദ്ധം.” (അൽ ജാമിഉ ലി ഇബ്നു വഹബ്: 26)

പ്രവാചകൻ (സ) പറഞ്ഞു: “തോൽപാത്രത്തിൽ ഉള്ളതല്ലാത്ത എല്ലാ നബീദും ഞാൻ നിങ്ങൾക്ക് വിരോധിച്ചിരുന്നു. എന്നാൽ (ഇനിമേൽ) എല്ലാ പാത്രത്തിലും നിങ്ങൾ നബീദ് കുടിച്ചു കൊള്ളുക. എന്നാൽ ലഹരിയുള്ളത് കുടിക്കരുത്.” (സ്വഹീഹു മുസ്‌ലിം: 977,1977, മുസ്നദുൽ ബസ്സാർ : 4435, സുനനു തുർമുദി: 1054, ത്വയാലിസി: 807, ഹാകിം: 1/375, മുസ്നദു അഹ്മദ്: 5/259, 261, സുനനു അബൂദാവൂദ്: 3235, ബൈഹകി: 4/76, ബഗ്‌വി: 1553, അബ്ദുർറസാക്: 6708)

പ്രവാചകൻ (സ) പറഞ്ഞു: “കളിമൺ പാത്രത്തിൽ സൂക്ഷിച്ച നബീദ് കുടിക്കുന്നതിൽ നിന്നും ഞാൻ നിങ്ങളെ വിലക്കിയിരുന്നു. എന്നാൽ ഇനിമേൽ എല്ലാ പാത്രത്തിലും നിങ്ങൾ നബീദ് കുടിച്ചു കൊള്ളുക. (എന്നാൽ) എല്ലാ ലഹരിയുള്ള (നബീദും) ഉപേക്ഷിക്കുക.” (മുസ്വന്നഫ് അബ്ദുർറസാക്: 6708, 6714, മുസ്വന്നഫ് ഇബ്നു അബീ ശൈബ: 23942,)

തബൂക്ക് യുദ്ധ സന്ദർഭത്തിൽ ചിലർക്ക്, ദുബ്ബാഅ്, ഹൻതം, മുസഫ്ഫത്ത് എന്നീ പാത്രങ്ങളിൽ നബീദ് കുടിക്കുന്നത് പ്രവാചകൻ (സ) നിരോധിച്ചു. പിന്നീട് അവർക്ക് വയറിന് അസുഖം ബാധിച്ചതായി അവർ പരാധിപ്പെട്ടപ്പോൾ ആ പാത്രങ്ങളിൽ കുടിക്കുന്നത് പ്രവാചകൻ (സ) ഇളവു നൽകി. എന്നാൽ ലഹരിയുള്ള (നബീദ്) കുടിക്കുന്നത് അദ്ദേഹം പ്രത്യേകം വിലക്കി. (അൽ ആസാർ: അബൂ യൂസുഫ്: 997)

ഇത്തരം പാത്രങ്ങൾ അനുവദിച്ച അവസരത്തിലും നബീദുകളിൽ ലഹരി വന്നാൽ കുടിക്കരുത് എന്ന് പ്രവാചകൻ (സ) പ്രത്യേകം ഉണർത്തുന്നത് ശ്രദ്ധിക്കുക. അതു പോലും ശ്രദ്ധിച്ച പ്രവാചകനെയാണോ നിങ്ങൾ മദ്യപാനിയായി ചിത്രീകരിക്കാൻ ശ്രമിക്കുന്നത് ?!!

നബീദ് ഒരുപാടു ദിവസം സൂക്ഷിച്ചാലും മത്തും ലഹരിയും ബാധിക്കാൻ സാധ്യതയുണ്ട്. ഈ സാധ്യത അറിയാവുന്നതു കൊണ്ട് തന്നെ പരമാവധി മൂന്ന് ദിവസ കാലയളവിനപ്പുറം – തോൽപാത്രത്തിൽ സൂക്ഷിച്ച നബീദാണെങ്കിൽ പോലും – അത് കുടിക്കാതിരിക്കാൻ പ്രവാചകൻ (സ) കാർക്കശ്യം കാണിച്ചിരുന്നു. മൂന്ന് ദിവസ കാലയളവിനപ്പുറമുള്ള നബീദ് കുടിക്കരുതെന്ന് മറ്റുള്ളവരോട് ഗൗരവകരമായ ശാസനയും നൽകി. ഈ പ്രവാചകനെയാണോ നിങ്ങൾ മദ്യപാനിയായി ചിത്രീകരിക്കാൻ ശ്രമിക്കുന്നത് ?!!

ഇബ്നു അബ്ബാസ് (റ) പറഞ്ഞു: അല്ലാഹുവിന്റെ ദൂതന്(സ) തോൽപാത്രത്തിൽ നബീദ് നൽകപെട്ടിരുന്നു. ഞായറാഴ്ച്ച രാത്രി നൽകപ്പെട്ട നബീദ് അദ്ദേഹം തിങ്കളാഴ്ച്ച രാവിലെ അദ്ദേഹം കുടിക്കുമായിരുന്നു. ചെവ്വാഴ്ച്ച അസ്ർ വരെയും കുടിക്കും. എന്നിട്ടും വല്ലതും അവശേഷിച്ചാൽ ഭൃത്യന് നൽകുകയോ അയാൾക്കും വേണ്ടെങ്കിൽ ഒഴിച്ച് കളയുകയോ ചെയ്യും (സ്വഹീഹു മുസ്‌ലിം: 2004)

അല്ലാഹുവിന്റെ ദൂതൻ (സ) ഒരു യാത്ര പുറപ്പെട്ടു. ശേഷം മടങ്ങിവന്നു. അപ്പോൾ ചില അനുചരർ ഹനാതിം, നകീർ, ദുബ്ബാഅ് എന്നീ പാത്രങ്ങളിൽ നബീദ് തയ്യാറാക്കിയിരുന്നു. അപ്പോൾ (അവയിൽ ലഹരി വന്നതിനാൽ) അത് ഒഴുക്കി കളയാൻ പ്രവാചകൻ (സ) കൽപ്പിച്ചു. ശേഷം തോൽ പാത്രത്തിൽ ഉണക്കമുന്തിരിയും വെള്ളവും നിക്ഷേപിക്കാൻ പറഞ്ഞു. രാത്രി ഉണ്ടാക്കിയത് പിറ്റേന്ന് പകലും വരുന്ന രാത്രിയും കുടിച്ചു. പിറ്റേ ദിവസം വൈകുന്നേരം വരെ കുടിക്കുകയും കുടിപ്പിക്കുകയും ചെയ്തു. അതിന് പിറ്റേന്ന് രാവിലെ ഭാക്കിയായവ ഒഴുക്കി കളയാൻ കൽപ്പിച്ചു. (സ്വഹീഹു മുസ്‌ലിം: 83- 2004)

ഈ ഹദീസുകളെ വ്യാഖ്യാനിച്ചു കൊണ്ട് ഇമാം നവവി (റ) പറഞ്ഞു:

“മൂന്ന് ദിവസം വരെ പ്രവാചകൻ (സ) നബീദ് കുടിക്കുകയും എന്നിട്ടും വല്ലതും അവശേഷിച്ചാൽ ഒഴിച്ച് കളയുകയും ചെയ്യുമായിരുന്നു (സ്വഹീഹു മുസ്‌ലിം: 2004) എന്ന് ഇബ്നു അബ്ബാസ് പറഞ്ഞ ഹദീസും സമാനമായ ഹദീസുകളും തെളിയിക്കുന്നത് നബീദ് ഉണ്ടാക്കുകയും, നബീദ് മധുര പാനീയമായി അവശേഷിക്കുകയും, മാറ്റം വരുകയോ ലഹരി ബാധിക്കുകയോ ചെയ്യാത്തിടത്തോളം കുടിക്കുകയും ചെയ്യൽ അനുവദനീയമാണെന്ന കാര്യത്തിൽ മുസ്‌ലിം സമുദായം മുഴുവൻ ഏകോപിച്ചിരിക്കുന്ന കാര്യമാണ്. മൂന്ന് ദിവസം കഴിഞ്ഞ് പ്രവാചകൻ (സ) അത് കുടിക്കാതെ ഒഴിച്ച് കളയാൻ കാരണം മൂന്ന് ദിവസത്തിനപ്പുറം നബീദിൽ മാറ്റം സംഭവിക്കുകയും ലഹരി ബാധിക്കുകയും ചെയ്യാൻ സാധ്യത ഉള്ളതിനാലാണ്. ലഹരിയുടെ പ്രാധമികമായ വല്ല ലക്ഷണങ്ങളോ മാറ്റമോ നബീദിൽ പ്രത്യക്ഷപ്പെട്ടാൽ അത് പ്രവാചകൻ (സ) ഒഴുക്കി കളയുമായിരുന്നു. കാരണം, ലഹരിയുള്ളതായി മാറിയാൽ നബീദ് ഹറാമും അശുദ്ധവുമാണ്. ഇത്തരം അവസ്ഥയിൽ പ്രവാചകൻ (സ) അവ ഒഴുക്കി കളയും, ഭൃത്യന്മാരെയും കുടിപ്പിക്കില്ല. കാരണം ലഹരിയുള്ള നബീദ് കുടിക്കലും കുടിപ്പിക്കലും അനുവദനീയമല്ല. എന്നാൽ തോൽ പാത്രത്തിൽ സൂക്ഷിച്ച, മൂന്ന് ദിവസ പരിധിക്കുള്ളിലുള്ള നബീദിൽ മാറ്റം സംഭവിക്കുകയോ ലഹരി ബാധിക്കുകയോ ചെയ്യില്ല. ” (ശർഹു മുസ്‌ലിം: 13: 173)

ഇതേ കാര്യം തന്നെ ഇമാം ഇബ്നു കുദാമ തന്റെ ‘മുഗ്നി'(3/144) എന്ന ഗ്രന്ഥത്തിൽ പറയുന്നു.

നബീദ് രണ്ടു വിധമുണ്ടെന്നും, സ്വമേധയാ അവ കേവല പഴച്ചാറും അനുവദനീയമായ പാനീയവുമാണെന്നും, ലഹരി വന്ന നബീദ് (അഥവാ മദ്യം) നിഷിദ്ധമാണെന്നും തെളിയിക്കുന്ന, പ്രവാചകന്റെ(സ) സമകാലികരുടെ വാചകങ്ങൾ ശ്രദ്ധിക്കുക: അലി (റ) പറഞ്ഞു: “ഖംറ് (الخمر) അഥവാ മദ്യമോ ലഹരിയുള്ള നബീദോ കുടിച്ച ആരെയെങ്കിലും എന്റെ അടുത്ത് കൊണ്ട് വരപ്പെട്ടാൽ അയാൾക്ക് (ഭരണാധികാരിയെന്ന നിലക്ക്) ഞാൻ അടി ശിക്ഷയായി നടപ്പാക്കുന്നതാണ്.” (മുസ്നദു ശാഫിഈ: 1322)

പ്രവാചക പൗത്രൻ ഹസനിബ്നു അലിയോട് നബീദിനെ സംബന്ധിച്ച് ചോദിക്കപ്പെട്ടു. അപ്പോൾ അദ്ദേഹം പറഞ്ഞു: “അത് കുടിച്ചു കൊള്ളു. എന്നാൽ അത് ലഹരിയുള്ളതായി മാറാൻ തുടങ്ങിയാൽ ഉപേക്ഷിക്കുക.” (മുസ്വന്നഫ് ഇബ്നു അബീ ശൈബ: 23872)

മുഖ്താർ ബിൻ ഫുൽഫുൽ പറഞ്ഞു: ഞാൻ പ്രവാചകാനുചരൻ അനസ് ബിൻ മാലികിനോട് നബീദിനെ പറ്റി ചോദിച്ചു. അപ്പോൾ അദ്ദേഹം പറഞ്ഞു: “അവയിൽ ലഹരിയുള്ളതെല്ലാം ഉപേക്ഷിക്കുക…” (മുസ്വന്നഫ് ഇബ്നു അബീ ശൈബ: 23803)

ആമിർ പറഞ്ഞു: “നിങ്ങൾ കല്യാണ സദ്യയിലെ നബീദ് കുടിക്കുക. ലഹരിയുള്ള നബീദ് കുടിക്കരുത്.” (മുസ്വന്നഫ് ഇബ്നു അബീ ശൈബ: 23886)

അബുൽ ആലിയ പറഞ്ഞു: “നിങ്ങളുടെ നബീദ് ലഹരിയുള്ളതിനാൽ വളരെ വ്യത്തികെട്ടതായി ഞങ്ങൾ കാണുന്നു. പണ്ടത്തെ നബീദ് ഈത്തപ്പഴം ഇട്ട മധുര വെള്ളമായിരുന്നു.” (സുനനുൽ കുബ്റാ: 34)

നബീദ് സ്വമേധയാ കേവല മധുര പാനീയമാണെന്നും അതിൽ ലഹരി വന്നാൽ പ്രവാചകൻ (സ) അവ സ്വയം വർജിക്കുകയും മറ്റുള്ളവർക്ക് വിലക്കുകയും ചെയ്തിട്ടുണ്ട് എന്ന് തെളിയിക്കുന്ന മറ്റു ചില ഹദീസുകൾ കൂടി കാണുക:

പ്രവാചകൻ (സ) പറഞ്ഞു: “ചോളം കൊണ്ടും തേൻ കൊണ്ടും ലഹരിയുള്ള നബീദ് കുടിക്കുന്നതിനെ സംബന്ധിച്ച് പ്രവാചകനോട് ചോദിച്ചപ്പോൾ അദ്ദേഹം പറഞ്ഞു: ലഹരിയുള്ളത് നിങ്ങൾ കുടിക്കരുത്.” (ബുഖാരി: 4343, മുസ്നദു അഹ്‌മദ്: 19598, 24652,)

സ്വിഹാർ ഇബ്നു സ്വഖ്റുൽ അബ്ദി, നബീദ് കുടിക്കാമോ എന്ന് ചോദിച്ചപ്പോൾ പ്രവാചകൻ (സ) മറുപടി പറഞ്ഞു: “നിങ്ങളുടെ ബോധത്തെ മത്തുപിടിപ്പിക്കാത്ത നബീദ് നിങ്ങൾക്ക് കുടിക്കാം…” (അൽമുഅ്ജമുൽ കബീർ: ത്വബ്റാനി: 7405)

അബ്ദുൽ കൈസ് നിവേദക സംഘത്തോട് പ്രവാചകൻ (സ) പറഞ്ഞു: (നകീർ (പാത്രത്തിൽ) സൂക്ഷിക്കപ്പെട്ട നബീദ് പെട്ടെന്ന് ലഹരി ബാധിക്കുന്നതിനാൽ) നിങ്ങൾ നകീർ (പാത്രത്തിൽ) നബീദ് കുടിക്കരുത്. കുടിച്ചാൽ നിങ്ങൾ (ലഹരി മൂലം) പരസ്പരം അടിപിടി കൂടുന്നതാണ്. (അൽമുഅ്ജമുൽ കബീർ: ത്വബ്റാനി: 122)

ഹദീസിൽ പരാമർശിക്കപ്പെട്ട നബീദിനെ സംബന്ധിച്ച് ചർച്ച ചെയ്ത പൗരാണികരായ ഹദീസ് പണ്ഡിതന്മാരും മനസ്സിലാക്കിയതും പഠിപ്പിച്ചതും ലഹരിയില്ലാത്ത പഴച്ചാറാണ് പ്രവാചകൻ (സ) കുടിച്ചതും അനുവദിച്ചതുമായ നബീദ് എന്നാണ്. അഥവാ അവയിൽ ലഹരി ബാധിച്ചാൽ അത് മദ്യമായി പരിണമിക്കുമെന്നും തദവസരത്തിൽ പ്രവാചകൻ (സ) അത് വലിച്ചെറിയുകയും ചെയ്തിരുന്നു. ഹദീസ് പണ്ഡിതന്മാരുടെ വിശദീകരണങ്ങൾ കാണുക:

1. ‘സ്വഹീഹു മുസ്‌ലിമി’ലെ വിവാദ വിധേയമായ ഹദീസിന്റെ തൊട്ടു മുമ്പത്തെ അധ്യായത്തിന്റെ പേര് തന്നെ ഇപ്രകാരമാണ്.

ﺑﺎﺏ ﺇﺑﺎﺣﺔ اﻝﻧﺒﻴﺬ اﻟﺬﻱ ﻟﻢ ﻳﺸﺘﺪ ﻭﻟﻢ ﻳﺼﺮ ﻣﺴﻜﺮا “മത്ത് വരാത്ത, ലഹരിയുള്ളതായി മാറിയിട്ടില്ലാത്ത നബീദ് അനുവദനീയമാണ്.”

2. ഇമാം നവവി പറഞ്ഞു: “ഉണക്കമുന്തിരിയും അല്ലാത്തതുമായ പഴങ്ങൾ ഇട്ട മധുരമുള്ള വെള്ളമാണ് ഹദീസിൽ പറഞ്ഞിരിക്കുന്ന നബീദ്. ഈ നബീദ് നല്ല രുചിയുള്ളതും എന്നാൽ ലഹരിയില്ലാത്തതുമാണ്. എന്നാൽ ഈ നബീദ് ദീർഘകാലം കൊണ്ട് ലഹരിയുള്ളതായി പരിണമിക്കും. ഇത് ഹറാം (നിഷിദ്ധം) ആകുന്നു.” (ശർഹു മുസ്‌ലിം: 9: 64)

3. ഇമാം ഇബ്നു ഹിബ്ബാന്റെ ‘സ്വഹീഹിലെ’ ചില അധ്യായങ്ങളുടെ പേരുകൾ കാണുക: “ബാർലിയും തേനും കലർത്തിയ മധുര പാനീയമായമായ നബീദ്, ലഹരിയുള്ളതായാൽ അത് നിഷിദ്ധമായി മാറുന്നതാണ്” (സ്വഹീഹു ഇബ്നു ഹിബ്ബാൻ: 12:190)

ലഹരി ബാധിച്ചിട്ടില്ലാത്ത നബീദ് കുടിക്കൽ ഒരാൾക്ക് അനുവദനീയമാണ് (സ്വഹീഹു ഇബ്നു ഹിബ്ബാൻ: 12:207)

“നബീദ് ലഹരി ബാധിച്ച അവസ്ഥയിൽ എത്തിയാൽ പ്രവാചകൻ (സ) അത് കുടിക്കുകയല്ല, വലിച്ചെറിയുകയാണ് ചെയ്തിരുന്നത്.” (സ്വഹീഹു ഇബ്നു ഹിബ്ബാൻ: 12:208)

4. ഇബ്നു അബ്ദി റബ്ബി പറഞ്ഞു: “നിഷിദ്ധമായ കാര്യങ്ങൾക്ക് പകരം അല്ലാഹു അനുവദനീയമായ കാര്യങ്ങൾ ബദലായി നമ്മുക്ക് നൽകിയിട്ടുണ്ട്. പലിശ നിഷിദ്ധമാക്കുകയും കച്ചവടം അനുവദനീയമാക്കുകയും ചെയ്തു. വ്യഭിചാരം നിഷിദ്ധമാക്കുകയും വിവാഹം അനുവദനീയമാക്കുകയും ചെയ്തു. പട്ട് നിഷിദ്ധമാക്കുകയും ചിത്രത്തുന്നലുകൾ അനുവദനീയമാക്കുകയും ചെയ്തു. മദ്യം നിഷിദ്ധമാക്കുകയും ലഹരിയില്ലാത്ത നബീദ് അനുവദനീയമാക്കുകയും ചെയ്തു; എന്നാൽ നബീദ് ലഹരിയുള്ളതായാൽ നിഷിദ്ധമാണ്.” (അൽ ഇക്ദുൽ ഫരീദ്: 8: 67)

5. അബ്ദുർറഹ്മാൻ മുബാറക് പുരി പറഞ്ഞു:

“ഈന്തപ്പഴത്തിൽ നിന്നും മുന്തിരിയിൽ നിന്നും നബീദ് ഉണ്ടാക്കി എന്ന് പറഞ്ഞാൽ അതിനർത്ഥം ആ പഴങ്ങൾ വെള്ളത്തിൽ ഇട്ട് പഴച്ചാറ് എടുത്തു എന്നാണ്. നബീദ് എന്നാൽ ഈ പഴച്ചാറാണ്; അത് ലഹരി ഉള്ളതാവാം ഇല്ലാത്തതാവാം. പഴത്തിൽ നിന്ന് പിഴിഞ്ഞെടുത്ത (ചാറ് കൂടുതൽ കാലം സൂക്ഷിക്കുന്നതിലൂടെ) ലഹരിയുള്ള നബീദിന് ഖംറ് (മദ്യം) എന്നാണ് വിളിക്കപ്പെടുക.” (തുഹ്‌ഫത്തുൽ അഹ്‌വദി: 1: 245)

“ജസ്‌രി നിഹായയിൽ പറഞ്ഞു: ഈത്തപ്പഴം, ഉണക്കമുന്തിരി, തേൻ, ഗോതമ്പ്, ബാർലി തുടങ്ങിയവയിൽ നിന്ന് ഉണ്ടാക്കപ്പെടുന്ന മധുര പാനീയങ്ങളാണ് നബീദ്. ഈന്തപ്പഴത്തിൽ നിന്നും മുന്തിരിയിൽ നിന്നും നബീദ് ഉണ്ടാക്കി എന്ന് പറഞ്ഞാൽ അതിനർത്ഥം ആ പഴങ്ങൾ വെള്ളത്തിൽ ഇട്ട് പഴച്ചാറ് എടുത്തു എന്നാണ്. നബീദ് എന്നാൽ ഈ പഴച്ചാറാണ്; അത് ലഹരി ഉള്ളതാവാം ഇല്ലാത്തതാവാം. രണ്ടിനേയും നബീദ് എന്ന് പറയും. മധുര പാനീയമായിരിക്കുന്നേടത്തോളം, ലഹരിയുടെ അവസ്ഥയിലേക്ക് എത്താത്തിടത്തോളം നബീദ് അനുവദനീയമാണെന്നതിൽ മുസ്‌ലിംകൾക്കിടയിൽ ഏകാഭിപ്രായമുണ്ട്.” (തുഹ്‌ഫത്തുൽ അഹ്‌വദി: 5: 494)

പൗരാണികരായ അറബി ഭാഷാ പണ്ഡിതർ തങ്ങളുടെ നിഖണ്ഡുക്കളിൽ നബീദിനെ സംബന്ധിച്ച് ചർച്ച ചെയ്യുന്നുണ്ട്. നബീദ് സാധാരണ ഗതിയിൽ ഒരു മധുര പാനീയമാണെന്നും അത് ലഹരിയുള്ളതായി പരിവർത്തിപ്പിക്കപ്പെട്ടാൽ മാത്രമാണ് മദ്യമായി മാറുന്നതെന്നും അവരും വ്യക്തമാക്കുന്നു. നബീദ് ലഹരിയുള്ളതും ഇല്ലാത്തതുമുണ്ടെന്നും ലഹരിയുള്ളത് ഖംറ് (മദ്യം الخمر) എന്ന് വിളിക്കപ്പെടുന്നുവെന്നും അറബി ഭാഷാ നിഖണ്ഡുക്കൾ വിശദീകരിക്കുന്നത് കാണുക:

1. “നബദ (ﻧﺒﺬ) എന്നാൽ ഒരു സാധനത്തെ മുമ്പിലോട്ടോ പിന്നിലോട്ടോ ഇടുക എന്നാണ്. നബദ്തു (ﻧَﺒَﺬْﺕُ) എന്നാൽ ഞാൻ ഒരു കാര്യത്തെ എറിഞ്ഞു, ഞാൻ അകലെ ഇട്ടു എന്നൊക്കെയാണർത്ഥം. നബ്ദ് (اﻟﻨَّﺒْﺬُ) എന്നാൽ ഇടുക (اﻟﻄَّﺮْﺡُ) എന്നാണ്. (പഴങ്ങൾ വെള്ളത്തിൽ ‘ഇട്ട്’ ഉണ്ടാക്കപ്പെടുന്ന ഈ നബീദ്) ലഹരി ബാധിക്കാത്തിടത്തോളം ഹലാൽ (അനുവദനീയം) ആകുന്നു. ലഹരി ബാധിച്ചാൽ ഹറാം (നിഷിദ്ധം) ആകുന്നു. നബീദ് എന്ന പദം ഹദീസുകളിൽ ധാരാളമായി ആവർത്തിക്കപ്പെട്ടിട്ടുണ്ട്. ഈത്തപ്പഴം, ഉണക്കമുന്തിരി, തേൻ, ഗോതമ്പ്, ബാർലി തുടങ്ങിയവയിൽ നിന്ന് ഉണ്ടാക്കപ്പെടുന്ന മധുര പാനീയങ്ങളാണ് നബീദ്. നബീദ് എന്നാൽ ഈ പഴച്ചാറാണ്; അത് ലഹരി ഉള്ളതാവാം ഇല്ലാത്തതാവാം. രണ്ടിനേയും നബീദ് എന്ന് പറയും. പഴത്തിൽ നിന്ന് പിഴിഞ്ഞെടുത്ത ചാറ് (നബീദ്) ലഹരിയുള്ളതായാൽ ഖംറ് (മദ്യം) എന്നാണ് വിളിക്കപ്പെടുക.” (ലിസാനുൽ അറബ്: 3: 511)

2. “നബദ എന്നാൽ അകലെ ഇട്ടു, എറിഞ്ഞു എന്നിവയാണ് (ഭാഷാ പരമായ) അർത്ഥം. ഹദീസുകളിൽ പറയപ്പെട്ട നബീദ് ഈത്തപ്പഴം, ഉണക്കമുന്തിരി, തേൻ, ഗോതമ്പ്, ബാർലി തുടങ്ങിയവയിൽ നിന്ന് ഉണ്ടാക്കപ്പെടുന്ന മധുര പാനീയങ്ങളാണ്. ഈന്തപ്പഴത്തിൽ നിന്നും മുന്തിരിയിൽ നിന്നും നബീദ് ഉണ്ടാക്കി എന്ന് പറഞ്ഞാൽ അതിനർത്ഥം ആ പഴങ്ങൾ വെള്ളത്തിൽ ഇട്ട് പഴച്ചാറ് എടുത്തു എന്നാണ്. നബീദ് എന്നാൽ ഈ പഴച്ചാറാണ്; അത് ലഹരി ഉള്ളതാവാം ഇല്ലാത്തതാവാം. രണ്ടിനേയും നബീദ് എന്ന് പറയും. പഴത്തിൽ നിന്ന് പിഴിഞ്ഞെടുത്ത (ജ്യൂസ് കൂടുതൽ കാലം സൂക്ഷിക്കുന്നതിലൂടെ ഉണ്ടാവുന്ന) ലഹരിയുള്ള നബീദിന് ഖംറ് (മദ്യം) എന്നാണ് വിളിക്കപ്പെടുക.” (അന്നിഹായ ഫീ ഗരീബിൽ ഹദീസ്: 5:7)

3. “നബ്ദ് (اﻟﻨَّﺒْﺬُ) എന്നാൽ ഇടുക (اﻟﻄَّﺮْﺡُ) എന്നാണ് അർത്ഥം. നബീദ് ലഹരി ബാധിക്കാത്തിടത്തോളം അനുവദനീയവും ലഹരി ബാധിച്ചാൽ നിഷിദ്ധവുമാകുന്നു. നബീദ് എന്ന പദം ഹദീസുകളിൽ ധാരാളമായി ആവർത്തിക്കപ്പെട്ടിട്ടുണ്ട്. നബീദ് എന്നാൽ ഈ പഴച്ചാറാണ്; അത് ലഹരി ഉള്ളതാവാം ഇല്ലാത്തതാവാം. രണ്ടിനേയും നബീദ് എന്ന് പറയും.” (താജുൽ ഉറൂസ്: 9: 480)

4. “നബ്ദ് (اﻟﻨَّﺒْﺬُ) എന്നാൽ ഒരു കാര്യം സന്നാഹങ്ങൾ ഒരുക്കാതെ ഇടുക എന്നാണ് അർത്ഥം… നബീദ്: ഈത്തപ്പഴം, ഉണക്കമുന്തിരി എന്നിവ വെള്ളം നിറച്ച പാത്രത്തിൽ ഇടുകയാണ്. പിന്നീടത് പ്രത്യേക പാനീയത്തിന്റെ നാമമായി പരിണമിച്ചു.” (അൽ മുഫ്റദാത്ത്: റാഗിബ് അൽ ഇസ്ബഹാനി: 788)

5. “ഈത്തപ്പഴം, ഉണക്കമുന്തിരി തുടങ്ങിയവ രുചി ലഭിക്കാനായി വെള്ളത്തിൽ ഇടുന്നതാണ് നബീദ്; എന്നാൽ മൂന്ന് ദിവസത്തിനപ്പുറം കടന്നുപോകരുത് എന്ന നിബന്ധനയോടെ. മൂന്ന് ദിവസത്തിനപ്പുറം കടന്നുപോയാൽ അത് നിഷിദ്ധമായ മദ്യമായി മാറുന്നു.” (അൽ മുഅ്ജമുൽ വസീത്വ്: പദം: നബദ نبذ)

6. الكلمة: النبيذ. الجذر: نبذ. الوزن: فَعِيل .[النبيذ]: المنبوذ .والنبيذ معروف، وهو من نبذ الشيءَ: إِذا ألقاه، لأنه يُلقى في الإِناء ثم يصب عليه الماء

“നബീദ് എന്നത് പ്രസിദ്ധമായ പദമാണ്. നബ്ദ എന്നാൽ ഒരു വസ്തു ദൂരെ ഇടുക എന്നാണ് അർത്ഥം… നബീദിന് ആ പേര് വരാൻ കാരണം പഴങ്ങൾ പാത്രത്തിൽ ഇടുകയും വെള്ളം ഒഴിക്കുകയും ചെയ്യലാണ്.” (ശംസുൽ ഉലൂം: നിശ്വാൻ ബിൻ സഈദ് അൽ ഹിംയരി: ഹിജ്റാബ്ദം: 573)

7. النبيذ: الشراب المتخذ من عصير التمر والعنب وغيرهما، ج أنبذة (ز) وهو نبيذ سواء كان مسكرا أو غير مسكر ويقال للخمر المعتصرة من العنب نبيذ. كما يقال للنبيذ خمر.

നബീദ്: (النبيذ) ഈത്തപ്പഴം മുന്തിരി പോലുള്ള പഴങ്ങളുടെ ചാറിൽ നിന്ന് ഉണ്ടാക്കുന്ന പാനീയം. നബീദ് എന്നതിന്റെ ബഹുവചനം അൻബിദ (أنبذة) എന്നാണ്. നബീദ് എന്നാൽ ഈ പഴച്ചാറാണ്; അത് ലഹരിയുള്ളതാകട്ടെ അല്ലാത്തതാകട്ടെ. മുന്തിരിയിൽ നിന്ന് ഉണ്ടാക്കപ്പെട്ട വൈനിനെ നബീദ് എന്ന് വിളിക്കപ്പെടാറുണ്ട്. (ലഹരിയുള്ള) നബീദിനെ ഖംറ് (മദ്യം) എന്നും വിളിക്കപ്പെടാറുണ്ട്. (മുഅ്ജമു മത്നുല്ലുഗ: അഹ്മദ് രിദാ)

8. “നബീദ്: ഈത്തപ്പഴം പോലെയുള്ള പഴങ്ങളിൽ നിന്ന് ഉണ്ടാക്കപ്പെടുന്ന എല്ലാ പാനീയത്തിന്റെയും പേരാണ് നബീദ്. ഈത്തപ്പഴ നബീദ് ഞാൻ നൽകി എന്ന ഹദീസിൽ ഉപയോഗിച്ച പദത്തിന്റെ അർത്ഥം ഈത്തപ്പഴം ഇട്ട വെള്ളമാണ്…” (അദുർറു നകിയ് ഫീ ശർഹി അൽഫാദിൽ ഖർകി: 1524: ഇബ്നുൽ മുബർരിദ്: ഹിജ്റാബ്ദം: 909)

9. نبيذ: 1- ملقى، منبوذ. 2- و 3- الشراب عموما.

“നബീദ്: ഇടപ്പെട്ടത്, നിക്ഷേപിക്കപ്പെട്ടത്. (പഴച്ചാറു കൊണ്ടുള്ള) പാനീയങ്ങളെ മൊത്തത്തിൽ നബീദ് എന്ന് പറയുന്നു.” (അർ റാഇദ്: ജിബ്റാൻ മസ്ഊദ്)

10. النبيذ: ما يُتَّخذ التمر والزبيب والعسلِ من غير غليان واشتداد.

“നബീദ്: ഈത്തപ്പഴം, ഉണക്കമുന്തിരി, തേൻ എന്നിവയിൽ നിന്ന് ഉണ്ടാക്കപ്പെടുന്ന -മത്തു വരികയോ ലഹരി ബാധിക്കുകയോ ചെയ്തിട്ടില്ലാത്ത- പാനീയമാണിത്.” (അത്തഅ്’രീഫാത്തുൽ ഫിക്ഹിയ്യ: മുഹമ്മദ് അമീം അൽ ഇഹ്സാൻ)

11. النبيذ: بفتح فسكون فعيل بمعنى مفعول، الملقى والمطروح. الماء الذي ينبذ فيه التمر أو الزبيب أو نحوهما ما لم ينقلب إلى مسكر، فإذا صار مسكرا فهو خمر… (Wine (of grapes, dates وعند الحنفية: الخمر هو النئ من ماء العنب إذا غلا واشتد وقذف بالزبد، وما عداه فهو نبيذ كله.

“നബ്ദ് (اﻟﻨَّﺒْﺬُ) എന്നാൽ ഒരു കാര്യം സന്നാഹങ്ങൾ ഒരുക്കാതെ ഇടുക എന്നാണ് അർത്ഥം… ഈത്തപ്പഴം, ഉണക്കമുന്തിരി തുടങ്ങിയവ ഇട്ട വെള്ളമാണ് നബീദ്; ലഹരിയുള്ളതായി പരിണമിക്കാത്തിടത്തോളം. ലഹരിയുള്ളതായി പരിണമിച്ചാൽ ഈ പഴച്ചാറിനെ ഖംറ് (മദ്യം Wine) എന്ന് വിളിക്കപ്പെടുന്നു.

ഹനഫി പണ്ഡിതർ പറയുന്നു: ഖംറ് എന്നാൽ മുന്തിരി ലഹരിയുള്ളതാവുകയും നുരയുള്ള കലർപ്പാവുകയും ചെയ്ത പാനീയമാണ്. ലഹരി ബാധിക്കാത്തവയാണ് നബീദ്.” (മുഅ്ജമു ലുഗത്തിൽ ഫുകഹാഅ്: മുഹമ്മദ് റവാസ് കൽഅ, ഹാമിദ് സ്വാദിക്)

ഇത്രയും ചർച്ച ചെയ്ത വിഷയത്തിന്റെ സംക്ഷിപ്തമായി ശൈഖുൽ ഇസ്‌ലാം ഇബ്നു തീമിയ്യയുടെ ഒരു വിശദീകരണം കൂടി ഉദ്ധരിച്ചു കൊണ്ട് നമ്മുക്ക് മറുപടിയിൽ നിന്ന് വിരമിക്കാം:

“നബീദ് അഥവാ പഴച്ചാറ് (النبيذ) കുടിക്കുന്നതിൽ ഇളവ് അനുവദിക്കുന്നതായി വന്നിട്ടുള്ള ചില നിവേദനങ്ങൾ ലഹരിയുള്ള പാനീയത്തെ സംബന്ധിച്ച് വന്നതാണെന്ന് ചിലർ തെറ്റിദ്ധരിക്കുകയുണ്ടായി. പഴങ്ങൾ വെള്ളത്തിൽ ഇട്ട് കുതിർത്ത് ലഹരി വരുന്നതിനു മുമ്പുള്ള അവസ്ഥയെയാണ് ‘നബീദ് ‘ കൊണ്ട് മുൻകാലക്കാർ ഉദ്ദേശിച്ചത്. അതല്ലാതെ പിൻകാലത്തോ, മറ്റു നാടുകളിലോ അറിയപ്പെടുന്നത് പോലെ ലഹരിയുള്ള വീഞ്ഞല്ല ‘നബീദ്’ എന്നതിന്റെ ഉദ്ദേശം.

ഇത് പോലെ മറ്റൊരു പദമാണ് ഖംറ് (الخمر) അഥവാ മദ്യം. പല പണ്ഡിതരും വിശ്വസിക്കുന്നത് പോലെ മുന്തിരിയിൽ നിന്നുമുള്ള മദ്യത്തെയല്ല ഇത് സൂചിപ്പിക്കുന്നത്; ഭാഷയിൽ അതാണ് അർത്ഥമെങ്കിലും. മത്തുണ്ടാകുന്ന എല്ലാ പാനീയങ്ങളെയും ആണ് ഖംറ് കൊണ്ട് ഉദ്ദേശിക്കുന്നത് എന്ന് ഹദീസുകൾ വിശദീകരിക്കുന്നു. (ബുഖാരി: 4619, മുസ്‌ലിം: 3032) (റഫ്ഉൽ മലാം അനിൽ അഇമ്മത്തിൽ അഅ്ലാം: 1:27)

സ്വന്തം നാട്ടിൽ നിന്നും മുഹമ്മദ് പുറത്താക്കിയതിന്റെ വിദ്വേഷത്താൽ ഖൈബറിലെ ജൂതന്മാർ മുഹമ്മദ് നബിയെ വിഷം കൊടുത്ത് കൊന്നതാണ്. മുഹമ്മദ് നബിയുടെ ക്രൂരത അസഹ്യമായതിനാലാണ് അപ്രകാരം അവർ ചെയ്തത്. (സ്വഹീഹുൽ ബുഖാരി) മറുപടി:

ഇസ്‌ലാം മതത്തെ സംബന്ധിച്ച പ്രാധമിക വിവരം പോലും വിമർശകർക്കില്ല എന്നതാണ് ഈ ഹദീസ് ദുർവ്യാഖ്യാനത്തിൽ അദിഷ്ടിതമായ കള്ള കഥ തെളിയിക്കുന്നത്. ഹദീസ് സ്വഹീഹ് (സ്വീകാര്യതയുടെ മാനദണ്ഡങ്ങൾ പൂർത്തിയായത്) ആണെന്നതിൽ സംശയമില്ല. പക്ഷെ പ്രവാചകൻ (സ) വിഷബാധയേറ്റാണ് മരിച്ചത് എന്നത് വിവരമില്ലായ്മയിൽ നിന്ന് ഉരുത്തിരിഞ്ഞ ഒരു അത്ഭുതാവഹമായ വാദമാണ്.

ഹിജ്റാബ്ദം ഏഴിനാണ് ഖൈബർ യുദ്ധം നടക്കുന്നത്.

ഇബ്നു ഇസ്ഹാക് (സീറത്തു ഇബ്നു ഇസ്ഹാക്), വാകിദി, ഇബ്നു സഅ്ദ് (ത്വബകാത്ത്) ഇബ്നു ഹജർ അൽ അസ്കലാനി (ഫത്ഹുൽ ബാരി) തുടങ്ങിയ ചരിത്ര പണ്ഡിതരും ഹദീസ് പണ്ഡിതരും ഇക്കാര്യം തെളിവു സഹിതം തങ്ങളുടെ ഗ്രന്ഥങ്ങളിൽ രേഖപ്പെടുത്തിയിട്ടുണ്ട്. (ഗസ്വത്ത് ഖൈബർ ദുറൂസ് വ ഇബർ: അമീർ ബിൻ മുഹമ്മദ് അൽ മുദ്‌രി : 8-9)

പ്രവാചകൻ (സ) മരണപ്പെടുന്നതാകട്ടെ ഹിജ്റാബ്ദം 11 ന് റബിഉൽ അവ്വൽ 12 നാണ്. അപ്പോൾ ഖൈബറിലെ ജൂത സ്ത്രീ ഒരുക്കിയ വിഷക്കെണിക്ക് ശേഷം നാലോ അഞ്ചോ വർഷം പ്രവാചകൻ (സ) വീണ്ടും ജീവിച്ചിട്ടുണ്ട്.

പ്രവാചകൻ (സ) മരണപ്പെടുന്നത് 63 വയസ്സിലാണ് (സ്വഹീഹുൽ ബുഖാരി: 4466)

പ്രവാചക ശിഷ്യൻ ഇബ്നു അബ്ബാസ് (റ) പറഞ്ഞു: അല്ലാഹുവിന്റെ ദൂതൻ (സ) നാൽപതാമത്തെ വയസ്സിൽ പ്രവാചകനായി നിയോഗിതനായി. ദിവ്യബോധനം നൽകപ്പെട്ടുകൊണ്ട് മക്കയിൽ 13 വർഷം താമസിച്ചു. ശേഷം മദീനയിലേക്ക് പലായനം ചെയ്യാൻ കൽപ്പിക്കപ്പെട്ടപ്പോൾ പത്ത് വർഷം മദീനയിൽ താമസിച്ചു. അറുപത്തിമൂന്നാമത്തെ വയസ്സിൽ മരണപ്പെട്ടു. (സ്വഹീഹുൽ ബുഖാരി: 3902, സ്വഹീഹു മുസ്‌ലിം: 2351)

ഈ പത്തു വർഷത്തെ മദീന ജീവിതത്തിൽ ഖൈബറിന് ശേഷം എത്രയെത്ര ചരിത്ര മൂഹൂർത്തങ്ങളിൽ പ്രവാചകൻ (സ) സന്നിഹിതനായി. മരണ സന്ദർഭത്തിൽ പ്രവാചകൻ (സ) ഇപ്രകാരം പറഞ്ഞതായി ചില ഹദീസുകളിൽ കാണാം:

مازالتْ أكْلَةُ خيبرَ تعاوِدُنِي كلَّ عامٍ ، حتى كان هذا أوانُ قطْعِ أَبْهَرِي

ഖൈബറിൽ കഴിച്ച വിഷം ‘എല്ലാ വർഷത്തിലും’ എനിക്ക് ബുദ്ധിമുട്ട് ഉണ്ടാക്കി കൊണ്ടിരുന്നു. എന്റെ രക്തധമനിയെ മുറിക്കുന്ന ഈ (മരണ) അവസരം എത്തിയതു വരെ. (സ്വഹീഹുൽ ജാമിഅ്: 5629, ത്വിബ്ബുന്നബി: അബൂ നുഐം: 83, ത്വിബ്ബുന്നബി: ഇബ്നു സുന്നി)

വിഷത്തിന്റെ സ്വാധീനത്താലുള്ള ചില ബുദ്ധിമുട്ടുകൾ ‘എല്ലാ വർഷത്തിലും’ പ്രവാചകന്(സ) അനുഭവപ്പെട്ടിരുന്നു എന്ന് മരണ സന്ദർഭത്തിൽ അദ്ദേഹം പറയുന്നതിൽ നിന്ന് എന്താണ് മനസ്സിലാവുന്നത്. ഖൈബർ വിഷബാധക്ക് ശേഷം ഒരുപാട് വർഷം അദ്ദേഹം ജീവിച്ചു എന്നു തന്നെ. (മരണ സന്ദർഭത്തിൽ ഈ വിഷത്തിന്റെ സ്വാധീന ഫലം ശക്തമായി അനുഭവപ്പെട്ടിരുന്നു എന്ന് മാത്രം)

തനിക്കു നൽകിയ ഭക്ഷണത്തിൽ ജൂതന്മാർ വിഷം പൂഴ്ത്തിയിരുന്നു എന്ന് ദിവ്യബോധനത്തിലൂടെ അറിഞ്ഞ മുഹമ്മദ് നബി (സ) ഖൈബറിലെ ജൂതന്മാരെ വിളിച്ചു കൂട്ടി നടത്തിയ പ്രഭാഷണമൊക്കെ സ്വഹീഹുൽ ബുഖാരിയിൽ തന്നെയുണ്ട് (ഹദീസ് നമ്പർ: 5441) പിന്നീട് എത്രയോ വർഷങ്ങൾ ജീവിച്ച് എത്രയോ പ്രഭാഷണങ്ങൾ അദ്ദേഹം നടത്തി. വിഷബാധയേറ്റാണ് അദ്ദേഹം മരിക്കുന്നതെങ്കിൽ അഞ്ച് വർഷം കഴിഞ്ഞാണോ മരിക്കുക ?! ഖൈബറിൽ നിന്ന് 180 കിലോമീറ്റർ ദൂരയുള്ള മദീനയിൽ, സ്വപത്നി ആഇശയുടെ വീട്ടിൽ, അവരുടെ മടിയിൽ കിടന്നാണ് പ്രവാചകൻ (സ) മരണപ്പെടുന്നത്. (സ്വഹീഹുൽ ബുഖാരി: 5959) പതിനാല് നൂറ്റാണ്ടുകൾക്കപ്പുറം ഊഷര മരുഭൂമിയിലൂടെ ഒട്ടകപ്പുറത്ത് സഞ്ചരിച്ചാൽ ഖൈബറിനും മദീനക്കുമിടയിൽ ദിവസങ്ങളോളം വരുന്ന യാത്രാ ദൂരം ഉണ്ട് എന്നതും ഖൈബറിലെ വിഷബാധയാണ് പ്രവാചകന്റെ (സ) മരണ കാരണമെന്ന വാദം മഹാവിഡ്ഢിത്തമാണെന്ന് തെളിയിക്കുന്നു.

ഇബ്നു മുഫ്‌ലിഹ് പറഞ്ഞു: പ്രവാചകനോടൊപ്പം ആട്ടിൻ മാംസം കഴിച്ച ചിലർ വിഷത്താൽ മരിക്കുകയുണ്ടായി. മുഹമ്മദ് പ്രവാചകൻ അല്ലെങ്കിൽ വിഷത്താൽ മരിക്കട്ടെ എന്നായിരുന്നു ജൂതന്മാരുടെ ഉദ്ദേശം. എന്നാൽ പ്രവാചകൻ (സ) വർഷങ്ങളോളം ജീവിച്ചു… ജൂതൻമാർക്ക് മറ്റുള്ളവരെ കൊല്ലാൻ കഴിഞ്ഞതു പോലെ പ്രവാചകനെ കൊല്ലാൻ കഴിഞ്ഞില്ല. (അൽ ആദാബു ശറഇയ്യ: 3:190-205)

ഇബ്നു കസീർ പറഞ്ഞു: പ്രവാചകന് (സ) അല്ലാഹു നൽകിയ സംരക്ഷണത്തിൽ പെട്ടതാണ്… ജൂതന്മാർ ഖൈബറിൽ ആടിൻ മാംസത്തിൽ വിഷം തേച്ച് നൽകിയപ്പോൾ അല്ലാഹു ആ വിഷത്തെ പറ്റി അദ്ദേഹത്തിന് അറിയിച്ചു കൊടുക്കുകയും അദ്ദേഹത്തിന് അതിൽ നിന്ന് സംരക്ഷണം നൽകുകയും ചെയ്തു എന്നത്… (തഫ്സീറു ഇബ്നു കസീർ: 2:81)

ശക്തമായ പനിയായിരുന്നു പ്രവാചകന്റെ മരണരോഗം. ഇത് ഒരുപാടു ഹദീസുകളിലൂടെ പ്രവാചകാനുചരന്മാർ പങ്കു വെച്ച വിവരമാണ്. (സ്വഹീഹുൽ ബുഖാരി: 198, 4437, 4436, 5660, സ്വഹീഹു മുസ്‌ലിം: 418) ആ രോഗാവസ്ഥയിൽ വർഷങ്ങൾക്ക് മുമ്പ് ജൂത വഞ്ചനയുടെ ഭാഗമായി കഴിച്ച വിഷത്തിന്റെ സ്വാധീന ഫലം ശക്തമായി അനുഭവപ്പെട്ടിരുന്നു എന്നത് കൊണ്ട് അക്കാരണത്താലാണ് മരിച്ചത് എന്ന് പറയുന്നത് എന്തൊരു വിഡ്ഢിത്തമാണ്. വിഷം കഴിച്ച് ആത്മഹത്യക്ക് ശ്രമിക്കുകയും രക്ഷിക്കപ്പെടുകയും ചെയ്തവർക്ക് ആ വിഷത്തിന്റെ സ്വാധീനഫലങ്ങൾ ജീവിതത്തിൽ പല ഘട്ടങ്ങളിലുമായി അനുഭവപ്പെടാറുള്ളത് ഒരു പൊതു വിവരമാണ്. ഇതു തന്നെയാണ് പ്രവാചകനും അനുഭവപ്പെട്ടത്. ഈ വേദനാനുഭവം പങ്കുവെക്കുക മാത്രമാണ് പ്രവാചകൻ (സ) ചെയ്തത്. അല്ലാതെ വിഷബാധയായിരുന്നു മരണ കാരണമെങ്കിൽ അഞ്ച് വർഷം കഴിഞ്ഞല്ല മരണപ്പെടുക എന്ന് മനസ്സിലാക്കാൻ സെമിനാരിയിൽ പഠിച്ചവർക്ക്‌ പ്രയാസമുണ്ടാകാം. ഖൈബറിലെ ജൂത സ്ത്രീ നൽകിയ വിഷം പ്രവാചകനോടൊപ്പം കഴിച്ച അനുചരൻ ബദ്ർ ഇബ്നു ബറാഅ് (റ) ഉടനെ മരണപ്പെടുകയാണുണ്ടായത് എന്നും ഓർക്കുക. മരണ വേളയിൽ ഒരു രോഗി അസഹ്യമായ തലവേദന അനുഭവിച്ചാൽ തലവേദനയാൽ മരിച്ചു എന്ന് ആരെങ്കിലും വാദിക്കുമോ ?! ജീവിതത്തിൽ ഏതോ ദശയിൽ ഒരു അപകടത്തിൽ പരിക്കേറ്റ വ്യക്തിക്ക് മരണ സമയത്ത് പരിക്കുണ്ടായിരുന്ന ഭാഗത്ത് വേദന അനുഭവപ്പെട്ടാൽ അപകടത്തിൽ മരിച്ചു എന്ന് വ്യാഖ്യാനിക്കാമോ?

സ്വഹീഹുൽ ബുഖാരിയിലെ ഹദീസിലേക്ക് തിരിച്ചു വരാം. സ്വഹീഹുൽ ബുഖാരിയിലെ ഹദീസ് പ്രവാചകന്റെ ക്രൂരതയെ തെളിയിക്കുന്നു എന്നതാണ് അടുത്ത വാദം. ഇതും ശുദ്ധമായ നുണയാണെന്നതിന് പുറമെ പ്രവാചകന്റെ കാരുണ്യമാണ് ഹദീസിൽ സ്ഫുരിക്കുന്നത് എന്നതാണ് യാതാർത്ഥ്യം.

أنَّ امرأةً يَهوديَّةً أتت رسولَ اللَّهِ صلَّى اللَّهُ عليْهِ وسلَّمَ بشاةٍ مسمومةٍ فأَكلَ منْها فجيءَ بِها إلى رسولِ اللَّهِ صلَّى اللَّهُ عليْهِ وسلَّمَ فسألَها عن ذلِكَ ؟ فقالت : أردتُ لأقتلَكَ قالَ : ما كانَ اللَّهُ ليسلِّطَكِ على ذلكَ أو قالَ عليَّ فقالوا ألا نقتلُها قالَ لا قالَ أنس فما زلتُ أعرِفُها في لَهواتِ رسولِ اللَّهِ صلَّى اللَّهُ عليْهِ وسلَّمَ

ഒരു ജൂത സ്ത്രീ അല്ലാഹുവിന്റെ ദൂതന് വിഷം തേച്ച ആട് (സമ്മാനമായി) നൽകുകയുണ്ടായി. അദ്ദേഹം അതിൽ നിന്ന് ഭക്ഷിച്ചു. പിന്നീട് (ജൂത സ്ത്രീ വിഷം തേച്ചതിനെ സംബന്ധിച്ച് അല്ലാഹു അദ്ദേഹത്തെ അറിയച്ചപ്പോൾ) ആ സ്ത്രീയെ അല്ലാഹുവിന്റെ ദൂതന്റെ അടുക്കൽ കൊണ്ട് വരപ്പെട്ടു. അദ്ദേഹം അതിനെ പറ്റി സ്ത്രീയോട് ചോദിച്ചപ്പോൾ അവർ പറഞ്ഞു: താങ്കളെ കൊല്ലാനാണ് ഞാൻ ഉദ്ദേശിച്ചത്. പ്രവാചകൻ (സ) പറഞ്ഞു: അല്ലാഹു അതിനുള്ള പരമാധികാരം നിനക്ക് നൽകില്ല. പ്രവാചകാനുചരന്മാർ ചോദിച്ചു: ഞങ്ങൾ അവൾക്ക് വധശിക്ഷ നൽകട്ടെ ? പ്രവാചകൻ (സ) പറഞ്ഞു: അരുത്. അനസ് പറഞ്ഞു: ആ വിഷത്തിന്റെ അടയാളം പ്രവാചകന്റെ തൊണ്ട ഭാഗങ്ങളിൽ ഞാൻ മനസ്സിലാക്കി കൊണ്ടിരുന്നു. (സ്വഹീഹുൽ ബുഖാരി: 2617, സ്വഹീഹു മുസ്‌ലിം: 2190)

പ്രവാചകന്റെ കാരുണ്യവും അന്നത്തെ ജൂതരുടെ ശത്രുതയും ഈ സംഭവത്തിൽ നിന്നും സുതരാം വ്യക്തമാണ്. തന്നെ വധിക്കാൻ ശ്രമിച്ച സ്ത്രീക്ക് വധശിക്ഷ നൽകട്ടെ എന്ന് ചോദിച്ച അനുചരന്മാരോട് അരുത് എന്നു പറഞ്ഞ ഈ മഹാ മനീഷിയുടെ ഹൃദയ വിശാലതയെയാണ് ഹദീസ് തെളിയിക്കുന്നത്. മത വർഗീയതയില്ലാതെ ജൂത സ്ത്രീ ഒരുക്കിയ വിരുന്നിൽ പങ്കെടുത്ത സഹിഷ്ണുതയെയാണ് ഹദീസ് പരിചയപ്പെടുത്തുന്നത്. മരണം വരെ ജീവിതത്തിന്റെ ഓരോ സന്തിയിലും ജൂത വിഷത്തിന്റെ സ്വാധീനം അനുഭവിച്ചിട്ടും ജൂതന്മാരോട് പക വെച്ചു പുലർത്താതെ… വിജയിച്ചടക്കിയ ഖൈബർ ഭൂമി ജൂതന്മാരുടെ അപേക്ഷ പ്രകാരം അവർക്കു തന്നെ തിരിച്ചു കൊടുത്ത ദയാപരനായ പ്രവാചകന്റെ (സ) ചിത്രമാണ് ഹദീസിൽ നാം ദർശിക്കുന്നത്. വിഷം നൽകിയ സ്ത്രീയുടെ സമൂഹം പ്രവാചകന്റെ മരണം വരെ ഖൈബറിൽ ജീവിച്ചു !!

സ്വഹീഹുൽ ബുഖാരിയിലെ ഹദീസ് പ്രവാചകന്റെ ക്രൂരതയെ തെളിയിക്കുന്നു എന്ന വാദം വിദ്വേഷ പ്രചാരണത്തിന്റെ മിഷണറി സ്റ്റൈൽ മാത്രമാണ്. അഥവാ, പരസ്പരം ബന്ധമില്ലാത്ത കാര്യങ്ങളെ ദുർവ്യാഖ്യാനിച്ച് ബന്ധം മെനഞ്ഞെടുക്കുന്ന മിഷനറി കുബുദ്ധിയുടെ സൃഷ്ടിപ്പാണ് ഈ ദുഷ്പ്രചാരണം. ഇത്തരം ഇല്ലാത്ത ബന്ധങ്ങൾ ദുർവ്യാഖ്യാനിച്ച് ഉണ്ടാക്കാനായിരുന്നെങ്കിൽ അതിന് ഏറ്റവും എളുപ്പം വഴങ്ങുന്ന ഗ്രന്ഥം ബൈബിളാണെന്നതിൽ ആർക്കാണ് സംശയം? യേശുവെ കുരിശിൽ തറച്ചത് യേശുവിന്റെ ക്രൂരമായ ദ്രോഹങ്ങൾ സഹിക്കാൻ കഴിയാത്തത് കൊണ്ടാണ് ജൂതന്മാർ കൊന്നത് എന്ന് വാദിച്ചാൽ എന്തു മറുപടിയാണ് ഈ മിഷണറി ദുർവ്യാഖ്യാനക്കാർക്ക് മറുപടി പറയാനുണ്ടാവുക.?!

പരീശകളെ “കപടവിശ്വാസികളിൽ ഏറ്റവും മോശം” (ബൈബിൾ: മത്തായി : 23:4-36) ജനതയെന്നും ജൂതന്മാരെ “പിശാചിന്റെ മക്കൾ” (ബൈബിൾ: യോഹന്നാന്‍: 8:44) എന്നും അവമതിക്കുകയും ദേവാലയത്തിലെ കച്ചവടക്കാരേയും അവരുടെ ആടുകളേയും കന്നുകാലികളേയും ചാട്ടവാറുകൊണ്ട് അടിച്ച് പുറത്താക്കുകയും നാണയങ്ങൾ തട്ടിതെറിപ്പിക്കുകയും ചെയ്ത (ബൈബിൾ: യോഹന്നാന്‍: 2:15) യേശുവിന്റെ ദ്രോഹങ്ങൾക്കുള്ള മറുപടിയായിരുന്നു ക്രൂശീകരണം എന്നാണ് ജൂതന്മാർക്ക് പറയാനുള്ളത്. ഈ വാദത്തിന് മിഷണറിമാർക്ക് എന്താണോ മറുപടി പറയാനുള്ളത് അതു തന്നെയാണ് മുസ്‌ലിംകൾക്ക് ഖൈബറിൽ പ്രവാചകനെ വിഷം കൊടുത്തു വധിക്കാൻ ശ്രമിച്ച ജൂതന്മാരുടെ കാര്യത്തിലും പറയാനുള്ളത്. (ഇസ്‌ലാം പഠിപ്പിച്ച നീതി ബോധവും സത്യസന്ധതയും ശിരസാവഹിക്കുന്ന മുസ്‌ലിംകളെ സംബന്ധിച്ചിടത്തോളം ബൈബിളിൽ യേശുക്രിസ്തു ചെയ്തെന്നും പഠിപ്പിച്ചെന്നും പറയുന്ന ശുദ്ധ നുണകളും അദ്ദേഹം ക്രൂശീകരിക്കപ്പെട്ടുവെന്ന പെരും നുണയും വിശ്വാസയോഗ്യമല്ല എന്നത് സാന്ദർഭികമായി ഉണർത്തട്ടെ.)

പ്രവാചകന്മാരെ കൊല്ലാനുള്ള ജൂത പ്രവണതയെ സംബന്ധിച്ച് കുർആൻ പലയിടത്തും സംസാരിച്ചിട്ടുണ്ട് (കുർആൻ:4:155, 3:181). ഇത് ആ ജനതയുടെ സ്വഭാവ വൈകൃതത്തിന്റേയും വഞ്ചനാത്മകതയുടേയും തെളിവാണ്. (ഇതിന് അപവാദമായ ഒരു വിഭാഗം അവരിലുണ്ട് എന്നത് വിസ്മരിക്കുന്നില്ല).

മുഹമ്മദ് നബിയുടെ(സ) ക്രൂരതയാണ് അവരെ അതിന് പ്രേരിപ്പിച്ചതെങ്കിൽ ബൈബിൾ തന്നെ കഥ പറഞ്ഞു തരുന്ന പ്രകാരം പ്രവാചകന്മാരായ ആബേൽ, സക്കരിയ്യ, യോസേഫ്, യേശു എന്നിവരെയുമൊക്കെ ജൂതന്മാർ വധിക്കാൻ ശ്രമിച്ചതിലുള്ള പ്രേരണ എന്തായിരുന്നു, അച്ചന്മാരേ ?! ‘പ്രവാചകന്മാരെ കൊല്ലുന്ന ധിക്കാരികളെന്ന്’ ജൂതന്മാരെ സംബന്ധിച്ച് യേശു ക്രിസ്തു തന്നെ പഠിപ്പിച്ചതിന് എന്ത് മറുപടിയുണ്ട് ?!! (ബൈബിൾ: മത്തായി 23:23-31, ലൂക്ക 11:47)

ഖൈബറിലെ ജൂത വിഷം ബാധിച്ചാണ് മുഹമ്മദ് നബി (സ) മരണപ്പെട്ടത് എന്ന വാദത്തിന് തെളിവായ ദുർവ്യാഖ്യാനിക്കപ്പെടുന്ന മറ്റൊരു ഹദീസാണ് ഇത്:

പ്രവാചകൻ (സ) തന്റെ മരണ കാരണമായ രോഗത്തിന്റെ അവസരത്തിൽ പറയുമായിരുന്നു: ആഇശാ, ഖൈബറിൽ ഞാൻ കഴിച്ച ഭക്ഷണത്തിന്റെ വേദന എനിക്ക് ഇപ്പോൾ അനുഭവപ്പെടുന്നു. ഈ ആവസരത്തിൽ ആ വിഷത്തിൽ നിന്ന് എന്റെ രക്തധമനിയെ മുറിക്കുന്ന വേദന എനിക്ക് അനുഭവപ്പെടുന്നു. (സ്വഹീഹുൽ ബുഖാരി: 4428)

മുമ്പ് വിശദീകരിച്ചത് പോലെ ഖൈബറിൽ ജൂത സ്ത്രീ വിഷം നൽകിയതിന് ശേഷം ചുരുങ്ങിയത് നാല് വർഷം പ്രവാചകൻ (സ) ജീവിച്ചിട്ടുണ്ട്. ജൂത സ്ത്രീ നൽകിയ വിഷത്താൽ മരിക്കുന്നതിൽ നിന്ന് അല്ലാഹു പ്രവാചകന് സംരക്ഷണമേകി.

“ദൈവദൂതരേ, നിന്റെ നാഥനില്‍നിന്ന് നിനക്ക് ഇറക്കിക്കിട്ടിയത് ജനങ്ങള്‍ക്കെത്തിച്ചുകൊടുക്കുക… ജനങ്ങളില്‍നിന്ന് അല്ലാഹു നിന്നെ സംരക്ഷിക്കുന്നതാണ്.” (കുർആൻ: 5:67) എന്ന അല്ലാഹുവിന്റെ വാഗ്ദാനമാണ് ഇതിലൂടെ പുലർന്നത്. മുഹമ്മദ് നബി (സ) അല്ലാഹുവിന്റെ ദൂതനാണെന്നാണ് ഈ മുഅ്ജിസത്ത് (അമാനുഷിക ദൃഷ്ടാന്തം) തെളിയിക്കുന്നത്.

“അവർ പറഞ്ഞു: താങ്കളെ കൊല്ലാനാണ് ഞാൻ ഉദ്ദേശിച്ചത്. പ്രവാചകൻ (സ) പറഞ്ഞു: അല്ലാഹു അതിനുള്ള പരമാധികാരം നിനക്ക് നൽകില്ല.” എന്ന ഹദീസിന്റെ ഭാഗത്ത് നിന്ന് മനസ്സിലാക്കാവുന്നത് ജനങ്ങളുടെ ഉപദ്രവത്തിൽ നിന്ന് പ്രവാചന് നൽകപ്പെട്ടിട്ടുള്ള ദിവ്യ സംരക്ഷണമാണ്.

മരണ കാരണമായ വിഷത്തിൽ നിന്ന് സംരക്ഷണം നൽകപ്പെട്ടത് പ്രവാചകന്റെ മുഅ്ജിസത്ത് ആകുന്നു. (ശർഹു മുസ്‌ലിം: 14: 34)

ജൂത സ്ത്രീ നൽകിയ വിഷത്താൽ മരിക്കുന്നതിൽ നിന്ന് അല്ലാഹു പ്രവാചകന് സംരക്ഷണമേകിയെങ്കിലും അതിന്റെ സ്വാധീനത്താലുള്ള ചില ബുദ്ധിമുട്ടുകൾ ചില സന്ദർഭങ്ങളിൽ പ്രവാചകന് അനുഭവപ്പെട്ടിരുന്നു. മരണ സമയത്ത് ഇത് ശക്തമായി അനുഭവപ്പെട്ടു എന്നു മാത്രമാണ് ഈ ഹദീസ് സൂചിപ്പിക്കുന്നത്. അതല്ലാതെ വിഷബാധയേറ്റ് മരിച്ചു എന്നല്ല. മുമ്പ് സൂചിപ്പിച്ചത് പോലെ, ജീവിതത്തിൽ ഏതോ ദശയിൽ ഒരു അപകടത്തിൽ പരിക്കേറ്റ വ്യക്തിക്ക് മരണ സമയത്ത് പരിക്കുണ്ടായിരുന്ന ഭാഗത്ത് വേദന അനുഭവപ്പെട്ടാൽ അപകടത്തിൽ മരിച്ചു എന്ന് വ്യാഖ്യാനിക്കാമോ?

ജൂതന്മാരെ തങ്ങളുടെ നാടായ മദീനയിൽ നിന്ന് പുറത്താക്കിയത് മുഹമ്മദ് നബി (സ) ചെയ്ത ക്രൂരതയല്ലെ എന്നതാണ് അടുത്ത വിമർശനം. ഇന്ന്, ഫലസ്തീനിലെ നാലു വയസ്സുകാരൻ തങ്ങളുടെ ഒരു പട്ടാളക്കാരനെ നോക്കി ഗോഷ്ടി കാണിച്ചു എന്ന ‘ന്യായീകരണവും’ പറഞ്ഞ് അധിനിവേശ രാഷ്ട്രങ്ങളിൽ നിന്ന് ഇരന്നുവാങ്ങിയ മിസൈലുകൾ എൽ.പി. സ്കൂൾ കുട്ടികളുടെ തലയിലേക്ക് വാരിയെറിയുന്ന ജൂത ക്രൂതകളുടെ സമകാലിക ചിത്രം ലോകത്തിന് മുമ്പിലുണ്ട്. അതുകൊണ്ട് തന്നെ, ജൂതന്മാരെ ഹിജാസിൽ നിന്ന് നാടുകടത്തിയതിന്റെ കാരണം മനസ്സിലാക്കി കൊടുക്കാൻ അധികം കഷ്ടപ്പെടേണ്ടതൊന്നുമില്ല.

ജൂതന്മാരെ മദീനയിൽ നിന്ന് പുറത്താക്കാനും ഖൈബർ യുദ്ധം നടത്താനുമുള്ള ചരിത്ര കാരണങ്ങൾ ഹ്രസ്വമായി ഇവിടെ വിവരിക്കാം:

പ്രവാചകൻ (സ) മദീനയിൽ എത്തിയപ്പോൾ മക്കയിൽ നിന്ന് പലായനം ചെയ്തുവന്നവർക്കും (മുഹാജിറുകൾ) മദീനയിലെ മുസ്‌ലിംകൾക്കും (അൻസ്വാറുകൾ) ഇടയിലും ഒരു കരാറുണ്ടാക്കി. മദീനയിലുണ്ടായിരുന്ന ജൂത ഗോത്രങ്ങളെയും ആ കരാറിൽ ഉൾപ്പെടുത്തി. (സീറത്തു ഇബ്നു ഹിശാം: 2/ 147, അൽ അംവാൽ: 1/307, അൻ സാബുൽ അശ്റാഫ്: 1/286, സീറത്തു ഇബ്നു ഇസ്ഹാഖ്:)

ജൂതന്മാർക്ക് അവരുടെ മതവും മുസ്‌ലിംകൾക്ക് അവരുടെ മതവുമനുസരിച്ച് ജീവിക്കാം; അക്രമം പ്രവർത്തിച്ചവരും കുറ്റവാളികളും ഒഴികെ. ഈ കരാറുകാർക്കെതിരെ യുദ്ധം ചെയ്യുന്നവരെ പ്രതിരോധിക്കാൻ കരാറിലേർപ്പെട്ടവർ പരസ്പരം സഹായിക്കണം. കരാറിലുള്ള ഇരു കക്ഷികൾക്കുമെതിരെ (മുസ്‌ലിംകളും ജൂതന്മാരും) പുറത്തു നിന്നും ആരെങ്കിലും യുദ്ധം ചെയ്യുകയാണെങ്കിൽ ശത്രു പക്ഷത്തെ സഹായിക്കരുത്, പരസ്പരം സഹായിക്കണം. പരസ്പരം ഗുണകാംക്ഷയും നന്മയും വെച്ചുപുലർത്തണം. ചരിത്രകാരന്മാരായ ഇബ്നു ഇസ്ഹാഖ് (സീറത്തു ഇബ്നു ഇസ്ഹാഖ്), ഇബ്നു ഹിശാം (സീറത്തു ഇബ്നു ഹിശാം: 2/ 147), വാഖിദി (മഗാസി: 1/176), തബ്‌രി (താരീഖു തബ്‌രി: 2/479), ഇബ്നുൽ അസീർ (അൽ കാമിൽ: 2/96), കാസിം ഇബ്നു സലാം (അൽ അംവാൽ: 1/446), ബലാദുരി (അൻസാബുൽ അശ്റാഫ്: 1/286) തുടങ്ങിയവർ ഈ സമാധാന കരാറിനെ സംബന്ധിച്ച് വിശദീകരിക്കുന്നുണ്ട്.

‘മദീനാ പ്രമാണം’ (وثيقة المدينة) എന്ന പേരിൽ ചരിത്ര പ്രസിദ്ധമായ ഈ കരാർ ഇസ്‌ലാമിക രാഷ്ട്രത്തിന്റെ ആദ്യ ഭരണഘടനയായി പരിചയപ്പെടുത്തപ്പെടുന്നു.

ഇബ്നുൽ ഖയ്യിം (റ) പറയുന്നു: അല്ലാഹുവിന്റെ ദൂതൻ മദീനയിലുണ്ടായിരുന്ന ജൂതന്മാരുമായി സമാധാന സന്ധിയുണ്ടാക്കി. ഇരു കൂട്ടർക്കുമിടയിൽ ഒരു കരാർ പത്രം എഴുതുകയും ചെയ്തു. ജൂതന്മാരുടെ കാരണവരും വേദശാസ്ത്ര പണ്ഡിതനുമായ അബ്ദുല്ല ഇബ്നുസ്സലാം ഈ കരാറിന് മുൻകൈയെടുത്തു. അദ്ദേഹം പിന്നീട് ഇസ്‌ലാം സ്വീകരിച്ചു, സത്യനിഷേധമല്ലാതെ മറ്റു ജുതന്മാരെ ഇസ്‌ലാം സ്വീകരിക്കുന്നതിൽ നിന്ന് തടഞ്ഞില്ല. അവർ മൂന്ന് ഗോത്രങ്ങളായിരുന്നു: ബനൂ ഖൈനുകാഅ്, ബനൂ നളീർ, ബനൂ ഖുറൈള. മൂന്നു കൂട്ടരും മുസ്‌ലിംകൾക്കെതിരെ കലാപമുണ്ടാക്കി, ബനൂ ഖൈനുകാഅ് ഗോത്രത്തോട് പ്രവാചകൻ (സ) അനുകമ്പ കാണിച്ചു, ബനൂ നളീറുകാരെ മദീനയിൽ നിന്ന് ഒഴിപ്പിച്ചു, ബനൂ ഖുറൈളക്കാരോട് ആദ്യം കാരുണ്യം കാണിച്ചെങ്കിലും അവർ യുദ്ധം ചെയ്തപ്പോൾ അവരിലെ പുരുഷന്മാരെ വധിച്ചു. സൂറത്തുൽ ഹശ്ർ അവതരിപ്പിക്കപ്പെട്ടത് ബനൂ നളീറുകാരുടെ വിഷയത്തിലും സൂറത്തുൽ അഹ്സാബ് അവതരിപ്പിക്കപ്പെട്ടത് ബനൂ ഖുറൈളക്കാരുടെ വിഷയത്തിലുമായിരുന്നു. (സാദുൽ മആദ്: 3/58,59)

ജർമ്മൻ ബൈബിൾ പണ്ഡിതനും രേഖാപരികൽപ്പനയുടെ ഉപജ്ഞാതാക്കളിൽ ഒരാളും (documentary hypothesis) ഓറിയന്റലിസ്റ്റുമായ ജൂലിയസ് വെൽ‌ഹൗസെൻ തന്റെ ‘Arab kingdom and it’s fall’ എന്ന ഗ്രന്ഥത്തിൽ (പേജ്: 11) പറയുന്നു:

“മക്കയിൽ നിന്നും മദീനയിലേക്ക് പാലായനം ചെയ്ത ശേഷമുള്ള ആദ്യ ഘട്ടത്തിൽ, ബദർ യുദ്ധത്തിന് മുമ്പായി, മുഹമ്മദ് ഉണ്ടാക്കിയ കരാറിന്റെ രേഖ, കാലം നമുക്കായി സൂക്ഷിക്കുകയുണ്ടായി. മദീനയിലെ ആദ്യ ഘട്ടത്തിലെ പൊതു ജീവിതത്തേയും രാഷ്ട്രീയത്തേയും നിയന്ത്രിച്ചിരുന്നതും സമൂഹത്തിൽ പ്രയോഗവൽകരിച്ചിരുന്നതുമായ നിയമത്തിലെ ചില പ്രധാന പോയിന്റുകൾ പ്രസ്തുത രേഖയിൽ വ്യക്തമാക്കപ്പെടുന്നുണ്ട്… മദീന എങ്ങനെ അഖണ്ഡതയുള്ള ഒരൊറ്റ സമൂഹമായി മാറി എന്ന് അതിൽ നിന്നും മനസ്സിലാക്കാം.” (അവലംബം: ഗ്രന്ഥത്തിന്റെ അറബി പരിഭാഷ)

ഓറിയന്റലിസ്റ്റായ ഡോ. ഇസ്രായേൽ ബെൻസീവ് (History of jews in Arabia: Pre-Islam and Early Islam: 114,115) തെളിവുകൾ നിരത്തി ഈ കരാർ പത്രത്തെ ചരിത്ര വസ്തുതയാണെന്ന് സമർത്ഥിക്കുന്നു. ഇറ്റാലിയൻ ഓറിയന്റലിസ്റ്റായ ലിയോൺ കൈതാനിയും ഈ സമാധാന സന്ധിയുടെ സത്യതയെ അംഗീകരിക്കുന്നുണ്ട്.

മദീനയുടെ വലിയൊരു ഭാഗം യുദ്ധവും കൊലപാതകവും നിഷിദ്ധമാക്കപ്പെട്ട പരിശുദ്ധ ഭൂമിയായി പ്രവാചകൻ (സ) പ്രഖ്യാപിച്ചു.

ജാബിർ (റ) നിവേദനം അല്ലാഹുവിന്റെ ദൂതൻ (സ) പറഞ്ഞു: തീർച്ചയായും ഇബ്രാഹീം മക്കയെ പവിത്രമായി പ്രഖ്യാപിച്ചു; നിശ്ചയം മദീനയെ -രണ്ട് പർവതങ്ങൾക്കിടയിലുള്ള പ്രദേശത്തെ – പവിത്രമായി ഞാനിതാ പ്രഖ്യാപിക്കുന്നു. അവിടെയുള്ള ഒരു വൃക്ഷവും (അനാവശ്യമായി) മുറിക്കപ്പെടരുത്, ഒരു ഉരുവും വേട്ടയാടപ്പെടരുത്.” (സ്വഹീഹുൽ ബുഖാരി: 2889, സ്വഹീഹു മുസ്‌ലിം: 3383, 459, 1363)

ജൂതന്മാരോട് പ്രവാചകൻ (സ) ദീക്ഷിച്ച കാരുണ്യവും നീതിയും, തിരിച്ച് യഹൂദർ ചെയ്തിരുന്ന ദ്രോഹങ്ങളും ശത്രുതയും ചരിത്രത്തിൽ നിന്നും ഹദീസുകളിൽ നിന്നും വായിച്ചെടുക്കാവുന്നതാണ്:

ഖൈബറുകാർ അബ്ദുല്ലാഹിബ്നു സഹ്‌ലിനെ (റ) വധിച്ചപ്പോൾ വ്യക്തമായ തെളിവില്ലാത്തതിനാൽ കൊലയാളികളായ ജൂതരെ പ്രവാചകൻ വെറുതെ വിട്ടു. (സ്വഹീഹുൽ ബുഖാരി: 6769, സ്വഹീഹു മുസ്‌ലിം: 1669)

മുസ്‌ലിമായ അശ്അസ് ഇബ്നു ഖൈസും ഒരു ജൂതനും തമ്മിൽ യമനിലെ ഒരു ഭൂമിയുടെ കാര്യത്തിൽ തർക്കമുണ്ടായി പ്രവാചക സന്നിധിയിൽ വിധിക്കായി വന്നപ്പോൾ അശ്അസിന് തെളിവ് ഹാജരാക്കാൻ കഴിയാഞ്ഞതിനാൽ ഭൂമി ജൂതന്റേതായി പ്രവാചകൻ (സ) വിധിച്ചു. (സ്വഹീഹുൽ ബുഖാരി: 2525, സ്വഹീഹു മുസ്‌ലിം: 138)

ജൂതന്മാർക്ക് അവർക്കിടയിലെ വ്യക്തിപരവും സാമൂഹികവും മതപരവുമായ കാര്യങ്ങളിൽ അവരുടെ മതവും വേദവും വെച്ച് തന്നെ വിധി പറയാനുള്ള സ്വാതന്ത്ര്യം മദീനയിൽ നടപ്പാക്കി.

“…അവര്‍ നിന്‍റെ അടുത്ത് വരുകയാണെങ്കില്‍ അവര്‍ക്കിടയില്‍ നീ തീര്‍പ്പുകല്‍പിക്കുകയോ, അവരെ അവഗണിച്ച് കളയുകയോ ചെയ്യുക. നീ അവരെ അവഗണിച്ച് കളയുന്ന പക്ഷം അവര്‍ നിനക്ക് ഒരു ദോഷവും വരുത്തുകയില്ല. എന്നാല്‍ നീ തീര്‍പ്പുകല്‍പിക്കുകയാണെങ്കില്‍ അവര്‍ക്കിടയില്‍ നീതിപൂര്‍വ്വം തീര്‍പ്പുകല്‍പിക്കുക. നീതിപാലിക്കുന്നവരെ തീര്‍ച്ചയായും അല്ലാഹു സ്നേഹിക്കുന്നു.” (ഖുർആൻ: 5:42)

സ്നേഹ സംവാദങ്ങളുടേയും മത സഹവർത്തിത്വത്തിന്റേയും മാർഗം ഇസ്‌ലാം വേദക്കാർക്ക് മുമ്പിൽ മലർക്കെ തുറന്നിട്ടു: “വേദക്കാരോട് ഏറ്റവും നല്ല രീതിയിലല്ലാതെ നിങ്ങള്‍ സംവാദം നടത്തരുത്‌- അവരില്‍ നിന്ന് അക്രമം പ്രവര്‍ത്തിച്ചവരോടൊഴികെ. നിങ്ങള്‍ (അവരോട്‌) പറയുക: ഞങ്ങള്‍ക്ക് അവതരിപ്പിക്കപ്പെട്ടതിലും നിങ്ങള്‍ക്ക് അവതരിപ്പിക്കപ്പെട്ടതിലും ഞങ്ങള്‍ വിശ്വസിച്ചിരിക്കുന്നു. ഞങ്ങളുടെ ദൈവവും നിങ്ങളുടെ ദൈവവും ഒരുവനാകുന്നു. ഞങ്ങള്‍ അവന് കീഴ്പെട്ടവരുമാകുന്നു.” (ഖുർആൻ: 29:46)

അവർക്ക് ധനം സമ്പാദിക്കാനും ഉടമപ്പെടുത്താനുമുള്ള പൂർണമായ സ്വാതന്ത്ര്യം നൽകി. ആഇശ (റ) പറഞ്ഞു: അല്ലാഹുവിന്റെ ദൂതൻ (സ) തന്റെ പടയങ്കി ജൂതന് പണയം വെച്ച് ധാന്യം കടം വാങ്ങുകയുണ്ടായി (സ്വഹീഹുൽ ബുഖാരി:1990, സ്വഹീഹു മുസ്‌ലിം: 1603)

കൊലയും കൊള്ളയുമായിരുന്നില്ല മദീനയിലെ ഇസ്‌ലാമിക സാമൂഹിക വ്യവസ്ഥ. മദീനയിലെ ഭരണാധികാരിയായിട്ടും ജൂതന്മാരിൽ നിന്ന് – അവർ യുദ്ധം ചെയ്യും വരെ – സമ്പത്ത് തട്ടിയെടുക്കുകയോ പിടിച്ച് പറിക്കുകയോ ചെയ്തിട്ടില്ല.

ജൂതന്മാരുടെ ഗൂഢാലോചനയുടെ ഭാഗമായി ലബീദിബ്നു അഅ്സം എന്ന ജൂതൻ പ്രവാചകനെ വധിക്കാനായി അദ്ദേഹത്തിന് (സ) മാരണം ചെയ്ത സംഭവവും ഹദീസുകളിൽ കാണാം. (സ്വഹീഹുൽ ബുഖാരി: 3268, സ്വഹീഹു മുസ്‌ലിം :2189)

പ്രവാചകന് (സ) ജൂതന്മാർ മാരണം ചെയ്തെന്ന് മുസ്‌ലിംകൾ അറിഞ്ഞാൽ അത് ജൂതന്മാർക്കും മുസ്‌ലിംകൾക്കുമിടയിൽ ശത്രുതക്ക് കാരണമായേക്കാം എന്നതിനാൽ ജനങ്ങൾ അധികം കാണാതെ അതു നശിപ്പിച്ചു കളയാൻ പ്രവാചകൻ (സ) നിർദേശിച്ചു. (സ്വഹീഹുൽ ബുഖാരി: 3268)

ആ ജൂതനോട് അതിനെ പറ്റി പ്രവാചകൻ (സ) ഒരിക്കൽ പോലും സ്മരിക്കുകയോ എന്തെങ്കിലും പരിഭവം മുഖത്ത് കാട്ടുകയോ ചെയ്തില്ല. (സുനനു നസാഈ: 4080)

എന്നാൽ ജൂത സമൂഹം മുസ്‌ലിംകളുമായി യാതൊരുവിധ സഹവർത്തിത്വത്തിനും തയ്യാറായിരുന്നില്ല. ബനൂ നളീർ, ഖുറൈള എന്നിവർ കരാർ ലംഘിക്കുകയാണുണ്ടായത്.

“അവര്‍ (യഹൂദര്‍) ഏതൊരു കരാര്‍ ചെയ്തു കഴിയുമ്പോഴും അവരില്‍ ഒരു വിഭാഗം അത് വലിച്ചെറിയുകയാണോ? തന്നെയുമല്ല, അവരില്‍ അധികപേര്‍ക്കും വിശ്വാസം തന്നെയില്ല. (ഖുർആൻ 2: 100)

മക്കക്കാർ മുസ്‌ലിംകൾക്കെതിരെ യുദ്ധം ചെയ്തപ്പോൾ ഖുറൈളക്കാർ സഹായിക്കാതെ വിട്ടു നിന്നു എന്നതു മാത്രമായിരുന്നു അവരുടെ കരാർ ലംഘനം എന്നതിനാൽ മദീനയിൽ തന്നെ താമസിക്കാൻ പ്രവാചകൻ (സ) അനുവദിച്ചു. അവരോട് കരുണ കാണിക്കുകയും നാടുകടത്താതിരിക്കുകയും ചെയ്തു. എന്നാൽ ബനൂ നളീറുകാർ കരാർ ലംഘിച്ച് മുസ്‌ലിംകളെ സഹായിക്കാതെ വിട്ടു നിൽക്കുക മാത്രമല്ല ചെയ്തത്. മറിച്ച് മക്കയിലേക്ക് ആളെ അയച്ച് യുദ്ധത്തിനായി അവരെ പ്രോത്സാഹിപ്പിക്കുകയും മുസ്‌ലിം സൈന്യത്തിന്റെ ദുർബലതകൾ അറിയിച്ചു കൊടുക്കുകയും ചെയ്തു. (മഗാസി: മൂസാ ഇബ്നു ഉഖ്ബ: 210)

ബദർ യുദ്ധത്തിന് ശേഷം മദീനയിലേക്ക് യാത്ര ചെയ്ത ബനൂ നളീറുകാരനും കവിയുമായിരുന്ന കഅ്ബിബ്നു അശ്റഫ് കുറൈശികളിൽ നിന്ന് യുദ്ധത്തിൽ കൊല്ലപെട്ടവർക്കായി വിലാപകാവ്യം രചിക്കുകയും യുദ്ധത്തിനായി അവരെ പ്രചോദിപ്പിക്കുകയും ചെയ്തു. കഅ്ബയുടെ വിരിയിൽ പിടിച്ച് മുസ്‌ലിംകളോട് യുദ്ധം ചെയ്യുമെന്ന് ഖുറൈശികളെ കൊണ്ട് കരാർ ചെയ്യിപ്പിച്ചു. (ഫത്ഹുൽ ബാരി: 7/337) മദീനയിൽ വന്ന ശേഷം പ്രവാചകനെ ആക്ഷേപിച്ചു കൊണ്ട് കവിത പാടി. പ്രകോപനം സൃഷ്ടിക്കാനായി പ്രവാചകാനുചരന്മാരുടെ ഭാര്യമാരെ അസഭ്യവാക്കുകൾ കൊണ്ട് വർണിച്ച് ഗസലുകൾ പാടി. ഈ സംഭവം ഒക്‌ലഹോമ സർവകലാശാലയിലെ ജൂഡായിക് ചരിത്രത്തിന്റെ എമെറിറ്റസ് ചെയറും ഒറിയന്റലിസ്റ്റുമായ നോർമൻ ആർതർ സ്റ്റിൽമാൻ തന്റെ ഗ്രന്ഥത്തിൽ എഴുതി ചേർത്തിട്ടുണ്ട്. (The Jews of Arab Lands: A History and Source Book: Page: 124, The Jewish publication society of America)

ദിയ്യത്തുമായി ബന്ധപ്പെട്ട് സഹായത്തിനായി ബനൂ നളീറുകാരുടെ അടുത്ത് ഒരുപറ്റം അനുചരന്മാരോടൊപ്പം പ്രവാചകൻ (സ) ചെന്നപ്പോൾ അദ്ദേഹത്തെ ബനൂ നളീറുകാർ ഒരിടത്ത് ഇരുത്തി. ഹുയൈയ് ഇബ്നു അഖ്തബ് ജൂതന്മാരോട് പറഞ്ഞു: “ജൂത സമൂഹമേ, മുഹമ്മദ് ഇതാ ഒരു ചെറിയ അനുയായി വൃന്ദവുമായി വന്നിരിക്കുന്നു. അവർ പത്തുപേർ പോലുമില്ല. അവർ ഇരിക്കുന്ന ഭവനത്തിന് മുകളിൽ നിന്ന് കല്ല് താഴേക്കിട്ട് മുഹമ്മദിനെ കൊല്ലുക. മുഹമ്മദിനെ കൊല്ലാൻ ഇതിലും നല്ല അവസരമില്ല.” അംറിബ്നു ജിഹശ് ഈ ദൗത്യമേറ്റെടുത്തു, ഭവനത്തിന് മുകളിൽ കയറി കല്ലെടുത്തു. പക്ഷെ ജൂതന്മാരുടെ ഈ കുതന്ത്രം അല്ലാഹു ദിവ്യഉൽബോധനം വഴി പ്രവാചകന് അറിയിച്ചു കൊടുത്തു. അദ്ദേഹം അവിടെ നിന്ന് പെട്ടെന്ന് എഴുന്നേറ്റു പോയി. (The Jews of Arab Lands: A History and Source Book: Page: 129, 130.The Jewish publication society of America )

ഇബ്നു ഉമർ (റ) പറഞ്ഞു: ബനൂ നളീർ, ഖുറൈള എന്നീ ജൂത ഗോത്രങ്ങൾ അല്ലാഹുവിന്റെ ദൂതനോട് യുദ്ധം ചെയ്തു. അപ്പോൾ അല്ലാഹുവിന്റെ ദൂതൻ (സ) അവരെ (മദീനയിൽ നിന്നും ബനൂ നളീറുകാരെ) പുറത്താക്കി. ഖുറൈളക്കാരെ (അവർ യുദ്ധത്തിൽ നേരിട്ട് പങ്കാളികളായില്ല എന്നതിനാൽ പ്രവാചകൻ (സ) മദീനയിൽ) താമസിപ്പിച്ചു; അതിന് ശേഷം ഖുറൈളക്കാരും യുദ്ധം ചെയ്യുന്നത് വരെ. (സ്വഹീഹുൽ ബുഖാരി: 4028, സ്വഹീഹു മുസ്‌ലിം: 1766)

‘നിങ്ങൾക്ക് കോട്ടയും സുരക്ഷയുമുണ്ടായിരിക്കെ നിങ്ങൾ എന്ത് കൊണ്ട് മുഹമ്മദിനെതിരെ സംഘടിക്കുന്നില്ല ?’ എന്ന് ആരാഞ്ഞു കൊണ്ട് ബദർ യുദ്ധാനന്തരം മക്കയിലെ ഖുറൈശികൾ മദീനയിലെ ജൂതന്മാർക്ക് കത്തെഴുതി. അങ്ങനെ ബനൂ നളീറുകാർ പ്രവാചകനെ ചതിക്കാൻ ഗൂഢാലോചന നടത്തി. പ്രവാചകനോട് അദ്ദേഹത്തിന്റെ മൂന്ന് അനുയായികളെ തങ്ങളിലേക്ക് അയക്കാനും ഞങ്ങളിലെ മൂന്ന് പുരോഹിതന്മാരുമായി അവർ സംസാരിച്ചതിന് ശേഷം അവർ ഇസ്‌ലാമിൽ വിശ്വസിക്കുകയാണെങ്കിൽ ഞങ്ങൾ മുഴുവനും ഇസ്‌ലാം സ്വീകരിക്കാം എന്നും പറഞ്ഞു. ഇത് കേട്ട് പ്രവാചകൻ (സ) അപ്രകാരം ദൂതന്മാരെ അയച്ചു. ബനൂ നളീറുകാർ സായുധരായി ഈ അനുചരന്മാരെ വധിക്കാനായി ഒരുങ്ങി നിന്നു. ബനൂ നളീറുകാരിൽ പെട്ട ഒരു സ്ത്രീ അൻസ്വാരികളിൽ പെട്ട തന്റെ സഹോദരന് ബനൂ നളീറുകാരുടെ ചതിയെ പറ്റി രഹസ്യമായി വിവരമറിയിച്ചു. ഇതറിഞ്ഞ പ്രവാചകൻ (സ) അനുചരന്മാരെ തിരിച്ചു വിളിച്ചു. ഈ വഞ്ചനക്കും കരാർ ലംഘനത്തിനും പകരമായി മുസ്‌ലിംകൾ ബനൂ നളീറുകാരുടെ കോട്ട ഉപരോധിച്ചു. ഉപരോധം ശക്തമായപ്പോൾ മദീനയിൽ നിന്നും നാടുകടത്തപെടുക എന്ന നിബന്ധനയോടെ അവർ കീഴടങ്ങി. ഈ വഞ്ചനകൾക്ക് ശേഷവും മദീനയിൽ താമസിക്കാൻ കഴിയില്ലെന്നും പത്ത് ദിവസം കൊണ്ട് ഒഴിഞ്ഞു പോകണമെന്നും മുഹമ്മദ് ഇബ്നു മസ്‌ലമയെ ദൂതനായി അയച്ച് ബനൂ നളീറുകാരെ പ്രവാചകൻ (സ) വിവരമറിയിച്ചു. സംഭവം സ്വഹീഹായ സനദു സഹിതം ഇമാം ഇബ്നു ഹജർ അൽ അസ്കലാനി തന്റെ ‘ഫത്ഹുൽ ബാരി’യിൽ (7/331, 332) ഉദ്ധരിക്കുന്നുണ്ട്.

കുറച്ച് കാലങ്ങൾക്ക് ശേഷം മക്കക്കാർ മുസ്‌ലിംകൾക്കെതിരെ ഖന്ദക്ക് യുദ്ധത്തിന് നേതൃത്വം നൽകിയപ്പോൾ ഖുറൈളക്കാർ അവരോടൊപ്പം സഖ്യം ചേരുകയും മദീനയിൽ ഒറ്റപ്പെട്ട സ്ത്രീകളെ ആക്രമിക്കുകയും ചെയ്തപ്പോൾ മാത്രമാണ് അവരേയും മദീനയിൽ നിന്ന് പ്രവാചകൻ (സ) പുറത്താക്കിയത്. (സീറത്തു ഇബ്നു ഹിശാം: 3/ 231- 233)

“അവരില്‍ ഒരു വിഭാഗവുമായി നീ കരാറില്‍ ഏര്‍പെടുകയുണ്ടായി. എന്നിട്ട് ഓരോ തവണയും തങ്ങളുടെ കരാര്‍ അവര്‍ ലംഘിച്ചുകൊണ്ടിരുന്നു. അവര്‍ (അല്ലാഹുവെ) സൂക്ഷിക്കുന്നുമില്ല.” (ഖുർആൻ 8: 56) ഈ വചനവും അവതരിപ്പിക്കപ്പെട്ടതും ബനൂ ഖൈനുകാഉകാരുടെ കാര്യത്തിലാണെന്ന് ഇമാം മുജാഹിദ് വ്യക്തമാക്കുന്നു. (അദ്ദുർറുൽ മൻസൂർ: സുയൂത്വി: 3/191, മഗാസി: വാഖിദി: 1/135)

ഖന്ദക്ക് യുദ്ധത്തോടെ മുസ്‌ലിംകളുമായുള്ള സന്ധി റദ്ദു ചെയ്തുവെന്ന് പരസ്യമായി പ്രഖ്യാപിച്ച ബനൂ ഖുറൈളക്കാരോട് കരാർ പുനസ്ഥാപിക്കാൻ ആവശ്യപ്പെട്ട് പ്രവാചകൻ നിയോഗിച്ച സഅ്ദിബ്നു ഉബാദ സഅ്ദിബ്നു മുആദ് (റ) എന്നിവരെ ബനൂ ഖുറൈളക്കാർ ശകാരിക്കുകയും തിരസ്കരിക്കുകയും ചെയ്തു. (സീറത്തു ഇബ്നു ഹിശാം: 2/222)

മുസ്‌ലിംകൾ ഖന്ദക്ക് യുദ്ധത്തിൽ മുഴുകിയിരിക്കെ മദീനയിലെ സ്ത്രീകളെ ആക്രമിക്കാൻ ബനൂ ഖുറൈളക്കാർ പദ്ധതിയിട്ടു. മുസ്‌ലിംകൾ സ്ത്രീകളേയും കുട്ടികളേയും താമസിപ്പിച്ചിരിക്കുന്ന കോട്ടയെ ഖുറൈളക്കാരിലെ ദൂതൻ വലം വെച്ചു. അയാൾ ഉള്ളിലേക്ക് എത്തി നോക്കിയപ്പോൾ സ്വഫിയ്യ ബിൻത്ത് അബ്ദുൽ മുത്വലിബ് (റ) കുന്തം കൊണ്ട് അയാളെ കുത്തിയിട്ടു. മുസ്‌ലിംകൾ കോട്ടയിലും സൈന്യത്തെ വിന്യസിച്ചിട്ടുണ്ടെന്ന് ബനൂ ഖുറൈളക്കാർ തെറ്റിദ്ധരിക്കുകയും ചെയ്തു. (അൽ ഇസ്വാബ: 2/56, 8/214, മുസ്തദ്റഖു ഹാകിം:6945, മജ്മഉ സവാഇദ്: 6/115)

ബനൂ ഖൈനുകാഅ് ഗോത്രത്തിൽ പെട്ടവർ ഒരു മുസ്‌ലിം സ്ത്രീയുടെ വസ്ത്രം ബലം പ്രയോഗിച്ച് അഴിക്കുകയും അവളുടെ നഗ്നത കണ്ട് കൂട്ടത്തോടെ ചിരിക്കുകയും ചെയ്തു. ഉടനെ ഒരു മുസ്‌ലിം, ആ സ്ത്രീയുടെ വസ്ത്രം വലിച്ചൂരിയ വ്യക്തിയെ വധിച്ചു. ബനൂ ഖൈനുകാഅ് സംഘം അദ്ദേഹത്തെ വധിച്ചു. ഈ സംഭവവുമായി ബന്ധപ്പെട്ട കലാപത്തെ തുടർന്നാണ് ബനൂ ഖൈനുകാഉകാർ നാടുകടത്തപെട്ടത്. (സീറത്തു ഇബ്നു ഹിശാം: 2/642, അൽ മഗാസി: വാകിദി: 1/176, 177)

ഈ കലാപകാരികളും യുദ്ധ കൊതിയരുമായ സമൂഹത്തെയാണോ മുഹമ്മദ് നബിയുടെ ക്രൂരതകളുടെ ഇരകളായി വിമർശകർ ദുർവ്യാഖ്യാനിക്കുന്നത് ?! ഇത്തരം കുൽസിത പ്രവർത്തനങ്ങളുടേയും രാജ്യദ്രോഹത്തിന്റെയും പേരിലാണ് മദീനയിൽ നിന്നും ജൂതന്മാരെ പ്രവാചകൻ (സ) പുറത്താക്കിയത്. എന്നിട്ടും ഖൈബറിൽ താമസമാക്കിയ ജൂത സമൂഹം മുസ്‌ലിം സമൂഹത്തേയും രാഷ്ട്രത്തേയും വെറുതെ വിടാൻ തയ്യാറായിരുന്നില്ല ! മദീനയിൽ നിന്നും നാടുകടത്തപ്പെട്ട ബനൂ നളീറുകാർക്ക് ഖൈബറിലെ ജൂതർ അഭയം നൽകി, അവർ അറബികളിലെ പല ഗോത്രങ്ങളേയും മുസ്‌ലിംകൾക്കെതിരെ യുദ്ധത്തിനായി പ്രേരിപ്പിക്കുവാനും ആരംഭിച്ചു. ബനൂ നളീറുകാരടങ്ങുന്ന ഖൈബറുകാർ ഗോത്രങ്ങളെ സംഘടിപ്പിക്കുകയും ഖുറൈശികളെ പ്രലോഭിപ്പിക്കുകയും ചെയ്തതിലൂടെയാണ് ഖുറൈശികൾ മുസ്‌ലിംകൾക്കെതിരെ ഖന്ദഖ് യുദ്ധം നടത്തുന്നത്. (ഉംദത്തുൽ കാരി: ഇമാം അൽ ഐനി: 17/176, സീറത്തു ഇബ്നു ഹിശാം: 2/441, സിയറു അഅ്ലാമിന്നു ബലാഅ്: 1/457, അൽ ബിദായ വന്നിഹായ: 8/50)

ഹിജ്റ ഏഴാം വർഷം നടന്ന ഖൈബർ യുദ്ധത്തിനൊടുവിൽ ജൂതർ പരാജയപ്പെട്ടു. ജൂതന്മാരെ ഒന്നടങ്കം ഖൈബറിൽ നിന്നും നാടുകടത്താൻ മുസ്‌ലിംകൾ തീരുമാനിച്ചെങ്കിലും ജൂതർ പ്രവാചകനോട് സന്ധിക്കായി കേണു. ഖൈബർ യുദ്ധാനന്തരവും ജൂതന്മാരോട് അനുനയത്തോടെയും കരുണയോടെയും പെരുമാറാൻ പ്രവാചകൻ (സ) മടിച്ചില്ല.

ഇബ്നു ഉമർ (റ) പറഞ്ഞു: അല്ലാഹുവിന്റെ റസൂൽ (സ) ഖൈബർ ദേശം യഹൂദന്മാർക്ക് തന്നെ നൽകി, അവർ അതിൽ പ്രവർത്തിക്കുകയും കൃഷിചെയ്യുകയും അതിന്റെ വിളവിന്റെ പകുതി മുസ്‌ലിംകൾക്ക് നൽകുകയും ചെയ്യുക എന്ന നിബന്ധനയോടെ. (സ്വഹീഹുൽ ബുഖാരി: 2165, സ്വഹീഹു മുസ്‌ലിം: 1551)

ഖൈബറിലെ ജൂതരോട് ഇത്തരമൊരു അനുനയ സമീപനം സ്വീകരിച്ചിട്ടും അവർ തിരിച്ച് വെച്ചുപുലർത്തിയ നയം ശത്രുത തന്നെയായിരുന്നു. ഈ ചരിത്ര ദശയിലാണ് ഖൈബറിലെ ജൂതന്മാരിൽ പെട്ട ഒരു സ്ത്രീ പ്രവാചകനേയും(സ) അനുയായികളേയും വിരുന്നു വിളിക്കുകയും വിഷം തേച്ച ആട്ടിൻ മാംസം വിളമ്പുകയും ചെയ്തത്. വിഷം ചേർത്തിട്ടുണ്ടെന്ന് അറിഞ്ഞപ്പോൾ ആ സ്ത്രീയെ വധിക്കട്ടെയെന്ന് അനുചരന്മാർ ചോദിച്ചുവെങ്കിലും പ്രവാചകൻ വേണ്ടെന്ന് പറഞ്ഞു. (സ്വഹീഹുൽ ബുഖാരി: 3169)

ഒരിക്കലും സമാധാന സന്ധികൾ നിലനിർത്താനും യുദ്ധ കലാപ നിരോധന കരാറുകൾ പാലിക്കാനും തയ്യാറല്ലാത്ത സമൂഹങ്ങൾ മദീനയിൽ നിലനിൽക്കുമ്പോൾ മക്കയിലേയും മദീനയിലേയും പുണ്യ ഭൂമി, യുദ്ധം നിഷിദ്ധമാക്കപ്പെട്ട ഹറമുകൾ ചോരയാൽ ചുവന്നുകൊണ്ട് തന്നെ നിലനിൽക്കും. അരക്ഷിതത്വവും അഹിംസയും തീരാ കഥകളാകുമെന്ന ബോദ്ധ്യത്തിൽ നിന്നാണ് ജൂതന്മാരെ ഹിജാസിൽ നിന്നും പുറത്താക്കാൻ പ്രവാചകൻ (സ) നിർബന്ധിതനാകുന്നത്. ജൂത ക്രൈസ്തവരടങ്ങുന്ന അമുസ്‌ലിംകൾ മക്കാ മദീന പള്ളികൾ ഒഴികെയുള്ള, ഹിജാസിലെ മറ്റു സ്ഥലങ്ങളിൽ താമസിക്കുന്നത് നിഷിദ്ധമായി ഇസ്‌ലാം കാണുന്നില്ല. അതുകൊണ്ടാണ് പ്രവാചകൻ (സ) തന്നെയും മരണം വരെ അവരെ പുറത്താക്കാതിരുന്നത്. പുറത്താക്കണമെന്ന് പറഞ്ഞതാകട്ടെ ജീവിതത്തിന്റെ അവസാന ശ്വാസം വരെ ജൂതന്മാരും ഖുറൈശികളും ഹിജാസിൽ സമാധാനാന്തരീക്ഷം പുലരുവാൻ സമ്മതിക്കില്ല എന്ന് മനസ്സിലാക്കിയപ്പോഴാണ്.

പ്രവാചകനായി നിയോഗിതനായപ്പോഴോ, മക്കയിൽ നിന്നും മദീനയിലേക്ക് വന്ന ഉടനെയോ, മദീനയുടെ ഭരണാധികാരി ആയ ഉടനെയോ ‘ജൂതന്മാരെ മദീനയിൽ നിന്ന് പുറത്താക്കണമെന്ന്’ പ്രവാചകൻ പറഞ്ഞിരുന്നെങ്കിൽ അദ്ദേഹത്തെ വർഗീയ വാദിയെന്നും ക്രൂരനെന്നും വിളിക്കുന്നതിന് ന്യായം കണ്ടെത്താമായിരുന്നു.

നബി(സ്വ)യും ‘ശവരതി’യാരോപണവും

വിമര്‍ശനം:

നബിജീവിതം വിശുദ്ധമായിരുന്നു എന്ന ധാരണ തെറ്റാണെന്ന് വ്യക്തമാക്കുന്ന സംഭവമാണ് ഫാത്തിമ ബിന്‍ത് അസദ് എന്ന സ്ത്രീയുടെ മൃതദേഹത്തെ ഖബ്റില്‍ (ശവക്കുഴി) വെച്ച് ഭോഗിച്ച പ്രവാചക നടപടി. ശവരതിപോലും നടത്താന്‍ മടിയില്ലായിരുന്ന ഒരാളെയാണ് മുസ്‌ലിംകള്‍ മാര്‍ഗദര്‍ശകനായി ഉള്‍ക്കൊള്ളുന്നത് എന്നത് എത്രമാത്രം ലജ്ജാവഹമാണ്. രക്തബന്ധമോ കുടുംബബന്ധമോ പോലും പരിഗണിക്കാതെ ഏത് സ്ത്രീയെയും ഭോഗിക്കുവാന്‍ നബിക്ക് അല്ലാഹു അനുവാദം നല്‍കുന്നതിനു മുമ്പായിരുന്നു (ക്വുര്‍ആന്‍ 33:50) ഈ ശവരതി നടന്നതെന്നു കാണാം. സ്ത്രീകളെ കേവലം ലൈംഗിക ഉപഭോഗ വസ്തു മാത്രമായി കണ്ടിരുന്ന ഒരാള്‍ എങ്ങിനെയാണ് ദൈവദൂതനും മാര്‍ഗദര്‍ശകനുമാവുക!?.

മറുപടി:

പ്രശസ്ത മനഃശാസ്ത്രജ്ഞന്‍ സ്റ്റീവന്‍ സ്റ്റോസ്നി (Steven Stonsy phD) ഓസ്‌കാര്‍ വൈല്‍ഡിന്റെ വാചകങ്ങള്‍ ഉദ്ദരിച്ചുകൊണ്ട് ഇപ്രകാരം പറയുകയുണ്ടായി ”ഓസ്‌കാര്‍ വൈല്‍ഡ് ഒരിക്കല്‍ പറഞ്ഞതുപോലെ ‘ആത്മകഥയുടെ വിശ്വസനീയമായ ഒരേയൊരു രൂപമാണ് നിരൂപണം’. ഒരു നിരൂപണം നിരൂപകന്‍ വിമര്‍ശിക്കുന്ന ആളുകളേക്കാള്‍ – അത് പുരുഷനാകട്ടെ സ്ത്രീയാകട്ടെ – കൂടുതല്‍ നിരൂപകന്റെ മനഃശാസ്ത്രത്തെയാണ് നമ്മോട് പറഞ്ഞു തരുന്നത്. ഒരാളുടെ വിമര്‍ശനങ്ങള്‍ കേട്ടാല്‍ മാത്രം വിദഗ്ധരായ പ്രൊഫഷണലുകള്‍ക്ക് വിമര്‍ശകന്റെ രോഗനിര്‍ണയ സിദ്ധാന്തം രൂപപ്പെടുത്താന്‍ കഴിയും” (www.spychologytoday.com)

സ്റ്റീവന്‍ സ്റ്റോസ്നി പറഞ്ഞതുപോലെ, പ്രവാചകനെതിരെയുള്ള ഈ ആരോപണം വാസ്തവത്തില്‍ അടിവരയിടുന്നത് ആരോപകരുടെ മനഃശാസ്ത്രത്തെയാണ്. എത്രമാത്രം നികൃഷ്ഠവും നിന്ദ്യവുമായ മനോവൈകൃതങ്ങള്‍ക്ക് അടിപ്പെട്ടവരാണ് നബിവിമര്‍ശകരെന്ന സത്യം അര്‍ഥശങ്കക്കിടയില്ലാത്ത വിധം ബോധ്യപ്പെടുത്തുന്നതാണ് ഈ വിമര്‍ശനം. തങ്ങളുടെ ഹൃദയങ്ങളില്‍ കട്ടപിടിച്ചു കിടക്കുന്ന വൈകൃതങ്ങളുടെ ചഷകങ്ങളിലൂടെ പ്രവാചകജീവിതത്തെ തലതിരിഞ്ഞു നോക്കികാണുന്ന ഇത്തരം നബിനിന്ദകരില്‍ നിന്നും വസ്തു നിഷ്ഠമായ നിരൂപണങ്ങള്‍ പ്രതീക്ഷിക്കുന്നതാണ് ഏറ്റവും വലിയ തെറ്റ്. തികഞ്ഞ അരാജകവാദികളും ജീവിത മൂല്യങ്ങളോട് കടുത്ത വിരോധം വെച്ചുപുലര്‍ത്തുകയും ചെയ്യുന്ന നവനാസ്തിക ഞരമ്പുരോഗികളാണ് ഈ കഠിനമായ നബിനിന്ദ സമൂഹമാധ്യമങ്ങളിലൂടെ പ്രചരിപ്പിച്ചുകൊണ്ടിരിക്കന്നതിന് മുഖ്യ കാര്‍മികത്വം വഹിച്ചുകൊണ്ടിരിക്കുന്നത്. തങ്ങളുടെ മനോവൈകൃതങ്ങള്‍ക്കനുസരിച്ച് അവര്‍ നബിജീവിതത്തെ ദുര്‍വ്യാഖ്യാനിച്ചും ദുഷിപ്പിച്ചും അവതരിപ്പിക്കുന്നു എന്നത് ആശ്ചര്യജനകമായ കാര്യമൊന്നുമല്ല. കാരണം അവര്‍ വിഭാവനം ചെയ്യുന്ന ഭോഗതൃഷ്ണാമയമായ കുത്തഴിഞ്ഞ ജീവിതാസ്വാധനങ്ങള്‍ക്കെതിരെ, ജീവിത വിശുദ്ധിയുടെ വഴിയടയാളങ്ങള്‍ ചൂണ്ടികാണിക്കുന്ന ഏറ്റവും കരുത്തുറ്റ വ്യക്തിത്വം മുഹമ്മദ് നബി(സ്വ)യാണ്. അതുകൊണ്ടു തന്നെ നബിജീവിതത്തെ അപകീര്‍ത്തിപ്പെടുത്താനും നുണകളിലൂടെ ആ വിശുദ്ധ ജീവിതത്തെ തമസ്‌കരിക്കാനും അവര്‍ അത്യാര്‍ത്തരാണ്; എന്നും എപ്പോഴും. മാനവികതക്ക് ജീവിത മൂല്യങ്ങള്‍ പകര്‍ന്നു നല്‍കിയ മഹാ മാനുഷികളുടെ ചരിത്രവും ജീവിതവും ദുര്‍വ്യാഖ്യാനിച്ച്, അവരെയും തങ്ങളെ പോലെ അരാജകത്വാസ്വാധകരും മൂല്യനിരാസ പ്രണയികളുമാക്കി ചിത്രീകരിക്കുക വഴി ജീവിത വിശുദ്ധിയുടെ പാഥേയത്തെ പ്രകാശ പൂരിതമാക്കുന്ന വഴിവിളക്കുകളെ കെടുത്തികളയാനാണ് അവര്‍ വെമ്പല്‍ കൊള്ളുന്നത്. നല്ലവരെയും നന്മയിലേക്ക് വിളിച്ചവരെയും ഇരുട്ടുകൊണ്ട് മറച്ചുപിടിക്കുക എന്നത് നവനാസ്തിക ഞരമ്പുരോഗികളുടെ പ്രഥമ അജണ്ടകളില്‍ പെട്ടതാണെന്ന് അവരെ അറിയുന്നവര്‍ക്കെല്ലാമറിയാവുന്ന കാര്യമാണ്. അതുകൊണ്ടു തന്നെ മുഹമ്മദ് നബി (സ്വ) ഒരു ഖബ്‌റിന്നരികെ ഇരിക്കുന്ന സന്ദര്‍ഭം ഹദീഥ് ഗ്രന്ഥങ്ങളില്‍ കാണുമ്പോഴേക്കും അതില്‍ നിന്നും ശവരതി വിഭാവനം ചെയ്യാനും, ക്രിസ്തുവിനരികെ മഗ്‌ദല മറിയം നില്‍ക്കുന്ന ചിത്രം കാണുമ്പോഴേക്കും അതില്‍ നിന്നും രഹസ്യവേഴ്ച്ചയുടെ ഇല്ലാ കഥകള്‍ മെനഞ്ഞെടുക്കാനും അവര്‍ നിതാന്ത പരിശ്രമത്തിലായിരിക്കും. കാരണം അവരുടെ മനോവൈകൃതം അത്രമേല്‍ കഠിനമാണ്; നിന്ദിതവും.

പക്ഷെ, അത്ഭുതകരമായ കാര്യം, മുഹമ്മദ് നബി(സ്വ)ക്കെതിരെയുള്ള ഹീനമായ ഈ നുണകഥ നവനാസ്തികരടക്കമുള്ള ഇസ്‌ലാംവിരോധികള്‍ക്കെല്ലാം വേവിച്ച് വിളമ്പിയത് ചില മിഷനറി അടുക്കളയിലാണെന്നതാണ്. ഒരു ശരാശരി ധാര്‍മികത പോലും പല മിഷനറി ഗ്രൂപ്പുകളില്‍ നിന്നും പ്രതീക്ഷിക്കാന്‍ സാധ്യമല്ലാതായിരിക്കുന്നു. മൂടുപടം മാറ്റിയത് മിഷനറിമാരുടെ ഇസ്‌ലാംവിരോധം മാത്രമല്ല, അവരുടെ മനോവൈകൃതങ്ങള്‍ കൂടിയാണ്. നിരൂപണങ്ങള്‍ നിരൂപകന്റെ മനഃശാസ്ത്രത്തെ കൂടി അടയാളപ്പെടുത്തുന്നുണ്ട് എന്ന മനഃശാസ്ത്ര നിരീക്ഷണം അതാണല്ലോ നമ്മെ പഠിപ്പിക്കുന്നത്. അടുത്ത കാലത്തായി ചില മിഷനറിമാരുടെയും സുവിശേഷ വേലക്കാരുടെയും ഇസ്‌ലാംവിമര്‍ശനങ്ങള്‍ സഭ്യതയുടെ സകല സീമകളും ഉല്ലെങ്കിക്കുന്നതാണ്. മനോവൈകൃതങ്ങളുടെ തള്ളിച്ചക്കൊപ്പം ഇസ്‌ലാംവിരോധവും കെട്ടുപിണയുമ്പോള്‍, ചില മിഷനറി-സുവിശേഷ വേലക്കാര്‍ സമൂഹ മാധ്യമങ്ങളില്‍ മേല്‍വിലാസമില്ലാത്ത ഇത്തരം നബിനിന്ദകള്‍ വിസര്‍ജിക്കുന്നത് ഒരു സ്ഥിരം ഏര്‍പ്പാടാക്കിയിട്ടുണ്ട്; എല്ലാവരുമല്ലെങ്കിലും ചിലര്‍ അല്ല പലരും. മഠങ്ങളിലും കോണ്‍വെന്റുകളിലും വര്‍ദ്ധിച്ചു വരുന്ന കന്യാസ്ത്രീ ‘അത്മഹത്യ’കള്‍ക്കു പിന്നില്‍ ഇത്തരം വൈകൃതംപേറികളുടെ തള്ളിച്ചകളാണോ എന്നാരെങ്കിലും സംശയിച്ചാല്‍, അത് തീര്‍ത്തും ഒരു ക്രൈസ്തവ വിരുദ്ധ നിലപാടുമാത്രമായി വിലയിരുത്താന്‍ എപ്പോഴും കഴിയില്ലല്ലോ!?.

ഇനി നമുക്കു പരിശോദിക്കുവാനുള്ളത് പ്രവാചകനെതിരെയുള്ള ‘ശവരതി’യാരോപണത്തിന്റെ നിജസ്ഥിതിയാണ്. ‘ഫാത്തിമ ബിന്‍ത് അസദ് എന്ന സ്ത്രീയുടെ മൃതദേഹത്തെ പ്രവാചകന്‍ ഖബ്റില്‍ വെച്ച് ഭോഗിച്ചു’ എന്നതാണ് ആരോപണം. എത്രമാത്രം കടുത്ത ദുരാരോപണം. കല്ലുവെച്ച നുണയല്ലാതെ മറ്റൊന്നുമല്ലിത്. പ്രസ്തുത ആരോപണത്തിന്റെ നിജസ്ഥിതി പരിശോദിക്കും മുമ്പ് ആരാണ് ഈ ‘ഫാത്തിമ ബിന്‍ത് അസദ്’ എന്ന് നാം അറിയേണ്ടിയിരിക്കുന്നു. നബി(സ്വ)യുടെ പിതൃവ്യന്‍ അബൂത്വാലിബിന്റെ ഭാര്യ. അഥവാ പ്രവാചകന്റെ പിതൃവ്യ. നബി(സ്വ)യുടെ മകള്‍ ഫാത്തിമ(റ)യുടെ ഭര്‍ത്താവ് അലി(റ)യുടെ മാതാവ്. ചെറുപ്രായത്തില്‍ തന്നെ മതാ-പിതാക്കള്‍ നഷ്ടപ്പെട്ട നബി(സ്വ)യെ എടുത്തു വളര്‍ത്തിയ, ഊട്ടിയ, ഉറക്കിയ പ്രിയ പോറ്റുമ്മ. പ്രവാചകന്‍ (സ്വ) അവരെ ‘ഉമ്മ’ എന്നാണ് തന്റെ മരണം വരെ സംബോധന ചെയ്തത്. അബൂത്വാലിബിനു ശേഷം നബി(സ്വ)യോട് ഏറ്റവുമധികം രക്തബന്ധപരിഗണന കാണിച്ച വ്യക്തി. ‘എന്റെ പെറ്റുമ്മക്കു ശേഷമുള്ള എന്റെ ഉമ്മ’ എന്നാണ്, ശവരതിയാരോപണത്തിനായി ഇസ്‌ലാംവിരോധികള്‍ ദുര്‍വ്യാഖ്യാനിക്കുന്ന നിവേദനത്തില്‍ പോലും പ്രവാചകന്‍ അവരെ സംബോധന ചെയ്തത്. പ്രവാചകനുമേല്‍ ശവരതിയാരോപിക്കുവാനായി ഇസ്‌ലാംവിദ്വേഷികള്‍ വളച്ചൊടിച്ച നിവേദനത്തില്‍ തന്നെ, നബി (സ്വ) അവരെ പറ്റി ഇങ്ങനെ പറഞ്ഞതായി രേഖപ്പെടുത്തപെട്ടിരിക്കുന്നു: ”എന്റെ ഉമ്മാ, നിങ്ങള്‍ക്ക് അല്ലാഹു കാരുണ്യം നല്‍കട്ടെ. നിങ്ങള്‍ എന്റെ ഉമ്മക്കു ശേഷം എന്റെ ഉമ്മയായിരുന്നു. നിങ്ങള്‍ വിശപ്പു സഹിക്കുകയും എന്നെ ഭക്ഷണം ഊട്ടുകയും ചെയ്തിരുന്നു. നിങ്ങള്‍ക്ക് വസ്ത്രമില്ലാതിരുന്നിട്ടും എനിക്കു വസ്ത്രം നല്‍കിയിരുന്നു. വിശിഷ്ടമായ ഭക്ഷണങ്ങള്‍ നിങ്ങളെന്നെ ഊട്ടുകയും നിങ്ങള്‍ക്കത് സ്വയം നിഷേധിക്കുകയും ചെയ്തിരുന്നു. അതു മൂലം നിങ്ങള്‍ ഉദ്ദേശിച്ചിരുന്നത് അല്ലാഹുവിന്റെ പ്രീതിയും പരലോക മോക്ഷവുമായിരുന്നു.” മാത്രമല്ല ”അബൂത്വാലിബിന്റെ വിയോഗാനന്തരം ഏറ്റവും മെച്ചപ്പെട്ട രീതിയില്‍ എന്നെ സംരക്ഷിച്ച സ്ത്രീയാണവര്‍” എന്നുകൂടി നബി (സ്വ) അവരെ പറ്റി അനുസ്മരിച്ചതും പ്രസ്തുത നിവേദനങ്ങളില്‍ തന്നെ കാണാവുന്നതാണ്. ഇത്രയെല്ലാം കണ്ടിട്ടും, പ്രവാചകന് അവരോടുണ്ടായിരുന്ന അളവറ്റ സ്നേഹവും വാത്സല്യവും കാരുണ്യവും ആദരവുമെല്ലാം തിരിച്ചറിഞ്ഞിട്ടും, പ്രസ്തുത നിവേദനങ്ങളില്‍ നിന്നും അതെല്ലാം അടര്‍ത്തിമാറ്റി ആ നിവേദനങ്ങളെ തന്നെയെടുത്ത് ക്രൂരമായി ദുര്‍വ്യാഖ്യാനിച്ച് അതില്‍ നിന്നും ശവരതി കണ്ടെടുത്തവരുടെ അന്തരംഗം എത്രമേല്‍ വൃത്തിഹീനമായിരിക്കും. ഏതു കുമ്പസാര കൂട്ടില്‍ കൊണ്ടുപോയി കഴുകികളയും ഈ നികൃഷ്ഠത!.

വൈകൃതാനുരാഗികള്‍ പ്രവാചകനെതിരെ ഉന്നയിച്ച ശവരതിയാരോപണത്തിന്റെ യാഥാര്‍ഥ്യമറിയാന്‍ പ്രസ്തുത നിവേദനങ്ങളും അവയുടെ സ്വീകാര്യതയും നമുക്ക് പരിശോദനാവിധേയമാക്കാം. ”അനസ് (റ) പറയുന്നു: അലി(റ)യുടെ മാതാവായ ഫാത്തിമ ബിന്‍ത് അസദ് ബ്നു ഹാശിം മരണപ്പെട്ടപ്പോള്‍ പ്രവാചകന്‍ അവരുടെ അടുത്തേക്ക് പ്രവേശിച്ചു. അവരുടെ (മൃതദേഹത്തിന്റെ) തല ഭാഗത്ത് അദ്ദേഹം ഇരുന്നു. എന്നിട്ടവിടുന്ന് പറഞ്ഞു: എന്റെ ഉമ്മാ, നിങ്ങള്‍ക്ക് അല്ലാഹു കാരുണ്യം നല്‍കട്ടെ. നിങ്ങള്‍ എന്റെ ഉമ്മക്കു ശേഷം എന്റെ ഉമ്മയായിരുന്നു. നിങ്ങള്‍ വിശപ്പു സഹിക്കുകയും എന്നെ ഭക്ഷണം ഊട്ടുകയും ചെയ്തിരുന്നു. നിങ്ങള്‍ക്ക് വസ്ത്രമില്ലാതിരുന്നിട്ടും എനിക്കു വസ്ത്രം നല്‍കിയിരുന്നു. വിശിഷ്ടമായ ഭക്ഷണങ്ങള്‍ നിങ്ങളെന്നെ ഊട്ടുകയും നിങ്ങള്‍ക്കത് സ്വയം നിഷേധിക്കുകയും ചെയ്തിരുന്നു. അതു മൂലം നിങ്ങള്‍ ഉദ്ദേശിച്ചിരുന്നത് അല്ലാഹുവിന്റെ പ്രീതിയും പരലോക മോക്ഷവുമായിരുന്നു. ശേഷം അവരുടെ മയ്യിത്ത് കുളിപ്പിക്കുവാന്‍ പ്രവാചകന്‍ കല്‍പ്പിച്ചു. മൂന്നുവട്ടം കഴുകുന്ന രീതിയിലാകണമെന്ന് നിര്‍ദ്ദേശിച്ചു. കര്‍പൂരം കലര്‍ത്തിയ വെള്ളമെത്തിയപ്പോള്‍ പ്രവാചകന്‍ തന്റെ കൈ കൊണ്ട് അതവരുടെ മേല്‍ ഒഴിച്ചു. ശേഷം പ്രവാചകന്‍ തന്റെ മേല്‍കുപ്പായം ഊരി. എന്നിട്ട് അവരുടെ വസ്ത്രത്തിന്റെ മുകളില്‍ പ്രവാചകന്‍ തന്റെ വസ്ത്രം കഫന്‍ ചെയ്തു. എന്നിട്ട് നബി (സ്വ) ഉസാമത്ത് ബ്നു സൈദ്, അബൂഅയ്യൂബുല്‍ അന്‍സാരി, ഉമ്മറിബ്നുല്‍ ഖത്താബ് എന്ന മൂന്നുപേരെ വിളിച്ചു. അവരുടെ കൂടെ കറുത്ത ഒരു ബാലനുമുണ്ടായിരുന്നു. എന്നിട്ടവരോട് കുഴി കുഴിക്കാന്‍ പറഞ്ഞു. അങ്ങനെ അവരുടെ ഖബ്ര്‍ അവര്‍ കുഴിച്ചു. അങ്ങനെ ലഹ്ദ് എത്തിയപ്പോള്‍ പ്രവാചകന്‍ തന്റെ കൈ കൊണ്ട് ലഹ്ദ് കുഴിച്ചു. എന്നിട്ടാ ലഹ്ദിന്റെ മണ്ണ് പ്രവാചകന്‍ തന്നെ തന്റെ കൈ കൊണ്ട് പുറത്തെടുത്തു. ശേഷം അതില്‍ നിന്നും വിരമിച്ചപ്പോള്‍ പ്രവാചകന്‍ ആ ലഹ്ദിന്റെ ഉള്ളിലേക്ക് പ്രവേശിച്ചു. എന്നിട്ടവിടെ കിടന്നു. എന്നിട്ട് പ്രവാചകന്‍ പ്രാർത്ഥിച്ചു: ജീവിപ്പിക്കുകയും മരിപ്പിക്കുകയും ചെയ്യുന്ന, ഒരിക്കലും മരിക്കാത്ത എന്നെന്നും ജീവിച്ചിരിക്കുന്നവനായ അല്ലാഹുവേ, എന്റെ ഉമ്മയായ ഫാത്തിമ ബിന്‍ത് അസദിന് നീ പൊറുത്തു കൊടുക്കേണമേ, അവര്‍ക്കനുകൂലമായ പ്രമാണങ്ങള്‍ നീ അവര്‍ക്ക് നല്‍കേണമേ, അവരുടെ പ്രവേശന സ്ഥാനം നീ വിശാലമാക്കേണമേ, നിന്റെ നബിയുടെയും എനിക്കു മുമ്പുള്ള നിന്റെ മറ്റു പ്രവാചകന്മാരുടെയും അവകാശം കൊണ്ട് ഞാന്‍ ചോദിക്കുന്നു. തീര്‍ച്ചയായും നീ കാരുണ്യവാന്മാരില്‍ അങ്ങേയറ്റം കാരുണ്യവാനാണ്. ശേഷം പ്രവാചകന്‍ നാല് തവണ തക്ബീര്‍ ചൊല്ലി (മയ്യിത്ത് നമസ്‌കരിച്ചു) ശേഷം അവരെ (ഫാത്തിമ ബിന്‍ത് അസദിന്റെ മൃതദേഹം) നബി(സ്വ)യും അബ്ബാസും അബൂബഖറും ചേര്‍ന്ന് (റ)ഖബ്‌റിലേക്ക് പ്രവേശിപ്പിച്ചു.” (ത്വബ്റാനി, അല്‍ കബീര്‍: 24/351, അല്‍ ഹില്‍യ: അബൂ നുഐം: 3/121).

ഇതാണ് പ്രവാചകനു മേല്‍ ശവരതിയാരോപിക്കുവാന്‍ നബിവൈരികള്‍ (ഹദീഥ് പൂര്‍ണരൂപത്തില്‍ കൊടുക്കാതെ ഹദീഥ് നമ്പര്‍ മാത്രം നല്‍കികൊണ്ട്) ഉദ്ദരിക്കുന്ന ‘തെളിവ്’. ഹദീഥിന്റെ സ്വീകാര്യത പരിശോദിക്കും മുമ്പ് അതില്‍ പറഞ്ഞ കാര്യങ്ങള്‍ നമുക്കു വിശകലനം ചെയ്യാം.

1, മരണപ്പെട്ടുപോയ തന്റെ പോറ്റുമ്മയോട് ഒരു മകനെന്ന നിലയില്‍ പ്രവാചകനുണ്ടായിരുന്ന അളവറ്റ ആദരവും സ്നേഹവും വാത്സല്യവും കാരുണ്യവും ആര്‍ദ്രതയുമെല്ലാം ഹദീഥില്‍ നിന്നും കാഴ്ചയുള്ള കണ്ണുകള്‍ക്കെല്ലാം വ്യക്തമായും കാണാവുന്നതാണ്. അതെല്ലാം കണ്ടിട്ടും, ആ മകനും മാതാവിന്നുമിടയിലെ വേര്‍പാടിന്റെ ദുഃഖം തളംകെട്ടിയ ചരിത്ര നിമിഷങ്ങളില്‍ നിന്നുപോലും അശ്ലീലതകള്‍ ചികഞ്ഞെടുക്കാന്‍ ലൈംഗിക വൈകൃതങ്ങള്‍കൊണ്ട് ഹൃദയം നുരുമ്പിച്ചവര്‍ക്കല്ലാതെ സാധ്യമല്ല.

2, പ്രവാചകന്‍ തന്റെ മേല്‍കുപ്പായം ഊരി ഫാത്തിമ ബിന്‍ത് അസദിന്റെ മൃതദേഹത്തെ അണിയിക്കുന്നത് അവരെ ഖബ്റില്‍ വെക്കുന്നതിനും എത്രയോ മുമ്പാണെന്ന് ഹദീഥില്‍ നിന്നും സുവ്യക്തമാണ്. അതും അവരെ മൂടിയ വസ്ത്രത്തിനു മീതെയായിരുന്നു എന്നതും ഹദീഥില്‍ കാണാം. ഖബ്‌റിലിറങ്ങി വസ്ത്രമുരിഞ്ഞ് ശവരതിയിലേര്‍പ്പെട്ടു എന്ന കല്ലുവെച്ച കളവ് ഞരമ്പുരോഗികളുടെ ഭാവനാ വിന്ന്യാസങ്ങള്‍ മാത്രമാണ്.

3, പ്രവാചകന്‍ കിടന്നത് ഖബ്‌റിലെ ലഹ്ദിലാണ്. അവിടുന്ന് തന്നെയാണ് ലഹ്ദ് കുഴിച്ചതും. എന്താണ് ലഹ്ദ്? ഖബ്റിന്റെ പാര്‍ശ്വം ഖിബ്‌ലയുടെ ഭാഗത്തേക്ക് തുരന്നുണ്ടാക്കുന്ന ഉള്‍ഖബ്‌റിനാണ് ലഹ്ദ് എന്നു പറയുക (ഫിഖ്ഹുസ്സുന്നഃ, ഭാഗം 4, പേജ്: 398). അഥവാ ലഹ്ദ് മയ്യിത്തിനെ മാത്രം ഉള്‍കൊള്ളാന്‍ പര്യാപ്തമായ ഖബ്‌റിന്റെ ഏറ്റവും താഴെ പാര്‍ശ്വം ഖിബ്‌ലയുടെ ഭാഗത്തേക്ക് തുരന്നുണ്ടാക്കുന്ന ഇടുങ്ങിയ ഉള്‍ഭാഗമാണ്. അവിടെ മയ്യിത്ത് വെക്കും മുമ്പാണ് പ്രവാചകന്‍ കിടന്നത്. ഖബ്‌റിന്റെ ആത്മീയവും ഭൗതികവുമായ ഇടുക്കത്തില്‍ നിന്നും തന്റെ പോറ്റുമ്മക്ക് വിശാലത ലഭിക്കാനായാണ് നബി (സ്വ) അങ്ങനെ ചെയ്തത്. മയ്യിത്തിനെ മാത്രം ഉള്‍കൊള്ളാന്‍ പര്യാപ്തമായ ഇടുങ്ങിയ ഉള്‍ഭാഗമായ ലഹ്ദില്‍ മയ്യിത്തിനൊപ്പം മറ്റൊരാള്‍ക്ക് കൂടി കിടക്കുവാന്‍ സാധ്യമല്ലെന്ന് ലഹ്ദ് കണ്ടിട്ടുള്ളവര്‍ക്കെല്ലാം അറിയാം. അത് കൃത്യമായി അറിയാവുന്നതു കൊണ്ടാണ് ശവരതിയാരോപകര്‍ നബി ലഹ്ദില്‍ കിടന്നു എന്നു പറയാതെ, ഖബ്‌റില്‍ കിടന്നു എന്നു പറഞ്ഞുകൊണ്ടിരിക്കുന്നത്. കാരണം ഹദീഥുകളില്‍ നബി (സ്വ) കിടന്നത് ലഹ്ദിലാണെന്ന് വ്യക്തമാക്കിയിട്ടും അതു മൂടിവെക്കുന്നത് ലഹ്ദ് എന്താണെന്നറിയുന്നവര്‍ക്കിടയില്‍ ശവരതിയാരോപണത്തിന്റെ കാറ്റുപോകുമെന്ന് കൃത്യമായ ധാരണയുള്ളതുകൊണ്ടു മാത്രമാണ്.

4, ലഹ്ദില്‍ കിടന്നതിന് ശേഷം അതില്‍ നിന്നും പുറത്തുവന്നതിനു ശേഷമാണ് നബി (സ്വ) ഫാത്തിമ ബിന്‍ത് അസദിന്റെ മയ്യിത്ത് നമസ്‌കരിക്കുന്നത്. അതിനും ശേഷമാണ് അവരുടെ മയ്യിത്ത്, അബ്ബാസിനോടും അബൂബഖറിനോടും ചേര്‍ന്ന് നബി (സ്വ) ഖബ്റിലെ ലഹ്ദിലേക്ക് ഇറക്കി വെക്കുന്നത്. അപ്പോള്‍ ഖബ്‌റില്‍ വെച്ച് പ്രവാചകന്‍ ശവരതി നടത്തിയെന്നത് കല്ലുവെച്ച കള്ളമാണെന്നര്‍ഥം.

5. എന്തിനാണ് പ്രവാചകന്‍ (സ) ലഹ്ദില്‍ അവരുടെ സ്ഥാനത്ത് കിടന്നത് എന്നതും സുവ്യക്തമാണ്.

‘അദ്ദേഹം കബ്‌റിന്റെ (ലഹ്ദില്‍) അതിരുകള്‍ തിക്കി. അത് വിശാലമാക്കുന്നതു പോലെ…’ (അല്‍ മുസ്തദ്‌റക്: ഹാകിം: 3/108?)

ഉമര്‍ ഇബ്‌നു ശബ്ബ: പറഞ്ഞു: ‘അദ്ദേഹം ലഹ്ദില്‍ കിടന്നു; അത് (അതിരുകള്‍ ഒതുക്കി) വിശാലമാക്കാനെന്ന പോലെ.’ (താരീഖുല്‍ മദീന: 1: 124)

ലഹ്ദില്‍ കിടന്നാല്‍ മാത്രമെ ലഹ്ദിന്റെ ഇടുക്കവും കുടുസ്സതയും ഒരു മയ്യിത്തിന് (മൃതശരീരം) എത്രയുണ്ടെന്ന് കൃത്യമായി മനസ്സിലാക്കാന്‍ സാധിക്കു. അതുകൊണ്ട് ഫാതിമയുടെ മയ്യിത്ത് ലഹ്ദില്‍ വെക്കുന്നതിന് മുമ്പ് പ്രവാചകന്‍ ആ സ്ഥാനത്ത് കിടന്നു നോക്കി. എന്നിട്ട് ലഹ്ദില്‍ ഇടുങ്ങി നില്‍ക്കുന്ന അതിരുകള്‍ തിക്കി ഒതുക്കി. ലഹ്ദില്‍ വെക്കാന്‍ പോകുന്ന മയ്യിത്തിന് വേണ്ടി അത് വിശാലമാക്കി. ഇപ്രകാരം ഭൗതീകമായി, പ്രതീകാത്മായി ലഹ്ദ് വിശാലമാക്കുന്നതിലൂടെ കബ്‌റിലെ ആത്മീയ വാസത്തിലും ഫാതിമക്ക് ഈ വിശാലത അനുഭവപ്പെടാന്‍ വേണ്ടി ആ കര്‍മ്മത്തിലൂടെ അദ്ദേഹം പ്രാര്‍ത്ഥിച്ചു. കാരണം എല്ലാ മനുഷ്യരും കബ്‌റില്‍ ആത്മീയമായി ‘ഇടുക്കം’ അനുഭവിക്കുമെന്ന് പ്രവാചകന്‍ (സ) തന്നെ പഠിപ്പിച്ചിട്ടുണ്ട്. (മുസ്‌നദു അഹ്‌മദ്: 24707, ശുഅ്ബുല്‍ ഈമാന്‍: 396)

‘അവളുടെ ഖബ്റില്‍ അവരോടൊപ്പം ഞാന്‍ കിടന്നത് അവള്‍ക്ക് ഖബ്‌റിന്റെ ഇടുക്കം ലളിതമാകാന്‍ വേണ്ടിയാണ്’ എന്ന് പറഞ്ഞതിന്റെ വിവക്ഷ ഇതാണ്. ലൈംഗിതയുമായി അയല്‍ബന്ധം പോലുമില്ലാത്ത, മാതാവിന്റെ പരലോക ജീവിതത്തെ സംബന്ധിച്ച് ഒരു മകന്റെ തീര്‍ത്തും നിഷ്‌കളങ്കമായ വ്യാകുലത മാത്രമാണ് ഈ ലഹ്ദിലെ കിടത്തം തെളിയിക്കുന്നത്.

6, സ്വാഭാവികമായും പകല്‍ വെളിച്ചത്ത്, പൊതുജന മധ്യത്തില്‍, മയ്യിത്തിന്റെ ബന്ധുക്കളുടെ സാന്നിധ്യത്തില്‍ വെച്ചു നടന്ന മൃതദേഹ സംസ്‌കരണ ചടങ്ങായിരുന്നു അതെന്ന മിനിമം ബോധമില്ലാതെയല്ല ഈ ലൈംഗിക വൈകൃതാനുരാഗികള്‍ ഇത്തരം കള്ള പ്രചരണം അഴിച്ചു വിട്ടുകൊണ്ടിരിക്കുന്നത്. അടങ്ങാത്ത നബിവൈര്യവും ഹൃദയത്തില്‍ കട്ടപിടിച്ചു കിടക്കുന്ന ലൈംഗിക വൈകൃതാനുരാഗവും കെട്ടുപിണഞ്ഞ മനോരോഗത്തെ ചികിത്സിക്കുവാനുള്ള കരുത്ത് അവരുടെയൊന്നും ശാസ്ത്രാവബോധത്തിനോ സുവിശേഷ ദര്‍ശനങ്ങള്‍ക്കോ സനാധന മൂല്യങ്ങള്‍ക്കോ അശേഷമില്ലാത്തതുകൊണ്ടാണ്.

ആരോപണ വിധേയമായ ഹദീഥിന്റെ ന്യൂനതകള്‍:

നബിവൈരികള്‍ പ്രവാചകനെതിരെ ഉന്നയിക്കുന്ന ശവരതിയാരോപണം കേവലം അവരുടെ മനോവൈകൃതങ്ങളുടെ ഭാവനാ വിന്ന്യാസങ്ങള്‍ മാത്രമാണെന്നു നാം മനസ്സിലാക്കി. ആരോപണം ആരോപിക്കപ്പെട്ട ഹദീഥുകളില്‍ ഭൂതകണ്ണാടി വെച്ചു തിരഞ്ഞാല്‍ പോലും കാണാനാവില്ലെന്ന വസ്തുത നിലനില്‍ക്കെ തന്നെ പ്രസ്തുത ഹദീഥുകളുടെ സ്വീകാര്യതയെ കൂടി നമുക്കു പഠനവിധേയമാക്കാം. ഹദീഥ്, സ്വീകാര്യതയുടെ മാനദണ്ഡങ്ങള്‍ പൂര്‍ത്തീകരിക്കപ്പെട്ടതാണോ അല്ലയോ എന്ന് പരിശോദിക്കുക ഇസ്‌ലാമിക വൈജ്ഞാനിക രംഗത്ത് വളരെ പ്രധാനപ്പെട്ട കാര്യമാണല്ലോ? കാരണം സ്വീകാര്യയോഗ്യമായ ഹദീഥുകള്‍ക്ക് മാത്രമാണല്ലോ പ്രമാണപരതയുണ്ടാവുക. അല്ലാത്തവ മുസ്‌ലിംകള്‍ പ്രമാണമായി സ്വീകരിക്കാന്‍ പാടുള്ളതല്ലല്ലോ. അതുകൊണ്ടു തന്നെ ഉപര്യുക്ത ഹദീഥുകളുടെ സ്വീകാര്യത കൂടി നമുക്കന്വേഷണവിധേയമാക്കാം.

അഞ്ചു പരമ്പരകളിലൂടെയാണ് ഈ സംഭവം ഉദ്ധരിക്കപ്പെട്ടിരിക്കുന്നത്.

1. (ത്വബ്റാനി: അല്‍കബീര്‍: 24/351, അല്‍ ഹില്‍യ: അബൂ നുഐം: 3/121)

നിവേദക പരമ്പര: അഹ്‌മദിബ്നു ഹമ്മാദ് അസ്സഗ്ബയില്‍ നിന്ന് – റൗഹിബ്നു സ്വലാഹ് നമ്മോട് പറഞ്ഞു – സുഫ്‌യാനു സൗരി നമ്മോട് പറഞ്ഞു – ആസ്വിം അല്‍ അഹ്‌വലില്‍ നിന്ന് – അനസില്‍ നിന്ന്….

പരമ്പരയിലെ റൗഹിബ്നു സ്വലാഹ് ദുര്‍ബലനാണെന്ന് ഹദീഥ് പണ്ഡിതനായ ഇബ്നുഅദിയ്യ് വ്യക്തമാക്കിയിട്ടുണ്ട്. വളരെ ദുര്‍ബലമായ ഹദീഥുകള്‍ ഉദ്ധരിക്കാറുണ്ടെന്ന് അഭിപ്രായപ്പെടുകയും ചെയ്തു. ഇബ്നു മാകൂലാ പറയുന്നു: റൗഹിബ്നു സ്വലാഹിനെ ഹദീഥ് പണ്ഡിതന്മാര്‍ ദുര്‍ബലനായാണ് കാണുന്നത്. ഇബ്നു യൂനുസ് പറയുന്നു: വിശ്വസ്ഥരായ നിവേദകര്‍ക്കെതിരായി വളരെ ദുര്‍ബലമായ ഹദീഥുകള്‍ അയാള്‍ ഉദ്ധരിക്കാറുണ്ട്. ദാറകുത്നി പറഞ്ഞു: ഹദീഥിന്റെ വിഷയത്തില്‍ ദുര്‍ബലന്‍. (അസ്സികാത്ത്: ഇബ്നുഹിബ്ബാന്‍: 8/ 244, അല്‍ കാമില്‍: 3/1006, മീസാന്‍: 2/58, അല്ലിസാന്‍: 2/465466)

കൂടാതെ ദുര്‍ബലനായ റൗഹിബ്നു സ്വലാഹ് മാത്രമാണ് സുഫ്‌യാനു സൗരിയില്‍ നിന്നും ഈ കഥ ഉദ്ധരിക്കുന്നത് എന്നതും റൗഹിബ്നു സ്വലാഹ് ഈജിപ്റ്റുകാരനും സുഫ്യാനു സൗരി കൂഫക്കാരനുമായതിനാല്‍ റൗഹിബ്നു സ്വലാഹ്, സുഫ്‌യാനു സൗരിയില്‍ നിന്ന് ഇങ്ങനെയൊരു കഥ കേള്‍ക്കാന്‍ സാധ്യതയില്ല എന്നതും നിവേദക പരമ്പരയുടെ മറ്റു ന്യൂനതകളായി ഹദീഥ് പണ്ഡിതര്‍ സൂചിപ്പിക്കുന്നുണ്ട്. (മുകദ്ദിമ സ്വഹീഹു മുസ്‌ലിം: 1/7, അല്‍ അവ്‌സത്: ത്വബ്റാനി: 1/153, അല്‍ ഹില്‍യ: 3/121, സില്‍സിലത്തു ദഈഫ: 1/32, അസ്സികാത്ത്: 8/244)

2. (മജ്‌മഉ സവാഇദ്:9/257, അവ്സത്ത്: ത്വബ്റാനി)

നിവേദക പരമ്പര: സഅ്ദാന്‍ ഇബ്നുല്‍ വലീദില്‍ നിന്ന് – അത്വാഅ് ഇബ്നു അബീ റബാഹില്‍ നിന്ന് – ഇബ്നു അബ്ബാസ് പറഞ്ഞു….

നിവേദക പരമ്പരയിലെ സഅ്ദാന്‍ ഇബ്നുല്‍ വലീദ് ‘മജ്ഹൂല്‍’ (വ്യക്തിത്വമോ വിശ്വസ്ഥതയോ അറിയപ്പെടാത്ത വ്യക്തി) ആണ്. (മജ്‌മഉ സവാഇദ്: 9/257)

3. (താരീഖുല്‍ മദീന: ഇബ്നു ശബ്ബ: 1/124)

നിവേദക പരമ്പര: കാസിം ഇബ്നു മുഹമ്മദുല്‍ ഹാശിമി പറഞ്ഞു- അയാള്‍ തന്റെ പിതാമഹനില്‍ നിന്ന് – അയാള്‍ ജാബിറില്‍ നിന്ന്….

പരമ്പര വളരെ ദുര്‍ബലമാണ്. കാരണം കാസിം ഇബ്നു മുഹമ്മദുല്‍ ഹാശിമി ഹദീഥ് നിവേദനത്തില്‍ പരിഗണനീയനേയല്ല എന്ന് സര്‍വ്വ ഹദീഥ് പണ്ഡിതരും വ്യക്തമാക്കിയിട്ടുണ്ട്. അബൂ ഹാതിം പറഞ്ഞു: കാസിം ഇബ്നു മുഹമ്മദുല്‍ ഹാശിമി, ‘മത്റൂക്’ (കളവ് പറയുന്നവനായി ആരോപിതന്‍) ആകുന്നു.

ഇമാം അഹ്‌മദ് പറഞ്ഞു: അയാള്‍ ഹദീഥിന്റെ വിഷയത്തില്‍ ഒന്നുമല്ല. അബൂ സര്‍അ പറഞ്ഞു: വിശ്വസ്ഥരായ നിവേദകര്‍ക്കെതിരായി വളരെ ദുര്‍ബലമായ ഹദീഥുകള്‍ അയാള്‍ ഉദ്ധരിക്കാറുണ്ട്. (മീസാനുല്‍ ഇഅ്തിദാല്‍: 3/379)

4. (താരീഖുല്‍ മദീന: ഇബ്നു ശബ്ബ: 1/123)

നിവേദക പരമ്പര: അബ്ദുല്‍ അസീസ് ഇബ്നു മുഹമ്മദ് അദ്ദുറാവര്‍ദി- അയാള്‍ അബ്ദുല്ലാഹിബ്നു ജഅ്ഫറില്‍ നിന്ന് – അയാള്‍ അംറിബ്നു ദീനാറില്‍ നിന്ന് – അദ്ദേഹം മുഹമ്മദിബ്നു അലിയില്‍ നിന്ന്….

പരമ്പര ദുര്‍ബലം: അബ്ദുല്‍ അസീസ് ഇബ്നു മുഹമ്മദ് അദ്ദുറാവര്‍ദി ദുര്‍ബലനാണ്. ഹൃദ്യസ്ഥ ശേഷി കുറവായതിനാല്‍ ധാരാളം അബദ്ധങ്ങള്‍ ഉദ്ധരിക്കാറുണ്ടെന്ന് ഹദീഥ് പണ്ഡിതര്‍ വ്യക്തമാക്കിയിട്ടുണ്ട്. (മീസാനുല്‍ ഇഅ്തിദാല്‍: 2/633634)

മാത്രമല്ല നിവേദക പരമ്പര ‘മുര്‍സല്‍’ ആകുന്നു അഥവാ പ്രവാചകനിലേക്കെത്താതെ കണ്ണി മുറിഞ്ഞതാകുന്നു. പ്രവാചക ശിഷ്യനല്ലാത്ത മുഹമ്മദുല്‍ ഹനഫിയ്യയാണ് കഥ പറയുന്നത്.

5. മുഹമ്മദിബ്നു ഉമറുബ്നു അലിയില്‍ നിന്ന് പരമ്പര മുറിഞ്ഞതാണ് മറ്റൊരു നിവേദനം. (ഉസ്ദുല്‍ ഗായ: 6/217)

നിവേദക പരമ്പര: ഇബ്നുല്‍ അസീറില്‍ നിന്ന്- അബ്ദുല്ലാഹിബ്നു മുഹമ്മദിബ്നു ഉമറുബ്നു അലി പിതാവില്‍ നിന്നും ഉദ്ധരിക്കുന്നു….

പരമ്പരയിലെ അബ്ദുല്ലാഹിബ്നു മുഹമ്മദിബ്നു ഉമറുബ്നു അലി തന്റെ പിതാമഹനായ അലിയില്‍ നിന്നും ഉദ്ധരിക്കുന്ന നിവേദനങ്ങളെല്ലാം പരമ്പര മുറിഞ്ഞവയാണെന്ന് ഹദീഥ് പണ്ഡിതര്‍ വ്യക്തമാക്കിയിട്ടുണ്ട്. (അത്തക്‌രീബ്: ഇബ്നു ഹജര്‍: 6170). പിന്നെ എങ്ങനെ പ്രവാചകനില്‍ നിന്ന് അദ്ദേഹം നിവേദനം ചെയ്യും.?!

ഹദീഥുകളില്‍ നിന്നും വിമര്‍ശകന്മാര്‍ മുഹമ്മദിന്റെ മുഖമൂടി ഊരിയെടുക്കുമ്പോള്‍, അതിനു മുമ്പില്‍ കുടുങ്ങുന്ന വേളയില്‍ മുസ്‌ലിംകള്‍ പുറത്തെടുക്കുന്ന അവസാനത്തെ അടവാണ് ‘ഹദീഥ് ദുര്‍ബലമാക്കുന്നു’ എന്ന പ്രഖ്യാപനമെന്ന്, ഹദീഥ് നിദാനശാസ്ത്രത്തിന്റെ ബാല പാഠം പോലും കേട്ടിട്ടില്ലാത്ത ചില നാസ്തിക-മിഷനറി ഇസ്‌ലാം വിമര്‍ശകര്‍ തട്ടിവിടാറുണ്ട്. അവരോടു പറയട്ടെ, മുസ്‌ലിംകള്‍ ഹദീഥുകളെ കാണുന്നത് ദൈവിക വെളിപാടുകളായാണ്. അതുകൊണ്ടു തന്നെ അവര്‍ക്ക് അതിന്റെ പ്രാമാണികത കണ്ടെത്താന്‍ കൃത്യമായ മാനദണ്ഡങ്ങളുണ്ട്. അത് പൗരാണിക കാലം തൊട്ടേ അവര്‍ സ്വീകരിച്ചിട്ടുള്ള നടപടിക്രമമാണ്. പൗരാണികരും ആധുനികരുമായ ലോക മുസ്‌ലിം പണ്ഡിതന്മാരെല്ലാം സര്‍വാംഗീകൃതമായി അംഗീകരിച്ചിരിക്കുന്ന ആ നിയമം നമുക്കും മനസ്സിലാക്കാം. ഒരു ഹദീഥ് തള്ളുന്നതും കൊള്ളുന്നതും എന്തിന്റെയടിസ്ഥാനത്തിലാണെന്ന് അപ്പോള്‍ ബോധ്യമാകും.

1, നിവേദക പരമ്പരയിലെ സര്‍വ്വ നിവേദകര്‍ക്കും അദാലത്ത് ഉണ്ടാവുക. അഥവാ നിവേദക പരമ്പരയിലെ സര്‍വ്വ നിവേദകരും വിശ്വസ്ഥരും നീതിമാന്മാരും ഭക്തരുമാവുക.

2, നിവേദക പരമ്പരയിലെ സര്‍വ്വ നിവേദകരുടേയും ഓര്‍മ്മശക്തി സമ്പൂര്‍ണമായിരിക്കുക.

3, നിവേദക പരമ്പര കണ്ണി ചേര്‍ന്നതാവുക; മുറിഞ്ഞതാവാതിരിക്കുക. അഥവാ പരമ്പരയിലെ നിവേദകര്‍ ഏത് നിവേദകരില്‍ നിന്നാണോ ഒരു ഹദീഥ് ഉദ്ധരിക്കുന്നത്, അവര്‍ പരസ്പരം കണ്ടുമുട്ടുകയും നേരിട്ട് ആ ഹദീഥ് കേള്‍ക്കുകയും ചെയ്തിട്ടുണ്ടെന്ന് ഉറപ്പുണ്ടാവുക.

4, നിവേദനത്തിന്റെ പരമ്പരയും (സനദ്), ഉള്ളടക്കവും (മത്നും) കൂടുതല്‍ വിശ്വസ്ഥരും ശ്രേഷ്ടരുമായ നിവേദകര്‍ക്കോ നിവേദനങ്ങള്‍ക്കോ എതിരാവാതിരിക്കുക.

5, നിവേദനം ഇല്ലത്ത് അഥവാ സൂക്ഷ്മമായ ന്യൂനതകളില്‍ നിന്ന് മുക്തമാകണം.

ഈ നിബന്ധനകള്‍ ഒത്ത ഹദീഥുകളെയാണ് ‘സ്വഹീഹ്’ എന്ന് മുസ്‌ലിംകള്‍ വിളിക്കുന്നത്. മുകളില്‍ പ്രസ്ഥാവിച്ച അഞ്ച് നിബന്ധനകളില്‍ ഒന്നില്‍ ന്യൂനതകള്‍ ഉള്ളതായ ഹദീഥുകള്‍ ള്വഈഫ് (ദുര്‍ബലം) ആയ ഹദീഥുകളാകുന്നു. അവ ഇസ്‌ലാമില്‍ പ്രമാണമല്ല. പൗരാണികരും ആധുനികരുമായ ലോക മുസ്‌ലിം പണ്ഡിതന്മാരെല്ലാം അര്‍ത്ഥശങ്കക്കിടയില്ലാത്ത വിധം വ്യക്തമാക്കിയ സര്‍വാംഗീകൃതമായ നിയമമാണിത്. (അര്‍രിസാല: ശാഫിഈ: 370-371, നുസ്ഹത്തുന്നളര്‍: 52, നുഖ്ബത്തുല്‍ ഫികര്‍: ഇബ്നു ഹജര്‍: 30, ഉലൂമുല്‍ ഹദീസ്: 30, മുഖദ്ദിമത്തു ഇബ്നു സ്വലാഹ്: 8, അല്‍ മൂകിദ: ദഹബി: 24, തദ്രീബുര്‍ റാവി: സുയൂത്വി: 1/6875, 155, അല്‍ഫിയ്യ: 19, മന്‍ദൂമത്തുല്‍ ബൈകൂനി: 30, ഹാശിയത്തുല്‍ അജ്ഹുരി: 6, ഇഖ്തിസ്വാറു ഉലൂമുല്‍ ഹദീസ്: ഇബ്നു കസീര്‍: 22, അല്‍ മുക്നിഅ്: ഇബ്നു മുലകിന്‍: 1/42, അല്‍ ജാമിഅ്: ഖത്തീബുല്‍ ബഗ്‌ദാദി: 2/295, അല്‍ ഇക്തിറാഹ്: ഇബ്നു ദകീകുല്‍ ഈദ്: 215 – 216, മുകദ്ദിമത്തു മുസ്‌ലിം: ഇമാം നവവി , അത്തക്‌രീബ്: 105,മജ്‌മഉല്‍ ഫതാവാ ഇബ്നു തീമിയ്യ: 1/250, മആലിമുസ്സുനന്‍: ഖത്താബി: 1/10, ശര്‍ഹുല്‍ അല്‍ഫിയ: ഇറാകി: 1/111, സ്വഹീഹു തര്‍ഗീബു വതര്‍ഹീബ്: 1/4767)

ശവരതിയാരോപണത്തിന്റെ അവസാന പുകയും ഇവിടെ കെട്ടടങ്ങുകയാണ്. വിശുദ്ധിയുടെ വിസ്മയ പ്രകാശമായ നബിജീവിതത്തെ അപകീര്‍ത്തിപ്പെടുത്താനുള്ള മിഷനറി-നവനാസ്തിക വൈകൃതാനുരക്തരുടെ എല്ലാ പരിശ്രമങ്ങളും പാഴ്‌വേലയായി പരിസമാപ്തി കുറിക്കപ്പെടുകയാണ് സ്ഥിരം പതിവ്. പ്രവാചകനെതിരെയുള്ള ശവരതിയാരോപണത്തിന്റെ ഊര്‍ദ്ധ്വശ്വാസവും ഇവിടെ നിലച്ചിരിക്കുന്നു. എങ്കിലും, എങ്ങനെയാണ് ഹദീഥുകളില്‍ നിന്നും – അവ പ്രബലമോ ദുര്‍ബലമോ എന്നു പോലും പരിഗണിക്കാതെ – പ്രവാചക വിരോധികള്‍ ദുരാരോപണങ്ങളും കള്ള കഥകളും മെനഞ്ഞെടുക്കുന്നതെന്ന് പഠിക്കുവാനുള്ള ഒരു സന്ദര്‍ഭമായി ഈ വിഷയസംബന്ധിയായ ചര്‍ച്ചയെ നാം ഉപയോഗപ്പെടുത്തേണ്ടിയിരിക്കുന്നു. ഈ ദുരാരോപണത്തിന്റെ, അഥവാ ശവരതിയാരോപണത്തിന്റെ നിര്‍മിതിയുടെ വഴിയിടങ്ങളിലൂടെ സഞ്ചരിച്ചാല്‍ മാത്രം മതി നമുക്കതു നിഷ്പ്രയാസം കണ്ടെത്താനാകും. പ്രവാചകനു മേല്‍ നികൃഷ്ഠമായ ഒരാരോപണം നിര്‍മിക്കുന്നതിനായി നബിവൈരികള്‍ ഹദീഥ് ഗ്രന്ഥങ്ങള്‍ തിരയുന്നു. നിര്‍മാതാക്കള്‍ ലൈംഗിക വൈകൃതങ്ങള്‍ക്ക് അടിപ്പെട്ടവരായതുകൊണ്ടു തന്നെ പ്രവാചകനെതിരെ ലൈംഗിക വൈകൃതം ആരോപിക്കുന്നതിലായിരിക്കും അവരുടെ ഉത്സാഹം മുഴുവനും. ഹദീഥ് ഗ്രന്ഥങ്ങള്‍ തിരഞ്ഞപ്പോള്‍ അവര്‍ക്കു കിട്ടിയത് ഫാത്തിമ ബിന്‍ത് അസദിന്റെ മൃതദേഹ സംസ്‌കരണത്തിന്റെ ഹദീഥുകള്‍. ഹദീഥുകള്‍ പരിശോദിച്ചപ്പോള്‍ എല്ലാം ദുര്‍ബലമായ നിവേദനങ്ങളാണെന്ന് കാണുന്നു. അതുകൊണ്ടു തന്നെ ഹദീഥുകളുടെ സമാഹാരങ്ങളും അവയിലെ ക്രമനമ്പറുകളും തെറ്റായി ചേര്‍ത്തുകൊണ്ട് തങ്ങളുടെ ദുര്‍വ്യാഖ്യാന അച്ചടിശാലകളില്‍ അച്ചുനിരത്തുന്നു. ശരിയായ നിലക്ക് അവ ഉദ്ദരിച്ചാല്‍ അവയുടെ ദുര്‍ബലതയും നിജസ്ഥിതിയും കൈയോടെ പെടുന്നനെ പിടിക്കപ്പെടുമെന്ന് അവര്‍ക്കറിയാം. സമാഹാരങ്ങളും ക്രമനമ്പറുകളും തെറ്റായി ചേര്‍ത്തവതരിപ്പിക്കുമ്പോള്‍ കൈക്രിയകള്‍ കണ്ടെത്താന്‍ ചെറിയ കാലതാമസം വരും. ആ ഹൃസ്വകാലയളവില്‍ മറുപടി പറയപ്പെടാതെ നില്‍ക്കുന്ന തങ്ങളുടെ കള്ള കഥകളെ സമൂഹ മധ്യത്തില്‍ വെല്ലുവിളികളോടെ പ്രചരിപ്പിച്ചുകൊണ്ടിരിക്കുക. നബിവിരോധം അത്രയെങ്കിലും സജീവമാക്കുക.

പ്രവാചകനെതിരെ ശവരതിയാരോപണമാണ് അവര്‍ ലക്ഷ്യം വെച്ചതെന്നതുകൊണ്ട് തന്നെ അതിനൊപ്പിച്ചു ദുര്‍വ്യാഖ്യാനം ചമക്കാനൊക്കുന്ന ചരിത്ര രംഗങ്ങള്‍ നിവേദനങ്ങളില്‍ ആദ്യം പരതുന്നു. അവിടെ നോക്കുമ്പോള്‍ ആകെ കൂടി കാണാന്‍ കഴിയുന്നത്, ഫാത്തിമ ബിന്‍ത് അസദ് മരണപ്പെടുന്നു. നബി (സ്വ) അവര്‍ക്കുവേണ്ടി പ്രാർത്ഥിക്കുന്നു. അവര്‍ നബിക്കു ചെയ്ത സേവനങ്ങള്‍ അനുസ്മരിക്കുന്നു. ശേഷം അവരെ കുളിപ്പിക്കാന്‍ നിര്‍ദ്ദേശിക്കുന്നു. അനന്തരം അവരെ പൊതിഞ്ഞ വസ്ത്രത്തിനുമേല്‍ നബിയുടെ മേല്‍കുപ്പായമണിയിക്കുന്നു. അതിനു ശേഷം അവര്‍ക്കു വേണ്ടി ഖബ്ര്‍ കുഴിക്കാനാവശ്യപ്പെടുന്നു. ഖബ്‌റിന്റെ ലഹദ് എത്താറായപ്പോള്‍ അത് നബിതന്നെ കുഴിക്കുന്നു. അനന്തരം അവിടുന്ന് ഒരാള്‍ക്ക് മാത്രം കിടക്കാന്‍ കഴിയുന്ന ലഹ്ദില്‍ കിടക്കുന്നു. പ്രാർത്ഥിക്കുന്നു. അതിനു ശേഷം തക്ബീര്‍ ചൊല്ലി മയ്യിത്ത് നമസ്‌കരിക്കുന്നു. അതിനും ശേഷം മറ്റുള്ളവരുടെ സഹായത്തോടെ മയ്യിത്ത് ഖബ്റിലെ ലഹദില്‍ വെക്കുന്നു. ജനങ്ങളും മയ്യിത്തിന്റെ ഉറ്റ ബന്ധുക്കളും പ്രസ്തുത ചടങ്ങുകള്‍ക്ക് സാക്ഷികളാകുന്നു. പക്ഷെ, കഥ ഇതല്ല അവര്‍ക്കുവേണ്ടത്. മയ്യിത്ത് നഗ്‌നമായി ഖബ്റില്‍ വെക്കുന്നു. അല്ലെങ്കില്‍ ഖബ്‌റില്‍ വെച്ച മയ്യിത്തില്‍ നിന്നും വസ്ത്രമുരിയുന്നു. എന്നിട്ട് തന്റെ വസ്ത്രമഴിക്കുന്നു. ശവഭോഗം ചെയ്യുന്നു. ശേഷം തന്റെ വസ്ത്രം മയ്യിത്തിനെ അണിയിക്കുന്നു. എന്നിട്ട് പ്രവാചകന്‍ പറയുന്നു: ‘ഞാന്‍ എന്റെ കുപ്പായം അവളെ അണിയിച്ചു. ഖബ്റിലെ വേദന അകറ്റുന്നതിന്’ ഇതാണ് അവരുടെ വൈകൃത ഭാവനകള്‍ മെനഞ്ഞെടുത്ത ചിത്രം. പക്ഷെ നബി തന്റെ വസ്ത്രം മയ്യിത്തിനെ അണിയിപ്പിച്ചത് മയ്യിത്ത് കുളിപ്പിച്ച ശേഷം, അതിനെ പൊതിഞ്ഞ പുടവക്കുമേല്‍. അവിടുന്ന് ലഹ്ദില്‍ കിടന്നത് മയ്യിത്ത് വെക്കും മുമ്പ്. അതിനു ശേഷം മയ്യിത്ത് നമസ്‌കരിക്കുന്നു. അതിനും ശേഷമാണ് മയ്യിത്ത് ഖബ്‌റിലെ ലഹദില്‍ വെക്കുന്നത്. കഥ ഒരു നിലക്കും ഭാവനകള്‍ക്കൊത്ത് ശരിയായി വരുന്നില്ല. അപ്പോള്‍ പിന്നെ തങ്ങളുടെ ഭാവനകള്‍ക്കൊത്ത് കഥ തിരുത്തിയെഴുതാന്‍ പറ്റിയ വല്ല വാചകങ്ങളും നിവേദനങ്ങളില്‍ നിന്നും (നിവേദനങ്ങളെല്ലാം ദുര്‍ബലമാണെങ്കിലും) അടര്‍ത്തിയെടുക്കാനുണ്ടോ എന്നതാണ് അടുത്ത റിസെര്‍ച്ച്. ഇതാ കിട്ടിപ്പോയി ത്വബ്റാനിയുടെ അല്‍ മുഅ്ജമുല്‍ ഔസത്ത്. അവിടെ 6935 മത്തെ നിവേദനത്തില്‍ നിന്നും ഒരു വാചകം മാത്രം അടര്‍ത്തിയെടുക്കാം. ‘വള്വ്ത്വജഅത്തു മഅഹാ ഫീ ഖബ്രിഹാ’ (അവരുടെ ഖബ്റില്‍ അവളോടൊപ്പം ഞാന്‍ കിടന്നത്…). അത്രമാത്രം അങ്ങ് അടര്‍ത്തിയെടുത്ത് തങ്ങളുടെ ഭാവനകള്‍ക്കൊത്ത് കഥ തിരുത്തിയെഴുതുന്നു. ശേഷം ശവരതിയാരോപണം ദുര്‍വ്യാഖ്യാന അച്ചടിശാലകളില്‍ മഷിയുണക്കിയെടുക്കുന്നു. ‘വള്വ്ത്വജഅത്തു മഅഹാ ഫീ ഖബ്രിഹാ’ (അവളുടെ ഖബ്റില്‍ അവളോടൊപ്പം ഞാന്‍ കിടന്നത്…) എന്നതുകൊണ്ട് എന്താണ് ഉദ്ദേശിക്കപ്പെട്ടത്?. അതൊരു പ്രയോഗം മാത്രമാണ്. ഇംഗ്ലീഷില്‍ “Think in her shoes’ എന്നു പറയുന്നതു പോലെ. അതിനാരെങ്കിലും ‘അവളുടെ ഷൂസില്‍ കയറി ചിന്തിക്കുക’ എന്നര്‍ഥം മനസ്സിലാക്കുമോ!. ‘അവളുടെ സ്ഥാനത്തു നിന്നു ചിന്തിക്കുക’ എന്നേ അതിനുദ്ധേശമുള്ളൂ. അതുപോലെ ‘വള്വ്ത്വജഅത്തു മഅഹാ ഫീ ഖബ്രിഹാ’ (അവളുടെ ഖബ്‌റില്‍ അവളോടൊപ്പം ഞാന്‍ കിടന്നു) എന്നതുകൊണ്ടുദ്ദേശിക്കപ്പെടുന്നത് ‘ഫീ മൗള്വിഇഹാ’ അഥവാ ‘അവളുടെ സ്ഥാനത്ത് ഞാന്‍ കിടന്നു’ എന്നു മാത്രമാണ്. മാത്രമല്ല മറ്റു പല നിവേദനങ്ങളിലും വന്നിട്ടുള്ള പദം ‘വള്വ്ത്വജഅത്തു ഫീ ഖബ്രിഹാ’ (അവളുടെ ഖബ്‌റില്‍ ഞാന്‍ കിടന്നു) എന്നാണ്. (കന്‍സുല്‍ ഉമ്മാല്‍)

ഇനി, ‘വള്വ്ത്വജഅത്തു മഅഹാ ഫീ ഖബ്രിഹാ’ (അവളുടെ ഖബ്റില്‍ അവളോടൊപ്പം ഞാന്‍ കിടന്നത്…) എന്നു പറയപ്പെട്ട ത്വബ്റാനിയുടെ അല്‍ മുഅ്ജമുല്‍ ഔസത്തിലും സംഭവ വിവരണത്തിന് മേല്‍ പ്രസ്താവിക്കപ്പെട്ടതില്‍ നിന്നും യാതൊരു വ്യത്യാസവും കാണാനാകില്ല. ഫാത്തിമ ബിന്‍ത് അസദ് മരണപ്പെടുന്നു. നബി (സ്വ) അവര്‍ക്കുവേണ്ടി പ്രാർത്ഥിക്കുന്നു. അവര്‍ നബിക്കു ചെയ്ത സേവനങ്ങള്‍ അനുസ്മരിക്കുന്നു. ശേഷം അവരെ കുളിപ്പിക്കാന്‍ നിര്‍ദ്ദേശിക്കുന്നു. അനന്തരം അവരെ പൊതിഞ്ഞ വസ്ത്രത്തിനുമേല്‍ നബിയുടെ മേല്‍കുപ്പായമണിയിക്കുന്നു. അതിനു ശേഷം ഖബ്ര്‍ കഴിക്കുന്നു. പിന്നെ ലഹ്ദ് നബി കുഴിക്കുന്നു. അതില്‍ കിടക്കുന്നു. പ്രാർത്ഥിക്കുന്നു. മയ്യിത്ത് നമസ്‌കരിക്കുന്നു. അതിനും ശേഷം മയ്യിത്ത് ഖബ്‌റിലെ ലഹ്ദില്‍ വെക്കുന്നു. സാധാരണഗതിയില്‍ നിന്നും വ്യത്യസ്തമായി മയ്യിത്തിന് ഇത്ര പരിചരണവും പ്രാര്‍ഥനയും നല്‍കിയതിന്റെ ഉദ്ദേശമെന്താണെന്ന സ്വഹാബാക്കളുടെ ചോദ്യത്തിന് മറുപടിയായി നബി(സ്വ) പറഞ്ഞു: ”ഞാനെന്റെ മേല്‍വസ്ത്രം അവളെ ധരിപ്പിച്ചു; സ്വര്‍ഗത്തിലെ പുടവ ധരിക്കപ്പെടാനുള്ള സൗഭാഗ്യം അവള്‍ക്കു ലഭിക്കാനായി. അവളുടെ ഖബ്‌റില്‍ അവളോടൊപ്പം ഞാന്‍ കിടന്നു; അവള്‍ക്ക് ഖബ്‌റിന്റെ ഇടുക്കം ലളിതമാകാന്‍ വേണ്ടി. കാരണം, അബൂത്വാലിനു ശേഷം എന്നോട് ഏറ്റവും നന്നായി പെരുമാറുന്ന ആളായിരുന്നു അവര്‍”. ത്വബ്റാനിയുടെ അല്‍ മുഅ്ജമുല്‍ ഔസത്ത് 6935 മത്തെ നിവേദനത്തില്‍ നിന്നും ഇതിനപ്പിറം ഒന്നും തരപ്പെടില്ല. ഈ നിവേദനത്തില്‍ ‘അവളുടെ ഖബ്‌റില്‍ അവളോടൊപ്പം ഞാന്‍ കിടന്നത് അവള്‍ക്ക് ഖബ്‌റിന്റെ ഇടുക്കം ലളിതമാകാന്‍ വേണ്ടി’ എന്നു പറഞ്ഞത് – മയ്യിത്ത് ഖബ്റിലെ ലഹദില്‍ വെക്കുന്നതിനും മയ്യിത്ത് നമസ്‌കരിക്കുന്നതിനും മുമ്പ് – നബി ലഹ്ദ് കുഴിച്ച് അതില്‍ കിടന്നു പ്രാർത്ഥിച്ച സംഭവത്തെ പറ്റി തന്നെയാണെന്ന് ആര്‍ക്കും ഒറ്റ വായനയില്‍ തന്നെ ബോധ്യമാകുന്ന കാര്യമാണ്. അപ്പോള്‍ ‘അവളുടെ ഖബ്റില്‍ അവളോടൊപ്പം ഞാന്‍ കിടന്നു’ എന്നു പറഞ്ഞതിന്റെ ഉദ്ദേശം ‘അവളുടെ സ്ഥാനത്ത് ഞാന്‍ കിടന്നു’ എന്നു മാത്രമാണെന്നു വ്യക്തം.

അല്ലാതെ ആഭാസന്മാരുടെ അശ്ലീല ഭാവനകള്‍ക്കൊത്ത് പ്രവാചക ജീവിതത്തെ വളച്ചൊടിക്കാനൊക്കുന്ന ഒന്നും ആ വിശുദ്ധ ജീവിതത്തില്‍ നിന്നും കിട്ടാന്‍ പോകുന്നില്ല. പക്ഷെ, കൂട്ടത്തില്‍ ഒന്നുകൂടി ഓര്‍മപ്പെടുത്തട്ടെ സംഭവം ദുര്‍ബലമായ നിവേദനമാണ്; തെളിവിനു കൊള്ളില്ല.

വിശുദ്ധിയുടെ നിതാന്ത പരിമളമയമായ ആ ജീവിതം ചരിത്രത്തിന്റെ സ്വര്‍ണ ലിപികളാല്‍ ആലേഖനം ചെയ്യപ്പെട്ടതാണ്. അവിടെ എത്ര ഉരച്ചു നോക്കിയാലും ശവരതി പോയിട്ട് ഒരു പരസ്ത്രീ കരസ്പര്‍ശനത്തിന്റെ ചെറുതരികള്‍ പോലും കാണ്ടുകിട്ടില്ല. നൂറ്റാണ്ടുകളായി, എത്രയോ വമ്പന്‍ വിമര്‍ശകന്മാര്‍ ഉരക്കാന്‍ തുടങ്ങിയിട്ട്. ഉരക്കല്ലുകള്‍ തേഞ്ഞുതീര്‍ന്നിട്ടും അവര്‍ക്കൊന്നും മരുന്നിനുപോലും ഇന്നോളം ഒന്നും തരപ്പെട്ടിട്ടില്ല. പിന്നെയല്ലേ ഈ മനോരോഗികള്‍ ഉരച്ചിട്ട്!.

”ആഇശ (റ) പറയുന്നു: പ്രവാചകന്‍ ഒരിക്കലും ഒരു (അന്യ) സ്ത്രീയുടെയും കൈപ്പടം തൊട്ടിട്ടില്ല. അവരോട് ബൈഅത്ത് (പ്രതിജ്ഞ) എടുക്കാന്‍ പോകുന്ന സമയത്ത് പോലും അവരോട് പറയാറുണ്ട് നിങ്ങളോട് ഞാന്‍ എന്റെ സംസാരത്തിലൂടെ ബൈഅത്ത് ചെയ്തിരിക്കുന്നു.” (മുസ്‌ലിം: 1866)

”ഉമൈമഃ ബിന്‍ത് റക്വീക്വ (റ) പറഞ്ഞു: അവര്‍ നബി(സ്വ)ക്ക് കൈകൊടുത്ത സമയത്ത് നബി (സ്വ) അവരോട് പറഞ്ഞു: തീര്‍ച്ചയായും ഞാന്‍ (അന്യ) സ്ത്രീകള്‍ക്ക് കൈകൊടുക്കയില്ല.” (നസാഈ: 4181, ഇബ്‌നു മാജ: 2874)

”മഅ്ക്വലുബ്നു യസാര്‍ നിവേദനം: നബി (സ്വ) പറഞ്ഞു: തനിക്ക് അനുവദനീയമല്ലാത്ത ഒരു സ്ത്രീയെ സ്പര്‍ശിക്കുന്നതിനെക്കാള്‍ അവന് ഉത്തമമായത് ഇരുമ്പാണി തലയില്‍ തറക്കുന്നതാണ്.” (ത്വബ്‌റാനി)

ഇതാണ് മുഹമ്മദ് നബി (സ്വ) വിശുദ്ധ ജീവിതത്തിന്റെ നിതാന്ത വിസ്മയം. ലൈംഗിക വൈകൃതം ഹൃദയങ്ങളില്‍ കട്ടപിടിച്ചു കിടക്കുന്നവര്‍ക്ക് ആ ജീവിതവിശുദ്ധി ഉള്‍കൊള്ളാനാകില്ല; സമ്മതിച്ചു തരാനാകില്ല. ഈ രോഗാധുരമായ മനസ്സുകളില്‍ നിന്നു മാത്രമാണ് ഇത്തരം നീചവും നികൃഷ്ടവുമായ ദുരാരോപണങ്ങള്‍ ജന്മമെടുക്കുന്നത്. അവര്‍ പ്രവാചകനെ തെറി വിളിച്ചുകൊണ്ടേയിരിക്കും. അപകീര്‍ത്തിപ്പെടുത്തികൊണ്ടേയിരിക്കും. കാരണം പരിശുദ്ധരെയും പരിശുദ്ധിയിലേക്ക് മാനവികതയെ വിളിക്കുന്നവരെയും അവര്‍ക്ക് സഹിക്കാനാകില്ല; ഉള്‍കൊള്ളാനും. ഇഛാവൈകൃതങ്ങളുടെ ലോകം പടുത്തുയര്‍ത്താന്‍ അവര്‍ക്കു മുമ്പിലുള്ള ഏറ്റവും വലിയ തടസ്സം നബിയും അവിടുത്തെ ജീവിതവും സന്ദേശവുമാണ്. അതാണ് അവരില്‍ പലരുടെയും നബിവിരോധത്തിന്റെ മനഃശാസ്ത്രം.

അടുത്ത വിമര്‍ശനവും കൂടി നമുക്കൊന്ന് വിശകലനം ചെയ്യാം.

‘രക്തബന്ധമോ കുടുംബബന്ധമോ പോലും പരിഗണിക്കാതെ ഏത് സ്ത്രീയെയും ഭോഗിക്കുവാന്‍ നബിക്ക് അല്ലാഹു തന്നെ അനുവാദം നല്‍കുന്നതിനു മുമ്പായിരുന്നു (ക്വുര്‍ആന്‍ 33: 50) ഈ ശവരതി നടന്നതെന്നു കാണാം’. എന്താണ് ക്വുര്‍ആന്‍ 33: 50 ല്‍ പറയുന്നത്. ആദ്യം നമുക്ക് സൂക്തം ഒന്നു പരിശോദിക്കാം.

”നബിയേ, നീ വിവാഹമൂല്യം കൊടുത്തിട്ടുള്ളവരായ നിന്റെ ഭാര്യമാരെ നിനക്ക് നാം അനുവദിച്ചു തന്നിരിക്കുന്നു. അല്ലാഹു നിനക്ക് (യുദ്ധത്തില്‍) അധീനപ്പെടുത്തിത്തന്ന കൂട്ടത്തില്‍ നിന്റെ വലതുകൈ ഉടമപ്പെടുത്തിയ (അടിമ) സ്ത്രീകളെയും നിന്നോടൊപ്പം സ്വദേശം വിട്ടുപോന്നവരായ നിന്റെ പിതൃവ്യന്റെ പുത്രിമാര്‍, നിന്റെ പിതൃസഹോദരിമാരുടെ പുത്രിമാര്‍, നിന്റെ അമ്മാവന്റെ പുത്രിമാര്‍, നിന്റെ മാതൃസഹോദരിമാരുടെ പുത്രിമാര്‍ എന്നിവരെയും (വിവാഹം ചെയ്യാന്‍ അനുവദിച്ചിരിക്കുന്നു.) സത്യവിശ്വാസിനിയായ ഒരു സ്ത്രീ സ്വദേഹം നബിക്ക് ദാനം ചെയ്യുന്ന പക്ഷം നബി അവളെ വിവാഹം കഴിക്കാന്‍ ഉദ്ദേശിക്കുന്നെങ്കില്‍ അതും (അനുവദിച്ചിരിക്കുന്നു.) ഇത് സത്യവിശ്വാസികളെ കൂടാതെ നിനക്ക് മാത്രമുള്ളതാകുന്നു. അവരുടെ ഭാര്യമാരുടെയും അവരുടെ വലതുകൈകള്‍ ഉടമപ്പെടുത്തിയവരുടേയും കാര്യത്തില്‍ നാം നിയമമായി നിശ്ചയിച്ചിട്ടുള്ളത് നമുക്കറിയാം. നിനക്ക് യാതൊരു വിഷമവും ഉണ്ടാവാതിരിക്കാന്‍ വേണ്ടിയത്രെ ഇത്. അല്ലാഹു ഏറെ പൊറുക്കുന്നവനും കരുണാനിധിയുമാകുന്നു”. (കുര്‍ആന്‍: 33: 50)

വിമര്‍ശകന്മാര്‍ ജല്‍പിക്കുന്നതു പോലെ പ്രവാചകന് ലൈംഗിക ബന്ധത്തിലേര്‍പെടാന്‍ അനുവദിക്കപ്പെട്ട സ്ത്രീജന ലിസ്റ്റ് അല്ലിത്. മറിച്ച് പ്രവാചകന് വിവാഹ ബന്ധത്തിലേര്‍പെടാന്‍ അനുവദിക്കപ്പെട്ട സ്ത്രീകള്‍ ആരൊക്കെ? വിവാഹം ചെയ്യാവുന്ന സ്ത്രീകള്‍ ആരൊക്കെ? പ്രവാചകന് മാത്രമായി അനുവദിക്കപ്പെട്ട വിവാഹ രീതി ഏത്? ഇതൊക്കെയാണ് സൂക്തം കൈകാര്യം ചെയ്യുന്ന വിഷയം. അതിനെയെല്ലാം ഒന്നിച്ച് ‘ലൈംഗികാനുവാദ’മെന്ന പായയില്‍ ചുരുട്ടികെട്ടിയതാണ് വിമര്‍ശകര്‍.

ആദ്യം പറഞ്ഞ അല്ലെങ്കില്‍ ലൈംഗിക ബന്ധത്തിലേര്‍പെടാന്‍ അനുവദിക്കപ്പെട്ട സ്ത്രീകള്‍ ആരെല്ലാമാണ്. ഭാര്യമാരും സ്വന്തം അടിമസ്ത്രീകളും. അടിമസ്ത്രീകളുമായി ലൈംഗിക ബന്ധം അനുവദിക്കുക മാനവിക വിരുദ്ധമല്ലേ? അടിമത്ത സമ്പ്രദായം നിലനിന്നിരുന്ന സാമൂഹ്യ സാഹചര്യങ്ങളെ പറ്റി യാതൊരു ധാരണയുമില്ലാത്തതുകൊണ്ട് മാത്രമാണ് ഈ സംശയം ഉണ്ടാകുന്നത്. അത് നിലനിന്നിരുന്ന ഭൂമികയില്‍ നിന്നും ആ സമ്പ്രദായത്തെ അടര്‍ത്തിയെടുത്ത്, ഇന്നത്തെ ലോകക്രമത്തില്‍ കൊണ്ടുവെച്ച് അതിനെ വിലയിരുത്തുമ്പോള്‍ അത് മാനവിക വിരുദ്ധമായി തോന്നാം. പക്ഷെ അതിന്റെ ഭൂമികയില്‍ നിന്നുകൊണ്ട് ഇസ്‌ലാം അതിനെ സമീപിച്ച രീതിയെ പറ്റി വിലയിരുത്തുമ്പോഴാണ് പ്രസ്തുത മേഖലയില്‍ ഇസ്‌ലാം സ്വീകരിച്ച മാനവിക നിലപാട് നമുക്ക് മനസ്സിലാവുകയുള്ളൂ. ഒന്നാമതായി നാം മനസ്സിലാക്കേണ്ടത് അത് ഇസ്‌ലാം കൊണ്ടുവന്ന സമ്പ്രദായമല്ല. പ്രവാചകന് എത്രയോ നൂറ്റാണ്ടുകള്‍ക്കു മുമ്പുതന്നെ അത് സമൂഹത്തില്‍ നിലനിന്നിരുന്നു എന്നതാണ്. അതുകൊണ്ടാണ് ബൈബിളില്‍ പോലും അടിമത്ത സമ്പ്രദായവുമായി ബന്ധപ്പെട്ട ധാരാളം പരാമര്‍ശങ്ങള്‍ കാണാന്‍ കഴിയുന്നത്. (ലേവ്യ, 25: 44-46) ബൈബിളിലെ പല പ്രവാചകന്മാരും അടിമസ്ത്രീകളുള്ളവരോ, യുദ്ധങ്ങളില്‍ ബന്ധികളാക്കപ്പെട്ട സ്ത്രീകളെ അടിമകളായി സ്വീകരിച്ചവരോ അയിരുന്നെന്ന് കാണാനാകും. അബ്രഹാമും ദാവീദും മോശെയുമെല്ലാം ഉദാഹരണം. കേവലം ഒരു രാഷ്ട്രത്തിന്റെ ഭൂമിശാസ്ത്രപരമായ നാലതിരുകള്‍ക്കുള്ളില്‍ അപൂര്‍വമായി കാണപ്പെട്ടിരുന്ന ഒറ്റപ്പെട്ട ഒരു സാമൂഹിക പ്രതിഭാസമായിരുന്നില്ല അടിമത്ത സമ്പ്രദായം. ആ കാലഘട്ടത്തില്‍ ലോകത്തിന്റെ തന്നെ സാമൂഹ്യ സാമ്പത്തിക രാഷ്ട്രീയ മേഖലകളില്‍ ആഴത്തില്‍ വേരൂന്നിയ ഒരു സാമൂഹിക പ്രതിഭാസമായിരുന്നു അത്. ഇന്നത്തെ ഭാഷയില്‍ പറഞ്ഞാല്‍, യുദ്ധരംഗങ്ങളില്‍ ബന്ധികളാക്കപ്പെടുന്നവര്‍ മോചിപ്പിക്കപ്പെടുന്നില്ലെങ്കില്‍ അടിമകളാക്കപ്പെടുകയെന്നത് അന്നത്തെ അന്താരാഷ്ട്ര നിയമമായിരുന്നു എന്നുപോലും വേണമെങ്കില്‍ പറയാവുന്നതാണ്. അത് അന്നത്തെ അന്താരാഷ്ട്ര നൈതികതക്കെതിരായിരുന്നില്ല. ആ കാലഘട്ടത്തിലെ ഒരു രാഷ്ട്രത്തിനോ സാമ്രാജ്യത്വത്തിനോ പോലും, ഒരൊറ്റ സുപ്രഭാതത്തില്‍ തുടച്ചു നീക്കാന്‍ കഴിയുന്ന ഒന്നായിരുന്നില്ല അതെന്ന് പൗരാണിക ചരിത്രത്തെ പറ്റി ശരാശരി അറിവെങ്കിലുമുള്ള എല്ലാവര്‍ക്കുമറിയാവുന്ന കാര്യമാണ്. ആ യാഥാര്‍ത്ഥത്തെ ഉള്‍കൊള്ളാതിരിക്കാന്‍ ഇസ്‌ലാമികസാമ്രാജ്യത്വത്തിനെന്നല്ല ഒരു സാമ്രാജ്യത്വത്തിനും സാധ്യമല്ല. അങ്ങനെ വന്നാല്‍ അത് ആ സാമ്രാജ്യത്വത്തിന്റെ നാശത്തിലായിരിക്കുമവസാനിക്കുക. കാരണം അത്രമേല്‍ അത് അന്നത്തെ സാമൂഹ്യ സാമ്പത്തിക രാഷ്ട്രീയ മേഖലകളില്‍ ശക്തമായ സ്വാധീനം ചെലുത്തുന്ന ഒരു ഘടകമായിരുന്നു. പക്ഷെ മാനവികതയുടെ മതമായ ഇസ്‌ലാം ആ പരിധിക്കുള്ളില്‍ നിന്നു കൊണ്ട് തന്നെ കഴിയുന്നത്ര മാനവികമായ പരിഷ്‌കരണങ്ങള്‍ പ്രസ്തുത രംഗത്ത് നടപ്പാക്കുകയാണുണ്ടായത്. ആ പരിഷ്‌കരണങ്ങളാകട്ടെ അടിമകള്‍ക്ക് സ്വതന്ത്ര തുല്ല്യമായ ആശ്വാസമാണ് പ്രദാനം ചെയ്തത്. ഇസ്‌ലാമികസാമ്രാജ്യം അടിമകള്‍ക്ക് ചരിത്രത്തില്‍ മറ്റൊരിടത്തുമില്ലാതിരുന്ന അസ്തിത്വവും വ്യക്തിത്വവും നല്‍കിയാദരിച്ചു. അവര്‍ക്ക് ചോദിക്കാനും പറയാനും ആളുണ്ടായി. പിഴുതെടുക്കാന്‍ സമൂഹത്തിനു സാധ്യമല്ലാത്ത വിധം വേരും വിലാസവുമുണ്ടായവര്‍ക്ക്. ജീവിതത്തില്‍ തിരഞ്ഞെടുപ്പുകള്‍ക്ക് അവകാശമുണ്ടായവര്‍ക്ക്; അത് സ്വതന്ത്രനത്ര വന്നില്ലെങ്കില്‍ പോലും. കാരണം അവിടെ പരിധി നിശ്ചയിച്ചത് ഇസ്‌ലാമല്ല, സാമൂഹികാന്തരീക്ഷമായിരുന്നു. അവര്‍ക്ക് ഉടുക്കാനും ഉണ്ണാനും ഉറങ്ങാനും അവകാശമുണ്ടായി. ആകാശത്തിനു കീഴെ ഭൂമിക്കു മുകളില്‍ അവര്‍ക്കാദ്യമായി അന്തസ്സുണ്ടായി. ഈ ചരിത്ര പശ്ചാതലത്തില്‍ നിന്നു കൊണ്ട് വേണം അടിമസ്ത്രീയുമായുള്ള ലൈംഗിക ബന്ധത്തിന് ഉടമക്ക് അവകാശം നല്‍കുന്ന ഇസ്‌ലാമിക നിയമത്തെ നോക്കി കാണാന്‍.

പ്രവാചക കാലഘട്ടത്തിനു മുമ്പ് തന്നെ, അടിമ സ്ത്രീ എന്നത് ഉടമയുടെ സ്വകാര്യ വരുമാന സ്രോതസ്സുകൂടിയായിരുന്നു. അവളെ ഉടമ ലൈംഗികമായി ഭോഗിക്കുന്നതിനു പുറമെ ഗാര്‍ഹികവും സാമൂഹികവുമായ പല മേഖലകളിലും വരുമാന സ്രോതസ്സായി ഉപയോഗപ്പെടുത്തുകയും ചെയ്യും. അതില്‍ ഏറ്റവും അധികം അവളെ ഉപയോഗിച്ച് വരുമാനമുണ്ടാക്കിയിരുന്നത് വേശ്യാവൃത്തിയിലൂടെയാണ്. അതിലൂടെ പണമുണ്ടാക്കി ഉടമക്കെത്തിക്കണം. അവളുടെ ഉപജീവനമോ പരിചരണമോ അയാളുടെ ബാധ്യതയായിരുന്നില്ല. അതെല്ലാം അവള്‍ കണ്ടെത്തണം. ഉടമക്കു പുറമെ വേശ്യാവൃത്തിയിലൂടെയും അല്ലാതെ ഉടമയുടെ അഥിതി സല്‍ക്കാരത്തിന്റെ ഭാഗമായും അവള്‍ക്ക് ഒരുപാട് പുരുഷന്മാരോടൊപ്പം തന്റെ ശരീരം പങ്കുവെക്കേണ്ടി വരുമായിരുന്നു. ആര്‍ക്കും എപ്പോള്‍ വേണമെങ്കിലും ഭോഗിക്കാവുന്ന ഒരു വില്‍പന ചരക്കായിരുന്നു അക്ഷരാര്‍ഥത്തില്‍ ഇസ്‌ലാംമിനു മുമ്പുള്ള അടിമ സ്ത്രീ. അനവധി പുരുഷന്മാരുമായി ശരീരം പങ്കുവെക്കപ്പെടുന്നതു കൊണ്ടു തന്നെ അവള്‍ ഗര്‍ഭിണിയാവുകയോ പ്രസവിക്കുകയോ ചെയ്താല്‍ ആ ഉത്തരവാദിത്തം ഏറ്റെടുക്കാന്‍ ആരുമുണ്ടാവില്ല. ഇനി കുഞ്ഞ് ആരുടേതാണ് എന്നു തിരിഞ്ഞാല്‍ പോലും ഏറ്റെടുക്കാന്‍ ആളുണ്ടാവില്ല. പിതൃത്വം ഒരാളിലേക്കും ചേര്‍ത്തിപറയാന്‍ അവള്‍ക്കവകാശമില്ല. കുഞ്ഞിനെ അവള്‍ തനിച്ചു വളര്‍ത്തണം. ആ കുഞ്ഞ് വലുതായാല്‍ അന്നത്തെ സാമൂഹിക രീതിയനുസരിച്ച് അതും അടിമയായി മാറും. പെണ്‍ കുഞ്ഞാണെങ്കില്‍ മാതാവിന്റെ ജീവിത വഴിയിടങ്ങളിലൂടെ തന്നെ അവളും സഞ്ചരിക്കേണ്ടി വരും; പലപ്പോഴും. വേരില്ല വിലാസമില്ല സ്വത്തില്ല സന്താനങ്ങള്‍ക്ക് തന്തയില്ല. ഇനി അവള്‍ ഭര്‍തൃമതിയാണെങ്കിലോ കാര്യമായ മാറ്റമൊന്നുമില്ല കുഞ്ഞിന്റെ പേരിനൊരു വാലുണ്ടായി എന്നു മാത്രം. ആരുടേതാണെങ്കിലും പിതൃകോളം ഭര്‍ത്താവിന്റെ പേരുകൊണ്ട് പൂരിപ്പിക്കുമെന്നര്‍ഥം.

അടിമ സ്ത്രീയുടെ ഈ ഖേദഘതിക്ക് ഇസ്‌ലാം അറുതി വരുത്തി. ഇസ്‌ലാം പറഞ്ഞു ആര്‍ക്കെങ്കിലും അടിമ സ്ത്രീയുണ്ടായാല്‍ ഒന്നുകില്‍ അവളെ മോചിപ്പിച്ച് നല്ല നിലയില്‍ വിവാഹം ചെയ്തുകൊടുക്കുക. അല്ലെങ്കില്‍ അവള്‍ക്കുമേലുള്ള അധികാരം (വിലായത്ത്) നിലനിര്‍ത്തികൊണ്ട്, പ്രായശ്ചിത്തമായോ പുണ്യകര്‍മമായോ അവളെ മോചിപ്പിക്കക. അതുമല്ലെങ്കില്‍ അവളുടെ ഉപജീവന-പരിചരണ ഉത്തരവാദിത്തമേറ്റെടുത്ത് ഉടമ മാത്രം അവളെ പ്രാപിക്കുക. അതു വഴി അവള്‍ക്ക് കുഞ്ഞുണ്ടായാല്‍ അതിന്റെ ഉത്തരവാദിത്തവും ഉടമക്കായിരിക്കും. അവന്റെ സ്വത്തില്‍ ആ കുഞ്ഞിന് അവകാശമുണ്ടായിരിക്കും. കുഞ്ഞിന്റെ സ്വത്തില്‍ സ്വാഭാവികമായും അമ്മക്കും ഒരവകാശമുണ്ടായിരിക്കും. വേശ്യാവൃത്തിയിലൂടെ അവളെ ഉപയോഗിച്ച് വരുമാനമുണ്ടാക്കുന്ന ഏര്‍പ്പാട് അവസാനിപ്പിക്കുകയും ഉടമയിലൂടെ മാത്രമായി അവള്‍ക്ക് ലൈംഗികവും ഗാര്‍ഹികവുമായ ഒരു ജീവിതം തുറന്നു കൊടുക്കുകയും ചെയ്തു ഇസ്‌ലാം. അവള്‍ക്കും അവളുടെ പരമ്പരകള്‍ക്കും വേരും വിലാസവും സ്വത്തും കുടുംബവും അവകാശവും സംരക്ഷണവും; അല്ല ജീവിതവും കൊടുത്തു ഇസ്‌ലാം. എത്ര മാനവികമായ ഇടപെടല്‍. ഒരേ സമയം ഇസ്‌ലാം, അവളെ ഉപഭോഗ വസ്തുവല്ലാതാക്കുകയും എന്നാല്‍ ഉടമയിലൂടെ അവളുടെ ലൈംഗിക മോഹങ്ങള്‍ക്ക് ഇടംകൊടുക്കുകയും ചെയ്തു. അതാണ് ഇസ്‌ലാമിലെ വലതുകൈ ഉടമപ്പെടുത്തിയ സ്ത്രീകള്‍ അഥവാ അടിമ സ്ത്രീകള്‍. ”നബിയേ, നീ വിവാഹമൂല്യം കൊടുത്തിട്ടുള്ളവരായ നിന്റെ ഭാര്യമാരെ നിനക്ക് നാം അനുവദിച്ചു തന്നിരിക്കുന്നു. അല്ലാഹു നിനക്ക് (യുദ്ധത്തില്‍) അധീനപ്പെടുത്തിത്തന്ന കൂട്ടത്തില്‍ നിന്റെ വലതുകൈ ഉടമപ്പെടുത്തിയ (അടിമ) സ്ത്രീകളെയും…” എന്നു പറഞ്ഞ സൂക്തത്തില്‍ മാനവിക വിരുദ്ധയില്ലെന്ന് ഇപ്പോള്‍ മനസ്സിലായല്ലോ?.

ഇനി രണ്ടാമതായി സൂക്തം എണ്ണി പറയുന്നത് ലൈംഗിക ബന്ധത്തിലേര്‍പ്പെടാന്‍ അനുവധിക്കപ്പെട്ട സ്ത്രീകളെ പറ്റിയല്ല. മറിച്ച് നബിക്ക് വിവാഹം ചെയ്യാന്‍ അനുവാദമുള്ള കുടുംബ ബന്ധുക്കളില്‍ ചിലരെ പറ്റിയാണെന്ന് ഇബ്‌നു അബ്ബാസി(റ)ല്‍ നിന്നും ഇമാം ത്വബ്‌രി ഉദ്ദരിക്കുന്നുണ്ട് (തഫ്‌സീറു ത്വബ്‌രി: 33:50 ന്റെ വ്യാഖ്യാനം). അതാരൊക്കെയാണ്. ”…നിന്നോടൊപ്പം സ്വദേശം വിട്ടുപോന്നവരായ നിന്റെ പിതൃവ്യന്റെ പുത്രിമാര്‍, നിന്റെ പിതൃസഹോദരിമാരുടെ പുത്രിമാര്‍, നിന്റെ അമ്മാവന്റെ പുത്രിമാര്‍, നിന്റെ മാതൃസഹോദരിമാരുടെ പുത്രിമാര്‍ എന്നിവരെയും (വിവാഹം ചെയ്യാന്‍ അനുവദിച്ചിരിക്കുന്നു.)…”. കുടുംബ ബന്ധുക്കളില്‍ നിന്നും നബി(സ്വ)ക്ക് വിവാഹം ചെയ്യാന്‍ പാടില്ലാത്തവരും പാടുള്ളവരും ആരൊക്കെയാണെന്ന് പഠിപ്പിക്കപ്പെടേണ്ടത് വളരെ പ്രധാനപ്പെട്ട കാര്യമാണ്. ഇത്തരം കുടുംബ ബന്ധത്തിലുള്ള സ്ത്രീകളെ വിവാഹം ചെയ്യാന്‍ എല്ലാ മുസ്‌ലിം പുരുഷന്മാര്‍ക്കും അനുവാദമുണ്ടെങ്കിലും പ്രവാചകന് പക്ഷെ ഹിജ്‌റക്കുശേഷം, അദ്ധേഹത്തോടൊപ്പം ഹിജ്‌റ ചെയ്ത കുടുംബ ബന്ധുക്കളെ മാത്രമേ -ബന്ധുക്കളില്‍ നിന്നും- വിവാഹം ചെയ്യാന്‍ പാടുള്ളൂ എന്ന് ഈ സൂക്തത്തിലുടെ പ്രത്യേകം നിയമമാക്കിയിരിക്കുന്നു എന്നു കുറിക്കുന്ന പല നിവേദനങ്ങളും തഫ്‌സീറുകളില്‍ കാണാം. (തഫ്‌സീറു ത്വബ്‌രി: 33:50 ന്റെ വ്യാഖ്യാനം നോക്കുക).

അപ്പോള്‍ ഹിജ്‌റക്കു ശേഷം കുടുംബ ബന്ധുക്കളില്‍ നിന്നും നബിക്ക് വിവാഹം ചെയ്യാവുന്നവരായി സൂക്തം എണ്ണി പറഞ്ഞത് ആരെയൊക്കെയാണ്. ‘നിന്നോടൊപ്പം സ്വദേശം വിട്ടുപോന്നവരായ’ അമ്മാവന്റെയും അമ്മായിയുടെയും പിതൃവ്യന്റെയും മാതൃസഹോദരിമാരുടെയും പുത്രിമാര്‍. ‘പുത്രിമാര്‍’ എന്നത് തെറ്റാതെ പ്രത്യേകം പഠിച്ചുവെക്കണം. കാരണം ചില മിഷനറി അച്ചടിശാലകളില്‍ പലപ്പോഴും ‘പുത്രിമാര്‍’ എന്നത്, അച്ച് നഷ്ടപ്പെട്ടിട്ടാണോ അതല്ല അച്ചടിപ്പിശാച് പിടികൂടിയിട്ടാണോ എന്നറിയില്ല; വിട്ട് പോകാറുണ്ട് പലപ്പേഴും. അങ്ങനെ വിട്ടുപോയവ മാത്രം പ്രസിദ്ധീകരിക്കാന്‍, ഏറ്റവും ഉളുപ്പ് നഷ്ടപ്പെട്ടവരെ തേടിയുള്ള മിഷനറി അച്ചടിശാലകളുടെ യാത്രകള്‍ നവനാസ്തികര്‍ക്കും എക്‌സ് മുസ്‌ലിംകള്‍ക്കും മുമ്പിലാണ് മിക്കവാറും ചെന്നവസാനിക്കാറുള്ളത്. ‘പുത്രിമാര്‍’ വിട്ടുപോയാല്‍ പിന്നെ ഭാക്കിയാവുന്നതാരൊക്കെയാണ്. അമ്മായിയും മാതൃസഹോദരിയും അമ്മാവനും പിതൃവ്യനും മാത്രമാകും. ‘ലേഡീസ് ഫസ്റ്റ്’ എന്നതാണല്ലോ പരിഷ്‌കാരികളുടെ ഒരു നിലപാട്. അപ്പോള്‍ അമ്മാവനും പിതൃവ്യനും മാറ്റി നിര്‍ത്തപ്പെടും. പിന്നെ പറയാനുണ്ടോ ആവശ്യത്തിനുള്ള വകയായല്ലോ. അമ്മായിയെയും മാതൃസഹോദരിയെയും കാമിക്കാനും കല്ല്യാണം കഴിക്കാനും മുഹമ്മദിനെ ക്വുര്‍ആന്‍ അനുവദിക്കുന്നു. കഷ്ടം തന്നെ മുസ്‌ലിംകളുടെ ധാര്‍മിക ബോധം. പ്രചരണ വാഹനത്തിന്റെ സ്റ്റിയറിംങ് പിന്നെ നവനാസ്തിക മനോരോഗികളുടെ കൈയ്യിലായിരിക്കും.

മൂന്നാമതായി സൂക്തം പരാമര്‍ശിക്കുന്ന വിഷയമെന്താണ്. ”…സത്യവിശ്വാസിനിയായ ഒരു സ്ത്രീ സ്വദേഹം നബിക്ക് ദാനം ചെയ്യുന്ന പക്ഷം നബി അവളെ വിവാഹം കഴിക്കാന്‍ ഉദ്ദേശിക്കുന്നെങ്കില്‍ അതും (അനുവദിച്ചിരിക്കുന്നു.) ഇത് സത്യവിശ്വാസികളെ കൂടാതെ നിനക്ക് മാത്രമുള്ളതാകുന്നു…”. എന്താണ് സംഭവം. ഒരു സ്ത്രീ സ്വദേഹം നബിക്ക് ദാനം ചെയ്യുന്ന പക്ഷം അവളെ നബിക്കു വിവാഹം ചെയ്യാം. അതു പ്രവാചകനു മാത്രം അനുവദിക്കപ്പെട്ട നിയമമാണ്. മുസ്‌ലിംകളില്‍ മറ്റാര്‍ക്കും ഈ അനുമതി ബാധകമല്ല. ‘സ്വദേഹം ദാനം ചെയ്യുക’ എന്നു പറഞ്ഞാല്‍ എന്താണ്. ശരീര ദാനമാണോ അവിടെ ഉദ്ധേശിക്കുന്നത്. അല്ലേ അല്ല. മറിച്ച് അതുകൊണ്ടര്‍ത്ഥമാക്കുന്നതെന്താണെന്ന് പണ്ഡിതന്മാര്‍ വിശദീകരിച്ചിട്ടുണ്ട്.

”ഞാന്‍ എന്നെ താങ്കള്‍ക്കു മുമ്പില്‍ വിവാഹത്തിനായി ഇഷ്ടദാനം ചെയ്യുന്നു’ എന്ന വാചകത്തില്‍ ഒരു മുള്വാഫ് (Possession) (ഭാഷാ പരമായ ഭംഗിക്കായി) വിട്ടുകളഞ്ഞതാണ്. യഥാർത്ഥത്തില്‍ വാചകത്തിന്റെ വിവക്ഷ ഇപ്രകാരമാണ്: ‘ഞാന്‍ എന്നെ അഥവാ എന്റെ വിവാഹ കാര്യത്തെ താങ്കള്‍ക്കു മുമ്പില്‍ ഇഷ്ടദാനം ചെയ്യുന്നു’. കാരണം ഒരു സ്വതന്ത്ര്യ സ്ത്രീ ഉടമപ്പെടുത്തപ്പെടുകയോ ദാനം ചെയ്യപ്പെടുകയോ ഇല്ലല്ലോ. മഹര്‍ ഇല്ലാതെ തന്നെ താങ്കളെ വിവാഹം ചെയ്യാന്‍ ഞാന്‍ തയ്യാറാണ് എന്നാണ് ആ സ്ത്രീ പറഞ്ഞതിന്റെ വിവക്ഷ.” (ഫത്ഹുല്‍ ബാരി: 9/112, ഫത്ഹുല്‍ മുന്‍ഇം: 5/540). എന്നാല്‍ അനുവാദമുണ്ടായിരുന്നിട്ടും പ്രവാചകന്‍ അത്തരത്തിലുള്ള വിവാഹം ഒരിക്കലും ചെയ്തിട്ടില്ലെന്ന് ഇബ്‌നു അബ്ബാസി(റ)ല്‍ നിന്ന് ഇമാം കുര്‍തുബിയും (തഫ്‌സീറുല്‍ കുര്‍തുബി, 33:50 ന്റെ വ്യാഖ്യാനം) ഇബ്‌നു ഹജറും (ഫത്ഹുല്‍ ബാരി, 8:526) ഉദ്ദരിച്ചിട്ടുണ്ട്.

അപ്പോള്‍ ഇത്രയുള്ളൂ കാര്യം. ‘സ്വദേഹം ദാനം ചെയ്യുക’ എന്നു പറഞ്ഞാല്‍ മഹര്‍ ഇല്ലാതെ തന്നെ താങ്കളെ വിവാഹം ചെയ്യാന്‍ ഞാന്‍ തയ്യാറാണ് എന്നുമാത്രമേ അതിനര്‍ത്ഥമുള്ളൂ. അത് സ്ത്രീയുടെ തീരുമാനാധികാരപരിധിയില്‍ വരുന്ന സംഗതിയാണ്. ധാര്‍മികതാലംഘനത്തിന്റെ ഒരു പ്രശ്‌നവും അവിടെ ഉരുത്തിരിയുന്നില്ല. അറബി ഭാഷയോ ഭാഷാ പ്രയോഗങ്ങളോ തിരിയാത്തവര്‍ ക്വുര്‍ആനും ഹദീഥും നിരൂപണം ചെയ്യാനൊരുമ്പെട്ടാല്‍ ഇതല്ല ഇതിനപ്പുറവും പറയും. വിവരക്കേട് ഇസ്‌ലാംവിമര്‍ശകര്‍ക്ക് ഒരലങ്കാരമാണെന്നു തോന്നുന്നു. കഷ്ടം

വിമർശനം:

(സോഷ്യൽ മീഡിയകളിൽ വ്യാപകമായി പ്രചരിക്കുന്ന വിമർശനം വിമർശകരുടെ തെറി വിളികൾ മാത്രം ഒഴിവാക്കി ഇവിടെ എടുത്തു ചേർക്കുന്നു:)

“മുഹമ്മദ് ഒരു സ്വവർഗഭോഗി കൂടി ആയിരുന്നു. അനസ് എന്ന മുഹമ്മദിന്റെ കൂട്ടാളിയുമായി മുഹമ്മദ് കാമകേളിയിൽ ഏർപ്പെട്ടു എന്ന് അനസ് തന്നെ വിവരിക്കുന്നു. Bukhari 4:52:143

മുഹമ്മദ് ആൺകുട്ടികളുടെ നാവും ചുണ്ടും നക്കിയിരുന്നതായി വിവരിക്കുന്നു. Musnad Ahmad 16245. (Pedophilic prophet)”

മറുപടി:

ഒരു തെറി പ്രവാചകന്റെ(സ) മേൽ സ്വന്തമായി കെട്ടിയുണ്ടാക്കുകയും അതിന് തെളിവുണ്ടെന്ന് വരുത്തി തീർക്കാൻ ചില ഗ്രന്ഥങ്ങളും ‘നമ്പറു’കളും കുറിക്കുകയും ചെയ്യുക എന്നത് ഇസ്‌ലാം വിമർശകരുടെ സ്ഥിരം ഒരു ‘നമ്പറാ’ണ്. പറഞ്ഞ തെറിക്ക് ഉപോൽബലകമായ യാതൊന്നും പ്രസ്‌തുത നമ്പറുകളിലുള്ള ഹദീസുകളിൽ ഉണ്ടാകില്ലെന്നതാണ് യാഥാർത്ഥ്യം. ഇവിടെയും അതു തന്നെയാണ് നടന്നിരിക്കുന്നത്. ലൈംഗികതയുമായി യാതൊരു ബന്ധവുമില്ലാത്ത രണ്ട് ഹദീസുകളെ സ്വവർഗാനുരാഗവും ബാലാനുരാഗവുമായി വളച്ചൊടിക്കാനുള്ള കുൽസിത ശ്രമം. അതിലെ കള്ളത്തരം തിരിച്ചറിയണമെങ്കിൽ വിമർശകർ ‘നമ്പറിറക്കിയ’ രണ്ട് ഹദീസുകളും ആദ്യം ഒന്ന് നേരിട്ട് വായിക്കാം.

1. ﻋﻦ ﺃﻧﺲ ﺑﻦ ﻣﺎﻟﻚ ﺭﺿﻲ اﻟﻠﻪ ﻋﻨﻪ: ﺃﻥ اﻟﻨﺒﻲ ﺻﻠﻰ اﻟﻠﻪ ﻋﻠﻴﻪ ﻭﺳﻠﻢ، ﻗﺎﻝ ﻷﺑﻲ ﻃﻠﺤﺔ: «اﻟﺘﻤﺲ ﻏﻼﻣﺎ ﻣﻦ ﻏﻠﻤﺎﻧﻜﻢ ﻳﺨﺪﻣﻨﻲ ﺣﺘﻰ ﺃﺧﺮﺝ ﺇﻟﻰ ﺧﻴﺒﺮ» ﻓﺨﺮﺝ ﺑﻲ ﺃﺑﻮ ﻃﻠﺤﺔ ﻣﺮﺩﻓﻲ، ﻭﺃﻧﺎ ﻏﻼﻡ ﺭاﻫﻘﺖ اﻟﺤﻠﻢ، ﻓﻜﻨﺖ ﺃﺧﺪﻡ ﺭﺳﻮﻝ اﻟﻠﻪ ﺻﻠﻰ اﻟﻠﻪ ﻋﻠﻴﻪ ﻭﺳﻠﻢ، ﺇﺫا ﻧﺰﻝ، ﻓﻜﻨﺖ ﺃﺳﻤﻌﻪ ﻛﺜﻴﺮا ﻳﻘﻮﻝ: اﻟﻠﻬﻢ ﺇﻧﻲ ﺃﻋﻮﺫ ﺑﻚ ﻣﻦ اﻟﻬﻢ ﻭاﻟﺤﺰﻥ، ﻭاﻟﻌﺠﺰ ﻭاﻟﻜﺴﻞ، ﻭاﻟﺒﺨﻞ ﻭاﻟﺠﺒﻦ، ﻭﺿﻠﻊ اﻟﺪﻳﻦ، ﻭﻏﻠﺒﺔ اﻟﺮﺟﺎﻝ

അനസ് ബിൻ മാലികിൽ (റ) നിന്ന്: പ്രവാചകൻ (സ) (അനസിന്റെ പിതാവായ) അബൂ ത്വൽഹയോട് പറഞ്ഞു: ഞാൻ ഖൈബർ (യുദ്ധത്തിന്) പുറപ്പെടുമ്പോൾ നിങ്ങളുടെ മക്കളിൽ ഒരാളെ എനിക്ക് സഹായങ്ങൾ ചെയ്തു തരാനായി കൂടെ കൂട്ടുക. അപ്പോൾ അബൂ ത്വൽഹ എന്നെ അദ്ദേഹത്തിന്റെ പിന്നിലിരുത്തി (കുതിരപ്പുറത്ത്) പുറപ്പെട്ടു. ഞാനന്ന് പ്രായപൂർത്തി എത്താറായ പയ്യനായിരുന്നു. (പടയോട്ടത്തിനിടയിൽ) ഒരു സ്ഥലത്ത് വിശ്രമിക്കാൻ ഇറങ്ങിയാൽ ഞാൻ അല്ലാഹുവിന്റെ ദൂതന്(സ) സഹായങ്ങൾ ചെയ്തു കൊടുക്കുമായിരുന്നു. അദ്ദേഹം (സ) ധാരാളമായി ഇപ്രകാരം പ്രാർത്ഥിക്കുന്നത് ഞാൻ കേൾക്കുമായിരുന്നു: “അല്ലാഹുവേ! ആകുലത, ദുഃഖം, ദുര്‍ബലത, മടി, പിശുക്ക്, ഭീരുത്വം, കടഭാരം, ആളുകള്‍ എന്നെ കീഴ്‌പ്പെടുത്തല്‍ എന്നിവയില്‍ നിന്നെല്ലാം ഞാന്‍ നിന്നോട് രക്ഷതേടുന്നു.” (സ്വഹീഹുൽ ബുഖാരി: 2893)

പ്രവാചകന്റെ (സ) മാതൃസഹോദരിയാണ് അനസിന്റെ (റ) മാതാവ് ഉമ്മു സുലൈം. അഥവാ അനസിന്റെ കൊച്ചാപ്പയാണ് പ്രവാചകൻ (സ) എന്ന് സാന്ദർഭികമായി സൂചിപ്പിക്കട്ടെ. സാധാരണക്കാർ ഈ ഹദീസ് വായിക്കുമ്പോൾ അതിൽ ലൈംഗികതയുടെ ലാഞ്ചന പോലും കണ്ടെന്നു വരില്ല. (അതുകൊണ്ടാണ് വിമർശകർ ഹദീസിന്റെ നമ്പർ മാത്രം കൊടുക്കുന്നത്) പക്ഷെ മനസ്സിൽ രോഗമുള്ളവർ ഇത് വായിച്ചപ്പോൾ സ്വവർഗരതിയും ബാലാനുരാഗവുമൊക്കെ കണ്ടെത്തി. ഉപ്പയോടൊപ്പം യാത്ര ചെയ്ത് യാത്രക്കിടയിൽ കൊച്ചാപ്പക്ക് സഹായങ്ങൾ ചെയ്ത് കൊടുക്കുന്നതിൽ ലൈംഗികത ചികഞ്ഞെടുക്കാൻ പോൺ സിനിമാ സംവിധായകന്മാരുടെ വൈദഗ്‌ദ്ധ്യം തന്നെ വേണം. ഈ പരിചയ സമ്പന്നതയുടെ വെളിച്ചത്തിൽ വിമർശകർ രണ്ട് ലൈംഗിക സൂചനകൾ മോർഫ് ചെയ്തെടുത്തു.

ഹദീസിൽ “(പടയോട്ടത്തിനിടയിൽ) ഒരു സ്ഥലത്ത് വിശ്രമിക്കാൻ ഇറങ്ങിയാൽ ഞാൻ അല്ലാഹുവിന്റെ ദൂതന് (സ) സഹായങ്ങൾ ചെയ്തു കൊടുക്കുമായിരുന്നു.” എന്ന വരിയാണ് ഒന്ന്. വിശ്രമിക്കാനിറങ്ങുമ്പോൾ ഒരാൾക്ക് സഹായങ്ങൾ ചെയ്യുക എന്ന് പറഞ്ഞാൽ അത് “കാമകേളിയിൽ ഏർപ്പെട്ടു” എന്നതിനുള്ള തെളിവാണ് പോലും!!

സ്ത്രീ, ബാലൻ, ശവം, കിടക്ക, ഉറക്കം, വിശ്രമം തുടങ്ങിയ പദങ്ങൾ ഒരു വാചകത്തിൽ വരുമ്പോഴേക്കും ലൈംഗിക ഉത്ഥാപനം ഉണ്ടാകുന്ന ഞരമ്പുരോഗികൾക്കല്ലാതെ ഈ ഹദീസിൽ എന്ത് ലൈംഗീകതയാണ് നിരീക്ഷിക്കാൻ കഴിയുക ?! ലൈംഗികതയുടെ ലാഞ്ചനയുള്ള ഒരു വരിയെങ്കിലും ഹദീസിൽ കാണിച്ചു തരാൻ ഈ ലൈംഗിക രോഗികൾക്ക് കഴിയുമോ ?!

പ്രവാചകന് ഭക്ഷണമെത്തിക്കുക, ആളുകളെ തന്റെ അടുത്തേക്ക് വിരുന്നിനായോ മറ്റൊ ക്ഷണിക്കുക (സ്വഹീഹു മുസ്‌ലിം:3580) ആവശ്യ സാധനങ്ങൾ വാങ്ങിപ്പിക്കുക (സ്വഹീഹു മുസ്‌ലിം: 2310) സന്ദേശങ്ങൾ അറിയിക്കാനുള്ളവരെ അറിയിക്കാനായി നിയോഗിക്കുക (സ്വഹീഹു ബുഖാരി: 5170) എന്നതൊക്കെയാണ് താൻ ചെയ്തിരുന്ന സഹായങ്ങൾ എന്നും പ്രവാചകൻ അശ്ലീലം പറയുന്നവനൊ പ്രവർത്തിക്കുന്നവനോ ആയിരുന്നില്ല എന്നും (സ്വഹീഹു ബുഖാരി: 6031) അനസ് (റ) മറ്റു ഹദീസുകളിൽ വിവരിക്കുന്നുണ്ടെങ്കിലും അതൊന്നും ഈ വെറുപ്പിന്റെ അപ്പോസ്തലന്മാർക്ക് ബാധകമല്ല.

ഹദീസിനെ ലൈംഗീകവൽക്കരിക്കാൻ വിമർശകർ പയറ്റുന്ന മറ്റൊരു ദുർവ്യാഖ്യാനം ഇപ്രകാരമാണ്:

ഹദീസിൽ പ്രവാചകൻ (സ) ധാരാളമായി പ്രാർത്ഥിക്കുമായിരുന്നു എന്ന് അനസ് വിവരിക്കുന്ന പ്രാർത്ഥനയിൽ “ആളുകള്‍ എന്നെ കീഴ്‌പ്പെടുത്തല്‍ എന്നിവയില്‍ നിന്നെല്ലാം ഞാന്‍ നിന്നോട് രക്ഷതേടുന്നു” എന്ന വാചകമുണ്ട്. ഇതിന് overcome by men എന്നാണ് ഇംഗ്ലീഷ് ഭാഷയിൽ നൽകിയിരിക്കുന്ന പരിഭാഷ. ഇത് അനസിനെ മുഹമ്മദ് നബി ലൈംഗീകമായി പീഡിപ്പിച്ചതിനെ തെളിയിക്കുന്നു എന്നാണ് (ദുർ)വ്യാഖ്യാനം.!!! അങ്ങനെയാണെങ്കിൽ ഈ പ്രാർത്ഥന പ്രാർത്ഥിക്കേണ്ടിയിരുന്നത് അനസ് (റ) അല്ലെ ? പ്രവാചകൻ അല്ലല്ലൊ എന്ന് ചിന്തിക്കാനുള്ള സാമാന്യ ബുദ്ധി വിമർശകർക്ക് – വർഗീയതയുടെ അതിപ്രസരത്താൽ – നഷ്ടപ്പെട്ടിരിക്കുന്നു.

പ്രാർത്ഥനയിലെ (ﻏﻠﺒﺔ اﻟﺮﺟﺎﻝ) ‘ആളുകള്‍ എന്നെ കീഴ്‌പ്പെടുത്തല്‍’ എന്നതിൽ ലൈംഗികതയൊന്നും ഇല്ല.

ശൈഖ് ശംസുൽ ഹക് അളീമാബാദി പറഞ്ഞു: ‘ശക്തിയും സ്വാധീനവുമുള്ളവർ ദുർബലരെ അടിച്ചമർത്തുന്നതാണ് ഗലബത്തുർ രിജാൽ. രിജാൽ എന്നത് കൊണ്ട് ഇവിടെ ഉദ്ദേശം അക്രമികളും കടം തന്നത് ചൂഷണം ചെയ്ത് സാമ്പത്തികമായി ഉപദ്രവിക്കുന്നവരുമാണ്. അതിൽ നിന്നും പ്രവാചകൻ ശരണം തേടാൻ കാരണം അത് മാനസികമായി ഒരാളെ തളർത്തിക്കളയും എന്നതിനാലാണ്.’ (ഔനുൽ മഅ്ബൂദ്: 4/281)

ആകുലത, ദുഃഖം, ദുര്‍ബലത, മടി, പിശുക്ക്, ഭീരുത്വം, കടഭാരം എന്നിവയിൽ നിന്ന് അല്ലാഹുവോട് രക്ഷ തേടുന്നതിനൊപ്പം ‘മറ്റുള്ളവർ തലയിൽ കയറി നിരങ്ങുന്നതിൽ നിന്നും’ പ്രവാചകൻ (സ) രക്ഷ തേടി. ഇതിലെവിടെയാണ് “കാമകേളിയിൽ ഏർപ്പെടൽ” ??? കടം കയറി പള്ളിയിൽ ദുഖിതനായി ഇരിക്കുന്ന അബൂഉമാമ എന്ന അനുചരനും ഇതേ പ്രാർത്ഥന പ്രവാചകൻ (സ) പഠിപ്പിച്ചു കൊടുത്തതായി ഹദീസ് ഉദ്ധരിക്കപ്പെട്ടിരിക്കുന്നു. (അബൂദാവൂദ്: 1555)

2. رأيْتُ رسولَ اللهِ صلَّى اللهُ عليه وسلَّمَ يَمُصُّ لِسانَه -أو قال: شَفَتَه، يعني: الحسَنَ بنَ عليٍّ صلواتُ اللهِ عليه-، وإنَّه لن يُعذَّبَ لِسانٌ أو شَفَتانِ مَصَّهُما رسولُ اللهِ صلَّى اللهُ عليه وسلَّمَ.

പ്രവാചകന് തന്റെ പേരകുഞ്ഞായിരുന്ന ഹസനോട് ഉണ്ടായിരുന്ന അതിയായ സ്നേഹത്തെ സംബന്ധിച്ച് സ്മരിക്കവെ മുആവിയ (റ) പറയുന്നു: അല്ലാഹുവിന്റെ ദൂതൻ ഹസൻ ഇബ്നു അലിയുടെ ചുണ്ട് അതൊ നാവ് (ശരിക്കും ഓർമ്മയില്ല) ചപ്പുന്നത് ഞാൻ കണ്ടിട്ടുണ്ട്…” (മുസ്നദു അഹ്മദ്: 16848)

“മുഹമ്മദ് ആൺകുട്ടികളുടെ നാവും ചുണ്ടും നക്കിയിരുന്നു” എന്ന് പറഞ്ഞ് ഹദീസിന്റെ നമ്പർ കൊടുത്തവർ ആ ‘ആൺകുട്ടികൾ’ എന്ന ബഹുവചനമല്ല ഹദീസിൽ ഉള്ളത് എന്നും തന്റെ പേരകുഞ്ഞാണെന്നും വ്യക്തമാകാതിരിക്കാനാണ് ‘നമ്പർ’ ഇറക്കിയത്. യഥാർത്ഥത്തിൽ, ഒരു സ്നേഹ നിധിയായ മുത്തച്ഛൻ മുലകുടി പ്രായമുള്ള തന്റെ പേരകുഞ്ഞിനെ ലാളിക്കുന്നതിനിടയിൽ ഗാഢമായി ചുംബിച്ചതാണ് ഹദീസിലെ രംഗം. അതും എല്ലാവരും കാൺകെ. ഇതിനെയാണ് വിമർശകർ സ്വവർഗ്ഗഭോഗമായും കുട്ടികളോടുള്ള ലൈംഗിക ആസക്തിയായും (pedophilia) ചിത്രീകരിക്കുന്നത് ! ഒരു പിതാവ് തന്റെ പിഞ്ചുകുഞ്ഞിനെ സ്നേഹത്താൽ ചുംബിച്ചെന്നും ചപ്പിയെന്നും വരാം. ആ പുത്രവാത്സല്യവും കുട്ടികളോടുള്ള ലൈംഗിക ആസക്തിയും തമ്മിലുള്ള പരസ്പര വ്യത്യാസം തിരിച്ചറിയാനുള്ള കഴിവ് ബന്ധങ്ങളുടെ പവിത്രത തിരിച്ചറിയുന്നവർക്ക് പറഞ്ഞിട്ടുള്ളതാണ്; അമ്മമാരെയും പെങ്ങമ്മാരെയും ഉൾപ്പെടെ ആരെയും ഭോഗിക്കാം എന്നും അഗമ്യഗമനം (incest) ലൈംഗിക സ്വാതന്ത്ര്യത്തിന്റെ ഭാഗമാണെന്നും സ്റ്റേജ് ഒരുക്കി പ്രസംഗിക്കുകയും യൂറ്റൂബിൽ പരസ്യമായി പ്രചരിപ്പിക്കുകയും ചെയ്യുന്നവർക്ക് ലഭിക്കുന്നതല്ല. (https://youtu.be/nkiBbtydlwA)

ഇനി, പ്രവാചകൻ (സ) മുലകുടി പ്രായമുള്ള പേരകുഞ്ഞിന്റെ നാവ് ചപ്പിയ സാഹചര്യം ചില നിവേദനങ്ങളിൽ വന്നതു കൂടി ചേർത്തു വായിക്കുമ്പോഴാണ് സംഭവത്തിന്റെ യഥാർത്ഥ ചിത്രം നമുക്ക് ലഭിക്കുക. സംഭവത്തിന്റെ ചുരുക്കമിതാണ്:

ഒരിക്കൽ ഒരു യാത്രാ വേളയിൽ മുല കുടി പ്രായക്കാരായ, പ്രവാചകന്റെ (സ) പേര കിടാങ്ങളായ ഹസൻ, ഹുസൈൻ എന്നിവർ (റ) ശക്തമായി കരയാൻ തുടങ്ങി. അവരോടൊപ്പം അവരുടെ മാതാവുമുണ്ട്. “എന്റെ കുഞ്ഞുമക്കൾ എന്തേ കരയുന്നത് ?” എന്ന് പ്രവാചകൻ (സ) ചോദിച്ചു. ഫാതിമ പറഞ്ഞു: ദാഹം കൊണ്ട്. പ്രവാചകൻ (സ) വെള്ളമന്വേഷിച്ചു. അന്ന് വരൾച്ചയുടെ സമയമാണ്. അദ്ദേഹം ജനങ്ങളോട് വിളിച്ച് ചോദിച്ചു: “നിങ്ങളുടെ ആരുടെയെങ്കിലും അടുക്കൽ വെള്ളമുണ്ടോ ?” ആരുടെ കയ്യിലുമുണ്ടായില്ല. കുഞ്ഞുങ്ങൾ ശക്തമായി കരഞ്ഞു കൊണ്ടിരുന്നു. അപ്പോൾ പ്രവാചകൻ (സ) പറഞ്ഞു: “അവരിൽ ഒരാളെ ഇങ്ങു തരൂ” കുഞ്ഞുങ്ങളിലൊരാളെ പ്രവാചകൻ (സ) കയ്യിലെടുത്തു, മടിയിൽ കിടത്തി, നാവ് നീട്ടി. കുഞ്ഞ് (ഹസൻ (റ)) അദ്ദേഹത്തിന്റെ നാവ് ചപ്പാൻ തുടങ്ങി. കരച്ചിൽ നിർത്തി ശാന്തമായി ഉറങ്ങി…. (അൽ മുഅ്ജമുൽ കബീർ: ത്വബ്റാനി: 2656)

3. ഒരു മനുഷ്യന് പ്രവർത്തിക്കാൻ കഴിയുന്നതിൽ വെച്ച് ഏറ്റവും വലിയ പാതകമായി സ്വവർഗരതിയെ പഠിപ്പിച്ച വ്യക്തിയാണ് പ്രവാചകൻ (സ). സ്വവർഗരതിയിൽ കൂട്ടായി നിർലജ്ജം അഭിരമിച്ച ലൂത്ത് നബിയുടെ ജനതയെ അല്ലാഹു ശിക്ഷിച്ച ചരിത്രം പ്രവാചകൻ (സ) ഈ ലോകത്തെ പഠിപ്പിച്ചു:

“അങ്ങനെ സൂര്യോദയത്തോടെ ആ ഘോരശബ്ദം അവരെ പിടികൂടി. അങ്ങനെ ആ രാജ്യത്തെ നാം തലകീഴായി മറിക്കുകയും, ചുട്ടുപഴുത്ത ഇഷ്ടികക്കല്ലുകള്‍ അവരുടെ മേല്‍ നാം വര്‍ഷിക്കുകയും ചെയ്തു.” (കുർആൻ 15: 73,74)

പ്രവാചകൻ (സ) പറഞ്ഞു: ആരെങ്കിലും ലൂത്ത് നബിയുടെ ജനതയുടെ പ്രവർത്തനം (സ്വവർഗരതി) പ്രവർത്തിച്ചാൽ ആ പ്രവർത്തനത്തിൽ ഏർപ്പെട്ടവന്റെയും അതിന് വിധേയനായി നിന്നു കൊടുത്തവന്റെയും മേൽ വധശിക്ഷ നടപ്പാക്കണം. (തുർമുദി: 1456, അബൂദാവൂദ്: 4462, ഇബ്നുമാജ: 2561)

പ്രവാചകൻ (സ) പറഞ്ഞു: ലൂത്ത് നബിയുടെ ജനതയുടെ പ്രവർത്തനം (സ്വവർഗരതി) പ്രവർത്തിച്ചവനെ അല്ലാഹു ശപിച്ചിരിക്കുന്നു. പ്രവാചകൻ (സ) മൂന്ന് തവണ ഇത് ആവർത്തിച്ചു പറഞ്ഞു. (മുസ്നദു അഹ്മദ്: 2915)

വിമർശനം:

“അജ്ഞാത കാലഘട്ടത്തിലെ പോലെ ആരെങ്കിലും തന്റെ ഗോത്ര വംശത്തെക്കുറിച്ച് വീമ്പിളക്കി അതിലേക്ക് ക്ഷണിക്കുന്നത് നിങ്ങൾ കേൾക്കുന്നുവെങ്കിൽ, പിതാവിന്റെ പുരുഷ അംഗത്തെ കടിക്കാൻ പറയുക, എന്ന് മുഹമ്മദ് നബി പറഞ്ഞു.

മറുപടി:

1. ഹദീസിന്റെ നിവേദക പരമ്പരകളെല്ലാം ദുർബലമാണ്. സ്വീകാര്യയോഗ്യവും സ്ഥിരപ്പെട്ടവയുമായ ഹദീസുകൾക്ക് എതിരുമാണ്.

കാരണങ്ങൾ:

ഹദീസ് സ്വഹീഹ് (സ്വീകാര്യതയുടെ നിബന്ധനകൾ പൂർത്തീകരിക്കപ്പെട്ടത്) അല്ല; ദുർബലമാണ്. ഹദീസിന്റെ ദുർബലതയെ സംബന്ധിച്ച ചർച്ചകളിലേക്ക് കടക്കും മുമ്പ് ഹദീസിന്റെ വിവർത്തനത്തിൽ സംഭവിച്ചു പോവുന്ന ആശയ ചോർച്ചയെ സാന്ദർഭികമായി ഒന്ന് വിലയിരുത്തേണ്ടതുണ്ട്.

مَنْ تَعَزَّى بِعَزَاءِ الْجَاهِلِيَّةِ، فَأَعِضُّوهُ، وَلَا تَكْنُوا

പിതാവിന്റെ പുരുഷ അംഗത്തെ കടിക്കാനൊന്നും ഹദീസിലില്ല. അറബി ഭാഷാ പ്രയോഗങ്ങളെ സംബന്ധിച്ച സൂക്ഷമമായ അറിവിന്റെ അഭാവമാണ് ഇത്തരമൊരു വിവർത്തനാർത്ഥത്തിലേക്ക് നയിക്കുന്നത്. ഹദീസിലെ ‘അൽ അദ്ദു’ (الْعَضُّ) എന്ന പദത്തിന് പല്ല് കൊണ്ട് കടിച്ചു പിടിക്കുക എന്ന ഒരു അർത്ഥമുണ്ടെങ്കിലും വാചകത്തിനനുസരിച്ച് അതിന്റെ അർത്ഥം വ്യത്യാസപെടുന്നുവെന്നത് അറബി ഭാഷാ പണ്ഡിതർ വ്യക്തമാക്കിയിട്ടുണ്ട്. ഒരു വസ്തുവെയോ, ആശയത്തെയോ, വ്യക്തിയെയോ വിട്ടുവീഴ്ച്ചയില്ലാതെ മുറുകെ പിടിക്കുക, വേർപിരിയാതെ കൂടെ കൂടുക, നിർബാധം ചേർന്നിരിക്കുക എന്നൊക്കെയാണ് ‘അദീദ്’ (العَضِيض) എന്ന പദത്തിന്റെ അടിസ്ഥാനപരമായ അർത്ഥം എന്ന് അൽ മുഗ്‌രിബ് ഫീ തർതീബിൽ മുഅ്രിബ്: മുത്വരിസി, അൽ മിസ്ബാഹുൽ മുനീർ: ഫയ്യൂമി, മുഅ്ജമുല്ലുഗ അൽ അറബിയ്യ: അഹ്മദ് മുഖ്താർ, അസാസുൽ ബലാഗ: സമഖ്ശരി, താജുല്ലുഗ വസിഹാഹുൽ അറബിയ്യ: ജൗഹരി, ലിസാനുൽ അറബ്: ഇബ്നു മൻദൂർ തുടങ്ങിയ അറബി ഭാഷാ നിഘണ്ടുക്കൾ പ്രസ്ഥാവിക്കുന്നുണ്ട്.

മറ്റു ഹദീസുകളിൽ ‘അൽ അദ്ദു’ (الْعَضُّ), ‘അദീദ്’ (العَضِيض) എന്നീ പദങ്ങൾ പ്രയോഗിച്ചേടത്തും ‘പല്ല് കൊണ്ട് കടിക്കുക’ എന്ന് അക്ഷരാർത്ഥത്തിൽ ഉദ്ദേശിക്കപ്പെടുന്നില്ല എന്ന് വ്യക്തമാവുന്നു. ഉദാഹരണങ്ങൾ കാണുക:

فَعَلَيْكُمْ بِسُنَّتِي وَسُنَّةِ الْخُلَفَاءِ الرَّاشِدِينَ الْمَهْدِيينَ، عَضُّوا عَلَيْهَا بِالنَّوَاجِذِ.

“നിങ്ങൾ എന്റെ ചര്യയും സച്ചരിതരായ ഖലീഫമാരുടെ ചര്യയും സ്വീകരിക്കുക. ‘അവയെ അണപ്പല്ല് കൊണ്ട് കടിച്ചു പിടിക്കുകയും ചെയ്യുക” എന്ന് പ്രവാചകൻ (സ) പറഞ്ഞതായി ഒരു ഹദീസിൽ കാണാം. (അബൂദാവൂദ്: 4607, തുർമുദി: 266)

ഹദീസിലെ عَضُّوا عَلَيْهَا بِالنَّوَاجِذِ എന്ന വാചകത്തിന്റെ പദാനുപദ വിവർത്തനമാണ് ‘അവയെ അണപ്പല്ല് കൊണ്ട് കടിച്ചു പിടിക്കുകയും ചെയ്യുക’ എന്ന് നൽകിയിരിക്കുന്നത്. എന്നാൽ പ്രവാചകന്റെയും(സ) ഖലീഫമാരുടെയും ജീവിതചര്യ അണപ്പല്ല് കൊണ്ട് കടിച്ചു പിടിക്കുക എന്നത് അക്ഷരാർത്ഥിൽ സംഭവ്യമല്ലല്ലൊ. ഇവിടെ ഉദ്ദേശ്യം അവരുടെ ജീവിതചര്യയെ ആദർശമായി മുറുകെ പിടിക്കുക, വേർപിരിയാതെ കൂടെ കൂട്ടുക എന്നൊക്കെയാണ്.

മറ്റൊരു ഹദീസിൽ പ്രവാചകൻ (സ) പറഞ്ഞതായി കാണാം:

ولَو أنْ تَعَضَّ بأصلِ شجَرَةٍ حتى يُدْرِكَكَ الموتُ وأنتَ كذلِكَ

“… മുസ്‌ലിംകളുടെ നേതാവിനേയും സംഘ ശക്തിയെയും നീ മുറുകെ പിടിക്കുക… മുസ്‌ലിംകൾക്ക് ഒരു നേതാവും സംഘ ശക്തിയും ഇല്ലെങ്കിൽ നീ എല്ലാ (ഒറ്റപ്പെട്ട അരക്ഷിത) കക്ഷികളേയും വേർപ്പിരിയുക. ‘ഒരു മരത്തിന്റെ മുരട് കടിച്ച് പിടിച്ച് കൊണ്ടെങ്കിലും’ (തനിച്ച് കഴിയുക). നിനക്ക് മരണം സംഭവിക്കുന്നത് വരെ അപ്രകാരം നിലകൊള്ളുക.” (സ്വഹീഹുൽ ബുഖാരി: 3606) ഹദീസിലെ ولَو أنْ تَعَضَّ بأصلِ شجَرَةٍ എന്ന വാചകത്തിന്റെ പദാനുപദ വിവർത്തനമാണ് ‘ഒരു മരത്തിന്റെ മുരട് കടിച്ച് പിടിച്ച് കൊണ്ടെങ്കിലും’ എന്ന് നൽകിയിരിക്കുന്നത്. ഇവിടെ മരത്തിന്റെ മുരട് കടിക്കുക എന്ന പദാനുപദ വിവർത്തനമല്ലല്ലൊ ഉദ്ദേശ്യം. ഉദ്ദേശ്യം മരത്തെ മുറുകെ പിടിക്കുക, വേർപിരിയാതെ കൂടെ കൂടുക എന്നൊക്കെയാണ്. അഥവാ ഒറ്റപ്പെട്ട അരക്ഷിത കക്ഷികൾ വിഹരിക്കുന്ന നാഗരികമായ പ്രദേശങ്ങളെ വിട്ടെറിഞ്ഞ് വൃക്ഷവന്യമായ ഗ്രാമങ്ങളിൽ ഗൃഹജമായ ജീവിതം നയിക്കുക എന്നാണ് ഹദീസിന്റെ താൽപര്യം.

ഇനി, ഈ നിഘണ്ടുക്കളുടെയൊന്നും സഹായമില്ലാതെ, അൽപം ‘ചന്ത പരിജ്ഞാന’മുണ്ടെങ്കിൽ തന്നെ ഹദീസിൽ ‘അൽ അദ്ദു’ (الْعَضُّ) എന്ന പദം കൊണ്ട്, ‘പല്ല് കൊണ്ട് കടിച്ചു പിടിക്കുക’ എന്ന അർത്ഥമല്ല ഉദ്ദേശിക്കപ്പെട്ടിരിക്കുന്നത് എന്ന് മനസ്സിലാക്കാം. കാരണം, ‘ലിംഗം കടിക്കുക’ എന്ന ഒരു തെറിയുണ്ടോ ? ലിംഗം ഈമ്പുക എന്നതാണ് തെറിയെന്ന് അൽപമെങ്കിലും തെറികൾ കേട്ടിട്ടുള്ളവർക്ക് അറിയാം. അപ്പോൾ ഹദീസിൽ വംശവെറിയനെ, ‘ലിംഗം ഈമ്പുക’ എന്ന ചന്ത തെറി വിളിക്കാനല്ല ആഹ്വാനം ചെയ്യുന്നത് എന്ന് വ്യക്തം.

പിന്നെ എന്താണ് ഹദീസിന്റെ വിവക്ഷ?

വംശ വെറിയെ (racism) ഇന്ന് ലോകത്ത് നിലനിൽക്കുന്ന ഏറ്റവും വലിയ പാതകങ്ങളിലും അപകടങ്ങളിലുമൊന്നായി ഇസ്‌ലാം കാണുന്നു. വർണ വർഗ വെറിയാണ് ലോകത്തെ രക്ത ചുഴിയിൽ ആഴ്ത്തിയ ലോക മഹാ യുദ്ധങ്ങളിലേക്ക് മനുഷ്യരെ നയിച്ച ചാലക ശക്തി. അടിമകളാക്കപ്പെട്ട കോടിക്കണക്കിന് മനുഷ്യരുടെ ചോരക്കൊണ്ട് എഴുതപ്പെട്ട സവർണ അധിനിവേശ ചരിത്ര കഥകൾ എല്ലാ ദേശക്കാർക്കും നാട്ടുകാർക്കും പറയാനുണ്ട്. എന്താണ് യഥാർത്ഥത്തിൽ വംശ വിരോധത്തിന്റെ അടിത്തറ? പിതാക്കന്മാരിലൂടെ ഒരു വംശത്തിനോ വർഗത്തിനോ മറ്റുള്ളവർക്ക് മേൽ പരിശുദ്ധിയും ശ്രേഷ്ഠതതയും സിദ്ധിക്കുന്നു എന്ന സങ്കൽപമാണല്ലൊ ഈ വിവേചന ആശയത്തിന് നിദാനം. ഒരു പ്രത്യേക വർണത്തിലൊ, ഗോത്രത്തിലോ, വംശത്തിലൊ ഉള്ള പിതാക്കന്മാരുടെ പുത്രന്മാരായി ജനിക്കാൻ കഴിഞ്ഞാൽ താൻ ശ്രേഷ്‌ഠനായി എന്നർത്ഥം. ഈ ശ്രേഷ്ഠത പരിശ്രമങ്ങളിലൂടെയൊ സേവനങ്ങളിലൂടെയൊ നേടിയെടുക്കാൻ സാധിക്കില്ല. പച്ചയായി പറഞ്ഞാൽ ഈ ശ്രേഷ്ഠതക്ക് നിദാനം ഏത് ലിംഗത്തിൽ നിന്നുള്ള ബീജത്തിൽ നിന്ന് പിറക്കുന്നു എന്നതാണ്. അഥവാ വംശവെറിയുടെ ആധാരം ലിംഗ ശ്രേഷ്ഠതയാണ്. ഇത്തരമൊരു തരം താഴ്ന്ന ആശയത്തിലേക്ക് ക്ഷണിക്കുന്നവനോട് “നീ നിന്റെ പിതാവിന്റെ ലിംഗത്തെ അവലംബിച്ചും ആദർശമായി ആചരിച്ചും ജീവിച്ചു കൊള്ളുക” എന്ന് പറയാനാണ് ഹദീസിൽ ഉള്ളടങ്ങിയ കൽപ്പന. ഇതിലൂടെ വംശ വെറിയെ (racism) പച്ചക്ക് തൊലിയുരിച്ചു കാണിക്കുകയാണ് ചെയ്യുന്നത്. വംശീയ വിരോധി യഥാർത്ഥത്തിൽ തങ്ങളുടെ പിതാക്കന്മാരുടെ ഏറ്റവും താഴ്ന്ന ഒരു അവയവമായ ലിംഗത്തെ ആദരിച്ചു പൂജിക്കുന്ന തനി വിഡ്ഢിയാണ്, വംശീയമായ യാഥാസ്ഥിതികത്വത്തിന്റെ ആരാധനാ മൂർത്തി താന്താങ്ങളുടെ പിതാക്കന്മാരുടെ ലിംഗങ്ങളാണ്. ആ ലിംഗങ്ങളുടെ മഹത്വവും പരിപാവനത്വവും ശ്രേഷ്ടതയുമൊക്കെയാണ് ഒരോ റേസിസ്റ്റും നാസിസ്റ്റും ഉൽഘോഷിക്കുന്നതും പ്രകീർത്തിക്കുന്നതും. അതിനപ്പുറം ഒരു ആശയ പൊലിമയും അവനില്ല എന്ന് പച്ചക്ക് പറഞ്ഞ് നാണം കെടുത്തലാണ് ഒരു റേസിസ്റ്റിന് നൽകാനുള്ള ശിക്ഷ.

ഈ അർത്ഥത്തിലാണ് ഹദീസ് നിലകൊള്ളുന്നത്. അല്ലാതെ വിമർശകർ ചിത്രീകരിക്കും പോലെ, അത് തർക്കമുണ്ടായാൽ പയറ്റാനായി ഉപയോഗിക്കേണ്ട ഒരു ചന്ത തെറിയല്ല. ഹദീസിനെ മുൻപത്തെ ഖണ്ഡികയിൽ വിശദീകരിച്ച ആശയത്തിൽ -വിമർശകർ ചിത്രീകരിക്കുന്ന രൂപത്തിലല്ല- സ്വീകരിച്ച ചില മുസലിം പണ്ഡിതന്മാരുണ്ട്. അവരെ മുൻ നിർത്തി ഈ ഹദീസ് സ്വീകാര്യയോഗ്യമായി അവതരിപ്പിക്കുകയാണ് വിമർശകർ സാധാരണ ചെയ്യാറുള്ളത്. പക്ഷെ ഹദീസിന്റെ നിവേദകപരമ്പരകളെല്ലാം ദുർബലമായതു കൊണ്ട് തന്നെ ഹദീസ് മുസ്‌ലിംകളുടെ അടുക്കൽ സ്വീകാര്യയോഗ്യമല്ല.

2. ഹദീസിന്റെ നിവേദകപരമ്പരകളെല്ലാം ദുർബലമാണ്.

നിവേദക പരമ്പര: ഒന്ന്:

ഔഫിബ്നു അബീ ജമീല അൽഅഅ്റാബി – ഹസനിൽ നിന്ന് – ഉത്ബി ഇബ്നു ദംറയിൽ നിന്ന് – അദ്ദേഹം പറഞ്ഞു:…

പരമ്പരയിലെ ഹസൻ, ഹസനുൽ ബസ്വരിയാണ്. അദ്ദേഹം പ്രസിദ്ധ ഹദീസ് പണ്ഡിതനാണെങ്കിലും ‘മുദല്ലിസാ’ണ്. (ത്വബകാത്തുൽ മുദല്ലിസീൻ: ഇബ്നു ഹജർ: 29, അത്തബ് യീൻ ഫീ അസ്മാഇൽ മുദല്ലിസീൻ: അബുൽ വഫാ,അത്തബ് യീൻ ഫീ അസ്മാഇൽ മുദല്ലിസീൻ: ഇബ്നുൽ അജ്മി: 20, അസ്സികാത്ത്: ഇബ്നു ഹിബ്ബാൻ)

ഹദീസ് പണ്ഡിതന്മാർ, ‘മുദല്ലിസെ’ന്ന സാങ്കേതിക പദമുപയോഗിച്ച് വിളിക്കുന്ന നിവേദകർ, തങ്ങൾ ആരിൽ നിന്നാണോ ഹദീസ് ഉദ്ധരിക്കുന്നത് അവരിൽ നിന്ന് ഇടയാളന്മാരില്ലാതെ നേരിട്ട് കേട്ടുവെന്ന് സൂചിപ്പിക്കുന്ന ‘എന്നോട് പറഞ്ഞു’ എന്ന വാചകം ഉപയോഗിക്കേണ്ടതുണ്ട്. ചർച്ച ചെയ്യപ്പെടുന്ന ഹദീസിൽ അപ്രകാരം ഒരു വാചകം ഹസനുൽ ബസ്വരി ഉപയോഗിച്ചിട്ടില്ല. “ഹസനിൽ നിന്ന് – ഉത്ബി ഇബ്നു ദംറയിൽ നിന്ന് ” എന്നാണ് വന്നിരിക്കുന്നത്. അതിനാൽ ഹദീസ്, ദുർബല ഹദീസുകളുടെ ഇനങ്ങളിലൊന്നായ ‘മുദല്ലസി’ൽ ഉൾപെടുന്നു.

കൂടാതെ, പരമ്പരയിലെ ഉത്ബി ഇബ്നു ദംറ അറിയപ്പെടാത്ത വ്യക്തിയാണെന്ന് ഇബ്നുൽ മദീനി പറയുന്നു. (തഹ്ദീബുതഹ്ദീബ്: 7:96)

നിവേദക പരമ്പര: രണ്ട്:

അബ്ദുല്ല പറഞ്ഞു – നമ്മോട് മുഹമ്മദ് ഇബ്നു അംറ് ഇബ്നുൽ അബ്ബാസ് അൽ ബാഹുലി അറിയിച്ചു – നമ്മോട് സുഫ്‌യാൻ പറഞ്ഞു – അദ്ദേഹം ആസ്വിമിൽ നിന്ന് – അദ്ദേഹം അബൂ ഉസ്മാനിൽ നിന്ന് – അദ്ദേഹം ഉബയ്യിൽ നിന്ന്….

പരമ്പരയിലെ മുഹമ്മദ് ഇബ്നു അംറ് ഇബ്നുൽ അബ്ബാസ് അൽ ബാഹുലിയെ വിശ്വസ്‌തനായി ആരും പ്രസ്താവിച്ചിട്ടില്ല. റാഫിദി- സിന്ദീക് എന്നിങ്ങനെ, ഹദീസ് പണ്ഡിതർ വ്യാജന്മാരെന്നും പിഴച്ച കക്ഷിക്കാരെന്നും വ്യക്തമാക്കിയ ചില തൽപര കക്ഷികൾ മാത്രമാണ് ഈ നിവേദകനെ വിശ്വസ്‌തനെന്ന് വിശേഷിപ്പിച്ച്. ഇത് ഈ നിവേദകനും തൽപരകക്ഷിക്കാരായ കള്ള കഥ നിർമാതാക്കളും തമ്മിലുള്ള അന്തർധാരയെ വെളിപ്പെടുത്തുന്നു. അതുകൊണ്ട് തന്നെ നിവേദനം ദുർബലമാണ്.

സമാനമായ ഹദീസ് ത്വബ്റാനി മറ്റൊരു പരമ്പരയിലൂടെ ഉദ്ധരിക്കുന്നുണ്ടെങ്കിലും അത് അങ്ങേയറ്റം ദുർബലമാണ്.

قال الطبراني : حدثنا أحمد بن مسعود ثنا عمرو بن أبي سلمة عن سعيد بن بشير عن قتادة عن الحسن عن مكحول عن عجرد بن مدراع التميمي

പരമ്പരയിലെ അംറിബ്നു അബീ സലമ നിവേദനത്തിൽ ധാരാളം തെറ്റുകൾ സംഭവിക്കുന്ന വ്യക്തിയാണ്. (തക്‌രീബു തഹ്ദീബ്: 1/736) പരമ്പരയിലെ മറ്റൊരു നിവേദകനായ സഈദിബ്നുൽ ബശീർ ദുർബലനാണ്. (അത്തഹ്ദീബ്) വേറെയും അനവധി ന്യൂനതകൾ പരമ്പരക്കുണ്ട്. ഇനി ഈ ദുർബലമായ പരമ്പരകളെല്ലാം ചേർത്തു വെച്ചാൽ അവ പരസ്പരം ശക്തിപകരുന്നു എന്ന് വാദിക്കാനും വകുപ്പില്ല. കാരണം വളരെയധികം വിശ്വസ്‌തരായ നിവേദകർ ഉദ്ധരിച്ച, സ്ഥിരപ്പെട്ട ഒട്ടനവധി ഹദീസുകൾക്ക് വിരുദ്ധമാണ് ഈ ദുർബല നിവേദനങ്ങൾ.

3. ഹദീസ് വളരെയധികം വിശ്വസ്‌തരായ നിവേദകർ ഉദ്ധരിച്ച, സ്ഥിരപ്പെട്ട ഒട്ടനവധി ഹദീസുകൾക്ക് വിരുദ്ധമാണ്.

പ്രസ്തുത ഹദീസുകളിൽ ചിലത്:

* അല്ലാഹുവിന്റെ തിരുദൂതൻ (സ) പറഞ്ഞു: “ഒരു വ്യക്തി തന്റെ പിതാവിനെ ശപിക്കുക എന്നത് വൻപാപങ്ങളിൽ ഒന്നാണ്.” അദ്ദേഹത്തോട് ചോദിക്കപ്പെട്ടു: അല്ലാഹുവിന്റെ തിരുദൂതരേ, എങ്ങനെയാണ് ഒരാൾ തന്റെ പിതാവിനെ ശപിക്കുക? അപ്പോൾ അദ്ദേഹം പറഞ്ഞു: “ഒരാൾ മറ്റൊരാളുടെ പിതാവിനെ ശകാരിക്കുന്നു. അപ്പോൾ മറ്റയാൾ തിരിച്ചും അയാളുടെ പിതാവിനേയും മാതാവിനേയും ശകാരിക്കുന്നു.” (സ്വഹീഹുൽ ബുഖാരി: 5636, സ്വഹീഹു മുസ്‌ലിം: 155)

* ആഇശ (റ) പറഞ്ഞു: ഒരു സ്ത്രീ പ്രവാചകനോട് ആർത്തവകാരിയായതിന് ശേഷമുള്ള കുളിയെ സംബന്ധിച്ച് ചോദിച്ചു. അവരോട് കുളിക്കാൻ അദ്ദേഹം കൽപിച്ചു, ഇപ്രകാരം പറയുകയും ചെയ്തു: കസ്‌തൂരി മുക്കിയ ഒരു പഞ്ഞിയെടുകയും അത് കൊണ്ട് ശുദ്ധിയാക്കുകയും ചെയ്യുക. സ്ത്രീ ചോദിച്ചു: എങ്ങനെയാണ് ഞാൻ ശുദ്ധിയാക്കേണ്ടത്? പ്രവാചകൻ (സ) പറഞ്ഞു: അത് കൊണ്ട് ശുദ്ധിയാക്കുകയും ചെയ്യുക. സ്ത്രീ ചോദിച്ചു: എങ്ങനെ? പ്രവാചകൻ (സ) പറഞ്ഞു: അല്ലാഹു പരിശുദ്ധൻ, ശുദ്ധിയാക്കുക. എന്നിട്ട് എന്നെ (ആഇശ) തന്റെ അടുത്തേക്ക് ക്ഷണിച്ച് (ആ സ്ത്രീക്ക് വ്യക്തമായി പറഞ്ഞു കൊടുക്കാൻ ആവശ്യപ്പെട്ടു). അപ്പോൾ ഞാൻ അവർക്ക് പറഞ്ഞു കൊടുത്തു: ആർത്തവ രക്തത്തിന്റെ പാട് പിന്തുടർന്ന് കഴുകി കളയുക… (സ്വഹീഹുൽ ബുഖാരി: 308, സ്വഹീഹു മുസ്‌ലിം: 332)

ചോദ്യകർത്താവിനോട് മുഖത്ത് നോക്കി ആർത്തവ രക്തത്തെ സംബന്ധിച്ചും സ്ത്രീകളുടെ അവയവങ്ങളെ സംബന്ധിച്ചും സംസാരിക്കാൻ ലജ്ജിച്ചത് കൊണ്ടാണ് “എങ്ങനെയാണ് ഞാൻ ശുദ്ധിയാക്കേണ്ടത്?” എന്ന് ചോദ്യകർത്താവ് ആവർത്തിച്ച് ചോദിച്ചിട്ടും പ്രവാചകൻ (സ) ഒഴിഞ്ഞു മാറിയതും അത് വിശദീകരിച്ചു കൊടുക്കാൻ ഭാര്യയോട് അപേക്ഷിച്ചതും. അതേ പ്രവാചകൻ (സ) ശിഷ്യന്മാരോട് പിതാക്കളുടെ അവയവം എടുത്തു പറഞ്ഞ് ശകാരിക്കാൻ ഉപദേശിക്കുകയെന്നത് മനശാസ്ത്രപരമായും സ്വാഭാവികമായും അസംഭവ്യമാണ്.

വിമർശനം:

മുഹമ്മദ് നബി, ശിഷ്യൻ സൈദിന്റെ ഭാര്യ സൈനബിനെ മോഹിക്കുകയും അദ്ദേഹത്തെ കൊണ്ട് ഭാര്യയെ നിർബന്ധപൂർവ്വം വിവാഹമോചനം ചെയ്യിപ്പിക്കുകയും ചെയ്തു. “അല്ലാഹു വെളിപ്പെടുത്താന്‍ പോകുന്ന ഒരു കാര്യം നിന്‍റെ മനസ്സില്‍ നീ മറച്ചു വെക്കുകയും ജനങ്ങളെ നീ പേടിക്കുകയും ചെയ്തിരുന്നു…” (കുർആൻ: 33:37) എന്ന് കുർആൻ സൂചിപ്പിച്ചത് മുഹമ്മദ് നബി സൈദിന്റെ ഭാര്യയെ വിവാഹം കഴിച്ചാല്‍ കൊള്ളാമെന്ന് ആഗ്രഹിച്ചതിനെ സംബന്ധിച്ചാണ്.

മറുപടി:

ചരിത്ര നിവേദനങ്ങൾ വളരെ ദുർബലവും സ്വീകാര്യയോഗ്യവും സ്ഥിരപ്പെട്ടവയുമായ ഹദീസുകൾക്ക് എതിരുമാണ്.

കാരണങ്ങൾ:

1. പ്രവാചകൻ (സ) സൈദിന്റെ(റ) ഭാര്യയെ മോഹിച്ചുവെന്നോ അദ്ദേഹത്തെ കൊണ്ട് ഭാര്യയെ നിർബന്ധപൂർവ്വം വിവാഹമോചനം ചെയ്യിപ്പിച്ചുവെന്നോ നേരിയ സൂചനയെങ്കിലും ഉൾകൊള്ളുന്ന നിവേദനങ്ങളുടെ നിവേദക പരമ്പരകൾ വളരെ ദുർബലങ്ങളാണ്: (അവലംബം: http://www.ebnmaryam.com/Zaynab.htm)

നിവേദക പരമ്പര: ഒന്ന്

ذكرها ابن سعد في طبقاته (8/101) و من طريقه ساقها ابن جرير في تاريخه (3/161) : قال : أخبرنا محمد بن عمر ، قال : حدثني عبد الله بن عامر الأسلمي ، عن محمد بن يحيى بن حبان ، قال :

ത്വബകാത്തു ഇബ്‌നു സഅ്ദ് (8/101): നമ്മോട് മുഹമ്മദിബ്‌നു ഉമർ പറഞ്ഞു – എന്നോട് അബ്ദുല്ലാഹിബ്‌നു ആമിറുൽ അസ്‌ലമി പറഞ്ഞു – മുഹമ്മദിബ്‌നു യഹ്‌യ ഇബ്‌നുഹിബ്ബാൻ പറഞ്ഞു:…

ഈ നിവേദക പരമ്പരയിൽ മൂന്ന് ന്യൂനതകൾ ഉണ്ട്:

ഒന്നാമത്തെ ന്യൂനത: നിവേദനം മുർസൽ (المرسل ) അഥവാ പരമ്പര കണ്ണിമുറിഞ്ഞതാണ്. പ്രവാചക കാലഘട്ടത്തിലെ ഈ സംഭവം പ്രവാചക കാലഘട്ടത്തിന് ശേഷം, ഒരു തലമുറക്ക് ശേഷം ജനിച്ച മുഹമ്മദിബ്‌നു യഹ്‌യ ഇബ്‌നുഹിബ്ബാൻ എന്ന താബിഈ (ജനനം: ഹിജ്റാബ്ദം: 47) യാണ് പ്രസ്ഥാവിക്കുന്നത്. അദ്ദേഹം ഈ സംഭവത്തിന് സാക്ഷിയല്ല. (തഹ്ദീബ്:9/507-508)

രണ്ടാമത്തെ ന്യൂനത: നിവേദക പരമ്പരയിലെ അബ്ദുല്ലാഹിബ്‌നു ആമിറുൽ അസ്‌ലമി ദുർബലനാണ് എന്നതിൽ ഹദീസ് പണ്ഡിതന്മാർ യോജിച്ചിരിക്കുന്നു. ഇമാം അബൂ ഹാതിം പറഞ്ഞു: വിശ്വസ്തരായ നിവേദകർക്കെതിരെ വളരെ ദുർബലമായ നിവേദനങ്ങൾ ഉദ്ധരിക്കുന്ന വ്യക്തിയാണ് അബ്ദുല്ലാഹിബ്‌നു ആമിറുൽ അസ്‌ലമി. (തഹ്ദീബ്: 5/275, മീസാനുൽ ഇഅ്തിദാൽ: 2/448 )

മൂന്നാമത്തെ ന്യൂനത: നിവേദക പരമ്പരയിലെ മുഹമ്മദിബ്‌നു ഉമർ വാക്കിദുൽ അസ്‌ലമി (സത്യസന്ധതയിൽ) ദുർബലനാണ്. അഹ്മദിബ്‌നു ഹമ്പൽ പറഞ്ഞു: മുഹമ്മദിബ്‌നു ഉമർ വാക്കിദുൽ അസ്‌ലമി നുണയനാണ്; അയാൾ ഹദീസുകളിൽ കോട്ടിമാട്ടുമായിരുന്നു. യഹ്‌യ പറഞ്ഞു: അയാൾ വിശ്വസ്തനല്ല. അയാളുടെ ഹദീസുകൾ എഴുതിവെക്കാൻ കൊള്ളാത്തത്രയും അവിശ്വസനീയമാണ്. ഇമാം ബുഖാരി, റാസി, നസാഈ എന്നിവർ പറഞ്ഞു: അയാൾ കളവു കൊണ്ട് ആരോപിതനാണ്. റാസി, നസാഈ എന്നിവർ പറഞ്ഞു: അയാൾ വ്യാജ ഹദീസുകൾ ഉണ്ടാക്കുന്ന വ്യക്തിയായിരുന്നു. ഇമാം ദാറക്കുത്നി പറഞ്ഞു: അയാളിൽ ദൗർബല്യമുണ്ട്. ഇസ്ഹാകിബ്‌നു റാഹൂയ പറഞ്ഞു: അയാൾ നുണയനാണ്. (അദ്ദുഅഫാഉ വൽ മത്റൂകീൻ: ഇബ്‌നുൽ ജൗസി: 3 / 87, അദ്ദുഅഫാഉ സ്സ്വഗീർ: ബുഖാരി: 334, അൽ ജർഹുവതഅദീൽ: അബൂഹാതിം: 8/21, അൽ കാമിൽ ഇബ്‌നു അദിയ്യ്: 7/ 481)

നിവേദക പരമ്പര: രണ്ട്:

ذكرها ابن جرير في تفسيره (22/13 ) قال : حدثني يونس ، قال : أخبرنا ابن وهب ، قال : قال ابن زيد : كان النبي صلى الله عليه وسلم

ത്വബ്‌രി തന്റെ തഫ്സീറിൽ (22/13) പറഞ്ഞു: എന്നോട് യൂനസ് പറഞ്ഞു – നമ്മോട് ഇബ്‌നു വഹബ് അറിയിച്ചു – ഇബ്‌നു സൈദ് പറഞ്ഞു:….

ഈ നിവേദക പരമ്പരയിൽ രണ്ട് ന്യൂനതകൾ ഉണ്ട്:

ഒന്നാമത്തെ ന്യൂനത: നിവേദനം മുഅ്ദൽ (المعضل) അഥവാ പരമ്പരയിലെ രണ്ട് കണ്ണികൾ മുറിഞ്ഞതാണ്. പ്രവാചക കാലഘട്ടത്തിലെ ഈ സംഭവം പ്രവാചക കാലഘട്ടത്തിന് രണ്ട് തലമുറക്ക് ശേഷം ജനിച്ച ഇബ്‌നു സൈദ് എന്ന അബ്ദുർറഹ്മാൻ ഇബ്‌നു സൈദ് ഇബ്‌നു അസ്‌ലം ആണ് ഉദ്ധരിക്കുന്നത്. അദ്ദേഹം ഈ സംഭവത്തിന് സാക്ഷിയല്ല. പ്രവാചകാനുചരനോ (സ്വഹാബി), പ്രവാചകാനുചരരുടെ അനുചരനോ (താബിഈ) അല്ല. അതുകൊണ്ട് തന്നെ പരമ്പരയിൽ രണ്ടോ രണ്ടിലധികമോ നിവേദകർ വിട്ടു പോയിട്ടുണ്ട്.

രണ്ടാമത്തെ ന്യൂനത: അബ്ദുർറഹ്മാൻ ഇബ്‌നു സൈദ് ഇബ്‌നു അസ്‌ലം എന്ന ഈ നിവേദകൻ തന്നെ അങ്ങേയറ്റം ദുർബലനാണ് എന്ന കാര്യത്തിൽ ഹദീസ് പണ്ഡിതർ യോജിച്ചിരിക്കുന്നു. ഇമാം ബുഖാരി, അബൂ ഹാതിം, അലിയ്യിബ്‌നുൽ മദീനി എന്നിവർ പറയുന്നത് ഇയാൾ അങ്ങേയറ്റം ദുർബലനാണ് എന്നാണ്. ചിലരെല്ലാം ഇയാൾ മത്റൂക് (متروك) അഥവാ കളവ് കൊണ്ട് ആരോപിതനാണെന്നും അഭിപ്രായപ്പെടുന്നു. (തഹ്ദീബ്: 6/178)

നിവേദക പരമ്പര: മൂന്ന്:

ذكرها أحمد في مسنده (3/149-150) ، قال : حدثنا مؤمل بن إسماعيل قال : حدثنا حماد بن زيد ، قال : حدثنا ثابت عن أنس قال :…

അഹ്മദ് തന്റെ മുസ്നദിൽ (3/149-150) പറഞ്ഞു: നമ്മോട് മുഅമ്മിൽ ഇബ്‌നു ഇസ്മാഈൽ പറഞ്ഞു – നമ്മോട് ഹമ്മാദി ബ്‌നു സൈദ് പറഞ്ഞു – നമ്മോട് സാബിത് പറഞ്ഞു – അദ്ദേഹം അനസിൽ നിന്ന്:….

നിവേദക പരമ്പരയിലെ മുഅമ്മിൽ ഇബ്‌നു ഇസ്മാഈലിനെ ഹദീസ് പണ്ഡിതന്മാരിൽ അധികപേരും വളരെ ദുർബലനായ നിവേദകനായാണ് കാണുന്നത്. അദ്ദേഹം തന്റെ ഗുരുവര്യമാരിൽ നിന്ന് മുൻകറായ ഹദീസുകൾ (വിശ്വസ്‌തരായ നിവേദകരുടെ നിവേദനങ്ങൾക്ക് ഘടക വിരുദ്ധമായ അങ്ങേയറ്റം ദുർബലമായ നിവേദനങ്ങൾ) ഉദ്ധരിക്കുമായിരുന്നു. മുഹമ്മദിബ്‌നു നസ്വർ അൽ മറൂസി പറഞ്ഞു: മുഅമ്മിൽ ഒരു ഹദീസ് ഒറ്റക്ക് നിവേദനം ചെയ്താൽ അത് സ്വീകരിക്കുന്നതിന് മുമ്പ് പരിശോധനക്ക് വിധേയമാക്കണം. കാരണം അയാൾ നിവേദനത്തിൽ ഒട്ടനവധി പിഴവുകൾ വരുത്തുന്ന, മനപാഠ ശേഷി തീർത്തും മോശപ്പെട്ട വ്യക്തിയാണ്. (തഹ്ദീബ്: 10/381)

നിവേദക പരമ്പര: നാല്:

رواها ابن جرير في تفسيره (22/13) ، قال : حدثنا بشر ، قال : حدثنا يزيد ، قال : حدثنا سعيد عن قتادة

ഇബ്‌നു ജരീർ തന്റെ തഫ്സീറിൽ (22/13) പറഞ്ഞു: നമ്മോട് ബിശ്ർ പറഞ്ഞു- നമ്മോട് യസീദ് പറഞ്ഞു – നമ്മോട് സഈദ് പറഞ്ഞു – കത്താദയിൽ നിന്ന്:….

കത്താദ പ്രവാചകാനുചരനല്ല എന്ന് മാത്രമല്ല സംഭവത്തിന് സാക്ഷിയായ ആരിൽ നിന്നുമല്ല ഉദ്ധരിക്കുന്നത്. അതുകൊണ്ട് തന്നെ പരമ്പര മുറിഞ്ഞതാണ്. മാത്രമല്ല കത്താദ ആരിൽ നിന്നാണോ ഒരു സംഭവം ഉദ്ധരിക്കുന്നത് അവരിൽ നിന്ന് നേരിട്ട് കേട്ടു എന്ന് സൂചിപ്പിക്കുന്ന പ്രയോഗം നിവേദനത്തിൽ ഉപയോഗിച്ചില്ലെങ്കിൽ കത്താദയുടെ നിവേദനം സ്വീകരിക്കപ്പെടില്ല എന്ന് പണ്ഡിതന്മാർ വ്യക്തമാക്കിയിട്ടുണ്ട്. (ജാമിഉ തഹ്സ്വീൽ ഫീ അഹ്കാമിൽ മറാസീൽ: 101)

കുർതുബി തന്റെ തഫ്സീറിൽ (14/190) മുകാതിൽ എന്ന നിവേദകനിൽ നിന്ന് ഉദ്ധരിക്കുന്നതാണ് മറ്റൊരു നിവേദനം. മുകാതിലിന്റെ മുഴുവൻ നാമം മുകാതിലിബ്‌നു സുലൈമാൻ എന്നാണ്. ഇയാൾ നുണയനും വ്യാജ ഹദീസ് നിർമാതാവുമാണെന്ന് ഹദീസ് പണ്ഡിതർ വ്യക്തമാക്കിയിട്ടുണ്ട്. (തഹ്ദീബ്: 10/279-285) മറ്റു ചില പണ്ഡിതർ തങ്ങളുടെ സീറകളിൽ സമാനമായ നിവേദനങ്ങൾ സൂചിപ്പിക്കുന്നുണ്ടെങ്കിലും അവക്കൊന്നും തന്നെ നിവേദക പരമ്പര പോലും നൽകാത്തതിനാൽ അവ അസ്വീകാര്യമാണ്.

2. പ്രവാചകൻ (സ) സൈദിനെ കൊണ്ട് ഭാര്യയെ നിർബന്ധപൂർവ്വം വിവാഹമോചനം ചെയ്യിപ്പിച്ചു എന്ന് സൂചിപ്പിക്കുന്ന ചരിത്ര നിവേദനങ്ങളെല്ലാം വ്യാജങ്ങളും തനി കളവുകളുമാണെന്നും മുസ്‌ലിം പണ്ഡിതന്മാർ തന്നെ – ഇസ്‌ലാമിക വിമർശകർ ജന്മമെടുക്കുന്നതിന് നൂറ്റാണ്ടുകൾക്ക് മുമ്പ് – വ്യക്തമാക്കിയിട്ടുണ്ട്.

ഇബ്‌നുൽ അറബി പറഞ്ഞു: ഈ നിവേദനങ്ങളുടെയെല്ലാം പരമ്പര അടിസ്ഥാനരഹിതങ്ങളാണ്. (അഹ്കാമുൽ കുർആൻ: 3/1543)

ഇമാം കുർതുബി പറഞ്ഞു: …. ഇത്തരം നിവേദനങ്ങളുടെ ഉൽഭവം പ്രവാചകന്റെ പാപസുരക്ഷിതത്വത്തെ സംബന്ധിച്ച് യാതൊരു അറിവുമില്ലാത്ത വിഡ്ഢികളിൽ നിന്നും പ്രവാചകന്റെ പാവനത്വം കുറച്ചു കാണുന്നവരിൽ നിന്നും മാത്രമാണ്. (അൽ ജാമിഉ ലി അഹ്കാമിൽ കുർആൻ: 14/191)

ഇമാം ഇബ്‌നുകസീർ, ഇബ്‌നു ഹജർ, ശൈഖ് ശംക്വീത്വി തുടങ്ങി ഒട്ടനവധി ഹദീസ് – ചരിത്ര പണ്ഡിതർ വേറെയും ഇത്തരം നിവേദനങ്ങളെല്ലാം അടിസ്ഥാനരഹിതമാണെന്ന അഭിപ്രായക്കാരാണ്. (തഫ്സീറുൽ കുർആനിൽ അളീം: 3/491, ഫത്ഹുൽ ബാരി: 8/524, അദ്‌വാഉൽ ബയാൻ: 6/639)

മഹാ ഭൂരിപക്ഷം വരുന്ന ഹദീസ് – ചരിത്ര പണ്ഡിതന്മാർ പ്രമാണബദ്ധമായി വ്യക്തമാക്കിയതിന് വിപരീതമായി, സൈദ് – സൈനബ് ദാമ്പത്യ കഥയുമായി ബന്ധപ്പെട്ട വ്യാജ നിവേദനങ്ങളുടെ അടിസ്ഥാനത്തിൽ പ്രവാചകന്റെ(സ) കണ്ണ് പതിഞ്ഞ സ്ത്രീകളെ ഭർത്താക്കന്മാർ വിവാഹം മോചനം ചെയ്യണമെന്നും അത്തരം സ്ത്രീകളെ വിവാഹം ചെയ്യൽ പ്രവാചകന് (സ) അനുവദനീയമാണ് എന്നുമൊക്കെ ചില മുസ്‌ലിം പണ്ഡിതന്മാരുടെ ഒറ്റപ്പെട്ട അഭിപ്രായങ്ങൾ ഗ്രന്ഥങ്ങളിൽ (തഫ്സീറുൽ കുർതുബി) എഴുതപ്പെട്ടിരിക്കുന്നു എന്നത് ഇസ്‌ലാം മതത്തിനോ, മുസ്‌ലിംകൾക്കോ ബാധകമല്ല. അവയൊന്നും ഇസ്‌ലാം മതത്തിൽ പ്രമാണമല്ല. ഇസ്‌ലാമിലെ പ്രമാണം കുർആനും സ്വഹീഹായ ഹദീസുകളുമാണ്. ഈ തെളിവുകളുടെ പിൻബലമില്ലാത്ത അത്തരം കേവലാഭിപ്രായങ്ങൾ, പ്രമാണ വിരുദ്ധമായ നിലപാടുകൾ യാതൊരു പരിഗണനയും കൂടാതെ തള്ളിക്കളയണമെന്ന് മുസ്‌ലിം പണ്ഡിതന്മാർ തന്നെ വ്യക്തമാക്കിയിട്ടുണ്ട്.

3. ഈ വ്യാജ നിവേദനങ്ങളുടെ നിവേദകർ ദുർബലരും അവിശ്വസ്ഥരുമാണ് എന്നതിന് പുറമെ ഇവയുടെ ഉള്ളടക്കവും (മത്ന് المتن) ധാരാളം വൈരുധ്യങ്ങൾ നിറഞ്ഞവയാണ്:

ചില നിവേദനങ്ങളിൽ സൈദ് നാട്ടിലില്ലാത്തപ്പോൾ പ്രവാചകൻ (സ) അദ്ദേഹത്തിന്റെ വീട്ടിൽ ചെന്നുവെന്നും അപ്പോൾ സൈനബിനെ കണ്ടുമുട്ടി എന്നും ചില നിവേദനങ്ങളിൽ സൈദ് രോഗിയായിരിക്കെ വീട്ടിൽ സന്ദർശിച്ചുവെന്നും അപ്പോഴാണ് സൈനബിനെ കണ്ടുമുട്ടിയത് എന്നും കാണാം! ഇത് രണ്ടും എങ്ങനെ യോജിക്കും?! സൈനബിനെ പ്രവാചകൻ (സ) കണ്ടത് എങ്ങനെയാണെന്നതിലും ഈ വ്യാജ നിവേദനങ്ങൾ പരസ്പരം വൈരുധ്യത്തിലാണ്. ജനലിലൂടെ സൈനബിനെ കണ്ടു എന്ന് ചില നിവേദനങ്ങൾ… മുടി കെട്ട് കൊണ്ടുള്ള മറ കാറ്റിൽ പാറിയപ്പോൾ കണ്ടു എന്ന് മറ്റു ചിലതിലും പറയുന്നു.

പ്രവാചകന് (സ) പന്ത്രണ്ട് വയസ്സുള്ളപ്പോൾ സൈനബ് (റ) ജനിച്ചിട്ടുണ്ട്. പ്രവാചകന്റെ (സ) പിതൃവ്യപുത്രിയാണ് സൈനബ് (റ). സൈനബ് (റ) വളർന്നതും യുവതിയായതും പ്രവാചകന്റെ (സ) കൺമുമ്പിൽ തന്നെയാണ്. അവരെ സൈദിന് (റ) വിവാഹം കഴിപ്പിച്ചു നൽകിയതും പ്രവാചകൻ (സ) തന്നെ. പലവുരു പ്രവാചകൻ (സ) സൈനബിനെ (റ) കണ്ടിട്ടുണ്ട് എന്ന വസ്തുത ഈ വ്യാജ നിവേദനങ്ങളെയെല്ലാം ഖണ്ഡിക്കുന്നു. സൈനബ് (റ) മക്കയിൽ വെച്ച് ഇസ്‌ലാം സ്വീകരിച്ചു. പ്രവാചകനോടൊപ്പം മദീനയിലേക്ക് പലായനം ചെയ്തു. പ്രവാചകന് (സ) സൈനബിനെ (റ) വിവാഹം ചെയ്യാൻ ആഗ്രഹമുണ്ടായിരുന്നെങ്കിൽ സൈദിന് വിവാഹം ചെയ്യിപ്പിക്കുന്നതിന് മുമ്പ് ഒരു സൂചനയെങ്കിലും നൽകിയിരുന്നെങ്കിൽ സൈനബ് (റ) അതിന് തയ്യാറാകുമായിരുന്നു. എന്ന് മാത്രമല്ല സൈദുമായി വിവാഹം നടക്കുന്നതിന് മുമ്പ് വിവാഹമൂല്യമൊന്നുമില്ലാതെ, തന്നെ വിവാഹം ചെയ്യാൻ തയ്യാറാണോ എന്ന് പ്രവാചകനോട് (സ) അങ്ങോട്ട്, സൈനബ് (റ) ചോദിച്ചതായും ചില ചരിത്ര നിവേദനങ്ങളിൽ കാണാം. (അഹ്കാമുൽ കുർആൻ: 3/1543) എന്നിരിക്കെ സൈദുമായുള്ള വിവാഹ ശേഷം സൈനബിനെ (റ) വിവാഹം കഴിക്കാൻ പ്രവാചകൻ (സ) നടത്തിയ ‘പരാക്രമ കഥകൾ’ ചിരിച്ചു തള്ളേണ്ട ഫലിതങ്ങൾ മാത്രമാണ്.

4. സ്വീകാര്യയോഗ്യമായ ഹദീസുകളും (الحديث الصحيح) സ്ഥിരപ്പെട്ട ചരിത്ര നിവേദനങ്ങളും തെളിയിക്കുന്നത് ദുർബലമായ ഈ കഥക്ക് നേർവിപരീതമാണ്:

പ്രവാചകൻ (സ) സൈദിന് (റ) സൈനബിനെ (റ) വിവാഹം കഴിപ്പിച്ചു കൊടുത്തു. എന്നാൽ ഇരുവർക്കും പരസ്പരം സ്നേഹ സമ്പന്നമായ ഒരു ദാമ്പത്യ ജീവിതത്തിൽ നിരാശ സംജാതമായി. അവർ പരസ്പരം അകന്നു. സൈദിന് (റ) ഭാര്യയുമായി യോജിച്ചു പോകാനാകില്ലെന്നു കണ്ടപ്പോള്‍ അവരെ വിവാഹമോചനം നടത്തുന്നതിനെപ്പറ്റി പ്രവാചകനോട് (സ) പരാതിപ്പെട്ടപ്പോഴും ആ ബന്ധം തുടരാനാണ് പ്രവാചകൻ (സ) അദ്ദേഹത്തോട് കൽപ്പിച്ചത് എന്നത് സ്വീകാര്യയോഗ്യമായ ഹദീസിലൂടെ സ്ഥിരപ്പെട്ട ഒരു വസ്തുതയാണ്.

جَاءَ زَيْدُ بْنُ حَارِثَةَ يَشْكُو ، فَجَعَلَ النَّبِيُّ صَلَّى اللَّهُ عَلَيْهِ وَسَلَّمَ يَقُولُ : ( اتَّقِ اللَّهَ وَأَمْسِكْ عَلَيْكَ زَوْجَكَ )

സൈദുബ്‌നു ഹാരിസ (റ) സങ്കടപ്പെട്ടുകൊണ്ടു പ്രവാചകന്റെ (സ) അടുക്കല്‍ വന്നു. അപ്പോള്‍ പ്രവാചകൻ (സ) ഇപ്രകാരം അദ്ദേഹത്തോട് ഉപദേശിച്ചു കൊണ്ടിരുന്നു: “നീ അല്ലാഹുവിനെ സൂക്ഷിക്കുക: നിന്‍റെ ഭാര്യയെ കൂടെ താമസിപ്പിക്കുകയും ചെയ്യുക (വിവാഹ മോചനം ചെയ്യരുത്)”. (സ്വഹീഹുൽ ബുഖാരി: 6984, ജാമിഉൽ ബയാൻ: ത്വബ്‌രി: 22/11, തഫ്‌സീറുൽ കുർആനിൽ അളീം: ഇബ്‌നുകസീർ: 3/489)

പ്രവാചകൻ (സ) സൈദിനെ(റ) കൊണ്ട് ഭാര്യയെ വിവാഹമോചനം ചെയ്യിപ്പിച്ചു എന്ന കളള കഥക്ക് നേർ വിപരീതമായി സൈനബുമായി ദാമ്പത്യ ജീവിതം തുടരാൻ പരമാവധി പരിശ്രമിക്കാനാണ് പ്രവാചകൻ (സ) ഉപദേശിച്ചത് എന്ന് വ്യക്തമായും ഹദീസ് തെളിയിക്കുന്നു. അങ്ങനെ ദാമ്പത്യം തീരെ തുടർന്നു പോകാൻ സാധിക്കാത്തതിനാൽ ദമ്പതികൾ പരസ്പരം വേർപിരിഞ്ഞുവെന്നും ശേഷം സൈനബ് (റ) തന്റെ ഇദ്ദാ കാലഘട്ടം കഴിഞ്ഞപ്പോൾ പ്രവാചകനെ വിവാഹം ചെയ്യുകയും അതിൽ അതിയായി സന്തോഷിക്കുകയും ചെയ്തുവെന്ന് സ്വീകാര്യയോഗ്യമായ ഹദീസുകളിലും സ്ഥിരപ്പെട്ട ചരിത്ര നിവേദനങ്ങളിലും നാം വായിക്കുന്നു. സൈനബ് (റ), പ്രവാചകനുമായുള്ള വിവാഹത്തോടുകൂടി സത്യവിശ്വാസികളുടെ മാതാക്കളില്‍ ഒരാളായിത്തീരുകയും അഭിമാനപൂര്‍വ്വം പ്രവാചകന്റെ(സ) ഭാര്യമാരോടു ഇങ്ങിനെ പറയുകയും ചെയ്യുമായിരുന്നുവെന്ന് ഹദീസില്‍ വന്നിരിക്കുന്നു: “നിങ്ങളെ നിങ്ങളുടെ വീട്ടുകാര്‍ വിവാഹം ചെയ്യിച്ചു കൊടുത്തതാണ്; എന്നെ ഏഴു ആകാശങ്ങള്‍ക്കു മീതെ നിന്നു അല്ലാഹു വിവാഹം ചെയ്യിച്ചു കൊടുത്തതാണ്.” (സ്വഹീഹുൽ ബുഖാരി: 6984).

ഇതെല്ലാം താഴെ പറയുന്ന കാര്യങ്ങൾ തെളിയിക്കുന്നു:

* പ്രവാചകൻ (സ) സൈദിനോട് (റ) തന്റെ ഭാര്യയെ വിവാഹ മോചനം ചെയ്യാൻ കൽപ്പിച്ചിട്ടില്ല. നേർ വിപരീതമാണ് പ്രവാചകൻ (സ) അദ്ദേഹത്തോട് ഉപദേശിച്ചത്.

* സൈദിന്(റ) സൈനബിനോടൊപ്പവും(റ) സൈനബിന്(റ) സൈദിനോടൊപ്പവുമുള്ള(റ) ദാമ്പത്യ ജീവിതം തൃപ്തികരമായിരുന്നില്ല. അവർ തമ്മിൽ നടന്ന വിവാഹ മോചനം അവർ സ്വയം എടുത്ത തീരുമാനത്തിന്റെ അടിസ്ഥാനത്തിലായിരുന്നു. പ്രവാചകന്(സ) അതിൽ യാതൊരു പങ്കുമില്ല.

* മാസങ്ങൾക്ക് ശേഷം പ്രവാചകനുമായി(സ) നടന്ന വിവാഹത്തിൽ സൈനബ്(റ) അങ്ങേയറ്റം സന്തുഷ്ടയും അഭിമാനിയുമായിരുന്നു എന്നത് സൈദ് – സൈനബ് (റ) ദമ്പതികളുടെ വിവാഹ മോചനം അവർ സ്വയം എടുത്ത തീരുമാനത്തിന്റെ അടിസ്ഥാനത്തിലായിരുന്നു എന്ന് തെളിയിക്കുന്നു.

5. “അല്ലാഹു വെളിപ്പെടുത്താന്‍ പോകുന്ന ഒരു കാര്യം നിന്‍റെ മനസ്സില്‍ നീ മറച്ചു വെക്കുകയും ജനങ്ങളെ നീ പേടിക്കുകയും ചെയ്തിരുന്നു. എന്നാല്‍ നീ പേടിക്കുവാന്‍ ഏറ്റവും അര്‍ഹതയുള്ളവന്‍ അല്ലാഹുവാകുന്നു.” (കുർആൻ: 33:37) എന്ന് കുർആൻ സൂചിപ്പിച്ചത് പ്രവാചകൻ (സ) സൈദിന്റെ(റ) ഭാര്യയെ മോഹിക്കുകയും അദ്ദേഹത്തെ കൊണ്ട് ഭാര്യയെ വിവാഹമോചനം ചെയ്യിപ്പിക്കുകയും ചെയ്തതിനെ സംബന്ധിച്ചാണ് എന്നതും ശുദ്ധ അസംബന്ധമാണ്. കാരണങ്ങൾ:

* അതായിരുന്നു മുഹമ്മദ് നബി (സ) തന്റെ മനസ്സിൽ മറച്ചുവെച്ച രഹസ്യമെങ്കിൽ അദ്ദേഹം തന്നെ രചിച്ചുവെന്ന് വിമർശകർ വാദിക്കുന്ന ‘ഖുറാനിൽ’ ആ കള്ളത്തരം സൂചിപ്പിക്കുന്ന ഒരു വചനം അദ്ദേഹം തന്നെ മാലോകർക്ക് ഓതി കൊടുക്കുമായിരുന്നോ ?

* സ്വീകാര്യയോഗ്യമായ ഒരു ഹദീസും ഇത്തരമൊരു വാദത്തിന് പിൻബലമേകുന്നില്ല.

* സ്വീകാര്യയോഗ്യമായ നിവേദനങ്ങൾ, എന്താണ് മുഹമ്മദ് നബി (സ) തന്റെ ‘മനസ്സിൽ മറച്ചുവെച്ച ഒരു കാര്യം’ എന്ന് വ്യക്തമാക്കിയിട്ടുണ്ട്. ആ രഹസ്യമെന്താണ് എന്ന് കാണുക: “സൈദ്‌ (റ) തന്‍റെ ഭാര്യയുമായി യോജിച്ചു പോകുകയില്ലെന്നു കണ്ടപ്പോള്‍ അവരെ വിവാഹമോചനം (طلاق) നടത്തുന്നതിനെപ്പറ്റി തിരുമേനി(സ)യോട് ആലോചിച്ചു. ഇരുഭാഗത്തെയും ഗുണകാംക്ഷിയാണല്ലോ നബി(സ). അപ്പോഴായിരുന്നു അദ്ദേഹത്തോടു നബി(സ) ഭാര്യയെ വെച്ചുകൊള്ളുവാനും, അല്ലാഹുവിനെ സൂക്ഷിക്കുവാനും പറഞ്ഞത്. ഈ വിവാഹം അധികം നീണ്ടുപോകുകയില്ലെന്നും, സൈദ്‌ (റ) ഭാര്യയെ വേര്‍പ്പെടുത്തുക തന്നെ ചെയ്യുമെന്നും, അനന്തരം സൈനബ(റ)യെ തിരുമേനി(സ) വിവാഹം ചെയ്‌വാനിരിക്കുന്നുവെന്നും തിരുമേനി(സ)ക്കു (ദിവ്യ ബോധനത്തിലൂടെ) അറിവു ലഭിച്ചിട്ടുണ്ടായിരുന്നു. പക്ഷേ, നടക്കുവാനിരിക്കുന്ന ആ വിഷയത്തെക്കുറിച്ച് സൈദി(റ)നോടു തിരുമേനി(സ) ഒന്നും പ്രസ്താവിച്ചില്ല. ഭാര്യയെ വിവാഹമോചനം നടത്താതിരിക്കുവാന്‍ സാധാരണമട്ടില്‍ ഉപദേശിക്കുകയാണ് ചെയ്തത്. ജനസംസാരത്തിനു ഇടയാകരുതെന്നും, അല്ലാഹു ഉദ്ദേശിച്ച കാര്യം അവന്‍ ഉദ്ദേശിക്കുമ്പോള്‍ അവന്‍ നടപ്പിലാക്കിക്കൊള്ളുമെന്നും തിരുമേനി(സ) കരുതി. ഇതിനെ സൂചിപ്പിച്ചുകൊണ്ടാണ് ‘അല്ലാഹു വെളിവാക്കുവാന്‍ പോകുന്ന കാര്യം നീ നിന്‍റെ മനസ്സില്‍ മറച്ചുവെക്കുകയും ചെയ്തിരിക്കുന്നു’, (وَتُخْفِي فِي نَفْسِكَ مَا اللَّـهُ مُبْدِيهِ) എന്നു പറഞ്ഞത്. (മറിച്ചു ചിലര്‍ പ്രസ്താവിക്കുന്നതുപോലെ, അവരെ വിവാഹംകഴിച്ചാല്‍ കൊള്ളാമെന്ന ആഗ്രഹമല്ല എന്നു താല്‍പര്യം.) യഥാര്‍ത്ഥവും സത്യവും തുറന്നു പറയുന്നതില്‍ ആരെയും ശങ്കിക്കേണ്ടതില്ല, അതില്‍ ജനസംസാരം ഭയപ്പെടേണ്ടതുമില്ല, അതു തുറന്നു പറയാതിരിക്കുന്നതില്‍ അല്ലാഹുവിനെ ഭയപ്പെടുകയാണ് വേണ്ടത് എന്ന് (അല്ലാഹു) നബി(സ)യെ ഓര്‍മ്മിപ്പിക്കുകയും ചെയ്യുന്നു.” (തഫ്സീറുൽ അമാനി: 33:37 ന്റെ വ്യാഖ്യാനം)

അഥവാ പ്രവാചകനായത് കൊണ്ട് തന്നെ – സൈദ് സൈനബിനെ വിവാഹമോചനം ചെയ്യുമെന്നും കാലങ്ങൾക്ക് ശേഷം താൻ സൈനബിനെ വിവാഹം ചെയ്യുമെന്നുമുൾപ്പെടെ – ഭാവിയിൽ നടക്കാനിരിക്കുന്ന കാര്യങ്ങൾ അദ്ദേഹത്തിന് അല്ലാഹു നൽകിയ ദിവ്യബോധനത്തിലൂടെ അറിയാമായിരുന്നു. എന്നിട്ടും അവ മനസ്സിൽ രഹസ്യമായി മറച്ചുവെച്ച് സൈനബുമായി ദാമ്പത്യ ജീവിതം തുടരാൻ പരമാവധി പരിശ്രമിക്കാനാണ് പ്രവാചകൻ (സ) സൈദിനെ ഉപദേശിച്ചത്. ഇതാകട്ടെ പ്രവാചകന്റെ(സ) മാന്യതയേയും ലൈംഗിക ശുദ്ധിയേയുമാണ് തെളിയിക്കുന്നത്. അദ്ദേഹം സ്ത്രീലമ്പടനായിരുന്നു എന്ന വിമർശകരുടെ വാദത്തെ ഈ സംഭവം തച്ചുടക്കുന്നു.

രാത്രി, പാത്രത്തിൽ ഒഴിച്ചിരുന്ന പ്രവാചകന്റെ(സ) മൂത്രം ഉമ്മു അയ്‌മൻ (റ) കുടിച്ചു എന്ന കഥ. (മുസ്തദ്റക് ഹാകിം: 70/ 4)

ഹദീസ് വളരെ ദുർബലമാണ്.

കാരണങ്ങൾ:

1, നിവേദകനായ നുബൈഹ് അൽ അനസിയുടേയും ഉമ്മു അയ്‌മന്റേയും ഇടയിൽ നിവേദക പരമ്പര മുറിഞ്ഞതാണെന്ന് ഇമാം ഇബ്‌നു ഹജർ വ്യക്തമാക്കുന്നു. (അത്തൽഖീസുൽ ഹബീർ ഫീ തഖ്‌രീജി അഹാദീസിർ റാഫിഈ: 1/171)

2. നിവേദക പരമ്പരയിലെ (സനദ്) ‘അബൂമാലിക് അന്നഖഇ’ യുടെ മുഴുവൻ നാമം അബ്ദുൽ മലിക് ഇബ്‌നു ഹുസൈൻ എന്നാണ്. ഇയാൾ ദുർബലനാണെന്ന കാര്യത്തിൽ ഹദീസ് പണ്ഡിതന്മാരെല്ലാം ഏകാഭിപ്രായക്കാരാണ്.

ഇമാം നസാഇ പറഞ്ഞു: അയാൾ കളവ് പറയുന്നവനായി ആരോപിതനാണ്. ഇമാം അബൂ ഹാതിം പറഞ്ഞു: അയാളുടെ ഹദീസുകൾ ദുർബലമാണ്. ഇമാം അംറിബ്‌നു അലി പറഞ്ഞു: ദുർബലൻ, വിശ്വസ്ഥരായ നിവേദകർക്കെതിരെ ദുർബലമായ ഹദീസ് ഉദ്ധരിക്കുന്ന വ്യക്തി. ഇമാം നസാഇയുടെ ‘ദുഅഫാഉ വൽ മത്റൂകീൻ’, ഇബ്‌നു അബീഹാതിമിന്റെ ‘ജർഹുവ തഅ്ദീൽ’ എന്നീ ഗ്രന്ഥങ്ങളിൽ ഇത് വ്യക്തമാക്കപ്പെട്ടിട്ടുണ്ട്.

കൂടാതെ ഇമാം ദാറകുത്നിയും (അൽ ഇലല്: 15 / 415) ഇമാം ഇബ്‌നു ഹജറുൽ അസ്‌കലാനിയും (അത്തൽഖീസുൽ ഹബീർ ഫീ തഖ്‌രീജി അഹാദീസിർ റാഫിഈ: 1/171) ഈ ഹദീസ് മുകളിൽ സൂചിപ്പിച്ച കാരണങ്ങളാൾ ദുർബലമാണെന്ന് വ്യക്തമാക്കിയിട്ടുണ്ട്.

3. പ്രവാചകൻ (സ) മലമൂത്ര വിസർജനത്തിന് ശേഷം കണിശതയോടെ അവയവങ്ങൾ വൃത്തിയാക്കിയിരുന്നതായി സ്വഹീഹായ ഒട്ടനവധി ഹദീസുകളിൽ സ്ഥിരപ്പെട്ട കാര്യമാണ്. പ്രവാചകന്റെ മലമൂത്ര വിസർജ്യങ്ങൾ ശുദ്ധമായിരുന്നെങ്കിൽ അദ്ദേഹം അപ്രകാരം ചെയ്യുമായിരുന്നില്ല. ഇത്തരം കാര്യങ്ങളിൽ പ്രവാചകൻ (സ) മറ്റുള്ള മനുഷ്യന്മാരെ പോലെ തന്നെയാണ്. “(നബിയേ,) പറയുക: ഞാന്‍ നിങ്ങളെപ്പോലെയുള്ള ഒരു മനുഷ്യന്‍ മാത്രമാകുന്നു.” (ഖുർആൻ: 18: 110)

വിമർശനം:

മുഹമ്മദ് ശവക്കുഴിയിൽ വെച്ച് ശവരതിക്ക് അഹ്വാനം നൽകുന്നു. Al bukhari 23-426,374.

മറുപടി:

കുറേ നുണകൾ പറയുക, എന്നിട്ട് കുറച്ച് ‘നമ്പറും’ എഴുതുക എന്ന വെറുപ്പിന്റെ അപ്പോസ്‌തലന്മാരുടെ സ്ഥിരം ‘നമ്പർ’ തന്നെയാണ് ഇവിടെയും പയറ്റിയിരിക്കുന്നത്. ആദ്യം ഹദീസ് നേരിട്ട് വായിക്കാം:

حَدَّثَنَا عَبْدُ اللَّهِ بْنُ مُحَمَّدٍ، حَدَّثَنَا أَبُو عَامِرٍ، حَدَّثَنَا فُلَيْحُ بْنُ سُلَيْمَانَ، عَنْ هِلاَلِ بْنِ عَلِيٍّ، عَنْ أَنَسِ بْنِ مَالِكٍ ـ رضى الله عنه ـ قَالَ شَهِدْنَا بِنْتًا لِرَسُولِ اللَّهِ صلى الله عليه وسلم قَالَ وَرَسُولُ اللَّهِ صلى الله عليه وسلم جَالِسٌ عَلَى الْقَبْرِ ـ قَالَ فَرَأَيْتُ عَيْنَيْهِ تَدْمَعَانِ قَالَ ـ فَقَالَ ‏”‏ هَلْ مِنْكُمْ رَجُلٌ لَمْ يُقَارِفِ اللَّيْلَةَ ‏”‏‏.‏ فَقَالَ أَبُو طَلْحَةَ أَنَا‏.‏ قَالَ ‏”‏ فَانْزِلْ ‏”‏‏.‏ قَالَ فَنَزَلَ فِي قَبْرِهَا‏.‏

അനസ് (റ) നിവേദനം: പ്രവാചകന്റെ(സ) ഒരു പുത്രിയുടെ മരണാനന്തര ചടങ്ങിന് ഞങ്ങൾ സാക്ഷികളായി. പ്രവാചകൻ (സ) ‘ഖബ്റി’ന്നരികിൽ (ശവക്കുഴി) ഇരിക്കുകയാണ്. അനസ് (റ) പറയുന്നു: അദ്ദേഹത്തിന്റെ ഇരുകണ്ണുകളിൽ നിന്നും കണ്ണുനീരൊഴുകുന്നത് ഞാൻ കണ്ടു. പ്രവാചകൻ (സ) ചോദിച്ചു: നിങ്ങളുടെ കൂട്ടത്തിൽ ഇന്നലെ രാത്രി സ്വഭാര്യയുമായി സംസർഗത്തിൽ ഏർപ്പെടാത്തവർ അല്ലെങ്കിൽ വഴക്കിൽ ഏർപ്പെടാത്തവർ ആരെങ്കിലുമുണ്ടോ? അബൂത്വൽഹ (റ) പറഞ്ഞു: ഞാൻ ഉണ്ട്. പ്രവാചകൻ (സ) പറഞ്ഞു: എങ്കിൽ താങ്കൾ ഖബറിൽ ഇറങ്ങുക. അങ്ങനെ അദ്ദേഹം അവരുടെ ഖബറിൽ ഇറങ്ങി. (ബുഖാരി. 2. 23. 374)

ഇതിൽ ശവരതി ഒളിഞ്ഞിരിക്കുന്നത് എവിടെയാണെന്ന് ഹദീസ് വായിച്ചിട്ട് മനസ്സിലാവാത്ത നിഷ്കളങ്കരുണ്ടെങ്കിൽ അവർക്കായി വിമർശകരുടെ ദുർവ്യാഖ്യാന കസർത്ത് വ്യക്തമാക്കാം. “നിങ്ങളുടെ കൂട്ടത്തിൽ ഇന്നലെ രാത്രി സ്വഭാര്യയുമായി സംസർഗത്തിലേർപ്പെടാത്തവർ ആരെങ്കിലുമുണ്ടോ?” എന്ന് പ്രവാചകൻ (സ) ചോദിച്ചതിന്റെ അർത്ഥം “ഭാര്യയുമായി സംസർഗത്തിലേർപ്പെടാത്തവർ ഖബറിലിറങ്ങി എന്റെ മകളുമായി ശവരതിയിൽ ഏർപ്പെട്ടു കൊള്ളു” എന്ന് സൂചിപ്പിക്കുകയാണ്. ഇതാണ് പുതു തലമുറയിലെ ഇസ്‌ലാം വിമർശകരായ ഞരമ്പ് രോഗികൾ ഹദീസിന് നൽകുന്ന വ്യാഖ്യാനം !!! മകളുടെ വിയോഗത്തിൽ അടക്കാൻ കഴിയാത്ത വ്യഥയാൽ കരഞ്ഞ് കണ്ണുനീരു വാർക്കുന്ന വൃദ്ധനായ ഒരു ഉപ്പ മകളുമായി ശവരതിയിൽ ഏർപ്പെടാൻ കൽപ്പിച്ചുവെന്ന് പറയാൻ മാത്രം കരിങ്കല്ലുകളാണല്ലൊ ഇക്കൂട്ടരുടെ മനസ്സുകളുടെ സ്ഥാനത്ത്…!!!

“പിന്നീട് അതിന് ശേഷവും നിങ്ങളുടെ മനസ്സുകള്‍ കടുത്തുപോയി. അവ പാറപോലെയോ അതിനെക്കാള്‍ കടുത്തതോ ആയി ഭവിച്ചു. പാറകളില്‍ ചിലതില്‍ നിന്ന് നദികള്‍ പൊട്ടി ഒഴുകാറുണ്ട്‌. ചിലത് പിളര്‍ന്ന് വെള്ളം പുറത്ത് വരുന്നു. ചിലത് ദൈവഭയത്താല്‍ താഴോട്ട് ഉരുണ്ടു വീഴുകയും ചെയ്യുന്നു. നിങ്ങള്‍ പ്രവര്‍ത്തിക്കുന്ന യാതൊന്നിനെപറ്റിയും അല്ലാഹു ഒട്ടും അശ്രദ്ധനല്ല.” (കുർആൻ: 2:74)

ഇത്തരം രോഗാതുര മനസ്സുള്ളവരാണല്ലൊ നമുക്ക് ചുറ്റും മാന്യന്മാരെ പോലെ ജീവിക്കുന്നത് എന്നോർക്കുമ്പോൾ ദുഖവും അതിലേറെ ഭയവുമാണ് അനുഭവപ്പെടുന്നത്. നമ്മുടെ ഉമ്മ പെങ്ങമ്മാർ ശ്മശാനത്തിലും സുരക്ഷിതരാണോ എന്ന് ചിന്തിച്ച് പോവുന്നു. അല്ലാഹുവിൽ ശരണം.

Vol. 2, Book 23, Hadith 426 എന്ന് നമ്പറിട്ട് കൊടുത്ത ഹദീസിൽ തന്നെ അബൂത്വൽഹ (റ) ഖബറിൽ ഇറങ്ങിയിട്ട് എന്താണ് ചെയ്തത് എന്നും വ്യക്തമായി പറയുന്നുണ്ട് എങ്കിലും അത് ഈ പോൺ ആർട്ടിസ്റ്റുകൾ മറച്ചുവെക്കുകയാണ് പതിവ്.

فَنَزَلَ فِي قَبْرِهَا فَقَبَرَهَا‏ അങ്ങനെ അബൂത്വൽഹ (റ) ഖബറിൽ ഇറങ്ങുകയും അവരെ മറമാടുകയും ചെയ്തു. (സ്വഹീഹുൽ ബുഖാരി: 1342, അഹ്‌മദ്: 12275)

“നിങ്ങളുടെ കൂട്ടത്തിൽ ഇന്നലെ രാത്രി സ്വഭാര്യയുമായി സംസർഗത്തിലേർപ്പെടാത്തവർ ആരെങ്കിലുമുണ്ടോ?” അല്ലെങ്കിൽ “ഇന്നലെ രാത്രി സ്വഭാര്യയുമായി സംസർഗത്തിലേർപ്പെടാത്തവർ മാത്രം ഖബറിൽ ഇറങ്ങുക” എന്ന് പ്രവാചകൻ (സ) പറയാൻ കാരണം ഇസ്‌ലാമിക നിയമ പ്രകാരം തലേന്ന് ഭാര്യമാരുമായി സംസർഗത്തിലേർപ്പെട്ടവരൊ, മനസ്സ് ഏകാഗ്രമല്ലാത്തവരൊ, കലുഷമായ മനസ്സുള്ളവരോ മൃതദേഹം മറമാടുന്ന കർമ്മം ചെയ്യാൻ പാടില്ല എന്നത് കൊണ്ടാണ്. (പ്രവാചകൻ നേരിട്ട് ഇറങ്ങാതിരുന്നത് പുത്രവിയോഗ ദുഖത്താലുമാണ്). ഏകാഗ്രമല്ലാത്ത മനസ്സുമായി ചെയ്യേണ്ട ഒന്നല്ല മൃതശരീര സംസ്ക്കരണം. മരണചിന്തയിലും ഭക്തി സാന്ദ്രതയിലും പ്രാർത്ഥനാ മനസ്സോടെയും നിർവഹിക്കേണ്ട കർമ്മമാണത്.

ശൈഖ് സ്വാലിഹ് അൽ മുനജ്ജിദ് പറയുന്നു: “പുലർച്ചയാണ് മൃതശരീരം മറമാടുന്നത്. അന്ന് രാത്രി ഭാര്യമാരുമായി സംസർഗത്തിൽ ഏർപ്പെട്ട ഒരു വ്യക്തിയെ സംബന്ധിച്ചിടത്തോളം അയാളുടെ മനസ്സിൽ ആ ചിന്തകളുടെ ശേഷിപ്പുകളുണ്ടാവും. എന്നാൽ മൃതദേഹം മറവു ചെയ്യുക എന്നത് ഭക്തിസാന്ദ്രവും ഭയവും മരണചിന്തയുമെല്ലാം പ്രബലമായി നിൽക്കേണ്ട സന്ദർഭമാണല്ലൊ. അപ്പോൾ തലേന്ന് രാത്രിയിലെ ഓർമകൾ ഉള്ള ഒരാൾ മൃതദേഹം മറവു ചെയ്യുമ്പോൾ ശ്രദ്ധിക്കേണ്ട ബാധ്യതകളിലും കർമ്മങ്ങളിലും ചിലത് മറന്നു പോകാൻ സാധ്യതയുണ്ട് എന്നതിനാലാണ് തലേന്ന് ഭാര്യമാരുമായി സംസർഗത്തിൽ ഏർപ്പെടാത്തവർ മൃതദേഹം മറമാടാൻ നിർദ്ദേശിച്ചത്.” (ഇസ്‌ലാം സുആൽ വൽജവാബ്: 246638 )

ഇബ്നുൽ ജൗസി പറഞ്ഞു: “ഒരാൾ തൊട്ടടുത്ത് ചെയ്ത ഒരു കാര്യം ഓർക്കാൻ സാധ്യത കൂടുതലാണ്. ഇതില്ലാതിരിക്കാനാണ് തൊട്ടു തലേന്ന് ഭാര്യമാരുമായി സംസർഗത്തിൽ ഏർപ്പെടാത്തവർ മൃതദേഹം മറമാടാൻ നിർദ്ദേശിച്ചത്.” (കശ്ഫുൽ മുശ്കിൽ: 3: 296)

قال الحافظ ابن حجر رحمه الله وَعَلَّلَ ذَلِكَ بَعْضهمْ بِأَنَّهُ حِينَئِذٍ يَأْمَن مِنْ أَنْ يُذَكِّرهُ الشَّيْطَان بِمَا كَانَ مِنْهُ تِلْكَ اللَّيْلَة

ഇബ്നു ഹജർ പറഞ്ഞു: “പിശാച് തലേ ദിവസത്തെ ഓർമകളിൽ വല്ലതും മനസ്സിൽ ഇട്ടു കൊടുക്കുന്നതിൽ നിന്ന് സുരക്ഷിതനായിരിക്കാനാണ് തലേന്ന് ഭാര്യമാരുമായി സംസർഗത്തിൽ ഏർപ്പെടാത്തവർ മൃതദേഹം മറമാടാൻ നിർദ്ദേശിച്ചത്.” (ഫത്ഹുൽ ബാരി: 3:159)

خَشِيَ صَلَّى اللَّهُ عَلَيْهِ وَسَلَّمَ إِنْ نَزَلَ أَنْ يَتَذَكَّرَ شَيْئًا، فَيَذْهَلَ مِنَ الْإِتْيَانِ بِكَمَالِ الْمَنْدُوبَاتِ الَّتِي تُفْعَلُ بِالْمَيِّتِ فِي الْقَبْرِ അലി അൽകാരി പറഞ്ഞു: “തലേന്ന് രാത്രിയിലെ ഓർമകൾ മയ്യത്ത് മറമാടുന്ന വേളയിൽ ഒരാൾക്കുണ്ടാകുന്നതിനെ പ്രവാചകൻ (സ) ഭയപ്പെട്ടു. അത്തരം ഓർമകൾ മനസ്സിൽ നില നിന്നാൽ മൃതദേഹം മറവു ചെയ്യുമ്പോൾ ശ്രദ്ധിക്കേണ്ട മര്യാദകളൊ ഐച്ഛിക പുണ്യകർമങ്ങളൊ മറന്നു പോകാൻ സാധ്യതയുണ്ടല്ലൊ.” (മിർകാത്തുൽ മഫാതീഹ്: 3:1227)

ആധുനിക വിമർശകരുടെ ശവരതി ആരോപണങ്ങളൊന്നും ദുർവ്യാഖ്യാന ഫാക്റ്ററികളുടെ മൂശയിലിട്ട് വാർത്തെടുക്കപ്പെടുന്നതിനും നൂറ്റാണ്ടുകൾക്ക് മുമ്പ് ജീവിച്ച് മരിച്ചു പോയ പൗരാണികരായ ഒട്ടനവധി ഹദീസ് പണ്ഡിതന്മാർ ഈ യാഥാർഥ്യം വ്യക്തമാക്കിയിട്ടുണ്ട്. (തുഹ്ഫത്തുൽ മുഹ്താജ്: 3: 170, നൈലുൽ ഔതാർ: 4: 106, മിർആത്തുൽ മഫാതീഹ്: 5: 451)

2. هَلْ فِيكُمْ مِنْ أَحَدٍ لَمْ يُقَارِفِ اللَّيْلَةَ എന്ന പ്രവാചകന്റെ ചോദ്യത്തിന് “ഇന്നലെ രാത്രി ഭാര്യമാരുമായി വഴക്കു കൂടാത്തവർ ആരുണ്ട്?” എന്നാണ് ഭാഷാർത്ഥം. ‘യുകാരിഫു’ (يُقَارِفِ) എന്ന പദത്തിന് ‘ഭാര്യമാരുമായി സംസർഗത്തിൽ ഏർപ്പെടുക’ എന്നല്ല അർത്ഥം ‘ഭാര്യമാരുമായി വഴക്കിടുക’, ‘തിന്മ വല്ലതും പ്രവർത്തിക്കുക’ എന്നൊക്കെയാണ് അർത്ഥം എന്ന് അറബി ഭാഷാ പണ്ഡിതരായ ഒരുപാട് മുഹദ്ദിസുകൾ പറഞ്ഞിട്ടുണ്ട്. (ശർഹു സുന്ന: ബഗ്‌വി: 1513, ഉംദത്തുൽ കാരി: 8:76, ഹുസ്നുൽ ഉസ്വ: 1:471, മിർആത്തുൽ മഫാതീഹ്: 5: 450, ശർഹുൽ കസ്തല്ലാനി, മുസ്നദു അഹ്മദ്: 13383, അൽ മജ്മൂഅ് ശർഹുൽ മുഹദ്ദബ്: 5:289)

തലേ ദിവസം ഭാര്യമാരുമായി വഴക്കിടുക, അല്ലെങ്കിൽ വല്ല പാപകൃത്യങ്ങളും ചെയ്യുക… ഇതിൽ രണ്ടിൽ ഏതായാലും മനസ്സ് ശാന്തവും നിശ്ചലവുമായിരിക്കില്ല. മനസ്സ് പ്രക്ഷുബ്‌ധവും വികാര തീവ്രവുമായിരിക്കും. അത്തരമൊരു മനസ്സുമായി മൃതദേഹം മറമാടൽ ക്രിയയിൽ ഏർപ്പെട്ടാൽ ചെയ്യേണ്ട പ്രസ്‌തുത കർമ്മത്തിൽ പല പാളിച്ചകൾ സംഭവിച്ചേക്കാം. ഭക്തിയോടെയും മരണചിന്തയോടെയും അത് നിർവഹിക്കാൻ കഴിയാതെ വരാം. അതുകൊണ്ടാണ് തലേ ദിവസം രാത്രി ഭാര്യമാരുമായി വഴക്കിടാത്തവർ അല്ലെങ്കിൽ ഇന്നലെ രാത്രി പാപമൊന്നും ചെയ്തിട്ടില്ലാത്തവർ മൃതദേഹം മറമാടാൻ നിർദേശിച്ചത്.

ﻭَﺣُﻜِﻲَ ﻋَﻦِ اﻟﻄَّﺤَﺎﻭِﻱِّ ﺃَﻧَّﻪُ ﻗَﺎﻝَ ﻟَﻢْ ﻳُﻘَﺎﺭِﻑْ ﺗَﺼْﺤِﻴﻒٌ ﻭَاﻟﺼَّﻮَاﺏُ ﻟَﻢْ ﻳُﻘَﺎﻭِﻝْ ﺃَﻱْ ﻟَﻢْ ﻳُﻨَﺎﺯِﻉْ ﻏَﻴْﺮَﻩُ اﻟْﻜَﻼَﻡَ

ഇമാം ത്വഹാവി പറഞ്ഞു: യുകാരിഫു (يُقَارِفِ) എന്ന പദത്തിന് ‘വഴക്കിടുക, തർക്കിക്കുക’ എന്നൊക്കെയാണ് അർത്ഥം. ഇതാണ് ഹദീസിന്റെ ശരിയായ അർത്ഥം. അതായത് മറ്റുള്ളവരുമായി വഴക്കിനും വക്കാണത്തിനുമൊന്നും പോയിട്ടില്ലാത്തവർ വേണം മൃതദേഹം മറമാടാൻ. (ഫത്ഹുൽ ബാരി: 3: 158)

അപ്പോൾ ഭാര്യമാരുമായി വഴക്കിട്ട മനസ്സുമായി മൃതദേഹ സംസ്കരണത്തിൽ ഏർപ്പെട്ടാലും ഭക്തിയോടെയും മരണചിന്തയോടെയും ചെയ്യേണ്ട പ്രസ്തുത കർമ്മത്തിൽ – തിടുക്കം കൂട്ടുക, അശ്രദ്ധ മൂലം പല മര്യാദകളും മറന്നു പോവുക തുടങ്ങിയ – പാളിച്ചകൾ സംഭവിച്ചേക്കാം. അതുകൊണ്ടാണ് ഭാര്യമാരുമായി വഴക്കിടാത്തവർ മൃതദേഹം മറമാടാൻ പ്രവാചകൻ (സ) നിർദേശിച്ചത് എന്ന് ഒരു കൂട്ടം പണ്ഡിതർ അഭിപ്രായപ്പെടുന്നു.

കർമാനി പറയുന്നു: “മനസ്സ് ശാന്തവും നിശ്ചലവുമായിരിക്കുന്നവരാവണം മൃതദേഹം മറമാടാൻ എന്നത് കൊണ്ട് ആണ് പ്രവാചകൻ (സ) അപ്രകാരം നിർദേശിച്ചത്. (ഉംദത്തുൽ കാരി: 8:76)

എന്ന് വച്ചാൽ ശവരതി ചിന്ത പോയിട്ട്, ഇഹലോക സംബന്ധമായ ചിന്തകളുടെ ഒരു കണിക പോലും മനസ്സിൽ അവശേഷിക്കാത്ത, പാപചിന്തകളിൽ മുക്തമായ ഏകാഗ്ര മനസ്സിനുടമകളാണ് ഖബർ സംസ്ക്കരണ പ്രവർത്തനങ്ങളിൽ ഏർപ്പെടേണ്ടത് എന്നായിരുന്നു ആ മഹാനുഭാവൻ കരഞ്ഞു കൊണ്ട് കൽപ്പിച്ച കൽപനയുടെ ഉദ്ദേശ്യം ! അത് നേർവിപരീത ദിശയുടെ അഗ്രത്തിലെത്തിച്ചു ഈ വെറുപ്പിന്റെ പ്രചാരകർ !!!

ശുദ്ധ അസംബന്ധം. ഏതോ മാനസിക രോഗികളുടെ തെറ്റിദ്ധരിപ്പിക്കൽ മാത്രം. മുഹമ്മദ് നബിയുടെ(സ്വ) ജീവിത വിശുദ്ധിയെ കുറിച്ച് അറിയുന്ന ഒരാളും ആരോപിക്കാത്ത നീച ആരോപണം. പ്രമാണങ്ങളിലെ പരാമർശങ്ങൾ ഹീനമായ താല്പര്യങ്ങൾക്ക് വളച്ചൊടിച്ചുകൊണ്ടുള്ള തരംതാണ പ്രോപഗണ്ട.

നീതിയും സത്യസന്ധതയും ഇല്ലാത്ത ഇസ്‌ലാം വിരോധികൾ തെറ്റിദ്ധരിപ്പിക്കുന്ന പ്രസ്തുത സംഭവത്തിന്റെ യാഥാർഥ്യം ഇങ്ങനെയാണ്: ബാല്യത്തിൽ ഉമ്മ മരണപ്പെട്ട മുഹമ്മദ് നബിയുടെ(സ്വ) വളർത്തുമ്മയാണ് ഫാത്തിമ ബിൻത് അസദ്. സ്വന്തം അമ്മയേക്കാൾ ഏറെക്കാലം നബിയെ പരിചരിച്ച നബിയുടെ സ്നേഹവതിയായ മാതാവ്. നബിയുടെ(സ്വ) വത്സല പിതൃവ്യൻ അബൂ ത്വാലിബിന്റെ ഭാര്യ. നബിയുടെ(സ്വ) വളർത്തുമകനും സന്തത സഹചാരിയും പിന്നീട് മരുമകനുമായ അലിയുടെ ഉമ്മ. നബിയുടെ(സ്വ) ധീര അനുയായികളായിരുന്ന ജഅഫറിന്റെയും അഖീലിന്റെയും ജുമാന(1)യുടെയും ഉമ്മ. ഭർത്താവ് ഇസ്‌ലാമിലേക്ക് വരാൻ മടിച്ചപ്പോഴും, നബിയുടെ അനുയായി ആയി പ്രവർത്തിച്ച ധീര വനിത. ഭർത്താവിന്റെ വിയോഗത്തിന് ശേഷം മക്കളോടൊപ്പം ഇസ്‌ലാമിനെയും നബിയെയും പ്രതിരോധിക്കാൻ മുന്നണിയിൽ നിരന്ന ആദർശശാലിനി. ഒരു ഹാശിമിക്ക് പിറന്ന ചരിത്രത്തിലെ ആദ്യത്തെ ഹാശിമി വനിതയെന്ന കുടുംബ അന്തസ്സിന്റെ ഉടമയും, ഒരു ഖലീഫയുടെ മാതാവായ ആദ്യത്തെ ഹാശിമി വനിതയെന്ന വ്യക്തിഗത നേട്ടത്തിന്റെ ഉടമയുമായ മഹതിയെകുറിച്ച് പ്രമുഖ ചരിത്രകാരൻമാരായ ഇബ്‌നു സഅദ് ‘ത്വബഖാത്തുൽ കുബ്റാ’യിലും, ഇബ്‌നുൽ അസീർ ‘ഉസുദുൽ ഗ്വാബ’യിലും (7168), ഇബ്‌നു അബ്ദിൽ ബർറ് ‘അൽ ഇസ്തീആബി’ലും, ഹാഫിള് ഇബ്‌നു ഹജർ അൽ അസ്ഖലാനി ‘അൽ ഇസ്വാബ’യിലും പരിചയപ്പെടുത്തുന്നു. പ്രമുഖരായ മൂന്നു മുഹദ്ദിസുകൾ മഹതിയിലൂടെ കടന്നുവന്ന ധാരാളം ഹദീസുകൾ ഉദ്ധരിച്ചിട്ടുണ്ട്.

തന്റെ മകൻ അലിയെ(റ) വീട്ടിലാക്കി നബി (സ്വ) മദീനയിലേക്ക് പലായനം ചെയ്തപ്പോൾ, മറ്റു വിശ്വാസികളോടൊപ്പം മദീനയിലേക്ക് ഹിജ്‌റ ചെയ്ത മുഹാജിറ. അലി വൈകാതെ മദീനയിലെത്തി. ജഅഫർ മറ്റു വിശ്വാസികളോടൊപ്പം ആഫ്രിക്കയിലെ എത്യോപ്പ്യയിലേക്ക് ഹിജ്‌റ പോയതാണ്). മദീനയിൽ കുറച്ചുകാലം ജീവിച്ച മഹതിയുടെ വിയോഗം മദീനയിൽ തന്നെയായിരുന്നുവെന്ന് ഇബ്‌നുൽ അസീർ, ഇബ്‌നു അബ്ദിൽ ബർറ്, ഇബ്‌നു അസാകിർ (താരീഖ് ദിമിശ്ഖ്), ദഹബി (താരീഖുൽ ഇസ്‌ലാം) മുതലായ എല്ലാ ജീവചരിത്രകാരന്മാരും രേഖപ്പെടുത്തുന്നു. ഹിജ്‌റ രണ്ടാം വർഷം ബദ്ർ പോരാട്ടത്തിന് ശേഷം, മകൻ അലി നബിയുടെ പുത്രി ഫാഥ്വിമയെ വിവാഹം ചെയ്ത ശേഷക്കാലവും മഹതി ജീവിച്ചിരുന്നിട്ടുണ്ട്. ഫാഥ്വിമയുടെ വീട്ടുജോലിഭാരങ്ങൾ സംബന്ധമായി അലി ഉമ്മയോട് സങ്കടപ്പെടുന്ന രംഗം ചരിത്രത്തിൽ രേഖപ്പെട്ടിട്ടുണ്ട്. അലിയുടെയും ഫാഥ്വിമയുടെയും കൂടെ അവരുടെ വീട്ടിൽ മഹതിയും ഉണ്ടായിരുന്നുവെന്നാണ് വിവിധ പരാമർശങ്ങൾ വ്യക്തമാക്കുന്നത്. ഹിജ്‌റ മൂന്നാം വർഷം ശവ്വാലിൽ നടന്ന ഉഹ്ദ് പോരാട്ടത്തിലായിരുന്നുവല്ലോ, നബിയുടെ പിതൃവ്യൻ ഹംസ വധിക്കപ്പെട്ടത്. ഹംസയുടെ മകൾ ഫാഥ്വിമയുടെ രക്ഷാധികാരം നബിയിലെത്തി. നബി (സ്വ) അവളെ സലമത്ത് ബ്‌നു അബീ സലമത്തിന് പിന്നീട് വിവാഹം ചെയ്തു കൊടുത്തു. നബിയ്ക്ക്(സ്വ) ഈയ്യിടെ കുറച്ചു കസവുപട്ടുതുണി സമ്മാനമായി ലഭിച്ചിരുന്നു. അത് ‘ഫാഥ്വിമമാർക്ക് മുഖമക്കനയാക്കി നൽകു’വാൻ നബി (സ്വ) നിർദ്ദേശിച്ചു. അതുപ്രകാരം, ആ തുണി ഉപയോഗിച്ച് നബിയുടെ പുത്രി ഫാഥ്വിമ, ചർച്ച ചെയ്യപ്പെടുന്ന അലിയുടെ ഉമ്മ ഫാഥ്വിമ, ഹംസയുടെ മകൾ ഫാഥ്വിമ, അലിയുടെ സഹോദരൻ അഖീലിന്റെ പത്നി ഫാഥ്വിമ എന്നിവർക്ക് മക്കനയുണ്ടാക്കി നൽകി. ഇക്കാര്യം പ്രമുഖ ഹദീസ് സമാഹാരമായ ഇബ്‌നു മാജയിൽ (ഹദീസ് നമ്പർ 3596) രേഖപ്പെടുത്തിയതിനു പുറമെ, ഹാഫിള് അസ്ഖലാനിയെ പോലുള്ള ചരിത്രകാരന്മാരും സ്ഥിരീകരിച്ചിട്ടുണ്ട്. ഹിജ്‌റ മൂന്നാം വർഷാവസാനം വരെയെങ്കിലും ഫാഥ്വിമ ബിൻത് അസദ് മദീനയിൽ ജീവിച്ചിരുന്നിട്ടുണ്ടെന്ന്‌ ഈ സംഭവം തെളിയിക്കുന്നു. പ്രവാചകൻ (സ്വ) മഹതിയുടെ ഭവനം സന്ദർശിക്കാറുണ്ടായിരുന്നുവെന്നും അവിടെ ‘ഉച്ചയുറക്കം’ (ഖൈലൂലത്ത്) നടത്താറുണ്ടായിരുന്നുവെന്നും ഇബ്‌നു സഅദ് രേഖപ്പെടുത്തുന്നു. وَكَانَ رَسُولُ اللَّهِ – صَلَّى اللَّهُ عَلَيْهِ وَسَلَّمَ – يزورها ويقيل في بيتها.

നബിയ്ക്ക് പ്രിയപ്പെട്ട മാതാവായിരുന്നു ഫാഥ്വിമ ബിൻത് അസദ്. അബൂ ത്വാലിബിന് ശേഷം മഹതിയെക്കാൾ തന്നോട് രക്തബന്ധപരിഗണന കാണിച്ച മറ്റാരേയും ഞാൻ കണ്ടിട്ടില്ല’ (لم نلق بعد أبي طالب أبرّ بي منها) എന്ന് നബി (സ്വ) പറയുകയുണ്ടായി. മഹതി മരണപ്പെട്ടപ്പോൾ, വീട്ടിലെത്തി, മഹതിയുടെ തലയുടെ ഭാഗത്തിരുന്ന് ഇങ്ങനെ അനുശോചിച്ചു: “എന്റെ പ്രിയ മാതാവേ, അങ്ങയെ അല്ലാഹു അനുഗ്രഹിക്കട്ടെ. എന്റെ പെറ്റുമ്മയുടെ വിയോഗാനന്തരം അങ്ങായിരുന്നു എന്റെ ഉമ്മ. വിശപ്പ് സഹിച്ചും ഉള്ളതെടുത്ത് അങ്ങ് എന്റെ വയറു നിറയെ തീറ്റി. ഉടുക്കാൻ ആവശ്യമുണ്ടായിട്ടും കിട്ടുന്ന വസ്ത്രങ്ങൾ അങ്ങെന്നെ അണിയിച്ചു. സ്വന്തം ഇഷ്ടങ്ങൾ വെടിഞ്ഞും അങ്ങെന്നെ രുചിയുള്ളത് ഭക്ഷിപ്പിച്ചു. അല്ലാഹുവിന്റെ പ്രീതിയും പരലോക മോക്ഷവും മാത്രമായിരുന്നു, ഇതുകൊണ്ടെല്ലാം അങ്ങ് ലക്ഷ്യം വെച്ചത്”. رَحِمَكِ اللَّهُ يَا أُمِّي، كُنْتِ أُمِّي بَعْدَ أُمِّي، تَجُوعِينَ وَتُشْبِعِينِي، وَتَعْرَيْنَ وَتَكْسِينِي، وَتَمْنَعِينَ نَفْسَكِ طَيِّبًا وَتُطْعِمِينِي، تُرِيدِينَ بِذَلِكَ وَجْهَ اللَّهِ وَالدَّارَ الْآخِرَةَ “. പിന്നെ, മഹതിയെ മൂന്നുവട്ടം ദേഹമാസകലം വെള്ളമൊഴിച്ചു കുളിപ്പിക്കാൻ നിർദ്ദേശിച്ചു. കർപ്പൂരം ഒഴിച്ചിട്ടുള്ള വെള്ളം മൃത ശരീരത്തിൽ ഒഴിക്കാൻ സമയമായപ്പോൾ, നബി (സ്വ) തന്നെ അതുവാങ്ങി, മഹതിയുടെ ശരീരത്തിൽ ഒഴിച്ചു. കുളിപ്പിച്ച് കഴിഞ്ഞപ്പോൾ, നബി അണിഞ്ഞിരുന്ന നീളക്കുപ്പായം (ഖമീസ്) അഴിച്ചു മഹതിയെ ഉടുപ്പിച്ചു. അതിനുമുകളിൽ രോമത്തിന്റെ കറുത്ത തുണികൊണ്ട് പൊതിഞ്ഞു. ശേഷം, ഉസാമത്ത് ബ്‌നു സൈദ്, അബൂ അയ്യൂബിൽ അൻസ്വാരി, ഉമർ ബ്‌നുൽ ഖത്വാബ്, ഒരു കറുത്ത സേവകൻ എന്നിവരെ വിളിച്ചു ഖബർ കുഴിക്കാൻ ആവശ്യപ്പെട്ടു. അവർ കുഴിക്കാൻ തുടങ്ങി. കുഴിയായപ്പോൾ, നബി (സ്വ) തന്നെ കുഴിയിലിറങ്ങി, തന്റെ കൈകൾ കൊണ്ട് കുഴിയുടെ ഒരുവശത്തുനിന്നും(2) മണ്ണ് മാന്തിയെടുക്കാൻ തുടങ്ങി. ആവശ്യത്തിന് മണ്ണെടുത്ത ശേഷം, നബി (സ്വ) അതിൽ ചെരിഞ്ഞു കിടന്നു (മയ്യിത്ത് വെക്കാറുള്ള പോലെ). എന്നിട്ടിങ്ങനെ പ്രാർത്ഥിച്ചു: “ജീവിച്ചിരിക്കുന്നവനും മരിക്കാത്തവനുമായ അല്ലാഹുവേ, എന്റെ പ്രിയമാതാവ് ഫാഥ്വിമ ബിൻത് അസദിന് നീ പൊറുത്തുകൊടുക്കണേ, അവർക്ക് (ഖബറിലെ ചോദ്യസമയത്ത്) സത്യപ്രമാണം നൽകി അനുഗ്രഹിക്കണേ. നിന്റെ ഈ പ്രവാചകന്റെയും എനിക്കുമുമ്പുള്ള പ്രവാചകന്മാരുടെയും മഹത്വത്തെ പരിഗണിച്ച് അവരുടെ ഖബർ നീ വിശാലമാക്കണേ, നിശ്ചയം നീ മഹാ കാരുണ്യവാനല്ലോ’. اللَّهُ الَّذِي يُحْيِي وَيُمِيتُ، وَهُوَ حَيٌّ لَا يَمُوتُ، اغْفِرْ لِأُمِّي فَاطِمَةَ بِنْتِ أَسَدٍ، وَلَقِّنْهَا حُجَّتَهَا، وَوَسِّعْ عَلَيْهَا مُدْخَلَهَا بِحَقِّ نَبِيِّكَ وَالْأَنْبِيَاءِ الَّذِينَ مِنْ قَبْلِي ; فَإِنَّكَ أَرْحَمُ الرَّاحِمِينَ” ശേഷം, നാല് തക്ബീറുകൾ ചൊല്ലി മയ്യിത്ത് നിസ്കരിച്ചു. നബിയും അബൂബക്കർ സ്വിദ്ധീഖും നബിയുടെ മൂത്ത പിതൃവ്യൻ അബ്ബാസും ചേർന്ന് മഹതിയെ ലഹ്‌ദിലേക്ക് ചെരിച്ചുകടത്തിവെച്ചു. ഹദീസ് സമാഹർത്താക്കളായ ത്വബ്റാനി തന്റെ കബീറിലും ഔസത്തിലും ഈ സംഭവം ഇതുപോലെ ഉദ്ധരിച്ചിരിക്കുന്നു(3). പതിവില്ലാത്ത ഒരു പ്രവൃത്തി നബിയിൽ നിന്നും കാണാനിടയായ അനുയായികൾ, മഹതിയെ അടക്കം ചെയ്ത ശേഷം നബിയോട് അന്വേഷിച്ചു: “അല്ലാഹുവിന്റെ ദൂതരേ, മറ്റാരുടെയും ഖബറടക്ക സമയത്ത് ചെയ്യാത്ത ചില കാര്യങ്ങൾ അങ്ങ് ഇവിടെ ചെയ്തല്ലോ?!”. അപ്പോൾ നബി പ്രതിവചിച്ചു: “എന്റെ ഖമീസ് ഞാൻ അവർക്ക് അണിഞ്ഞുകൊടുത്തത്, അവർക്ക് സ്വർഗ്ഗത്തിലെ പുടവകൾ അണിയക്കപ്പെടുവാനായിട്ടാകുന്നു. അവർ കിടക്കുന്ന ഖബ്‌റിൽ ഞാൻ കയറിക്കിടന്നത് അവർക്ക് ഖബറിന്റെ ഇടുക്കം ലഘൂകരിക്കപ്പെടാൻ വേണ്ടിയാകുന്നു. കാരണം, അബൂ ത്വാലിബ് മരണപ്പെട്ട ശേഷം ഏറ്റവും മെച്ചപ്പെട്ട രീതിയിൽ എന്നെ സംരക്ഷിച്ച സ്ത്രീയാണവർ.” (ത്വബ്റാനി)

أَلْبَسْتُهَا قَمِيصِي ; لِتَلْبَسَ مِنْ ثِيَابِ الْجَنَّةِ، وَاضْطَّجَعْتُ مَعَهَا فِي قَبْرِهَا ; خُفِّفَ عَنْهَا مِنْ ضَغْطَةِ الْقَبْرِ؛ إِنَّهَا كَانَتْ مِنْ أَحْسَنِ خَلْقِ اللَّهِ إِلَيَّ صَنِيعًا بَعْدَ أَبِي طَالِبٍ. അനേകം ചരിത്രകാരന്മാരും ഈ സംഭവം ഉദ്ധരിച്ചിട്ടുണ്ട്. തന്റെ വളർത്തുമ്മയായ അമ്മായിയോട് നബി (സ്വ) കാണിച്ച മാതൃകാപരമായ സ്നേഹകടപ്പാടുകൾ എത്ര മഹോന്നതമായിരുന്നു എന്ന് ഈ സംഭവം കാണിക്കുന്നു.

എന്നാൽ, ഇസ്‌ലാമിനെയും നബിയെയും എങ്ങനെയെങ്കിലും അവഹേളിക്കാൻ ആഗ്രഹിക്കുന്ന മലിനമനസ്‌കർ ഈ സംഭവത്തെ ശവഭോഗത്തിലേക്ക് തിരിച്ചുവിട്ടത് കാണുമ്പോൾ, വിരോധം മനുഷ്യരെ എത്ര അധമരാക്കിത്തീർക്കുന്നു എന്ന് മനസ്സിലാകും. ഒരു സംഭവം ഹദീസുകളിൽ രണ്ടുവിധത്തിൽ ഉദ്ധരിക്കാറുണ്ട്. സാമാന്യം വിശദമായും, വളരെ ചുരുക്കിയും. രണ്ടും രണ്ടു സന്ദർഭങ്ങളിൽ പ്രസക്തവുമാണ്. എന്നാൽ, ചുരുക്ക വിവരണത്തിൽ പറയുന്ന കാര്യങ്ങളെ തെറ്റിദ്ധരിപ്പിച്ചു, യഥാർത്ഥ വശം മറച്ചുവെക്കുകയായിരുന്നു ആരോപകർ. ഈ സംഭവത്തെ നരാധമർ അവതരിപ്പിച്ച രീതി ഇങ്ങനെയായിരുന്നു: മദീനയിൽ ഒരു സ്ത്രീ മരണപ്പെടുന്നു. അവരുടെ കുഴിമാടത്തിൽ പ്രവാചകൻ ഇറങ്ങുന്നു. തന്റെ വസ്ത്രം അഴിച്ചു ആ ശവത്തോടൊപ്പം കിടക്കുന്നു?! നോക്കൂ, ആ സ്ത്രീ നബിയുടെ അമ്മായിയും മാതാവുമാണെന്ന കാര്യം മറച്ചുവെച്ചു. മയ്യിത്തിനെ പുടവ അണിയിക്കുന്ന സമയത്ത് അവിടെവെച്ചാണ് തന്റെ ഖമീസ് അഴിച്ചു അതിൽ ഉമ്മയെ പൊതിഞ്ഞതെന്ന കാര്യം ഒളിച്ചുവെച്ചു. മദീനയിൽ തന്റെ അനുയായികൾ ചേർന്ന് ഖബർ കുഴിക്കുന്നതിനിടയിൽ, അവർക്ക് മുന്നിൽ വെച്ചായിരുന്നു നബി, മയ്യിത്ത് കിടത്തേണ്ട മടയിൽ ചെരിഞ്ഞുകിടന്നതെന്നും അതിനു ശേഷമായിരുന്നു മയ്യിത്ത് മറ്റുള്ളവരുടെ സഹായത്തോടെ അതേസ്ഥാനത്തേക്ക് കടത്തിവെക്കുന്നതെന്നുമുള്ള സംഗതി പുറത്തുപറഞ്ഞില്ല. മയ്യിത്ത് കിടത്തുന്ന ഇത്തരം ഖബറിലെ മടയിൽ രണ്ടാമതൊരാൾക്ക് കിടക്കാൻ കഴിയില്ലെന്ന നാട്ടനുഭവം പൂഴ്ത്തിവെച്ചു. എന്തിനാണിതെല്ലാം?! മുഹമ്മദ് ശവത്തെ ഭോഗിച്ചുവെന്നു വരുത്താൻ!! അസൂയയും വിദ്വേഷവും മനുഷ്യരെ കൊണ്ടെത്തിക്കുന്ന അനാവസ്ഥ എത്ര ഭീകരമാണ്!!

സംഭവം വളച്ചൊടിക്കാൻ ദുഷ്ടമനസ്കർ നിവേദനത്തിൽ വന്ന وَاضْطَّجَعْتُ مَعَهَا فِي قَبْرِهَا എന്ന പദത്തിന്, സംഭവത്തിന്റെ സന്ദർഭവും സാഹചര്യവും മറച്ചുവെച്ചുകൊണ്ട്, തെറ്റായി അർത്ഥം കല്പിക്കുകയായിരുന്നു. ‘ഇള്തജഅ’ =’ഭോഗിച്ചു’ എന്ന അർത്ഥത്തിൽ അവർ ചാടിപ്പിടിക്കുകയായിരുന്നു. പദപ്രയോഗത്തെ തെറ്റിദ്ധരിപ്പിച്ചുകൊണ്ട്, താന്തോന്നികൾക്ക് ശവഭോഗത്തെ അനുക്കൂലമാക്കിയെടുക്കാനും സംസ്കാരസമ്പന്നർക്ക് മുന്നിൽ പ്രവാചകനെ അവഹേളിക്കാനും നടത്തുന്ന സാഹസം പക്ഷേ, പഠിതാക്കൾക്ക് നബിയിലെ മനുഷ്യനെ മനസ്സിലാക്കാൻ കൂടുതൽ അവസരമുണ്ടാക്കി എന്നതാണ് ഗുണഫലം.

കുറിപ്പുകൾ

(1) ഹിജ്‌റ ഏഴാം വർഷം മുഹറമിലെ ഖൈബർ പോരാട്ട ശേഷം, മഹതിയുടെ മകൾ ജുമാനയ്ക്ക് നബി (സ്വ) മുപ്പത് വസ്ഖ് (ഏതാണ്ട് നൂറ്റി അമ്പത് കിലോ ഭാരം വരുന്ന ഒട്ടകച്ചുമടാണ്‌ വസ്ഖ് (30 * 150 = 4500 kg) വസ്തുക്കൾ സമ്മാനമായി കൊടുത്തയച്ച സംഭവം ചരിത്രത്തിലുണ്ട്. (അർറൗളത്തുൽ ഫൈഹാ/ യാസീനിൽ ഉമരി/മ. ഹി. 1232 ) (2)രണ്ടുതരം ഖബ്ർ ഉണ്ട്. നേരെ താഴേക്ക് കുഴിക്കുന്ന സാധാരണ രൂപം. താഴേക്ക് കുഴിച്ച ശേഷം, ഒരാളെ ചെരിച്ച് കിടത്താവുന്നവിധം ഖിബ്‌ലയുടെ ഭാഗത്തുനിന്നും മണ്ണെടുക്കുന്ന രണ്ടാം രൂപം. മുകളിൽ നിന്നും നോക്കിയാൽ മയ്യിത്ത് ഒറ്റനോട്ടത്തിൽ കാണില്ല. മുകൾ ഭാഗം അടയ്ക്കുമ്പോൾ മയ്യിത്തിനു മേൽ മണ്ണോ മറ്റോ വീഴാതെ സംരക്ഷിക്കുകയാണ് ഒരു ലക്ഷ്യം. ഉറച്ച മണ്ണുള്ളിടത്താണ് ഇത് പ്രായോഗികമാവുക. ലഹ്ദ് എന്നാണിതിന് പേര്. (3) ഹദീസ് നിവേദകരിൽ റൗഹ്ബ്നു സ്വലാഹ് എന്ന വ്യക്തി ദുർബ്ബലനാണെന്ന അഭിപ്രായം നൂറുദ്ധീൻ അൽഹൈസമി (ഹി 807) പ്രകടിപ്പിക്കുന്നുവെങ്കിലും, ആദ്യകാല നിരൂപകനായ ഇബ്നു ഹിബ്ബാൻ, ഹാകിം എന്നിവർ അദ്ദേഹം വിശ്വസ്തനാണെന്ന് സമ്മതിച്ചകാര്യം ശ്രദ്ധേയമാണ്. ഒരു മതവിധി പറയാനുള്ള ബലം ഇല്ലെന്നു വന്നാലും, ഒരു സംഭവം സ്ഥിരീകരിക്കാൻ ഈ നിവേദനം ധാരാളം മതി.

വിമർശനം:

മുഹമ്മദ് നബിയുടെ കുടുംബ പരമ്പര കിന്ദക്കാരിലേക്കാണ് എത്തുന്നത് എന്ന് കിന്ദക്കാർ വാദിച്ചത് മുഹമ്മദ് നബിയുടെ പിതൃത്വത്തെ ചോദ്യം ചെയ്യുന്ന തെളിവല്ലെ ?

മറുപടി:

ഹദീസ് ദുർബലമാണെന്നതിനു പുറമെ ഹദീസിന്റെ ആശയത്തെ വിമർശകർ ദുർവ്യാഖ്യാനിക്കുക കൂടി ചെയ്തിട്ടുണ്ട്.

കാരണങ്ങൾ:

1. ഹദീസിന്റെ നിവേദക പരമ്പര ഇപ്രകാരമാണ്:

നമ്മോട് അബുൽ ഹസൻ അലിയ്യിബ്‌നു അഹ്‌മദ് ഇബ്‌നു ഉമറിബ്‌നു ഹഫ്സ് നമ്മെ അറിയിച്ചു – അബൂ ഈസാ ബക്കാറിബ്‌നു അഹ്‌മദിബ്‌നു ബക്കാർ നമ്മോട് പറഞ്ഞു – അബൂ ജഅ്ഫറിബ്‌നു മൂസാ ഇബ്‌നു സഅ്ദ് നമ്മോട് പറഞ്ഞു – അബൂജഅ്ഫർ മുഹമ്മദിബ്‌നു അബ്ബാൻ അൽ കലാനസി നമ്മോട് പറഞ്ഞു – നമ്മോട് അബൂ മുഹമ്മദ് അബ്ദുല്ലാഹിബ്‌നു മുഹമ്മദിബ്‌നു റബീഅ അൽകുദാമി പറഞ്ഞു – മാലികിബ്‌നു അനസിൽ നിന്ന് – അദ്ദേഹം സുഹ്‌രിയിൽ നിന്ന് – അദ്ദേഹം അനസിബ്‌നു മാലികിൽ നിന്നും – അദ്ദേഹം അബൂ ബക്കറിബ്‌നു അബ്ദുർറഹ്‌മാനിൽ നിന്നും, അദ്ദേഹം പറഞ്ഞു:…

നിവേദക പരമ്പരയിലെ ‘അബൂ മുഹമ്മദ് അബ്ദുല്ലാഹിബ്‌നു മുഹമ്മദിബ്‌നു റബീഅ അൽകുദാമി’ അങ്ങേയറ്റം ദുർബലനാണ്.

ഇബ്‌നുകസീർ പറഞ്ഞു: കുദാമി തനിച്ച് നിവേദനം ചെയ്യുന്നതാണ് ഈ ഹദീസ്. അയാൾ ദുർബലനാണ്. ഇമാം ദഹബി പറഞ്ഞു: അയാൾ ദുർബലനാണ്. അയാൾ ഇമാം മാലികിൽ നിന്ന് വ്യാജ നിവേദനങ്ങൾ ഉദ്ധരിക്കുമായിരുന്നു. (മീസാനുൽ ഇഅ്തിദാൽ) ഹാകിം പറഞ്ഞു: ഇമാം മാലികിൽ നിന്ന് അയാൾ കള്ള ഹദീസുകൾ ഉദ്ധരിക്കുമായിരുന്നു. കുടുംബ പരമ്പരകളെ സംബന്ധിച്ച ഗ്രന്ഥമായ ‘അൽ അൻസാബ്’ ൽ ഇമാം സംആനി കുദാമിയെ സംബന്ധിച്ച് ഇപ്രകാരമെഴുതി: അദ്ദേഹം ചരിത്ര നിവേദനങ്ങളിൽ കൃത്രിമങ്ങൾ കാണിക്കുമായിരുന്നു എന്നതുകൊണ്ട് തന്നെ അയാളുടെ നിവേദനങ്ങൾ പ്രമാണമായി സ്വീകരിക്കപ്പെടില്ല. (ലിസാനുൽ മീസാൻ: ഇബ്‌നു ഹജർ: 3/335, അൽ മജ്‌റൂഹീൻ: 2/39)

കൂടാതെ, ഇബ്‌നുഹിബ്ബാൻ, ഇബ്‌നു അദിയ്യ്, ദാറകുത്നി എന്നിവരും അയാൾ ദുർബലനാണെന്ന് കട്ടായം പറയുന്നു.

ഇബ്‌നു സഅ്ദ് – മഅ്നിബ്‌നു ഈസായിൽ നിന്നും – അദ്ദേഹം ഇബ്‌നു അബി ദിഅ്ബിൽ നിന്നും – അദ്ദേഹം പിതാവിൽ നിന്നും ഉദ്ധരിക്കുന്ന ഒരു നിവേദനം ഇതേ സംഭവത്തെ സംബന്ധിച്ച് ഉദ്ധരിക്കപ്പെട്ടിട്ടുണ്ടെങ്കിലും (ത്വബകാത്തു ഇബ്‌നു സഅ്ദ്: 1:23) ആ നിവേദനവും ദുർബലമാണ്; നിവേദക പരമ്പര പ്രവാചകനിലേക്ക് എത്തുന്നത് (മുസ്നദ് المسند) അല്ല. വേറേയും നിവേദക പരമ്പരകളിലൂടെ ഇതേ സംഭവം ഉദ്ധരിക്കപ്പെട്ടിട്ടുണ്ടെങ്കിലും എല്ലാം അങ്ങേയറ്റം ദുർബലങ്ങളാണ്.

2. ഹദീസ് സ്വീകാര്യയോഗ്യമാണെന്ന് വാദത്തിന് അംഗീകരിച്ചാൽ തന്നെ പ്രവാചകന്റെ(സ) പിതൃത്വവുമായി യാതൊരു ബന്ധവുമില്ലാത്ത ഒരു നിവേദനത്തെ, പ്രവാചകൻ (സ) പിതൃശൂന്യനാണ് എന്ന് വരുത്തി തീർക്കാനായി നിവേദനത്തിന്റെ ആശയത്തെ വിമർശകർ അങ്ങേയറ്റം പണിപ്പെട്ട് ദുർവ്യാഖ്യാനിച്ചിരിക്കുകയാണ്.

നിവേദനത്തിന്റെ ആശയ സംഗ്രഹം:

ആകിലുൽ മുറാർ കിന്ദക്കാരുടെ രാജാവും നേതാവുമായിരുന്നു. ഇസ്‌ലാമിക കാലഘട്ടത്തിന് മുമ്പ്, ജാഹിലി കാലഘട്ടത്തിൽ കച്ചവടക്കാരായ അറബികൾക്ക് -വിശിഷ്യ കുറൈശികൾക്ക് – പല നാടുകളിലും കച്ചവടാർത്ഥം സഞ്ചരിക്കുകയും താമസിക്കുകയും ചെയ്യേണ്ടിവരുമായിരുന്നു. അബ്ബാസിബ്‌നു അബ്ദുൽ മുത്വലിബ്, റബീഅത്തിബ്‌നു ഹാരിസ്, അബൂസുഫ്‌യാൻ തുടങ്ങിയ കുറൈശി നേതാക്കൾ കച്ചവടാർത്ഥം അന്യ നാടുകളിൽ ചെന്ന് താമസിക്കുമ്പോൾ, ആ നാട്ടുകാർ “നിങ്ങൾ ആരാണ്” എന്ന് ചോദിച്ചാൽ ” ഞങ്ങൾ ആകിലുൽ മുറാർ സന്തതികളാണ്” എന്ന് പ്രതികരിക്കുമായിരുന്നു. അന്യനാട്ടിൽ സ്ഥാനവും സുരക്ഷയും ലഭിക്കാൻ വേണ്ടിയും കിന്ദക്കാരെ കൊണ്ട് അഭിമാനിച്ചു കൊണ്ടുമായിരുന്നു അവർ ഇപ്രകാരം പറഞ്ഞിരുന്നത്. അല്ലാതെ കുടുംബ ബന്ധം ഉള്ളതു കൊണ്ടായിരുന്നില്ല. പക്ഷെ കുറൈശി നേതാക്കളുടെ ഈ സംസാരം കിന്ദക്കാരുടെ അടുത്തെത്തിയപ്പോൾ കുറൈശികൾക്ക് തങ്ങളുമായി കുടുംബ ബന്ധമുണ്ടെന്ന് കിന്ദക്കാർ തെറ്റിദ്ധരിച്ചു. കുറൈശികൾക്ക് തങ്ങളുമായി കുടുംബ ബന്ധമുണ്ടെന്നതിൽ അവർ അഭിമാനം കൊണ്ടു. പിന്നീട് കുറൈശികളിൽ നിന്നും പ്രവാചകൻ (സ) നിയോഗിതനായപ്പോൾ കിന്ദക്കാരിൽ ഈ തെറ്റിദ്ധാരണ വെച്ചുപുലർത്തുന്ന ചിലർ കൂടുതൽ അഭിമാനിച്ചു. അവർ പ്രവാചകനോട്(സ) പറഞ്ഞു: “ഞങ്ങൾ ആകിലുൽ മുറാർ സന്തതികളാണ്. നിങ്ങളും ആകിലുൽ മുറാർ സന്തതികളല്ലെ ?” അപ്പോൾ പ്രവാചകൻ (സ) അവരുടെ തെറ്റിദ്ധാരണ തിരുത്തി. അബ്ബാസിബ്‌നു അബ്ദുൽ മുത്വലിബ്, റബീഅത്തിബ്‌നു ഹാരിസ്, അബൂസുഫ്‌യാൻ തുടങ്ങിയ കുറൈശി നേതാക്കൾ കച്ചവടാർത്ഥം അന്യ നാടുകളിൽ ചെന്ന് താമസിക്കുമ്പോൾ,”ഞങ്ങൾ ആകിലുൽ മുറാർ സന്തതികളാണ്” എന്ന് പറഞ്ഞിരുന്നത്, അന്യനാട്ടിൽ സ്ഥാനവും സുരക്ഷയും ലഭിക്കാൻ വേണ്ടിയും കിന്ദക്കാരെ കൊണ്ട് അഭിമാനം പ്രകടിപ്പിച്ചു കൊണ്ടുമായിരുന്നു. അല്ലാതെ കുടുംബ ബന്ധം ഉള്ളതു കൊണ്ടായിരുന്നില്ല. കുടുംബബന്ധത്തിലാണെങ്കിൽ “ഞങ്ങൾ നദ്ർ ഇബ്‌നു കിനാനയുടെ മക്കളാണ്” എന്ന് പ്രവാചകൻ (സ) അവരെ പഠിപ്പിക്കുകയും ചെയ്തു. (ത്വബകാത്തുൽ കുബ്റാ: ഇബ്‌നു സഅ്ദ്: 1/708, അൽ ബിദായ വന്നിഹായ: 5/85, സാദുൽ മആദ്: 3/676)

ഇവിടെ കിന്ദക്കാർ സംസാരിക്കുന്നത് പ്രവാചകന്റെ(സ) പിതൃത്വത്തെ സംബന്ധിച്ചേയല്ല എന്ന് നിവേദനം ഒരു വട്ടം വായിച്ചാൽ തന്നെ വ്യക്തം. മറിച്ച് കുറൈശി ഗോത്രത്തെ സംബന്ധിച്ചാണ്. മുസ്‌ലിംകളും അമുസ്‌ലിംകളുമടങ്ങുന്ന കുറൈശി ഗോത്രത്തെ സംബന്ധിച്ച്. പ്രവാചകൻ അവരോട് പറഞ്ഞ മറുപടിയും ശ്രദ്ധിക്കുക: نحن بنو النضر بن كنانة

“ഞങ്ങൾ നദ്ർ ഇബ്‌നു കിനാനയുടെ മക്കളാണ്”. നദ്ർ ഇബ്‌നു കിനാനയുടെ മറ്റൊരു പേരാണ് കുറൈശ്. (ലിസാനുൽ അറബ്: 6: 335, അൽ ബിദായ വന്നിഹായ: 3: 219, താരീഖുൽ അറബിൽ കദീം: 1: 172) അപ്പോൾ ഞങ്ങൾ കുറൈശികൾ നദ്ർ ഇബ്‌നു കിനാനയുടെ സന്തതികളാണ്, ആകിലുൽ മുറാറിന്റേതല്ല എന്നാണ് പ്രവാചകന്റെ മറുപടി. സ്വന്തം പിതൃത്വത്തെയാണ് പ്രവാചകൻ ഈ നിവേദനത്തിൽ പ്രതിരോധിക്കുന്നത് എങ്കിൽ “ഞാൻ അബ്ദുല്ലയുടെ മകനാണ്” എന്നല്ലെ പറയേണ്ടിയിരുന്നത്.?!

മാത്രമല്ല കിന്ദക്കാരിൽ ചിലർക്ക് മാത്രമായിരുന്നു ഈ തെറ്റിദ്ധാരണ ഉണ്ടായിരുന്നത്. അവരിലെ ഭൂരിഭാഗത്തിനും കുറൈശികൾ വിശിഷ്യാ ഹാശിം കുടുംബം കിന്ദക്കാരിൽ പെട്ടവരല്ല എന്ന് നന്നായി അറിയാമായിരുന്നു. അതുകൊണ്ടാണ് പ്രവാചകൻ (സ) കിന്ദക്കാരോട് ഏക ദൈവ വിശ്വാസം പ്രബോധനം ചെയ്തപ്പോൾ അവർ ഇപ്രകാരം പ്രതികരിച്ചത്: “താങ്കൾ താങ്കളുടെ ഗോത്രത്തിലേക്ക് മടങ്ങി പോവുക. താങ്കളെ ഞങ്ങൾക്ക് യാതൊരു ആവശ്യവുമില്ല…” (ഇംതാഉൽ അസ്മാഅ്: മക്‌രീസി: 8:313)

3. പ്രവാചകൻ (സ) മക്കയിൽ ഇസ്‌ലാമിക പ്രബോധനം ആരംഭിച്ചപ്പോൾ കലി പൂണ്ട കുറൈശികൾ പ്രവാചകന്റെ പിതൃവ്യൻ (പിതാവ് അബ്ദുല്ലയുടെ സഹോദരൻ) അബൂ ത്വാലിബിന്റെ അടുത്തു വന്ന് പരാതിപ്പെടുകയുണ്ടായി. അവർ അവരുടെ സംഭാഷണം തുടങ്ങുന്നത് ഇപ്രകാരമാണ്: “താങ്കളുടെ സഹോദര പുത്രൻ നമ്മളുടെ ദൈവങ്ങളെ അവമതിക്കുന്നു, ഇന്നയിന്നതെല്ലാം ചെയ്യുന്നു, ഇന്നതിന്നതെല്ലാം പറയുന്നു. താങ്കൾ അവനെ തടയണം…” അപ്പോൾ അബൂ ത്വാലിബ് പ്രവാചകന്റെ അടുത്ത് ചെന്ന് ചോദിച്ചു: “എന്റെ സഹോദര പുത്രാ, താങ്കളുടെ സമുദായം താങ്കളെ സംബന്ധിച്ച് പരാതി പറയുന്നുവല്ലോ ?….” (തഫ്സീറുത്വബ്‌രി: 23/ 149)

കുറൈശി നേതാവായിരുന്ന അബ്ദുൽ മുത്വലിബ് സൈഫിബ്‌നു ദിയസിനോട് നടത്തിയ സംഭാഷണത്തിൽ ഇപ്രകാരം കാണാം: “അല്ലയോ രാജാവെ, എനിക്ക് ഒരു പുത്രനുണ്ടായിരുന്നു (അബ്ദുല്ല). അവൻ എനിക്ക് വളരെ പ്രിയപ്പെട്ടവനായിരുന്നു. അവനോടെനിക്ക് വലിയ വാത്സ്യല്യവുമായിരുന്നു. എന്റെ സമൂഹത്തിലെ തന്നെ വിശിഷ്ട വനിതകളിൽ ഒരുവളായ ആമിന ബിൻത്ത് വഹബ് ഇബ്‌നു അബ്ദു മനാഫിബ്‌നു സഹ്‌റയെ ഞാൻ അവന് വിവാഹം കഴിപ്പിച്ചു നൽകി. അങ്ങനെ അവർക്കൊരു കുഞ്ഞുണ്ടായി. അവന് ഞാൻ മുഹമ്മദ് എന്ന് പേരിട്ടു. അവന്റെ പിതാവും മാതാവും മരണപ്പെടുകയുണ്ടായി. അങ്ങനെ അവന്റെ ചുമതല ഞാനും അവന്റെ പിതൃവ്യനും ഏറ്റെടുത്തു…” (ദലാഇലുന്നുബുവ്വ: 2:13, താരീഖു മദീനത്തി ദിമശ്ക്ക്: 3/449)

കുറൈശി പ്രമുഖനായിരുന്ന ത്വൽഹ (റ) ഒരിക്കൽ കച്ചവടത്തിനായി യാത്രയിലായിരിക്കെ ഒരു സന്യാസിവര്യൻ ‘ഈ മാസം പ്രവാചകന്മാരിൽ അന്തിമൻ പുറപ്പെടുന്ന മാസമാണെന്ന്’ സൂചിപ്പിക്കുകയുണ്ടായി. ത്വൽഹ (റ) പറയുന്നു “… അങ്ങനെ ഞാൻ ഉടൻ യാത്ര പുറപ്പെട്ടു. മക്കയിലെത്തി, മക്കക്കാരോട് എന്റെ അസാന്നിധ്യത്തിൽ വല്ലതും നടന്നോ എന്ന് ആരാഞ്ഞു. അവർ പറഞ്ഞു: “അബ്ദുല്ലയുടെ പുത്രൻ മുഹമ്മദ് പ്രവാചകത്വം വാദിക്കുകയുണ്ടായി. അബൂ കുഹാഫയുടെ മകൻ അദ്ദേഹത്തെ വിശ്വാസത്തിൽ പിന്തുടരുകയും ചെയ്തു…” (തഹ്ദീബുൽ കമാൽ: 5:1-11)

ബുഹൈരിയുടെ കഥയിൽ, പ്രവാചകത്വ അടയാളങ്ങൾ ബുഹൈരി കണ്ടെത്തിയ യുവാവിനെ സംബന്ധിച്ച് അദ്ദേഹം കുറൈശികളോട് ചോദിച്ചപ്പോൾ കുറൈശികൾ ഇപ്രകാരം പറഞ്ഞു: “അല്ലാഹുവാണെ, ഈ യുവാവ് ഞങ്ങളിലെ ഉന്നത കുടുംബത്തിൽ ജനിച്ച വ്യക്തിയാണ്.” അബീത്വാലിബിനെ ചൂണ്ടി കൊണ്ട് അവർ പറഞ്ഞു: “ഇത് ഈ വ്യക്തിയുടെ സഹോദര പുത്രനാണ്.” (സീറത്തു ഇബ്‌നു ഹിശാം: 1:117)

‘അബ്ദുല്ലയുടെ പുത്രൻ’ എന്ന് പ്രവാചകനെ കുറൈശികൾ പല സന്ദർഭങ്ങളിലും വിളിച്ചിരുന്നതായുള്ള ചരിത്ര ഗ്രന്ഥങ്ങളിലെ വാചകങ്ങൾ എണ്ണമറ്റതാണ്. (അസ്സീറത്തുൽ ഹലബിയ:1:194, സീറത്തു ഇബ്‌നു ഹിശാം: 1:117, അൽ ബിദായ വന്നിഹായ)

പ്രവാചകൻ (സ) ജനിച്ചു വളർന്ന സമൂഹം മുഴുവൻ -അദ്ദേഹത്തോട് ആദർശപരമായി ശത്രുക്കളായിരിക്കെ തന്നെ – പലയാവർത്തി അദ്ദേഹം അബ്ദുല്ലയുടെ പുത്രനും കുലീനനുമാണെന്ന് പ്രഖ്യാപിക്കുന്നു. എന്നിട്ടും അദ്ദേഹത്തെ പിതൃശൂന്യനായി ദുർവ്യാഖ്യാനിക്കുന്നതിലെ കുബുദ്ധി വിചിത്രം തന്നെ !!!

പുറമ്പോക്കില്‍ നില്‍ക്കുന്ന ഒരു ഈന്തപ്പന മരത്തോട് ഉപമിച്ചു കൊണ്ട് മുഹമ്മദ് നബിയുടെ പിതൃത്വവും കുടുംബ പരമ്പരയും കുറൈശികൾ അധിക്ഷേപിക്കുകയും നിഷേധിക്കുകയും ചെയ്തിരുന്നലോ ?

മറുപടി:

ഹദീസ് ദുർബലമാണെന്നതിനു പുറമെ ഹദീസിന്റെ ആശയത്തെ വിമർശകർ ദുർവ്യാഖ്യാനിക്കുക കൂടി ചെയ്തിട്ടുണ്ട്.

കാരണങ്ങൾ:

വിമർശകർ തങ്ങളുടെ വാദത്തിനായി തെളിവുപിടിച്ച നിവേദനം (തുർമുദി: 5/584: ഹദീസ് നമ്പർ: 3607) വളരെ ദുർബലമാണ് (ദഈഫ് ضعيف). നിവേദനത്തിന്റെ നിവേദക പരമ്പര (സനദ് السند) ഇപ്രകാരമാണ്:

حدثنا يوسف بن موسى البغدادي حدثنا عبيد الله بن موسى عن إسماعيل بن أبي خالد عن يزيد بن أبي زياد عن عبد الله بن الحارث عن العباس بن عبد المطلب قال

യൂസുഫുബ്‌നു മുസാ അൽ ബഗ്ദാദി നമ്മോട് പറഞ്ഞു – ഉബൈദുല്ലാഹിബ്‌നു മൂസാ നമ്മോട് പറഞ്ഞു – ഇസ്മാഈൽ ഇബ്‌നു അബീ ഖാലിദിൽ നിന്ന് – യസീദ് ഇബ്‌നു അബീസിയാദിൽ നിന്ന് – അദ്ദേഹം അബ്ദുല്ലാഹിബ്‌നുൽ ഹാരിസിൽ നിന്ന് – അദ്ദേഹം അബ്ബാസിബ്‌നു അബ്ദുൽ മുത്വലിബിൽ നിന്ന്, അദ്ദേഹം പറഞ്ഞു:…

നിവേദക പരമ്പരയിലെ യസീദ് ഇബ്‌നു അബീസിയാദ് വളരെ ദുർബലനാണെന്ന് ഹദീസ് നിദാനശാസ്ത്ര പണ്ഡിതർ വ്യക്തമാക്കിയിട്ടുണ്ട്.

ബൈഹകി പറഞ്ഞു: വിശ്വാസ്യതയുടെ കാര്യത്തിൽ ഒട്ടും ശക്തനല്ല. ഔസാഈ പറഞ്ഞു: യസീദ് ഹദീസിന്റെ കാര്യത്തിൽ ദുർബലനാണ്, അയാളുടെ ഹദീസ് പ്രവാചക ചര്യക്ക് വിരുദ്ധമാണ്. ഇബ്‌നു ഹിബ്ബാൻ പറഞ്ഞു: വിശ്വസ്ഥരായ നിവേദകർക്ക് എതിരായ ദുർബല ഹദീസുകൾ ഉദ്ധരിക്കുമായിരുന്നു. റാസി പറഞ്ഞു: അയാളുടെ ഹദീസ് എഴുതി വെക്കാമെങ്കിലും അവ പ്രാമാണികമല്ല. ദാറകുത്നി പറഞ്ഞു: ദുർബലൻ, നിവേദനങ്ങളിൽ ധാരാളം തെറ്റുകൾ സംഭവിക്കുന്ന വ്യക്തി. യഹ്‌യബ്‌നു മഈൻ പറഞ്ഞു: അയാളുടെ ഹദീസ് പ്രമാണമായി സ്വീകരിക്കപ്പെടില്ല, ദുർബലൻ. (http://hadith.islam-db.com/narrators/8368/يزيد-بن-أبي-زياد)

ചുരുക്കത്തിൽ വിമർശകർ ഉയർത്തിപ്പിടിക്കുന്ന ഹദീസ് ദുർബലമാണ്. മാത്രമല്ല, ഹദീസിന്റെ ആശയം വിമർശകർ ദുർവ്യാഖ്യാനിക്കുക കൂടി ചെയ്തിട്ടുണ്ട്. കാരണം, പ്രവാചകന്റെ (സ) പിതൃത്വമൊ അതിൽ കുറൈശികൾക്കുള്ള സംശയമൊ അല്ല ഈ (ദുർബല) ഹദീസിന്റെ ഉള്ളടക്കം. അതിനായി ആദ്യം നിവേദനത്തിന്റെ വിവർത്തനം കാണുക:

عن العباس بن عبد المطلب قال قلت… يا رسول الله إن قريشا جلسوا فتذاكروا أحسابهم بينهم فجعلوا مثلك كمثل نخلة في كبوة من الأرض فقال النبي صلى الله عليه وسلم إن الله خلق الخلق فجعلني من خيرهم من خير فرقهم وخير صليت ثم تخير القبائل فجعلني من خير قبيلة ثم تخير البيوت فجعلني من خير بيوتهم فأنا خيرهم نفسا وخيرهم بيتا

അബ്ബാസ്‌ ഇബ്‌നു അബ്ദുൽ മുത്വലിബിൽ (റ) നിന്ന്, അദ്ദേഹം പറഞ്ഞു: “ഞാന്‍ പ്രവാചകനോട് (സ) പറഞ്ഞു: അല്ലാഹുവിന്റെ ദൂതരേ, തീര്‍ച്ചയായും, കുറൈശ് ഗോത്രക്കാര്‍ ചേര്‍ന്ന് ഇരുന്ന്, അവരുടെ ‘അഹ്സാബി’നെ ( الأحساب ) സംബന്ധിച്ച് പരസ്പരം സംസാരിക്കുകയുണ്ടായി. എന്നാല്‍ താങ്കളെ അവര്‍ ഉപമിച്ചത്, പുറമ്പോക്കില്‍ നില്‍ക്കുന്ന ഒരു ഈന്തപ്പനമരത്തോട് ആണ്. (മറ്റൊരു നിവേദനത്തിൽ: ‘പുറമ്പോക്കിലെ തുളസി ചെടി പോലെയാണ് മുഹമ്മദ്’ എന്നും കാണാം: മജ്മഉസ്സവാഇദ്: 8/215 )

അന്നേരം പ്രവാചകന്‍ (ﷺ) പറഞ്ഞു: “തീര്‍ച്ചയായും അല്ലാഹു സൃഷ്ടികളെ സൃഷ്ടിച്ചപ്പോൾ അവരിൽ ഏറ്റവും ഉത്തമരിലാണ് എന്നെ ആക്കിയത്, അവരിൽ ഏറ്റവും ഉത്തമമായ വിഭാഗത്തിലും, ആ വിഭാഗത്തിലെ ഏറ്റവും മികച്ച രണ്ടു സംഘത്തിലും എന്നെ ഉൾപ്പെടുത്തി. പിന്നെ അവന്‍ ഗോത്രത്തില്‍ നിന്നും ഏറ്റവും നല്ല ഗോത്രത്തെ എനിക്കായി തിരഞ്ഞെടുത്തു, അതില്‍ നിന്നുള്ള കുടുംബങ്ങളില്‍ നിന്നും ഏറ്റവും മികച്ച കുടുബത്തെ തിരഞ്ഞെടുത്തു. ഞാന്‍ അവരുടെ ഇടയില്‍ ഏറ്റവും ഉത്തമനാണ്, അവരുടെ ഇടയിൽ ഏറ്റവും നല്ല കുടുംബത്തില്‍ നിന്നുള്ളവനും.” (തുർമുദി: 5/584: ഹദീസ് നമ്പർ: 3607)

കുറൈശികൾ കൂടിയിരുന്ന് പരസ്പരം ചർച്ച ചെയ്തത് അവർക്കിടയിലെയോ, പ്രവാചകന്റെയോ കുടുംബ പരമ്പരയെ (النسب) സംബന്ധിച്ചല്ല, അവർക്കിടയിലെ ‘അഹ്സാബി’നെ (മഹിമ الأحساب) സംബന്ധിച്ചാണ്. അഹ്സാബ് എന്നതിന്റെ ഏകവചനം ഹസബ് (الحسب) എന്നാണ്. ഹസബും നസബും രണ്ടും വ്യത്യസ്ഥമായ കാര്യങ്ങളാണ്. ഇത് അറബി ഭാഷാ ഗ്രന്ഥങ്ങളിൽ നിന്നും നമുക്ക് മനസ്സിലാക്കാം.

“ഒരു മനുഷ്യൻ തന്നിലും തന്റെ പ്രപിതാക്കന്മാരിലും അഭിമാന ഹേതുവായി പരിഗണിക്കുന്നവയാണ് ഹസബ്. ഒരാളുടെ ഹസബ് അയാളുടെ മതമോ ധനമോ സംഘശക്തിയോ ആകാം. പ്രവർത്തനങ്ങളിലൂടെയും കർമ്മങ്ങളിലൂടെയും നേടിയെടുക്കുന്ന മഹിമയാണ് ഹസബ്. പിതാക്കന്മാരെ കൂടാതെ തന്നെ ഹസബും കറമും (ഉദാരത) ഉണ്ടാകാം. എന്നാൽ ബഹുമാനവും മഹത്വവും പിതാക്കന്മാരിലൂടെയല്ലാതെ സിദ്ധിക്കില്ല. അഥവാ പിതൃത്വമോ കുടുംബ പരമ്പരയോ ആയി ബന്ധപ്പെട്ട ഒന്നല്ല ഹസബ്.” (മുഖ്താറുസ്സിഹാഹ്: 1/57, അന്നിഹായ ഫീ ഗരീബിൽ ഹദീസ്: 1/381)

പുറമ്പോക്കില്‍ നില്‍ക്കുന്ന ഒരു ഈന്തപ്പനമരത്തോട് ഉപമിച്ചു കൊണ്ട് മുഹമ്മദ് നബിയുടെﷺ പിതൃത്വവും കുടുംബ പരമ്പരയും അധിക്ഷേപിക്കുകയും നിഷേധിക്കുകയുമല്ല കുറൈശികൾ ചെയ്തത്. മറിച്ച് അവർക്കിടയിൽ പരസ്പരം നിലനിൽക്കുന്ന ഗോത്ര വിധ്വേഷമാണ് അവർ പ്രകടിപ്പിച്ചത്. ഹാശിം കുടുംബത്തെയാണ് അവർ പുറമ്പോക്കെന്ന് പറഞ്ഞ് ആക്ഷേപിച്ചത്. അതേസമയം മുഹമ്മദ് നബിയെ, അറബികളുടെ അടുക്കൽ ഏറ്റവും ആദരണീയമായ ഈന്തപ്പനമരത്തോടും തുളസി ചെടിയോടുമാണ് ഉപമിച്ചത്. ഇതിൽ നിന്ന് തന്നെ പ്രവാചകനെ ആക്ഷേപിക്കുകയോ അദ്ദേഹത്തിന്റെ പിതൃത്വത്തെ ചോദ്യം ചെയ്യുകയോ അല്ല എന്ന് വ്യക്തമായി മനസ്സിലാക്കാം.

മുഹമ്മദ് നബിയുടെﷺ പിതൃത്വവും കുടുംബ പരമ്പരയും കുറൈശികൾ അധിക്ഷേപിക്കുകയും നിഷേധിക്കുകയും ചെയ്തു എന്നത് വിചിത്ര വാദവും മൗഢ്യവുമാണ്. കാരണം സ്ഥാപിതവും സ്വീകാര്യയോഗ്യവുമായ ചരിത്ര നിവേദനങ്ങളും ഹദീസുകളും തെളിയിക്കുന്നത് നേർ വിപരീതമാണ്:

1. പ്രവാചകന്റെ(സ) ജനനവുമായി ബന്ധപ്പെട്ട ഒരു ദീർഘമായ ഹദീസിന്റെ ആശയ സംഗ്രഹം ഇപ്രകാരമാണ്:

ജൂതന്മാരിൽ നിന്ന് ഒരാൾ മക്കയിൽ കച്ചവട ആവശ്യാർത്ഥം താമസിക്കുകയുണ്ടായി. പ്രവാചകൻ (സ) ജനിച്ച രാത്രി അയാൾ കുറൈശികളുടെ സദസ്സിൽ വന്ന് ‘ഇന്ന് രാത്രി വല്ല കുഞ്ഞും പിറന്നോ’ എന്ന് ചോദിക്കുകയുണ്ടായി. വല്ല കുഞ്ഞും പിറന്നോ എന്ന് അന്വേഷിക്കാനായി ജൂതൻ കുറൈശികളോട് ആവശ്യപ്പെട്ടു, ഇന്ന് ഈ സമുദായത്തിൽ ഒരു പ്രവാചകൻ പിറക്കുമെന്നും ആ കുഞ്ഞിന്റെ മുതുകിൽ ഒരു അടയാളമുണ്ടാകുമെന്നും അവരോട് പറയുകയും ചെയ്തു. കുറൈശികൾ തങ്ങളുടെ വീടുകളിലേക്ക് മടങ്ങി പോവുകയും ഭാര്യമാരോട് കാര്യം പറയുകയും ചെയ്തു. അപ്പോൾ ഭാര്യമാർ പറഞ്ഞു: അബ്ദുല്ല ഇബ്‌നു അബ്ദുൽ മുത്വലിബിന് ഇന്ന് ഒരു കുഞ്ഞു പിറന്നിട്ടുണ്ട്. അവർ ആ കുഞ്ഞിന് മുഹമ്മദ് എന്ന് പേരിടുകയും ചെയ്തു. ഈ വിവരം കുറൈശികൾ ജൂതനെ അറിയിച്ചു. ഹിശാമിബ്‌നു വലീദിബ്‌നു മുഗീറ, മുസാഫിർ ഇബ്‌നു അബീ അംറ്, ഉബൈദ ഇബ്‌നുൽ ഹാരിസ് ഇബ്‌നു അബ്ദുൽ മുത്വലിബ്, ഉത്ബത്തിബ്‌നു റബീഅ തുടങ്ങി കുറൈശികളിലേയും ബനൂ മനാഫ് കുടുംബത്തിലേയും നേതാക്കളുമൊത്ത് ജൂതൻ ആമിനയുടെ വീട്ടിലേക്ക് പോയി. കുഞ്ഞിന്റെ മുതുകിൽ പ്രവാചകനുണ്ടാകുമെന്ന് പ്രവചിക്കപ്പെട്ട മുദ്ര /അടയാളം കണ്ട ജൂതൻ ബോധരഹിതനായി വീണു. ഉണർന്നപ്പോൾ കാര്യമന്വേഷിച്ച കുറൈശികളോട്, പ്രവാചകത്വം ഇസ്രാഈല്യരിൽ നിന്ന് കുറൈശികളിലേക്ക് നീക്കപ്പെട്ടല്ലൊ എന്ന് ജൂതൻ പരിതപിച്ചു. (അൽ മുസ്തദ്റക് അല സ്വഹീഹൈൻ: 2/656: ഹദീസ് നമ്പർ: 4177)

2. പ്രവാചകനും കുറൈശികളും തമ്മിൽ ഹുദൈബിയ സമാധാന കരാർ എഴുതവെ പ്രവാചകൻ (സ) അനുചരൻ അലിയോട് പറഞ്ഞു: അല്ലാഹുവിന്റെ ദൂതൻ മുഹമ്മദിൽ നിന്ന് എന്നെഴുതുക. അപ്പോൾ കുറൈശി നേതാവ് സുഹൈൽ ഇബ്‌നു അംറ് പറഞ്ഞു: നീ അല്ലാഹുവിന്റെ ദൂതനാണെന്ന് ഞങ്ങൾ അംഗീകരിച്ചിരുന്നുവെങ്കിൽ ഞങ്ങൾ നിന്നെ (മതത്തിൽ) പിന്തുടരുമായിരുന്നല്ലോ. അതുകൊണ്ട് നിന്റെ പേരും നിന്റെ പിതാവിന്റെ പേരും എഴുതുക. അപ്പോൾ പ്രവാചകൻ (സ) അലിയോട് (റ) പറഞ്ഞു: അബ്ദുല്ലയുടെ മകൻ മുഹമ്മദിൽ നിന്ന് എന്നെഴുതുക. (സ്വഹീഹു മുസ്‌ലിം: 3/1411: ഹദീസ് നമ്പർ: 1784)

3. അബ്‌സീനിയ ഭരിച്ചിരുന്ന ക്രിസ്ത്യൻ രാജാവ് നജ്ജാശിയോട് ജഅ്ഫർ (റ) പറഞ്ഞു: “രാജാവേ! ഞങ്ങള്‍ അജ്ഞരായ ഒരു ജനതയായിരുന്നു. ഞങ്ങള്‍ ബിംബങ്ങളെ ആരാധിക്കുകയും ശവം തിന്നുകയും മ്ലേച്ഛതകൾ പ്രവർത്തിക്കുകയും കുടുബബന്ധങ്ങള്‍ മുറിക്കുകയും അയല്‍ക്കാരെ ഉപദ്രവിക്കുകയും ഞങ്ങളിലെ ശക്തർ ദുർബലരുടെ ധനം ഭുജിക്കുകയും ചെയ്യുന്നവരായിരുന്നു. അങ്ങനെയിരിക്കെ അല്ലാഹു ഞങ്ങള്‍ക്ക് ഞങ്ങളില്‍നിന്നു തന്നെ ഒരു ദൈവദൂതനെ നിയോഗിച്ചു. അദ്ദേഹത്തിന്റെ കുടുംബ പരമ്പരയും സത്യസന്ധതയും വിശ്വസ്തതയും സൽസ്വഭാവവും ഞങ്ങള്‍ക്ക് നന്നായി അറിയാം. അദ്ദേഹം ഞങ്ങളെ ദൈവത്തിലേക്ക് ക്ഷണിച്ചു. ദൈവം ഏകനാണെന്ന് വിശ്വസിക്കാനും അവനെ മാത്രം ആരാധിക്കാനും, അവനു പുറമെ ഞങ്ങളും ഞങ്ങളുടെ പൂര്‍വ്വ പിതാക്കളും ആരാധിച്ചിരുന്ന ബിംബങ്ങളേയും കൽ പ്രതിമകളേയും മറ്റും ഉപേക്ഷിക്കുവാനും ഞങ്ങളോടു കല്‍പ്പിച്ചു. കൂടാതെ, സത്യം പറയുക, വിശ്വസിച്ചേൽപ്പിച്ചവ സത്യസന്ധമായി നിർവ്വഹിക്കുക, കുടുംബബന്ധം ചേര്‍ക്കുക, അയല്‍ക്കാർക്ക് നന്മ ചെയ്യുക എന്നെല്ലാം ഞങ്ങളോട് കൽപ്പിച്ചു. കൊലയും, രക്തച്ചൊരിച്ചിലും, വ്യഭിചാരവും, കളവും, അനാഥകളുടെ സമ്പത്ത് തട്ടിയെടുത്ത് ഭുജിക്കലും മാന്യവതികളായ സ്ത്രീകളെ സംബന്ധിച്ച് ദുരാരോപണങ്ങൾ ഉന്നയിക്കുന്നതുമെല്ലാം അദ്ദേഹം ഞങ്ങളോട് വിരോധിക്കുകയും ചെയ്തു. നമസ്‌ക്കരിക്കാനും വ്രതമനുഷ്ടിക്കാനും (പാവപ്പെട്ടവർക്ക് നിർബന്ധ ദാനമായ) സക്കാത്ത് നൽകാനും ഞങ്ങളോട് കൽപ്പിച്ചു. (സ്വഹീഹു ഇബ്‌നു ഖുസൈമ: 4/13: ഹദീസ് നമ്പർ: 2260, സീറത്തു ഇബ്‌നു ഇസ്ഹാക്: 213, മുസ്നദു അഹ്മദ്: 3/180, സീറത്തു ഇബ്‌നു ഹിശാം: 3/413, സ്വഹീഹു സീറത്തു ന്നബവിയ്യ: ത്വർഹൂനി: 1/340, ത്വബ്റാനി, ബസ്സാർ, ത്വയാലിസി)

മുകളിലെ ഹദീസിലെ نَعرِف نسبه وصدقه وأمانته وعفافه “അദ്ദേഹത്തിന്റെ കുടുംബ പരമ്പരയും സത്യസന്ധതയും വിശ്വസ്തതയും സൽസ്വഭാവവും ഞങ്ങള്‍ക്ക് നന്നായി അറിയാം….” എന്ന വരി പ്രത്യേകം ശ്രദ്ധ അർഹിക്കുന്നു.

4. പ്രവാചക കാലഘട്ടത്തിലെ റോമൻ ചക്രവർത്തി ഹിർക്കലിന്റെ ചരിത്രവും ഇതിന് തെളിവാണ്:

ആവിർഭാവ കാലഘട്ടത്തിൽ, ഇസ്‌ലാമിനെ സംബന്ധിച്ച് കേൾക്കുകയും അതിൽ കൗതുകം ജനിക്കുകയും ചെയ്ത ഹിർക്കൽ ചക്രവർത്തി ശാമിൽ കച്ചവടത്തിനായി വന്നു ചേർന്ന, പ്രവാചകന്റെ എതിരാളിയും കുറൈശി നേതാവുമായ അബൂസുസുഫ്‌യാനെ ചക്രവർത്തിയുടെ സന്നിധിലേക്ക് ക്ഷണിച്ചു വരുത്തി. സംഭവം അബൂസുഫ്‌യാൻ തന്നെ വിശദീകരിക്കുന്നു:

“ഞങ്ങൾ ഒരു കച്ചവട സംഘത്തിലായിരിക്കെ ഹിർക്കൽ ഒരു ദൂതനെ അയച്ചു അദ്ദേഹത്തിന്റെ അടുത്ത് ഹാജരാകാൻ കൽപ്പിച്ചു. കുറൈശികളും അല്ലാഹുവിന്റെ ദൂതനും അക്കാലത്ത് സമാധാന സന്ധിയിലേർപ്പെട്ടിരിക്കുകയായിരുന്നു. അങ്ങനെ അബൂസുഫ്‌യാനും കച്ചവട സംഘവും ഈലിയായിൽ ഹിർക്കലിന്റെ അടുത്തു ചെന്നു. ഹിർഖൽ അവരെ തന്റെ സദസ്സിലേക്ക് ക്ഷണിച്ചു. ചക്രവർത്തിക്ക് ചുറ്റും റോമിലെ പ്രധാനികളും പ്രമാണികളും ഹാജരുണ്ടായിരുന്നു. ചക്രവർത്തി തന്റെ പരിഭാഷകരേയും കൊണ്ടുവന്നു:

“നിങ്ങളിൽ ആർക്കാണ്, താൻ പ്രവാചകനാണെന്ന് വാദിക്കുന്ന ഈ വ്യക്തിയുമായി അടുത്ത കുടുംബ ബന്ധമുള്ളത് ?” എന്ന് ചക്രവർത്തി ചോദിച്ചു. “ഞാൻ പറഞ്ഞു: ഞാനാണ് ഈ കൂട്ടത്തിൽ അദ്ദേഹത്തോട് ഏറ്റവും അടുത്ത കുടുംബ ബന്ധമുള്ളയാൾ.” ഹിർക്കൽ പറഞ്ഞു; “അദ്ദേഹത്തെ എന്റെ അടുത്തേക്ക് കൊണ്ടു വരൂ, അദ്ദേഹത്തിന്റെ കൂട്ടാളികളെ അദ്ദേഹത്തിന്റെ പിറകിൽ നിർത്തൂ.” എന്നിട്ട് ഹിർക്കൽ പരിഭാഷകനോട് പറഞ്ഞു: “അവരോട് പറയൂ: ഞാൻ മുഹമ്മദെന്ന വ്യക്തിയെ പറ്റി അബൂസുഫ്‌യാനോട് ചിലത് ചോദിക്കും, അബൂസുഫ്‌യാൻ കളവ് പറഞ്ഞാൽ അദ്ദേഹം പറയുന്നത് കളവാണെന്ന് പറയണമെന്ന് അദ്ദേഹത്തിന്റെ കൂട്ടാളികളോട് പറയുക.” ശേഷം, ഹിർഖൽ ചക്രവർത്തി ചോദിച്ച ആദ്യത്തെ ചോദ്യം: “അദ്ദേഹത്തിന്റെ (മുഹമ്മദ് നബി (ﷺ)) കുടുംബ പരമ്പര എങ്ങനെയാണ്?” എന്നായിരുന്നു. ഞാൻ പറഞ്ഞു: “അദ്ദേഹം (സ) കുടുംബ മഹിമയുള്ള വ്യക്തിയാണ്.” (ദീർഘമായ സംഭാഷണത്തിന്റെ അവസാന ഭാഗത്തിൽ ഇപ്രകാരം കാണാം:) …ശേഷം ഹിർക്കൽ പരിഭാഷകനോട് പറഞ്ഞു: “അദ്ദേഹത്തോട് പറയുക: മുഹമ്മദിന്റെ കുടുംബമഹിമയെക്കുറിച്ച് ചോദിച്ചപ്പോള്‍ അദ്ദേഹം കുടുംബമഹിമയുള്ളവനാണെന്ന് താങ്കള്‍ പറഞ്ഞു. അപ്രകാരമാണ് പ്രവാചകന്മാരും, സാധാരണ ഗതിയിൽ അവർ കുടുംബ മഹിമയുള്ളവരായിരിക്കും….” (സ്വഹീഹുൽ ബുഖാരി: ഹദീസ് നമ്പർ: 7)

മുഹമ്മദിനെ(സ) ഗർഭം ധരിച്ചു എന്ന് പറയുന്നതിനപ്പുറം, ആമിനയ്ക്ക് എത്ര മാസമായിരുന്നു അബ്ദുല്ലയുടെ മരണം സംഭവിക്കുന്ന സമയം എന്ന് ഉറപ്പിക്കാവുന്ന തെളിവുകള്‍ ഒന്നും വന്നുകിട്ടിയിട്ടില്ല. അബ്ദുല്ല – ആമിന ദമ്പതികളുടേയും അബ്ദുൽമുത്തലിബ് – ഹാല ദമ്പതികളുടേയും വിവാഹം ഒരുമിച്ചാണ് നടന്നത്. അബ്ദുൽമുത്തലിബ് – ഹാല ദമ്പതികളുടെ ദാമ്പത്യം പിന്നീടും വര്‍ഷങ്ങള്‍ നീണ്ടുനിന്നു. ആയതിനാൽ അബ്ദു-മുത്തലിബ് ഹാല ദമ്പതിമാരുടെ മകന്‍ ഹംസ ബിന്‍ അബ്ദുൽ മുത്തലിബ്, അബ്ദുല്ല – ആമിന ദമ്പതിമാരുടെ പുത്രനായ മുഹമ്മദിനെക്കാളും ഇളയവന്‍ ആകണമല്ലൊ. കാരണം, അബ്ദുല്ല വിവാഹത്തിന് ശേഷം മാസങ്ങൾ മാത്രമെ ആമിനയുടെ കൂടെ ദാമ്പത്യം നടത്തിയിട്ടുള്ളു, ശേഷം മരണപ്പെട്ടു. പക്ഷെ ഹംസ മുഹമ്മദിനെക്കാള്‍ മൂന്ന്-നാലു വയസ്സിന് മുതിര്‍ന്നതാണ് എന്നതിലേയ്ക്കാണ് തെളിവുകൾ വിരല്‍ ചൂണ്ടുന്നത്. അപ്പോൾ അബ്ദുല്ല മരണപ്പെട്ടിട്ടും രണ്ടോ മൂന്നോ വര്‍ഷങ്ങള്‍ കഴിഞ്ഞാണ് ആമിന ഗര്‍ഭിണിയായത്‌ എന്നല്ലേ വരിക. ഇത് ആമിനയുടെ അവിഹിത ഗർഭത്തിലേക്കല്ലെ വിരൽ ചൂണ്ടുന്നത് ?

ആരോപണം തെളിവുകളുടെ പിൻബലമില്ലാത്തതും ചരിത്ര നിവേദനങ്ങൾ വളരെ ദുർബലവുമാണ്.

കാരണങ്ങൾ:

മുകളിൽ സൂചിപ്പിച്ച വിമർശകരുടെ ആരോപണത്തിൽ മൂന്ന് വാദങ്ങൾ ഉൾക്കൊള്ളുന്നു:

1. അബ്ദുല്ല – ആമിന ദമ്പതികളുടേയും, അബ്ദുൽ മുത്തലിബ് – ഹാല ദമ്പതികളുടേയും വിവാഹം ഒരുമിച്ചായിരുന്നു.

2. മുഹമ്മദിനേക്കാൾ(സ) മൂന്ന്-നാലു വയസ്സിന് മുതിര്‍ന്ന വ്യക്തിയാണ് ഹംസ (റ).

3. അബ്ദുല്ലയുടെ ജീവിത കാലഘട്ടത്തിൽ തന്നെ ആമിന മുഹമ്മദിനെ (സ) ഗർഭം ചുമന്നിട്ടുണ്ടോ എന്നതിന് വ്യക്തമായ തെളിവൊന്നുമില്ല എന്ന വ്യംഗ്യമായ സൂചന.

ഈ മൂന്ന് വാദവും തീർത്തും വാസ്തവ വിരുദ്ധവും കളവുമാണ്.

1. അബ്ദുല്ല – ആമിന ദമ്പതികളുടേയും, അബ്ദുൽ മുത്തലിബ് – ഹാല ദമ്പതികളുടേയും വിവാഹം ഒരേ ദിവസമായിരുന്നു എന്ന് സൂചിപ്പിക്കുന്ന നിവേദനങ്ങൾ വളരെ ദുർബലങ്ങളാണ് (ദഈഫ് ضعيف).

ഇബ്നു സഅ്ദ് തന്റെ ത്വബക്കാത്തുൽ കുബ്റായിൽ (1/94) ഉദ്ധരിച്ച ഈ നിവേദനത്തിന്റെ നിവേദക പരമ്പര (സനദ് السند) ഇപ്രകാരമാണ്:

قال حدثنا محمد بن عمر بن واقد الأسلمي قال حدثني عبد الله بن جعفر الزهري عن عمته أم بكر بنت المسور بن مخرمة عن أبيها قال وحدثني عمر بن محمد بن عمر بن علي بن أبي طالب عن يحيى بن شبل عن أبي جعفر محمد بن علي بن الحسين قالا…

മുഹമ്മദിബ്നു ഉമർ വാക്കിദുൽ അസ്ലമി നമ്മോട് പറഞ്ഞു – എന്നോട് അബ്ദുല്ലാഹിബ്നു ജഅ്ഫർ അസ്സുഹ്‌രി പറഞ്ഞു – അദ്ദേഹം തന്റെ പിതൃവ്യയായ ഉമ്മു ബകർ ബിൻത്ത് അൽ മിസ്വർ ഇബ്നു മഖ്‌റമയിൽ നിന്ന് – അദ്ദേഹം തന്റെ പിതാവിൽ നിന്ന് – എന്നോട് ഉമർ ഇബ്നു മുഹമ്മദിബ്നു ഉമറിബ്നു അലിയ്യിബ്നു അബീത്വാലിബ് പറഞ്ഞു – അദ്ദേഹം യഹ്‌യബ്നു ശിബലിൽ നിന്ന് – അദ്ദേഹം അബൂജഅ്ഫർ മഹമ്മദിബ്നു അലിയ്യിബ്നുൽ ഹുസൈനിൽ നിന്ന് – അവർ രണ്ടു പേരും പറഞ്ഞു: …..

പരമ്പരയിലെ മുഹമ്മദിബ്നു ഉമർ വാക്കിദുൽ അസ്‌ലമി (സത്യസന്ധതയിൽ) ദുർബലനാണ്. അഹ്മദിബ്നു ഹമ്പൽ പറഞ്ഞു: മുഹമ്മദിബ്നു ഉമർ വാക്കിദുൽ അസ്‌ലമി നുണയനാണ്; അയാൾ ഹദീസുകളിൽ കോട്ടിമാട്ടുമായിരുന്നു.

യഹ്‌യ പറഞ്ഞു: അയാൾ വിശ്വസ്ഥനല്ല. അയാളുടെ ഹദീസുകൾ എഴുതിവെക്കാൻ കൊള്ളാത്തത്രയും അവിശ്വസനീയമാണ്. ഇമാം ബുഖാരി, റാസി, നസാഈ എന്നിവർ പറഞ്ഞു: അയാൾ കളവു കൊണ്ട് ആരോപിതനാണ്. റാസി, നസാഈ എന്നിവർ പറഞ്ഞു: അയാൾ വ്യാജ ഹദീസുകൾ ഉണ്ടാക്കുന്ന വ്യക്തിയായിരുന്നു. ഇമാം ദാറക്കുത്നി പറഞ്ഞു: അയാളിൽ ദൗർബല്യമുണ്ട്. ഇസ്ഹാകിബ്നു റാഹൂയ പറഞ്ഞു: അയാൾ നുണയനാണ്. (അദ്ദുഅഫാഉ വൽ മത്റൂകീൻ : ഇബ്നുൽ ജൗസി: 3 / 87, അദ്ദുഅഫാഉ സ്സ്വഗീർ: ബുഖാരി: 334, അൽ ജർഹുവതഅദീൽ: അബൂഹാതിം: 8/21, അൽ കാമിൽ ഇബ്നു അദിയ്യ്: 7/ 481)

മറ്റൊരു നിവേദനത്തിന്റെ (അൽ മുസ്തദ്റക്ക്: 2:656) പരമ്പര (സനദ് السند) ഇപ്രകാരമാണ്:

4176 أخبرنا أبو جعفر محمد بن محمد بن عبد الله البغدادي حدثنا هاشم بن مرثد الطبراني حدثنا يعقوب بن محمد الزهري حدثنا عبد العزيز بن عمران حدثنا عبد الله بن جعفر عن أبي عون عن المسور بن مخرمة عن بن عباس عن أبيه قال قال عبد المطلب

അബൂ ജഅ്ഫർ മുഹമ്മദിബ്നു മുഹമ്മദിബ്നു അബ്ദുല്ല അൽ ബഗ്ദാദി നമ്മോട് പറഞ്ഞു – ഹാശിം ഇബ്നു മിർസദ് – യഅ്കൂബിബ്നു മുഹമ്മദ് അസ്സുഹ്‌രി നമ്മോട് പറഞ്ഞു – അബ്ദുൽ അസീസിബ്നു ഇംറാൻ നമ്മോട് പറഞ്ഞു – അബ്ദുല്ലാഹിബ്നു ജഅ്ഫർ – അബൂ ഔനിൽ നിന്ന് – അദ്ദേഹം മിസ്വറിബനു മഖ്റമയിൽ നിന്ന് – അദ്ദേഹം അബ്ബാസിൽ നിന്ന് – അദ്ദേഹം തന്റെ പിതാവായ അബ്ദുൽ മുത്വലിബിൽ നിന്ന്….

നിവേദക പരമ്പരയിൽ അബ്ദുൽ അസീസിബ്നു ഇംറാൻ എന്ന നിവേദകനുണ്ട്. അയാൾ വിശ്വസ്ഥനല്ലെന്ന് ഹദീസ് നിദാനശാസ്ത്ര പണ്ഡിതന്മാർ വ്യക്തമാക്കിയിട്ടുണ്ട്.

യഹ്‌യ പറഞ്ഞു: അയാൾ വിശ്വസ്ഥനല്ല. ഇമാം ബുഖാരി പറഞ്ഞു: അയാളുടെ ഹദീസുകൾ എഴുതിവെക്കാൻ കൊള്ളാത്തത്രയും അവിശ്വസനീയമാണ്. നസാഈ പറഞ്ഞു: അയാൾ കളവു കൊണ്ട് ആരോപിതനാണ്. ഇമാം ഇബ്നു ഹിബ്ബാൻ പറഞ്ഞു: അയാൾ വിശ്വസ്ഥരായ നിവേദകർക്കെതിരായി ദുർബല ഹദീസുകൾ ഉദ്ധരിക്കുന്ന വ്യക്തിയായിരുന്നു. ഇമാം തുർമുദി, ദാറക്കുത്നി എന്നിവർ പറഞ്ഞു: അയാൾ ദുർബലനാണ്. (അദ്ദുഅഫാഉ വൽ മത്റൂകീൻ: ഇബ്നുൽ ജൗസി: 2 / 111, അദ്ദുഅഫാഉ സ്സ്വഗീർ: ബുഖാരി: 223, അൽ ജർഹുവതഅദീൽ: അബൂഹാതിം: 5/391)

2. മുഹമ്മദിനേക്കാൾ(സ) മൂന്ന്-നാലു വയസ്സിന് മുതിര്‍ന്ന വ്യക്തിയാണ് ഹംസ (റ) എന്ന് സൂചിപ്പിക്കുന്ന നിവേദനത്തിന്റെ (ത്വബക്കാത്തുൽ കുബ്റാ: ഇബ്നു സഅ്ദ്: 3:10) നിവേദക പരമ്പര ഇപ്രകാരമാണ്:

قال أخبرنا محمد بن عمر قال حدثني موسى بن محمد بن إبراهيم عن أبيه قال

മുഹമ്മദിബ്നു ഉമർ വാക്കിദുൽ അസ്‌ലമി നമ്മെ അറിയിച്ചു – മൂസ ഇബ്നു മുഹമ്മദ് ഇബ്നു ഇബ്രാഹീം തന്റെ പിതാവിൽ നിന്ന് ഉദ്ധരിക്കുന്നു – അദ്ദേഹം പറഞ്ഞു:….

നിവേദക പരമ്പരയിലെ മുഹമ്മദിബ്നു ഉമർ വാക്കിദുൽ അസ്‌ലമി (സത്യസന്ധതയിൽ) ദുർബലനാണ് എന്ന് നാം മുമ്പ് സൂചിപ്പിച്ചു. പരമ്പരയിലെ മറ്റൊരു നിവേദകനായ മൂസ ഇബ്നു മുഹമ്മദ് ഇബ്നു ഇബ്രാഹീമും ദുർബലാണ്. ഇമാം അബൂ ഹാതിം പറഞ്ഞു: അയാൾ തന്റെ പിതാവിൽ നിന്ന് പിതാവ് പറഞ്ഞിട്ടില്ലാത്ത വ്യാജവർത്തമാനങ്ങൾ ഉദ്ധരിക്കുമായിരുന്നു…വിശ്വസ്ഥരായ നിവേദകർക്കെതിരായി ദുർബല ഹദീസുകൾ ഉദ്ധരിക്കുന്ന വ്യക്തിയായിരുന്നു… യാതൊരു വിധത്തിലും പ്രമാണികനല്ലാത്ത വ്യക്തി. (അൽ മജ്റൂഹീൻ: 2:241)

ഇമാം ഇബ്നു അബ്ദുൽ ബിർറ് പറഞ്ഞു: ഹംസ അല്ലാഹുവിന്റെ ദൂതനേക്കാൾ(സ) നാലു വയസ്സ് മുതിര്‍ന്നതാണ് എന്ന് സൂചിപ്പിക്കുന്ന നിവേദനമുണ്ട്. ഇത് സ്വീകാര്യയോഗ്യമല്ല. കാരണം സ്ഥാപിതമായ ഹദീസിൽ വന്നിരിക്കുന്നത് സുവൈബ ഹംസയേയും പ്രവാചകനേയും ഒരുമിച്ച് മുലയൂട്ടിയിട്ടുണ്ട് എന്നാണ്. (മുലയൂട്ടുന്ന പ്രായം രണ്ടു വയസ്സിന് താഴെയാണ്. അപ്പോൾ രണ്ടു പേർക്കും ഏകദേശം ഒരേ പ്രായമാണെന്ന് സ്ഥാപിതമാവുന്നു.) (അൽ ഇസ്തീആബ്: 1/369)

3. അബ്ദുല്ലയുടെ ജീവിത കാലഘട്ടത്തിൽ തന്നെ ആമിന മുഹമ്മദിനെ(സ) ഗർഭം ചുമന്നിട്ടുണ്ടോ എന്നതിന് വ്യക്തമായ തെളിവൊന്നുമില്ല എന്ന വ്യംഗ്യമായ സൂചന തീർത്തും കുതന്ത്രപരവും തെറ്റിദ്ധരിപ്പിക്കലുമാണ്. “ചരിത്രകാരനായ ഇബ്നു ഇസ്ഹാക് പറഞ്ഞു: അല്ലാഹുവിന്റെ തിരുദൂതനെ മാതാവ് ഗർഭം ചുമക്കുന്ന ഘട്ടത്തിലാണ് പിതാവ് അബ്ദുല്ല മരണപ്പെടുന്നത്.

ഇതാണ് ചരിത്രകാരന്മാരായ ഇബ്നു ഇസ്ഹാക് തറപ്പിച്ചു പറയുന്നത്. ഈ അഭിപ്രായത്തെ തന്നെയാണ് വാക്വിദി, ഇബ്നു സഅ്ദ്, ബലാദുരി എന്നിവർ മുന്തിക്കുന്നതും ദഹബി ശരിവെക്കുന്നതും. ഇബ്നുകസീർ പറഞ്ഞു: ഇത് വളരെ പ്രസിദ്ധമാണ്. ഇബ്നുൽ ജൗസി പറഞ്ഞു: (പ്രവാചകൻ തൊട്ടിലിൽ കിടക്കുന്ന പ്രായത്തിലാണ് പിതാവ് അബ്ദുല്ല മരണപ്പെടുന്നത് എന്ന് ചരിത്ര വിശാരദരിൽ ഒരു ന്യൂനപക്ഷത്തിന് അഭിപ്രായമുണ്ട് എന്നതൊഴിച്ചാൽ) പ്രവാചക ചരിത്രം രചിച്ച ഭൂരിഭാഗവും ഇതേ അഭിപ്രായക്കാരാണ്. ഇതിന് ഉപോൽബലകമായ നിവേദനം ഹാകിം ഉദ്ധരിക്കുകയും സ്വീകാര്യയോഗ്യമാണെന്ന് അഭിപ്രായപ്പെടുകയും ചെയ്തിരിക്കുന്നു. കൈസിബ്നു മഖ്റമ എന്ന പ്രവാചക ശിഷ്യനിൽ നിന്ന് തന്നെ ഇക്കാര്യം ഉദ്ധരിച്ച് ഉറപ്പിക്കുന്നുണ്ട് ഇമാം ദഹബി. ഇബ്നു ഇസ്ഹാക് അല്ലാത്ത ചരിത്രകാരന്മാർ പറയുന്നത് ആമിന പ്രവാചകനെയും കൊണ്ട് രണ്ട് മാസം ഗർഭിണിയായിരിക്കെയാണ് അബ്ദുല്ല മരണപ്പെട്ടത് എന്നാണ്.” (സുബുലുൽ ഹുദാ വർറശാദ്: അസ്സ്വാലിഹി അശ്ശാമി: 1/331)

സ്ത്രീകളെ അധികാരം ഏല്‍പ്പിച്ച ഒരു ജനതയും വിജയിക്കില്ലെന്ന് പഠിപ്പിച്ചത് മുഹമ്മദ് നബിയാണ്. അതുകൊണ്ടുതന്നെ പ്രവാചകാനുചരന്മാര്‍ സ്ത്രീകളുടെ അധികാരം അംഗീകരിക്കാന്‍ തയ്യാറായിരുന്നില്ല. ജമല്‍ യുദ്ധം നയിച്ച ആഇശ പ്രവാചകപത്‌നിയായിരുന്നിട്ടുപോലും, ഒരു സ്ത്രീ ആയതിനാല്‍ അവരുടെ നേതൃത്വത്തിനോട് പ്രവാചക ശിഷ്യന്‍ അബൂബക്‌റത് മുഖം തിരിച്ചു നിന്നത്,’സ്ത്രീയെ അധികാരം ഏല്‍പ്പിച്ച ജനത വിജയിക്കില്ലെന്ന’ പ്രവാചകോപദേശം മുഖവിലക്കെടുത്തതിനാലായിരുന്നു എന്നത് അദ്ദേഹം തന്നെ പറഞ്ഞതായി ഹദീഥ് ഗ്രന്ഥങ്ങള്‍ രേഖപ്പെടുത്തിയിട്ടുണ്ട്. പ്രത്യക്ഷമായ പെണ്‍വിരുദ്ധതയാണ് പ്രവാചകന്‍ പഠിപ്പിച്ചിരുന്നതെന്ന് ഇത് വ്യക്തമാക്കുന്നുണ്ടല്ലോ ?

ഒരു ഹദീഥ് ഉദ്ദരിക്കുകയും എന്നിട്ട് തങ്ങള്‍ക്ക് എന്താണോ തോന്നുന്നത് അപ്രകാരം അതിനെ വിലയിരുത്തുകയും വിമര്‍ശനവിധേയമാക്കുകയും ചെയ്യുക എന്നത് ഇസ്‌ലാംവിദ്വേഷികളുടെ സ്ഥിരം പതിവാണ്. ആ ഹദീഥിനെ പ്രമാണമായി കാണുന്ന മുസ്‌ലിം സമൂഹം അതിനെ എപ്രകാരമാണ് വിലയിരുത്തിയതും സ്വീകരിച്ചതുമെന്നും ഹദീഥിന്റെ ആധികാരികവും പ്രാമാണികവുമായ വ്യഖ്യാനം എങ്ങനെയാണെന്നും ഇസ്‌ലാമിക പ്രമാണങ്ങളുടെ ആകത്തുക ഈ വിഷയത്തെ എങ്ങനെ വീക്ഷിക്കുന്നുവെന്നുമുള്ള വളരെ പ്രധാനപ്പെട്ട ഘടകങ്ങളെ അവഗണിക്കുക എന്നത് നിരൂപണ രംഗത്തെ സത്യസന്ധതയില്ലായ്മയെയാണ് അടിവരയിടുന്നത്. സ്ത്രീ ഭരണാധികാരിയാവുക അല്ലെങ്കില്‍ അധികാരത്തിന്റെ ഏതെങ്കിലും ഒരു പങ്ക് സ്ത്രീയെ ഏല്‍പ്പിക്കുക എന്നത് ഇസ്‌ലാമിക പ്രമാണങ്ങള്‍ക്ക് വിരുദ്ധമാണോ? അല്ല എന്നതാണ് വസ്തുത. കാരണം, തന്റെ സമൂഹത്തെ ഏറ്റവും നല്ല നിലയില്‍ നയിച്ച ഒരു സ്ത്രീയുടെ ചരിത്രം വിശുദ്ധ ക്വുര്‍ആന്‍ തന്നെ മുസ്‌ലിം സമൂഹത്തെ പഠിപ്പിച്ചിട്ടുണ്ട്; സൂറത്തു’ന്നംലി’ല്‍ ക്വുര്‍ആന്‍ വിവരിച്ച ‘സബഇ’ലെ രാജ്ഞി ബല്‍ഖീസിന്റെ ചരിത്രം. നല്ല യുക്തിയോടും വിവേകത്തോടും കൂടിയാണ് അവര്‍ ഭരണം നടത്തിയിരുന്നത്. തന്റെ തീരുമാന ശക്തിയും വിവേചനാധികാരവും ഉപയോഗിച്ച് സര്‍വനാശത്തില്‍ എത്തിപ്പെടുമായിരുന്ന ഒരു യുദ്ധത്തില്‍ നിന്നും അവര്‍ സ്വന്തം ജനതയെയും രാഷ്ട്രത്തെയും രക്ഷിച്ച ചരിത്രം വിശുദ്ധ ക്വുര്‍ആന്‍ പരാമര്‍ശിച്ചിട്ടുണ്ട്. നയതന്ത്രപരമായ പക്വതയുള്ള ഒരു ഭരണാധികാരിയായാണ് ബല്‍ഖീസ് രാജ്ഞിയെ ക്വുര്‍ആന്‍ അടയാളപ്പെടുത്തിയത്. തന്റെ അധികാര പരിധിയില്‍ കീഴൊതുങ്ങുവാനുള്ള സര്‍വാധിപതിയായ സുലൈമാന്റെ(അ) ആഹ്വാനദൂത് എത്തിയപ്പോള്‍ അവര്‍ തന്റെ കൂടിയാലോചനാ സദസ്സ് വിളിച്ചുകൂട്ടി. പ്രസ്തുത സംഭവം ക്വുര്‍ആന്‍ ഇപ്രകാരം വിവരിച്ചു:

”അവള്‍ പറഞ്ഞു: ഹേ, പ്രമുഖന്മാരേ, എനിക്കിതാ മാന്യമായ ഒരെഴുത്ത് നല്‍കപ്പെട്ടിരിക്കുന്നു” ”അത് സുലൈമാന്റെ പക്കല്‍ നിന്നുള്ളതാണ്. ആ കത്ത് ഇപ്രകാരമത്രെ: ‘പരമകാരുണികനും കരുണാനിധിയുമായ അല്ലാഹുവിന്റെ നാമത്തില്‍’ ”എനിക്കെതിരില്‍ നിങ്ങള്‍ അഹങ്കാരം കാണിക്കാതിരിക്കുകയും, കീഴൊതുങ്ങിയവരായിക്കൊണ്ട് നിങ്ങള്‍ എന്റെ അടുത്ത് വരികയും ചെയ്യുക” ”അവള്‍ പറഞ്ഞു: ഹേ, പ്രമുഖന്മാരേ, എന്റെ ഈ കാര്യത്തില്‍ നിങ്ങള്‍ എനിക്ക് നിര്‍ദ്ദേശം നല്‍കുക. നിങ്ങള്‍ എന്റെ അടുക്കല്‍ സന്നിഹിതരായിട്ടല്ലാതെ യാതൊരു കാര്യവും ഖണ്ഡിതമായി തീരുമാനിക്കുന്നവളല്ല ഞാന്‍” ”അവര്‍ പറഞ്ഞു: ‘നാം ശക്തിയുള്ളവരും ഉഗ്രമായ സമരവീര്യമുള്ളവരുമാണ്. അധികാരം അങ്ങയ്ക്കാണല്ലോ, അതിനാല്‍ എന്താണ് കല്‍പിച്ചരുളേണ്ടതെന്ന് ആലോചിച്ചു നോക്കുക” ”അവള്‍ പറഞ്ഞു: തീര്‍ച്ചയായും രാജാക്കന്മാര്‍ ഒരു നാട്ടില്‍ കടന്നാല്‍ അവര്‍ അവിടെ നാശമുണ്ടാക്കുകയും, അവിടത്തുകാരിലെ പ്രതാപികളെ നിന്ദ്യരാക്കുകയും ചെയ്യുന്നതാണ്. അപ്രകാരമാകുന്നു അവര്‍ ചെയ്തുകൊണ്ടിരിക്കുന്നത്” ”ഞാന്‍ അവര്‍ക്ക് ഒരു പാരിതോഷികം കൊടുത്തയച്ചിട്ട് എന്തൊരു വിവരവും കൊണ്ടാണ് ദൂതന്മാര്‍ മടങ്ങി വരുന്നതെന്ന് നോക്കാന്‍ പോകുകയാണ്.” (ക്വുര്‍ആന്‍ 27: 29-35)

സുലൈമാന്‍ എന്ന സര്‍വാധിപതിക്കെതിരെ ഒരു യുദ്ധമുണ്ടായാല്‍ തന്റെ സമൂഹത്തിന്റെ സര്‍വനാശമായിരിക്കും പരിണിതഫലം എന്നു ദീര്‍ഘദര്‍ശനം ചെയ്ത അവള്‍, തനിക്കു യുദ്ധ പിന്തുണ പ്രഖ്യാപിച്ച പ്രമുഖന്മാരുടെ തീരുമാനത്തെ അവഗണിച്ചുകൊണ്ട് യുദ്ധം ഒവിവാക്കുക എന്ന വിവേക പൂര്‍ണ്ണമായ നിലപാട് സ്വീകരിക്കുകയും സ്വന്തം രാഷ്ട്രത്തെയും ജനങ്ങളെയും നാശത്തില്‍ നിന്നും രക്ഷിക്കുകയുമാണുണ്ടായത്. അവസാനം അവര്‍ സുലൈമാന്റെ(അ) അരികിലെത്തി സത്യവിശ്വാസം സ്വീകരിക്കുകയും ചെയ്തതായി ക്വുര്‍ആന്‍ പഠിപ്പിക്കുന്നുണ്ട്. (27:44)

ഭരണാധികാരിയായ ഒരു സ്ത്രീയുടെ വിവേകപൂര്‍ണ്ണമായ ഭരണത്തെ എടുത്തു പരാമര്‍ശിച്ച ക്വുര്‍ആന്‍, ഒരിക്കലും സ്ത്രീ ഭരണാധികാരം കൈയ്യാളാന്‍ പാടില്ലെന്ന് പറഞ്ഞില്ല. മാത്രമല്ല ബല്‍ഖീസ് രാജ്ഞിയുടെ സത്യവിശ്വാസപൂര്‍വകാലത്തെ തെറ്റുകളെ പരാമര്‍ശിച്ച ക്വുര്‍ആന്‍ (27: 24) ഭരണാധികാരം അവര്‍ കയ്യാളിയത് ഒരു തെറ്റായി സൂചിപ്പിച്ചുമില്ല.

എങ്കില്‍ പിന്നെ ‘തങ്ങളുടെ ഭരണനേതൃത്വം സ്ത്രീയെ ഏല്‍പ്പിച്ച ജനത വിജയിക്കുകയില്ല’ എന്ന ഹദീഥിനെ എങ്ങനെയാണ് മനസ്സിലാക്കേണ്ടത്?

ഒന്ന്, പ്രവാചക കാലഘട്ടത്തില്‍ നിലനിന്നിരുന്ന വമ്പിച്ച രണ്ട് സാമ്രാജ്യത്വ ശക്തികളായിരുന്നു റോമും പേര്‍ഷ്യയും. സാമ്രാജ്യത്വ വികസനത്തിനായി പരസ്പരം ശക്തമായ പോരാട്ടം ഇരു സാമ്രാജ്യത്വങ്ങളും നടത്തിയിരുന്ന ഘട്ടത്തിലാണ് പേര്‍ഷ്യന്‍ ചക്രവര്‍ത്തി കിസ്റാ മരണപ്പെടുന്നത്. തുടര്‍ന്ന് പേര്‍ഷ്യന്‍ സാമ്രാജ്യത്വത്തിന്റെ അധികാരത്തിനുവേണ്ടി രാജകുടുംബങ്ങള്‍ക്കിടയില്‍ ശക്തമായ അധികാര വടംവലി നടക്കുകയുണ്ടായി. രാജകുടുംബങ്ങളിലെ പലരും കൊലചെയ്യപ്പെട്ടു. അവസാനം ചക്രവര്‍ത്തിയുടെ മകളെ സമൂഹം ഭരണനേതൃത്വം ഏല്‍പ്പിച്ചു. ഈ സന്ദര്‍ഭത്തിലാണ് നബി (സ) ‘തങ്ങളുടെ ഭരണനേതൃത്വം സ്ത്രീയെ ഏല്‍പ്പിച്ച ജനത വിജയിക്കുകയില്ല’ (ബുഖാരി) എന്നു പറഞ്ഞത്. പ്രവാചകന്‍ (സ) ഇതു പറഞ്ഞ് ഏറെ താമസിയാതെ തന്നെ പേര്‍ഷ്യന്‍ സാമ്രാജ്യം തകരുകയാണുണ്ടായത്. അതുകൊണ്ടുതന്നെ ഇത് പേര്‍ഷ്യന്‍ സാമ്രാജ്യത്വത്തെ സംബന്ധിച്ച ഒരു പ്രവചനമാണോ – അഥവാ ഹദീഥ് പശ്ചാത്തലത്തോട് മാത്രം ബന്ധപ്പെട്ടതാണോ – അതല്ല ഹദീഥിലെ ആശയത്തെ സാമാന്യവല്‍ക്കരിക്കാമോ എന്ന കാര്യത്തില്‍ പണ്ഡിതന്മാര്‍ക്കിടയില്‍ അഭിപ്രായാന്തരമുണ്ട്.

മുഹമ്മദ് അമാനി മൗലവി എഴുതുന്നു: ”പേര്‍ഷ്യക്കാരുടെ ഭരണം ഒരു രാജ്ഞിയുടെ കൈയ്യിലായിരുന്ന ഘട്ടത്തിലാണ് നബി (സ) ഇതു പറഞ്ഞത്. ഈ ഹദീഥില്‍ സ്ത്രീനേതൃത്വത്തെ ആക്ഷേപിച്ചിരിക്കുന്നത് പേര്‍ഷ്യക്കാരെ മാത്രം ബാധിക്കുന്ന ഒരു സംഗതിയാണോ, അതോ സ്ത്രീനേതൃത്വത്തെ തത്ത്വത്തില്‍ എതിര്‍ത്തിരിക്കുകയാണോ എന്ന കാര്യത്തില്‍ ഭിന്നാഭിപ്രായങ്ങളുണ്ട്. ഏതായാലും പേര്‍ഷ്യക്കാരുടെ കാര്യത്തില്‍ നബി (സ) പറഞ്ഞത് താമസിയാതെ തന്നെ പുലര്‍ന്നുവെന്നത് ഒരു ചരിത്ര യാഥാര്‍ത്ഥ്യമാണ്” (ഇസ്‌ലാമിക ജീവിതം, പേജ്: 163,164)

ശറഫുദ്ദീന്‍ ത്വീബി പറഞ്ഞു: ”പേര്‍ഷ്യക്കാര്‍ രാഷ്ട്രീയ വിജയം കൈവരിക്കില്ല എന്ന് ഉറപ്പ് നല്‍കിക്കൊണ്ടുള്ള പ്രവചനമാണിത്. അറബികള്‍ക്ക് (അവരുടെ മേല്‍) കൈവരുന്ന വിജയത്തെ സംബന്ധിച്ച സൂചന ഈ പ്രസ്താവനയില്‍ ഉണ്ട്. അപ്പോള്‍ ഈ പ്രസ്താവന മുഅ്ജിസത്ത് (പ്രവാചകത്വത്തിനുള്ള അമാനുഷികമായ തെളിവ്) ആകുന്നു” (ഫൈദുല്‍ കദീര്‍: 5/303)

‘ഹദ്യുല്‍ ഇസ്‌ലാം’ എന്ന ഗ്രന്ഥത്തില്‍ ഡോ. യൂസുഫുല്‍ ഖറദാവി പറഞ്ഞു: ”ഈ ഹദീഥിന്റെ ആശയം സാമാന്യവല്‍ക്കരിക്കാമോ അതോ അതിന്റെ പശ്ചാത്തലത്തോട് മാത്രം ബന്ധപ്പെട്ടതാണോ? പേര്‍ഷ്യക്കാര്‍ വിജയിക്കാന്‍ പോകുന്നില്ല എന്ന് തിരുമേനി പറഞ്ഞത്, സമുദായത്തില്‍ അവളേക്കാള്‍ ആയിരം മടങ്ങ് പ്രാപ്തരും ശ്രേഷ്ഠരുമായ ആളുകളുണ്ടായിരുന്നിട്ടും ചക്രവര്‍ത്തിയുടെ മകളെ പാരമ്പര്യമായി വാഴിച്ചു എന്ന അര്‍ത്ഥത്തിലാണ്. അത്തരക്കാര്‍ വിജയിക്കുകയില്ല.”

ഈജിപ്റ്റിലെ പണ്ഡിത സഭയുടെ ഫത്‌വ ബോര്‍ഡ് ‘ദാറുല്‍ ഇഫ്താഉല്‍ മിസ്രിയ്യ’ പറയുന്നു: ”കിസ്റാ ചക്രവര്‍ത്തിക്ക് പ്രവാചകന്‍ (സ) കത്തയച്ചപ്പോള്‍ അയാള്‍ അത് പിച്ചിചീന്തുകയാണ് ചെയ്തത്. അയാള്‍ തന്റെ കത്ത് പിച്ചിചീന്തിയതുപോലെ അയാളുടെ അധികാരവും ചിന്നഭിന്നമാക്കപ്പെടാന്‍ പ്രവാചകന്‍ (സ) പ്രാര്‍ത്ഥിച്ചു. അപ്പോള്‍ പ്രവാചകന്റെ പ്രാര്‍ത്ഥനക്കുത്തരമെന്നോണം അല്ലാഹു അയാളുടെ മകനെ അയാള്‍ക്കെതിരെ തിരിച്ചു. മകന്‍ പിതാവിനെ വധിച്ചു. തുടര്‍ന്ന് സഹോദരങ്ങളേയും. അങ്ങനെ പ്രവാചകന്‍ പ്രാര്‍ത്ഥിച്ചതുപോലെ ഭരണകൂടം ചിന്നഭിന്നമാക്കപ്പെട്ടു. ഒരു സ്ത്രീയെ ചക്രവര്‍ത്തി ആക്കുന്നതിലേക്ക് കാര്യം ചെന്നെത്തി. അത് ഭരണകൂടത്തിന്റെ ഉന്മൂല നാശത്തിന് കാരണമാവുകയും ചെയ്തു. രാജകുടുംബത്തിലെ പുരുഷന്മാര്‍ അധികാര വടംവലിയില്‍ മുഴുകിയതിനാലും പരസ്പരം കൊന്നൊടുക്കി കൊണ്ടിരുന്നതിനാലും ഒരു സ്ത്രീയെ ചക്രവര്‍ത്തിയാക്കി എന്നറിഞ്ഞപ്പോള്‍ അത് അവരുടെ ഭരണകൂടത്തിന്റെ ഉന്മൂലനാശത്തിന്റെ അടയാളമാണെന്ന് പ്രവാചകന്‍ (സ) അറിയിച്ചു. ലോകത്ത് ഏത് സ്ത്രീയും വിജയം കൈവരിക്കില്ല എന്നല്ല പ്രവാചന്‍ അറിയിച്ചത്. പ്രത്യേക വ്യക്തികളെ സംബന്ധിച്ച പ്രസ്താവന പൊതുവല്‍ക്കരിക്കാന്‍ പാടില്ല എന്നത് ഇസ്‌ലാമിലെ അടിസ്ഥാന തത്ത്വങ്ങളെ സംബന്ധിച്ച വിജ്ഞാന ശാഖയിലെ ഒരു അംഗീകൃത തത്ത്വമാണ്. ഇമാം ശാഫിഈ പറഞ്ഞതായി ഉദ്ധരിക്കപ്പെടുന്നു.”പ്രത്യേക സാഹചര്യങ്ങളുമായി ബന്ധപ്പെട്ട വിഷയങ്ങളില്‍ (പ്രത്യേകവും പൊതുവുമായ ഉദ്ദേശതലങ്ങളില്‍) ഏതും ഉദ്ദേശിക്കപ്പെട്ടതായിരിക്കാന്‍ സാധ്യത നിലനില്‍ക്കുന്നുവെങ്കില്‍ അതുകൊണ്ട് തെളിവു പിടിക്കാന്‍ കഴിയില്ല.” ചര്‍ച്ചാ വിഷയകമായ ഹദീഥ് ഒരു വ്യക്തിയുടെ വിഷയത്തില്‍ ആയതുകൊണ്ട് തന്നെ അതിനെ പൊതുവായ തെളിവായി സ്വീകരിക്കല്‍ ശരിയാവുകയില്ല; അത് പൊതുവായ വിധിയാണെന്ന് സൂചിപ്പിക്കുന്ന മറ്റു തെളിവുകള്‍ ഉണ്ടായാല്‍ ഒഴികെ.” (htthp://www.fatawa.com/view/15196)

ഒരു പ്രത്യേക വ്യക്തിയുടെ വിഷയത്തിലൊ പ്രത്യേക അവസ്ഥയിലൊ പ്രമാണങ്ങലില്‍ വന്ന വിധിയെ പൊതുവായി എല്ലാവര്‍ക്കുമുള്ള തെളിവായി സ്വീകരിക്കല്‍ അനുവദനീയമല്ല; അത് പൊതുവായ വിധിയാണെന്ന് സൂചിപ്പിക്കുന്ന മറ്റു തെളിവുകള്‍ ഉണ്ടായാല്‍ ഒഴികെ. ‘വകാഇഉല്‍ അഅ്യാന്‍’ (وقا ئع الأعيان) എന്നാണ് ഇത്തരം കേസുകളുടെ ഇസ്‌ലാമിക കര്‍മ്മശാസ്ത്രത്തിലെ സാങ്കേതിക നാമം. ‘വകാഇഉല്‍ അഅ്യാന്‍’ ആയ വിഷയങ്ങളെ വ്യവച്ഛേദിക്കുന്നതില്‍ – ചില അഭിപ്രായ വ്യത്യാസങ്ങള്‍ നിലനില്‍ക്കുന്നതോടൊപ്പം തന്നെ, കര്‍മ്മശാസ്ത്രവിധി നിര്‍ധാരണത്തിന്റെ അടിസ്ഥാന തത്ത്വങ്ങളില്‍ സര്‍വാംഗീകൃതമായ ഒരു തത്ത്വമാണിത്. (അല്‍ കവാഇദുല്‍ ഹിസ് നി: 3/78, വകാഇഉല്‍ അഅ്‌യാന്‍ ഫില്‍ ഇബാദത്ത്: ദിറാസ ഫിക്ഹിയ്യ: 1/4, കള്വായ അല്‍ അഅ്‌യാന്‍: ദിറാസ ഉസൂലിയ തത്ബീകിയ്യ : 16)

ഈ വിഷയകമായി ജോര്‍ദാനിലെ ഫത്‌വ സമിധി നടത്തിയ പഠനവും ഇവിടെ ഉദ്ധരിക്കുന്നത് പ്രസക്തമായിരിക്കും: ”നബി (സ) അബൂദർറിനോട്, അദ്ദേഹം അധികാരം ചോദിച്ച സന്ദര്‍ഭത്തില്‍ ഇപ്രകാരം പറയുകയുണ്ടായി ‘അബൂദറെ തീര്‍ച്ചയായും താങ്കള്‍ ഒരു ദുര്‍ബല പ്രകൃതിയിലുള്ള മനുഷ്യനാണ്. അധികാരം എന്നു പറയുന്നതാകട്ടെ അമാനത്താണ്. അത് പരലോകത്ത് നിന്ദ്യതയും ഖേദവുമായി പരിണമിക്കുന്നതാണ്; അതിന്റെ അവകാശങ്ങള്‍ പൂര്‍ണമായും വകവെച്ചു കൊടുത്തു കൊണ്ട് അതിനെ സ്വീകരിച്ച ആളുകള്‍ ഒഴികെ, ഒരാളുടെ മേല്‍ ആ അധികാരം കിട്ടികഴിഞ്ഞാല്‍ എന്തൊക്കെ ബാധ്യതകളുണ്ടോ അതു നിര്‍വഹിക്കുകയും ചെയ്തവനൊഴികെ’ (മുസ്‌ലിം). ഈ സമുദായത്തില്‍ നിന്ന് സ്ത്രീയാകട്ടെ പുരുഷനാകട്ടെ പ്രവാചകന്‍ (സ) പറഞ്ഞ നിബന്ധനകളും ഘടകങ്ങളും ആരിലാണോ പൂര്‍ത്തിയാക്കപ്പെടുന്നത് എന്നുള്ളതില്‍ വ്യത്യാസമില്ല. കാരണം പ്രവാചകന്‍ (സ) ഇങ്ങനെയും പറഞ്ഞിട്ടുണ്ട്. ‘സ്ത്രീകള്‍ പുരുഷന്മാരുടെ കൂടെപിറപ്പുകളാണ്’ (അബൂദാവൂദ്). സ്ത്രീകളുമായും കുടുംബവുമായുമൊക്കെ ബന്ധപ്പെട്ട വ്യവസ്ഥകളുടെയും നിയമങ്ങളുടെയും കാര്യത്തില്‍ സമത്വവും നേരെചൊവ്വെയുള്ള നിലപാടുകളും ഭരണകൂടത്തിനു മുമ്പില്‍ എത്തണമെങ്കില്‍ യുക്തിദീക്ഷിതമായി ചിന്തിക്കുന്ന സ്ത്രീയുടെ അഭിപ്രായത്തിന് വളരെ അധികം പ്രാധാന്യമുണ്ട്. ‘തങ്ങളുടെ ഭരണനേതൃത്വം ഒരു സ്ത്രീയെ ഏല്പിച്ച ജനത വിജയിക്കുകയില്ല’ (ബുഖാരി) എന്ന ഹദീഥില്‍ അത് ഉള്‍പ്പെടുകയില്ല. കാരണം ഈ ഹദീഥ് അതിന്റെ പ്രത്യേകമായ സാഹചര്യവും അതുപോലെ തന്നെ സന്ദര്‍ഭത്തില്‍ നിന്നും മാത്രമേ മനസ്സിലാക്കാന്‍ പാടുള്ളൂ. പൊതുകാര്യങ്ങളിലായാല്‍ പോലും വളരെ പ്രധാനപ്പെട്ട കാര്യങ്ങള്‍ തന്നെ വിശ്വാസികളുടെ മാതാക്കളായ പ്രവാചക പത്നിമാരോട് പ്രവാചകന്‍ കൂടിയാലോചന നടത്താറുണ്ടെന്നത് സ്ഥാപിതമായ കാര്യമാണ്.” (ലജനത്തുല്‍ ഇഫ്ത്വാഅ് അല്‍ ഉര്‍ദുനിയ)

‘തങ്ങളുടെ ഭരണനേതൃത്വം ഒരു സ്ത്രീയെ ഏല്‍പ്പിച്ച ജനത വിജയിക്കുകയില്ല’ എന്ന ഹദീഥ് സാമാന്യവല്‍ക്കരിക്കേണ്ടതില്ലെന്നും അത് പ്രത്യേകമായ സാഹചര്യവും സന്ദര്‍ഭവും കൊണ്ട് വിശദീകരിക്കപ്പെടേണ്ടതാണെന്ന വീക്ഷണം തന്നെയാണ് യു എ ഇയിലേയും കുവൈത്തിലെയും പണ്ഡിത സഭകളുടെയും നിലപാട് (https:www.aljarida.com/articles/146360661264961300/). യൂറോപ്യന്‍ രാജ്യങ്ങളിലെ ഇസ്‌ലാമിക സംഘടനകളുടെ യൂണിയനു കീഴില്‍ നിലനില്‍ക്കുന്ന ഫത്‌വ ബോര്‍ഡിന്റെയും (അല്‍ മജ്‌ലിസുല്‍ ഔറുബി ലില്‍ ഇഫ്താഇ വല്‍ ബുഹൂസ്) പഠനങ്ങളും സമാനമാണ്. (tthps://bit.ly/3fHBRwD)

രണ്ട്, കൂര്‍മബുദ്ധി, പ്രായപൂര്‍ത്തി, (അടിമയല്ലാത്ത) സ്വതന്ത്ര്യനാവുക, പുരുഷനാവുക, ധീരനും ശൂരനുമാവുക, കല്‍പ്പനാധികാരശേഷി, കാഴ്ച, കേള്‍വി, സംസാരം എന്നിങ്ങനെയുള്ള അവയവങ്ങളുടെ ശേഷി, ജനസമ്മതി, തന്റേയും ഗോത്രത്തിന്റേയും ഔന്നത്യത്തെ സംബന്ധിച്ച ജനങ്ങളുടെ അംഗീകാരം, പ്രജകളുടെ അനുസരണ സമ്മതി, പ്രജകളുടെ പിന്തുണയും വിശ്വാസവും എന്നിങ്ങനെ പൗരാണിക കാലഘട്ടത്തിലെ ജനങ്ങള്‍ ഒരു ഭരണാധികാരിയില്‍ സമ്മേളിക്കേണ്ട നിബന്ധനകളായി കരുതിപ്പോന്നിരുന്ന പല ഗുണവിശേഷങ്ങളുമുണ്ടായിരുന്നു. ജാതി-മത-രാഷ്ട്ര-ഭാഷാ വ്യത്യാസമില്ലാത്ത അന്നത്തെ ജനങ്ങള്‍ ഈ നിബന്ധനകളെ ഭരണസാരഥ്യത്തിന്റേയും നേതൃത്വത്തിന്റെയും നിര്‍ബന്ധ ഘടകങ്ങളായി മനസ്സിലാക്കി പോന്നു. അതുകൊണ്ടുതന്നെ ഈ നിബന്ധനകള്‍ ഉള്‍ക്കൊള്ളുന്ന ഭരണാധികാരികള്‍ അനുസരിക്കപ്പെട്ടു. ഭരണത്തിന്റെ വിജയം ഭരണീയരുടെ അനുസരണ മനോഭാവവും സമ്മതിയുമാണല്ലോ. ഈ നിബന്ധനകള്‍ പൂര്‍ത്തിയാക്കപ്പെടാതെ വരുമ്പോള്‍ ഭരണീയരില്‍ നിന്ന് അനുസരണക്കേടുകളും, മാനസീകവും പ്രാവര്‍ത്തികവുമായ വിയോജിപ്പുകളും സംഭവിക്കും. പ്രതികരിക്കാന്‍ തീരെ ശേഷിയില്ലാത്ത പ്രജകളാണെങ്കില്‍ വെറുപ്പോടെ ഭരണാധികാരിയെ അംഗീകരിച്ചതായി നടിക്കുന്നു. ഇത്തരം ഒരു ഭരണ നേതൃത്വത്തിന് അസന്തുഷ്ട സമൂഹത്തില്‍ അല്‍പ്പകാലം മാത്രമേ നിലനില്‍പ്പ് ഉണ്ടാകൂ എന്നത് ആ കാലഘട്ടത്തിലെ ചരിത്ര വസ്തുതയാണ്. ഉടനെ ഭരണനേതൃത്വത്തിനെതിരെ പ്രതിലോമ വിപ്ലവങ്ങളും, അധികാര വടംവലിയും ഉടലെടുക്കുക തന്നെ ചെയ്യും. പേര്‍ഷ്യക്കാര്‍ ഒരു സ്ത്രീയെ പരമാധികാരിയായി അവരോധിച്ചു എന്നറിഞ്ഞപ്പോള്‍ ‘ഒരു സ്ത്രീയെ തങ്ങളുടെ ഭരണനേതൃത്വം ഏല്‍പ്പിച്ച ഒരു ജനത വിജയിക്കില്ല’ എന്ന് പ്രവാചകന്‍ (സ) പറഞ്ഞത് ഇക്കാരണത്താലാണ് എന്ന് ശാഹ് വലിയുല്ലാഹി ദ്ദഹ്ലവി തന്റെ ‘ഹുജ്ജത്തുല്ലാഹില്‍ ബാലിഗ’: 2/131’യില്‍ വ്യക്തമാക്കുന്നുണ്ട്.

അതായത് ഒരു സ്ത്രീ പരമാധികാരം കൈയ്യാളല്‍ ആത്മീയമായ പരാജയമാണെന്നോ, മതപരമായി നിഷിദ്ധമാണെന്നോ ഒന്നും പ്രവാചകന്‍ (സ) ഉദ്ദേശിച്ചിട്ടില്ല. മറിച്ച് ആ കാലഘട്ടത്തിലെ രാഷ്ട്രീയമാണ് പ്രവാചകന്‍ (സ) ചര്‍ച്ച ചെയ്യുന്നത്. പ്രവാചകന്റെ(സ) ഈ രാഷ്ട്രീയ പ്രവചനം പുലരുകയും ചെയ്തു. പേര്‍ഷ്യക്കാരുടെ ഈ വനിതാ ഭരണാധികാരിക്കെതിരെ അവരുടെ ഉള്ളില്‍ നിന്ന് തന്നെ പ്രതിവിപ്ലവങ്ങള്‍ ഉടലെടുക്കുകയും സാമ്രാജ്യം ക്ഷയിക്കുകയും നേതൃത്വം ദുര്‍ബലമാവുകയും ചെയ്തു. മുസ്‌ലിംകള്‍ പിന്നീട് പേര്‍ഷ്യന്‍ സാമ്രാജ്യത്തിനു മേല്‍ അധികാരം കൈയ്യാളുകയും ചെയ്തു.

”ഭരണാധികാരികള്‍ കുറൈശികളില്‍ നിന്നാവണം” (മുസ്‌നദ് അഹ്‌മദ്: 3/129) എന്ന് പ്രവാചകന്‍ (സ) പറഞ്ഞതും ഇതേ രാഷ്ട്രീയ കാരണത്താലാണ്. അറബികളെ ഭരിക്കാന്‍ കുറൈശി ഗോത്രത്തിനേ കഴിയൂ എന്നതാണ് അന്നത്തെ അറേബ്യയുടെ രാഷ്ട്രീയം. അത് മതപരമായ ഒരു നിര്‍ബന്ധ ബാധ്യതയായിരുന്നെങ്കില്‍ അക്കാലം തൊട്ട് ഇക്കാലം വരെ ഇസ്‌ലാമിക ലോകത്ത് കുറൈശികളല്ലാത്ത അനേകായിരം ഭരണാധികാരികള്‍ ഉണ്ടാകുമായിരുന്നോ?! അന്ന് അറബികള്‍ക്കിടയില്‍ ഭാഷകൊണ്ടും അംഗശക്തികൊണ്ടും ധനംകൊണ്ടും മതനിഷ്ഠകൊണ്ടും ഭക്തി, ധീരത, സംസ്‌കാരം, പൈതൃകം എന്നിവ കൊണ്ടെല്ലാം പ്രബലര്‍ കുറൈശികള്‍ ആയിരുന്നു. അപ്പോള്‍ അന്നത്തെ അറബികളെ ഭരിക്കാന്‍ മറ്റൊരു ഗോത്രക്കാരെ തിരഞ്ഞെടുക്കപ്പെട്ടാല്‍ ജനസമ്മതിയും അനുസരണവും ഭരണനേതൃത്വത്തില്‍ കേന്ദ്രീകരിക്കപ്പെടില്ല. നേതൃത്വത്തെ വെല്ലുവിളിക്കുന്നവര്‍ ഉയര്‍ന്നുവരാനും ഭരണം സ്തംഭിക്കാനുമാണ് സാധ്യത. ഈ രാഷ്ട്രീയമാണ് ഹദീഥിലെ ചര്‍ച്ച.

മൂന്ന്, ഉപര്യുക്ത ഹദീഥ് കേവലം അതിന്റെ പ്രത്യേകമായ പശ്ചാതലത്തില്‍ നിന്നുകൊണ്ടാണ് മനസ്സിലാക്കേണ്ടത് എന്നു നിഷ്‌കര്‍ഷിച്ച പണ്ഡിത വീക്ഷണങ്ങളെയാണ് നാം ഇതുവരെ വിലയിരുത്തിയതെങ്കില്‍ ഇനി നമുക്കു പരിശോധിക്കുവാനുള്ളത്, പ്രസ്തുത ഹദീഥിലെ ആശയത്തെ സാമാന്യവല്‍ക്കരിച്ചുകൊണ്ട് മനസ്സിലാക്കണമെന്നു നിഷ്‌കര്‍ഷിച്ചാല്‍ തന്നെ ഹദീഥ് സ്ത്രീ വിരുദ്ധമാകുന്നില്ല എന്ന വസ്തുതയെയാണ്. ‘തങ്ങളുടെ ഭരണനേതൃത്വം ഒരു സ്ത്രീയെ ഏല്‍പ്പിച്ച ജനത വിജയിക്കുകയില്ല’ എന്ന ഹദീഥ് പേര്‍ഷ്യക്കാരുടെ കാര്യത്തില്‍ പറയപ്പെട്ടതാണെങ്കിലും അതിലെ ആശയം സാമാന്യവല്‍ക്കരിച്ചു മനസ്സിലാക്കേണ്ടതുണ്ട് എന്നു നിഷ്‌കര്‍ഷിച്ചാല്‍ തന്നെ ഹദീഥില്‍ നിരുത്സാഹപ്പെടുത്തപ്പെട്ട പെണ്ണധികാരം ‘പരമാധികാര’ത്തിന്റെ വിഷയത്തില്‍ മാത്രമാണ്. അഥവാ ‘സര്‍വ്വാധിപത്യം’ ഒരു പെണ്ണില്‍ അര്‍പ്പിതമാകുന്നതിനെയാണ് ഹദീഥ് പ്രശ്നവല്‍കരിച്ചിരിക്കുന്നതെന്നര്‍ഥം.

ഇമാം ഇബ്‌നു ഹസം പറഞ്ഞു: ‘സ്ത്രീ ന്യായാധിപത്യം ഏറ്റെടുക്കല്‍ അനുവദനീയമാണ്. അതാണ് ഇമാം അബൂഹനീഫയുടേയും അഭിപ്രായം. ഖലീഫ ഉമര്‍ ബിന്‍ ഖത്താബ് തന്റെ ഗോത്രത്തില്‍പ്പെട്ട ശിഫാഅ് എന്ന സ്ത്രീയെ മാര്‍ക്കറ്റിലെ (നിയമപാലനവുമായി ബന്ധപ്പെട്ട ഉത്തരവാദിത്തം) ഏല്‍പ്പിച്ചതായി നിവേദനം ചെയ്യപ്പെട്ടിട്ടുണ്ടല്ലോ. ‘തങ്ങളുടെ കാര്യം സ്ത്രീയെ ഏല്‍പ്പിച്ച ഒരു സമൂഹം വിജയിക്കില്ല’ എന്നു പ്രവാചകന്‍ (സ) പറഞ്ഞിട്ടുണ്ടല്ലോ എന്ന് ആരെങ്കിലും ചോദിച്ചാല്‍ നാം ഇപ്രകാരം മറുപടി നല്‍കും: പ്രവാചകന്‍ (സ) അതുകൊണ്ടുദ്ദേശിച്ചത് ഖിലാഫത്ത് ഏല്‍പ്പിക്കുന്നതിനെ സംബന്ധിച്ചാണ്. കാരണം, ഭര്‍ത്താവിന്റെ ധനം, ഭവനം, സ്വകുടുംബം തുടങ്ങി മറ്റു പല അധികാര അവകാശങ്ങളും പ്രവാചകന്‍ (സ) സ്ത്രീക്കു വക വെച്ചുകൊടുത്തിട്ടുണ്ട് എന്നതാണ് അതിനുള്ള തെളിവ്. സ്ത്രീക്ക് വക്കീലയും (കാര്യസ്ഥത, പ്രതിനിധി) വസ്വിയ്യയും (ഒരാളുടെ മരണാനന്തരം അയാളുടെ കാര്യങ്ങളുടെ കൈകാര്യകര്‍ത്താവായി അയാള്‍ തന്നെ – ജീവിത കാലത്ത് നിശ്ചയിക്കുന്ന പ്രതിനിധി) ആകല്‍ അനുവദനീയമാണെന്ന് മാലികി മദ്ഹബുകാര്‍ അഭിപ്രായപ്പെടുന്നു. പരമാധികാരം അല്ലാത്ത (അതിനു താഴെ വരുന്ന) പല കാര്യങ്ങളുടേയും അധികാരം സ്ത്രീ ഏറ്റെടുക്കുന്നത് ഇസ്‌ലാമിക പ്രമാണങ്ങള്‍ തടയുന്നില്ല…” അല്ലാഹു നിങ്ങളോടിതാ കല്‍പ്പിക്കുന്നു: നിങ്ങളെ വിശ്വസിച്ചേല്‍പ്പിച്ച വസ്തുക്കള്‍ അവയുടെ അവകാശികളെ തിരിച്ചേല്‍പ്പിക്കുക. ജനങ്ങള്‍ക്കിടയില്‍ തീര്‍പ്പ് കല്‍പ്പിക്കുകയാണെങ്കില്‍ നീതിപൂര്‍വ്വം വിധി നടത്തുക” (ക്വുര്‍ആന്‍ 4:58) ക്വുര്‍ആനിലെ ഈ വചനം അഭിമുഖീകരിക്കുന്നത് പുരുഷനേയും സ്ത്രീയേയും സ്വതന്ത്രനേയും അടിമയേയും ഉള്‍കൊള്ളുന്ന നിലക്ക് പൊതുവായിട്ടാണ്. മതം മൊത്തത്തില്‍ സ്ത്രീക്കും പുരുഷനും ഒന്നാണ്; സ്ത്രീയേയും പുരുഷനേയും അവരുടെ (പ്രകൃതി സഹജമായ വ്യത്യാസങ്ങളെ പരിഗണിച്ച്) വ്യത്യാസമുള്ള വിധികള്‍ വല്ലതും പ്രമാണങ്ങളില്‍ വന്നിട്ടുള്ള ചില വിഷയങ്ങളൊഴിച്ച്” (അല്‍ മുഹല്ലാ: 9/429)

ഇമാം ത്വബ്‌രി പറഞ്ഞു: ”സ്ത്രീയെ ഏതു കാര്യത്തിലും, നിരുപാധികം വിധി കര്‍തൃത്വം/ന്യായാധിപത്യം ഏല്‍പ്പിക്കാന്‍ അനുവദനീയമാണ്” (ബിദായത്തുല്‍ മുജ്തഹിദ്: 3/445)

ഇമാം ഹസുനല്‍ ബസ്വരി (ജനനം: ഹിജ്റ 21), മാലികീ മദ്ഹബിലെ ഇമാം ഇബ്നുല്‍ ക്വാസിം (ജനനം: ഹ്ജ്റ 132) എന്നിവരും സ്ത്രീകളെ വിധി കര്‍തൃത്വം/ന്യായാധിപത്യം ഏല്‍പ്പിക്കാന്‍ അനുവദനീയമാണ് എന്ന അഭിപ്രായപ്പെടുന്നു (മവാഹിബുല്‍ ജലീല്‍: ഹത്താബ് അര്‍റുഅ്യാനി: 8:65)

ഇമാം ഇബ്നു ജരീര്‍ (ജനനം: ഹിജ്റ 224) സ്ത്രീകള്‍ക്ക് നിരുപാധികം ന്യായാധിപത്യം നല്‍കപ്പെടുമെന്ന് അഭിപ്രായപ്പെടുന്നു. സ്ത്രീകളുടെ ഫത്‌വകള്‍ (മത വിധികള്‍) ഏതു വിഷയത്തിലും സ്വീകരിക്കപ്പെടുമെന്നതാണ് അവരുടെ ന്യായാധിപത്യവും നിരുപാധികം സ്വീകരിക്കാന്‍ അദ്ദേഹം ന്യായമായി കാണുന്നത് (അല്‍ മുഗ്‌നി 9:39, അദബുല്‍ ക്വാളി: 1:625, റൗളത്തുല്‍ ക്വുള്വാത്ത്: 1:53, ഫത്ഹുല്‍ കദീര്‍: 5:485, ഫത്ഹുല്‍ബാരി: 13/147. ഉദ്ദരണം: അല്‍ മൗസൂഅത്തുല്‍ ഫിക്ഹിയ്യ: 33:294)

”യഹ്‌യബ്നു അബീ സുലൈം പറഞ്ഞു: പ്രവാചകാനുചരയായ (സ്വഹാബി വനിത) സംറാഅ് ബിന്‍ത്ത് നുഹൈക് പരുക്കന്‍ മുഖമക്കനയും പടച്ചട്ടയും ധരിച്ച്, കയ്യില്‍ ചാട്ടവാറേന്തി ജനങ്ങളില്‍ മര്യാദ നടപ്പാക്കിയും നന്മ കല്‍പ്പിച്ചും തിന്മ വിരോധിച്ചും നടക്കുന്നത് ഞാന്‍ കാണുകയുണ്ടായി” (അല്‍ മുഅ്ജമുല്‍ കബീര്‍: ത്വബ്റാനി: 24/311, നമ്പര്‍: 785, നിവേദക പരമ്പരയിലെ എല്ലാവരും വിശ്വസ്തരാണെന്ന് ഇമാം ഹൈഥമി പറയുന്നു (9/264). ശൈഖ് അല്‍ബാനി പരമ്പര നല്ലതെന്ന് അഭിപ്രായപ്പെട്ടിരിക്കുന്നു (24/311:785). (ഉമര്‍ (റ) ശിഫാഅ് ബിന്‍ത് അബ്ദില്ല എന്ന സ്ത്രീയെ അങ്ങാടിയിലെ നിയമപാലന ചുമതല ഏല്‍പ്പിച്ചിരുന്നു എന്ന നിവേദനത്തില്‍ ദുര്‍ബലത കാണുന്നവര്‍ക്ക് സംറാഅ് ബിന്‍ത്ത് നുഹൈകിന്നെ പറ്റിയുള്ള പ്രബലമായ ഈ നിവേദനം സ്വീകരിക്കാവുന്നതാണ്)

‘തങ്ങളുടെ ഭരണനേതൃത്വം ഒരു സ്ത്രീയെ ഏല്‍പ്പിച്ച ഒരു ജനത വിജിയിക്കില്ല’ എന്ന ഹദീഥിനെ പ്രത്യേക പശ്ചാത്തലത്തില്‍ ബന്ധിപ്പിക്കരുതെന്നും സാമാന്യര്‍ത്ഥത്തില്‍ തന്നെ അതിനെ മനസ്സിലാക്കിയാലും അതുകൊണ്ടുദ്ദേശിക്കപ്പെടുന്നത്; സ്ത്രീക്ക് – പൗരാണിക രാഷ്ട്ര വ്യവസ്ഥയില്‍ നില നിന്നിരുന്ന സ്വഭാവത്തിലുള്ള – സര്‍വ്വാധിപത്യം നല്‍കുന്നതിനെപ്പറ്റിയാണ്. അല്ലാതെ അധികാരത്തില്‍ നിന്നും ഒരു പങ്കും സ്ത്രീക്ക് നല്‍കരുതെന്ന നിലപാടല്ല. അതുകൊണ്ട് തന്നെ ആധുനിക ജനാധിപത്യ വ്യവസ്ഥകളുടെ കീഴില്‍ സ്ത്രീയെ മന്ത്രിസ്ഥാനം, അഡ്മിനിസ്ട്രേഷന്‍, ജനപ്രതിനിധി തുടങ്ങിയ അധികാരങ്ങള്‍ ഏല്‍പ്പിക്കുന്നത് ഹദീഥിന്റെ പ്രതലം പറ്റികൊണ്ട് എതിര്‍ക്കപ്പെടേണ്ട കാര്യമല്ല. കാരണം ആധുനിക ജനാധിപത്യ വ്യവസ്ഥകളുടെ കീഴില്‍ സ്ത്രീയെ മന്ത്രിസ്ഥാനം, അഡ്മിനിസ്ട്രേഷന്‍, ജനപ്രതിനിധി തുടങ്ങിയ അധികാരങ്ങള്‍ ഏല്‍പ്പിക്കുമ്പോള്‍ ഭരണഭാരം പൂര്‍ണ്ണമായും സ്ത്രീയുടെ തലയില്‍ ചുമത്തുന്നില്ല. മറിച്ച് കൂട്ടുത്തരവാദിത്വത്തിലെ ഒരു പങ്കു മാത്രമാണ് അവിടെ സ്ത്രീക്ക് വഹിക്കേണ്ടി വരുന്നത്.

ബ്രിട്ടനിലെ താച്ചറുടേയും ഇന്ത്യയിലെ ഇന്ദിരാഗാന്ധിയുടേയും അധിനിവേശ ഫലസ്തീനിലെ ഗോള്‍ഡാമീറിന്റേയും ഭരണരീതി സമൂഹത്തില്‍ ഒരു സ്ത്രീയുടെമാത്രം ഭരണമല്ല. മറിച്ച് ഭരിക്കുന്ന ഗ്രൂപ്പുകളുടേയും വ്യവസ്ഥയുടേയും ഭരണമാണ്. തലപ്പത്ത് സ്ത്രീയാണെന്ന് മാത്രം. ഭരണനിര്‍വ്വഹണം നടത്തുന്നത് കൂട്ടുത്തരവാദിത്വമുള്ള മന്ത്രിസഭകളാണ്. അവിടെ സ്ത്രീ സര്‍വ്വ ഭാരവും ഉത്തരവാദിത്തവും മുഴുവന്‍ പേറേണ്ടിവരുന്ന ഭരണാധികാരിയല്ല. പാര്‍ട്ടിയിലേയോ വ്യവസ്ഥയിലേയോ ഭൂരിപക്ഷത്തിന്റെ പിന്തുണയും ആനുകൂല്യം സഹായവും അവള്‍ക്ക് ലഭ്യമാകുന്നുണ്ട്. വിമര്‍ശനവിധേയമായ ഹദീഥിനെ സാമാന്യ അർത്ഥത്തില്‍ തന്നെ എടുക്കണമെന്ന നിലപാട് സ്വീകരിച്ച പണ്ഡിതന്മാരില്‍ പലരും ഇതിനപ്പുറത്തേക്ക് ഹദീഥിനെ വ്യാഖ്യാനിച്ചിട്ടില്ല. ഈ നിലപാടാകട്ടെ ആധുനിക ജനാധിപത്യ വ്യവസ്ഥകളുടെ നിലപാടുകള്‍ക്ക് വിരുദ്ധവുമല്ല. കാരണം സര്‍വ്വാധിപത്യമെന്നത് ജനാധിപത്യവിരുദ്ധമായ ഒരു നിലപാടാണല്ലൊ. ഒരു ജനാധിപത്യവ്യവസ്ഥയുടെ ഏതു പ്രതലത്തില്‍ നിന്നുകൊണ്ടും, പ്രസ്തുത ഹദീഥിനെ പ്രശ്നവല്‍ക്കരിച്ച് പെണ്‍വിരുദ്ധതയുടെ ആലയില്‍ കൊണ്ടുപോയി കെട്ടാന്‍ സാധ്യമല്ല; വിശിഷ്യ ജനാധിപത്യവാദികള്‍ക്ക്.

‘തങ്ങളുടെ ഭരണനേതൃത്വം ഒരു സ്ത്രീയെ ഏല്‍പ്പിച്ച ഒരു ജനത വിജയിക്കില്ല’ എന്ന ഹദീഥിന് നല്‍കപ്പെട്ട രണ്ട് രൂപത്തിലുള്ള വിശദീകരണങ്ങളില്‍ ഏതു സ്വീകരിച്ചാലും ശരി (പ്രത്യേക പശ്ചാത്തലത്തില്‍ ബന്ധിതമാണ് ഹദീഥ്; അല്ല അതിനെ സാമാന്യ അര്‍ത്ഥത്തില്‍ എടുക്കണം) സ്ത്രീയുടെ അധികാരത്തിലുള്ള എല്ലാ വിധ പങ്കിനേയും ഹദീഥ് എതിര്‍ക്കുന്നില്ല എന്നത് സ്പഷ്ടമാണ്.

പൗരാണിക രാഷ്ട്ര – രാഷ്ട്രീയ വ്യവസ്ഥയില്‍ നിലനിന്നിരുന്ന – സാധാരണ ഗതിയില്‍ പുരുഷ സമൂഹത്താല്‍ കയ്യാളപ്പെട്ടിരുന്ന – പരമാധികാരത്തിന്റെ സ്വഭാവവും പ്രകൃതിയും ചരിത്രത്തിലൂടെ കണ്ണോടിക്കുമ്പോള്‍, എന്തുകൊണ്ട് സ്ത്രീത്വത്തിന് അത് യോജിക്കില്ലെന്നുള്ള അഭിപ്രായം രൂപീകൃതമായി എന്ന് മനസ്സിലാക്കാന്‍ എളുപ്പമായിരിക്കും. അതിനായി അക്കാലഘട്ടത്തിലെ പേര്‍ഷ്യന്‍ റോമന്‍ സാമ്രാജ്യങ്ങളിലെ ‘പരമാധികാരവും’ അതിനു കീഴില്‍ നടമാടിയിരുന്ന രക്താഭിഷിക്തമായ സാമൂഹിക-രാഷ്ട്രീയ അന്തരീക്ഷവും തന്നെ പരിശോധനക്കെടക്കാം:

ഇസ്‌ലാമിന്റെ ആവിര്‍ഭാവത്തിനു മുമ്പ് തന്നെ പേര്‍ഷ്യന്‍ റോമന്‍ സാമ്രാജ്യങ്ങള്‍ക്കിടയിലും പാഴ്‌സികള്‍, ക്രിസ്ത്യാനികള്‍, ജൂതന്മാര്‍ എന്നിവര്‍ക്കിടയിലും നടമാടിയിരുന്ന ഭരണ വടം വലികളും അധികാരമുപയോഗിച്ച് നടപ്പാക്കിയിരുന്ന മത ധ്വംസനങ്ങളും അന്നത്തെ ലോക വ്യവസ്ഥയിലെ അസഹിഷ്ണുതയുടെ ആഴം വ്യക്തമാക്കി തരുന്നുണ്ട്. ലബ്‌നാനിലെ പ്രസിദ്ധ സാഹിത്യകാരനും ചരിത്രകാരനുമായ ജോര്‍ജി സൈദാന്റെ ‘താരീഖുത്തമദ്ദുനുല്‍ ഇസ്‌ലാമി’ (تاريخ التمدن الإسلامي) എന്ന വിശ്രുതമായ ഗ്രന്ഥത്തില്‍ നിന്ന് ചില ചരിത്ര സാക്ഷ്യങ്ങള്‍ ചുവടെ ഉദ്ധരിക്കാം. (ജോര്‍ജി സൈദാന്‍ മുസ്‌ലിമായിരുന്നില്ല, ക്രിസ്ത്യാനിയായിരുന്നു എന്ന് സാന്ദര്‍ഭികമായി സൂചിപ്പിക്കട്ടെ.)

പേര്‍ഷ്യയും റോമും തമ്മിലുളള ശത്രുത പൗരാണികമാണ്. ഒരു പക്ഷെ ബി.സി അഞ്ചാം നൂറ്റാണ്ടിനുമപ്പുറം അതിന്റെ വേരുകള്‍ എത്തി നില്‍ക്കുന്നുണ്ടാകാം. ലോകത്തെ അടക്കി ഭരിക്കാനുള്ള ഇരു സാമ്രാജ്യങ്ങളുടെയും അത്യാഗ്രഹമായിരുന്നു ഈ ശത്രുതയുടെ അടിത്തറ. നൂറ്റാണ്ടുകളോളം നീണ്ടു നിന്ന ഈ അധികാര വടം വലി ഇസ്‌ലാമിന്റെ ആവിര്‍ഭാവ കാലഘട്ടത്തിലും തുടര്‍ക്കഥയായിരുന്നു. പേര്‍ഷ്യന്‍ സാമ്രാജ്യത്തിന്റെ ആധിപത്യം ഇസ്ര അനൂഷര്‍വാന്‍ ചക്രവര്‍ത്തിയില്‍ എത്തിച്ചേര്‍ന്നപ്പോള്‍ റോമന്‍ സാമ്രാജ്യത്തെ അല്‍പാല്‍പ്പമായി പിടിച്ചടക്കാന്‍ അദ്ദേഹം സൈന്യ വ്യൂഹത്തെ വിന്യസിച്ചു. സിറിയ പിടിച്ചടക്കുകയും അന്താഖിയ ചുട്ടു നശിപ്പിക്കുകയും ഏഷ്യാ മൈനര്‍ കൊള്ളയടിക്കുകയും ചെയ്തു. അന്നത്തെ റോമന്‍ ചക്രവര്‍ത്തിയായിരുന്ന ജസ്റ്റീനിയന്‍ ചക്രവര്‍ത്തിയും വിട്ടുവീഴ്ചയ്ക്ക് തയ്യാറല്ലായിരുന്നു. ക്രിസ്താബ്ദം 541 മുതല്‍ 561 വരെ ഇരുപതു വര്‍ഷം ഇരു രാഷ്ട്രങ്ങളും യുദ്ധത്തില്‍ മുഴുകി.

പര്‍വേസ് ചക്രവര്‍ത്തിയുടെ കാലഘട്ടത്തില്‍ തന്റെ സുഹൃത്ത് മോറിസിന്റെ കൊലപാതകത്തിന് പ്രതികാരമെന്ന പേരില്‍ റോമന്‍ സാമ്രാജ്യത്തെ പര്‍വേസ് ചക്രവര്‍ത്തി ആക്രമിക്കുകയുണ്ടായി. ക്രിസ്താബ്ദം 614 ല്‍ സിറിയ പിടിച്ചടക്കി… ജൂതന്മാരുടെ അകമഴിഞ്ഞ സഹായത്താല്‍ ബൈസാന്റിയന്‍ പടയെ പര്‍വേസ് ചക്രവര്‍ത്തി ഒന്നൊന്നായി കീഴടക്കി. ഈജിപ്ത്, അന്താഖിയ, ദമാസ്‌ക്കസ്, ബൈത്തുല്‍ മുഖദസ് തുടങ്ങിയവ പിടിച്ചടക്കി. ജറുസലേമിലെ ബൈത്തുല്‍ മുഖദസ് കൊള്ളയടിക്കാനും ക്രിസ്ത്യന്‍ പള്ളികളും പുണ്യപുരുഷന്മാരുടെ കല്ലറകള്‍ തീയിടാനും അവിടെയുള്ള വിലമതിക്കാനാകാത്ത സ്വത്തുക്കള്‍ പിടിച്ചു പറിക്കാനും തന്റെ സൈന്യത്തിന് പര്‍വേസ് ചക്രവര്‍ത്തി അനുവാദം നല്‍കി. സിറിയ വരെ ഈ കൊലയും കൊള്ളയും തുടര്‍ന്നു. 90000 ക്രിസ്ത്യാനികളെ സൈന്യം കൊന്നൊടുക്കി… ഇതെല്ലാം കണ്ടിട്ടും കാണാത്ത മട്ടില്‍ ഭീരുവായി (അന്നത്തെ) ഹെറാക്ലിയസ് ചക്രവര്‍ത്തി കൊട്ടാരത്തില്‍ തന്നെ ഇരുന്നു; ക്രിസ്താബ്ദം 632 ല്‍ ഏഷ്യാ മൈനറില്‍ വെച്ച് കൊല്ലപ്പെടുന്നത് വരെ… (ഈ വര്‍ഷമാണ് ഇങ്ങു അറേബ്യയില്‍, മുസ്‌ലിംകള്‍ മക്കയില്‍ നിന്ന് പലായനം ചെയ്യുന്നത്) (താരീഖുത്തമദ്ദുനുല്‍ ഇസ്‌ലാമി: ജോര്‍ജി സൈദാന്‍: 1: 43-48) ക്രിസ്ത്യന്‍ റോമും ജൂതന്മാരും തമ്മിലുള്ള അതിര്‍ത്തി യുദ്ധങ്ങളും കലാപങ്ങളും അക്കാലഘട്ടത്തിന്റെ പ്രധാന ഇതിവൃത്തം ആയിരുന്നു.

പ്രൊഫ. പി. എസ്. വേലായുധന്‍ എഴുതുന്നു: ”കോണ്‍സ്റ്റന്റൈന്‍ ചക്രവര്‍ത്തി ചരമം പ്രാപിച്ചപ്പോള്‍ രണ്ടു പുത്രന്മാരും ചക്രവര്‍ത്തിമാരായി. അവരുടെ ചാര്‍ച്ചക്കാരനായ ജൂലിയന്‍ ചക്രവര്‍ത്തിയായി ഏ.ഡി. 360 മുതല്‍ 363 വരെ ഭരിച്ച അദ്ദേഹം ഒരു ക്രൈസ്തവ വിരോധിയായിരുന്നു. അദ്ദേഹത്തില്‍ നിന്ന് ക്രിസ്ത്യാനികള്‍ക്ക് പീഢനം സഹിക്കേണ്ടി വന്നു. അദ്ദേഹത്തിനു ശേഷം വന്ന ചക്രവര്‍ത്തിമാരില്‍ ഏറ്റവും പ്രമാണി തിയോഡോഷ്യസ് ഒന്നാമനായിരുന്നു. അദ്ദേഹം എ.ഡി. 378 മുതല്‍ 395 വരെ ഭരിച്ചു. അദ്ദേഹത്തിന്റെ ഭരണകാലത്താണ് ക്രിസ്തുമതം ഔദ്യോഗികമായി അംഗീകരിക്കപ്പെട്ടത്. തന്റെ സമസ്ത പ്രജകളും ക്രിസ്തുമതം സ്വീകരിക്കണമെന്നുള്ള രാജകീയശാസനം എ.ഡി. 380 ല്‍ അദ്ദേഹം പുറപ്പെടുവിച്ചു. പിന്നീട് ക്രൈസ്തവേതര്‍ക്ക് മതാനുഷ്ഠാനത്തിനുള്ള സര്‍വാവകാശങ്ങളും നിഷേധിച്ചുകൊണ്ടും അവര്‍ക്ക് ഗവണ്‍മെന്റുദ്യോഗങ്ങളില്‍ പ്രവേശനം നിരസിച്ചു കൊണ്ടും ഉത്തരവു പുറപ്പെടുവിച്ചു. അങ്ങനെ അദ്ദേഹം ക്രിസ്തുമതത്തെ രാഷ്ട്ര മതമായി സ്വീകരിച്ചു.” (ലോക ചരിത്രം: ഒന്നാം ഭാഗം: പ്രൊഫ. പി. എസ്. വേലായുധന്‍: പേജ്: 174: കേരള ഭാഷാ ഇന്‍സ്റ്റിറ്റ്യൂട്ട്)

ഹെറാക്ലിയസിന്റെ കാലഘട്ടത്തില്‍ അന്‍താഖിയായില്‍ നടന്നിരുന്ന കൂട്ടക്കുരുതികള്‍ ഭയാനകമായിരുന്നു. ജൂതന്മാര്‍ വിപ്ലവത്തിലൂടെ ക്രിസ്ത്യാനികളെ വധിക്കുകയും ശരീരം കഷ്ണങ്ങള്‍ ആക്കുകയും അതി നിഷ്ഠൂരമായ വൈകൃത-ക്രൂരതകള്‍ ഒരു വശത്ത് നടപ്പാക്കിയപ്പോള്‍ മറുവശത്ത് ഹെറാക്ലിയസ് ചക്രവര്‍ത്തി (ഹെറാക്ലിയസ് എന്നത് റോമന്‍ ചക്രവര്‍ത്തിമാരുടെ സ്ഥാനപേരാണ്) എണ്ണമറ്റ ജൂതന്മാരെ കൊന്നൊടുക്കിക്കൊണ്ടിരുന്നു. നാട്ടില്‍ കാണപ്പെടുന്ന ജൂതന്മാരെയെല്ലാം വധിക്കുക എന്ന ഉത്തരവിറക്കുകയും ചെയ്തു. ഫലസ്തീനിലും പരിസര പ്രദേശങ്ങളിലും ഇത്തരം സംഘട്ടനങ്ങള്‍ പതിവായിരുന്നു… അന്നത്തെ റോമന്‍ ചക്രവര്‍ത്തിയുടെ സഹോദരന്‍ ആയിരുന്ന തിയഡോര്‍ ഫലസ്തീനിലെ ഭരണ കാര്യങ്ങള്‍ക്ക് നിയോഗിതനായപ്പോള്‍ നാട്ടിലെ സര്‍വ്വ ജൂതന്മാരെയും തിരഞ്ഞു പിടിച്ച് വധിക്കാന്‍ തുടങ്ങി. ജൂതന്മാര്‍ ആകട്ടെ തങ്ങളുടെ സഹായികളായ പേര്‍ഷ്യക്കാരില്‍ നിന്നും 80000 ക്രിസ്ത്യന്‍ ബന്ദികളെ വിലയ്ക്ക് വാങ്ങി അറുത്തു കൊന്നു… (താരീഖുത്തമദ്ദുനുല്‍ ഇസ്‌ലാമി: ജോര്‍ജി സൈദാന്‍: 1: 43-48)

ഈ വിധ്വംസനമയമായ രാഷ്ട്രവ്യവസ്ഥകളിലും ക്രൂരമായ രാഷ്ട്രീയവടംവലികളിലും സ്ത്രീയെ തള്ളിയിടുന്നതിനെ ‘സ്ത്രീ സ്വാതന്ത്ര്യ’മെന്നോ ‘സ്ത്രീ വിമോചന’മെന്നോ പറയാന്‍ വിവേകമതികളായ ആര്‍ക്കെങ്കിലും സാധിക്കുമോ ? സ്‌നേഹം, കാരുണ്യം, ആര്‍ദ്രത എന്നീ മനുഷ്യ മൂല്യങ്ങളുടെ പുനരുദ്ധാരകരും വാഹകരുമായി മാനവ ചരിത്രത്തിലുടനീളം നിലകൊണ്ടവരും ഇന്നും നിലകൊള്ളുന്നവരുമാണ് സ്ത്രീകള്‍. ലോകം സുബുദ്ധിയും സമാധാനവും കൈവിട്ട് ഊഷരമായി പരിണമിച്ച സന്ധികളില്‍, മനുഷ്യ മനസ്സുകളിലെ വൈകാരിക മണ്ഡലത്തില്‍ സഹാനുഭൂതിയുടേയും ആര്‍ദ്രതയുടേയും വിത്തുകളായി വര്‍ത്തിച്ച സ്ത്രീത്വത്തിന്റെ മനോഹാരിതയും സ്വത്വസൗന്ദര്യവും തിരിച്ചറിയാത്തവര്‍ക്കേ അവളെ ചോര കൊണ്ട് അലങ്കരിക്കാന്‍ തോന്നൂ.

വിഷയത്തിലേക്ക് തിരിച്ചു വരാം. സ്ത്രീയുടെ അധികാരത്തിലുള്ള എല്ലാ വിധ പങ്കിനേയും ഹദീഥ് എതിര്‍ക്കുന്നില്ല, ഏറിവന്നാല്‍ പൗരാണിക രാഷ്ട്ര വ്യവസ്ഥയിലെ അവളുടെ സര്‍വ്വാധിപത്യത്തെ മാത്രമാണ് അത് ചോദ്യം ചെയ്യുന്നത് എന്ന് സൂചിപ്പിച്ചുവല്ലൊ. അതുകൊണ്ടു തന്നെയാണ് ചരിത്രത്തിന്റെ വ്യത്യസ്ഥ ദിക്കുകളില്‍, ഇസ്‌ലാമിക പ്രവിശ്യകളില്‍, വൈവിധ്യമാര്‍ന്ന ചിന്താധാരകളില്‍ സ്ത്രീ ഭരണങ്ങളുടെ എണ്ണിയാല്‍ തീരാത്തത്ര ചരിത്രങ്ങള്‍ രേഖപ്പെടുത്തപ്പെട്ടത്. സ്ത്രീക്ക് അധികാരത്തില്‍ നിന്നും ഒരു പങ്കും നല്‍കാന്‍ ഇസ്‌ലാം അനുവദിച്ചിട്ടില്ലായിരുന്നെങ്കില്‍ അത്തരം ചരിത്രങ്ങള്‍ മുസ്‌ലിം നാടുകളില്‍ നിന്നും നമുക്ക് ഒരിക്കലും വായിച്ചെടുക്കാനാവില്ല. ഈജിപ്തിലെ ഫത്‌വ ബോര്‍ഡ്, ദാറുല്‍ ഇഫ്താഉല്‍ മിസ്‌രിയ്യയുടെ ഫത്‌വയില്‍ നിന്നും ആ ചരിത്ര നിമിഷങ്ങളെ പറ്റിയുമുള്ള സ്മൃതികള്‍ നമുക്കു വായിക്കുക. ”വ്യത്യസ്ഥങ്ങളായ കാലഘട്ടത്തില്‍ ഇസ്‌ലാമിക രാജ്യങ്ങളുടെ വ്യത്യസ്ഥ ഭാഗങ്ങളില്‍ സ്ത്രീകള്‍ ഭരിച്ചിട്ടുണ്ട്. അവര്‍ വ്യത്യസ്ഥങ്ങളായ നാമദേയത്തിലാണ് അറിയപ്പെട്ടത്. സുല്‍ത്താന, മലിക, ഹുര്‍റ, ഖാത്തൂന്‍ എന്നിങ്ങനെയുള്ള പേരുകളിലാണ് അവര്‍ അറിയപ്പെട്ടിരുന്നത്. ചരിത്രത്തിന്റെ വ്യത്യസ്ഥ ദിക്കുകളില്‍ ഇസ്‌ലാമിക രാജ്യങ്ങളില്‍ പല നാടുകളും അമ്പതിലേറെ സ്തീകള്‍ ഭരിച്ചതായി ഇസ്‌ലാമിക ചരിത്രം നമുക്ക് വ്യക്തമാക്കിതരുന്നുണ്ട്. ഇബ്‌നു കസീറിന്റെ ‘അല്‍ബിദായഃ വന്നിഹായഃ’യിലും ഇബ്നുല്‍ ജൗസിയുടെ ‘അല്‍മുന്‍തള്വിം’ എന്നു പറയുന്ന ഗ്രന്ഥത്തിലും സുമല്‍ അല്‍ കഹ്റുമാന എന്നു പറയുന്ന ഒരു സ്ത്രീ ക്വാളിയായി (ജഡ്ജിയായി) പ്രവര്‍ത്തിച്ചതായി ചരിത്രം നമുക്ക് വിവരിച്ചുതരുന്നുണ്ട്. അവരുടെ സദസ്സില്‍ ജഡ്ജിമാരും കര്‍മ്മശാസ്ത്രപണ്ഡിതന്മാരും മഹാന്മാരുമെല്ലാം ഹാജരായിരുന്നു. ഹിജ്റ 317ലാണ് അവര്‍ മരണപ്പെടുന്നത്. ഭരണം നടത്തിയിരുന്ന സ്ത്രീകളില്‍ ചിലര്‍ ക്രിമിനല്‍ കേസുകളില്‍ വിധി പറയുന്നവരായും ഉണ്ടായിരുന്നു. സുല്‍ത്താന തുര്‍ക്കാന്‍ ഖാത്തൂന്‍ ഒരുദാഹരണമാണ്. അവരുടെ അടുക്കല്‍ ക്രിമിനല്‍ കേസുകള്‍ വന്നിട്ടുണ്ടെങ്കില്‍ അവര്‍ നീതിപൂര്‍വവും നന്മയോടും കൂടി വിധി നല്‍കുമായിരുന്നു. ജിഹാദിലും യുദ്ധങ്ങളിലുമെല്ലാം സ്ത്രീകള്‍ പങ്കാളികളാകുന്നതിനെ നബി (സ) അംഗീകരിച്ചിട്ടുണ്ട് എന്നതാണ് മറ്റൊരു കാര്യം. സ്ത്രീകള്‍ പ്രവാചകന്റെ ഒപ്പം യുദ്ധം ചെയ്തിട്ടുണ്ട്. ഉമ്മുസുലൈം, ഉമ്മുഹറാം, ബിന്‍ത് മില്‍ഹാന്‍, ഉമ്മുല്‍ ഹാരിസ അല്‍ അന്‍സാരി, റബീഅ ബിന്‍ത് മുഅവ്വദ്ബ്നു അഫ്റാഅ്, ഉമ്മുസിനാന്‍ അല്‍ അസ്‌ലമിയ്യ, ഹംന ബിന്‍ത് ജഹ്ശ്, ഉമ്മു സിയാദ് അല്‍ അശ്ജഈയ്യ പോലെയുള്ള സ്ത്രീകള്‍ അതിനുദാഹരണമാണ്. അതുപോലെതന്നെ ഇസ്‌ലാമിക ചരിത്രത്തിന്റെ വിവിധ ഘട്ടങ്ങളില്‍ ആയിരകണക്കിന് നിപുണകളായ പണ്ഡിതകളും, അതുപോലെതന്നെ ഇസ്‌ലാമികവും അറബിയുമായും ബന്ധപ്പെട്ട വിജ്ഞാനശാസ്ത്രങ്ങളിലുമെല്ലാം കഴിവുറ്റ പ്രതിഭകളും ഉണ്ടായിട്ടുണ്ട് ‘അല്‍ ഇസ്വാബഃ ഫീതമീസിസ്സ്വഹാബ’ എന്ന ഇബ്നു ഹജര്‍ അല്‍ അസ്‌ക്വലാനിയുടെ ഗ്രന്ഥത്തില്‍, അദ്ദേഹം ഇത്തരത്തില്‍ ഉള്ള ആയിരത്തി അഞ്ഞൂറ്റി നാല്‍പത്തിമൂന്ന് സ്ത്രീകളുടെ ചരിത്രം ഉള്‍പ്പെടുത്തിയിട്ടുണ്ട്. അവരുടെ കൂട്ടത്തില്‍ കര്‍മ്മശാസ്ത്ര പണ്ഡിതകളും, ഹദീഥ് പണ്ഡിതകളും, സാഹിത്യകാരികളും, വിധിയും നിയമനിര്‍മ്മാണവും നീതിപാലനവുമായും ബന്ധപ്പെട്ട സ്ഥാനങ്ങള്‍ വഹിച്ച സ്ത്രീകളും, ഇസ്‌ലാമിക ഫിഖ്ഹില്‍ ‘അല്‍ ഹസബ’ എന്നറിയപ്പെട്ടിരുന്ന പ്രത്യേക സ്ഥാനങ്ങള്‍ സ്ത്രീകള്‍ വഹിച്ചതായും ഒരുപാട് നിവേദനങ്ങള്‍ ഉദ്ദരിക്കപ്പെട്ടിട്ടുണ്ട്. ഇതെല്ലാം ഹിജ്റ ഒന്നാം നൂറ്റാണ്ടിലായിരുന്നു എന്നത് പ്രത്യേകം ഓര്‍ക്കണം. (ഫത്‌വ ബോര്‍ഡ് ദാറുല്‍ ഇഫ്താഉല്‍ മിസ്രിയ്യ, ഈജിപ്ത് (tthps://bit.ly/38GZKjD)

മൂന്നാം നൂറ്റാണ്ടുകാരിയായ സിത്തുല്‍ മുല്‍ക്, അഞ്ചാം നൂറ്റാണ്ടിലെ സ്വന്‍ആഅ് ഭരിച്ച അസ്മാഅ്, നാലാം നൂറ്റാണ്ടില്‍ യമനില്‍ ജനിച്ച അര്‍വ്വ ബിന്‍ത് അഹ്‌മദ്, സ്പെയിനിലെ സൈനബ് നഫ്സാവിയ്യ, സുല്‍ത്താന റദ്വിയ, ഏഴാം നൂറ്റാണ്ടില്‍ ഈജിപ്ത് ഭരിച്ച ശജറുദ്ദുര്‍റ്, സ്പെയിനിലെ ആഇശ ഹുര്‍റ, സിത്തുല്‍ അറബ്, സിത്തുല്‍ അജം, സിത്തുല്‍ വുസറാഅ് അത്തന്നൂഖിയ്യ, ശരീഫ ഫാത്തിമിയ്യ, ഗാലിയ്യ വഹ്ഹാബിയ്യ, ഖാത്തൂന്‍ ഖത്ലഅ് താര്‍കാന്‍, ഖാത്തൂന്‍ ബാദ്ശാഹ്, ഗസാല ശബീബ, സുല്‍ത്താന ഖദീജ, അബിശ് ഖാത്തൂന്‍, ദൗലത്ത് ഖാത്തൂന്‍, തുര്‍ഖാന്‍ ഖാത്തൂന്‍ തുടങ്ങി വ്യത്യസ്ത മദ്ഹഹബ്‌കാരും, വിഭാഗക്കാരുമായ സ്ത്രീകള്‍ ഭരണ- സൈനിക സാരഥ്യം വഹിച്ചിട്ടുണ്ടെന്നതിന് ചരിത്രം സാക്ഷിയാണ്. (tthps://www.dtoosr.org/699052)

ഭരണം, വിധി, നിയമനിര്‍മ്മാണം, നീതിപാലനം, സൈനികം, കര്‍മ്മശാസ്ത്രം, ഹദീഫ്, സാഹിത്യം, ഭാഷാശാസ്ത്രം, വിദ്യഭ്യാസ പ്രവര്‍ത്തനങ്ങള്‍, വൈദ്യം തുടങ്ങി വിശാലമായ പല മേഖലകളിലും സ്ത്രീ നേതാക്കള്‍ ഇസ്‌ലാമിക ചരിത്രത്തില്‍ സുലഭമായിരുന്നു. ‘മുഅ്ജമു അഅ്ലാമി നിസാഅ്’ അഥവാ ‘സ്ത്രീ നേതാക്കളെ പറ്റിയുള്ള നിഖണ്ഡു’ എന്ന ഒരു ഗ്രന്ഥം മുഹമ്മദ് തന്നൂഖിയുടേതായി കാണാം. ചരിത്രത്തിന്റെ വ്യത്യസ്ത ദിക്കുകളില്‍, ഇസ്‌ലാമിക രാജ്യങ്ങളില്‍ മേല്‍പറയപ്പെട്ട സ്തുത്യാര്‍ഹമായ മേഖലകളില്‍ വിരാജിച്ച നിപുണകളായ മുസ്‌ലിം സ്ത്രീ രത്നങ്ങളുടെ ജീവചരിത്രശേഖരമാണ് പ്രസ്തുത ഗ്രന്ഥം. പതിനായിരത്തിലതികം സ്ത്രീകളെപറ്റി ആ ഗ്രന്ഥം ലോകത്തോട് സംസാരിക്കുന്നുണ്ട്. അത്തരത്തിലുള്ള ഒട്ടനവധി ഗ്രന്ഥസമുച്ചയങ്ങള്‍ തന്നെ ഇസ്‌ലാമിക ലോകത്ത് ഇന്നും സുലഭമാണ്.

ഉര്‍വത്തിബ്നു സുബൈര്‍ (റ) പറഞ്ഞു: ”ഖുര്‍ആന്‍, അനന്തരാവകാശ നിയമങ്ങള്‍, ഹറാം ഹലാലുകള്‍ (അഥവാ കര്‍മ്മശാസ്ത്രം), കവിത, അറബികളുടെ നാട്ടറിവുകള്‍, കുടുംബപരമ്പരകള്‍ എന്ന് തുടങ്ങി ഒരു വിഷയത്തിലും ആഈശയേക്കാള്‍ അറിവുള്ള ഒരാളെയും ജനങ്ങളില്‍ ഞാന്‍ കണ്ടിട്ടില്ല.” (തദ്കിറത്തുല്‍ ഹുഫ്ഫാദ്, ദഹബി: 1/25) മസ്റൂക് പറഞ്ഞു: ”എന്റെ ആത്മാവ് ആരുടെ കൈയ്യിലാണോ അവന്‍ തന്നെ സത്യം. മുഹമ്മദ് നബി (സ)യുടെ അനുചരന്മാരില്‍ മഹാപണ്ഡിതരായ തലമുതിര്‍ന്നവര്‍ ആഇശ(റ)യോട് അനന്തരാവകാശ നിയമങ്ങളെ സംബന്ധിച്ച് ചോദിച്ച് പഠിക്കുന്നത് ഞാന്‍ കണ്ടിട്ടുണ്ട്.” (മുസ്വന്നഫ് ഇബ്നു അബീശൈബ: 30387) അലി (റ) പറഞ്ഞു: ”ആരെയൊക്കെ കൊണ്ടാണ് ഞാന്‍ പരീക്ഷിക്കപ്പെട്ടിരിക്കുന്നത് എന്ന് നിനക്കറിയാമോ? ജനങ്ങളില്‍ ഏറ്റവും, ജനങ്ങളാല്‍ അനുസരിക്കപ്പെടുന്ന ആഇശയെകൊണ്ടും, ജനങ്ങളില്‍ ഏറ്റവും ശക്തനായ സുബൈറിനെകൊണ്ടും, ജനങ്ങളില്‍ ഏറ്റവും ബുദ്ധിശാലിയായ ത്വല്‍ഹയെകൊണ്ടും, ജനങ്ങളില്‍ ഏറ്റവുംശുദ്ധനായ യഅ്ലബ്നു ഉമയ്യയെകൊണ്ടുമാണ് ഞാന്‍ പരീക്ഷിക്കപ്പെട്ടിരിക്കുന്നത്”. (ഫത്ഹുല്‍ ബാരി: 13/57)

ആഇശ എന്ന സ്ത്രീയുടെ മതപരമായ വിധികര്‍തൃത്വം മുസ്‌ലിം സമൂഹം അംഗീകരിച്ചിരുന്നതിന്റെ ചരിത്ര സാക്ഷ്യങ്ങളാണിത്. ‘ജനങ്ങളില്‍ ഏറ്റവും, ജനങ്ങളാല്‍ അനുസരിക്കപ്പെട്ടിരുന്ന വ്യക്തി’ എന്നാണ് അലി (റ) ആഇശയെപറ്റി പരാമര്‍ശിക്കുന്നത്. ജമല്‍ യുദ്ധത്തില്‍ തനിക്കെതിരായി ആഇശ നേതൃത്വം നല്‍കിയ കാര്യത്തെ അനുസ്മരിക്കുമ്പോള്‍ പോലും, അവരുടെ നേതൃപാടവത്തേയും ജനസമ്മതിയേയും അംഗീകരിച്ചുകൊണ്ടാണ് അലി(റ) സംസാരിക്കുന്നത്. സ്ത്രീ രാഷ്ട്രീയ നേതൃത്വം കൈയ്യാളുന്നത് ഇസ്‌ലാമില്‍ നിഷിദ്ധമായിരുന്നെങ്കില്‍ അദ്ദേഹം പ്രതിയോഗിക്കെതിരെ ആദ്യമായി ഉന്നയിക്കുമായിരുന്നു ഏറ്റവും പ്രധാനപ്പെട്ട വിമര്‍ശനം അതാകുമായിരുന്നില്ലേ?! ഒരു സ്ത്രീ ആയിരുന്നിട്ടും ആഇശയുടെ മതപരമായ വിധികര്‍തൃത്വം അംഗീകരിക്കാന്‍ ഇസ്‌ലാം ജനങ്ങള്‍ക്ക് തടസ്സം നിന്നിട്ടില്ലെങ്കില്‍ ഭൗതിക കാര്യങ്ങളില്‍പെട്ട വിധികര്‍തൃത്വം (ഭരണം) അംഗീകരിക്കാന്‍ അത് തടസ്സമാകുന്നതെങ്ങനെ?!

ജമല്‍ യുദ്ധത്തില്‍ ഒട്ടകത്തിന്റെ സംഘക്കാരുടെ (ആഇശയുടെ) കൂടെ ചേര്‍ന്ന് യുദ്ധം ചെയ്യാന്‍ ഞാനാഗ്രഹിച്ചപ്പോള്‍ പ്രവാചകനില്‍ നിന്ന് കേട്ട ഒരു വചനം എനിക്ക് ഏറെ ഉപകാരപ്പെട്ടു. പേര്‍ഷ്യക്കാര്‍ കിസ്റയുടെ മകളെ അധികാരമേല്‍പിച്ച വിവരം പ്രവാചകന് ലഭിച്ചപ്പോള്‍ അവിടുന്ന് പറഞ്ഞു: ‘സ്ത്രീയെ തങ്ങളുടെ ഭരണനേതൃത്വം ഏല്‍പ്പിച്ച ഒരു ജനത വിജയിക്കുകയില്ല’ (ബുഖാരി) എന്ന അബൂബക്റത്തി(റ)ന്റെ നിലപാടിനെ നാം അപ്പോള്‍ എങ്ങനെയാണ് മനസ്സിലാക്കേണ്ടത്.?

അബൂബക്റത് (റ) സ്വഹാബിയാണ് എന്നതുകൊണ്ട് അദ്ദേഹത്തിന്റെ ഗ്രാഹ്യതയില്‍ അബദ്ധം സംഭവിക്കില്ലെന്ന് പറയാനാവില്ല. ‘ഒരു സ്ത്രീയെ തങ്ങളുടെ ഭരണനേതൃത്വം ഏല്‍പ്പിച്ച ഒരു ജനത വിജയിക്കില്ല’ എന്ന ഹദീഥാണ് ജമല്‍ യുദ്ധത്തില്‍ ആഇശയുടെ പക്ഷം ചേരാതിരിക്കാന്‍ തന്നെ പ്രേരിപ്പിച്ച ഘടകമെന്ന അദ്ദേഹത്തിന്റെ വാചകം ധാരണപിശക് മാത്രമാണ്. കാരണം അതേ ഹദീഥ് കേട്ട അനേകം സ്വഹാബിമാര്‍ ആഇശ(റ)യുടെ പക്ഷം ചേര്‍ന്നിരുന്നു എന്നു നാം മനസ്സിലാക്കണം. സ്വര്‍ഗ്ഗം കൊണ്ട് സുവിശേഷം അറിയിക്കപ്പെട്ട സുബൈര്‍ (റ), ത്വല്‍ഹ (റ) തുടങ്ങിയവരും അബ്ദുല്ലാബിബ്നു ആമിര്‍ (റ), യഅ്ലബ്നു മനിയ്യ (റ), മുഹമ്മദിബ്നു ത്വല്‍ഹ (റ) തുടങ്ങിയ സ്വഹാബികളും ആഇശയുടെ നേതൃത്വം അംഗീകരിച്ചവരായിരുന്നു. അവരാരും തന്നെ, അബൂബക്റത്ത് (റ) ഹദീഥിനെ ഗ്രഹിച്ചതുപോലെ മനസ്സിലാക്കിയിട്ടുണ്ടായിരുന്നില്ല. ഉണ്ടായിരുന്നെങ്കില്‍ അവരും ആഇശയില്‍ നിന്നും വിട്ടുനില്‍ക്കുമായിരുന്നു. അതുകൊണ്ടാണ് അബൂബക്റത്തിന് സംഭവിച്ച ധാരണപിശക് മാത്രമാണിതെന്ന് പറഞ്ഞത്.

മാത്രമല്ല തന്റെ ഭാര്യമാരില്‍ ഒരാള്‍ ഭാവിയില്‍ ഈ യുദ്ധത്തിനു (ജമല്‍) നേതൃത്വം നല്‍കുമെന്ന് പ്രവാചകന്‍ (സ) തന്നെ അറിയിച്ചിരുന്ന കാര്യമായിരുന്നു. പ്രസ്തുത നിവേദനം നാം കാണുക. ”രാത്രിയില്‍ ബനൂ ആമിറുകാരുടെ നദീജല സ്രോതസ്സിനടുത്തെത്തിയപ്പോള്‍ നായ്ക്കള്‍ കുരക്കുന്നതായി ആഇശ (റ) കേട്ടു. ഇതേതാണ് ഈ ജലസ്രോതസ്സെന്ന് അവര്‍ ചോദിച്ചു. ഹൗഅബ് ജലതടമാണെന്ന് പറയപ്പെട്ടു. അപ്പോള്‍ അവര്‍ പറഞ്ഞു: മടങ്ങുവാനല്ലാതെ മറ്റൊന്നും ഞാന്‍ ഉദ്ദേശിക്കുന്നില്ല. തീര്‍ച്ചയായും അല്ലാഹുവിന്റെ തിരുദൂതന്‍ പറയുന്നത് ഞാന്‍ കേട്ടിട്ടുണ്ട്. ഹൗഅബിലെ നായ്ക്കള്‍ നിങ്ങളില്‍ (ഭാര്യമാരില്‍) ഒരാളുടെ മേല്‍ കുരക്കുന്നതിനെപറ്റി എങ്ങനെയുണ്ടെന്ന് തോന്നുന്നു. അപ്പോള്‍ സുബൈര്‍ (റ) പറഞ്ഞു: മടങ്ങുകയോ നിങ്ങള്‍ നിമിത്തം അല്ലാഹു ജനങ്ങള്‍ക്കിടയില്‍ രഞ്ചിപ്പുണ്ടാക്കിയാലോ? (അത്തരം ഒരവസരം നഷ്ടപ്പെടുത്തരുതെന്നര്‍ത്ഥം) (മുസ്‌നദ് അഹ്‌മദ്: 24654, ഇബ്‌നുഅബീശൈബ: 7/536, സില്‍സിലത്തു സ്വഹീഹ: 474, ഇബ്‌നു ഹിബ്ബാന്‍: 6732, ഹാകിം: 4613)

തന്റെ ഭാര്യമാരില്‍ ഒരാള്‍ തനിക്കുശേഷം അത്തരം ഒരു യുദ്ധത്തിന് നേതൃത്വം നല്‍കുമെന്ന് പ്രവചിച്ച നബി (സ), ആ ഘട്ടത്തില്‍ ഹൗഅബിലെ നായ്ക്കള്‍ ആര്‍ക്കുമേലായിരിക്കുമോ കുരക്കുന്നത് അവരായിരിക്കും അതെന്ന് അടയാളവും പറഞ്ഞു കൊടുത്തു. ഭാവിയില്‍ മുസ്‌ലിംങ്ങള്‍ക്കിടയില്‍ രൂപപ്പെടുന്ന ഒരു വലിയ കുഴപ്പത്തിന്റെ മുന്നോടിയായുള്ള സൂചനയായി അത് തന്റെ ഭാര്യമാരെ പഠിപ്പിച്ച പ്രവാചകന്‍, ആ ഘട്ടത്തില്‍ പോലും സ്ത്രീ നേതൃത്വത്തെ പഴിച്ചില്ലെന്നത് നാം പ്രത്യേകം മനസ്സിലാക്കുക. അഥവാ തെറ്റായ ഒരു പടപ്പുറപ്പാടായി ജമല്‍ യുദ്ധത്തെപറ്റി പ്രവാചകന്‍ (സ) പരാമര്‍ശിച്ചത്; അതിനു നേതൃത്വം നല്‍കുന്നത് സ്ത്രീയായതുകൊണ്ടല്ല. മറിച്ച് അത് മുസ്‌ലിംകള്‍ക്കിടയില്‍ വലിയ കുഴപ്പങ്ങള്‍ക്ക് നിമിത്തമാകും എന്നതുകൊണ്ടായിരുന്നു. സ്ത്രീ നേതൃത്വം ഇസ്‌ലാം വെച്ചുപൊറുപ്പിക്കാത്ത ഒന്നായിരുന്നെങ്കില്‍ ഈ സന്ദര്‍ഭത്തില്‍ പ്രവാചകന്‍ അത് പ്രത്യേകം പരാമര്‍ശിക്കുമായിരുന്നു. അപ്പോള്‍ ഭാവിയില്‍ സംഭവിക്കാനിരിക്കുന്ന ജമല്‍ യുദ്ധത്തെയും അതിന് ഒരു സ്ത്രീയായിരിക്കും നേതൃത്വം നല്‍കുകയെന്നും അത് തന്റെ ഭാര്യമാരില്‍ ഒരാളായിരിക്കും എന്നുമെല്ലാം മനസ്സിലാക്കിയ പ്രവാചകന്‍ അതിനെപറ്റി തന്റെ ഭാര്യമാര്‍ക്ക് മുന്നറിയിപ്പ് നല്‍കുമ്പോള്‍ സ്ത്രീ നേതൃത്വത്തെ പ്രശ്നവല്‍കരിക്കാതെ മുസ്‌ലിം സമൂഹത്തിനിടയില്‍ സംഭവിക്കുന്ന ഭിന്നതയേയും കുഴപ്പത്തേയും പറ്റി മാത്രം പ്രശ്നവല്‍കരിച്ചു സംസാരിച്ചു എന്നതില്‍ നിന്നു തന്നെ അബൂബക്റത്തിന്റെ (റ) നിലപാട് ധാരണപിശകില്‍നിന്ന് രൂപപ്പെട്ട ഒന്നാണെന്ന് വ്യക്തം. ചുരുക്കത്തില്‍, ‘സ്ത്രീയെ തങ്ങളുടെ ഭരണനേതൃത്വം ഏല്‍പ്പിച്ച ഒരു ജനത വിജയിക്കുകയില്ല’ എന്ന ഹദീഥ് സ്ത്രീയുടെ അധികാരത്തിലുള്ള എല്ലാ വിധ പങ്കിനേയും എതിര്‍ക്കുന്നില്ല; ഏറിവന്നാല്‍, പൗരാണിക രാഷ്ട്ര വ്യവസ്ഥയിലെ അവളുടെ സര്‍വ്വാധിപത്യത്തെ മാത്രമാണ് അത് ചോദ്യം ചെയ്യുന്നത്. അതുകൊണ്ടുതന്നെ പ്രസ്തുത ഹദീഥും പൊക്കിപ്പിടിച്ച് പൊഴിക്കുന്ന മുതലകണ്ണീരുകൊണ്ടൊന്നും കാര്യമായ പ്രയോജനം ഉണ്ടാകില്ലെന്ന് തിരിച്ചറിഞ്ഞ് ഒന്ന് മാറ്റിപിടിക്കുന്നതാണ് ഇസ്‌ലാംവിമര്‍ശകര്‍ക്ക് നല്ലത്. ഉഡായിപ്പുകള്‍കൊണ്ടൊന്നും വൈജ്ഞാനിക രംഗത്ത് മേല്‍വിലാസമുണ്ടാക്കാനാവില്ലെന്ന് ഇവരൊക്കെ ഇനി എന്നാണാവോ തിരിച്ചറിയുക.

സ്ത്രീകളെ തികഞ്ഞ അശ്ലീലതകള്‍ക്കു പ്രേരിപ്പിക്കുന്ന പല ഉപദേശങ്ങളും ഹദീഥ് ഗ്രന്ഥങ്ങളില്‍, നബിക്കു ലഭിച്ച ദൈവിക വെളിപാടുകളായി ഇന്നും രേഖപ്പെടുത്തപ്പെട്ടു കിടക്കുന്നതു കാണാം. സുഹൈലിന്റെ മകള്‍ സഹ്‌ല എന്ന സ്ത്രീയോട് സാലിം എന്ന പരപുരുഷനെ മുലയൂട്ടി മകനായി സ്വീകരിക്കാന്‍ ആവശ്യപ്പെട്ട ഹദീഫുകള്‍ സ്വഹീഹു മുസ്‌ലിമില്‍ വരെ കാണാം. ഈ പ്രവാചകോപദേശം സ്വീകരിക്കാന്‍ മാന്യതയുള്ള മുസ്‌ലിം സ്ത്രീകള്‍ക്കു സാധിക്കുമോ? മാത്രമല്ല പ്രവാചക പത്‌നി ആഇശ ഇത്തരത്തില്‍ പരപുരുഷന്മാരെ മുലയൂട്ടാറുണ്ടായിരുന്നുവെന്നും പലപ്പോഴും ആഇശയുടെ അരികില്‍ പരപുരുഷന്മാരെ കാണുക വഴി നബിക്കുതന്നെ അതില്‍ അനിഷ്ടമുണ്ടാവുകയും ചെയ്തിരുന്നു എന്ന് സ്വഹീഹ് മുസ്‌ലിമിലെ 1455-ാം ഹദീഥായി രേഖപ്പെടുത്തിയിട്ടുണ്ട്. സ്വയം ചെയ്തിരുന്നു എന്നു മാത്രമല്ല തന്റെ സഹോദര-സഹോദരി പുത്രിമാരോട് അപ്രകാരം ചെയ്യാന്‍ ആഇശ ഉപദേശിക്കുമായിരുന്നു എന്നു കൂടി ഹദീഥ് ഗ്രന്ഥങ്ങളില്‍ രേഖപ്പെടുത്തപ്പെട്ടിട്ടുണ്ട്. മാത്രമല്ല, കല്ലെറിയലിന്റെ വചനവും പത്ത് പ്രാവശ്യമാണ് മുതിര്‍ന്ന പുരുഷന്മാരുടെ മുലകുടിയെന്ന വചനവും (ആദ്യകാലത്ത് ക്വുര്‍ആനില്‍) അവതരിപ്പിക്കപ്പെട്ടിരുന്നെന്നും അതെഴുതിയ രേഖ എന്റെ തലയിണക്കടിയിലുണ്ടായിരുന്നെന്നും ദൈവദൂതന്‍ മരണപ്പെട്ടപ്പോള്‍, ഞങ്ങളെല്ലാം അതുമായി ബന്ധപ്പെട്ട തിരക്കുകള്‍ക്കടിയിലായിരുന്ന സന്ദര്‍ഭത്തില്‍ ഒരു ആട് അകത്ത് കടന്ന് അത് തിന്നുകളഞ്ഞു എന്നും ആഇശയില്‍ നിന്ന് സുനനു ഇബ്‌നുമാജയും മുസ്‌നദ് അഹ്‌മദും ഉദ്ദരിച്ചിട്ടുണ്ട്. സ്ത്രീകളെക്കൊണ്ട് പരപുരുഷന്മാര്‍ക്ക് മുലയൂട്ടുന്ന നടപടിയാണോ ഇസ്‌ലാം പഠിപ്പിച്ച ധാര്‍മ്മികത?

അസത്യങ്ങളും അര്‍ദ്ധസത്യങ്ങളും കോര്‍ത്തിണക്കി നെയ്‌തെടുത്ത ഒരു വ്യാജപ്രചരണമാണിത്. തെറിവിളികള്‍ക്കും പരമതനിന്ദക്കും ‘വിമര്‍ശനം’ എന്നുപേരിട്ട് വൈജ്ഞാനിക രംഗത്ത് കൃത്രിമ മേല്‍വിലാസമുണ്ടാക്കി വെറുപ്പുകച്ചവടം നടത്താനുള്ള കുടില വ്യഗ്രത മാത്രമാണ് പ്രസ്തുത ആരോപണങ്ങള്‍ക്കു പിന്നിലുള്ളത്. ‘ദൈവസ്‌നേഹം’ നെറ്റിയിലൊട്ടിച്ച് പരമത വിദ്വേഷം ഹൃദയത്തില്‍ ഒളിപ്പിച്ചുവെച്ച ചില മിഷണറി നുണ ഫാക്ടറികള്‍ നിര്‍മ്മിച്ച ഈ നബിനിന്ദാ പ്രചരണം, യുക്തിവാദികളും ഫെമിനിസ്റ്റുകളും ഫാഷിസ്റ്റുകളുമെല്ലാം മൊത്തമായും ചില്ലറയായും വിപണനം ചെയ്തുകൊണ്ടിരിക്കുകയാണ്. അതുകൊണ്ടുതന്നെ പ്രസ്തുത ആരോപണം ഇഴയടര്‍ത്തി പരിശോധിക്കേണ്ടിയിരിക്കുന്നു.

അറബികള്‍ക്കിടയില്‍ ബന്ധം സ്ഥാപിതമാകാനുള്ള കാരണങ്ങള്‍ നാലെണ്ണമായിരുന്നു; പ്രസവം, വിവാഹം, മുലകുടി, ദത്തെടുക്കല്‍. ഇതില്‍ ദത്തെടുക്കല്‍, ബന്ധം സ്ഥാപിതമാകാനുള്ള കാരണങ്ങളില്‍ നിന്നും ഇസ്‌ലാം ഒഴിവാക്കിയപ്പോഴാണ് സാലിമിന്റെ(റ) വിഷയത്തിലുള്ള പ്രശ്‌നം ഉണ്ടാകുന്നത്. അഥവാ അബൂഹുദൈഫഃ(റ)യുടെയും സഹ്‌ല(റ)യുടെയും ദത്തുപുത്രനായിരുന്നു സാലിം (റ). ഇസ്‌ലാം ദത്തുപുത്ര സമ്പ്രദായത്തിനു അറുതിവരുത്തിയപ്പോള്‍ ഇന്നലെവരെ തന്റെ മകനായി ജീവിച്ച സാലിം, സഹ്‌ല എന്ന വളര്‍ത്തു മാതാവിന് അന്യപുരുഷനായി മാറി. ഇതില്‍ തീവ്ര ദുഃഖമറിയിച്ചുകൊണ്ട് അവര്‍ പ്രവാചകനെ(സ) സമീപിക്കുകയുണ്ടായി. പ്രസ്തുത ചരിത്ര നിമിഷങ്ങള്‍ ഹദീഥ് ഗ്രന്ഥങ്ങളില്‍ നിന്നും നമുക്കു വായിച്ചെടുക്കാം. ആഇശ (റ) നിവേദനം: സുഹൈലിന്റെ മകള്‍ സഹ്‌ല ഒരിക്കല്‍ നബി(സ)യുടെ അരികില്‍ വന്നുപറഞ്ഞു. ‘അല്ലാഹുവിന്റെ ദൂതരേ, സാലിം എന്റെ അടുത്ത് പ്രവേശിക്കുന്നതില്‍ അബൂഹുദൈഫഃയുടെ (സഹ്‌ലയുടെ ഭര്‍ത്താവാണ് അബൂഹുദൈഫഃ) മുഖത്ത് വെറുപ്പുള്ളതായി തോന്നുന്നു. അപ്പോള്‍ നബി (സ) പറഞ്ഞു: ‘അയാള്‍ക്കു നീ മുലപ്പാല്‍ കൊടുക്കുക’. അവള്‍ ചോദിച്ചു ‘അയാള്‍ വലിയ മനുഷ്യനാണല്ലോ എങ്ങനെ ഞാന്‍ മുലപ്പാല്‍ കൊടുക്കും?!’. അപ്പോള്‍ നബി (സ) പുഞ്ചിരിച്ചുകൊണ്ട് പറഞ്ഞു: ‘അയാള്‍ വലിയ മനുഷ്യനാണെന്നു എനിക്കറിയാം. (സ്വഹീഹു മുസ്‌ലിം 1453)

മുസ്‌ലിമിന്റെ തന്നെ മറ്റൊരു നിവേദനം ഇപ്രകാരമാണ്. ”അബൂഹുദൈഫഃ(റ)യുടെ അടിമ സാലിം (റ), അബൂഹുദൈഫഃയുടെയും കുടുംബത്തിന്റെയും കൂടെ അവരുടെ വീട്ടിലായിരുന്നു താമസം. സുഹൈലിന്റെ മകള്‍ നബി(സ)യുടെ അടുത്ത് വന്നു പറഞ്ഞു: ‘തീര്‍ച്ചയായും സാലിമിന്നു സാധാരണ പുരുഷന്മാരാകുന്ന നിലയില്‍ ബുദ്ധിയും പ്രായവും തികഞ്ഞിട്ടുണ്ട്. അവന്‍ ഞങ്ങളുടെ അടുത്ത് കടന്നുവരാറുണ്ട്. അബൂഹുദൈഫഃയുടെ മനസ്സില്‍ അതില്‍ വെറുപ്പുള്ളതായി എനിക്ക് തോന്നുന്നു’. അപ്പോള്‍ നബി (സ) പറഞ്ഞു: ‘നീ അയാള്‍ക്ക് മുലപ്പാല്‍ കൊടുക്കുക. എന്നാല്‍ അയാള്‍ അടുത്ത ബന്ധുവായിത്തീരും. അബൂഹുദൈഫഃയുടെ മനസ്സില്‍ വെറുപ്പും ഇല്ലാതാകും’. അവള്‍ വീണ്ടും നബി(സ)യുടെ അരികെ വന്നു പറഞ്ഞു: ‘ഞാന്‍ അയാള്‍ക്ക് മുലപ്പാല്‍ കൊടുത്തു. അങ്ങനെ അബൂഹുദൈഫഃയുടെ മനസ്സില്‍ വെറുപ്പ് ഇല്ലാതാകുകയും ചെയ്തു’. (സ്വഹീഹു മുസ്‌ലിം)

ഈ ഹദീഥുകള്‍ മനസ്സിലാക്കിത്തരുന്ന കാര്യങ്ങളെ ഇങ്ങനെ സംഗ്രഹിക്കാം

1) അബൂഹുദൈഫഃ(റ)യുടെ മോചിത അടിമയായ സാലിമി(റ)നെ നന്നെ ചെറുപ്പത്തില്‍ തന്നെ അബൂഹുദൈഫ-സഹ്‌ല ദമ്പതിമാര്‍ ദത്തുപുത്രനായി സ്വീകരിക്കുകയും സ്വപുത്രന്റെ സ്ഥാനം നല്‍കി വളര്‍ത്തുകയും ചെയ്തു. 2) ഇസ്‌ലാം ദത്തുപുത്രസമ്പ്രദായം അവസാനിപ്പിച്ചതോടെ, മതപരമായി സാലിം(റ) ആ കുടുംബത്തിന് മകനല്ലാതായിത്തീരുകയും, വളര്‍ത്തു മാതാവാണെങ്കിലും സഹ്‌ല(റ)യെ സംബന്ധിച്ച് സാലിമി(റ)നുമേല്‍ അന്യപുരുഷന്റെ വിധി ബാധകമാവുകയും ചെയ്തു. 3) വളര്‍ത്തുപുത്രനാണെങ്കിലും മതപരമായി അന്യപുരുഷന്റെ വിധി ബാധകമായ ഒരാള്‍, തന്റെ ഭാര്യയെ സന്ദര്‍ശിക്കുന്നതും അവരുടെ അടുക്കല്‍ പ്രവേശിക്കുന്നതും ഭര്‍ത്താവായ അബൂഹുദൈഫഃ(റ)യില്‍ വെറുപ്പുളവാക്കി. 4) മകന്റെ സ്ഥാനം നല്‍കി താന്‍ വളര്‍ത്തിയ ഒരാളെ ഒരു നിമിഷം കൊണ്ട് അന്യനെപ്പോലെ ഒഴിവാക്കുവാന്‍ സഹ്‌ല(റ)യുടെ മാതൃഹൃദയം അവരെ അനുവദിച്ചില്ല. അതിനാല്‍ തങ്ങളുടെ കാര്യത്തില്‍ ഒരു പോംവഴി തേടിക്കൊണ്ട് നീറുന്ന ഹൃദയവുമായി അവള്‍ പ്രവാചക(സ)ന്റെ അരികിലെത്തി. 5) അവരുടെ പ്രശ്‌നത്തിനു പരിഹാരമായി ‘സാലിമിന് മുലപ്പാല്‍ നല്‍കാനും അങ്ങനെ സ്വപുത്രനെ പോലെ സ്വീകരിക്കുവാനും’ പ്രവാചകന്‍ (സ) അവര്‍ക്ക് ഇളവുനല്‍കി. അതോടെ അബൂഹുദൈഫഃ(റ)യുടെ മനസ്സില്‍നിന്നും ആ വെറുപ്പ് ഇല്ലാതാവുകയും ചെയ്തു.

സാലിം എന്ന മുതിര്‍ന്ന ആണ്‍കുട്ടിക്ക് മുലപ്പാല്‍ നല്‍കി മകനായി സ്വീകരിക്കുവാന്‍ സഹ്‌ലയോട് നിര്‍ദ്ദേശിച്ചതാണ്, പ്രവാചകനെ പെണ്‍വിരുദ്ധനും സ്ത്രീകളെ അശ്ലീലതകള്‍ക്കു പ്രേരിപ്പിച്ച വ്യക്തിത്വമായും മുദ്ര കുത്താന്‍ വിമര്‍ശകരെ പ്രേരിപ്പിക്കുന്ന ഘടകം. വസ്തുതകള്‍ പലതും മറച്ചുപിടിച്ചുകൊണ്ടാണ് ഈ വിമര്‍ശനം നെയ്‌തെടുത്തിരിക്കുന്നത്. ‘മുലപ്പാല്‍ നല്‍കുക’ എന്നു പറഞ്ഞതിനെ ‘പാല്‍ കുടിക്കാന്‍ സ്തനം നല്‍കുക’ എന്ന്, അശ്ലീല ഭാവനകള്‍ കൊണ്ട് ലൈംഗിക ഛായം പൂശുകയാണ് വാസ്തവത്തില്‍ വിമര്‍ശകന്മാര്‍ ചെയ്തിരിക്കുന്നത്. ‘ബാബു രിദ്വാഅത്തില്‍ കബീരി’ അഥവാ വലിയവരുടെ രിദ്വാഅഃ (മുലപ്പാല്‍ കുടിക്കല്‍) എന്ന അദ്ധ്യായങ്ങള്‍ക്കു കീഴിലാണ് പ്രസ്തുത ഹദീഥുകല്‍ ഉദ്ദരിക്കപ്പെട്ടിരിക്കുന്നത്. രിദ്വാഅഃ (الرضاعة) എന്ന പദം സ്ത്രീയുടെ മുലപ്പാല്‍ സ്തനത്തില്‍ നിന്ന് നേരിട്ട് ഈമ്പിക്കുടിക്കുന്ന രീതിക്കു മാത്രമാണോ അറബി ഭാഷയില്‍ ഉപയോഗിക്കുന്നത്? അല്ല എന്നതാണ് വസ്തുത. പക്ഷേ ആ വസ്തുതകളൊന്നും കാണാന്‍ വിമര്‍ശകര്‍ തയ്യാറല്ല. തയ്യാറായാല്‍ ഈ അശ്ലീല ഭാവനകള്‍ക്കൊന്നും നിലനില്‍പ്പുണ്ടാവില്ലെന്ന് അവര്‍ക്ക് നല്ലപോലെ അറിയാം.

എന്താണ് രിദ്വാഅഃ? (الرضاعة)

ഒരു സ്ത്രീയുടെ മുലപ്പാല്‍ മറ്റൊരാളുടെ അല്ലെങ്കില്‍ കുഞ്ഞിന്റെ വയറ്റില്‍ എത്തുക എന്നതിനെയാണ് ഇര്‍ദ്വാഅ് (الارضاع), രിദ്വാഅഃ (الرضاعة) എന്നൊക്കെ പറയുന്നത്. അതാകട്ടെ പല രീതിയിലും നടക്കാം. സ്ത്രീയുടെ മുലപ്പാല്‍ സ്തനത്തില്‍ നിന്ന് നേരിട്ട് ഈമ്പിക്കുടിക്കുന്ന രീതി അഥവാ ‘മസ്വ്’ (المص), സ്തനം സ്പര്‍ശിക്കാതെ കുട്ടിയുടെ വായിലേക്ക് പാല്‍ ചുരന്ന് ഒഴുക്കുന്ന രീതി അഥവാ ‘സ്വബ്ബ്’ (الصب), സ്തനം സ്പര്‍ശിക്കാതെ സ്ത്രീയുടെ മുലപ്പാല്‍ പിഴിഞ്ഞെടുത്ത് തളികയിലാക്കി കുട്ടിയുടെ തൊണ്ടയിലേക്ക് ചൊരിയുന്ന രീതി അഥവാ ‘വുജൂര്‍’ (الوجور), സ്തനം സ്പര്‍ശിക്കാതെ സ്ത്രീയുടെ മുലപ്പാല്‍ പിഴിഞ്ഞെടുത്ത് തളികയിലാക്കി കുട്ടിയുടെ നാസദ്വാരത്തില്‍ ഒഴിക്കുന്ന രീതി അഥവാ ‘സുഊത്വ്’ (السعوط), സ്തനം സ്പര്‍ശിക്കാതെ സ്ത്രീയുടെ മുലപ്പാല്‍ പൈപ്പിലൂടെയോ, സിറിഞ്ചിലൂടെയോ കുട്ടിക്കു നല്‍കുന്ന രീതി അഥവാ ‘ഹിക്‌നത്ത്’ (الحقنة). ഈ ഏത് രീതിയിലും മുലപ്പാല്‍ നല്‍കുന്നതിനും ഇസ്‌ലാമിലെ സാങ്കേതിക പദവും കര്‍മ്മശാസ്ത സംജ്ഞ്യയുമായ ‘രിദ്വാഅഃ’ എന്ന് പറയുമെന്ന് പൗരാണികരായ അറബി ഭാഷാ പണ്ഡിതരും സര്‍വ്വ മദ്ഹബുകളിലും പെട്ട കര്‍മ്മശാസ്ത്ര വിശാരദന്മാരും വ്യക്തമാക്കിയിട്ടുണ്ട്. (മുഖ്തസറു ഖലീല്‍: 162, മവാഹിബുല്‍ ജലീല്‍: 4/178, ശര്‍ഹുല്‍ കബീര്‍: 2/502, മന്‍ഹുല്‍ ജലീല്‍: 4/371, അല്‍ ഫവാകിഹുദ്ദവാനി: 2/54, മുഗ്നി അല്‍ മുഹ്താജ്: 3/414, ഫത്ഹുല്‍ വഹാബ്: 2/194, അസ്സിറാജുല്‍ വഹ്ഹാജ്: 460, അല്‍ മുബ്ദിഅ് 8/160, ശര്‍ഹു മുന്‍തഹല്‍ ഇറാദത്ത്: 3/213, കശ്ശാഫുല്‍ കനാഅ്- 5: 442, മതാലിബു ഉലിന്നുഹ- 5: 596, അല്‍ ബഹ്‌റു റാഇഖ്- 3: 238, മജ്മഉല്‍ അന്‍ഹുര്‍ : 1/551) (ഉദ്ദരണം: ഡോ. രിയാദ് മിശ്അലിന്റെ ലേഖനത്തില്‍ നിന്ന് Midad Al-Adab Refereed Quarterly journal : Vol.1 : Issue 1 : Article 10, ഇറാക് സര്‍വ്വകലാശാല)

പൗരാണിക ഭാഷാ പണ്ഡിതനായ ഇബ്‌നു മന്‍ളൂര്‍ (ജനനം: ഹിജ്‌റ 630) തന്റെ അറബി നിഘണ്ഡുവായ ‘ലിസാനുല്‍ അറബി’ല്‍ (2/92) പറഞ്ഞു: ”സ്തനത്തില്‍ നിന്ന് നേരിട്ട് മുലയൂട്ടുന്നത് വിവാഹബന്ധം നിഷിദ്ധമാക്കുന്നത് പോലെ തന്നെ സ്ത്രീയുടെ സ്തനത്തില്‍ നിന്ന് നേരിട്ടല്ലാതെ പാല്‍ ചുരന്നെടുത്ത് കുട്ടിയെ കുടിപ്പിക്കുന്നതും (രിദ്വാഅഃ ആയതിനാല്‍) വിവാഹബന്ധം നിഷിദ്ധമാക്കുന്നതാണ്. സ്തനത്തില്‍ നിന്ന് വേറിട്ടതിനാല്‍ അത് രിദ്വാഅഃ ആകാതിരിക്കുന്നില്ല.”

അപ്പോള്‍ ഭാഷാപരമായി പോലും രിദ്വാഅഃ എന്നു പറഞ്ഞാല്‍, സ്ത്രീയുടെ മുലപ്പാല്‍ സ്തനത്തില്‍ നിന്ന് നേരിട്ട് ഈമ്പി കുടിക്കുന്ന രീതിയാകണം എന്നില്ല. പ്രവാചകന്‍ (സ) സഹ്‌ലയോട് പറഞ്ഞതാകട്ടെ ‘അവന് മുലപ്പാല്‍ നല്‍കുക’ അര്‍ദ്വഈഹി (ارضعيه) എന്നാണ്. അല്ലാതെ നിന്റെ സ്തനം അവന് കുടിക്കാന്‍ കൊടുക്കുക എന്നല്ല. സഹ്‌ല-സാലിം സംഭവം കേവലം ഇസ്‌ലാംവിരോധികളുടെ മ്ലേഛമായ ഭാവനാവിലാസങ്ങള്‍ക്കപ്പുറം, യാതൊരു ആധികാരികതയുമില്ലാത്ത ദുരാരോപണം മാത്രമാണെന്ന് വ്യക്തമാക്കുന്ന തെളിവുകളാണ്, പൂര്‍വ്വകാലം മുതല്‍ മുസ്‌ലിം പണ്ഡിതന്മാര്‍ പ്രസ്തുത ഹദീഥുകള്‍ക്കു നല്‍കിയ വ്യാഖ്യാനം. അതിലൂടെ നമുക്കൊന്ന് കണ്ണോടിക്കാം.

ഇമാം ഇബ്‌നു കുതൈബ (ജനനം: ഹിജ്‌റ: 213) പറഞ്ഞു: ”പ്രവാചകന്‍ (സ) സഹ്‌ലയോട് പറഞ്ഞത് ‘അവന് മുലപ്പാല്‍ നല്‍കുക'(ارضعيه) എന്നാണ്. നിന്റെ സ്തനം അവന് കുടിക്കാന്‍ കൊടുക്കുക എന്ന് (പ്രവാചകന്‍) ഊദ്ദേശിച്ചിട്ടേ ഇല്ല. നിന്റെ പാല്‍ ചുരന്നെടുത്ത് തളികയിലാക്കി കുറച്ച് അവനെ കുടിപ്പിക്കുക എന്നേ അദ്ദേഹം ഉദ്ദേശിച്ചിട്ടുള്ളൂ. അതല്ലാതെ സ്തനത്തില്‍ നിന്ന് നേരിട്ട് കുടിക്കല്‍ അനുവദനീയമല്ല. കാരണം സഹ്‌ലയുടെ സ്തനത്തിലേക്ക് നോക്കുന്നത് തന്നെ സാലിമിന് നിഷിദ്ധമാണ്. പിന്നെയെങ്ങനെ നിഷിദ്ധമായ ഒരു കാര്യം പ്രവാചകന്‍ അദ്ദേഹത്തിന് അനുവദിച്ച് കൊടുക്കും!” (തഅ്‌വീലു മുക്തലിഫില്‍ ഹദീഥ്: 1/437) ഇതേ വ്യാഖ്യാനം തന്നെ ഇമാം ക്വാള്വീ ഇയാള്വും (ജനനം: ഹിജ്‌റ: 476) ആവര്‍ത്തിക്കുന്നതു കാണാം (ശര്‍ഹുന്നവവി അലാ സ്വഹീഹു മുസ്‌ലിം : 10/30,31)

ഇമാം മാവര്‍ദി (ജനനം : ഹിജ്‌റ : 364) പറയുന്നു: ”സാലിമിന്റെ വിഷയത്തില്‍ സ്തനത്തില്‍ നിന്ന് മുലപ്പാല്‍ ഊട്ടിയെന്ന പ്രസ്താവന ഒന്നും തന്നെയില്ല എന്നത് സുവിദിതമാണ്. അതുകൊണ്ട് തന്നെ പ്രവാചകന്‍ ഉദ്ദേശിച്ചത് ‘വുജൂര്‍’ (الوجور സ്തനം സ്പര്‍ശിക്കാതെ സ്ത്രീയുടെ മുലപ്പാല്‍ പിഴിഞ്ഞെടുത്ത് തളികയിലാക്കി കുട്ടിയുടെ തൊണ്ടയില്‍ ചൊരിയുന്ന രീതി) ആണെന്ന് സ്ഥാപിതമാകുന്നു” (അല്‍ ഹാവി അല്‍അല്‍ കബീര്‍ : 11/372)

ഇബ്‌നു അബ്ദുല്‍ ബിര്‍റ് (ജനനം: ഹിജ്‌റ: 368) പറഞ്ഞു: ”മുതിര്‍ന്നവരുടെ മുലകുടി എന്നാല്‍ പാല്‍ പാത്രത്തില്‍ ചുരന്ന് അതില്‍ നിന്ന് കുടിപ്പിക്കലാണ്. എന്നാല്‍ കുട്ടികള്‍ക്ക് നല്‍കുന്നതുപോലെ സ്തനത്തില്‍ നിന്ന് നേരിട്ട് കുടിപ്പിക്കല്‍ മുതിര്‍ന്നവരുടെ കാര്യത്തില്‍ നിഷിദ്ധമാണ് എന്ന് പണ്ഡിതസംഘം വ്യക്തമാക്കിയിട്ടുണ്ട്” (അത്തംഹീദ്: 8/257, അല്‍ ഇസ്തിദ്കാര്‍: 6/255)

ആധുനിക കാലഘട്ടത്തിലെ മുസ്‌ലിം പണ്ഡിതന്മാര്‍ ‘മുലയൂട്ടല്‍’ വിവാദവുമായി ബന്ധപ്പെട്ട ആരോപണങ്ങള്‍ക്ക് മറുപടി പറയുമ്പോള്‍ അതെല്ലാം ‘വ്യാഖ്യാന ഫാക്ടറികള്‍’ എന്നു പറഞ്ഞ് ഒഴിഞ്ഞുമാറുന്ന ഇസ്‌ലാംവിരോധികള്‍ക്ക്, നൂറ്റാണ്ടുകള്‍ക്കു മുമ്പേ പൂര്‍വ്വകാല മുസ്‌ലിം പണ്ഡിതന്മാര്‍ പ്രസ്തുത വിഷയത്തില്‍ നല്‍കിയ ഈ മറുപടികളെപ്പറ്റി എന്തു പറയാനുണ്ട്?!. മിഷണറി നുണഫാക്ടറികളില്‍ ‘മുലയൂട്ടല്‍’ വിവാദം നിര്‍മ്മിച്ചെടുക്കപ്പെടുന്നതിനും എത്രയോ നൂറ്റാണ്ടുകള്‍ക്ക് മുമ്പാണ് ഇമാം ഇബ്‌നു കുതൈബയും, ഇമാം ക്വാള്വീ ഇയാള്വും, ഇമാം മാവര്‍ദിയും, ഇബ്‌നു അബ്ദുല്‍ ബിര്‍റുമെക്കെ ഈ വിവാദങ്ങള്‍ക്ക് മറുപടി പറഞ്ഞതെന്ന വസ്തുതയെങ്കിലും ഇസ്‌ലാം വിരോധം കൊണ്ട് അന്ധത ബാധിച്ചവരുടെ കണ്ണുകള്‍ തുറപ്പിച്ചിരുന്നെങ്കില്‍!! പണ്ഡിത വ്യാഖ്യാനങ്ങള്‍ മാത്രമല്ല ചരിത്ര രേഖകളും പ്രസ്തുത ‘മുലപ്പാല്‍ നല്‍കല്‍’ അപ്രകാരമായിരുന്നെന്ന് വ്യക്തമാക്കുന്നുണ്ട്. ‘ഒരു പാത്രത്തില്‍ മുലപ്പാല്‍ ചുരന്നൊഴിച്ച് എല്ലാ ദിവസവും സാലിം കുടിച്ചു കൊണ്ടിരുന്നുവെന്നും അതിനു ശേഷമാണ് സഹ്‌ലയുടെ വീട്ടില്‍ പ്രവേശിച്ചത്’ എന്നും ചില ചരിത്ര നിവേദനങ്ങള്‍ കാണാം. (ത്വബകാത്തു ഇബ്‌നു സഅ്ദ്: 8/271, ശര്‍ഹു സര്‍ക്കാനി : 3/316)

അശ്ലീല ഭാവനകള്‍ കൊണ്ട് പ്രവാചക ജീവിതത്തിനുമേല്‍ ലൈംഗിക ഛായം പൂശുവാന്‍ വെമ്പല്‍ കൊള്ളുന്ന ഇസ്‌ലാംവിരോധികള്‍ നബി(സ)യില്‍ നിന്നും നിവേദനം ചെയ്യപ്പെട്ട ഈ ഹദീഥെങ്കിലും ഒന്നു പരിഗണിച്ചിരുന്നെങ്കില്‍ അവരുടെ വായകൊണ്ട് ഇത്ര ഗുരുതരമായ അപവാദം പറയാന്‍ ഒന്നു മടിക്കുമായിരുന്നു. നബി(സ) പറഞ്ഞു: ”നിന്റെ തലയില്‍ ഇരുമ്പിന്റെ സൂചികൊണ്ട് കുത്തി തറക്കുന്നതാണ് അനുവദനീയമല്ലാത്ത ഒരു സ്ത്രീയെ സ്പര്‍ശിക്കുന്നതിനേക്കാള്‍ നിനക്ക് നല്ലത്” (അല്‍ മുഅ്ജമുല്‍ കബീര്‍: ത്വബ്‌റാനി: 20:211) അന്യസ്ത്രീകളെ സ്പര്‍ശിക്കുന്നതിനേക്കാള്‍ നല്ലത് തലയില്‍ ഇരുമ്പുസൂചി കുത്തി തറക്കുന്നതാണെന്നു പഠിപ്പിച്ച ഒരു വിശുദ്ധ വ്യക്തിത്വത്തിനുമേല്‍, പരപുരുഷന്മാരെ മുലയൂട്ടാന്‍ സ്ത്രീകളെ പ്രേരിപ്പിച്ചെന്ന അപവാദം പറഞ്ഞു പ്രചരിപ്പിച്ചവര്‍ ദൈവത്തിന്റെ ‘കുഞ്ഞാടാ’വാന്‍ എന്തുകൊണ്ടും യോഗ്യരാണ്?!.

വിമര്‍ശകരുടെ കാപട്യം

വിമര്‍ശനവിധേയമായ ഹദീഥുകളുമായി ബന്ധപ്പെട്ട് മ്ലേച്ഛഭാവനകളുടെ മേച്ചില്‍പ്പുറങ്ങള്‍ തേടിയിറങ്ങിയ ഇസ്‌ലാംവിരോധികള്‍ ബോധപൂര്‍വ്വം മറച്ചുപിടിച്ച ചില വസ്തുതകള്‍ കൂടി നാം തിരിച്ചറിയേണ്ടിയിരിക്കുന്നു. 1) ഹദീഥുകളില്‍ പരാമര്‍ശിക്കപ്പെട്ട ഈ ഇളവ് കേവലം സാലിമിന്റെ വിഷയത്തില്‍ മാത്രം പരിമിതമാണ്. അതൊരു പൊതുവിധിയല്ലെന്നര്‍ത്ഥം. അതിനാല്‍ സഹ്‌ലയല്ലാത്ത ഒരു മുസ്‌ലിം സ്ത്രീക്കും ആ ഇളവ് ബാധകമല്ല. ‘മുഹമ്മദ് നബി(സ)യുടെ ഈ അദ്ധ്യാപനം പിന്‍പറ്റാന്‍ മാന്യതയുള്ള മുസ്‌ലിം സ്ത്രീകള്‍ക്കു സാധിക്കുമോ’ എന്നു ചോദിക്കുന്നവരുടെ ആശങ്കകള്‍ അടിസ്ഥാന രഹിതമാണ്. സ്വഹീഹു മുസ്‌ലിമില്‍ തന്നെ ആ ഇളവ് സാലിമിന് മാത്രമുള്ളതാണെന്ന് വ്യക്തമാക്കുന്ന നിവേദം കാണാവുന്നതാണ്. പ്രവാചക പത്‌നി ഉമ്മുസലമ (റ) പറയാറുണ്ടായിരുന്നു: ”ആ നിലക്ക് മുലകുടി ബന്ധത്തിലുള്ള (രണ്ടു വയസ്സിനുള്ളില്‍ മുലകുടി ബന്ധം സ്ഥാപിതമായിട്ടില്ലാത്ത) ഒരാളും നബി (സ) യുടെ ഭാര്യമാരുടെ അരികെ പ്രവേശിക്കുന്നതിനെ അവര്‍ വിസമ്മതിച്ചിരുന്നു. അവര്‍ ആഇശ(റ)യോട് പറഞ്ഞു: ‘ഇത് സാലിമിന്നു മാത്രമായി റസൂല്‍ (സ) പറഞ്ഞ ഒരു ഇളവാകുന്നു. അതിനാല്‍ ആ നിലക്ക് മുലകുടി ബന്ധമുള്ള ഒരാളും നമ്മുടെ അടുത്ത് പ്രവേശിക്കേണ്ടതില്ല. നമ്മളെ കാണുകയും ചെയ്യേണ്ട.” (സ്വഹീഹു മുസ്‌ലിം: 1454, മുസ്‌നദു അഹ്‌മദ്: 26660, സുനനു അബൂദാവൂദ്: 2061)

2) തന്റെ ഭാര്യയുടെ അടുത്ത് വളര്‍ത്തുപുത്രന്‍ സാലിം പ്രവേശിക്കുന്നത്- ദത്തുപുത്ര സമ്പ്രദായം അവസാനിപ്പിക്കപ്പെട്ടതോടെ- മതപരമായി അനുവദനീയമല്ലാതായി. അതുകൊണ്ടാണ് അബൂഹുദൈഫഃക്ക് സാലിമിന്റെ പ്രവേശനം നീരസമുള്ളതായി മാറിയത്. ആ നീരസം മാറാന്‍ വേണ്ടിയാണ് മുലപ്പാല്‍ നല്‍കി മാതൃബന്ധം സ്ഥാപിക്കുവാന്‍ പ്രവാചകന്‍ (സ) ഉപദേശിച്ചത്. അതേസമയം സ്തനങ്ങളില്‍ നിന്ന് നേരിട്ട് കുടിക്കാനായിരുന്നു നബി(സ)യുടെ ഉപദേശമെങ്കില്‍ അത് അബൂഹുദൈഫഃയുടെ നീരസവും മനഃപ്രയാസവും അധികരിപ്പിക്കുകയാണുണ്ടാവുക. ഈ വസ്തുത വിമര്‍ശകര്‍ ബോധപൂര്‍വ്വം മറച്ചുപിടിക്കുകയാണ്. പ്രവേശനം തന്നെ നീരസവും മനഃപ്രയാസവും സൃഷ്ടിക്കുന്ന ഒരാളെ കൊണ്ട് തന്നെ, സ്തനങ്ങളില്‍ നിന്ന് നേരിട്ട് മുലപ്പാല്‍ കുടിക്കുവാന്‍ കല്‍പ്പിക്കില്ലെന്ന് ഏതു വിഡ്ഢിക്കാണ് മനസ്സിലാകാതിരിക്കുക?!

‘മുലയൂട്ടല്‍’ ആഇശ(റ)യുടെ പേരിലെ പച്ചക്കള്ളങ്ങള്‍

അശ്ലീല ഭാവനകള്‍ കൊണ്ട് ഹദീഥുകളെ ദുഷിപ്പിച്ചവതരിപ്പിക്കുക മാത്രമല്ല, പച്ചകള്ളങ്ങള്‍ പോലും ഇസ്‌ലാമിനെതിരെ പടച്ചുണ്ടാക്കാന്‍ വരെ മടിയില്ലാതായിരിക്കുന്നു ഇസ്‌ലാംവിമര്‍ശകര്‍ക്ക്. ലജ്ജാവഹവും നിന്ദാപരവുമായ ഇത്തരം ഏര്‍പ്പാടുകളെ ‘സുവിശേഷ വേല’ എന്നുപേരിട്ടു വിളിക്കുന്നത് അല്പം കടന്ന കയ്യാണ്. ‘മുലയൂട്ടല്‍’ വിവാദവുമായി ബന്ധപ്പെട്ട് പ്രവാചക പത്‌നി ആഇശ(റ)ക്കെതിരെ മിഷണറി നുണഫാക്ടറികള്‍ ഉല്‍പ്പാദിപ്പിച്ച അത്തരം രണ്ടു കളവുകള്‍ നാം കാണുക.

‘പ്രവാചക പത്‌നി ആഇശ (റ) അത്തരത്തില്‍ പരപുരുഷന്മാരെ മുലയൂട്ടാറുണ്ടായിരുന്നുവെന്നും പലപ്പോഴും ആഇശയുടെ അരികില്‍ പരപുരുഷന്മാരെ കാണുക വഴി നബി(സ)ക്കുതന്നെ അതില്‍ അനിഷ്ടമുണ്ടാവുകയും ചെയ്തിരുന്നു എന്ന് സ്വഹീഹു മുസ്‌ലിമിലെ 1455-ാം ഹദീഥായി രേഖപ്പെടുത്തിയിട്ടുണ്ട്’. ഇതാണ് സോഷ്യല്‍ മീഡിയ വഴി ചില മിഷണറി നുണ ഫാക്ടറികള്‍ നിരന്തരം പ്രചരിപ്പിച്ചതും, യുക്തിവാദികളും ഫെമിനിസ്റ്റുകളും ഫാഷിസ്റ്റുകളുമടങ്ങുന്ന ഇസ്‌ലാംവിരോധികള്‍ ഏറ്റെടുത്തതുമായ ഒന്നാമത്തെ കളവ്. സ്വഹീഹു മുസ്‌ലിമിലെ ആ നിവേദനം നാം ആദ്യം കാണുക. ”മസ്‌റൂഖ് (റ) നിവേദനം. ആഇശ (റ) പറഞ്ഞതായി അദ്ദേഹം പറഞ്ഞു: റസൂല്‍ (സ) എന്റെ അടുത്ത് കടന്നുവന്നു. എന്റെ അരികെ ഒരാള്‍ ഇരിക്കുന്നുണ്ടായിരുന്നു? അത് നബി(സ)ക്ക് അതിയായ പ്രയാസമുണ്ടാക്കി. നബി(സ)യുടെ മുഖത്ത് ഞാന്‍ കോപം കണ്ടു. അപ്പോള്‍ ഞാന്‍ ചോദിച്ചു: ‘അല്ലാഹുവിന്റെ പ്രവാചകരെ, നിശ്ചയം ഇദ്ദേഹം മുലകുടി ബന്ധത്തില്‍ എന്റെ സഹോദരനാകുന്നു’. അപ്പോള്‍ നബി (സ) പറഞ്ഞു: നിങ്ങള്‍ മുലകുടി ബന്ധത്തിലുള്ള സഹോദരന്മാരെപ്പറ്റി ശരിക്കും നോക്കണം. വിശപ്പടങ്ങുന്ന നിലക്ക് മുല കുടിച്ചാല്‍ മാത്രമെ മുലകുടി ബന്ധം ഉണ്ടാവുകയുള്ളൂ”. (സ്വഹീഹു മുസ്‌ലിം: 1455) ഈ സംഭവത്തെ ദുര്‍വ്യാഖ്യാനിച്ചുകൊണ്ടാണ് ആഇശ (റ) പരപുരുഷന്മാരെ മുലയൂട്ടിയിരുന്നെന്ന് പച്ചകള്ളം ഇവര്‍ പ്രചരിപ്പിച്ചത്. ഹദീഥുകള്‍ നേര്‍ക്കുനേരെ ഒരാവര്‍ത്തി വായിച്ചാല്‍ തന്നെ അതൊരു ദുരാരോപണം മാത്രമാണെന്ന് മനസ്സിലാക്കാന്‍ കഴിയും. ‘അല്ലാഹുവിന്റെ പ്രവാചകരേ, നിശ്ചയം ഇദ്ദേഹം മുലകുടി ബന്ധത്തില്‍ എന്റെ സഹോദരനാകുന്നു’ എന്ന ആഇശ(റ)യുടെ വാക്കുകള്‍ സ്പഷ്ടമായും മനസ്സിലാക്കി തരുന്നത്; ആഇശ(റ)യും അവരുടെ സമീപത്തിരുന്ന വ്യക്തിയും കുഞ്ഞായിരുന്നപ്പോള്‍ ഒരേ സ്ത്രീയുടെ മുലപ്പാല്‍ കുടിച്ചിട്ടുണ്ട് എന്നതാണ്. അല്ലാതെ ആഇശ (റ) അദ്ദേഹത്തെ ‘മുലയൂട്ടി’ എന്നല്ല. ഒന്നുകില്‍ ആഇശ (റ) കൈകുഞ്ഞായിരിക്കുമ്പോള്‍ അദ്ദേഹത്തിന്റെ മാതാവിന്റെ മുലപ്പാല്‍ കുടിച്ചിട്ടുണ്ടാകും. അല്ലെങ്കില്‍ അദ്ദേഹം ശൈശവ ഘട്ടത്തില്‍ ആഇശ(റ)യുടെ മാതാവിന്റെ പാല്‍കുടിച്ചതുമാകാം. ഇതുരണ്ടുമല്ലെങ്കില്‍ അവര്‍ രണ്ടുപേരും മറ്റേതോ ഒരു സ്ത്രീയുടെ പാല്‍കുടിച്ചിട്ടുണ്ടാകാം. അതുകൊണ്ടാണ് ആഇശ(റ)ക്കും അദ്ദേഹത്തിനുമിടയില്‍ സഹോദര ബന്ധം സ്ഥാപിതമായത്. ഈ സംഭവത്തെ ദുര്‍വ്യാഖ്യാനിച്ചുകൊണ്ടാണ് ഈ ദുരാരോപണം ഇസ്‌ലാംവിരോധികള്‍ പ്രചരിപ്പിക്കുന്നത്. ആഇശ (റ) പ്രസവിക്കാത്ത ഒരു സ്ത്രീയായിരുന്നു. അതുകൊണ്ടുതന്നെ അവര്‍ക്ക് ഒരു കുഞ്ഞിനും മുലയൂട്ടാന്‍ സാധ്യവുമല്ല. ഈ വസ്തുതകളെല്ലാം അറിഞ്ഞിട്ടും വിമര്‍ശകര്‍ ഇത്തരം കള്ളപ്രചരണം നടത്തിക്കൊണ്ടിരിക്കുന്നത്, ഇസ്‌ലാംവിരോധം അവരുടെ മനഃസാക്ഷിയെപോലും കൊന്നുകളഞ്ഞിരിക്കുന്നു എന്നതിന്റെ സൂചനയാണ്. എങ്കില്‍പിന്നെ പ്രവാചകന്‍ കോപിച്ചതെന്തിനാണ്? അതിന്റെ ഉത്തരവും ഹദീഥിലുണ്ട്. ‘നിങ്ങള്‍ മുലകുടി ബന്ധത്തിലുള്ള സഹോദരന്മാരെപ്പറ്റി ശരിക്കും നോക്കണം. വിശപ്പടങ്ങുന്ന നിലക്ക് മുലകുടിച്ചാല്‍ മാത്രമേ മുലകുടി ബന്ധം ഉണ്ടാവുകയുള്ളു’ എന്ന നബി(സ)യുടെ വിശദീകരണം അതിന്റെ മറുപടിയാണ്. മുലപ്പാല്‍ കുടിക്കുന്നതിലൂടെ വിശപ്പടങ്ങുന്ന പ്രായത്തിലുള്ള മുലകുടിയെ മാത്രമേ ഇസ്‌ലാം – വിവാഹബന്ധം നിഷിദ്ധമായ- സഹോദരബന്ധമായി പരിഗണിക്കുകയുള്ളൂ. അത് രണ്ടു വയസ്സിനുള്ളില്‍ അഞ്ച് തവണയെങ്കിലും നടന്നിരിക്കണം. മുലകുടിയുമായി ബന്ധപ്പെട്ട ഈ നിബന്ധനകള്‍ എല്ലാം യോജിക്കുന്നതാണോ നിങ്ങള്‍ക്കിടയിലുള്ള സഹോദര ബന്ധങ്ങള്‍ എന്ന് നിങ്ങള്‍ ഉറപ്പു വരുത്തണമെന്ന കാര്‍ക്കശ്യമാണ് പ്രവാചക(സ)ന്റെ കോപത്തിനു പിന്നിലെ കാരണം. അല്ലാതെ ആഇശ(റ)യുടെ സമീപത്ത് അവരുടെ മുലകുടി ബന്ധത്തിലെ സഹോദരനെ കണ്ടപ്പോള്‍ എന്തെങ്കിലും സംശയം ഹൃദയത്തില്‍ വന്നുപോയതുകൊണ്ടല്ല. കാരണം ആഇശ (റ) അത്തരം ബന്ധുക്കള്‍ (മുലകുടിയിലൂടെയുള്ള ബന്ധുക്കള്‍) അവരെ സന്ദര്‍ശിക്കാന്‍ അനുവാദം തേടുമ്പോള്‍ വളരെ സൂക്ഷ്മതയോടെ മാത്രമേ അനുവാദം നല്‍കാറുള്ളൂ എന്ന് നബി(സ)ക്ക് അറിയാമായിരുന്നു. ഒരു സംഭവം നാം കാണുക: ”ആഇശ (റ) പറഞ്ഞു: മുലകുടി ബന്ധത്തിലൂടെയുള്ള എന്റെ പിതാമഹന്‍ എന്റെ അടുക്കല്‍ (വീട്ടില്‍)കയറാന്‍ സമ്മതം ചോദിച്ചു. അപ്പോള്‍ അല്ലാഹുവിന്റെ ദൂതരുടെ സമ്മതം കിട്ടാത്തതിനാല്‍ ഞാന്‍ വിസ്സമ്മതിച്ചു. അപ്പോള്‍ അല്ലാഹുവിന്റെ ദൂതന്‍ (സ) പറഞ്ഞു: നിന്റെ പിതാമഹനെ നീ അടുത്ത് പ്രവേശിപ്പിച്ചുകൊള്ളുക…” (സ്വഹീഹു മുസ്‌ലിം: 1445)

വിവാഹബന്ധം നിഷിദ്ധമായവര്‍ മാത്രമേ തന്റെ വീട്ടില്‍ പ്രവേശിക്കാവൂ എന്ന കാര്യത്തില്‍ അതീവ ശ്രദ്ധ പുലര്‍ത്തിയിരുന്ന ഒരു വ്യക്തിത്വമായിരുന്നു ആഇശ (റ). അല്ലാത്ത പുരഷന്മാരെ മറക്കു പിറകില്‍ നിന്നുകൊണ്ടല്ലാതെ അവര്‍ അഭിസംബോധന ചെയ്തിട്ടില്ല. അത്തരം ഒരു വ്യക്തിയെ പറ്റി ഇത്രമാത്രം ദുഷിച്ച കള്ളകഥകള്‍ പ്രചരിപ്പിക്കുവാന്‍ മിഷണറിമാര്‍ക്കല്ലാതെ മറ്റാര്‍ക്കു കഴിയും. ഇതിനെയെല്ലാം നാം ‘സുവിശേഷ വേല’ എന്നുതന്നെ വിളിക്കണോ?!. ദുഷിച്ച ഭാവനകള്‍കൊണ്ട് ഹദീഥുകളില്‍ നിന്നും അശ്ലീലതകള്‍ മെനഞ്ഞെടുത്ത ‘സുവിശേഷ വേല’യാണ് നാം മുകളില്‍ കണ്ടതെങ്കില്‍, വ്യാജതൂലികകള്‍ കൊണ്ട് ഹദീഥുകളില്‍ നേര്‍ക്കുനേരെ കൃത്രിമത്വം നടത്തുന്ന ‘സുവിശേഷ വേല’യാണ് ഇനി കാണാനുള്ളത്. വിഷയം ഒരു ദുര്‍ബലമായ ഹദീഥാണ്. മുസ്‌ലിംകള്‍ പ്രമാണമായി കാണാത്ത ഒരു വാറോല. പക്ഷെ അതില്‍പ്പോലും പച്ചയായ കാപട്യം കാണിക്കുവാന്‍ മിഷണറികള്‍ക്ക് മടിയില്ലെന്നതാണ് അതിശയകരമായ സംഗതി.

‘കല്ലെറിയലിന്റെ വചനവും പത്ത് പ്രാവശ്യമാണ് മുതിര്‍ന്ന പുരുഷന്മാര്‍ക്ക് മുലകുടിയെന്ന വചനവും (ആദ്യകാലത്ത് കുര്‍ആനില്‍) അവതരിപ്പിക്കപ്പെട്ടിരുന്നുവെന്നും, അതെഴുതിയ രേഖ തന്റെ തലയിണക്കടയില്‍ സൂക്ഷിച്ചുവെച്ചിരുന്നെന്നും പ്രവാചകന്‍ മരണപ്പെട്ടപ്പോള്‍, ഞങ്ങളെല്ലാം അതുമായി ബന്ധപ്പെട്ട തിരക്കുകള്‍ക്കിടയിലായിരുന്ന സന്ദര്‍ഭത്തില്‍ ഒരു ആട് അകത്ത് കടന്ന് അത് തിന്നുകളഞ്ഞെന്നും ആഇശയില്‍ നിന്നും സുനനു ഇബ്‌നു മാജയും മുസ്‌നദു അഹ്‌മദും നിവേദനം ചെയ്തിട്ടുണ്ടെന്നതാണ് വ്യാജമായ ആരോപണം. ആയതിനാല്‍ ദുര്‍ബലമായ ആ ഹദീഥ് നമുക്ക് ആദ്യം പരിശോധിക്കാം.

"ആഇശ (റ) പറഞ്ഞു: കല്ലെറിയലിന്റെ വചനവും പത്ത് പ്രാവശ്യമാണ് മുലകുടിയെന്ന വചനവും അവതരിക്കപ്പെട്ടിരുന്നു. അതെഴുതിയ രേഖ എന്റെ തലയിണക്കടിയിലുണ്ടായിരുന്നു. ദൈവദൂതന്‍ മരണപ്പെട്ടപ്പോള്‍, ഞങ്ങളെല്ലാം അതുമായി ബന്ധപ്പെട്ട തിരക്കുകള്‍ക്കിടയിലായിരുന്ന സന്ദര്‍ഭത്തില്‍ ഒരു ആട് അകത്ത് കടന്ന് അത് തിന്നു കളഞ്ഞു” (സുനനു ഇബ്‌നു മാജ: 1944, മുസ്‌നദു അഹ്‌മദ്: 43/343) ഹദീഥ് ദുര്‍ബലമാണ്. അതുകൊണ്ടുതന്നെ മുസ്‌ലിംകള്‍ക്ക് അത് പ്രമാണവുമല്ല. ഈ നിവേദനത്തില്‍ മുഹമ്മദ്ബ്‌നു ഇസ്ഹാഖ് എന്ന ഒരു നിവേദകനുണ്ട്. അദ്ദേഹത്തെ തെളിവിനു കൊള്ളില്ലെന്ന് ഹദീഥ് നിവേദനം ചെയ്ത അഹ്‌മദ് തന്നെ വ്യക്തമാക്കിയിട്ടുണ്ട്. അയാള്‍ ദുര്‍ബലനാണെന്ന് യഹ്‌യബിന്‍ മഈനും, ശക്തനല്ലെന്ന് നസാഇയും പറഞ്ഞിട്ടുണ്ട്. (തഹ്ദീബ് അത്തഹ്ദീബ് 9/45)

പക്ഷെ നമ്മുടെ പ്രശ്‌നം അതല്ല. ദുര്‍ബലമായ ആ നിവേദനത്തില്‍ പോലും പച്ചയായ കൃത്രിമത്വം നടത്തിയിരിക്കുകയാണ് ചില മിഷണറി നുണ ഫാക്ടറികള്‍. ‘കല്ലെറിയലിന്റെ വചനവും പത്ത് പ്രാവശ്യമാണ് മുലകുടിയെന്ന വചനവും അവതരിക്കപ്പെട്ടിരുന്നു’ എന്ന ഭാഗത്ത് മിഷണറിമാര്‍ എഴുതി പിടിപ്പിച്ചത് ‘കല്ലെറിയലിന്റെ വചനവും പത്ത് പ്രാവശ്യമാണ് മുതിര്‍ന്ന പുരുഷന്മാര്‍ക്ക് മുലകുടിയെന്ന വചനവും അവതരിപ്പിക്കപ്പെട്ടിരുന്നു എന്നാണ്. ‘മുതിര്‍ന്ന പുരുഷന്മാര്‍’ എന്നത് ‘സുവിശേഷകന്മാരു’ടെ കള്ളകോലാണ്. അവര്‍ കൃത്രിമമായി എഴുതിച്ചേര്‍ത്തു പ്രചരിപ്പിച്ചതാണെന്നര്‍ത്ഥം. എത്രമാത്രം ലജ്ജാവഹവും നിന്ദാപരവുമാണ് ഇവരുടെ പ്രവര്‍ത്തനങ്ങള്‍. ഇത്തരം ചവറുകളാണ്, ‘സയന്‍സ് ടെമ്പര്‍’ ഉണ്ടാക്കാന്‍ കൂട്ടയോട്ടം നടത്തുന്ന യുക്തിവാദികളും സ്ത്രീ ശാക്തീകരണം പത്യമാക്കിയ ഫെമിനിസ്റ്റുകളും സനാതനധര്‍മ്മം സ്ഥാപിക്കാന്‍ നെട്ടോട്ടമോടുന്ന ഫാഷിസ്റ്റുകളും കൊണ്ടുനടക്കുന്നതെന്നോര്‍ക്കുമ്പോള്‍ സഹതാപം തോന്നുന്നു. ‘ദൈവം സ്‌നേഹ’മാണെന്ന് നാഴികക്ക് നാല്‍പ്പത് വട്ടവും പാട്ടുപാടുന്നവര്‍ക്കെങ്കിലും, അവര്‍ പാടിയതിലും പറഞ്ഞതിലും തെല്ല് ആത്മാര്‍ത്ഥത ഉണ്ടായിരുന്നെങ്കില്‍ എത്ര നന്നായിരുന്നു.

‘മുലയൂട്ടല്‍’ ആഇശ(റ)യുടെ നിലപാടെന്തായിരുന്നു

വിവാഹബന്ധം നിഷിദ്ധമാക്കപ്പെടുന്ന ബന്ധങ്ങള്‍ സ്ഥാപിക്കാനായി ആഇശ (റ) തന്റെ സഹോദരി പുത്രിമാരോടും സഹോദര പുത്രിമാരോടും, മുതിര്‍ന്ന പുരുഷന്മാര്‍ക്ക് മുലപ്പാല്‍ നല്‍കുവാന്‍ ഉപദേശിച്ചിരുന്നു എന്നതാണ് വിഷയ സംബന്ധിയായ അടുത്ത വിമര്‍ശനം. വാസ്തവത്തില്‍ ആഇശ (റ) അപ്രകാരം ആരോടെങ്കിലും ഉപദേശിക്കുകയോ നിര്‍ദ്ദേശിക്കുകയോ ചെയ്തിരുന്നോ? പ്രസ്തുത വിഷയം ചര്‍ച്ച ചെയ്യും മുന്‍പ് ഇസ്‌ലാമില്‍ മുലകുടി ബന്ധം എപ്രകാരമാണ് സ്ഥാപിതമാകുന്നതെന്ന് നാം മനസ്സിലാക്കേണ്ടിയിരിക്കുന്നു. എങ്കില്‍ മാത്രമേ ആഇശ(റ)യുടെ നിലപാട് എന്തായിരുന്നു എന്ന് നമുക്ക് വേര്‍തിരിച്ചു മനസ്സിലാക്കാനാകൂ. രണ്ടു വയസ്സിനുള്ളില്‍, വിശപ്പ് അടങ്ങുംവിധം അഞ്ചു തവണയെങ്കിലും മുലപ്പാല്‍ നല്‍കിയാല്‍ മാത്രമാണ്, വിവാഹം നിഷിദ്ധമാക്കപ്പെടുന്ന ബന്ധം സ്ഥാപിതമാവുകയുള്ളൂ എന്നതാണ് ഈ വിഷയകമായി ഇസ്‌ലാമിക നിയമം. അഥവാ കുഞ്ഞുങ്ങള്‍ക്ക് മുലയൂട്ടുന്നതിലൂടെ മാത്രമേ മുലകുടി ബന്ധം സ്ഥാപിതമാകൂ എന്നതാണ് ഇസ്‌ലാമിക വിധി.

”മാതാക്കള്‍ തങ്ങളുടെ സന്താനങ്ങള്‍ക്ക് പൂര്‍ണ്ണമായ രണ്ടുകൊല്ലം മുല കൊടുക്കേണ്ടതാണ്. (കുട്ടിയുടെ) മുലകുടി പൂര്‍ണ്ണമാക്കണം എന്ന് ഉദ്ദേശിക്കുന്നവര്‍ക്കത്രെ ഇത്”. (ക്വുര്‍ആന്‍: 2:233) ”അവന്റെ മുലകുടി നിര്‍ത്തുന്നതാകട്ടെ രണ്ടുവര്‍ഷം കൊണ്ടുമാണ്” (ക്വുര്‍ആന്‍: 31:14) ”അവന്റെ ഗര്‍ഭകാലവും മുലകുടി നിര്‍ത്തലും കൂടി മുപ്പത് മാസക്കാലമാകുന്നു” (ക്വുര്‍ആന്‍: 46:15) ”വയറിനെ പുഷ്ടിപ്പിക്കുന്നതില്‍ അല്ലാതെ രിദ്വാഅ് (പാലൂട്ടല്‍) ഇല്ല” (ഇബ്‌നുമാജ: 1946, ഇബ്‌നുഹിബ്ബാന്‍: 4225) ”അസ്ഥിയെ ശക്തിപ്പെടുത്തുകയും മാംസം പുഷ്ടിപ്പെടുത്തുകയും ചെയ്യുന്നതല്ലാത്തത് രിദ്വാഅ് അല്ല” (സുനനുല്‍ ക്വുബ്‌റാ, ബൈഹകി: 15653) ”രണ്ടു വയസ്സിനുള്ളില്‍ ഊട്ടുന്നതല്ലാത്ത രിദ്വാഅ് ഇല്ല.” (സുനനുദ്ദാറക്വുത്‌നി : 4364) ”വിശപ്പടങ്ങുന്ന കുട്ടി മാത്രമാണ് (ബന്ധം സ്ഥാപിക്കുന്ന) രിദ്വാഅ് (മുലകുടി).” (സ്വഹീഹു മുസ്‌ലിം:32, അബൂദാവൂദ് : 12/8 നസാഈ : 26/51, ഇബ്‌നുമാജ : 9/37, അഹ്‌മദ്: 6/93) മുലപ്പാല്‍ കുടിക്കുന്നതിലൂടെ വിശപ്പടങ്ങുക കുഞ്ഞുങ്ങള്‍ക്കാണ്. അതുകൊണ്ടുതന്നെ മുലകുടിബന്ധം സ്ഥാപിതമാവുക കുഞ്ഞായിരിക്കുമ്പോള്‍ മുലപ്പാല്‍ കുടിക്കുന്നതിലൂടെ മാത്രമാണ് എന്ന് ഹദീഥിനെ വ്യാഖ്യാനിച്ച പണ്ഡിതന്മാരെല്ലാം വ്യക്തമാക്കുന്നുണ്ട്. (ശര്‍ഹു മുസ്‌ലിം: 1/434, ഫത്ഹുല്‍ ബാരി: 1/100, ഉംദത്തുല്‍ കാരി: 20/97, ഇര്‍ശാദുസ്സരി: 8/33, ഫൈദുല്‍ കദീര്‍: 3/360, മിര്‍ക്കാത്തുല്‍ മഫാത്തീഹ്: 3168, ശര്‍ഹു സര്‍ക്കാനി അലാല്‍ മുവത്വഃ: 3/374, ഹാശിയത്തു സിന്ദി അലാ സുനനി ഈബ്‌നുമാജ: 1/600, ഔനുല്‍ മഅ്ബൂദ്: 6/47) പ്രവാചകാനുചരന്മാരായ ഉമ്മര്‍, അലി, അബ്ദുല്ലാഹിബ്‌നു മസ്ഊദ്, അബ്ദുല്ലാഹിബ്‌നു അബ്ബാസ്, ഇബ്‌നു ഉമ്മര്‍, അബൂഹുറൈറ, ജാബിര്‍, പ്രവാചക പത്‌നിമാര്‍ തുടങ്ങി ഹസനുല്‍ ബസ്വരി, സുഹ്‌രി, ഇക്‌രിമ, ഔസാഈ, ശുഅ്ബി, കത്താദ, ഇബ്‌നു ശബ്‌റമ, ഇബ്‌നു അബീ ലൈല, ഹസനിബ്‌നു സ്വാലിഹ്, ഇബ്‌നു അബി ദിഅ്ബ്, സൗരി, ഇസ്ഹാക്, അബൂസൗര്‍, അബൂ ഉബൈദ്, ത്വബ്‌രി, ലൈസ് എന്നിങ്ങനെ പൗരാണിക കര്‍മ്മശാസ്ത്ര പണ്ഡിതരായ സര്‍വ്വരുടേയും അഭിപ്രായവും ഇതുതന്നെയാണ്. (മുഖ്തസറു ഇഖ്തിലാഫുല്‍ ഉലമ: 2/314, അല്‍ഹാവി അല്‍കബീര്‍: മാവര്‍ദി 11/366, മുഹല്ലാ: 10/17, മുഗ്നി: 11/319, സാദുല്‍ മആദ്: 5/577, തഫ്‌സീറു ഇബ്‌നു കസീര്‍ : 1/284, ഉംദത്തുല്‍ കാരി : 20/85) ഹനഫീ, മാലികീ, ശാഫിഈ, ഹമ്പലി കര്‍മ്മശാസ്ത്ര മദ്ഹബുകളുടെ പണ്ഡിതന്മാരുടെ അഭിപ്രായവും തഥൈവ. (അല്‍ മബ്‌സൂത്: 5/135, അല്‍ മുദവ്വനത്തുല്‍ കുബ്‌റാ: 5/407, അല്‍ ഉമ്മ്: 5/428, മുഗ്‌നി: 11/319)

അപ്പോള്‍ ഇസ്‌ലാമിന്റെ വിധി മുതിര്‍ന്നവര്‍ക്ക് മുലകുടി ഇല്ല എന്നതാണ്. എന്നാല്‍ സാലിമിന്റെ വിഷയത്തില്‍ ഉണ്ടായത് തീര്‍ത്തും അവരുടെ വിഷയത്തില്‍ മാത്രം ബാധകമായ ഒരു ഇളവു മാത്രമായിരുന്നു. അതും മുലപ്പാല്‍ സ്തനങ്ങളില്‍ നിന്നും നേരിട്ടു നല്‍കാതെ, പാല്‍ പിഴിഞ്ഞ് തളികയിലാക്കി കൊടുക്കുക മാത്രമാണുണ്ടായതെന്ന് നാം മുമ്പ് സൂചിപ്പിച്ചിട്ടുണ്ട്. എന്നാല്‍ ‘സാലിമിന്റെ വിഷയത്തിലെ പ്രവാചക നടപടി തെളിവ് പിടിച്ചുകൊണ്ട് ആഇശ (റ) തന്റെ വീട്ടില്‍ പ്രവേശിക്കാന്‍ താല്‍പര്യപ്പെടുന്ന മുതിര്‍ന്നവര്‍ക്ക് മുലപ്പാല്‍ നല്‍കാനായി തന്റെ സഹോദര പുത്രിമാരോടും, സഹോദരി പുത്രിമാരോടും നിര്‍ദ്ദേശിക്കാറുണ്ടായിരുന്നു എന്നും, അപ്രകാരം അവര്‍ (സഹോദര- സഹോദരി പുത്രിമാര്‍) അഞ്ചുതവണ മുലപ്പാല്‍ നല്‍കി മുലകുടിബന്ധം സ്ഥാപിതമായതിന് ശേഷമേ ആഇശ (റ) അവരെ തന്റെ വീട്ടില്‍ പ്രവേശിപ്പിക്കാറുള്ളൂ’ (സുനനു അബീദാവൂദ്: 2061) എന്ന നിവേദനത്തെ ദുര്‍വ്യാഖ്യാനിച്ചു കൊണ്ടാണ് ഇസ്‌ലാമില്‍ മുതിര്‍ന്ന പുരുഷന്മാര്‍ക്കും മുലകുടിയുണ്ടെന്ന വ്യാജപ്രചരണം ഇസ്‌ലാംവിരോധികള്‍ അഴിച്ചുവിട്ടു കൊണ്ടിരിക്കുന്നത്. എന്നാല്‍ ഇവിടെ നാം പ്രത്യേകം മനസ്സിലാക്കേണ്ടുന്ന ചില കാര്യങ്ങളെ ഇപ്രകാരം സംഗ്രഹിക്കാം:

ഒന്ന്: ആഇശ (റ) ആര്‍ക്കും മുലപ്പാല്‍ – സ്തനങ്ങളില്‍ നിന്ന് നേരിട്ടോ അല്ലാതെയോ കൊടുത്തിട്ടില്ല. ഒരിക്കലും പ്രസവിച്ചിട്ടില്ലാത്ത അവര്‍ക്ക് എങ്ങനെ മുലപ്പാലുണ്ടാകും? അതുകൊണ്ടാണ് മുലപ്പാല്‍ നല്‍കാനായി ആഇശ (റ) തന്റെ സഹോദരി പുത്രിമാരോടും സഹോദര പുത്രിമാരോടും നിര്‍ദ്ദേശിച്ചത്. ആഇശ (റ) മുതിര്‍ന്ന പുരുഷന്മാരെ മുലയൂട്ടിയെന്ന മിഷണറി പ്രചരണം കല്ലുവെച്ച കളവാണെന്ന് വ്യക്തം.

രണ്ട്: സാലിമിന് സഹ്‌ല മുലപ്പാല്‍ നല്‍കിയത് സ്തനങ്ങളില്‍ നിന്ന് നേരിട്ടല്ലെന്നും, ഒരു തളികയില്‍ പാല്‍ പിഴിഞ്ഞ് കുടിക്കാന്‍ കൊടുക്കുന്ന രീതി – ‘വുജൂര്‍’ (الوجور) – യാണ് അവലംബിച്ചിരുന്നതെന്നും നാം മുമ്പ് വ്യക്തമായ തെളിവുകളുടെ അടിസ്ഥാനത്തില്‍ സ്ഥാപിച്ചതാണ്. അതെ രീതിയില്‍ (‘വുജൂര്‍’) തന്നെ മുലപ്പാല്‍ കൊടുക്കാനാണ് ആഇശ (റ) തന്റെ സഹോദര പുത്രിമാരോടും, സഹോദരി പുത്രിമാരോടും നിര്‍ദ്ദേശിച്ചതെന്ന വസ്തുത മറച്ചുവെച്ചുകൊണ്ട്, ആഇശ(റ)യുടെ നിര്‍ദ്ദേശം – (സഹോദര, സഹോദരി പുത്രിമാരോട്) നേരിട്ട് സ്തനങ്ങളില്‍ നിന്നും മുലയൂട്ടുവാനായിരുന്നു എന്ന പച്ചക്കള്ളം പ്രചരിപ്പിക്കുകയാണ് ഇസ്‌ലാംവിമര്‍ശകര്‍ ചെയ്യുന്നത്. സാലിമിന്റെ വിഷയത്തിലെ പ്രവാചക നടപടി തെളിവ് പിടിച്ചുകൊണ്ട്, മുലകുടി പ്രായം (രണ്ടു വയസ്സ്) പിന്നിട്ട ഒരാള്‍ക്ക് മുലപ്പാല്‍ നല്‍കുക വഴി വിവാഹം നിഷിദ്ധമാക്കപ്പെടുന്ന ബന്ധം സ്ഥാപിക്കാമെന്ന് വിലയിരുത്തിയ ആഇശ (റ), പക്ഷെ ആ വിഷയത്തില്‍ പ്രവാചകന്‍ നിര്‍ദ്ദേശിച്ച മുലപ്പാല്‍ നല്കുന്ന രീതി (വുജൂര്‍) പരിഗണിച്ചില്ലെന്നത് കേവലം വിമര്‍ശകരുടെ ലൈംഗിക ഭാവന മാത്രമാണ്.

മൂന്ന്: ഇസ്‌ലാമില്‍ ബന്ധം സ്ഥാപിതമാക്കുന്നത് മൂന്ന് മാര്‍ഗങ്ങളിലൂടെയാണ്. പ്രസവം, വിവാഹം, മുലകുടി. ഈ ബന്ധങ്ങളില്‍ പെടാത്ത, വിവാഹബന്ധം നിഷിദ്ധമാക്കപ്പെട്ടുവെന്ന് സ്ഥിരപ്പെടാത്ത ഒരു അന്യപുരുഷനേയും തന്റെ വീട്ടില്‍ പ്രവേശിപ്പിക്കില്ലെന്ന ദൃഢനിശ്ചയം ആഇശ(റ)യുടെ കണിശമായ പാതിവ്രത്യത്തേയും പരിശുദ്ധിയേയുമാണ് തെളിയിക്കുന്നത്. അത്തരം മഹത്തരമായ ഒരു നിലപാടി നെയാണ്, തങ്ങളുടെ ദുഷിച്ച ലൈംഗിക ഭാവനയിലൂടെ അശ്ലീലതയുടെ ചായം പൂശാന്‍ മിഷണറി നുണ ഫാക്ടറികള്‍ ഒരുമ്പെട്ടതെന്നത് എത്രമാത്രം നെറികെട്ട ‘സുവിശേഷ വേല’യായിപ്പോയി.

നാല്: ‘തന്റെ വീട്ടില്‍ പ്രവേശിക്കാന്‍ താല്‍പര്യപ്പെടുന്ന മുതിര്‍ന്നവര്‍ക്ക് മുലപ്പാല്‍ നല്‍കാനായി അവര്‍ കല്‍പ്പിക്കുമായിരുന്നു’ എന്ന ഹദീഥില്‍ പരാമര്‍ശിക്കപ്പെട്ട ‘മുതിര്‍ന്നവര്‍’ (الكبير) എന്ന പദപ്രയോഗം യഥാര്‍ത്ഥത്തില്‍ മുതിര്‍ന്ന പുരുഷന്മാരെയല്ല ഉദ്ദേശിക്കുന്നത്. മറിച്ച്, മുലകുടിപ്രായം കഴിഞ്ഞ വലിയ കുട്ടികളെയാണ്. ഇത് ചില ഹദീഥുകളില്‍ വ്യക്തമായി തന്നെ പരാമര്‍ശിച്ചിട്ടുണ്ട്.

ഉമ്മുസലമ (റ) ആഇശ(റ)യോട് പറഞ്ഞു ”നിങ്ങളുടെ അടുക്കല്‍ മുലകുടി പ്രായം കഴിഞ്ഞ കുട്ടി – الغلام الايفع – പ്രവേശിക്കുന്നുണ്ടല്ലോ. (അവര്‍ക്ക് മുലപ്പാല്‍ നല്‍കിയാല്‍ മുലകുടിബന്ധം സ്ഥാപിതമാകുമെന്ന് വിശ്വസിക്കാത്തതുകൊണ്ട്) അവര്‍ എന്റെ അടുക്കല്‍ പ്രവേശിക്കുന്നത് ഞാനിഷ്ടപ്പെടുന്നില്ല. അപ്പോള്‍ അവര്‍ക്ക് മുലപ്പാല്‍ നല്‍കുക വഴി മുലകുടിബന്ധം സ്ഥാപിതമാകുമെന്നതിന് തെളിവായി ആഇശ (റ) സാലിമിന്റെ സംഭവം ഉദ്ധരിക്കുകയും ചെയ്തു…” (സ്വഹീഹു മുസ്‌ലിം : 1453, മുസ്‌നദു അഹ്‌മദ്: 25415) ഹദീഥില്‍ പരാമര്‍ശിക്കപ്പെട്ട, അല്‍ ഗുലാം അല്‍ അയ്ഫഅ് (الغلام الايفع) എന്നതുകൊണ്ട് വിവക്ഷിക്കുന്നത് പ്രായപൂര്‍ത്തിയാകാത്ത ആണ്‍കുട്ടിയെയാണെന്ന് ഇമാം നവവി (റ) ഹദീഥിനെ വ്യാഖ്യാനിക്കവെ വ്യക്തമാക്കുന്നുണ്ട്. (ശര്‍ഹു മുസ്‌ലിം: 10/33)

തന്റെ വീട്ടില്‍ പ്രവേശിക്കാന്‍ താല്‍പര്യപ്പെടുന്ന ‘പുരുഷന്മാര്‍ക്ക്’ മുലപ്പാല്‍ നല്‍കി മുലകുടിയിലെ ബന്ധുവാക്കാന്‍ ആഇശ (റ) ശ്രദ്ധിച്ചിരുന്നു എന്ന് സൂചിപ്പിക്കുന്ന ഹദീഥുകളിലെ ‘പുരുഷന്മാര്‍’ (الرجال) എന്നതുകൊണ്ട് ഉദ്ദേശിക്കുന്നതും മുലകുടി പ്രായം കഴിഞ്ഞ, എന്നാല്‍ പ്രായപൂര്‍ത്തിയായിട്ടില്ലാത്ത ആണ്‍കുട്ടികളെയാണ്. അല്ലാതെ വലിയ പുരുഷന്മാരെയല്ല. പുല്ലിംഗത്തെ (Male Gender) സൂചിപ്പിക്കാനായി രിജാല്‍ (الرجال) എന്ന് അറബിയില്‍ ഉപയോഗിക്കാറുണ്ട്. എന്ന് മാത്രമല്ല, ശാരീരികമായി പുരുഷനായി മാറിയിട്ടില്ലാത്ത ആണ്‍കുട്ടിയെ അവന്റെ ലിംഗഭേദം (Gender) പുല്ലിംഗമായതിനാല്‍ അവന്‍ ഭാവിയില്‍ പുരുഷനായി മാറും എന്നത് പരിഗണിച്ച് റജുല്‍ (الرجل) ‘പുരുഷന്‍’ എന്ന് അറബിഭാഷാ നിയമപ്രകാരം വിളിക്കാവുന്നതാണ്. ഇതിനെ അറബി അലങ്കാര ശാസ്ത്രത്തില്‍ (Rhetoric) ‘ഭാവിയില്‍ ആയി മാറുന്നതിനെ പരിഗണിച്ചുകൊണ്ടുള്ള പ്രയോഗം’ (اعتبار ما يكون) എന്നാണ് പറയുക. (അല്‍ മജാസുല്‍ മുര്‍സല്‍ ഫീ ലിസാനില്‍ അറബ്: അഹ്‌മദ് ഹിന്ദാവി അബ്ദുല്‍ ഗഫ്ഫാര്‍, കിത്താബു മിന്‍ഹാജുല്‍ വാള്വിനി ലില്‍ ബലാഗ: 3/300, ഗായത്തുല്‍ മുസൂല്‍ ഫീ ശര്‍ഹി ലുബ്ബുല്‍ ഉസൂല്‍: 1/51)

ഇത് കേവലം ഭാഷ ശാസ്ത്ര നിയമപ്രകാരമുള്ള ഒരു സമര്‍ത്ഥനമല്ല; മറിച്ച് ഹദീഥുകള്‍ വ്യക്തമായി സൂചിപ്പിച്ച ഒരു വിഷയം ഭാഷ പ്രയോഗങ്ങള്‍ക്കുപോലും അന്യമല്ലെന്ന ബോധ്യപ്പെടുത്തലാണ്. ആഇശ (റ) ഉദ്ദേശിച്ച ‘പുരുഷന്മാര്‍’ (الرجال) മുലകുടി പ്രായം പിന്നിട്ട, എന്നാല്‍ പ്രായപൂര്‍ത്തിയായിട്ടില്ലാത്ത ആണ്‍കുട്ടികളാണെന്ന് വ്യക്തമാക്കുന്ന മറ്റു ചരിത്ര നിവേദനങ്ങളും നമുക്ക് കാണാം.

(a) ”സാലിമിബ്‌നു അബ്ദുല്ലാഹിബ്‌നു ഉമര്‍ (സഹ്‌ല മുലപ്പാല്‍ നല്‍കിയ സാലിം അല്ല ഇത്) പറയുന്നു: ഞാന്‍ കുഞ്ഞായിരിക്കെ ആഇശ (റ) എന്നെ അവരുടെ സഹോദരി ഉമ്മുകുല്‍സും ബിന്‍ത് അബൂബക്കറിന്റെ അടുത്തേക്ക് അയച്ചു. എന്നിട്ടവരോട് ഇങ്ങനെ പറഞ്ഞു: ‘ഇവന് പത്തു തവണ മുലപ്പാല്‍ ഊട്ടുക എങ്കില്‍ ഇവന് എന്റെ അടുത്ത് (വീട്ടില്‍) പ്രവേശിക്കാമല്ലോ.’ സാലിം പറയുന്നു: ഉമ്മുകുല്‍സും എനിക്ക് മൂന്നു തവണ മുലപ്പാല്‍ തന്നു. പിന്നീടെനിക്ക് രോഗമായി. അതിനാല്‍ എനിക്ക് മൂന്നു തവണ മാത്രമേ മുലപ്പാല്‍ തരാന്‍ സാധിച്ചുള്ളൂ. ഉമ്മുകുല്‍സൂം എനിക്ക് പത്തു തവണ മുലപ്പാല്‍ തരാതിരുന്നത് കാരണത്താല്‍ (വലുതായപ്പോള്‍) ഞാന്‍ ആഇശയുടെ അടുത്ത് പ്രവേശിക്കില്ലായിരുന്നു” (മുവത്വഅ്: 2/603)

ചെറുപ്രായത്തില്‍ ഉമ്മുകുല്‍സൂമിന്റെ മുലപ്പാല്‍, ബന്ധം സ്ഥാപിതമാകുന്ന നിലയ്ക്ക് പൂര്‍ണമായും കുടിക്കാതിരിക്കുന്നത് മൂലം ആഇശ(റ)യുമായി മുലകുടിയിലെ ബന്ധം സ്ഥാപിതമായില്ല. അതു കാരണത്താല്‍ ആഇശ (റ) തന്നെ വലുതായപ്പോള്‍ അവരുടെ വീട്ടില്‍ പ്രവേശിപ്പിക്കില്ലായിരുന്നു എന്ന് ചുരുക്കം. വലിയ പുരുഷന്മാര്‍ മുലപ്പാല്‍ കുടിച്ചാല്‍ മുലകുടി ബന്ധം സ്ഥാപിതമാകും എന്നതായിരുന്നു ആഇശ(റ)യുടെ വാദമെങ്കില്‍ സാലിമിബ്‌നു അബ്ദുല്ലാഹിബ്‌നു ഉമ്മറിനെ വലിയ പുരുഷനായതിനുശേഷവും, സഹോദരിയുടെ മുലപ്പാല്‍ നല്‍കിയതിനുശേഷം തന്റെ വീട്ടില്‍ പ്രവേശിപ്പിക്കാമായിരുന്നില്ലേ? അപ്പോള്‍ കാര്യം വളരെ വ്യക്തമാണ്. മുലപ്പാല്‍ ഊട്ടപ്പെടുന്ന ‘രിജാല്‍’ (പുരുഷന്മാര്‍) എന്നതുകൊണ്ട് ആഇശ (റ) ഉദ്ദേശിച്ചത്, മുലകുടി പ്രായം പിന്നിട്ട, എന്നാല്‍ പ്രായപൂര്‍ത്തിയെത്തിയിട്ടില്ലാത്ത ആണ്‍കുട്ടികളെ മാത്രമാണ്.

(b) ”കുട്ടികള്‍ക്ക് (الصبيان) അവര്‍ പുരുഷന്മാരായാല്‍ തന്റെ അടുത്ത് പ്രവേശിക്കുന്നതിന് തടസ്സമാകാതിരിക്കാന്‍ – ചെറുപ്രായത്തില്‍ മുലപ്പാല്‍ ഊട്ടുവാന്‍ ആഇശ (റ) തന്റെ സഹോദരന്‍ അബ്ദുര്‍റഹ്‌മനിബ്‌നു അബൂബക്കറിനോട് കല്‍പ്പിക്കുമായിരുന്നു”. (ബദാഇഉ സ്വനാഇഅ്: 4/6, ശര്‍ഹു മുഖ്തസറു ത്വഹാവി: 3/325) ഇവിടെ ആഇശ (റ) ഉദ്ദേശിച്ച പ്രായം വളരെ വ്യക്തമാണ്. കുട്ടികള്‍ പുരുഷന്മാരായാല്‍ തന്റെ അടുത്ത് പ്രവേശിക്കുന്നതിന് തടസ്സമാകാതിരിക്കുവാന്‍- ചെറുപ്രായത്തില്‍ തന്നെ മുലപ്പാല്‍ ഊട്ടുവാനാണ് അവര്‍ നിര്‍ദേശിക്കുന്നത്. ഇവിടെയും, ‘പുരുഷന്മാര്‍’ എന്നതുകൊണ്ട് മുതിര്‍ന്ന പുരുഷന്മാരെയല്ല, പ്രായപൂര്‍ത്തിയെത്താത്ത എന്നാല്‍ മുലകുടി പ്രായം കഴിഞ്ഞ ആണ്‍കുട്ടികളെയാണ് അവര്‍ ഉദ്ദേശിച്ചതെന്ന് വ്യക്തം. ഹദീഥില്‍ ‘രിജാല്‍’ (പുരുഷന്മാര്‍) എന്നത് ലിംഗഭേദത്തെ കുറിക്കാനാണ്, പ്രായത്തെ കുറിക്കാനല്ല ആഇശ (റ) ഉപയോഗിച്ചതെന്ന് പൗരാണികവും പ്രാമാണികവുമായ അറബി ഡിക്ഷണറികളിലെല്ലാം വ്യക്തമാക്കപ്പെട്ടിട്ടുണ്ട്. (താജുല്‍ ഉറൂസ്: 29/34, അല്‍ കാമൂസുല്‍ മുഹീത്: 1/1297, ലിസാനുല്‍ അറബ്: 11/265, അല്‍ കാമില്‍ ഫില്ലുഗത്തി വല്‍ അദബ്: 1/100)

(c) വലിയ പുരുഷന്മാര്‍ക്ക് മുലപ്പാല്‍ നല്‍കുക വഴി ബന്ധം സ്ഥാപിതമാകും എന്നതല്ല, മറിച്ച് മുലകുടി പ്രായമായ രണ്ടു വയസ്സു കഴിഞ്ഞ മുതിര്‍ന്ന ആണ്‍കുട്ടികള്‍ക്ക് മുലപ്പാല്‍ നല്‍കുക വഴി മുലകുടിബന്ധം സ്ഥാപിതമാകും എന്ന് മാത്രമാണ് ആഇശയുടെ അഭിപ്രായം എന്നതിനാലാണ് അതിനെതിരെ, ആഇശ (റ)യുടെ അഭിപ്രായം തിരുത്തിക്കൊണ്ട് എതിരഭിപ്രായം പ്രകടിപ്പിച്ച ഉമ്മുസലമ (റ) ‘തൊട്ടിലില്‍ വെച്ച് തന്നെ മുലകുടിച്ചാലെ’ (حتي يرضع في المهد) മുലകുടി ബന്ധം സ്ഥാപിതമാകൂ എന്ന പദപ്രയോഗം തന്നെ ഉപയോഗിച്ചത്. (മുസ്‌നദു അഹ്‌മദ്: 6/270, മുസ്‌നദു ഇബ്‌നു ഉവാന: 3/122, സുനനു അബൂദാവൂദ്: 2/223) അതുകൊണ്ടു തന്നെ ഈ വിഷയത്തില്‍ ആഇശയുടെ ഒറ്റപ്പെട്ട അഭിപ്രായം, വലിയ പുരുഷന്മാര്‍ക്ക് മുലപ്പാല്‍ നല്‍കുക വഴി ബന്ധം സ്ഥാപിതമാകും എന്നല്ല മറിച്ച്, മുലകുടി പ്രായമായ രണ്ട് വയസ്സ് കഴിഞ്ഞ ആണ്‍കുട്ടികള്‍ക്കും മുലപ്പാല്‍ നല്‍കുക വഴി മുലകുടി ബന്ധം സ്ഥാപിതമാകും എന്നു മാത്രമാണ്. അതും തീര്‍ത്തും അവരുടെ ഒറ്റപ്പെട്ട ഒരഭിപ്രായം മാത്രമാണത്. അതുകൊണ്ടാണ് മറ്റു പ്രവാചകപത്‌നിമാരെല്ലാം അവരെ ആ വിഷയത്തില്‍ എതിര്‍ത്തതും തിരുത്തിയതും.

അതിനാല്‍ ആഇശ(റ)യുടെ ഈ ഒറ്റപ്പെട്ട അഭിപ്രായത്തെ ദുര്‍വ്യാഖ്യാനിച്ച് അശ്ലീലതകള്‍ വിതറുവാനുള്ള മിഷണറി നുണ ഫാക്ടറികളുടെ ഉദ്യമം വളരെ തരംതാഴ്ന്ന പ്രവര്‍ത്തനമായിപ്പോയി. ചെറിയ ആണ്‍കുട്ടികള്‍ക്കു മുലയൂട്ടുന്നതിലൂടെ, അതും സ്തനങ്ങളില്‍ നിന്ന് നേരിട്ടല്ലാതെ തളികയില്‍ പിഴിഞ്ഞ് കുടിപ്പിക്കപ്പെടുന്നതിലൂടെ (വുജൂര്‍) ബന്ധുക്കളായാല്‍ അവര്‍ക്ക് പ്രായപൂര്‍ത്തിയെത്തിയാലും അവരുടെ സാന്നിധ്യം അന്യപുരുഷ സാന്നിധ്യമാകില്ലല്ലോ എന്ന് കണക്കുകൂട്ടിയാണ് ഇത്തരമൊരു നയം ആഇശ (റ) സ്വീകരിച്ചത്.

‘മുലയൂട്ടല്‍’ വിവാദം ഇനിയും അവസാനിക്കാത്ത ‘സുവിശേഷ വേലകള്‍’

രണ്ടു വയസ്സിനുള്ളില്‍ വിശപ്പടങ്ങും വിധം അഞ്ചു തവണയെങ്കിലും മുലയൂട്ടിയാല്‍ മാത്രമേ, വിവാഹം നിഷിദ്ധമാക്കപ്പെടുന്ന ബന്ധം സ്ഥാപിതമാകൂ എന്ന കണിശമായ കര്‍മശാസ്ത്ര വിധി നിഷ്‌കര്‍ഷിച്ച ഒരു മതത്തെ അപകീര്‍ത്തിപ്പെടുത്താനും വൈകൃതവും പ്രാകൃതവുമായ ഒരു ധര്‍മ്മശാസ്ത്രമായി അതിനെ ചിത്രീകരിക്കുവാനും വേണ്ടി മിഷണറി നുണ ഫാക്ടറികള്‍ കൈവെച്ചത് ‘സഹ്‌ല-സാലിം മുലപ്പാലൂട്ടല്‍’ സംഭവമാണ്. സാലിമിന്റെ വിഷയത്തില്‍ മാത്രം, അദ്ദേഹത്തിന്റെ മാതാവ് സഹ്‌ലയുടെ തീവ്ര ദുഃഖത്തിന് ഒരു പോംവഴിയായി – ഇളവ് നല്‍കപ്പെട്ട ഒരു കാര്യമാണ് അതെന്ന് വ്യക്തമായിട്ടും ആ സംഭവത്തെ ഇത്രമേല്‍ മ്ലേച്ഛമായവതരിപ്പിച്ചു എന്നതു മാത്രമല്ല ‘മുലയൂട്ടല്‍’ വിവാദവുമായി ബന്ധപ്പെട്ട് ‘കുഞ്ഞാടുകള്‍’ ചെയ്ത ക്രൂരത. ഒരു കുഞ്ഞിനും ഒരിക്കലും മുലയൂട്ടാന്‍ സാധ്യമല്ലാത്ത, ഒരിക്കലും പ്രസവിച്ചിട്ടില്ലാത്ത പ്രവാചക പത്‌നി ആഇശ (റ) അത്തരത്തില്‍ ‘മുലയൂട്ടല്‍’ നടത്തിയിരുന്നു എന്ന പച്ചക്കള്ളം പടച്ചുണ്ടാക്കാന്‍ ഇസ്‌ലാമിന്റെ അടിസ്ഥാന പ്രമാണങ്ങളില്‍ ഒന്നായ ഹദീഥുകളില്‍ കൈവെച്ചു. എന്നിട്ടും അരിശം തീരാതെ ആഇശയുടെ പ്രസ്തുത വിഷയകമായ ഒറ്റപ്പെട്ട അഭിപ്രായത്തെ ഉയര്‍ത്തിക്കാട്ടി വീണ്ടും അശ്ലീലതകള്‍ക്ക് പഴുതു തപ്പി നടന്നു. അതും അവരുടെ അഭിപ്രായത്തെ വസ്തുതാവിരുദ്ധമായ നിലയില്‍ ദുര്‍വ്യാഖ്യാനിച്ചു കൊണ്ട്. ഇനിയും തീര്‍ന്നിട്ടില്ല ഇവരുടെ മഹത്തായ ‘സുവിശേഷ വേല’കള്‍. തന്റെ ഒറ്റപ്പെട്ട അഭിപ്രായത്തില്‍ നിന്നും അഥവാ മുലകുടി പ്രായം (രണ്ടു വയസ്സ്) കഴിഞ്ഞതും എന്നാല്‍ പ്രായപൂര്‍ത്തിയെത്തിയിട്ടില്ലാത്തതുമായ ആണ്‍കുട്ടികള്‍ക്ക് മുലപ്പാല്‍ തളികയില്‍ പിഴിഞ്ഞു കുടിപ്പിച്ചാല്‍ ‘ബന്ധം’ സ്ഥാപിതമാകും എന്ന അഭിപ്രായത്തില്‍ നിന്നും പിന്നീട് ആഇശ (റ) മടങ്ങിയിട്ടുണ്ട് എന്ന വസ്തുത ഈ ‘കുഞ്ഞാടുകള്‍’ മറച്ചുവെച്ചിട്ടുണ്ടെന്നതും കൂടി നാം അറിയണം. അപ്പോള്‍ നമുക്ക് മനസ്സിലാകും മിഷണറി പക്ഷം വിമര്‍ശനമാണോ അല്ല വിരോധമാണോ പ്രകടിപ്പിക്കുന്നതെന്ന്.

ഇമാം കാസാനി പറഞ്ഞു: ”എന്നാല്‍ ആഇശ (ഈ വിഷയത്തിലുള്ള) നയം അവര്‍ തിരുത്തുകയുണ്ടായി എന്ന് സൂചിപ്പിക്കുന്ന നിവേദനങ്ങള്‍ ഉദ്ദരിക്കപ്പെട്ടിട്ടുണ്ട്. അവര്‍ ഇപ്രകാരം പറഞ്ഞതായി നിവേദനം ചെയ്യപ്പെട്ടിരിക്കുന്നു: ‘രക്തവും മാംസവും മുളപ്പിക്കുന്നതല്ലാത്ത (കൈകുഞ്ഞായിരിക്കുമ്പോളല്ലാത്ത) മുലകുടി വിവാഹബന്ധം നിഷിദ്ധമാക്കില്ല (മുലകുടി ബന്ധം സ്ഥാപിതമാകില്ല)” (ബദാഇഉസ്വനാഇ: 4/6) ‘തൊട്ടിലില്‍ വെച്ചുതന്നെ മുല കുടിച്ചാലെ’ മുലകുടി ബന്ധം സ്ഥാപിതമാകൂ എന്നും (മുസ്‌നദു അഹമ്ദ്: 6/270, മുസ്‌നദു ഇബ്‌നു ഉവാന: 3/122, സുനനു അബൂദാവൂദ്: 2/223) ‘അല്ലാഹുവാണേ, സാലിമിന്റെ വിഷയത്തില്‍ മാത്രമായി (ഖാസ്) പ്രവാചകന്‍ അനുവദിച്ചു കൊടുത്ത ഇളവായിരുന്നു അത് (ആ ഇളവ് എല്ലാവര്‍ക്കും ബാധകമല്ല)’ (സ്വഹീഹു മുസ്‌ലിം: 1454, മുസ്‌നദു അഹ്‌മദ്: 26660, സുനനു അബൂദാവൂദ്: 2061) എന്നുമുള്ള പ്രവാചക പത്‌നി ഉമ്മുസലമ(റ)യുടെ തിരുത്ത് ആഇശ (റ) സ്വീകരിച്ചു എന്നും, തന്റെ ഒറ്റപ്പെട്ട പൂര്‍വ്വ അഭിപ്രായത്തില്‍ നിന്നും അവര്‍ മടങ്ങിയെന്നും ഹദീഥ് ഗ്രന്ഥങ്ങളില്‍ തന്നെ വ്യക്തമാക്കിയിരിക്കെ അതെല്ലാം മറച്ചുവച്ചുകൊണ്ട് ഇത്തരം കുത്സിത പ്രവര്‍ത്തനങ്ങള്‍ നടത്താന്‍ ‘ദൈവത്തിന്റെ കുഞ്ഞാടുകള്‍’ക്കെങ്ങിനെ കഴിഞ്ഞു. മിഷണറി നുണ ഫാക്ടറികള്‍ നിര്‍മ്മിച്ച ഈ നുണകഥയും പൊക്കിപ്പിടിച്ച് ഇപ്പോഴും യുക്തിവാദികളും ഫെമിനിസ്റ്റുകളും ഫാഷിസ്റ്റുകളുമടങ്ങുന്ന ഇസ്‌ലാംവിമര്‍ശകര്‍ ചോദിക്കുന്നു ‘പ്രവാചകന്റെ ഈ കല്പന നടപ്പിലാക്കുവാന്‍ ഈ കാലഘട്ടത്തിലെ മുസ്‌ലിം സ്ത്രീകള്‍ തയ്യാറാകുമോ? തങ്ങളുടെ ഭാര്യമാരെ കൊണ്ട് ഈ പ്രവാചക നിര്‍ദ്ദേശം നടപ്പില്‍ വരുത്തുവാന്‍ മുസ്‌ലിം പുരുഷന്മാര്‍ ഒരുക്കമാണോ?’ എന്ന്. പാവം! സാധുക്കള്‍ വല്ലതും അറിയുന്നുണ്ടോ ഇതു മിഷണറി ‘സുവിശേഷ വേല’ മാത്രമാണെന്ന കഥ.!!!

മുഹമ്മദ് നബി സ്ത്രീ പീഢകനായിരുന്നു എന്ന് വ്യക്തമാക്കുന്ന നിവേദനങ്ങള്‍ സ്വഹീഹുല്‍ ബുഖാരിയില്‍ തന്നെ കാണാം. ‘ശൗത്ത്’ എന്നു വിളിക്കപ്പെട്ടിരുന്ന ഒരു തോട്ടത്തില്‍ വെച്ച് ‘ജൗന്‍’ ഗോത്രത്തിലെ ഉമൈമ:ബിന്‍ത് ശറാഹീല്‍ എന്ന സ്ത്രീയെ പ്രവാചകന്‍ കടന്നുപിടിക്കാന്‍ ശ്രമിക്കുകയും അവര്‍ ശക്തമായി പ്രതിരോധിക്കുകയും ചെയ്ത സംഭവം ഹദീഥ് ഗ്രന്ഥങ്ങള്‍ തന്നെ റിപ്പോര്‍ട്ട് ചെയ്തിട്ടുണ്ട്. സ്ത്രീകളെ കടന്നുപിടിക്കാന്‍ പോലും മടിയില്ലാതിരുന്ന ഒരു വ്യക്തിയെ എങ്ങനെയാണ് മാനവികതയുടെ പ്രവാചകനായി വിലയിരുത്തുക?

ഇമാം ബുഖാരി തന്റെ സ്വഹീഹില്‍ വ്യത്യസ്ത പരമ്പരകളിലൂടെ ഉദ്ധരിച്ച ഒരു സംഭവത്തിന്റെ ഏതാനും ഭാഗങ്ങള്‍ അടര്‍ത്തിയെടുത്ത് പച്ചക്ക് ദുര്‍വ്യാഖാനിച്ചിരിക്കുകയാണിവിടെ. ഉമൈമ: ബിന്‍ത് ശറാഹീലുമായി ബന്ധപ്പെട്ട് ഇമാം ബുഖാരി അദ്ദേഹത്തിന്റെ സ്വഹീഹില്‍ ഉദ്ധരിച്ച നിവേദനങ്ങള്‍ വായിച്ചാല്‍ സംഭവം പ്രവാചക ജീവിതത്തിലെ ഒരു പുഴുകുത്തല്ലെന്നും മറിച്ച് പ്രവാചകന്റെ മഹാമനസ്‌കതയും സഹിഷ്ണുതയും കാരുണ്യവും അനുവാചകര്‍ക്ക് ബോധ്യപ്പെടുക മാത്രമാണുണ്ടാവുക എന്ന ഉത്തമബോധ്യമുള്ളതുകൊണ്ടു തന്നെ ഹദീഥുകളില്‍ നിന്നും തങ്ങള്‍ക്ക് കൈവെക്കാനൊക്കുന്നതു മാത്രം തിരഞ്ഞുപിടിച്ചു ദുര്‍വ്യാഖ്യാനിച്ചു ദുഷിപ്പിക്കുക എന്ന തന്ത്രമാണ് ഇസ്‌ലാംവിമര്‍ശകര്‍ ചെയ്തുകൊണ്ടിരിക്കുന്നത്. അതിനാല്‍ പ്രസ്തുത സംഭവവുമായി ബന്ധപ്പെട്ട് ഇമാം ബുഖാരി അദ്ദേഹത്തിന്റെ സ്വഹീഹില്‍ ഉദ്ധരിച്ച ഏതാനും ചില നിവേദനങ്ങള്‍ താഴെ കൊടുക്കുകയാണ്. ഒരൊറ്റ വായനയിലൂടെ തന്നെ ഏതൊരാള്‍ക്കും എളുപ്പം ഗ്രഹിക്കാന്‍ സാധിക്കുന്ന ഒരു വിഷയത്തെ പോലും ഇത്തരത്തില്‍ ദുര്‍വ്യാഖ്യാനിക്കുവാന്‍ ഇസ്‌ലാം വിമര്‍ശകര്‍ക്ക് യാതൊരു ലജ്ജയുമില്ലെന്നത് അവരുടെ സംസ്‌കാരത്തിന് നേരെയുള്ള ചൂണ്ടുവിരലാണ്.

”ഹംസത്തിബ്‌നു അബീ ഉസൈദ് നിവേദനം: അബൂ ഉസൈദ് (റ) പറഞ്ഞു: ഒരിക്കല്‍ പ്രവാചകനോടൊപ്പം(സ) ഞങ്ങള്‍ ഒരു യാത്ര പുറപ്പെട്ടു. ‘ശൗത്ത്’ എന്ന് വിളിക്കപ്പെടുന്ന ഒരു തോട്ടത്തെ ലക്ഷ്യമാക്കി ഞങ്ങള്‍ നീങ്ങി. ഞങ്ങള്‍ രണ്ട് തോട്ടങ്ങള്‍ക്കിടയിലെത്തിയപ്പോള്‍, അവിടെ ഞങ്ങളിരുന്നു. ഞങ്ങളോട് അവിടെ ഇരിക്കാൻ പ്രവാചകൻ (സ) പറഞ്ഞു. എന്നിട്ട് അദ്ദേഹം ആ തോട്ടത്തിലേക്ക് പോയി. ജൗന്‍ ഗോത്രത്തിലെ സ്ത്രീയെ അവിടേക്ക് കൊണ്ടുവരപ്പെട്ടിരുന്നു. ഈത്തപ്പന കൊണ്ടുണ്ടാക്കിയ വീട്ടിലായിരുന്നു അവര്‍. ഉമൈമ: ബിന്‍ത് ശറാഹീല്‍ എന്നായിരുന്നു അവരുടെ നാമം. അവരോടൊപ്പം അവരുടെ മുലകുടി ബന്ധത്തിലെ പോറ്റുമ്മയും ഉണ്ടായിരുന്നു. അവരുടെ അടുത്തേക്ക് അല്ലാഹുവിന്റെ ദൂതൻ (സ) പ്രവേശിക്കുകയും ‘നീ നിന്നെ എനിക്ക് സമര്‍പ്പിക്കുക’ എന്ന് പറയുകയും ചെയ്തു. അപ്പോള്‍ അവര്‍ പറഞ്ഞു: ‘ഒരു രാജ്ഞി അവരെ ഏതെങ്കിലും ഒരു സാധാരണക്കാരന് സമര്‍പ്പിക്കുമോ?’ അവരുടെ മേല്‍ കൈവെച്ച് അവരെ ശാന്തയാക്കാനായി തന്റെ കൈകള്‍ അദ്ദേഹം നീട്ടി. അപ്പോള്‍ അവര്‍ പറഞ്ഞു: ‘നിങ്ങളില്‍ നിന്നും ഞാൻ അല്ലാഹുവില്‍ ശരണം തേടുന്നു’. അപ്പോൾ പ്രവാചകൻ (സ) പറഞ്ഞു: ‘ശരണം തേടുവാന്‍ ഏറ്റവും അർഹനായവനിലാണ് നീ ശരണം തേടിയിരിക്കുന്നത്. പിന്നീട് അദ്ദേഹം ഞങ്ങളുടെ അടുത്തേക്ക് വന്നുകൊണ്ട് പറഞ്ഞു. “അബൂ ഉസൈദ്, അവര്‍ക്ക് രണ്ട് റാസിഖിയ്യാ വസ്ത്രങ്ങള്‍ നല്‍കുകയും അവരുടെ കുടുംബത്തിലേക്ക് അവരെ തിരിച്ചെത്തിക്കുകയും ചെയ്യുക.” (ബുഖാരി: 5255)

”പ്രവാചകൻ (സ) ഉമൈമ: ബിന്‍ത് ശറാഹീലിനെ വിവാഹം ചെയ്തു. അവരെ അദ്ദേഹത്തിന്റെ അടുത്തേക്ക് പ്രവേശിക്കപ്പെട്ടപ്പോള്‍, അദ്ദേഹം തന്റെ കൈകൾ നീട്ടി സ്വീകരിച്ചു. അവര്‍ക്കത് ഇഷ്ടപെടാത്തത് പോലെ അവര്‍ പ്രതികരിച്ചു. അപ്പോള്‍ അവര്‍ക്ക് തിരികെ സ്വഗൃഹത്തിലേക്ക് പോകാന്‍ യാത്രാ സൗകര്യങ്ങള്‍ ചെയ്യാനും, രണ്ട് റാസിഖിയ്യാ വസ്ത്രങ്ങള്‍ സമ്മാനമായി നല്‍കാനും പ്രവാചകൻ (സ) അബൂ ഉസൈദിനോട് കല്പിച്ചു.” (ബുഖാരി: 5256)

”ഇമാം ഔസാഇ (റ) പറഞ്ഞു: ഞാന്‍ സുഹ്‌രിയോട് ചോദിച്ചു: ‘പ്രവാചകൻ(സ)യുടെ ഭാര്യമാരില്‍ ആരാണ് അദ്ദേഹത്തില്‍ നിന്നും ശരണം തേടിയത്?’ അദ്ദേഹം പറഞ്ഞു: ‘എന്നോട് ആഇശ(റ)യില്‍ നിന്നും ഇപ്രകാരം ഉര്‍വ അറിയിക്കുകയുണ്ടായി. ജൗന്‍ ഗോത്രക്കാരിയെ പ്രവാചകന്റെ(സ) അരികിലേക്ക് (അദ്ദേഹത്തിന്റെ പത്‌നിയായി) ആനയിക്കപ്പെടുകയും അദ്ദേഹം അവളുടെ അരികിലേക്ക് ചെല്ലുകയും ചെയ്തപ്പോള്‍ അവള്‍ പറഞ്ഞു: ‘ഞാന്‍ താങ്കളില്‍ നിന്നും അല്ലാഹുവിനോട് ശരണം തേടുന്നു.’ അപ്പോള്‍ അദ്ദേഹം പറഞ്ഞു: ‘അതിമഹത്വമുള്ളവനിലാണ് നീ ശരണം തേടിയിരിക്കുന്നത്. നീ നിന്റെ കുടുംബത്തിലേക്ക് മടങ്ങിക്കൊള്ളുക.” (ബുഖാരി: 5254)

ഉമൈമ: ബിന്‍ത് ശറാഹീലീനെ പ്രവാചകൻ (സ) വിവാഹം ചെയ്തിരുന്നു എന്ന വസ്തുത മറച്ചു പിടിച്ചു കൊണ്ട് പ്രസ്തുത സംഭവം വിമര്‍ശകര്‍ അവതരിപ്പിക്കാറ് എന്ന് മുകളിലെ മൂന്ന് ഹദീസുകളും ഒരുമിച്ചു വെച്ച് വായിക്കുമ്പോൾ സുതരാം വ്യക്തമാവുന്നു. പ്രവാചകനെ പെണ്ണു പിടുത്തക്കാരനായും പരസ്ത്രീകളെ കടന്നുപിടിക്കുന്ന വ്യക്തിയായും താറടിക്കുകയാണ് ഈ ദുര്‍വ്യാഖ്യാന കസര്‍ത്ത് നടത്തുന്നതിന് പിന്നിലെ ചേതോവികാരം. താന്‍ വിവാഹം ചെയ്ത സ്ത്രീക്ക് തന്നോടൊപ്പം ജീവിക്കുവാന്‍ താല്‍പര്യമില്ലെന്നറിഞ്ഞപ്പോള്‍, നിര്‍ബന്ധിച്ച് കൂടെ താമസിപ്പിക്കാതെ മാന്യമായി അവരെ സ്വഗൃഹത്തിലേക്ക് യാത്രയാക്കുകയും വേര്‍പിരിയും മുമ്പ് അവര്‍ക്ക് സമ്മാനങ്ങള്‍ നല്‍കുകയും ചെയ്ത മാതൃകാപരമായ ഒരു നപടിയെ എത്ര നികൃഷ്ടമായാണ് ഇസ്‌ലാംവിമര്‍ശകര്‍ അവതരിപ്പിച്ചിരിക്കുന്നത്.

ﻻَ ﺗَﺤْﻤِﻠُﻮا اﻟﻨِّﺴَﺎءَ ﻋَﻠَﻰ ﻣَﺎ ﻳَﻜْﺮَﻫْﻦَ

“സ്ത്രീകളെ അവർക്ക് വെറുക്കുന്നത് ചെയ്യാൻ നിങ്ങൾ നിർബന്ധിക്കരുത്.” (മുസ്വന്നഫ് അബ്ദുർ റസാഖ്: 10320) എന്ന് അനുചരന്മാരെ പഠിപ്പിക്കുക മാത്രമല്ല കാരുണ്യ മൂർത്തിയായ പ്രവാചകൻ (സ) ചെയ്തത്, പ്രത്യുത ഉമൈമയോട് അനുവർത്തിച്ച നിലപാടിലൂടെ തന്റെ ആദർശനിഷ്ട സ്വജീവിതത്തിൽ പ്രാവർത്തികമായി തെളിയിക്കുക കൂടി അദ്ദേഹം ചെയ്തു.

പ്രവാചകന്‍ (സ) ഉദ്ദേശിച്ചിരുന്നെങ്കില്‍ അവരെ നിര്‍ബന്ധപൂര്‍വ്വം കൂടെ താമസിപ്പിക്കുവാന്‍ യാതൊരു തടസ്സവുമുണ്ടായിരുന്നില്ല. കാരണം അവിടുന്ന് ഇസ്‌ലാമിക സാമ്രാജ്യത്തിന്റെ അധിപനായിരുന്നു. രാജാക്കന്മാരും ചക്രവര്‍ത്തിമാരും ഒരു പെണ്ണിനെ ആഗ്രഹിച്ചു കഴിഞ്ഞാല്‍ അവളുടെ താല്‍പര്യം അന്വേഷിക്കുന്ന പതിവില്ലെന്ന് എല്ലാവര്‍ക്കുമറിയാവുന്ന വസ്തുതയാണ്. അതിനെതിരെ ഒരു ശബ്ദവുമവിടെ ഉയരുകയില്ല. ഇവിടെ പ്രവാചകന്‍ (സ) മാതൃകയാവുകയാണ്. താന്‍ വിവാഹം ചെയ്ത ഒരു സ്ത്രീക്ക് തന്നോടൊപ്പം ജീവിക്കുവാന്‍ താല്‍പര്യമില്ലെന്നറിഞ്ഞ നിമിഷം അവളെ ആശ്വസിപ്പിക്കുകയും സമാധാനിപ്പിക്കുകയും നിര്‍ഭയത്വത്തോടെ സ്വഗൃഹത്തിലേക്ക് മടങ്ങാന്‍ അവസരമൊരുക്കുകയും ചെയ്യുന്ന പ്രവാചകന്‍, ഒരു രാഷ്ട്രത്തിന്റെ ചോദ്യം ചെയ്യപ്പെടാത്ത ഭരണാധിപനാണെന്ന വസ്തുതയും ചേര്‍ത്തു മനസ്സിലാക്കുമ്പോള്‍ എത്രമാത്രം ആദരവും താല്‍പര്യവുമാണ് ആ വ്യക്തിത്വത്തിനോട് തോന്നേണ്ടത്. ഇസ്‌ലാം വിമര്‍ശകര്‍ക്ക് പക്ഷെ അത്തരം ഊഷ്മളമായ ചിന്തയും വികാരവുമൊന്നും ഉണ്ടാവുകയില്ല. കാരണം അവരുടെ ഹൃദയം കടുത്തു പോയിരിക്കുന്നു. ഊഷരമായ ചിന്തയും വികാരവുമാണ് അവരെ നയിക്കുന്നത്. വെറുപ്പും വിദ്വേഷവും മാത്രമാണ് അവരെ ഭരിക്കുന്നത്.

സമാധാനിപ്പിക്കാനായി -അതും സ്വന്തം ഭാര്യയുടെ മേൽ- കൈ വെക്കാൻ തുനിഞ്ഞതിനെ ‘കേറിപ്പിടിക്കലാക്കി’ ചിത്രീകരിക്കാനുള്ള ‘അപാരമായ കഴിവ് ‘ അഗമ്യഗമനത്തേയും, ബലാൽസംഗത്തേയും ശവരതിയേയുമെല്ലാം പ്രണയിക്കുന്നവരുടെ ‘ഹൈപ്പർ സെക്ഷ്വാലിറ്റിയുടെ'(Hypersexuality)ഭാഗമാണ്. ഇത്തരക്കാർക്ക് ഒരു തെറാപ്പിസ്റ്റിനെ സംഘടിപ്പിച്ച് കൊടുക്കുന്നതിന് പകരം സ്റ്റേജും, പേജും നൽകി സമൂഹത്തിലേക്ക് അഴിച്ചു വിടുന്നതാണ് ഭൗതികവാദികൾ മത വിശ്വാസികളോട് ചെയ്യുന്ന ഏറ്റവും വലിയ ദ്രോഹം.

ചരിത്രനിമിഷങ്ങളെ നാം തിരിച്ചൊന്ന് വിഭാവനം ചെയ്തു നോക്കൂ; ഉമൈമ: ബിന്‍ത് ശറാഹീലിന്റെ മനോഗതങ്ങളിലൂടെ. എന്തായിരിക്കും അവരുടെ ഹൃദയത്തില്‍ നിറഞ്ഞു നിന്ന വികാരങ്ങളും വിചാരങ്ങളും. തന്നെ വിവാഹം ചെയ്ത ആളോട്, അദ്ധേഹത്തെ തനിക്ക് ഇഷ്ടമല്ലെന്ന് അറിയിച്ചപ്പോള്‍ അവള്‍ കണ്ടത് ശാന്തവും മാന്യവുമായ പ്രതികരണമാണ്. ഒരു നിമിഷം പോലും അവളെ അദ്ദേഹത്തിന്റെ താല്‍പര്യങ്ങള്‍ക്കു വിധേയമാക്കാന്‍ ശ്രമിക്കാതെ, സമ്മാനങ്ങള്‍ നല്‍കി സ്വന്തം ഗൃഹത്തിലേക്ക് അവള്‍ക്ക് യാത്രാസൗകര്യമൊരുക്കിയ ഇസ്‌ലാമിക രാഷ്ട്രനായകനെ അവളുടെ ഹൃദയം എത്രമാത്രം ആദരിച്ചിട്ടുണ്ടാകും. തന്റെ നാട്ടിൽ തിരിച്ചെത്തിയ ഉമൈമ പ്രവാചകനുമായുള്ള ദാമ്പത്യ ജീവിതം ഉപേക്ഷിച്ചതിൽ പിന്നീട് ഖേദിക്കുകയും, ‘തനിക്ക് അമളി പറ്റി പോയി’ എന്ന് പറയുകയും ചെയ്തതായി ചരിത്രത്തിൽ തന്നെ കാണാം. (ഫത്ഹുൽ ബാരി: 9:314)

പക്ഷേ ഇസ്‌ലാം വിമര്‍ശകരുടെ വരണ്ട ഹൃദയങ്ങള്‍ക്ക് അത്തരം വിഭാവനങ്ങള്‍ അന്യമാണ്. ഹൃദയങ്ങളില്‍ അവര്‍ സൂക്ഷിച്ചിവെച്ചിരിക്കുന്ന ഇസ്‌ലാമിനോടുള്ള വെറുപ്പും വിദ്വേഷവും അവരെ എത്രമാത്രം ഊഷരവും വൃത്തിഹീനവുമായ മനോഗതിക്കാരാക്കിയിരുന്നു.! കഷ്ടം.!!

 അബ്രഹാമും ഇശ്മയേലും ഏകദൈവാരാധക്കുവേണ്ടി സ്ഥാപിച്ച പ്രാര്‍ത്ഥനാമന്ദിരത്തോടനുബന്ധിച്ച് പില്‍ക്കാലത്ത് ബഹുദൈവവിശ്വാസപരമായ ചടങ്ങുകള്‍ രൂപം കൊണ്ടതാണെന്നും കഅ്ബയുടെ സാക്ഷാല്‍ ലക്ഷ്യം പ്രപഞ്ചനാഥനെ മാത്രം ആരാധിക്കുകയാണെന്നും പല അറബികള്‍ക്കും പ്രവാചകനിയോഗത്തിന്റെ കാലഘട്ടത്തില്‍പോലും അറിയാമായിരുന്നുവെന്നും അവരാണ് ഹനീഫുകള്‍ എന്നറിയപ്പെട്ടതെന്നും ഉള്ള മുസ്‌ലിം ചരിത്രകാരന്‍മാരുടെ വാദം അടിസ്ഥാനരഹിതമാണ്. ഹനീഫുകള്‍ എന്ന പേരില്‍ ജാഹിലിയ്യ അറബികള്‍ക്കിടയില്‍ ജീവിച്ചിരുന്നവരൊന്നും ഈ വിശ്വാസമുള്ളവരല്ലായിരുന്നുവെന്ന് അവരെക്കുറിച്ചുള്ള നിവേദനങ്ങളില്‍ നിന്ന് സ്പഷ്ടമാണ്. മക്കയുടെ അബ്രഹാമിക പാരമ്പര്യത്തെ നിഷേധിക്കുന്ന ഓറിയന്റലിസ്റ്റ്-മിഷനറി രചനകളില്‍ സര്‍വസാധാരണമായ ഈ വാദങ്ങള്‍ ഹനീഫിയ്യത്തിനെ സംബന്ധിച്ച മുസ്‌ലിം അവകാശവാദത്തെ പുനപരിശോധിക്കാന്‍ പ്രേരിപ്പിക്കുന്നവയല്ലേ?

ല്ല. മക്കയുടെ ഇബ്‌റാഹീമീ പാരമ്പര്യം മുഹമ്മദ് നബി (സ) പറഞ്ഞുണ്ടാക്കിയതാണ് എന്ന വിമര്‍ശനത്തെ എല്ലാ അര്‍ത്ഥത്തിലും കടപുഴക്കുന്നതാണ് പ്രവാചകനിയോഗത്തിനുമുമ്പേ അറേബ്യയിലുണ്ടായിരുന്ന ഹനീഫുകളുടെ സാന്നിധ്യം. അറബികള്‍ ഇബ്‌റാഹീമീ ഏകദൈവാരാധനയില്‍നിന്ന് വ്യതിചലിച്ചുപോയതായി മനസ്സിലാക്കുകയും ബഹുദൈവാരാധനാപരമായ അറബ് അനുഷ്ഠാനങ്ങളോട് വിരക്തി പ്രകടിപ്പിച്ച് ഇബ്‌റാഹീമീ മാര്‍ഗത്തിന്റെ വീണ്ടെടുപ്പ് സ്വന്തം ജീവിതത്തില്‍ ആഗ്രഹിക്കുകയും ചെയ്ത ന്യൂനപക്ഷമാണ് മക്കയിലും പരിസരപ്രദേശങ്ങളിലും ഹനീഫുകള്‍ എന്നു വിളിക്കപ്പെട്ടത് എന്ന് അവരെ സംബന്ധിച്ച നിവേദനങ്ങളെല്ലാം വ്യക്തമാക്കുന്നുണ്ട്. ഇബ്‌റാഹീമീ രക്തത്തോടൊപ്പം ആദര്‍ശവും കുറേയെങ്കിലും അറേബ്യയില്‍ മുഹമ്മദ് നബി(സ)യുടെ കാലം വരെ നിലനിന്നുവെന്ന് ഹനീഫുകളുടെ ചരിത്രം ബോധ്യപ്പെടുത്തുന്നു എന്നതിനാലാണ് ഹനീഫുകള്‍ ഇബ്‌റാഹീമീ നിലപാടുകളുടെ പുനരുജ്ജീവനത്തിന് പരിശ്രമിച്ചവരായിരുന്നില്ലെന്ന് ചില നിവേദനങ്ങളിലെ പരാമര്‍ശങ്ങളുടെ വെളിച്ചത്തില്‍ സ്ഥാപിച്ചെടുക്കാന്‍ ഓറിയന്റലിസ്റ്റുകള്‍ പരിശ്രമിച്ചു നോക്കിയിട്ടുള്ളത്.

വാസ്തവത്തില്‍, അറേബ്യയില്‍ നിലനിന്നിരുന്ന ഹനീഫിയ്യത്തിനെ സംബന്ധിച്ച ചരിത്രനിവേദനങ്ങളുടെ വിശകലനം ഇബ്‌റാഹീം നബി(അ)യുടെ ആദര്‍ശമനുസരിച്ച് ജീവിക്കാനുളള അദമ്യമായ ആഗ്രഹമാണ് ഹനീഫുകളെ വ്യതിരിക്തരാക്കിയത് എന്നുതന്നെയാണ് ബോധ്യപ്പെടുത്തുന്നത്. ഇബ്‌റാഹീമിലേക്ക് മടങ്ങുവാനുള്ള ത്വര മുഹമ്മദ് നബി (സ) പുതിയ ദേശപാരമ്പര്യം മെനഞ്ഞുണ്ടാക്കി മക്കക്കാരില്‍ കൃത്രിമമായി സന്നിവേശിപ്പിച്ചതാണെന്ന വിമര്‍ശക വീക്ഷണം പ്രസ്തുത നിവേദനങ്ങള്‍ക്കുമുന്നില്‍ ഒരിക്കലും നിലനില്‍ക്കുകയില്ല. തങ്ങള്‍ തുടര്‍ന്നുകൊണ്ടിരിക്കുന്ന വിശ്വാസ-കര്‍മ മാര്‍ഗം പൂര്‍ണമായും ഇബ്‌റാഹീമിന്റെയും ഇസ്മാഈലിന്റേതുമാണ് എന്ന് വലിയൊരു വിഭാഗം അറബികള്‍ തെറ്റിദ്ധരിച്ചപ്പോഴും അങ്ങനെയല്ലെന്നും അതില്‍ കലര്‍പ്പുകള്‍ വന്നിട്ടുണ്ടെന്നും ശരിയായി തിരിച്ചറിഞ്ഞ ഒറ്റപ്പെട്ട വ്യക്തികളായിരുന്നു ഹനീഫുകള്‍. അല്ലാഹുവിനു മാത്രം ആരാധനകള്‍ സമര്‍പ്പിക്കണമെന്നു വാദിച്ചിരുന്നതുകൊണ്ടാണ് അവര്‍ ഹനീഫുകള്‍ (ഋജുമാനസ്‌കര്‍-കലര്‍പ്പുകള്‍ അനുവദിക്കാത്തവര്‍) എന്നറിയപ്പെട്ടത്.

മക്കന്‍ മുഖ്യധാരയോട് കലഹിച്ച് ഇബ്‌റാഹീമീ സരണിയോട് വിഗ്രഹാരാധനയും അറബ് അനാചാരങ്ങളും ഒത്തുപോവുകയില്ലെന്ന് ഒറ്റയാനായി പ്രഖ്യാപിച്ച സയ്ദ്ബ്‌നു അംറുബ്‌നു നുഫയ്ല്‍ ആണ് ചരിത്രത്തിലെ ഏറ്റവും പ്രഖ്യാതനായ ഹനീഫ്. മുഹമ്മദ് നബി(സ)യുടെ സമകാലീനനായിരുന്നെങ്കിലും അദ്ദേഹത്തിന്റെ പ്രവാചകത്വത്തിനുമുമ്പ് മരണപ്പെട്ടുപോയ വ്യക്തിയായിരുന്നു സയ്ദ് എന്നാണ് മനസ്സിലാകുന്നത്. മക്കന്‍ വിഗ്രഹാരാധന ശരിയല്ലെന്ന് മനസ്സിലാക്കി ശരിയായ ദൈവികപാത തേടി സിറിയയിലേക്കടക്കം യാത്ര പോയ സയ്ദിന് ജൂത, ക്രൈസ്തവ പണ്ഡിതരടക്കം ഉപദേശിച്ചുകൊടുത്തത് ഇബ്‌റാഹീമീ ഹനീഫിയ്യത്തായിരുന്നുവെന്ന് സ്വഹീഹുല്‍ ബുഖാരിയിലെ തീര്‍ത്തും പ്രബലമായ നിവേദനത്തിലുണ്ട്. സിറിയയില്‍ നിന്നു മടങ്ങിയപ്പോള്‍ അദ്ദേഹം കൈകളുയര്‍ത്തി ”എന്റെ രക്ഷിതാവേ, ഞാന്‍ ഇബ്‌റാഹീമിന്റെ മതത്തിലാണെന്നതിന് നീ സാക്ഷ്യം വഹിച്ചുകൊള്ളുക” എന്ന് പ്രഖ്യാപിച്ചതായി ഇബ്‌നു ഉമര്‍ (റ) റിപ്പോര്‍ട്ട് ചെയ്തിട്ടുണ്ട് (ബുഖാരി). താന്‍ അല്ലാഹുവിനെ മാത്രമേ ആരാധിക്കുകയുള്ളൂവെന്നും അതാണ് ഇബ്‌റാഹീമിന്റെ ശരിയായ മതം എന്നും താനാണ് അതില്‍ നിലനില്‍ക്കുന്നതെന്നും ഇബ്‌റാഹീമിന്റെ പൈതൃകം അവകാശപ്പെടുമ്പോഴും മറ്റു മക്കക്കാര്‍ അദ്ദേഹത്തിന്റെ മതത്തില്‍ നിന്നും വ്യതിചലിച്ചുപോയിരിക്കുന്നുവെന്നും വ്യക്തമാക്കിക്കൊണ്ട് സയ്ദ്ബ്‌നു അംറ് കഅ്ബയുടെ ചാരത്തുനിന്ന് ”ഖുറയ്ശികളേ, സയ്ദിന്റെ ആത്മാവ് ആരുടെ കയ്യിലാണോ, അവനാണ് (അല്ലാഹു) സത്യം; നിങ്ങളിലൊരാളുമല്ല, മറിച്ച് ഞാനാണ് ഇബ്‌റാഹീമിന്റെ മതത്തിലുള്ളത്” (മാ അസ്ബ്ഹ മിന്‍കും അഹദുന്‍ അലാ ദീനി ഇബ്‌റാഹീമ ഗ്വയ്‌രീ) എന്നു പ്രഖ്യാപിച്ചത് ഇബ്‌നു ഇസ്ഹാക്വിന്റെ സീറയിലുണ്ട് (Guillaume, 99-100).

താന്‍ ഇബ്‌റാഹീമിന്റെ മാര്‍ഗം തെരഞ്ഞെടുക്കുന്നു എന്ന് സയ്ദ് പറയുന്നത് ആ മാര്‍ഗം മക്കക്കാര്‍ നേരത്തെ അവകാശപ്പെട്ടുകൊണ്ടിരുന്നതാണ് എന്ന അവബോധത്തോടെയാണ് എന്ന് അദ്ദേഹത്തിന്റെ വാക്കുകള്‍ സുതരാം വ്യക്തമാക്കുന്നുണ്ട്. ഹനീഫിയ്യാ ഏകദൈവാരാധനാനിഷ്ഠ സ്വീകരിച്ചതിന്റെ ഫലമായി, അദ്ദേഹം വിഗ്രഹാരാധനയും വിഗ്രഹങ്ങള്‍ക്ക് നിവേദിക്കപ്പെട്ട ഭക്ഷണം കഴിക്കുന്നതും പൂര്‍ണമായി ഉപേക്ഷിച്ചുവെന്നും ഇബ്‌നു ഇസ്ഹാക്വ് രേഖപ്പെടുത്തുന്നു (Ibid, p. 99). മദീനയില്‍ പ്രവാചകാഗമനത്തിനു മുമ്പുതന്നെ ഹനീഫ് ആയി ജീവിച്ചിരുന്ന ബനൂ അദിയ്യ ഗോത്രക്കാരന്‍ അബൂ ക്വയ്‌സ് ബിന്‍ അബൂ അനസിന്റെയും കഥ ഏതാണ്ട് സമാനം തന്നെയാണ്. വിഗ്രഹങ്ങളുപേക്ഷിക്കുകയും ജൂതനോ ക്രൈസ്തവനോ ആകുന്നതിനുപകരം ശുദ്ധമായ ഇബ്‌റാഹീമീ സരണി പുല്‍കുന്നുവെന്ന് പ്രഖ്യാപിക്കുകയും ചെയ്ത അദ്ദേഹം ”ഞാന്‍ ഇബ്‌റാഹീമിന്റെ നാഥനെയാണ് ആരാധിക്കുന്നത്” എന്ന് വിശദീകരിച്ച് പ്രാര്‍ത്ഥന നിര്‍വഹിക്കുവാന്‍ വേണ്ടി ഒരു ആരാധനാലയം പണിതതായി ഇബ്‌നു ഇസ്ഹാക്വില്‍ തന്നെയുണ്ട്. ഇദ്ദേഹം നബി (സ) മദീനയിലെത്തിയപ്പോള്‍ ഇസ്‌ലാം സ്വീകരിച്ചു (Ibid, pp. 236-9).

സയ്ദിനെയും അബൂക്വയ്‌സിനെയും സംബന്ധിച്ചുള്ള നിവേദനങ്ങള്‍ വ്യക്തമാക്കുന്ന ഒരു കാര്യം, ഇബ്‌റാഹീം നബി(അ)യും ഇസ്മാഈല്‍ നബി(അ)യും പഠിപ്പിച്ചിരുന്നത് ശുദ്ധ ഏകദൈവാരാധനയാണെന്നും എന്നാല്‍ അറബികള്‍ അവരുടെ മാര്‍ഗത്തില്‍നിന്ന് വ്യതിചലിച്ച് വിഗ്രഹാരാധനയിലും അധാര്‍മികതകളിലും എത്തിപ്പെട്ടു എന്നുമുള്ള അടിസ്ഥാന ബോധ്യങ്ങളാണ് അവര്‍ക്കുണ്ടായിരുന്നത് എന്നാണ്. ഇബ്‌റാഹീമീ സരണിക്ക് നിരക്കുന്നതല്ലെന്ന് തങ്ങള്‍ക്ക് ബോധ്യം വന്ന തിന്മകളില്‍ നിന്ന് വിട്ടുനില്‍ക്കുകയും സാധ്യമാകുന്ന തരത്തില്‍ ഏകദൈവാരാധന നിര്‍വഹിക്കുകയും ചെയ്ത് മനസ്സാക്ഷിയെ തൃപ്തിപ്പെടുത്താന്‍ ശ്രമിച്ചവരായിരുന്നു അവര്‍. അതല്ലാതെ, അല്ലാഹുവിനുവേണ്ടി നിര്‍വഹിക്കേണ്ടുന്ന ആരാധനകളുടെ വിശദമായ കര്‍മശാസ്ത്രത്തെക്കുറിച്ചോ അനുഷ്ഠിക്കേണ്ട സല്‍പ്രവര്‍ത്തനങ്ങളെക്കുറിച്ചോ ജീവിതവിശുദ്ധി നിലനിര്‍ത്താന്‍ ഉപേക്ഷിക്കേണ്ട തിന്മകളെക്കുറിച്ചോ കൃത്യവും സമഗ്രവുമായ ധാരണകളൊന്നും അവര്‍ക്കുണ്ടായിരുന്നില്ല. വഹ്‌യ് ലഭിക്കുന്ന ഒരു പ്രവാചകന്റെ അസാന്നിധ്യമായിരുന്നു ഈ പ്രതിസന്ധിക്കു കാരണം.

അബൂക്വയ്‌സും സയ്ദും ആര്‍ത്തവകാരികളുമായുള്ള ലൈംഗിക ബന്ധത്തില്‍നിന്ന് വിട്ടുനിന്നതും സയ്ദ് ശവവും രക്തവും ഭക്ഷിക്കുന്നത് ഒഴിവാക്കിയതും പെണ്‍മക്കളെ ജീവനോടെ കുഴിച്ചുമൂടുന്നതിനെ എതിര്‍ത്തതുമെല്ലാം ഇബ്‌നു ഇസ്ഹാക്വ് വിവരിക്കുന്നുണ്ട്. ഇവയെല്ലാം അവരുടെ അന്വേഷണങ്ങളില്‍ നിന്ന് അവരെത്തിപ്പെട്ട ധാര്‍മിക നിലപാടുകളായിരുന്നു. കുറേക്കൂടി നിഷ്‌കൃഷ്ടമായ മാര്‍ഗദര്‍ശനത്തിനുവേണ്ടി അവര്‍ ദാഹിച്ചിരുന്നുവെന്ന് സയ്ദ്ബ്‌നു അംറിന്റെ വാക്കുകള്‍ വ്യക്തമാക്കുന്നുണ്ട്. അദ്ദേഹം പറഞ്ഞു : ”എന്റെ രക്ഷിതാവേ, നിന്നെ ആരാധിക്കാനുള്ള കൂടുതല്‍ നല്ല മാര്‍ഗങ്ങള്‍ അറിയുമായിരുന്നുവെങ്കില്‍ ഞാനത് സ്വീകരിക്കുമായിരുന്നു; പക്ഷേ എന്തു ചെയ്യാം, എനിക്കതറിയില്ല!” തുടര്‍ന്ന് കഅ്ബക്കുനേരെ തിരിഞ്ഞ് അല്ലാഹുവിനു സുജൂദ് ചെയ്ത് അദ്ദേഹം വാക്കുകള്‍ ഇങ്ങനെ മുഴുമിപ്പിച്ചു: ”എന്റെ നാഥന്‍ ഇബ്‌റാഹീമിന്റെ നാഥനാണ്, എന്റെ മതം ഇബ്‌റാഹീമിന്റെ മതവുമാണ്.” (Ibid, p. 100; Dr. Mahdi  Rizqullah Ahmad, A Biography of the Prophet of Islam in the light of Original Sources  (Riyadh: Darussalam, 2005), p. 58).

മക്കയിലും മദീനയിലുമുണ്ടായിരുന്ന ഹനീഫുകളായി അറിയപ്പെട്ടിരുന്ന  ചില വ്യക്തികള്‍ നബി(സ)യുടെ പ്രവാചകത്വം അംഗീകരിക്കുവാന്‍ വിസമ്മതിക്കുകയും അദ്ദേഹത്തോട് ആശയപരമായ വിയോജിപ്പുകള്‍ രേഖപ്പെടുത്തുകയും ചെയ്തതായി പറയുന്ന ചില നിവേദനങ്ങളുടെ അടിസ്ഥാനത്തിലാണ് വിമര്‍ശകര്‍ ഹനീഫുകള്‍ ഇബ്‌റാഹീമീ മാര്‍ഗത്തിന്റെ പുനരുജ്ജീവനത്തിനു ശ്രമിച്ചവരല്ലായിരുന്നു എന്ന് സ്ഥാപിക്കുവാന്‍ ശ്രമിക്കുന്നത്. അവര്‍ക്കുള്ള മറുപടി സയ്ദിന്റെ വാക്കുകളില്‍ തന്നെയുണ്ട് എന്നുള്ളതാണ് വാസ്തവം. ഇബ്‌റാഹീമീ ഏകദൈവാരാധനയുടെ അടിസ്ഥാനമൂല്യങ്ങള്‍ ഉള്‍ക്കൊണ്ടിരുന്നവരെല്ലാം അറേബ്യയില്‍ ഹനീഫുകളായാണ് അറിയപ്പെട്ടിരുന്നത്. അവരില്‍ നബി(സ)യുടെ പ്രവാചകത്വം അംഗീകരിച്ചവരും നിഷേധിച്ചവരുമുണ്ടാകാം, പ്രവാചകന്‍ സ) പ്രബോധനം ചെയ്ത ധാര്‍മിക പദ്ധതിയുടെ വിശദാംശങ്ങളോട് യോജിച്ചവരും വിയോജിച്ചവരുമുണ്ടാകാം, ഏകദൈവാരാധന മനസ്സിലുള്‍ക്കൊണ്ടാല്‍ മതിയെന്നും നബി (സ) ചെയ്യുന്നതുപോലെ സമൂഹത്തില്‍ അത് വ്യാപകമായി പ്രചരിപ്പിക്കേണ്ടതില്ലെന്നും കരുതിയ ആദര്‍ശ പ്രതിബദ്ധത കുറഞ്ഞ വ്യക്തികളുമുണ്ടാകാം. ഹനീഫുകള്‍ ആരാണെന്ന് മനസ്സിലാക്കിയവര്‍ക്ക് ചരിത്രപരമായി ഇവയിലൊന്നും യാതൊരു അസാംഗത്യവും അനുഭവപ്പെടുകയില്ല.

ഏകശിലാത്മകമായ ഒരു സമൂഹമായിരുന്നില്ല ഹനീഫുകളുടേത്; മറിച്ച് ഇബ്‌റാഹീമീ ഏകദൈവാരാധനയെക്കുറിച്ച് നിശ്ചയവും വ്യക്തതയുമുണ്ടായിരുന്ന, എന്നാല്‍ അനുബന്ധങ്ങളില്‍ ആശയക്കുഴപ്പങ്ങളും അഭിപ്രായാന്തരങ്ങളുമുണ്ടായിരുന്ന ഒറ്റയും തെറ്റയുമായ വ്യക്തിത്വങ്ങളാണ്. അതുകൊണ്ടുതന്നെ വിമര്‍ശകര്‍ എടുത്തുദ്ധരിക്കുന്ന നിവേദനങ്ങള്‍ നിദാനശാസ്ത്രപരമായി ആധികാരികമാണെങ്കിലും അല്ലെങ്കിലും, അവരുടെ വാദം സ്ഥാപിക്കുവാന്‍ പര്യാപ്തമായവയല്ല എന്നതാണ് വാസ്തവം. ചില ഹനീഫുകള്‍ പ്രവാചകന്റെ കൂടെ നിന്നില്ല എന്നുമാത്രമാണ് പരാമൃഷ്ട നിവേദനങ്ങള്‍ വ്യക്തമാക്കുന്നത്. എന്നാല്‍ ഇബ്‌റാഹീം നബി(അ)യുടെ ആശയങ്ങളിലേക്ക് മടങ്ങിപ്പോകണമെന്ന കാര്യത്തില്‍ അവര്‍ക്ക് നബി(സ)യുമായി യാതൊരു അഭിപ്രായവ്യത്യാസവുമുണ്ടായിരുന്നില്ല  എന്ന് അതേ നിവേദനങ്ങള്‍ തന്നെ വ്യക്തമാക്കുന്നുണ്ട്. മക്കയില്‍ പൂര്‍വപിതാക്കളായ ഇബ്‌റാഹീമിെനയും ഇസ്മാഈലിനെയും കുറിച്ചുള്ള ബോധ്യവും അവരുടെ ചര്യകള്‍ മുറുകെപ്പിടിക്കണമെന്ന വികാരവും പ്രവാചകന്‍ (സ) പുതുതായി സൃഷ്ടിച്ചെടുത്തതല്ലെന്ന് എല്ലാ അര്‍ത്ഥത്തിലും സ്ഥാപിക്കുന്നവയാണ് ആ ഉദ്ധരണികള്‍. ഹനീഫുകള്‍ എന്നാല്‍ മുഹമ്മദീയ ഇസ്‌ലാമിന്റെ എല്ലാ ആശയങ്ങളും അംഗീകരിച്ചവരായിരുന്നു എന്ന് മുസ്‌ലിംകള്‍ വാദിക്കുന്നതായി തെറ്റിദ്ധരിപ്പിച്ചുകൊണ്ടാണ് വിമര്‍ശകര്‍ അവയുടെ വിശകലനം നിര്‍വഹിക്കുന്നത്. ഈ തെറ്റിദ്ധാരണയില്‍ നിന്ന് മോചിതരായാല്‍  വിമര്‍ശകര്‍ സ്വന്തം അടിസ്ഥാനങ്ങളെയാണ് തകര്‍ത്തുകളയുന്നത് എന്ന് ആര്‍ക്കും ബോധ്യമാകും. മദീനയിലെ ഔസ് ഗോത്രത്തിന്റെ നേതാക്കളിലൊരാളായിരുന്ന അബൂ ആമിര്‍ അംറുബ്‌നു സയ്ഫും ഔസ് ഗോത്രക്കാരന്‍ തന്നെയായിരുന്ന കവി അബുക്വയ്‌സ് ബിന്‍ അസ്‌ലതും ത്വാഇഫുകാരനായ ഉമയ്യയുമാണ് ഹനീഫുകളായി അറിയപ്പെട്ടിരുന്ന, ഇസ്‌ലാം സ്വീകരിച്ചിട്ടില്ലെന്ന് ചില നിവേദനങ്ങള്‍ പറയുന്ന വ്യക്തികള്‍. അബൂ ആമിറും നബി(സ)യും തമ്മില്‍ മദീനയില്‍വെച്ച് നേരില്‍ കണ്ടുമുട്ടിയപ്പോള്‍ അവര്‍ തമ്മില്‍ നടന്ന സംഭാഷണം ഇബ്‌നു ഇസ്ഹാക്വിന്റെ, വിമര്‍ശകര്‍ ആശ്രയിക്കുന്ന നിവേദനത്തില്‍ തന്നെയുണ്ട്. ഏതു മതവുമായാണ് പ്രവാചകന്‍ (സ) നിയോഗിക്കപ്പെട്ടതെന്ന അബൂ ആമിറിന്റെ ചോദ്യത്തിന് ‘ഹനീഫിയ്യ; ഇബ്‌റാഹീമിന്റെ മതം’ എന്ന് നബി (സ) മറുപടി പറഞ്ഞപ്പോള്‍ താനും ആ മതത്തില്‍ തന്നെയാണ് എന്നായിരുന്നു അബൂ ആമിറിന്റെ പ്രത്യുത്തരം. അബൂ ആമിര്‍ ഇബ്‌റാഹീമിന്റെ മതം ശരിയായി പിന്തുടരുന്നില്ലെന്ന് പറഞ്ഞ പ്രവാചകന്‍(സ)യോട് അദ്ദേഹം പറഞ്ഞത് ‘ഹനീഫിയ്യത്തില്‍ ഇല്ലാത്ത പലതും, മുഹമ്മദ്, താങ്കള്‍ അതില്‍ കൂട്ടിച്ചേര്‍ത്തിരിക്കുന്നു’ (ഇന്നക്ക അദ്ഖല്‍ത, യാ മുഹമ്മദ്, ഫില്‍ ഹനീഫിയ്യ മാ ലയ്‌സ മിന്‍ഹാ) എന്നായിരുന്നുവെന്നും ഇബ്‌റാഹീമിന്റെ മതത്തെ അതിന്റ ശുദ്ധതയില്‍ അപ്പടി പ്രബോധനം ചെയ്യുക മാത്രമാണ് താന്‍ ചെയ്യുന്നതെന്ന് നബി (സ) ഇതിനോട് പ്രതിവചിച്ചുവെന്നും ഇബ്‌നു ഇസ്ഹാക്വ് രേഖപ്പെടുത്തുന്നു. ഇബ്‌റാഹീമീ സരണി പിന്തുടരുന്നുവെന്ന് അവകാശപ്പെട്ടതിനാല്‍ തന്നെയാണ് അബൂ ആമിര്‍ ഹനീഫ് ആയി അറയപ്പെട്ടതെന്നും ഇബ്‌റാഹീമിന്റെ മാര്‍ഗത്തിന്റെ വിശദാംശങ്ങളില്‍ എന്തെല്ലാം വരുമെന്ന കാര്യത്തിലാണ് അദ്ദേഹത്തിന് പ്രവാചകനുമായി അഭിപ്രായവ്യത്യാസമുണ്ടായതെന്നും നിവേദനത്തില്‍ പരാമര്‍ശിക്കപ്പെട്ടിരിക്കുന്ന സംവാദത്തില്‍ നിന്ന് വ്യക്തമാണ്. അറേബ്യയില്‍ പ്രവാചകനുമുമ്പേ ഇബ്‌റാഹീമീ വികാരങ്ങള്‍ നിലനിന്നിരുന്നുവെന്നതിന്റെ സൂചകമാണ് ഫനീഫുകള്‍ എന്ന നിരീക്ഷണത്തെ ഈ നിവേദനം എങ്ങനെ തകര്‍ക്കുമെന്നാണ് വിമര്‍ശകര്‍ പറയുന്നത്? രണ്ടാമത്തെയാളായ അബൂ ക്വയ്‌സ് ബിന്‍ അസ്‌ലത്, ‘ഞാന്‍ ഇബ്‌റാഹീമിന്റെ മതം പിന്തുടരുന്നു; മരണം വരെ ഞാനതില്‍ നിന്ന് പിന്‍മാറുകയില്ല’ എന്ന് പ്രസ്താവിക്കുമായിരുന്നുവെന്ന് ഓറിയന്റലിസ്റ്റുകള്‍ ആശ്രയിക്കുന്ന ഇബ്‌നു സഅദിന്റെ നിവേദനത്തില്‍ തന്നെയുണ്ട്. ഹനീഫിയ്യത്തിന് ഏകദൈവാരാധനാ നിലപാടുകളുമായി ബന്ധമുണ്ടായിരുന്നില്ലെന്ന വിമര്‍ശകവാദത്തെ എല്ലാ അര്‍ത്ഥത്തിലും പൊളിച്ചുകളയുന്നതാണ് ഉമയ്യയെക്കുറിച്ചുള്ള നിവേദനങ്ങളെല്ലാം. അദ്ദേഹം വിഗ്രഹാരാധനയില്‍ നിന്ന് വിട്ടുനില്‍ക്കുകയും അതിന്റെ പേരില്‍ ജൂതനായിപ്പോലും വിചാരിക്കപ്പെടുകയും ചെയ്ത, ഇബ്‌റാഹീമിന്റെയും ഇസ്മാഈലിന്റെയും യഥാര്‍ത്ഥ പൈതൃകം ഹനീഫിയ്യത്താണെന്ന് സ്ഥാപിച്ചുകൊണ്ട് സമൃദ്ധമായി കവിതകളെഴുതിയ ആളായിരുന്നുവെന്നാണ് ചരിത്രഗ്രന്ഥങ്ങള്‍ പറയുന്നത്. അദ്ദേഹത്തിന്റെ ‘ഹനീഫീ’ കവിതള്‍ അറബ് വാമൊഴി പാരമ്പര്യത്തില്‍ സജീവമായി നിലനിന്നതുകൊണ്ടുതന്നെ, ജാഹിലിയ്യാ കാലഘട്ടം മുതല്‍ ഒന്‍പതാം നൂറ്റാണ്ടുവരെയുള്ള അറബിക്കവിതകളുടെ ബൃഹദ്‌ശേഖരം അരനൂറ്റാണ്ടു കാലത്തെ അധ്വാനംകൊണ്ട് അബുല്‍ ഫറജ് ഇസ് ഫഹാനി സി. ഇ പത്താം നൂറ്റാണ്ടില്‍ അഗാനി എന്ന പേരില്‍ പ്രസിദ്ധീകരിച്ചപ്പോള്‍ അതില്‍പോലും ഇടം പിടിച്ചിട്ടുണ്ട്. വിമര്‍ശകര്‍ക്കാവശ്യമുള്ളതൊന്നും ഉമയ്യയെ വിശകലനം ചെയ്തതുകൊണ്ട് ലഭിക്കുകയില്ലെന്ന് ഇതില്‍ നിന്നുതന്നെ വ്യക്തമാണ്. ”ഹനീഫിയ്യത്ത് സത്യമാണെന്നെനിക്കറിയാം; എന്നാല്‍ മുഹമ്മദിന്റെ കാര്യത്തിലാണ് എനിക്ക് തീര്‍ച്ചയില്ലാത്തത്” (വ അന അഅ്‌ലമു അന്നല്‍ ഹനീഫിയ്യ ഹക്വുന്‍ വലാകിന്നശ്ശക്ക്വ യുദാഖിലുനീ ഫീ മുഹമ്മദ്) എന്നാണ് ഉമയ്യ പ്രവാചകനുമായുള്ള തന്റെ അഭിപ്രായവ്യത്യാസത്തെക്കുറിച്ച് പറഞ്ഞത് (ഫത്ഹുല്‍ബാരി). മുഹമ്മദ് നബി(സ)യുടെ പ്രവാചകത്വം അംഗീകരിക്കാന്‍ വിമുഖത കാണിച്ചുവെന്നതുകൊണ്ട് ഉമയ്യ ഇബ്‌റാഹീമീ ആദര്‍ശ പിന്തുടര്‍ച്ച അവകാശപ്പെട്ടിരുന്നുവെന്ന വസ്തുത എങ്ങനെയാണ് ഇല്ലാതാവുക? അറബികള്‍ക്കിയടിലുണ്ടായിരുന്ന ഇബ്‌റാഹീമീ ബോധത്തെ അദ്ദേഹത്തെ സംബന്ധിച്ച നിവേദനങ്ങള്‍ സാധൂകരിക്കുകയല്ലാതെ എങ്ങനെയാണ് നിരാകരിക്കുക? ചുരുക്കത്തില്‍, ഹനീഫുകളെക്കുറിച്ചുള്ള ചരിത്ര/ഹനീഥ് നിവേദനങ്ങള്‍ അവര്‍ ഇബ്‌റാഹീമീ ആദര്‍ശ വ്യതിരിക്തത അവകാശപ്പെട്ടിരുന്നവരല്ല എന്ന് സൂചിപ്പിക്കുന്നുവെന്ന വിമര്‍ശകരുടെ വാദം ഒരു കഴമ്പുമില്ലാത്തതാണ്.

സ്വന്തത്തേക്കാളധികം മുഹമ്മദ് നബി(സ)യെ സ്‌നേഹിക്കുന്ന മുസ്‌ലിംകള്‍ക്ക് നബി(സ)യെന്ന പേരിട്ടുകൊണ്ട് വൃത്തികെട്ട ചിത്രങ്ങള്‍ വര ക്കപ്പെടുകയും ലോകത്തിന് പ്രകാശം നല്‍കുകയും നന്മ മാത്രം വിതറുകയും ചെയ്ത പ്രവാചകന്‍ ക്രൂരമായി അഭിശംസിക്കപ്പെടുകയും ചെയ്യുമ്പോള്‍ വേദനയുണ്ടാവുക സ്വാഭാവികമാണ്. സ്വന്തത്തെയല്ലാതെ മറ്റൊന്നിനെയും ആത്മാര്‍ത്ഥമായി സ്‌നേഹിക്കാന്‍ പഠിച്ചിട്ടില്ലാ ത്ത ആത്മരതിയുടെ സംസ്‌കാരം പേറുന്നവര്‍ക്ക് മുസ്‌ലിംകളുടെ നബിസ്‌നേഹത്തിന്റെ ആഴവും അര്‍ത്ഥവുമറിയാന്‍ കഴിയില്ല. ഒരാളു ടെ കാരിക്കേച്ചറുകളും കോമാളി വരകളും എന്തിനാണ് ഒരു ജനസമൂഹത്തെ വൈകാരികമായി സ്വാധീനിക്കുന്നതെന്ന് അവര്‍ക്ക് മനസ്സിലാ കാത്തത് ദൈവസ്‌നേഹത്തിന്റെയും അതില്‍ നിന്ന് ഉയിരെടുക്കുന്ന നബിസ്‌നേഹത്തിന്റെയും അവാച്യമായ അനുഭൂതി അവര്‍ക്ക് ആസ്വ ദിക്കാന്‍ കഴിയാത്തതുകൊണ്ടാണ്. വിശ്വാസം രൂഢമൂലമായ മനസ്സിനേ പ്രസ്തുത അനുഭൂതിയുടെ ആസ്വാദ്യത അനുഭവിക്കുവാന്‍ ഭാഗ്യം ലഭിക്കൂ. അവിശ്വാസത്തിന്റെ ആന്ധ്യവും ആസ്വാദനമെന്നാല്‍ ഇന്ദിയാസ്വാദനം മാത്രമാണെന്ന അധകൃത ബോധവും ബാധിച്ചവര്‍ പ്രസ്തു ത ഭാഗ്യമില്ലാതെ പോയവരാണ് എന്നതിനാല്‍ അവരോട് സഹതപിക്കാന്‍ മാത്രമെ വിശ്വാസികള്‍ക്കാവൂ. ലോകനാഥനോടുള്ള സ്‌നേഹ വും പ്രസ്തുത സ്‌നേഹം എങ്ങനെയെന്ന് പഠിപ്പിച്ച ദൂതനോടുള്ള പ്രേമവുമാണ് മുസ്‌ലിമിന്റെ വ്യക്തിത്വത്തെ പ്രകാശമാനമാക്കുന്നത്. മറ്റൊരാളെ സ്വന്തത്തെക്കാളേറെ സ്‌നേഹിക്കുന്നതെങ്ങനെയെന്ന് മനസ്സിലാക്കുവാന്‍ പോലും അവിശ്വാസവും ആത്മാരാധനയും മൂലം കടു ത്തുപോയ ഹൃദയത്തിന്റെ ഉടമകള്‍ക്ക് കഴിയില്ല. അതിന് ദൈവസമര്‍പ്പണത്തിന്റെ വെളിച്ചം ലഭിച്ച മനസ്സുണ്ടാകണം. അതില്ലാത്തവര്‍ ക്ക് നബിനിന്ദ മുസ്‌ലിം മനസ്സില്‍ സൃഷ്ടിക്കുന്ന വേദനയുടെ ആഴമളക്കാനാവില്ല. സ്വന്തം ഹൃദയത്തിനകത്തേക്ക് അമ്പ് തുളച്ചുകയറുമ്പോഴു ണ്ടാകുന്ന വേദനയേക്കാള്‍ അസഹ്യമാണ് മുസ്‌ലിമിന് നബിനിന്ദ കേള്‍ക്കുമ്പോഴുള്ള വേദന. അളന്നു തിട്ടപ്പെടുത്തുവാനോ അഭിനയിച്ചു കാണിച്ചുകൊടുക്കുവാനോ കഴിയുന്നതല്ല അത്. നബിസ്‌നേഹത്തിന്റെ ആസ്വാദ്യത അനുഭവിക്കുവാന്‍ ഭാഗ്യം ലഭിച്ചവര്‍ക്കേ പ്രസ്തുത വേദനയുടെ ആഴം മനസ്സിലാകൂ; ആത്മാരാധകരായ നബിനിന്ദകര്‍ക്ക് അത് മനസ്സിലാവില്ല.

നബിസ്‌നേഹത്തിന്റെ പ്രകടനത്തിന് ഹാവഭാവങ്ങളുടെ അകമ്പടിയുണ്ടാവണമെന്നില്ല. ചരിത്രത്തില്‍ ജീവിച്ചുകഴിഞ്ഞവരില്‍ നിന്നെല്ലാം നബി(സ) വ്യത്യസ്തമായതുപോലെ അദ്ദേഹത്തോടുള്ള സ്‌നേഹപ്രകടനവും വ്യത്യസ്തമാണ്. നബിസ്‌നേഹത്തിന്റെ പ്രയോഗവല്‍ക്കര ണമെന്നാല്‍ നബിജീവിതത്തിന്റെ അനുധാവനമാണെന്നാണ് അന്തിമ പ്രവാചകന്‍(സ) വിശ്വാസികളെ പഠിപ്പിച്ചത്. ജീവിതത്തിന്റെ ഏതേ ത് തുറകളിലുള്ളവര്‍ക്കാണെങ്കിലും മാതൃകയാക്കാന്‍ പറ്റിയതായി അവതരിപ്പിക്കാനാവുന്ന മറ്റൊരു വ്യക്തിത്വമില്ല; ഞങ്ങളുടെ പ്രവാച കനല്ലാതെയെന്ന വസ്തുത ആരുടെ മുന്നിലും ഉറക്കെപ്പറയാന്‍ കഴിയുന്ന ഒരേയൊരു ജനസമൂഹമാണ് മുസ്‌ലിംകള്‍. നബിജീവിതത്തിന്റെ മാതൃക വെളിച്ചം വീശാത്ത മേഖലകളൊന്നുമില്ല. സ്‌നേഹിക്കുന്നവരോടും വെറുക്കുന്നവരോടുമെല്ലാം എങ്ങനെ പെരുമാറണമെന്ന് പ്രവാ ചകന്‍ (സ) പഠിപ്പിച്ചിട്ടുണ്ട്. അനുയായികളെയും എതിരാളികളെയും എങ്ങനെ കൈകാര്യം ചെയ്യണമെന്നും സ്തുതിപാഠകന്‍മാരോടും നിന്ദകരോടുമുള്ള നിലപാടുകള്‍ എങ്ങനെയാകണമെന്നും നബി(സ) മാതൃക കാണിക്കാതെ വിട്ടുപോയിട്ടില്ല. ഏതാനും ചില അനുയായിക ളല്ലാതെ മക്കയിലുള്ള മറ്റുള്ളവരെല്ലാം പ്രവാചകന്റെ പ്രബോധനകാലത്തിന്റെ തുടക്കത്തില്‍ നബിനന്ദകരായിരുന്നു. പ്രസ്തുത നിന്ദക രില്‍ നിന്നാണ് പ്രവാചകാനുചരന്‍മാരെല്ലാം പരിണമിച്ചുണ്ടായത് എന്നുള്ള വസ്തുത നബിചരിത്രം നമ്മെ പഠിപ്പിക്കുന്നു. ഉമറും അബൂ സുഫ്‌യാനും ഖാലിദുബ്‌നു വലീദും സുഹൈലുബ്‌നു അംറും ഹിന്ദും തുടങ്ങി അറിയപ്പെട്ട നബിനിന്ദകരെയെല്ലാം ഇസ്‌ലാമിന്റെ കൊടി ക്കീഴില്‍ അണിനിരത്തി നബിസ്‌നേഹത്തിന്റെ നിതാന്ത മാതൃകകളാക്കിത്തീര്‍ത്ത ചരിത്രമാണ് പ്രവാചകന്റേത്. നിന്ദകരെ കൊന്നൊടു ക്കുകയല്ല, സ്വന്തം ജീവിതവിശുദ്ധിയിലൂടെ അനുയായികളാക്കിത്തീര്‍ക്കുകയാണ് വേണ്ടതെന്ന പാഠമാണ് നബി (സ) സ്വന്തം കര്‍മങ്ങളി ലൂടെ പഠിപ്പിച്ചത്. നബിനിന്ദകര്‍ക്കുള്ള പ്രവാചകകാലത്തെ മറുപടി നബിജീവിതത്തിന്റെ വെളിച്ചമായിരുന്നുവെങ്കില്‍ ഇന്നത്തെയും മറുപടി പ്രസ്തുത വെളിച്ചം തന്നെയാണ്. പതിനാല് നൂറ്റാണ്ടുകള്‍ക്ക് മുമ്പ് നബി(സ) പ്രസരിപ്പിച്ച വെളിച്ചം അതിന്റെ ഒളി ഒരല്‍പ്പംപോ ലും മങ്ങാതെ ഇന്നും ഇവിടെയുണ്ട്. പ്രസ്തുത വെളിച്ചത്തെ നബിനിന്ദകര്‍ക്കു നേരെ തിരിച്ചുവെക്കുകയെന്നതാണ് മുസ്‌ലിംകളുടെ ദൗത്യം. ലോകത്തെങ്ങുമുള്ള നബിനിന്ദകരെ കൊന്നൊടുക്കുവാന്‍ നബി (സ) പഠിപ്പിച്ചിട്ടില്ലാത്തതിനാല്‍ അങ്ങനെ ചെയ്യാനൊരുമ്പെടുന്നവര്‍ ഇക്കാ ര്യത്തില്‍ നബി(സ)യില്‍ നിന്ന് വെളിച്ചം സ്വീകരിക്കാത്തവരാണ്. ഇക്കാര്യത്തില്‍ അവരും നബിനിന്ദകരും ഒരേ പോലെയാണെന്ന് പറയു ന്നത് അതുകൊണ്ടാണ്.

ബി (സ) യുടെ ജീവിതത്തിലെ ഏതെങ്കിലും ഒരു സംഭവം ഉയർത്തിക്കാണിച്ച് അദ്ദേഹം വർഗീയവാദിയായിരുന്നുവെന്ന് സ്ഥാപിക്കുവാൻ കൊടിയ നബി വിമർശകർക്കു പോലും കഴിഞ്ഞിട്ടില്ല.വർഗീയതയുടെ ലാഞ്ചനപോലുമില്ലാതെ ജീവിച്ചയാളായിരുന്നു പ്രവാചകനെന്ന് വ്യക്തമാക്കുന്ന നൂറുകണക്കിന് സംഭവങ്ങൾ ആ മഹത്ജീവിതത്തിൽ കാണാൻ കഴിയും.

ഹിജ്റയോടനുബന്ധിച്ച ഒരു സംഭവം നോക്കുക. അല്ലാഹുവിന്റെ മാര്ഗെത്തില്‍ ഇറങ്ങിത്തിരിച്ചവരോടൊപ്പം അല്ലാഹുവുണ്ടാകുമെന്ന പാഠം നൽകുന്ന സംഭവമാണ് ഹിജ്‌റ. മൂന്നുദിവസത്തെ ഗുഹാവാസത്തിനു ശേഷം, റബീഉല്‍ അവ്വല്‍ ഒന്നാം തിയ്യതി രാത്രിയാണ് യഥ്‌രിബ് ലക്ഷ്യമാക്കിയുള്ള നബിസംഘത്തിന്റെ യാത്രയാരംഭിച്ചത്. നബി (സ) യും അബൂബക്കറും (റ) അദ്ദേഹത്തിന്റെ ദാസനായ ആമിറുബ്‌നു ഫുഹൈറയും വഴികാട്ടിയായ അബ്ദുല്ലാഹിബ്‌നു ഉറൈഖത്ത് അല്‍ ലൈഥിയുമടങ്ങുന്ന നാലുപേരുടെ സംഘം എട്ടാം തിയ്യതി തിങ്കളാഴ്ച പകൽ സമയത്ത് യഥ്‌രിബിന്റെ കവാട നഗരിയായ ഖുബായില്‍ എത്തുന്നതിനിടയിലെ സംഭവങ്ങളിലൂടെ നബി (സ) നിരവധി പാഠങ്ങള്‍ ലോകത്തിന് നല്കു്കയുണ്ടായി. നബിസംഘത്തിലെ വഴികാട്ടിയെപ്പറ്റി ബുഖാരിയിലെ നിവേദനത്തില്‍ പറയുന്നത് 'സത്യനിഷേധികളായ ഖുറൈശികളുടെ മതത്തിലായിരുന്നു അയാള്‍'(സ്വഹീഹുല്‍ ബുഖാരി, കിതാബുഫദാഇലില്‍ അന്സാ്ര്‍) എന്നാണ്. കീല്‍ ഗോത്രക്കാരനും അബ്ദുബ്‌നു അദിയ്യിന്റെ സന്തതികളില്‍ പെട്ടയാളും വിദഗ്ധനായ വഴികാട്ടിയുമായിരുന്ന അദ്ദേഹമായിരുന്നു മക്കയില്‍ നിന്ന് യഥ്‌രിബിലെത്തുവോളം പ്രവാചകന് (സ) വഴികാട്ടിയായി കുടെയുണ്ടായിരുന്നതെന്ന യാഥാര്ഥ്യംക.

തങ്ങളുടെ പലായന വിവരവും ഥൗര്‍ ഗുഹയിലെ താമസവിവരവും മൂന്നു ദിവസങ്ങൾക്കുശേഷമുള്ള യാത്രാവിവരവുമെല്ലാം മുസ്‌ലിമല്ലാത്ത ഒരു വിശ്വസ്തനുമായി നേരത്തെതന്നെ പങ്കുവെക്കുന്നതില്‍ പ്രവാചകൻ (സ)യാതൊരുവിധ അനൗചിത്യവും ദർശിച്ചില്ല. അബ്ദുല്ലാഹിബ്‌നു ഉറൈഖത്താകട്ടെ തന്നെ ഏല്പിതച്ച ദൗത്യം ഭംഗിയായി നിര്വഹഹിക്കുകയും ചെയ്തു. യാത്രയിലോ യാത്രയ്ക്കു മുമ്പോ നബി (സ) യുടെ ശത്രുക്കളെ അറിയാതെ, അവര്ക്ക് പരിചയമില്ലാത്ത വഴികളിലൂടെ, തികച്ചും സുരക്ഷിതവുമായി മുഹമ്മദ് നബി (സ) യെ അദ്ദേഹം യഥ്‌രിബിലെത്തിച്ചു. നബി (സ) യെ ഒറ്റിക്കൊടുക്കുകയോ അപായപ്പെടുത്തുകയോ ചെയ്യാന്‍ അദ്ദേഹം സന്നദ്ധമായില്ല; അങ്ങനെ ചെയ്താല്‍ അദ്ദേഹത്തിന് നൂറ് ഒട്ടകം ലഭിക്കുമായിരുന്നിട്ടും നബി (സ) യോടുള്ള വാഗ്ദത്തം പാലിച്ചുകൊണ്ട് മുശ്‌രിക്കുകളുടെ പാരിതോഷികം വേണ്ടെന്നുവെക്കുകയാണ് ഇബ്‌നു ഉറൈഖത്ത് ചെയ്തത്. വിശ്വസ്തരാണെങ്കില്‍, അമുസ്‌ലിംകളെപ്പോലും സുപ്രധാനമായ ദൗത്യങ്ങള്‍ ഏൽപിക്കാമെന്ന് പഠിപ്പിക്കുന്നതാണ് അബ്ദുല്ലാഹിബ്‌നു ഉറൈഖത്തിനെ ഹിജ്‌റയിലെ വഴികാട്ടിയായി നിശ്ചയിച്ച മുഹമ്മദ് നബി (സ) യുടെ നടപടി. വഞ്ചിക്കുകയില്ലെന്നുറപ്പുള്ളവരെ, അവരുടെ മതം നോക്കാതെത്തന്നെ രഹസ്യങ്ങള്‍ വരെ ഏൽപിക്കാമെന്ന് സ്വന്തം ജീവിതത്തിലൂടെ പഠിപ്പിച്ച പ്രവാചകനില്‍ വർഗീതയവിദ്വേഷത്തിന്റെ കുഴലൂത്തുകാരനെ തിരയുന്നവര്‍ ഹിമപാളികൾക്കിടയില്‍ അഗ്നി അന്വേഷിക്കുന്നതുപോലുയുള്ള വൃഥാവ്യായാമമാണ് ചെയ്യുന്നത്.

ഹിജ്‌റ നിർബന്ധമാണെന്ന് വ്യക്തമാക്കുന്ന ഖുർആൻ വചനങ്ങൾ മറുനാട്ടിൽ പോയി ജനിച്ച നാടിനെതിരെ പോരാടുവാനുള്ള പ്രചോദനം നൽകി ഭീകരത വളർത്തുന്നതാണെന്ന ആരോപണം പ്രസ്തുത വചങ്ങളുടെ അവതരണ പശ്ചാത്തലത്തെ കുറിച്ച് അറിയാത്തതുകൊണ്ടുണ്ടാവുന്നതാണ്.

''തീര്‍ച്ചയായും വിശ്വസിക്കുകയും, സ്വദേശം വെടിഞ്ഞ് പോകുകയും തങ്ങളുടെ സ്വത്തുക്കളും ശരീരങ്ങളും കൊണ്ട് അല്ലാഹുവിന്റെ മാര്‍ഗത്തില്‍ സമരത്തില്‍ ഏര്‍പെടുകയും ചെയ്തവരും, അവര്‍ക്ക് അഭയം നല്‍കുകയും സഹായിക്കുകയും ചെയ്തവരും അന്യോന്യം ഉറ്റമിത്രങ്ങളാകുന്നു. വിശ്വസിക്കുകയും എന്നാല്‍ സ്വദേശം വെടിഞ്ഞ് പോകാതിരിക്കുകയും ചെയ്തവരോട് അവര്‍ സ്വദേശം വെടിഞ്ഞ് പോരുന്നത് വരെ നിങ്ങള്‍ക്ക് യാതൊരു സംരക്ഷണ ബാധ്യതയുമില്ല. ഇനി മതകാര്യത്തില്‍ അവര്‍ നിങ്ങളുടെ സഹായം തേടുകയാണെങ്കില്‍ സഹായിക്കാന്‍ നിങ്ങള്‍ക്ക് ബാധ്യതയുണ്ട്. എന്നാല്‍ നിങ്ങളുമായി കരാറില്‍ ഏര്‍പെട്ടുകഴിയുന്ന ജനതയ്‌ക്കെതിരെ (നിങ്ങളവരെ സഹായിക്കാന്‍) പാടില്ല. അല്ലാഹു നിങ്ങള്‍ പ്രവര്‍ത്തിക്കുന്നതെല്ലാം കണ്ടറിയുന്നവനാകുന്നു.'' (8:72)

''വിശ്വസിക്കുകയും സ്വദേശം വെടിയുകയും തങ്ങളുടെ സ്വത്തുക്കളും ശരീരങ്ങളും കൊണ്ട് അല്ലാഹുവിന്റെ മാര്‍ഗത്തില്‍ സമരം നടത്തുകയും ചെയ്തവര്‍ അല്ലാഹുവിങ്കല്‍ ഏറ്റവും മഹത്തായ പദവിയുള്ളവരാണ്. അവര്‍ തന്നെയാണ് വിജയം പ്രാപിച്ചവര്‍. അവര്‍ക്ക് അവരുടെ രക്ഷിതാവ് അവന്റെ പക്കല്‍ നിന്നുള്ള കാരുണ്യത്തെയും പ്രീതിയെയും സ്വര്‍ഗത്തോപ്പുകളെയും പറ്റി സന്തോഷവാര്‍ത്ത അറിയിക്കുന്നു. അവര്‍ക്ക് അവിടെ ശാശ്വതമായ സുഖാനുഭവമാണുള്ളത്. അവരതില്‍ നിത്യവാസികളായിരിക്കും. തീര്‍ച്ചയായും അല്ലാഹുവിന്റെ അടുക്കലാണ് മഹത്തായ പ്രതിഫലമുള്ളത്.'' (9:20-22)

''(അവിശ്വാസികളുടെ ഇടയില്‍ തന്നെ ജീവിച്ചുകൊണ്ട്) സ്വന്തത്തോട് അന്യായം ചെയ്തവരെ മരിപ്പിക്കുമ്പോള്‍ മലക്കുകള്‍ അവരോട് ചോദിക്കും: നിങ്ങളെന്തൊരു നിലപാടിലായിരുന്നു? അവര്‍ പറയും: ഞങ്ങള്‍ നാട്ടില്‍ അടിച്ചൊതുക്കപ്പെട്ടവരായിരുന്നു. അവര്‍ (മലക്കുകള്‍) ചോദിക്കും: അല്ലാഹുവിന്റെ ഭൂമി വിശാലമായിരുന്നില്ലേ? നിങ്ങള്‍ക്ക് സ്വദേശം വിട്ട് അതില്‍ എവിടെയെങ്കിലും പോകാമായിരുന്നല്ലോ. എന്നാല്‍ അത്തരക്കാരുടെ വാസസ്ഥലം നരകമത്രെ. അതെത്ര ചീത്ത സങ്കേതം! എന്നാല്‍ യാതൊരു ഉപായവും സ്വീകരിക്കാന്‍ കഴിവില്ലാതെ, ഒരു രക്ഷാമാര്‍ഗവും കണ്ടെത്താനാകാതെ അടിച്ചൊതുക്കപ്പെട്ടവരായിക്കഴിയുന്ന പുരുഷന്‍മാരും സ്ത്രീകളും കുട്ടികളും ഇതില്‍ നിന്നൊഴിവാകുന്നു.'' (4:97,98)

മുഹമ്മദ് നബി (സ)  മദീനയിലെത്തുകയും അവിടെ ഒരു ഇസ്‌ലാമികസമൂഹം രൂപപ്പെടുകയും ചെയ്തതോടെ മക്കയിലുള്ള മുസ്‌ലിംകള്‍ക്കെല്ലാം ഹിജ്‌റ നിര്‍ബന്ധബാധ്യതയായിത്തീര്‍ന്നുവെന്ന് വ്യക്തമാക്കുന്നതാണ് ഈ വചനങ്ങൾ. പീഡനങ്ങള്‍ സഹിക്കേണ്ടിവരുമെന്ന് പേടിച്ച് ആദര്‍ശം പുറത്തുപറയാതെ ജീവിക്കേണ്ടുന്ന അവസ്ഥ വെടിഞ്ഞ് ഹിജ്‌റ ചെയ്ത് മദീനയിലെത്തുവാന്‍ വിശ്വാസികളെ ആഹ്വാനം ചെയ്യുന്നവയാണിവ. ഇസ്‌ലാമിക രാഷ്ട്രങ്ങളുണ്ടായതിനു ശേഷം പിന്നെ മക്കയില്‍ ജീവിക്കുന്നവര്‍ യഥാര്‍ഥത്തില്‍ സ്വീകരിച്ചിരിക്കുന്നത് ആത്മഹത്യാപരമായ നിലപാടാണെന്നും അവരുടെ ആദര്‍ശം സ്വാഭാവികമായും ചോര്‍ന്നുപോകാന്‍ സാധ്യതയുണ്ടെന്നും നബി (സ) വ്യക്തമാക്കിയിട്ടുണ്ട്. അതുകൊണ്ടാണ് ‘ബഹുദൈവാരാധകരോടൊപ്പം ജീവിക്കുന്ന മുസ്‌ലിംകളുടെ കാര്യത്തില്‍ എനിക്കൊരു ഉത്തരവാദിത്തവുമില്ല; 'അവരുടെ തീയില്‍ നിന്ന് നിങ്ങള്‍ വെളിച്ചമെടുക്കേണ്ടിവരരുത്'(സുനനു അബൂദാവൂദ്) എന്ന് പ്രവാചകൻ (സ) പഠിപ്പിച്ചത്. പ്രവാചകന്റെ നേതൃത്വത്തില്‍ ഇസ്‌ലാമികരാഷ്ട്രമുണ്ടാവുകയും അവിടെ സ്വതന്ത്രമായി മതമനുഷ്ഠിച്ച് ജീവിക്കുവാന്‍ കഴിയുന്ന അവസ്ഥയുണ്ടാവുകയും ചെയ്തശേഷം മുശ്‌രിക്കുകളോടൊപ്പം ജീവിച്ച് അവരുടെ പീഡനങ്ങള്‍ സഹിക്കേണ്ടതില്ലെന്നും അങ്ങനെ ചെയ്യുന്നപക്ഷം അവരുടെ ആശയങ്ങളില്‍ പലതും സ്വീകരിക്കേണ്ട അവസ്ഥയുണ്ടാകുമെന്നും അത്തരക്കാരുടെ കാര്യത്തില്‍ എനിക്ക് ഉത്തരവാദിത്തമൊന്നുമില്ലെന്നുമാണ് ഇവിടെ പ്രവാചകൻ (സ) നിഷ്‌കര്‍ഷിക്കുന്നത്.

ഇസ്‌ലാമികാദര്‍ശമനുസരിച്ച് ജീവിക്കുവാന്‍ സ്വാതന്ത്ര്യം നല്‍കപ്പെടുന്ന നാട്ടില്‍ നിന്ന് ഹിജ്‌റ പോവേണ്ടതില്ലെന്ന വസ്തുത ഹിജ്‌റ ആറാം വര്‍ഷം വരെ അബ്‌സീനിയയില്‍ കഴിയാന്‍ ജഅ്ഫറുബ്‌നു അബീത്വാലിബിനെയും  (റ)കൂടെയുള്ളവരെയും അനുവദിച്ച പ്രവാചകനടപടിയില്‍ നിന്ന് വ്യക്തമാവുന്നുണ്ട്. മദീനാരാഷ്ട്രത്തിന്റെ സൈനിക ശക്തി വര്‍ധിപ്പിക്കുവാനാണ് എല്ലാ മുസ്‌ലിംകളോടും ഹിജ്‌റ ചെയ്യാന്‍ മുഹമ്മദ് നബി (സ)കല്‍പിച്ചതെന്ന് വിമര്‍ശിക്കുന്നവര്‍ നബി (സ) ക്ക് അത്തരമൊരു ഉദ്ദേശമുണ്ടായിരുന്നെങ്കില്‍ തന്നെ അതിലെന്താണ് തെറ്റെന്ന് പറഞ്ഞുതരുവാന്‍ ബാധ്യസ്ഥരാണ്. മക്കയിലും മറ്റു സ്ഥലങ്ങളിലും പീഡനങ്ങളനുഭവിച്ചുകൊണ്ടിരിക്കുന്നവര്‍ മദീനയിലെത്തുകയും അതുവഴി ഇസ്‌ലാമികരാഷ്ട്രത്തിന്റെ സൈനികശക്തി വര്‍ധിക്കുകയും ചെയ്യണമെന്ന് ഒരു രാഷ്ട്രനേതാവെന്ന നിലയില്‍ മുഹമ്മദ് നബി (സ)ആഗ്രഹിച്ചുവെങ്കില്‍, ഏതു തരം നൈതികയുടെ അടിസ്ഥാനത്തിലാണ് അതില്‍ തെറ്റുകാണാന്‍ കഴിയുകയെന്ന് അവര്‍ വ്യക്തമാക്കേണ്ടത്.

അസ്വാതന്ത്ര്യത്തില്‍ നിന്ന് സ്വാതന്ത്ര്യത്തിലേക്കുള്ള പലായനമാണ് ഹിജ്‌റ. ഒരു രാജ്യം ഇസ്‌ലാമികമാണോ അല്ലേയെന്നതല്ല പ്രത്യുത മുസ്‌ലിമായി ജീവിക്കുവാന്‍ അനുവദിക്കുന്നതാണോ അല്ലേയെന്നതാണ് അവിടെ നിന്ന് മുസ്‌ലിംകള്‍ ഹിജ്‌റ പോകേണ്ടതുണ്ടോ ഇല്ലേയെന്ന് തീരുമാനിക്കുന്നത്. രാജ്യങ്ങളെ ദാറുല്‍ ഇസ്‌ലാം, ദാറുല്‍ ഹര്‍ബ്, ദാറുല്‍ അഹദ് എന്നിങ്ങനെ വര്‍ഗീകരിക്കുന്ന പില്‍ക്കാല രീതിയനുസരിച്ച് ദാറുല്‍ ഹര്‍ബില്‍ നിന്ന്- (മുസ്‌ലിംകളുമായി യുദ്ധത്തിലേര്‍പ്പെട്ടിരിക്കുന്ന അമുസ്‌ലിം രാഷ്ട്രം) ദാറുല്‍ ഇസ്‌ലാമിലേക്കാണ് (ഇസ്‌ലാമികരാഷ്ട്രം) ഹിജ്‌റ നടക്കേണ്ടത്. ദാറുല്‍ അഹദ് ഇസ്‌ലാമികരാഷ്ട്രവുമായി സന്ധിയിലേര്‍പ്പെട്ടിരിക്കുന്ന രാജ്യമാണ്; അവിടെ മുസ്‌ലിംകള്‍ക്ക് സ്വാതന്ത്ര്യത്തോടെ മതമനുഷ്ഠിച്ച് ജീവിക്കുവാന്‍ സാധിക്കുന്നതിനാല്‍ അത്തരം രാജ്യങ്ങളില്‍ നിന്ന് ഹിജ്‌റ ചെയ്യേണ്ടതില്ലെന്ന് വ്യക്തമാക്കുന്നതാണ് ഇസ്‌ലാമികരാഷ്ട്രമുണ്ടായി വര്‍ഷങ്ങള്‍ കഴിഞ്ഞിട്ടും അബ്‌സീനിയയില്‍ ജീവിക്കുവാന്‍ മുസ്‌ലിംകളെ അനുവദിച്ച പ്രവാചകന്റെ നടപടി. ജനാധിപത്യക്രമം നിലനില്‍ക്കുന്ന രാഷ്ട്രസംവിധാനങ്ങളില്‍ മതസ്വാതന്ത്ര്യം അനുവദിക്കപ്പെടുന്നുവെങ്കില്‍ അതിനെ വിളിക്കുവാന്‍ ഏറ്റവും അനുയോജ്യമായ പദം ‘ദാറുല്‍ അംന്’-സുരക്ഷിത നാട്- എന്നാണ്. മുസ്‌ലിമായി ജീവിക്കുവാന്‍ സ്വാതന്ത്ര്യം നല്‍കുകയും അത് പ്രബോധനം ചെയ്യാന്‍ അനുവദിക്കുകയും ചെയ്യുന്ന രാഷ്ട്രമാണ് ദാറുല്‍ അംന്. അത്തരം രാജ്യങ്ങളോട് കലാപത്തിലേര്‍പ്പെടുകയോ അവിടെ നിന്ന് പലായനം ചെയ്യുകയോ അല്ല; മാതൃകായോഗ്യരായി ജീവിക്കുകയും സഹജീവികള്‍ക്ക് സത്യസന്ദേശമെത്തിക്കുകയുമാണ് മുസ്‌ലിമിന്റെ കടമ.

'ഇനി ഹിജ്‌റയില്ല'(സ്വഹീഹുല്‍ ബുഖാരി) യെന്ന് മക്കാ വിജയ ദിവസം പ്രവാചകൻ (സ) പ്രഖ്യാപിച്ചതില്‍ നിന്നും 'മക്കാ വിജയത്തിനു ശേഷം ഹിജ്‌റയില്ല'(സ്വഹീഹുമുസ്‌ലിം) യെന്ന് അദ്ദേഹം വ്യക്തമാക്കിയതില്‍ നിന്നും സമാധാനത്തോടെ ജീവിക്കുവാന്‍ കഴിയുന്ന സാഹചര്യങ്ങള്‍ സംജാതമായിക്കഴിഞ്ഞാല്‍ പിന്നെ പലായനം വേണ്ടതില്ലെന്ന് മനസ്സിലാവുന്നുണ്ട്. മക്കയില്‍ മുസ്‌ലിമായി ജീവിക്കുവാനുള്ള സാഹചര്യങ്ങള്‍ ഒരുങ്ങിക്കഴിഞ്ഞ സ്ഥിതിക്ക് ഇനി മക്കയില്‍ നിന്ന് മദീനയിലേക്കുള്ള ഹിജ്‌റയുടെ ആവശ്യമില്ലെന്നാണ് ഇതു പഠിപ്പിക്കുന്നത്. എന്നാല്‍, മതത്തിന്റെ പേരിലുള്ള പീഡനങ്ങള്‍ നിലനില്‍ക്കുന്നിടത്തോളം ഹിജ്‌റ അനിവാര്യമായിത്തീരുമെന്നും പ്രവാചകൻ (സ)പഠിപ്പിച്ചിട്ടുണ്ട്. 'പശ്ചാത്താപം നിലയ്ക്കുന്നതുവരെ ഹിജ്‌റ അവസാനിക്കുകയില്ല. സൂര്യന്‍ പടിഞ്ഞാറ് നിന്ന് ഉദിച്ചുയരുന്നതുവരെ പശ്ചാത്താപം അവസാനിക്കുകയില്ല'(സുനനു അബൂദാവൂദ് ) എന്ന പ്രവാചകവചനം വ്യക്തമാക്കുന്നത് ഇക്കാര്യമാണ്. മതത്തിന്റെ പേരിലുള്ള പീഡനങ്ങള്‍ നിലനില്‍ക്കുന്നിടത്തോളം, അവസാനനാളു വരെ ഹിജ്‌റ പ്രസക്തമായിരിക്കുമെന്ന് സാരം.

ജനിച്ചു വളര്‍ന്ന നാടിനെക്കാളും പോറ്റി വളര്‍ത്തിയ മാതാപിതാക്കളേക്കാളും അധ്വാനിച്ചുണ്ടാക്കിയ സമ്പാദ്യത്തേക്കാളുമെല്ലാം മുസ്‌ലിംകള്‍ വിലമതിക്കുന്നത് ദൈവിക മാര്‍ഗദര്‍ശനപ്രകാരമുള്ള ജീവിതവും അതെപ്പറ്റി സഹജീവികള്‍ക്ക് പറഞ്ഞുകൊടുക്കുവാനുള്ള അവസരവുമാണെന്നതാണ് ഹിജ്‌റ നല്‍കുന്ന സന്ദേശം. കലാപവും കുഴപ്പവുമുണ്ടാക്കി ജീവിക്കുന്ന നാട്ടില്‍ അരാജകത്വം സൃഷ്ടിക്കുന്നതിനല്ല ആദര്‍ശജീവിതത്തിന് പറ്റുന്നിടത്തേക്ക് പലായനം ചെയ്യുവാനാണ് ഇസ്‌ലാം അനുശാസിക്കുന്നത്. അങ്ങനെ പലായനം ചെയ്യുന്നവര്‍ക്കാണ് അല്ലാഹുവിന്റെയടുക്കല്‍ ഉന്നതമായ പദവിയും സ്വര്‍ഗീയ സുഖസൗകര്യങ്ങളുമുണ്ടെന്ന് ക്വുര്‍ആന്‍ സന്തോഷവാര്‍ത്തയറിയിക്കുന്നത്.

''വിശ്വസിക്കുകയും സ്വദേശം വെടിയുകയും തങ്ങളുടെ സ്വത്തുക്കളും ശരീരങ്ങളും കൊണ്ട് അല്ലാഹുവിന്റെ മാര്‍ഗത്തില്‍ സമരം നടത്തുകയും ചെയ്തവര്‍ അല്ലാഹുവിങ്കല്‍ ഏറ്റവും മഹത്തായ പദവിയുള്ളവരാണ്. അവര്‍ തന്നെയാണ് വിജയം പ്രാപിച്ചവര്‍. അവര്‍ക്ക് അവരുടെ രക്ഷിതാവ് അവന്റെ പക്കല്‍ നിന്നുള്ള കാരുണ്യത്തെയും പ്രീതിയെയും സ്വര്‍ഗത്തോപ്പുകളെയും പറ്റി സന്തോഷവാര്‍ത്ത അറിയിക്കുന്നു. അവര്‍ക്ക് അവിടെ ശാശ്വതമായ സുഖാനുഭവമാണുള്ളത്. അവരതില്‍ നിത്യവാസികളായിരിക്കും. തീര്‍ച്ചയായും അല്ലാഹുവിന്റെ അടുക്കലാണ് മഹത്തായ പ്രതിഫലമുള്ളത്.'' (9:20-22)

മുഹമ്മദ് നബി (സ) പ്രവാചകത്വം വാദിച്ചത് തന്നെ അധികാരം ലക്ഷ്യമാക്കിയായിരുന്നു. കുറച്ച് അനുയായികളായപ്പോൾ തനിക്ക് അധികാരമുള്ള ഒരു നാട് അദ്ദേഹം അന്വേഷിക്കാൻ തുടങ്ങി. പ്രസ്തുത അന്വേഷണമാണ് അദ്ദേഹത്തെ മദീനയിലെത്തിക്കുകയും അവിടുത്തെ അധികാരിയായിത്ത്തീരുന്നതിന് കാരണമാവുകയും ചെയ്തത്. ഈ ആരോപണത്തെക്കുറിച്ച് എന്ത് പറയുന്നു?

ധികാരം മുന്നില്‍ കണ്ടുകൊണ്ട്, തന്റെ കീഴിലുള്ള ഒരു രാഷ്ട്ര നിര്‍മിതിക്കുവേണ്ടി മുഹമ്മദ് ല നടത്തിയ ശ്രമത്തിന്റെ പ്രാരംഭമായി ഒരു തലസ്ഥാന നഗരി അന്വേഷിച്ചതിന്റെ ഫലമായാണ് അദ്ദേഹം മദീനയിലെത്തിയതെന്ന് വിമര്‍ശിക്കുന്നവര്‍ മദീനാ ഹിജ്‌റയ്ക്കുമുമ്പ് നടന്ന സംഭവങ്ങളെല്ലാം കണ്ടില്ലെന്ന് നടിക്കുകയാണ് ചെയ്യുന്നത്. തനിക്ക് ഭരിക്കാന്‍ പറ്റുന്ന ഒരു പ്രദേശം തിരഞ്ഞ് കണ്ടെത്തുകയായിരുന്നില്ല, ഇസ്‌ലാമിക പ്രബോധനത്തിന് സ്വാതന്ത്ര്യമുള്ള ഒരിടത്ത് അഭയം ലഭിക്കുകയായായിരുന്നു നബി (സ)യുടെ ഉദ്ദേശമെന്ന് ദുഃഖവര്‍ഷമെന്ന് വിളിക്കപ്പെട്ട, അബൂത്വാലിബിന്റെയും ഖദീജ(റ) യുടെയും മരണം നടന്ന വര്‍ഷത്തിന് ശേഷമുള്ള നബിവര്‍ത്തനങ്ങള്‍ പരിശോധിക്കുന്ന ആര്‍ക്കും മനസ്സിലാകും. 'അബൂത്വാലിബ് ജീവിച്ചിരുന്നെങ്കില്‍ ഖുറൈശികള്‍ എന്നോട് ഇങ്ങനെയൊന്നും ചെയ്യുകയില്ലായിരുന്നു' വെന്ന് നബി (സ)യെക്കൊണ്ട് പറയിപ്പിക്കുന്ന രീതിയിലുള്ള ഖുറൈശീ ക്രൂരതകളാണ് മക്കയില്‍ നിന്ന് പോവുകയെന്ന തീരുമാനത്തിലേക്ക് നബി (സ) എത്തുന്നതിനുള്ള കാരണം.

തനിക്ക് സംരക്ഷണം തേടിക്കൊണ്ട് ഒന്നാമതായി നബി (സ) നടത്തിയ ത്വാഇഫിലേക്കുള്ള യാത്രയുടെ ലക്ഷ്യമെന്തായിരുന്നുവെന്ന് അവിടെ എത്തിയശേഷം അദ്ദേഹം എന്താണ് ആവശ്യപ്പെട്ടതെന്ന് പരിശോധിച്ചാല്‍ വ്യക്തമാകും. ഥഖീഫ് ഗോത്രത്തിലെ പ്രധാനികളായ, അംറുബ്‌നു ഉമൈറിന്റെ പുത്രന്‍മാരായ അബ്ദ് യഅ്‌ലില്‍, മസ്ഊദ്, ഹബീബ് എന്നിവരോട് ഇസ്‌ലാമിനെക്കുറിച്ച് പറഞ്ഞുകൊടുക്കുകയും അതിലേക്ക് ക്ഷണിക്കുകയും തന്നെ സംരക്ഷിക്കണമെന്ന് ആവശ്യപ്പെടുകയുമാണ് നബി (സ) ചെയ്തത്. ത്വാഇഫിന്റെ അധികാരം ചോദിക്കുകയോ അധികാരത്തില്‍ പങ്കുനല്‍കണമെന്ന് ആവശ്യപ്പെടുകയോ ചെയ്തതുകൊണ്ടാണ് ഥഖീഫ് ഗോത്രക്കാര്‍ നബി (സ)യെ അവമതിക്കുകയും അപമാനിക്കുകയും ചെയ്തതെന്ന് നബി (സ)യെക്കുറിച്ച് നബി വിരോധികള്‍ എഴുതിയ ചരിത്ര ഗ്രന്ഥങ്ങള്‍പോലും ആരോപിക്കുന്നില്ല.  ത്വാഇഫില്‍ നിന്ന് തുടങ്ങിയ സംരക്ഷണം ആവശ്യപ്പെട്ടുകൊണ്ടുള്ള നബി (സ)യുടെ യാത്രയാണ് യഥ്‌രിബില്‍ ചെന്ന് അവസാനിക്കുകയും അത് മദീനത്തുര്‍റസൂലായി മാറുന്നതിലേക്ക് നയിക്കുകയും ചെയ്തതെന്നുള്ളതാണ് യാഥാര്‍ഥ്യം.

ത്വാഇഫില്‍നിന്ന് മുത്വ്ഇം ബിന്‍ ഹാദിയുടെ സംരക്ഷണത്തില്‍ മക്കയിലേക്ക് തിരിച്ചെത്തിയതുമുതല്‍ മുഹമ്മദ് നബി (സ)യുടെ അന്വേഷണങ്ങള്‍ കേന്ദ്രീകരിച്ചിരുന്നത് തനിക്ക് അധികാരം ലഭിക്കുവാന്‍ എന്തെങ്കിലും മാര്‍ഗങ്ങളുണ്ടോയെന്നതിലായിരുന്നില്ലെന്നും പ്രത്യുത തനിക്കും ഇസ്‌ലാമിനും സംരക്ഷണം നല്‍കാന്‍ ആരെങ്കിലുമുണ്ടോയെന്നതിലായിരുന്നെന്നുമുള്ള വസ്തുത ഹിജ്‌റക്ക് പിന്നില്‍ അധികാര സ്ഥാപനം ആരോപിക്കുന്നവരുടെ കണ്ണ് തുറപ്പിക്കേണ്ടതാണ്. മക്കക്ക് പുറത്തുനിന്ന് ഹജ്ജിനും ഉംറക്കും വരുന്നവരോട് തന്നെ സംരക്ഷിക്കുവാന്‍ സന്നദ്ധരായ ആരെങ്കിലുമുണ്ടോയെന്ന് അന്വേഷിച്ച് മുഹമ്മദ് നബി (സ) ചുറ്റി നടന്നിരുന്നുവെന്ന് സ്വഹീഹായി നിവേദനം ചെയ്യപ്പെട്ടിട്ടുണ്ട്. 'എന്റെ നാഥന്റെ വചനങ്ങള്‍ ജനങ്ങള്‍ക്ക് എത്തിച്ചുകൊടുക്കുന്നതില്‍ നിന്ന് ഖുറൈശികള്‍ എന്നെ തടഞ്ഞിരിക്കുന്നു; തങ്ങളുടെ ജനതയിലേക്ക് എന്നെ കൊണ്ടുപോകുവാന്‍ ആരെങ്കിലും തയാറുണ്ടോ?'യെന്ന് ഹജ്ജിന് വരുന്നവരോടായി അറഫയില്‍വെച്ച് നബി പറഞ്ഞുകൊണ്ടിരുന്നതായി ജാബിറുബ്‌നു അബ്ദില്ല (റ) നിവേദനം ചെയ്യുന്നുണ്ട്.(അബൂദാവൂദ്, തിര്‍മിദി, ഇബ്‌നുമാജ) തന്നെക്കൊണ്ടാവുന്ന വിധത്തില്‍ ഓരോ തീര്‍ഥാടകരെയും നേരില്‍ കണ്ട് അവരുടെ പേര് വിളിച്ചുകൊണ്ട് പ്രവാചകൻ (സ) ഇങ്ങനെ പറയുമായിരുന്നുവത്രെ: 'ഞാന്‍ നിങ്ങളിലേക്കുള്ള അല്ലാഹുവിന്റെ ദൂതനാണ്. അവനെ ആരാധിക്കണമെന്നും അവനില്‍ ആരെയും പങ്കുചേര്‍ക്കരുതെന്നും അവന്‍ കല്‍പിച്ചിരിക്കുന്നു. അവനെയല്ലാതെ നിങ്ങള്‍ ആരാധിച്ചുകൊണ്ടിരിക്കുന്നവയെയെല്ലാം നിങ്ങള്‍ കയ്യൊഴിക്കുക; എന്നില്‍ വിശ്വസിക്കുകയും എന്നെ സംരക്ഷിക്കുകയും ചെയ്ത് എന്തിനാണോ ഞാന്‍ അയക്കപ്പെട്ടത് അത് ജനങ്ങള്‍ക്ക് സമര്‍പ്പിക്കുവാന്‍ എനിക്ക് അവസരം നല്‍കുക'(മുസ്‌നദ് ഇമാം അഹ്മദ് ). ഈ സംസാരങ്ങളിലൊന്നും തന്നെ അധികാര സംസ്ഥാപനത്തിന്റെയോ അതിനുള്ള ഒരുക്കത്തിന്റെയോ പ്രതീക്ഷയുടെയോ ലാഞ്ചനകളൊന്നും കാണാനാവുന്നില്ലെങ്കില്‍ പിന്നെ ഹിജ്‌റയുടെ ലക്ഷ്യം രാഷ്ട്രസംസ്ഥാപനമായിരുന്നുവെന്ന് കരുതുന്നതിന് എന്ത് ന്യായീകരണമാണുള്ളത്.

താന്‍ സഹായമഭ്യര്‍ഥിക്കുന്ന തീര്‍ഥാടകരുടെയെല്ലാം പിന്നില്‍ നടന്ന് അബൂലഹബ് പിന്തിരിപ്പിക്കുകയും അപവാദ പ്രചരണം നടത്തുകയും ചെയ്യുന്ന അവസ്ഥ സംജാതമായപ്പോള്‍ രഹസ്യമായി ഗോത്ര നേതാക്കളെ സന്ദര്‍ശിച്ച് സംരക്ഷണം അഭ്യര്‍ഥിക്കുന്ന പ്രവാചകനിലും (സ) നാം കാണുന്നത് ഒരു അധികാര കേന്ദ്രമന്വേഷിക്കുന്ന രാഷ്ട്രീയ നേതാവിനെയല്ല, പ്രത്യുത ആദര്‍ശ പ്രചരണ സ്വാതന്ത്ര്യത്തിന് ദാഹിക്കുന്ന ദൈവദൂതനെയാണ്. കിന്‍ക, കല്‍ബ്, ബനൂ ആമിര്‍, ബനൂ ഹനീഫ, മുഹാരിബ്, ഫുസാറ, ഗസ്സാന്‍, മുര്‍റ, ഹനീഫ, സുലൈം, അബ്‌സ്, ബനൂ നസ്വ്‌റ്, ബനൂ അല്‍ ബക്കാഅ്, അല്‍ ഹാരിഥ്, ഉദ്‌റ, ഹദാറമ തുടങ്ങിയ ഗോത്രങ്ങള്‍ക്കടുത്തെത്തി തന്നെയും തന്റെ ആദര്‍ശ പ്രചരണത്തെയും സംരക്ഷിക്കണമെന്ന് ആവശ്യപ്പെടുകയുമാണ് പ്രവാചകൻ (സ) ചെയ്തത്. അവിടെയൊന്നും തന്നെ അധികാര സ്ഥാപനത്തിന്റെ യാതൊരു പരാമര്‍ശവും കാണാന്‍ കഴിയില്ല. പ്രവാചകനോട്‌ (സ) വളരെ മോശമായി പെരുമാറിയ ഗോത്രങ്ങളിലൊന്ന് ബനൂ ആമിറുബ്‌നു സഅ്‌സാഅ് ആയിരുന്നു. തന്റെ സംരക്ഷണാഭ്യര്‍ഥനക്ക് മറുപടിയായി ബനൂ ആമിര്‍ ഗോത്രത്തിലെ ബൈഹറാബ്‌നു ഫിറാസ് 'താങ്കളുമായി ഞങ്ങള്‍ ഉടമ്പടി ചെയ്യുകയും എന്നിട്ട് അറബികള്‍ക്കുമേല്‍ അല്ലാഹു താങ്കളെ വിജയിപ്പിക്കുകയും ചെയ്താല്‍ താങ്കള്‍ക്കുശേഷം ഞങ്ങള്‍ക്ക് അധികാരം നല്‍കുമോ?'യെന്ന് ചോദിച്ചപ്പോള്‍ 'കാര്യം അല്ലാഹുവിന്റെ പക്കലാണ്; അവര്‍ ഉദ്ദേശിക്കുന്നയിടത്താണ് അവനത് വെക്കുക'യെന്നാണ് പ്രവാചകൻ (സ) മറുപടി പറഞ്ഞത്. 'നിനക്കുവേണ്ടി ഞങ്ങള്‍ ഞങ്ങളുടെ കഴുത്തുകള്‍ അറബികളുടെ വാളുകള്‍ക്ക് മുന്നില്‍ വെക്കുകയും അവസാനം വിജയിക്കുമ്പോള്‍ നേതൃത്വം മറ്റുള്ളവര്‍ക്കും അല്ലേ... നിന്റെ ഈ കാര്യം ഞങ്ങള്‍ക്കാവശ്യമില്ല'യെന്ന് പറഞ്ഞാണ് അവര്‍ അല്ലാഹുവിന്റെ ദൂതനെ തിരിച്ചയച്ചത്.

ഹജ്ജിന് വരുന്നവരെ ഇസ്‌ലാമിലേക്ക് ക്ഷണിക്കുന്ന സന്ദര്‍ഭത്തില്‍ സത്യദീനിന്റെ സൗരഭ്യം മനസ്സിലാക്കി അത് സ്വീകരിക്കുവാന്‍ ഭാഗ്യം ലഭിച്ചവരിലൂടെയാണ് ഇസ്‌ലാം ആദ്യമായി യഥ്‌രിബിലെത്തുന്നതും നബി (സ)യുടെ പലായനത്തിന്റെ വാതില്‍ തുറന്നതും. കവിയും ബുദ്ധിജീവിയും ഉന്നതകുലജാതനും ധൈര്യശാലിയുമെല്ലാമായതിനാല്‍ പൂര്‍ണനെന്ന് (അല്‍ കാമില്‍) വിളിക്കപ്പെട്ടിരുന്ന സുബൈദ് ബ്‌നു സാമിതും ഖസ്‌റജുകാര്‍ക്കെതിരെ കക്ഷിചേരുവാന്‍ ഖുറൈശികളോട് അഭ്യര്‍ഥിക്കാന്‍ വന്ന സംഘത്തിലെ യുവാവായിരുന്ന ഇയാസ് ബ്‌നു മുആദുമെല്ലാം പ്രവാചകന്റെ (സ) പ്രബോധനത്തില്‍ ആകൃഷ്ടരായി ഇസ്‌ലാം സ്വീകരിച്ച യഥ്‌രിബുകാരായിരുന്നു.

വ്യക്തിപരമായ പ്രബോധന പ്രവര്‍ത്തനങ്ങളിലൂടെ മെല്ലെ മെല്ലെ യഥ്‌രിബിലേക്ക് പ്രവേശിച്ചുകൊണ്ടിരുന്ന ഇസ്‌ലാമിന്റെ വെളിച്ചം പടര്‍ന്ന് പരന്ന് പ്രവാചകന്റെ (സ) പട്ടണമായിത്തീരുന്നതിലേക്ക് തിരിയുന്നതിന് കാരണമായിത്തീര്‍ന്ന ആദ്യ സംഭവം പ്രവാചക നിയോഗത്തിന്റെ പതിനൊന്നാം വര്‍ഷം നബിയുംല ഖസ്‌റജ് ഗോത്രക്കാരും തമ്മില്‍ നടന്ന ഒരു കൂടിക്കാഴ്ചയായിരുന്നു. ഖസ്‌റജുകാരുമായി നബി (സ) നടത്തിയ കൂടിക്കാഴ്ചയില്‍ ഇസ്‌ലാമിന്റെ അടിസ്ഥാന തത്ത്വങ്ങളെക്കുറിച്ച് പറഞ്ഞുകൊടുക്കുകയും താന്‍ ദൈവദൂതനാണെന്ന വസ്തുത വ്യക്തമാക്കുകയും ചെയ്തപ്പോള്‍ അവരോടൊപ്പം യഥ്‌രിബില്‍ വസിച്ചിരുന്ന ജൂതന്‍മാരില്‍നിന്ന് അന്തിമപ്രവാചകന്റെ ആഗമനമുണ്ടാകുമെന്ന് മനസ്സിലാക്കിയിരുന്ന അവര്‍ക്ക് അദ്ദേഹത്തില്‍ വിശ്വസിക്കാന്‍ പ്രയാസമുണ്ടായില്ല. അവരെല്ലാവരും മുസ്‌ലിംകളായിത്തീരുകയും അവരുടെ നാട്ടുകാര്‍ക്ക് ഇസ്‌ലാം എത്തിച്ചുകൊടുക്കാമെന്ന് സമ്മതിക്കുകയും ചെയ്തു. ആഭ്യന്തര യുദ്ധങ്ങളാല്‍ പരസ്പരം ശത്രുതയില്‍ വര്‍ത്തിക്കുന്ന തങ്ങളുടെ ജനത ഇസ്‌ലാമിലൂടെ ഒറ്റക്കെട്ടായിത്തീര്‍ന്നാല്‍ പ്രവാചകൻ (സ)നെക്കാള്‍ പ്രതാപവാനായി ആരുമില്ലാത്ത അവസ്ഥാവിശേഷമുണ്ടാകുമെന്ന് പ്രത്യാശ പുറപ്പെടുവിച്ചുകൊണ്ടാണ് ആ ഖസ്‌റജുകാര്‍ പിന്നീട് നബി (സ) യുടെ പട്ടണമായി-മദീനത്തുല്‍ റസൂല്‍ അറിയപ്പെട്ട യഥ്‌രിബിലേക്ക് യാത്രതിരിച്ചത്.

അധികാരകേന്ദ്രമന്വേഷിച്ചുള്ള പ്രയാണമല്ല, പ്രത്യുത ആദർശമനുസരിച്ച് ജീവിക്കുവാനും അത് പ്രബോധനം ചെയ്യുന്നതിനുമുള്ള സ്വാതന്ത്യവും തേടിയുള്ള യാത്രയാണ് യഥാർത്ഥത്തിൽ പ്രവാചകനെ മദീനയിലെത്തിച്ചതെന്ന വസ്തുതയാണ് ഇതെല്ലാം വ്യക്തമാക്കുന്നത്.

ല്ല. പ്രവാചകാനുചരന്മാരുടെ ഒന്നാമത്തെ ഹിജ്റ തന്നെ അമുസ്‌ലിം നാട്ടിലേക്കായിരുന്നു. മദീനയിലേക്കുള്ള ഹിജ്റയും അബ്സീനിയയിലേക്കുള്ള ഹിജ്റയും തമ്മിൽ എന്തെങ്കിലും വ്യത്യാസമുണ്ടെന്ന് പ്രവാചകൻ (സ) പഠിപ്പിച്ചിട്ടില്ല.

ആദര്‍ശമനുസരിച്ച് ജീവിക്കാനാവാത്ത സാഹചര്യമാണ് ജനിച്ചുവളര്‍ന്ന നാട്ടില്‍ നിലനില്‍ക്കുന്നതെങ്കില്‍ അതിന് സ്വാതന്ത്ര്യമുള്ള നാട്ടിലേക്ക്, അത് ആര് ഭരിക്കുന്നുവെന്നോ അവിടെ നിലനില്‍ക്കുന്ന വ്യവസ്ഥിതിയോ നോക്കാതെ പലായനം ചെയ്യുകയാണ് മുസ്‌ലിം ചെയ്യേണ്ടതെന്ന പാഠമാണ് ഹിജ്രകളെല്ലാം നൽകുന്നത്.. അബ്‌സീനിയ ക്രൈസ്തവ രാജ്യമാണെന്നതോ അവിടുത്തെ രാജാവ് ക്രിസ്തുമതക്കാരനാണെന്നതോ ഹിജ്‌റയില്‍നിന്ന് അനുചരന്മാരെ പിന്തിരിപ്പിക്കുവാനുള്ള കാരണമായി പ്രവാചകൻ (സ) കരുതിയില്ല. ത്രിത്വദൈവത്തില്‍ വിശ്വസിക്കുകയും യേശുക്രിസ്തുവിനെ ദൈവപുത്രനായി പരിഗണിച്ച് ആരാധിക്കുകയും ചെയ്തവര്‍തന്നെയായിരുന്നു അബ്‌സീനിയയിലെ ക്രിസ്ത്യാനികള്‍. വിശുദ്ധ ക്വുര്‍ആന്‍ ശക്തമായി വിമര്‍ശിക്കുകയും ശാസിക്കുകയും ചെയ്ത വിശ്വാസാചാരങ്ങള്‍ വെച്ചുപുലര്‍ത്തുന്നവരുടെ നാടാണെങ്കിലും സത്യമതമനുസരിച്ച് ജീവിക്കുവാനും അത് പ്രബോധനം ചെയ്യുന്നതിനുമുള്ള സ്വാതന്ത്ര്യം ലഭിക്കുകയാണെങ്കില്‍ അവിടെ സമാധാനപൂര്‍വം ജീവിക്കുകയാണ് മുസ്‌ലിംകള്‍ ചെയ്യേണ്ടതെന്ന പാഠവും അബ്‌സീനിയന്‍ ഹിജ്‌റ നല്‍കുന്നുണ്ട്.  ജീവിക്കുന്ന നാട്ടിലെ വ്യവസ്ഥിതിയേതാണെങ്കിലും അത് ഇസ്‌ലാം വിരുദ്ധമോ മതവിരുദ്ധമോ ആണെങ്കില്‍പോലും അവിടെ മതസ്വാതന്ത്ര്യമുണ്ടെങ്കില്‍ പൂര്‍ണ മുസ്‌ലിംകളും നല്ല പൗരന്‍മാരുമായി ജീവിക്കുവാന്‍ സത്യവിശ്വാസികള്‍ക്ക് കഴിയുമെന്ന് പാഠം.

മക്കയിലെ പീഡനങ്ങളില്‍നിന്ന് രക്ഷപ്പെട്ട്, സ്വസ്ഥതയോടെ മുസ്‌ലിംകളായി ജീവിക്കുവാനാഗ്രഹിച്ച് അബ്‌സീനിയയിലേക്ക് ഹിജ്‌റ പോയവരെ പ്രശംസിച്ചുകൊണ്ട് അവതരിപ്പിക്കപ്പെട്ട ക്വുര്‍ആന്‍ വചനങ്ങളും അവര്‍ക്ക് ഉയര്‍ന്ന പ്രതിഫലമുണ്ടെന്ന് അറിയിച്ചുകൊണ്ടുള്ള പ്രവാചക മൊഴികളും ആദര്‍ശ സ്വാതന്ത്ര്യത്തിനുവേണ്ടിയുള്ള ഹിജ്‌റ, പോകുന്ന പ്രദേശത്തിലെ വ്യവസ്ഥിതി പരിഗണിക്കാതെതന്നെ, മഹത്തരവും ഉജ്ജ്വലവുമാണെന്ന വസ്തുത വ്യക്തമാക്കുന്നവയാണ്. അബ്‌സീനിയയിലേക്ക് പലായനം ചെയ്തവരെക്കുറിച്ച് അവതരിപ്പിക്കപ്പെട്ടതാണെന്ന് വ്യാഖ്യാതാക്കളില്‍ ചിലര്‍ വ്യക്തമാക്കിയ ചില ക്വുര്‍ആന്‍ വചനങ്ങള്‍ നോക്കുക.

''അക്രമത്തിന് വിധേയരായതിന് ശേഷം അല്ലാഹുവിന്റെ മാര്‍ഗത്തില്‍ സ്വദേശം വെടിഞ്ഞ് പോയവരാരോ അവര്‍ക്ക് ഇഹലോകത്ത് നാം നല്ല താമസസൗകര്യം ഏര്‍പെടുത്തികൊടുക്കുകതന്നെ ചെയ്യും. എന്നാല്‍, പരലോകത്തെ പ്രതിഫലം തന്നെയാകുന്നു ഏറ്റവും മഹത്തായത്. അവര്‍ (അത്) അറിഞ്ഞിരുന്നുവെങ്കില്‍!'' (16:41)

''പറയുക: വിശ്വസിച്ചവരായ എന്റെ ദാസന്‍മാരേ, നിങ്ങള്‍ നിങ്ങളുടെ രക്ഷിതാവിനെ സൂക്ഷിക്കുക. ഈ ഐഹികജീവിതത്തില്‍ നന്‍മ പ്രവര്‍ത്തിച്ചവര്‍ക്കാണ് സല്‍ഫലമുള്ളത്. അല്ലാഹുവിന്റെ ഭൂമിയാകട്ടെ വിശാലമാകുന്നു. ക്ഷമാശീലര്‍ക്കു തന്നെയാകുന്നു തങ്ങളുടെ പ്രതിഫലം കണക്കുനോക്കാതെ നിറവേറ്റികൊടുക്കപ്പെടുന്നത്.'' (39:10)

അബ്‌സീനിയയിലേക്ക് ഹിജ്‌റ പോവുകയും പിന്നീട് അവിടെനിന്ന് മദീനയിലെത്തി പ്രവാചകനോടൊപ്പം ചേരുകയും ചെയ്തവര്‍ക്ക് രണ്ട് ഹിജ്‌റയുടെ പ്രതിഫലമുണ്ടെന്നാണ് ബുഖാരിയും മുസ്‌ലിമും ഉദ്ധരിച്ച ഒരു ദീര്‍ഘമായ ഹദീഥിൽ നബി (സ) വ്യക്തമാക്കിയിട്ടുള്ളത്.

അബ്‌സീനിയയിലെത്തി സമാധാന ജീവിതം നയിച്ചുവന്ന മുസ്‌ലിംകളില്‍ പലരും നബി (സ) മദീനയിലെത്തി അവിടെ ഇസ്‌ലാമിക രാഷ്ട്രം സ്ഥാപിച്ചുകഴിഞ്ഞ ശേഷവും അവിടെതന്നെ തങ്ങളുടെ ജീവിതം തുടര്‍ന്നുവെന്നും മദീനയിലേക്ക് വന്ന് ഇസ്‌ലാമിക രാഷ്ട്രത്തിന്റെ പ്രജയാകുവാന്‍ തിടുക്കം കാട്ടിയില്ലെന്നുമുള്ള വസ്തുതകള്‍ ശ്രദ്ധേയമാണ്. മദീനാ രാഷ്ട്രത്തിലേക്ക് അവരില്‍ ചിലര്‍ മാത്രമാണ് രാഷ്ട്ര നിര്‍മാണത്തിന്റെ ആദ്യഘട്ടത്തില്‍ മടങ്ങിയെത്തിയത്. മക്കയിലേക്ക് മടങ്ങിയെത്തിയത് മുപ്പത്തിമൂന്ന് പുരുഷന്‍മാരും എട്ട് സ്ത്രീകളുമായിരുന്നുവെന്നും അതില്‍ രണ്ടുപേര്‍ മക്കയില്‍വെച്ചുതന്നെ മരണപ്പെട്ടുവെന്നും ഏഴുപേരെ മക്കക്കാര്‍ ബന്ധനസ്ഥരാക്കിയെന്നും ഇരുപത്തിനാലുപേര്‍ ബദ്ര്‍ യുദ്ധത്തില്‍ പങ്കെടുത്തുവെന്നും ഇബ്‌നുസഅദ് രേഖപ്പെടുത്തുന്നുണ്ട്. അബ്സീനിയാഹിജ്‌റയിലെ നേതാവായിരുന്ന ജഅ്ഫര്‍ ബ്ന്‍ അബീത്വാലിബും ബാക്കിയുള്ളവരുമെല്ലാം മദീനയിലെത്തിയത് ഖൈബര്‍ ജയിച്ചടക്കിയതിന് ശേഷമാണെന്ന് ബുഖാരിയും മുസ്ലിമും നിവേദനം ചെയ്ത ഹദീഥിലുണ്ട്. അവര്‍ മദീനയിലെത്തിയപ്പോള്‍ പ്രവാചകൻ (സ) ഒട്ടകത്തിന്മേല്‍നിന്ന് ചാടിയിറങ്ങുകയും അദ്ദേഹത്തിന്റെ പിതൃവ്യപുത്രനായ ജഅ്ഫര്‍ ബിന്‍ അബീത്വാലിബിനെ ആശ്ലേഷിക്കുകയും ചെയ്തുകൊണ്ട് പറഞ്ഞതിങ്ങനെയാണ്: 'ഖൈബര്‍ വിജയമാണോ ജഅ്ഫറിന്റെ തിരിച്ചുവരവാണോ എന്നെ കൂടുതല്‍ സന്തോഷിപ്പിച്ചതെന്ന് എനിക്കറിയില്ല'.( ഇമാം ഇബ്‌നുല്‍ ഖയ്യിം അല്‍ ജൗസിയ്യയുടെ സാദുല്‍മആദില്‍ നിന്ന്)

മദീനാ രാഷ്ട്രത്തിന്റെ നിര്‍മാണം കഴിഞ്ഞ് നീണ്ട ഏഴുവര്‍ഷങ്ങള്‍ക്ക് ശേഷമാണ് ജഅ്ഫര്‍ ബ്ന്‍ അബീതാലിബിന്റെ നേതൃത്വത്തിലുള്ള മുസ്‌ലിം സംഘം അബ്‌സീനിയയിലേക്ക് മടങ്ങിയതെന്ന വസ്തുതയും അങ്ങനെ മടങ്ങിയെത്തിയപ്പോള്‍ പൂര്‍ണമനസ്സോടെ പ്രവാചകൻ (സ) അവരെ സ്വീകരിക്കുകയും അവര്‍ക്ക് രണ്ട് ഹിജ്‌റയുടെ പ്രതിഫലമുണ്ടെന്ന സന്തോഷവാര്‍ത്ത അറിയിക്കുകയുമാണ് ചെയ്തതെന്ന യാഥാര്‍ഥ്യവും പഠിപ്പിക്കുന്നത് ഇസ്‌ലാമികേതര രാഷ്ട്ര സംവിധാനത്തിലാണെങ്കിലും മുസ്‌ലിമായി ജീവിക്കുവാന് സ്വാതന്ത്ര്യവും അവസരവുമുണ്ടെങ്കില്‍ ജീവിക്കുന്നതിന് വിരോധമില്ലെന്നും അങ്ങനെ ജീവിക്കുന്നവരുടെ മതത്തെ പ്രസ്തുത ജീവിതം ഒരുതരത്തിലും പ്രതികൂലമായി ബാധിക്കുകയില്ലെന്നുംതന്നെയാണ്. അബ്‌സീനിയയില്‍വെച്ച് മരണപ്പെട്ട നിരവധി മുസ്‌ലിംകളുടെ പട്ടിക ഇബ്‌നു ഇസ്ഹാഖ് രേഖപ്പെടുത്തുന്നുണ്ട്.

ഇസ്‌ലാമിക രാഷ്ട്രമുണ്ടായിരിക്കെതന്നെ ഒരു ഇസ്‌ലാമികേതര രാഷ്ട്രത്തില്‍ ജീവിച്ച് മരണപ്പെട്ടതിനാല്‍ അവരുടെ മതത്തിന് എന്തെങ്കിലും ഗ്ലാനി സംഭവിച്ചുവെന്നോ അവരുടെ മതം അപൂര്‍ണമായിരുന്നുവെന്നോ പ്രവാചകൻ (സ) സൂചിപ്പിച്ചതായി രേഖകളൊന്നുംതന്നെയില്ല. പ്രവാചകന്റെ ഹിജ്‌റയെക്കുറിച്ചറിഞ്ഞപ്പോള്‍ യമനിലായിരുന്ന അബൂമൂസല്‍ അശ്അരിയും (റ)അമ്പതിലധികം പേരുമടങ്ങുന്ന സംഘം മദീനയിലെത്തണമെന്ന ആഗ്രഹത്തോടെ യാത്രതിരിക്കുകയും വഴിമധ്യെ അവര്‍ അബ്‌സീനിയയിലെത്തുകയും ജഅ്ഫര്‍ ബ്ന്‍ അബീത്വാലിബിനെ (റ)കണ്ടുമുട്ടുകയും അദ്ദേഹത്തിന്റെ നിര്‍ദേശപ്രകാരം അവര്‍ അവിടെ ഏറെക്കാലം തങ്ങുകയും അവര്‍ ഒന്നിച്ച് മദീനയിലേക്ക് യാത്രയാവുകയും ഖൈബര്‍ വിജയ സന്ദര്‍ഭത്തില്‍ പ്രവാചകനുമായി സന്ധിക്കുകയും ചെയ്തതായി വ്യക്തമാക്കുന്ന ഹദീഥ് സ്വഹീഹുല്‍ ബുഖാരിയിലുണ്ട്. ഇസ്‌ലാമിക രാഷ്ട്രമായ മദീനയിലേക്ക് പോകാതെ ഇസ്‌ലാമികേതര രാഷ്ട്രമായ അബ്‌സീനിയയില്‍ തങ്ങുവാനുള്ള കാരണമായി ജഅ്ഫര്‍ ബിന്‍ അബീത്വാലിബ് പറഞ്ഞത് ഇങ്ങനെയാണ്: 'ഇവിടേക്ക് ഞങ്ങളെ പറഞ്ഞയച്ചതും ഇവിടെ താമസിക്കുവാന്‍ ആവശ്യപ്പെട്ടതും അല്ലാഹുവിന്റെ ദൂതനാണ്; അതിനാല്‍ നിങ്ങളും ഞങ്ങളോടൊപ്പം താമസിക്കുക'.( സ്വഹീഹുല്‍ ബുഖാരി, കിതാബുല്‍മഗാസി)  ഇസ്‌ലാമിക രാഷ്ട്രത്തില്‍ മാത്രമെ മുസ്‌ലിമിന് പൂര്‍ണ മുസ്‌ലിമായി ജീവിക്കുവാനാകൂവെന്നതിനാല്‍ ലോകത്ത് എവിടെ ജീവിക്കുന്ന മുസ്‌ലിമും ആ പ്രദേശം ഇസ്‌ലാമിക രാഷ്ട്രമാക്കുവാനോ അതല്ലെങ്കില്‍ നിലവിലുള്ള ഏതെങ്കിലും ഇസ്‌ലാമിക രാഷ്ട്രത്തിന് കീഴിലാക്കുകയോ ശ്രമിച്ചുകൊണ്ടിരിക്കുമെന്നും അതുകൊണ്ടാണ് മുസ്‌ലിംകള്‍ കലാപകാരികളായിത്തീരുന്നതെന്നും വിമര്‍ശിക്കുന്നവര്‍ കാണാതെ പോകുന്ന ചരിത്ര സംഭവങ്ങളാണിവ. രാഷ്ട്രം ഇസ്‌ലാമികമാണോ അല്ലേയെന്നതല്ല, മുസ്‌ലിമായി ജീവിക്കുവാന്‍ അനുവദിക്കുന്നതാണോ അല്ലേയെന്നതാണ് ഒരു രാഷ്ട്ര സംവിധാനത്തോടുള്ള മുസ്‌ലിമിന്റെ സമീപനമെന്തായിരിക്കണമെന്ന് നിശ്ചയിക്കുന്ന സൂചികയെന്ന് ഈ സംഭവങ്ങള്‍ സുതരാം വ്യക്തമാക്കുന്നു.

ല്ല. മുസ്‌ലിമായി ജീവിക്കുവാൻ സ്വാതന്ത്ര്യം ലഭിക്കുന്നതിന് വേണ്ടിയുള്ള പലായനമാണത്. നബി ശിഷ്യന്മാരുടെ അബ്സീനിയ പലായനം ഇക്കാര്യം വ്യക്തമാക്കുന്നുണ്ട്.

പ്രവാചകത്വത്തിന്റെ ആറാം വര്‍ഷത്തിലെ റജബ് മാസത്തില്‍ പതിനൊന്ന് പുരുഷന്‍മാരും നാല് സ്ത്രീകളുമടങ്ങുന്ന മുസ്‌ലിംകളുടെ ആദ്യസംഘം അഭയാര്‍ഥികളായി അബ്‌സീനിയയിലേക്ക് പലായനം ചെയ്യുന്നത് മുതല്‍ ആരംഭിക്കുന്നു ഇസ്‌ലാമിലെ ഹിജ്‌റയുടെ ചരിത്രം. മക്കയിലെ പീഡനങ്ങളില്‍നിന്ന് രക്ഷപ്പെടുന്നതിനായാണ് അവര്‍ അബ്‌സീനിയയിലേക്ക് പോയത്. ഏറെ ക്രൂരമായി പീഡിപ്പിക്കപ്പെട്ടിരുന്ന അടിമകള്‍ക്ക് യജമാനന്‍മാരുടെ കണ്ണുവെട്ടിച്ച് ഓടിപ്പോവുക പ്രയാസകരമായതുകൊണ്ടാവാം ആദ്യത്തെ പലായന സംഘത്തിലുണ്ടായിരുന്നത് പ്രധാനമായും ഖുറൈശീ ഗോത്രത്തില്‍നിന്ന് ഇസ്‌ലാം സ്വീകരിച്ചവരും അവരുടെ ഭാര്യമാരുമായിരുന്നു. തന്റെ മകളായ റുഖിയ്യ (റ) യെയും ഭര്‍ത്താവായിരുന്ന ഉഥ്മാനു ബ്‌നു അഫ്ഫാനെ(റ)യുമാണ് പ്രവാചകൻ (സ) ആദ്യമായി പലായനത്തിന് പറഞ്ഞയച്ചത്. ആദര്‍ശജീവിതത്തിലേക്ക്, സ്വന്തമായതെല്ലാം ഉപേക്ഷിച്ചുകൊണ്ട് പലായനം ചെയ്യാന്‍ പ്രവാചകൻ (സ) മുന്നില്‍ നടത്തിയത് സ്വന്തം പുത്രിയെയും പുത്രി ഭര്‍ത്താവിനെയുമായിരുന്നു.

ഉഥ്മാനുബ്‌നു മദ്വ്ഊനിന്റെ(റ) നേതൃത്വത്തില്‍ അബ്‌സീനിയയിലേക്ക് പലായനം ചെയ്ത ആദ്യസംഘം ഏറെ താമസിയാതെ ആ വര്‍ഷം ശവ്വാലില്‍തന്നെ മക്കയിലേക്ക് തിരിച്ചുവന്നത് തങ്ങളുടെ നാട്ടില്‍തന്നെ മുസ്‌ലിമായി ജീവിക്കുവാനുള്ള സാഹചര്യമുണ്ടായിട്ടുണ്ടെന്ന് തെറ്റായി മനസ്സിലാക്കിയതിനാലായിരുന്നു. ഉമറി (റ) ന്റെയും ഹംസ (റ) യുടെയും ഇസ്‌ലാം ആശ്ലേഷണത്തെക്കുറിച്ചറിഞ്ഞ അവര്‍ അതുവഴി മക്കയില്‍ സ്വസ്ഥമായി ആദര്‍ശജീവിതം നയിക്കുവാനുള്ള സ്വാതന്ത്ര്യമുണ്ടായിട്ടുണ്ടെന്ന് തെറ്റിദ്ധരിക്കുകയായിരുന്നു. ദൈവികബോധന പ്രകാരം ജീവിക്കുകയും അത് പ്രബോധിപ്പിക്കുകയും ചെയ്യാനുള്ള സ്വാതന്ത്ര്യം ജന്മനാട്ടില്‍ ഒരുങ്ങിയിട്ടുണ്ടെന്ന് കരുതി മക്കയില്‍ തിരിച്ചെത്തിയ അവരെ സ്വാഗതം ചെയ്തത് മുസ്‌ലിംകള്‍ക്കെതിരെ ബഹുദൈവാരാധകര്‍ അഴിച്ചുവിട്ടുകൊണ്ടിരിക്കുന്ന ക്രൂരമായ പീഡനങ്ങളെക്കുറിച്ച വര്‍ധമാനമായ വാര്‍ത്തകളായിരുന്നു. തങ്ങള്‍ സത്യമെന്ന് മനസ്സിലാക്കിയ ആദര്‍ശമുള്‍ക്കൊണ്ട് ജീവിക്കുവാന്‍ സ്വദേശത്ത് സാഹചര്യമൊരുങ്ങിയിട്ടുണ്ടെന്ന് മനസ്സിലാക്കിയ ഉടനെ സന്തോഷത്തോടെ ജന്മനാട്ടിലേക്ക് തിരിച്ച അബ്‌സീനിയര്‍ മുഹാജിറുകളുടെ നടപടിയില്‍നിന്ന് അവരുടെ പലായനത്തിന്റെ ലക്ഷ്യമെന്തായിരുന്നുവെന്ന്  വ്യക്തമാവുന്നുണ്ട്. ആദര്‍ശ ജീവിതത്തിനുള്ള സ്വാതന്ത്ര്യം നേടിയെടുക്കുകയെന്നതില്‍ കവിഞ്ഞ ലക്ഷ്യങ്ങളൊന്നും തന്നെ അവര്‍ക്കുണ്ടായിരുന്നില്ലെന്ന് സ്പഷ്ടം. ഇതുതന്നെയായിരുന്നു പിന്നീട് നടന്ന പലായനങ്ങളുടെയും ലക്ഷ്യം.

തങ്ങൾ കരുതിയത് പോലെ മക്കയിൽ മതസ്വാതന്ത്ര്യമുള്ള അവസ്ഥ സംജാതമായിട്ടില്ലെന്ന് മനസ്സിലാക്കിയപ്പോഴാണ് അബ്‌സീനിയയില്‍നിന്ന് തിരിച്ചെത്തിയവര്‍ വീണ്ടും അവിടേക്ക് തന്നെ പലായനം ചെയ്തത്. എണ്‍പതിലധികം പുരുഷന്‍മാരും പത്തൊന്‍പത് സ്ത്രീകളുമടങ്ങുന്ന താരതമ്യേന വലിയ പലായനസംഘത്തെ ക്രൈസ്തവ ഭരണാധികാരിയായിരുന്ന നജ്ജാശിയുടെ നാട്ടിലേക്ക് നബി (സ) പറഞ്ഞയച്ചതും സ്രഷ്ടാവിനെ മാത്രം ആരാധിച്ച് ജീവിക്കുകയും അവന്റെ നിയമനിര്‍ദേശങ്ങള്‍ അനുസരിക്കുകയും ചെയ്യുന്നതിനുള്ള സ്വാതന്ത്ര്യത്തിനുവേണ്ടിയായിരുന്നു. അബ്‌സീനിയയിലേക്ക് പലായനം ചെയ്യാന്‍ പ്രചോദിപ്പിച്ചുകൊണ്ടുള്ള പ്രവാചക നിര്‍ദേശത്തില്‍നിന്ന്  ഇക്കാര്യം സുതരാം വ്യക്തമാകുന്നുണ്ട്. മക്കാ മുശ്‌രിക്കുകളുടെ അക്രമങ്ങളാല്‍ പൊറുതിമുട്ടിയിരുന്ന തന്റെ അനുചരന്‍മാരോട് പ്രവാചകൻ (സ) പറഞ്ഞതായി, അബ്‌സീനിയന്‍ പലായനത്തില്‍ പങ്കെടുത്ത, പിന്നീട് പ്രവാചക പത്‌നിയായിത്തീര്‍ന്ന ഉമ്മുസല്‍മ (റ)നിവേദനം ചെയ്യുന്നു: 'ആരെയും അക്രമിക്കുകയോ അടിച്ചമര്‍ത്തുകയോ ചെയ്യാത്ത ഒരു രാജാവാണ് അബ്‌സീനിയന്‍ പ്രദേശങ്ങള്‍ ഭരിക്കുന്നത്. അതിനാല്‍ നിങ്ങള്‍ അങ്ങോട്ട് പോവുകയും അദ്ദേഹത്തിന്റെ പ്രദേശങ്ങളില്‍ അഭയം തേടുകയും ചെയ്യുക; ഇപ്പോഴുള്ള അവസ്ഥയ്ക്ക് അല്ലാഹു ഒരു മാറ്റമുണ്ടാക്കുന്നതും നമുക്ക് മറ്റൊരു മാര്‍ഗമുണ്ടാവുന്നതുംവരെ'.( ഇബ്‌നു ഹിശാം സ്വീകാര്യമായ പരമ്പരയോടു (ഹസന്‍) നിവേദനം ചെയ്തത്)

മക്കയ്ക്ക് ചുറ്റും നിരവധി സ്ഥലങ്ങളുണ്ടായിട്ടും തന്റെ അനുചരന്‍മാരെ എന്തുകൊണ്ടാണ് പ്രവാചകൻ (സ) അവിടങ്ങളിലേക്കൊന്നും പറഞ്ഞയക്കാതെ അബ്‌സീനിയതന്നെ തെരഞ്ഞെടുത്തതെന്ന ചോദ്യത്തിനുള്ള ഉത്തരം ഉമ്മുസല്‍മ (റ)നിവേദനം ചെയ്ത ഹദീഥിലുണ്ട്. 'ആരെയും അക്രമിക്കുകയോ അടിച്ചമര്‍ത്തുകയോ ചെയ്യാത്ത ഒരു രാജാവാണ് അബ്‌സീനിയ ഭരിക്കുന്നത്'. അബ്‌സീനിയയിലേക്ക് കച്ചവടത്തിന് പോയ മക്കക്കാരില്‍നിന്ന് അവിടുത്തെ രാജാവിന്റെ നീതിനിഷ്ഠയെയും മാന്യതയെയുംകുറിച്ച് മനസ്സിലാക്കിയതില്‍നിന്നാവണം അദ്ദേഹം മുസ്‌ലിംകളെ സ്വീകരിക്കുകയും അവരുടെ മതമനുസരിച്ച് ജീവിക്കാന്‍ അനുവദിക്കുകയും ചെയ്യുമെന്ന് പ്രവാചകൻ (സ) കരുതിയത്. അദ്ദേഹത്തിന്റെ നിഗമനം തെറ്റിയിട്ടില്ലെന്ന് അബ്‌സീനിയയില്‍ മുസ്‌ലിംകള്‍ക്ക് ലഭിച്ച സ്വീകരണം തെളിയിക്കുകയും ചെയ്തു. 'ഞങ്ങള്‍ക്ക് അവിടെ ചെന്നപ്പോള്‍ നല്ല സ്വീകരണം ലഭിക്കുകയും നല്ല അയല്‍ക്കാരെ കിട്ടുകയും ചെയ്തു. ഞങ്ങളുടെ മതത്തിന്റെ കാര്യത്തില്‍ ശാന്തി അനുഭവിക്കുകയും പീഡനങ്ങളില്ലാത്ത സ്ഥിതി സംജാതമാവുകയും ചെയ്തു'വെന്ന പ്രവാചക പത്‌നി ഉമ്മുസല്‍മേയുടെ വാക്കുകള്‍ മുസ്‌ലിംകള്‍ക്ക് അബ്‌സീനിയയില്‍ ലഭിച്ച സ്വീകാര്യതയെ വെളിപ്പെടുത്തുന്നുണ്ട്.

പീഡനങ്ങളും പ്രയാസങ്ങളും അധികരിച്ചപ്പോള്‍ അവയില്‍നിന്ന് രക്ഷപ്പെടാന്‍ പ്രവാചകൻ (സ) നിര്‍ദേശിച്ചത് ജീവിക്കുന്ന നാട്ടില്‍ കലാപങ്ങളും കുഴപ്പങ്ങളുമുണ്ടാക്കി, അവിടെയുള്ളവരെ ഭീഷണിപ്പെടുത്തിക്കൊണ്ട് കാര്യം നേടുകയെന്ന മാര്‍ഗമല്ല, പ്രത്യുത ആദര്‍ശമനുസരിച്ച് ജീവിക്കുവാനാകുന്ന ഏതെങ്കിലുമൊരു സ്ഥലമുണ്ടെങ്കില്‍ അവിടെ പോയി ജീവിക്കുകയും ആശയപ്രബോധനം നടത്തുകയും ചെയ്യുകയാണെന്ന വസ്തുത വ്യക്തമാക്കുന്നതാണ് അബ്‌സീനിയയിലേക്ക് പലായനം ചെയ്യുവാനുള്ള നബിനിര്‍ദേശങ്ങള്‍. എണ്‍പതിലധികം പുരുഷന്‍മാരും അവര്‍ക്ക് നേതൃത്വം നല്‍കാനാവുന്ന ഉമറിനെയും (റ)ഹംസയെയും (റ)പോലെയുള്ള ശക്തരും ധൈര്യശാലികളുമായ പ്രമുഖരും ആദര്‍ശത്തിനായി എന്തുതരം ത്യാഗങ്ങളും സഹിക്കാനാകുമെന്ന് തെളിയിച്ച ബിലാലിനെ (റ) പോലെയുള്ള മഹത്തുക്കളും തനിക്ക് ചുറ്റുമുണ്ടായിരുന്നിട്ടും മക്കയില്‍ തന്നെയും അനുയായികളെയും പീഡിപ്പിക്കുന്നവര്‍ക്കെതിരെ ശക്തമായ ഒരു കലാപമഴിച്ചുവിട്ട് പീഡനത്തിന്റെ ചങ്ങലയറുക്കുവാനല്ല, പ്രത്യുത പീഡനങ്ങളില്ലാത്ത അയല്‍ ദേശത്തേക്ക് കുടിയേറുവാനാണ് പ്രവാചകൻ (സ) നിര്‍ദേശിച്ചതെന്ന വസ്തുത കലാപകാരിയായിരുന്നു മുഹമ്മദെന്നും (സ) മക്കയില്‍വെച്ച് അവസരമില്ലാത്തതുകൊണ്ടാണ് അദ്ദേഹം കലാപങ്ങളുടെ കെട്ടഴിക്കാതിരുന്നതെന്നും വിമര്‍ശിക്കുന്നവര്‍ക്ക് മറുപടി നല്‍കുന്നതാണ്.

കലാപമുണ്ടാക്കി കലുഷിതമായ സാമൂഹ്യാന്തരീക്ഷം സൃഷ്ടിക്കുകയും അതില്‍നിന്ന് തന്റെ താല്‍പര്യങ്ങള്‍ സംരക്ഷിക്കുകയുമല്ല, പ്രത്യുത സമാധാനത്തോടെ ദൈവികമതമനുസരിച്ച് ജീവിക്കുകയും അത് പ്രബോധിപ്പിക്കുകയും ചെയ്യാനാവശ്യമായ സാഹചര്യം സൃഷ്ടിക്കുകയുമായിരുന്നു പ്രവാചകന്റെ ലക്ഷ്യം. മക്കയിലെ നിലവിലുള്ള സാമൂഹ്യ സാഹചര്യത്തില്‍ ഗോത്രങ്ങളുടെ സംരക്ഷണത്തിന്റെ തണലില്‍ താനും അത് ലഭിച്ച സ്വഹാബികളും നിലയുറപ്പിക്കുകയും ആദര്‍ശജീവിതവും പ്രബോധനവും നിര്‍വഹിച്ചുകൊണ്ട് മുന്നോട്ട് പോവുകയും ചെയ്തുകൊണ്ടിരുന്നപ്പോള്‍തന്നെ, പ്രസ്തുത സംരക്ഷണം ലഭിക്കാത്തവരെ മതപീഡനമില്ലാത്ത അയല്‍നാട്ടിലേക്ക് പോകാന്‍ പ്രചോദിപ്പിക്കുകയും അവിടേക്ക് പറഞ്ഞയക്കുകയും ചെയ്ത പ്രവാചകനില്‍ (സ) ശാന്തമായി വിശുദ്ധജീവിതം നയിക്കുകയും അത് പ്രബോധിപ്പിക്കുകയും ചെയ്യാന്‍ അഭിവാഞ്ചയുള്ള ഒരു സാമൂഹ്യ പരിഷ്‌കര്‍ത്താവിനെയാണ് ആര്‍ക്കും കാണാനാവുക.

വിടവാങ്ങുക, ഉപേക്ഷിക്കുക, കയ്യൊഴിയുക, പരിത്യജിക്കുക, വേര്‍പെടുക, നിരാകരിക്കുക, സ്ഥലം വിടുക തുടങ്ങിയ അര്‍ഥങ്ങളില്‍ പ്രയോഗിക്കപ്പെടുന്ന ഹാ, ജീം, റാ എന്നീ അക്ഷരത്രയങ്ങളില്‍നിന്ന് നിഷ്പന്നമായ 'ഹിജ്‌റ'യെന്ന പദമാണ് പലായനം എന്ന് പരിഭാഷപ്പെടുത്തപ്പെടാറുള്ളത്. സ്വദേശം വെടിഞ്ഞുപോകുന്നതിനും തെറ്റുകള്‍ വെടിഞ്ഞ് വിശുദ്ധമാകുന്നതിനും പൈശാചിക പാതവെടിഞ്ഞ് സത്യമാര്‍ഗത്തിലെത്തിച്ചേരുന്നതിനുമെല്ലാം 'ഹിജ്‌റ'യെന്ന് പറയാവുന്നതാണ്. 'തെറ്റുകളും പാപങ്ങളും ഉപേക്ഷിക്കുന്നവനാണ് മുഹാജിര്‍' (പലായനം ചെയ്യുന്നവന്‍)( ഇമാമുമാര്‍ അഹ്മദ്, ഹാക്കിം, ത്വബ്‌റാനി എന്നിവര്‍ നിവേദനം ചെയ്ത സ്വഹീഹായ ഹദീഥ് ) എന്ന് പ്രവാചകൻ (സ) പറഞ്ഞത് ഈ അര്‍ഥങ്ങളെയെല്ലാം ദ്യോതിപ്പിച്ചുകൊണ്ടാണ്. പാപങ്ങള്‍ വെടിയുകയും നന്മകളില്‍ വ്യാപൃതരാവുകയും ചെയ്യുകയെന്ന ഹിജ്‌റ ചെയ്യേണ്ടവനാണ് ഓരോ മുസ്‌ലിമും. എലാ മുസ്ലിംകളും ആത്യന്തികമായി മുഹാജിറുകളാണെന്ന് സാരം.

പൈശാചികപാത വെടിഞ്ഞ് സത്യമാര്‍ഗത്തിലെത്തിച്ചേര്‍ന്ന ഒരാള്‍ക്ക് തന്റെ സത്യമെന്ന് തിരിച്ചറിഞ്ഞ ആദര്‍ശപ്രകാരം ജീവിക്കാൻ കഴിയാത്ത സാഹചര്യമുണ്ടാകുമ്പോഴാണ് അയാള്‍ സ്വന്തം നാട്ടില്‍ നിന്ന് പലായനം ചെയ്യുന്നത്. മാനസികമായ ഹിജ്‌റയില്‍നിന്നാണ് നാട് വെടിയേണ്ടിവരികയെന്ന ശാരീരികമായ ഹിജ്‌റയുണ്ടാവുന്നത്. പൈശാചിക പ്രലോഭനങ്ങളില്‍നിന്ന് രക്ഷപ്പെടുകയെന്ന മാനസികമായ ഹിജ്‌റ ചെയ്യാത്തവര്‍ ശാരീരിക ഹിജ്‌റ ചെയ്യുന്നതുകൊണ്ട് കാര്യമൊന്നുമില്ലെന്ന് പ്രവാചകൻ (സ) പഠിപ്പിച്ചിട്ടുണ്ട്. "പ്രവര്‍ത്തനങ്ങള്‍ ഉദ്ദേശ്യങ്ങള്‍ക്കനുസരിച്ചാണ് (പ്രതിഫലാര്‍ഹമാവുക) ഓരോ മനുഷ്യനും അവനിദ്ദേശിച്ചത് ലഭിക്കും. ഭൗതികനേട്ടത്തിനോ ഏതെങ്കിലും സ്ത്രീയെ വിവാഹം ചെയ്യാനോ ആണ് ആരുടെയെങ്കിലും ഹിജ്‌റയെങ്കില്‍ അവനതാണ് ലഭിക്കുക.( സ്വഹീഹുല്‍ ബുഖാരി, കിതാബുബദഉല്‍ വഹ്‌യ്; സ്വഹീഹുമുസ്‌ലിം, കിതാബുല്‍ഇമാറ)

സത്യമതത്തിന്റെ നിര്‍ദേശങ്ങളനുസരിച്ച് ജീവിക്കുന്നതിനും പ്രബോധനം ചെയ്യുന്നതിനും വേണ്ടി സ്വദേശം വെടിഞ്ഞുപോകുന്നത് വളരെ വലിയ ത്യാഗമാണ്. ജനിച്ചുവളര്‍ന്ന നാടും പിച്ചവെച്ചുവളര്‍ന്ന മണ്ണും താലോലിച്ച് വളര്‍ത്തിയ ബന്ധുമിത്രാദികളെയും അധ്വാനിച്ചുണ്ടാക്കിയ സമ്പാദ്യവുമെല്ലാം വെടിഞ്ഞ് ആദര്‍ശജീവിതത്തിനുള്ള സ്വാതന്ത്ര്യത്തിനുവേണ്ടി മാത്രമായുള്ള പലായനമെന്ന അല്ലാഹുവിന്റെ മാര്‍ഗത്തിലുള്ള ത്യാഗത്തിന് ഉന്നതമായ പ്രതിഫലമുണ്ടെന്ന് ക്വുര്‍ആന്‍ വ്യക്തമാക്കുന്നുണ്ട്.

''വിശ്വസിക്കുകയും സ്വദേശം വെടിയുകയും തങ്ങളുടെ സ്വത്തുക്കളും ശരീരങ്ങളും കൊണ്ട് അല്ലാഹുവിന്റെ മാര്‍ഗത്തില്‍ സമരം നടത്തുകയും ചെയ്തവര്‍ അല്ലാഹുവിങ്കല്‍ ഏറ്റവും മഹത്തായ പദവിയുള്ളവരാണ്. അവര്‍ തന്നെയാണ് വിജയം പ്രാപിച്ചവര്‍. അവര്‍ക്ക് അവരുടെ രക്ഷിതാവ് അവന്റെ പക്കല്‍ നിന്നുള്ള കാരുണ്യത്തെയും പ്രീതിയെയും സ്വര്‍ഗത്തോപ്പുകളെയും പറ്റി സന്തോഷവാര്‍ത്ത അറിയിക്കുന്നു. അവര്‍ക്ക് അവിടെ ശാശ്വതമായ സുഖാനുഭവമാണുള്ളത്. അവരതില്‍ നിത്യവാസികളായിരിക്കും. തീര്‍ച്ചയായും അല്ലാഹുവിന്റെ അടുക്കലാണ് മഹത്തായ പ്രതിഫലമുള്ളത്.'' (9:20-22)

''അല്ലാഹുവിന്റെ മാര്‍ഗത്തില്‍ വല്ലവനും സ്വദേശം വെടിഞ്ഞ് പോകുന്ന പക്ഷം ഭൂമിയില്‍ ധാരാളം അഭയസ്ഥാനങ്ങളും ജീവിതവിശാലതയും അവന്‍ കണ്ടെത്തുന്നതാണ്. വല്ലവനും തന്റെ വീട്ടില്‍ നിന്ന് - സ്വദേശം വെടിഞ്ഞ് കൊണ്ട് - അല്ലാഹുവിലേക്കും അവന്റെ ദൂതനിലേക്കും ഇറങ്ങി പുറപ്പെടുകയും, അനന്തരം (വഴി മധ്യേ) മരണമവനെ പിടികൂടുകയും ചെയ്യുന്ന പക്ഷം അവന്നുള്ള പ്രതിഫലം അല്ലാഹുവിങ്കല്‍ സ്ഥിരപ്പെട്ടു കഴിഞ്ഞു. അല്ലാഹു ഏറെ പൊറുക്കുന്നവനും കരുണാനിധിയുമാകുന്നു.'' (4:100)

സ്വര്‍ഗപ്രാപ്തിയെന്ന ലക്ഷ്യവും ആദര്‍ശജീവിതം നയിക്കുവാന്‍ അവസരമുണ്ടാകണമെന്ന ആഗ്രഹവും മാത്രമാണ് മുസ്‌ലിമിന്റെ പലായനത്തിന് പിന്നിലുള്ളത്. സ്വാര്‍ഥമായ താല്‍പര്യങ്ങള്‍ക്കോ ഭൗതികമായ ലക്ഷ്യങ്ങള്‍ക്കോ വേണ്ടിയുള്ള പലായനം ചിലപ്പോള്‍ പ്രസ്തുത ലക്ഷ്യങ്ങള്‍ നേടിയെടുക്കുവാന്‍ പ്രയോജനീഭവിക്കാമെങ്കിലും അത് അല്ലാഹുവിന്റെ മാര്‍ഗത്തിലുള്ള ഹിജ്‌റയാവുകയില്ലെന്നും ആത്യന്തികമായി അത് നന്മയ്ക്ക് നിമിത്തമാകുകയില്ലെന്നുമാണ് പ്രവാചകൻ (സ) പഠിപ്പിച്ചത്. ഭൗതികമായ ഒരു സാമ്രാജ്യ സ്ഥാപനമായിരുന്നു മുഹമ്മദ് നബി (സ) യുടെ ഹിജ്‌റക്കുപിന്നിലുള്ള ലക്ഷ്യമെന്ന് വിമര്‍ശിക്കുന്നവര്‍ നബി ദൗത്യത്തെക്കുറിച്ച് വിവരമില്ലാത്തവരാണ്. പ്രവാചകത്വം വാദിച്ചതുതന്നെ അറബികള്‍ക്കുമേലുള്ള അധീശത്വത്തിന് വേണ്ടിയായിരുന്നുവെന്ന് വാദിക്കുന്നവരാണ് മുഹമ്മദ് നബി (സ) യുടെ ഹിജ്‌റക്കുപിന്നിലും സാമ്രാജ്യസ്ഥാപനമെന്ന ലക്ഷ്യമുണ്ടായിരുന്നുവെന്ന് സമര്‍ഥിക്കാന്‍ ശ്രമിക്കുന്നത്. ദൈവികബോധനങ്ങള്‍ പ്രകാരമുള്ള ജീവിതത്തിനും അത് പ്രബോധനം ചെയ്യുന്നതിനുമുള്ള സ്വാതന്ത്ര്യമുണ്ടാവുകയെന്നതില്‍ കവിഞ്ഞ ലക്ഷ്യങ്ങളെന്തെങ്കിലും ഹിജ്‌റക്ക് പിന്നിലുണ്ടായിരുന്നുവെന്ന് സ്ഥാപിക്കുവാന്‍ വിമര്‍ശകരുടെ പക്കല്‍ തെളിവുകളൊന്നും തന്നെയില്ല. ഹിജ്‌റക്ക് മുമ്പുള്ള സംഭവങ്ങളെ സൂക്ഷ്മമായി അപഗ്രഥിക്കുകയും തലനാരിഴ കീറി പരിശോധിക്കുകയും ചെയ്താല്‍പോലും നബി (സ)യുടെ പലായനത്തിന് പിന്നില്‍ സ്വാര്‍ഥമായ വല്ല ലക്ഷ്യവുമുണ്ടായിരുന്നുവെന്ന് സ്ഥാപിക്കുവാനാവശ്യമായ തെളിവുകളൊന്നും ലഭിക്കുകയില്ലെന്നുറപ്പാണ്.

മദീനയിലെത്തി അവിടത്തെ ഭരണാധികാരിയായിത്ത്തീർന്നതോടെ നിരവധി യുദ്ധങ്ങൾ നയിക്കുകയും നാടുകൾ പിടിച്ചടക്കുയും ചെയ്ത മുഹമ്മദ് നബിയുടെ (സ) നടപടികൾ അദ്ദേഹം ഒരു അധികാരമോഹിയായിരുന്നുവെന്നല്ലേ വ്യക്തമാക്കുന്നത് ?

ദീനയിലെ ഭരണാധികാരിയായിത്ത്തീർന്ന ശേഷം നബി(സ) യുദ്ധം ചെയ്തിട്ടുണ്ടെന്നത് നേരാണ്. അയാൾ നാടുകളുടെ അധികാരം അദ്ദേഹത്തിന്റെ കൈകളിൽ അർപ്പിക്കപ്പെട്ടിട്ടുണ്ടെന്നതും ശരിയാണ്. എന്നാൽ അതിനെ അധികാഹാരമോഹത്തിന്റെ അളവുകോലുകളുപയോഗിച്ച് അനീതിയായിരിക്കും. അധീശത്വത്തിനോ അധികാര വിപുലീകരണത്തിനോ ആഡംബര ജീവിതത്തിനോ വേണ്ടി നടക്കേണ്ടവയാണ് യുദ്ധങ്ങളെന്ന രാഷ്ട്രമീമാംസയുടെ പരമ്പരാഗത പാഠങ്ങളില്‍ അഭിരമിക്കുന്നവർക്ക് മുഹമ്മദ് നബി (സ) ചെയ്ത യുദ്ധങ്ങളുടെ ആത്മാവെന്താണെന്ന് മനസ്സിലാവുകയില്ല. അധികാര വിപുലീകരണത്തിനുവേണ്ടി നടക്കുന്ന യുദ്ധങ്ങളെ അപഗ്രഥിക്കുവാന്‍ ഉപയോഗിക്കുന്ന അതേ മൂശയിലിട്ട് ആദര്ശ സംരക്ഷണത്തിനും ആദര്ശപമനുസരിച്ച് ജീവിക്കുവാനും വേണ്ടി നടത്തുന്ന യുദ്ധങ്ങളെ മനസ്സിലാക്കുവാന്‍ ശ്രമിച്ചതുകൊണ്ടാണ് മുഹമ്മദ് നബി(സ)യെ യുദ്ധക്കൊതിയനായി കാണാന്‍ അവർ ധൃഷ്ടരാവുന്നത്. ചരിത്രത്തിന് പരിചയമുള്ള അധികാര പ്രമത്തതയുടെ യുദ്ധങ്ങളെവിടെ, ആദര്ശമത്തിനുവേണ്ടി മുഹമ്മദ് നബി (സ) നയിച്ച യുദ്ധങ്ങളെവിടെ? താരതമ്യത്തിനുപോലും പറ്റാത്തത്രയും വ്യത്യസ്തങ്ങളാണ് ഇവ രണ്ടുമെന്നതാണ് വാസ്തവം.

നബി (സ) നയിച്ച യുദ്ധങ്ങളെ നബിജീവിതത്തിന്റെ മറ്റു വശങ്ങളെയുംകൂടി പരിഗണിച്ചുകൊണ്ട് അപഗ്രഥിക്കുവാന്‍ സന്നദ്ധമായാല്‍ തങ്ങളുടെ വാദങ്ങളെല്ലാം ശുദ്ധ അസംബന്ധമാണെന്ന് നബി (സ) യിെല യുദ്ധക്കൊതിയനെ ഗവേഷണം ചെയ്‌തെടുക്കുവാന്‍ ശ്രമിക്കുന്നവര്ക്കുവതന്നെ ബോധ്യമാകും. അധികാരപ്രമത്തതയാണ് പ്രവാചക പോരാട്ടങ്ങള്ക്ക് പിന്നിലെന്ന് പറയുന്നവര്ക്ക്െ പ്രത്യുത പ്രമത്തതയുടെ പ്രചോദനമെന്തായിരിക്കുമെന്ന് വിശദീകരിക്കേണ്ട ബാധ്യതയുണ്ട്. ഒരാള്ക്ക് അധികാരമെന്തിനാണ് എന്ന ചോദ്യത്തിന് ഇവരുടെ കയ്യിലുള്ള ഉത്തരം ആഡംബര ജീവിതത്തിന് എന്നാണ്. ആഡംബര ജീവിതത്തിനാണ് അധികാരമെങ്കില്‍, അതിനോടുള്ള പ്രതിപത്തിയുള്ളയാള്‍ തീര്ച്ചെയായും ആഡംബര പ്രിയനായിരിക്കണം. ഓരോ യുദ്ധവും കഴിയുകയും തന്റെ അധികാരപരിധി വര്ധിപക്കുകയും ചെയ്യുന്നതിനനുസരിച്ച് അയാളുടെ ആഡംബര ജീവിതത്തിന്റെ പൊലിമയും വര്ധിതച്ചുകൊണ്ടിരുന്നിരിക്കണം. അങ്ങനെ ആഡംബരങ്ങളില്‍ ആറാടിക്കൊണ്ട് താനും തന്റെ കുടുംബവും ജീവിക്കുന്നതിനിടയിലായിരിക്കണം അയാള്‍ ഇൗ ലോകത്തുനിന്ന് വിടപറഞ്ഞിരിക്കുക. അധികാര വിപുലീകരണത്തിനായി യുദ്ധം ചെയ്ത ലോകത്തിന് പരിചയമുള്ളവരുടെയെല്ലാം അവസ്ഥയിതാണ്. ഈ വസ്തുതകളുടെ വെളിച്ചത്തില്‍ നബിജീവിതത്തെ മുന്ധാിരണകളില്ലാതെ പരിശോധിക്കുവാന്‍ ആത്മാര്ഥഥമായി സന്നദ്ധമാവുകയാണെങ്കില്‍ അധികാര ദുര മൂത്ത് ചെയ്തതല്ല പ്രവാചക പോരാട്ടങ്ങളെന്ന വസ്തുത വിമര്ശടകര്ക്ക് പോലും സുതരാം വ്യക്തമാകും.

അറേബ്യയുടെ അധികാരിയായിരിക്കെ മരണപ്പെട്ട മുഹമ്മദ് നബി (സ) യുടെ 'ആഡംബര' ജീവിതത്തെക്കുറിച്ച് വ്യക്തമാക്കുന്ന ഹദീഥുകൾ പരിശോധിച്ചാൽ മതി, അധികാരമോഹമായിരുന്നു അദ്ദേഹത്തിന്റെ യുദ്ധങ്ങൾക്ക് പിന്നിലെന്ന ആരോപണത്തിന്റെ നിജസ്ഥിതിയറിയുവാൻ.

മാസങ്ങളോളം പച്ചവെള്ളവും കാരക്കയും തിന്ന് ജീവിതം തള്ളിനീക്കുന്ന പ്രവാചകനും കുടുംബവും!

ഒരു ദിവസം രണ്ടുനേരം പോലും വയര്നിനറച്ച് ഭക്ഷണം കഴിക്കാത്ത രാഷ്ട്രനേതാവ്!

വിലകുറഞ്ഞ കാരക്കപോലും ഭക്ഷിക്കുവാനില്ലാതെ വിശപ്പ് സഹിക്കുന്ന ജനനായകന്‍!

ശരീരത്തില്‍ ചുവന്ന പാടുകളുണ്ടാക്കുമാറുള്ള പരുക്കന്‍ ഈത്തപ്പനപ്പായയില്‍ അന്തിയുറങ്ങുന്ന അന്തിമ പ്രവാചകന്‍!

ദ്രവിച്ച ചെരിപ്പുകളും കഷ്ണംവെച്ച കുപ്പായവുമണിഞ്ഞ് രാഷ്ട്രഭരണം നടത്തുന്ന ദൈവദൂതന്‍!

സ്വന്തം കുടുംബത്തിന്റെ പട്ടിണി മാറ്റാനായി തന്റെ പടയങ്കി ജൂതന്റെ പക്കല്‍ പണയംവെച്ച രീതിയില്‍ മരണപ്പെട്ട രാഷ്ട്രപതി!

പൊട്ടിപ്പോയിട്ട് വിളക്കിവെച്ച് ഉപയോഗിക്കുന്ന പാനപാത്രം ഉപയോഗിക്കുന്ന ജനനേതാവ്! അനന്തരാവകാശമായി തന്റെ കുടുംബത്തിന് ഒന്നും അവശേഷിപ്പിക്കാതെ ഇഹലോകവാസം വെടിഞ്ഞ അന്തിമ പ്രവാചകന്‍!

പൊതുമുതലില്നിാന്ന് ഒരു കാരക്ക തിന്നാനൊരുമ്പെട്ട പിഞ്ചുബാലനായ പൗത്രെന തടയുകയും ശാസിക്കുകയും ചെയ്യുന്ന മാര്ഗുദര്ശാകന്‍!

അധ്വാനിച്ച് പൊട്ടിയ കൈകളുമായി തനിക്ക് ഒരു ഭൃത്യനെ അനുവദിച്ച് തരണമെന്നപേക്ഷിച്ച സ്വന്തം മകളോട് അതിന് കഴിയില്ലെന്ന് നിസ്സങ്കോചം മറുപടി പറഞ്ഞ രാഷ്ട്ര നേതാവ്!

നിരവധി യുദ്ധങ്ങള്‍ ചെയ്യുകയും അധികാരമുറപ്പിക്കുകയും ചെയ്തതിനുശേഷമുള്ള നബിജീവിതത്തിന്റെ നഖചിത്രമാണിത്. ഇനി നബിവിമര്ശകര്‍ പറയട്ടെ, പ്രവാചകൻ (സ) യുദ്ധം ചെയ്തത് തന്റെ ആഡംബര ജീവിതത്തെ പൊലിപ്പിക്കുവാനായിരുന്നുവെന്ന്! തനിക്ക് ജീവിതസുഖങ്ങള്‍ നല്കുൊന്ന അധികാരത്തോടുള്ള മോഹംകൊണ്ടാണ് നബി (സ) യുദ്ധം ചെയ്തതെന്ന് അദ്ദേഹത്തിന്റെ ജീവതത്തെക്കുറിച്ച് സമഗ്രമായി പഠിച്ചാല്‍ കടുത്ത നബിവിരോധിക്കുപോലും പറയാനാവില്ല എന്നതാണ് വാസ്തവം. അരപ്പട്ടിണിയും ഉണക്കറൊട്ടിയും കാരക്കയും പച്ചവെള്ളവും കഴിച്ചുള്ള ജീവിതവും മകളോട് അടുക്കളയില്‍ അധ്വാനിക്കുവാനുപദേശിച്ചുകൊണ്ടുള്ള സമാശ്വസിപ്പിക്കലും പനയോലപ്പായയില്‍ കിടന്നുറങ്ങിയും കഷ്ണംവെച്ച പരുക്കല്‍ കുപ്പായമിട്ടുകൊണ്ടുള്ള ഭരണനിര്വ്ഹണവുമായിരുന്നു അന്നത്തെ ആഡംബര ജീവിതമെന്ന ഗവേഷണഫലം പുറത്തുവിടാന്‍ നബിവിരോധികളൊന്നുംതന്നെ ധൃഷ്ടരാവുകയില്ലെന്ന് നാം പ്രത്യാശിക്കുക!

ഗോത്രങ്ങളുടെമേല്‍ അധീശത്വം സ്ഥാപിക്കുന്നതിനും അവരെ കൊള്ളയടിച്ച് യുദ്ധാര്ജിത സ്വത്തുക്കള്‍ നേടിയെടുക്കുന്നതിനും അതുവഴി ആഡംബര ജീവിതം നയിക്കുന്നതിനുംവേണ്ടി നടത്തിയിരുന്ന അറേബ്യന്‍ ഗോത്രവര്ഗീ യുദ്ധങ്ങള്‍ അവസാനിപ്പിക്കുകയും ആദര്ശത്തിനുവേണ്ടി മാത്രമാണ് യുദ്ധം ചെയ്യേണ്ടതെന്ന പാഠം പഠിപ്പിക്കുകയുമെന്ന ദൗത്യമാണ് മുഹമ്മദ് നബി (സ) താന്‍ നിയോഗിക്കപ്പെട്ട സമൂഹത്തില്‍ നിര്വഹിച്ചത്. അതോടൊപ്പംതന്നെ, സാമ്രാജ്യത്വ മോഹങ്ങളെ സാക്ഷാല്ക്കരിക്കുന്നതിനായി നിരപരാധികളെ കൊന്നൊടുക്കുന്നത് ന്യായീകരണമര്ഹി്ക്കാത്ത കുറ്റകൃത്യമാണെന്നും ആദര്ശ്മനുസരിച്ച് ജീവിക്കുവാനുള്ള സ്വാതന്ത്ര്യത്തിനുവേണ്ടി, പ്രസ്തുത സ്വാതന്ത്ര്യം തടയപ്പെടുമ്പോഴും സ്വന്തം സുരക്ഷയെ തകര്ത്തു കൊണ്ട് നാടിനെ അടിമപ്പെടുത്തുവാനായി സായുധ മുന്നേറ്റങ്ങളുണ്ടാകുമ്പോള്‍ അതിനെ പ്രതിരോധിക്കുവാനും വേണ്ടിയാണ് യുദ്ധം ചെയ്യേണ്ടതെന്നും ലോകത്തുള്ള മുഴുവന്‍ മനുഷ്യരെയും പഠിപ്പിക്കുകകൂടി ചെയ്തു, മുഹമ്മദ്‌ല. ഗോത്രവൈര്യത്തിലും അക്രമണോല്സുപകതയിലും എതിര്‍ ഗോത്രങ്ങളെ കൊള്ളയടിച്ച് അവരുടെ സ്വത്തുപയോഗിക്കുകയും തരുണീമണികളെ വെപ്പാട്ടികളാക്കിത്തീര്ത്ത്െ അവരില്നികന്ന് രതിസുഖം നുകരുകയും ചെയ്യുന്നതില്‍ എത്രമാത്രം അഭിരമിക്കുന്നവരായിരുന്നു അറബികളെന്ന് കഅ്ബാലയത്തില്‍ തൂക്കിയിട്ടിരിക്കുന്ന (മുഅല്ലഖാത്ത്) ഇംറുല്‍ ഖൈസ് ബ്‌നു ഹുജ്‌റ്, ത്വറഫതുബ്‌നു അബ്ദ്, സുഹൈറുബ്‌നു അബീസുല്മ്, ലബീദുബ്‌നു റബീഅഃ, അന്ത,രബ്‌നു ശദാദ്, അംറുബ്‌നു കുല്ഥൂംമ, ഹാരിഥുബ്‌നു ഹില്ലിസ എന്നീ ജാഹിലിയ്യാ കവികളുടെ രചനകള്‍ വ്യക്തമാക്കുന്നുണ്ട്. അത്തരക്കാരെ ആദര്ശ്ത്തിനുവേണ്ടി സമരം ചെയ്യുന്നവരാക്കിത്തീര്ത്ത് യുദ്ധത്തെ മാനവവല്ക്ക്രിക്കേണ്ടത് എങ്ങനെയെന്ന് ലോകത്തെ പഠിപ്പിക്കുകയാണ് മുഹമ്മദ് നബി (സ) ചെയ്തത്.

മദീനാരാഷ്ട്രത്തിന്റെ സുരക്ഷയായിരുന്നു മുഹമ്മദ് നബി (സ) നയിച്ച യുദ്ധങ്ങളുടെ ലക്ഷ്യമെങ്കിൽ പിന്നെയെന്തിനാണ് അദ്ദേഹം ഇങ്ങോട്ട് യുദ്ധത്തിന് വരാത്ത അയൽനാടുകളെ ആക്രമിച്ചത്? ഇത്തരം ആക്രമണങ്ങൾ മുഹമ്മദ് നബിക്ക് ശേഷം വ്യാപകമായി നടന്നിട്ടുണ്ടെന്ന് ചരിത്രം പഠിപ്പിക്കുന്നുമൂണ്ടല്ലൊ ?

രാഷ്ട്രസുരക്ഷയ്ക്കും സമാധാനത്തിനും,  പ്രതിരോധത്തെ പോലെ, ചിലപ്പോൾ പ്രത്യാക്രമണവും അനിവാര്യമായിത്തീരും. മദീനാ നാടിനെ തകർക്കാൻ വന്നവരെ പ്രതിരോധിക്കുകയായിരുന്നു ഉഹ്ദിലും ഖൻദഖിലും പ്രവാചകൻ(സ) ചെയ്തത്. തങ്ങളുടെ നാട് തകർക്കാനായി വരുന്നവരെ സ്വദേശത്ത് വെച്ച് പ്രതിരോധിക്കുകയെന്ന തന്ത്രം തുടർന്നാൽ അത് നാടിന്റെയും നാട്ടുകാരുടെയും നാശത്തിലാണ് കലാശിക്കുകയെന്ന് പ്രവാചകനിലെ രാഷ്ട്രനേതാവ് മനസ്സിലാക്കി. പതിനായിരത്തോളം വരുന്ന സഖ്യകക്ഷീസൈന്യത്തിനെങ്ങാനും ദിവസങ്ങളെടുത്ത് മദീനാപ്രവാസികൾ നിർമിച്ച കിടങ്ങ് കടന്ന് മദീനയെ ആക്രമിക്കാൻ കഴിഞ്ഞിരുന്നെങ്കിൽ മദീനാനാട് തന്നെ ഇല്ലാതാകുമായിരുന്നു. അവിടെയുള്ള പുരുഷന്മാരെല്ലാം കൊല്ലപ്പെടുകയും സ്ത്രീകൾ വെപ്പാട്ടികളായിതത്തീർന്ന് അടിമച്ചന്തകളിൽ വില്‍ക്കപ്പെടുകയും ചെയ്യുമായിരുന്നു. അല്ലാഹുവിന്റെ അപാരമായ അനുഗ്രഹത്താലാണ് മദീന രക്ഷപ്പെട്ടത്. ഇനിയും ശത്രുക്കൾക്ക് മദീനയെ വളഞ്ഞ് അതിനെ നശിപ്പിക്കാൻ അവസരം കൊടുത്തുകൂടെന്ന് യുദ്ധതന്ത്രജ്ഞനായ ആ രാഷ്ട്രനേതാവ് തീരുമാനിച്ചു. ഖന്‍ദഖില്‍വെച്ച് സഖ്യകക്ഷികള്‍ നിരാശയോടെ തിരിച്ചുപോയതിനുശേഷമുള്ള പ്രവാചകന്റെ (സ) പ്രഖ്യാപനത്തിൽ കാണാൻ കഴിയുന്നത് നാടിനെ തകർക്കാൻ വരുന്നവരെ അവരുടെ നാടുകളിൽ പോയി പ്രത്യാക്രമിക്കുകയെന്ന രാഷ്ട്രതന്ത്രമാണ്. അദ്ദേഹം പറഞ്ഞു: "ഇനി മുതല്‍ നാം അവരോട് യുദ്ധം ചെയ്യും. അവര്‍ നമ്മോട് യുദ്ധത്തിനൊരുമ്പെടുകയില്ല. (സ്വഹീഹുല്‍ ബുഖാരി കിത്താബുല്‍ മഗാസി)

രാഷ്ട്രതന്ത്രജ്ഞനായ നേതാവിന്റെ പ്രഖ്യാപനം. ശത്രുക്കള്‍ മദീനാ രാജ്യത്തേക്ക് കടന്നുവന്ന് അവിടെയുള്ളവരെ കൊന്നൊടുക്കുകയും രാജ്യസമ്പത്ത് കൊള്ളയടിക്കുകയും നാടിനെ തകര്‍ക്കുകയും ചെയ്യുന്നതിന് സമ്മതിക്കാതെ അവരുടെ കേന്ദ്രങ്ങളില്‍ പോയി ആക്രമിക്കുകയും മദീനയുടെ സുരക്ഷിതത്വം ഉറപ്പുവരുത്തുകയും ചെയ്യുകയെന്ന തന്ത്രമാണ് ഇതിനുശേഷമുള്ള പ്രവാചക യുദ്ധങ്ങളിലെല്ലാം കാണാന്‍ കഴിയുന്നത്. പ്രത്യാക്രമണത്തിലൂടെ സ്വന്തം നാടിനെ പ്രതിരോധിക്കുകയെന്ന തന്ത്രം. അതിന്ന് ആദ്യമായി സ്വന്തം നാടിനകത്തെ ആന്തരികശത്രുക്കളെ മാതൃകാപരമായി ഉന്‍മൂലനം ചെയ്യേണ്ടതുണ്ടായിരുന്നു, പ്രവാചകന്. അനുസരണയുള്ള പ്രജകളായി ജീവിക്കാമെന്ന് ആവര്‍ത്തിച്ച് കരാര്‍ ചെയ്യുകയും യുദ്ധസന്ദര്‍ഭത്തില്‍ കൂറുമാറുകയും ചെയ്ത ബനൂഖുറൈദക്കാരെ ഉന്‍മൂലനം ചെയ്തുകൊണ്ട് ആന്തരികശത്രുക്കള്‍ക്കെതിരെ കര്‍ശനമായ നടപടികളാണ് സ്വീകരിക്കപ്പെടുകയെന്ന് കപടവിശ്വാസികളടക്കമുള്ള മദീന പൗരന്‍മാരെ ബോധ്യപ്പെടുത്തുകയാണ് പ്രവാചകൻ(സ) ഖന് ദഖ്‌ യുദ്ധത്തിന് ശേഷം ആദ്യമായി ചെയ്തത്. .

മദീനാരാഷ്ട്രത്തെ തകര്‍ക്കാന്‍ ഗൂഢാലോചനകള്‍ നടത്തുന്നവരോട് അവര്‍ ഇങ്ങോട്ട് ആക്രമിക്കുന്നതിന് മുമ്പ് അവരുടെ കേന്ദ്രങ്ങളില്‍ പോയി ആക്രമിക്കുകയെന്ന പ്രതിരോധ തന്ത്രം പ്രഖ്യാപിക്കപ്പെട്ടതിനു ശേഷവും ശാന്തിയും സമാധാനവും നിലനിര്‍ത്താന്‍ ഏതുവിധത്തിലുള്ള വിട്ടുവീഴ്ചയ്ക്കും പ്രവാചകന്‍ സന്നദ്ധനായിരുന്നുവെന്ന് വ്യക്തമാക്കുന്നതാണ് ഹുദൈബിയ സന്ധി. സത്യനിഷേധികള്‍ സമാധാനത്തിലേക്ക് ചായവ് കാണിച്ചാല്‍ പ്രവാചകനും(സ) സമാധാനത്തിന് സന്നദ്ധനാകണമെന്ന ക്വുര്‍ആന്‍ നിര്‍ദേശം പ്രയോഗവത്ക്കരിക്കപ്പെടുകയാണ് ഹുദൈബിയ സന്ധിയില്‍ ഉണ്ടായത്. ഹിജ്‌റ ആറാം വര്‍ഷത്തില്‍ ഉംറ നിര്‍വഹിക്കാനായി മക്കയിലേക്ക് പുറപ്പെട്ട പ്രവാചകനും ആയിരത്തിഅഞ്ഞൂറോളം വരുന്ന അനുചരന്‍മാരും വഴിയിലുള്ള ഹുദൈബിയയില്‍ വെച്ച് തടയപ്പെട്ടതിനോടനുബന്ധിച്ച് ഉണ്ടാക്കിയ കരാറാണ് ഹുദൈബിയ സന്ധി എന്നറിയപ്പെടുന്നത്. 'പത്തുവര്‍ഷം ഇരുവിഭാഗവും പരസ്പരം യുദ്ധം ചെയ്യാന്‍ പാടില്ല'യെന്ന സമാധാനക്കരാറിനുവേണ്ടി ഏകപക്ഷീയമായി ഖുറൈശികള്‍ക്ക് അനുകൂലമെന്ന് ഒറ്റനോട്ടത്തില്‍ തോന്നുന്ന, പ്രവാചകാനുചരന്‍മാരടക്കം പ്രതിഷേധിച്ച കരാര്‍ വ്യവസ്ഥകള്‍ അംഗീകരിക്കാന്‍ മുഹമ്മദ് നബി(സ) സന്നദ്ധനായി. ഉമറിെേന  പോലെയുള്ള പ്രവാചകാനുചരന്‍മാര്‍ പോലും ശക്തമായ പ്രതിഷേധമറിയിച്ച കരാറിനെക്കുറിച്ചാണ് 'തീര്‍ച്ചയായും നിനക്ക് പ്രത്യക്ഷമായ ഒരു വിജയം നല്‍കിയിരിക്കുന്നു'(48:1)വെന്ന് അല്ലാഹു പറഞ്ഞതെന്ന കാര്യം പ്രത്യേകം ശ്രദ്ധിക്കുക.

ഹുദൈബിയ സന്ധി സൃഷ്ടിച്ച സമാധാനകാലത്താണ് അയല്‍നാടുകളിലെ ഭരണാധികാരികള്‍ക്ക് അവരെ ഇസ്‌ലാമിലേക്ക് ക്ഷണിച്ചുകൊണ്ട് പ്രവാചകൻ(സ)  കത്തുകളയച്ചത്; അയല്‍പ്രദേശങ്ങളിലേക്ക് ആദര്‍ശപ്രബോധനത്തിനായി അനുചരന്‍മാരെ പറഞ്ഞയച്ചതും ഇക്കാലത്തുതന്നെ. മക്കാ മുശ്‌രിക്കുകളുമായി സമാധാന സന്ധിയുണ്ടാക്കിയശേഷം വഞ്ചകരായ ജൂതന്‍മാര്‍ക്കെതിരെ ശക്തമായ നടപടികള്‍ സ്വീകരിക്കുകയാണ് പ്രവാചകൻ(സ)ചെയ്തത്. മദീനാ രാഷ്ട്രത്തില്‍നിന്ന് പുറത്താക്കപ്പെട്ടതിനുശേഷം ഖൈബറില്‍ ഒരുമിച്ച് ചേര്‍ന്ന് മദീനയെയും ഇസ്‌ലാമിക സമൂഹത്തെയും തകര്‍ക്കാന്‍ ശ്രമിച്ചുകൊണ്ടിരുന്ന ജൂതന്‍മാര്‍ക്കെതിരെ 1600 പേര്‍ അടങ്ങിയ സൈന്യത്തോടെ നടത്തിയ പടയോട്ടമാണ്' അവരുടെ നാടുകളില്‍ പോയി നാം യുദ്ധം ചെയ്യും' എന്ന പ്രവാചക പ്രഖ്യാപനത്തിന്റെ ആദ്യത്തെ പ്രയോഗവത്ക്കരണം. ഖൈബറില്‍ വച്ചു നടന്ന ശക്തമായ യുദ്ധത്തില്‍ മുസ്‌ലിംകള്‍ വിജയിച്ചു. 93 ജൂതന്‍മാരും 15 മുസ്‌ലിംകളും മരണപ്പെട്ട യുദ്ധത്തോടെ ഖൈബര്‍ മുസ്‌ലിംകളുടെ അധീനതയില്‍ വന്നുവെങ്കിലും അവിടുത്തെ യഹൂദന്‍മാരുടെ ആവശ്യങ്ങള്‍ അംഗീകരിക്കാന്‍ നബി (സ) സന്നദ്ധനായി. ഫലഭൂയിഷ്ടമായ ഖൈബറിലെ ഭൂമി പരിപാലിക്കുകയും കൃഷി ചെയ്തു വരുമാനമുണ്ടാക്കുകയും ചെയ്യാന്‍ തങ്ങളെ അനുവദിക്കണമെന്ന യഹൂദ ആവശ്യം ഉത്പന്നത്തിന്റെ പകുതി മുസ്‌ലിംകളുമായി പങ്കുവയ്ക്കണമെന്ന വ്യവസ്ഥയില്‍ അംഗീകരിക്കപ്പെട്ടു. ഖൈബറില്‍നിന്ന് പാഠമുള്‍ക്കൊണ്ട ഫദക്വിലെയും തയ്മാഇലെയും ജൂതന്‍മാര്‍ നബി(സ)യുമായി സമാധാനക്കരാറിലേര്‍പ്പെട്ടപ്പോള്‍ വാദില്‍ ഖുറയിലെ ജൂതന്‍മാര്‍ അക്രമത്തിന്റെ മാര്‍ഗമാണ് കൈക്കൊണ്ടത്. അതുകൊണ്ടുതന്നെ അവര്‍ക്കെതിരെ സൈനിക നടപടിയുണ്ടായി; അവര്‍ പരാജയപ്പെട്ടപ്പോള്‍ സ്വന്തം ഭൂമിയില്‍ കൃഷി ചെയ്യാന്‍ തങ്ങളെ അനുവദിക്കണമെന്ന അവിടെയുള്ള ജൂതന്മാരുടെ ആവശ്യവും അംഗീകരിക്കുകയാണ് പ്രവാചകൻ(സ)ചെയ്തത്.

റോമിലെ കൈസര്‍, സിറിയയിലെ കിസ്‌റ, അബ്‌സീനിയയിലെ നജ്ജാശി, ഈജിപ്തിലെ മുഖൗഖിസ്, ദമസ്‌കസിലെ ഹാരിസ്, ബഹ്‌റൈനിലെ മുന്‍ദിര്‍, സിറിയന്‍ അതിര്‍ത്തിയിലെ ഗസ്സാന്‍, ബുസ്‌റായിലെ അമീര്‍, യമാമയിലെ ഹൗദ, ഒമാനിലെ അമീര്‍ എന്നിവര്‍ക്ക് പ്രവാചകൻ(സ)അയച്ച ഇസ്‌ലാമിലേക്ക് ക്ഷണിച്ചുകൊണ്ടുള്ള കത്തുകള്‍ക്ക് വ്യത്യസ്ത രീതിയിലുള്ള പ്രതികരണങ്ങളാണുണ്ടായത്. റോമാ ചക്രവര്‍ത്തിയുടെ കീഴില്‍ ഭരണം നടത്തിയിരുന്ന ഗസ്സാന്‍ രാജാവ് പ്രവാചകൻ(സ)ന്റെ കത്തിനെ അവഹേളിക്കുകയും അതുമായി ചെന്ന ദൂതനെ വധിക്കുകയും ചെയ്തുകൊണ്ട് ഇസ്‌ലാമിനോടും പ്രവാചകനു (സ)മെതിരെയുള്ള ശാത്രവം പരസ്യമായി പ്രഖ്യാപിച്ചപ്പോള്‍ അദ്ദേഹത്തിനെതിരെ സൈനിക നടപടിയെടുക്കുകയല്ലാതെ നിര്‍വാഹമില്ലെന്ന അവസ്ഥയുണ്ടായി. ഹിജ്‌റ എട്ടാം വര്‍ഷം പ്രവാചകൻ(സ) പറഞ്ഞയച്ച മുവ്വായിരം മുസ്‌ലിംകള്‍ റോമാക്കാരുമായി നടത്തിയ പോരാട്ടമാണ് മുഅ്ത്വ യുദ്ധം എന്നറിയപ്പെടുന്നത്. മൂന്ന് സേനാനായകരടക്കം പന്ത്രണ്ട് പേര്‍ രക്തസാക്ഷികളായ ഈ യുദ്ധത്തിന്റെ ഗതി മുസ്‌ലിംകള്‍ക്ക് എതിരായി തീര്‍ന്നതിനാല്‍ സൈനിക നേതൃത്വമേറ്റെടുത്ത ഖാലിദുബ്‌നു വലീദ്‌ േതന്ത്രപരമായി യുദ്ധരംഗത്തുനിന്ന് പിന്‍മാറുകയാണ് ചെയ്തത്.

ഹുദൈബിയ സന്ധിയിലെ വ്യവസ്ഥകള്‍ പാലിക്കുന്നതില്‍ വീഴ്ചവരുത്തിയ മുശ്‌രിക്കുകള്‍ക്കെതിരെ നടന്ന സൈനിക നടപടിയാണ് മക്കാ വിജയമായി പരിണമിച്ചത്. ഹിജ്‌റ എട്ടാം വര്‍ഷം റമദാനില്‍ പതിനായിരത്തോളം വരുന്ന മുസ്‌ലിംകളുടെ സൈന്യം മക്കിയിലേക്ക് മാര്‍ച്ച് ചെയ്തു. ചരിത്രത്തില്‍ തുല്യതയില്ലാത്ത ഒരു രക്തരഹിത വിപ്ലവത്തിന് മക്ക വിധേയമായി. ഖുറൈശി നേതാവായിരുന്ന അബൂസുഫ്‌യാനടക്കം പലരും മുസ്‌ലിംകളായി. പ്രവാചകനെ പീഡിപ്പിക്കുകയും ജന്‍മനാട്ടില്‍നിന്ന് പുറത്താക്കുകയും അനുചരന്‍മാരെ ക്രൂരമായി കൊലപ്പെടുത്തുകയും ചെയ്തവര്‍ ഉള്‍ക്കൊള്ളുന്ന മക്കക്കാര്‍ക്ക് പ്രവാചകൻ(സ)ന്‍ല നിരുപാധികം മാപ്പു നല്‍കി. കഅ്ബാലയത്തിനകത്ത് പില്‍ക്കാലത്ത് സ്ഥാപിക്കപ്പെട്ട മുന്നൂറ്റി അറുപത് ശിലാവിഗ്രഹങ്ങളും നീക്കം ചെയ്ത് ശുദ്ധീകരിക്കുകയും സ്രഷ്ടാവിനെ മാത്രം ആരാധിക്കാനായി ആദ്യമായി നിര്‍മ്മിക്കപ്പെട്ട ഗേഹത്തില്‍നിന്ന് സൃഷ്ടിപൂജയുടെ ചിഹ്നങ്ങളെ പുറത്താക്കുകയും ചെയ്തു.(15) മക്കാ മണലാരണ്യത്തില്‍ വെച്ച് ക്രൂരമായി പീഡിപ്പിക്കപ്പെട്ട അടിമയായിരുന്ന ബിലാലിനോട് ഈ മഹാവിജയത്തിന്റെ പ്രഖ്യാപനമായി കഅ്ബാലയത്തിന് മുകളില്‍ കയറി ബാങ്ക് വിളിക്കാന്‍ ആവശ്യപ്പെട്ടുകൊണ്ട് മക്കാവിജയത്തെ സകലവിധ സങ്കുചിതത്വങ്ങളുടെയും അടിവേരറുക്കുന്നതിനുള്ള പ്രഖ്യാപനമാക്കിത്തീര്‍ത്തു. മക്കക്കാര്‍ക്കെല്ലാം പൊതുമാപ്പ് നല്‍കിയെങ്കിലും വഞ്ചകരും കൊടുംക്രൂരതകള്‍ കാണിച്ചവരുമായി ഏതാനും പേര്‍ക്കെതിരെ നടപടികളെടുത്തുകൊണ്ട് ചില കുറ്റകൃത്യങ്ങളോടുള്ള ഇസ്‌ലാമിന്റെ നിലപാട് കര്‍ക്കശമാണെന്ന് പ്രവാചകൻ(സ) മക്കാ വിജയത്തോടനുബന്ധിച്ച് വ്യക്തമാക്കി.

മക്കാ വിജയത്തോടനുബന്ധിച്ച് ഖുറൈശികള്‍ കീഴടങ്ങിയെങ്കിലും ശക്തരായ തങ്ങളെ കീഴടക്കാനാവില്ലെന്ന് അഹങ്കരിക്കുകയും നബി(സ)ക്കെതിരില്‍ പടയൊരുക്കം നടത്തുകയും ചെയ്ത ഹവാസീന്‍ ഗോത്രത്തെയും ഥഖീഫ് ഗോത്രത്തെയും പ്രതിരോധിക്കുന്നതിനുവേണ്ടിയാണ് ഹുനൈനില്‍ തുടങ്ങുകയും ത്വാഇഫില്‍ അവസാനിക്കുകയും ചെയ്ത യുദ്ധം നടന്നത്. മാലിക്കുബ്‌നു ഔഫിന്റെ നേതൃത്വത്തില്‍ വ്യത്യസ്തങ്ങളായ ഗോത്രങ്ങള്‍ സംഘടിച്ച് മുസ്‌ലിംകള്‍ക്കെതിരെ അണിനിരക്കുന്നുവെന്നറിഞ്ഞപ്പോഴാണ് പന്ത്രണ്ടായിരം പേരടങ്ങുന്ന വമ്പിച്ച ഒരു സൈന്യവുമായി മക്കാ വിജയം കഴിഞ്ഞ് രണ്ട് ആഴ്ചകള്‍ക്കുശേഷം പ്രവാചകൻ(സ) ഹുനൈനിലേക്ക് പുറപ്പെട്ടത്. എണ്ണപ്പെരുപ്പം സൃഷ്ടിച്ച മതിപ്പും ഇത്ര വലിയ സൈന്യത്തെ ജയിക്കാന്‍ ആര്‍ക്കും സാധിക്കില്ലെന്ന ആത്മസംതൃപ്തിയും അല്ലാഹുവിന്റെ പരീക്ഷണത്തിന് കാരണമായിയെന്ന് ക്വുര്‍ആന്‍ വ്യക്തമാക്കുന്നു.( 9:25) ഇടുങ്ങിയ മലമ്പാതകളിലൂടെ ഹുനൈനിലേക്ക് നീങ്ങിയ മുസ്‌ലിം സൈന്യത്തിന്റെ നേരെ താഴ്‌വരകളിലും കുന്നുകളിലും ഒളിഞ്ഞിരുന്ന ഒളിപ്പോരാളികള്‍ നടത്തിയ അപ്രതീക്ഷിതമായ മിന്നലാക്രമണം മുസ്‌ലിംകളുടെ അണിയെ ഛിന്നഭിന്നമാക്കി. മുസ്‌ലിംകള്‍ ചിന്നിച്ചിതറി നാലുഭാഗത്തേക്കും ഓടി. അവര്‍ക്കിടയിലൂടെ നടന്ന്'ഞാന്‍ നബിയാണ്; കള്ളനല്ല'; അബ്ദുല്‍മുത്തലിബിന്റെ പുത്രനാണ്' എന്ന് പ്രഖ്യാപിച്ചുകൊണ്ട് മുഹമ്മദ് നബി(സ) യുദ്ധരംഗം വിട്ടോടുന്നവരെ തിരിച്ചുവിളിച്ചു. അതോടെ അല്ലാഹുവിന്റെ അനുഗ്രഹത്താല്‍ മുസ്‌ലിംകള്‍ക്ക് ധൈര്യം ലഭിക്കുകയും യുദ്ധരംഗത്ത് സജീവമാകുകയും ചെയ്തു. അതോടെ ശത്രുക്കള്‍ തോറ്റോടി. ചിലര്‍ ത്വാഇഫിലേക്കാണ് ഓടിപ്പോയത്. അവിടെയുള്ള കോട്ടകള്‍ക്കകത്ത് അഭയം തേടിയവര്‍ക്കെതിരെ ശ്ക്തമായ നടപടികളുമായി പ്രവാചകനും പ്രവാചകൻ(സ) അനുചരന്‍മാരും മുന്നേറിയെങ്കിലും അവരെ കോട്ടകള്‍ക്കകത്തുനിന്ന് പുറത്തുകൊണ്ടുവരാന്‍ സാധിച്ചില്ല. ഹുെനെന്‍ യുദ്ധത്തില്‍ പിടിച്ചുവെക്കപ്പെട്ടവരെ തങ്ങള്‍ക്കുതന്നെ തിരിച്ചുതരണമെന്ന് യുദ്ധത്തില്‍ പരാജയപ്പെട്ട ഹവാസിന്‍ ഗോത്രക്കാര്‍ പ്രവാചകനുടുത്ത് വന്ന് ആവശ്യപ്പെട്ടപ്പോള്‍ പ്രവാചകൻ(സ)അത് അംഗീകരിക്കുകയാണ് ചെയ്തത്.

അറേബ്യന്‍ ഉപഭൂഖണ്ഡത്തിലെ പ്രധാനപ്പെട്ട പ്രദേശങ്ങളെല്ലാം ഇസ്‌ലാമിന് കീഴിലാവുമെന്ന് മനസ്സിലായപ്പോള്‍, അത് പ്രതിരോധിക്കാനായി റോമാ സാമ്രാജ്യം സായുധ ശേഖരണവും സൈനിക വിന്യാസവും നടത്തിയപ്പോള്‍ അവര്‍ക്കെതിരെ നടന്ന തബൂക്ക് യുദ്ധവും നാടിനെയും ആദര്‍ശത്തെയും സംരക്ഷിക്കുന്നതിനുവേണ്ടി നടന്നതായിരുന്നു. കഠിനമായ ചൂടുകാലത്താണ് മുപ്പതിനായിരം മുസ്‌ലിംകളുള്‍ക്കൊള്ളുന്ന പ്രവാചകന്റെ സൈന്യം തബൂക്കിലേക്ക് പോയത്. നിശ്ചയദാര്‍ഢ്യത്തോടെ വരുന്ന മുസ്‌ലിം സൈന്യത്തിന്റെ ഗതി കണ്ട് ഭയന്ന റോമന്‍ സൈന്യം തബൂക്കില്‍നിന്ന് പിന്‍മാറിയതിനാല്‍ യുദ്ധം നടന്നില്ല. പ്രവാചകൻ(സ) ന്‍നേരിട്ട് പങ്കെടുത്ത അവസാനത്തെ സൈനിക നീക്കവും കാര്യമായ ആള്‍നാശമൊന്നുമുണ്ടാകാതെ വിജയത്തിന്റെ പാതക ഉയര്‍ത്തുകയാണുണ്ടായത്-എന്നര്‍ഥം.

ആദര്‍ശത്തിന്റെ അടിസ്ഥാനത്തില്‍ സ്ഥാപിക്കപ്പെട്ട ഒരു രാഷ്ട്രത്തിന്റെ നേതാവ് എന്ന നിലയ്ക്കുള്ള തന്റെ ഉത്തരവാദിത്ത നിര്‍വഹണമാണ് പ്രവാചകൻ(സ) നയിച്ച യുദ്ധങ്ങളിലെല്ലാം നമുക്ക് കാണാന്‍ കഴിയുന്നത്. പൗരന്‍മാരുടെ ജീവനും സ്വത്തും അഭിമാനവും മതവും സംരക്ഷിക്കുക, നാടിന്റെ സുരക്ഷ ഉറപ്പുവരുത്തുക, ആന്തരിക ശത്രുക്കളെ ഇല്ലായ്മ ചെയ്യുക, ഇസ്‌ലാമികാദര്‍ശമനുസരിച്ച് ജീവിക്കാന്‍ സ്വാതന്ത്ര്യമുള്ള അവസ്ഥ എല്ലായിടത്തും നിലനില്‍ക്കുന്നുണ്ട് എന്ന് ഉറപ്പുവരുത്തുക എന്നിങ്ങനെ ഒരു ഭരണാധികാരിയുടെ ഉത്തരവാദിത്തങ്ങളുടെ നിര്‍വഹണത്തിന് വേണ്ടിയാണ് പ്രവാചകൻ(സ) യുദ്ധം ചെയ്തത്. മക്കയിലും മറ്റ് പ്രദേശങ്ങളിലുമെല്ലാം പീഡനങ്ങളനുഭവിക്കുന്ന മുസ്‌ലിംകളുടെ മോചനവും അവര്‍ക്ക് സ്വതന്ത്രമായി ഇസ്‌ലാം അനുധാവനം ചെയ്യാന്‍ കഴിയുന്ന അവസ്ഥയുടെ സംസ്ഥാപനവും പ്രവാചക യുദ്ധങ്ങളുടെ ലക്ഷ്യങ്ങളിലൊന്നായിരുന്നു.

ല്ല. അധികാരമോഹമായിരുന്നില്ല അധികാരമേല്പിക്കപ്പെട്ട നാടിന്റെ സംരക്ഷണമെന്ന ഉത്തരവാദിത്തത്തിൻറെ നിർവഹണമായിരുന്നു ബദർ യുദ്ധത്തിന് കാരണമായിത്തതീർന്നത്.

മുഹമ്മദ് നബി(സ) നേതൃത്വം നല്‍കിയ ആദ്യത്തെ പോരാട്ടമാണ് ബദ്‌റ് യുദ്ധം. തികച്ചും ആകസ്മികമായി ഉണ്ടായ ഒരു യുദ്ധമായിരുന്നു അത്. മദീനയിലെ ഇസ്‌ലാമിക സമൂഹത്തിന്റെ ഭാഗമാകാന്‍ സമ്മതിക്കാതെ മക്കയില്‍ ക്രൂരമായി പിടിച്ചുവെക്കപ്പെട്ട വിശ്വാസികളുടെ മോചനം, പ്രവാചകനെയും അനുയായികളെയും നശിപ്പിക്കുകയും മദീനയില്‍ വളര്‍ന്നുകൊണ്ടിരിക്കുന്ന മുസ്‌ലിം സമൂഹത്തെ തകര്‍ക്കുകയും ചെയ്യുമെന്ന മക്കാമുശ്‌രിക്കുകളുടെ ഭീഷണിക്ക് അറുതിവരുത്തുക, മദീനക്ക് ചുറ്റുമുള്ള അറബ് ഗോത്രങ്ങളുടെ അക്രമങ്ങളില്‍നിന്ന് നാടിനെ രക്ഷിക്കുകയും അവരുമായി സമാധാനസന്ധിയുണ്ടാക്കുകയും ചെയ്യുക. ആദര്‍ശമനുസരിച്ച് ജീവിക്കുവാനുള്ള സ്വാതന്ത്ര്യത്തിനുവേണ്ടി നാടുവിട്ടപ്പോള്‍ മുസ്‌ലിംകള്‍ മക്കയില്‍ ഉപേക്ഷിച്ചുപോന്ന സമ്പത്ത് അതിന്റെ അവകാശികള്‍ക്ക് തിരിച്ച് നല്‍കുന്നതിനുവേണ്ടി പരിശ്രമിക്കുക തുടങ്ങിയ ലക്ഷ്യങ്ങള്‍ക്കുവേണ്ടി മുഹമ്മദ് നബി(സ)യുടെ നേരിട്ടുള്ള നേതൃത്വത്തിലും അല്ലാതെയും നടന്ന സൈനിക നീക്കങ്ങളിലൊന്നാണ് ബദ്‌റ് യുദ്ധമായി പരിണമിച്ചത്.

മക്കയില്‍ മുശ്‌രിക്കുകള്‍ നയിക്കുന്ന വ്യാപാര സംഘങ്ങളെ തടയുകയും നടേ പറഞ്ഞ ലക്ഷ്യങ്ങള്‍ നേടുകയും ചെയ്യുകയെന്ന ഉദ്ദേശത്തോടെ ഹിജ്‌റക്കുശേഷം ഏഴാം മാസം നടത്തിയ സൈഫില്‍ ബഹ്‌റ് മുതലുള്ള സൈനിക നീക്കങ്ങളെപോലെയുള്ള ഒരു നീക്കം മാത്രമായിരുന്നു നബി(സ)യും സ്വഹാബിമാരും ബദ്‌റിലേക്ക് നടത്തിയത്. അതുകൊണ്ടുതന്നെ മദീനയിലുണ്ടായിരുന്ന സ്വഹാബിമാരില്‍ പലരും ബദ്‌റിലേക്ക് പോവുകയോ യുദ്ധത്തില്‍ പങ്കെടുക്കുകയോ ചെയ്തിരുന്നില്ല.(1) മുന്നൂറ്റിപ്പത്തില്‍പരം പേര്‍ മുഹമ്മദ് നബി(സ)യോടൊപ്പം ബദ്‌റില്‍ പങ്കെടുത്തുവെന്ന് ബുഖാരിയും(2) അവരുടെ എണ്ണം എണ്ണൂറ്റിപത്തൊന്‍പതായിരുന്നുവെന്ന് മുസ്‌ലിമും(3) നിവേദനം ചെയ്തിട്ടുണ്ട്. യുദ്ധത്തിന് ഒരുങ്ങിപോയവരല്ലാത്തതിനാല്‍തന്നെ അവരുടെ പക്കല്‍ ആവശ്യത്തിനുള്ള ആയുധങ്ങളൊന്നും ഉണ്ടായിരുന്നില്ല. അവര്‍ക്ക് പോരാടാനുണ്ടായിരുന്നതാകട്ടെ സര്‍വ്വായുധ സന്നദ്ധരായി യുദ്ധത്തിന് ഒരുങ്ങിവന്ന(4) യുദ്ധസജ്ജരും നിപുണരുമായ ഖുറൈശികളോടായിരുന്നു. രണ്ട് കുതിരകളും എഴുപത് ഒട്ടകങ്ങളും മുന്നൂറിലധികം പേരുമടങ്ങുന്ന മുഹമ്മദ് നബി(സ)യുടെ നേതൃത്വത്തിലുള്ള മുസ്‌ലിം സൈന്യവും നൂറിലധികം കുതിരകളും നിരവധി ഒട്ടകങ്ങളും ആയിരത്തിമുന്നൂറോളം വരുന്ന സായുധസജ്ജരുമടങ്ങുന്ന അബൂജഹ്‌ലിന്റെ നേതൃത്വത്തിലുള്ള ഖുറൈശീ സൈന്യവും തമ്മിലാണ് ബദ്‌റില്‍ ഏറ്റുമുട്ടിയത്.(5) പക്ഷെ, ഇസ്‌ലാമിക ചരിത്രത്തിലെ പ്രഥമ യുദ്ധത്തില്‍ മുസ്‌ലിംകള്‍ വിജയിച്ചു. ബഹുദൈവാരാധകരിലെ നേതാക്കളായ അബൂജഹ്‌ലും ഉമയ്യത്തുബിനു ഖലഫുമടക്കമുള്ള എഴുപത് പേര്‍ മരണപ്പെടുകയും എഴുപത് പേരെ മുസ്‌ലിംകള്‍ ബന്ദികളായി പിടിക്കുകയും െചയ്തപ്പോള്‍(6) പതിനാല് മുസ്‌ലിംകളാണ് ബദ്‌റ് യുദ്ധത്തില്‍ രക്ഷസാക്ഷികളായത്.(7) ചെറിയൊരു സംഘം അല്ലാഹുവിന്റെ സഹായത്താല്‍ വലിയൊരു സംഘത്തെ ജയിച്ചടക്കി. ബദ്‌റിലുണ്ടായ അല്ലാഹുവിന്റെ സഹായത്തെക്കുറിച്ച് വ്യക്തമാക്കുന്ന ക്വുര്‍ആന്‍ സൂക്തങ്ങള്‍ കാണുക: ''നിങ്ങള്‍ ദുര്‍ബലരായിരിക്കെ ബദ്‌റില്‍ വെച്ച് അല്ലാഹു നിങ്ങളെ സഹായിച്ചിട്ടുണ്ട്. അതിനാല്‍ നിങ്ങള്‍ അല്ലാഹുവെ സൂക്ഷിക്കുക. നിങ്ങള്‍ നന്ദിയുള്ളവരായേക്കാം. നബി(സ)യേ നിങ്ങളുടെ രക്ഷിതാവ് മൂവായിരം മലക്കുകളെ ഇറക്കികൊണ്ട് നിങ്ങളെ സഹായിക്കുക എന്നത് നിങ്ങള്‍ക്ക് മതിയാവുകയില്ലേ എന്ന് നീ സത്യവിശ്വാസികളോട് പറഞ്ഞിരുന്ന സന്ദര്‍ഭം (ഓര്‍ക്കുക)'' (3:123,124)

ആദര്‍ശത്തിന്റെ സംരക്ഷണത്തിനായി അന്തിമ പ്രവാചകനും അനുയായികളും തോളോട് തോളുരുമ്മി പടവെട്ടിയപ്പോള്‍ അവിടെ അല്ലാഹുവിന്റെ സഹായമുണ്ടായി. അല്ലാഹുവിന്റെ തൃപ്തി മാത്രം കാംക്ഷിച്ചുകൊണ്ടാണ് മുസ്‌ലിംകള്‍ പടവെട്ടിയതെന്ന വസ്തുത വ്യക്തമാക്കുന്നതാണ് നടേ വിവരിച്ച സംഭവം. അധികാരവും യുദ്ധാര്‍ജ്ജിത സമ്പത്തും അങ്ങനെ ലഭിക്കുന്ന അടിമപ്പെണ്‍കൊടികളുമൊത്തുള്ള ശയനവുമായിരുന്നു മുസ്‌ലിംകളെ യുദ്ധത്തിന് പ്രചോദിപ്പിച്ചതെന്ന വിമര്‍ശനത്തിന്റെ നട്ടെല്ലൊടിക്കുന്ന ഇത്തരം നിരവധി സംഭവങ്ങള്‍ നിവേദനം ചെയ്യപ്പെട്ടിട്ടുണ്ട്. അല്ലാഹുവിന്റെ പ്രീതി ആഗ്രഹിച്ചുകൊണ്ടും അവന്റെ സഹായം പ്രതീക്ഷിച്ചുകൊണ്ടും വിശ്വാസികള്‍ പടപൊരുതിയപ്പോള്‍ അല്ലാഹു അവരെ സഹായിച്ചു. യുദ്ധത്തില്‍ മുസ്‌ലിംകള്‍ വിജയിച്ചത് അല്ലാഹുവിന്റെ സഹായം കൊണ്ടാണ്. കൂടാരത്തില്‍ പ്രാര്‍ഥനാനിമഗ്‌നനായിരുന്ന പ്രവാചകൻ(സ) ഇടയ്ക്ക് പുറത്തുവന്നുകൊണ്ട് അബൂബക്കറിനോട്‌ (റ)പറഞ്ഞു: 'സന്തോഷവാര്‍ത്തയുണ്ട് അബൂബക്ര്‍. അല്ലാഹുവിന്റെ സഹായം വന്നെത്തിയിരിക്കുന്നു. തലപ്പാവ് വെച്ചുകൊണ്ട് തന്റെ കുതിരയെ നയിച്ച് ഇതാ ജിബ്‌രീല്‍ എത്തിക്കഴിഞ്ഞു. അദ്ദേഹത്തിന്റെ കുതിരയുടെ പല്ലില്‍ പൊടി പിടിച്ചിട്ടുണ്ട്. അല്ലാഹു വാഗ്ദാനം ചെയ്ത സഹായം ഇതാ വന്നെത്തിയിരിക്കുന്നു'(8)

അധികാരമോഹവും അധീശത്വ സ്വപ്നങ്ങളുമാണ് മുഹമ്മദ് നബി(സ)യെ ആയുധമെടുക്കുവാന്‍ പ്രചോദിപ്പിച്ചത് എന്ന് പറയുന്നവര്‍ക്ക് ബദ്‌റില്‍വെച്ച് അദ്ദേഹം മറ്റെന്ത് ചെയ്യണമെന്നാണ് നിര്‍േദശിക്കുവാനുള്ളതെന്ന് അറിയുവാന്‍ കൗതുകമുണ്ട്. മുസ്‌ലിംകള്‍ക്കെതിരെ പടപൊരുതിക്കൊണ്ട് തങ്ങളുടെ യശസ്സ് അറബ് ലോകത്തെ അറിയിക്കണമെന്ന് അഹങ്കരിക്കുന്ന അബൂജഹ്‌ലിന്റെ നേതൃത്വത്തിലുള്ള ഒരു വന്‍സൈന്ന്യം അതിക്രമങ്ങള്‍ക്കൊരുങ്ങി ബദ്‌റില്‍ തമ്പടിച്ചപ്പോള്‍ അവര്‍ക്കെതിരെ സായുധസന്നാഹം നടത്താതിരിക്കണമെന്ന് പറയുവാന്‍ രാഷ്ട്രതന്ത്രത്തിന്റെ ബാലപാഠമെങ്കിലും അറിയുന്നവര്‍ സന്നദ്ധമാകുമോ? അത് നടന്നിട്ടില്ലായിരുന്നുവെങ്കില്‍ പ്രസ്തുത സൈന്യം മദീനയിലേക്ക് മാര്‍ച്ച് ചെയ്യുകയും മദീനാപട്ടണം തന്നെ നാമാവശേഷമാകുന്ന സ്ഥിതി ഉണ്ടാവുകയും ചെയ്യുമായിരുന്നുവെന്ന് സംഭവങ്ങളെ നിരീക്ഷിക്കുന്ന ആര്‍ക്കാണ് മനസിലാക്കാന്‍ കഴിയാത്തത്! യുദ്ധത്തിന് ഒരുങ്ങിവന്ന ശത്രുക്കളില്‍നിന്ന് നാടിനെയും നാട്ടുകാരെയും സംരക്ഷിക്കുകയും അങ്ങനെ സമാധാനപൂര്‍ണമായി ഇസ്‌ലാമിക ജീവിതം നയിക്കുവാനും അതിലേക്ക് ആളുകളെ ക്ഷണിക്കുവാനുമുള്ള സാഹചര്യങ്ങള്‍ ഉണ്ടാക്കുകയും ചെയ്യുവാന്‍ വേണ്ടിയാണ് ബദ്‌റ് യുദ്ധം നടത്തണമെന്ന വസ്തുത സത്യസന്ധമായി ചരിത്രപഠനം നിര്‍വഹിക്കുന്നവര്‍ക്കൊന്നും നിഷേധിക്കാനാവില്ല, തീര്‍ച്ച.

  1. സ്വഹീഹുല്‍ ബുഖാരി, കിത്താബുല്‍ മഗാസി
  2. സ്വഹീഹുല്‍ ബുഖാരി, കിത്താബുല്‍ മഗാസി
  3. സ്വഹീഹു മുസ്‌ലിം, കിത്താബുല്‍ ജിഹാദ്, വസ്സിയീര്‍
  4. സ്വഹീഹു മുസ്‌ലിം, കിത്താബുല്‍ ജിഹാദ്, വസ്സിയീര്‍
  5. ഇമാം ഇബ്‌നുകഥീര്‍ തന്റെ അല്‍ ബിദായയില്‍ (3/284-285) മുര്‍സലായി നിവേദനം ചെയ്തത്. Mahdi Rizqulla Ahmed: A Biography of the Prophet of Islam; Riyadh 2005, page 391
  6. സ്വഹീഹു മുസ്‌ലിം, കിത്താബുല്‍ ജിഹാദ് വസ്സിയീര്‍
  7. ഇമാം ഇബ്‌നു കഥീര്‍ അല്‍ ബിദായയില്‍ (3/230) മൂസബ്‌നു ഉഖ്ബയില്‍ (റ) നിന്ന് നിവേദനം ചെയ്തത് M.R. Ahmad Opt.cit page 414
  8. സ്വഹീഹു മുസ്‌ലിം, കിത്താബുല്‍ ജിഹാബു വസ്സയീര്‍
  9. ഇബ്‌നു കഥീര്‍ അല്‍ബിദായയില്‍ ഹസനായി നിവേദനം ചെയ്തത് (ഇമാം അല്‍ബാനി: ഫിഖ്ഹുസ്‌സീറക്ക് എഴുതിയ അടിക്കുറിപ്പുകള്‍ പുറം 243)

ഭരണാധികാരം ഒരു ഉത്തരവാദിത്തമാണെന്നും അതിന്റെ നിര്‍വഹണം എങ്ങനെയായിരുന്നുവെന്ന വിഷയത്തില്‍ ഭരണാധികാരികള്‍ അല്ലാഹുവിന്റെ മുമ്പില്‍ വിചാരണ നേരിടുമെന്നും പഠിപ്പിച്ച ആദർശമാണ് ഇസ്‌ലാം. സുഖിക്കുവാനുള്ള മാർഗ്ഗമല്ല, പ്രത്യുത അല്ലാഹുവിന്റെ മുന്നിൽ ഉത്തരം പറയേണ്ട വലിയ ഉത്തരവാദിത്തമാണ് മുസ്ലിംകളെ സംബന്ധിച്ചിടത്തോളം ഭരണാധികാരം. വലിയൊരു ഉത്തരവാദിത്തമാണ് അധികാരമെന്ന് പഠിപ്പിക്കപ്പെടുമ്പോള്‍ ഭരണനിര്‍വഹണരംഗത്തെ വ്യത്യസ്തങ്ങളായ സാഹചര്യങ്ങളില്‍ എങ്ങനെ പെരുമാറണമെന്നുകൂടി വ്യക്തമാക്കേണ്ട ചുമതല അങ്ങനെ പഠിപ്പിക്കുന്നവര്‍ക്കുണ്ട്. യുദ്ധം അനിവാര്യമാകുന്ന സാഹചര്യങ്ങളില്‍ അത് നടത്തേണ്ടതെങ്ങനെയെന്ന് അവസാനനാളു വരെയുള്ളവരെയെല്ലാം പഠിപ്പിക്കുകയെന്ന ദൗത്യം നിര്‍വഹിക്കേണ്ടയാളാണ് അന്തിമ പ്രവാചകനായ മുഹമ്മദ് നബി(സ).

തിന്മയെ കൈകൊണ്ട് തടുക്കാനാകുമെങ്കില്‍ അങ്ങനെതന്നെ ചെയ്യണമെന്ന് നിഷ്‌കര്‍ഷിച്ച പ്രവാചകന്‌(സ), ഒരു ഭരണാധികാരിയെന്ന നിലയില്‍ എങ്ങനെയാണ് തിന്മകളെ കൈകൊണ്ട് പ്രതിരോധിക്കേണ്ടത് എന്നുകൂടി പ്രായോഗികമായി പഠിപ്പിക്കേണ്ടതുണ്ട്. ഇസ്‌ലാമിക നിയമങ്ങള്‍ നടപ്പിലാക്കിക്കൊണ്ട് തന്റെ ഭരണപ്രദേശത്ത് നടക്കുന്ന തിന്മകളെ എങ്ങനെ കൈകൊണ്ട് തടുക്കാമെന്ന് കാണിച്ചുതന്നതോടൊപ്പംതന്നെ തന്റെ പൗരന്മാര്‍ക്കും നാടിനുമെതിരെയുള്ള അതിക്രമങ്ങളോട് എന്ത് നിലപാടാണ് സ്വീകരിക്കേണ്ടത് എന്നുകൂടി സൈനികനീക്കങ്ങളിലൂടെയും നേര്‍ക്കുനേരെയുള്ള പോരാട്ടങ്ങളിലൂടെയും അദ്ദേഹം  അനുയായികളെ തെര്യപ്പെടുത്തി. അങ്ങനെ, ജീവിതത്തിന്റെ എല്ലാ രംഗങ്ങളെയും മാനവവല്‍ക്കരിക്കേണ്ടത് എങ്ങനെയെന്ന് മുഹമ്മദ് നബി(സ) മനുഷ്യരെ പഠിപ്പിച്ചു. 'തീര്‍ച്ചയായും താങ്കള്‍ മഹത്തായ സ്വഭാവത്തിലാകുന്നു' (ക്വുര്‍ആന്‍ 68:4) എന്നും 'തീര്‍ച്ചയായും നിങ്ങള്‍ക്ക് അല്ലാഹുവിന്റെ ദൂതനില്‍ മഹത്തായ മാതൃകയുണ്ട്' (33:21) എന്നുമുള്ള അല്ലാഹുവിന്റെ വചനങ്ങളെ അന്വര്‍ഥമാക്കിക്കൊണ്ട് താനുള്‍ക്കൊള്ളുന്ന ആദര്‍ശത്തെയും പ്രസ്തുത ആദര്‍ശത്തിന്റെ അടിത്തറയില്‍ സ്ഥാപിക്കപ്പെട്ട രാഷ്ട്രസംവിധാനത്തെയും തകര്‍ക്കുവാനുദ്ദേശിച്ച് യുദ്ധത്തിന് വരുന്നവരോടും അതിന് കോപ്പുകൂട്ടുന്നവരോടും എങ്ങനെ പ്രതികരിക്കണമെന്ന വിഷയത്തിലുള്ള മഹത്തായ മാതൃകയാണ് താന്‍ നയിച്ച യുദ്ധങ്ങളിലൂടെയും സൈനിക നടപടികളിലൂടെയും പ്രവാചകന്‍ല മാനവരാശിക്ക് മുമ്പില്‍വെച്ചത്.

ദൈവിക മാര്‍ഗദര്‍ശനപ്രകാരം ജീവിച്ച് ശാന്തിയും മരണാനന്തരം ശാന്തിയുടെ ഭവനവും നേടിയെടുക്കുവാന്‍ ശ്രമിക്കുകയും അത് സഹജീവികള്‍ക്ക് പറഞ്ഞുകൊടുക്കുവാനായി ത്യാഗപരിശ്രമങ്ങളില്‍ ഏര്‍പ്പെടുകയും ചെയ്യുന്നവരെ ജീവിക്കുവാൻ അനുവദിക്കുകയില്ലെന്ന് നിശ്ചയിച്ചുറച്ച് അതിക്രമങ്ങളില്‍ ഏര്‍പ്പെടുന്നവരെ കൈകൊണ്ട് തടയേണ്ടതെങ്ങനെയെന്ന് പഠിപ്പിക്കുകവഴി ഒരു ഭരണാധികാരിയുടെ ദൗത്യനിര്‍വഹണം എങ്ങനെയെന്ന് മനസ്സിലാക്കിക്കൊടുക്കുകയായിരുന്നു അദ്ദേഹം. ശാന്തമായ ഭൗതികജീവിതം നയിച്ച് ശാന്തിയുടെ ശാശ്വത ഭവനത്തിലെത്തിച്ചേരുവാന്‍ ആത്മാര്‍ഥമായി പരിശ്രമിക്കുന്നവരുടെ മുമ്പില്‍ വിഘ്‌നങ്ങളുണ്ടായിക്കൂടായെന്ന് നിഷ്‌കര്‍ഷിക്കുകയും ഉണ്ടാകുന്ന വിഘ്‌നങ്ങള്‍ എങ്ങനെ നീക്കം ചെയ്യാമെന്ന് കാണിച്ചുകൊടുക്കുകയും ചെയ്യുകവഴി 'ലോകര്‍ക്ക് മുഴുവന്‍ കാരുണ്യമായിക്കൊണ്ടല്ലാതെ താങ്കളെ നാം നിയോഗിച്ചിട്ടില്ല' (21:107)യെന്ന അല്ലാഹുവിന്റെ പ്രഖ്യാപനം അന്വർത്ഥമാക്കുക കൂടിയാണ് മുഹമ്മദ് നബി(സ) ചെയ്തത്.

നിയമം നടപ്പാക്കുകയാണ് ഭരണാധികാരിയുടെ ധര്‍മം. പൗരന്മാര്‍ക്ക് അടിസ്ഥാനപരമായ അഞ്ച് അവകാശങ്ങളുണ്ടെന്നും അവ സംരക്ഷിക്കുവാനാവശ്യമായ നിയമങ്ങളാണ് ഇസ്‌ലാം മുന്നോട്ടുവെക്കുന്നതെന്നും ക്വുര്‍ആനിലും ഹദീഥുകളിലുമുള്ള മാര്‍ഗനിര്‍ദേശങ്ങളുടെ വെളിച്ചത്തില്‍ അവയില്‍നിന്ന് നിയമങ്ങള്‍ നിര്‍ധരിച്ച് വിശദീകരിച്ച കര്‍മശാസ്ത്ര പണ്ഡിതന്മാര്‍ വ്യക്തമാക്കിയിട്ടുണ്ട്. മതം, ജീവന്‍, മനസ്സ്, അഭിമാനം, സ്വത്ത് എന്നീ അഞ്ച് അടിസ്ഥാന കാര്യങ്ങളുടെ സംരക്ഷണമാണ് നിയമത്തിന്റെ ധര്‍മം (മഖാസിദശ്ശരീഅ:). മനുഷ്യനും അവന്റെ സ്രഷ്ടാവും തമ്മിലുള്ള ബന്ധമായിരിക്കണം അവന്റെ ജീവിത നിലപാടുകളെ മൊത്തത്തില്‍ നിര്‍ണയിക്കേണ്ടത് എന്നതുകൊണ്ടുതന്നെ ദൈവിക മതം അനുധാവനം ചെയ്യുവാനുള്ള അവന്റെ അവകാശവും സ്വാതന്ത്ര്യവുമാണ് നിയമംമൂലം സംരക്ഷിക്കപ്പെടേണ്ട ഒന്നാമത്തെ കാര്യമെന്ന് ഇസ്‌ലാം നിഷ്‌കര്‍ഷിക്കുന്നു. കൊല, പീഡനം, ഭീതി, പട്ടിണി എന്നിവയുടെ ഭീഷണിയില്ലാതെ ജീവിക്കുവാന്‍ ഓരോ പൗരനും കഴിയണമെന്നതാണ് രണ്ടാമത്തെ കാര്യം. ലഹരി ഉപയോഗിക്കുന്നതുമൂലമുണ്ടാകുന്ന മാനസിക സ്ഥിരതയില്ലായ്മ, കളവ്, പരദൂഷണം, തെറ്റായ ആരോപണങ്ങള്‍ തുടങ്ങിയവ മൂലമുണ്ടാകുന്ന മാനസികസംഘര്‍ഷം തുടങ്ങിയവ ഇല്ലാതെയാക്കുകയാണ് നിയമത്തിന്റെ മൂന്നാമത്തെ ധര്‍മം. അഭിമാനവും അസ്തിത്വവും സംരക്ഷിക്കപ്പെടും. ഒരു സാമൂഹ്യ സംവിധാനത്തിന്റെ നിലനില്‍പാണ് നിയമത്തിന്റെ നാലാമത്തെ ധര്‍മം. കൊള്ള, ചൂഷണം, തകര്‍ച്ച, വഞ്ചന എന്നിവയില്‍നിന്ന് മുക്തമായ ഒരു സാമ്പത്തികക്രമത്തിന്റെ നിലനില്‍പാണ് അഞ്ചാമത്തേത്. ഈ അഞ്ച് കാര്യങ്ങളും നിര്‍വഹിക്കുവാന്‍ പര്യാപ്തമായ, ക്വുര്‍ആനും പ്രവാചകചര്യയും വരച്ചുകാണിക്കുന്ന നിയമങ്ങള്‍ നടപ്പാക്കുകയാണ് ഭരണാധികാരിയുടെ ഉത്തരവാദിത്തം. പ്രമാണങ്ങളില്‍ നേര്‍ക്കുനേരെ പരാമര്‍ശിക്കപ്പെട്ടിട്ടുള്ള നിയമങ്ങള്‍ മനസ്സിലാക്കുവാനും ഇല്ലാത്തവ അവയുടെ വെളിച്ചത്തില്‍ നിര്‍ധരിച്ചെടുക്കുവാനുമുള്ള അറിവ് അയാള്‍ക്കില്ലെങ്കില്‍, ഇക്കാര്യത്തിന് പണ്ഡിതന്മാരുടെ സഹായത്തോടെയാണ് അയാള്‍ ഭരണം നിര്‍വഹിക്കേണ്ടത്.

പൗരന്മാരുടെ മതം, ജീവന്‍, മനസ്സ്, അഭിമാനം, സ്വത്ത് എന്നിവയുടെ സംരക്ഷണത്തിനാവശ്യമായ നിയമങ്ങള്‍ സ്വന്തം നാട്ടിലെ പൗരന്‍മാരുടെ മേല്‍ മാത്രമെ നടപ്പാക്കുവാന്‍ ഭരണാധികാരിക്ക് കഴിയുകയുള്ളൂ. ഇവയെ ചോദ്യം ചെയ്യുന്നത് മറ്റൊരു നാട്ടുകാരാണെങ്കില്‍ പ്രസ്തുത നാട്ടിലെ അധികാര സ്ഥാപനങ്ങളിലൂടെ മാത്രമെ പ്രശ്‌നം പരിഹരിക്കുവാന്‍ കഴിയുകയുള്ളൂ. ഇസ്‌ലാമിക സമൂഹത്തിലെ പൗരന്‍മാരുടെ മതവും ജീവനും മനസ്സമാധാനവും അഭിമാനവും സ്വത്തുമെല്ലാം അപകടപ്പെടുത്തുവാന്‍ അന്യനാട്ടില്‍ അവിടുത്തെ അധികാരസ്ഥാപനങ്ങളുടെ സമ്മതത്തോടെയോ നേതൃത്വത്തിലോ ശ്രമം നടക്കുന്നുണ്ടെന്ന് മനസ്സിലായാല്‍ അതിനെ പ്രതിരോധിച്ചുകൊണ്ട് പൗരന്‍മാരുടെ അടിസ്ഥാനാവകാശങ്ങള്‍ സംരക്ഷിക്കേണ്ടത് ഭരണാധികാരിയുടെ ഉത്തരവാദിത്തമാണ്. സമാധനപൂര്‍ണമായ മാര്‍ഗങ്ങള്‍ പരാജയപ്പെടുമ്പോള്‍ ഇക്കാര്യത്തിനുവേണ്ടി അനിവാര്യമെങ്കില്‍ ആയുധമെടുക്കണമെന്നുതന്നെയാണ് ഇസ്‌ലാം ഭരണാധികാരികളോട് അനുശാസിക്കുന്നത്. യുദ്ധം ജിഹാദായിത്തീരുന്നത് ഇത്തരം സാഹചര്യങ്ങളിലാണ്.

മുസ്‌ലിംകളുടെ മതത്തിനും ജീവനും മനസ്സമാധാനത്തിനും അഭിമാനത്തിനും സ്വത്തിനും വെല്ലുവിളി ഉയര്‍ത്തിക്കൊണ്ട് ശത്രുക്കള്‍ സമരസജ്ജരാകുമ്പോള്‍ അതിനെ പ്രതിരോധിച്ച് പൗരന്‍മാരുടെ അടിസ്ഥാന അവകാശങ്ങള്‍ സംരക്ഷിക്കുന്നതിനായി അവരുടെ ഭരണാധികാരിയാണ് യുദ്ധം ചെയ്യേണ്ടത്. നിയമനിര്‍വഹണത്തിലൂടെ ഭരിക്കുന്ന നാടിനകത്ത് അടിസ്ഥാനാവകാശങ്ങളുടെ സംരക്ഷണത്തിനാവശ്യമായ സംവിധാനമുണ്ടാക്കേണ്ട ഉത്തരവാദിത്തം ആരുടെ മേലാണോ ബാധ്യതയായിട്ടുള്ളത് അവരുടെമേല്‍തന്നെയാണ് അയല്‍നാട്ടില്‍നിന്ന് ഈ അവകാശങ്ങള്‍ ചോദ്യം ചെയ്യപ്പെടുമ്പോള്‍ അത് നീക്കുവാനാവശ്യമായ കാര്യങ്ങള്‍ നിര്‍വഹിക്കേണ്ട ഉത്തരവാദിത്തവുമുള്ളത്. അതിന് സമാധാനസന്ധിയാണ് ആവശ്യമെങ്കില്‍ അതും സായുധസമരം അനിവാര്യമാണെങ്കില്‍ അതും ചെയ്യേണ്ടത് ഭരണാധികാരിയാണ്. യുദ്ധമാണ് വേണ്ടതെന്ന് അയാള്‍ തീരുമാനിക്കുകയും സായുധ ജിഹാദിന് സന്നദ്ധമാകുവാന്‍ പൗരന്‍മാരോട് ആവശ്യപ്പെടുകയും ചെയ്താല്‍ അതിന് സന്നദ്ധമാവുകയാണ്  അതിന് സാധിക്കുന്നവരുടെ ബാധ്യത. ഭരണാധികാരി  നയിക്കുന്നതോ ആഹ്വാനം നടത്തുകയോ ചെയ്യുന്ന യുദ്ധത്തില്‍ അണിചേരുക മാത്രമാണ് പൗരന്‍മാര്‍ ചെയ്യുന്നത് എന്നര്‍ഥം.

സാമ്രാജ്യത്വ സംസ്ഥാപനത്തിനും അധീശത്വത്തിനും അങ്ങനെയുള്ള സുഖലോലുപമായ ജീവിതത്തിനും വേണ്ടിയുള്ള യുദ്ധമെന്ന സങ്കല്പത്തെ മാറ്റിയെഴുതിയ മുഹമ്മദ് നബി(സ), മാനവികതക്ക് ‌ വേണ്ടി എങ്ങനെ യുദ്ധം ചെയ്യാമെന്നാണ് ലോകത്തെ പഠിപ്പിച്ചത്. അടിച്ചമർത്തപ്പെട്ടവയുടെ വിമോചനത്തിനും ആദർശമനുസരിച്ച് ജീവിക്കുവാനുള്ള അവകാശത്തിനും വേണ്ടിയുള്ളതാണ് ഇസ്‌ലാം പഠിപ്പിക്കുന്ന യുദ്ധം. മനുഷ്യാവകാശങ്ങളുടെ സംസ്ഥാപനത്തിന് വേണ്ടി നടത്തുന്ന, ഇസ്‌ലാമിലെ യുദ്ധസങ്കൽപം പൂർണമായും മാനവികമാണെന്ന് പറയുന്നത് അത് കൊണ്ടാണ്.

മുസ്ലിംകളുടെ യുദ്ധവെറിയല്ല, ശത്രുക്കളുടെ ഇസ്ലാം വിരോധത്തിന്റെ ക്രൂരമുഖമാണ് കുരിശുയുദ്ധങ്ങളിലൂടെ ലോകം കണ്ടത്. ഇസ്‌ലാമിനെയും മുസ്ലിംകളെയും ഉന്മൂലനം ചെയ്യുകയെന്ന ലക്ഷ്യത്തോടോടെ ആയുധമെടുത്തവരെപ്പോലും മാനവികതയെന്തെന്ന് പഠിപ്പിക്കുവാൻ മുസ്ലിംകൾക്ക് ലഭിച്ച അവസരമായിരുന്നു കുരിശുയുദ്ധങ്ങൾ എന്ന് പറയുന്നതാവും ശരി. മതത്തിന്റെ പേരിൽ  ഏറ്റവും വലിയ നരനായാട്ട് നടന്നത് കുരിശു യുദ്ധങ്ങളോട് അനുബന്ധിച്ചായിരുന്നവെന്ന് സമ്മതിക്കുമ്പോൾ തന്നെ, അവയുടെ കാരണം തികച്ചും മതപരമായിരുന്നില്ലെന്ന വസ്തുത കൂടി മനസ്സിലാക്കേണ്ടതുണ്ട്. തകർന്നുകൊണ്ടിരുന്ന ബൈസന്റൈൻ സാമ്രാജ്യം മതത്തെ ഉപയോഗപ്പെടുത്തി തങ്ങളുടെ അധികാരം സംരക്ഷിക്കാൻ നടത്തിയ പരിശ്രമമായിരുന്നു കുരിശുയുദ്ധങ്ങളിൽ കലാശിച്ചത്. ബൈസന്റൈൻ സാമ്രാജ്യത്തെ സംരക്ഷിക്കേണ്ടത് തങ്ങളുടെ ഉതതരവാദിത്തമായിക്കരുത്തിയ കത്തോലിക്കാ സഭ മുസ്ലിംകൾക്കും മുസ്ലിം നാടുകൾക്കുമെതിരെ കൊലവിളി നടത്താൻ ക്രിസ്ത്യാനികളെ പ്രചോദിപ്പിക്കുകയായിരുന്നു.

ബൈസന്റൈന്‍ ചക്രവര്‍ത്തിയായിരുന്ന അലക്‌സിയോസ് ഒന്നാമന്‍, അന്നത്തെ മാര്‍പാപ്പയായിരുന്ന പോപ്പ് അര്‍ബന്‍ രണ്ടാമന് തന്നെ സഹായിക്കണമെന്ന് ആവശ്യപ്പെട്ട് എഴുതിയ കത്തില്‍ നിന്ന് തുടങ്ങുന്നു കുരിശുയുദ്ധങ്ങളുടെ ചരിത്രം. 1071 ആഗസ്ത് 26ന് നടന്ന മാന്‍സികെര്‍ട്ട് യുദ്ധത്തില്‍ സില്‍ജൂക്ക് തുര്‍ക്കികള്‍ ബൈസന്റയിന്‍ സൈന്യത്തെ തുരത്തുകയും അനത്തോളിയ, അര്‍മേനിയ തുടങ്ങിയ പ്രദേശങ്ങള്‍ പിടിച്ചെടുക്കുകയും ചെയ്തതിന്റെ പശ്ചാത്തലത്തിലുള്ളതായിരുന്നു പ്രസ്തുത കത്ത്. 1095 നവംബര്‍ 18 മുതല്‍ 28 വരെ ഫ്രാന്‍സിലെ ക്ലെര്‍മണ്ടില്‍ വെച്ചു നടന്ന സഭാധ്യക്ഷന്‍മാരുടെ സമ്മേളനത്തില്‍, 27ാം തിയതി പോപ്പ് തന്നെ സമ്മേളനത്തെ അഭിസംബോധന ചെയ്തുകൊണ്ട് മുസ്‌ലിംകള്‍ക്കെതിരെ നടത്തേണ്ട യുദ്ധത്തെക്കുറിച്ച് സംസാരിക്കുകയും സദസ്സിനെ ആവേശഭരിതരാക്കി പ്രചോദിപ്പിക്കുകയും ചെയ്തു. ക്രൈസ്തവരുടെ വിശുദ്ധ സ്ഥലങ്ങള്‍ ഭരിക്കുന്ന അറബികളും തുര്‍ക്കികളുമാകുന്ന മുസ്‌ലിംകള്‍ പ്രാകൃതരാണെന്നും അതുകൊണ്ടു തന്നെ അവരെ തുരത്തേണ്ടത് ലോകത്തെങ്ങുമുള്ള ക്രൈസ്തവരുടെ ഉത്തരവാദിത്തമാണെന്നും ഇത് ക്രിസ്തുവിന്റെ കല്‍പനയാണെന്നും പ്രാകൃതരായ മുസ്‌ലിംകള്‍ക്കെതിരെയുള്ള യുദ്ധത്തില്‍ മരണപ്പെടുന്നവര്‍ക്ക് ഉടനടി പാപമോചനം ലഭിക്കുമെന്നും പിശാചുക്കളെ (demons) ആരാധിക്കുന്നവരുടെ മേല്‍ സര്‍വശക്തനായ ദൈവെത്ത പൂജിക്കുന്നവരുടെ ആധിപത്യത്തിനു വേണ്ടിയുള്ള നമ്മുടെ യുദ്ധത്തില്‍ ദൈവസഹായമുണ്ടാകുമെന്നും സഹോദരങ്ങള്‍ക്കും ബന്ധുക്കള്‍ക്കുമെതിരെ യുദ്ധം ചെയ്തിരുന്നവരെല്ലാം അവ നിര്‍ത്തി കാടന്‍മാര്‍ക്കെതിരെ(barbarians)യുള്ള ഈ യുദ്ധത്തിനു വേണ്ടി ഐക്യപ്പെടണമെന്നും മാര്‍പാപ്പ ആഹ്വാനം ചെയ്തു.(1)

മാര്‍പാപ്പയുടെ പ്രസംഗത്തില്‍ നിന്ന് പ്രചോദനമുള്‍ക്കൊണ്ട് 1096 ആഗസ്ത് 15ന് ഫ്രാന്‍സില്‍ നിന്നും ജര്‍മനിയില്‍ നിന്നും ഇറ്റലിയില്‍ നിന്നുമായി സന്യാസിയായ പത്രോസിന്റെ (Peter the hermit) നേതൃത്വത്തില്‍ 35000ത്തോളം വരുന്ന വലിയൊരു ക്രൈസ്തവസംഘം കോണ്‍സ്റ്റന്റിനോപ്പിളിലേക്ക് മാര്‍ച്ച് ചെയ്തു. വഴിയില്‍ കണ്ടവരെയെല്ലാം കൊന്നൊടുക്കിയും സാധാരണക്കാരെ ക്രൂരമായി പീഡിപ്പിച്ചും കൊണ്ടാണ് കുരിശുപടയാളികള്‍ മുന്നോട്ടു നീങ്ങിയത്. മൈല്‍ഗ്രേഡില്‍ വെച്ച് ഹംഗറിക്കാരുമായി അവര്‍ ഏറ്റുമുട്ടി; സ്‌പ്രെയറിലും മോംസിലും മെയിന്‍സിലുമുള്ള ജൂതന്‍മാരെ കൂട്ടക്കൊല നടത്തി. 1092ലെ മലിക്ഷാ ഒന്നാമന്റെ മരണത്തിന് ശേഷം ശിഥിലമായ സില്‍ജൂക്ക് തുര്‍ക്കി സാമ്രാജ്യത്തിന്റെ പതിതാവസ്ഥയെ മുതലെടുത്ത് കുരിശ് യോദ്ധാക്കള്‍ മുസ്‌ലിം നാടുകളിലേക്ക് ഇരച്ചു കയറുകയും അവിടങ്ങളില്‍ ക്രൂരതകൊണ്ട് സംഹാരതാണ്ഡവമാടുകയും ചെയ്തു. ക്രിസ്താബ്ദം 638ല്‍ ഉമറുല്‍ ഫാറൂഖിന്റെ ഭരണകാലം മുതല്‍ മുസ്‌ലിംകള്‍ ഭരിച്ചിരുന്ന ജറൂസലേം അങ്ങനെ 1099 ജൂലൈ 15ന്, ഫാത്വമിയ്യാക്കളുടെ കൈകളില്‍ നിന്ന് കുരിശുയോദ്ധാക്കള്‍ പിടിച്ചടക്കി.(2)

ഒന്നാമത്തെ കുരിശുയുദ്ധത്തിനു മുതല്‍ കൂട്ടക്കൊലകളുടെയും ക്രൂരതകളുടെയും ചരിത്രമാണ് പറയാനുള്ളത്. പടയാളികളെ മാത്രമായിരുന്നില്ല കുരിശുയോദ്ധാക്കള്‍ കൊന്നൊടുക്കിയത്; പ്രത്യുത സാധാരണക്കാരെക്കൂടിയായിരുന്നു. കുരിശുയുദ്ധത്തിനു മുമ്പ് ജറൂസലേം നഗരത്തിന്റെ ജനസംഖ്യ എഴുപതിനായിരമായിരുന്നെങ്കില്‍ യുദ്ധം കാരണത്താല്‍ അത് മുപ്പതിനായിരമായി ചുരുങ്ങിയെന്ന് ചരിത്രകാരന്‍മാര്‍ രേഖപ്പെടുത്തുന്നുണ്ട്.(3) നഗരത്തിലെ പകുതിയിലധികം പേരെയും കുരിശുയോദ്ധാക്കള്‍ കൊന്നുവെന്നര്‍ഥം.

മസ്ജിദുല്‍ അഖ്‌സയുടെ ഉള്ളില്‍ അഭയം തേടിയവരെയൊ സ്ത്രീകളേയൊ കുട്ടികളെയൊ അവര്‍ വെറുതെ വിട്ടില്ല. കണ്ണില്‍ കണ്ട എല്ലാവരെയും അവര്‍ കൊന്നൊടുക്കി. സിനഗോഗുകള്‍ക്കകത്ത് ജൂതന്മാര്‍ ഒളിച്ചപ്പോള്‍ അവരെ അതിനുള്ളിലിട്ട് പൂട്ടി അവര്‍ സിനഗോഗുകള്‍ക്ക് തീ വെച്ചു. യുദ്ധത്തില്‍ ബന്ദികളായി പിടിച്ചവരെയെല്ലാം കുരിശുദ്ധോക്കള്‍ കൂട്ടക്കൊല ചെയ്തു. തലകളുടെയും കൈകളുടെയും കാലുകളുടെയും  കൂമ്പാരങ്ങളായിരുന്നു ജെറുസെലേം നഗരത്തിലെന്നും അവയ്ക്കിടയിലൂടെ ഒരു പരവതാനിയില്‍ എന്നതുപോലെ നടന്നുല്ലസിക്കുകയായിരുന്നു കുരിശുദ്ധോക്കളെന്നും ''നമുക്ക് വേണ്ടി കര്‍ത്താവ് ഈ ദിവസം ഒരുക്കിത്തന്നിരിക്കുന്നത് ആഹ്ലാദിക്കുവാനും സന്തോഷം പങ്കിടാനുമാണ്'' എന്നു പ്രഖ്യാപിച്ച കുരിശുയോദ്ധാവായ അഗ്വിലേഴ്‌സ്‌കാരന്‍ റെയ്മണ്ട് രേഖപ്പെടുത്തുന്നുണ്ട്.(4) കുരിശു യുദ്ധത്തെക്കുറിച്ച് എഴുതപ്പെട്ട ലത്തീന്‍ ദിനവൃത്താന്തമായ ജെസ്റ്റ ഫ്രാന്‍ കോറം (Gesta Fran corum- ഫ്രാങ്കകളുടെ ചെയ്തികള്‍)യിലെ വരികള്‍ കാണുക: ''നമ്മുടെയാളുകള്‍ സോളമന്റെ ദേവാലയത്തിനടുത്തുവെച്ചുപോലും ആളുകളെ വെറുതെ വിട്ടില്ല. അവരെയെല്ലാം കൂട്ടക്കൊല ചെയ്തു. നെരിയാണിവരെ മുങ്ങുന്ന രക്തപ്പുഴയിലൂടെ നീന്തിയാണ് അവരിതെല്ലാം ചെയ്തത്. പ്രാകൃതന്‍മാരെ കീഴടക്കിയതിനു ശേഷം നമ്മുടെ ആള്‍ക്കാര്‍ അവരിലെ സ്ത്രീകളെയും കുട്ടികളെയുമടക്കം എല്ലാവരെയും ബന്ദികളാക്കി. ഓരോരുത്തരും ഇഷ്ടപ്രകാരം ബന്ദികളെ കൊല്ലുകയോ അടിമകളാക്കിത്തീര്‍ക്കുകയോ ചെയ്തു''.(5)

സോളമന്റെ ദേവാലയത്തില്‍ വെച്ച് നടന്ന കൂട്ടക്കൊലയെപ്പറ്റി ചര്‍ട്രസുകാരനായ ഫുള്‍ച്ചര്‍ വിവരിക്കുന്നത് ഇങ്ങനെയാണ്: ''ഈ ദേവാലയത്തില്‍വെച്ച് മാത്രം പതിനായിരം പേര്‍ കൊല്ലപ്പെട്ടു. അപ്പോള്‍ നിങ്ങള്‍ ഞങ്ങളെ കണ്ടിരുന്നെങ്കില്‍ കൊല്ലപ്പെട്ടവരുടെ ചോരയാല്‍ ഞങ്ങളുടെയെല്ലാം കണങ്കാലുകള്‍ ചുവന്നത് നിങ്ങള്‍ക്ക് കാണാമായിരുന്നു. സ്ത്രീകളെയും കുട്ടികളെയുമടക്കം ഒരാളെയും ഞങ്ങള്‍ വെറുതെ വിട്ടില്ല''.(6)

ദേവാലയത്തിലും സോളമന്റെ വരാന്തയിലുംകൂടി അവരുടെ മുട്ടുവരെ നിറഞ്ഞ ചോരയിലൂടെ ചോരപൂരണ്ട കടിഞ്ഞാണ്‍ പിടിച്ച് ജനം സവാരി നടത്തി(7)യെന്ന കുരിശുയോദ്ധാവിന്റെ ദൃക്‌സാക്ഷ്യം സംഭവത്തിന്റെ ഭീകരത വ്യക്തമാക്കുന്നുണ്ട്. സിനഗോഗുകള്‍ക്കകത്ത് അഭയം പ്രാപിച്ച ജൂതന്‍മാരെ അതിനുള്ളില്‍ പൂട്ടിയിട്ട് അതിന് തീക്കൊളുത്തിയശേഷം അഗ്നിസ്ഫുലിംഗങ്ങള്‍ ഉയര്‍ന്നു പൊങ്ങിയപ്പോള്‍ അതിനുചുറ്റും ആനന്ദനൃത്തമാടിക്കൊണ്ട് കുരിശുയോദ്ധാക്കള്‍ പാടി. 'ക്രിസ്തുവേ... നിന്നെ ഞങ്ങള്‍ ആരാധിക്കുന്നു; നീയാണ് ഞങ്ങളുടെ വെളിച്ചം. നീയാണ് ഞങ്ങളുടെ ദിശ; നീയാണ് ഞങ്ങളുടെ സ്‌നേഹം.'(8)

ഒന്നാമത്തെ കുരിശുയുദ്ധത്തോടെ മുസ്‌ലിം ലോകത്തിെന്റ അന്നത്തെ ദൗര്‍ബല്യം മനസ്സിലാക്കിയ കുരിശു പടയാളികള്‍ അര്‍ബന്‍ രണ്ടാമന്റെ നിര്‍ദേശം പാലിച്ചുകൊണ്ട് മുസ്‌ലിം ലോകത്ത് നടത്തിയ ക്രൂരതകളുടെ ഭീതിപ്പെടുത്തുന്ന കഥകളാണ് ഓരോ കുരിശു യുദ്ധത്തിനും പറയാനുള്ളത്. നിയമങ്ങളോ നീതിയോ ധാര്‍മികതയോയില്ലാത്ത കാടന്‍ യുദ്ധങ്ങളാണ് കുരിശുയുദ്ധക്കാര്‍ നടത്തിയത്. ഇവയാല്‍ ലക്ഷക്കണക്കിന് മനുഷ്യര്‍ കൊല്ലപ്പെട്ടു. മരിച്ചവരുടെ ശവശരീരങ്ങളെ കുരിശു പടയാളികള്‍ വികൃതമാക്കി. എട്ടും പൊട്ടും തിരിച്ചറിയാത്ത പൈതങ്ങളെപ്പോലും കൊല്ലുന്നതില്‍ നിന്ന് അവരെ പിന്‍തിരിപ്പിക്കുവാന്‍ 'ഒരു മുഖത്തടിച്ചാല്‍ മറ്റേ മുഖവും കാണിച്ചു കൊടുക്കണം' എന്ന ബൈബിള്‍ പാഠത്തിെന്റ അടിസ്ഥാനത്തില്‍ ജനങ്ങളെ നയിക്കുന്ന വരെന്ന് അവകാശപ്പെടുന്ന സഭാനേതൃത്വം സന്നദ്ധമായില്ല. ക്രിസ്തുവിന്റെ പേരില്‍ ആരോപിക്കപ്പെട്ട, അദ്ദേഹത്തിന്റേതല്ലാത്ത ഉപദേശം അപ്രായോഗികവും അക്രമികളെ സഹായിക്കുന്നതുമാണെന്ന വസ്തുതക്ക് സഭാനേതൃത്വം തന്നെ സ്വയം തെളിവായിത്തീരുകയായിരുന്നു എന്നു പറയുന്നതാവും ശരി. അതല്ലായിരുന്നുവെങ്കില്‍ ഈ കുരിശുയുദ്ധങ്ങളുടെ പേരില്‍ ഏഴു നൂറ്റാണ്ടുകള്‍ക്കു ശേഷം ലോകത്തെ പരമോന്നത ക്രിസ്ത്യാനിക്ക് കുമ്പസരിക്കേണ്ടി വരികയില്ലായിരുന്നല്ലോ!

ആയുധങ്ങളെടുത്ത് ആളുകളെ കൂട്ടക്കൊല ചെയ്യുക മാത്രമാണ് യുദ്ധമെന്ന് പഠിച്ച കുരിശുയോദ്ധാക്കള്‍ക്ക് മാന്യമായി എങ്ങെന യുദ്ധം ചെയ്യണമെന്ന് പഠിപ്പിച്ചത് സുല്‍ത്താന്‍ സ്വലാഹുദ്ദീന്‍ അയ്യൂബിയായിരുന്നു. നീണ്ട രണ്ടു നൂറ്റാണ്ടു കാലത്തെ കുരിശുയോദ്ധാക്കളുടെ ദുര്‍ഭരണത്തില്‍ നിന്ന് മസ്ജിദുല്‍ അഖ്‌സയെയും ജറൂസലേമിനെയും മോചിപ്പിച്ചത് അദ്ദേഹത്തിന്റെ കീഴിലുള്ള അബ്ബാസിയ്യാ മുസ്‌ലിം പടയാളികളായിരുന്നു. 1187 ജൂലൈ 4 ന് ശനിയാഴ്ച നടന്ന ഹത്തീന്‍ യുദ്ധത്തില്‍ കുരിശു പടയാളികള്‍ പരാജയപ്പെടുകയും ജറൂസലേമിന്റെ അധികാരം സ്വലാഹുദ്ദീന്റെ കൈകളില്‍ വന്നു ചേരുകയും ചെയ്തു. കുരിശുയോദ്ധാക്കളുടെ നേതാക്കളായ ഗൈ രാജാവും സഹോദരങ്ങളുമടക്കം നിരവധിപേരെ ബന്ധികളാക്കി സ്വലാഹുദ്ദീനു മുന്നില്‍ ഹാജരാക്കപ്പെട്ടു. അവര്‍ക്കെല്ലാം തണുത്ത പാനീയം നല്‍കി സ്വീകരിക്കുവാനാണ് സ്വലാഹുദ്ദീന്‍ തന്റെ സേവകന്‍മാരോട് ആവശ്യപ്പെട്ടത്. സ്വലാഹുദ്ദീനെയും മുസ്‌ലിം പടയാളികളെയും വഞ്ചിച്ചതിന് രാജസഹോദരനായ റെയ്‌നാള്‍ഡും മറ്റു ചിലരും മാത്രമാണ് വധിക്കപ്പെട്ടത്. ഗൈ രാജാവടക്കമുള്ളവരെ ആദ്യം ജയിലിലടക്കുകയും പിന്നെ വെറുതെ വിടുകയുമാണ് ചെയ്തത്. കുരിശുയോദ്ധാക്കള്‍ ജറൂസലേമില്‍ നിന്ന് പുറത്താക്കിയ ജൂതന്‍മാരുടെ പിന്‍ഗാമികളെ വരുത്തി അവിടെ പുനരധിവസിപ്പിക്കുകയാണ് ഭരണമേറ്റെടുത്തയുടനെ സ്വലാഹുദ്ദീന്‍ ചെയ്തത്.(9) മനുഷ്യരക്തം നീന്തി രക്തദാഹം തീര്‍ക്കുകയും പൈതങ്ങളെയടക്കം കൊന്നൊടുക്കുകയും ചെയ്ത് ജറൂസലേം പിടിച്ചടക്കിയവര്‍ക്ക് അതു തിരിച്ചുപിടിച്ചശേഷം യുദ്ധനേതൃത്വത്തിലുണ്ടായിരുന്നവര്‍ക്കടക്കം മാപ്പുനല്‍കി വിട്ടയച്ചുകൊണ്ട് എന്തായിരിക്കണം യുദ്ധമര്യാദയെന്ന് പഠിപ്പിക്കുന്നതായിരുന്നു സ്വലാഹുദ്ദീന്റെ നടപടി.

സ്വലാഹുദ്ദീന്‍ അയ്യൂബി ജറൂസലേം പിടിച്ചടക്കിയതറിഞ്ഞ പോപ്പ് അര്‍ബന്‍ മൂന്നാമന്‍ മാര്‍പാപ്പ ഹൃദയാഘാതം മൂലം മരണെപ്പട്ടു.(10) ജറൂസലേം നഷ്ടപ്പെട്ടതറിഞ്ഞ ക്രിസ്ത്യന്‍ യൂറോപ്പ് ഞെട്ടി. പുതുതായി സ്ഥാനമേറ്റെടുത്ത പോപ്പ് ഗ്രിഗറി എട്ടാമന്‍ മാര്‍പാപ്പ ജറൂസലേമിന്റെ നഷ്ടം ക്രൈസ്തവരുടെ പാപങ്ങള്‍ക്കുള്ള ശിക്ഷയാണെന്നും അതു തിരിച്ചുപിടിച്ചാല്‍ മാത്രമെ പ്രസ്തുത പാപങ്ങള്‍ക്ക് പരിഹാരമാവുകയുള്ളുവെന്നും പ്രഖ്യാപിച്ചു. പരസ്പരം യുദ്ധത്തിലായിരുന്ന ഇംഗ്ലണ്ടിലെ ഹെന്‍ട്രി രണ്ടാമനും ഫ്രാന്‍സിലെ ഫിലിപ്പ് രണ്ടാമനും വൈര്യങ്ങള്‍ വെടിഞ്ഞ് ഐക്യപ്പെടാനും സ്വലാഹുദ്ദീനെതിരെ ഒരുമിച്ച് പോരാടി ജറൂസലേം വീണ്ടെടുക്കുവാനും തിരുമാനിച്ചു. സലാദിന്‍ നികുതി(Saladin tithe)എന്ന പേരില്‍ രണ്ടു നാട്ടിലുമുള്ള പൗരന്‍മാരില്‍ നിന്ന് ചുങ്കംപിരിച്ച് യുദ്ധത്തിനുള്ള സമ്പത്ത് സമാഹരിച്ചു. 1189 ജൂലൈ 6ന് ഇംഗ്ലണ്ടിലെ രാജാവായ ഹെന്‍ട്രി രണ്ടാമന്‍ മരണപ്പെട്ടതോടെ യുദ്ധനേതൃത്വം മകന്‍ സിംഹഹൃദയനായ റിച്ചാര്‍ഡ് (Richard the lion-hearted) എന്നറിയപ്പെട്ട റിച്ചാര്‍ഡ് ഒന്നാമന്റെ ചുമതലയായിത്തീര്‍ന്നു. റിച്ചാര്‍ഡ് ഒന്നാമന്റെയും ഫിലിപ്പ് രണ്ടാമന്റെയും നേതൃത്വത്തിലുള്ള കുരിശുയോദ്ധാക്കളുടെ സൈന്യത്തോട് സ്വലാഹുദ്ദീന്‍ ജയില്‍ മോചിതനാക്കിയ ഗൈരാജാവും ഒന്നിക്കുകയും ഒരു വലിയ സൈന്യം സ്വലാഹുദ്ദീനെ ആക്രമിക്കാനെത്തുകയും ചെയ്തു. വഴിയില്‍വെച്ചു നടന്ന അക്രമങ്ങളിലൂടെ സ്ത്രീകളും കുട്ടികളുമടക്കമുള്ള മുസ്‌ലിംകളെ കൊന്നുകൂട്ടിക്കൊണ്ടായിരുന്നു കുരിശ് സൈന്യം മുന്നേറിയത്. 1191 സെപ്റ്റംബര്‍ 7ന് ജറൂസലേം പിടിച്ചടക്കുകയെന്ന ലക്ഷ്യത്തോട സിംഹഹൃദയനായ റിച്ചാര്‍ഡിന്റെ നേതൃത്വത്തിലുള്ള സൈന്യം ഫലസ്തിനിലുള്ള അര്‍സൂഫില്‍ വെച്ച് സ്വലാഹുദ്ദീനോട് ഏറ്റുമുട്ടിയെങ്കിലും അദ്ദേഹത്തിന് ലക്ഷ്യംനേടാന്‍ കഴിഞ്ഞില്ല. നിരന്തമായി നടന്ന യുദ്ധങ്ങള്‍ക്ക് വിരാമമുണ്ടായത് 1192 സെപ്റ്റംബര്‍ 2ന് സ്വലാഹുദ്ദീനും റിച്ചാര്‍ഡും തമ്മിലുണ്ടാക്കിയ സന്ധിയോടു കൂടിയാണ്. ജറൂസലേം മുസ്‌ലിംകളുടെ നിയന്ത്രണത്തിലായിരിക്കുമെന്നും എന്നാല്‍ നിരായുധരായ ക്രൈസ്തവരെ അവിടെയുള്ള ക്രിസ്ത്യന്‍ പുണ്യസ്ഥലങ്ങള്‍ സന്ദര്‍ശിക്കാന്‍ അനുവദിക്കുമെന്നുമായിരുന്നു കരാര്‍.(11) പുണ്യസ്ഥലങ്ങള്‍ സന്ദര്‍ശിക്കുന്നതില്‍ നിന്ന് മുമ്പും സ്വലാഹുദ്ദീന്‍ ക്രൈസ്തവെര വിലക്കിയിട്ടില്ല എന്നതുകൊണ്ടുതന്നെ കരാര്‍ വഴി കുരിശുയോദ്ധാക്കള്‍ക്ക് ഒന്നും നേടാനായില്ലെങ്കിലും, 1192 ഒക്‌ടോബര്‍ 9ന് റിച്ചാര്‍ഡ് ജറൂസലേമില്‍ നിന്ന് മടങ്ങിയത് യുദ്ധരംഗത്ത് പാലിക്കപ്പെടേണ്ട മര്യാദകളെന്തൊക്കെയാണെന്ന് പഠിച്ചുകൊണ്ടായിരുന്നു. ശത്രുവിനെ സ്‌നേഹിക്കേണ്ടത് എങ്ങനെയാണെന്ന് സഭയില്‍ നിന്ന് പഠിക്കാത്ത അദ്ദേഹത്തിന് അത് പഠിപ്പിച്ചത് സ്വലാഹുദ്ദീന്‍ അയ്യൂബിയുടെ യുദ്ധരംഗത്തെ വര്‍ത്തനങ്ങളായിരുന്നു.

യുദ്ധരംഗത്തായിരിക്കുമ്പോള്‍ പോലും ശത്രുവിന്റെ മാനുഷികമായ ആവശ്യങ്ങള്‍ നിര്‍വഹിച്ചു കൊടുക്കുവാന്‍ സ്വലാഹുദ്ദീന്‍ അയ്യൂബി സന്നദ്ധമായി. റിച്ചാര്‍ഡിന് പനിപിടിച്ചപ്പോള്‍ തന്റെ വൈദ്യനെ അയച്ച് ചികില്‍സിക്കുകയും മരുന്ന് നല്‍കുകയും നല്ല പഴങ്ങളും ശുദ്ധമായ തണുത്ത ജലവും കൊടുത്തയക്കുകയും ചെയ്തത് സ്വലാഹുദ്ദീനായിരുന്നു. യുദ്ധത്തിനിടയില്‍ റിച്ചാര്‍ഡിന് കുതിര നഷ്ടപ്പെട്ടപ്പോള്‍ പകരം രണ്ടു കുതിരകളെ നല്‍കിയതും അദ്ദേഹം തന്നെ. അതുകൊണ്ടു തന്നെ കുരിശുയോദ്ധാക്കള്‍ക്കിടയില്‍ പോലും  സ്വലാഹുദ്ദീന്‍ നായകനായിത്തീര്‍ന്നു. തന്റെ സേഹാദരിയും സിസിലിയിലെ രാജ്ഞിയുമായ ജോനിനെ സ്വലാഹുദ്ദീന്റെ സഹോദരന് വിവാഹം ചെയ്തു കൊടുക്കുവാന്‍ സന്നദ്ധമാണെന്ന് അറിയിച്ചുകൊണ്ടാണ് അദ്ദേഹം ജറൂസലേം വിട്ടത്.(12) കാടനായി വന്നയാളെ മാന്യനായിത്തീര്‍ത്ത് തിരിച്ചയച്ചത് ശത്രുവിനെ സ്‌നേഹിക്കണമെന്ന പ്രവാചകന്‍മാരുടെ നിര്‍ദേശം യഥാവിധി പാലിക്കുവാന്‍ സ്വലാഹുദ്ദീന്‍ അയ്യൂബി തയ്യാറായതു കൊണ്ടായിരുന്നു.

ജീവിതത്തിന്റെ മറ്റു രംഗങ്ങളിലേതു പോലെത്തന്നെ യുദ്ധരംഗത്തും പ്രവാചകന്‍മാര്‍ പഠിപ്പിച്ച സമാധാന സന്ദര്‍ഭത്തിലും യുദ്ധരംഗത്തുമെല്ലാം പാലിക്കേണ്ട നിയമങ്ങളെന്തൊക്കെയാണെന്ന് അദ്ദേഹം പഠിപ്പിച്ചു തരികയും പ്രായോഗികമാക്കി കാണിച്ചുതരികയും ചെയ്തിട്ടുണ്ട്. പീഡനങ്ങള്‍ സഹിച്ച് പ്രബോധനം ചെയ്യുമ്പോയും ആദര്‍ശജീവിതത്തിന് നിവൃത്തിയില്ലാതെ പാലായനം ചെയ്യുമ്പോഴും വിവിത സമുദായങ്ങള്‍ ജീവിക്കുന്ന നാട് ഭരിക്കേണ്ടിവരുമ്പോഴും ഭരണാധികാരിയായി ശത്രുരാജ്യങ്ങളോട് യുദ്ധം ചെയ്യുമ്പോഴും യുദ്ധത്തില്‍ ജയിക്കുമ്പോയും ശത്രുരാജ്യങ്ങളോട് സന്ധിയുണ്ടാക്കുമ്പോഴും ശത്രുരാജ്യത്തിനുമേല്‍ അധിശത്വം ലഭിക്കുമ്പോഴുമെല്ലാം എങ്ങനെ പെരുമാറണമെന്ന് അദ്ദേഹം കൃത്യമായി പഠിപ്പിച്ചിട്ടുണ്ട് യുദ്ധരംഗവും സമാധാനരംഗവുമെല്ലാം മാനവവല്‍ക്കരിക്കുവാന്‍ ആ ജീവിതം തന്നെയാണ് മനുഷ്യര്‍ മാതൃകയാക്കേണ്ടത്. എല്ലാ രംഗത്തുമുള്ള സല്‍സ്വഭാവങ്ങളുടെ പൂര്‍ത്തീകരണത്തിനു വേണ്ടിയാണല്ലോ നബിനിയോഗമുണ്ടായിട്ടുള്ളത്. ശാന്തിക്കും സമാധാനത്തിനും വേണ്ടി പ്രവാചകന്മാരെല്ലാം പഠിപ്പിച്ച പാതയിലേക്കുള്ള തിരിച്ചുപോക്കാണ് വേണ്ടതെന്നും അതുവഴി മാത്രമെ ശത്രുതയുടെ മാറ്റുരയ്ക്കപ്പെടുന്ന രണഭൂമിയെപ്പോലും മാനവവല്‍ക്കരിക്കാന്‍ കഴിയുകയുള്ളൂവെന്നുമാണ് കുരിശുയുദ്ധഭൂമിയിൽ വെച്ച് മുസ്ലിംകൾ ശത്രുക്കളെ പഠിപ്പിച്ചത്.

  1. Medieval Source book: Urban II (1085 - 1099) Speech at Counsil of Clerm-ont, 1095 (www.fordham-edu/halsall/source/urban 2-5 verntml.
  2. Thomas Asbridge: The Crusades. The Authoritative History of the war for the Holy Land, Chicago, 2010.
  3. Michael D Hull: First Crusade: Siege of Jerusalem, June 1999, Military History Magazine (www..historynet.com)
  4. Ibid
  5. Gesta Francorum” (fodham.edu/halsall/source/gesta-ede-asp
  6. Fulcher of chartresa: The siege of the city of Jerusalem; Gesta francorum (fordham.edu)
  7. Benjamin 2 Kedar & Jonathan SC: In the Crusades, New York 2004, Vol 3, page 65
  8. David A Rausch: A Legacy of Hatred: Why Christians Must not forget the Holocaust, Chicago, 1984, Page 63.
  9. Crusades’ Encyclopaedia Brittanica, Online edition, brittanica.com
  10. Pope Urban III”, Encyclopaedia of Keywords, Keg 1(keywen.com)
  11. Thomas Madden: The New Concise History of the Crusades, Mary Land, 2006, Page 90-98
  12. James Reston Jr: Richard the Lionheart and Saladin in the third crusade, Harpswell, 2002 Page 358-378
ല്ല. ലോകം മുഴുവൻ ഇസ്‌ലാമീകരിക്കുക എന്ന ഒരു ലക്‌ഷ്യം പ്രവാചകൻ ഉണ്ടായിരുന്നതായി വ്യക്തമാക്കുന്ന ആയത്തുകളോ ഹദീഥുകളോ ഒന്നുമില്ല.

യുദ്ധത്തിലൂടെയോ നിര്‍ബന്ധമോ ബലാല്‍ക്കാരമോ ചെലുത്തിക്കൊണ്ടോ സ്വീകരിക്കപ്പെടേണ്ടതല്ല മതവിശ്വാസമെന്നും സത്യവും അസത്യവും വിവേചിച്ച് മനസ്സിലാക്കിക്കൊടുക്കുകയാണ് പ്രബോധകനെന്ന നിലയില്‍ പ്രവാചകന്റെ ബാധ്യതയെന്നും അദ്ദേഹത്തിന് സ്വന്തമായി ആരെയും സന്മാര്‍ഗത്തിലാക്കുവാന്‍ സാധ്യമല്ലെന്നും സത്യവിശ്വാസം സ്വീകരിക്കുവാന്‍ മനസ്സ് പാകപ്പെട്ടവരെ അതിലേക്ക് നയിക്കുന്നത് അല്ലാഹുവാണെന്നും വ്യക്തമാക്കുന്ന ഗ്രൻഥമാണ് ക്വുര്‍ആൻ. പ്രവാചകജീവിതത്തിലെപ്പോഴെങ്കിലും ആരെയെങ്കിലും നിർബന്ധിച്ച് ഇസ്‌ലാം സ്വീകരിപ്പിച്ചതായി വ്യക്തമാക്കുന്ന രേഖകളൊന്നും തന്നെയില്ല. പിന്നെയെങ്ങനെയാണ് യുദ്ധത്തിലൂടെ ലോകം മുഴുവന്‍ ഇസ്‌ലാം സ്വീകരിക്കുന്ന അവസ്ഥയുണ്ടാക്കുകയാണ് മുസ്‌ലിംകളുടെ ധര്‍മമെന്ന് പറയാനാവുക ?! ലോകത്തുള്ള മനുഷ്യരെല്ലാം സത്യസന്ദേശം സ്വീകരിക്കുന്ന സ്ഥിതി ഉണ്ടാവുകയില്ലെന്ന് വ്യക്തമാക്കുന്ന ക്വുര്‍ആനെങ്ങനെ അത്തരമൊരു സ്ഥിതി സംജാതമാക്കുന്നതിന് യുദ്ധത്തിലേര്‍പ്പെടണമെന്ന് അതിന്റെ അനുയായികളെ ആഹ്വാനം ചെയ്യാന്‍ കഴിയും? ഒരിക്കലും ഉണ്ടാവുകയില്ലെന്നുറപ്പുള്ള സാമൂഹ്യാവസ്ഥയുടെ സൃഷ്ടിയെ യുദ്ധലക്ഷ്യമായി നിര്‍ണയിക്കുന്ന അന്ധമായ ദര്‍ശനമല്ല ഇസ്‌ലാമെന്ന് അതിന്റെ അടിസ്ഥാനാദര്‍ശങ്ങളെങ്കിലും പഠിച്ചാല്‍ ആര്‍ക്കും മനസ്സിലാക്കാവുന്നതാണ്. ലോകം മുഴുവന്‍ ഇസ്‌ലാമീകരിക്കുകയാണ് ഇസ്‌ലാമിന്റെ യുദ്ധലക്ഷ്യമെന്ന ആരോപണം അടിസ്ഥാനരഹിതമാണെന്നര്‍ഥം.

മതവിശ്വാസത്തിന്റെ സ്വീകരണത്തിന്റെയും തിരസ്‌കരണത്തിന്റെയും മാനദണ്ഡമെന്തായിരിക്കണമെന്ന് വ്യക്തമായി പഠിപ്പിക്കുന്ന ഗ്രന്ഥമാണ് ക്വുര്‍ആന്‍. ഇവ്വിഷയമായി ഏറെ പ്രസിദ്ധമായ ക്വുര്‍ആന്‍ സൂക്തങ്ങളുടെ സാരം ഇങ്ങനെയാണ്: ''മതത്തിന്റെ കാര്യത്തില്‍ ബലപ്രയോഗമേ ഇല്ല. സന്‍മാര്‍ഗം ദുര്‍മാര്‍ഗത്തില്‍ നിന്ന് വ്യക്തമായി വേര്‍തിരിഞ്ഞ് കഴിഞ്ഞിരിക്കുന്നു. ആകയാല്‍ ഏതൊരാള്‍ ദുര്‍മൂര്‍ത്തികളെ അവിശ്വസിക്കുകയും അല്ലാഹുവില്‍ വിശ്വസിക്കുകയും ചെയ്യുന്നുവോ അവന്‍ പിടിച്ചിട്ടുള്ളത് ബലമുള്ള ഒരു കയറിലാകുന്നു. അത് പൊട്ടി പോകുകയേ ഇല്ല. അല്ലാഹു (എല്ലാം) കേള്‍ക്കുന്നവനും അറിയുന്നവനുമാകുന്നു. വിശ്വസിച്ചവരുടെ രക്ഷാധികാരിയാകുന്നു അല്ലാഹു. അവന്‍ അവരെ ഇരുട്ടുകളില്‍ നിന്ന് വെളിച്ചത്തിലേക്ക് കൊണ്ടു വരുന്നു. എന്നാല്‍ സത്യനിഷേധികളുടെ രക്ഷാധികാരികള്‍ ദുര്‍മൂര്‍ത്തികളാകുന്നു. വെളിച്ചത്തില്‍ നിന്ന് ഇരുട്ടുകളിലേക്കാണ് ആ ദുര്‍മൂര്‍ത്തികള്‍ അവെര നയിക്കുന്നത്. അവരത്രെ നരകാവകാശികള്‍. അവരതില്‍ നിത്യവാസികളാകുന്നു.'' (2:256,257)

ഈ വചനങ്ങളുടെ അവതരണ പശ്ചാത്തലംകൂടി പരിശോധിച്ചാല്‍ നിര്‍ബന്ധ മതപരിവര്‍ത്തനമെന്ന ആശയത്തോട് ഇസ്‌ലാം എങ്ങനെ പ്രതികരിക്കുന്നുവെന്ന് സുതരാം വ്യക്തമാവും. ഏതെങ്കിലും സ്ത്രീകള്‍ക്ക് മക്കളുണ്ടായിട്ടില്ലെങ്കില്‍ തങ്ങള്‍ക്ക് കുഞ്ഞുണ്ടായാല്‍ അവനെ യഹൂദനാക്കി വളര്‍ത്താമെന്ന് നേര്‍ച്ചയാക്കുന്ന സ്രമ്പദായമുണ്ടായിരുന്നു, യഥ്‌രിബ് വാസികള്‍ക്കിടയില്‍. ജൂതന്മാരായ ബനൂ നദ്വീര്‍ ഗോത്രക്കാരെ മദീനയില്‍ നിന്ന് നാടുകടത്തിയപ്പോള്‍ അവരോടൊപ്പം ഇങ്ങനെ ജൂതന്മാരായിത്തീര്‍ന്ന ചില അന്‍സ്വാരി സ്ത്രീകളുടെ കുഞ്ഞുങ്ങളുമുണ്ടായിരുന്നു. അവര്‍ പ്രവാചകനോട് പറഞ്ഞു: ദൈവദൂതരേ, അവര്‍ ഞങ്ങളുടെ മക്കളാണ്. അവരെ ഞങ്ങള്‍ ജൂതന്മാരോടൊപ്പം പറഞ്ഞയക്കുകയില്ല. ഈ സന്ദര്‍ഭത്തിലാണ് ഭമതത്തില്‍ ബലാല്‍ക്കാരമില്ല' എന്ന് തുടങ്ങുന്ന സൂക്തം അവതരിക്കപ്പെട്ടതെന്ന് ഇബ്‌നു അബ്ബാസില്‍ നിന്ന് സ്വീകാര്യമായ പരമ്പരയോടെ (സ്വഹീഹ്) സുനനു അബീദാവൂദില്‍ ഉദ്ധരിച്ചിട്ടുണ്ട്.(സുനനു അബൂദാവൂദ്, കിതാബുല്‍ ജിഹാദ്‌) സ്വേച്ഛപ്രകാരമല്ലാതെ ജൂതമതം സ്വീകരിക്കേണ്ടിവന്ന സ്വന്തം മക്കളെപ്പോലും നിര്‍ബന്ധപൂര്‍വം ഇസ്‌ലാം സ്വീകരിപ്പിക്കേണ്ടതില്ലെന്ന് അവരുടെ മാതാക്കളെ നിഷ്‌കര്‍ഷിക്കുന്ന ക്വുര്‍ആന്‍ നിര്‍ബന്ധ മതപരിവര്‍ത്തനമെന്ന ആശയത്തെ ഒരു തരത്തിലും അംഗീകരിക്കുകയില്ലെന്ന് ഉറപ്പാണ്. അതുകൊണ്ട് തന്നെ ഇസ്‌ലാം യുദ്ധംചെയ്യാന്‍ പറയുന്നത് ലോകത്തുള്ള മനുഷ്യരെയെല്ലാം മുസ്‌ലിംകളാക്കുന്നതിന് വേണ്ടിയാണെന്ന വിമര്‍ശനം ശുദ്ധഭോഷ്‌ക്കാണ്.

മതത്തിന്റെ പേരില്‍ മര്‍ദിക്കപ്പെടുകയും, ഏകനായ സ്രഷ്ടാവിനെ മാത്രം ആരാധിച്ചുകൊണ്ട് സ്വസ്ഥമായി ജീവിക്കുകയും ആ മാര്‍ഗം മറ്റുള്ളവര്‍ക്ക് പകര്‍ന്നുനല്‍കാനുള്ള സ്വാതന്ത്ര്യം നിേഷധിക്കപ്പെടുകയും ചെയ്യുമ്പോള്‍ മര്‍ദനങ്ങള്‍ക്ക് അറുതി വരുത്തുന്നതിനും സ്വാതന്ത്ര്യം സ്ഥാപിക്കുന്നതിനും വേണ്ടി നടത്തുന്നതാണ് ഇസ്‌ലാം അനുവദിച്ച യുദ്ധം. ഇത്തരം സാഹചര്യങ്ങളില്‍ സായുധമായി പ്രതികരിക്കാതിരുന്നാല്‍ പിന്നെ എങ്ങനെയാണ് സത്യമതത്തിന് നിലനില്‍പുണ്ടാവുക? യുദ്ധം അനുവദിച്ച ഇസ്‌ലാമിന്റെ നടപടി തെറ്റായിപ്പോയെന്ന് പറയുന്നവര്‍ക്ക് ഇത്തരം സാഹചര്യങ്ങളില്‍ എന്ത് പരിഹാര മാര്‍ഗമാണ് നിര്‍ദേശിക്കുവാനുള്ളത്? യുദ്ധം അനുവദിക്കുക മാത്രമല്ല, ആരോട് എപ്പോള്‍ എങ്ങനെയെല്ലാം യുദ്ധംചെയ്യണമെന്നു കൂടി കൃത്യവും വ്യക്തവുമായി ഇസ്‌ലാം മുസ്‌ലിംകളെ പഠിപ്പിച്ചിട്ടുണ്ട് . ദൈവിക മാര്‍ഗദര്‍ശനപ്രകാരം അവര്‍ മുന്നേറിയപ്പോള്‍ മതത്തിന്റെ പേരിലുള്ള മര്‍ദനങ്ങള്‍ക്ക് അറുതി വരുത്തുവാനും ആരെയും ഭയപ്പെടാതെ അല്ലാഹുവിനെ മാത്രം ആരാധിച്ചുകൊണ്ട് ജീവിക്കുവാന്‍ കഴിയുന്ന സ്ഥിതി ഉണ്ടാക്കുവാനും അല്ലാഹുവിന്റെ നാം ഉയര്‍ത്തിപ്പിടിച്ച് അഭിമാനത്തോടെ ജീവിക്കാനും അവര്‍ക്ക് കഴിഞ്ഞു. ഫിത്‌ന(പീഢനം) ഇല്ലാതെയാവുകയും മതം മുഴുവന്‍ അല്ലാഹുവിന് വേണ്ടി മാത്രമായിത്തീര്‍ക്കുകയുമായിരുന്നു അവരുടെ യുദ്ധലക്ഷ്യം. പ്രസ്തുത ലക്ഷ്യം നേടുന്നത് വരെ അവര്‍ യുദ്ധം ചെയ്തു. മുസ്‌ലിമായിപ്പോയെന്ന കാരണത്താല്‍ മാത്രം മനുഷ്യര്‍ പീഡിപ്പിക്കപ്പെട്ടിരുന്ന അവസ്ഥക്ക് വിരാമമായി. മുസ്‌ലിമായി ജീവിക്കുവാനും സത്യമതപ്രബോധനത്തിനുമുള്ള സ്വാതന്ത്ര്യമുണ്ടായി. പ്രസ്തുത സ്വാതന്ത്ര്യമുപയോഗിച്ച് ലോകത്തിന്റെ മുക്കുമൂലകളിലേക്ക് സത്യമതത്തിന്റെ സന്ദേശം പ്രസരിപ്പിക്കുവാന്‍ അവര്‍ പരിശ്രമിച്ചു. നമസ്‌ക്കാരത്തിനായി ക്ഷണിക്കുന്ന ബാങ്ക്‌വിളി മുഴങ്ങാത്ത ഒരു നിമിഷം പോലും ഭൂഗോളത്തിലില്ലാത്ത സ്ഥിതി സംജാതമായത് അങ്ങനെയാണ്.

ആദര്‍ശത്തിന് വേണ്ടിയുള്ളതാണ് ഇസ്‌ലാമിലെ യുദ്ധം. എതിരാളികളെയെല്ലാം ഉന്മൂലനം ചെയ്യുകയോ ലോകത്തിന്റെ മേല്‍ മുസ്‌ലിംകളുടെ രാഷ്ട്രീയാധിനിവേശം സ്ഥാപിക്കുകയോ ചെയ്യുന്നതിന് വേണ്ടിയുള്ളതല്ല അത്. ദൈവത്തമായ ജീവിതവ്യവസ്ഥയനുസരിച്ച് ജീവിക്കുവാനും അതു മറ്റുള്ളവര്‍ക്ക് പകര്‍ന്നുകൊടുക്കുവാനുമുള്ള സ്വാതന്ത്ര്യമുണ്ടാക്കുകയാണ് യുദ്ധത്തിന്റെ ലക്ഷ്യം. മുസ്‌ലിംകളായി ജീവിക്കാന്‍ അനുവദിക്കാതെ ജനിച്ചുവളര്‍ന്ന നാട്ടില്‍ നിന്ന് പലായനം ചെയ്യേണ്ടി വന്നവര്‍ക്ക് തങ്ങളുടെ അവകാശങ്ങള്‍ വീണ്ടെടുക്കുന്നതിന് വേണ്ടി പേരാടുവാന്‍ അനുവദിച്ചുകൊണ്ടുള്ളതാണ് യുദ്ധത്തെക്കുറിച്ച് പരാമര്‍ശിക്കുന്ന ആദ്യമായി അവതരിക്കപ്പെട്ട വചനങ്ങള്‍. സൂറത്തുല്‍ ഹജ്ജിലെ 39,40 സൂക്തങ്ങള്‍ അവതരിക്കപ്പെട്ടപ്പോഴാണ് യുദ്ധമുണ്ടാകുവാന്‍ പോകുന്നുണ്ടെന്ന് തനിക്ക് മനസ്സിലായത് എന്ന് അബൂബക്കർ (റ) പറഞ്ഞതായി  സ്വീകാര്യമായ പരമ്പരയോടെ ഉദ്ധരിക്കപ്പെട്ടിട്ടുണ്ട്. (സുനനുന്നസാഇയിലും (ഹദീഥ് 365) ജാമിഉത്തിര്‍മിദിയിലും (3171) മുസ്‌നദ് ഇമാം അഹ്മദിലും (1216) അല്‍മുസ്തദ്‌റക്ക് അല്‍ ഹാകിമിലും (266) ഉദ്ധരിച്ചിരിക്കുന്ന ഈ ഹദീഥ് സ്വഹീഹാണെന്ന് ഇമാം നസാഈ രേഖപ്പെടുത്തിയതായി ഇമാം നൈസാബൂരി തന്റെ അസ്ബാബുന്നുസൂലില്‍’പറഞ്ഞിട്ടുണ്ട്.(Aiman Saleh Sha’aban & Mohammed Ismail: Al-Imam Al-Nais aburi’s"Reasons of Revelation of the Holy Qur'an'', Kaulalampur, 2010, page457))

വീടും സമ്പത്തുകളും ഉപേക്ഷിച്ചുകൊണ്ട് ആദര്‍ശമനുസരിച്ച് ജീവിക്കുവാന്‍ വേണ്ടി മാത്രമായി പലായനം ചെയ്തവര്‍ക്ക് അവര്‍ പലായനം ചെയ്‌തെത്തി ജീവിക്കുന്ന നാട്ടില്‍ സൈ്വര്യമായിരിക്കുവാന്‍ അവസരം നല്‍കാതിരിക്കുന്നവര്‍ക്കെതിരെ യുദ്ധം ചെയ്യാന്‍ അനുവാദമുണ്ടെന്നും ആയുധമെടുക്കുന്നവരെ അല്ലാഹു സഹായിക്കുമെന്ന് അറിയിക്കുകയും ചെയ്തുകൊണ്ടുള്ള പ്രസ്തുത സൂക്തങ്ങളുടെ സാരം ഇങ്ങനെയാണ്: ''യുദ്ധത്തിന്ന് ഇരയാകുന്നവര്‍ക്ക്, അവര്‍ മര്‍ദിതരായതിനാല്‍ (തിരിച്ചടിക്കാന്‍) അനുവാദം നല്‍കപ്പെട്ടിരിക്കുന്നു. തീര്‍ച്ചയായും അല്ലാഹു അവരെ സഹായിക്കാന്‍ കഴിവുള്ളവന്‍ തന്നെയാകുന്നു. യാതൊരു ന്യായവും കൂടാതെ, ഞങ്ങളുടെ രക്ഷിതാവ് അല്ലാഹുവാണ് എന്ന് പറയുന്നതിന്റെ പേരില്‍ മാത്രം തങ്ങളുടെ ഭവനങ്ങളില്‍ നിന്ന് പുറത്താക്കപ്പെട്ടവരെ്രത അവര്‍. മനുഷ്യരില്‍ ചിലരെ മറ്റുചിലരെക്കൊണ്ട് അല്ലാഹു തടുക്കുന്നില്ലായിരുന്നുവെങ്കില്‍ സന്യാസിമഠങ്ങളും, ക്രിസ്തീയദേവാലയങ്ങളും, യഹൂദദേവാലയങ്ങളും, അല്ലാഹുവിന്റെ നാമം ധാരാളമായി പ്രകീര്‍ത്തിക്കപ്പെടുന്ന മുസ്‌ലിം പള്ളികളും തകര്‍ക്കപ്പെടുമായിരുന്നു. തന്നെ സഹായിക്കുന്നതാരോ അവനെ തീര്‍ച്ചയായും അല്ലാഹു സഹായിക്കും. തീര്‍ച്ചയായും അല്ലാഹു ശക്തനും പ്രതാപിയും തന്നെയാകുന്നു.'' (22:39,40)

മതമര്ദനം ഇല്ലാതെയാക്കുന്നതിനും ആക്രമത്തെ പ്രതിരോധിക്കുന്നതിനും വേണ്ടിയാണ് മുസ്‌ലിംകള്‍ യുദ്ധം ചെയ്യേണ്ടതെന്ന് വ്യക്തമാക്കുന്ന ഈ സൂക്തങ്ങള്‍ അത്തരം പ്രതിരോധങ്ങളുണ്ടായിട്ടില്ലെങ്കില്‍ മുസ്‌ലിംകള്‍ക്ക് മാത്രമല്ല മറ്റു മതവിശ്വാസികള്‍ക്കും സൈ്വര്യമായി ആരാധനകള്‍ നിര്‍വഹിച്ചുകൊണ്ട് ജീവിക്കുവാന്‍ കഴിയാത്ത സാഹചര്യമാണുണ്ടാവുകയെന്ന് വ്യക്തമാക്കുന്നു. സന്യാസി മഠങ്ങളും ചര്‍ച്ചുകളും സിനഗോഗുകളും മസ്ജിദുകളുമെല്ലാം തകര്‍ക്കപ്പെടുകയും വിശ്വാസികള്‍ക്കൊന്നും സ്വസ്ഥമായി ആരാധനകള്‍ നിര്‍വഹിക്കാനാവാത്ത അവസ്ഥ ഉണ്ടാവുകയുമായിരിക്കും മതപീഡകന്മാരെ സ്വതന്ത്രമായി അഴിഞ്ഞാടുവാന്‍ അനുവദിച്ചാലുണ്ടാവുന്ന ഫലങ്ങള്‍. മതത്തിന്റെ പേരിലുള്ള പീഡനം ഇല്ലാതെയാക്കുന്നതിന് വേണ്ടി മുസ്‌ലിംകള്‍ നടത്തുന്ന പോരാട്ടം മറ്റു മതവിശ്വാസികള്‍ക്കും സമാധാനപൂര്‍ണമായി ആരാധനകള്‍ നിര്‍വഹിക്കുവാനുള്ള സാഹചര്യമൊരുക്കുകയാണ് സ്വാഭാവികമായും ചെയ്യുകയെന്നര്‍ഥം.

സൂറത്തുല്‍ ബക്വറയിലെ 190 മുതല്‍ 193 വരെ സൂക്തങ്ങളും സൂറത്തുന്നിസാഇലെ 75-ാം സൂക്തവുമെല്ലാം മുസ്‌ലിംകള്‍ എന്തിനുവേണ്ടി, എങ്ങനെയെല്ലാം യുദ്ധം ചെയ്യണമെന്ന് വ്യക്തമാക്കുന്നവയാണ്. പ്രസ്തുത സൂക്തങ്ങളുടെ സാരം കാണുക:

''നിങ്ങളോട് യുദ്ധം ചെയ്യുന്നവരുമായി അല്ലാഹുവിന്റെ മാര്‍ഗത്തില്‍ നിങ്ങളും യുദ്ധം ചെയ്യുക. എന്നാല്‍ നിങ്ങള്‍ പരിധിവിട്ട് പ്രവര്‍ത്തിക്കരുത്. പരിധിവിട്ട് പ്രവര്‍ത്തിക്കുന്നവരെ അല്ലാഹു ഇഷ്ടപ്പെടുകയില്ല തന്നെ. അവരെ കണ്ടുമുട്ടുന്നേടത്ത് വെച്ച് നിങ്ങളവരെ കൊന്നുകളയുകയും, അവര്‍ നിങ്ങളെ പുറത്താക്കിയേടത്ത് നിന്ന് നിങ്ങള്‍ അവരെ പുറത്താക്കുകയും ചെയ്യുക. (കാരണം, അവര്‍ നടത്തുന്ന) മര്‍ദനം കൊലയേക്കാള്‍ നിഷ്ഠൂരമാകുന്നു. മസ്ജിദുല്‍ ഹറാമിന്നടുത്ത് വെച്ച് നിങ്ങള്‍ അവരോട് യുദ്ധം ചെയ്യരുത്; അവര്‍ നിങ്ങളോട് അവിടെ വെച്ച് യുദ്ധം ചെയ്യുന്നത് വരെ. ഇനി അവര്‍ നിങ്ങളോട് (അവിടെ വെച്ച്) യുദ്ധത്തില്‍ ഏര്‍പെടുകയാണെങ്കില്‍ അവരെ കൊന്നുകളയുക. അപ്രകാരമാണ് സത്യനിഷേധികള്‍ക്കുള്ള പ്രതിഫലം. ഇനി അവര്‍ (പശ്ചാത്തപിച്ച്, എതിര്‍പ്പില്‍ നിന്ന്) വിരമിക്കുകയാണെങ്കിലോ തീര്‍ച്ചയായും ഏറെ പൊറുക്കുന്നവനും കരുണാനിധിയുമാണ് അല്ലാഹു. മര്‍ദനം ഇല്ലാതാവുകയും, മതം അല്ലാഹുവിന് വേണ്ടിയാവുകയും ചെയ്യുന്നത് വരെ നിങ്ങളവരോട് യുദ്ധം നടത്തിക്കൊള്ളുക. എന്നാല്‍ അവര്‍ (യുദ്ധത്തില്‍ നിന്ന്) വിരമിക്കുകയാണെങ്കില്‍ (അവരിലെ) അക്രമികള്‍ക്കെതിരിലല്ലാതെ പിന്നീട് യാതൊരു കയ്യേറ്റവും പാടുള്ളതല്ല.'' (2:190-193)

''അല്ലാഹുവിന്റെ മാര്‍ഗത്തില്‍ നിങ്ങള്‍ക്കെന്തുകൊണ്ട് യുദ്ധം ചെയ്തു കൂടാ? ഞങ്ങളുടെ രക്ഷിതാവേ, അക്രമികളായ ആളുകള്‍ അധിവസിക്കുന്ന ഈ നാട്ടില്‍ നിന്ന് ഞങ്ങളെ നീ മോചിപ്പിക്കുകയും, നിന്റെ വകയായി ഒരു രക്ഷാധികാരിയെയും നിന്റെ വകയായി ഒരു സഹായിയെയും ഞങ്ങള്‍ക്ക് നീ നിശ്ചയിച്ച് തരികയും ചെയ്യേണമേ. എന്ന് പ്രാര്‍ഥിച്ച് കൊണ്ടിരിക്കുന്ന മര്‍ദിച്ചൊതുക്കപ്പെട്ട പുരുഷന്‍മാര്‍ക്കും സ്ത്രീകള്‍ക്കും കുട്ടികള്‍ക്കും വേണ്ടിയും (നിങ്ങള്‍ക്കെന്തുകൊണ്ട് യുദ്ധം ചെയ്തു കൂടാ?)'' (4:75)

ഈ സൂക്തങ്ങളില്‍ നിന്ന് നിര്‍ധരിക്കപ്പെടുന്ന യുദ്ധവിധികള്‍ താഴെ പറയുന്നവയാണ്:

1. മുസ്‌ലിംകളോട് യുദ്ധം ചെയ്യുന്നവരോട് അവര്‍ക്ക് സായുധമായി പ്രതിരോധിക്കാവുന്നതാണ്.

2. മുസ്‌ലിംകളെ അക്രമിക്കുകയും പീഡിപ്പിക്കുകയും അവരുടെ ഭവനങ്ങള്‍ കയ്യേറുകയും സമ്പത്തുകള്‍ പിടിച്ചെടുക്കുകയും അവകാശങ്ങള്‍ അനുവദിക്കാതിരിക്കുകയും ചെയ്യുന്നവര്‍ക്കെതിരില്‍ അവര്‍ക്ക് യുദ്ധം ചെയ്യാവുന്നതാണ്.

3. മതവിശ്വാസത്തിന്റെ പേരില്‍ മര്‍ദിക്കപ്പെടുകയും പീഡിക്കപ്പെടുകയും ശത്രുരാജ്യത്ത് മതവിശ്വാസം പുറത്തുപറയാതെ ജീവിക്കുവാന്‍ നിര്‍ബന്ധിക്കപ്പെടുകയും ചെയ്യുന്നവരെ മോചിപ്പിക്കുന്നതിന് വേണ്ടി മുസ്‌ലിംകള്‍ക്ക് അടരാടാവുന്നതാണ്.

4. യുദ്ധം ചെയ്യുന്നത് ശത്രുക്കളോട് പ്രതികാരം ചെയ്യുവാനോ പ്രശസ്തിക്കുവേണ്ടിയോ അല്ല, പ്രത്യുത അല്ലാഹുവിന്റെ മാര്‍ഗത്തില്‍ അവന്റെ മതമനുസരിച്ച് ജീവിക്കുവാനും അതു പ്രബോധനം ചെയ്യുന്നതിനുമുള്ള സ്വാതന്ത്ര്യം നേടിയെടുക്കാന്‍ വേണ്ടിയായിരിക്കണം.

5. യുദ്ധത്തില്‍ അതിക്രമങ്ങള്‍ ഉണ്ടാകുവാന്‍ പാടില്ല.

6. യുദ്ധത്തിലോ യുദ്ധസന്ദര്‍ഭത്തിലോ ശത്രുക്കളെ വകവരുത്തുവാന്‍ സന്ദര്‍ഭങ്ങളൊന്നും പാഴാക്കരുത്, അവരെ കണ്ടിടത്തുവെച്ച് കൊല്ലുക തന്നെ വേണം.

7. മര്‍ദനം ഇല്ലാതായാവുകയും മതം അല്ലാഹുവിന് വേണ്ടിയാവുകയും ചെയ്യുന്നത് വരെയാണ് മുസ്‌ലിംകള്‍ യുദ്ധം ചെയ്യേണ്ടത്.

8. അക്രമകാരികള്‍ കുഴപ്പമുണ്ടാക്കുന്നതില്‍ നിന്ന് വിരമിച്ചാല്‍ പിന്നീട് അവരില്‍പെട്ട അതിക്രമകാരികള്‍ക്കെതിരല്ലാതെ മറ്റുള്ളവര്‍ക്കെതിരില്‍ യാതൊരുവിധ നടപടികളുണ്ടാകുവാന്‍ പാടുള്ളതല്ല.

സാമ്രാജ്യസ്ഥാപനത്തിനോ എതിരാളികളെ കൊന്നൊടുക്കുന്നതിനോ അധീശത്വം സ്ഥാപിച്ച് ആസ്വാദ്യകരമായ ജീവിതം ആസ്വദിക്കുന്നതിനോ വേണ്ടിയല്ല സത്യമെന്ന് ബോധ്യമുള്ള ആദര്‍ശമനുസരിച്ച് ജീവിക്കുന്നതിനുള്ള സ്വാതന്ത്ര്യത്തിന് വേണ്ടിയാണ് അനിവാര്യമായ സാഹചര്യത്തില്‍ ആയുധമെടുക്കുവാന്‍ ഇസ്‌ലാം അനുവദിച്ചിട്ടുള്ളത്. തങ്ങളെയും തങ്ങളുള്‍ക്കൊള്ളുന്ന ആദര്‍ശത്തെയും നശിപ്പിക്കുവാനൊരുങ്ങി സായുധ സമരം നടത്തുന്നവരോട് ആവശ്യമാണെങ്കില്‍ സായുധരായിത്തന്നെ പ്രതികരിക്കണമെന്നുതന്നെയാണ് ഇസ്‌ലാമിന്റെ നിലപാട്. ആക്രമിക്കുകയും പീഡിപ്പിക്കുകയും ഭവനങ്ങള്‍ കയ്യേറുകയും സമ്പത്തുക്കള്‍ പിടിച്ചടക്കുകയും ചെയ്യുന്നവരെ പ്രതിരോധിക്കുകയും സത്യമതമനുസരിച്ച് ജീവിക്കാന്‍ അനുവദിക്കാത്ത സാഹചര്യങ്ങളില്‍ ജീവിക്കുന്നവരെ സ്വതന്ത്രരാക്കുകയും സംരക്ഷിക്കുകയും ചെയ്യുവാനുള്ള യുദ്ധത്തില്‍പോലും അതിരുകള്‍ ലംഘിച്ചുകൂടായെന്നും അതിക്രമങ്ങള്‍ കാണിച്ചുകൂടായെന്നുമാണ് ഇസ്‌ലാം അനുശാസിക്കുന്നത്. സ്വയം ജീവിക്കുകയും മറ്റുള്ളവരെ ജീവിക്കുവാന്‍ അനുവദിക്കുകയും ചെയ്യുകയെന്നാണ് ഇസ്‌ലാം എല്ലാവരോടും അനുശാസിക്കുന്നത്. സ്വാര്‍ഥമായ ജീവിതത്തിനുവേണ്ടി മറ്റുള്ളവരെ ജീവിക്കുവാനനുവദിക്കാത്ത സാഹചര്യങ്ങളുണ്ടാകുമ്പോള്‍ അത്തരക്കാര്‍ക്കെതിരെ നടത്തുന്നതാണ് ഇസ്‌ലാം അനുവദിച്ച സായുധ സമരം. അതിക്രമകാരികളെ പ്രതിരോധിക്കുവാനായുള്ള പ്രസ്തുത സമരം നടന്നിട്ടില്ലെങ്കില്‍ സമാധാനപൂര്‍ണമായി ജീവിക്കുവാനുള്ള സ്വാതന്ത്ര്യമാണ് ഹനിക്കപ്പെടുകയെന്ന് വ്യക്തമാക്കിക്കൊണ്ടാണ് ക്വുർആൻ സായുധസമരത്തിന് അനുമതി നൽകിയതെന്ന വസ്തുത ശ്രദ്ധേയമാണ് (22: 39, 40)

വിഷയവുമായി ബന്ധപ്പെട്ട വീഡിയോ

മനുഷ്യജീവന് ഏറെ പവിത്രതയുണ്ടെന്ന് പഠിപ്പിക്കുന്ന ആദര്‍ശമാണ് ഇസ്‌ലാം. ജീവിക്കുവാനുള്ള മനുഷ്യരുടെ അവകാശെത്ത അന്യായമായി ഹനിക്കുവാന്‍ ആര്‍ക്കും അവകാശമില്ലെന്നാണ് അത് പഠിപ്പിക്കുന്നത്. ആദ്യത്തെ കൊലപാതകത്തെക്കുറിച്ച് പരാമര്‍ശിച്ചുകൊണ്ട് ക്വുര്‍ആന്‍ പറഞ്ഞു:

''അക്കാരണത്താല്‍ ഇസ്രായീല്‍ സന്തതികള്‍ക്ക് നാം ഇപ്രകാരം വിധിനല്‍കുകയുണ്ടായി: മറ്റൊരാളെ കൊന്നതിന് പകരമായോ, ഭൂമിയില്‍ കുഴപ്പമുണ്ടാക്കിയതിന്റെ പേരിലോ അല്ലാതെ വല്ലവനും ഒരാളെ കൊലപ്പെടുത്തിയാല്‍, അത് മനുഷ്യരെ മുഴുവന്‍ കൊലപ്പെടുത്തിയതിന് തുല്യമാകുന്നു. ഒരാളുടെ ജീവന്‍ വല്ലവനും രക്ഷിച്ചാല്‍, അത് മനുഷ്യരുടെ മുഴുവന്‍ ജീവന്‍ രക്ഷിച്ചതിന് തുല്യമാകുന്നു.'' (5:32)

ഒരാളും അന്യായമായി വധിക്കപ്പെട്ടുകൂടാ എന്ന് ഇസ്രാഈല്യര്‍ക്ക് നല്‍കിയ ദൈവിക ബോധനം അവര്‍ക്ക് നല്‍കപ്പെട്ട പത്ത് കല്‍പനകളില്‍ രേഖപ്പെടുത്തിയിട്ടുണ്ട്. ബൈബിള്‍ പഴയനിയമത്തിലെ പുറപ്പാട് പുസ്തകത്തിലും(പുറപ്പാട് 20:1-17)ആവര്‍ത്തന പുസ്തകത്തിലും(ആവര്‍ത്തനം 5:6-22) ഉദ്ധരിച്ചിരിക്കുന്ന പത്ത് കല്‍പനകളില്‍ ആറാമത്തേത് 'നീ കൊല്ലരുത്’എന്നാണ്.  ആദമിന് ശേഷം വന്ന പ്രവാചകന്മാരിലൂടെയെല്ലാം അന്യായമായ കൊലക്കെതിരെയുള്ള ബോധവല്‍ക്കരണമുണ്ടായിട്ടുണ്ടെന്ന് നടേ ഉദ്ധരിച്ച ക്വുര്‍ആന്‍ സൂക്തം സൂചിപ്പിക്കുന്നുണ്ട്. മറ്റൊരാളെ മാത്രമല്ല, സ്വന്തത്തെ വധിക്കാന്‍ പോലും ആര്‍ക്കും അവകാശമില്ലെന്നാണ് ഇസ്‌ലാമിന്റെ അധ്യാപനം. ക്വുര്‍ആന്‍ പറയുന്നു: ''നിങ്ങള്‍ നിങ്ങളെത്തന്നെ കൊലപ്പെടുത്തുകയും ചെയ്യരുത്. തീര്‍ച്ചയായും അല്ലാഹു നിങ്ങളോട് കരുണയുള്ളവനാകുന്നു. ആരെങ്കിലും അതിക്രമമായും അന്യായമായും അങ്ങനെ ചെയ്യുന്ന പക്ഷം നാമവനെ നരകാഗ്‌നിയിലിട്ട് കരിക്കുന്നതാണ്. അത് അല്ലാഹുവിന് എളുപ്പമുള്ള കാര്യമാകുന്നു.'' (4:29,30) ആത്മഹത്യ ചെയ്യുന്നവര്‍ക്ക് സ്വര്‍ഗം നിഷിദ്ധമാണെന്ന് മുഹമ്മദ് നബി (സ) പഠിപ്പിച്ചു: ജുന്‍ദുബില്‍നിന്ന്: നബി പറഞ്ഞു: മുറിവേറ്റിരുന്ന ഒരാള്‍ അക്കാരണത്താല്‍ ആത്മഹത്യ ചെയ്തു. അപ്പോള്‍ അല്ലാഹു പറയുന്നു:ന്നുഅവന്റെ ജീവന്റെ കാര്യത്തില്‍ എന്നെ മുന്‍കടന്ന് പ്രവര്‍ത്തിച്ചിരിക്കുന്നു. അതിനാല്‍ ഞാനവന്ന് സ്വര്‍ഗം നിഷിദ്ധമാക്കി.(സ്വഹീഹുല്‍ ബുഖാരി, കിത്താബുല്‍ ജനാഇസ്) അന്യായമായി ഒരാളുടെ രക്തം ചിന്തുവാന്‍ വഴികളന്വേഷിച്ച് നടക്കുന്നവന്‍ അല്ലാഹുവിന്റെ കോപത്തിന് ഇരയാകുന്നയാളുകളില്‍ മുന്‍പന്തിയിലായിരിക്കും ഉണ്ടാവുകയെന്നാണ് മുഹമ്മദ് നബി(സ)പഠിപ്പിച്ചത്: ഇബ്‌നുഅബ്ബാസില്‍ നിന്ന്: നബില പറഞ്ഞു: ജനങ്ങളില്‍ അല്ലാഹുവിന് ഏറ്റവും കോപമുണ്ടാവുക മൂന്ന് വിഭാഗം ആളുകളാണ്: മസ്ജിദുല്‍ ഹറാമില്‍ വെച്ച് അക്രമം പ്രവര്‍ത്തിക്കുന്നവന്‍, ജാഹിലീ കാലഘട്ടത്തിലെ നടപടികള്‍ പാലിക്കാന്‍ ആഗ്രഹിക്കുന്നവന്‍, അന്യായമായി ഒരാളുടെ രക്തം ചിന്താന്‍ വഴി അനേഷിക്കുന്നവന്‍.(സ്വഹീഹുല്‍ ബുഖാരി, കിതാബു ദ്ദിയാത്ത്)

അന്ത്യനാളില്‍ ആദ്യമായി വിധി തീര്‍പുകല്‍പിക്കപ്പെടുക കൊലക്കുറ്റത്തിന്റെ കാര്യത്തിലായിരിക്കുമെന്നും നബി (സ) പഠിപ്പിച്ചു: അബ്ദുല്ലയില്‍ നിന്ന്: നബി (സ)  പറഞ്ഞു: പരലോകത്ത് ആദ്യമായി വിധി തീര്‍പ്പുണ്ടാക്കുക കൊലക്കുറ്റങ്ങള്‍ക്കാണ്.(സ്വഹീഹുല്‍ ബുഖാരി, കിതാബു ര്‍റിഖാഖ്)

ഇസ്‌ലാമിക സമൂഹത്തില്‍ ജീവിക്കുന്ന ഒരാള്‍ക്ക് മതം നല്‍കുന്ന വിശാലത അനുഭവിക്കാനാവുക നിരപരാധികളുടെ രക്തം ചിന്താത്ത കാലത്തോളം മാത്രമായിരിക്കുമെന്ന് നബില വ്യക്തമാക്കി: ഇബ്‌നുഉമറില്‍നിന്ന്: റസൂല്‍(സ)പറഞ്ഞു: ആദരണീയ (നിരപരാധിയുടെ) രക്തം ചീന്താതിരിക്കുന്നിടത്തോളം സത്യവിശ്വാസിക്ക് ദീനിന്റെ വിശാലത ലഭിച്ചുകൊണ്ടിരിക്കും.(സ്വഹീഹുല്‍ ബുഖാരി, കിതാബു ദ്ദിയാത്ത്)

സത്യവിശ്വാസിയുടെ കരങ്ങളില്‍ നിന്ന് നിരപരാധികള്‍ എപ്പോഴും സുരക്ഷിതരായിരിക്കുമെന്ന് ക്വുര്‍ആന്‍ വ്യക്തമാക്കുന്നുണ്ട്. സത്യവിശ്വാസികളുടെ സ്വഭാവ വിശേഷങ്ങളെക്കുറിച്ച് പരാമര്‍ശിക്കവെ ക്വുര്‍ആന്‍ പറഞ്ഞു: ''അല്ലാഹുവോടൊപ്പം വേറെയൊരു ദൈവത്തെയും വിളിച്ചു പ്രാര്‍ഥിക്കാത്തവരും, അല്ലാഹു പവിത്രമാക്കി വെച്ചിട്ടുള്ള ജീവനെ ന്യായമായ കാരണത്താലല്ലാതെ ഹനിച്ചു കളയാത്തവരും, വ്യഭിചരിക്കാത്തവരുമാകുന്നു അവര്‍. ആ കാര്യങ്ങള്‍ വല്ലവനും ചെയ്യുന്ന പക്ഷം അവന്‍ പാപഫലം കണ്ടെത്തുക തന്നെ ചെയ്യും.'' (25:68)

നിഷിദ്ധമാക്കപ്പെട്ട കാര്യങ്ങളെക്കുറിച്ച് എണ്ണിപ്പറഞ്ഞപ്പോഴും കൊലപാതകത്തെ പ്രത്യേകമായി ക്വുര്‍ആന്‍ എടുത്തുപറഞ്ഞതായി കാണാന്‍ കഴിയും: ''(നബിയേ) പറയുക: നിങ്ങള്‍ വരൂ! നിങ്ങളുടെ രക്ഷിതാവ് നിങ്ങളുടെ മേല്‍ നിഷിദ്ധമാക്കിയത് നിങ്ങള്‍ക്ക് ഞാന്‍ പറഞ്ഞ് കേള്‍പിക്കാം. അവനോട് യാതൊന്നിനെയും നിങ്ങള്‍ പങ്കുചേര്‍ക്കരുത്. മാതാപിതാക്കള്‍ക്ക് നന്‍മചെയ്യണം. ദാരിദ്ര്യം കാരണമായി സ്വന്തം മക്കളെ നിങ്ങള്‍ കൊന്നുകളയരുത്. നാമാണ് നിങ്ങള്‍ക്കും അവര്‍ക്കും ആഹാരം തരുന്നത്. പ്രത്യക്ഷവും പരോക്ഷവുമായ നീചവൃത്തികളെ നിങ്ങള്‍ സമീപിച്ച് പോകരുത്. അല്ലാഹു പരിപാവനമാക്കിയ ജീവനെ ന്യായപ്രകാരമല്ലാെത നിങ്ങള്‍ ഹനിച്ചുകളയരുത്. നിങ്ങള്‍ ചിന്തിച്ച് മനസ്സിലാക്കുവാന്‍ വേണ്ടി. അവന്‍ (അല്ലാഹു) നിങ്ങള്‍ക്ക് നല്‍കിയ ഉപേദശമാണത്.'' (6:151)

മാതാവിന്റെ ഗര്‍ഭാശയത്തില്‍ ഉരുവം പ്രാപിച്ചു കഴിഞ്ഞാല്‍ എല്ലാവര്‍ക്കും ജനിക്കുവാനും ജീവിക്കുവാനും അവകാശമുണ്ടെന്നതാണ് ഇസ്‌ലാമിന്റെ നിലപാട്. പട്ടിണി ഭയന്ന് മക്കളെ കൊല്ലുവാന്‍ ധൃഷ്ടരായിരുന്ന ജാഹിലിയ്യാ അറബികളുടെ സമ്പ്രദായെത്ത വിമര്‍ശിക്കുന്ന ക്വുര്‍ആന്‍ വചനങ്ങള്‍ ഇന്നു യോജിക്കുക ജനസംഖ്യാ വര്‍ധനവിനെക്കുറിച്ച ഭീതിപരത്തി ഭ്രൂണഹത്യക്ക് പ്രേരിപ്പിക്കുന്നവരുടെ ചെയ്തികള്‍ക്കാണ്. ക്വുര്‍ആന്‍ നിഷ്‌കര്‍ഷിക്കുന്നത് കാണുക: ''ദാരിദ്ര്യഭയത്താല്‍ നിങ്ങള്‍ നിങ്ങളുടെ കുഞ്ഞുങ്ങളെ കൊന്നുകളയരുത്. നാമാണ് അവര്‍ക്കും നിങ്ങള്‍ക്കും ഉപജീവനം നല്‍കുന്നത്. അവരെ കൊല്ലുന്നത് തീര്‍ച്ചയായും ഭീമമായ അപരാധമാകുന്നു.'' (17:31) 'അല്ലാഹു പവിത്രമാക്കിയ ജീവന്‍ ഹനിക്കപ്പെടരുത്'എന്ന ക്വുര്‍ആനിക നിര്‍ദേശത്തോടൊപ്പം തന്നെ ന്യായമായ കാരണങ്ങളാലല്ലാതെ'യെന്നുകൂടി പറയുന്നുണ്ടെന്ന വസ്തുത ശ്രദ്ധേയമാണ്. ഒരു കാരണവശാലും ജീവന്‍ ഹനിക്കപ്പെട്ടുകൂടാ എന്ന അപ്രായോഗികമായ അഹിംസാ നിയമമല്ല ഇസ്‌ലാം പഠിപ്പിക്കുന്നത്. ന്യായമായ കാരണങ്ങളാല്‍ ചിലപ്പോള്‍ ജീവന്‍ ഹനിക്കേണ്ടിവരുമെന്ന വസ്തുത ഇസ്‌ലാം അംഗീകരിക്കുന്നു. ജീവന്‍ ഹിംസിക്കപ്പെടേണ്ട അനിവാര്യമായ സാഹചര്യങ്ങളെന്തൊക്കെയാണെന്ന് കുര്‍ആന്‍ തന്നെ വ്യക്തമാക്കുകയും ചെയ്യുന്നു. നിരപരാധിയുടെ കൊലപാതകം മനുഷ്യരാശിയെ മുഴുവന്‍ കൊല്ലുന്നതിന് സമാനമാണെന്ന് പ്രഖ്യാപിക്കുന്ന ക്വുര്‍ആന്‍ സൂക്തം തന്നെ ഇക്കാര്യം കൂടി പരാമര്‍ശിക്കുന്നുണ്ട്. (5:32) 'ഒരാളെ കൊന്നതിന് പകരമായോ ഭൂമിയില്‍ കുഴപ്പമുണ്ടാക്കുന്നതിന് പകരമായോ ആണ് ഒരാള്‍ വധിക്കപ്പെടേണ്ടത്' എന്ന് ഈ സൂക്തം വ്യക്തമാക്കുന്നു. കൊലപാതകം, കുഴപ്പം എന്നിവ കൊലക്കുറ്റം അര്‍ഹിക്കുന്ന തെറ്റുകളാണെന്നര്‍ഥം. ഈ തെറ്റുകള്‍ ചെയ്തയാളുടെ ജീവനെടുക്കുന്നത് ന്യായവും അതല്ലാത്തവ അന്യായവുമായ കൊലപാതകങ്ങളാണെന്ന് ഇതില്‍ നിന്ന് സുതരാം വ്യക്തമാണ്. തെറ്റുകള്‍ക്കുള്ള ശിക്ഷ വിധിക്കേണ്ടത് ന്യായാധിപനാണ്. കൊലപാതകത്തിനും കുഴപ്പത്തിനുമുള്ള ശിക്ഷയും വിധിക്കേണ്ടത് അയാള്‍ തന്നെ. ഭരണാധികാരിയുടെയോ ന്യായാധിപന്റെയൊ നിര്‍ദേശപ്രകാരമുള്ളതല്ലാതെയുള്ള കൊലപാതകങ്ങളെല്ലാം അന്യായമാണെന്നര്‍ഥം

. അബദ്ധത്തില്‍ സംഭവിച്ചുപോകുന്നതാണെങ്കില്‍ പോലും കൊലപാതകം അന്യായമായി തന്നെ മനസ്സിലാക്കുകയും അതിനുള്ള പ്രായശ്ചിത്തം ചെയ്യുകയും വേണമെന്ന് ഇസ്‌ലാമിക പാഠത്തില്‍ നിന്ന് ഇത് സുതരാം വ്യക്തമാകുന്നുണ്ട്. ക്വുര്‍ആന്‍ പഠിപ്പിക്കുന്നത് കാണുക: ''യാതൊരു വിശ്വാസിക്കും മറ്റൊരു വിശ്വാസിയെ കൊല്ലാന്‍ പാടുള്ളതല്ല; അബദ്ധത്തില്‍ വന്നുപോകുന്നതല്ലാതെ. എന്നാല്‍ വല്ലവനും ഒരു വിശ്വാസിയെ അബദ്ധത്തില്‍ കൊന്നുപോയാല്‍ (പ്രായശ്ചിത്തമായി) ഒരു വിശ്വാസിയായ അടിമയെ മോചിപ്പിക്കുകയും, അവന്റെ (കൊല്ലപ്പെട്ടവന്റെ) അവകാശികള്‍ക്ക് നഷ്ടപരിഹാരം നല്‍കുകയുമാണ് വേണ്ടത്. അവര്‍ (ആ അവകാശികള്‍) അത് ഉദാരമായി വിട്ടുതന്നെങ്കിലൊഴികെ. ഇനി അവന്‍ (കൊല്ലപ്പെട്ടവന്‍) നിങ്ങളോട് ശത്രുതയുള്ള ജനവിഭാഗത്തില്‍ പെട്ടവനാണ്; അവനാണെങ്കില്‍ സത്യവിശ്വാസിയുമാണ് എങ്കില്‍ സത്യവിശ്വാസിയായ ഒരു അടിമയെ മോചിപ്പിക്കുക മാത്രമാണ് വേണ്ടത്. ഇനി അവന്‍ (കൊല്ലപ്പെട്ടവന്‍) നിങ്ങളുമായി സഖ്യത്തിലിരിക്കുന്ന ഒരു ജനവിഭാഗത്തില്‍ പെട്ടവനാണെങ്കില്‍ അവന്റെ അവകാശികള്‍ക്ക് നഷ്ടപരിഹാരം നല്‍കുകയും വിശ്വാസിയായ ഒരു അടിമയെ മോചിപ്പിക്കുകയും ചെയ്യേണ്ടതാണ്. വല്ലവന്നും അത് സാധിച്ച് കിട്ടിയില്ലെങ്കില്‍ തുടര്‍ച്ചയായി രണ്ടുമാസം നോമ്പനുഷ്ഠിക്കേണ്ടതാണ്. അല്ലാഹു നിശ്ചയിച്ച പശ്ചാത്താപ(മാര്‍ഗ)മാണത്. അല്ലാഹു എല്ലാം അറിയുന്നവനും യുക്തിമാനുമാകുന്നു.'' (4:92)

കൊലപാതകത്തിനും കുഴപ്പത്തിനുമുള്ള ശിക്ഷയായി നല്‍കുന്ന വധശിക്ഷയാകട്ടെ സമൂഹത്തിന്റെ സുസ്ഥിതിക്കുവേണ്ടി നടപ്പാക്കപ്പെടേണ്ട പ്രതിക്രിയാ നടപടിയാണെന്നാണ് ഇസ്‌ലാമിന്റെ പക്ഷം. അത് നടപ്പാക്കപ്പെടാത്തപക്ഷം സമൂഹത്തില്‍ അന്യായമായ കൊലപാതകങ്ങള്‍ പെരുകും. കൊലയാളിയും കുഴപ്പമുണ്ടാക്കി സമൂഹത്തില്‍ കലാപങ്ങള്‍ വിതയ്ക്കുന്നവരും മാതൃകാപരമായി ശിക്ഷിക്കപ്പെടുമ്പോഴാണ് സ്വന്തം ജീവന്‍ ഹനിക്കപ്പെടുമോയെന്ന ഭയപ്പാടില്ലാതെ സമാധാനത്തോടെ പൗരന്മാര്‍ക്കെല്ലാം ജീവിക്കാന്‍ കഴിയുക. വധശിക്ഷ നിഷ്‌കര്‍ഷിക്കുന്ന ക്വുര്‍ആന്‍ സൂക്തം ഇക്കാര്യം സുതരാം വ്യക്തമാക്കുന്നുണ്ട്. ''സത്യവിശ്വാസികളേ, കൊലചെയ്യപ്പെടുന്നവരുടെ കാര്യത്തില്‍ തുല്യശിക്ഷ നടപ്പാക്കുക എന്നത് നിങ്ങള്‍ക്ക് നിയമമാക്കപ്പെട്ടിരിക്കുന്നു. സ്വതന്ത്രനു പകരം സ്വതന്ത്രനും, അടിമയ്ക്കു പകരം അടിമയും, സ്ത്രീക്കു പകരം സ്ത്രീയും (കൊല്ലപ്പെടേണ്ടതാണ്.) ഇനി അവന്ന് (കൊലയാളിക്ക്) തന്റെ സഹോദരന്റെ പക്ഷത്ത് നിന്ന് വല്ല ഇളവും ലഭിക്കുകയാണെങ്കില്‍ അവന്‍ മര്യാദ പാലിക്കുകയും, നല്ല നിലയില്‍ (നഷ്ടപരിഹാരം) കൊടുത്തു വീട്ടുകയും ചെയ്യേണ്ടതാകുന്നു. നിങ്ങളുടെ രക്ഷിതാവിങ്കല്‍ നിന്നുള്ള ഒരു വിട്ടുവീഴ്ചയും കാരുണ്യവുമാകുന്നു അത്. ഇനി അതിനു ശേഷവും ആരെങ്കിലും അതിക്രമം പ്രവര്‍ത്തിക്കുകയാണെങ്കില്‍ അവന് വേദനയേറിയ ശിക്ഷയുണ്ടായിരിക്കും. ബുദ്ധിമാന്‍മാരേ, (അങ്ങനെ) തുല്യശിക്ഷ നല്‍കുന്നതിലാണ് നിങ്ങളുടെ ജീവിതത്തിന്റെ നിലനില്‍പ്. നിങ്ങള്‍ സൂക്ഷ്മത പാലിക്കുന്നതിനു വേണ്ടിയത്രെ (ഈ നിയമനിര്‍േദശങ്ങള്‍).'' (2:178, 179)

കുറ്റവാളി മാത്രമാണ് ശിക്ഷിക്കപ്പെടേണ്ടതെന്നും ഒരാളുടെ അടിമയെ വധിച്ചാല്‍ തിരിച്ച് കൊലയാളിയുടെ അടിമയെ വധിക്കുന്നത് പോലെയുള്ള പ്രാകൃത സമ്പ്രദായങ്ങള്‍ അനീതിയാണെന്നും അവ വര്‍ജിക്കേണ്ടവയാണെന്നും കൊലയാളിയെ ശിക്ഷിക്കണമോ അതല്ല നഷ്ടപരിഹാരം സ്വീകരിച്ച് വെറുതെ വിടണമോ നഷ്ടപരിഹാരത്തില്‍ ഇളവ് ചെയ്യണമോ എന്നെല്ലാം തീരുമാനിക്കുവാനുള്ള ആത്യന്തികമായ അധികാരം കൊല്ലപ്പെട്ടവന്റെ അവകാശികള്‍ക്കാണെന്നും വ്യക്തമാക്കുന്ന ഈ ക്വുര്‍ആന്‍ സൂക്തങ്ങളിലെ പരാമര്‍ശങ്ങള്‍ അവസാനിപ്പിക്കുന്നത് ഭബുദ്ധിമാന്‍മാരേ, തുല്യശിക്ഷ നല്‍കുന്നതിലാണ് നിങ്ങളുടെ ജീവിതത്തിന്റെ നിലനില്‍പ്’ എന്ന് പറഞ്ഞുകൊണ്ടാണെന്ന വസ്തുത ശ്രദ്ധേയമാണ്. കുറ്റവാളികള്‍ ശിക്ഷിക്കപ്പെടാതിരിക്കുകയും സമൂഹത്തില്‍ അവര്‍ക്ക് സൈ്വര്യവിഹാരം നടത്തുവാന്‍ അവസരമുണ്ടാവുകയും ചെയ്താല്‍ സമാധാനപൂര്‍ണമായി ജീവിതം സാധ്യമാകാത്ത സ്ഥിതിയാണ് സംജാതമാവുകയെന്നും അതുകൊണ്ട് തന്നെ പ്രതിക്രിയ െചയ്യേണ്ടത് സമൂഹത്തിന്റെ ആവശ്യമാണെന്നും വ്യക്തമാക്കുകയാണ് ഈ സൂക്തങ്ങള്‍ ചെയ്യുന്നത്. നാട്ടില്‍ കലാപങ്ങളും കുഴപ്പങ്ങളുമുണ്ടാക്കുകയും സൈ്വര്യ ജീവിതത്തെ താറുമാറാക്കുകയും ചെയ്യുന്നവര്‍ക്കും നിഷ്‌കൃഷ്ടമായ ശിക്ഷകള്‍ തന്നെ നല്‍കണമെന്നാണ് ക്വുര്‍ആനിന്റെ അനുശാസന: ''അല്ലാഹുവോടും അവെന്റ ദൂതനോടും പോരാടുകയും, ഭൂമിയില്‍ കുഴപ്പമുണ്ടാക്കാന്‍ ശ്രമിക്കുകയും ചെയ്യുന്നവര്‍ക്കുള്ള പ്രതിഫലം അവര്‍ കൊന്നൊടുക്കപ്പെടുകയോ, ക്രൂശിക്കപ്പെടുകയോ, അവരുടെ കൈകളും കാലുകളും എതിര്‍വശങ്ങളില്‍ നിന്നായി മുറിച്ചുകളയപ്പെടുകയോ, നാടുകടത്തപ്പെടുകയോ ചെയ്യുക മാത്രമാകുന്നു. അതവര്‍ക്ക് ഇഹലോകത്തുള്ള അപമാനമാകുന്നു. പരലോകത്ത് അവര്‍ക്ക് കനത്ത ശിക്ഷയുമുണ്ടായിരിക്കും. എന്നാല്‍, അവര്‍ക്കെതിരില്‍ നടപടിയെടുക്കാന്‍ നിങ്ങള്‍ക്ക് കഴിയുന്നതിന്റെ മുമ്പായി പശ്ചാത്തപിച്ച് മടങ്ങിയവര്‍ ഇതില്‍ നിന്നൊഴിവാണ്. അല്ലാഹു ഏറെ പൊറുക്കുന്നവനും കരുണാനിധിയുമാണെന്ന് നിങ്ങള്‍ മനസ്സിലാക്കുക.'' (5:33,34)

കൊലപാതകങ്ങളും കുഴപ്പങ്ങളുമില്ലാത്ത ഒരു സമൂഹത്തിന്റെ സൃഷ്ടിയാണ് ഇസ്‌ലാം വിഭാവനം ചെയ്യുന്നത്. കൊലയാളിയെയും കുഴപ്പക്കാരെയും കലാപകാരികളെയും സൈ്വര്യമായ സാമൂഹ്യ ജീവിതത്തെ തകര്‍ക്കുന്നവരെയും മാതൃകാപരമായി ശിക്ഷിക്കണമെന്ന് പറയുമ്പോള്‍ അതുവഴി ഇത്തരം പ്രശ്‌നങ്ങളൊന്നുമില്ലാത്ത സമാധാനപൂര്‍ണമായ സാമൂഹ്യാന്തരീക്ഷം സൃഷ്ടിക്കണമെന്ന ലക്ഷ്യമാണ് ഇസ്‌ലാമിനുള്ളത്. ആദർശവിരോധികളെയെല്ലാം കൊന്നൊടുക്കണമെന്നല്ല, മുസ്ലിംകളുമായി യുദ്ധത്തിന് വരുന്നവർക്കെതിരെ സായുധമായി പോരാടണമെന്നാണ് ഇസ്‌ലാമിന്റെ കല്പന. ഭരണാധികാരിയുടെ കല്പനപ്രകാരമുള്ള വധശിക്ഷയും യുദ്ധത്തിൽ ശത്രുവിനെതിയുള്ള സായുധമുന്നേറ്റവുമൊഴിച്ചുള്ള സന്ദര്ഭങ്ങളിലൊന്നും ആർക്കെതിരെയും ആയുധമെടുക്കുവാൻ ഇസ്‌ലാം വിശ്വാസികളെ അനുവദിച്ചിട്ടില്ല. ഇസ്‌ലാമിനെതിരില്‍ യുദ്ധം ചെയ്യുകയും നാട്ടില്‍ കുഴപ്പങ്ങളുണ്ടാക്കുകയും ചെയ്തവർ പോലും പിടിക്കപ്പെടുന്നതിന് മുമ്പ്  പശ്ചാത്തപിക്കുകയും തങ്ങളുടെ നിലപാട് നന്നാക്കിത്തീര്‍ക്കുകയും ചെയ്താല്‍ അവര്‍ ശിക്ഷയില്‍ നിന്ന് ഒഴിവാണെന്ന് പ്രത്യേകം എടുത്തുപറഞ്ഞിരിക്കുന്നത് അതുകൊണ്ടാണ്. ശിക്ഷിക്കുകയല്ല, സമാധാനപൂര്‍ണമായ സാമൂഹ്യാന്തരീക്ഷം സൃഷ്ടിക്കുകയാണ് ഇസ്‌ലാമിക നിയമങ്ങളുടെ ലക്ഷ്യം. കുഴപ്പങ്ങളും കലാപകങ്ങളും കൊലപാതകങ്ങളും അധാര്‍മികതകളും ഇല്ലാത്ത എല്ലാവര്‍ക്കും ശാന്തിയോടെ ജീവിക്കാനാവുന്ന ഒരു സമൂഹത്തിന്റെ സൃഷ്ടി.

പീഡനങ്ങളുടെ മക്കാനാളുകളില്‍ പ്രവാചകന്‍(സ)സായുധമായി പ്രതികരിക്കാതിരുന്നത് ശക്തിയില്ലാത്തതിനാലായിരുന്നുവെന്നും മദീനയിലെത്തി താന്‍ ശക്തനാണെന്ന് തോന്നിയതു മുതല്‍ക്കാണ് സായുധപ്രതികരണങ്ങളും അതിക്രമങ്ങളുമാരംഭിച്ചതെന്നും വിമര്‍ശിക്കുന്നവര്‍ രണ്ടുസ്ഥലത്തെയും നബി(സ)യുടെ ഉത്തരവാദിത്തങ്ങളിലുള്ള വ്യത്യാസത്തെക്കുറിച്ച് നിശ്ശബ്ദരാവുകയാണ് ചെയ്യുന്നത്. രഹസ്യമായി ഇസ്‌ലാമികപ്രബോധനം നടന്നിരുന്ന ആദ്യകാലത്ത് മുസ്‌ലിംകള്‍ തീരെ ദുര്‍ബലരായിരുന്നുവെന്നും തിരിച്ചടിക്കുന്നതിനെക്കുറിച്ച് ചിന്തിക്കുവാന്‍ പോലും കഴിയാത്തവരായിരുന്നുവെന്നും പറയുന്നത് വാസ്തവമാണ്.എന്നാൽ മക്കാകാലം മുഴുവൻ തിരിച്ചടിയെക്കുറിച്ച് ആലോചിക്കാൻ പോലും കഴിയാത്തവിധം മുസ്ലിംകൾ ദുര്ബലരായിരുന്നുവെന്ന് പറയുന്നത് ശരിയല്ല. തിരിച്ചടിയെപ്പറ്റി പ്രവാചകാനുചരന്മാർ പോലും ചിന്തിച്ചിരുന്ന സന്ദർഭങ്ങൾ മക്കാകാലഘട്ടത്തിലുണ്ടായിരുന്നു എന്നതാണ് നേര്.

ധീരവീരശൂര പരാക്രമികളായി അറിയപ്പെട്ടിരുന്ന ഹംസ(റ)യു  ഉമറിന്റെയും(റ) ഇസ്‌ലാം സ്വീകരണത്തോടെത്തന്നെ തങ്ങളുടെ ആദർശം പരസ്യമായി പ്രഖ്യാപിക്കുവാനുള്ള ധൈര്യം മുസ്ലിംകൾക്കുണ്ടാകാൻ തുടങ്ങിയിരുന്നു. കഅ്ബാലയത്തിനു മുമ്പില്‍ വെച്ച് പരസ്യമായി നമസ്കരിക്കുവാനും  ഇസ്‌ലാമിനെക്കുറിച്ച് ഉറക്കെ ചര്‍ച്ച ചെയ്യുവാനും സത്യമതത്തിലേക്ക് പൊതുസ്ഥലങ്ങളില്‍ വെച്ച് ആളുളെ ക്ഷണിക്കുവാനുമെല്ലാമുള്ള ധൈര്യം അതിന്നുശേഷം മുസ്‌ലിംകള്‍ പ്രകടിപ്പിക്കാൻ തുടങ്ങി. . മക്കയില്‍ തങ്ങളെ പ്രയാസപ്പെടുത്തിക്കൊണ്ടിരിക്കുന്നവര്‍ക്കെതിരെ ഒരു കലാപം അഴിച്ചുവിടുവാന്‍ പ്രവാചകന്‍(സ)കല്‍പിച്ചാല്‍ അത് ശിരസ്സാവഹിക്കുവാന്‍ അവര്‍ക്ക് കഴിയുമായിരുന്നു. ഉമറിനെ(റ)പ്പോലുള്ളവരുടെ നേതൃത്വത്തില്‍ അത്തരമൊരു കലാപമുണ്ടായാല്‍ അതുമൂലം മക്കാമുശ്‌രിക്കുകള്‍ക്ക് വമ്പിച്ച നാശനഷ്ടങ്ങളുണ്ടാക്കുവാനും അത് നിമിത്തമാകുമായിരുന്നു. എന്നാല്‍ തങ്ങള്‍ ജീവിക്കുന്ന നാട്ടില്‍ കുഴപ്പുമുണ്ടാക്കുവാനല്ല, പ്രത്യുത നിലനില്‍ക്കുന്ന സാമൂഹ്യസാഹചര്യങ്ങളുടെ സാധ്യതകളുപയോഗിച്ച് തങ്ങള്‍ അനുഭവിച്ചുകൊണ്ടിരിക്കുന്ന പീഡനങ്ങള്‍ക്ക് അറുതിവരുത്തുവാനും തങ്ങള്‍ക്ക് മുസ്‌ലിംകളായി നിലനില്‍ക്കുവാനുമുള്ള സാഹചര്യങ്ങളൊരുക്കുവാനാണ് പ്രവാചകന്‍(സ)പരിശ്രമിച്ചത്. കലാപങ്ങള്‍ സമൂഹത്തിന്റെ സുസ്ഥിതിയെ ദോഷകരമായി ബാധിക്കുമെന്നതുകൊണ്ടാവാം, അത്തരമൊരു മുന്നേറ്റത്തിന് പ്രവാചകന്‍(സ)  പ്രചോദനം നല്‍കാതിരുന്നത്.

സായുധസന്നാഹങ്ങളും സംഘട്ടനങ്ങളുമെല്ലാം നന്നായി അറിയുവന്നവരായിരുന്നു മുസ്‌ലിംകളായിത്തീര്‍ന്ന മക്കയിലുള്ളവര്‍. ഗോത്രാഭിമാനത്തിനുവേണ്ടി തലമുറകള്‍ നീണ്ട യുദ്ധം ചെയ്തു പരിചയമുള്ള അവര്‍ക്ക് ശത്രുവിനെതിരെ സായുധസമരം നടത്തുന്നതിനുള്ള നിര്‍ദേശം മാത്രം മതിയായിരുന്നു, തങ്ങളെ പീഡിപ്പിക്കുന്നവര്‍ക്കെതിരെ ആഞ്ഞടിക്കുവാന്‍. ഉമര്‍(റ)ഇസ്‌ലാം സ്വീകരിച്ചതോടെ മുശ്‌രിക്കുകള്‍ പോലും തിരിച്ചടി പ്രതീക്ഷിച്ചിരുന്നുവെന്ന് അവരുടെ പ്രതികരണങ്ങളില്‍ നിന്ന് വ്യക്തമാകുന്നുണ്ടെന്ന വസ്തുത ഹദീഥ് ഗ്രൻഥങ്ങളിൽ നിന്ന് മനസ്സിലാകുന്നുണ്ട്. സ്വഹാബിമാരില്‍ പലരും അത്തരമൊരു തിരിച്ചടി ആഗ്രഹിക്കുകയും അതാണ് തങ്ങള്‍ക്ക് അഭിമാനകരമായ അസ്തിത്വം പ്രദാനം ചെയ്യുകയെന്ന് കരുതുകയും ചെയ്തുവെങ്കിലും പ്രവാചകന്‍(സ)അതിന് അനുവദിച്ചില്ല. ഒരു ഹദീഥ് നോക്കുക: ഇബ്‌നു അബ്ബാസില്‍ നിന്ന്: മക്കയില്‍ വെച്ച് അബ്ദുര്‍റഹ്മാനു ബ്‌നു ഔഫും അദ്‌ദേഹത്തിന്റെ സഖാക്കളും കൂടി പ്രവാചകന്റെ(സ)യടുക്കല്‍ ചെന്നു ചോദിച്ചു: 'ദൈവദൂതരേ, ഞങ്ങള്‍ ബഹുദൈവാരാധകരായിരുന്നപ്പോള്‍ ഞങ്ങള്‍ ആത്മാഭിമാനമുള്ളവരും ആദരിക്കപ്പെട്ടവരുമായിരുന്നു; വിശ്വാസികളായേതാടെ ഞങ്ങള്‍ അധഃസ്ഥിതരും അടിച്ചമര്‍ത്തപ്പെടുന്നവരുമായിത്തീര്‍ന്നു.' പ്രവാചകന്‍(സ)പറഞ്ഞു: 'ഞാന്‍ ക്ഷമിക്കുവാനാണ് കല്‍പിക്കപ്പെട്ടരിക്കുന്നത്; അതിനാല്‍ നിങ്ങള്‍ സായുധ സമരം നടത്തരുത്.'(സുനനുന്നസാഈ, കിതാബുല്‍ ജിഹാദ്, മുഹമ്മദ് ബിന്‍ അലി ബിന്‍ അല്‍ ഹസനില്‍ നിന്ന് ഇമാം ത്വബ്‌രി തന്റെ തഫ്‌സീറില്‍ (5/108) ഉദ്ധരിച്ച ഈ ഹദീഥ് സ്വഹീഹാണെന്ന്ഇമാം ഹാകിം (2/66) വ്യക്തമാക്കിയിട്ടുണ്ട്. Hafiz Abu Tahir Zubair Ali Za'i (Ed& Ref) English Translation of Sunan An-Nasai, Riyadh, 2008, Page 16)

ആളുകളുടെ എണ്ണക്കുറവാണ് പ്രവാചകനെ(സ) മക്കയില്‍വെച്ച് സായുധപ്രതികരണങ്ങളില്‍ നിന്ന് തടഞ്ഞുനിര്‍ത്തിയതെന്ന വിമര്‍ശനവും അടിസ്ഥാനരഹിതമാണ്. എണ്ണക്കുറവ് മുസ്‌ലിംകളെ സായുധ സമരത്തില്‍ നിന്ന് തടഞ്ഞുനിര്‍ത്തുകയില്ലെന്ന് ബദ്‌റിന്റെ ചരിത്രം പഠിച്ചാല്‍ ആര്‍ക്കും ബോധ്യപ്പെടും. മുസ്‌ലിം സൈന്യത്തിന്റെ എണ്ണം ശത്രുക്കളുടേതിന്റെ മൂന്നിലൊന്ന് മാത്രമാണന്ന വസ്തുതയോ അവരുടെയത്ര ആയുധങ്ങളോ വാഹനങ്ങളോ തങ്ങളുടെ പക്കലില്ലെന്ന ബോധമോ മുസ്‌ലിംകളെ ബദ്‌റില്‍ നിന്ന് പിന്തിരിപ്പിച്ചില്ലെങ്കില്‍ മക്കയില്‍ വെച്ച് മാത്രം തങ്ങള്‍ എണ്ണത്തില്‍ കുറവാണെന്ന ബോധം അവരെ സായുധ പ്രതികരണത്തില്‍ നിന്ന് തടഞ്ഞുവെന്ന് കരുതുന്നതില്‍ ന്യായമില്ല. അല്ലാഹുവിന്റെ അനുഗ്രഹമുണ്ടെങ്കില്‍ ചെറിയ സംഘങ്ങള്‍ക്ക് വലിയ സംഘങ്ങളെ തോല്‍പിക്കാനാകുമെന്ന വസ്തുത ക്വുര്‍ആന്‍ വ്യക്തമാക്കുന്നുമുണ്ട്. ''എത്രയെത്ര ചെറിയ സംഘങ്ങളാണ് അല്ലാഹുവിന്റെ അനുമതിയോടെ വലിയ സംഘങ്ങളെ കീഴ്‌പെടുത്തിയിട്ടുള്ളത്! അല്ലാഹു ക്ഷമിക്കുന്നവരുടെ കൂടെയാകുന്നു.'' (2:249)

വിശ്വാസികള്‍ക്ക് തങ്ങളെക്കാള്‍ പത്തിരട്ടി വരുന്നവരെ നേരിടാനാകുമെന്നാണ് ക്വുര്‍ആന്‍ നല്‍കുന്ന ഉല്‍ബോധനം. ''നബിയേ, നീ വിശ്വാസികളെ യുദ്ധത്തിന് പ്രോത്‌സാഹിപ്പിക്കുക. നിങ്ങളുടെ കൂട്ടത്തില്‍ ക്ഷമാശീലരായ ഇരുപത് പേരുണ്ടായിരുന്നാല്‍ ഇരുനൂറ് പേരെ അവര്‍ക്ക് ജയിച്ചടക്കാവുന്നതാണ്. നിങ്ങളുടെ കൂട്ടത്തില്‍ നൂറ് പേരുണ്ടായിരുന്നാല്‍ സത്യനിഷേധികളില്‍ നിന്ന് ആയിരം പേരെ അവര്‍ക്ക് ജയിച്ചടക്കാവുന്നതാണ്. അവര്‍ കാര്യം ഗ്രഹിക്കാത്ത ഒരു ജനവിഭാഗമാണ് എന്നതുകൊണ്ടത്രെ അത്.'' (8:65).

അനുയായികളുടെ എണ്ണക്കുറവോ മുശ്‌രിക്കുകളെ തങ്ങള്‍ക്ക് തോല്‍പിക്കാന്‍ കഴികയില്ലെന്ന തിരിച്ചറിവോ അനുചരന്‍മാര്‍ യുദ്ധസന്നദ്ധരല്ലാത്തതോ അല്ല മക്കയിലെ പീഡകര്‍ക്കെതിരെ ഒരു സായുധ കലാപത്തിന് ആഹ്വാനം ചെയ്യുന്നതില്‍ നിന്ന് മുഹമ്മദ് നബി(സ)യെ തടഞ്ഞു നിര്‍ത്തിയതെന്ന് ഇതില്‍ നിന്ന് വ്യക്തമാണ്. ജനിച്ചു വളര്‍ന്ന നാട്ടില്‍ കലാപമുണ്ടാക്കുകയോ നാട്ടിലെ സമാധാനാന്തരീക്ഷം തകര്‍ത്ത് തന്റെ ആധിപത്യം സ്ഥാപിക്കുകയോ തങ്ങളെ എതിര്‍ത്തവരെയെല്ലാം സായുധമായി സംഹരിക്കുകയോ ആയിരുന്നില്ല അദ്‌ദേഹത്തിന്റെ ലക്ഷ്യം. പ്രത്യുത സത്യമതത്തിലേക്ക് ജനങ്ങളെ ക്ഷണിക്കുകയും അങ്ങനെ അവര്‍ക്ക് സമാധാന സംതൃപ്തമായ ഇഹലോക ജീവിതവും ശാന്തസുന്ദരമായ മരണാനന്തര ജീവിതവും നേടിയെടുക്കുവാനുള്ള മാര്‍ഗമൊരുക്കുകയുമായിരുന്നു അദ്‌ദേഹം ചെയ്തത്. തന്നെ പീഡിപ്പിക്കുന്നവര്‍ക്കെതിരെ കലാപമുണ്ടാക്കാതെയും തന്നെ നശിപ്പിക്കാന്‍ ശ്രമിച്ചവരെ നശിപ്പിക്കുവാന്‍ ശ്രമിക്കാതെയും തന്നിലേര്‍പിക്കപ്പെട്ട ദൗത്യം നിര്‍വഹിക്കുകയായിരുന്നു മക്കയില്‍ വെച്ച് പ്രവാചകന്‍(സ) ചെയ്തത്. ആളോ അര്‍ഥമോ ഇല്ലാത്തതുകൊണ്ടല്ല ജീവിക്കുന്ന നാട്ടില്‍ കുഴപ്പങ്ങളുണ്ടാക്കുവാന്‍ ദൈവകല്‍പനയില്ലാത്തതിനാലാണ് മക്കയില്‍വെച്ച് സായുധ കലാപത്തിന് അദ്ദേഹം മുതിരാതിരുന്നത്. ത്യാഗങ്ങള്‍ സഹിച്ചും പീഡനങ്ങളില്ലാതെയാക്കുവാന്‍ പ്രാര്‍ഥിച്ചും നിയതമായ മാര്‍ഗങ്ങളിലൂടെ പ്രവര്‍ത്തിച്ചും കൊണ്ട് തന്നിലര്‍പിക്കപ്പെട്ട ആദര്‍ശ പ്രചരണമെന്ന ദൗത്യം നിര്‍വഹിക്കുന്നതില്‍ മാത്രമാണ് മക്കയില്‍ വെച്ച് മുഹമ്മദ് നബി(സ) ബദ്ധശ്രദ്ധനായത്. അതായിരുന്നു മക്കയിലെ ജിഹാദ്; ക്വുര്‍ആന്‍ പറഞ്ഞ, ക്വുര്‍ആനുപയോഗിച്ചുകൊണ്ടുള്ള ഏറ്റവും വലിയ ജിഹാദ്.

മുസ്ലിമിന്റെ ജീവിതത്തിൽ നിന്ന് ഒഴിച്ചുനിർത്താനാത്ത കാര്യമാണ് ജിഹാദ് എന്നാണ് ഖുർആനും നബിവചനങ്ങളും വ്യക്തമാക്കുന്നത്. നബിജീവിതത്തിന്റെ തുടക്കം മുതൽ തന്നെ ജിഹാദ് നിറഞ്ഞുനിന്നിരുന്നുവെന്നതാണ് സത്യം. ജിഹാദിനെക്കുറിച്ച ക്വുര്‍ആന്‍ നിര്‍ദേശങ്ങള്‍ അവതരിക്കുവാനാരംഭിച്ചത് മക്കയില്‍വെച്ചാണ്. പീഡനങ്ങളും പ്രയാസങ്ങളും സഹിച്ച് ഇസ്‌ലാമിക പ്രബോധനം നിര്‍വഹിച്ചുകൊണ്ടിരിക്കുന്നതിനിടയില്‍ അവതരിപ്പിക്കപ്പെട്ട ക്വുര്‍ആന്‍ സൂക്തങ്ങളില്‍ ജിഹാദിന് പ്രേരിപ്പിക്കുന്ന വചനങ്ങള്‍ കാണാം. മക്കയില്‍വെച്ച് അവതരിപ്പിക്കപ്പെട്ട സൂറത്തുല്‍ അന്‍കബൂത്തിലെ അവസാനത്തെ വചനം കാണുക: ''നമ്മുടെ മാര്‍ഗത്തില്‍ ജിഹാദില്‍ ഏര്‍പെട്ടവരാരോ, അവരെ നമ്മുടെ വഴികളിലേക്ക് നാം നയിക്കുക തന്നെ ചെയ്യുന്നതാണ്. തീര്‍ച്ചയായും അല്ലാഹു സദ്‌വൃത്തരോടൊപ്പമാകുന്നു'' (29:69).

മക്കയില്‍വെച്ച് ചെയ്യേണ്ട ജിഹാദ് എന്താണ്?

പൂര്‍ണമായും മക്കയില്‍വെച്ച് അവതരിച്ചതെന്ന് വ്യാഖ്യാതാക്കള്‍ വ്യക്തമാക്കിയ സൂറത്തുല്‍ ഫുര്‍ഖാനിലെ 52-ാമത്തെ സൂക്തം ഈ ചോദ്യത്തിനുള്ള കൃത്യമായ ഉത്തരം നല്‍കുന്നുണ്ട്: ''അതിനാല്‍ സത്യനിഷേധികളെ നീ അനുസരിച്ചു പോകരുത്. ഇത് (ക്വുര്‍ആന്‍) കൊണ്ട് നീ അവരോട് വലിയൊരു ജിഹാദ് നടത്തിക്കൊള്ളുക'' (25:52)

ക്വുര്‍ആന്‍ ഉപയോഗിച്ച് ശത്രുക്കളോട് മഹത്തായ ജിഹാദ് നടത്തുക! ഏതാണീ ജിഹാദ്? ക്വുര്‍ആനുപയോഗിച്ചു കൊണ്ടുള്ള ജിഹാദ് ആശയസമരമാണെന്ന് വ്യക്തം. മുഹമ്മദ് നബി(സ)യുടെ നീണ്ട പതിമൂന്ന് വര്‍ഷത്തെ ജിഹാദിനുള്ള ആയുധം ക്വുര്‍ആനായിരുന്നു. അവതരിപ്പിക്കപ്പെടുന്ന മുറയ്ക്ക് ക്വുര്‍ആന്‍ വചനങ്ങളുപയോഗിച്ച് സമൂഹത്തില്‍ നിലനിന്നിരുന്ന തിന്‍മകള്‍ക്കും അധര്‍മങ്ങള്‍ക്കുമെതിരെ അദ്ദേഹം ജിഹാദ് നടത്തുകയായിരുന്നു. ഈ ജിഹാദാണ് മക്കയിലെ പലരുടെയും മനസ്സുകളില്‍ പരിവര്‍ത്തനത്തിന്റെ കൊടുങ്കാറ്റ് സൃഷ്ടിച്ചത്. ബഹുദൈവാരാധനയും തന്നിഷ്ടപ്രകാരമുള്ള ജീവിതവും കാരണം ശിലാഹൃദയരായിത്തീര്‍ന്നവരുടെ മനസ്സുകളില്‍ നിന്ന് ഏകദൈവാരാധനയും ദൈവിക വെളിപാടുപ്രകാരമുള്ള ജീവിതവും സൃഷ്ടിക്കുന്ന ആര്‍ദ്രതയുടെ തെളിനീരുകള്‍ അരിച്ചിറങ്ങുകയും അത് നദികളായിത്തീര്‍ന്ന് അറേബ്യയിലും പ്രാന്തപ്രദേശങ്ങളിലുമുള്ളവര്‍ക്ക് ദൈവിക ദര്‍ശനത്തിന്റെ തീര്‍ത്ഥമായിത്തീരുകയും ചെയ്തത് ക്വുര്‍ആന്‍ ഉപയോഗിച്ചുള്ള ജിഹാദ് കൊണ്ടായിരുന്നു. എത്രയെത്ര ശിലാഹൃദയരെയാണ് ക്വുര്‍ആന്‍ നിര്‍മലമാനസന്‍മാരാക്കിത്തീര്‍ത്തത്! പ്രവാചകശിഷ്യരില്‍ പ്രമുഖനായിത്തീര്‍ന്ന ഉമറുബ്‌നുല്‍ ഖത്ത്വാബിന്റെ പരിവര്‍ത്തന ചരിത്രം ഉദാഹരണമാണ്. നബിയോടുള്ള ശത്രുത കാരണം അദ്ദേഹത്തെ വധിച്ചുകളയാനായി വാളുമേന്തി പുറപ്പെട്ട ഉമറാണ് (റ) സഹോദരിയുടെ ക്വുർആൻ പാരായണം കേട്ട് അതിൽ ആകൃഷ്ടനായി നബിയുടെ (സ) ശക്തനായ അനുയായിയായിത്ത്തീർന്നത്. ക്വുര്‍ആന്‍ വചനങ്ങളുടെ വശ്യതയില്‍ പെട്ട് പ്രവാചകവിരോധികളായ മക്കാമുശ്‌രിക്കുകള്‍ പോലും അതിന്റെ കല്‍പന സ്വീകരിക്കുന്ന സ്ഥിതിക്ക് മക്കാരാജ്യം സാക്ഷിയായിട്ടുണ്ട്. നബില ക്വുര്‍ആനിലെ 53ാം അധ്യായമായ സൂറത്തുന്നജ്മ് പാരായണം ചെയ്തപ്പോള്‍ സ്രഷ്ടാവിന് സാഷ്ടാംഗം നമിക്കുവാനുള്ള അതിലെ കല്‍പനക്ക് വിധേയമായി അദ്ദേഹവും അനുചരന്‍മാരും സാഷ്ടാംഗം ചെയ്തപ്പോള്‍ അതോടൊപ്പം പാരായണം കേട്ടുനിന്ന മുശ്‌രിക്കുകയും സാഷ്ടാംഗം ചെയ്തതായി പ്രബലമായ ഹദീഥുകളില്‍ കാണാതാവും. പരിശുദ്ധ ക്വുര്‍ആനിന് ജനഹൃദയങ്ങളില്‍ മാറ്റമുണ്ടാക്കുവാനുള്ള അപാരമായ ശേഷിയുണ്ടെന്ന് മക്കാമുശ്‌രിക്കുകള്‍ക്ക് അറിയാമായിരുന്നു. അതുകൊണ്ടു തന്നെ തങ്ങളുടെ ഭാര്യമാരും മക്കളുമൊന്നും ക്വുര്‍ആന്‍ പാരായണം കേള്‍ക്കരുതെന്ന് അവര്‍ ആഗ്രഹിക്കുകയും അതിന്നായുള്ള കരുതല്‍ നടപടികളെടുക്കുകയും ചെയ്തു. സത്യസന്ധരും സദ്‌വൃത്തരുമായ മുസ്‌ലിംകള്‍ മക്കയില്‍ തന്നെയുണ്ടാകണമെന്നാണ്, അവരെ പീഡിപ്പിച്ചുകൊണ്ടിരുന്നപ്പോള്‍ പോലും മക്കാമുശ്‌രിക്കുകള്‍ ആഗ്രഹിച്ചതെങ്കിലും അവരുടെ ക്വുര്‍ആന്‍ പാരായണം ജനഹൃദയങ്ങളെ മാറ്റിമറിക്കുമെന്ന് അവര്‍ ഭയപ്പെട്ടു. മുസ്‌ലിംകള്‍ക്ക് സംരക്ഷണം നല്‍കിയവരോട് അവര്‍ ഉറക്കെ ക്വുര്‍ആന്‍ പാരായണം ചെയ്യരുതെന്നും അതുകേട്ട് തങ്ങളുടെ നാട്ടുകാരും കുടുംബക്കാരും അതിന്റെ ആശയങ്ങളിലേക്ക് ആകൃഷ്ടരാകാന്‍ ഇടയാകരുതെന്നും അവര്‍ നിബന്ധനവെക്കുവാന്‍ കാരണമതായിരുന്നു

ക്വുര്‍ആനിന്റെ സ്വാധീനവലയത്തില്‍ പെടാത്തവരായി മക്കയില്‍ ആരും തന്നെയുണ്ടായിരുന്നില്ല. തങ്ങള്‍ക്ക് ലഭിച്ചുകൊണ്ടിരിക്കുന്ന സൗകര്യങ്ങള്‍ ഇല്ലാതെയാവുകയും സ്ഥാനമാനങ്ങള്‍ നഷ്ടപ്പെടുകയും ചെയ്യുമോയെന്ന ഭയമായിരുന്നു സത്യമതം സ്വീകരിക്കുന്നതില്‍ നിന്ന് അവരെ തടഞ്ഞു നിര്‍ത്തിയത്. ഖുറൈശി നേതാവും കവിയും സാഹിത്യകാരനുമായിരുന്ന വലീദുബ്‌നു മുഗീറ തന്നെ ഉദാഹരണം. ക്വുര്‍ആനില്‍ ആകൃഷ്ടനായ അദ്ദേഹം, പക്ഷേ, തനിക്ക് ലഭിച്ചുകൊണ്ടിരിക്കുന്ന സ്ഥാനമാനങ്ങള്‍ നഷ്ടപ്പെടുമോയെന്ന ഭയം കാരണം അതിനെ നിഷേധിക്കുവാനും തള്ളിപ്പറയുവാനും ധൃഷ്ടനായി. സൂറത്തുല്‍ മുദ്ദഥിറിലെ പതിനൊന്ന് മുതല്‍ മുപ്പതു വരെയുള്ള വചനങ്ങളുടെ അവതരണ പശ്ചാത്തലത്തെക്കുറിച്ച് വിവരിക്കുന്ന വ്യാഖ്യാതാക്കള്‍ ഈ സംഭവം ഉദ്ധരിക്കുന്നുണ്ട്. അതിന്റെ സംക്ഷിപ്തം ഇങ്ങനെയാണ്: ഖുറൈശികള്‍ക്കിടയിലെ ബനൂമഖ്‌സൂം ഗോത്രത്തിലെ ധനാഢ്യനും നേതാവും സാഹിത്യകാരനുമായിരുന്ന വലീദുബ്‌നു മുഗീറ 'ഖുറൈശികളുടെ സുഗന്ധച്ചെടി' (റൈഹാനത്തു ഖുറൈശ്) എന്ന പേരില്‍ പ്രസിദ്ധനായിരുന്നു. അദ്ദേഹം ഒരിക്കല്‍ മുഹമ്മദ് നബിയെ സമീപിച്ചു. പ്രവാചകന്‍ല വലീദിന് ക്വുര്‍ആനിലെ 41ാം അധ്യായമായ 'ഹാമിം സജദഃ' പാരായണം ചെയ്തു കേള്‍പിച്ചു. ക്വുര്‍ആന്‍ വചനങ്ങളില്‍ ആകൃഷ്ടനായ അദ്ദേഹം അതിനെ പുകഴ്ത്തിക്കൊണ്ടുള്ള പ്രസ്താവന നടത്തി. ഈ വിവരമറിഞ്ഞ, നബിലയുടെ കഠിന ശത്രുവായിരുന്ന അബൂജഹല്‍ അദ്ദേഹത്തെ പരിഹസിക്കുകയും മുഹമ്മദ് നബി(സ)യെയും ക്വുര്‍ആനിനെയും വിമര്‍ശിച്ചു പറഞ്ഞിട്ടില്ലെങ്കില്‍ ജനങ്ങളെല്ലാം അദ്ദേഹത്തിന് എതിരാകുമെന്ന് ബോധ്യപ്പെടുത്തുകയും ചെയ്തു. അപ്പോള്‍ അദ്ദേഹം പറഞ്ഞു: 'അവനെപ്പറ്റി ഞാനെന്തു പറയാനാണ്? അല്ലാഹുവാണ! എന്നെക്കാള്‍ കവിതയും പദ്യവും പാട്ടും ജിന്നുകളുടെ കാവ്യവുമറിയുന്നവരായി നിങ്ങളില്‍ ഒരാളും തന്നെയില്ല. അല്ലാഹുവാണ! അവന്‍ പറയുന്ന വചനത്തിനൊരു മാധുര്യമുണ്ട്. അതിനു താഴെയുള്ളതെല്ലാം അതു ചവിട്ടിത്താഴ്ത്തുക തന്നെ ചെയ്യും. അത് ഉന്നതമായിത്തീരും. അതിനുപരിയായി ഒന്നും നിലകൊള്ളുകയില്ല'. മുഹമ്മദ് നബി(സ)യെയും ക്വുര്‍ആനിനെയും നിന്ദിച്ചുകൊണ്ട് എന്തെങ്കിലും പറയണമെന്ന അബൂജഹ്‌ലിന്റെ നിര്‍ബന്ധത്തിന് വഴങ്ങി അവസാനം 'ക്വുര്‍ആന്‍ ഒരു മാരണമാണെന്ന്' പറഞ്ഞ് രക്ഷപ്പെടുകയാണ് വലീദ് ചെയ്തത്. ക്വുര്‍ആനിന്റെ വശ്യതയിലും ആശയ ഗാംഭീര്യത്തിലും ആകൃഷ്ടരായി ഇസ്‌ലാമില്‍ എത്തിച്ചേര്‍ന്ന നിരവധിപേരെക്കുറിച്ച് ചരിത്രം നമുക്ക് പറഞ്ഞുതരുന്നുണ്ട്. ക്വുര്‍ആനിന് മറുപടിയെഴുതാന്‍ വേണ്ടിയുള്ള ശ്രമത്തിനിടയില്‍ ഒരു മുസ്‌ലിം ബാലന്‍ സൂറത്തുല്‍ ഹൂദിലെ 44-ാമത്തെ വചനം പാരായണം ചെയ്യുന്നത് കേട്ട് മനഃപരിവര്‍ത്തനമുണ്ടാവുകയും ''അല്ലാഹുവാണെ! ക്വുര്‍ആനിന് മറുപടിയെഴുതുക അസാധ്യമാണെന്നും അത് മനുഷ്യവചനമല്ലെന്നും ഞാന്‍ സാക്ഷ്യം വഹിക്കുന്നു''വെന്ന് പ്രഖ്യാപിക്കുകയും ചെയ്ത ജാഹിലിയ്യാകാലത്തെ അറിയപ്പെടുന്ന എഴുത്തുകാരനായിരുന്ന അബ്ദുല്ലാഹിബ്‌നു മുഖാഫ്; പ്രവാചകനില്‍നിന്ന് സൂറതുത്ത്വൂറിലെ ഏഴ്, എട്ട് വചനങ്ങള്‍ ശ്രവിച്ച് മനഃപരിവര്‍ത്തനമുണ്ടായി ഇസ്‌ലാമിലെത്തിയ ജുബൈറുബ്‌നു മുത്അം; മദീനയില്‍ പ്രവാചക പ്രതിനിധിയായെത്തിയ മുസ്അബുബ്‌നു ഉമൈറിനോട് യഥ്‌രിബുകാരായ മൃദുലഹൃദയരെ വഴിതെറ്റിക്കുന്നവിധത്തില്‍ ക്വുര്‍ആന്‍ പാരായണം ചെയ്യരുതെന്ന് ആവശ്യപ്പെട്ടെത്തിയപ്പോള്‍ മുസ്അബിന്റെ അഭ്യര്‍ഥനയെ മാനിച്ച് അല്‍പനേരം ക്വുര്‍ആന്‍ പാരായണം ശ്രവിച്ചതോടെ അതില്‍ ആകൃഷ്ടനായി ഇസ്‌ലാം സ്വീകരിച്ച ഉസൈദുബ്‌നു ഹുദൈര്‍; ഉസൈദിന്റെ നിര്‍ദേശപ്രകാരം മുസ്അബില്‍നിന്ന് ക്വുര്‍ആന്‍ ശ്രവിച്ചതോടെ അതിന്റെ അനുയായിയായിത്തീര്‍ന്ന ഔസ് ഗോത്രനേതാവ് സഅദ്ബ്‌നു മുആദ്... ഇങ്ങനെ ക്വുര്‍ആന്‍ പാരായണം കേട്ടതുകൊണ്ടുമാത്രം ഇസ്‌ലാമിലെത്തിച്ചേര്‍ന്ന നിരവധിയാളുകളെക്കുറിച്ച് ചരിത്രം നമുക്ക് പറഞ്ഞുതരുന്നുണ്ട്. മുസ്അബുബ്‌നു ഉമൈറിന്റെ േക്വുര്‍ആന്‍ പാരായണം വഴിയാണ്, വാളുകൊണ്ടല്ല മദീനയെ ഇസ്‌ലാം കീഴടക്കിയതെന്ന വസ്തുത ചരിത്രകാരന്‍മാരെല്ലാം അംഗീകരിക്കുന്നതാണ് .

പ്രവാചകന്റെ കാലത്ത് മാത്രമല്ല, അതിന് ശേഷം ഇന്നുവരെയും ക്വുര്‍ആന്‍ ജനമനസ്സുകളില്‍ മാറ്റത്തിന്റെ കൊടുങ്കാറ്റ് വിതയ്ക്കുകയെന്ന ദൗത്യം നിര്‍വഹിച്ചുകൊണ്ടേയിരിക്കുന്നുണ്ട്. ആംഗ്ലിക്കന്‍ പാതിരിയുടെ മകനായി ജനിച്ച് ക്വുര്‍ആന്‍ പഠനത്തിലൂടെ ഇസ്‌ലാമിലെത്തിച്ചേരുകയും പിന്നീട് പ്രസിദ്ധമായ ഇംഗ്ലീഷ് ക്വുര്‍ആന്‍ വിവര്‍ത്തനം രചിക്കുകയും ചെയ്ത ബ്രിട്ടീഷ് നോവലിസ്റ്റ് മുഹമ്മദ് മാര്‍മഡ്യൂക് പിക്താള്‍; യഹൂദ റബ്ബിമാരുടെ പാരമ്പര്യമുള്‍ക്കൊള്ളുന്ന കുടുംബത്തില്‍ ജനിക്കുകയും ക്വുര്‍ആന്‍ പഠനത്തിലൂടെ ഇസ്‌ലാമിലെത്തിച്ചേര്‍ന്ന് മുഹമ്മദ് അസദ് എന്ന പേര് സ്വീകരിക്കുകയും സ്വന്തമായി ഇംഗ്ലീഷിലുള്ള ക്വുര്‍ആന്‍ വിവരണം രചിക്കുകയും ചെയ്ത ഓസ്ട്രിയന്‍ സഞ്ചാരിയും യഹൂദ പണ്ഡിതനുമായിരുന്നു ലിയോ പോള്‍ഡ് വെയ്സ്സ്. ക്വുര്‍ആനിന്റെ വെളിച്ചത്തിലൂടെ ഇസ്‌ലാമില്‍ എത്തുകയും അങ്ങനെ ക്വുര്‍ആനിന്റെ വെളിച്ചം ലോകത്തിന് എത്തിച്ചുകൊടുക്കുവാന്‍ മുന്നില്‍ നടക്കുകയും ചെയ്ത രണ്ട് മഹാരഥന്‍മാര്‍-പാശ്ചാത്യന്‍ സംഗീത ലോകത്തിന്റെ ഉന്നത വിതാനത്തില്‍ വിരാജിക്കവെ ക്വുര്‍ആനിന്റെ സന്ദേശം മനസ്സിലാക്കുവാന്‍ അവസരമുണ്ടാവുകയും യൂസുഫ് ഇസ്‌ലാം എന്ന പേര് സ്വീകരിച്ച് ഇസ്‌ലാമിക പ്രബോധന രംഗത്തേക്ക് കടന്നുവരികയും ചെയ്ത കാറ്റ് സ്റ്റീവന്‍സ്; പന്ത്രണ്ടാം വയസ്സില്‍ അഭിനയരംഗത്തെത്തുകയും നിരവധി ഹോളിവുഡ് സിനിമകളില്‍ അഭിനയിക്കുകയും തന്റെ അഭിനയ ജീവിതത്തിനിടയ്ക്ക് ക്വുര്‍ആനിനെ പരിചയപ്പെട്ട് അതിന്റെ പഠനത്തിലൂടെ ഇസ്‌ലാമിലെത്തുകയും ഇപ്പോള്‍ മറിയം ഫ്രാന്‍കോയിസ് എന്ന പേര് സ്വീകരിച്ച് ക്വുര്‍ആനിന്റെ ആശയ പ്രചാരണത്തിനായി പരിശ്രമിക്കുകയും ചെയ്യുന്ന എമിനി ഫ്രാന്‍കോയ്‌സ് സിര്‍റ; ബൈബിള്‍ പഠനത്തിനിടയില്‍ അവിചാരിതമായി ക്വുര്‍ആന്‍ പരിഭാഷ വായിക്കുവാന്‍ അവസരമുണ്ടാവുകയും അത് ക്വുര്‍ആനിന്റെ പ്രചാരകനാക്കിത്തീര്‍ക്കുന്നതിലേക്ക് നയിക്കുകയും ചെയ്ത ക്രിസ്ത്യന്‍ മിഷനറി പ്രവര്‍ത്തകന്‍ ജോഷുവ ഇവാന്‍സ്, സ്വിറ്റ്‌സര്‍ലാന്റില്‍ മുസ്‌ലിംകള്‍ക്ക് പള്ളി നിര്‍മിക്കുവാന്‍ അവസരം നല്‍കരുതെന്ന് വാദിക്കുകയും അതിനായി ജനഹിതമുണ്ടാക്കിയെടുക്കാന്‍ പരിശ്രമിക്കുകയും ചെയ്ത സ്വിസ് പീപ്പിള്‍സ് പാര്‍ട്ടിയുടെ മുന്നില്‍നിന്ന് പ്രവര്‍ത്തിക്കുന്നതിനിടെ ക്വുര്‍ആനികാശയങ്ങളെപ്പറ്റി പഠിക്കുവാന്‍ അവസരമുണ്ടാകുകയും ഇസ്‌ലാം സ്വീകരിക്കുന്നതിലേക്ക് അത് നയിക്കുകയും ചെയ്ത ഡാനിയല്‍ സ്ട്രീക്ക്-അന്ധകാര നിബിഡമായ മനസ്സുകളില്‍ ക്വുര്‍ആന്‍ സൃഷ്ടിച്ചുകൊണ്ടിരിക്കുന്ന മാറ്റത്തിന് പാശ്ചാത്യനാടുകളില്‍ നിന്നും ഇന്നും ജീവിച്ചിരിക്കുന്ന ഉദാഹരണങ്ങളാണിവ. നാടോടികളും അധര്‍മങ്ങളില്‍ നിമഗ്‌നരുമായിരുന്ന അറബികളെ ലോകത്തിന് വഴി കാണിക്കുവാന്‍ പര്യാപ്തരാക്കുകയെന്ന അത്ഭുതമാണ് ക്വുര്‍ആന്‍ ഉപയോഗിച്ചുകൊണ്ടുള്ള ജിഹാദ് വഴി പ്രവാചകന്‍(സ) സൃഷ്ടിച്ചത്. അന്ധവിശ്വാസങ്ങളില്‍നിന്നും അനാചാരങ്ങളില്‍നിന്നും ആധുനികത സൃഷ്ടിക്കുന്ന പുതിയ അടിമത്തങ്ങളില്‍നിന്നും മനുഷ്യരെ രക്ഷപ്പെടുത്തുകയും വിമലീകൃതമായ വിശുദ്ധ ജീവിതംവഴി കുടുംബത്തിലും സമൂഹത്തിലുമെല്ലാം പരിവര്‍ത്തനങ്ങള്‍ ഉണ്ടാക്കുവാന്‍ കഴിയുന്നവരാക്കി അവരെ മാറ്റിയെടുക്കുകയും ചെയ്തുകൊണ്ട് ക്വുര്‍ആനിന്റെ ജിഹാദ് ഇന്നും തുടര്‍ന്നുകൊണ്ടിരിക്കുന്നു. ആയുധങ്ങളല്ല ക്വുര്‍ആനികാശയങ്ങളാണ് അന്നും ഇന്നും ജനഹൃദയങ്ങളെ മാറ്റുവാനുള്ള നിമിത്തങ്ങളായിത്തീരുന്നത്. ജനഹൃദയങ്ങളെ മാറ്റിമറിക്കുവാനാകുന്ന ക്വുര്‍ആന്‍കൊണ്ടുള്ള ജിഹാദാണ് ഏറ്റവും വലിയ ജിഹാദ് എന്നാണല്ലോ ക്വുര്‍ആന്‍തന്നെ പരിചയപ്പെടുത്തുന്നത്.

സ്വന്തം തൃഷ്ണകളോട് ചെയ്യുന്ന പോരാട്ടമാണ് ഏറ്റവും വലിയ ജിഹാദെന്നും ജീവത്യാഗമാവശ്യപ്പെടുന്ന സായുധാജിഹാദിന് പോവുന്നവർ ആദ്യം ചെയ്യേണ്ടത് സ്വന്തത്തോടുള്ള ജിഹാദാണെന്നും പ്രവാചകവചങ്ങളുടെ അടിസ്ഥാനത്തിൽ പണ്ഡിതന്മാർ വ്യക്തമാക്കിയിട്ടുണ്ട്.  ജിഹാദിനെക്കുറിച്ച് വര്‍ഗീകരിച്ചു വിശദീകരിച്ച പണ്ഡിതന്‍മാരില്‍ പ്രമുഖനാണ് ഹിജ്‌റ എട്ടാം നൂറ്റാണ്ടില്‍ മരണപ്പെട്ട ഇമാം ഇബ്‌നുല്‍ ഖയ്യിം അല്‍ ജൗസിയ്യഃ (ക്രി. 1292-1350). സ്വന്തത്തോടുള്ളത് (ജിഹാദുന്നഫ്‌സ്), ചെകുത്താനോടുള്ളത് (ജിഹാദുശ്‌ശൈത്വാന്‍), സത്യനിഷേധികളോടും കപട വിശ്വാസികളോടുമുള്ളത് (ജിഹാദുല്‍ കുഫ്ഫാറു വല്‍ മുനാഫിഖീന്‍), അനീതിയുടെയും തിന്‍മകളുടെയും അനാചാരങ്ങളുടെയും ആളുകള്‍ക്കെതിരെയുള്ളത് (ജിഹാദു അര്‍ബാബുദ്ദ്വുല്‍മി വല്‍ ബിദ്ഇ വല്‍ മുന്‍കറാത്ത്) എന്നിങ്ങനെ നാലായി ജിഹാദിനെ വര്‍ഗീകരിക്കാമെന്നാണ് അദ്ദേഹത്തിന്റെ പക്ഷം.

സ്വന്തത്തോടുള്ള ജിഹാദില്‍ നിന്നാണ് ഒരാള്‍ തന്റെ ഇസ്‌ലാമിക ജീവിതം ആരംഭിക്കുന്നത്. സ്വന്തം താല്‍പര്യങ്ങളുടെയും ആഗ്രഹങ്ങളുടെയും തൃഷ്ണയുടെയും തടവറയില്‍ നിന്ന് സ്വയം മോചിതനായി ദൈവിക നിയമങ്ങള്‍ക്ക് വിധേയനാവലാണ് സ്വന്തത്തോടുള്ള ജിഹാദ്. നാലു രൂപത്തിലാണ് ഒരാള്‍ സ്വന്തത്തോട് പോരാടുന്നത്. ദൈവിക മതത്തിന്റെ യഥാര്‍ഥമായ മാര്‍ഗദര്‍ശനത്തെക്കുറിച്ച കൃത്യമായ പഠനം, അറിവു ലഭിച്ച കാര്യങ്ങളുടെ പ്രയോഗവല്‍ക്കരണം, താന്‍ അറിഞ്ഞ കാര്യങ്ങള്‍ അത് അറിയാത്തവര്‍ക്ക് പഠിപ്പിച്ചുകൊടുക്കുക, ദൈവിക മാര്‍ഗത്തിലേക്കുള്ള ക്ഷണത്തിന്റെ പാതയില്‍ ത്യാഗങ്ങള്‍ സഹിക്കേണ്ടി വരുമ്പോള്‍ ക്ഷമിക്കുക എന്നിവയാണ് ജിഹാദുന്നഫ്‌സിന്റെ നാല് രൂപങ്ങള്‍.

ജിഹാദുശ്‌ശൈത്വാനിന് രണ്ടു രൂപങ്ങളാണുള്ളത്. ശരിയായ വിശ്വാസത്തെ നശിപ്പിക്കാനായി പിശാച് മനസ്സില്‍ ചെലുത്തുന്ന സംശയങ്ങളെ ദൂരീകരിക്കുകയും അതിനെതിരെ സജ്ജമാവുകയും ചെയ്യുന്നതും തെറ്റായ താല്‍പര്യങ്ങളും അനനുവദനീയമായ തൃഷ്ണകളും വളര്‍ത്തി പ്രലോഭിപ്പിക്കുവാന്‍ വേണ്ടിയുള്ള പിശാചിന്റെ പരിശ്രമങ്ങളെ ചെറുക്കുകയും അവഗണിക്കുകയും ചെയ്യുന്നതുമാണ് ജിഹാദുശ്‌ശൈത്വാനിന്റെ രൂപങ്ങള്‍. നാല് രൂപത്തിലാണ് ഒരാള്‍ ജിഹാദുല്‍ കുഫ്ഫാര്‍ വല്‍ മുനാഫിഖീന്‍ നിര്‍വഹിക്കുന്നത്. ഹൃദയം കൊണ്ടും നാവുകൊണ്ടും സമ്പത്തുകൊണ്ടും ശരീരംകൊണ്ടും നടത്തുന്നവയാണ് അവ.

അനീതിയുടെയും അനാചാരങ്ങളുടെയും തിന്‍മകളുടെയും വക്താക്കള്‍ക്കെതിരെ നടത്തുന്ന ജിഹാദിന് മൂന്ന് രൂപങ്ങളാണുള്ളത്. കൈകൊണ്ടുള്ളത്, നാവുകൊണ്ടുള്ളത്, മനസ്സുകൊണ്ടുള്ളത് എന്നിവയാണവ.(1)

ഇബ്‌നുല്‍ ഖയ്യിം വര്‍ഗീകരിച്ച നാലുതരം ജിഹാദുകളും മക്കയില്‍ വെച്ചുതന്നെ പ്രവാചകനും(സ) അനുചരന്‍മാരും പ്രവാര്‍ത്തികമാക്കിയിരുന്നുവെന്ന് കാണാനാവും. മൂന്നാമത്തെ തരമായി എണ്ണിയ സത്യനിഷേധികളോടും കപടവിശ്വാസികളോടുമുള്ള ജിഹാദില്‍, കപടവിശ്വാസികളോടുള്ള ജിഹാദ് മാത്രമാണ് മദീനയിലെത്തിയശേഷം മാത്രമായി നിര്‍വഹിക്കപ്പെട്ടത്. മക്കയിലെ പീഡിപ്പിക്കപ്പെട്ട നാളുകളില്‍ പ്രവാചകാനുചരന്‍മാരോടൊപ്പം മുനാഫിഖുകള്‍ ഉണ്ടായിരുന്നില്ല എന്നതുകൊണ്ടായിരുന്നു അത്. സ്വന്തത്തോടും ചെകുത്താനോടും സത്യനിഷേധികളോടും തിന്‍മയുടെ വാഹകരോടുമുള്ള ജിഹാദുകള്‍ മക്കയില്‍ വെച്ചുതന്നെ നിര്‍വഹിക്കുവാന്‍ തുടങ്ങിയിരുന്നു. മക്കയിലെ വ്യത്യസ്ത സാഹചര്യങ്ങളില്‍ വ്യത്യസ്ത രൂപഭാവങ്ങളായാണ് പ്രസ്തുത ജിഹാദുകള്‍ നിര്‍വഹിക്കപ്പെട്ടത് എന്നു മാത്രമെയുള്ളു.

എല്ലാ ജിഹാദുകളുടെയും അടിസ്ഥാനം ജിഹാദുന്നഫ്‌സാണെന്ന് പണ്ഡിതന്‍മാര്‍ വ്യക്തമാക്കിയിട്ടുണ്ട്. 'സത്യനിഷേധികള്‍ക്കും കപടവിശ്വാസികള്‍ക്കുമെതിരെയുള്ള ജിഹാദിന്റെ അടിത്തറയുണ്ടാകുന്നത് സ്വന്തത്തിനും സ്വന്തം ഇച്ഛകള്‍ക്കുമെതിരെയുള്ള ജിഹാദില്‍ നിന്നാണ്; സ്വന്തത്തിനും സ്വന്തം ഇച്ഛകള്‍ക്കെതിരെയുമുള്ള ജിഹാദ് ചെയ്യാത്തവര്‍ക്ക് സത്യനിഷേധികള്‍ക്കും കപടവിശ്വാസികള്‍ക്കുമെതിരെയുള്ള ജിഹാദ് ചെയ്യുവാനോ അതിന്നായി പുറപ്പെട്ടിറങ്ങുവാനോ സാധ്യമല്ല'യെന്ന് ശൈഖുല്‍ ഇസ്‌ലാം ഇബ്‌നുതൈമിയ്യഃ പറഞ്ഞതായി ഇബ്‌നുല്‍ ഖയ്യിം ഉദ്ധരിക്കുന്നുണ്ട്.(2)

ഇസ്‌ലാമികമായ ആദര്‍ശങ്ങള്‍ക്കനുസരിച്ച് സ്വന്തം ജീവിതത്തെ ചിട്ടപ്പെടുത്തുകയും സത്യമെന്ന് ഉറപ്പുള്ള ആദര്‍ശത്തെ മറ്റുള്ളവര്‍ക്ക് എത്തിച്ചുകൊടുക്കുവാനായി പ്രയത്‌നിക്കുകയും അതിന്റെ മാര്‍ഗത്തില്‍ ത്യാഗങ്ങള്‍ സഹിക്കുകയും ചെയ്യുകയെന്ന ജിഹാദുന്നഫ്‌സ് ചെയ്യാത്തവര്‍ക്ക് എങ്ങനെയാണ് പിശാചിനെ തോല്‍പിക്കാനും അവിശ്വാസികളോടും കപടവിശ്വാസികളോടും ആദര്‍ശ സമരം നടത്തുവാനും അനിവാര്യമായ അവസരത്തില്‍ ആയുധമെടുത്ത് അടരാടുവാനും കഴിയുക? എല്ലാ ജിഹാദുകളുടെയും അടിത്തറ ജിഹാദുന്നഫ്‌സ് ആണെന്നതിനാല്‍ അതിനെ 'ജിഹാദുല്‍ അക്ബര്‍' എന്ന് ചില പണ്ഡിതന്‍മാര്‍ വിളിച്ചിട്ടുണ്ട്.(3)

സ്വന്തം ഇച്ഛകള്‍ക്കും തൃഷ്ണകള്‍ക്കുമെതിരെയുള്ള പോരാട്ടമാണ് അടര്‍ക്കളത്തിലിറങ്ങിക്കൊണ്ടുള്ള യുദ്ധത്തെക്കാള്‍ മഹത്തരമെന്ന് സ്ഥാപിക്കുവാനായി നബി(സ)യില്‍ നിന്ന് ഉദ്ധരിക്കപ്പെടുന്ന ഒരു സംഭവവിവരണം അടിസ്ഥാനരഹിതമാണെന്നും നിവേദക പരമ്പര ദുര്‍ബലമാണെന്നും ഹദീഥ് പണ്ഡിതന്‍മാര്‍ ചൂണ്ടിക്കാട്ടിയിട്ടുണ്ട്. അനുചരന്‍മാര്‍ ഒരു യുദ്ധം കഴിഞ്ഞു മടങ്ങിവന്നപ്പോള്‍ 'നല്ലൊരു വരവാണ് നിങ്ങള്‍ വന്നത്; ചെറിയൊരു ജിഹാദില്‍ നിന്ന് വലിയൊരു ജിഹാദിലേക്ക്-സ്വന്തം ഇച്ഛകള്‍ക്കെതിരെ ദൈവദാസന്‍മാര്‍ നടത്തുന്ന പോരാട്ടം-ആണ് നിങ്ങള്‍ വന്നിരിക്കുന്നത്' എന്ന് പ്രവാചകന്‍(സ) പറഞ്ഞതായി ജാബിറുബ്‌നു അബ്ദുല്ല(റ)നിവേദനം ചെയ്തതായി 'താരീഖുല്‍ ബാഗ്ദാദി'യില്‍ രേഖപ്പെടുത്തിയ ഹദീഥാണത്. ഈ പദങ്ങള്‍ യഥാര്‍ഥത്തില്‍ താബിഇയായ(4) ഇബ്‌റാഹിമുബ്‌നു അബ്‌ലഹിന്റെതാണെന്ന് പ്രമുഖ ഹദീഥ് നിദാനശാസ്ത്രജ്ഞനായ അന്നസാഈ രേഖപ്പെടുത്തിയിട്ടുണ്ടെന്നും ഹദീഥ് വ്യാഖ്യാതാക്കളില്‍ പ്രസിദ്ധനായ ഇമാം ഇബ്‌നു ഹജറുല്‍ അസ്ഖലാനി രേഖപ്പെടുത്തുന്നുണ്ട്.(5) ഇമാം സുയൂത്വി തന്റെ 'ജാമിഉസ്‌സഗീറി'ലും ഇമാം ദഹബി തന്റെ 'അല്‍മീസാനിലും' ഇമാം ഇബ്‌നു തൈമിയ്യ തന്റെ 'അല്‍ഫുര്‍ഖാനു ബൈനല്‍ ഔലിയാ ഉര്‍റഹ്മാനി വ ഔലിയാഉശ്ശൈത്വാനി'ലും ഇൗ ഹദീഥ് ദുര്‍ബലമാണെന്ന് വ്യക്തമാക്കിയിട്ടുമുണ്ട്.(6)

സ്വന്തം ഇച്ഛക്കെതിരെ ദൈവദാസന്‍മാര്‍ നടത്തുന്ന പോരാട്ടമാണ് വലിയ ജിഹാദെന്ന് പ്രസ്താവിക്കുന്ന ഹദീഥിന്റെ പരമ്പര ദുര്‍ബലമായതിനാല്‍ അത് പ്രവാചകന്‍ പറഞ്ഞതാണെന്ന് ഉറപ്പിക്കുവാന്‍ കഴിയാത്തതുകൊണ്ടുതന്നെ മതവിധികള്‍ നിര്‍ധരിക്കുന്നതിന് അത് ഉപയോഗിക്കുവാന്‍ തീരെ പാടില്ലാത്തതാണെങ്കിലും, സ്വന്തത്തോടുള്ള പോരാട്ടത്തെ 'ജിഹാദുല്‍ അക്ബര്‍' എന്നും പടക്കളത്തിലിറങ്ങി നടത്തുന്ന യുദ്ധത്തെ 'ജിഹാദുല്‍ അസ്ഗര്‍' എന്നും വിളിക്കുന്ന സമ്പ്രദായം നബി(സ)ക്കു ശേഷമുള്ള രണ്ടാം തലമുറ മുതല്‍ക്കെങ്കിലും നിലനിന്നിരുന്നുവെന്ന യാഥാര്‍ഥ്യം വ്യക്തമാക്കുന്നതാണ് ഈ ഹദീഥ് എന്ന വസ്തുത വിസ്മരിച്ചു കൂടാ. ആയുധങ്ങളെടുത്ത് അടരാടാന്‍ അവസരം ലഭിച്ചവരും അത് ലഭിക്കാത്തവരുമെല്ലാം ചെയ്യേണ്ട മഹത്തായ ജിഹാദാണ് സ്വന്തം ഇച്ഛയോടുള്ള പോരാട്ടമെന്നായിരുന്നു പ്രവാചകാനുചരന്‍മാരില്‍ നിന്ന് മതം നേര്‍ക്കുനേരെ അനുഭവിച്ചറിഞ്ഞവര്‍ മനസ്സിലാക്കിയിരുന്നത് എന്നാണല്ലോ ഇത് വ്യക്തമാക്കുന്നത്.

സ്വന്തം ഇച്ഛയോട് നടത്തേണ്ട പോരാട്ടത്തിന്റെ പ്രാധാന്യം വ്യക്തമാക്കുന്ന പ്രബലമായ ഹദീഥുകള്‍ അതിനെ ജിഹാദുല്‍ അക്ബര്‍ എന്നു വിളിക്കുന്നതിനെ സാധൂകരിക്കുന്നുണ്ടെന്ന് അഭിപ്രായപ്പെടുന്നവരുണ്ട്. താന്‍ നിര്‍വഹിച്ച ഹജ്ജ് കര്‍മത്തോടനുബന്ധിച്ച് പ്രവാചകന്‍(സ)നിര്‍വഹിച്ച വിടവാങ്ങല്‍ പ്രസംഗത്തിന്റെ, ഇമാം അഹ്മദും ത്വബ്‌റാനിയും ഇബ്‌നുമാജയും ഹാക്കിമുമെല്ലാം നിവേദനം ചെയ്ത സമ്പൂര്‍ണ രൂപത്തില്‍ 'അല്ലാഹുവിനെ അനുസരിക്കുന്ന വിഷയത്തില്‍ സ്വന്തത്തോടു ജിഹാദു ചെയ്യുന്നവനാണ് മുജാഹിദ്' എന്ന് നബി(സ) പ്രസ്താവിച്ചതായി രേഖപ്പെടുത്തിയത് സ്വഹീഹാണെന്ന് ഇമാം ഇബ്‌നു ഹിബ്ബാനും ഇമാം ഹാകിമും(7) ഇമാം അല്‍ബാനിയും(8) സാക്ഷ്യപ്പെടുത്തിയിട്ടുണ്ട്. അല്ലാഹുവിനെ അനുസരിക്കുന്നതില്‍ നിന്ന് സ്വന്തത്തെ തടയുന്ന തൃഷ്ണകളോട് പോരാടി അവനോടുള്ള അനുസരണത്തെ ദൃഢീകരിച്ച് ഉറപ്പിക്കുന്നവനാണ് യഥാര്‍ഥ മുജാഹിദെന്ന് പഠിപ്പിക്കുന്ന ഈ ഹദീഥ് ജിഹാദുന്നഫ്‌സിന്റെ പ്രാധാന്യം വ്യക്തമാക്കുന്നുണ്ട്.

അല്ലാഹുവെക്കുറിച്ച സ്മരണകളും അവന്റെ പ്രീതി നേടിയെടുക്കണമെന്ന അഭിലാഷവുമാണ് മുസ്‌ലിമിനെ സ്വന്തത്തോടും മറ്റുള്ളവരോടുമെല്ലാം ജിഹാദ് ചെയ്യുവാന്‍ പ്രേരിപ്പിക്കുന്ന ഘടകം. വ്യക്തിയുടെ വിമലീകരണത്തിന്റെ അടിത്തറയാണ് ദൈവസ്മരണ. ഒരു മുസ്‌ലിം ചെയ്യുന്ന മറ്റേത് കര്‍മത്തെക്കാളും മഹത്തരമായതാണ് അല്ലാഹുവിനെക്കുറിച്ച സ്മരണയെന്ന് അഹ്മദും തിര്‍മിദിയും ഇബ്‌നുമാജയും ഹാകിമുമെല്ലാം നിവേദനം ചെയ്ത സ്വഹീഹാണെന്ന് ഇമാം അല്‍ബാനി വ്യക്തമാക്കിയ(9) ഒരു ഹദീഥ് പഠിപ്പിക്കുണ്ട്. ഹദീഥിന്റെ സാരം ഇങ്ങനെയാണ്: അബിദ്ദര്‍ദാഇല്‍ േനിന്ന്: പ്രവാചകന്‍(സ) പറഞ്ഞു: നിങ്ങളുടെ രാജാധിരാജന് ഏറ്റവും ഇഷ്ടപ്പെട്ട, നിങ്ങളുടെ പദവികളെ ഉയര്‍ത്തുന്ന, സ്വര്‍ണവും വെള്ളിയും ദാനം ചെയ്യുന്നതിനെക്കാള്‍ ഉത്തമമായ, യുദ്ധത്തില്‍ നിങ്ങളുടെ ശത്രുവിനെ കണ്ടുമുട്ടുകയും അയാളുടെ തലകൊയ്യുകയും നിങ്ങളുടെ തലകൊയ്യപ്പെടുകയും ചെയ്യുന്നതിനെക്കാള്‍ മഹത്തരമായ, അത്യുത്തമമായ കര്‍മത്തെക്കുറിച്ച് ഞാന്‍ നിങ്ങള്‍ക്ക് പറഞ്ഞുതരട്ടയോ? അതാണ് 'അല്ലാഹുവെക്കുറിച്ച സ്മരണ' (ദിക്‌റുല്ലാ).(10)

തന്റെ നാല്‍പതാമത്തെ വയസ്സില്‍ പ്രവാചകത്വം ലഭിച്ചത് മുതല്‍ അറുപത്തി മൂന്നാമത്തെ വയസ്സില്‍ ഏല്‍പിക്കപ്പെട്ട ദൗത്യം പൂര്‍ത്തിയാക്കി ഈ ലോകത്തുനിന്ന് വിട പറയുന്നതു വരെയുള്ള നബി(സ)യുടെ ജീവിതം മുഴുവന്‍ ജിഹാദ് നിറഞ്ഞുനില്‍ക്കുകയായിരുന്നു. അതില്‍ സ്വന്തത്തോടുള്ള പോരാട്ടവും പിശാചിനോടുള്ള പോരാട്ടവും സത്യനിഷേധികള്‍ക്കും കപടവിശ്വാസികള്‍ക്കുമെതിരെയുള്ള പോരാട്ടവും അനീതിയുടെയും അധര്‍മത്തിന്റെയും അക്രമത്തിന്റെയും വക്താക്കള്‍ക്കെതിരെയുള്ള പോരാട്ടവുമുണ്ടായിരുന്നു. നബി(സ)യെ പിന്‍തുടര്‍ന്ന് സത്യമതത്തിന്റെ അനുയായികളായിത്തീര്‍ന്നവരും ഈ പോരാട്ടത്തില്‍ അദ്ദേഹത്തോടൊപ്പം ചേര്‍ന്നു. ഇസ്‌ലാം സ്വീകരിച്ചതു മുതല്‍ ജീവിതാവസാനം വരെ അവര്‍ പോരാട്ടത്തിലായിരുന്നു. മക്കയില്‍ വെച്ചു തന്നെ മരണപ്പെട്ടവരും മദീനയിലെത്തിയ ശേഷം ഏറെനാള്‍ ജീവിക്കുവാന്‍ അവസരം ലഭിച്ചവരുമെല്ലാമുണ്ട് അവര്‍ക്കിടയില്‍. അവരിലൊരാളും തന്നെ ജിഹാദ് ചെയ്യാതിരുന്നിട്ടില്ല. ആദര്‍ശപ്രബോധനത്തിന്റെ ആദ്യകാലത്ത് നബി(സ)ക്ക് താങ്ങും തണലുമായി നിലനിന്ന പ്രവാചക പത്‌നി ഖദീജയും(റ)സത്യത്തിന്റെ ശത്രുക്കളുടെ ക്രൂരമായ പീഡനങ്ങള്‍ക്കു മുമ്പില്‍ ക്ഷമ അവലംബിച്ച് അവസാനം മരണത്തിന് കീഴ്‌പെട്ട യാസിറും(റ) സുമയ്യ(റ)യെയുമല്ലാം മക്കയില്‍ വെച്ച് മരണപ്പെട്ട മുജാഹിദുകളായിരുന്നു. ജീവിതത്തില്‍ ഒരിക്കലും ആയുധമേന്തുവാന്‍ അവസരം ലഭിക്കാത്ത മുജാഹിദുകള്‍. സ്വന്തത്തിനും പിശാചിനും ശത്രുക്കള്‍ക്കുമെതിരെ ഏറ്റവും വലിയ ജിഹാദ് ചെയ്തുകൊണ്ടാണ് അവരെല്ലാം മരണപ്പെട്ടത്. ക്വുര്‍ആന്‍ ഉപയോഗിച്ചു കൊണ്ടുള്ള പോരാട്ടമാണ് ഏറ്റവും വലിയ ജിഹാദ് എന്നാണല്ലോ ക്വുര്‍ആന്‍ അവരെ പഠിപ്പിച്ചത്; ത്യാഗങ്ങള്‍ സഹിച്ചുകൊണ്ടുള്ള ഇസ്‌ലാമിക ജീവിതവും ആദര്‍ശ പ്രബോധനവുമായിരുന്നു അവര്‍ നയിച്ച ജിഹാദ്.

പ്രബോധകജീവിതത്തിന്റെ ആദ്യത്തെ നാളുകള്‍ മുതല്‍ യാതൊരു വിട്ടുവീഴ്ചയുമില്ലാതെ പ്രവാചകനും(സ) അനുയായികളും നയിച്ച ജിഹാദായിരുന്നു പ്രതിബന്ധങ്ങളെ തൃണവല്‍ഗണിച്ചു കൊണ്ടുള്ള ആദര്‍ശജീവിതവും ക്ഷമയും സഹനവും അവലംബിച്ചുകൊണ്ടുള്ള സത്യമത സന്ദേശപ്രചരണവും. മറ്റെന്ത് ത്യജിക്കേണ്ടി വന്നാലും ഈ ജിഹാദിന്റെ നിര്‍വഹണത്തില്‍ യാതൊരുവിധ വിട്ടുവീഴ്ചക്കും പ്രവാചകന്‍(സ) ഒരുക്കമല്ലായിരുന്നു. അതുകൊണ്ടാണ് സത്യസന്ധനെന്ന് വിളിച്ച് ആദരിച്ചിരുന്ന നാട്ടുകാരുടെ മുഴുവന്‍ വെറുപ്പിന് അദ്ദേഹം പാത്രമായത്; അല്ലലില്ലാതെയുള്ള ജീവിതത്തില്‍ നിന്ന് പട്ടിണിയിലേക്കും പീഡനങ്ങളിലേക്കും അദ്ദേഹം എടുത്തെറിയപ്പെട്ടത്; തനിക്ക് പ്രിയപ്പെട്ടവരും തന്നെ ഇഷ്ടപ്പെട്ടവരുമായിരുന്ന ബന്ധുക്കളുടെയും സ്വന്തക്കാരുടെയും വെര വിരോധത്തിന് അദ്ദേഹം പാത്രമായത്; തന്റെ അനുയായികൡല്‍ പലരും ക്രൂരമായ പീഡനങ്ങള്‍ക്ക് വിധേയമാകുന്നതും അവസാനം കൊല്ലപ്പെടുന്നതുമെല്ലാം നിസ്സഹായനായി അദ്ദേഹത്തിന് നോക്കി നില്‍ക്കേണ്ടി വന്നത്; ജനിച്ചു വളര്‍ന്ന നാടും വീടുമുപേക്ഷിച്ച് മറ്റൊരു ഗത്യന്തരവുമില്ലാത്തതു കൊണ്ട് പലായനം ചെേയ്യണ്ട ഗതിയുണ്ടായതും അതുകൊണ്ടു തന്നെ. വിട്ടുവീഴ്ചയില്ലാത്ത ആദര്‍ശജീവിതത്തിന് സ്വന്തത്തെ സജ്ജമാക്കുകയും സത്യത്തിന്റെ സല്‍പാന്ഥാവിലേക്ക് സഹജീവികളെ ആനയിക്കുകയും ചെയ്യുകയെന്ന ജിഹാദാണ് ഒരു വിട്ടുവീഴ്ചയുമില്ലാതെ, മുസ്‌ലിമിന്റെ ജീവിതത്തില്‍ എപ്പോഴുമുണ്ടാകേണ്ട ജിഹാദെന്ന് പഠിപ്പിക്കുകയായിരുന്നു, തിരുനബി(സ) തന്റെ ജീവിതത്തിലുടനീളം ചെയ്തത്.

  1. ഇമാം ഇബ്‌നുല്‍ ഖയ്യിം അല്‍ ജൗസിയ്യ: സാദുല്‍ മആദ്, ബൈറൂത്ത്, 2000, വാല്യം3, പുറം 9,10
  2. ഇമാം ഇബ്‌നുല്‍ ഖയ്യിം അല്‍ ജൗസിയ്യ: റൗദത്തുല്‍ മുഹിബ്ബീന്‍ വ നുസ്ഹത്തുല്‍ മുഷ്തക്കീന്‍. ബൈറൂത്ത്, 2000, പുറം 208
  3. അബൂബക്ര്‍ അല്‍ ഖാത്തിബ് അല്‍ ബാഗ്ദാദി: താരീഖുല്‍ ബഗ്ദാദി, 13/493 (http://theunjustmedia.com)
  4. പ്രവാചകാനുചരന്മാരായ സ്വഹാബികളില്‍ നിന്ന് മതം പഠിച്ച രണ്ടാം തലമുറയിലുള്ളവരെയാണ് താബിഉകള്‍ എന്ന് വിളിക്കുന്നത്
  5. ഇബ്‌നു ഹജറുല്‍ അസ്ഖലാനിയുടെ 'തസ്ദീദ് അല്‍ ഖൗസില്‍'’നിന്ന് ശൈഖ് ഹിഷാം കബ്ബാനി ഉദ്ധരിച്ചത്. (http://www.sunnah.org/tasawwuf/jihad004.html)
  6. Greater and "Lesser' Jihad?(http://the unjustmedia.com)
  7. സ്വഹീഹു ഇബ്‌നു ഹിബ്ബാന്‍ 11:203(http://www.sunnah.org/tasawwuf/jihad0 04.html)
  8. ഇമാം അല്‍ബാനി: സില്‍സിലത്തുല്‍ അഹാദീഥു സ്‌സ്വഹീഹ, വാല്യം 2, ഹദീഥ് 549 (http://www.alalbany.net)
  9. ഇമാം അല്‍ബാനി: സ്വഹീഹ് ഇബ്‌നുമാജ, ഹദീഥ് 3057 (http://www.alal-bany.net)
  10. ഇബ്‌നു മാജ, കിതാബുല്‍ ആദാബ്, ബാബു ഫദ്‌ലുദ്ദിക്ര്‍

റെ തെറ്റിദ്ധരിക്കപ്പെട്ട പദമാണ് ജിഹാദ്. വിവിധ ദേശങ്ങളില്‍ നടക്കുന്ന വിമോചന പോരാട്ടങ്ങളെക്കുറിച്ച പരാമര്‍ശങ്ങളിലും മാനവികതക്കു തന്നെ ഭീഷണിയായിത്തീരുന്ന ഭീകരവാദത്തെപ്പറ്റിയുള്ള അപഗ്രഥനങ്ങളിലും ഇസ്‌ലാമിനെയും മുഹമ്മദ് നബി(സ)യെയും പ്രതിചേര്‍ത്തുകൊണ്ടുള്ള പഠനങ്ങളിലുമെല്ലാം നിറഞ്ഞുനില്‍ക്കുന്ന പദമാണത്. ജിഹാദെന്ന ശബ്ദം കേള്‍ക്കുമ്പോഴേക്ക് ഒഴുകുന്ന രക്തവും കരയുന്ന വ്രണിതരും തകരുന്ന കെട്ടിടങ്ങളും നിരയായി കിടക്കുന്ന ശവശരീരങ്ങളും സ്‌ഫോടനത്തില്‍ നിന്നുയരുന്ന അഗ്‌നിസ്ഫുലിംഗങ്ങളും തോക്കും വാളും കഠാരയും മുഖംമൂടിയണിഞ്ഞ ആയുധധാരികളുമാണ് സാധാരണക്കാരന്റെ മനസ്സിലേക്ക് കടന്നു വരുന്നത്.ജിഹാദ് എന്ന പദവുമായി യാതൊരു ബന്ധവുമില്ലാത്ത ഈ പരികല്പനയുടെ സ്വാധീനം നമ്മുടെ മാധ്യമങ്ങളിലും പൊതുവ്യവഹാരങ്ങളിലുമെല്ലാം കാണാൻ കഴിയും.

കോടിക്കണക്കിന് കോപ്പികള്‍ വിറ്റഴിയുകയും മേശപ്പുറത്തെ എളുപ്പ റഫറന്‍സിന് ഏറ്റവുമധികം ഉപയോഗിക്കപ്പെടുകയും ചെയ്യുന്ന ഇംഗ്ലീഷ് നിഘണ്ടുവായ 'റീഡേഴ്‌സ് ഡൈജസ്റ്റ് ഓക്‌സ്‌ഫോര്‍ഡ് കംപ്ലീറ്റ് വേള്‍ഡ് ഫൈന്‍ഡര്‍' ജിഹാദിനെ നിര്‍വചിക്കുന്നത് 'അവിശ്വാസികള്‍ക്കെതിരെ മുസ്‌ലിംകള്‍ നടത്തുന്ന വിശുദ്ധയുദ്ധം'(1) എന്നാണ്. വിശദമായ ഇംഗ്ലീഷ് പദപഠനത്തിന് ഉപയോഗിക്കപ്പെടുന്ന 'വെബ്‌സ്റ്റേഴ്‌സ് എന്‍സൈക്ലോപീഡിക് ഡിക്ഷണറിയില്‍ 'മുസ്‌ലിംകള്‍ പരിശുദ്ധ ഉത്തരവാദിത്തമായി പരിഗണിക്കുന്ന വിശുദ്ധയുദ്ധം' എന്നും 'ഒരു ആശയത്തിനോ തത്ത്വത്തിനോ വേണ്ടി നടത്തുന്ന ഉജ്വലമായ, പലപ്പോഴും കയ്‌പേറിയ കുരിശു യുദ്ധം' എന്നുമാണ് ജിഹാദിന് നല്‍കിയിരിക്കുന്ന രണ്ട് നിര്‍വചനങ്ങള്‍.(2) 'മുഹമ്മദീയര്‍ നടത്തുന്ന പോര്; മതപ്രചാരണാര്‍ത്ഥം ഇസ്ലാം നിയമം അനുശാസിച്ചിട്ടുള്ള വിശുദ്ധയുദ്ധം' എന്നാണല്ലോ മലയാള ഭാഷാകാരന്‍മാരില്‍ പ്രമുഖനായ ശ്രീകണ്‌ഠേശ്വരം പത്മനാഭപിള്ള തന്റെ പ്രസിദ്ധമായ 'ശബ്ദ താരാവലി'യില്‍ ജിഹാദിനെ നിര്‍വചിച്ചത്.(3) പ്രൊഫ: ഇ.പി. നാരായണ ഭട്ടതിരിയുടെ അസ്സീസി മലയാളം നിഘണ്ടുവിലാകട്ടെ, 'മുസ്‌ലിംകളുടെ വിശുദ്ധയുദ്ധം; മതത്തിനു വേണ്ടിയുള്ള യുദ്ധം; കുരിശു യുദ്ധം; മിന്നലാക്രമണം പോലെ പെട്ടെന്നുള്ള ആക്രമണം' എന്നീ നാല് അര്‍ഥങ്ങള്‍ നല്‍കിയിട്ടുണ്ട്, ജിഹാദിന്.(4) നിഘണ്ടുകള്‍ വ്യാപകമായി നിര്‍മിക്കുവാനാരംഭിച്ച പതിനാറാം നൂറ്റാണ്ടിന്റെ തുടക്കം മുതലെങ്കിലും തന്നെ ജിഹാദെന്നാല്‍ കുരിശുയുദ്ധത്തിന് സമാനമായ ഇസ്‌ലാമിക സംജ്ഞയാണെന്ന ധാരണ നിലനിന്നിരുന്നുവെന്ന് ഇവ വ്യക്തമാക്കുന്നു. പദപരമായി നോക്കിയാൽ ജിഹാദ് എന്ന പദത്തിന് യുദ്ധവുമായി ബന്ധമൊന്നുമില്ല. ഹര്‍ബ്, വഇയ്യ്, ഇദാഅ്, ഖിസ്വാമ് എന്നിവയാണ് യുദ്ധത്തിനുപയോഗിക്കുന്ന അറബിപദങ്ങള്‍.(5) വിശുദ്ധ യുദ്ധത്തിന് 'അല്‍ ഹര്‍ബുല്‍ മുഖദ്ദസ' എന്നാണ് അറബിയില്‍ പറയുക. ഇങ്ങനെയൊരു സാങ്കേതിക ശബ്ദം ഇസ്‌ലാമിക ശബ്ദതാരാവലിയില്‍ കാണാന്‍ കഴിയില്ല. ജിഹാദിന് ഇങ്ങനെയൊരു അര്‍ഥം അംഗീകരിക്കപ്പെട്ട അറബി നിഘണ്ടുക്കളിലൊന്നും നല്‍കിയിട്ടുമില്ല. 'ജാഹദയുടെ നാമരൂപമായ ജിഹാദ് യഥാവിധി സൂചിപ്പിക്കുന്നത് ഒരാളുടെ പരമാവധി കഴിവും ശക്തിയും പരിശ്രമങ്ങളും അധ്വാനങ്ങളുമെല്ലാം അനിഷ്ടകരമായ എന്തിനെങ്കിലുമെതിരെ പോരാടുവാന്‍ വിനിയോഗിക്കുകയെന്നാണ്. ഇത് മൂന്നുതരമുണ്ട്. പ്രത്യക്ഷ ശത്രുവിനോടുള്ളത്, പിശാചിനോടുള്ളത്, സ്വന്തത്തോടുള്ളത്. ക്വുര്‍ആനിലെ 22:77 ലെ ഈ പ്രയോഗത്തില്‍ ഇവ മൂന്നും ഉള്‍പ്പെടുന്നു'െവന്നാണ് പ്രസിദ്ധമായ ലെയിന്‍സ് അറബി ഇംഗ്ലീഷ് ലെക്‌സിക്കണ്‍ പറയുന്നത്.(6) ഇതിനു സമാനമായ അര്‍ഥം തന്നെയാണ് ക്ലാസിക്കല്‍ അറബി നിഘണ്ടുക്കളും ജിഹാദിന് നല്‍കിയിട്ടുള്ളത്.(7)

ആദ്യകാലത്തോ പില്‍ക്കാലത്തോ ഉള്ള ക്വുര്‍ആന്‍ വ്യാഖ്യാതാക്കളാരും തന്നെ ജിഹാദിനെക്കുറിച്ച ചര്‍ച്ചയില്‍ അത് 'അല്‍ഹര്‍ബുല്‍ മുഖദ്ദസ'യാണെന്ന് സമര്‍ഥിച്ചതായി കാണാനാവില്ല. പ്രമുഖ ബ്രിട്ടീഷ് ഓറിയന്റലിസ്റ്റായ ആര്‍തര്‍ ജോണ്‍ ആര്‍ബറി തന്റെ ക്വുര്‍ആന്‍ പരിഭാഷയിലുടനീളം 'ജിഹാദി'നെ പരിഭാഷപ്പെടുത്തിയിരിക്കുന്നത്"struggle'എന്നാണ്, holywar എന്നല്ല എന്ന കാര്യവും ശ്രദ്ധേയമാണ്.(8) പോരാട്ടത്തിനും യുദ്ധത്തിനുമെല്ലാംstruggleഎന്നു പറയുമെങ്കിലും, എന്തെങ്കിലുമൊരു കാര്യം നേടിയെടുക്കുന്നതിനു വേണ്ടിയുള്ള തീവ്രമായ യജ്ഞത്തിനാണല്ലോ പൊതുവായി ആ ആംഗലേയ ശബ്ദം പ്രയോഗിക്കപ്പെടാറുള്ളത്; ഏതായിരുന്നാലും,struggle എന്ന ഇംഗ്‌ളീഷ് പദത്തിന് വിശുദ്ധയുദ്ധമെന്ന് അര്‍ഥമില്ലെന്ന് വ്യക്തമാണ്. ഇതു തന്നെയാണ് ജിഹാദ് എന്ന അറബി പദത്തിന്റെയും അവസ്ഥ. 'തീവ്രമായ പരിശ്രമമെന്നാണ്' പ്രസ്തുത പദത്തിന്റെ വിവക്ഷ. നന്‍മക്കു വേണ്ടിയുള്ള പോരാട്ടം തിന്‍മയെ വിപാടനം ചെയ്യുന്നതിനായി സ്വന്തം ജീവന്‍ നല്‍കിക്കൊണ്ട് നടത്തുന്ന തീവ്രയജ്ഞമായതിനാല്‍ അത് ജിഹാദാണ്. ജിഹാദ് എന്നാല്‍ പോരാട്ടമല്ല; എന്നാല്‍ പോരാട്ടം ജിഹാദായിത്തീരുമെന്നര്‍ഥം.

ജിഹാദായിത്തീരുന്ന പോരാട്ടത്തെക്കുറിച്ചു പോലും മുസ്‌ലിംകള്‍ 'വിശുദ്ധയുദ്ധം' എന്നു വിളിക്കാറില്ല. സ്വതവേ അശുദ്ധമായ രക്തച്ചൊരിച്ചിലുകളെ മാര്‍പാപ്പയാല്‍ മാമോദീസ മുക്കി വിശുദ്ധമാക്കുകയാണ് മധ്യകാല ക്രൈസ്തവ നേതൃത്വം ചെയ്തത്. അധീശത്വത്തിനുവേണ്ടി നടത്തുന്ന അതിക്രമങ്ങളുടെ സമാഹാരമായി നിലനില്‍ക്കുന്ന സാമൂഹ്യരൂപമായ യുദ്ധത്തെ അത് സഭയുടെ താല്‍പര്യസംരക്ഷണത്തിനാണ് എന്ന ഒരേയൊരു കാരണത്താല്‍ മാമോദീസ മുക്കി വിശുദ്ധമാക്കിയപ്പോള്‍ അതിന്റെ പേരില്‍ നടന്നത് നിരപരാധികളും നിഷ്‌കളങ്കരുമായ മനുഷ്യരുടെ കൂട്ടക്കുരുതികളായിരുന്നു.

പോരാട്ടം ജിഹാദായിത്തീരുന്ന സന്ദര്‍ഭത്തെക്കുറിച്ച് വിശദീകരിക്കുന്ന ഇസ്‌ലാമിക പ്രമാണങ്ങള്‍ അത് എന്തിന്, എങ്ങനെയാണ് നടത്തേണ്ടതെന്നു കൂടി കൃത്യമായി പഠിപ്പിക്കുന്നുണ്ട്.വിശ്വാസആരാധനാപ്രബോധന സ്വാതന്ത്ര്യങ്ങള്‍ നേടിയെടുക്കുന്നതിനായി നടക്കുന്ന പോരാട്ടങ്ങളുടെ ലക്ഷ്യവും മാര്‍ഗവും ദൈവിക മാര്‍ഗദര്‍ശന പ്രകാരമുള്ളതായിത്തീരുമ്പോള്‍ മാത്രമാണ് അത് ജിഹാദായിത്തീരുക. സ്വതവെ അശുദ്ധമായ യുദ്ധത്തെ പരിശുദ്ധാത്മ സാമീപ്യത്താലുള്ള മാര്‍പാപ്പ പ്രഖ്യാപനം വഴി വിശുദ്ധമാക്കി, അതിന്റെ പേരില്‍ അണികളെ ആയുധമണിയിച്ച് എന്തും ചെയ്യാനുള്ള അനുമതിയും കൊടുത്ത് തുറന്ന് വിടുന്ന ഏര്‍പ്പാടല്ല അത,് പ്രത്യുത അനിവാര്യമായ സാഹചര്യത്തില്‍ നടത്തുന്ന, നിയതമായ മാര്‍ഗവും കൃത്യമായ ലക്ഷ്യവുമുള്ള പോരാട്ടമാണത്; അശുദ്ധമായ ഒന്നിനെ വിശുദ്ധവല്‍ക്കരിച്ചതല്ല ജിഹാദ്; അത് എല്ലായ്‌പ്പോഴും വിശുദ്ധമാണ്; വിശുദ്ധമായതേ ജിഹാദാവൂ എന്നർത്ഥം. 'ജിഹാദ്' എന്ന നാമരൂപമോ അതിന്റെ ക്രിയാരൂപങ്ങളോ ആകെ 41 തവണയാണ് ക്വുര്‍ആനില്‍ പ്രയോഗിക്കപ്പെട്ടിരിക്കുന്നത്. നാമരൂപത്തില്‍ ജിഹാദ് എന്നു പ്രയോഗിച്ചിരിക്കുന്നത് നാലു തവണയാണ്; സൂറത്തുത്തൗബയിലെ 24ാം വചനത്തിലും സൂറത്തുല്‍ ഹജ്ജിലെ 78ാം വചനത്തിലും സൂറത്തുല്‍ ഫുര്‍ഖാനിലെ 52ാം വചനത്തിലും സൂറത്തുല്‍ മുംതഹനയിലെ ഒന്നാം വചനത്തിലുമാണ് അവ. മറ്റു സൂക്തങ്ങളില്‍, ജാഹദൂ, ജാഹിദൂ, യുജാഹിദു, ജാഹദ, യുജാഹിദൂന, തുജാഹിദൂന, മുജാഹിദൂന തുടങ്ങിയ വ്യത്യസ്ത ജാഹദാ രൂപങ്ങള്‍ പ്രയോഗിക്കുന്നുണ്ട് ക്വുര്‍ആന്‍. 'യുദ്ധംചെയ്യുക'യെന്ന കേവലമായ അര്‍ഥത്തിനപ്പുറം, യുദ്ധമടക്കമുള്ള ത്യാഗപരിശ്രമങ്ങളെക്കുറിക്കുവാനാണ് പ്രയോഗങ്ങള്‍ നടത്തുന്നതെന്ന് വചനങ്ങള്‍ പരിശോധിച്ചാല്‍ വ്യക്തമാവും.

യുദ്ധത്തെ മാത്രം സൂചിപ്പിക്കുന്നതിന് ക്വുര്‍ആനില്‍ പ്രയോഗിക്കുന്നത് 'ഖിതാല്‍' എന്ന പദമോ അതിന്റെ ക്രിയാരൂപങ്ങളോ ആണ്. പോരാട്ടം എന്ന് അര്‍ഥം വരുന്ന 'ഖിതാല്‍' എന്ന അറബിപദമാണ്, ധനസമ്പാദനം ലക്ഷ്യമാക്കി അറബികള്‍ ചെയ്തിരുന്ന യുദ്ധത്തെക്കുറിക്കുവാന്‍ അറബികള്‍ പ്രയോഗിച്ചിരുന്ന 'ഹര്‍ബിനും' ഗോത്രവൈരം തീര്‍ക്കാനുള്ള ഏറ്റുമുട്ടലുകള്‍ക്ക് അവര്‍ പറഞ്ഞിരുന്ന 'വഇയ്യിനും' പകരം ക്വുര്‍ആന്‍ പ്രയോഗിച്ചിരിക്കുന്നത് എന്ന കാര്യം പ്രത്യേകം പ്രസ്താവ്യമാണ്. ഖിതാലും ജിഹാദും പര്യായപദങ്ങളല്ല. ജിഹാദിന്റെ അര്‍ഥമായി 'ഖിതാല്‍' എന്ന് ഭാഷാകാരന്‍മാരൊന്നും രേഖപ്പെടുത്തിയിട്ടുമില്ല. ഖിതാല്‍ മാത്രമല്ല ജിഹാദ്; എന്നാല്‍ ഖിതാലും ജിഹാദായിത്തീരും എന്നാണ് ജിഹാദ്, ഖിതാല്‍ എന്നീ പ്രയോഗങ്ങള്‍ നടത്തിയ ക്വുര്‍ആന്‍ വചനങ്ങളും ഹദീഥുകളും പരിശോധിച്ചാല്‍ നമുക്ക് മനസ്സിലാവുക. മുസ്‌ലിംകളായ തങ്ങളുടെ സന്താനങ്ങളെ ബഹുദൈവാരാധനക്ക് നിര്‍ബന്ധിക്കുന്ന സത്യനിഷേധികളായ മാതാപിതാക്കളുടെ ചെയ്തിയെ ക്വുര്‍ആന്‍ വിശേഷിപ്പിച്ചിരിക്കുന്നത് 'ജിഹാദ്' എന്നാണ്: (29:8) ബഹുദൈവാരാധനക്ക് നിര്‍ബന്ധപൂര്‍വം ആവശ്യപ്പെടുന്നതിനാണ് ഈ വചനത്തില്‍ ജിഹാദ് എന്ന് പ്രയോഗിച്ചിരിക്കുന്നത് എന്ന് വ്യക്തമാണ്.

ജിഹാദിനെക്കുറിച്ച് പരാമര്‍ശിക്കുന്ന ക്വുര്‍ആന്‍ സൂക്തങ്ങളില്‍ ചിലതില്‍ സ്വശരീരംകൊണ്ടും സമ്പത്തുകൊണ്ടും ജിഹാദ് ചെയ്യാന്‍ കല്‍പിക്കുന്നതായി കാണാന്‍ കഴിയും: ''അല്ലാഹുവിലും അവന്റെ ദൂതനിലും വിശ്വസിക്കുകയും പിന്നീട് സംശയിക്കാതിരിക്കുകയും, തങ്ങളുടെ സ്വത്തുക്കളും ശരീരങ്ങളും കൊണ്ട് അല്ലാഹുവിന്റെ മാര്‍ഗത്തില്‍ സമരം നടത്തുകയും ചെയ്തവരാരോ അവര്‍ മാത്രമാകുന്നു സത്യവിശ്വാസികള്‍. അവര്‍ തന്നെയാകുന്നു സത്യവാന്‍മാര്‍.'' (49:15) രണഭൂമിയിലെ പോരാട്ടം അടക്കമുള്ള ത്യാഗപരിശ്രമങ്ങളെല്ലാം ശരീരംകൊണ്ടുള്ള ജിഹാദാണ്. നന്മയുടെ മാര്‍ഗത്തില്‍ സ്വത്തുക്കള്‍ ചെലവഴിക്കുകയെന്ന സാമ്പത്തിക ത്യാഗപരിശ്രമമാണ് ഇവിടെ സമ്പത്തുകൊണ്ടുള്ള ജിഹാദുകൊണ്ട് വിവക്ഷിക്കപ്പെട്ടിരിക്കുന്നത് എന്ന് വ്യക്തമാണ്.

ആദര്‍ശ ശത്രുക്കളോട് ക്വുര്‍ആന്‍ ഉപയോഗിച്ച് ജിഹാദ് ചെയ്യുവാനാണ് അല്ലാഹുവിന്റെ കല്‍പന: ''അതിനാല്‍ സത്യനിഷേധികളെ നീ അനുസരിച്ചു പോകരുത്. ഇത് (ഖുര്‍ആന്‍) കൊണ്ട് നീ അവരോട് വലിയൊരു സമരം നടത്തിക്കൊള്ളുക.'' (25:52) പീഡനങ്ങളും പ്രയാസങ്ങളും സഹിച്ച് മക്കയില്‍ ആദര്‍ശപ്രബോധനം നിര്‍വഹിക്കുന്നതിനിടയില്‍ അവതരിപ്പിക്കപ്പെട്ട ഈ സൂക്തത്തില്‍ ക്വുര്‍ആന്‍ ഉപയോഗിച്ചുകൊണ്ടാണ് ഏറ്റവും വലിയ ജിഹാദ് ചെയ്യേണ്ടതെന്നാണ് കല്‍പിക്കപ്പെട്ടിരിക്കുന്നതെന്ന വസ്തുത ശ്രദ്ധേയമാണ്. ക്വുര്‍ആനികാദര്‍ശങ്ങളെ സത്യനിഷേധികള്‍ക്ക് മുമ്പില്‍ സമര്‍പ്പിക്കുന്നതിനെയും അതുപയോഗിച്ച് ആദര്‍ശസമരം നടത്തുന്നതിനെയുംകുറിച്ചാണ് ഇവിടെ ക്വുര്‍ആന്‍ കൊണ്ടുള്ള ജിഹാദ് എന്ന് പരിചയപ്പെടുത്തിയിരിക്കുന്നത് എന്ന് വ്യക്തം.

'അല്ലാഹുവിന്റെ മാര്‍ഗത്തില്‍ ജിഹാദ് ചെയ്യേണ്ട മുറപ്രകാരം നിങ്ങള്‍ ജിഹാദ് ചെയ്യുക' (22:78) എന്ന് കല്‍പിക്കുന്ന ക്വുര്‍ആന്‍ വചനം 'നിങ്ങളുടെ പിതാവായ ഇബ്‌റാഹീമിന്റെ മാര്‍ഗമാണത്' എന്നാണ് അനുബന്ധമായി പറഞ്ഞിരിക്കുന്നത്. ത്യാഗനിര്‍ഭരമായ ആദര്‍ശ സമരമായിരുന്നു ഇബ്‌റാഹീം നബി(അ)യുടെ ജീവിത്തില്‍ നിറഞ്ഞുനിന്നിരുന്നത് എന്ന വസ്തുത ക്വുര്‍ആനിലൂടെ കണ്ണോടിക്കുന്ന ആര്‍ക്കും മനസ്സിലാകും. ഇബ്‌റാഹീം നബി(അ)നടത്തിയ സായുധ സമരങ്ങളെക്കുറിച്ച പരാമര്‍ശങ്ങളൊന്നുംതന്നെ ക്വുര്‍ആനിലില്ല. സ്രഷ്ടാവ് മാത്രമാണ് ആരാധനകള്‍ അര്‍ഹിക്കുന്നതെന്ന തത്ത്വം പ്രബോധനം ചെയ്യുകയും പ്രസ്തുത മാര്‍ഗത്തില്‍ ത്യാഗങ്ങള്‍ അനുഷ്ഠിക്കുകയും ചെയ്തതാണ് ഇബ്‌റാഹീം നബിൗയുടെ ജീവിതത്തില്‍ നിറഞ്ഞുനിന്നിരുന്ന ജിഹാദ്. വിശ്വാസികള്‍ക്ക് എന്നും മാതൃകയായ ജിഹാദാണത്; അവരുടെ ജീവിതത്തില്‍നിന്ന് ഒരിക്കലും മാറ്റിനിര്‍ത്താനാവാത്ത ജിഹാദ്.

1. "Jihad: a holy war undertaken by Muslims against unbelievers', Sarah Tulloch (Ed): Reader's Digest Oxford Complete Word finder, Milan, 1994, Page 819

2. "Jihad:a holy war undertaken as a sacred duty by Muslims; any vigoro-us, often bitter crusade for an idea or principle.'”Webster's Encyclopaedic Unabridged Dictionary of the English Language, New York, 1989, Page 767

3. ശബ്ദതാരാവലി, 9ാം പതിപ്പ്, ശ്രീ കണ്‌ഠേശ്വരം ജി. പത്മനാഭപ്പിള്ള, പ്രസാധകര്‍: സാഹിത്യ പ്രവര്‍ത്തക സഹകരണ സംഘം ലിമിറ്റഡ്, നാഷണല്‍ ബുക്ക്സ്റ്റാള്‍ കോട്ടയം, പേജ്: 871

4. പ്രൊഫ: ഇ.പി. നാരായണ ഭട്ടതിരി: അസ്സീസി മലയാളം നിഘണ്ടു, ചങ്ങനാശേരി, 1997,പുറം 328

5. حرب، وغي، عداء، خصام

6. "Jihad, infinitive noun of jaahada, properly signifies the using or exerting of one's utm-ost power, eÅorts, endeavours or ability, in contending with an object of disappro-bation; and this is of three kinds, namely, a visible enemy, the devil, and one's self; all of which are included in the term as used in the Quran 22:77.'' (Edward William Lane: An Arabic-English Lexicon, Beirut, 1968, Page 473)

7. ലിസാനുല്‍ അറബ് 4/107, താജുല്‍ അറൂസ് 2/329

8. A. J. Arberry: The Koran Interpreted: A Translation, Touchstone, 1996, Look 9:24, 22:78, 25:52, 60:1 etc.

 

അല്ല. ഒരാൾ തൻറെ ഇച്ഛയനുസരിച്ച് ഭരിക്കുന്നതാണ് ഏകഛത്രാധിപത്യമെങ്കിൽ ഇസ്‌ലാം അത്തരമൊരു വ്യവസ്ഥയെ ഒരു തരത്തിലും അംഗീകരിക്കുന്നില്ല. ഇസ്‌ലാമികരാഷ്ട്രത്തിലെ അടിസ്ഥാന ധാര്മിക നിയമങ്ങൾ തീരുമാനിക്കുന്നത് ഏതെങ്കിലുമൊരാളോ ഒരു കൂട്ടം ആളുകളോ അല്ല; രാജാധിരാജനായ അല്ലാഹുവാണ്. അവന്റെ നിയമങ്ങൾ മാറ്റിമറിക്കുവാൻ ആർക്കും അവകാശമില്ല. ക്വുർആനും സുന്നത്തുമനുസരിച്ച് കാര്യനിര്വഹണം നടത്തുക മാത്രമാണ് ഇസ്‌ലാമികരാഷ്ട്രത്തിലെ ഭരണാധികാരി ചെയ്യുന്നത്. രാഷ്ട്രനിർമ്മാണപരമായ കാര്യങ്ങൾ തീരുമാനിക്കേണ്ടത് അയാളുടെ ചുമതലയാണ്. അവ പോലും എകാധിപത്യപരമായി തീരുമാനിക്കണമെന്നല്ല പ്രവാചകൻ (സ) പഠിപ്പിച്ചത്. അവ തീരുമാനിക്കേണ്ടത് കൂടിയാലോചനയിലൂടെയാണ്.

ഇസ്‌ലാമിക രാഷ്ട്രത്തിന്റെ നായകനാണ് ഖലീഫ. വിശുദ്ധ ക്വുര്‍ആനിലും പ്രവാചകചര്യയിലും വ്യക്തമാ ക്കപ്പെട്ട അടിസ്ഥാനതത്ത്വങ്ങള്‍ക്കനുസൃതമായി രാഷ്ട്രത്തെ നയി ക്കുകയാണ് ഖലീഫയുടെ ഉത്തരവാദിത്തം. ക്വുര്‍ആനിലും സുന്ന ത്തിലും വ്യക്തമായി വിവരിക്കപ്പെടാത്ത ഒട്ടനവധി പ്രശ്‌നങ്ങള്‍ ഒരു രാഷ്ട്രത്തിന് കൈകാര്യം ചെയ്യേണ്ടതായി വരുമല്ലോ. രാഷ്ട്രീയപ്രശ്‌നങ്ങളില്‍ ഖണ്ഡിതമായ നിയമങ്ങളെക്കാളധികം പാലിക്കപ്പെടേണ്ട തത്ത്വങ്ങളാണ് വിശുദ്ധക്വുര്‍ആന്‍ പരാമര്‍ശിക്കുന്നത്. ഈ തത്ത്വങ്ങളുടെ വെളിച്ചത്തില്‍, ഇസ്‌ലാമിന്റെ ആത്മാവിനിണങ്ങുന്ന വിധത്തില്‍ നിയമനിര്‍മാണം നടത്തേണ്ടത് ഖലീഫയുടെ ബാധ്യത യാണ്. ഈ നിയമങ്ങള്‍ നിര്‍മിക്കേണ്ടത് ഖലീഫയുടെ തന്നിഷ്ടപ്ര കാരമല്ല; പ്രത്യുത കൂടിയാലോചനയിലൂടെയാണ്. കൂടിയാലോചനാ സമിതിയെയാണ് ഇസ്‌ലാമികരാഷ്ട്രത്തില്‍ 'ശൂറാ' എന്നു വിളി ക്കുന്നത്.

പ്രവാചകന്‍പോലും രാഷ്ട്രീയമായ പ്രശ്‌നങ്ങള്‍ കൂടിയാ ലോചനയിലൂടെയായിരുന്നു തീരുമാനിച്ചിരുന്നതെന്ന് കാണാന്‍ കഴിയും. 'കാര്യങ്ങളില്‍ നീ അവരോട് കൂടിയാലോചിക്കുകയും ചെയ്യുക'(ക്വുർആൻ 3:159) എന്നാണ് പ്രവാചകനോടുള്ള ദൈവികകല്‍പന. അബൂഹുറയ്‌റ(റ)പറയുന്നു: 'പ്രവാചകനെപ്പോലെ സ്വന്തം അനുയായികളുമായി ഇത്രയേറെ കൂടിയാലോചന ചെയ്യുന്ന മറ്റൊരു വ്യക്തിയെ യും ഞാന്‍ കണ്ടിട്ടില്ല'.(ബുഖാരി) അനുയായികളുമായി കൂടിയാലോചിക്കുകയും അവരുടെ അഭിപ്രായങ്ങള്‍ മാനിക്കുകയും ചെയ്ത ആളായിരുന്നു പ്രവാചകന്‍(സ). ബദർ യുദ്ധവേളയില്‍ ആദ്യം തെരഞ്ഞെടുത്ത താവളം മാറ്റിയതും ഉഹ്ദ്‌യുദ്ധത്തിന് പുറപ്പെട്ടതും ഖന്‍ദഖ് യുദ്ധസമയത്ത് ഗത്ഫാന്‍ ഗോത്രവുമായി സന്ധിചെയ്യാനുള്ള തന്റെ തീരുമാനം മാറ്റിയതു മെല്ലാം അനുയായികളുമായി കൂടിയാലോചിച്ചശേഷമായിരുന്നു. ചുരുക്കത്തിൽ, ഏകഛത്രാധിപത്യത്തിന്റെ ലാഞ്ഛന പോലുമില്ലാത്തതാണ് ഇസ്‌ലാമിക രാഷ്ട്രം.

ന്ന് വ്യവഹിക്കപ്പെടുന്ന രീതിയിലുള്ള ജനാധിപത്യത്തെക്കുറിച്ച് നാം ചിന്തിക്കാനാരംഭിച്ചതു തന്നെ പതിനെട്ടാം നൂറ്റാണ്ടിനു ശേഷമാണ്. അതിനു മുൻപ് നിലനിന്നിരുന്ന വ്യത്യസ്തങ്ങളായ രാഷ്ട്രമീമാംസകളുണ്ട്. ഗോത്രാധിപത്യവും രാജാധിപത്യവുമെല്ലാം അവയിൽ ചിലതാണ്.സമൂഹത്തിന്റെ വളർച്ചയ്ക്കനുസരിച്ച് രൂപപ്പെട്ടു വന്നവയാണ് ഈ മീമാംസകളെല്ലാം. ഒരാൾ ഭരണാധികാരിയാവുന്നതെങ്ങനെയെന്ന് തീരുമാനിക്കുന്നതിന് അടിസ്ഥാനത്തിലാണ് അത് ജനാധിപത്യമാണോ രാജ്യാധിപത്യമാണോ എന്നെല്ലാം പറയുന്നത്. തെരഞ്ഞെടുക്കപ്പെടുന്നത് എങ്ങനെയെന്നതിലധികം പ്രധാനം തെരെഞ്ഞെടുക്കപ്പെടുന്നവർ എങ്ങനെ ഭരിക്കണം എന്ന വിഷയമാണെന്ന് പൗരന്മാരെ സംബന്ധിച്ചിടത്തോളം ഏറ്റവും പ്രധാനം. ജനാധിപത്യരീതിയിൽ തെരെഞ്ഞെടുക്കപ്പെട്ടവരെക്കൊണ്ട് പൊരുതി മുട്ടുന്ന സമൂഹങ്ങളെക്കുറിച്ച വാർത്തകൾ ദിനംപ്രതി വായിക്കുന്നവരാണ് നാം. നന്മ നിറഞ്ഞ നല്ല രാജാക്കന്മാരെ നൂറ്റാണ്ടുകൾ കഴിഞ്ഞും ആദരവോടെ സ്മരിക്കുന്ന സമൂഹങ്ങളുമുണ്ട്. ഭരണാധികാരി നന്മയുള്ളവരാവുകയെന്നതാണ് എങ്ങനെ തെരെഞ്ഞെടുക്കപ്പെടുന്നുവെന്നതിനേക്കാൾ പ്രധാനമെന്നർത്ഥം.

ഭരണാധികാരിയെ എങ്ങനെ തെരെഞ്ഞെടുക്കണമെന്ന വിഷയം സാമൂഹികവളർച്ചയ്ക്കനുസരിച്ച് തീരുമാനിക്കാൻ സ്വാതന്ത്ര്യം നൽകിയ മതമാണ് ഇസ്‌ലാം. പ്രവാചകന് ശേഷമുള്ള നാല് ഖലീഫമാരെയും തെരെഞ്ഞെടുത്ത രീതികൾ വ്യത്യസ്തമായത് ആ രംഗത്ത് നിഷ്‌കൃഷ്ടമായ നിയമങ്ങൾ പഠിക്കാത്തത് കൊണ്ടാണ്. പ്രവാചകന്‍(സ)തന്റെ പിന്‍ഗാമിയെ നിശ്ചയിക്കാതെയാണ് ദേഹവിയോഗം ചെയ്തത്. സർക്കാർ രൂപീകരണത്തിന്റെ നിശ്ചിതരൂപം പ്രവാചകന്‍(സ)കാണിച്ചു തന്നിട്ടില്ല. എന്നാല്‍, രാഷ്ട്രത്തിന്റെ മൗലികസ്വഭാവങ്ങളും സവിശേഷതകളുമെന്തെല്ലാമായിരിക്കണമെന്ന് കര്‍മപഥത്തിലൂടെ അദ്ദേഹം കാണിച്ചുതന്നിട്ടുണ്ട്. പ്രസ് തുത സവിശേഷതകളിലൂന്നി സന്ദര്‍ഭത്തിനൊത്ത ഗവണ്‍മെന്റ് രൂപീകരണരീതി സ്വീകരിക്കുവാനുള്ള വിപുലമായ സാധ്യതക്ക് അംഗീകാരം നല്‍കുകയാണ് പ്രവാചകന്‍ ചെയ്തത്. രൂപീകരണത്തിന്റെ രീതിയേക്കാളധികം പ്രധാനം ഗവണ്‍മെന്റ് എങ്ങനെ നീതിനിര്‍വഹിക്കണമെന്നതും പൗരന്മാരും ഗവണ്‍മെ ന്റും തമ്മില്‍ നിലനില്‍ക്കേണ്ട ബന്ധമെന്തായിരിക്കണമെന്നത മാണ്. ഇക്കാര്യങ്ങളിലേക്കാണ് പ്രവാചകന്‍(സ)തന്റെ ജീവിതത്തിലൂടെ പ്രധാനമായി വെളിച്ചം പകര്‍ന്നിരിക്കുന്നത്.

ഇസ് ലാമികഭരണം തന്റെ കുടുംബത്തിന്റെ അവകാശമായി മാറ്റണമെന്ന് പ്രാവാകന്ന് (സ) ഉദ്ദേശമുണ്ടായിരുന്നെങ്കിൽ അദ്ദേഹം അത് ചെയ്യുമായിരുന്നു. ഭരണാധികാരിയെ തെരെഞ്ഞെടുക്കേണ്ടതെങ്ങനെയെന്ന കാര്യം ജനങ്ങളുടെ തീരുമാനത്തിന് വിട്ടുകൊടുക്കുകയാണ് തിരുനബി(സ) ചെയ്തത്. പ്രവാചക വിയോഗത്തിനുശേഷം ഗവണ്‍മെന്റ് രൂപീകരണത്തിന്റെ പ്രശ്‌നം അനുചരന്മാര്‍ കൈകാര്യം ചെയ്തത് അദ്ദേഹം പഠിപ്പിച്ച മൗലികതത്ത്വങ്ങളുടെ അടിസ് ഥാനത്തില്‍ നിന്നുകൊണ്ടായിരുന്നു. മുഹമ്മദ് നബി(സ)യുടെ മരണശേഷം മൃതദേഹം സംസ്‌കരിക്കുന്നതിന് മുമ്പുതന്നെ അനുചരന്മാര്‍ സഖീഫത്തു ബനീസഈദില്‍ ഒത്തുകൂടി. ചര്‍ച്ചകള്‍ക്കു ശേഷം പ്രവാച കാനുചരന്മാരില്‍ പ്രമുഖനായ അബൂബക്കര്‍(റ)എഴുന്നേറ്റുനിന്ന് ഉമര്‍, അബൂ ഉബൈദത്തുല്‍ ജര്‍റാഹ് എന്നിവരുടെ പേരു നിര്‍ദേ ശിച്ചുകൊണ്ട് അവരിലൊരാളെ തെരഞ്ഞെടുക്കുവാനാവശ്യപ്പെട്ടു. ഉടനെ ഉമര്‍(റ)'താങ്കളെക്കാള്‍ അതിന്നര്‍ഹനായി മറ്റാരുമില്ല' എന്നു പറഞ്ഞ് അബൂബക്കറിനോട് കൈനീട്ടുവാന്‍ ആവശ്യപ്പെടുകയും അപ്പോള്‍ തന്നെ അദ്ദേഹത്തിന്റെ കരംഗ്രഹിച്ചുകൊണ്ട് അനുസരണ പ്രതിജ്ഞ (ബൈഅത്ത്) ചെയ്യുകയും ചെയ്തു. അതിന്നുശേ ഷം എല്ലാവരും അതിന്ന് അംഗീകാരം നല്‍കുകയും അബൂബക്കറി(റ)നെ ബൈഅത്ത് ചെയ്യുകയും ചെയ്തു.

രണ്ടാം ഖലീഫയായ ഉമറി(റ)ന നിയമിച്ചത് അബൂബക്കര്‍(റ)ആയിരുന്നു. അദ്ദേഹം തന്റെ പിന്‍ഗാമിയായി ഒരാളെ തെരഞ്ഞെ ടുക്കുവാന്‍ ജനങ്ങളോട് ആവശ്യപ്പെട്ടു. അപ്പോള്‍ പ്രസ്തുത തെര ഞ്ഞെടുപ്പിനുള്ള മുഴുവന്‍ അധികാരവും ജനങ്ങള്‍ അദ്ദേഹത്തിന് തന്നെ നല്‍കി. ഒന്നാം ഖലീഫ അബൂബക്കര്‍(റ)തന്റെ പിന്‍ഗാമി ആരാകണമെന്ന വിഷയത്തെക്കുറിച്ച് അബ്ദുറഹ്മാനുബ്‌നു ഔഫ്, ഉസ്മാനുബ്‌നു അഫ്ഫാന്‍, സഈദുബ്‌നുസൈദ്, ഉസൈദുബ്‌നു ഹുസൈനെ(റ) തുടങ്ങിയ പ്രഗല്‍ഭരുമായി ആലോചിച്ച ശേഷം പ്രസ്തുത സ്ഥാനത്തിന് അര്‍ഹന്‍ ഉമര്‍(റ)തന്നെയാണെന്ന് തീരുമാന ത്തിലെത്തി. ഖലീഫയായി നിയമിക്കപ്പെടയുടന്‍ ഉമർ (റ) ചെയ്ത പൊതുപ്രസംഗത്തില്‍ അദ്ദേഹം പറഞ്ഞു: 'നിങ്ങളുടെമേലുള്ള ഈ അധികാരസ്ഥാനം ഞാനൊരിക്കലും ആഗ്രഹിച്ചിരുന്നതല്ല. പരേതന്റെ മനസ്സില്‍ അല്ലാഹു തോന്നിച്ച ഒരു കാര്യമാണത്. എന്നില്‍ ഭരമേല്‍പിക്കപ്പെട്ട ഈ 'അനാമത്ത്' അനര്‍ഹരായ ആരെയെങ്കിലും ഏല്‍പിക്കണമെന്ന് ഞാന്‍ വിചാരിക്കുന്നില്ല. എന്നാല്‍, മുസ്‌ലിം കളുടെ അന്തസ്സ് ഉയര്‍ത്തുവാന്‍ കഴിയുമെന്ന് പ്രതീക്ഷിക്കുന്ന ആര്‍ക്കും ഈ സ്ഥാനം ഏല്‍പിക്കുവാന്‍ ഞാനൊരുക്കമാണ്. അവരാണ് അതിന്ന് കൂടുതല്‍ അര്‍ഹര്‍.' ഇത് കേട്ട ജനങ്ങള്‍ ഒന്നടങ്കം തങ്ങളി ഷ്ടപ്പെടുന്നത് ഉമറിനെത്തന്നെയാണെന്ന് പ്രഖ്യാപിക്കുകയാണു ണ്ടായത്.

ഒരു ഘാതകന്റെ കുത്തേറ്റ് മരണശയ്യയില്‍ കിടക്കുന്ന ഉമറി(റ)നോട് തന്റെ പിന്‍ഗാമിയെ നിശ്ചയിക്കുവാന്‍ വേണ്ടി മുസ്‌ലിംകള്‍ ആവശ്യപ്പെട്ടു. അദ്ദേഹം ഉസ്മാനുബ്‌നു അഫ്ഫാന്‍, അലിയ്യുബ്‌നു അബീത്വാലിബ്, അബ്ദുര്‍റഹ്മാനുബ്‌നു ഔഫ്, സുബൈറുബ്‌നു അവ്വാം, ത്വല്‍ഹത്തുബ്‌നു ഉബൈദില്ലാഹ്, സഅ്ദുബ്‌നു അബീവ ഖാസ്(റ) എന്നിവരുള്‍ക്കൊള്ളുന്ന ഒരു ആറംഗ സമിതി രൂപീകരിച്ച് അവരില്‍ നിന്ന് ഒരാളെ ഖലീഫയായി തെരഞ്ഞെടുക്കുവാന്‍ പ്രസ്തുത സമിതിയെത്തന്നെ ചുമതലപ്പെടുത്തി. തന്റെ മകനായ അബ്ദുല്ലാഹ്ബ്‌നു ഉമ(റ)ന കൂടിയാലോചനയില്‍ സഹായിക്കുവാനായി സമിതിയിലേക്ക് ക്ഷണിക്കണമെന്ന് നിര്‍ദേശിച്ചതോടൊപ്പം തന്നെ അദ്ദേഹത്തെ തെരഞ്ഞെടുക്കരുതെന്നും ഉമര്‍(റ)നിഷ്‌കര്‍ഷിച്ചിരുന്നു. സമിതി സമ്മേളിച്ചപ്പോള്‍ തന്നെ അബ്ദുറഹ്മാനുബ്‌നു ഔഫ്‌(റ)സ്ഥാനാര്‍ഥിത്വത്തില്‍ നിന്ന് പിന്‍വാങ്ങി. അപ്പോള്‍ ജനങ്ങളുമായി സമ്പര്‍ക്കത്തിലേര്‍പെട്ട് അവര്‍ ആഗ്രഹിക്കുന്ന വ്യക്തിയുടെ പേര് നിര്‍ദേശിക്കനായി സമിതി അദ്ദേഹത്തെ ചുമതലപ്പെടുത്തി. ജനങ്ങളുമായുള്ള സമ്പര്‍ക്കത്തില്‍ നിന്ന് അവര്‍ ഇഷ്ടപ്പെടുന്നത് ഉസ്മാനെയും അലിയെയുമാണെന്നും ഭൂരിപക്ഷം ഉസ്മാനോടൊപ്പമാണെന്നും അദ്ദേഹം മനസ്സിലാക്കി. മുസ്‌ലിംകളെ മുഴുവന്‍ അതി ന്നുശേഷം അദ്ദേഹം മദീനയിലെ മസ്ജിദുന്നബവിയില്‍ വിളിച്ചു കൂട്ടി. 'താങ്കളെ അധികാരമേല്‍പിച്ചാല്‍ അല്ലാഹുവിന്റെ ഗ്രന്ഥവും തിരുനബിയുടെ ചര്യയും പൂര്‍വികരായ രണ്ടു ഖലീഫമാരുടെ ചര്യയും അനുസരിച്ച് ഭരണം നടത്തുമോ?' അലിയെ അഭിസംബോധന ചെയ്തുകൊണ്ട് അബറുഹ്മാനിബ്‌നു ഔഫ് ചോദിച്ചു. 'കഴിവനുസരിച്ച് നടത്തും' എന്നാണ് അലി(റ)മറുപടി നല്‍കിയത്. ഇതേ ചോദ്യം ഉസ്മാ(റ)നോട് ചോദിച്ചപ്പോള്‍ 'അതെ'യെന്ന് അനുബന്ധമൊന്നും കൂടാതെ അദ്ദേഹം മറുപടി നല്‍കി. ഉടനെത്തന്നെ അബ്ദുറഹ്മാനുബ്‌നുഔഫ്‌(റ)ഉസ്മാന്റെ(റ) കരംഗ്രഹിച്ചുകൊണ്ട് അനുസരണ പ്രതിജ്ഞ ചെയ്തു. തുടര്‍ന്ന് മറ്റു മുസ്‌ലിംകളെല്ലാം അദ്ദേഹത്തിന് ബൈഅത്ത് ചെയ്തു.

ഉസ്മാന്‍(റ)വധിക്കപ്പെട്ടപ്പോള്‍ സൈന്യത്തലവന്മാര്‍ വന്ന് അലി(റ)യോട് അധികാരമേറ്റെടുക്കാനാവശ്യപ്പെട്ടു. അലി(റ)അധികാരമേ റ്റെടുക്കുവാന്‍ വിസമ്മതിക്കുകയാണുണ്ടായത്. മദീനയിലെ തലമുതിര്‍ന്ന നേതാക്കള്‍ ചെന്ന് കണ്ട് അധികാരമേറ്റെടുക്കാന്‍ ആവശ്യപ്പെട്ടപ്പോള്‍ പള്ളിയില്‍വെച്ച് പൊതുബൈഅത്ത് നടക്കുകയാണെങ്കില്‍ താന്‍ അധികാരമേറ്റെടുക്കാമെന്ന് അദ്ദേഹം സമ്മതിച്ചു. അങ്ങനെ മദീനയിലെ മസ്ജിദുന്നബവിയില്‍ മുസ്‌ലിം ബഹുജനങ്ങള്‍ സമ്മേളിച്ചു ബൈഅത്ത് ചെയ്ത ശേഷമാണ് അലി(റ)അധികാരമേറ്റെടുത്തത്.

സച്ചരിതരായിരുന്ന നാലു ഖലീഫമാരുടെ തെരഞ്ഞെടുപ്പ് രീതികള്‍ വ്യത്യസ്തമായിരുന്നുവെങ്കിലും അവയിലെല്ലാം പൊതുജനാഭിപ്രായത്തിന് അര്‍ഹമായ സ്ഥാനം നല്‍കപ്പെട്ടിരുന്നുവെന്ന് മുകളില്‍ വിവരിച്ച ചരിത്രത്തില്‍ നിന്ന് വ്യക്തമാണ്. ഇസ്‌ലാമിന്റെ അടിസ്ഥാനതത്ത്വങ്ങളില്‍ നിന്ന് വ്യതിചലിക്കാതെ സമയാനുസൃതമായ തെരഞ്ഞെടുപ്പുരീതി സ്വീകരിക്കുവാന്‍ ഇസ്‌ലാം സ്വാതന്ത്ര്യം നല്‍കിയിരിക്കുന്നുവെന്ന് സാരം. ഏത് തരത്തിൽ തെരെഞ്ഞെടുക്കപ്പെട്ടവരാണെങ്കിലും അവർ ഖുർആനും സുന്നത്തുമനുസരിച്ച് സത്യസന്ധവും നീതിനിഷ്ഠവുമായി ഭരണം നടത്തണമെന്നാണ് ഇസ്‌ലാമിന്റെ അനുശാസന.

രണ്ട് രീതിയിലാണ് ഒരു രാജ്യം ഇസ്‌ലാമികമായിത്തീരുന്നത്. അവിടെ ജീവിക്കുന്നവരിലെ മനഃപരിവർത്തനത്തിലൂടെയാണ് ഒന്നാമത്തേത്. മദീനാരാജ്യമുണ്ടായത് അങ്ങനെയാണ്. ഇസ്‌ലാമികരാഷ്ട്രത്തിന്റെ വികാസത്തിന്റെ ഭാഗമായി അയാൾ നാടുകൾ ഇസ്‌ലാമികമായിത്തത്തീരുന്നതാണ് രണ്ടാമത്തേത്. പ്രവാചകാനുചരന്മാരുടെ കാലത്ത് പലരാജ്യങ്ങളും ഇസ്‌ലാമികസാമ്രാജ്യത്തിന്റെ ഭാഗമായത് ഇങ്ങനെയാണ്. രണ്ടായിരുന്നാലും അന്ന് നിലവിലുള്ള രാഷ്ട്രനൈതികത അംഗീകരിച്ചവയാണവ. അട്ടിമറിയിലൂടെയോ തീവ്ര വാദപ്രവര്‍ത്തനങ്ങളിലൂടെയോ നാടിനെ ഇസ്‌ലാമികമാക്കുവാൻ കല്പിക്കുന്ന പ്രവാചകവചനങ്ങളൊന്നും തന്നെയില്ല. അട്ടിമറിയിലൂടെ ഇസ്‌ലാ മികരാഷ്ട്രം സ്ഥാപിക്കണമെന്ന് പ്രവാചകന്(സ) വിചാരമുണ്ടായി രുന്നുവെങ്കില്‍ അദ്ദേഹത്തിന്റെ പ്രവാചക ജീവിതത്തിന്റെ ആദ്യ ഘട്ടത്തില്‍ തന്നെ അത് സാധ്യമാകുമായിരുന്നു. അദ്ദേഹത്തിനടുത്ത് ക്വുറൈശീ പ്രതിനിധിയായ ഉത്ബത്തുബ്‌നു റബീഅ വന്ന് തന്റെ പ്രബോധനപ്രവര്‍ത്തനങ്ങള്‍ നിറുത്തിവെച്ചാല്‍ പണമോ പ്രതാപ മോ തരുണികളോ അധികാരമോ എന്താണ് ആവശ്യമെന്ന് വെച്ചാല്‍ അത് നല്‍കാമെന്ന് പറഞ്ഞപ്പോള്‍ ഒരു ചെറിയ അട്ടിമറി നടത്തിയാ ല്‍ മതിയായിരുന്നു. പ്രവാചകനിയോഗത്തിന്റെ ലക്ഷ്യം പക്ഷേ, അട്ടിമറിയിലൂടെ ഇസ്‌ലാമികരാഷ്ട്രം നിര്‍മിക്കുകയായിരുന്നില്ല. പ്ര ത്യുത, ജനങ്ങളെ സംസ്‌കരിക്കുകയും സംസ്‌കൃതമായ ഒരു സമൂഹ ത്തെ സൃഷ്ടിച്ച് ലോകത്തിന് മാതൃകയാവുകയുമായിരുന്നു. ഒരു ഇസ്‌ലാമികരാഷ്ട്രം നിര്‍മിച്ച് അതിന്റെ സാരഥിയാവുകയല്ല പ്രവാചകന്‍ ചെയ്തത്; മറിച്ച് ഒരു രാഷ്ട്രത്തിലെത്തിച്ചേരുവാനാ വശ്യമായ സാമൂഹ്യവിപ്ലവത്തിന് പാതയൊരുക്കുകയായിരുന്നു.

ഒരു രാഷ്ട്രത്തിലെ നിയമങ്ങളും അവിടുത്തെ പൗരന്മാരും തമ്മി ലുള്ള ബന്ധം രാഷ്ട്രമീമാംസയിലെ പ്രധാനപ്പെട്ട വിഷയങ്ങളിലൊ ന്നാണ്. ജനാധിപത്യ രാഷ്ട്രത്തില്‍ നിയമനിര്‍മാണം നടത്തുന്നത് ജനപ്രതിനിധി സഭയാണെങ്കിലും പൗരന് തന്നില്‍ അടിച്ചേല്‍പിക്കപ്പെടുന്നവയായിട്ടാണ് നിയമങ്ങള്‍ അനുഭവപ്പെടുന്നതെന്നാണ് സാമൂഹികശാസ്ത്രജ്ഞന്മാരുടെ പക്ഷം. ഉപഭോഗസംസ്‌കാരം നിലനില്‍ക്കുന്ന സമൂഹത്തില്‍ ഈ വികാരം അല്‍പം തീഷ്ണമാ യിരിക്കും. തന്റെ സുഖിക്കുവാനുള്ള അവകാശത്തിന്മേല്‍ രാഷ്ട്രം അടിച്ചേല്‍പിക്കുന്ന വിലക്കുകളായിട്ടാണ് നിയമങ്ങള്‍ അത്തരം സമൂഹങ്ങളിലെ പൗരന്ന് അനുഭവപ്പെടുക. അതുകൊണ്ടുതന്നെ നിയമത്തിന്റെ കുരുക്കുകളില്‍ നിന്ന് രക്ഷപ്പെടാന്‍ അവന്‍ പരമാ വധി ശ്രമിക്കും. അഴിമതിയും വഞ്ചനയും ജനാധിപത്യത്തിന്റെ ഉ പോല്‍പന്നമായി മാറുന്നത് അതുകൊണ്ടാണ്. മനംമാറ്റത്തിലാണ്

ഇസ്‌ലാമികരാഷ്ട്ര ത്തിലാകട്ടെ, താന്‍ അനുസരിക്കുന്നത് ദൈവികനിയമങ്ങളാണെന്ന ധാരണയുള്ളതിനാല്‍ നിയമലംഘനങ്ങള്‍ കുറവായിരിക്കും. തന്റെ ഭൗതികവും പാരത്രികവുമായ മോക്ഷത്തിന് അനിവാര്യമായ നിയമ ങ്ങളാണ് താന്‍ അനുസരിക്കുന്നതെന്ന ധാരണ പൗരനെ രാഷ്ട്ര ത്തോടു കൂറുള്ളവനാക്കിത്തീര്‍ക്കും. സര്‍വോപരിയായി നിയമപാല കരുടെ കണ്ണുവെട്ടിച്ചാലും നിയമദാതാവായ പടച്ചതമ്പുരാന്റെ കണ്ണു വെട്ടിക്കാന്‍ സാധ്യമല്ലെന്ന വിശ്വാസം അവനെ ഇസ്‌ലാമിക രാഷ് ട്രത്തിലെ നിയമങ്ങള്‍ പരസ്യമായും രഹസ്യമായും അനുസരി ക്കുന്നവനാക്കിത്തീര്‍ക്കും. അതാണ് ഒരു ദുര്‍ബലനിമിഷത്തില്‍ വ്യഭിചരിച്ചുപോയ ഇസ്‌ലാമിക രാഷ്ട്രത്തിലെ പൗരനെ സർക്കാരിന് മുമ്പില്‍ പ്രത്യക്ഷപ്പെട്ട് താന്‍ ചെയ്ത തെറ്റ് ഏറ്റുപറഞ്ഞ് ശിക്ഷ ശിരസാവഹിക്കുന്നവനാക്കിത്തീര്‍ത്തത്. രഹസ്യമായുള്ള നിയമ ലംഘനത്തില്‍ നിന്നുപോലും രാഷ്ട്രത്തിലെ പൗരന്മാരെ തടയുവാന്‍ തക്കവണ്ണമുള്ള രാഷ്ട്രസംവിധാനം സൃഷ്ടിക്കുവാന്‍ ഇസ്‌ലാ മിന്നല്ലാതെ മറ്റേത് തത്ത്വശാസ്ത്രത്തിനാണ് കഴിഞ്ഞിട്ടുള്ളത്? ഭരണമാറ്റം വഴി ഉണ്ടാകാനാവുന്നതല്ല ഈ പരിവർത്തനം; മനംമാറ്റത്തിലാണ് ഇസ്‌ലാം ഒന്നാമതായി ശ്രദ്ധിക്കുന്നത് എന്ന് പറയുന്നത് അത് കൊണ്ടാണ്.

രാഷ്ട്രത്തീയത്തെക്കുറിച്ച ഇസ്‌ലാമിന്റെ വീക്ഷണം അതിന്റെ സമൂഹസങ്കല്‍പത്തിന്റെ ഭാഗമാണ്. രാഷ്ട്രമില്ലെങ്കില്‍ ഇസ്‌ലാമിന്ന് തന്നെ നിലനില്‍പില്ലെന്ന വാദം അടിസ്ഥാനരഹിതമാണ്. അതേ പോലെത്തന്നെയാണ് രാഷ്ട്രീയത്തില്‍ ഇസ്‌ലാമികനിയമങ്ങള്‍ ക്കൊന്നും പ്രസക്തിയില്ലെന്ന വാദവും. ഇതു രണ്ടും രണ്ട് ആത്യന്തി കതകളാണ്. ഇവ്വിഷയകമായ ഇസ്‌ലാമിക വീക്ഷണമാകട്ടെ ഈ ആത്യന്തതകള്‍ക്ക് മധ്യെയാണ് താനും.

ഒരു ഇസ്‌ലാമികേതര രാഷ്ട്രത്തില്‍ ജീവിക്കുന്ന മുസ്‌ലിമിനെ സംബന്ധിച്ചിടത്തോളം അവന്ന് പാലിക്കുവാന്‍ കഴിയുന്ന രാഷ്ട്രീ യ നിയമങ്ങള്‍ ഇസ്‌ലാം പഠിപ്പിക്കുന്നുണ്ട്. രാഷ്ട്രം ഇസ്‌ലാമിനു വിരുദ്ധമാകാത്തിടത്തോളം തന്റെ രാജ്യത്തിന്റെ പുരോഗതിക്കു വേണ്ടി പ്രവര്‍ത്തിക്കുന്നതോ രാജ്യകാര്യങ്ങളില്‍ പങ്കുവഹിക്കുന്ന തോ മുസ്‌ലിമിന് നിഷിദ്ധമല്ല. മഹാനായ യൂസുഫ് നബിൗയുടെ ചരിത്രം വിവരിക്കുന്നിടത്ത് ക്വുര്‍ആന്‍ ഇക്കാര്യത്തിലേക്കാണ് വെളി ച്ചം വീശുന്നത്. ഇസ്‌ലാംമതം സ്വീകരിച്ചിട്ടില്ലാതിരുന്ന ഒരു രാജാവി നു കീഴില്‍ മന്ത്രിപദം വഹിച്ച ആ മഹാനായ പ്രവാചകന്‍ നീതിപാ ലിക്കുകയും രാഷ്ട്രസേവനം നിര്‍വഹിക്കുകയും ചെയ്തുവെന്ന് ക്വുര്‍ആനിലെ ചരിത്രകഥനം വ്യക്തമാക്കുന്നു. ഇസ്‌ലാമിനു വിരു ദ്ധമല്ലാത്ത അടിസ്ഥാനാദര്‍ശങ്ങളിന്‍മേല്‍ സ്ഥാപിതമായ ഒരു രാ ഷ്ട്രത്തിന്റെ കുഞ്ചികസ്ഥാനങ്ങള്‍ വഹിക്കുമ്പോള്‍ മുസ്‌ലിംകള്‍ എങ്ങനെയായിരിക്കണമെന്ന് പ്രസ്തുത കഥനത്തില്‍ നിന്ന് മനസ്സി ലാക്കാന്‍ കഴിയും.

മുഹമ്മദ് നബിയുടെ മക്കാ കാലഘട്ടത്തിലെ നടപടികളും ഇസ്‌ലാമികമല്ലാത്ത ഭരണസംവിധാനങ്ങൾക്ക് കീഴിൽ എങ്ങനെ മുസ്ലിംകൾ വർത്തിക്കണമെന്ന് പഠിപ്പിക്കുന്നുണ്ട്. നാടിനോട് കലാപമുണ്ടാക്കാതെ, നിലനിൽക്കുന്ന സാഹചര്യങ്ങളെ പരമാവധി പ്രയോജനപ്പെടുത്തിക്കൊണ്ട് മതജീവിതം നയിക്കുകയും മതപ്രബോധനത്തിലേർപ്പടുകയുമാണ് നബി (സ) ചെയ്തത്. മുസ്ലിംകൾ അല്ലാത്തവരുടെ സഹായം തെറ്റുകയും നിലവിലുള്ള വ്യവസ്ഥയുടെ സാധ്യതകൾ ഉപയോഗപ്പെടുത്തുകയും ചെയ്ത പ്രവാചകന്റെ മക്കാജീവിതനടപടികൾ ഇസ്‌ലാമികേതരമായ രാഷ്ടസംവിധാനങ്ങളോട് എത്രത്തോളം മുസ്ലിംകൾക്ക് സഹകരിക്കാനാവുമെന്ന് വ്യക്തമാക്കുന്നുണ്ട്.

മത-ധാര്‍മികമൂല്യങ്ങളില്‍ നിന്ന് വ്യതിചലിക്കാതെ, ഒരു ഇസ്‌ലാ മികേതര രാഷ്ട്രത്തിലെ രാഷ്ട്രീയ പ്രവര്‍ത്തനങ്ങളില്‍ പങ്കാളിയാ വുന്നതില്‍ നിന്ന് സത്യവിശ്വാസികളെ തടയുന്നതിന്ന് വിശുദ്ധ ക്വുര്‍ആനിന്റെയോ തിരുനബിചര്യയുടെയോ യാതൊരു പിന്‍ബലവു മില്ല. ഒരു മുസ്‌ലിം ജീവിത്തില്‍ എല്ലാ രംഗങ്ങളിലും ഇസ്‌ലാമികമാ യ വിധിവിലക്കുകള്‍ പാലിക്കാന്‍ ബാധ്യസ്ഥനായതുപോലെ രാഷ് ട്രീയത്തിലും പ്രസ്തുത വിധിവിലക്കുകള്‍ പാലിക്കുവാന്‍ ബാധ്യസ് ഥനാണ്. രാഷ്ട്രീയത്തില്‍ മതധാര്‍മികമൂല്യങ്ങള്‍ക്ക് യാതൊരു വിലയുമില്ലെന്ന വീക്ഷണം ഇസ്‌ലാമിന്ന് അന്യമാണ്. 'ദൈവത്തിനു ള്ളത് ദൈവത്തിനും സീസര്‍ക്കുള്ളത് സീസര്‍ക്കും' എന്ന ചര്‍ച്ചി ന്റെ വിഭജനരീതിയും ഇസ്‌ലാം അംഗീകരിക്കുന്നില്ല. ഭരണരംഗ ത്തുള്ളവര്‍ ദൈവികമായ വിധിവിലക്കുകള്‍ പാലിക്കണമെന്ന് ശക് തമായി നിഷ്‌കര്‍ഷിക്കുന്ന മതമാണ് ഇസ്‌ലാം. അഴിമതിയും സ്വജനപക്ഷപാതിത്വവും വഞ്ചനയും ചൂഷണവുമൊന്നും രാഷ്ട്രീയ പ്രവര്‍ത്തനത്തിന്റെ ഭാഗമായി ഒരു മുസ്‌ലിം സ്വീകരിക്കുവാന്‍ പാടി ല്ലാത്തതാണ്.

മുഹമ്മദ് നബി(സ)യുടെ പ്രവാചകത്വത്തിന്റെ ദൃഷ്ടാന്തമെന്ന നിലയില്‍ അല്ലാഹു ചന്ദ്രനെ പിളര്‍ത്തിയെന്നും മക്കക്കാര്‍ അതിന് സാക്ഷികളായെന്നും പറയുന്ന ഇസ്‌ലാമിക പാരമ്പര്യങ്ങള്‍ അടിസ്ഥാനരഹിതവും അസ്വീകാര്യവുമാണ്. കാരണം, ചാന്ദ്രപിളര്‍പ്പ് ചരിത്രപരമായി സ്ഥാപിക്കപ്പെട്ടതോ ശാസ്ത്രീയമായി സംഭവ്യമോ അല്ല. ഇനി ചാന്ദ്രപ്പിളര്‍പ്പും പുനസമാഗമവും അത്യത്ഭുതകരമായി സംഭവിച്ചാല്‍ തന്നെ അതിന്റെ ഭൗതികാഘാതങ്ങള്‍ ചന്ദ്രോപരിതലത്തില്‍ ദൃശ്യമാകേണ്ടതാണ്. അങ്ങനെ യാതൊന്നും ഇന്നുവരെ കണ്ടെത്താനായിട്ടില്ല. പ്രവാചകന്റെ മഹത്വം വര്‍ധിപ്പിച്ചു കാണിക്കുന്നതിനുവേണ്ടി നിര്‍മിക്കപ്പെട്ട ഒരു വ്യാജകഥയാണ് ചാന്ദ്രപ്പിളര്‍പ്പിന്റേത് എന്നാണ് ഇതെല്ലാം വ്യക്തമാക്കുന്നത്. മിഷനറി വെബ്‌സൈറ്റുകളില്‍ സര്‍വസാധാരണമായ ഈ വാദങ്ങളില്‍ കഴമ്പില്ലേ?

 ഇല്ല. പ്രവാചകത്വത്തിന് തെളിവുകള്‍ ആവശ്യപ്പെട്ടുകൊണ്ടുള്ള മക്കന്‍ ബഹുദൈവാരാധകരുടെ അന്വേഷണങ്ങള്‍ക്കുള്ള മറുപടിയായും സത്യവിശ്വാസികള്‍ക്കുള്ള അനുഗ്രഹമായും അല്ലാഹു സംഭവിപ്പിച്ച നിരവധി അമാനുഷിക ദൃഷ്ടാന്തങ്ങള്‍കൊണ്ട് നിബിഢമാണ് നബിജീവിതമെന്ന് ചരിത്രം വ്യക്തമാക്കുന്നുണ്ട്. പ്രവാചകത്വത്തിന്റെ ഇരുപത്തിമൂന്ന് വര്‍ഷക്കാലയളവിനെ സമൃദ്ധമാക്കി ആകാശത്തുനിന്ന് ഭൂമിയിലേക്കവതരിപ്പിക്കപ്പെടുകയും ക്വുര്‍ആന്‍ ആയി ക്രോഡീകരിക്കപ്പെടുകയും ചെയ്ത ആറായിരത്തില്‍പരം ദിവ്യവചസ്സുകള്‍ തന്നെയായിരുന്നു അവയില്‍ ഏറ്റവും പ്രധാനപ്പെട്ടത്. ക്വുര്‍ആനിനു പുറമെ, അല്ലാഹുവില്‍നിന്ന് ലഭിച്ച വിസ്മയകരങ്ങളായ സഹായങ്ങളായും സംരക്ഷണങ്ങളായും കിറുകൃത്യമായ പ്രവചനപ്പുലര്‍ച്ചകളായും മനുഷ്യസാധ്യമല്ലാത്ത യാത്രാനുഭവങ്ങളായും പ്രകൃതിയിലും തീന്‍മേശയിലും മനുഷ്യശരീരങ്ങളിലും വരെ പ്രകടമായ അത്ഭുതങ്ങളായും ദിവ്യദൃഷ്ടാന്തങ്ങള്‍ മുഹമ്മദ് നബി(സ)യുടെ പ്രവാചകത്വത്തിന് സാക്ഷി പറഞ്ഞുകൊണ്ടേയിരിന്നിട്ടുണ്ട്.

അത്തരത്തില്‍ ചരിത്രം രേഖപ്പെടുത്തിയ അനേകം അത്ഭുത സംഭവങ്ങളിലൊന്നു മാത്രമാണ് ചന്ദ്രന്റെ പിളര്‍പ്പും പുനസമാഗമവും. പരിശുദ്ധ ക്വുര്‍ആന്‍ തന്നെ നേര്‍ക്കുനേരെ പരാമര്‍ശിച്ചിട്ടുള്ളതും അതുകൊണ്ടുതന്നെ മുസ്‌ലിം ലോകത്ത് സുപ്രസിദ്ധവുമാണ് ഈ സംഭവം. ചാന്ദ്രപിളര്‍പ്പിനെക്കുറിച്ച് ക്വുര്‍ആന്‍ സംസാരിച്ചതിന്റെ ആശയമിങ്ങനെയാണ്:

”ആ (അന്ത്യ) സമയം അടുത്തു. ചന്ദ്രന്‍ പിളരുകയും ചെയ്തു. ഏതൊരു ദൃഷ്ടാന്തം അവര്‍ കാണുകയാണെങ്കിലും അവര്‍ പിന്തിരിഞ്ഞു കളയുകയും, ഇത് നിലനിന്നു വരുന്ന ജാലവിദ്യയാകുന്നു എന്ന് അവര്‍ പറയുകയും ചെയ്യും. അവര്‍ നിഷേധിച്ചു തള്ളുകയും തങ്ങളുടെ തന്നിഷ്ടങ്ങളെ പിന്‍പറ്റുകയും ചെയ്തിരിക്കുന്നു. ഏതൊരു കാര്യവും ഒരു നിശ്ചിത സ്ഥാനം പ്രാപിക്കുന്നതാകുന്നു.” (54: 1-3)

അന്തിമ പ്രവാചകന്റെ ദൈവിക ദൃഷ്ടാന്തമെന്ന നിലയില്‍ ചന്ദ്രന്‍ പിളര്‍ന്നുവെന്നും പക്ഷേ എത്ര വലിയ അത്ഭുതങ്ങള്‍ കണ്ടാലും മാരണമോ ജാലവിദ്യയോ ആയി തള്ളുകയാണ് മക്കയിലെ പല സത്യനിഷേധികളുടെയും രീതിയെന്നും അത്തരക്കാര്‍ ഈ മഹാത്ഭുതത്തെയും ആ ഗണത്തില്‍പ്പെടുത്തി തള്ളിയെന്നുമുള്ള വിവരങ്ങള്‍ ചാന്ദ്രപിളര്‍പ്പിനെക്കുറിച്ച് ഈ ക്വുര്‍ആന്‍ വചനങ്ങള്‍ നല്‍കുന്നുണ്ട്. ക്വുര്‍ആന്‍ വചനങ്ങളെല്ലാം പ്രവാചകകാലഘട്ടത്തിലേതാണെന്ന കാര്യം ചരിത്രപരമായി അവിതര്‍ക്കിതമാംവിധം സ്ഥാപിക്കപ്പെട്ടതാണ്. ആകാശത്ത് ചന്ദ്രന്റെ പിളര്‍പ്പ് ദൃശ്യമാവുകയും അതിനെ ദൈവിക ദൃഷ്ടാന്തമായി പരിഗണിക്കാന്‍ വിസമ്മതിച്ച ചില മക്കക്കാര്‍ പ്രസ്തുത ദൃശ്യത്തിന് മറ്റു വ്യാഖ്യാനങ്ങള്‍ നല്‍കാന്‍ ശ്രമിക്കുകയും ചെയ്തുവെന്ന് പ്രഖ്യാപിക്കുന്ന ക്വുര്‍ആന്‍ വചനങ്ങള്‍ നബിനാവിലൂടെ മക്ക കേട്ടതാണ്; അദ്ദേഹത്തിന്റെ അനുചരന്‍മാര്‍ മറ്റെല്ലാ ക്വുര്‍ആന്‍ വചനങ്ങളെയും പോലെ സമകാലീനരായ അവിശ്വാസികള്‍ക്കിടയില്‍ നിരന്തരമായി അവ പാരായണം ചെയ്ത് പ്രഘോഷിച്ചുകൊണ്ടിരിക്കുകയും ചെയ്തിട്ടുണ്ട്. എന്നിട്ടും മക്കയിലെ ഒരു ബഹുദൈവാരാധകനും ഈ ക്വുര്‍ആന്‍ വചനങ്ങളെ ഒരിക്കല്‍ പോലും ചോദ്യം ചെയ്തു രംഗത്തുവന്നില്ല. അതിനര്‍ത്ഥം ചന്ദ്രന്‍ പിളര്‍ന്നതിന് ബഹുദൈവാരാധകരടക്കമുള്ള പല മക്കക്കാരും സാക്ഷിയായിയെന്നും അതിന്റെ സ്വഭാവത്തെക്കുറിച്ചുള്ള കുതര്‍ക്കങ്ങളുന്നയിക്കുക മാത്രമാണവര്‍ ചെയ്തതെന്നുമുള്ളതിനുള്ള അനിഷേധ്യമായ ചരിത്രരേഖകളായി പരാമൃഷ്ട ക്വുര്‍ആന്‍ വചനങ്ങള്‍ മാറുന്നുവെന്ന് തന്നെയാണ്.

മദീനയിലേക്കുള്ള പലായനത്തിന്റെ ഉദ്ദേശം അഞ്ചു വര്‍ഷം മുമ്പ് ഒരു പൗര്‍ണമി രാവിലാണ് നബി തിരുമേനി (സ) തന്റെ ചുറ്റുമുണ്ടായിരുന്നവര്‍ക്ക് ചന്ദ്രന്‍ പിളര്‍ന്നുമാറുന്നത് കാണിച്ചുകൊടുത്തത് എന്നാണ് ചരിത്രകാരന്‍മാരുടെ പൊതുവായ അഭിപ്രായം. ചാന്ദ്രപിളര്‍പ്പ് ദൃശ്യമായതിനുള്ള ചരിത്രരേഖ ക്വുര്‍ആന്‍ മാത്രമല്ല; പ്രത്യുത മക്കയില്‍ പ്രവാചകന്റെ സമകാലീനരായവരില്‍ നിന്നുള്ള വിശ്വസ്തമായ ധാരാളം നിവേദനങ്ങള്‍ കൂടിയാണ്. മുഹമ്മദ് നബി(സ)യുടെ കാലത്ത് ചന്ദ്രന്‍ രണ്ടുഭാഗങ്ങളായി പിളര്‍ന്നുമാറിയെന്നു പറയുന്ന ഒട്ടനവധി ഹദീഥുകളാണ് ആധികാരികതയില്‍ ആര്‍ക്കും സംശയമില്ലാത്ത അനേകം പരമ്പരകള്‍ വഴി ഏറ്റവും പ്രാമാണികമായ ഹദീഥ് സമാഹാരങ്ങളില്‍ തന്നെ രേഖപ്പെടുത്തപ്പെട്ടിരിക്കുന്നത്. പ്രവാചകകാലം മുതല്‍ക്കുള്ള ഓരോ തലമുറയിലും അനേകമാളുകള്‍ ഒരുമിച്ച് കൈമാറി വന്നുവെന്ന് (മുതവാതിര്‍) പറയാനാകുംവിധം വിഖ്യാതവും അനിഷേധ്യവുമാണ് ഈ ഹദീഥുകളിലെ സംഭവവിവരണങ്ങള്‍. സ്വഹീഹുല്‍ ബുഖാരിയിലെ കിതാബുത്തഫ്‌സീറിലും കിതാബുല്‍ മനാക്വിബില്‍ അന്‍സ്വാറിലും സ്വഹീഹുമുസ്‌ലിമിലെ ക്വിതാബുസ്വിഫാത്തില്‍ ക്വിയാമതി വല്‍ജന്നതി വന്നാറിലും ചന്ദ്രന്‍ രണ്ടായി പിളര്‍ന്നുനീങ്ങിയെന്ന് ഖണ്ഡിതമായി പ്രസ്താവിക്കുന്ന ഹദീഥുകളുണ്ട്. മക്കക്കാര്‍ പ്രവാചകനോട് ദൃഷ്ടാന്തങ്ങള്‍ക്കുവേണ്ടി ചോദിച്ചപ്പോഴാണ് നബി (സ) ചന്ദ്രന്‍ പിളരുന്നത് കാണിച്ചുകൊടുത്തതെന്ന് സൂചിപ്പിക്കുന്ന ഹദീഥുകള്‍ സ്വഹീഹുല്‍ ബുഖാരിയിലെ കിതാബുല്‍ മനാക്വിബില്‍ അന്‍സ്വാറിലും കിതാബു ഫദാഇലി അസ്‌വ്ഹാബിന്നബിയിലും കാണാന്‍ കഴിയും. സംഭവം നടക്കുമ്പോള്‍ നബി (സ) മക്ക നഗരത്തിന് തൊട്ടപ്പുറത്തുള്ള മിനായിലായിരുന്നുവെന്ന് സ്വഹീഹുല്‍ ബുഖാരിയിലെ കിതാബുല്‍ മനാക്വിബില്‍ അന്‍സ്വാറില്‍നിന്ന് മനസ്സിലാക്കാനാകും.

ചന്ദ്രന്‍ പിളര്‍ന്നുണ്ടായ രണ്ട് കഷ്ണങ്ങളില്‍ ഒന്ന് ഒരു പര്‍വതത്തിന്റെ ഒരു വശത്തും മറ്റേത് മറുവശത്തുമായി നിന്നുവെന്നും അപാരമായ വ്യക്തതയുണ്ടായിരുന്ന ആ പിളര്‍ന്നുമാറല്‍ ദൃശ്യത്തെ ചൂണ്ടി പ്രവാചകന്‍ (സ) കൂടെയുണ്ടായിരുന്നവരോട് ‘സാക്ഷിയായിക്കൊള്ളുക’ എന്നു പറഞ്ഞുവെന്നും സ്വഹീഹുല്‍ ബുഖാരിയിലെ കിതാബുത്തഫ്‌സീറിലെയും സ്വഹീഹുമുസ്‌ലിമിലെ ക്വിതാബുസ്വിഫാത്തില്‍ ക്വിയാമതി വല്‍ജന്നതി വന്നാറിലെയും ഹദീഥുകളിലുണ്ട്. വിസ്മയകരമായ ഈ അനുഭവങ്ങള്‍ക്ക് സാക്ഷിയായി പ്രവാചക സന്നിധിയിലുണ്ടായിരുന്നവരില്‍ താനുമുള്‍പ്പെട്ടിരുന്നതായി അബ്ദുല്ലാഹിബ്‌നു മസ്ഊദ് (റ) സാക്ഷ്യപ്പെടുത്തിയത് മുസ്‌ലിമിലെ ക്വിതാബുസ്വിഫാത്തില്‍ ക്വിയാമതി വല്‍ജന്നതി വന്നാറില്‍ ഉദ്ധരിക്കപ്പെട്ടിരിക്കുന്നു.

മുഹമ്മദ് നബി(സ)യുടെ ജീവിതകാലത്ത് തന്റെ പ്രവാചകത്വത്തിന് തെളിവായി അല്ലാഹു ചന്ദ്രനെ രണ്ടു ഭാഗമാക്കി മാറ്റിനിര്‍ത്തുന്നത് അദ്ദേഹം തന്റെ ചുറ്റുമുണ്ടായിരുന്ന ചിലര്‍ക്ക് കാണിച്ചുകൊടുത്തുവെന്ന ഇസ്‌ലാമിക പാരമ്പര്യത്തിന് ചരിത്രപരമായ അടിത്തറയില്ലെന്ന മിഷനറി വാദം ശുദ്ധ അസംബന്ധമാണെന്ന് ക്വുര്‍ആനും ഹദീഥുകളും പരിശോധിക്കുന്ന ആര്‍ക്കും മനസ്സിലാകുമെന്നാണ് പറഞ്ഞുവന്നതിന്റെ ചുരുക്കം. ഒട്ടനേകം അനിഷേധ്യമായ രേഖീകരണങ്ങളുള്ള ചാന്ദ്രപ്പിളര്‍പ്പ് നടന്നത് ചരിത്രത്തിന്റെ വെള്ളിവെളിച്ചത്തിലാണെന്ന് പറയുന്നതായിരിക്കും ശരി.

ഇങ്ങനെയൊരു സംഭവം കണ്ടതായി ഹിജാസിനു പുറത്തുള്ള ആരും രേഖപ്പെടുത്തിയിട്ടില്ലെന്നു വാദിച്ചാണ് ചില മിഷനറിമാര്‍ ചന്ദ്രന്‍ പിളര്‍ന്നതിന്റെ ചരിത്രപരത നിഷേധിക്കുവാന്‍ ശ്രമിക്കാറുള്ളത്. യഥാര്‍ത്ഥത്തില്‍ ചന്ദ്രന്റെ ദൃശ്യത ഒരു സമയത്തും ഭൂഗോളത്തില്‍ എല്ലാ ഭാഗത്തും ഒരു പോലെയല്ലെന്നും മക്കയുടെ ചക്രവാളം പങ്കിടുന്ന ഭൂപ്രദേശങ്ങളില്‍ തന്നെ മുന്‍പ്രഖ്യാപനമോ ആഗോളശ്രദ്ധ നേടിയ വിളംബരങ്ങളോ ഇല്ലാതെ ആകാശത്ത് പൊടുന്നനെ സംഭവിക്കുകയും ഏതാനും സമയത്തിനകം അവസാനിക്കുകയും ചെയ്ത ഒരു സംഭവം ശ്രദ്ധിക്കപ്പെടാതെ പോവുക തികച്ചും സ്വാഭാവികമാണെന്നും വാനനിരീക്ഷണം ആധുനിക കാലത്തേതുപോലെ വ്യവസ്ഥാപിതമായിത്തീരാത്ത ഒരു കാലത്ത് അസാധാരണമായ ഒരു ആകാശക്കാഴ്ചക്ക് നിരീക്ഷകക്കുറിപ്പുകളുടെ ആധിക്യമുണ്ടാകുമെന്ന് പ്രതീക്ഷിക്കുന്നത് ബാലിശമാണെന്നും മേഘപടലങ്ങളോ ഏതാനും സമയം എന്തു സംഭവിച്ചുവെന്ന് മനസ്സിലാകാതിരിക്കലോ മതി ഇത്തരമൊരു സന്ദര്‍ഭം കുറേ പ്രദേശങ്ങളുടെ ‘ചരിത്ര’ത്തില്‍ നിന്ന് ‘പുറത്താകാന്‍’ എന്നും കണ്ടതെല്ലാം എല്ലാ കാലത്തും എല്ലാ മനുഷ്യരും രേഖപ്പെടുത്താറില്ലെന്നും രേഖപ്പെടുത്തിയതെല്ലാം ചരിത്രഗവേഷകര്‍ കണ്ടെത്തിക്കഴിഞ്ഞിട്ടില്ലെന്നുമുള്ള ലളിതവസ്തുതകള്‍ മാത്രം കണക്കിലെടുത്താല്‍ മതി ഈ പരിശ്രമങ്ങള്‍ എന്തുമാത്രം അര്‍ത്ഥശൂന്യമാണെന്നു മനസ്സിലാകാന്‍.

മക്കയില്‍ സാക്ഷികളുണ്ടായതിന് അനിഷേധ്യമായ ചരിത്രരേഖകളുള്ള ഒരു ആകാശദൃശ്യത്തെ തള്ളിക്കളയുവാനുള്ള യാതൊരു ന്യായവും മിഷനറിമാരുന്നയിക്കുന്ന മറുവാദങ്ങളിലൊന്നുമില്ലെന്നു സാരം. പ്രവാചകകാലഘട്ടത്തിലെ ചാന്ദ്രപ്പിളര്‍പ്പിന് സാക്ഷിയാവുകയോ അതിന്റെ വാര്‍ത്തകളോട് പ്രതികരിക്കുകയോ ചെയ്ത ഇന്‍ഡ്യന്‍ രാജാക്കന്‍മാരെ സംബന്ധിച്ച പ്രചുരപ്രചാരമാര്‍ജ്ജിച്ച പാരമ്പര്യങ്ങളിലേക്ക് ഇവിടെ പ്രവേശിക്കാത്തത് അവയ്ക്ക് ചരിത്രപരത ഇല്ലെന്നുവന്നാലും ചാന്ദ്രപ്പിളര്‍പ്പ് ദൃശ്യം ഒരു ചരിത്രസംഭവമാണെന്ന യാഥാര്‍ത്ഥ്യത്തെ അതൊരിക്കലും ബാധിക്കാന്‍ പോകുന്നില്ല എന്നതിനാലാണ്.

ചാന്ദ്രപ്പിളര്‍പ്പ് ആകാശത്ത് മക്കക്കാര്‍ക്ക് ദൃശ്യമായിയെന്നുറപ്പിക്കുകയാണ് ചരിത്രത്തിന്റെ ദൗത്യം; ആ ദൗത്യമാണ് ചരിത്രശാസ്ത്രത്തിന്റെ വീക്ഷണത്തില്‍ നാം ചര്‍ച്ച ചെയ്ത രേഖകള്‍ നിര്‍വഹിക്കുന്നത്. പ്രസ്തുത ദൃശ്യം എങ്ങനെയുണ്ടായതാണെന്ന് പറയാന്‍ ചരിത്രത്തിന് കഴിഞ്ഞുകൊള്ളണമെന്നില്ല. ചന്ദ്രനെ അല്ലാഹു ഒരമാനുഷിക ദൃഷ്ടാന്തമെന്ന നിലയില്‍ പിളര്‍ത്തിയതാണെന്ന മുഹമ്മദ് നബി(സ)യുടെ വിശദീകരണം വിശ്വാസത്തിന്റെ മണ്ഡലത്തിലുള്ളതാണ്. മിഷനറിമാര്‍ക്ക് പരമാവധി ചെയ്യാനാവുക ആ വിശദീകരണത്തോട് വിയോജിക്കുകയാണ്; മക്കയിലെ ബഹുദൈവാരാധകര്‍ ചെയ്തതുപോലെ. ചന്ദ്രന്‍ പിളര്‍ന്നതായുള്ള ഒരു പ്രതീതി തങ്ങളുടെ കണ്ണുകള്‍ക്കുണ്ടാകുന്ന കണ്‍കെട്ടാണ് മുഹമ്മദ് നബി (സ) നടത്തിയതെന്നാണ് അവര്‍ ആരോപിച്ചത്. ഇത് ചാന്ദ്രപ്പിളര്‍പ്പിന്റെ കാര്യത്തില്‍ മാത്രമല്ല, ക്വുര്‍ആന്‍ അടക്കമുള്ള മുഴുവന്‍ നബിദൃഷ്ടാന്തങ്ങളുടെയും കാര്യത്തില്‍ അവരുയര്‍ത്തിയ നിലപാടാണ്. തങ്ങള്‍ നിഷേധിച്ചുതള്ളിയ പ്രവാചകനിലൂടെ അത്യത്ഭുകരമായ ദൃഷ്ടാന്തങ്ങള്‍ പ്രത്യക്ഷപ്പെടുന്നത് കണ്ടുണ്ടായ അമ്പരപ്പ് മറച്ചുവെക്കാന്‍ യാതൊരാത്മാര്‍ത്ഥതയുമില്ലാതെ അവര്‍ പറഞ്ഞുേനാക്കിയ ഒഴിവുകഴിവ് മാത്രമാണതെന്ന് ചരിത്രം സൂക്ഷ്മമായി പരിശോധിക്കുന്നവര്‍ക്കു മനസ്സിലാകും. കണ്‍കെട്ടിന്റെ പരിമിതശേഷികള്‍കൊണ്ട് സാധ്യമാകുന്നവയല്ല തങ്ങള്‍ സാക്ഷ്യം വഹിച്ചുകൊണ്ടിരുന്ന നബിയത്ഭുതങ്ങളെന്നും അവ ദൈവികമായ ഇടപെടലാകാനേ തരമുള്ളൂ എന്നും അവര്‍ക്ക് വ്യക്തമായിരുന്നു. ദുര്‍വാശിയില്‍ നിന്നുണ്ടായ കപട ആരോപണമായിരുന്നു ജാലവിദ്യയുടേത്. ചാന്ദ്രപിളര്‍പ്പ് സംഭവത്തിന്റെ വിശദാംശങ്ങള്‍ തന്നെ ഇക്കാര്യം വ്യക്തമാക്കുന്നുണ്ട്.

സംഭവം മാരണജന്യമായ വ്യാജപ്രതീതിയോ മറ്റോ ആണെന്ന് വരുത്താന്‍, നമ്മള്‍ മാത്രമേ (നബിയുടെ തൊട്ടടുത്തുള്ളവര്‍) ഇങ്ങനെ കാണുന്നുണ്ടായിരിക്കുകയുള്ളൂ എന്നും അങ്ങനെ സംഭവിക്കുന്നത് കണ്‍കെട്ടായതുകൊണ്ടാണെന്നും മറ്റാരെങ്കിലും കണ്ടതായി തെളിഞ്ഞാല്‍ മാത്രമേ ഇത് ജാലവിദ്യയല്ലെന്നു വരൂ എന്നും അവര്‍ പ്രഖ്യാപിച്ചതായി ജാമിഉത്തിര്‍മിദിയിലെ കിതാബു തഫ്‌സീറില്‍ ക്വുര്‍ആനി അന്‍ റസൂലില്ലാഹിയില്‍ രേഖപ്പെടുത്തിയിട്ടുള്ള ഹദീഥില്‍ വായിക്കാന്‍ കഴിയും. സിഹ്ര്‍ എന്നവര്‍ വിളിച്ചിരുന്ന പലതരം കണ്‍കെട്ടു വിദ്യകളുടെ പരിമിതിയെക്കുറിച്ചുള്ള ബോധ്യമാണ് ഈ വര്‍ത്തമാനത്തില്‍ നിഴലിക്കുന്നത്. പിന്നീടെന്ത് സംഭവിച്ചുവെന്ന് പ്രഗല്‍ഭനായ ആദ്യകാല ക്വുര്‍ആന്‍ വ്യാഖ്യാതാവും ചരിത്രകാരനുമായ അബൂ ജഅ്ഫര്‍ ഇബ്‌നു ജരീറുത്ത്വബ്‌രി തന്റെ ബൃഹദ് ക്വുര്‍ആന്‍ വ്യാഖ്യാനഗ്രന്ഥമായ ജാമിഉല്‍ ബയാനി അന്‍ തഅ്‌വീലി അയ്യുല്‍ ക്വുര്‍ആനില്‍ ചാന്ദ്രപ്പിളര്‍പ്പിനെ പരാമര്‍ശിക്കുന്ന ക്വുര്‍ആന്‍ വചനങ്ങളുടെ (54: 1-3) വിശദീകരണക്കുറിപ്പില്‍ ഉള്‍പ്പെടുത്തിയ നിവേദനത്തിലുണ്ട്. ചന്ദ്രന്‍ പിളര്‍ന്നത് കണ്ടുവോ എന്ന് ആ രാത്രി മക്കക്കു പുറത്തായിരുന്ന ഏതെങ്കിലും യാത്രക്കാരോട് ചോദിക്കാന്‍ അവര്‍ തീരുമാനിച്ചു. അങ്ങനെ ഒരു യാത്രാസംഘത്തോട് അവര്‍ അന്വേഷിച്ചു. ‘അതെ, ഞങ്ങള്‍ അത് കണ്ടു’വെന്നായിരുന്നു അവരുടെ മറുപടി. അപ്പോഴാണ് ഈ ക്വുര്‍ആന്‍ വചനങ്ങള്‍ അവതരിപ്പിക്കപ്പെട്ടത്. (കെയ്‌റോ, ദാറുല്‍ ഹിജ്ര്‍, 2001, Vol. 23, pp.106-7).

തങ്ങള്‍ കണ്ട ആകാശദൃശ്യം കണ്‍കെട്ടല്ലെന്ന് ഇതോടുകൂടി സത്യനിഷേധികള്‍ക്ക് ബോധ്യം വന്നുവെന്ന കാര്യമുറപ്പാണ്. കണ്‍കെട്ടിന് തങ്ങള്‍ തന്നെ വെച്ച മാനദണ്ഡങ്ങള്‍ ചാന്ദ്രപിളര്‍പ്പിനു ബാധകമല്ലെന്നു മനസ്സിലായിട്ടും പ്രസ്തുത വാദത്തിലവര്‍ ഉറച്ചുനിന്നത് സത്യത്തോട് വിമുഖരാകുമാറ് അവരില്‍ രൂഢമൂലമായിരുന്ന അഹങ്കാരം കൊണ്ടുമാത്രമാണെന്നര്‍ത്ഥം. ചന്ദ്രന്‍ പിളര്‍ന്നുനിന്ന ആകാശദൃശ്യം എന്ന അനിഷേധ്യമായ ചരിത്രസംഭവത്തെ മക്കന്‍ ബഹുദൈവാരാധകരുടെ കൂടെനിന്ന് മന്ത്രവാദവല്‍കരിക്കുവാനുള്ള ശ്രമങ്ങള്‍ ഇന്നും അഹങ്കാരത്തില്‍ നിന്നുമാത്രമാണ് നിര്‍ഗളിക്കുക; അതിനോട് സംവദിക്കാന്‍ ചരിത്രത്തിന്റെ ആയുധങ്ങള്‍ മാത്രം മതിയാകില്ല.

യഥാര്‍ത്ഥത്തില്‍, പ്രവാചകന്‍മാരിലൂടെ വെളിപ്പെടുന്ന അത്ഭുതസംഭവങ്ങളെ ജാലവിദ്യയോ മാരണമോ വഴിയുള്ള കണ്‍കെട്ടായി തള്ളിക്കളഞ്ഞ് പടച്ചവനു കാണിക്കാന്‍ കഴിയുന്നതൊക്കെ കാണിക്കാന്‍ കഴിയുന്ന സൂപ്പര്‍ പവറുകളായി കണ്‍കെട്ടുകാരെ അവതരിപ്പിക്കാന്‍ മുതിര്‍ന്ന അസംബന്ധ നാടകക്കാര്‍ ചരിത്രത്തിന്റെ എല്ലാ ഘട്ടങ്ങളിലുമുണ്ടായിട്ടുണ്ട്. കണ്‍കെട്ടുകാരുടെ ചെപ്പടിവിദ്യകളുമായി താരതമ്യം പോലുമില്ലാത്ത മഹാത്ഭുതങ്ങളാണ് സംഭവിക്കുന്നതെന്നു മനസ്സിലായിട്ടും അവയെ ഇപ്രകാരം തള്ളിക്കളഞ്ഞ നിര്‍ഭാഗ്യവാന്‍മാരെ സംബന്ധിച്ച് വിവിധ പ്രവാചകന്‍മാരെകുറിച്ച ബൈബിള്‍ കഥനങ്ങളില്‍ നിന്ന് മനസ്സിലാക്കിയിട്ടും നബി(സ)യുടെ കാര്യം വരുമ്പോള്‍ അതേ നിര്‍ഭാഗ്യത്തിലേക്ക് മുഖം കുത്തുന്ന ദുര്യോഗമാണ് മിഷനറിമാര്‍ക്ക് സംഭവിക്കുന്നത്.

വടി നിലത്തിട്ടാല്‍ സര്‍പ്പമാകുന്ന ദൈവികദൃഷ്ടാന്തവുമായി രാജസദസ്സിലേക്കു കടന്നുവന്ന മോശെയോടും അഹറോനോടും അത് കണ്‍കെട്ടാണെന്ന് വാദിക്കുകയും കൊട്ടാരത്തിലെ ആസ്ഥാന കണ്‍കെട്ടുകാരെക്കൊണ്ടുവന്ന് അവരുടെ കയറുകളും വടികളുംവെച്ച് ‘പ്രതീതി സര്‍പ്പങ്ങളെ’ ഉണ്ടാക്കിപ്പിക്കുകയും മോശെയുടെ സര്‍പ്പം തങ്ങളുടെ വ്യാജസര്‍പ്പങ്ങളെ വിഴുങ്ങുന്നതുകണ്ട് മോശെയുടേത് കണ്‍കെട്ടല്ലെന്നു കണ്‍കെട്ടുകാര്‍ക്കടക്കം ബോധ്യം വന്നിട്ടും ‘കണ്‍കെട്ടുവാദ’ത്തില്‍ ധാര്‍ഷ്ഠ്യത്തോടെ നിലനില്‍ക്കുകയും ചെയ്ത ഫറോവയെക്കുറിച്ച് ക്വുര്‍ആന്‍ വിശദമായും (20: 9-71) ബൈബിള്‍ അല്‍പം കൂടി ചുരുക്കിയും (പുറപ്പാട് 7: 1-13) -വിശദാംശങ്ങളില്‍ ചില വ്യത്യാസങ്ങളുണ്ടെങ്കിലും- വിശദീകരിക്കുന്നുണ്ട്. ഫറോവയുടെ അനന്തരഗാമിത്വമാണ് തങ്ങള്‍ക്കുവേണ്ടതെന്ന് തീരുമാനിക്കുവാന്‍ മിഷനറിമാര്‍ക്ക് തീര്‍ച്ചയായും സ്വാതന്ത്ര്യമുണ്ട്; പക്ഷേ അത് മോശെയുടെയും യേശുവിന്റെയും മേല്‍വിലാസത്തിലാകുമ്പോഴാണ് നമുക്ക് സങ്കടം! ഫറോവയുടെ മനഃശാസ്ത്രം കടമെടുക്കുന്നതു കൊണ്ടല്ലാതെ മറ്റെന്തുകൊണ്ടാണ്.

 പ്രപഞ്ചനാഥന്‍ മോശെക്കുവേണ്ടി ചെങ്കടല്‍ പിളര്‍ത്തിയതും (പുറപ്പാട് 14: 21-31) ജലം രക്തമാക്കി മാറ്റിയതും (പുറപ്പാട് 7: 14-25) അഹറോനുവേണ്ടി വെറും വടിയില്‍ തളിരും പൂവും കായുമുണ്ടാക്കിയതും (സംഖ്യ 17: 1-10) യോശുവക്കുവേണ്ടി പ്രളയകാലത്ത് ജോര്‍ദ്ദാന്‍ നദിയുടെ ഒഴുക്കു നിര്‍ത്തിയതും (യോശുവ 3: 1-17) ബാലാമിനോട് അദ്ദേഹത്തിന്റെ കഴുതയെക്കൊണ്ട് സംസാരിപ്പിച്ചതും (സംഖ്യ 22: 22-30) ഏലിശക്കുവേണ്ടി മരിച്ച കുട്ടിക്ക് ജീവന്‍ നല്‍കിയതും (1 രാജാക്കന്‍മാര്‍ 17: 5-24) അടക്കമുള്ള പഴയനിയമപ്രകാരമുള്ള അത്ഭുതങ്ങളിലും യേശുക്രിസ്തുവിന്റെ ജനനവും ജീവിതവും മരണവുമെല്ലാം അത്ഭുതങ്ങളില്‍ കുതിര്‍ത്തുനിര്‍ത്തുന്ന പുതിയ നിയമവിവരണങ്ങളിലും കണ്ണുമടച്ച് വിശ്വസിക്കുന്ന മിഷനറിമാര്‍ മുഹമ്മദ് നബി(സ)യെ വായിക്കുമ്പോള്‍ മാത്രം മുഴുവന്‍ പ്രവാചകന്‍മാരുടെയും ശത്രുപക്ഷത്തുനിന്ന കണ്‍കെട്ടു സിദ്ധാന്തത്തില്‍ അഭയം തേടുന്നത്. ബൈബിള്‍ കഥകളില്‍ വിവരിക്കപ്പെടുന്ന അത്ഭുതങ്ങളെക്കാള്‍ എന്ത് ‘അവിശ്വസനീത’യാണ് നബിജീവിതത്തിലെ ദൃഷ്ടാന്തങ്ങളില്‍ മിഷനറിമാര്‍ ‘കൂടുതലായി’ കാണുന്നത്?

ബൈബിള്‍, വസ്തുനിഷ്ഠമായി പറഞ്ഞാല്‍ ആധികാരികമായ ഒരു ചരിത്രസ്രോതസ്സല്ല. അതുകൊണ്ടുതന്നെ ബൈബിള്‍ വിവരണങ്ങളുടെ മാത്രം അടിസ്ഥാനത്തില്‍ ഒരത്ഭുതത്തെയും സ്ഥിരീകരിക്കാനാവില്ല. മിഷനറിമാര്‍ വിശ്വസിക്കുന്ന അത്ഭുതങ്ങള്‍ മിക്കതും യാതൊരു ചരിത്രപരതയുമില്ലാത്തതാണെന്ന് അതിനാല്‍ തന്നെ പറയാനാകും. എന്നാല്‍ ചന്ദ്രവിഭജനമടക്കമുള്ള നബിദൃഷ്ടാന്തങ്ങളുടെ കാര്യമതല്ലെന്ന് നാം കണ്ടു. ചരിത്രപരമായി സ്ഥാപിക്കാനാവാത്ത അത്ഭുതങ്ങളില്‍ വിശ്വസിക്കുന്നവര്‍ ചരിത്രം പൂര്‍ണമായി രേഖീകരിച്ച മുഹമ്മദ് നബി(സ)യുടെ ജീവിതത്തിലെ അത്ഭുതങ്ങളെ തള്ളിക്കളയുന്നതിലെ യുക്തി എന്താണ്?

അത്ഭുതങ്ങളെ സംബന്ധിച്ച വിശ്വാസിയുടെ സങ്കല്‍പത്തെ പരിഗണിക്കുന്ന ആര്‍ക്കും ചാന്ദ്രപിളര്‍പ്പുമായി ബന്ധപ്പെട്ട മിഷനറി വിമര്‍ശനങ്ങളുടെ അന്തസാര ശുന്യത ബോധ്യപ്പെടാതിരിക്കില്ല. ഒരു പ്രവാചകന്റെ കൈക്ക് സംഭവിച്ചുവെന്ന് ചരിത്രപരമായി സ്ഥിരീകരിക്കപ്പെടുന്നതോടുകൂടിത്തന്നെ പ്രസ്തുത അത്ഭുതങ്ങളെ ദൈവിക ദൃഷ്ടാന്തങ്ങള്‍ എന്ന നിലക്ക് മനസ്സിലാക്കുക എന്നതാണ് വിശ്വാസപരമായ സമീപനം. ‘അത്ഭുത’ങ്ങളുടെ മണ്ഡലം വിശ്വാസമാണെന്നു പറയുവാനുള്ള കാരണമതാണ്. ശാസ്ത്രത്തെ ഈ ചര്‍ച്ചയിലേക്കു വലിച്ചിഴക്കുവാനുള്ള മിഷനറി ശ്രമം ഏറ്റവും വലിയ അശ്ലീലമായിത്തീരുന്നതും അതുകൊണ്ടു തന്നെ. പ്രപഞ്ചവും പദാര്‍ത്ഥവും അവ സാധാരണഗതിയില്‍ പിന്തുടരുന്ന നിയമങ്ങള്‍വെച്ച് മനസ്സിലാക്കുകയാണ് ശാസ്ത്രത്തിന്റെ ധര്‍മം. അത്ഭുതങ്ങള്‍ ആ നിയമങ്ങളെ മറികടന്ന് സംഭവിക്കുന്ന കാര്യങ്ങളാണെന്നും അതുകൊണ്ടാണ് അവ അത്ഭുതങ്ങളാകുന്നതെന്നും മിഷനറിമാര്‍ക്കറിയില്ലേ? പിന്നെ അവയെ ‘ശാസ്ത്രീയമായി’ തെളിയിക്കാന്‍ ആവശ്യപ്പെടുന്നതിന്റെ ന്യായമെന്താണ്?

പ്രാപഞ്ചിക നിയമങ്ങളെ അതിലംഘിച്ചുകൊണ്ട് എപ്പോഴെങ്കിലും എന്തെങ്കിലും നടക്കുമോ എന്ന ചോദ്യത്തിന് ‘ഇല്ല’യെന്ന് തീര്‍ത്തും നിഷേധാത്മകമായി മറുപടി പറയുക ഭൗതികവാദികള്‍ മാത്രമാണ്. പ്രപഞ്ചത്തിനൊരു സ്രഷ്ടാവുണ്ടെന്ന വസ്തുത അവരംഗീകരിക്കുന്നില്ലെന്നതാണ് അതിന്റെ കാരണം. പ്രപഞ്ചസ്രഷ്ടാവില്‍ വിശ്വസിക്കുന്നവരെ സംബന്ധിച്ചടുത്തോളം അവനാണ് സൃഷ്ടിലോകത്തിന്റെ നിയാമകന്‍. ‘സാധാരണ’യായി പിന്തുടരപ്പെടുന്ന പ്രാപഞ്ചിക നിയമങ്ങള്‍ നിര്‍മിച്ചത് അവനാണ്. ആ നിയമങ്ങള്‍ പ്രകാരമല്ലാതെ ചില കാര്യങ്ങള്‍ സംഭവിപ്പിച്ച് ‘അസാധാരണ’മായ സ്ഥിതിവിശേഷങ്ങള്‍ സൃഷ്ടിക്കുക സര്‍വശക്തനായ അവനെ സംബന്ധിച്ചേടുത്തോളം തീര്‍ത്തും സാധ്യമാണ്. അതുകൊണ്ടാണ് പ്രവാചകന്‍മാര്‍ക്ക് സാക്ഷീകരണമായി പ്രപഞ്ചനാഥന്‍ സംവിധാനിക്കുന്ന അത്ഭുതങ്ങളില്‍ വിശ്വാസി സംശയലേശമന്യേ വിശ്വസിക്കുന്നത്. അതുകൊണ്ടുതന്നെയാണ് മിഷനറിമാര്‍ ബൈബിളിലെ അത്ഭുതകഥകളില്‍ വിശ്വസിക്കുന്നതും. മുകളിലുദ്ധരിച്ച ബൈബിള്‍ അത്ഭുതങ്ങളൊന്നും തന്നെ ശാസ്ത്രീയമായി വിശദീകരിക്കാനാവില്ല; ആകുമായിരുന്നെങ്കില്‍ അവ അത്ഭുതങ്ങളാവുകയും ചെയ്യുമായിരുന്നില്ല.

ശരീരത്തില്‍ തീയേറ്റാല്‍ ചൂടും പൊള്ളലുമുണ്ടാകുമെന്നേ ശാസ്ത്രത്തിന് പറയാനാകൂ. നംറൂദ് അഗ്നികുണ്ഠത്തിലേക്കെറിഞ്ഞ അബ്രഹാമിന് തീ തണുപ്പായത് ക്വുര്‍ആന്‍ പറയുന്നുണ്ട് (21: 68-70). വിഗ്രഹപൂജകനായ ബാബിലോണിയന്‍ രാജാവ് പ്രതിമാപൂജക്ക് വിസമ്മതിച്ച മൂന്ന് ജൂതപുരോഹിതന്‍മാരെ തീയിലെറിഞ്ഞപ്പോള്‍ അവര്‍ ഒരു പൊള്ളലുമേല്‍ക്കാതെ അഗ്നിനാളങ്ങളില്‍ സൈ്വരവിഹാരം നടത്തി പുറത്തുവന്ന കഥ ബൈബിളിലുണ്ട് (ദാനിയേല്‍ 3: 10-27). ഇവിടെ ശാസ്ത്രം നിസ്സഹായമാണ്. ദൈവം ഇടപെട്ടുവെന്നു കരുതുന്ന വിശ്വാസിയോടും ദൈവമില്ലെന്നതിനാല്‍ ഇത് അസംഭ്യവമാണെന്നു കരുതുന്ന അവിശ്വാസിയോടും ‘വിഷയം പരിധിക്കു പുറത്താണ്’ എന്നുമാത്രമേ ശാസ്ത്രത്തിന് പറയാന്‍ കഴിയൂ. ചന്ദ്രന്‍ പിളര്‍ന്നത് ‘ശാസ്ത്രീയമായി’ വിശദീകരിക്കാന്‍ ആവശ്യപ്പെടുന്ന മിഷനറിമാര്‍ മറക്കുന്നത് ഈ വസ്തുതയാണ്; അത്ഭുതങ്ങളെ സംബന്ധിച്ച് അവരും മുസ്‌ലിംകളുമെല്ലാം ഒരുപോലെയംഗീകരിക്കുന്ന പ്രാഥമിക വസ്തുതയെ. പ്രകൃതിനിയമങ്ങള്‍വെച്ച് സ്വാഭാവികമായി സംഭവിച്ചിട്ടില്ലാത്തുകൊണ്ടാണ് ചന്ദ്രന്റെ പിളര്‍പ്പ് ഒരു അത്ഭുതവും പ്രവാചകത്വത്തിനുള്ള തെളിവുമായതെന്നും അത്തരമൊരു പ്രതിഭാസത്തിന്റെ പ്രായോഗികതയെക്കുറിച്ചുള്ള സംശയം ദൈവം തമ്പുരാന്റെ ശക്തിവിശേഷങ്ങളെക്കുറിച്ചുള്ള അജ്ഞതയില്‍ നിന്നുണ്ടാകുന്നതാണെന്നും തിരിച്ചറിയാന്‍ മതവും ശാസ്ത്രവും എന്താണെന്നതിനെക്കുറിച്ച പ്രാഥമിക ധാരണകള്‍ മാത്രമേ ആവശ്യമുള്ളൂ.

അത്ഭുതങ്ങളില്‍ വിശ്വസിക്കുകയും അവ ശാസ്ത്രീയമായി വിശദീകരണക്ഷമമാവുകയില്ലെന്നു തിരിച്ചറിയുകയും ഏതെങ്കിലും നിയമങ്ങളുടെ നിര്‍ബന്ധിത വരുതിക്കുള്ളിലുള്ളവനല്ല പടച്ചവനെന്ന് മനസ്സിലാക്കുകയും ചെയ്യുമ്പോള്‍ തന്നെ ചാന്ദ്രപിളര്‍പ്പിനെ ശാസ്ത്രീയമായി നിഷേധിക്കുവാന്‍ ശ്രമിക്കുന്നത് അത് ഒരു വാനലോകാത്ഭുതമായതു കൊണ്ടാണെന്നാണ് ചില മിഷനറിമാര്‍ തങ്ങളുടെ ഊന്നലുകള്‍ കൊണ്ട് ധ്വനിപ്പിക്കുവാന്‍ ശ്രമിക്കാറുള്ളത്. വാനവും ചന്ദ്രനുമെല്ലാം സര്‍വശക്തന്റെ അധികാരപരിധിക്കു പുറത്താണെന്നാണോ ഇവര്‍ ധരിച്ചിരിക്കുന്നത്?

മണ്‍തരിയും പുല്‍ക്കൊടിയും മാത്രമല്ല, ഗ്രഹവും ഉപഗ്രഹവും നക്ഷത്രവുമെല്ലാം അവന്റേതു മാത്രമാണെന്നും അവയ്ക്കു നിയമങ്ങള്‍ നിശ്ചയിക്കുന്നതും ഇച്ഛിക്കുമ്പോള്‍ നിയമങ്ങള്‍ക്കു പുറത്തേക്ക് അവയെ കൊണ്ടുപോകുന്നതും അവന്‍ മാത്രമാണെന്നും എന്നാണ് ഇവരെല്ലാം തിരിച്ചറിയുക? ഭൂമിയും ചന്ദ്രനും സൂര്യനും ഗ്രഹങ്ങളുമെല്ലാമുള്‍ക്കൊള്ളുന്ന ഒരു സംവിധാനത്തില്‍ ചാന്ദ്രപിളര്‍പ്പ് സൃഷ്ടിക്കാവുന്ന ഭൗതിക ‘പ്രത്യാഘാത’ങ്ങളെക്കുറിച്ചാകുലപ്പെടുന്നവര്‍, ഇപ്പറഞ്ഞതൊക്കെയും അവന്റേതാണെന്നും ഭൗതികശാസ്ത്ര സമവാക്യങ്ങള്‍ പ്രകാരമുള്ള പ്രതിഫലനങ്ങള്‍ക്കിടം നല്‍കി മാത്രമല്ല, ‘ചുറ്റപാടുകളെ അറിയിക്കാതെ’യും കാര്യങ്ങള്‍ നടത്താന്‍ കഴിവുള്ളവനാണവന്‍ എന്നുമുള്ള സത്യങ്ങളാണ് മറന്നുപോകുന്നത്. ചന്ദ്രോപരിതലത്തില്‍ പിളര്‍പ്പിന്റെ അടയാളങ്ങള്‍ ശേഷിക്കുന്നുണ്ടോ എന്ന ചോദ്യമുയര്‍ത്തുന്നവര്‍, പിളര്‍ത്താന്‍ മാത്രമല്ല അടയാളങ്ങളൊന്നും ബാക്കിവെക്കാതെ ചേര്‍ക്കാനും പ്രപഞ്ചനാഥനു കഴിയുമെന്നും ചന്ദ്രോപരിതലത്തിലുള്ള എല്ലാ ‘അടയാള’ങ്ങളും നാം വായിച്ചുകഴിഞ്ഞിട്ടില്ലെന്നും ഓര്‍ക്കുന്നതില്‍ ദയനീയമായി പരാജയപ്പെടുന്നതെന്തുകൊണ്ടാണ്?

അന്ധമായ നബിവിരോധം എന്താണ് ഒരു അത്ഭുതം എന്നും ആരാണ് പടച്ചവന്‍ എന്നുമുള്ള അടിസ്ഥാന യാഥാര്‍ത്ഥ്യങ്ങളെപ്പോലും വിസ്മരിച്ചുകൊണ്ടുള്ള പരിമിതബുദ്ധിയുടെ ഉന്മാദത്തിലേക്കും ചോദ്യങ്ങളുടെ പ്രളയത്തിലേക്കും മിഷനറിമാരുടെ കൈപിടിച്ചുകൊണ്ടുപോകുന്നുണ്ടെങ്കില്‍, ഇതേ ഖഗോളശാസ്ത്ര’ജ്ഞാനം’ ബൈബിള്‍ വായിക്കാന്‍ കൂടി അവര്‍ക്കുപയോഗിക്കാവുന്നതാണ്! അഹറോന്റെയും മോശെയുടെയും മരണശേഷം മോവാബ് ദേശത്തുവെച്ച് ഇസ്രയേല്‍ സമൂഹത്തിന്റെ നായകത്വമേറ്റെടുത്തശേഷം അവരെയുംകൊണ്ട് ജോര്‍ദ്ദാന്‍ നദി മുറിച്ചുകടന്ന് വാഗ്ദത്ത കാനാന്‍ ദേശത്തെത്തിയ യോശുവ, അമോര്യ രാജാക്കന്‍മാരുമായി നടത്തിയ യുദ്ധത്തില്‍ സമയം കൂടുതല്‍ ലഭിക്കാന്‍ വേണ്ടി സൂര്യനെ ഗിബയോണ്‍ താഴ്‌വരക്കു മുകളിലും ചന്ദ്രനെ അയ്യലോണ്‍ താഴ്‌വരക്കുമുകളിലും ഒരു ‘ദിവസം’ മുഴുവന്‍ ‘നിശ്ചലമാക്കി’ നിര്‍ത്തിയതിനെക്കുറിച്ച് (യോശുവ 10: 12-14) അവരുടെ അഭിപ്രായമെന്താണ്?

യേശു കുരിശില്‍ തറക്കപ്പെട്ടതിന്റെ ‘ഫലമായി’ ‘സൂര്യപ്രകാശം’ നിലച്ച് രാജ്യം മുഴുക്കെ മണിക്കൂറുകളോളം കനത്ത ‘ഇരുട്ട് പരക്കുകയും’ ദേവാലയത്തിലെ തിരശ്ശീല കീറുകയും ഭൂമി വിറക്കുകയും പാറകള്‍ പിളരുകയും ചെയ്തതിലെ (മത്തായി 27: 45-51, മാര്‍ക്കോസ് 15-33, ലൂക്കോസ് 23-44) ‘കാര്യകാരണ ബന്ധ’ത്തെയും ‘ശാസ്ത്രീയത’യെയും കുറിച്ച് അവര്‍ക്കെന്താണ് പറയാനുള്ളത്? ഒടുവിലൊടുവില്‍ നിങ്ങള്‍ നബി(സ)യെത്തോല്‍പിക്കാന്‍ പറഞ്ഞെത്തുന്നത് ഭൗതികവാദത്തിന്റെ അരമനയിലാണോ?

പ്രവാചകാനുചരന്‍ സഅദ്ബ്‌നു അബീവക്വാസ് ഒരു അമുസ്ലിമിനെ ഒട്ടകത്തിന്റെ താടിയെല്ലുകൊണ്ട് തലക്കടിച്ചുകൊന്ന സംഭവം മക്കയിൽ വെച്ചും മുസ്ലിംകൾ ആയുധമെടുത്തുവെന്നും അവർ കലാപകാരികളായിരുന്നുവെന്നുമല്ലേ വ്യക്തമാക്കുന്നത് ?

അദ്ബ്‌നു അബീവക്വാസ് മക്കയില്‍വെച്ച് ബഹുദൈവാരാധകനായ ഒരാളെ ഒട്ടകത്തിന്റെ താടിയെല്ലുകൊണ്ട് തലക്കടിച്ചുകൊന്ന സംഭവം മുസ്ലിംകൾ എന്നും കലാപകാരികളാണെന്നാണ് വ്യക്തമാക്കുന്നതെന്നും മക്കയിലെ സംഘര്‍ഷങ്ങളുടെ സ്വഭാവം എന്തെന്ന് മനസ്സിലാക്കാൻ ഈ സംഭവം പര്യാപ്തമാണെന്നും വാദിച്ചുകൊണ്ട് മക്കയിലെ മുസ്‌ലിംകള്‍ കലാപശ്രമങ്ങള്‍ നടത്തി എന്നു സമര്‍ത്ഥിക്കുവാന്‍ വേണ്ടി മിഷനറിമാരും ഓറിയന്റലിസ്റ്റുകളും ശ്രമിക്കാറുണ്ട്

സ്വന്തം നാടായ മക്കയില്‍ ശരിയെന്നു വിശ്വസിക്കുന്ന ആദര്‍ശം പ്രബോധനം ചെയ്തതിന്റെ പേരില്‍ തനിക്കും അനുയായികള്‍ക്കും പതിമൂന്നു വര്‍ഷം കടുത്ത മര്‍ദ്ദനങ്ങളേല്‍ക്കേണ്ടി വന്നിട്ടും മക്കയുടെ രാഷ്ട്രീയ നിയന്ത്രണം കൈക്കലാക്കാനോ എതിരാളികളെ വകവരുത്തുവാനോ വേണ്ടിയുള്ള സായുധ ഓപ്പറേഷനുകള്‍ക്കായുള്ള ഒരു ഗറില്ലാ പദ്ധതിയും ഒരിക്കല്‍ പോലും ആസൂത്രണം ചെയ്യാതെ ഗോത്രാധിപത്യ മക്കയില്‍ ക്ഷമാപൂര്‍വം നിയമവാഴ്ചക്കുള്ളില്‍ മാതൃകാപരമായി നിലനില്‍ക്കുകയാണ് നബി (സ) ചെയ്തതെന്ന യാഥാര്‍ത്ഥ്യം ഇസ്ലാംവിമര്‍ശകരുടെ പുസ്തങ്ങള്‍ തന്നെ വ്യക്തമാക്കുന്നുണ്ട് എന്നതാണ് യാഥാര്‍ത്ഥ്യം. പതിമൂന്നു വര്‍ഷക്കാലം ചരിത്രത്തിന്റെ വെള്ളിവെളിച്ചത്തില്‍. സംഭവബഹുലമായി നിലനിന്ന മക്കയിലെ നബിജീവിതത്തില്‍ നിന്ന് കലാപം കുഴിച്ചെടുക്കാനുള്ള സിദ്ധവൈഭവമുള്ള ഗവേഷകബുദ്ധികളുടെ അധ്വാനങ്ങള്‍ കേവലം ഒരു ‘ഒട്ടകയെല്ലില്‍’ തടഞ്ഞുനില്‍ക്കുന്നതുതന്നെ നബി(സ)യില്‍ ഗറില്ലാ പോരാളികള്‍ക്ക് പൂര്‍വമാതൃക കണ്ടെത്താനുള്ള എല്ലാ പരിശ്രമങ്ങളും അടിസ്ഥാനരഹിതമാണെന്നാണ് വ്യക്തമാക്കുന്നത്.

മതസ്വാതന്ത്ര്യവും പൗരാവകാശങ്ങളും നിഷേധിക്കപ്പെട്ട് പീഡനപര്‍വത്തില്‍ കിടന്നുപിടഞ്ഞ് പ്രക്ഷുബ്ധമായ ഒരു ജനസമൂഹമാണ് നബിനേതൃത്വത്തില്‍ സായുധമായ പ്രതികരണങ്ങള്‍ക്കും പ്രതികാരങ്ങള്‍ക്കുമുള്ള ഒരു ശ്രമവും നടത്താതെ മക്കയില്‍ ഒന്നര പതിറ്റാണ്ടിനടുത്ത് ശാന്തജീവിതം നയിച്ചത് എന്ന ചരിത്രവസ്തുത, ഇവ്വിഷയകമായ മുഴുവന്‍ വിമര്‍ശകഭാവനകളെയും കടപുഴക്കിയെറിയാന്‍ പോന്നതാണ്. സഹജമായ പ്രതികരണശേഷിയില്‍ നിന്നുടലെടുത്തേക്കാവുന്ന കലാപത്തിന്റെ ആശയങ്ങളൊന്നും അവരില്‍നിന്ന് നിര്‍ഗളിക്കാതിരുന്നത് പ്രവാചകന്‍ (സ) ബോധപൂര്‍വം അതിനെതിരായ ഉല്‍ബോധനങ്ങള്‍ നല്‍കിയതുകൊണ്ടാണെന്ന കാര്യം വ്യക്തമാണ്. പീഡനങ്ങളും പ്രയാസങ്ങളുമെല്ലാമുണ്ടായപ്പോള്‍ പോലും കലാപം പരിഹാരമല്ലെന്ന് മതപരമായിത്തന്നെ വിശ്വസിക്കുകയും പഠിപ്പിക്കുകയും ചെയ്ത ഒരു സമാധാനവാദിയെ ഭീകരവാദിയുടെ കുപ്പായമണിയിക്കുവാന്‍ നുരുമ്പിച്ച ഒരൊട്ടകയെല്ല് മതിയാകുമെന്നു കരുതുന്നവര്‍ വിഡ്ഡികളുടെ സ്വര്‍ഗത്തിലാണെന്ന് പറയാതിരിക്കാന്‍ യാതൊരു നിര്‍വാഹവുമില്ല തന്നെ.

വേണമെങ്കില്‍ പ്രതികാരസംഘങ്ങള്‍ക്ക് നിഷ്പ്രയാസം നേതൃത്വം കൊടുക്കുവാന്‍ കഴിയുമായിരുന്ന ഹംസയും ഉമറും കൂടെവന്നിട്ടും കലാപത്തിന്റെ വഴി ചെറുതായിപ്പോലും പുണരാതിരുന്ന നബിമാതൃക, അച്ചടക്കമുള്ള പൗരജീവിതം അദ്ദേഹത്തിന് ശക്തി കൈവരുന്നതിനനുസരിച്ച് കയ്യൊഴിക്കാനുള്ള ഒരു ‘സൗകര്യ’മായിരുന്നില്ല. മറിച്ച് കര്‍ക്കശമായ ഒരു മൂല്യവും നിലപാടുമായിരുന്നുവെന്ന് അസന്നിഗ്ധമായി വ്യക്തമാക്കുന്നുണ്ട്. വധഭീഷണികള്‍ ശക്തിപ്രാപിച്ച് പ്രവാചകനെ വിട്ടുകൊടുക്കാന്‍ സന്നദ്ധമാകാത്തതിന്റെ പേരില്‍ അദ്ദേഹത്തിന്റെ കുടുംബത്തിന് മൂന്നുവര്‍ഷത്തോളം പച്ചിലകള്‍ മാത്രം ഭക്ഷിച്ച് സാമൂഹികമായ ഭ്രഷ്ട് അനുഭവിക്കേണ്ടി വന്നപ്പോഴും അബൂത്വാലിബിന്റെ മരണത്തെത്തുടര്‍ന്ന് ഭീഷണികള്‍ക്ക് ഊക്ക് വര്‍ദ്ധിച്ചപ്പോഴും പ്രതികാരത്തിനുള്ള യാതൊരു രഹസ്യ/പരസ്യ നീക്കവുമില്ലാതെ ശാന്തനായി നിലനില്‍ക്കുകയാണ് നബി (സ) ചെയ്തത്. നബി(സ)യെ കലാപകാരിയാക്കി അവതരിപ്പിക്കാനാഗ്രഹമുള്ളവര്‍ മഷിയിട്ടു തിരഞ്ഞിട്ടും അവിടുത്തെ ജീവിതത്തില്‍ നിന്ന് കലാപത്തിന്റെ ചെറിയൊരു പൊട്ടുപോലും അവര്‍ക്ക് കണ്ടെടുക്കാന്‍ കഴിയാത്തതും അതുകൊണ്ടുതന്നെയാണ്.

കലാപം നബിചര്യയാണെന്ന ആരോപണത്തെ ചരിത്രം പൂര്‍ണമായി കരിച്ചുകളയുന്നുവെന്ന് മനസ്സിലാക്കിയ മിഷനറിമാര്‍, അവസാനത്തെ അവലംബം എന്ന നിലയിലാണ് സഅ്ദ്ബ്‌നു അബീ വക്വാസിനെക്കുറിച്ചുള്ള പരാമൃഷ്ട കഥയുദ്ധരിക്കുന്നത്. ഇബ്‌നു ഇസ്ഹാക്വ് തന്റെ സീറതുറസൂലില്ലയില്‍ സഅദ്ബ്‌നു അബീവക്വാസ് മക്കയില്‍വെച്ച് ഒരു അവിശ്വാസിയെ ഒട്ടകത്തിന്റെ താടിയെല്ലുകൊണ്ട് പ്രഹരിച്ചതായി പറയുന്നുണ്ട്. സത്യവും അസത്യവുമെല്ലാം കൂടിക്കുഴഞ്ഞു കിടക്കുന്ന ഒരു ചരിത്രവിവരണ ഗ്രന്ഥമായതുകൊണ്ടു തന്നെ, ഇബ്‌നു ഇസ്ഹാക്വിന്റെ സീറയില്‍ ഉള്ളതുകൊണ്ടുമാത്രം ഒരു സംഭവം വസ്തുതാപരമാകണമെന്നില്ല. സംഭവം ഉദ്ധരിക്കപ്പെട്ടിരിക്കുന്ന നിവേദക പരമ്പരയെ നിദാനശാസ്ത്രനിയമങ്ങള്‍ വെച്ചപഗ്രഥിച്ചാല്‍ മാത്രമേ മിഷനറിമാര്‍ ചൂണ്ടിക്കാണിക്കുന്ന കഥയുടെ ചരിത്രപരത നിര്‍ണയിക്കാനാകൂ. അതെന്തായിരുന്നാലും, കഥയിലേക്കുവരാം. കഥ ആധികാരികമാണോ അല്ലേ എന്ന ചര്‍ച്ചയ്ക്കപ്പുറത്ത് ആധികാരികമാണെങ്കില്‍തന്നെ മിഷനറിമാരുടെ വാദത്തിനനുകൂലമായ യാതൊന്നും പ്രസ്തുത കഥയിലില്ലെന്ന് സീറതു റസൂലില്ല വായിക്കുന്ന ആര്‍ക്കും ബോധ്യമാകും.

മക്കയിലെ അവിശ്വാസികളില്‍ പലരും പ്രവാചകനെയും ശിഷ്യന്‍മാരെയും കഠിനമായി എതിര്‍ക്കുകയും തരം കിട്ടിയാല്‍ മര്‍ദ്ദിക്കുകയും ചെയ്തിരുന്നതുകൊണ്ടുതന്നെ ജനങ്ങള്‍ അധികം ശ്രദ്ധിക്കാത്ത താഴ്‌വരകളില്‍ പോയാണ് നബിശിഷ്യന്‍മാരില്‍ പലരും മതം അനുശാസിക്കുന്ന പ്രാര്‍ത്ഥനകള്‍ നിര്‍വഹിച്ചിരുന്നത് എന്നു പറഞ്ഞുകൊണ്ടാണ് ഇബ്‌നു ഇസ്ഹാക്വ് ഈ സംഭവത്തിന്റെ വിവരണം ആരംഭിക്കുന്നത്. ഇങ്ങനെ ഒരു താഴ്‌വരയില്‍ സഅദ്ബ്‌നു അബീ വക്വാസ് (റ) ഉള്‍പ്പെടെയുള്ള ഒരു സംഘം പ്രവാചകാനുചരന്‍മാര്‍ പ്രാര്‍ത്ഥന നിര്‍വഹിച്ചുകൊണ്ടിരിക്കെ അതുവഴി കടന്നുവന്ന ബഹുദൈവാരാധകരുടെ ഒരു സംഘം തികഞ്ഞ ധാര്‍ഷ്ട്യത്തോടുകൂടി വളരെ പരുഷമായി പ്രാര്‍ത്ഥന തടസ്സപ്പെടുത്തുകയും വിശ്വാസികളെ ആക്ഷേപിക്കുകയും ചെയ്തുവെന്നും അത് വാക്കുതര്‍ക്കവും സംഘര്‍ഷവും ഏറ്റമുട്ടലും ഉണ്ടാക്കിയെന്നുമാണ് ഇബ്‌നു ഇസ്ഹാക്വ് പറയുന്നത്. ശാന്തമായി പ്രാര്‍ത്ഥന നിര്‍വഹിച്ചുകൊണ്ടിരിക്കുന്നവരെ കയ്യേറാന്‍ വന്ന സംഘവുമായുണ്ടായ ഉന്തിലും തള്ളിലും കയ്യില്‍ കിട്ടിയ ഒരൊട്ടകയെല്ല് സഅദ്ബ്‌നു അബീ വക്വാസ് ഒരു ശത്രുവിനുനേരെ പ്രയോഗിച്ചുവെന്നും അത് അയാളുടെ രക്തം ചിന്തിയെന്നുമാണ് നിവേദനം. (Alfred, Guillaume. The Life of Muhammad – A Translation of Ibn Ishaq’s Sirat Rasul Allah).

ഇബ്‌നു ഇസ്ഹാക്വിന്റെ ഈ വിവരണത്തെ സ്വീകരിച്ചിട്ടുള്ള ചില പണ്ഡിതന്‍മാര്‍ ഈ സംഭവത്തിനുശേഷം ഇത്തരം സംഘര്‍ഷങ്ങള്‍ ആവര്‍ത്തിക്കാതിരിക്കുവാന്‍ വേണ്ടിയാണ് വിദൂരവും വിജനവുമായ താഴ്‌വരകളുപേക്ഷിച്ച് കഅ്ബയ്ക്ക് തൊട്ടരുകില്‍ തന്നെയായിരുന്ന അര്‍ക്വമിന്റെ വീട്ടില്‍ (ദാറുല്‍ അര്‍ക്വം) ആളുകള്‍ ശ്രദ്ധിക്കാത്തവിധം പ്രാര്‍ത്ഥനക്കും പഠനത്തിനും ഒത്തുചേരാന്‍ നബി(സ)യും അനുയായികളും തീരുമാനിച്ചതെന്ന് അഭിപ്രായപ്പെട്ടിട്ടുണ്ട്. (അലി ബ്ന്‍ ബുര്‍ഹാനുദ്ദീന്‍ അല്‍ഹലബി, അസ്സീറത്തുല്‍ ഹലബിയ്യ, 1/456). ഇത് ശരിയാണെങ്കില്‍, എന്താണ് താഴ്‌വരാ സംഭവത്തിലുള്ളത്? നാം പരിശോധിക്കുക. പ്രകോപനങ്ങള്‍ ഒഴിവാക്കാനും മര്‍ദ്ദനത്തില്‍ നിന്ന് രക്ഷനേടാനും വേണ്ടി നഗരമധ്യത്തില്‍ നിന്നുമാറി ആളുകളില്ലാത്ത ഒരു താഴ്‌വരയില്‍ പോയി ശാന്തമായി പ്രാര്‍ത്ഥന നിര്‍വഹിക്കുന്ന വിശ്വാസികളുടെ ഒരു സംഘത്തെ അതുവഴി കടന്നുപോയ അവരുടെ തന്നെ നാട്ടുകാരായ ചിലര്‍ ഒരു കാരണവുമില്ലാതെ കടന്നാക്രമിക്കുന്നു. കയ്യില്‍ കിട്ടിയ വസ്തുക്കളുപയോഗിച്ച് കയ്യേറ്റക്കാരെ തുരത്താന്‍ വിശ്വാസികള്‍ ശ്രമിക്കുന്നു. അതിനിടക്ക് സ്വാഭാവികമായി സംഭവിച്ചതാണ് സഅദ്ബ്‌നു അബീ വക്വാസിന്റെ പ്രഹരം.

ശാന്തമായി നാം കടന്നുപോകുമ്പോള്‍ നമ്മെ കടന്നാക്രമിക്കുവാന്‍ വരുന്ന ഒരാളെ തുരത്താന്‍വേണ്ടി സാധ്യമായ എല്ലാ മാര്‍ഗങ്ങളും ഉപയോഗിക്കുന്നതിന്റെ പേരാണോ ‘കലാപം’? ആണെന്ന് വിവരമുള്ള ഒരാളും പറയില്ല. താഴ്‌വരയില്‍ വിശ്വാസികള്‍ പോയത് ആയുധപരിശീലനം നടത്താനോ അട്ടിമറി പ്രവര്‍ത്തനങ്ങള്‍ ആസൂത്രണം ചെയ്യാനോ സായുധ പ്രതികരണങ്ങള്‍ക്ക് പദ്ധതി തയ്യാറാക്കാനോ ആയിരുന്നുവെങ്കില്‍ അത് കലാപത്തിനുള്ള ശ്രമമായിരുന്നുവെന്ന് പറയാം. അങ്ങനെയാണെന്ന് മിഷനറിമാര്‍ക്കുപോലും വാദമില്ല. സ്വയം പ്രതിരോധത്തിന് ഒട്ടകത്തിന്റെ താടിയെല്ലു വരെയെടുക്കേണ്ടി വന്നതില്‍ നിന്ന് യാതൊരുവിധ ആയുധസന്നാഹങ്ങളും സംഘത്തിന്റെ കയ്യിലുണ്ടായിരുന്നില്ലെന്ന് വ്യക്തമാകുന്നുണ്ട്. ഏതെങ്കിലും മക്കക്കാരനെ പ്രതികാരമെന്ന നിലയിലോ അല്ലാതെയോ തിരഞ്ഞുചെന്നോ കണ്ടിടത്തുവെച്ചോ ആക്രമിച്ചതായിരുന്നുവെങ്കിലും അതിനെ മിഷനറിമാര്‍ സ്ഥാപിക്കാന്‍ ശ്രമിക്കുന്ന വകുപ്പുകളില്‍ ഉള്‍പ്പെടുത്താം. എന്നാല്‍ പരാമര്‍ശിക്കപ്പെടുന്ന കഥയിലുള്ളത് ഇതൊന്നും തന്നെയല്ല. ഇങ്ങോട്ടുവന്ന് ആക്രമിച്ച ഒരാളെ അതുണ്ടാക്കിയ ഏറ്റുമുട്ടലിനിടെ സ്വാഭാവികമായി തിരിച്ചടിച്ചതിനെക്കുറിച്ചാണ് നിവേദനം. ഏതു കോടതിയും പ്രകൃതിപരമായ പ്രതിരോധ നടപടിയെന്ന നിലയില്‍ നിയമവിധേയമായി അംഗീകരിക്കുന്ന ആത്മരക്ഷാ പ്രതികരണം മാത്രമാണത്. അതിനെ ‘കലാപ’മെന്നു പറയാന്‍ മിഷനറിമാര്‍ക്കു മാത്രമേ കഴിയൂ!

പ്രവാചകന്റെ (സ) പതിമൂന്നുവര്‍ഷത്തെ മക്കാജീവിതം പണിപ്പെട്ടു തിരഞ്ഞിട്ട് മിഷനറിമാര്‍ക്കു കിട്ടിയ ‘എല്ലിന്‍ കഷ്ണം’ തീര്‍ത്തും ഉപയോഗശൂന്യമാണെന്ന് സംഭവത്തിന്റെ പശ്ചാത്തലം അസന്നിഗ്ധമായി വ്യക്തമാക്കുന്നുണ്ടെന്നു ചുരുക്കം. പ്രവാചകന്റെ (സ) അസാന്നിധ്യത്തില്‍ അദ്ദേഹത്തിന്റെ നിര്‍ദ്ദേശപ്രകാരമല്ലാതെ തികച്ചും അപ്രതീക്ഷിതമായി ഒരുപറ്റം അനുയായികള്‍ അകപ്പെട്ടുപോയ സംഘര്‍ഷത്തില്‍ മനുഷ്യസഹജമായുണ്ടായ ഒരു പ്രഹരത്തെ നബി(സ)യുടെ കലാപമനസ്സായി വിവര്‍ത്തനം ചെയ്യുന്നര്‍ വഞ്ചിക്കുന്നത് അവരവരുടെ മനസാക്ഷിയെത്തന്നെയാണ്. സംഭവമറിഞ്ഞപ്പോള്‍ അത്തരം സംഭവങ്ങള്‍ ആവര്‍ത്തിക്കാതിരിക്കുവാനുള്ള ഇടപെടലാണ്, അല്ലാതെ വ്യാപിപ്പിക്കുവാനുള്ള പരിശ്രമമല്ല നബി (സ) നടത്തിയത് എന്ന് ദാറുല്‍ അര്‍ക്വമിനെ പുതിയ പാഠശാലയും പ്രാര്‍ത്ഥനാശാലയുമായി നിശ്ചയിച്ചതില്‍ നിന്ന് സുതരാം മനസ്സിലാകുന്നുണ്ട്. പ്രവാചകന് കലാപമനസ്സുണ്ടെന്ന് കാണിക്കുവാനുള്ള ഒരു തെളിവും സഅദ്ബ്‌നു അബീ വക്വാസിന്റെ താഴ്‌വരാ പ്രഹരത്തില്‍ -കഥ ആധികാരികമാണെങ്കില്‍ പോലും- ഇല്ലെന്നു സാരം.

റ്റേതൊരു രാഷ്ട്രത്തെയും പോലെ ഇസ്‌ലാമിക രാഷ്ട്രത്തിലും വ്യത്യസ്ത ഉത്തരവാദിത്തങ്ങളുള്ള പൗരന്മാരുണ്ടായിരിക്കും. പട്ടാളക്കാരന്റെ ഉത്തരവാദിത്തവും സർക്കാർ ജീവനക്കാരന്റെ ഉത്തരവാദിത്തവും സാധാരണ തൊഴിലാളിയുടെ ഉത്തരവാദിത്തവും ജനാധിപത്യരാഷ്ട്രത്തിൽ പോലും വ്യത്യസ്തമാണല്ലോ.ഇസ്‌ലാമികരാഷ്ട്രത്തിലെ സാമൂഹികനിയമങ്ങളും അന്തരീക്ഷവുമെല്ലാം ഇസ്‌ലാമികമായിരിക്കും. അതുകൊണ്ട് തന്നെ മുസ്ലിംകൾക്ക് ഉത്തരവാദിത്തം കൂടുതലുണ്ടായിരിക്കും. സൈനികസേവനത്തതിന് സദാസന്നദ്ധരായിരിക്കണം ഓരോ മുസ്ലിമും. എന്നാൽ മുസ്ലിംകളല്ലാത്തവർക്ക് ആ ഉത്തരവാദിത്തമില്ല. മുസ്ലിംകളിലെ പണക്കാരിൽ നിന്ന് നിന്ന് രാഷ്ട്രം സകാത്ത് പിരിച്ചെടുക്കുകയും പാവങ്ങൾക്ക് വിതരണം നടത്തുകയും ചെയ്യും. ഈ സകാത്തിൽ നിന്ന് നാടിന്റെ സംരക്ഷണത്തിനുള്ള യുദ്ധത്തിന് വേണ്ടിയും രാഷ്ട്രം ചിലവഴിക്കും. എന്നാൽ അമുസ്ലിംകൾ സകാത്ത് നൽകേണ്ടതില്ല. എന്നാൽ അവർ സംരക്ഷണ നികുതിയായ ജിസ് യ നൽകുവാൻ കടപ്പെട്ടവരാണ്. രാഷ്ട്രസംരക്ഷണത്തിനു വേണ്ടിയുള്ള യുദ്ധങ്ങളിൽ പങ്കെടുക്കാത്ത അവരുടെ ജീവനും സ്വത്തിനും നൽകുന്ന സംരക്ഷണത്തിന് പകരമായുള്ള നികുതി മാത്രമാണിത്.

എന്നാൽ, ശിക്ഷാസമ്പ്രദായങ്ങളിലും അതല്ലാത്ത നിയമങ്ങളിലുമെല്ലാം ഇസ്‌ലാമിക രാഷ്ട്രത്തിലെ എല്ലാ പൗരന്മാരും തുല്യരായാണ് പരി ഗണിക്കപ്പെടുന്നത്. സമ്പന്നനും ദരിദ്രനുമിടയിലോ ഭരണാധികാരി ക്കും സാധാരണ പൗരന്നുമിടയിലോ യാതൊരുരീതിയിലുള്ള വി വേചനവും നീതിനിര്‍വഹണത്തിന്റെ കാര്യത്തില്‍ ഉണ്ടാവാന്‍ പാടില്ലെന്നാണ് ഇസ്‌ലാമിന്റെ അധ്യാപനം. 'സത്യവിശ്വാസികളേ, നിങ്ങള്‍ അല്ലാഹുവിനുവേണ്ടി സാക്ഷ്യം വഹിക്കുന്നവരെന്ന നില യയില്‍ കണിശമായി നീതി നിലനിര്‍ത്തുന്നവരായിരിക്കണം. അത് നിങ്ങള്‍ക്ക്തന്നെയോ, നിങ്ങളുടെ മാതാപിതാക്കള്‍, അടുത്ത ബ ന്ധുക്കള്‍ എന്നിവര്‍ക്കോ പ്രതികൂലമായിത്തീര്‍ന്നാലും ശരി' എ ന്നാണ് ക്വുര്‍ആന്‍ പഠിപ്പിക്കുന്നത്.

ഒരിക്കല്‍ കുലീന കുടുംബത്തില്‍പ്പെട്ട ഒരു സ്ത്രീ മോഷണക്കു റ്റത്തിന് പിടിക്കപ്പെട്ടു. പ്രവാചക(സ)ന്റെ അടുത്ത അനുയായികളിലൊ രാളായ ഉസാമഃ (റ ) അവരുടെ കാര്യത്തില്‍ ശുപാര്‍ശ ചെയ്യുവാനായി പ്രവാചകനെ സമീപിച്ചു. അപ്പോള്‍ പ്രവാചകന്‍ (സ)പറഞ്ഞ വാക്കു കള്‍ ശ്രദ്ധേയമാണ്: 'നിങ്ങള്‍ക്ക് മുമ്പുള്ള സമുദായങ്ങള്‍ നശിപ്പിക്ക പ്പെട്ടത് ഇത്തരം പ്രവര്‍ത്തനങ്ങള്‍ മൂലമാണ്. സമ്പന്നര്‍ കുറ്റംചെയ് താല്‍ വെറുതെ വിടുകയും ദരിദ്രര്‍ കുറ്റംചെയ്താല്‍ ശിക്ഷിക്കുകയു മായിരുന്നു അവരുടെ സമ്പ്രദായം. അല്ലാഹുവാണെ സത്യം. ഈ പ്രവൃത്തി ചെയ്തത് മുഹമ്മദിന്റെ മകള്‍ ഫാത്തിമ തന്നെയായാ ലും അവളുടെ കരം ഞാന്‍ ഛേദിച്ചുകളയുന്നതാണ്'. നീതിയുടെ മുമ്പില്‍ ഭരണാധിപനും ഭരണീയനും സമമാണ് എന്നതാണ് ഇസ്‌ലാമിക രാഷ്ട്രസങ്കല്‍പത്തിന്റെ അടിസ്ഥാന സ്തംഭങ്ങളിലൊന്ന്.

ഇക്കാര്യത്തിലേക്ക് വെളിച്ചംവീശുന്ന ഒട്ടന വധി സംഭവങ്ങള്‍ ഇസ്‌ലാമിക ചരിത്രത്തില്‍ കാണാനാവും. ഒരു സംഭവമിതാ:ഉമറി(റ )ന്റെ ഭരണകാലത്ത് ഈജിപ്തിലെ ഗവര്‍ണറായിരുന്ന അംറുബ്‌നുല്‍ ആസി(റ )ന്റെ പുത്രന്‍ മുഹമ്മദുബ്‌നു അംറ് കോപ്റ്റിക് വിഭാഗത്തില്‍പ്പെട്ട ഒരു സാധാരണക്കാരനെ തന്റെ കുതി രയുടെ മുമ്പില്‍ സ്വന്തം കുതിരയെ ഓടിച്ചതിന്റെ പേരില്‍ അടിച്ചു. 'ഞാന്‍ മാന്യന്മാരുടെ പുത്രനാണ്' എന്നു പറഞ്ഞുകൊണ്ടായിരുന്നു പ്രഹരം. ഈ പ്രശ്‌നം ഖലീഫയുടെ അടുക്കലെത്തി. ഗവര്‍ണറെയും പുത്രനെയും മദീനയിലേക്ക് വിളിപ്പിച്ചു കൊണ്ട് ഉമര്‍(റ )ചോദിച്ചു: 'എന്നുമുതല്‍ക്കാണ് നിങ്ങള്‍ ജനങ്ങളെ അടിമകളാക്കിത്തുടങ്ങിയ ത്? അവരുടെ മാതാക്കള്‍ അവരെ പ്രസവിച്ചത് സ്വതന്ത്രരായിട്ടല്ലേ'. തുടര്‍ന്ന് മര്‍ദിതനായ ആ കോപ്റ്റിക് വംശജന്റെ കയ്യില്‍ ചാട്ടവാറു നല്‍കിക്കൊണ്ട് പ്രതികാരം ചെയ്യുവാന്‍ ഉമര്‍ കല്‍പിച്ചു. നിയമ ത്തിനു മുന്‍പില്‍ എല്ലാവരും സമന്മാരാണെന്ന തത്ത്വം ഇത്രത്തോ ളം പ്രായോഗികമാക്കിയ ഇസ്‌ലാമിക സമൂഹമല്ലാത്ത മറ്റൊരു സമൂ ഹത്തെ ലോകചരിത്രത്തില്‍ കാണുക സാധ്യമല്ല.

ഇസ്‌ലാമികരാഷ്ട്രത്തില്‍ ജീവിക്കുന്ന അമുസ്‌ലിം പൗരന്മാരോ ടും നീതിനിര്‍വഹണത്തിന്റെ കാര്യത്തില്‍ വിവേചനമൊന്നും കാണിക്കാന്‍ പാടില്ലെന്നാണ് ഇസ്‌ലാം പഠിപ്പിക്കുന്നത്. വിശുദ്ധക്വുര്‍ആന്‍ നിഷ്‌കര്‍ഷിക്കുന്നത് കാണുക: 'സത്യവിശ്വാസികളേ, നിങ്ങ ള്‍ അല്ലാഹുവിനുവേണ്ടി നിലകൊള്ളുന്നവരും നീതിക്ക് സാക്ഷ്യം വഹിക്കുന്നവരുമായിരിക്കുക. ഒരു ജനതയോടുള്ള അമര്‍ഷം നീതി പാലിക്കാതിരിക്കാന്‍ നിങ്ങള്‍ക്ക് പ്രേരകമാകരുത്. നിങ്ങള്‍ നീതി പാലിക്കുക. അതാണ് ധര്‍മനിഷ്ഠയോട് ഏറ്റവും അടുത്തത.

ഈ ക്വുര്‍ആനിക കല്‍പനയെ പൂര്‍ണമായി ഉള്‍ക്കൊണ്ടുകൊണ്ടായിരു ന്നു പ്രവാചകനും സച്ചരിതരായ ഖലീഫമാരും ഭരണം നടത്തിയത്. ക്രൈസ്തവനായ കോപ്റ്റിക് വംശജനും മുസ്‌ലിമായ ഗവര്‍ണ റുടെ പുത്രനും തമ്മിലുണ്ടായ പ്രശ്‌നത്തില്‍ മുസ്‌ലിം അനീതിചെയ് തുവെന്ന് മനസ്സിലാക്കിയ ഖലീഫാ ഉമറിെേന്റ കാലത്തുതന്നെ നടന്ന മറ്റൊരു സംഭവമിതാ: ഗവര്‍ണറായിരുന്ന അംറുബ്‌നുല്‍ ആസ്വ്‌(റ ) ഒരു ക്രൈസ്തവസ്ത്രീയുടെ വീട് നിര്‍ബന്ധപൂര്‍വം പള്ളി യോട് ചേര്‍ക്കുകയുണ്ടായി. സ്ത്രീ ഇതേക്കുറിച്ച് ഖലീഫയോട് പ രാതി പറഞ്ഞു: ഖലീഫാ ഉമര്‍ ഗവര്‍ണറെ വിളിച്ച് വിശദീകരണ മാവശ്യപ്പെട്ടു. അദ്ദേഹം പറഞ്ഞു: 'മുസ്‌ലിംകളുടെ അംഗസംഖ്യ വര്‍ധിച്ചതു കാരണം പള്ളിയില്‍ സ്ഥലമില്ലാതെ വിഷമംനേരിട്ടു. പള്ളി വിശാലമാക്കേണ്ടതിന്നായി അതിന്റെ അടുത്തുള്ള ക്രൈസ് തവസ്ത്രീയുടെ ഭവനം ആവശ്യപ്പെടുകയും അതിന്ന് ന്യായമായ വില നല്‍കാമെന്ന് ഉറപ്പുനല്‍കുകയും ചെയ്തു. അവര്‍ വിസമ്മതി ച്ചതുകാരണം വീടുപൊളിച്ച് പള്ളിയോട് ചേര്‍ക്കാന്‍ നിര്‍ബന്ധിത മാവുകയാണുണ്ടായത്. അതിന്റെ വില ആ സ്ത്രീക്ക് ആവശ്യമുള്ള ഏതുസമയത്തും സ്വീകരിക്കുവാന്‍ സാധിക്കുമാറ് പൊതുഖജനാ വില്‍ നിക്ഷേപിക്കുകയും ചെയ്തിട്ടുണ്ട്.' സാധാരണഗതിയില്‍ ന്യാ യമെന്നു പറയാവുന്ന ഗവര്‍ണറുടെ നടപടിയെ ശരിവെക്കാന്‍ ഉമര്‍ തയാറായില്ല. പള്ളിയുടെ പ്രസ്തുത ഭാഗം പൊളിച്ചുമാറ്റി പകരം ആ ക്രൈസ്തവസ്ത്രീയുടെ വീട് യഥാസ്ഥാനത്ത് നിര്‍മിച്ചു കൊടു ക്കുവാന്‍ കല്‍പിക്കുകയാണ് നീതിമാനായ ആ ഭരണാധികാരി ചെ യ്തത്.

നാലാം ഖലീഫയായിരുന്ന അലി(റ )ടെ കാലത്ത് നടന്ന ഒരു സംഭവം നോക്കുക: ഖലീഫ തന്റെ പടയങ്കി ഒരു ജൂതന്‍ മോഷ്ടിച്ചു വെന്ന് പറഞ്ഞ് കോടതിയെ സമീപിച്ചു. ന്യായാധിപനായ ശുറൈഹ് ജൂതന്റെയും ഖലീഫയുടെയും അവകാശവാദങ്ങള്‍ പരിശോധിച്ചു. തന്റെ കൈവശമുള്ള പടയങ്കി തന്‍േറതു തന്നെയാണെന്നു ജൂതന്‍ വാദിച്ചപ്പോള്‍ അതു തന്‍േറതാണെന്നും ജൂതന്‍ മോഷ്ടിച്ചതാണെ ന്നും ഖലീഫ അലി മൊഴി നല്‍കി. തന്റെ വാദം തെളിയിക്കാനായി ഖലീഫ ഹാജരാക്കിയ സാക്ഷികള്‍ മകന്‍ ഹസനും ഭൃത്യന്‍ ഖമറുമായിരുന്നു. ഇവര്‍ രണ്ടുപേരും ഖലീഫയുടെ സ്വന്തക്കാരായ തിനാല്‍ സാക്ഷ്യം സ്വീകാര്യമല്ലെന്നും മറ്റു തെളിവുകള്‍ ഇല്ലാത്തതി നാല്‍ പടയങ്കി ജൂതനില്‍നിന്നു വാങ്ങുവാന്‍ നിയമം അനുവദിക്കു ന്നില്ലെന്നും ന്യായാധിപന്‍ വിധിച്ചു. ന്യായധിപന്റെയടുത്തുനിന്ന് പടയങ്കിവാങ്ങി നടന്നുനീങ്ങിയ ജൂതന്‍ അല്‍പദൂരം മുന്നോട്ടുപോ യി തിരിച്ചുവന്നുകൊണ്ട് പറഞ്ഞു: 'നിശ്ചയമായും ഇത് പ്രവാചക ന്മാരുടെ നിയമം തന്നെയെന്ന് ഞാന്‍ സാക്ഷ്യം വഹിക്കുന്നു. മതി യായ തെളിവില്ലാത്തതിനാല്‍ കോടതി അദ്ദേഹത്തിനെതിരെ വിധി പ്രസ്താവിക്കുന്നു. ഇതു വളരെ മഹത്തരം തന്നെ. അതിനാല്‍ അ ല്ലാഹുവല്ലാതെ ആരാധ്യനില്ലെന്നും മുഹമ്മദ്(സ) അല്ലാഹുവിന്റെ ദൂത നാണെന്നും ഞാന്‍ സാക്ഷ്യം വഹിക്കുന്നു'. തുടര്‍ന്ന് ഖലീഫയുടെ നേരെ തിരിഞ്ഞ് അദ്ദേഹം ഇങ്ങനെ പറഞ്ഞു: 'ഈ പടയങ്കി താങ്ക ളുടേത് തന്നെയാണ്. നിങ്ങള്‍ സ്വിഫ്ഫീനിലേക്ക് പോകുമ്പോള്‍ പിന്നിലുണ്ടായിരുന്ന ഞാന്‍ താങ്കളുടെ ചാരനിറത്തിലുള്ള ഒട്ടക ത്തിന്റെ പുറത്തുനിന്ന് അത് തട്ടിയെടുത്തതാണ്'. ഖലീഫ ചിരിച്ചു കൊണ്ട് പ്രതിവചിച്ചു. 'ഞാനത് താങ്കള്‍ക്ക് തന്നെ ദാനമായി തന്നി രിക്കുന്നു'.

ഇസ്‌ലാമിക രാഷ്ട്രത്തില്‍ ജീവിക്കുന്ന അമുസ്‌ലിം പൗരന്മാരുടെ വിശ്വാസസ്വാതന്ത്ര്യമോ ആരാധനാസ്വാതന്ത്ര്യമോ ഹനിക്കുവാന്‍ ഖലീഫമാര്‍ക്കവകാശമില്ല. അമുസ്‌ലിം പൗരന്മാരോട് അനീതി കാണിക്കുന്നത് അക്ഷന്തവ്യമായ അപരാധമാണെന്നാണ് പ്രവാചകന്‍പഠിപ്പിച്ചിരിക്കുന്നത്. അദ്ദേഹം പറഞ്ഞു: 'സൂക്ഷിച്ചുകൊ ള്ളുക. അമുസ്‌ലിം പൗരന്മാരെ ആരെങ്കിലും അടിച്ചമര്‍ത്തുകയോ അവരുടെമേല്‍ കഴിവിന്നതീതമായി നികുതിഭാരം കെട്ടിയേല്‍പി ക്കുകയോ അവരോട് ക്രൂരമായി പെരുമാറുകയോ അവരുടെ അവ കാശങ്ങള്‍ വെട്ടിക്കുറക്കുകയോ ചെയ്യുകയാണെങ്കില്‍ അന്ത്യനാളി ല്‍ അവന്നെതിരില്‍ ഞാന്‍ സ്വയംതന്നെ പരാതി ബോധിപ്പിക്കുന്ന താണ്'. ചരിത്രപണ്ഡിതനായ സര്‍ തോമസ് ആര്‍നോള്‍ഡ് എഴുതുന്നത് കാണുക: 'മുഹമ്മദ് തന്നെ പല അറബ് ക്രൈസ്തവ ഗോത്രങ്ങളുമാ യും സന്ധിയിലേര്‍പെട്ടിരുന്നു. അവര്‍ക്ക് അദ്ദേഹം സംരക്ഷണവും സ്വന്തം മതമനുസരിച്ച് ജീവിക്കുവാനുള്ള സ്വാതന്ത്ര്യവും ഉറപ്പുവരു ത്തി. അവരുടെ പുരോഹിതര്‍ക്കുണ്ടായിരുന്ന സവിശേഷാധികാ രങ്ങള്‍ പഴയതുപോലെ നിലനിര്‍ത്തി'

പ്രവാചകന്റെ പാത പിന്‍തുടര്‍ന്ന അദ്ദേഹത്തിന്റെ ഖലീഫമാ രും അമുസ്‌ലിം പൗരന്മാരുടെ വിശ്വാസങ്ങളും ആരാധനാസമ്പ്രദായ ങ്ങളും തുടരാന്‍ അനുവദിച്ചിരുന്നതായും അവരുടെ ജീവനും സ്വ ത്തിനും പരിപൂര്‍ണ സംരക്ഷണം ഉറപ്പുവരുത്തിയിരുന്നതായും കാണാന്‍ കഴിയും. ഖലീഫ ഉമര്‍ േഈലിയാ നിവാസികള്‍ക്ക് നല്‍കിയ രക്ഷാകരാ റിലെ വ്യവസ്ഥകള്‍ കാണുക: 'ദൈവദാസനും വിശ്വാസികളുടെ നേതാവുമായ ഉമര്‍ ഈലിയാവാസികള്‍ക്ക് നല്‍കുന്ന സംരക്ഷണം. എല്ലാവരുടെയും ജീവന്നും സ്വത്തിനും ആരാധനാലയങ്ങള്‍ക്കും കുരിശുകള്‍ക്കും മതസംബന്ധിയായ എല്ലാറ്റിനും സംരക്ഷണം ഉറ പ്പുനല്‍കുന്നു. ആരുടെയും ചര്‍ച്ചുകള്‍ വാസസ്ഥലമാക്കുകയോ ന ശിപ്പിക്കുകയോ ചെയ്യരുത്. അവയോട് ചേര്‍ന്നു നില്‍ക്കുന്ന വസ്തു വഹകളോ കുരിശുകളോ പിടിച്ചെടുക്കരുത്. ആരെയും ഉപദ്രവിക്ക രുത്'.

ഇങ്ങനെ, ഇസ്‌ലാമിക രാഷ്ട്രത്തിലെ അമുസ്ലിം പൗരന്മാർക്ക് യാതൊരു വിധ അനീതിയും അനുഭവിക്കേണ്ടി വരാത്ത രീതിയിലുള്ളതാണ് ഇസ്‌ലാമിക നിയമങ്ങൾ എന്ന കാണാൻ കഴിയും.

മുഹമ്മദ് നബി(സ)യുടെ പ്രവാചകത്വത്തിന്റെ ആരംഭത്തെക്കുറിച്ച് മുസ്‌ലിംകള്‍ പറയുന്ന കഥ, വഹ്‌യിനെക്കുറിച്ച അദ്ദേഹത്തിന്റെ അവകാശവാദങ്ങള്‍ അടിസ്ഥാനരഹിതമാണെന്നാണ് വ്യക്തമാക്കുന്നുണ്ട്. ദൈവികമായ പ്രചോദനമുണ്ടാകുമ്പോള്‍ ഉണ്ടാകേണ്ട സമാധാനത്തിനും സന്തോഷത്തിനും പകരം ഭയവിഹ്വലനായി വീട്ടില്‍ തിരിച്ചെത്തുന്ന മുഹമ്മദിനെയാണ് വെളിപാട് തുടങ്ങിയെന്ന് പറയപ്പെടുന്ന ദിവസം ചരിത്രത്തില്‍ നാം കാണുന്നത്. ദൈവദൂതനായ ജിബ്‌രീല്‍ തന്നെയാണ് പ്രവാചകനടുക്കല്‍ വന്നിരുന്നതെങ്കില്‍ ഇതാകുമായിരുന്നില്ല അനുഭവം. അതിനാല്‍ മുഹമ്മദ് നബിയ്ക്ക് നാല്‍പതാം വയസ്സിലുണ്ടായിയെന്ന് പറയപ്പെടുന്ന വെളിപാട് ദൈവത്തില്‍ നിന്നല്ലെന്ന കാര്യം സ്പഷ്ടമാണ്. പ്രവാചകത്വാരംഭവുമായി ബന്ധപ്പെട്ട നിവേദനങ്ങളെ വിശകലനം ചെയ്തുകൊണ്ട് നബി (സ) അനുഭവിച്ച വെളിപാട് ദിവ്യമല്ലെന്ന് വരുത്തിത്തീര്‍ക്കാന്‍ വേണ്ടി നബിവിമര്‍ശകര്‍ ഉന്നയിക്കുന്ന പരാമൃഷ്ടവാദങ്ങള്‍ ന്യായമല്ലേ?

ല്ല. അന്തിമപ്രവാചകന് ദിവ്യവെളിപാടുകള്‍ കിട്ടിത്തുടങ്ങിയ സന്ദര്‍ഭത്തെക്കുറിച്ചുള്ള ചരിത്രരേഖകളൊന്നും പ്രസ്തുതവെളിപാടുകളുടെ ദൈവികതയെ സംശയാസ്പദമാക്കുന്ന യാതൊരു പരാമര്‍ശവും ഉള്‍ക്കൊള്ളുന്നില്ല. പ്രവാചകന്റെ നാല്‍പതാം വയസ്സിലുണ്ടായ പ്രവാചകത്വലബ്ധിയെയും വഹ്‌യിന്റെ ആരംഭത്തെയും കുറിച്ചുള്ള കുറ്റമറ്റ നിവേദകപരമ്പരയിലൂടെയുള്ള വിവരണമുള്ളത് സ്വഹീഹുല്‍ ബുഖാരിയിലാണ്. പ്രവാചകപത്‌നി ആഇശ (റ), തന്റെ സഹോദരീപുത്രന്‍ ഉര്‍വക്ക് ആദ്യവെളിപാടിന്റെ സമയത്തെ പ്രവാചകാനുഭവങ്ങള്‍ പ്രവാചകന്റെ തന്നെ ആത്മകഥനാപരമായ വാചകങ്ങളുദ്ധരിച്ചുകൊണ്ട് സ്വന്തം വാക്കുകളില്‍ വിശദീകരിച്ചുകൊടുത്തതാണ് ബുഖാരി തന്റെ സ്വഹീഹിന്റെ തുടക്കത്തില്‍ ഹദീഥായി രേഖപ്പെടുത്തിയിട്ടുള്ളത് (കിതാബുല്‍ വഹ്‌യ്). പ്രവാചകത്വത്തിന്റെ ആരംഭത്തെക്കുറിച്ച് പരാമൃഷ്ട ഹദീഥ് നല്‍കുന്നത് താഴെക്കൊടുത്തിരിക്കുന്ന വിവരങ്ങളാണ്.

1. മുഹമ്മദ് നബി(സ)ക്ക് പകല്‍വെളിച്ചം പോലെ സത്യമായിപ്പുലരുന്ന സ്വപ്‌നദര്‍ശനങ്ങള്‍ നിരന്തരമായി ഉണ്ടാകാന്‍ തുടങ്ങി. ഇതായിരുന്നു പ്രവാചകനുമായുള്ള ദൈവിക ആശയവിനിമയത്തിന്റെ ആരംഭം.

2. ഉറക്കത്തില്‍ താന്‍ കാണുന്ന സ്വപ്‌നങ്ങളുടെ പുലര്‍ച്ച പ്രവാചകനെ ചിന്താകുലനാക്കുകയും അദ്ദേഹം മക്കയിലെ ഒരു പര്‍വതത്തിനുമുകളിലുള്ള ഹിറാഗുഹയില്‍ ഏകാന്തനായി പ്രാര്‍ത്ഥനകളില്‍ മുഴുകിയിരിക്കുന്ന ശീലം ആരംഭിക്കുകയും ചെയ്തു. രണ്ടോ മൂന്നോ ദിവസങ്ങള്‍ക്കാവശ്യമായ ജീവിതവിഭവങ്ങളുമായി ഗുഹയില്‍ തങ്ങുകയും ശേഷം വീട്ടിലേക്ക് മടങ്ങി വീണ്ടും വിഭവശേഖരണം നടത്തി ഗുഹയിലേക്കുതന്നെ തിരിച്ചുപോവുകയുമായിരുന്നു പതിവ്.

3. ഇങ്ങനെ പ്രവാചകന്‍ ഗുഹയിലിരിക്കുന്ന ഒരു ദിവസമാണ് ക്വുര്‍ആന്‍ അവതരണത്തിന് സമാരംഭം കുറിച്ചുകൊണ്ട് ഒരു മലക്ക് പ്രവാചകനരികില്‍ പ്രത്യക്ഷപ്പെട്ടത്.

4. മലക്ക് പ്രവാചകനോട് ‘ഇക്വ്‌റഅ്’ (വായിക്കുക/ഓതുക) എന്ന് കല്‍പിച്ചു. ‘ഞാന്‍ വായിക്കാനറിയുന്നവനല്ല’ (മാ അന ബി ക്വാരിഅ്) എന്നായിരുന്നു നബി(സ)യുടെ മറുപടി. അപ്പോള്‍ മലക്ക് പ്രവാചകന് താങ്ങാന്‍ കഴിയുന്നതിലുമപ്പുറത്തുള്ള ശക്തിയോടെ അദ്ദേഹത്തെ പിടിച്ചുഞെരുക്കുകയും ശേഷം സ്വതന്ത്രനാക്കുകയും ചെയ്ത് ‘ഇക്വ്‌റഅ്’ എന്ന കല്‍പന ആവര്‍ത്തിച്ചു. വീണ്ടും പ്രവാചകന്‍ പഴയ മറുപടി തന്നെ പറഞ്ഞു. മലക്ക് വീണ്ടും പ്രവാചകനെ ഞെരുക്കുകയും സ്വതന്ത്രനാക്കുകയും കല്‍പന ആവര്‍ത്തിക്കുകയും ചെയ്തു. മൂന്നാം തവണയും ഇതേഘട്ടങ്ങള്‍ കടന്നപ്പോള്‍ മലക്ക് പരിശുദ്ധ ക്വുര്‍ആനില്‍ 96-ാം അധ്യായം സൂറത്തുല്‍ അലക്വിലെ ആദ്യത്തെ അഞ്ച് വചനങ്ങള്‍ പ്രവാചകനെ ഓതിക്കേള്‍പിച്ചു. ഇതായിരുന്നു ക്വുര്‍ആന്‍ അവതരണത്തിന്റെയും പ്രവാചകത്വത്തിന്റെയും തുടക്കം. പ്രസ്തുത വചനങ്ങളുടെ സാരം ഇപ്രകാരമാണ്: ”സൃഷ്ടിച്ചവനായ നിന്റെ രക്ഷിതാവിന്റെ നാമത്തില്‍ വായിക്കുക. മനുഷ്യനെ അവന്‍ ഭ്രൂണത്തില്‍ നിന്ന് സൃഷ്ടിച്ചിരിക്കുന്നു. നീ വായിക്കുക: നിന്റെ രക്ഷിതാവ് പേനകൊണ്ട് പഠിപ്പിച്ചവനായ ഏറ്റവും വലിയ ഔദാര്യവാനാകുന്നു. മനുഷ്യന് അറിയാത്തത് അവന്‍ പഠിപ്പിച്ചിരിക്കുന്നു.”

5. ഗുഹയില്‍ ഏകാന്തനായിരിക്കെ മലക്കുമായുണ്ടായ മുഖാമുഖവും തത്സസമയത്തെ അനുഭവങ്ങളും പ്രവാചകനെ ഭയവിഹ്വലനാക്കി. പരിഭ്രാന്തനായി അതിവേഗത്തില്‍ മിടിച്ചുകൊണ്ടിരിക്കുന്ന ഹൃദയവുമായി വീട്ടിലേക്ക് മടങ്ങിയെത്തിയ അദ്ദേഹം പത്‌നി ഖദീജ ബിന്‍ത് ഖുവയ്‌ലിദിനോട് (റ) തന്നെ പുതപ്പിക്കുവാന്‍ ആവശ്യപ്പെട്ടു. വിറ നിന്ന് സാധാരണ നില കൈവരിക്കുവോളം പ്രവാചകന്‍ (സ) പുതച്ചുമൂടി നിന്നു. 6. അനന്തരം പുതപ്പില്‍നിന്ന് പുറത്തുവന്ന പ്രവാചകന്‍ ഉണ്ടായ സംഭവങ്ങള്‍ വിവരിക്കുകയും തനിക്ക് ഭയം അനുഭവപ്പെടുന്നുവെന്ന് പറയുകയും ചെയ്തു. അപ്പോള്‍ ഖദീജ (റ) ‘ഇല്ല, അല്ലാഹുവാണ് സത്യം, അല്ലാഹു അങ്ങയെ നിന്ദിക്കുകയില്ല. കാരണം അങ്ങ് ബന്ധുക്കളുടെ കാര്യം ശ്രദ്ധിക്കുന്നു, ദുര്‍ബലരുടെ ഭാരങ്ങള്‍ ചുമക്കുന്നു, ദരിദ്രര്‍ക്കും അശരണര്‍ക്കും സംരക്ഷണമേകുന്നു, അതിഥികളെ ആദരിക്കുന്നു, പ്രയാസപ്പെടുന്നവരെ സഹായിക്കുന്നു’ എന്നുപറഞ്ഞുകൊണ്ട് പ്രവാചകനെ ആശ്വസിപ്പിച്ചു. 7. പ്രവാചകനെ ആശ്വസിപ്പിച്ചശേഷം ഖദീജ അദ്ദേഹത്തെയും കൂട്ടി തന്റെ ബന്ധുവും ക്രൈസ്തവ-ജൂതഗ്രന്ഥങ്ങളില്‍ പ്രാവിണ്യമുണ്ടായിരുന്ന പണ്ഡിതനുമായിരുന്ന വറക്വത്ബ്‌നു നൗഫലിനടുത്തേക്കുപോയി. വറക്വ കാഴ്ചശക്തിയടക്കം നഷ്ടപ്പെട്ട് അങ്ങേയറ്റം വൃദ്ധനായിത്തീര്‍ന്നിരുന്നു. പ്രവാചകനുണ്ടായ അനുഭവങ്ങള്‍ കേട്ട വറക്വ, മോശെ (മൂസ) പ്രവാചകന്റെയടുക്കലേക്ക് ദിവ്യവെളിപാടുമായി വന്ന മലക്ക് ജിബ്‌രീല്‍ തന്നെയാണ് ഹിറാ ഗുഹയില്‍ വന്നതെന്ന് പറയുകയും പ്രവാചകന്‍ തനിക്ക് ലഭിച്ചുതുടങ്ങിയ ദിവ്യവെളിപാടുകള്‍ പ്രബോധനം ചെയ്യാന്‍ തുടങ്ങുമ്പോള്‍ പൂര്‍വപ്രവാചകന്‍മാരെപ്പോലെ കടുത്ത എതിര്‍പ്പുകള്‍ നേരിടുമെന്നും മക്കയില്‍നിന്ന് പുറത്താക്കപ്പെടുമെന്നും അന്ന് ഒരു യുവാവായി നാട്ടിലുണ്ടാകണമെന്നും പ്രവാചകനെ പിന്തുണക്കാന്‍ കഴിയണമെന്നും താന്‍ ആഗ്രഹിച്ചുപോകുന്നുവെന്നും പറഞ്ഞു. പക്ഷേ ഈ കൂടിക്കാഴ്ച കഴിഞ്ഞ് അധികമാകും മുമ്പെ അദ്ദേഹം മരണപ്പെട്ടുപോയി.

പരിശുദ്ധ ക്വുര്‍ആനിന്റെ അവതരണസമയത്ത് പ്രവാചകനുണ്ടായ പരാമൃഷ്ടാനുഭവങ്ങളില്‍ അസംഗതമായിട്ടെന്താണുള്ളതെന്ന് വ്യക്തമാക്കേണ്ടത് വിമര്‍ശകര്‍ തന്നെയാണ്. ഭൗതികവാദികളായ നബിവിമര്‍ശകര്‍ക്ക് വഹ്‌യ് എന്ന ആശയത്തെ തന്നെ ഉള്‍ക്കൊള്ളാന്‍ കഴിയാതിരിക്കുക സ്വാഭാവികമാണ്. പ്രപഞ്ചത്തിന് ഒരു സ്രഷ്ടാവ് തന്നെയില്ലെന്ന് വിശ്വസിക്കുന്നവര്‍ക്ക് ആ സ്രഷ്ടാവ് മനുഷ്യര്‍ക്കായുളള ധാര്‍മികപദ്ധതി വെളിപാടുകള്‍ വഴി പ്രവാചകന്‍മാര്‍ക്കെത്തിച്ചു കൊടുക്കുന്നുവെന്ന ആശയത്തെ ഒരിക്കലും അംഗീകരിക്കാനാകില്ല. അവരുടെ നിഷേധത്തിന് വഹ്‌യിന്റെ സമയത്തെ പ്രവാചകാനുഭവങ്ങളെ വിശകലനം ചെയ്തുകൊണ്ട് മറുപടി പറയുന്നത് അര്‍ത്ഥശൂന്യമാണ്. ദൈവാസ്തിത്വത്തെക്കുറിച്ചും ദൈവിക മാര്‍ഗദര്‍ശനത്തിന്റെ അനിവാര്യതയെക്കുറിച്ചുമെല്ലാമുളള സംവാദങ്ങളാണ് അവരുമായി നടക്കേണ്ടത്. പടച്ചവനുണ്ടെന്നും വഹ്‌യ് എന്നൊരു സംവിധാനമുണ്ടെന്നും അംഗീകരിക്കുന്നവര്‍ക്ക് മാത്രമാണ് വഹ്‌യിന്റെ പ്രവാചകാനുഭവങ്ങളെ അപഗ്രഥിക്കേണ്ട ആവശ്യമുണ്ടാകുന്നത്.

മിഷനറിമാര്‍ തീര്‍ച്ചയായും രണ്ടാമതുപറഞ്ഞ ഗണത്തില്‍ വരുന്നവരാണ്. പ്രവാചകനുണ്ടായത് ദൈവിക വെളിപാടല്ലെന്ന് വരുത്തിത്തീര്‍ക്കാന്‍ ഹദീഥുകളിലുള്ള ഹിറാ വിവരണങ്ങളെ ഓറിയന്റലിസ്റ്റ് വ്യാഖാനങ്ങളെ ഉപജീവിച്ചുകൊണ്ട് അവര്‍ വിമര്‍ശനാത്മകമായി വിശകലനം ചെയ്തതിന് കയ്യും കണക്കുമില്ല. ആ വിമര്‍ശനങ്ങളെല്ലാം, സ്വാഭാവികമായും വഹ്‌യിന്റെ സമയത്ത് പ്രവാചകന്‍മാര്‍ക്കുണ്ടാകുന്ന അനുഭവങ്ങളെന്തൊക്കെയാണെന്ന് ബൈബിളുപയോഗിച്ച് വിശദീകരിച്ചശേഷം പ്രവാചകനുണ്ടായത് അത്തരത്തിലുള്ള അനുഭവങ്ങളൊന്നുമല്ലെന്ന് പറയാനാണ് അടിസ്ഥാനപരമായി ശ്രമിക്കുന്നത്. വഹ്‌യ് പ്രവാചകനില്‍ ഭൗതികമായി ഏതുതരം പ്രതിഫലനങ്ങളാണ് സൃഷ്ടിക്കുക എന്നതിനെ സംബന്ധിച്ച തെറ്റുപറ്റാത്ത ധാരണകള്‍ നാം സ്വരൂപിക്കേണ്ടത് ബൈബിളില്‍ നിന്നാണ് എന്ന നിലപാടാണ് ഈ വര്‍ത്തമാനത്തിന്റെ അടിത്തറ.

ബൈബിള്‍ നൂറുശതമാനവും ദൈവികമാണെന്നും അതില്‍ മനുഷ്യ നിഗമനങ്ങളിടം പിടിച്ചിട്ടില്ലെന്നും എഴുതപ്പെട്ട അതേ രീതിയില്‍ തന്നെ ബൈബിള്‍ പുസ്തകങ്ങള്‍ ഇപ്പോഴും നിലനില്‍ക്കുന്നുവെന്നും മിഷനറിമാര്‍ തെളിയിക്കുമ്പോള്‍ മാത്രമേ ബൈബിളുപയോഗിച്ച് മുഹമ്മദ് നബി(സ)യുടെ വഹ്‌യനുഭവങ്ങളെ നിരൂപണം ചെയ്യുന്നതിന് സാധൂകരണമുണ്ടാവുകയുള്ളൂ. ബൈബിളില്‍ വഹ്‌യിനെക്കുറിച്ച് ഏഴുതപ്പെട്ടിരിക്കുന്നതെന്നല്ലാം സത്യമാണെന്ന് വിശ്വസിക്കുന്നതില്‍ -ബൈബിളിന്റെ ദൈവികത തെളിയിക്കപ്പെടാത്തിടത്തോളം കാലം- യാതൊരു യുക്തിയുമില്ല. ഇതുപോലെത്തന്നെ പ്രധാനമാണ് എല്ലാ പ്രവാചകന്‍മാരുടെയും വെളിപാടനുഭവങ്ങള്‍ ഒരുപോലെയായിരിക്കണമെന്ന് ശഠിക്കുന്നത് അര്‍ത്ഥശൂന്യമാണെന്നതും. ദൈവം അവന്റെ പ്രവാചകന്‍മാരോട് സംസാരിക്കാന്‍ വ്യത്യസ്ത മാര്‍ഗങ്ങള്‍ സ്വീകരിക്കുകയും ആശയവിനിമയത്തിന് അവരെ വ്യത്യസ്ത അനുഭവങ്ങളിലൂടെ കൂട്ടിക്കൊണ്ടുപോവുകയും ചെയ്യുന്നതില്‍ എന്ത് അസാംഗത്യമാണുള്ളത്?

ചില പ്രവാചകന്‍മാരുടെ വെളിപാട് സ്വീകരണസമയത്തെ അനുഭവങ്ങള്‍ തന്നെ എല്ലാ പ്രവാചകന്‍മാര്‍ക്കും അതുപോലെ ആവര്‍ത്തിക്കുമെന്ന് ദൈവം പറയാത്തിടത്തോളം കാലം അത്തരം താരതമ്യങ്ങള്‍ തന്നെ അപ്രസക്തമാണെന്നതാണ് വസ്തുത. നബി(സ)യുടെ വ്യക്തിനിഷ്ഠമായ വെളിപാടനുഭവങ്ങളെ ബൈബിളിന്റെ മാത്രം അടിസ്ഥാനത്തില്‍ തള്ളിക്കളയാന്‍ ശ്രമിക്കുന്നത് തീര്‍ത്തും ബാലിശമായ ഒരു മിഷനറി സങ്കുചിതത്വമാണെന്ന് ചുരുക്കം.

ഇനി, ഹിറാ അനുഭവങ്ങളെ ബൈബിളുപയോഗിച്ച് വിശകലനം ചെയ്താല്‍ അവയുടെ ദൈവികതയെ നിഷേധിക്കേണ്ടി വരുമോ എന്ന് നാം പരിശോധിക്കുക. ബൈബിളിനെ അന്ധമായി പിന്‍പറ്റുന്നവര്‍ക്കുമാത്രം ബാധകമായ ഒരു ചര്‍ച്ചയാണ് ഇത്. എങ്കിലും നാം പരിശോധനയ്ക്ക് മുതിരുക. പ്രവാചകന് സത്യമായിപ്പുലരുന്ന സ്വപ്‌നദര്‍ശനങ്ങള്‍ ഉണ്ടായിത്തുടങ്ങിയെന്നും അതായിരുന്നു അല്ലാഹു അദ്ദേഹവുമായി സംവദിക്കാനാരംഭിച്ച രീതി എന്നുമാണ് ഹദീഥ് ഒന്നാമതായി പറയുന്നത്. ബൈബിളികമായ വീക്ഷണത്തില്‍ ഇതിന് എന്ത് കുഴപ്പമാണുള്ളത്? നേരായിപ്പുലരുന്ന സ്വപ്‌നങ്ങള്‍ ഉറക്കില്‍ പ്രവാചകന്‍മാരെ കാണിക്കുന്നത് ദൈവത്തിന്റെ രീതിയാണെന്നു തന്നെയല്ലേ ബൈബിളും പറയുന്നത്? ജോസഫും (ഉല്‍പത്തി 37: 5-8), ജേക്കബും (ഉല്‍പത്തി 28: 12-14), അബ്രഹാമും (ഉല്‍പത്തി 15: 1), സോളമനും (1 രാജാക്കന്‍മാര്‍ 8: 5) എല്ലാം ദൈവത്താല്‍ സത്യസ്വപ്‌നങ്ങള്‍ കാണിക്കപ്പെട്ടതിനെക്കുറിച്ചുള്ള ബൈബിള്‍ വചനങ്ങള്‍ ഉണ്ടെന്നിരിക്കെ മുഹമ്മദ് നബി(സ)ക്കുണ്ടായ സമാനമായ അനുഭവത്തെ ബൈബിളികമായ ഏത് പ്രതലമുപയോഗിച്ചാണ് മിഷനറിമാര്‍ക്ക് തള്ളിക്കളയേണ്ടി വരുന്നത്?

താന്‍ കാണാന്‍ തുടങ്ങിയ, അക്ഷരംപ്രതി പുലര്‍ന്നുകൊണ്ടിരിക്കുന്ന സ്വപ്‌നങ്ങള്‍ പ്രവാചകനെ (സ) അത്യധികം ചിന്താകുലനാക്കുകയും ഒറ്റയ്ക്കിരിക്കുവാന്‍ അദ്ദേഹം ഇഷ്ടപ്പെടുന്ന സ്ഥിതിയുണ്ടാക്കുകയും ചെയ്തതാണ് ഹിറയില്‍ പോയി പ്രാര്‍ത്ഥനകളില്‍ ശാന്തി കണ്ടെത്തുവാന്‍ ശ്രമിക്കുന്ന ഒരു സന്ദര്‍ഭം പ്രവാചകജീവിതത്തിലേക്ക് കൊണ്ടുവന്നത് എന്ന ചരിത്രവസ്തുത, തിരുനബി(സ)ക്ക് കാണിക്കപ്പെട്ട സ്വപ്‌നങ്ങള്‍ക്കുപിന്നില്‍ ജിബ്‌രീല്‍ മലക്കുമായി മുഖാമുഖം കാണാനുള്ള ഇടത്തിലേക്ക് അദ്ദേഹത്തെ നയിച്ചുകൊണ്ടുപോവുക എന്ന ദൈവിക പദ്ധതിയുണ്ടായിരുന്നുവെന്ന് സുതരാം വ്യക്തമാക്കുന്നുണ്ട്. അന്തിമവേദഗ്രന്ഥം അവതരിപ്പിച്ചു തുടങ്ങാനുള്ള സാഹചര്യം പ്രവാചകജീവിതത്തില്‍ സൃഷ്ടിക്കുവാനും പ്രവാചകത്വത്തിന്റെ മൂര്‍ത്തമായ ആരംഭം കുറിക്കുവാനും വേണ്ടി പടച്ചവന്‍ മുഹമ്മദ് നബി(സ)യുമായുള്ള സംവേദനത്തിന്റെ ആമുഖം സ്വപ്‌നങ്ങള്‍ വഴി നിര്‍വഹിച്ചുതുടങ്ങിയതില്‍ ശുദ്ധമായ ബൈബിള്‍ കണ്ണിലൂടെ നോക്കിയാല്‍ പോലും അസാധാരണമായി യാതൊന്നുമില്ലെന്ന് നാം കണ്ടു.

ഇനി ഹിറയിലേക്ക് വരാം. ഹിറാ ഗുഹയില്‍ വന്ന് മലക്ക് പ്രവാചകനെ ക്വുര്‍ആന്‍ വചനങ്ങള്‍ കേള്‍പിച്ച അനുഭവത്തെയാണ് ഓറിയന്റലിസ്റ്റുകളും മിഷനറിമാരും ഏറ്റവും പ്രധാനമായി ആക്രമിക്കാറുള്ളത്. മനഃശാസ്ത്രപരമായി പ്രസ്തുത സന്ദര്‍ഭത്തെ ഉപജീവിച്ചുകൊണ്ടുള്ള വിമര്‍ശനകാഠിന്യം തികച്ചും സ്വാഭാവികമാണ് എന്നുപറയുന്നതായിരിക്കും ശരി. കാരണം വെളിപാട് എന്നു പറയുമ്പോള്‍ അതിനെ കേവലം ഒരു ആന്തരിക പ്രചോദനമായി മാത്രം ചുരുക്കിയെഴുതാനാഗ്രഹിക്കുന്നവരാണ് മിഷനറിമാരെല്ലാം തന്നെ. തങ്ങള്‍ വേദഗ്രന്ഥമായി അവതരിപ്പിക്കുന്ന പഴയ-പുതിയ നിയമ പുസ്തകങ്ങളൊന്നും ദൈവമോ, ദൈവദൂതനോ ഏതെങ്കിലും പ്രവാചകനെ ഓതിക്കേള്‍പ്പിച്ചവയല്ലെന്നും പലരും പലകാലങ്ങളില്‍ സ്വന്തമായി എഴുതിയുണ്ടാക്കിയവയാണെന്നും ബോധ്യമുള്ളവര്‍ക്ക്, ദൈവം മനസ്സില്‍ സൃഷ്ടിക്കുന്ന ‘തോന്നലു’കള്‍ക്കനുസൃതമായി മനുഷ്യര്‍ സ്വന്തം വാചകങ്ങളില്‍ എഴുതിയുണ്ടാക്കുന്നവയാണ് വേദഗ്രന്ഥങ്ങള്‍ എന്ന് പറയേണ്ടി വരുന്നതില്‍ യാതൊരു അത്ഭുതത്തിനും വകയില്ല. ‘ദൈവനിവേശിതവും പരിശുദ്ധാത്മ പ്രചോദിതവുമാണ്, വാചകങ്ങള്‍ ദൈവികമല്ലെങ്കിലും വിശുദ്ധ ബൈബിള്‍’ എന്ന് സമാധാനിക്കുന്നവര്‍ക്കെങ്ങനെയാണ് തങ്ങള്‍ ശത്രുവായി കാണുന്ന മുഹമ്മദ് നബി(സ)ക്ക് ക്വുര്‍ആനിന്റെ പാഠം (text) ദൈവത്തിന്റെ മലക്ക് മൂര്‍ത്തമായി പ്രത്യക്ഷപ്പെട്ട് ഓതിക്കേള്‍പ്പിച്ചുതുടങ്ങി എന്ന വൃത്താന്തം അസ്വസ്ഥതയുണ്ടാക്കാതിരിക്കുക?

ക്വുര്‍ആനിലെ വചനങ്ങള്‍ നൂറുശതമാനം ദൈവികമാണെന്നും അവ അപ്പടി പ്രവാചകന് അവതരിപ്പിക്കപ്പെടുകയാണുണ്ടായതെന്നും വരുമ്പോള്‍ ബൈബിള്‍ ക്വുര്‍ആനുമായി താരതമ്യം പോലുമര്‍ഹിക്കുന്നില്ലെന്ന് തിരിച്ചറിയുന്നവരുടെ ആര്‍ക്കും പ്രവചിക്കാവുന്ന അസഹിഷ്ണുത മാത്രമാണ് ഹിറയില്‍ ചുറ്റിത്തിരിയുന്ന വിമര്‍ശകത്തൂലികകളുടെ മഷിയും ഊര്‍ജ്ജവുമെന്ന കാര്യം സ്പഷ്ടമാണ്. സത്യം സ്വാര്‍ത്ഥതാല്‍പര്യങ്ങള്‍ക്കെതിരായി വരുമ്പോള്‍ അസഹിഷ്ണുത കാണിക്കലല്ല മറിച്ച് അതിനോടുള്ള ശാത്രവം അവസാനിപ്പിക്കലാണ് രക്ഷയുടെ മാര്‍ഗമെന്ന് ഇവര്‍ തിരിച്ചറിഞ്ഞിരുന്നെങ്കില്‍ എന്ന് പ്രാര്‍ത്ഥിക്കുവാന്‍ മാത്രമേ നമുക്ക് കഴിയൂ. വെളിപാടിനെ ഒരു അന്തപ്രചോദനത്തിന്റെ മാത്രം തലമുള്ള മാനസികാനുഭൂതിയില്‍ ഒതുക്കി നിര്‍ത്തുകയല്ല അന്തിമപ്രവാചകന്റെ കാര്യത്തില്‍ പ്രപഞ്ചനാഥന്‍ ചെയ്തത്; മറിച്ച് അതിന്റെ ആദ്യ സന്ദര്‍ഭത്തില്‍ തന്നെ തികച്ചും മൂര്‍ത്തമായി തന്റെ മലക്കിനെ പ്രവാചകനുമുന്നില്‍ കൊണ്ടുചെന്നു നിര്‍ത്തുകയും ശാരീരികമായിത്തന്നെ പ്രവാചകനെ അതിശക്തമായി മലക്കിനെക്കൊണ്ട് സ്പര്‍ശിപ്പിക്കുകയും ശേഷം സ്ഫുടമായ വാചകങ്ങളിലുള്ള സംഭാഷണത്തിന് അവസരമൊരുക്കയും ചെയ്യുകയാണ്. അകത്തുനിന്ന് നിര്‍ഗളിക്കുന്ന യാതൊന്നുമല്ല, മറിച്ച് പുറത്തുനിന്ന് പ്രവാചകനിലേക്ക് പ്രവഹിക്കുന്നതാണ് വഹ്‌യ് എന്ന് ഇതിനേക്കാള്‍ വ്യക്തമായി ഭൗതിക പരിതസ്ഥിതികളുപയോഗിച്ച് പ്രതിഫലിപ്പിക്കാനാവുകയില്ല തന്നെ. വെളിപാടു സ്വീകരണത്തിന്റെ ഏറ്റവും ഉയര്‍ന്നതും സുവ്യക്തവും തീവ്രവുമായ വൈയക്തികാനുഭവങ്ങളാണ് പ്രവാചകശ്രേഷ്ഠനുണ്ടായതെന്ന് സാരം. നബിവിദ്വേഷത്താല്‍ ഉന്മാദം ബാധിച്ച തലച്ചോറുകള്‍ക്ക് പടച്ചവന്‍ പ്രവാചകനു നല്‍കിയ ഈ തുല്യതയില്ലാത്ത ആദരവിനെ തമസ്‌കരിക്കേണ്ടി വരുന്നതിന്റെ രാഷ്ട്രീയം യാതൊരു വിശദീകരണവുമില്ലാതെ തന്നെ വ്യക്തമാണ്.

ഹിറയില്‍വെച്ച് പ്രവാചകനുണ്ടായ അനുഭവങ്ങളെ നാം പരിഗണിക്കുക. പ്രവാചകത്വമോ വെളിപാടുകളോ ദീര്‍ഘകാലത്തേക്ക് ഉണ്ടായിട്ടില്ലാത്ത മക്കയിലെ അറബ് സമൂഹത്തില്‍ ജനിച്ചുവളര്‍ന്ന, വേദപുസ്തകങ്ങളുമായി യാതൊരു പരിചയവുമില്ലാത്ത സാധാരണക്കാരനായ ഒരു മനുഷ്യന്‍ ആള്‍പാര്‍പ്പില്ലാത്ത ഒരു കൂറ്റന്‍ പര്‍വതത്തിന്റെ ചെങ്കുത്തായ ചെരുവിലുള്ള ഹിറ എന്ന ചെറിയ ഗുഹയില്‍ ഒറ്റയ്ക്കിരിക്കുമ്പോള്‍ ഒരു ദിവസം പൊടുന്നനെ അല്ലാഹുവിന്റെ ദിവ്യസന്ദേശവുമായി അതിശക്തനായ ജിബ്‌രീല്‍ എന്ന മലക്ക് പ്രത്യക്ഷപ്പെടുകയും എഴുത്തോ വായനയോ അറിയാത്ത പ്രവാചകനോട് വായിക്കുവാനാജ്ഞാപിക്കുകയും വായിക്കാനറിയില്ലെന്നു പറഞ്ഞപ്പോള്‍ ആരും ഉലഞ്ഞുപോകുംവിധം ചേര്‍ത്തുപിടിച്ചമര്‍ത്തുകയും ഇത് രണ്ടുതവണ കൂടിയാവര്‍ത്തിച്ചശേഷം ഗാംഭീര്യം തുളുമ്പുന്ന അഞ്ച് ദിവ്യവചനങ്ങള്‍ ഓതികേള്‍പിക്കുകയും അപ്രത്യക്ഷനാവുകയും ചെയ്യുന്നു. പ്രവാചകന്‍ ഇത്തരമൊരനുഭവം ജീവിതത്തിലൊരിക്കലും പ്രതീക്ഷിച്ചിട്ടുപോലുമില്ലെന്ന കാര്യം കൂടി നമ്മളോര്‍ക്കണം. പ്രവാചകത്വത്തിന്റെയും വെളിപാടുകളുടെയും രീതിശാസ്ത്രവുമായുള്ള തികഞ്ഞ അപരിചിതത്വം, താന്‍ ഒരു പ്രവാചനായി നിയോഗിക്കപ്പെടുമെന്നോ തന്റെയടുക്കലേക്ക് മലക്ക് വരും എന്നോ ഉള്ള യാതൊരു വിചാരവുമില്ലാത്ത ഒരാളുടെ തയ്യാറെയുപ്പില്ലായ്മ. അതിഭീകരമായ ഏകാന്തത. ആരും ഭയന്നുപോകുംവിധമുള്ള ആശ്ലേഷം. ലോകര്‍ക്കു മുഴുവന്‍ ദിവ്യസന്ദേശമെത്തിക്കാനുള്ള ദൗത്യം ജഗന്നിയന്താവിനാല്‍ ഏല്‍പിക്കപ്പെടുന്നത് സൃഷ്ടിക്കുന്ന മനോവിസ്‌ഫോടനം. ദൈവത്തിന്റെ വചനങ്ങള്‍ അവന്റെ ദൂതനില്‍നിന്ന് നേരിട്ടുകേള്‍ക്കുന്നത് സൃഷ്ടിക്കുന്ന ശാരീരിക വ്യതിയാനങ്ങള്‍. ആരാണ് ഭയന്നുപോകാതിരിക്കുക? ആരാണ് വിറച്ചുപോകാതിരിക്കുക?

വിശുദ്ധ ക്വുര്‍ആനിലെ വചനങ്ങള്‍, അവയുടെ ആശയങ്ങളും പദങ്ങളും ഒരുപോലെ ദൈവത്തില്‍നിന്ന് നിര്‍ഗളിച്ചവയായതിനാല്‍, ശുദ്ധ മനസ്സുള്ള ആരിലും പ്രകമ്പനങ്ങള്‍ സൃഷ്ടിക്കുക തന്നെ ചെയ്യും. ക്വുര്‍ആന്‍ വചനങ്ങളുടെ കേവലമായ ശ്രുതി തന്നെ അതിന്റെ ദൈവികതയെക്കുറിച്ച ബോധമുള്ളവരുടെ മനസ്സില്‍ ആന്ദോളനങ്ങളുളവാക്കാന്‍ പോന്നതാണ്. ആശയങ്ങളറിഞ്ഞു കേള്‍ക്കുന്നവരില്‍ അത് ചെലുത്തുന്ന സ്വാധീനം അതിനേക്കാള്‍ ശക്തമാണ്. മനുഷ്യന്റെ നിസ്സാരതയും ദൈവത്തിന്റെ മഹത്വവും ദൈവതൃപ്തിക്കായുള്ള അധ്വാനങ്ങളില്‍ മനുഷ്യന്‍ വരുത്തുന്ന അലംഭാവത്തിന്റെ ഗൗരവവും ബോധ്യപ്പെടുത്തി ജീവിതത്തെ പൂര്‍ണമായി സംസ്‌കരിക്കുവാന്‍ ശേഷിയുള്ള അതിശക്തമായ ദൈവഭയം കേള്‍വിക്കാരനില്‍ നിറയ്ക്കുകയാണ് ഓരോ ക്വുര്‍ആന്‍ വചനവും ചെയ്യുന്നത്. സത്യവിശ്വാസികളില്‍ ക്വുര്‍ആന്‍ ശ്രവണം ഉണ്ടാക്കുന്ന ഫലത്തെക്കുറിച്ച് ക്വുര്‍ആന്‍ തന്നെ പറയുന്നതിപ്രകരമാണ്: ”അല്ലാഹുവെപ്പറ്റി പറയപ്പെട്ടാല്‍ ഹൃദയങ്ങള്‍ പേടിച്ചുനടുങ്ങുകയും അവന്റെ വചനങ്ങള്‍ വായിച്ചുകേള്‍പിക്കപ്പെട്ടാല്‍ വിശ്വാസം വര്‍ധിക്കുകയും തങ്ങളുടെ രക്ഷിതാവിന്റെ മേല്‍ ഭരമേല്‍പിക്കുകയും ചെയ്യുന്നവര്‍ മാത്രമാണ് സത്യവിശ്വാസികള്‍.” (ക്വുര്‍ആന്‍ 8: 2).

ഈ ഫലം സൃഷ്ടിക്കുന്ന ക്വുര്‍ആന്‍ വചനങ്ങള്‍, ദൈവത്തിന്റെ ഘനഗംഭീരമായ സംസാരം, ആകാശത്തുനിന്ന് ഭൂമിയിലേക്കിറങ്ങിവന്ന അതിശക്തനായ ഒരു മലക്കില്‍നിന്ന് മനുഷ്യരുടെ കൂട്ടത്തില്‍വെച്ച് ആദ്യമായി കേള്‍ക്കുകയാണ് മുഹമ്മദ് നബി (സ). ക്വുര്‍ആന്‍ അവതരണം പ്രവാചകനെ മാനസികമായും ശാരീരികമായും വിറപ്പിച്ച അനുഭവമായിത്തീരാതിരിക്കുക പിന്നെയെങ്ങനെയാണ്? ക്വുര്‍ആന്‍ അവതരണപ്രക്രിയയുടെ ഭാരത്തെക്കുറിച്ച് സുന്ദരമായ ഒരുപമയിലൂടെ ക്വുര്‍ആന്‍ തന്നെ വര്‍ണിക്കുന്നുണ്ട്: ”ഈ ക്വുര്‍ആനിനെ നാം (അല്ലാഹു) ഒരു പര്‍വതത്തിനുമുകളില്‍ അവതരിപ്പിച്ചിരുന്നെങ്കില്‍ അത് (പര്‍വതം) വിനീതമാകുന്നതും അല്ലാഹുവെപ്പറ്റിയുള്ള ഭയത്താല്‍ പൊട്ടിപ്പിളരുന്നതും നിനക്ക് കാണാമായിരുന്നു. ആ ഉദാഹരണങ്ങള്‍ നാം ജനങ്ങള്‍ക്കുേവണ്ടി വിവരിക്കുന്നു. അവര്‍ ചിന്തിക്കുവാന്‍വേണ്ടി.” (59: 21) വഹ്‌യ് പ്രവാചകനില്‍ സൃഷ്ടിച്ച വിഹ്വലത, തികച്ചും സ്വാഭാവികമായിരുന്നുവെന്നര്‍ത്ഥം. വഹ്‌യ് ലഭിച്ച സ്ഥലവും രീതിയും മുതല്‍ അതിന്റെ ഉള്ളടക്കവും ധ്വനികളും വരെ ആ വിഹ്വലതയില്‍ പ്രതിഫലിക്കുന്നുണ്ട്; ഇത്ര വലിയ പ്രകമ്പനങ്ങള്‍ സൃഷ്ടിക്കുന്ന ശാരീരിക-മാനസികാനുഭവങ്ങള്‍ സമ്മാനിക്കുന്ന ദൈവികബോധനപ്രക്രിയയെ താങ്ങാനുള്ള കരുത്ത് തനിക്കുണ്ടോ എന്ന ആവലാതി മുതല്‍ അന്തിമപ്രവാചകന്‍ എന്ന അതിഭയങ്കരമായ ഉത്തരവാദിത്തം ശിരസ്സാവഹിക്കാനുള്ള വലുപ്പം തനിക്കുണ്ടോ എന്ന ഭയപ്പാടുവരെ ആ വിറയലില്‍ പ്രതിധ്വനിക്കുന്നുണ്ട്.

പ്രവാചകന്‍ ഒരു മനുഷ്യനാണ് എന്ന വസ്തുത മാത്രമാണ് ഹിറാ സംഭവം തെളിയിക്കുന്നത്; നിഷ്‌കളങ്കനും ആത്മാര്‍ത്ഥതയുള്ളവനുമായ മനുഷ്യന്‍. പ്രവാചകത്വം ഏല്‍പിക്കുവാന്‍വേണ്ടി അതേ സാഹചര്യത്തിലുള്ള ഏതു മനുഷ്യനെ അതേ അനുഭവങ്ങളിലൂടെ കൊണ്ടുപോയാലും ഇതേ കാര്യങ്ങള്‍ തന്നെയാണ് സംഭവിക്കുക. മുഹമ്മദ് നബി (സ) മലക്കാണെന്നോ ദൈവമാണെന്നോ അല്ല മുസ്‌ലിംകള്‍ വിശ്വസിക്കുന്നത്; മനുഷ്യനാണെന്നു തന്നെയാണ്. ആ മനുഷ്യന് ദിവ്യബോധനങ്ങള്‍ ലഭിച്ചു എന്നാണ് മുസ്‌ലിംകള്‍ പറയുന്നത്. അതിനെ ഖണ്ഡിക്കുവാനുതകുന്ന യാതൊന്നും പ്രവാചകന്റെ ഹിറാ അനുഭവങ്ങളിലില്ല തന്നെ. ക്വുര്‍ആന്‍ പറയട്ടെ: ”( നബിയേ,) പറയുക: ഞാന്‍ നിങ്ങളെപ്പോലെയുള്ള ഒരു മനുഷ്യന്‍ മാത്രമാകുന്നു. നിങ്ങളുടെ ദൈവം ഏകദൈവം മാത്രമാണെന്ന് എനിക്ക് ബോധനം നല്‍കപ്പെടുന്നു. അതിനാല്‍ വല്ലവനും തന്റെ രക്ഷിതാവുമായി കണ്ടുമുട്ടണമെന്ന് ആഗ്രഹിക്കുന്നുവെങ്കില്‍ അവന്‍ സല്‍കര്‍മ്മം പ്രവര്‍ത്തിക്കുകയും, തന്റെ രക്ഷിതാവിനുള്ള ആരാധനയില്‍ യാതൊന്നിനെയും പങ്കുചേര്‍ക്കാതിരിക്കുകയും ചെയ്തുകൊള്ളട്ടെ.” (18: 110)

ജിബ്‌രീലിന്റെ ആഗമനം പ്രവാചകനില്‍ നിറച്ച ഭയവും അത് സൃഷ്ടിച്ച ശാരീരിക പ്രതിഫലനങ്ങളും ബൈബിള്‍ വിവരിക്കുന്ന പ്രവാചകാനുഭവങ്ങളുമായി ചേര്‍ന്നുപോകുന്നതല്ല എന്ന മിഷനറി വാദത്തിന്റെ (അങ്ങനെ ചേര്‍ന്നുപോകുക ഒരാളുടെ പ്രവാചകത്വം തെളിയിക്കുവാന്‍ ഒരു നിലയ്ക്കും ആവശ്യമല്ലെന്ന് നാം സൂചിപ്പിച്ചുകഴിഞ്ഞു) വസ്തുനിഷ്ഠതയാണ് ഇനി പരിശോധിക്കപ്പെടേണ്ടത്. ജിബ്‌രീല്‍ (ഗബ്രിയേല്‍) മലക്കുകളില്‍ ശക്തികൊണ്ടും ഗാംഭീര്യംകൊണ്ടും വേറിട്ടുനില്‍ക്കുന്നയാളാണെന്ന സങ്കല്‍പം ബൈബിള്‍ പ്രദാനം ചെയ്യുന്നുണ്ട്. പ്രശസ്തമായ ബൈബിള്‍ ഓണ്‍ലൈന്‍ വിജ്ഞാനകോശം www.newadvent.org പറയട്ടെ: ”ഗബ്രിയേല്‍, പേര് സൂചിപ്പിക്കുന്നതുപോലെ ദൈവത്തിന്റെ ശക്തിയുടെ ദൂതനാണ്. ഗബ്രിയേലിനെ പരാമര്‍ശിക്കുന്ന ബൈബിള്‍ വചനങ്ങളില്‍ മഹത്വത്തെയും ശക്തിയെയും അധികാരത്തെയും ബലത്തെയുമെല്ലാം സൂചിപ്പിക്കുന്ന പദങ്ങള്‍ നിരന്തരമായി ആവര്‍ത്തിക്കുന്നത് ശ്രദ്ധേയമാണ്… ജൂതന്‍മാര്‍ ഗബ്രിയേലിന്റെ ഈ വിശേഷണങ്ങളെ ആഴത്തില്‍ ഉള്‍ക്കൊണ്ടിട്ടുള്ളതുപോലെയാണ് തോന്നുന്നത്… സൊദോമിന്റെ നാശം ഗബ്രിയേലിന്റെ കൈകളിലൂടെയാണുണ്ടായതെന്ന് അവര്‍ മനസ്സിലാക്കുന്നു.” അതിശക്തനും ഗംഭീരഭാവമുള്ളയാളും ഒരു പ്രദേശത്തെയൊന്നടങ്കം നശിപ്പിക്കുവാന്‍ ദൈവം നിയോഗിച്ച ബലവാനുമെല്ലാമായാണ് ബൈബിള്‍ പഴയനിയമം ജിബ്‌രീലിനെ അവതരിപ്പിക്കുന്നതെന്ന് സാരം. ഹിറാ ഗുഹയില്‍ പ്രവാചകനുമുന്നില്‍ ജിബ്‌രീല്‍ പ്രത്യക്ഷപ്പെട്ടതിനെത്തുടര്‍ന്നുണ്ടായ സംഭവങ്ങളെ ന്യായീകരിക്കുക മാത്രമല്ലേ ഈ ബൈബിള്‍ വര്‍ണനകള്‍ ചെയ്യുന്നത്?

ഇനി ജിബ്‌രീല്‍ പ്രവാചകന്‍മാര്‍ക്ക് പ്രത്യക്ഷപ്പെട്ടതിനെ സംബന്ധിച്ച ബൈബിള്‍ വിവരണങ്ങളിലേക്കു വരാം. ഗബ്രിയേല്‍ ദൂതനുമായി ആശയവിനിമയം നടത്തിയ ഏറ്റവും ശ്രദ്ധേയനായ പഴയനിയമ കഥാപാത്രം ദാനിയേല്‍ ആണ്. എന്തായിരുന്നു ദാനിയേലിന്റെ അനുഭവം? ഭാവിയെക്കുറിച്ചുള്ള ഭീഷണമായ സുചനകളുള്‍ക്കൊള്ളുന്നുവെന്ന് തോന്നിപ്പിക്കുന്ന വിചിത്രദൃശ്യങ്ങളുള്ള ഒരു ദര്‍ശനം ദാനിയേലിനുണ്ടാകുന്നു. ദര്‍ശനത്തിന്റെ ആഘാതത്തില്‍ നില്‍ക്കവെ ഗബ്രിയേല്‍ മനുഷ്യരൂപത്തില്‍ ദാനിയേലിനു പ്രത്യക്ഷപ്പെട്ടതിനെക്കുറിച്ച് ബൈബിള്‍ പറയട്ടെ: ”ദാനിയേലായ ഞാന്‍ ഈ ദര്‍ശനം ഗ്രഹിക്കാന്‍ ശ്രമിച്ചുകൊണ്ടിരിക്കെ, ഇതാ എന്റെ മുമ്പില്‍ മനുഷ്യരൂപമുള്ള ഒരുവന്‍ നില്‍ക്കുന്നു. ഉലായ് തീരങ്ങളില്‍നിന്ന് ഒരുവന്‍ വിളിച്ചുപറയുന്നത് ഞാന്‍ കേട്ടു; ഗബ്രിയേല്‍, ദര്‍ശനം ഇവനെ ഗ്രഹിപ്പിക്കുക. ഞാന്‍ നിന്നിടത്തേക്ക് അവന്‍ (ഗബ്രിയേല്‍) വന്നു. അവന്‍ വന്നപ്പോള്‍ ഞാന്‍ ഭയവിഹ്വലനായി സാഷ്ടാംഗം വീണു… അവന്‍ എന്നോട് സംസാരിച്ചുകൊണ്ടിരിക്കുമ്പോള്‍ ഞാന്‍ മൂര്‍ഛിച്ചുവീണു. എന്നാല്‍ അവന്‍ എന്നെ തൊട്ട് എഴുന്നേല്‍പിച്ചു നിര്‍ത്തി.” (ദാനിയേല്‍ 8: 15-18)

മറ്റൊരിക്കല്‍ ഗബ്രിയേല്‍ ടൈഗ്രീസ് തീരത്തുവെച്ച് ദാനിയേലിനു പ്രത്യക്ഷപ്പെട്ടതിനെക്കുറിച്ച് പത്താം അധ്യായത്തിലുണ്ട്. അതിപ്രാകാരം: ”ഞാന്‍ കണ്ണുയര്‍ത്തി നോക്കിയപ്പോള്‍ ചണവസ്ത്രവും ഊഫാസിലെ സ്വര്‍ണം കൊണ്ടുള്ള അരപ്പട്ടയും ധരിച്ച ഒരുവനെ കണ്ടു. അവന്റെ ശരീരം ഗോമേദകം പോലെയും മുഖം മിന്നല്‍ പോലെയും കണ്ണുകള്‍ ജ്വലിക്കുന്ന ഒരു പന്തം പോലെയും ആയിരുന്നു. അവന്റെ കൈകാലുകള്‍ മിനുക്കിയ ഓടിന്റെ ഭംഗിയുള്ളവയും സ്വരം ജനക്കൂട്ടത്തിന്റെ ഇരമ്പല്‍ പോലെയും ആയിരുന്നു. ദാനിയേലായ ഞാന്‍ മാത്രം ഈ ദര്‍ശനം കണ്ടു; എന്നോടൊപ്പമുണ്ടായിരുന്നവര്‍ അതു കണ്ടില്ല. മഹാഭീതി പിടിപെട്ട് അവര്‍ ഓടിയൊളിച്ചു. അങ്ങനെ തനിച്ചായ ഞാന്‍ ഈ മഹാദര്‍ശനം കണ്ടു; എന്റെ ശക്തി ചോര്‍ന്നുപോയി. എന്റെ മുഖം തിരിച്ചറിയാന്‍ വയ്യാത്തവിധം മാറിപ്പോയി. എന്റെ ശക്തിയറ്റു. അപ്പോള്‍ ഞാന്‍ അവന്റെ സ്വരം കേട്ടു, അവന്റെ സ്വരം ശ്രവിച്ച ഞാന്‍ പ്രജ്ഞയറ്റ് നിലം പതിച്ചു. എന്നാല്‍, ഒരു കരം എന്നെ സ്പര്‍ശിച്ചു. അവന്‍ എന്നെ എഴുന്നേല്‍പിച്ചു. വിറയലോടെയാണെങ്കിലും മുട്ടും കയ്യും ഊന്നി ഞാന്‍ നിന്നു. അവന്‍ എന്നോട് പറഞ്ഞു: ഏറ്റവും പ്രിയങ്കരനായ ദാനിയേലേ, എഴുന്നേല്‍ക്കുക. ഞാന്‍ നിന്നോട് പറയുന്ന വാക്കുകള്‍ ശ്രദ്ധിച്ചുകേള്‍ക്കുക. എന്നെ നിന്റെയടുത്തേക്ക് അയച്ചിരിക്കുകയാണ്. അവന്‍ ഇതുപറഞ്ഞപ്പോള്‍ ഞാന്‍ വിറയലോടെ നിവര്‍ന്നുനിന്നു.” (ദാനിയേല്‍ 10: 5 – 11)

ഹിറയിലുണ്ടായ വെളിപാടിന്റെ ദൈവികതയെ നിഷേധിക്കാന്‍ ബൈബിളുപയോഗിച്ച് കഴിയില്ലെന്നും പ്രവാചകാനുഭവങ്ങളെ സാധൂകരിക്കുക മാത്രമാണ് ബൈബിള്‍ ഈ വിഷയത്തില്‍ ചെയ്യുന്നത് എന്നുമുള്ള കേവല വസ്തുതകളാണ് ഇവിടെ അനാവൃതമാകുന്നത്. ഗബ്രിയേലുമായുള്ള മനുഷ്യമുഖാമുഖത്തെക്കുറിച്ച് ബൈബിള്‍ പുതിയ നിയമവും ഇതേദിശയിലുള്ള സൂചനകള്‍ നല്‍കുന്നുണ്ട്. യോഹന്നാന്‍ സ്‌നാപകന്റെ പിതാവ് സെഖര്യാവിന്റെ അനുഭവങ്ങള്‍ ശ്രദ്ധിക്കുക: ”അപ്പോള്‍, കര്‍ത്താവിന്റെ ദൂതന്‍ ധൂപപീഠത്തിന്റെ വലതുവശത്ത് നില്‍ക്കുന്നതായി അവന് പ്രത്യക്ഷപ്പെട്ടു. അവനെക്കണ്ട് സഖറിയാ അസ്വസ്ഥനാവുകയും ഭയപ്പെടുകയും ചെയ്തു.” (ലൂക്കോസ് 1: 11, 12). പ്രവാചകന്‍ (സ) ഭയവിഹ്വലനായത് ചൂണ്ടിക്കാണിച്ച് അദ്ദേഹത്തിനുണ്ടായത് വഹ്‌യല്ലെന്നും വന്നത് മലക്കല്ലെന്നുമെല്ലാം സമര്‍ത്ഥിക്കുവാന്‍ പാടുപെടുന്ന മിഷനറിമാര്‍ തങ്ങളുടെ സ്വന്തം ബൈബിളിന്റെയടക്കം കഴുത്തിലാണ് ഈ അഭ്യാസപ്രകടനങ്ങള്‍ക്കിടയില്‍ കത്തിവെക്കുന്നതെന്നര്‍ത്ഥം.

ബൈബിള്‍ ശരിയായ രീതിയില്‍ വായിച്ചവര്‍ക്കൊന്നും മുഹമ്മദ് നബി(സ)യ്ക്ക് ഹിറാ ഗുഹയില്‍ വെച്ചുണ്ടായ വെളിപാടിനെ സാക്ഷീകരിക്കുവാനല്ലാതെ നിരാകരിക്കാന്‍ കഴിയില്ലെന്ന വസ്തുതയാണ് ഹദീഥിന്റെ അവസാന ഭാഗത്തുള്ള വറക്വത്ബ്‌നു നൗഫലിന്റെ വാചകങ്ങള്‍ തെളിയിക്കുന്നത്. ജൂത-ക്രിസ്തു ദര്‍ശനങ്ങളോട് ആഭിമുഖ്യവും അവരുടെ ഗ്രന്ഥങ്ങളില്‍ അഗാധപരിജ്ഞാനവുമുണ്ടായിരുന്ന വറക്വ, നബി(സ)യുടെ വെളിപാടനുഭവങ്ങള്‍ കേട്ടപ്പോള്‍ അത് ജിബ്‌രീല്‍ തന്നെയാണെന്ന് സാക്ഷ്യപ്പെടുത്തുകയും നബി(സ)ക്ക് പിന്തുണ പ്രഖ്യാപിക്കുകയുമാണ് ചെയ്തതെന്ന യാഥാര്‍ത്ഥ്യം മിഷനറിമാരുടെ കണ്ണുതുറപ്പിക്കേണ്ടതുണ്ട്. അതെ, ഖദീജ (റ) ആണയിട്ടു പറഞ്ഞതുപോലെ ബന്ധുക്കള്‍ക്കും ദുര്‍ബലര്‍ക്കും ദരിദ്രര്‍ക്കും അശരണര്‍ക്കും അതിഥികള്‍ക്കും പ്രയാസപ്പെടുന്നവര്‍ക്കും തണല്‍മരമായി നിന്നിരുന്ന മുഹമ്മദ് (സ) എന്ന നന്മകളുടെ ഉടല്‍ രൂപത്തെ പ്രവാചകനായി നിയോഗിക്കുവാന്‍ ജിബ്‌രീല്‍ എന്ന മലക്കു തന്നെയാണ് ഒന്നര സഹസ്രാബ്ദത്തോളം മുമ്പ് ഹിറാ ഗുഹയില്‍ പ്രത്യക്ഷപ്പെട്ടത്; മുഹമ്മദ് നബി (സ) ജിബ്‌രീലിനോട് സ്വയം സാക്ഷ്യപ്പെടുത്തിയതുപോലെ എഴുത്തോ വായനയോ അറിയാതിരുന്ന, വേദപുസ്തക പരിജ്ഞാനം അശേഷമില്ലാതിരുന്ന ആ സാധാരണക്കാരന്റെ നാവില്‍ നിന്ന് ലോകം വിസ്മയത്തോടുകൂടി ക്വുര്‍ആന്‍ കേട്ടത് ജിബ്‌രീല്‍ അദ്ദേഹത്തിനത് പഠിപ്പിച്ചുകൊടുത്തതുകൊണ്ടാണ്. ഹിറയില്‍ നിന്ന് പ്രസരിച്ച വെളിച്ചത്തിന് ബൈബിള്‍ മറയാകുമെന്ന് കരുതുന്നവര്‍ വിഡ്ഢികളുടെ സ്വര്‍ഗത്തിലാണെന്നു മാത്രമാണ് നമുക്ക് പറയാനുള്ളത്.

ഒരേ സമയം ഒൻപത് ഭാര്യമാരുണ്ടായിരുന്ന നബിയുടെ (സ) ദാമ്പത്യജീവിതം മാതൃകാപരമാണെന്ന് പറയുന്നതിൽ എന്ത് അർത്ഥമാണുള്ളത്?

ജീവിതത്തിലെവിടെയും കളങ്കത്തിന്റെ ലാഞ്ചനകളൊന്നുമില്ലാതെ ജീവിക്കുകയും തന്റെ വിശുദ്ധജീവിതം വഴി അവസാനനാളുവരെ ലോകത്തിന് മാതൃകയാക്കാന്‍പറ്റുന്ന പതിനായിരങ്ങളെ സൃഷ്ടിക്കുകയും ചെയ്ത നബി(സ)യുടെ ദാമ്പത്യജീവിതം അവസാന നാളുവരെയുള്ള മനുഷ്യർക്കെല്ലാം സംതൃപ്തദാമ്പത്യത്തിനുള്ള മാതൃകയാനിന്ന കാര്യത്തിൽ ആ ജീവിതത്തെ മുൻധാരണയില്ലാതെ പഠനവിധേയമാക്കിയവർക്കൊന്നും സംശയമുണ്ടാവില്ല.

നബിയുടെ (സ) ബഹുഭാര്യാത്വം നബിനിന്ദകരുടെ വിഷയമാകുന്നതും അതിനെ വൃത്തികെട്ട രൂപത്തിൽ ചിത്രീകരിക്കുന്നതും തങ്ങള്‍ രതിയിലേര്‍പ്പെടുന്നവരുടെ പൂര്‍ണമായ ഉത്തരവാദിത്വമേറ്റെടുക്കാന്‍ പുരുഷന് ബാധ്യതയുണ്ടെന്ന അദ്ദേഹത്തിന്റെ നിഷ്‌കൃഷ്ടമായ അനുശാസന നിന്ദകർക്കും അവരുടെ ഭൗതികാസ്വാദനത്തിന്റെ പ്രത്യയശാസ്ത്രത്തിനും അരോചകമാകുന്നതുകൊണ്ടാണ്. പെണ്ണിന്റെ ഉത്തരവാദിത്വങ്ങളൊന്നും ഏറ്റെടുക്കാതെ അവളില്‍നിന്ന് ലൈംഗികസുഖം നുണയാവുന്ന സാമൂഹ്യസാഹചര്യങ്ങള്‍ക്കാവശ്യമായ നിയമങ്ങള്‍ നിര്‍മിക്കുന്ന മുതലാളിത്ത ആണ്‍കോയ്മയുടെ കണ്ണട ധരിച്ചവര്‍ക്ക് പ്രവാചകന്റെ (സ) വിശുദ്ധജീവിതത്തിന്റെ അര്‍ത്ഥപൂര്‍ണിമ അറിയാന്‍ കഴിയില്ല. വിവാഹവും വ്യഭിചാരവും തമ്മിലുള്ള വ്യത്യാസമെന്താണെന്നുപോലും അവര്‍ക്ക് മനസ്സിലാവില്ല. മുഹമ്മദ് നബി(സ)ക്ക് ഒരേസമയം ഒന്‍പ്ത ഭാര്യമാരുണ്ടായിരുന്നുവെന്ന് നിന്ദാസ്വരത്തില്‍ പറയുന്നവര്‍ കരുതുന്നത് നിരവധി സ്ത്രീകളുമായി രമിക്കുകയും അവര്‍ക്കുള്ളത് അപ്പോള്‍തന്നെ കൊടുത്തുവിടുകയും ചെയ്യുന്ന വൃത്തികെട്ട സ്വതന്ത്രരതീ സംസ്‌കാരത്തിന് തുല്യമാണ് പ്രവാചകന്‍ (സ) തന്റെ ബഹുഭാര്യത്വത്തിലൂടെ പഠിപ്പിച്ച സംസ്‌കാരവുമെന്നാണ്. ഇണകളുടെയെല്ലാം പൂര്‍ണമായ ഉത്തരവാദിത്തമേറ്റെടുക്കുകയും അവര്‍ക്കെല്ലാം സംതൃപ്തമായ ദാമ്പത്യജീവിതം നല്‍കുകയും ചെയ്ത പ്രവാചകന്‍ ലോകത്തിനുപഠിപ്പിച്ച ധാര്‍മികതയെവിടെ? കൂടെ കിടന്നവളില്‍ മാംസം മാത്രം കാണുകയും അവളുടെ മാംസത്തിന് വില പറഞ്ഞുറപ്പിച്ച് അവളില്‍ സുഖമനുഭവിച്ച് അവളെ വലിച്ചെറിയുന്ന ആണ്‍കോയ്മാ ലൈംഗിക സംസ്‌കാരത്തിന്റെ ധാര്‍മികതയെവിടെ?

ഇണകളില്‍ നിന്നേ ലൈംഗികതയാസ്വദിക്കുവാന്‍ പാടുള്ളുവെന്ന് കരുതുന്നവര്‍ക്ക് മാത്രമാണ് നബി(സ)യുടെ ജീവിതത്തില്‍ മാതൃകയുള്ളതെന്നാണ് സ്വതന്ത്രരതിയുടെ ഭൂമികയില്‍ നിന്നുകൊണ്ട് നബി(സ)യെ തെറി പറയുകയും അദ്ദേഹത്തില്‍ മാതൃകയില്ലെന്ന് കരയുകയും ചെയ്യുന്നവരോട് ഒന്നാമതായി പറയാനുള്ളത്. ലൈംഗികസുഖം അനുഭവിക്കുകയും സ്വന്തം ഇണക്ക് അതിനുള്ള അവസരമുണ്ടാക്കുകയും ചെയ്യുന്നത് പുണ്യമാണെന്നും അതിന് പടച്ചവന്‍ പ്രതിഫലം നല്‍കുമെന്നും പഠിപ്പിച്ച ആത്മീയാചാര്യനാണ് മുഹമ്മദ് നബി(സ). സമ്പൂര്‍ണജീവിതത്തിന്റെ സവിശേഷമാതൃകയാണ് പ്രവാചകവ്യക്തിത്വത്തിലുടനീളം നമുക്ക് കാണാനാവുക.

ഏകഭാര്യത്വവും ബഹുഭാര്യത്വവും സ്വീകരിക്കുവാന്‍ സ്വാതന്ത്ര്യവും അവകാശവുമുള്ള മുസ്‌ലിംകള്‍ക്ക് ഈ രണ്ട് രംഗങ്ങളിലുമുള്ള പ്രവാചകമാതൃകയില്‍ നിന്ന് ഏറെ പഠിക്കുവാനുണ്ട്. തന്റെ ഇരുപത്തിയഞ്ചാമത്തെ വയസ്സില്‍ തന്നേക്കാള്‍ പ്രായക്കൂടുതലുള്ള ഒരു വിധവയെ വിവാഹം ചെയ്യുകയും നീണ്ട ഇരുപത്തിയഞ്ച് വര്‍ഷക്കാലം ആ ദാമ്പത്യം തുടരുകയും മക്കളുണ്ടാവുകയും പേരക്കുട്ടികളുണ്ടാവുകയുമെല്ലാം ചെയ്തിട്ടും ദാമ്പത്യജീവിതത്തിന്റെ മധുരിമ നിലനിര്‍ത്തിക്കൊണ്ട് തന്റെ ഇണയുടെ മരണം വരെ അവരോടൊപ്പം ജീവിക്കുകയും ചെയ്ത മുഹമ്മദ് (സ)-ഖദീജ (റ) ദാമ്പത്യത്തില്‍ ഏകഭാര്യത്വം സ്വീകരിക്കുന്ന മുസ്‌ലിംകള്‍ക്ക് കൃത്യമായ മാതൃകയുണ്ട്.

വൈധവ്യവും അനാഥത്വവും പേറേണ്ടി വരുന്ന പെണ്ണവസ്ഥകളില്‍ അവര്‍ക്ക് താങ്ങും തണലുമായിത്തീരുവാനും അതോടൊപ്പം ദാമ്പത്യജീവിതം ആസ്വദിക്കുവാനും അങ്ങനെ ബഹുഭാര്യത്വം ഉപയോഗിക്കാനാകുമെന്നും, എല്ലാ ഇണകള്‍ക്കും സംതൃപ്തമായ ദാമ്പത്യജീവിതം പ്രദാനം ചെയ്തുകൊണ്ട് മുസ്‌ലിം പുരുഷന് എങ്ങനെ നീതിമാനായ ഒരു ബഹുഭാര്യനായിത്തീരാമെന്നും പ്രവാചകന്റെ (സ) ബഹുഭാര്യത്വം നമുക്ക് മാതൃക കണിച്ചുതരുന്നുണ്ട്. ലൈംഗികസുഖമാസ്വദിക്കുന്നതിനുവേണ്ടി മാത്രമുള്ളതാണ് ഭാര്യ-ഭര്‍തൃ ജീവിതമെന്നു കരുതുന്നവര്‍ക്ക് പ്രവാചകന്റെ ഏകഭാര്യത്വത്തിലോ ബഹുഭാര്യത്വത്തിലോ മാതൃക കണ്ടെത്താനാവുകയില്ല. ഒന്നാണെങ്കിലും ഒന്നിലധികമാണെങ്കിലും ഇണകളുടെയെല്ലാം ഉത്തരവാദിത്തമേറ്റെടുക്കുകയും അവരുടെ അബലതകളില്‍ താങ്ങും തണലുമായി നില്‍ക്കുകയുമാണ് മുസ്‌ലിം പുരുഷന്റെ ഉത്തരവാദിത്തമെന്ന് പഠിപ്പിക്കപ്പെട്ടവര്‍ക്കു പ്രവാചകനില്‍ (സ) സമ്പൂര്‍ണമായ മാതൃക കാണാന്‍ കഴിയുകയും ചെയ്യും. പ്രവാചകജീവിതത്തിനല്ല നബിനിന്ദകരുടെ ഭൂമികയ്ക്കാണ് കുഴപ്പമെന്ന് സാരം.

ദുരിതമനുഭവിക്കുന്ന സഹോദരിമാരുടെ കരച്ചില്‍ കേള്‍ക്കാതെ അവരെ തെരുവിന് നല്‍കുന്നതാണ് ആധുനികതയെന്ന് കരുതുന്നവര്‍ പ്രയാസങ്ങളനുഭവിക്കുന്നവര്‍ക്ക് സ്വന്തം ജീവിതത്തിന്റെ തണലേകി സംരക്ഷിക്കണമെന്ന് സ്വന്തം മാതൃകയിലൂടെ കാണിച്ചുകൊടുത്ത അന്തിമപ്രവാചകനില്‍ (സ) നിന്ന് ഏറെ പഠിക്കേണ്ടതുണ്ട്. തന്റെ ആദ്യഭാര്യയായിരുന്ന ഖദീജ(റ)യുടെ വിയോഗാനന്തരമുള്ള ഓരോ പ്രവാചകവിവാഹങ്ങളിലും ദാമ്പത്യജീവിതമെന്നതിലുപരിയായ വലിയ ലക്ഷ്യങ്ങള്‍ കാണാന്‍ കഴിയും. വിധവയും മാതാവുമായ സൗദ ബിന്‍ത് സംഅയെന്ന അറുപതുകാരിയാണ് പ്രവാചകന്റെ (സ) ബഹുഭാര്യത്വജീവിതത്തിലേക്ക് ആദ്യമായി കടന്നുവന്ന സഖി. ഭര്‍തൃവിയോഗത്തിനുശേഷം അനാഥത്വത്തിന്റെ കയ്പുനീര്‍ കുടിച്ചുകൊണ്ടിരുന്ന, ആദ്യകാലത്ത് ഇസ്‌ലാം സ്വീകരിച്ച് ധൈര്യം കാണിച്ച ആ മാതൃകാവനിതയെ അവരുടെ വാര്‍ദ്ധക്യത്തിന്റെ തുടക്കത്തില്‍ തന്റെ സഖിയായി സ്വീകരിച്ച് മാതൃകയാവുകയായിരുന്നു നബി(സ). അബുസല്‍മയുടെ വിയോഗത്തിനുശേഷം പിഞ്ചുപൈതങ്ങളോടെ അവരുടെ മാതാവായ ഉമ്മുസല്‍മയെ ഏറ്റെടുക്കുകയും അനാഥത്വത്തില്‍നിന്ന് അവരെയും മക്കളെയും കരകയറ്റുകയും ചെയ്ത പ്രവാചകനില്‍ (സ) പറക്കമുറ്റാത്ത കുഞ്ഞുങ്ങളുണ്ടായിരിക്കെ മരണപ്പെടുന്നവരുടെ മക്കളെ സംരക്ഷിക്കേണ്ടത് എങ്ങനെയെന്ന വലിയ മാതൃകയുണ്ട്. തന്റെ സന്തതസഹചാരിയായിരുന്ന ഉമറി(റ)ന്റെ മകള്‍ ഹഫ്‌സ(റ)യെ അവരുടെ ഭര്‍ത്താവായ ഖുനൈസുബ്‌നു ഹുദൈഫ മരണപ്പെട്ടപ്പോള്‍ ഏറ്റെടുത്ത പ്രവാചകനില്‍ (സ) അനുയായികളുടെ സങ്കടങ്ങള്‍ക്ക് പ്രായോഗികപരിഹാരം നിര്‍ദ്ദേശിക്കുകയും അവരുടെ ബാധ്യതകളെ സ്വന്തം ഉത്തരവാദിത്വമായി ഏറ്റെടുക്കുകയും ചെയ്യുന്ന നേതാവിന്റെ മാതൃകയുണ്ട്.

ബദ്‌റില്‍ വെച്ച് മരണപ്പെട്ട ഭര്‍ത്താവിന്റെ വിയോഗത്താല്‍ സങ്കടമനുഭവിക്കുന്ന സൈനബ് ബിന്‍ത് ഖുസൈമയെ അവരുടെ തന്നെ ആവശ്യപ്രകാരം വിവാഹം ചെയ്തുകൊണ്ട് യുദ്ധത്തില്‍ മരണപ്പെട്ട മറ്റുള്ളവരുടെ വിധവകളെ ഏറ്റെടുക്കാന്‍ അനുചരന്‍മാരെ പ്രേരിപ്പിക്കുകയായിരുന്നു നബി(സ). ഇസ്‌ലാം സ്വീകരിച്ച ആദ്യപുരുഷനും തന്റെ സഖാക്കളില്‍ പ്രഥമഗണനീയനുമായ അബുബക്കറി(റ)ന്റെ മകള്‍ ആയിശ(റ)യുമായുള്ള വിവാഹത്തില്‍ കുടുംബബന്ധങ്ങള്‍ സൃഷ്ടിച്ചുകൊണ്ട് സൗഹൃദത്തെ ദൃഢീകരിക്കുന്നതെങ്ങനെയെന്ന മാതൃകയുണ്ട്. പ്രവാചകന്‍ (സ) സ്വീകരിച്ച ഒരേയൊരു കന്യകയായ ആയിശ (റ) ദാമ്പത്യബന്ധത്തില്‍ സ്വീകരിച്ചുപോന്ന പ്രവാചകമാതൃകകളുള്‍ക്കൊള്ളുന്ന നിരവധി ഹദീഥുകള്‍ നിവേദനം ചെയ്തുകൊണ്ട് പ്രസ്തുത വിവാഹത്തിലൂടെയുള്ള സാമൂഹികലക്ഷ്യം നിറവേറ്റി മാതൃകയാവുകയും ചെയ്തു. അബ്‌സീനിയായിലേക്ക് പാലായനം ചെയ്ത് തന്നോടൊപ്പം താങ്ങായി നിന്നിരുന്ന ഭര്‍ത്താവ് മതം മാറി മദ്യപനായിത്തീരുകയും മക്കയിലുള്ള പിതാവും സഹോദരങ്ങളും ഇസ്‌ലാമിനോടുള്ള ശത്രുത പരസ്യമായി പ്രഖ്യാപിക്കുകയും ചെയ്ത സാഹചര്യത്തില്‍ തികച്ചും അനാഥയും അഗതിയുമായിത്തീര്‍ന്ന ഉമ്മു ഹബീബ(റ)യെ മദീനാരാഷ്ട്രത്തിന്റെ തലവനായ പ്രവാചകന്‍ (സ) സ്വീകരിച്ചുകൊണ്ട് അശരണര്‍ക്ക് താങ്ങായിത്തീരുകയെന്ന ദൗത്യനിര്‍വഹണത്തിന് വിവാഹബന്ധത്തെ എങ്ങനെ ഉപയോഗിക്കാമെന്ന് പഠിപ്പിക്കുകയാണ് ചെയ്തത്.

പുതിയ സൗഹൃദങ്ങളിലൂടെ വ്യത്യസ്ത ഗോത്രങ്ങളുടെ ശാത്രവം കുറച്ചുകൊണ്ടുവരികയെന്ന ലക്ഷ്യമാണ് മൈമുന(റ)യുമായുള്ള വിവാഹത്തില്‍ നമുക്ക് കാണാനാവുക. യുദ്ധത്തില്‍ പിടിക്കപ്പെടുന്ന വനിതകളെ യുദ്ധത്തടവുകാരാക്കി അടിമകളാക്കുന്ന, അന്ന് നിലനിന്നിരുന്ന സമ്പ്രദായത്തില്‍ നിന്നുമാറി, അവരെ ഇണകളാക്കാനാവുന്നവര്‍ അങ്ങനെ ചെയ്യുകയാണ് വേണ്ടതെന്ന പാഠമാണ് ജുവൈരിയ(റ)യുമായും സ്വഫിയ്യ(റ)യുമായുള്ള വിവാഹങ്ങളിലൂടെ നബി (സ) പഠിപ്പിച്ചത്. ബനൂ മുസ്തലഖ് യുദ്ധത്തില്‍ പിടിക്കപ്പെട്ട ഗോത്രനേതാവിന്റെ മകളായ ജുവൈരിയ(റ)യെ സ്വതന്ത്രമാക്കി  വിവാഹം ചെയ്യുകവഴി അവരില്‍ നിന്ന് പിടിക്കപ്പെട്ടവരെയെല്ലാം സ്വതന്ത്രമാക്കുവാന്‍ മുസ്‌ലിം പടയാളികള്‍ സന്നദ്ധമാവുകയും അതുവഴി ആ ഗോത്രം മുഴുവന്‍ ഇസ്‌ലാം സ്വീകരിക്കുകയും ചെയ്തുവന്നതാണ് ചരിത്രം.

മാതാക്കളായും മക്കളായും തങ്ങള്‍ക്കിഷ്ടപ്പെട്ടവരെ വിളിക്കുകയും അവരുമായി ആ രൂപത്തിലുള്ള ബന്ധം പ്രഖ്യാപിക്കുകയും ചെയ്തിരുന്ന അജ്ഞാതകാലത്തെ സമ്പ്രദായത്തിന് അറുതി വരുത്തുകയെന്ന വലിയ ലക്ഷ്യത്തിനുവേണ്ടി അല്ലാഹുവിന്റെ നിര്‍ദ്ദേശപ്രകാരം നടന്നതാണ് സൈനബ് ബിന്‍ത് ജഹ്ശു (റ)മായുള്ള പ്രവാചകവിവാഹം.

ജീവിതത്തിന്റെ എല്ലാ രംഗങ്ങളിലുമെന്നതുപോലെ ഓരോ പ്രവാചകവിവാഹങ്ങളിലും വലിയ മാതൃകകളുണ്ടെന്ന വസ്തുതയാണ് ആ ജീവിതത്തെ വസ്തുനിഷ്ഠമായി പഠിച്ചവർക്കെല്ലാം മനസ്സിലാവുക.

വിഷയവുമായി ബന്ധപ്പെട്ട വീഡിയോ

ക്രൈസ്തവ വിമര്‍ശകരാണ് മുഹമ്മദ് നബിക്ക് ലഭിച്ച വെളിപാടുകള്‍ പിശാചില്‍നിന്നാണെന്ന ആരോപണം ഉന്നയിക്കുന്നത്. മുഹമ്മദി(സ)ന് ലഭിച്ച വെളിപാടുകള്‍ പിശാചുബാധയുടെ ഫലമായുണ്ടായതാണെന്ന് വരുത്തിത്തീര്‍ക്കാനാണ് സി.ഡി. ഫാണ്ടര്‍, ക്ലേയ്ര്‍ ടിസ്ഡാല്‍, ജോഷ്മാക്ഡവല്‍, ജോണ്‍ജില്‍ ക്രിസ്റ്റ്, ജി. നെഹ്ല്‍സ് തുടങ്ങിയ ക്രൈസ്തവ ഗ്രന്ഥകാരന്മാരെല്ലാം ശ്രമിച്ചിരിക്കുന്നത്. യേശുക്രിസ്തുവിന്റെ ക്രൂശീകര ണത്തെയും അതുമൂലമുള്ള പാപപരിഹാരത്തെയും നിഷേധിച്ചു കൊണ്ട് മനുഷ്യരാശിയെ പാപത്തിന്റെ ഗര്‍ത്തത്തില്‍തന്നെ തളച്ചിടുവാനുള്ള പി ശാചിന്റെ പരിശ്രമമാണ് ഖുര്‍ആനിന്റെ രചനക്കു പിന്നിലുള്ളതെന്ന് അവര്‍ വാദിക്കുന്നു. മനുഷ്യശരീരത്തില്‍ പിശാച് കയറിക്കൂടുമോ? പിശാചുബാധ കൊണ്ട് ഒരാള്‍ക്ക് രോഗങ്ങളുണ്ടാവുമോ? പിശാചുബാധിച്ച ഒരാള്‍ക്ക് വെളിപാടുണ്ടാവുമോ? തുടങ്ങിയ ചര്‍ച്ചകള്‍ ഇവിടെ അപ്രസക്തമാണ്. ബൈ ബിള്‍ പ്രകാരം പിശാചുബാധിച്ച ഒരാളില്‍ കാണപ്പെടുന്ന അസുഖങ്ങള്‍ എന്തെല്ലാമാണെന്ന് പരിശോധിക്കുക.

1. ബുദ്ധിഭ്രമത്താല്‍ അലറി വിളിക്കല്‍ (മാര്‍ക്കോസ് 1:24, ലൂക്കോസ് 9:39, യോഹന്നാന്‍ 10:20)

2. സ്വയം നശീകരണ പ്രവണത (മത്തായി 55:9, 18: 17, 15:32, മര്‍ക്കോസ് 5: 13, ലൂക്കോസ്, 8:33)

3. നഗ്‌നമായി നടക്കുന്നതിനുള്ള പ്രവണത (ലൂക്കോസ് 8:2, 8:35)

4. പിശാചിനാല്‍ തള്ളയിടപ്പെടുക (മത്തായി 17:15, മര്‍ക്കോസ് 1:26,9:18, 9:20,9:26)

5. മൂകത (മര്‍ക്കോസ് 9:25, 9:32, 12:22, ലൂക്കോസ് 11:14)

6. ബധിരത (മര്‍ക്കോസ് 9: 25)

7. അന്ധത (മത്തായി 12:22)

8. മറ്റാരും കാണാത്തത് കാണുകയും അറിയുകയും ചെയ്യുക (മര്‍ക്കോസ് 1:24, ലൂക്കോസ് 4:3, മത്തായി 8:29)

പിശാചുബാധിതനില്‍ കാണപ്പെടുന്നതെന്ന് ബൈബിള്‍ ഉദ്‌ഘോഷിക്കു ന്ന ലക്ഷണങ്ങളൊന്നും മുഹമ്മദി(സ)ല്‍ ഉണ്ടായിരുന്നതായി നമുക്ക് കാ ണാന്‍ കഴിയുന്നില്ല. ദൈവിക വെളിപാടുകള്‍ ലഭിക്കുമ്പോള്‍ അവ ഒരു മണിനാദം പോലെ തനിക്ക് അനുഭവപ്പെടാറുണ്ടെന്നും അതാണ് ഏറ്റവും പ്രയാസകരമായ വെളിപാടു രീതിയെന്നും മുഹമ്മദ്(സ) പറഞ്ഞതാണ് അദ്ദേഹത്തെ പിശാചുബാധിച്ചിരുന്നുവെന്നും പൈശാചിക വെളിപാടുകളാണ് ഖുര്‍ആനെന്നും വാദിക്കുന്നവരുടെ ഒരു തെളിവ്. വെളിപാട് ലഭിച്ചു കൊ ണ്ടിരുന്ന അതിശൈത്യമുള്ള ഒരു ദിവസം പ്രവാചകന്റെ നെറ്റിയില്‍ വിയര്‍പ്പുതുള്ളിയുണ്ടായിരുന്നതായി ഞാന്‍ കണ്ടുവെന്ന പ്രവാചകപത്‌നി ആഇശ(റ) യുടെ നിവേദനമാണ് മറ്റൊരു തെളിവ്. ഇവിടെ പ്രസക്തമായ ഒരു ചോദ്യമുണ്ട്. പിശാചുബാധിതന് ചെവിയില്‍ മണിയടിക്കുന്നതുപോലെ തോന്നുമെന്നോ അവന്റെ നെറ്റിത്തടം അതിശൈത്യമാണെങ്കിലും വിയര്‍പ്പുതുള്ളികളാല്‍ നിറയുമെന്നോ ബൈബിളില്‍ എവിടെയെങ്കിലുമുണ്ടോ? ഇല്ലെങ്കില്‍, പ്രവാചക(സ)നില്‍ പിശാചുബാധ ആരോപിക്കുവാന്‍ ബൈബിളിന്റെ അനുയായികള്‍ക്ക് എന്തടിസ്ഥാനമാണുള്ളത്?

പ്രവാചകന് ലഭിച്ച ദൈവിക സന്ദേശങ്ങള്‍ പിശാചുബാധയുടെ ഉല്‍പ ന്നങ്ങളാണെന്ന് പറയുന്നവര്‍ തങ്ങളുടെതന്നെ വിശുദ്ധന്മാരാണ് പിശാചു ബാധയേറ്റവരെന്ന് പറയാന്‍ നിര്‍ബന്ധിതരാവുമെന്നതാണ് വാസ്തവം.

യേശുവിന്റെ ജീവിതകാലമത്രയും അദ്ദേഹത്തെയും അദ്ദേഹം പഠിപ്പിച്ച ആശയങ്ങളെയും നശിപ്പിക്കുവാന്‍ വേണ്ടി അഹോരാത്രം പരിശ്രമിക്കുക യും (അപ്പോസ്തല പ്രവൃത്തികള്‍ 9:1, 26:10, 8:1) അദ്ദേഹത്തിനുശേഷം ക്രിസ്തു തനിക്ക് വെളിപ്പെട്ടിട്ടുണ്ടെന്ന് അവകാശവാദമുന്നയിക്കുകയും ചെയ്തയാളാണ് 'വിശുദ്ധ പൗലോസ്'. അദ്ദേഹത്തിന് ക്രിസ്തുദര്‍ശനം ലഭിച്ച രീതിയെക്കുറിച്ച് ബൈബിള്‍ വിവരിക്കുന്നത് കാണുക: ''പിന്നെ അയാള്‍ യാത്ര പുറപ്പെട്ട് ഡമാസ്‌കസിനെ സമീപിച്ചപ്പോള്‍, പെട്ടെന്ന് ആകാശത്തുനിന്ന് ഒരു പ്രകാശം അയാളുടെ ചുറ്റും മിന്നലൊളി പരത്തി. സാവൂള്‍ നിലം പതിച്ചു. 'സാവൂള്‍, സാവൂള്‍ നീ എന്നെ പീഡിപ്പിക്കുന്നത് എന്തിന്? എന്ന് തന്നോട് ചോദിക്കുന്ന ഒരു സ്വരം കേള്‍ക്കയായി. അപ്പോള്‍ അയാള്‍ ചോദിച്ചു: 'പ്രഭോ നീ ആരാണ്? അവന്‍ പറഞ്ഞു: നീ പീഡിപ്പിക്കുന്ന യേശുവാണ് ഞാന്‍. എഴുന്നേറ്റ് നഗരത്തില്‍ ചെല്ലുക. നീ ചെയ്യേണ്ടത് എന്തെന്ന് അവിടെ വെച്ച് നിനക്ക് അറിവ് കിട്ടും'. 'അയാളോടൊപ്പം യാത്ര ചെയ്തിരുന്ന ആളുകള്‍ സ്വരം കേട്ടെങ്കിലും ആരെയും കാണായ്കയാല്‍ വിസ്മയ സ്തബ്ധരായി നിന്നുപോയി. വീണുകിടന്നിടത്തുനിന്ന് സാവൂള്‍ എഴുന്നേറ്റു. കണ്ണുതുറന്നിട്ടും അയാള്‍ക്ക് ഒന്നും കാണാന്‍ കഴിഞ്ഞില്ല. അതിനാല്‍ അവര്‍ അയാളെ കൈക്കുപിടിച്ച് ഡമാസ്‌കസിലേക്കു കൊണ്ടുപോയി. മൂന്നു ദിവസത്തേക്ക് അയാള്‍ക്ക് കാഴ്ചയില്ലായിരുന്നു; അയാള്‍ തിന്നുകയോ കുടിക്കുകയോ ചെയ്തതുമില്ല'' (അപ്പോസ്തല പ്രവൃത്തികള്‍ 9:3-9)

നിലംപതിക്കുന്നതും കൂടെയുള്ളവര്‍ കാണാത്തത് കാണുന്നതും കേള്‍ ക്കാത്തത് കേള്‍ക്കുന്നതും കണ്ണു കാണാതാവുന്നതുമെല്ലാം പിശാചുബാധ യുടെ ലക്ഷണങ്ങളായി സുവിശേഷങ്ങളില്‍ സൂചിപ്പിക്കപ്പെട്ടിട്ടുള്ളത് നാം കണ്ടു. ക്രിസ്തുവിനെ താന്‍ കണ്ടുവെന്ന് പൗലോസ് അവകാശപ്പെട്ട സംഭവത്തില്‍ ഇതെല്ലാം അദ്ദേഹം അനുഭവിക്കുന്നുമുണ്ട്. പൗലോസിന് പിശാചുബാധയാണ് ഉണ്ടായതെന്ന് വാദിച്ചാല്‍ അത് അംഗീകരിക്കാന്‍ ക്രൈസ്തവ സമൂഹം സന്നദ്ധമാവുമോ? മുഹമ്മദി(സ)ന് പിശാചുബാധയായിരുന്നുവെന്ന് സമര്‍ഥിക്കുവാന്‍ ബൈബിളില്‍നിന്ന് ഒരു തെളിവെങ്കിലുമുദ്ധരിക്കാ ന്‍ ക്രൈസ്തവ വിമര്‍ശകര്‍ക്ക് കഴിയില്ല. അതേസമയം, നിലവിലുള്ള ക്രിസ്തുമതത്തിന്റെ സ്ഥാപകനായ പൗലോസിന് പിശാചുബാധയാണ് അനുഭ വപ്പെട്ടതെന്ന് ബൈബിള്‍ ഉപയോഗിച്ചു കൊണ്ട് സ്ഥാപിക്കാന്‍ ഒരാള്‍ക്ക് കഴിയും. അപ്പോള്‍ ആര്‍ക്കാണ് പിശാചുബാധ? ഇനി, മുഹമ്മദ് നബി(സ)ക്ക് പിശാച് ബാധിച്ചതുകൊണ്ടാണ് ഖുര്‍ആന്‍ എഴുതിയുണ്ടാക്കിയതെന്ന ക്രൈസ്തവവാദത്തിന്റെ ആണിക്കല്ല് പരിശോധിക്കുക. യേശുക്രിസ്തുവിന്റെ കുരിശുമരണത്തിലൂടെയുള്ള പാപപരിഹാരം എന്ന ആശയത്തെ വിമര്‍ശിക്കുന്നതുമൂലമാണല്ലോ ഖുര്‍ആന്‍ പിശാചിന്റെ സൃഷ്ടിയാണെന്ന് വാദിക്കുന്നത്.

എന്നാല്‍, യാഥാര്‍ഥ്യമെന്താണ്? യേശുക്രിസ്തു പരിശുദ്ധനായിരുന്നുവെന്ന് മുസ്‌ലിംകളും ക്രൈസ്തവരും വിശ്വസിക്കുന്നു. അദ്ദേഹം സര്‍വശക് തനാല്‍ നിയുക്തനായ വ്യക്തിയാണെന്ന് ഇരുകൂട്ടരും സമ്മതിക്കുന്നു. അദ്ദേഹത്തിന് പിശാചുബാധയുണ്ടായിട്ടില്ലെന്ന് ഇരുകക്ഷികളും പറയുന്നു. എങ്കില്‍, മുഹമ്മദ് നബി(സ)ക്കോ പൗലോസിനോ ആര്‍ക്കാണ് പിശാചില്‍നിന്ന് വെളിപാടുണ്ടായതെന്ന് പരിശോധിക്കാന്‍ നമുക്കെന്തുകൊണ്ട് യേശുക്രിസ്തുവിന്റെ ഉപദേശങ്ങളുമായി അവരുടെ ഉപദേശങ്ങളെ താരതമ്യം ചെയ്തുകൂടാ? പിശാചില്‍നിന്ന് വെളിപാടുണ്ടായ വ്യക്തി യേശുവിന്റെ ശത്രുവായിരിക്കുമല്ലോ. ഒരു ദൈവദൂതന്റെ ശത്രു അയാള്‍ പ്രബോധനം ചെയ്യുന്ന ആശയങ്ങളുടെ ശത്രുവായിരിക്കും എന്നോര്‍ക്കുക.

യേശു പറഞ്ഞു: നിയമത്തെ (തോറ)യോ പ്രവാചകന്മാരെയോ റദ്ദാക്കാനല്ല ഞാന്‍ വന്നത്' (മത്തായി 5:17). ഖുര്‍ആന്‍ പറയുന്നു: 'തീര്‍ച്ചയായും നാം തന്നെയാണ് തൗറാത്ത് അവ തരിപ്പിച്ചിരിക്കുന്നത്, അതില്‍ മാര്‍ഗദര്‍ശനവും പ്രകാശവുമുണ്ട്' (5:44).

'മര്‍യമിന്റെ മകന്‍ ഈസ പറഞ്ഞ സന്ദര്‍ഭം: ഇസ്രായേല്‍ സന്തതികളേ, എനിക്കുമുമ്പുള്ള തൗറാത്തിനെ സത്യപ്പെടുത്തുന്നവനായിക്കൊണ്ടും എനിക്ക് ശേഷം വരുന്ന അഹ്മദ് എന്നു പേരുള്ളൊരു ദൂതനെപ്പറ്റി സന്തോ ഷവാര്‍ത്ത അറിയിക്കുന്നവനായിക്കൊണ്ടും നിങ്ങളിലേക്ക് അല്ലാഹുവിന്റെ ദൂതനായി നിയോഗിക്കപ്പെട്ടവനാകുന്നു ഞാന്‍' (61:6).

പൗലോസ് എഴുതി: 'നിയമാനുഷ്ഠാനങ്ങളെ (തോറ) ആശ്രയിക്കുന്നവ രെല്ലാം ശാപഗ്രസ്തരാണ് (ഗലാത്തിയക്കാര്‍ 3:10).

'ക്രിസ്തു നിയമത്തിന്റെ ശാപത്തില്‍നിന്നു നമ്മെ മോചിപ്പിച്ചിരിക്കു ന്നു' (ഗലാത്തിയക്കാര്‍ 3:13). 'അവന്‍ (യേശു) തന്റെ ശരീരത്തില്‍, നിയമത്തെ അതിന്റെ കല്‍പന കളോടും അനുശാസനങ്ങളോടുംകൂടി റദ്ദാക്കി' (എഫേസോസുകാര്‍2:15)

ഞാന്‍ നിയമത്തെ റദ്ദാക്കാനല്ല വന്നതെന്ന് യേശു, ഖുര്‍ആനും അതുതന്നെ പറയുന്നു. പൗലോസാകട്ടെ യേശു നിയമത്തില്‍നിന്ന് ലോകത്തെ രക്ഷിക്കാനാണ് വന്നത് എന്നു സമര്‍ഥിക്കുന്നു. ആര്‍ക്കാണ് പിശാചിന്റെ വെളിപാട്?

യേശുക്രിസ്തു താന്‍ ദൈവമാണെന്ന് പഠിപ്പിച്ചില്ല (മര്‍ക്കോസ് 12:29, മത്തായി 4:10) ഇക്കാര്യം ഖുര്‍ആന്‍ അര്‍ഥശങ്കക്കിടയില്ലാത്തവണ്ണം വ്യക്ത മാക്കുന്നു (3:51), എന്നാല്‍ പൗലോസ് പറഞ്ഞതാകട്ടെ 'പ്രകൃത്യാതന്നെ ദൈവമായിരുന്നിട്ടും ദൈവത്തോടു തനിക്കുള്ള തുല്യതയെ, മുറുകെപ്പിടിച്ചുകൊണ്ടിരിക്കേണ്ട ഒരു കാര്യമായി അവന്‍ പരിഗണിച്ചില്ല? (ഫിലിപ്പിയര്‍ 2:6). 'അവന്‍ അദൃശ്യനായ ദൈവത്തിന്റെ പ്രതിരൂപമാണ്; സര്‍വസൃഷ്ടികളിലും ആദ്യജാതന്‍' (കൊളോസിയക്കാര്‍ 1:15) എന്നിങ്ങനെയാണ്. യേശുക്രിസ്തുവിന് സ്വയം താന്‍ ദൈവമാണെന്ന വെളിപാട് ലഭിച്ചിട്ടില്ല. അങ്ങനെ ലഭിച്ചിരുന്നുവെങ്കില്‍ അദ്ദേഹം അത് പറയുമായിരുന്നു. എന്നാല്‍, പൗലോസിന് യേശു ദൈവമായിരുന്നുവെന്ന് വെളിപാട് കിട്ടി. പ്രസ്തുത വെളിപാട് എവിടെനിന്നായിരിക്കണം?

അബ്രഹാമിനോട് ദൈവം ചെയ്ത ഉടമ്പടിയായിട്ടാണ് പരിച്‌ഛേദനാകര്‍മത്തെ ബൈബിള്‍ പരിചയപ്പെടുത്തുന്നത്. 'നീയും നിനക്കു ശേഷം തലമുറയായി നിന്റെ സന്തതികളും പാലിക്കേണ്ട ഉടമ്പടി'യെന്നു പറഞ്ഞുകൊണ്ടാണ് അബ്രഹാമിനോട് കര്‍ത്താവ് പരിച്‌ഛേദന ചെയ്യുന്നതിനുള്ള കല്‍പന നല്‍കുന്നത് (ഉല്‍പത്തി 17:9-14) കര്‍ത്താവ് മോശയോടു പറഞ്ഞ തായി ബൈബിള്‍ ഉദ്ധരിക്കുന്നു: 'എട്ടാം ദിവസം ശിശുവിന്റെ പരിച്‌ഛേ ദനം നടത്തണം (ലേവിയര്‍ 12:3) ഈ ദൈവിക കല്‍പന യേശുവും അനുസ രിച്ചിരുന്നു. 'എട്ടു ദിവസം പൂര്‍ത്തിയായപ്പോള്‍ ശിശുവിന് പരിച്‌ഛേദനം നടത്തി' (ലൂക്കോസ് 2:21). പരിച്‌ഛേദനം ചെയ്യേണ്ടതില്ലെന്ന് യേശു ആരോടും പറഞ്ഞില്ല. കാരണം അദ്ദേഹത്തിന് അത്തരത്തിലുള്ള ഒരു ബോധനം ലഭിച്ചിരുന്നില്ല. എന്നാല്‍ പൗലോസ് പറയുന്നത് കാണുക: 'പരിച്‌ഛേദനം സ്വീകരിക്കുന്നുവെങ്കില്‍ നിങ്ങള്‍ക്ക് ക്രിസ്തുവിനെക്കൊണ്ട് നേട്ടമില്ല' (ഗലാത്തിയക്കാര്‍ 5:2). ഈ വെളിപാട് പൗലോസിന് എവിടെനിന്ന് കിട്ടി? ദൈവത്തില്‍ നിന്നാകാന്‍ വഴിയില്ല. പിന്നെയോ?

പിശാചില്‍നിന്നാണ് മുഹമ്മദി(ല)ന് വെളിപാടുണ്ടായത് എന്നുപറയാ നുള്ള പ്രധാനപ്പെട്ട കാരണം കുരിശുമരണത്തെയും പാപപരിഹാരബലി യെയും ഖുര്‍ആന്‍ നിഷേധിക്കുന്നുവെന്നതാണല്ലോ. യേശുവിനെയും മാതാവിനെയും പുകഴ്ത്തുകയും ആദരിക്കുകയും ചെയ്യുന്ന ഒട്ടനവധി സൂക്ത ങ്ങള്‍ ഖുര്‍ആനിലുണ്ട്. ഖുര്‍ആനില്‍ പേരു പരാമര്‍ശിക്കപ്പെട്ട ഏക വനിത മര്‍യമാണെന്നോര്‍ക്കുക. യേശു ചെയ്തതായി ബൈബിളില്‍ പറയാത്ത കളിമണ്‍പക്ഷികളില്‍ ഊതി അവയ്ക്ക് ജീവനിടുക തുടങ്ങിയ അത്ഭുതങ്ങ ളെക്കുറിച്ച് ഖുര്‍ആന്‍ പ്രതിപാദിക്കുന്നുമുണ്ട് (3:49). തൊട്ടിലില്‍ വെച്ച് ഉണ്ണിയേശു സംസാരിച്ചതായുള്ള ഖുര്‍ആനിക പരാമര്‍ശം (19:30) ബൈബി ളിലൊരിടത്തും കാണുവാന്‍ സാധ്യമല്ല. യേശുവിന്റെ വിശുദ്ധ വ്യക്തിത്വത്തില്‍ കളങ്കമുണ്ടാക്കുന്ന യാതൊന്നും ഖുര്‍ആനിലില്ല. യോഹന്നാന്റെ സുവിശേഷ പ്രകാരം ക്രിസ്തുവിന്റെ ആദ്യത്തെ അത്ഭുതം കാനായിലെ കല്യാണവിരുന്നില്‍ വെച്ച് മദ്യം നിര്‍മിച്ചു നല്‍കിയതാണെന്ന കാര്യം പ്രത്യേകം പ്രസ്താവ്യമാണ് (യോഹന്നാന്‍ 2:1-11). ഖുര്‍ആനില്‍ ഇത്തരം യാതൊരു പരാമര്‍ശവുമില്ല.

'മരത്തില്‍ തൂക്കിക്കൊല്ലപ്പെടുന്നവന്‍ ദൈവത്താല്‍ ശപിക്കപ്പെട്ടവനാണ്' (ആവര്‍ത്തനം 21:23)എന്നാണ് ബൈബിളിന്റെ സിദ്ധാന്തം. കുരിശില്‍ തറക്കുക വഴി യേശുവിനെ ശപിക്കപ്പെട്ടവനായി മുദ്രയടിക്കുകയാണ് തങ്ങ ള്‍ ചെയ്തതെന്നാണ് യഹൂദര്‍ കരുതിയത്. പൗലോസ് പറയുന്നതും മറ്റൊന്നല്ല. 'മരത്തില്‍ തൂക്കപ്പെടുന്നവരെല്ലാം ശപിക്കപ്പെട്ടവര്‍ എന്ന് എഴുതിയിരിക്കുന്നതുപോലെ ക്രിസ്തു നമുക്കുവേണ്ടി ശാപമായിത്തീരുന്നു' (ഗലാത്യര്‍ 3:13). അപ്പോള്‍ ക്രൂശീകരണം യേശുവിനെ ശപിക്കപ്പെട്ടവനാക്കുകയാണ് ചെയ്യുന്നത്. ലോകത്തിനുവേണ്ടി യേശു ശാപമായിത്തീര്‍ന്നുവെന്ന വാദം ഖുര്‍ആന്‍ അംഗീകരിക്കുന്നില്ല. ശാപത്തിന്റെ മരക്കുരിശില്‍നിന്ന് തന്നെ രക്ഷിക്കേണമേയെന്ന ക്രിസ്തുവിന്റെ പ്രാര്‍ഥന (മത്തായി 26:39) ദൈവം കേട്ടില്ലെന്നു കരുതുന്നത് ദൈവിക കാരുണ്യത്തിന്റെ നിഷേധമല്ലാതെ മറ്റെന്താണ്? ശപിക്കപ്പെട്ട മരക്കുരിശില്‍നിന്ന് പടച്ചതമ്പുരാന്‍ യേശുവിനെ രക്ഷിച്ചുകൊണ്ട് യഹൂദന്മാരുടെ ഗൂഢാലോചനയെ തകര്‍ക്കുകയാണ് ചെയ്തത് എന്നാണ് ഖുര്‍ആന്‍ പഠിപ്പിക്കുന്നത് (4:157,158).

മരക്കുരിശില്‍ ക്രൂശിക്കുക വഴി യേശുവിനെ ശപിക്കപ്പെട്ടവനാക്കിയെന്ന് യഹൂദന്മാര്‍. മരക്കുരിശില്‍ മരിച്ച് യേശു ശപിക്കപ്പെട്ടവനായിത്തീര്‍ന്നുവെന്ന് പൗ ലോസ്. മരക്കുരിശില്‍നിന്ന് പരിശുദ്ധനായ യേശുവിനെ ദൈവം രക്ഷിച്ചുവെന്ന് ഖുര്‍ആന്‍.

ഏതാണ് പിശാചിന്റെ വെളിപാട്? യേശുവിനെ മഹത്വപ്പെടുത്തുന്നതോ അതല്ല ശാപഗ്രസ്തനാക്കുന്നതോ? ചുരുക്കത്തില്‍, ഖുര്‍ആന്‍ പൈശാചിക വെളിപാടാണെന്ന് സമര്‍ഥിക്കു വാന്‍ വേണ്ടി തെളിവുകള്‍ പരതുന്നവര്‍ കുഴിക്കുന്ന കുഴികളില്‍ തങ്ങള്‍തന്നെയാണ് വീഴുന്നത് എന്നുള്ളതാണ് യാഥാര്‍ഥ്യം.

യുക്തിവാദികളായ വിമര്‍ശകന്മാര്‍ പ്രധാനമായും ഉന്നയിക്കുന്ന ആരോ പണമാണ് മുഹമ്മദ്(സ) നബിക്ക് ഉന്മാദരോഗ (Schizophrenia) മായിരുന്നുവെന്നത്. ദൈവത്തിന്റെ അസ്തിത്വം അംഗീകരിക്കാത്തവരെ സംബന്ധിച്ചിടത്തോളം അവര്‍ക്ക് വെളിപാടുകളുടെ സത്യതയെക്കുറിച്ച് എത്രതന്നെ പറഞ്ഞാലും ഉള്‍ക്കൊള്ളാന്‍ കഴിയില്ല. അതുകൊണ്ടുതന്നെ നിരീശ്വരവാദികളോടുള്ള ചര്‍ച്ച തുടങ്ങേണ്ടത് ദൈവാസ്തിത്വത്തെക്കുറിച്ച് സംസാരിച്ചുകൊണ്ടാണ്. പടച്ചതമ്പുരാന്റെ അസ്തിത്വംതന്നെ അംഗീകരിക്കാത്തവരെ അവനില്‍നിന്നുള്ള വെളിപാടുകള്‍ സത്യസന്ധമാണെന്ന് സമ്മതിപ്പിക്കുന്ന തെങ്ങനെ?

ചോദ്യത്തിന്റെ രണ്ടാം ഭാഗമാണ് ആദ്യമായി ചര്‍ച്ച ചെയ്യപ്പെടേണ്ടത്. സമകാലികരാല്‍ മുഹമ്മദ് (സ) ഭ്രാന്തനെന്ന് വിളിക്കപ്പെട്ടിരുന്നുവോ? ഉണ്ടെ ങ്കില്‍ ഭ്രാന്തിന്റെ എന്തെല്ലാം ലക്ഷണങ്ങളുടെ അടിസ്ഥാനത്തിലാണ് അവര്‍ ഈ ആരോപണം ഉന്നയിച്ചത്?

നാല്‍പതു വയസ്സുവരെ സത്യസന്ധനും സര്‍വരാലും അംഗീകരിക്ക പ്പെട്ട വ്യക്തിത്വത്തിന്റെ ഉടമയുമായിരുന്നു മുഹമ്മദ് (സ) . സുദീര്‍ഘമായ ഈ കാലഘട്ടത്തിനിടയ്ക്ക് ആരെങ്കിലും അദ്ദേഹത്തില്‍ ഏതെങ്കിലും തരത്തിലുള്ള മാനസിക വിഭ്രാന്തി ആരോപിച്ചിട്ടില്ല.

പ്രവാചകത്വത്തിനുശേഷം അദ്ദേഹം ഭ്രാന്തനെന്ന് ആരോപിക്കപ്പെട്ടിരുന്നുവെന്നത് ശരിയാണ്. ഭ്രാന്ത നെന്ന് മാത്രമല്ല മുഹമ്മദ്(സ) അധിക്ഷേപിക്കപ്പെട്ടത്; ജ്യോല്‍സ്യന്‍, മാരണ ക്കാരന്‍, മാരണം ബാധിച്ചവന്‍, കവി എന്നിങ്ങനെയുള്ള അധിക്ഷേപങ്ങളെ ല്ലാം അദ്ദേഹത്തിനുനേരെ ഉന്നയിക്കപ്പെട്ടിരുന്നു. അദ്ദേഹത്തിന്റെ വ്യക്തിത്വത്തിലോ മാനസിക സംതുലനത്തിലോ വല്ല വ്യത്യാസവും പ്രകടമായതുകൊണ്ടാണോ അവര്‍ അങ്ങനെ അധിക്ഷേപിച്ചത്? ആണെന്ന് അവരാരുംതന്നെ വാദിച്ചിട്ടില്ല. അവരുടെ പ്രശ്‌നം ഖുര്‍ആനും അതുള്‍ക്കൊള്ളുന്ന ആശയങ്ങളുമായിരുന്നു. തങ്ങളുടെ പാരമ്പര്യ വിശ്വാസങ്ങള്‍ക്കെതിരെയാണ് മുഹമ്മദ്(സ) സംസാരിക്കുന്നത്. അദ്ദേഹം ദൈവികമാണെന്ന് പറഞ്ഞുകൊണ്ട് ഓതിക്കേള്‍പ്പിക്കുന്ന ഖുര്‍ആനിലേക്ക് ജനങ്ങള്‍ ആകൃഷ്ടരാവുക യും ചെയ്യുന്നു. മുഹമ്മദി(സ)നെ സ്വഭാവഹത്യ നടത്താതെ ജനങ്ങളെ അദ്ദേഹത്തില്‍നിന്ന് അകറ്റാന്‍ മറ്റു മാര്‍ഗങ്ങളൊന്നുമില്ലെന്ന് കണ്ട പാരമ്പ ര്യമതത്തിന്റെ കാവല്‍ക്കാര്‍ ബോധപൂര്‍വം കെട്ടിച്ചമച്ച സ്വഭാവഹത്യയായിരുന്നു ഇവയെല്ലാം.

മുഹമ്മദ്(സ) പ്രവാചകത്വം പരസ്യമായി പ്രഖ്യാപിച്ചകാലം. ഹജ്ജ് മാസം ആസന്നമായി. അറേബ്യയുടെ വിവിധ ഭാഗങ്ങളില്‍നിന്നും ഹജ്ജിനു വരുന്നവരോട് മുഹമ്മദ് (സ) മതപ്രബോധനം നടത്തുമെന്നും ഖുര്‍ആനിന്റെ വശ്യതയില്‍ അവര്‍ ആകൃഷ്ടരാവുമെന്നും മക്കയിലെ പ്രമാണിമാര്‍ ഭയന്നു. അവര്‍ യോഗം ചേര്‍ന്നു. ഹജ്ജിന് എത്തിച്ചേരുന്നവരോട് ആദ്യമേതന്നെ മുഹമ്മദി(സ)നെതിരെ പ്രചാരവേലകള്‍ നടത്താന്‍ തീരുമാനിച്ചു. മുഹമ്മദി(സ)നെ എങ്ങനെ വിശേഷിപ്പിക്കണം, എന്നതായി പിന്നീടുള്ള ചര്‍ച്ച. പലരും പല രൂപത്തില്‍ പറയുന്നത് തങ്ങളുടെ വിശ്വാസ്യത തകര്‍ക്കും. എല്ലാവര്‍ക്കും ഒരേ രൂപത്തില്‍ പറയാന്‍ പറ്റുന്ന ആരോപണമെന്ത്? ചിലര്‍ പറഞ്ഞു: ''നമുക്ക് മുഹമ്മദ് ഒരു ജ്യോല്‍സ്യനാണെന്ന് പറയാം''. പൗരപ്രമുഖനായ വലീദുബ്‌നുമുഗീറ പറഞ്ഞു: ''പറ്റില്ല, അല്ലാഹുവാണ് സത്യം അവ ന്‍ ജ്യോല്‍സ്യനല്ല. ജ്യോല്‍സ്യന്മാരെ നാം കണ്ടിട്ടുണ്ട്. മുഹമ്മദിന്റെ വാക്കുകള്‍ ജ്യോല്‍സ്യന്മാരുടെ പ്രവചനങ്ങളല്ല''. മറ്റു ചിലര്‍ പറഞ്ഞു: ''നമുക്ക് അവന്‍ ഭ്രാന്തനാണെന്ന് പറയാം''. വലീദ് പറഞ്ഞു: ''അവന്‍ ഭ്രാന്തനല്ല. ഭ്രാന്തന്മാരെ നാം കണ്ടിട്ടുണ്ട്. അവരുടെ ഭ്രാന്തമായ സംസാരങ്ങളോ ഗോഷ്ഠികളോ പിശാചുബാധയോ ഒന്നും അവനില്ല''. അവര്‍ പറഞ്ഞു: ''എങ്കില്‍ അവന്‍ കവിയാണെന്ന് പറയാം''. വലീദ് പ്രതിവചിച്ചു: ''അവന്‍ കവിയല്ല. കവിതയുടെ എല്ലാ ഇനങ്ങളും നമുക്കറിയാം. അവന്‍ പറയുന്ന ത് കവിതയല്ല''. ജനം പറഞ്ഞു: ''എങ്കില്‍ അവന്‍ മാരണക്കാരനാണെന്ന് പറയാം' വലീദ് പ്രതികരിച്ചു: ''അവന്‍ മാരണക്കാരനുമല്ല. മാരണക്കാരെ നമുക്കറിയാം. അവരുടെ കെട്ടുകളോ, ഊത്തുകളോ ഒന്നും അവന്‍ പ്രയോഗിക്കുന്നില്ല''. അവര്‍ ചോദിച്ചു: ''പിന്നെ എന്താണ് നിങ്ങളുടെ നിര്‍ദേശം?'' അദ്ദേഹം പറഞ്ഞു: ''തീര്‍ച്ചയായും അവന്റെ വചനങ്ങളില്‍ മാധുര്യമുണ്ട്. അതിന്റെ മൂലം വിസ്തൃതവും ശാഖകള്‍ ഫലസമൃദ്ധവുമാണ്. നിങ്ങള്‍ അവനെപ്പറ്റി എന്തു പറഞ്ഞാലും അതു നിരര്‍ഥകമാണെന്നു തെളിയും. പിതാവിനും മക്കള്‍ക്കുമിടയിലും ഭാര്യക്കും ഭര്‍ത്താവിനുമിടയിലും ജ്യേഷ്ഠനും അനുജനുമിടയിലും പിളര്‍പ്പുണ്ടാക്കുവാന്‍ വേണ്ടി വന്ന ജാലവിദ്യക്കാരനാണ് അവനെന്ന് പറയുന്നതാണ് നല്ലത്!'' ജനം ഇതംഗീകരിച്ചു. അവര്‍ പ്രചാര ണം തുടങ്ങി.

ഈ സംഭവം മനസ്സിലാക്കിത്തരുന്ന വസ്തുതയെന്താണ്? പ്രവാചകപ്ര ബോധനങ്ങളില്‍ നിന്ന് ജനങ്ങളെ പിന്തിരിപ്പിക്കുവാന്‍ വേണ്ടി ശത്രുക്കള്‍ മെനഞ്ഞെടുത്ത പലതരം ദുഷ്പ്രചാരണങ്ങളിലൊന്നു മാത്രമാണ് അദ്ദേ ഹം ഭ്രാന്തനാണെന്ന ആരോപണം. ഈ പ്രചാരണം നടത്തിയിരുന്നവര്‍ക്കു തന്നെ അതില്‍ വിശ്വാസമുണ്ടായിരുന്നില്ല. അതുകൊണ്ടുതന്നെ, അവരുടെ പ്രചാരണത്തെ ഒരു തെളിവായി സ്വീകരിക്കുന്നത് അബദ്ധമാണ്.

പ്രവാചകന്‍ ജീവിച്ചത് പതിനാലു നൂറ്റാണ്ടുകള്‍ക്കുമുമ്പാണ്. അദ്ദേഹ ത്തിന് ഉന്മാദരോഗമുണ്ടായിരുന്നുവോയെന്ന് വസ്തുനിഷ്ഠമായി പരിശോധിക്കുവാന്‍ ഇപ്പോള്‍ അദ്ദേഹം നമ്മുടെ മുന്നില്‍ ജീവിച്ചിരിക്കാത്തതിനാല്‍ ഇന്ന് നമുക്ക് കഴിയില്ല. അദ്ദേഹത്തിനുണ്ടായ വെളിപാടുകളും സ്വപ്‌നദര്‍ശ നങ്ങളുമാണ് പ്രവാചകന്‍ ഉന്മാദരോഗിയായിരുന്നുവെന്ന് വാദിക്കുന്നവര്‍ക്കുള്ള തെളിവ്. വെളിപാടുകള്‍ സ്വീകരിക്കുമ്പോള്‍ പ്രവാചകനില്‍ കാണ പ്പെട്ട ഭാവവ്യത്യാസങ്ങളെയും വഹ്‌യ് എങ്ങനെയാണെന്നുള്ള പ്രവാചക ന്റെ വിവരങ്ങളെയും വിശദീകരിക്കുന്ന ഹദീഥുകളുടെ വെളിച്ചത്തിലാണ് വിമര്‍ശകന്മാര്‍ ഈ വാദമുന്നയിക്കുന്നത്. ഉന്മാദരോഗത്തിന്റെ ലക്ഷണ ങ്ങള്‍ പ്രവാചകനില്‍ കാണപ്പെട്ടിരുന്നുവോയെന്ന് വസ്തുനിഷ്ഠമായി പരി ശോധിച്ചാല്‍ ഈ വാദത്തില്‍ യാതൊരു കഴമ്പുമില്ലെന്ന് സുതരാം വ്യക് തമാവും.

ഒന്ന്: ഉന്മാദരോഗികളുടെ സ്വഭാവം നിരന്തരം മാറിക്കൊണ്ടിരിക്കും. മറ്റുള്ളവരോടുള്ള പെരുമാറ്റത്തിലും സംസാരത്തിലുമെല്ലാം ഈ വൈരുധ്യം പ്രകടമായിരിക്കും.

മുഹമ്മദി(സ)ന്റെ ജീവിതവും സംസാരങ്ങളും പരിശോധിക്കുക.

യാതൊരു രീതിയിലുള്ള സ്വഭാവ വൈരുദ്ധ്യങ്ങളും അദ്ദേഹത്തില്‍ നമുക്ക് കാണാന്‍ കഴിയില്ല. മാറിക്കൊണ്ടിരിക്കുന്ന പെരുമാറ്റ രീതികളുടെയും പൂര്‍വാപരബന്ധമില്ലാത്ത സംസാരത്തിന്റെയും ഉടമസ്ഥനായിരുന്നു മുഹമ്മദ് നബി(ല)യെങ്കില്‍ അദ്ദേഹത്തിന് പരശ്ശതം അനുയായികളുണ്ടായതെങ്ങ നെ? സാധാരണയായി നാം മനസ്സിലാക്കുന്ന 'ദിവ്യന്‍'മാരുടെ അനുയായികളെപ്പോലെയായിരുന്നില്ല മുഹമ്മദി(സ)ന്റെ അനുചരന്മാര്‍. അദ്ദേഹത്തിന്റെ ഉപദേശങ്ങള്‍ പ്രാവര്‍ത്തികമാക്കുന്നതിനുവേണ്ടി മല്‍സരിക്കുകയായിരുന്നു അവര്‍. ഒരു ഉന്മാദരോഗിയുടെ വാക്കുകള്‍ അനുസരിക്കുവാന്‍ വേണ്ടി ജനസഹസ്രങ്ങള്‍ മല്‍സരിച്ചുവെന്ന് പറഞ്ഞാല്‍ അത് വിശ്വസിക്കാനാവുമോ?

രണ്ട്: ഉന്മാദരോഗികളുടെ പ്രതികരണങ്ങള്‍ വൈരുധ്യാത്മകമായിരിക്കും. സന്തോഷവേളയില്‍ പൊട്ടിക്കരയുകയും സന്താപവേളയില്‍ പൊട്ടിച്ചിരിക്കുകയും ചെയ്യും. വെറുതെ ചിരിക്കുകയും കരയുകയും ചെയ്യുന്ന സ്വഭാവവും കണ്ടുവരാറുണ്ട്.

മുഹമ്മദ് നബി(സ) യുടെ പ്രതികരണങ്ങള്‍ സമചിത്തതയോടുകൂടിയുള്ളതായിരുന്നു. ഒരു സംഭവം: പ്രവാചകന്‍(സ)ഒരു മരത്തണലില്‍ വിശ്രമിക്കുകയാണ്. പെട്ടെന്ന് ഊരിപ്പിടിച്ച വാളുമായി മുന്നില്‍ ഒരു കാട്ടാളന്‍ പ്രത്യക്ഷപ്പെ ട്ടു. അയാള്‍ ചോദിച്ചു: ''എന്നില്‍നിന്ന് നിന്നെ ഇപ്പോള്‍ ആര് രക്ഷിക്കും?'' പ്രവാചകന്‍ അക്ഷോഭ്യനായി മറുപടി പറഞ്ഞു: 'അല്ലാഹു'. ഈ മറുപടിയു ടെ ദൃഢത കേട്ട് കാട്ടാളന്റെ കൈയില്‍നിന്ന് വാള്‍ വീണുപോയി. (ബുഖാരി, മുസ്‌ലിം)

ഒരു ഉന്മാദരോഗിയില്‍നിന്ന് ദൃഢചിത്തതയോടുകൂടിയുള്ള ഇത്തരം പെരുമാറ്റങ്ങള്‍ പ്രതീക്ഷിക്കുവാന്‍ കഴിയുമോ?

മൂന്ന്: ഉന്മാദരോഗികള്‍ അന്തര്‍മുഖരായിരിക്കും. പുറമെയുള്ള ലോക ത്ത് നടക്കുന്ന സംഭവങ്ങളിലൊന്നും അവര്‍ക്ക് യാതൊരു താല്‍പര്യവും കാണുകയില്ല.

മുഹമ്മദ് നബി(സ)അന്തര്‍മുഖനായിരുന്നില്ല. തന്റെ ചുറ്റുപാടും നടക്കുന്ന സംഭവങ്ങള്‍ അതീവ താല്‍പര്യത്തോടെ നിരീക്ഷിക്കുകയും തന്റെ പങ്ക് ആവശ്യമെങ്കില്‍ നിര്‍വഹിക്കുകയും ചെയ്തിരുന്ന വ്യക്തിയാണദ്ദേ ഹം. ജനങ്ങള്‍ക്ക് ധാര്‍മിക നിര്‍ദേശങ്ങള്‍ നല്‍കുക മാത്രമല്ല, അവര്‍ക്ക് മാതൃകയായി ജീവിച്ച് കാണിച്ചുകൊടുക്കുകകൂടി ചെയ്ത വ്യക്തിയായിരുന്നു അദ്ദേഹം.

ലാമാര്‍ട്ടിന്‍ എഴുതി: 'തത്ത്വജ്ഞാനി, പ്രസംഗകന്‍, ദൈവദൂതന്‍, നിയമ നിര്‍മാതാവ്, പോരാളി, ആശയങ്ങളുടെ ജേതാവ്, അബദ്ധ സങ്കല്‍പങ്ങളില്‍നിന്ന് മുക്തമായ ആചാര വിശേഷങ്ങളുടെയും യുക്തിബന്ധുരമായ വിശ്വാസപ്രമാണങ്ങളുടെയും പുനഃസ്ഥാപകന്‍, ഇരുപത് ഭൗതിക സാമ്രാജ്യങ്ങളുടെ സ്ഥാപകന്‍ -അതായിരുന്നു മുഹമ്മദ്. മനുഷ്യത്വത്തിന്റെ എല്ലാ മാനദണ്ഡങ്ങളും വെച്ച് പരിഗണിക്കുമ്പോള്‍ നാം വ്യക്തമായും ചോദിച്ചേക്കാം. മുഹമ്മദിനേക്കാള്‍ മഹാനായ മറ്റു വല്ല മനുഷ്യനുമുണ്ടോ?''(Historie De La turquie., Vol, 2 Page 277)

അന്തര്‍മുഖനായ ഒരു ഉന്മാദരോഗിയെക്കുറിച്ച വിലയിരുത്തലാണോ ഇത്?

നാല്: ഉന്മാദരോഗികള്‍ക്ക് നിര്‍ണിതമായ എന്തെങ്കിലും ലക്ഷ്യത്തിനുവേണ്ടി വ്യവസ്ഥാപിതമായി പ്രവര്‍ത്തിക്കാന്‍ കഴിയില്ല. കാര്യമായി യാതൊന്നും ചെയ്യാനാവാത്ത ഇവര്‍ ശാരീരികമായും മാനസികമായും തളര്‍ന്നവരായിരിക്കും.

മുഹമ്മദ് നബി(സ) ജനങ്ങളെ സത്യമാര്‍ഗത്തിലേക്ക് നയിക്കുന്നതിനുവേണ്ടി അയക്കപ്പെട്ട ദൈവദൂതന്മാരില്‍ അന്തിമനായിരുന്നു. തന്നിലേല്‍പിക്കപ്പെട്ട ഉത്തരവാദിത്തം രണ്ടു ദശാബ്ദത്തിലധികം ഭംഗിയായി നിര്‍വഹിക്കാന്‍ അദ്ദേഹത്തിന് കഴിഞ്ഞു. ചിട്ടയോടുകൂടിയുള്ള പ്രബോധന പ്രവര്‍ ത്തനങ്ങള്‍ വഴി ജനസഹസ്രങ്ങളെ ദൈവികമതത്തിലേക്ക് ആകര്‍ഷിക്കുവാന്‍ മുഹമ്മദി(സ)ന് സാധിച്ചു. സാംസ്‌കാരിക രംഗത്ത് വട്ടപ്പൂജ്യത്തിലായിരുന്ന ഒരു ജനവിഭാഗത്തെ ലോകത്തിന് മുഴുവന്‍ മാതൃകയാക്കി പരിവര്‍ത്തിപ്പിക്കുവാന്‍ വേണ്ടിവന്നത് കേവലം ഇരുപത്തിമൂന്ന് വര്‍ഷങ്ങള്‍ മാത്രം. ലോകചരിത്രത്തെ ഏറ്റവുമധികം സ്വാധീനിച്ച വ്യക്തിയാണ് മുഹമ്മദ്(സ) എന്ന് ചരിത്രത്തെ നിഷ്പക്ഷമായി നോക്കിക്കണ്ടവരെല്ലാം അഭിപ്രായപ്പെട്ടിട്ടുണ്ട്.

ഇതെല്ലാം ഒരു ഉന്മാദരോഗിക്ക് കഴിയുന്നതാണെന്ന് പ്രസ്തുത രോഗ ത്തെക്കുറിച്ച് അല്‍പമെങ്കിലും അറിയുന്നവരാരെങ്കിലും സമ്മതിക്കുമോ?

അഞ്ച്: ഉന്മാദരോഗി അശരീരികള്‍ കേള്‍ക്കുകയും(Auditory Hallucination) മിഥ്യാഭ്രമത്തിലായിരിക്കുകയും (Delusion)  മായാദൃശ്യങ്ങള്‍ കാണുക യും(Hallucination) ചെയ്യും. ഈ അശരീരികളും മായാദൃശ്യങ്ങളും യാഥാര്‍ഥ്യവുമായി യാതൊരു ബന്ധവുമുള്ളതായിരിക്കില്ല.

മുഹമ്മദ് നബി(സ)ക്കുണ്ടായ വെളിപാടുകളും ദര്‍ശനങ്ങളും ഈ ഗണത്തില്‍ പെടുത്തിക്കൊണ്ടാണ് വിമര്‍ശകര്‍ അദ്ദേഹത്തില്‍ ഉന്മാദരോഗം ആരോപിക്കുന്നത്. ഉന്മാദരോഗത്തിന്റെ മറ്റു ലക്ഷണങ്ങളൊന്നും നബി(സ)യില്‍ ഉണ്ടായിരുന്നില്ലെന്ന് നാം മനസ്സിലാക്കി. അപ്പോള്‍ ഈ വെളിപാടുകളുടെ മാത്രം വെളിച്ചത്തില്‍ അദ്ദേഹം ഉന്മാദരോഗിയാണെന്ന് പറയുന്ന തെങ്ങനെ? ഉന്മാദരോഗിക്കുണ്ടാവുന്ന 'വെളിപാടു'കള്‍ അയാളുടെ രോഗത്തിന്റെ ലക്ഷണമാണ്. ഈ വെളിപാടുകള്‍ അയാളുടെ വൈയക്തിക മേഖലകളുമായി മാത്രം ബന്ധപ്പെട്ടതായിരിക്കും. എന്നാല്‍, മുഹമ്മദി(സ)നുണ്ടായ വെളിപാടുകളോ? ആ വെളിപാടുകള്‍ ഒരു ഉത്തമ സമൂഹത്തെ പടിപടിയായി വാര്‍ത്തെടുക്കുകയായിരുന്നു. ആദ്യം ദൈവബോധവും പര ലോകചിന്തയും ജനങ്ങളില്‍ വളര്‍ത്തി. ഘട്ടം ഘട്ടമായി സമൂഹത്തെ മുച്ചൂടും ബാധിച്ചിരുന്ന എല്ലാ തിന്മകളുടെയും അടിവേരറുത്തു. അങ്ങനെ ഒരു മാതൃകാ സമൂഹത്തിന്റെ സൃഷ്ടിക്ക് നിമിത്തമാകുവാന്‍ മുഹമ്മദി(സ)ന് ലഭിച്ച വെളിപാടുകള്‍ക്ക് കഴിഞ്ഞു. അത് സൃഷ്ടിച്ച വിപ്ലവം മഹത്തരമാണ്. ചരിത്രകാലത്ത് അതിനു തുല്യമായ മറ്റൊരു വിപ്ലവം നടന്നിട്ടില്ല.

ഉന്മാദരോഗി കേള്‍ക്കുന്ന അശരീരികള്‍ക്ക് ഒരു മാതൃകാ സമൂഹത്തിന്റെ സൃഷ്ടിക്കോ നിസ്തുലമായ ഒരു വിപ്ലവത്തിനോ നിമിത്തമാകുവാന്‍ കഴിയുമോ?

മുഹമ്മദി(സ)ന് ഉന്മാദരോഗമായിരുന്നുവെന്നും അദ്ദേഹം ശ്രവിച്ച അശ രീരികളാണ് ഖുര്‍ആനിലുള്ളതെന്നുമുള്ള വാദം പരിഗണനപോലും അര്‍ഹിക്കാത്ത ആരോപണം മാത്രമാണെന്നാണ് ഇതില്‍നിന്നെല്ലാം വ്യക്തമാകുന്നത്.

നങ്ങളെ ധാര്‍മികതയിലേക്ക് നയിക്കുന്ന ഗ്രന്ഥമാണ് ഖുര്‍ആന്‍. മദ്യ ത്തിലും മദിരാക്ഷിയിലും യുദ്ധങ്ങളിലും സായൂജ്യമടഞ്ഞിരുന്ന ഒരു സമൂഹ ത്തെ കേവലം 23 വര്‍ഷക്കാലം കൊണ്ട് ധാര്‍മികതയുടെ പ്രയോക്താക്കളും പ്രചാരകരുമാക്കിയ ഗ്രന്ഥമെന്ന ഖ്യാതി ഖുര്‍ആനിനു മാത്രം അവകാശപ്പെ ട്ടതാണ്. എന്നാല്‍ ധാര്‍മിക നവോത്ഥാനത്തിനുവേണ്ടി മുഹമ്മദ്(സ) രചി ച്ചുകൊണ്ട് ദൈവത്തില്‍ ആരോപിച്ച ഗ്രന്ഥമാണ് ഖുര്‍ആന്‍ എന്ന വാദ ഗതി അടിസ്ഥാന രഹിതമാണെന്ന് അത് ഒരാവര്‍ത്തി വായിക്കുന്ന ഏവര്‍ക്കും ബോധ്യമാവും. താഴെപ്പറയുന്ന വസ്തുതകള്‍ ശ്രദ്ധിക്കുക.

ഒന്ന്: സത്യസന്ധനായിരുന്നു മുഹമ്മദ് (സ) എന്ന കാര്യത്തില്‍ പക്ഷാ ന്തരമില്ല. അത്തരമൊരാള്‍ ധാര്‍മിക നവോത്ഥാനത്തിനുവേണ്ടി ദൈവത്തിന്റെ പേരില്‍ ഒരു പച്ചക്കള്ളം പറഞ്ഞുവെന്നു കരുതുന്നത് യുക്തി സഹമ ല്ല. ധാര്‍മിക നവോത്ഥാനത്തിനുവേണ്ടി ആത്മാര്‍ഥമായി പരിശ്രമിക്കുന്ന ഒരു വ്യക്തി അക്കാര്യത്തിനുവേണ്ടി സ്വന്തമായി ഒരു വലിയ അധര്‍മം ചെയ്യുകയെന്നത് അവിശ്വസനീയമാണ്. ദൈവത്തിന്റെ പേരില്‍ കളവ് പറയുന്നതിനേക്കാള്‍ വലിയ പാപമെന്താണ്?

രണ്ട്: പടച്ചവന്റെ പേരില്‍ കളവു പറയുകയും സ്വയം കൃതരചന കള്‍ ദൈവത്തിന്‍േറതാണെന്ന് വാദിക്കുകയും ചെയ്യുന്നവനാണ് ഏറ്റവും വലിയ അക്രമിയെന്നാണ് ഖുര്‍ആന്‍ പറയുന്നത്. ''അല്ലാഹുവിന്റെ പേരില്‍ കള്ളം കെട്ടിച്ചമയ്ക്കുകയോ, തനിക്ക് യാതൊരു ബോധനവും നല്‍കപ്പെടാതെ 'എനിക്ക് ബോധനം ലഭിച്ചിരിക്കുന്നു' എന്ന് പറയുകയോ ചെയ്തവനേ ക്കാളും അല്ലാഹു അവതരിപ്പിച്ചതുപോലെയുള്ളത് ഞാനും അവതരിപ്പിക്കാമെന്ന് പറഞ്ഞവനേക്കാളും വലിയ അക്രമി ആരുണ്ട്?''(6:93). ഖുര്‍ആന്‍ മുഹമ്മദി(സ)ന്റെ രചനയാണെങ്കില്‍ ഈ സൂക്തത്തില്‍ പറഞ്ഞ 'ഏറ്റവും വലിയ അക്രമി' അദ്ദേഹം തന്നെയായിരിക്കുമല്ലോ. തന്നെത്തന്നെ 'ഏറ്റവും വലിയ അക്രമി'യെന്ന് വിളിക്കുവാനും അതു രേഖപ്പെടുത്തുവാനും അദ്ദേ ഹം തയാറാകുമായിരുന്നുവോ?

മൂന്ന്: സ്വയംകൃത രചനകള്‍ നടത്തി അത് ദൈവത്തില്‍ ആരോപിക്കു ന്നവരെ ഖുര്‍ആന്‍ ശപിക്കുന്നുണ്ട്. ''എന്നാല്‍ സ്വന്തം കൈകള്‍ കൊണ്ട് ഗ്രന്ഥം എഴുതിയുണ്ടാക്കുകയും എന്നിട്ട് അത് അല്ലാഹുവില്‍നിന്ന് ലഭിച്ച താണെന്ന് പറയുകയും ചെയ്യുന്നവര്‍ക്ക് നാശം!''(2:79) ഖുര്‍ആന്‍ മുഹ മ്മദി(സ)ന്റെ സൃഷ്ടിയാണെങ്കില്‍ ഈ ശാപം അദ്ദേഹത്തിനുകൂടി ബാധക മാണല്ലോ. സ്വന്തമായി ഒരു രചന നിര്‍വഹിക്കുക. ആ രചനയില്‍ സ്വന്ത ത്തെത്തന്നെ ശപിക്കുക. ഇത് വിശ്വസനീയമാണോ?

നാല്: ഖുര്‍ആന്‍ ഒന്നിച്ച് അവതരിപ്പിക്കപ്പെട്ട ഗ്രന്ഥമല്ല. നീണ്ട ഇരുപ ത്തിമൂന്ന് വര്‍ഷങ്ങള്‍ക്കിടയില്‍ വ്യത്യസ്ത സാഹചര്യങ്ങളിലാണ് ഖുര്‍ആ ന്‍ സൂക്തങ്ങള്‍ അവതരിപ്പിക്കപ്പെട്ടത്. ഓരോ വിഷയങ്ങളിലും ജനങ്ങളുടെ ചോദ്യങ്ങള്‍ക്കുള്ള മറുപടിയായിട്ടാണ് ചില സന്ദര്‍ഭങ്ങളില്‍ ഖുര്‍ആന്‍ സൂക്തങ്ങള്‍ അവതരിപ്പിച്ചിരിക്കുന്നത്. ഖുര്‍ആനില്‍ പതിനഞ്ചോളം സ്ഥ ലങ്ങളില്‍ 'അവര്‍ നിന്നോട്...നെക്കുറിച്ചു ചോദിക്കുന്നു. പറയുക: ...' എന്ന ശൈലിയിലുള്ള സൂക്തങ്ങളുണ്ട്. ഓരോ വിഷയങ്ങളിലും പ്രവാചകനോട് അവര്‍ ചോദിച്ച സമയത്ത് അദ്ദേഹത്തിന് ഉത്തരം നല്‍കാന്‍ സാധിച്ചില്ലെ ന്നും പിന്നീട് ഖുര്‍ആന്‍ വാക്യം അവതരിപ്പിച്ചതിനുശേഷം മാത്രമാണ് അത് സാധിച്ചതെന്നുമാണല്ലോ ഇതില്‍നിന്ന് മനസ്സിലാവുന്നത്. ധാര്‍മിക നവോത്ഥാനം ലക്ഷ്യമാക്കിക്കൊണ്ടുള്ള പ്രവാചക(സ)ന്റെ രചനയായിരുന്നു ഖുര്‍ആനെങ്കില്‍ ജനം ചോദിച്ചപ്പോള്‍ ഉടന്‍തന്നെ അദ്ദേഹത്തിന് മറുപടി പറയാന്‍ കഴിയുമായിരുന്നു.

ഉദാഹരണത്തിന്, മദ്യത്തില്‍നിന്നും ചൂതാട്ടത്തില്‍നിന്നും ജനങ്ങളെ രക്ഷിക്കണമെന്നായിരുന്നു പ്രവാചകന്റെ ഉദ്ദശ്യമെങ്കില്‍ അവയെക്കുറിച്ച് ചോദിച്ച ഉടന്‍തന്നെ അവ പാപമാണ് എന്ന് അ ദ്ദേഹം മറുപടി പറയുമായിരുന്നു. എന്നാല്‍, അദ്ദേഹം ചെയ്തത് അതല്ല; സ്വയം മറുപടി പറയാതെ ദൈവിക വെളിപാട് പ്രതീക്ഷിക്കുകയായിരുന്നു. ദൈവവചനങ്ങള്‍ വെളിപ്പെട്ടതിനുശേഷമാണ് ഈ തിന്മകള്‍ക്കെതിരെയുള്ള നടപടികള്‍ അദ്ദേഹം സ്വീകരിച്ചത്.

അഞ്ച്: മുഹമ്മദ് നബി(സ)യെ തിരുത്തുന്ന ചില ഖുര്‍ആന്‍ സൂക്തങ്ങളുണ്ട്. ഖുറൈശി പ്രമുഖരുമായി സംസാരിച്ചുകൊണ്ടിരിക്കുമ്പോള്‍ കടന്നുവന്ന അന്ധനായ അബ്ദുല്ലാഹിബ്‌നുഉമ്മിമക്തൂമിനെ പ്രസന്നതയോടെ സ്വീകരിക്കാതിരുന്ന പ്രവാചക(സ)ന്റെ നടപടിയെ തിരുത്തിയ ഖുര്‍ആന്‍ സൂക്തങ്ങള്‍ (80:1-10) സുവിദിതമാണ്.

മറ്റൊരു സംഭവം: മുസ്‌ലിംകള്‍ക്ക് ഏറെ നാശനഷ്ടങ്ങള്‍ വിതച്ച ഉഹ്ദ് യുദ്ധത്തില്‍ പ്രവാചകന്റെ ശരീരത്തിലും ഒരുപാട് മുറിവുകള്‍ ഉണ്ടായി. യുദ്ധശേഷം അദ്ദേഹം അവിശ്വാസികളി ല്‍ ചിലരെ ശപിക്കുകയും 'അവരുടെ പ്രവാചകനെ മുറിപ്പെടുത്തിയ സമൂഹ മെങ്ങനെയാണ് നന്നാവുക?' എന്ന് ആത്മഗതം നടത്തുകയും ചെയ്തു. ഉടന്‍ ഖുര്‍ആന്‍ സൂക്തമവതരിച്ചു; പ്രവാചക(സ)നെ തിരുത്തിക്കൊണ്ട്. ''(നബിയേ), കാര്യത്തിന്റെ തീരുമാനത്തില്‍ നിനക്ക് യാതൊരവകാശവുമില്ല. അവന്‍ (അല്ലാഹു) ഒന്നുകില്‍ അവരുടെ പശ്ചാത്താപം സ്വീകരിച്ചേ ക്കാം. അല്ലെങ്കില്‍ അവരെ അവന്‍ ശിക്ഷിച്ചേക്കാം. തീര്‍ച്ചയായും അവര്‍ അക്രമികളാകുന്നു''(3:128)(തിര്‍മിദി,ഇബ്‌നുമാജ).

ഇതൊന്നും പ്രവാചകനി ല്‍ ബോധപൂര്‍വ്വം വന്ന തെറ്റുകളല്ല. താന്‍ സ്വീകരിച്ച നിലപാടുകളിലുണ്ടാ യ അബദ്ധം മാത്രം. എന്നിട്ടും അവ തിരുത്തുന്ന വചനങ്ങള്‍ ഖുര്‍ആനി ലുണ്ടായി. ജനങ്ങളെ ധര്‍മനിഷ്ഠരാക്കുവാന്‍ വേണ്ടി പ്രവാചകന്‍(സ) പടച്ച ഗ്രന്ഥമായിരുന്നു ഖുര്‍ആനെങ്കില്‍ അദ്ദേഹത്തിന്റെ നടപടികളെ വിമര്‍ശിക്കുന്ന സൂക്തങ്ങള്‍ ഖുര്‍ആനിലുണ്ടാവുമായിരുന്നുവോ?

ധികാരമോഹമെന്നാല്‍ എന്താണ്? രാജ്യത്തിന്റെ അധികാരം കൈ ക്കലാക്കി സുഖസമൃദ്ധമായ ജീവിതം നയിക്കാനുള്ള ആഗ്രഹം. പതിമൂന്ന് വര്‍ഷത്തെ കഷ്ടപ്പാടുകള്‍ക്കും പീഡനങ്ങള്‍ക്കും ശേഷം പലായനം ചെയ്തു മദീനയിലെത്തിയ പ്രവാചകന് അധികാരം ലഭിച്ചുവെന്നത് നേരാണ്. എന്നാല്‍, അദ്ദേഹത്തിന് അധികാരം സുഖലോലുപതയ്ക്കുള്ള മാര്‍ഗമായി രുന്നില്ല. ഭരണാധികാരിയായിരിക്കുമ്പോഴും ഈത്തപ്പനപ്പായയില്‍ അന്തിയുറങ്ങുകയും വസ്ത്രങ്ങള്‍ സ്വയം അലക്കുകയും പാദരക്ഷകള്‍ തുന്നുകയും ആടിനെ കറക്കുകയും ചെയ്യുന്ന മനുഷ്യനെ അധികാരമോഹിയെന്നു വിളിക്കാന്‍ ആര്‍ക്കാണ് സാധിക്കുക? അധികാരത്തിന്റെ പേരില്‍ ജനങ്ങളാല്‍ ആദരിക്കപ്പെടുകയും അവരില്‍ നിന്ന് ഉയര്‍ന്നുനില്‍ക്കുകയും ചെയ്യാന്‍ ആഗ്രഹിക്കുന്നവരാണ് അധികാരം മോഹിക്കുക. പ്രവാചക(സ)നാവട്ടെ ജനങ്ങളെ സേവിച്ച് ജനങ്ങളോടൊപ്പം ജീവിച്ചയാളായിരുന്നു. തന്നെ ബഹുമാനിച്ചുകൊണ്ട് ആളുകള്‍ എഴുന്നേറ്റു നില്‍ക്കുന്നതുപോലും അവിടുന്ന് ഇഷ്ടപ്പെട്ടിരുന്നില്ല.

അദ്ദേഹം ഉപദേശിച്ചു: ''ക്രിസ്ത്യാനികള്‍ മര്‍യമിന്റെ പുത്രനായ യേശുവിനെ പുകഴ്ത്തിയതുപോലെ എന്നെ നിങ്ങള്‍ പുകഴ്ത്തരുത്'' (ബുഖാരി, മുസ്‌ലിം). ഇതെല്ലാംതന്നെ മുഹമ്മദ് (സ) ഒരു അധികാര മോഹിയായിരുന്നില്ലെന്ന് വ്യക്തമാക്കുന്നു.

മാത്രവുമല്ല, തന്റെ പ്രബോധന പ്രവര്‍ത്തനങ്ങള്‍ നിര്‍ത്തിവെക്കുകയാ ണെങ്കില്‍, മക്കയിലെ പ്രയാസപൂര്‍ണമായ ആദ്യനാളുകളില്‍തന്നെ അധികാരം നല്‍കാമെന്ന് അദ്ദേഹം വാഗ്ദാനം ചെയ്യപ്പെട്ടിരുന്നു. സമൂഹത്തിലെ നേതാക്കന്മാരെല്ലാംകൂടി ഒരു ദിവസം മുഹമ്മദി(സ)ന്റെ അടുത്തുചെന്ന് അദ്ദേഹത്തെ വശീകരിക്കാനായി ആവത് ശ്രമിച്ചു നോക്കി. പക്ഷെ, നിരാശ മാത്രമായിരുന്നു ഫലം. മക്കയിലെ പ്രബോധന പ്രവര്‍ത്തനങ്ങളുടെ ആദ്യനാളുകളിലായിരുന്നു ഈ സംഭവം. ഖുര്‍ആന്‍ രചിച്ചുകൊണ്ട് താന്‍ ദൈവദൂതനാണെന്ന് വരുത്തിത്തീര്‍ത്ത് അധികാരം കൈക്കലാക്കുകയായിരുന്നു പ്രവാചക(ല)ന്റെ ലക്ഷ്യമെങ്കില്‍ പ്രയാസങ്ങള്‍ ഏറെയൊന്നും സഹിക്കാതെ അധികാരം തന്റെ കാല്‍ക്കീഴില്‍ വന്ന സമയത്ത് അദ്ദേഹം അത് സ്വീകരിക്കുവാന്‍ വൈമനസ്യം കാണിച്ചതെന്തിനാണ്? മുഹമ്മദ്അ(സ)ധികാരം കാംക്ഷിച്ചിരുന്നില്ലെന്ന് ഇതില്‍നിന്ന് സുതരാം വ്യക്തമാണ്. ഖുര്‍ആന്‍ കൊണ്ടുവന്നതിനു പിന്നില്‍ അധികാരമോഹമായിരുന്നില്ലെന്ന് സാരം.

നാഥനായി വളര്‍ന്ന മുഹമ്മദ്(സ) ചെറുപ്പത്തില്‍ ഒരുപാട് കഷ്ടപ്പാടുകള്‍ അനുഭവിച്ചിരിക്കാം. എന്നാല്‍, തന്റെ 25-ാം വയസ്സില്‍ നാല്‍പതുകാരിയായ കച്ചവടക്കാരി ഖദീജ( (റ)യെ വിവാഹം ചെയ്തതിനുശേഷം അദ്ദേ ഹത്തിന്റെ ജീവിതനിലവാരം സ്വാഭാവികമായും മെച്ചപ്പെട്ടതായി മാറിയിരിക്കണം. അത്യാവശ്യം നല്ല സാമ്പത്തിക ശേഷിയുണ്ടായിരുന്ന ഖദീജ( (റ)യുടെ ഭര്‍ത്താവായിരുന്ന അദ്ദേഹം സാമ്പത്തിക ക്ലേശങ്ങള്‍ അനുഭവിച്ചിരിക്കാനുള്ള സാധ്യത വിരളമാണ്.

ഖദീജയുമായുള്ള മുഹമ്മദി(സ) ന്റെ വിവാഹം നടന്നത് പ്രവാചകത്വം ലഭിക്കുന്നതിന് 15 വര്‍ഷങ്ങള്‍ക്കുമുമ്പാണ്. പതി നഞ്ച് വര്‍ഷം സാമ്പത്തികക്ലേശം കൂടാതെ ജീവിച്ചതിനുശേഷമാണ് താന്‍ പ്രവാചകനാണെന്നും ഖുര്‍ആന്‍ ദൈവവചനമാണെന്നുമുള്ള അവകാശവാദങ്ങളുമായി മുഹമ്മദ്(സ) രംഗപ്രവേശം ചെയ്യുന്നതെന്നര്‍ഥം. ഖുര്‍ആന്‍ ദൈവികമാണെന്ന് വാദിക്കുക വഴി ഭൗതികലാഭമാണ് അദ്ദേഹം ഇച്ഛിച്ചതെങ്കില്‍ ഈ വാദം ഉന്നയിച്ചതിനുശേഷം അദ്ദേഹത്തിന്റെ സാമ്പത്തിക സ്ഥിതി മെച്ചപ്പെട്ടിരിക്കണമല്ലോ.

എന്നാല്‍, എന്തായിരുന്നു സ്ഥിതി?

പ്രവാചകപത്‌നി ആഇശ(റ) പറയുന്നു: ''ഞങ്ങളുടെ വീട്ടില്‍ ഒന്നും പാചകം ചെയ്യാനില്ലാത്തതിനാല്‍ അടുപ്പു പുകയാതെ ഒന്നോ രണ്ടോ മാസ ങ്ങള്‍ കഴിഞ്ഞുപോകാറുണ്ടായിരുന്നു. ഈത്തപ്പഴവും വെള്ളവുമായിരുന്നു ഞങ്ങളുടെ ഉപജീവനം. ചിലപ്പോള്‍ മദീനത്തുകാര്‍ കൊണ്ടുവന്ന ആട്ടി ന്‍പാലും ഈത്തപ്പഴത്തോടു കൂടെയുണ്ടാവും''. (ബുഖാരി, മുസ്‌ലിം)

ആഇശ(റ) ഒരാളോട് പഴയകാര്യങ്ങള്‍ പറഞ്ഞുകൊണ്ടിരിക്കുകയാണ്. മദീനയിലേക്കുള്ള പലായനത്തിനുശേഷം പ്രവാചകനും കുടുംബവും സ ഹിച്ച പ്രയാസങ്ങളാണ് പ്രതിപാദ്യം. ഒരു രാത്രി തപ്പിത്തടഞ്ഞുകൊണ്ട് വീട്ടുജോലികള്‍ ചെയ്തകാര്യം അവര്‍ പറഞ്ഞു. അയാള്‍ ചോദിച്ചു: ''വിള ക്കില്ലായിരുന്നുവോ? അവര്‍ പ്രതിവചിച്ചു: ''വിളക്കു കത്തിക്കാനുള്ള എണ്ണ ഞങ്ങളുടെ പക്കലുണ്ടായിരുന്നുവെങ്കില്‍ വിശപ്പ് മാറ്റാന്‍ അത് കുടിക്കുമാ യിരുന്നു; കത്തിക്കുന്നതിനു പകരം''. (അഹ്മദ്, ത്വബ്‌റാനി)

ഇത് പ്രവാചകന്റെ ആദ്യകാലത്തെ മാത്രം അവസ്ഥയല്ല. മുഹമ്മദ്(സ) ശക്തമായ ഒരു സാമ്രാജ്യത്തിന്റെ ഭരണാധികാരിയായിരുന്നപ്പോഴും അദ്ദേ ഹത്തിന്റെ അവസ്ഥ ഇതില്‍നിന്ന് ഒട്ടും മെച്ചമായിരുന്നില്ല. ഇസ്‌ലാമിക സാമ്രാജ്യത്തിന്റെ അധിപന്റെ അന്തപുരത്തെക്കുറിച്ച് അദ്ദേഹത്തിന്റെ സഹചാരിയായിരുന്ന ഉമര്‍ (റ)തന്നെ പറയട്ടെ: ''പ്രവാചകന്റെ മുറിയില്‍ ഊറക്കിട്ട മൂന്ന് തോല്‍കഷ്ണങ്ങളും ഒരു മൂലയില്‍ അല്‍പം ബാര്‍ലിയുമല്ലാതെ മറ്റൊന്നുംതന്നെ ഞാന്‍ കണ്ടില്ല. ഞാന്‍ കരഞ്ഞുപോയി. പ്രവാചകന്‍ ചോദിച്ചു: 'എന്തിനാണ് താങ്കള്‍ കരയുന്നത്?' ഞാന്‍ പറഞ്ഞു: 'അല്ലാഹുവിന്റെ ദൂതരേ! ഞാനെങ്ങനെ കരയാതിരിക്കും? താങ്കളുടെ ശരീരത്തില്‍ ഈത്തപ്പനയോലകളുടെ പാട് ഞാന്‍ കാണുന്നു. ഈ മുറിയില്‍ എന്തെല്ലാമുണ്ടെന്നും ഞാനറിയുന്നു. അല്ലാഹുവിന്റെ ദൂതരേ! സമൃദ്ധമായ വിഭവങ്ങള്‍ക്കുവേണ്ടി അല്ലാഹുവിനോട് പ്രാര്‍ ഥിച്ചാലും. അവിശ്വാസികളും അല്ലാഹുവില്‍ പങ്കുചേര്‍ക്കുന്നവരുമായ പേര്‍ഷ്യക്കാരുടെയും റോമാക്കാരുടെയും രാജാക്കന്മാര്‍-സീസറും കൈസറുമെല്ലാം-അരുവികള്‍ ഒഴുകുന്ന തോട്ടങ്ങളില്‍ വസിക്കുമ്പോള്‍ അല്ലാഹുവിന്റെ തെരഞ്ഞെടുക്കപ്പെട്ട പ്രവാചകന്‍ ജീവിക്കുന്നത് ദാരുണമായ പട്ടിണിയില്‍!' എന്റെ ഈ സംസാരം കേട്ടപ്പോള്‍ തലയിണയില്‍ വിശ്രമിക്കുകയായിരുന്ന പ്രവാചകന്‍ എഴുന്നേറ്റിരുന്നു. എന്നിട്ടു പറഞ്ഞു: 'ഉമര്‍! താങ്കള്‍ ഈ വിഷയത്തില്‍ ഇനിയും സംശയാലുവാണോ? ഭൗതിക ജീവിതത്തിലെ സുഖസൗകര്യങ്ങളേക്കാള്‍ നല്ലത് മരണാനന്തര ജീവിതത്തിലെ സുഖസൗ കര്യങ്ങളാണ്. അവിശ്വാസികള്‍ അവരുടെ നന്മയുടെ വിഹിതം ഈ ജീവിതത്തില്‍ ആസ്വദിക്കുന്നു. നമ്മുടേതാകട്ടെ, മരണാനന്തര ജീവിതത്തിലേക്കുവേണ്ടി ബാക്കിവെച്ചിരിക്കുകയാണ്'. ഞാന്‍ അദ്ദേഹത്തോട് അപേക്ഷിച്ചു: 'ദൈവദൂതരെ! എനിക്കുവേണ്ടി മാപ്പിനപേക്ഷിച്ചാലും. എനിക്കു തെറ്റിപ്പോയി''.

ഖുര്‍ആന്‍ ഭൗതിക ലാഭങ്ങള്‍ക്കുവേണ്ടി പടച്ചുണ്ടാക്കിയ മുഹമ്മദി(സ) ന്റെ കൃതിയാണെന്ന വാദമാണിവിടെ തകരുന്നത്. ആകെ സ്വത്തായി ബാക്കിയുണ്ടായിരുന്ന ഏഴു ദീനാര്‍ മരണത്തിനുമുമ്പ് ദാനം ചെയ്യുകയും യഹൂദന് തന്റെ പടച്ചട്ട പണയം വെച്ചുകൊണ്ട് മരണപ്പെടുകയും ചെയ്ത മനുഷ്യന്‍ ധനമോഹിയായിരുന്നുവെന്ന് പറയുന്നത് അടിസ്ഥാനരഹിതമാണ്.

ഖുര്‍ആനിന്റെ രചനക്കുപിന്നില്‍ ധനമോഹമായിരുന്നുവെന്ന വാദം അടിസ്ഥാന രഹിതമാണെന്ന് ദി ന്യു കാത്തോലിക് എന്‍സൈക്ലോപീഡിയ പോലും സമ്മതിച്ചിട്ടുണ്ട്. ''മുഹമ്മദി(സ) ന്റെ മതവിപ്ലവത്തിനു പിന്നില്‍ ധനമോഹമായിരുന്നുവെന്ന ഒരു ധാരണ സൃഷ്ടിക്കപ്പെട്ടിട്ടുണ്ട്. വ്യക്തമാ യി അറിയപ്പെടുന്ന വസ്തുതകള്‍ ഈ ധാരണക്കെതിരാണ്'' (The New Catholic Encyclopedia Vol IX, Page 1001).

മുഹമ്മദ് നബി(സ) ജീവിച്ചത് ചരിത്രത്തിന്റെ വെളിച്ചത്തിലാണ്. അദ്ദേ ഹത്തിലൂടെയാണ് ലോകം ഖുര്‍ആന്‍ ശ്രവിച്ചത്. അതുകൊണ്ടുതന്നെ ഖുര്‍ആനിന്റെ ദൈവികത അംഗീകരിക്കാത്തവരെ സംബന്ധിച്ചിടത്തോളം അവര്‍ക്ക് പറയാനുള്ളത് ഇത് മുഹമ്മദി(സ)ന്റെ രചനയാണെന്നാണ്. ഈ വാദം വിശദമായി ചര്‍ച്ച ചെയ്യേണ്ടതാണ്. ചര്‍ച്ചയുടെ ആമുഖമായി നാം മനസ്സിലാക്കേണ്ട ചില വസ്തുതകളുണ്ട്. അവയുടെ അടിത്തറയില്‍നിന്നുകൊണ്ടായിരിക്കണം മുഹമ്മദ് നബി(സ)യില്‍ ഖുര്‍ആനിന്റെ കര്‍തൃത്വം ആരോപിക്കുന്നത്.

ഒന്ന്: നാല്‍പതു വയസ്സുവരെ അറബികള്‍ക്കിടയില്‍ സുസമ്മതനായ വ്യക്തിയായിരുന്നു മുഹമ്മദ്(സ). ഖുര്‍ആന്‍ ദൈവികമാണെന്നും അതിലെ വിധിവിലക്കുകള്‍ അനുസരിക്കേണ്ടതുണ്ടെന്നും പ്രബോധനം ചെയ്തതുകൊണ്ടാണ് അദ്ദേഹം വെറുക്കപ്പെട്ടവനായത്; ബഹിഷ്‌കരിക്കപ്പെട്ടത്; ജനിച്ച് വളര്‍ന്ന നാട്ടില്‍ നിന്ന് പലായനം ചെയ്യേണ്ടിവന്നത്.

രണ്ട്: സത്യസന്ധനായിരുന്നു മുഹമ്മദ്(സ)എന്ന കാര്യത്തില്‍ അദ്ദേഹ ത്തിന്റെ കഠിന ശത്രുക്കള്‍ക്കുപോലും അഭിപ്രായവ്യത്യാസമുണ്ടായിരുന്നി ല്ല. നാല്‍പതു വയസ്സുവരെ സത്യസന്ധനായി ജീവിച്ച അദ്ദേഹം ഒരു ദിവ സം പടച്ചതമ്പുരാന്റെ പേരില്‍ ഒരു പച്ചക്കള്ളം പറഞ്ഞുവെന്നും അത് പ്രചരിപ്പിക്കുന്നതിനുവേണ്ടി സ്വന്തം ജീവന്‍ തൃണവത്ഗണിച്ചുവെന്നും വിശ്വസിക്കുക പ്രയാസമാണ്.

മൂന്ന്: സാഹിത്യകാരന്മാര്‍ക്ക് അറേബ്യയില്‍ ഉന്നതമായ സ്ഥാനം നല്‍ കപ്പെട്ടിരുന്നു. ഖുര്‍ആന്‍ അത്യുന്നതമായ ഒരു സാഹിത്യ സൃഷ്ടിയാണെന്ന കാര്യത്തില്‍ ആര്‍ക്കും ഭിന്നാഭിപ്രായമുണ്ടായിരുന്നില്ല. അത് തന്‍േറതാണ് എന്ന് അവകാശപ്പെട്ടിരുന്നുവെങ്കില്‍ അദ്ദേഹത്തിന് അറബികള്‍ക്കിടയില്‍ ഉന്നതമായ സ്ഥാനമാനങ്ങള്‍ ലഭിക്കുമായിരുന്നു.

നാല്: മുഹമ്മദി(സ)ന്റെ ചില നടപടികളെ വിമര്‍ശിക്കുന്ന വാക്യങ്ങള്‍ ഖുര്‍ആനിലുണ്ട്. അഞ്ച്: മുഹമ്മദി(സ)നെ ശക്തമായി താക്കീത് ചെയ്യുന്ന വചനങ്ങളും ഖുര്‍ആനിലുണ്ട്. ഈ വസ്തുതകള്‍ മുന്നില്‍ വെച്ചുകൊണ്ടാണ് ഖുര്‍ആന്‍ മുഹമ്മദി(സ)ന്റെ സൃഷ്ടിയാണ് എന്ന വാദത്തിലെ ശരിയും തെറ്റും പരിശോധിക്കേ ണ്ടത്.

സാഹിത്യമൂല്യമുള്ള ഒരു സൃഷ്ടി നടത്തി അത് ദൈവത്തിന്റെ പേരില്‍ ആരോപിച്ചതാണെങ്കില്‍ അതിനു പിന്നില്‍ സ്വാര്‍ഥമായ വല്ല ലക്ഷ്യങ്ങളുമുണ്ടാവണമല്ലോ. അതെന്തായിരുന്നുവെന്നാണ് വിമര്‍ശകര്‍ ആദ്യം വ്യക്തമാക്കേണ്ടത്. അതിന്റെ അടിസ്ഥാനത്തിലാണ് ഈ വാദത്തിന്റെ സത്യത പരിശോധിക്കപ്പെടേണ്ടത്.

ക്രൂരവും രക്തരൂക്ഷിതവുമാണല്ലോ യുദ്ധങ്ങളെല്ലാം. മാനവികതയുടെ മതമാണെന്ന് അവകാശപ്പെടുന്ന ഇസ്‌ലാം യുദ്ധം അനുവദിച്ചത് അതിന്റെ ഈ അവകാശവാദം പൊള്ളയാണെന്നല്ലേ വ്യക്തമാക്കുന്നത്?

യുദ്ധം എക്കാലത്തുമുണ്ടായിരുന്നു. മാനവ സംസ്‌കാരത്തിന്റെ ആദ്യകാലം മുതല്‍ അത് നിലനിന്നിരുന്നു. യുദ്ധം നിലനിന്നിരുന്ന സമൂഹങ്ങളിലേക്കാണ് ദൈവിക മാര്‍ഗദര്‍ശനങ്ങള്‍ക്കനുസരിച്ച് പ്രസ്തുത സമൂഹങ്ങളിലെ ജനങ്ങളെ നയിക്കുവാനായി പ്രവാചകന്‍മാര്‍ നിയോഗിക്കപ്പെട്ടത്. മനുഷ്യരെ നേരായ പാതയിലൂടെ നയിക്കാന്‍ ശ്രമിച്ച ദൈവദൂതന്‍മാര്‍ യുദ്ധരംഗത്തേക്കും ആവശ്യമായ മാര്‍ഗനിര്‍ദ്ദേശങ്ങള്‍ നല്‍കിയിരിക്കണം. അക്രമത്തിന്റെയും അധര്‍മത്തിന്റെയും കറ പുരളാതെ യുദ്ധരംഗത്തെ നിലനിര്‍ത്താനാവശ്യമായ, അവര്‍ നല്‍കിയ നിര്‍ദ്ദേശങ്ങള്‍ പ്രകാരം നടന്ന യുദ്ധങ്ങള്‍ വഴി നന്മ പുലരുകയും തിന്മ തകരുകയും ചെയ്തിരിക്കണം. സ്വന്തം നാടുകളില്‍ നിന്ന് പുറത്താക്കപ്പെട്ട ഇസ്രാഈല്യര്‍ യുദ്ധം ചെയ്തു വിജയിക്കുകയും അങ്ങനെ അത് ദാവൂദിന്റെ രാജാധിപത്യത്തിലേക്ക് നയിക്കുകയും ചെയ്ത സംഭവം ക്വുര്‍ആന്‍ വിശദീകരിക്കുന്നുണ്ട്. (2:246)

സ്വന്തം താമസസ്ഥലത്ത് നിന്നും സന്തതികള്‍ക്കിടയില്‍ നിന്നും പുറത്താക്കപ്പെട്ട ഇസ്രാഈല്യർ യുദ്ധം ചെയ്തപ്പോൾ അവര്‍ക്ക് ദൈവിക സഹായം ഉണ്ടായി എന്നും പ്രാര്‍ത്ഥനയോടു കൂടി അവര്‍ യുദ്ധത്തിനിറങ്ങിയപ്പോൾ അഹങ്കാരിയായ ജാലൂത്തിനെ വധിക്കാന്‍ കഴിഞ്ഞുവെന്നും ഖുർആൻ പറയുന്നുണ്ട്. (2:250,251) താലൂത്തിന്റെ നേതൃത്വത്തില്‍ ജാലൂത്തിനെതിരെ നടന്ന യുദ്ധത്തെയും അതില്‍വെച്ച് ദാവീദ് ജാലൂത്തിനെ വധിച്ചതിനെയും അങ്ങനെ അദ്ദേഹം ഇസ്രാഈല്യരുടെ രാജാവായിത്തീര്‍ന്നതിനെയും പറ്റി പരാമര്‍ശിച്ചുകൊണ്ട് ഈ വിഷയം അല്ലാഹു അവസാനിപ്പിക്കുന്നത് ‘മനുഷ്യരില്‍ ചിലരെ ചിലര്‍മ മുഖേന അല്ലാഹു തടുക്കുന്നില്ലായിരുന്നുവെങ്കില്‍ ഭൂലോകം കുഴപ്പത്തിലാകുമായിരുന്നു. പക്ഷേ, അല്ലാഹു ലോകരോട് വളരെ ഉദാരനത്രെ’ എന്ന് പ്രസ്താവിച്ചുകൊണ്ടാണെന്ന കാര്യം പ്രത്യേകം ശ്രദ്ധേയമാണ്. മുഹമ്മദ് നബി(സ)ക്ക് യുദ്ധാനുമതി നല്‍കിക്കൊണ്ടുള്ള ആദ്യ കല്‍പനയിലും ‘മനുഷ്യരില്‍ ചിലരെ മറ്റു മനുഷ്യരെക്കൊണ്ട് തടുക്കുന്നില്ലായിരുന്നുവെങ്കില്‍ സന്യാസിമഠങ്ങളും, ക്രിസ്തീയദേവാലയങ്ങളും, യഹൂദദേവാലയങ്ങളും, അല്ലാഹുവിന്റെ നാമം ധാരാളമായി പ്രകീര്‍ത്തിക്കപ്പെടുന്ന മുസ്‌ലിം പള്ളികളും തകര്‍ക്കപ്പെടുമായിരുന്നു’ (22:40) എന്ന് പറയുന്നതായി കാണാം.

പ്രവാചകന്‍മാരിലൂടെ പഠിപ്പിക്കപ്പെട്ട യുദ്ധങ്ങളുടെ ലക്ഷ്യം മനുഷ്യരില്‍ ചിലരെ മറ്റു ചിലരെക്കൊണ്ട് തടഞ്ഞുനിര്‍ത്തിക്കൊണ്ട് അവര്‍ നടത്തുന്ന മര്‍ദ്ദനങ്ങള്‍ക്കും പീഡനങ്ങള്‍ക്കും അറുതിവരുത്തി ഭൂമിയെ കുഴപ്പത്തില്‍നിന്ന് സംരക്ഷിക്കുവാനും ആരാധനാ സ്വാതന്ത്ര്യവും പ്രബോധന സ്വാതന്ത്ര്യവും സ്ഥാപിച്ചെടുക്കുകയുമായിരുന്നുവെന്നാണ് ഈ വചനങ്ങള്‍ വ്യക്തമാക്കുന്നത്. യുദ്ധം ചെയ്ത പ്രവാചകന്‍മാരെല്ലാം ഈയൊരു ലക്ഷ്യത്തിനുവേണ്ടി തന്നെയായിരിക്കണം ആയുധമെടുത്തത്. പ്രവാചകന്‍മാര്‍ അതിക്രമകാരികളോ പീഡകരോ ആയിരുന്നില്ല, മനുഷ്യരുടെ വിമോചകരായിരുന്നു. അവര്‍ ചെയ്ത യുദ്ധങ്ങളെല്ലാം, അതുകൊണ്ട് തന്നെ മനുഷ്യരെ പീഡനങ്ങളില്‍ നിന്നും മോചിപ്പിക്കാന്‍ വേണ്ടിയുള്ളതായിരുന്നു. മുഹമ്മദ് നബി (സ) നയിച്ച യുദ്ധങ്ങളെടുത്ത് പരിശോധിച്ചാൽ നമുക്ക് കാണാനാവുക മനുഷ്യവിമോചനമെന്ന ഈ ഉന്നത ലക്ഷ്യമായിരുന്നു; ആളുകളെ കൂട്ടക്കൊല ചെയ്യാനോ സാമ്രാജ്യം സ്ഥാപിക്കുവാനോ വേണ്ടിയുള്ളതായിരുന്നില്ല പ്രസ്തുത യുദ്ധങ്ങളൊന്നുമെന്ന വസ്തുത സത്യസന്ധമായി ചരിത്രം പരിശോധിക്കുന്ന ആർക്കും ബോധ്യപ്പെടും.

പ്രവാചകനിൽ നിന്ന് മതം പഠിച്ച ശിഷ്യന്മാരും പിൽക്കാല മുസ്ലിംകളുമെല്ലാം യുദ്ധം ചെയ്തത് വലിയ മാനവികലക്ഷ്യങ്ങളോട് കൂടിയായിരുന്നു. അതുകൊണ്ടാണ് ഭരണാധികാരികളുടെ പീഡനങ്ങളാൽ പൊരുതി മുട്ടിയിരുന്ന ജനങ്ങൾ മുസ്ലിം ഭരണാധികാരികളെ തങ്ങളുടെ നാടുകളിലേക്ക് ക്ഷണിച്ചു വരുത്തിയ സംഭവങ്ങളുണ്ടായത്. ഇങ്ങനെ പോയ മുസ്ലിം പടയാളികളാണ് മാന്യവും മാനവികവുമായി എങ്ങനെ യുദ്ധം ചെയാമെന്ന് ലോകത്തെ പഠിപ്പിച്ചത്.

കുരിശുയുദ്ധങ്ങളുടെ കാര്യമെടുക്കുക. മുസ്‌ലിം ലോകത്ത് നടത്തിയ ക്രൂരതകളുടെ ഭീതിപപെടുത്തുന്ന കഥകളാണ് ഓരോ കുരിശു യുദ്ധത്തിനും പറയാനുളളത്. നിയമങ്ങളോ നീതിയോ ധാര്‍മികതയോ ഇല്ലാത്ത കാടന്‍ യുദ്ധങ്ങളാണ് കുരിശുയുദ്ധക്കാര്‍ നടത്തിയത്. ഇവയിൽ ലക്ഷക്കണക്കിന് മനുഷ്യര്‍ കൊല്ലപ്പെട്ടു. മരിച്ചവരുടെ ശവശരീരങ്ങളെ കുരിശു പടയാളികള്‍ വികൃതമാക്കി. എട്ടും പൊട്ടും തിരിച്ചറിയാത്ത പൈതങ്ങളെപ്പോലും കൊല്ലുന്നതില്‍ നിന്ന് അവരെ പിന്‍തിരിപ്പിക്കുവാന്‍ 'ഒരു മുഖത്തടിച്ചാല്‍ മറ്റേ മുഖവും കാണിച്ചു കൊടുക്കണം' എന്ന ബൈബിള്‍ പാഠത്തിന്റെ അടിസ്ഥാനത്തില്‍ ജനങ്ങളെ നയിക്കുന്നവരെന്ന് അവകാശപ്പെടുന്ന സഭാനേതൃത്വം സന്നദ്ധമായില്ല. ക്രിസ്തുവിന്റെ പേരില്‍ ആരോപിക്കപ്പെട്ട, അദ്ദേഹത്തിന്റെതല്ലാത്ത ഉപദേശം അപ്രായോഗികവും അക്രമികളെ സഹായിക്കുന്നതുമാണെന്ന വസ്തുതയ്ക്ക് സഭാനേതൃത്വം തന്നെ സ്വയം തെളിവായിത്തീരുകയായിരുന്നു എന്ന് പറയുന്നതാവും ശരി. അതല്ലായിരുന്നുവെങ്കില്‍ ഈ കുരിശുയുദ്ധങ്ങളുടെപേരില്‍ ഏഴു നൂറ്റാണ്ടുകള്‍ക്ക് ശേഷം ലോകത്തെ പരമോന്നത ക്രിസ്ത്യാനിക്ക് കുമ്പസരിക്കേണ്ടി വരികയില്ലായിരുന്നല്ലോ!

ആയുധങ്ങളെടുത്ത് ആളുകളെ കൂട്ടക്കൊല ചെയ്യുക മാത്രമാണ് യുദ്ധമെന്ന് പഠിച്ച കുരിശുയോദ്ധാക്കള്‍ക്ക് മാന്യമായി എങ്ങനെ യുദ്ധം ചെയ്യണമെന്ന് പഠിപ്പിച്ചത് സുല്‍ത്താന്‍ സ്വലാഹുദ്ദീന്‍ അയ്യൂബിയായിരുന്നു. നീണ്ട രണ്ട് നൂറ്റാണ്ടു കാലത്തെ കുരിശു യോദ്ധാക്കളുടെ ദുര്‍ഭരണത്തില്‍ നിന്ന് മസ്ജിദുല്‍ അക്വ്‌സയെയും ജറുസലേമിനേയും മോചിപ്പിച്ചത് അദ്ദേഹത്തിന്റെ കീഴിലുളള അബ്ബാസിയ്യാ മുസ്‌ലിം പടയാളികളായിരുന്നു. 1187 ജൂലൈ നാല് ശനിയാഴ്ച നടന്ന ഹത്തീന്‍ യുദ്ധത്തില്‍ കുരിശു പടയാളികള്‍ പരാജയപ്പെടുകയും ജറുസലേമിന്റെ അധികാരം സ്വലാഹുദ്ദീന്റെ കൈകളില്‍ വന്നു ചേരുകയും ചെയ്തു. കുരിശുയോദ്ധാക്കളുടെ നേതാക്കളായ ഗൈ രാജാവും സോഹദരങ്ങളുമടക്കം നിരവധി പേരെ ബന്ധികളാക്കി സ്വലാഹുദ്ദീനുമുന്നില്‍ ഹാജരാക്കപ്പെട്ടു. അവര്‍ക്കെല്ലാം തണുത്ത പാനീയം നല്‍കി സ്വീകരിക്കുവാനാണ് സ്വലാഹുദ്ദീന്‍ തന്റെ സേവകന്മാരോട് ആവശ്യപ്പെട്ടത്. സ്വലാഹുദ്ദീനെയും മുസ്‌ലിം പടയാളികളെയും വഞ്ചിച്ചതിന് രാജസഹോദരനായ റെയ്‌നാള്‍ഡും മറ്റു ചിലരും മാത്രമാണ് വധിക്കപ്പെട്ടത്. ഗൈ രാജാവടക്കമുള്ളവരെ ആദ്യം ജയിലിലടയ്ക്കുകയും പിന്നെ വെറുതെ വിടുകയുമാണ് ചെയ്തത്. കുരിശുയോദ്ധാക്കള്‍ ജറുസലേമില്‍ നിന്ന് പുറത്താക്കിയ ജൂതന്മാരുടെ പിന്‍ഗാമികളെ വരുത്തി അവിടെ പുനരധിവസിപ്പിക്കുകയാണ് ഭരണമേറ്റെടുത്തയുടനെ സ്വലാഹുദ്ദീന്‍ ചെയ്തത്. മനുഷ്യ രക്തം നീന്തി രക്തദാഹം തീര്‍ക്കുകയും പൈതങ്ങളെയടക്കം കൊന്നൊടുക്കുകയും ചെയ്ത് ജെറുസലേം പിടിച്ചടക്കിയവര്‍ക്ക് അത് തിരിച്ചു പിടിച്ചശേഷം യുദ്ധനേതൃത്വത്തിലുണ്ടായിരുന്നവര്‍ക്കടക്കം മാപ്പ് നല്‍കി വിട്ടയച്ചുകൊണ്ട് എന്തായിരിക്കണം യുദ്ധമര്യാദ എന്ന് പഠിപ്പിക്കുന്നതായിരുന്നു സ്വലാഹുദ്ദീന്റെ നടപടി.

സ്വലാഹുദ്ദീന്‍ അയ്യൂബി ജറുസലേം പിടിച്ചടക്കിയതറിഞ്ഞ പോപ്പ് അര്‍ബന്‍ മൂന്നാമന്‍ മാര്‍പാപ്പ ഹൃദയാഘാതം മൂലം മരണപ്പെട്ടു. ജറുസലേം നഷ്ടപ്പെട്ടതറിഞ്ഞ ക്രിസ്ത്യന്‍ യൂറോപ്പ് ഞെട്ടി. പുതുതായി സ്ഥാനമേറ്റെടുത്ത പോപ്പ് ഗ്രിഗറി എട്ടാമന്‍ മാര്‍പാപ്പ ജറുസലേമിന്റെ നഷ്ടം ക്രൈസ്തവരുടെ പാപങ്ങള്‍ക്കുള്ള ശിക്ഷയാണെന്നും അത് തിരിച്ചു പിടിച്ചാല്‍ മാത്രമെ പ്രസ്തുത പാപങ്ങള്‍ക്ക് പരിഹാരമാവുകയുള്ളൂ എന്നും പ്രഖ്യാപിച്ചു. പരസ്പരം യുദ്ധത്തിലായിരുന്ന ഇംഗ്ലണ്ടിലെ ഹെന്‍ട്രി രണ്ടാമനും ഫ്രാന്‍സിലെ ഫിലിപ്പ് രണ്ടാമനും വൈര്യങ്ങള്‍ വെടിഞ്ഞ് ഐക്യപ്പെടാനും സ്വലാഹുദ്ദീനെതിരെ ഒരുമിച്ചു പോരാടി ജറുസലേം വീണ്ടെടുക്കുവാനും തീരുമാനിച്ചു. സലാദിന്‍ നികുതി (Saladin tithe) എന്ന പേരില്‍ രണ്ടു നാട്ടിലുമുള്ള പൗരന്മാരില്‍ നിന്ന് ചുങ്കം പിരിച്ച് യുദ്ധത്തിനുള്ള സമ്പത്ത് സമാഹരിച്ചു. 1189 ജൂലൈ ആറിന് ഇംഗ്ലണ്ടിലെ രാജാവായ ഹെന്‍ട്രി രണ്ടാമന്‍ മരണപ്പെട്ടതോടെ യുദ്ധനേതൃത്വം മകന്‍ സിംഹഹൃദയനായ റിച്ചാര്‍ഡ് (Richard the lion hearted) എന്നറിയപ്പെട്ട റിച്ചാര്‍ഡ് ഒന്നാമന്റെ ചുമതലയായിത്തീര്‍ന്നു. റിച്ചാര്‍ഡ് ഒന്നാമന്റെയും ഫിലിപ്പ് രണ്ടാമന്റെയും നേതൃത്വത്തിലുള്ള കുരിശുയോദ്ധാക്കളുടെ സൈന്യത്തോട് സ്വലാഹുദ്ദീന്‍ ജയില്‍ മോചിതനാക്കിയ ഗൈ രാജാവും ഒന്നിക്കുകയും ഒരു വലിയ സൈന്യം സ്വലാഹുദ്ദീനെ അക്രമിക്കാനെത്തുകയും ചെയ്തു. വഴിയില്‍ വച്ച് നടന്ന അക്രമങ്ങളിലൂടെ സ്ത്രീകളും കുട്ടികളും അടക്കമുള്ള മുസ്‌ലിംകളെ കൊന്നുകൂട്ടിക്കൊണ്ടായിരുന്നു കുരിശ് സൈന്യം മുന്നേറിയത്. 1191 സെപ്റ്റംബര്‍ ഏഴിന് ജെറുസലേം പിടിച്ചടക്കുക എന്ന ലക്ഷ്യത്തോടെ സിംഹഹൃദയനായ റിച്ചാര്‍ഡിന്റെ നേതൃത്വത്തിലുള്ള സൈന്യം ഫലസ്തീനിലുള്ള അല്‍സൂഫില്‍ വച്ച് സ്വലാഹുദ്ദീനോട് ഏറ്റുമുട്ടിയെങ്കിലും അദ്ദേഹത്തിന് ലക്ഷ്യം നേടാന്‍ കഴിഞ്ഞില്ല. നിരന്തരമായുണ്ടായ യുദ്ധങ്ങള്‍ക്ക് വിരാമമുണ്ടായത് 1192 സെപ്റ്റംബര്‍ രണ്ടിന് സ്വലാഹുദ്ദീനും റിച്ചാര്‍ഡും തമ്മിലുണ്ടാക്കിയ സന്ധിയോടുകൂടിയാണ്. ജറുസലേം മുസ്‌ലിംകളുടെ നിയന്ത്രണത്തിലായിരിക്കുമെന്നും എന്നാല്‍ നിരായുധരായ ക്രൈസ്തവരെ അവിടെയുള്ള ക്രിസ്ത്യന്‍ പുണ്യസ്ഥലങ്ങള്‍ സന്ദര്‍ശിക്കാന്‍ അനുവദിക്കുമെന്നും ആയിരുന്നു കരാര്‍. പുണ്യസ്ഥലങ്ങള്‍ സന്ദര്‍ശിക്കുന്നതില്‍ നിന്ന് മുമ്പും സ്വലാഹുദ്ദീന്‍ ക്രൈസ്തവരെ വിലക്കിയിട്ടില്ല. എന്നതുകൊണ്ടു തന്നെ കരാര്‍ വഴി കുരിശുയോദ്ധാക്കള്‍ക്ക് ഒന്നും നേടാനായില്ലെങ്കിലും, 1192 ഒക്‌ടോബര്‍ ഒന്‍പതിന് റിച്ചാര്‍ഡ് ജറുസലേമില്‍ നിന്ന് മടങ്ങിയത് യുദ്ധരംഗത്ത് പാലിക്കപ്പെടേണ്ട മര്യാദകളെന്തൊക്കെയാണെന്ന് പഠിച്ചുകൊണ്ടായിരുന്നു. ശത്രുവിനെ സ്‌നേഹിക്കേണ്ടത് എങ്ങനെയാണെന്ന് സഭയില്‍ നിന്ന് പഠിക്കാത്ത അദ്ദേഹത്തിന് അത് പഠിപ്പിച്ചത് സ്വലാഹുദ്ദീന്‍ അയ്യൂബിയുടെ യുദ്ധരംഗത്തെ വര്‍ത്തനങ്ങളായിരുന്നു.

യുദ്ധരംഗത്തായിരിക്കുമ്പോള്‍ പോലും ശത്രുവിന്റെ മാനുഷികമായ ആവശ്യങ്ങള്‍ നിര്‍വ്വഹിച്ചു കൊടുക്കുവാന്‍ സ്വലാഹുദ്ദീന്‍ അയ്യൂബി സന്നദ്ധമായി. റിച്ചാര്‍ഡിന് പനി പിടിച്ചപ്പോള്‍ തന്റെ വൈദ്യനെ അയച്ച് ചികില്‍സിക്കുകയും മരുന്ന നല്‍കുകയും നല്ല നല്ല പഴങ്ങളും ശുദ്ധമായ തണുത്ത ജലവും കൊടുത്തയക്കുകയും ചെയ്തത് സ്വലാഹുദ്ദീനായിരുന്നു. യുദ്ധത്തിനിടയില്‍ റിച്ചാര്‍ഡിന് കുതിര നഷ്ടപ്പെട്ടപ്പോള്‍ പകരം രണ്ടു കുതിരകളെ നല്‍കിയതും അദ്ദേഹം തന്നെ. അതുകൊണ്ട് തന്നെ കുരിശുയോദ്ധാക്കള്‍ക്കിടയില്‍ പോലും സ്വലാഹുദ്ദീന്‍ നായകനായിത്തീര്‍ന്നു. തന്റെ സഹോദരിയും സിസിലിയിലെ രാജ്ഞിയുമായ ജോനിനെ സ്വലാഹുദ്ദീന്റെ സഹോദരന് വിവാഹം ചെയ്തു കൊടുക്കുവാന്‍ സന്നദ്ധമാണന്ന് അറിയിച്ചുകൊണ്ടാണ് അദ്ദേഹം ജറുസലേം വിട്ടത്. കാടനായി വന്നയാളെ മാന്യനാക്കിത്തീര്‍ത്ത് തിരിച്ചയച്ചത് ശത്രുവിനെ സ്‌നേഹിക്കണമെന്ന പ്രവാചകന്‍മാരുടെ നിര്‍ദ്ദേശം യഥാവിധി പാലിക്കുവാന്‍ സ്വലാഹുദ്ദീന്‍ അയ്യൂബി തയ്യാറായതു കൊണ്ടായിരുന്നു.

യുദ്ധഭൂമികളില്‍ പൈശാചികമായ രക്തദാഹവുമായി ഉറഞ്ഞു തുള്ളുകയാണ് മതധര്‍മ്മമെന്ന് പഠിപ്പിക്കപ്പെട്ട ക്രൈസ്തവ പടയാളികള്‍ക്ക് മനുഷ്യപക്ഷത്തുനിന്ന് യുദ്ധം ചെയ്യേണ്ടതെങ്ങനെയെന്ന് കാണിച്ചുകൊടുക്കാന്‍ ഒരു സ്വലാഹുദ്ദീന്‍ വേണ്ടി വന്നത് തീരെ യാദൃഛികമായിരുന്നില്ല. യുദ്ധ ധാര്‍മികത അജ്ഞാതമായിരുന്ന ക്രൈസ്തവ ലോകത്തിന് ആര്‍ദ്രതയെ മത പ്രചോദിതമായി രണവീറിലേക്ക് ചേര്‍ത്തുപിടിക്കേണ്ടതുണ്ടെന്ന് മനസ്സിലാക്കിക്കൊടുക്കാന്‍ മുഹമ്മദ് നബി(സ)യുടെ അനുയായികള്‍ക്കല്ലാതെ കഴിയുമായിരുന്നില്ല, ഇസ്‌ലാമിക ഭീകരതയുടെ മീഡിയാ വിശകലനങ്ങള്‍ അന്ധമാക്കിയ നമ്മുടെ പൊതുബോധം അതുള്‍ക്കൊള്ളാന്‍ ഇന്ന് പ്രയാസപ്പെടുമെങ്കിലും. ജീവിതത്തിന്റെ മറ്റു രംഗങ്ങളിലേതു പോലെത്തന്നെ സമാധാന സന്ദര്‍ഭത്തിലും യുദ്ധരംഗത്തുമെല്ലാം പാലിക്കേണ്ട നിയമങ്ങളെന്തൊക്കെയാണെന്ന് മുഹമ്മദ് നബി(സ) പഠിപ്പിച്ചു തരികയും പ്രായോഗികമാക്കി കാണിച്ചുതരുകയും ചെയ്തിട്ടുണ്ട്. പീഡനങ്ങള്‍ സഹിച്ച് പ്രബോധനം ചെയ്യുമ്പോഴും ആദര്‍ശജീവിതത്തിന് നിവൃത്തിയില്ലാതെ പാലായനം ചെയ്യുമ്പോഴും വിവിധ സമുദായങ്ങള്‍ ജീവിക്കുന്ന നാട് ഭരിക്കേണ്ടി വരുമ്പോഴും ഭരണാധികാരിയായി ശത്രുരാജ്യങ്ങളോട് യുദ്ധം ചെയ്യുമ്പോഴും യുദ്ധത്തില്‍ ജയിക്കുമ്പോഴും ശത്രുരാജ്യങ്ങളോട് സന്ധിയുണ്ടാക്കുമ്പോഴും ശത്രുരാജ്യത്തിനുമേല്‍ അധീശത്വം ലഭിക്കുമ്പോഴുമെല്ലാം എങ്ങനെ പെരുമാറണമെന്ന് അദ്ദേഹം കൃത്യമായി പഠിപ്പിച്ചിട്ടുണ്ട്. യുദ്ധരംഗത്തും സമാധാനരംഗത്തുമെല്ലാം മാനവവല്‍ക്കരിക്കുവാന്‍ ആ ജീവിതം തന്നെയാണ് മനുഷ്യര്‍ മാതൃകയാക്കേണ്ടത്. എല്ലാ രംഗത്തുമുള്ള സല്‍സ്വഭാവങ്ങളുടെ പൂര്‍ത്തീകരണത്തിനു വേണ്ടിയാണല്ലോ നബി നിയോഗമുണ്ടായിട്ടുള്ളത്.